Sei sulla pagina 1di 1246

PROMETRIC EXAM PRACTICE QUESTIONS

(specially for Gulf Countries Exam)

Based on past papers questions collection


3rd Edition (2016-2017)

By:
DR. IMRAN ASGHAR






















PREFACE
Praise be to Allah who helped us to complete this exam questions collection book
for Gulf countries 3RD edition. These questions we collected from different sources
and we discussed on our whatsapp group named: Prometric Exam Prep. This
extensive collection of questions will give you idea and exposure to all kind
of promtric exams i.e Gulf countries and some speciality exams too. Mostly for
General Practice exams. This book will help you to understand question nature
and correct answer with explanations. I would like to express my special thanks
of gratitude to those who participated actively on our whatsapp group and helped
in issuing this book.

DR. MUKESH KUMAR

DR. BABAR BILAL

DR. NAZIA NAZ

DR. SHAHINA SALEEM

DR. VIMLA

DR. SHAMIM AKHTAR

DR. KOKAB

DR. KANTA SUNIL

DR. MANAN SOOMRO

DR. HAYAT

This book is to help you, so any


suggestions corrections please contact
us, it will be pleasure to help out in
clearing confusions and queries: Email:
drimran823@gmail.com Contact #
+923363896197(whatsapp)


Internal medicine

Table of Contents

Cardiology ............................................................................................................................................................. 2
Pulmonology ........................................................................................................................................................ 48
Nephrology........................................................................................................................................................... 70
Hematology .......................................................................................................................................................... 90
Neurology ........................................................................................................................................................... 134
Infectious disease............................................................................................................................................... 157
Rheumatology.................................................................................................................................................... 214
Endocrinology ................................................................................................................................................... 233
Gastroenterology ............................................................................................................................................... 259
Immunology & Allergy ..................................................................................................................................... 283
Bullet points ....................................................................................................................................................... 289

1
Cardiology

2
1. What valve lesion you'll find in acute infective endocarditis?
A. Mitral stenosis
B. Mitral regurge
C. Aorta stenosis
D. Aorta regurge.

Answer: b or d
The most common affected valve is mitral valve, aortic valve, tricuspid valve then pulmonary valve. However, in left side vegetation,
the most common underlying lesions being mitral valve prolapse and degenerative mitral and aortic regurgitation. In mitral valve the
is some degree of regurgitation.
References:
Http://patient.info/doctor/infective-endocarditis-pro
Http://www.ncbi.nlm.nih.gov/books/nbk2208/
Http://emedicine.medscape.com/article/216650-overview#a3
Http://emedicine.medscape.com/article/216650-clinical#b2
Http://emedicine.medscape.com/article/150490-overview
Http://www.texasheart.org/hic/heartdoctor/answer_1428.cfm
Http://www.wilkes.med.ucla.edu/mvpmain.htm



2. Pt had urti for 2 weeks later developed orthopnea, severe pulmonary edema, what is the dx?
A. Infective endocarditis
B. Acute epicarditis
C. Acute myocarditis
D. Acute bronchitis

Answer: c
Patients with myocarditis have a clinical history of acute decompensation of heart failure, (e.g. Tachycardia, gallop, mitral
regurgitation, edema). In viral myocarditis, patients may present with a history of recent (within 1-2 wk) flulike syndrome of fevers,
arthralgia, and malaise or pharyngitis, tonsillitis, or upper respiratory tract infection.
Http://emedicine.medscape.com/article/156330-clinical#b1


3. Mid diastolic murmur in left sternum?
A. Mitral stenosis
B. Mitral regurge
C. Aortic stenosis
D. Aortic regurge

Answer: a
Ms: mid diastolic to late diastolic , rumbling
Mr: holosyatolic ‘pansystolic “, blowing radiate to left axilla
As: midsystic , harsh radiated to carotid artery
Ar: early diastolic


4. Best initial screening test for pt suspected with coarctation of aorta?
A. Echo/doppler
B. CT cardiac
C. MRI cardiac
D. Cardiac angio

Answer: chest xray (best initial test according to master the boards, first aid, kaplan)
Uptodate: “ in children with coarctation, echocardiography often provides adequate anatomic and hemodynamic information for
the surgeon or interventional cardiologist without the need for a further imaging study. However, MRI or ct is generally used as a
complementary diagnostic tool in adolescent and adult patients, and provides important anatomic data prior to intervention.

3
In adults, cranial mr angiography (or CT angiography) is also appropriate to search for intracranial aneurysms


5. Patient has history of mi suddenly he became breathless and harsh systolic murmur heard what the cause?
A. PE
B. Rupture aortic cusp
C. Tricu regurge
D. Rupture

Answer: interventricular septum rupture


''Rupture of the ventricular septum is often accompanied by a new harsh holosystolic murmur best heard at the left lower sternal
border''.
Reference: http://www.clevelandclinicmeded.com/medicalpubs/diseasemanagement/cardiology/complications-of-acute-
myocardial-infarction/



6. Mid diastolic murmur in left sternum?
A. Mitral stenosis
B. Mitral regurgitation
C. Aortic stenosis
D. Aortic regurgitation

Answer: a
Causes of mid diastolic murmur :
Mitral stenosis is best heard at apex.
Tricuspid stenosis is best heard along the left sternal border.
Atrial myxoma.
Reference : uptodate


7. With thyrotoxicosis, what is the most common arrhythmia?
A. SVT
B. Vt
C. Vfib
D. Wpw

Answer: SVT (toronto note mentions AFib as most common)


According to medscape, arrhythmias in thyroid storm are usually supraventricular, with atrial fibrillation being the most common.
Atrial flutter and paroxysmal atrial tachycardia are uncommon, whereas ventricular premature contractions and ventricular
fibrillation are rare.
They are due to the direct augmenting effect of the thyroid hormones on the β-adrenergic receptor's sensitivity to catecholamines
and increasing myocardial excitability, and on the intrinsic sinoatrial electrophysiologic function, thus decreasing conduction time
and resulting in nodal blockade in longstanding disease
Image bellow is from toronto notes.

4



8. The best initial screening test for patient suspected with coarctation of aorta
A. Echo/doppler
B. CT cardiac
C. MRI cardiac
D. Cardiac angio

Answer: a
Reference: http://cursoenarm.net/uptodate/contents/mobipreview.htm?17/11/17584#h17


9. Patient has history of mi suddenly he became breathlessness and harsh systolic murmur heard what the cause:
A. Pe
B. Rupture aortic cusp
C. Tricuspid regurgitation
D. Rupture septum

Answer: d
Acute sob with high pitched systolic murmur post mi is usually due to acute mitral regurgitation with or without injury to the
papillary muscle chordae tendineae
Reference: merck manual


10. Old lady with sharp chest pain and fever diagnosed with pericarditis what will you do to dx the case, most accurate test is :
A. Acid fact stain
B. Pericardial biopsy (my answer)
C. Pleural aspiration
D. Forget the last choice( the answers look like they are talking about tb as the cause )

11. Hypertensive patient on ACEI but not controlled Blood pressure , what to add?
A. Thiazide
B. Beta blocker
C. Furosemide
D. Nefidipine (calcium channel blocker)

5

Answer: d
In nonblack hypertensive patients, begin treatment with either a thiazide-type diuretic, CCB , ACE inhibitor, or arb
If a patient's goal Blood pressure is not achieved within 1 month of treatment, increase the dose of the initial agent or add an agent
from another of the recommended drug classes; if 2-drug therapy is unsuccessful for reaching the target Blood pressure , add a third
agent from the recommended drug classes
Http://emedicine.medscape.com/article/241381-treatment


12. Mitral stenosis cause enlargement in which chamber :
A. Left atrium
B. Right atrium
C. Left ventricle
D. Right ventricle

Answer: a
Mitral stenosis (ms) is characterized by obstruction to left ventricular inflow at the level of mitral valve due to structural abnormality
of the mitral valve apparatus. The most common cause of mitral stenosis is rheumatic fever. The normal mitral valve orifice area is
approximately 4-6 cm2. Patients will not experience valve-related symptoms until the valve area is 2-2.5 cm2 or less, at which point
moderate exercise or tachycardia may result in exertional dyspnea.
Http://emedicine.medscape.com/article/155724-overview#a5



13. The same scenario as the above question, how to treat?
A. Aspirin
B. Heparin
C. Lmwh
D. Fracion heparin
Answer c
According to medscape: regular unfractionated heparin was the standard of care until the introduction of lmwh products.
Reference: http://emedicine.medscape.com/article/1911303-treatment#d10

14. Most specific for coronary artery disease


A. Cholesterol
B. Triglycerides
C. Hdl
D. Ldl
Answer: d
Reference: http://emedicine.medscape.com/article/2087735-overview#a2

15. What types of hypersensitive rapid onset of ? I did not understand the q
Fast acting anti-hypertensives:
A. Captopril
B. Clonidine
C. Labetalol
D. Nifedipine
E. Sodium nitroprusside

16. Cardiac notch is at which intercostal level?


A. 3
B. 4
C. 5
D. 6
6
Answer:???
The cardiac notch is an angular notch that lies along the fifth and sixth intercostals space, where the
pericardium is exposed.
Http://www.anatomyexpert.com/app/structure/10604/1786/

17. Which is more at risk for cad


A. 25 male, smoker, sedentary life
B. 35 female, type 1dm, obese
C. 55 male, hypertension, high cholesterol
D. 50 male, obese, dm
Answer: c or d, age over 45, HTN hypercholesterolemia, dm, and obesity are all major risk factors but i think c because the older age.
Http://emedicine.medscape.com/article/164163-overview#a1

18. A case of rheumatic fever he had a severe arthritis and mitral regurgitation what is your short management?
A. Steroid and high dose aspirin
B. Daily steroid and aspirin
C. Im penicillin
D. Monthly
Answer: d
Anti-inflammatory agents are used to control the arthritis, fever, and other acute symptoms. Salicylates are the preferred agents,
although other nonsteroidal agents are probably equally efficacious.
Http://emedicine.medscape.com/article/236582-medication#2


19. Newly diagnosed hypertensive patient came to the primary clinic complaining of dry cough and shortness of breath. Which
medication he used?
A. Perindopril
B. Valsartan
C. Atenolol
D. Thiazide
Answer: a- perindopril (dry cough is one of the sides effect of ACEIs) drugs.com
Https://www.drugs.com/cdi/perindopril.html

20. Which murmur is associated with mitral stenosis?


A. Mid systolic
B. Holo systolic
C. Mid diastolic
D. Early systolic
Answer: c- mid diastolic. Easyauscultation.com
Http://www.easyauscultation.com/cases?Coursecaseorder=14&courseid=31

21. What condition should to take more attention regarding prevention of coronary artery disease
A. 25 yo with dm1 and htn
B. 55 yo with DM and htn
C. 55 (or 50) yo with DM and high cholesterol
D. 25 yo with DM and something
Answer: b
Http://emedicine.medscape.com/article/164163-overview

22. Which of the following is a minor criterion in duke’s criteria for rheumatic fever?
A. Fever
B. Lymphocytosis
7
C. Carditis
D. Arthritis
Answer: a
Major criteria:
carditis†
• clinical and/or subclinical
arthritis
• polyarthritis only
chorea
erythema marginatum
subcutaneous nodules
Minor criteria
polyarthralgia
fever (≥38.5°c)

esr ≥60 mm in the first hour and/or crp ≥3.0 mg/dl§


prolonged pr interval, after accounting for age variability (unless carditis is a major
criterion)



23. 60 years old male, c/c chest pain, he was brought to er of tertiary center. Ecg showed st elevation in leads ii, v1, v2, v3, after
initial management in the er, next step is?
A. Heparin
B. Streptokinase
C. Tpa
D. Stenting of coronary artery
Answer: d
Read more in: http://circ.ahajournals.org/content/110/5/588.full

24. 3 yo known case of cardiac disease. What will you do?


A. Eco
B. Ecg
C. Catheterization
D. Observation
Answer: a, incomplete


25. Pt c/o sever uncontrolled htn, “renography” not sure”, showed l renal artery stenosis, next step is to?
A. Venography
B. Ivp
C. CT angiograpgy
D. Renography
Answer: c
Reference: http://emedicine.medscape.com/article/245023-workup#c7

26. Man with chest pain while exercises, hx of chest pain in rest and exercise. Ejection systolic murmur at left sternal, not
radiating, ecg > left atrial enlargement with no specific st changes. Dx?
A. Aortic stenosis
B. Pulmonary stenosis,
C. Restrictive cardiomyopathy,
D. Hypertrophic cardiomyopathy.

8
Answer: a?

27. 50 y\o male presented to er with severe respiratory distress, no other history obtained, investigations were done to him cxr:
pulmonary edema with infiltrate ecg: he was having atrial fibrillation, his pulse rate was 125 what investigation you will do to
him:
A. Echocardiography
B. Spiral ct
C. V\q match
D. Arteriogram

Answer : a

Reference: stepup to medicine +
Kumar and clarck’s clinical medicine






28. Newly diagnosed hypertensive patient came to the primary clinic complaining of dry cough and shortness of breath. Which
medication he used?
A. Pernidopril
B. Valsartan
C. Atenolol
D. Thiazide
Answer: a
Http://www.rxlist.com/ace_inhibitors-page2/drugs-condition.htm

29. 60 years old patient has only htn, what’s the best drug to start with?
A. ARB
B. ACEI
C. BB
D. Diuretics
Answer: d

10







30. Which murmur is associated with mitral stenosis?
A. Mid systolic
B. Holo systolic
C. Mid diastolic
D. Early systolic
Answer: c
Http://www.merckmanuals.com/professional/cardiovascular-disorders/valvular-disorders/mitral-stenosis
Http://www.wilkes.med.ucla.edu/msmain.htm

31. Elderly patient presented with chest pain he is a smoker with positive family history of coronary artery disease and he is
dyslipedmic he is also over weight and can not tolerate excersise his ecg is normal what will do:
A. Stress echo
B. Stress test
C. Resting perfusion scan
D. Angiography

Answer: c
Http://www.hopkinsmedicine.org/healthlibrary/test_procedures/cardiovascular/myocardial_perfusion_scan_resting_92,p0797
8/

32. Patient with high level of cholesterol, what to avoid?

11
A. Organ meat
B. Avocado
C. Chicken
D. White egg
Answer: A

33. Most specific for coronary artery disease


A. Cholesterol
B. Triglycerides
C. Hdl
D. Ldl
Answer: d
Http://www.medscape.com/viewarticle/545401_2


34. Which of the following is most likely to be the presentation of a patient with early stemi?
A. Troponin of 0.12 with t inversion in v1-v4
B. Pathological q wave with subsided chest pain
C. Presence of chest pain with 0.3 mm elevation in st segment in leads 2,3, avf
D. St depression in 2,3, avf

Answer: c?
Normal ecg prior to mi
Hyperacute t wave changes - increased t wave amplitude and width; may also see st elevation
Marked st elevation with hyperacute t wave changes (transmural injury)
Pathologic q waves, less st elevation, terminal t wave inversion (necrosis)
(pathologic q waves are usually defined as duration ≥ 0.04 s or ≥ 25% of r-wave amplitude)
Pathologic q waves, t wave inversion (necrosis and fibrosis)
Pathologic q waves, upright t waves (fibrosis)

35. Pt with past history valve heart disease. 2 weeks ago he had tooth procedure & now present with symptoms of infective
endocarditis. Which of the following is the most likely organism?
A. Staph aureus
B. Strep pneumonia
C. Strep viridans
D. Moraxella catarrhalis
Answer: a?
Toothbrushing and chewing can cause bacteremia (usually due to viridans streptococci) in patients with gingivitis

Reference: merck manual

36. Ecg with t inverted in some leads otherwise normal.


Labs: tropi high ck high what dx?
A. Low risk unstable angina.
B. High risk unstable angina.
C. Nstemi.
D. Acute st elevation mi.
Answer

37. The best for lifestyle tt for HTN patient:


A. Na restriction less than 6
B. Give to him k
C. Run every day 1.5 km

12
D. Do something 4 time per week

•reduce sodium intake to no more than 100 mmol/d (2.4 g sodium or 6 g sodium chloride; range of approximate sbp reduction, 2-8
mm hg)
•engage in aerobic exercise at least 30 minutes daily for most days (range of approximate sbp reduction, 4-9 mm hg)

38. Ecg finding of ostium secundum atrial septal defect.


A. Lt axis deviation
B. Rbbb
C. Lvh
D. Delta wave
Answer: b
The most frequent ecg abnormalities in asd are right bundle branch block (rbbb).
Source: http://www.metealpaslan.com/ecg/asden.htm



39. Most important risk factor for cardiac?
A. Ldl
B. Total cholesterol
C. Blood pressure
D. Smoking
Answer: c?

40. Which of the following is side effect of atropine:


A. Vasoconstriction
B. Decrease iop
C. Decrease urine output
D. Dry of mouth
Answer: d
Most of the side effects are directly related to its antimuscarinic action. Dryness of the mouth, blurred
vision, photophobia and tachycardia commonly occur with chronic administration of therapeutic doses. Anhidrosis also may occur
and produce heat intolerance or impair temperature regulation in persons living in a hot environment. Constipation and difficulty
in micturition may occur in elderly patients.
[http://www.rxlist.com/atropine-drug/side-effects-interactions.htm]

41. Which in lipid profile is most important risk for coronary heart disease ?
A. Ldl,
B. Hdl,
C. Triglyceride ,
D. Total cholesterol
Answer a

42. Mitral stenosis cause enlargement in which chamber:


A. Left atrium
B. Right atrium
C. Left ventricle
D. Right ventricle
Answer a
Chest radiographic findings suggestive of mitral stenosis include left atrial enlargement (eg, double shadow in the cardiac silhouette,
straightening of left cardiac border due to the large left atrial appendage

13
43. Patient has history of mi suddenly he became breathless and a harsh systolic murmur was heard:
A. Pe
B. Rupture aortic cusp
C. Tricuspid regurge
D. Rupture .......
Answer: if answer d is ruptured papillary muscle then it’s true. If not go with b

44. Post mi, patient develops atrial fibrillation. Which medication is required to prevent stroke?
A. Warfarin
B. Heparin
C. Aspirin
D. Dabigatran

Answer: a
Taking an anticoagulant (blood thinner) can reduce the risk of having a stroke by approximately 50 to 70 percent.
Reference:
Http://www.uptodate.com/contents/atrial-fibrillation-beyond-the-basics#h17

45. Long scenario asking how amlodipine cause ll edema


A. Increase capillary oncotic pressure ?
B. Decrease plasma ...
C. Decrease cardiac output
D. Increased hydrostatic pressure

Answer: d
Reference: medscape: http://www.medscape.com/viewarticle/460070_1


46. Mitral stenosis cause enlargement in which chamber
A. Left atrium
B. Right atrium
C. Left ventricle
D. Right ventricle
Answer: a
Medscape

47. Ecg finding of ostium secundum atrial septal defect


A. Left axis deviation
B. Rbbb
C. LVH
D. Delta wave

Answer: b

14
toronto: ecg: rad, mild rvh, rbbb



48. Phytosterol mode of action;
A. Decrease cholesterol level
B. Decrease cholesterol synthesis
C. Decrease triglyceride synthesis
D. Decrease de no vo cholesterol synthesis
Answer:
Decreases cholesterol absorption from small intestine
Https://en.wikipedia.org/wiki/phytosterol

49. Ecg with AFib pattern. A patient present with irregular cardiac palpitation some duration in the past for which he was taking
a drug, lab values given demonstrating normal upper limit of aptt, normal pt, normal inr, normal platelet, low hemoglobin,
what is the mechanism of action of the drug?
A. Anti-thrombin iii
B. Decrease factor viii
C. Inhibit platelets aggregation
D. Decrease vitamin k dependent factors

50. Acute mi patient will present with?


A. High troponin i with st elevation in v2-v6
B. Ongoing chest pain with high st elevation in lead ii - iii and avf
C. ST elevation with no chest pain
D. ST depression with .....

Answer: missing information

51. Case presented with st elevations mi for 6 hours no neurological symptoms after CPR the patient died in autopsy you well
find:
A. Brain absess
B. Interventcular hemorrhage
C. Red cells in the hypocample area
D. Necrosis in the area associated with midcerbral infarct
Answer: d

52. Which medication will delay the surgery for chronic aortic regurgitation?
A. Digoxin
B. Verapamil
C. Nifedipine

15
D. Enalapril

Answer: C
o Nifedipine is the best evidence-based treatment in this indication.
o ACE inhibitors are particularly useful for hypertensive patients with AR.
o Beta blockers may be indicated to slow the rate of aortic dilatation and delay the need for surgery in patients with AR associated
with aortic root disease. Furthermore, they may improve cardiac performance by reducing cardiac volume and LV mass in
patients with impaired LV function after AVR for AR.
Reference: 3rd Edition UQU>Cardiology> Q3
In severe aortic valve insufficiency, the excess in afterload increases burden on the left side of the heart. Theoretically, any medication
that can reduce afterload could be expected to improve left ventricular function and decrease regurgitant backflow from the aorta. This
would provide a temporizing measure by which surgical intervention can be postponed. One study showed that the use of nifedipine in
asymptomatic patients with severe aortic regurgitation who had normal LV function could delay the need for surgery by 2-3 years. This
result may also be expected with the use of similar vasodilating agents.
Reference: Medscape.

53. (long scenario) man with chest pain and abnormal EKG. Which one of the following will be elevated?
A. ESR.
B. M2 Protein.
C. CRP.
D. Creatinine.

Answer: C
References:
Http://circ.ahajournals.org/content/113/6/e72.full
Http://circ.ahajournals.org/content/123/10/1092.full
Http://circ.ahajournals.org/content/107/3/499.figures-only

54. Patient with decreascendo-increasendo murmure, in midsternal not radiating, high pitched first sound?
A. Ejection diastol
B. Ejection systolic
C. Physiologic
D. Innocent
Answer: b
Systolic ejection (not sure 100%) maybe some points missed in the questions because answer overlapping. Systolic ejection murmurs
may be functional or organic.The most common functional systolic ejection murmur in adults is probably a variant of Still's murmur,
the so-called innocent murmur of childhood. It is a short, buzzing, pure, medium-pitched, nonradiating, midsystolic murmur heard best
along the upper left sternal border.

55. Best initial screening test for pt suspected with coarctation of aorta?
A. Echo/doppler
B. CT cardiac
C. MRI cardiac
D. Cardiac angio

Answer: A
No specific laboratory tests are necessary for coarctation of the aorta. Imaging studies that may be helpful include the following: Chest
radiography (Findings vary with the clinical presentation of the patient), Barium esophagography (Classic “E sign,” representing
compression from the dilated left subclavian artery and poststenotic dilatation of the descending aorta) Echocardiography (2-dimensional
echocardiography, pulsed-wave Doppler, and color flow mapping). In another reference once suspected, an echocardiogram is the
most commonly used test to confirm the diagnosis. It is more accurate. Cardiac catheterization helps to confirm the diagnosis when
echocardiography findings are not completely clear.
Http://www.cdc.gov/ncbddd/heartdefects/coarctationofaorta.html
Http://emedicine.medscape.com/article/150369-overview
16
56. Patient has history of MI, suddenly he became breathlessness and harsh systolic murmur heard what is the cause?
A. PE
B. Rupture aortic cusp
C. Tricu regurge
D. Rupture .......

Answer: Rupture of papillary muscle cause Mitral Regurgitation (pansystolic), ventricular septal defect cause harsh pansystolic
murmur and Left Ventricular Outflow Tract Obstruction cause systolic murmur
Https://books.google.com.sa/books?Id=anbzecmvdyic&pg=PA107&lpg=PA107&dq=myocardial+infarction+harsh+systolic&source=bl
&ots=Zc4qWUx2t6&sig=6Vaup9OPqC3FhVYwI7ElgI7b2bA&hl=en&sa=X&redir_esc=y#v=onepage&q&f=true
Http://www.clevelandclinicmeded.com/medicalpubs/diseasemanagement/cardiology/complications-of-acute-myocardial-
infarction/



57. What is diagnosis?

A. SVT
B. AF
C. VT
D. VF
Answer: A
“same ECG that shows SVT in UQU cardiology section” << couldn’t find it, so I uploaded another ECG strip.
Read about other arrhythmias.

58. Patient with chest pain. A picture of an ECG (NO ST ELEVATION). Lab results: HIGH
TROPONIN, HIGH LDH, HIGH ASPARTAT.
A. A HIGH RISK UNSTBLE ANGINA
B. Low risk
C. NSTEMI
D. STEMI

Answer: NSTEMI
• Increase LDH, AST, and Troponin in skeletal muscle injury and MI.
• Since there are no ECG changes (ST elevation), the answer would be NSTEMI.
Reference: http://medicinembbs.blogspot.com/2011/06/what-is-difference-between-nstemi-and.html
NSTEMI indicate myocardial necrosis marked by elevation in troponin I and CK-MP without ST-segment elevation on the ECG.
Reference: First Aid USMLE STEP 2 CK


59. Obese male pt presented to family physician complaining of chest pain for 2 days, ecg normal, what to do next?
A. Treadmill
B. Coronary angio
C. 24 hrs monitoring

Answer: a
Chest x-ray, exercise stress test, ecg and angiography as definitive diagnostic test are considered in patient with chest pain
suspecting angina.

17
Reference: http://emedicine.medscape.com/article/150215-overview



60. Patient with chest pain and st changes, you will find elevation in:
A. ALT
B. AST
C. Troponin

Answer: c


61. Hypertensive patient on ACEI but not controlled Blood pressure , what to add?
A. Furosemide
B. Thiazide
C. Beta blocker

Answer: b (CCB if no option> thiazide)




62. Smoker and obese patient can't exercise, with family history of mi, came with vague chest pain. Ecg is normal. What’s the
next step?
A. 24 hours ecg
B. Exercise with ecg
C. Perfusion cardiac scan

Answer: c
According to medscape, if the patient cannot tolerate stress tests, one should proceed to myocardial perfusion scans

63. A patient with sudden chest pain, diaphoresis, what’s the best test to order?
A. Echo
B. ECG
C. Cardiac enzymes

Answer: b
Ecg is the first investigation in chest pain.
Reference: master the boards


64. Urti after 1 week complaining of severe shortness of breath and orthopnea with no fever
A. Acute myocarditis
B. Acute pericarditis
C. Ie
18

Answer is a. Patients with myocarditis have a clinical history of acute decompensation of heart failure,( e.g. Tachycardia, gallop, mitral
regurgitation, edema ).
In viral myocarditis, patients may present with a history of recent (within 1-2 wk) flulike syndrome of fevers, arthralgias, and malaise
or pharyngitis, tonsillitis, or upper respiratory tract infection.
Http://emedicine.medscape.com/article/156330-clinical#b1


65. Case of mi with ecg showing t wave inversion in v3 - v6 with elevated troponin and ck, what's the diagnosis:
A. Unstable angina
B. NSTEMI
C. STEMI
Answer: b

66. Patient come with friction rub what is your next action
A. Echo and cardiology consultation
B. X – ray
C. Pericardiocentesis
Answer: a
Pericarditis:
Ecg: initially diffuse elevated st segments ± depressed pr segment, the elevation in the st segment is concave upwardsg2-5 d later st
isoelectric with t wave flattening and inversion
• cxr: normal heart size, pulmonary infiltrates
• echo: performed to assess for pericardial effusion

67. Pt came with cough, dyspnea, distended neck vein, peripheral edema & ascites. By examination there is bilateral rales. What
is the underlying cause
A. Right heart failure
B. Left heart failure
C. Aortic valve regurge others

Answer: should be congestive heart failure

68. Patient presented with chest pain ecg showing st segment elevation in lead i ,avl,v5andv6 ?
A. Lateral ischemia
B. Ant ischemia
C. Post ischemia
Answer: a
Http://lifeinthefastlane.com/ecg-library/lateral-stemi/
Answer: a

69. What is the most common cause of secondary hypertension?


A. Kidney
B. Liver
C. Spleen
Answer: a

70. Patient has proximal nocturnal dyspnea orthopnea and dyspnea with a history of mitral stenosis ... What is the most likely
diagnosis:
A. Acute respiratory distress syndrome
B. Left heart failure
C. Right heart failure
Answer: c

19
71. Trauma patient with hypotension, distended jugular veins, good bilateral equal air entry, diagnosis;
A. Tension pneumothorax
B. Hemothorax
C. Cardiac tamponade
Answer: c
Good bilateral air entry makes you avoid a &b. Also hypotension and distended jvp are directing to it also.
Http://emedicine.medscape.com/article/152083-clinical

72. Patient came with irregularly irregular pulse:


A. Atrial fibrillation
B. Supraventricular tachycardia
C. Heart block

Answer: a
Kaplan step 2ck pediatrics 2014

73. Black man e high Blood pressure , +ve fx hx of HTN . Best med to give?' (cardio)
A. B blocker
B. ACEI
C. CCB
Answer: c thiazide diuretics and calcium channel blockers are preferred in black patients.
Reference: http://www.aafp.org/afp/2015/0201/p172.html

74. Which of the following clinically suggests an early onset st elevation mi ?


A. Elevated troponin1 and long inverted t wave in lead 1,2 avf
B. St elevation of .3 mm in lead v1-v6 with chest pain
C. St elevation without chest pain

The answer is b
T wave inversion is considered to be a late finding. The changes should be corresponding to a side of the heart. St elevation without
chest pain is atypical presentation.
Reference: step up to medicine


75. Ecg picture
A. AFib
B. Vt
C. SVT ?
Reference :for more information about SVT http://www.practicalclinicalskills.com/supraventricular-tachycardia

76. On physical examination, patient shows friction rib. What is your next step?
A. Echo and cardiology consultant
B. X – ray
C. Pericardiocentesis

Answer: a
Pericardial friction rub is specific for pericarditis. Echo is part of its initial approach. If the patient is unstable, pericardiocentesis will
be needed.
X- ray doesn't have a strong rule in diagnosing pericarditis
Extra note: echocardiography is recommended in all cases of pericarditis. Any form of pericardial inflammation can induce pericardial
effusion.
- Http://emedicine.medscape.com/article/156951-overview

20
77. Elderly patient presented with right leg swelling, pitting edema and history of knee swelling and pain. What will you doe next?
A. Echocardiogram
B. Doppler
C. Chest x-ray
Answer: b
Dvt diagnosis is typically by ultrasonography with doppler flow studies (duplex ultrasonography).

78. A medication that will decrease cardiac output and decrease peripheral resistance?
A. Carvedilol
B. Hydralazine
C. CCB
Answer: ??

79. 22 year old male patient presents with chest pain increase when he is lying flat, ecg shows diffuse st-elevation in all leads
what is the diagnosis?
A. Myocardial infarction
B. Pericarditis
C. Infective endocarditis
Answer: b, in mi, st elevation in leads that correspond to the cardiac muscle damage.
Http://www.usmleforum.com/files/forum/2011/2/594549.php

80. Very obese smoker patient complain of chest pain how to test for myocardial perfusion:
A. Exercise stress test
B. 24 hrs holter monitor
C. Dobutamine stress test
Answer: c, pharmacologic stress testing is usually used when patients cannot walk on a treadmill long enough to reach their target
heart rate because of deconditioning, musculoskeletal disorders, obesity, peripheral arterial disease, or other disorders.
- holter is for arrhythmias.
Http://www.merckmanuals.com/professional/cardiovascular-disorders/cardiovascular-tests-and-procedures/stress-testing

81. Newborn is irritable and sweating, chest is clear, vitals was provided, hr 300 beat/min, what's your action:
A. Cardiac dextroversion
B. Vagal massage
C. Digoxin

Answer: b vagal massage. Maneuvers that stimulate the vagus delay av conduction and thus block the reentry mechanism: the valsalva
maneuver, carotid sinus massage, breath holding, and head immersion in cold water (or placing an ice bag to the face)
Acute treatment: pharmacologic therapy
• IV adenosine—agent of choice due to short duration of action and effective- ness in terminating svts; works by decreasing sinoatrial
and av nodal activity.
• IV verapamil (calcium channel blocker) and IV esmolol (β-blocker) or digoxin
Are alternatives in patients with preserved left ventricular function.
• dc cardioversion if drugs are not effective or if unstable; almost always suc-
Cessful.
R
step up to medicine

82. Aspirin side effect:


A. Diarrhea
B. Constipation
C. Bleeding
Answer: c
Reference: http://reference.medscape.com/drug/zorprin-bayer-buffered-aspirin-343279#4

21
83. A 68-year-old diabetic, started 10 days ago on amlodipine 10mg for hypertension, now he is complaining of gross ankle
edema, on examination, jvp was not raised, by auscultation the chest no base crepitations
labs: na, k and ca all within normal range
what is the most likely cause of his edema?
A. Na and water retention
B. Increased capillary hydrostatic pressure
C. Decreased oncotic pressure
Answer: b
Its etiology relates to a decrease in arteriolar resistance that goes unmatched in the venous circulation. T this disproportionate change in
resistance increases hydrostatic pressures in the precapillary circulation and permits fluid shifts into the interstitial compartment.
Link: http://www.medscape.com/viewarticle/460070_1

84. Which of following related to rheumatic heart disease?


A. Mitral stenosis
B. Mitral regurgitation
C. Aortic stenosis
Answer: a
Chronic rheumatic heart disease remains the leading cause of mitral valve stenosis and valve replacement in adults in the united states.

85. Which heart disease is common in down syndrome:


A. VSD
B. Atrioventricular septal defect
C. Coartication of aorta
Answer: b
AVSD is the most common cardiac defect in trisomy 21.
[https://www.pediatricheartspecialists.com/blog/55-down-syndrome-and-congenital-heart-disease]

86. Patient with murmur in left sternal border change with stand or sitting what is dx:
A. Pulmonary stenosis
B. Aortic stenosis
C. Cardiomyopathy
Answer: a
Ps murmur is a left upper sternal border murmur that is affected by standing.

87. Smokers obese patient can't exercise with family history of mi came with
Vague chest pain . But ecg is normal next step ?
A. 24 hours ecg
B. Exercise with ecg
C. Perfusion cardiac scan
Answer : c
From master the boards: perfusion cardiac scan, when the patient can’t exercise . Abnormality will be detected by seeing decreased
thallium uptake.
Nb: if the question is describing a scenario in the er, acute chest pain do ecg and cardiac enzymes .
If the question is describing a scenario in the clinic or chest pain etiology is not clear or the ecg is not diagnostic then do exercise
tolerance test.

88. A patient with renal failure and mitral regurgitation, valve was 0.7 cm. Other findings in aortic valve? What is the proper
management?
A. Total valve replacement
B. Valvoplasty
C. Balloon
Answer: http://www.uptodate.com/contents/mitral-regurgitation-beyond-the-basics

89. Athletic come for check-up all thing normal except xanthelasma on achllis tendon and cholesterol?

22
A. Ldl receptor
B. Apo ll
C. Apo c

Answer: a
Familial hypercholesterolemia (fh) is an autosomal dominant disorder that causes severe elevations in total cholesterol and low-density
lipoprotein cholesterol (ldlc).
Xanthomas are noted commonly on the achilles tendons and metacarpal phalangeal extensor tendons of the hands of patients with
untreated fh.



90. Exercise for cardiac patients:
A. Yoga
B. Isometric
C. Isotonic
Answer: c
The usual mode of exercise recommended is dynamic, aerobic or isotonic
Exercise, including walking, running, cycling, swimming, aerobic dancing, cross-country skiing, and elliptical machines. Isometric
exercise does not improve cardiac performance. Http://www.ncbi.nlm.nih.gov/pmc/articles/pmc3116747/

91. ECG Picture?


A. AF
B. VT
C. SVT

92. Treatment of acute myocarditis?


A. Abx.
B. Immunoglobulin
C. Steroid
Answer: c
Medscape:
Avoid: NSAID, BB (unstable)
• Standard treatment of clinically significant disease includes the detection of dysrhythmia with cardiac monitoring, the
administration of supplemental oxygen, and the management of fluid status.
• Left ventricular dysfunction developing from myocarditis should be approached in much the same manner as other causes of
congestive heart failure (chf), with some exceptions. In general, sympathomimetic drugs should be avoided, because the
increase the extent of myocardial necrosis and mortality.
• Intensive immunosuppressants “e.g. Steroids” has been shown to have some benefits only in small scale clinical trials, and
has not been validated in a larger scale. At this time, it is not recommended until a clear evidence is available.

93. Mid diastolic rumble with opening snap and dysphagia. Where is the lesion?
A. Left atrium
B. Aortic arch
C. Left ventricle
Answer: a

This is an auscultation finding of mitral stenosis. In mitral stenosis, blood pools in the left atrium causing it to enlarge. Left atrium is
located in front of esophagus so any enlargement of left atrium will compress on the esophagus causing dysphagia.
Usmle step 1 anatomy kaplan lecture notes

94. What lower blood pressure the most ??


A. Weight loss

23
B. Salt restricted diet
C. Aerobic exercise
Answer: a

• Weight loss helps to prevent hypertension (range of approximate systolic Blood pressure reduction [sbp], 5-20 mm Hg
per 10 kg).
• Reduce sodium intake to no more than 100 mmol/d (2.4 g sodium or 6 g sodium chloride; range of approximate sbp
reduction, 2-8 mm hg)
• Engage in aerobic exercise at least 30 minutes daily for most days (range of approximate sbp reduction, 4-9 mm hg)
Reference: medscape: http://emedicine.medscape.com/article/241381-treatment#d9

95. Patient with heart failure and af, you added digoxin, what is the benefit:
A. Decrease heart failure
B. Slow ventricular rate
C. Decrease ventricular efficacy
Answer: b

Reference: uptodate

96. Which murmur is a crescendo-decrescendo murmur?


A. Systolic ejection (aortic stenosis)
B. Physiologic murmur
C. Innocent murmur

Answer: a

• Classic crescendo-decrescendo systolic murmur of aortic stenosis begins shortly after the first heart sound.

Reference: medscape: http://emedicine.medscape.com/article/150638-overview




97. Patient with mitral valve disease and murmur radiating to the axilla, right ventricle is enlarged. How will you manage?
A. Mitral valve replacement
B. Medical treatment
C. Pci
Answer: a
It’s mitral regurgitation.valve replacement is indicated when the heart starts to dilate. Do not wait for left ventricular end systolic
diameter (lvesd) to become too large because the damage will be irreversible.(many references)

98. Pt with mi he is on nitroglycerin and morphine. After 20 minutes he worse with raise jvp and no lung finding. Bp drop.

24
A. Ruptured
B. Arrhythmia
C. Rv infarction
Answer: c


99. Patient with mi. What is the drug that decrease the mortality rate?
A. Captopril
B. Digoxin
C. No b-blocker in the choices
Answer: a

100.High pitched? Murmur crescendo decrescendo, midsternal


A. Ejection systolic
B. Ejection regurgitate
C. Innocent

Answer: a

101.Which one of these can cause LBBB?


A. Aortic stenosis
B. PE
C. Cardiomyopathy

Answer: A (Most common of these choices)


Arteriosclerotic coronary artery disease is the most common adult cardiovascular disease. As a result, it is the most common cause of
left bundle branch block. Hypertension, aortic valve disease and cardiomyopathies continue to be important but less common etiologic
disorders.
Reference: Journal of the American College of Cardiology.

102.Patient with chest pain and ST changes, you will find elevation in:
A. ALT
B. AST
C. Troponin

Answer: C

103.Prophylactic antiarrhythmic therapy?


A. Procainamide
B. Lidocaine
C. Amiodarone

Answer: C
Don’t give the patient any anti-arrhythmic medication unless he has an arrhythmia.
Reference: Step up to Medicine.
C is the best answer compared to A and B (if not post MI)
The question might be similar to this (Post MI):

25
In this case the answer will be Metoprolol, which is a beta blocker (Prophylaxis for arrhythmias after MI: Beta blocker).
Reference: Lippincott Illustrated Reviews Pharmacology.

104.According to the modified criteria of rheumatic fever, which is considered as a minor criteria?
A. Carditis
B. Fever
C. Arthritis

Answer: B



105.(long scenario) case of endocarditis with negative bacterial culture, there is diastolic murmur radiate to the left axilla. What is
the most likely diagnosis?
A. SLE
B. Rheumatoid arthritis.
C. Acute myocarditis.

Answer: A
Libman-Sacks endocarditis (otherwise known as verrucous, marantic, or nonbacterial thrombotic endocarditis) is the most characteristic
cardiac manifestation of the autoimmune disease systemic lupus erythematosus.
One cohort study reported that pure mitral regurgitation was the most common valvular abnormality, followed by aortic regurgitation,
combined mitral stenosis and regurgitation, and combined aortic stenosis and regurgitation.
Reference: http://emedicine.medscape.com/article/155230-overview#showall
Blood culture-negative infective endocarditis (IE):
Http://www.uptodate.com/contents/epidemiology-microbiology-and-diagnosis-of-culture-negative-endocarditis

106.A male patient known case of HTN on ACEI but with poor control, which drug you will add?
A. Thiazide
B. Furosemide
C. Vasodilator

Answer: A

26
107.Hypertensive patient on ACEI but not controlled BLOOD PRESSURE , what to add?
A. Furosemide
B. Thiazide
C. Beta blocker

Answer: B
Step 1 antihypertensive treatment with an angiotensin-converting enzyme (ACE) inhibitor or a low-cost angiotensin-II receptor blocker
(ARB). If diuretic treatment is to be initiated or changed, offer a thiazide-like diuretic, such as chlortalidone. A combination of ACE
inhibitors and diuretics instead of ACE inhibitors alone is recommended for preventing recurrence of stroke based on findings of
(PROGRESS).
Http://www.clevelandclinicmeded.com/medicalpubs/diseasemanagement/nephrology/arterial-hypertension/
Https://www.nice.org.uk/guidance/cg127/chapter/1-guidance?UNLID=8546402482015836347
Http://www.pharmacology2000.com/Cardio/antihyper/antihype.htm





108.Pt with retrosternal chest pain for 3 days increase in the last 24 hrs relieved by sublingual nitro Dx?
A. Unstable angina
B. Pericardidits
C. MI

Answer: ???
The duration of chest pain here is more than 3 days it is possible to be cardiac cause but it is unlikely. Suggestion of Non
Cardiac cause eg. Esophageal spasm that will relieved by nitro.
Http://www.health.harvard.edu/heart-health/chest-pain-a-heart-attack-or-something-else
Http://emedicine.medscape.com/article/159383-clinical
Http://emedicine.medscape.com/article/174975-clinical
Http://patient.info/health/heart-attack-myocardial-infarction-leaflet
Http://www.merckmanuals.com/professional/cardiovascular-disorders/coronary-artery-disease/acute-coronary-syndromes-acs

109.Tx of acute myocrditis?!


A. Ab
B. Immunoglobulin
C. Steroids
Answer:

27
th
Reference: short text book of medical diagnosis & management , danish 11 edition pg127


110.Down fix spilt s2 +harsh systolic murmur + biventricular hypertrophy +.... Dx?
A. ASD
B. VSD
C. Atrioventrical septal defect



111.Academic professor present with headache sometimes resolve with 2 tablets of panadol. His Blood pressure is 170/100, what
is the type of his htn?
A. Essential
B. Secondary
C. Pain causing
Answer: a
Primary or essential hypertension accounts for 90-95% of adult cases, and a small percentage of patients (2-10%) have a secondary
cause.
Source: http://emedicine.medscape.com/article/241381-overview#a4


112.What is the best to dx coarctation of aorta?
A. Cardiac mri
B. Echo

Answer: ??
No specific laboratory tests are necessary for coarctation of the aorta. Imaging studies that may be helpful include the following: chest
radiography (findings vary with the clinical presentation of the patient), barium esophagography (classic “e sign,” representing
compression from the dilated left subclavian artery and poststenotic dilatation of the descending aorta) echocardiography (2-dimensional
echocardiography, pulsed-wave doppler, and color flow mapping). In another reference once suspected, an echocardiogram is the
most commonly used test to confirm the diagnosis. It is more accurate. Cardiac catheterization helps to confirm the diagnosis when
echocardiography findings are not completely clear.
References: http://www.cdc.gov/ncbddd/heartdefects/coarctationofaorta.html
Http://emedicine.medscape.com/article/150369-overview


113.Young pt with HTN discrepancy, what is the tx?
A. Trans-aortic stenting
B. Thiazides

28

Answer: b


114.Patient with acute rheumatic fever show acute cardiac symptoms, what is the treatment?
A. IV penicillin
B. Im steroid

Answer: b
Treatment of arf consists of anti-inflammatory therapy, antibiotic therapy, and heart failure management.
Patients with severe carditis (significant cardiomegaly, congestive heart failure, and/or third-degree heart block) should be treated with
conventional therapy for heart failure.
Corticosteroids should be reserved for the treatment of severe carditis.
Reference: http://emedicine.medscape.com/article/333103-medication#4

115.Ischemic heart patient with lab result of high lipid. What is the next order?
A. TFT
B. LFT

Answer: b
It is recommend treating all patients with cvd with high-dose statin therapy.
In 2012, the us food and drug administration revised its labeling information on statins to only recommend liver function testing prior
to initiation of statin therapy and to only repeat such testing for clinical indications.
Reference: uptodate

116.An ecg with st elevation in v1-v5:


A. Extensive anterior mi
B. High lateral mi

Answer: anterior or anteroseptal mi


In extensive mi there has to be st elevation of 6 or more leads.
Extensive anterior mi includes st elevation in anterior leads (v3, v4) + septal leads (v1, v2), and lateral leads (i, avl, v5, v6).
Reference: toronto notes



117.What is the commonest cause of HTN in adolescents:
A. Idiopathic
29
B. Renal

Answer: a
Essential HTN applies to more than 95% of cases of htn.
Renal artery stenosis (most common cause of secondary htn), birth control pills are the most common secondary cause of HTN in
young women.
Reference: step-up of medicine


118.HTN patient with decrease gfr ;
A. Bilateral renal artery stenosis
B. DM nephropathy
Answer:


119.A case of myocardial infarction .. What is the complications if he didn't managed after six hour?
A. Myocardial rupture
B. Arrhythmias
Answer: b!
Pericarditis:
post infarction pericarditis usually begins several days after the infarct, due to an inflammatory exudate in the pericardium.
Ventricular septal rupture:
acute ventricular septal rupture can occur usually several days following the acute infarction, due to softening of the necrotic
portion of the septum.
Complications may occur due to ischemic or injured tissue and therefore may begin within 20 minutes of the onset of m.i., when
myocardial tissue injury begins. These complications include arrhythmias and heart block (due to injured or ischemic conduction
system tissue), and hypotension and congestive heart failure (due to ischemic or injured muscle tissue, resulting in abnormal filling
{"diastolic dysfunction"} or abnormal emptying {"systolic dysfunction"}).
Referance:
http://www.brown.edu/courses/bio_281-cardio/cardio/handout4.htm

120.Traveling women 18hr in flight developed ll edema what will you do:
A. Compression ultrasonography
B. Angiography

Answer: a
According to bmj best practice long flight is strong risk factor and lower limb swelling is a common symptom of dvt." first-line test in
all high-probability patients (wells' score of 2 or more) or in low-probability patients (wells' score <2) with an elevated d-dimer level is
venous duplex ultrasound (dus)."
Reference: http://bestpractice.bmj.com/best-practice/monograph/70/diagnosis/tests.html


121.Mi patient presented to er after resuscitation he developed coma and then died, what postmortem change you will find
A. Inter ventricular hemorrhage
B. Brown colored area supplied by middle meningeal artery
Answer: b
Reference: https://annalsofintensivecare.springeropen.com/articles/10.1186/2110-5820-1-45

30

122.In patients with an ostium seccundum defect, electrocardiogram (ecg) results usually demonstrate the following:
A. Left -axis deviation
B. Right ventricular hypertrophy
Answer b
Atrial septal defect (asd) is one of the most common congenital cardiac abnormalities identified in adults. There are 3 types of asd:
ostium secundum, ostium primum, and sinus venosus. In patients with an ostium secundum defect, electrocardiogram (ecg) results
usually demonstrate the following:
• Right-axis deviation
• Right ventricular hypertrophy
• Rsr' pattern in the right precordial leads with a normal qrs duration
Http://emedicine.medscape.com/article/348121-overview

123.A patient with left bundle branch block will go for dental procedure, regarding endocarditis prophylaxis:
A. Amoxicillin before procedure
B. No need
Answer: b
antibiotic prophylaxis is indicated for the following high-risk cardiac conditions: prosthetic cardiac valve, history of infective
endocarditis, congenital heart disease, cardiac transplantation recipients with cardiac valvular disease.
Http://emedicine.medscape.com/article/1672902-overview#a2

124.Patient with mi, blood pressure 80/65 ecg shows bradycardia (i think complete heart block) what is your action.
A. Dopamin/ norepinephrin
B. IV fluid bolus / percutaneous pacemaker
Answer: b in case of severe bradycardia or hemodynamic unstably: atropine (if inf. Mi) and placement of pacemaker
step up to medicine
http://www.clevelandclinicmeded.com/medicalpubs/diseasemanagement/cardiology/complications-of-acute-myocardial-infarction/
31
125.Which of the following is major criteria of infective endocarditis?
A. Positive blood culture
B. Echo shows valvular …
Answer: b we need positive blood cultures from 2 separate blood cultures or 2 positive cultures from samples drawn > 12 hrs.
Http://reference.medscape.com/calculator/endocarditis-diagnostic-criteria-duke

126.A patient complains of chest pain that radiates to left shoulder and jaw what’s best analgesics?
A. All answers are painkillers medications
B. Morphine
Answer: b
Morphine should be given for the relief of chest pain or anxiety. This also decrease work demand on cardiac muscle. The use of other
analgesic agents, such as nonsteroidal anti-inflammatory drugs (NSAIDs) should be avoided if at all possible, as the use of these agents
has been associated with adverse cardiovascular events.
Http://emedicine.medscape.com/article/155919-treatment#showall
Http://cursoenarm.net/uptodate/contents/mobipreview.htm?39/40/40576#h6

127.Patient post delivery by one week presented with orthopnea and increase jvp symptoms of heart failure the x-ray will show:
A. Bilateral infiltration
B. Thoracocardiac ratio <50%
Answer: not complete options.
Chest x ray heart failure findings include: pleural effusions, cardiomegaly (enlargement of the cardiac silhouette cardio-thoracic ratio
> 50%), kerley b lines (horizontal lines in the periphery of the lower posterior lung fields), upper lobe pulmonary venous congestion
and interstitial edema.

128.Most common cause of 2dry hypertension


A. Kidneys disease
B. Cons syndrome
Answer: a
Link: http://www.aafp.org/afp/2010/1215/p1471.html

129.Digoxin antidote
A. Lidocaine
B. Immune fab
Answer: b
In the case of severe digoxin intoxication, an antidote digoxin immune fab (digibind) is available. Digibind binds and inactivates digoxin.

130.Indication of defibrillator (pace maker) is injury in?


A. Sa node
B. Av node
This question is incorrect since defibrillators and pacemakers are different devices and have totally different indications
Defibrillators:
Electrical cardioversion and defibrillation have become routine procedures in the management of patients with cardiac arrhythmias.
Cardioversion is the delivery of energy that is synchronized to the qrs complex, while defibrillation is the nonsynchronized delivery of
a shock randomly during the cardiac cycle.
Most defibrillators are energy-based, meaning that the devices charge a capacitor to a selected voltage and then deliver a prespecified
amount of energy in joules. The amount of energy which arrives at the myocardium is dependent upon the selected voltage and the
transthoracic impedance (which varies by patient).

Pacemakers
Cardiac pacemakers are effective treatments for a variety of bradyarrhythmias. By providing an appropriate heart rate and heart rate
response, cardiac pacing can reestablish effective circulation and more normal hemodynamics that are compromised by a slow heart
rate.

32
131.Most common risk factor for cad:
A. Smoker and 50 y m
B. Dyslipidemia and obesity
Answer: b
Link:http://www.heart.org/heartorg/conditions/more/myheartandstrokenews/coronary-artery-disease---coronary-heart-
disease_ucm_436416_article.jsp#.v2fir8dbfq0

132.50 years old patient with third reading of persistent hypertension wasn't started on medication yet, lab shows high na of 147
and low k of 3 other parameters were normal what's most likely the diagnosis?
A. Essential hypertension
B. Hyperaldosteronism

Answer: b

Liddle syndrome is a rare autosomal dominant disorder characterized by severe hypertension and hypokalemia. Liddle syndrome is
caused by unrestrained sodium reabsorption in the distal nephron due to mutations found in genes encoding for epithelial sodium channel
subunits. Inappropriately high reabsorption of sodium results in both hypertension and renal potassium wasting.
Reference: toronto note, merck manual.

133.Patient with cad angio done for him showed left coronary artery 90% stenosis and the right is 40%, what is the management
A. Left coronary bypass
B. Bypass all vessel
C. Stent
answer: a
Check the reference for the classification and indication for cabg done by the american college of cardiology (acc) and the american
heart association (aha)
reference: http://emedicine.medscape.com/article/164682-overview#a2

134.Patient HTN and hyperlipidemia with chest pain and when he is coming to hospital take drug relieve his symptoms which
drug:
A. Nitric oxide
B. Digoxin
Answer: a.
Nitroglycerin is a nitrate. It works by relaxing blood vessels which allows blood to flow more easily. This reduces the heart's workload
and the amount of oxygen needed by the heart.

135.Patient with chest pain for 6 hours what to give?:


A. Tpa
B. Aspirin
Answer:b.
Management:
o Immediate assessment with cardiac monitoring
o Early therapy (e.g., intravenous access, oxygen, aspirin, nitroglycerin, morphine, unfractionated heparin or low-molecular-weight
heparin [lmwh], nitrates, beta-blockers, p2y12 inhibitors, glycoprotein iib/iia receptor antagonist)
o Initial management according to the american heart association advanced cardiac life support guideline for those with
compromised vital signs
o Percutaneous coronary intervention or coronary artery bypass graft if indicated
o Thrombolytic
Https://www.guideline.gov/content.aspx?Id=39320

136.Female with leg pain when she walk 300 m. Relief by rest:
A. Claudication
B. Dvt
Answer: a
33
Reference: toronto notes
claudication
• Pain with exertion: usually in calves or any exercising muscle group relieved by short rest: 2-5 min, and no postural
changes necessary reproducible: same distance to elicit pain, same location of pain, same amount of rest to relieve pain


137.What can cause Coarctation of aorta?
A. Coronary artery disease
B. Aortic dissection

Answer: B
Complications of untreated patient include: HTN, stroke, aortic aneurysm, aortic dissection, premature coronary artery disease, HF,
brain aneurysm or hemorrhage. The most common complication in adult is HTN and CAD.

138.A patient with renal function test abnormalities. Tests show beads on string appearance. What is the diagnosis?
A. Renal artery disease
B. Fibromuscular dysplasia

Answer: B
The string-of-beads appearance is considered pathognomonic for medial fibroplasia on diagnostic angiography.
Reference: http://emedicine.medscape.com/article/417771-overview#a

139.A Patient had an MI and was treated for that, after that he developed chest pain that worsen with movement and taking
deep breath. On examination there was distant heart sounds and pericardial rub. What is the most ECG changes associated
with this condition?
A. ST changes
B. PR prolongation

Answer: A
A case of pericarditis.
ECG: initially diffuse elevated ST segments ± depressed and shorten PR segment, the elevation in the ST segment is concave upwards
>> 2-5 d later ST isoelectric with T wave flattening and inversion.
Reference: Toronto Notes.

140.A male patient came to the ER complaining of palpitations, tachycardia ... ECG shows
deep S wave in lead (?) And tall R wave in lead (?) , Dx ?
A. LBBB
B. RBBB

Answer:?
LBBB:
QRS duration >120 msec
V1 and V2: W pattern and wide deep slurred S wave
V5 and V6: wide QRS complex with M pattern or rabbit ear pattern

RBBB: the opposite of LBBB


QRS duration >120 msec
V1 and V2: wide QRS ( more than 3 small squares) with ear rabbit pattern or M shape pattern
V5 and V6: wide and deep/slurred S

LVH:
S inv1 + R in V5 or V6 > 35mm above age 40, (>40 mm for age 31-40, > 45 mm for age 21-30)
R in avl >11mm
R in I + S in III >25mm

RVH:
Right axis deviation
34
R/S ratio > 1or qr in lead V 1
RV strain pattern: ST segment depression and T wave inversion in leads V1-2
Reference: Toronto Notes

141.What is the commonest cause of HTN in adolescents?


A. Idiopathic
B. Renal

Answer:A
Secondary hypertension is more common in preadolescent children, with most cases caused by renal disease. Primary or essential
hypertension is more common in adolescents and has multiple risk factors, including obesity and a family history of hypertension.
Http://emedicine.medscape.com/article/889877-overview#a4
Http://www.aafp.org/afp/2006/0501/p1558.html

142.A patient comes to you for routine check up, his dad died of hear disease aged 66 what is the best screening test:
A. Lipid profile
B. Ecg
Answer: a http://www.lifelinescreening.com/what-we-do/what-we-screen-for/heart-disease

143.60 Year-old has only HTN the best drug for him?!!
A. Diuretics
B. CCB
Answer: i think the answer is diuretics!
Uptodate:-in general, three classes of drugs are considered first-line therapy for the treatment of hypertension in elderly patients: low-
dose thiazide diuretics (eg, 12.5 to 25 mg/day of chlorthalidone), long-acting calcium channel blockers (most often dihydropyridines),
and ace inhibitors or angiotensin ii receptor blockers. A long-acting dihydropyridine or a thiazide diuretic is generally preferred in
elderly patients because of increased efficacy in blood pressure lowering.
Among elderly patients in whom there is a reasonable likelihood of requiring a second drug (eg, more than 10/5 mmHg above goal),
some physicians who practice according to the results of the accomplish trial would prefer initial therapy with a long-acting
dihydropyridine calcium channel blocker. This is because, if additional therapy is required, a long-acting ace inhibitor/arb can be
added to achieve the desired combination regimen.
Medscape: on the basis of Blood pressure -lowering efficacy and outcomes data, CCB s are acceptable alternatives to diuretics for
first-line treatment of hypertension in the elderly and may offer advantages in some patient groups, eg, those with the metabolic
syndrome.

American family physician:
Low-dose thiazide diuretics remain first-line therapy for older patients. Beta blockers, angiotensin-converting enzyme inhibitors,
angiotensin-receptor blockers, and calcium channel blockers are second-line medications that should be selected based on
comorbidities and risk factors.

Refrence: http://emedicine.medscape.com/article/241381-treatment
Http://www.aafp.org/afp/2005/0201/p469.html

144.What is the initial management of hypotension induced lumber puncture ?


A. IV fluid
B. Bed rest
Answer:?


145.Early detection of st segment elevation mi :
A. Typical chest pain with … some ecg changes
B. Atypical chest pain with some ecg changes
35
Answer: a (question not clear, and choices incomplete)
Explanation: presents with central chest pain that is classically heavy in nature, like a sensation of pressure or squeezing. St-elevation
mi (stemi) is suspected when a patient presents with persistent st-segment elevation in 2 or more anatomically contiguous ecg leads in
the context of a consistent clinical history. Cardiac enzymes confirm diagnosis. Treatment should, however, be started immediately in
patients with a typical history and ecg changes, without waiting for laboratory results.
Reference: http://bestpractice.bmj.com/best-practice/monograph/150/highlights/summary.html


146.Ecg showed wide complex tachycardia, jvp: a cannon wave, diagnosis?
A. Vt
Answer: a
This patient has ventricular tachycardia based on the presence of a wide complex tachycardia and cannon "a" waves in the jugular veins.
Cannon "a" waves are due to the unsynchronized contraction of the ventricles and the atria. This results in a retrograde flow of blood
back to the jugular veins with atrial systole. The variation of the intensity of s1 is caused by the ventricle contracting at times when the
av valves are open and at other times when they are closed. Procainamide, amiodarone, and lidocaine are the most effective treatments
for a hemodynamically stable patient.

147. Mi patient has big thrombus in left coronary artery what is drug that cause thrombolytic action?
Thrombolytic therapy is indicated in patients with evidence of st-segment elevation mi (stemi) or presumably new left bundle-branch
block (lbbb) presenting within 12 hours of the onset of symptoms if there are no contraindications to fibrinolysis.
Http://emedicine.medscape.com/article/811234-overview#a3


148.Patient presented with chest pain was diagnosed with stem. What is the other finding you can detect on his ecg?
Answer: inverted t wave
Other ecg findings: reciprocal st segment depression, q wave indicating old infarct.
Source: step-up to medicine

149.A patient presented with inferior mi. Which artery is affected?


Answer: right coronary artery (rca)
Inferior mi present in ecg with st elevation in ii, iii and avf. Rca supply the right atrium and ventricle, which lie on the diaphragm and
constitute the inferior wall of the heart muscle.

36
150.Most common cause HTN in adult?
Hypertension may be primary, which may develop as a result of environmental or genetic causes, or secondary, which has multiple
etiologies, including renal, vascular, and endocrine causes. Primary or essential hypertension accounts for 90-95% of adult cases, and
secondary hypertension accounts for 2-10% of cases.
Http://emedicine.medscape.com/article/241381-overview


151.Mitral stenosis ecg:
A. Bifid p wave left axis deviation
B. Elevated p wave right axis deviation

Answer ; ?
In mitral stenosis the ecg is often normal. Pathological changes in the ecg include:
• Left atrial hypertrophy:
o Causes bifid p wave
o Increased voltage in the later part of the p wave gives it a large, bifid appearance in leads ii, iii and avf
o The second half of the p wave is negative in v1
o Atrial fibrillation is present in 60 to 70%
• Right ventricular hypertrophy:
o A dominant r wave in v1 and v2 indicates pulmonary hypertension
• Right axis deviation
• There may be digitalis effects

152.Heart sound that supports pulmonary htn.


o Loud pulmonic component to the second heart sound.
o A right ventricular fourth heart sound the murmur of tricuspid regurgitation, which is holosystolic, located at the le lower
sternal border, and augments with inspiration, is common in patients with moderate to severe pulmonary hypertension.
o Other findings on auscultation may include an early systolic click and the murmur of pulmonic regurgitation. A right
ventricular third heart sound o en signifies advanced disease and right heart failure.

153.A man lost his consciousness during playing football i.e. Syncopal attack. He had history of aortic stenosis. How can you explain
this case: (no low cardiac output not mentioned in the choices!)
A. Cardiac arrhythmia.
Answer: a might be right because aortic stenosis is common cause of left bundle branch block

154.Which of the following is a thrombolytic?


Answer: streptokinase
Streptokinase (sk) is an enzyme secreted by several species of streptococci that can bind and activate human plasminogen. Sk is used
as an effective and inexpensive thrombolysis medication in some cases of myocardial infarction (heart attack) and pulmonary
embolism.
Belongs to a group of medications known as fibrinolytics.
Https://en.m.wikipedia.org/wiki/streptokinase

37


155.Patient come with pericardial pain, ecg st segment elevation, patient given aspirin and nitrate, but no relieve of pain what
next step you will do?
Answer: give morphine iv, refractory or severe pain should be treated symptomatically with IV morphine.
Http://emedicine.medscape.com/article/155919-treatment#d6

156.Child with episodic of cyanotic lip with cold extremities, what is the diagnostic investigation: echocardiography to prevent
cyanosis in chd?
A. Prostaglndin e

157.St elevation at lead 2, 3, and avf , that represent which cardiac parts? Inferior mi

158.?? Year old male came with history of leg pain after walking, improved after resting, he notices loss of hair in the shaft of his
leg and become shiny?
A. Chronic limb ischemia

159. Acs which enzyme to order after 1 hour

Cardiac markers
Marker Initial elevation Peak elevation Return to baseline
Myoglobin 1-4 h 6-7 h 18-24 h

Ck-mb 4-12 h 10-24 h 48-72 h


Cardiac trop i 3-12 h 10-24 h 3-10 d
Cardiac trop t 3-12 h 12-48 h 5-14

160.Ischemic heart disease first test to diagnose?

bmjhttp://bestpractice.bmj.com/best-practice/monograph/152.html

38
161.Atrial septal defect changes on ecg:
§ Not diagnostic: normal; or tall p waves, large r waves in v1, crochetage pattern in inferior limb leads. Bmj
http://bestpractice.bmj.com/best-practice/monograph/1099/diagnosis/tests.html

162. Asd: . Clinical features mcq.


o Mild systolic ejection murmur at pulmonary area secondary to increased pulmonary blood flow
o Fixed split s2 ( important)
o Diastolic flow “rumble” murmur across tricuspid valve area secondary to increased blood flow
o In advanced disease, signs of rvf may be seen
Step up to medicine

163.Signs of chronic stable angina?


Chest pain or substernal pressure sensation that lasts less than 10 to 15 minute, gradual in onset, brought on by exertion or
emotion, relieved with rest or nitroglycerin, pain does not change with breathing nor with body position. Stepuptomedicine

164.Which one is criteria of TOF?


Pulmonary outflow stenosis

Tetralogy of fallot involves four heart defects:


- A large ventricular septal defect (vsd)
- Pulmonary stenosis
- Right ventricular hypertrophy
- An overriding aorta

165.Patient present with typical symptoms of angina relieve by using sublingual drug what is the mechanism of action of this
drug?
Answer: relax smooth muscle and dilation of vein and artery and decrease resistance.
Http://www.uptodate.com/contents/nitrates-in-the-management-of-stable-angina-pectoris#h2

166.Description of congenital heart disease there is decrease vascularization of the lungs on x-ray, cardiac shadow (they
described boot shaped but not in a direct way)
A. Teratology of fallot was my answer
Answer: a
Link: http://radiopaedia.org/articles/boot-shaped-heart-1

167.Old woman has ht it’s not controlled even with multi drugs. She sleeps afternoon a lot and feels fatigue most of the time,
what is the cause of her resistance ht?
A. Obstructive sleep apnea
Answer: a
Severe obstructive sleep apnea may interfere with blood pressure (Blood pressure )–lowering treatment in patients at high
cardiovascular disease risk or with established cardiovascular disease, results of a multicenter clinical trial suggest.
Link: http://www.medscape.com/viewarticle/832716

168.Patient has history of endocarditis and underwent to extract his teeth the doctor what will do before extraction:
Answer: 2 g amoxicillin before or 2 g clindamycin before

169.Treatment of acute myocarditis?

39
Treatment of myocarditis includes supportive therapy for symptoms of acute heart failure with use of diuretics,
nitroglycerin/nitroprusside, and angiotensin-converting enzyme (ace) inhibitors. Inotropic drugs (eg, dobutamine, milrinone) may be
necessary for severe decompensation, although they are highly arrhythmogenic.


170.Patient with moderate mitral stenosis, syncope, what is the cause of syncope?
Most likely is by causing afib


171.2 yo ejection systolic murmur cause?
Http://emedicine.medscape.com/article/1894036-overview#a8

172.Cause of delayed arterial radiofemoral pulse:


Answer: coarctation of aorta

173.Heart defect result from malrotation and truncus arteriosus


A. Transposition of great arteries
Answer: a

174.Management of the acute attack


A single dose of benzyl penicillin (1.2 million u im) or oral phenoxymethylpenicillin (250 mg 4 times daily for 10 days) should be
given on diagnosis to eliminate any residual streptococcal infection. If the patient is penicillin-allergic, erythromycin or a
cephalosporin can be used. Treatment is then directed towards limiting cardiac damage and relieving symptoms.

Reference: davidsone

175.Patient elderly k/c of DM and HTN with hx of previous tia and ecg showed atrial fibrillation controlled on digoxin, what’s next
for him?
A. Anticoagulate with warfarin.
Answer: a

176.Most common cause of HTN in saudi arabia.


Answer: essential htn.

177.Pt with decreased lower limbs pulses and intercostal notching in x-ray. What is the diagnosis?
A. Coarctation of aorta
Answer: a
Coarctation of the aorta
• HTN in upper extremities with hypotension in lower extremities
• Cxr
• Notching of the ribs
• “figure 3” appearance due to indentation of the aorta at site of coarctation, with dilation before and after the stenosis
Reference: step up to medicine

178.69 Year-old female on antiarrhythmic and she developed hyperpigmentation. Which drug cause that?
Answer: amiodarone
1. Amiodarone has been used both in the treatment of acute life-threatening arrhythmias as well as the chronic suppression
of arrhythmias. Long-term administration of amiodarone (usually more than eighteen months) is associated with a light-
sensitive blue-grey discoloration of the skin; such patients should avoid exposure to the sun and use sunscreen that
40
protects against ultraviolet-a and -b. The discoloration will slowly improve upon cessation of the drug, however, the skin
colour may not return completely

Reference: wikipedia: https://en.wikipedia.org/wiki/amiodarone

179.Lipid profile of a patient shows high level and patient is on simvastatin, what to add?
Answer:
Reference: davidsons 22

180.Mi patient within 6 hours what is the most expected complication? :


Answer: arrhythmias mostly vf

Complication of mi:

o Electrical dysfunction (arrhythmias) the commonly causes mortality in the first 72 h


o Mechanical dysfunction (heart failure, myocardial rupture or aneurysm, papillary muscle dysfunction)
o Thrombotic complications (recurrent coronary ischemia, mural thrombosis)
o Inflammatory complications (pericarditis, dressler syndrome)

Reference: merck manual

181.HTN patient develops gout, which drug side effect


Answer: thiazide
Hyperuricemia is a common finding in patient treated with thiazide due to decrease renal execration of uric acid and may lead to gouty
arthritis
Reference: uptodate

182.Patient had mi, he was given sublingual nitrate and analgesia, the moa of the analgesic that was given ?
Answer: morphine bind full agonist to mu receptor
Reference: rxlist.

183.Scenario of a patient with chest pain that relieved when lying down and increase when lying forward with chest x-ray show
the heart (globular like a ball) what is the dx?
A. Pericardial effusion
Answer: a
41


184.Drug causing malignant syndrome.

185.This is a classical presentation of aortic stenosis, high pitched, crescendo-decrescendo, midsystolic murmur located at the
aortic area
Http://www.healio.com/cardiology/learn-the-heart/cardiology-review/heart-murmurs

186.SVT scenario what most important to test?


A. TSH
Answer
Electrolyte levels - should be checked because electrolyte abnormalities can contribute to paroxysmal supraventricular tachycardia
(paroxysmal svt)
CBC help to assess anemia
TSH to assess hyperthyroidism



187.Ecg showing ventricular tachycardia the patient is unstable what is the management?
Answer:
Hemodynamic compromise: electrical cardioversion.
No hemodynamic compromise: electrical cardioversion, lidocaine, amiodarone, type ia agents (procainamide, quinidine).
Source: toronto notes 2014 c20


188.Side effect of nitroglycerin (ttt of stable angina ) :
A. Headache
Common se of the drug are severe headache, severe hypotension, and, in certain cases, and bradycardia.
Https://en.wikipedia.org/wiki/nitroglycerin_(drug)#adverse_effects

189.Patient came with history of heart failure & pulmonary edema. Treatment?
A. Diuretics.
Answer: a
Acute treatment of pulmonary edema.
Treat acute precipitating factors (e.g. Ischemia, arrhythmias). Mnemonic: lmnop.
o L: lasix® (furosemide) 40-500 mg IV
o M: morphine 2-4 mg iv: decreases anxiety and preload (vasodilation)
o N: nitroglycerin: topical/iv/sl
o O: oxygen: in hypoxemic patients
o P: positive airway pressure (cpap/bipap): decreases preload and need for ventilation when appropriate
o P: position: sit patient up with legs hanging down unless patient is hypotensive

190.Right dominant coronary artery, branch of it?


A. Marginal
Answer: a. Toronto:
Right coronary artery (rca) branches:
o Acute marginal branches.
o Atrioventricular (av) nodal artery.
o Posterior interventricular artery (piv) = posterior descending artery (pda).

191.Black man with high Blood pressure -, +ve fx h-x of -htn


42
A. CCB
Or thiazide or combination (jnc8).

192.Aspirin in high dose cause hyperthermia through which mechanism:


A. Aspirin and hyperthermia
Uncoupling of oxidative phosphorylation: electron transport continues but without the coupled atp synthesis. Result is heat
generation.
Http://www.acmt.net/ijmt/ijmt/4_5/case1c/tsld009.htm
Salicylate poisoning is manifested clinically by disturbances of several organ systems, including the central nervous system (cns)
and the cardiovascular, pulmonary, hepatic, renal, and metabolic systems. Salicylates directly or indirectly affect most organ
systems in the body by uncoupling oxidative phosphorylation, inhibiting krebs cycle enzymes, and inhibiting amino acid
synthesis.
Http://emedicine.medscape.com/article/1009987-overview#a5

193.Patient use anti-hyperlipidemia, liver enzymes normal, creatin kinase high:


A. Statins
Reference: http://www.medsafe.govt.nz/profs/puarticles/statinmyop.htm

194.27 Year-old female , recurrent palpitations ecg :


A. Supraventricular tachycardia
Http://www.practicalclinicalskills.com/mobile/ekg/supraventricular-tachycardia.aspx

195.Tetralogy of fallot presents with a vsd.


A. Tetralogy of fallot: http://emedicine.medscape.com/article/2035949-overview

196.Case of cardio change in ecg show , inverted t and st change , what is cause ?Ischemia - ( mi - infarction )


197.Cardiac case ..his brother died when walk dx?
A. Cardiomyopathy..

198.Which type of the angina present when go to bed


A. Variant prinzmetal angina.
Vasospasm in diseased or normal artery and during the episode patient will have st elevation not depression.

122- pt with STEMI, what you will find in the ECG?


Answer:
Https://www.heart.org/idc/groups/heart-public/@wcm/@mwa/documents/downloadable/ucm_467056.pdf

199.Cardiac patient on Aspirin, no new complaints. He is having low platelet (less than 10) for the last 6 months. What is your
management?
Answer: ?
The diagnosis is rare and called aspirin induced thrombocytopenia
Stop aspirin and find the cause (Answered by a senior cardiologist consultant)

200.Which of these anti HTN medications decrease afterload and preload?


A. ACEI
Answer: A

201.A patient presented with frothy red sputum, flushed cheeks, etc (long scenario). What is the diagnosis?
43
A. Mitral stenosis

Answer: A
Reference: Master the boards + Medical diagnosis and Management by Mohammad Inam Danish

202.A patient known case of aortic stenosis had syncope. What is the cause?
A. Systemic hypotension

Answer: A
Syncope from aortic stenosis often occurs upon exertion when systemic vasodilatation in the presence of a fixed forward stroke
volume causes the arterial systolic blood pressure to decline. It also may be caused by atrial or ventricular tachyarrhythmias.
Syncope at rest may be due to transient ventricular tachycardia, atrial fibrillation, or (if calcification of the valve extends into the
conduction system) atrioventricular block. Another cause of syncope is abnormal vasodepressor reflexes due to increased LV
intracavitary pressure (vasodepressor syncope).
Reference: http://emedicine.medscape.com/article/150638-clinical

203.The most important risk factor for stroke ?


A. HTN
B. High cholesterol
C. Smoking
D. Age process

Hypertension, is the single most important treatable risk factor for stroke
Reference: uptodate.com

204.Anomaly between aortic arch and pulmonary trunk?


A. Patent ductus arteriosus .
Answer: A

205.X-ray of pericardial effusion


Answer

44
Answer: Plain film: chest radiograph
Globular enlargement of the cardiac shadow giving a water bottle configuration. Lateral CXR may show a vertical opaque line
(pericardial fluid) separating a vertical lucent line directly behind sternum (epicardial fat) anteriorly from a similar lucent vertical
lucent line (pericardial fat) posteriorly; this is known as the Oreo cookie sign. Widening of the subcarinal angle without other
evidence of left atrial enlargement may be an indirect clue.
Echocardiography is the imaging modality of choice for the diagnosis of pericardial effusion.
Http://radiopaedia.org/articles/pericardial-effusion
Http://emedicine.medscape.com/article/157325-workup#c10

206.What’s the acute management in rheumatic fever?


A. IV penicillin
B. Im steroid

Answer:
o Antibiotic treatment for GAS infection eradication: either oral penicillin OR amoxicillin OR single dose intramuscular benzathine
penicillin
o Aspirin for fever, arthritis, and/or MILD carditis: 15 to 25 mg/kg PO QID (to a maximum daily dose of 4 to 6 g) for 2 to 4 weeks
and then taper the dose over another 4 weeks.
o Prednisone for MODERATE/SEVERE carditis: 1 mg/kg po bid (up to 60 mg/day)
o 10 years to life-long prophylaxis to recurrent GAS infection.
READ MORE DETAILS HERE: http://www.merckmanuals.com/professional/pediatrics/miscellaneous-bacterial-infections-in-
infants-and-children/rheumatic-fever#v1092890

207.Infective endocarditis (migratory arthritis)

Answer:
o Duke criteria for diagnosis of infective endocarditis
o The presence of 2 major OR 1 major + 3 minor OR 5 minor is DIAGNOSTIC.

45
208.Wide complex ECG

Answer: see picture

209.Enzyme that get elevated in MI?


A. Creatine phosphokinase (Note: no troponin I in the options)
Answer: A. Creatine Kinase-MB
• Serial measurements of CK-MB isoenzyme levels were previously the standard criterion for the diagnosis of myocardial
infarction. However, sensitivity and specificity are not as high as they are for troponin levels. The trend has favored using

46
troponins and they are now considered to be the criterion standard for defining and diagnosing myocardial infarction,
according to the American College of Cardiology.
Reference: http://emedicine.medscape.com/article/155919-workup#c9

47



Pulmonology

48
1. Calcified lesion in the upper lung?
A. Bronchoscopy biopsy
B. Percutaneous biopsy
C. Thoracotomy
D. Follow up with serial x-ray
Answer: d
In asymptomatic pts with calcified nodule, we follow the pt with serial x ray for about 3 months, if it is not changed without sx
development, the calcification is mostly benign

2. A 40 year old patient has right upper lobe nodule, no symptom, other information, what will you do?
A. Transcutaneous biopsy
B. Bronchoscopy
C. Thoracotomy
D. Reassurance

Http://www.ncbi.nlm.nih.gov/pmc/articles/pmc3507065/

3. Lung tumor, size: 3.5 cm, enlarged ipsilateral bronchopulmonary ln, what is the tnm stage?(med)
A. T1 n0 m0
B. T2 n1 m0
C. T3 n2 m0
D. T4 n3 m0

Answer: b
Lung cancer staging
0 1 2 3 4

T No evidence of 3 cm or less 3-7 cm 7 cm or more A tumor of any


primary tumor size that
invades
adjacent
structures

49
N No regional Ipsilateral Ipsilateral Contralateral
lymph node peribronchial mediastinal mediastinal
and\or hilar and \or Contralateral
lymph node subcarinal hilar
and Ipsi or
intrapulmonar contralateral
y node scalene or
supracavicular

M No distant Distant
metastasis metastasis



Reference:http://cancerstaging.org/references-tools/quickreferences/documents/lungmedium.pdf


4. Asthma medication question. Patient on saba prn, symptoms not controlled. What to add?:
A. Inhaled corticosteroid
B. Long acting beta agonist
C. Iatropium bromide
D. Steorid oral

Answer: a
Reference:
Http://www.nhlbi.nih.gov/health-pro/guidelines/current/asthma-guidelines/quick-reference-html

5. High pco2, low ph ;


A. Metabolic acidosis
B. Metabolic alkalosis
C. Respiratory acidosis
D. Respiratory alkalosis
Answer: c
Http://emedicine.medscape.com/article/301574-workup#c8


6. Patient admitted to you with diarrhea and dry cough after that he presented with tachypnea and during p/e there was
rhonchi on the base of the lung cxr showing opacification hyperdense area on the lower parts of the lung , so what is the
most likely organism ?
A. Pasteurella multocida
B. Mannheimia haemolytica
C. Staphylococcus aureus
D. Legionella pneumophila

Answer: d
S.aureus pneumonia presents with multiple nodular infiltrates, and bilateral pleural effusions.
Both b&c should have history of animal contact
Diarrhea is more with legionella pneumophila (legionnaires' disease)
Https://www.nlm.nih.gov/medlineplus/ency/article/000616.htm


7. Which of the following lung cancer contraindication of doing surgery ?
A. Small cell carcinoma
B. Squamous cell carcinoma
C. Adenocarcinoma
D. Fibroma related to something(ican not remember) i think this is the answer*
50
Answer:

8. In acute pe with no signs of dvt; what will you give?


A. Im warfarin
B. Im heparin
C. Warfarin + heparin
D. Streptomycin
◌answer
ِ :c

9. Upper limb edema, intercostal vein engorgement, lesion in right lung, compression in which side?
A. Ant mediastinum
B. Post mediastinum
C. Rt hilum
D. Median mediastinum

Answer: d
(d) .. The tumor in the right lung is compressing the svc in the middle mediastinum causing these symptoms.

(a) .. Would obstruct only a small portion of the thoracic wall drainage and would not produce the neck and upper limb effects.

(b) .. Obstruction of the azygos vein would not influence the neck and upper limb drainage because of collateral connections with
the anterior intercostal veins and would not produce the neck and upper limb effects.

(c) .. Compression of the hilum would obstruct pulmonary functions would not produce the neck and upper limb effects.

Reference: lippincott. Uptodate




10. Long scenario about tb and what is the antibiotic to start:
A. Rifampicin, inh, ethambutol and pyrazinamide
B. Rifampicin, inh and ethambutol
C. Rifampicin and inh
D. Inh

Answer: a
A. For initial empiric treatment of tb, start patients on a 4-drug regimen: isoniazid, rifampin, pyrazinamide, and either
ethambutol or streptomycin.
Reference: medscape: http://emedicine.medscape.com/article/230802-treatment

51
11. A middle-aged man presents with a cough and fever lasting several weeks. Posteroanterior chest radiograph shows a
prominent paratracheal area on the right, lymphadenopathy, a cavitary opacity in the right upper lobe, and a focal
consolidation in the middle lung zone on the right. Cxr shown below. What is the dx?
A. Copd
B. Ba
C. Pneumonia
D. Tb

Answer: d
• Common findings include segmental or lobar airspace consolidation, ipsilateral hilar and mediastinal lymphadenopathy,
and/or pleural effusion. Atelectasis may occur in primary pulmonary tuberculosis, often as a consequence of tuberculous
airway involvement.
Reference: medscape: http://emedicine.medscape.com/article/358610-overview#a2


12. Which lobe is most commonly affected in lobar pneumonia?
A. Right mid
B. Right upper
C. Right lower
D. Left upper
Answer: c
Reference: https://goo.gl/ciwlo8

13. Which of the following decrease the recurrence of asthma?


A. Salbutamol
B. Aminophylline
C. Ipratropium
D. Montilukust
Answer d?
Leukotriene receptor antagonists are one of the controller medications for asthma

14. Asthmatic with 3/week of frequency on short acting and last sever attack was 3 months ago. What the appropriate
management?
A. Short
B. Long
C. Ipratropim
D. Dexamethas

Answer: d

52
15. In emphysema, which part is affected in alfa one antitrypsin deficiency?
A. Interstitial
B. Centroacinar
C. Peripheral
D. Subseptal or something like this
Answer:
Generally, pulmonary emphysema is classified into three types related to the lobular anatomy: centrilobular emphysema, panlobular
emphysema, and paraseptal emphysema. Panlobular (panacinar) emphysema: seen in patients with α1-antitrypsin deficiency.
Destruction involves both proximal and distal acini with predilection for lung bases. *step up to medicine.

16. Smoker + hemoptysis what to do (not specified first or best)?


A. Chest x-ray
B. Chest ct
C. Ppd
D. Coagulation profile
Answer: a
Dd for hemoptysis & tobacco use: acute bronchitis, chronic bronchitis, lung cancer & pneumonia. After a careful history and
examination, a chest radiograph should be obtained. If a diagnosis remains unclear, further imaging with chest computed tomography
(ct) or direct visualization with bronchoscopy often is indicated.
Http://www.aafp.org/afp/2005/1001/p1253.html

17. 40 years old present with 1.5 cm lung nodule what is the most useful to do?
A. Transthoracic biopsy.
B. Thoracic biopsy.
C. Evaluate by multiple x-ray .
D. Look at prior x rays.
Answer: d. Look at prior x-rays. Diagnostic evaluation of a lung nodule includes : cinical features and radiographic features: assess
size – larger lesions are more likely to be malignant than smaller lesions ,border, calcification, density ,and growth – review of
available prior imaging studies is a critical part of the diagnostic evaluation. Uptodate: diagnostic evaluation and management of the
solitary pulmonary nodule.
Https://yhdp.vn/uptodate/contents/mobipreview.htm?43/31/44537

18. Asymptomatic Patient. Chest X ray shows a unilateral calcified nodule on the upper zone of his lung?
A. Adenoma
B. Granuloma
C. Hamartoma
D. SCC

Answer: b
The most common cause of nodule calcification is granuloma formation, usually in the response to healed infection.
Reference: http://radiopaedia.org/articles/calcified-pulmonary-nodules

19. An asthmatic who needs daily short acting beta 2 inhalers, oral steroids and daily spirometry monitoring of pfts. What is his
asthma stage?

53
A. Mild intermittent
B. Mild persistent
C. Moderate
D. Severe

Answer: D
Once the patient on oral steroid he/she classified as a severe asthma.
Reference: http://www.med.umich.edu/1info/FHP/practiceguides/asthma/EPR-3_pocket_guide.pdf

20. Which of the lung tumar can tt by anti epithelium


A. Adeno
B. Small lung
C. Seqoums
D. Carcinoid

80% of lung adenocarcinomas are associated with egfr mutation, in which cetuximab, a monoclonal antibody that binds the egfr
receptor can be used.
Http://emedicine.medscape.com/article/279960-treatment#d12

21. Biopsy of lung found some material in the macrophage ( i cant remebre the material )
A. (pcp )pneocyteic jnera
B. CMV
C. Bacterial
D. Hf

22. Acute onset chest pain + image coronary angio not so clear ..
A. Tpa
B. Pci
C. Heparin
D. The q not clear


23. Most common cause of acute bronchiolitis?
A. Rsv
B. Adeno
C. Parainfluenza

Answer: a
References: http://emedicine.medscape.com/article/961963-overview
Http://www.ncbi.nlm.nih.gov/books/nbk11786/

24. Case of pneumonia, what is the finding on auscultation?


A. Dispred crackles
B. Bronchial breath sound
C. Absence of vesicular breath sound

Answer: b
(short textbook of medical diagnosis and management, inam danish, page 344) (step up to medicine)

On physical examination, approximately 80 percent are febrile, although this finding is frequently absent in older patients and
temperature may be deceptively low in the morning. A respiratory rate above 24 breaths/minute is noted in 45 to 70 percent of patients
and may be the most sensitive sign in older adult patients; tachycardia is also common. Chest examination reveals audible crackles in
most patients, while approximately one-third have evidence of consolidation. However, no clear constellation of symptoms and signs
has been found to accurately predict whether or not the patient has pneumonia.

25. Most common cause of chronic cough in adult?

54
A. Gerd
B. Postnasal drip
C. Asthma

Answer: b
The most common causes of chronic cough are postnasal drip, asthma, and acid reflux from the stomach.
Http://www.uptodate.com/contents/chronic-cough-in-adults-beyond-the
basics?Source=outline_link&view=text&anchor=h3#h3
The most common causes of chronic cough are postnasal drip, asthma, and acid reflux from the stomach. These three causes are
responsible for up to 90 percent of all cases of chronic cough. Less common causes include infections, medications, and lung diseases.


26. Patient with asthma exacerbation. Which drug will decrease the mucous secretion more than bronchodilation?
A. Oral steroids
B. Ipratropium
C. Leukotriene
Answer b
Corticosteroids reduce the mucus secretion by inhibiting the release of secretagogue from macrophages.
Reference: http://www.ncbi.nlm.nih.gov/pubmed/3026210
Anticholinergic agents will dilate bronchi and decrease secretions. They are
Very effective in copd.
Ref : http://err.ersjournals.com/content/19/116/127

27. Smoking withdrawal symptoms peak at ?


A. 1-3days (48hours)
B. 4-7 days
C. 2 weeks
Answer: a
Http://www.webmd.com/smoking-cessation/understanding-nicotine-withdrawal-basics

28. Old patient with small cell lung cancer treated by chemotherapy on examination there is crepitation on the lung no ll swelling
lab result showed hyponatremia what is your advice ?
A. IV furosemide
B. Fluid restriction
C. Desmopressin

The answer is b
This is a case of paraneoplastic syndrome (sidah ).to treat the hyponatremia : treat the underlying cause and restrict free water
intake.
Reference: toronto notes



29. Asthmatic boy on muntelukast present to er, he has symptoms everyday exacerbated by exercise what to give for
maintenance
A. Oral steroid daily with long acting when needed
B. B- inhaled steroid twice with short acting when needed
C. Muntelukast with long acting
Answer:

55
Step 3 - moderate persistent asthma the preferred controller medication is either a low-dose inhaled corticosteroid plus a long-acting beta-agonist
[78]
(laba) (combination medication preferred choice to improve compliance) or an inhaled medium-dose corticosteroid. Alternatives include an inhaled
low-dose ics plus either a leukotriene receptor antagonist (montelukast) or low-dose theophylline.
Reference:http://emedicine.medscape.com/article/296301-treatment#d8

30. After delivery shortness of breath at night. What findings in the x-ray support diagnosis?
A. Increase in mediastinal width.
B. Increase shadowing
C. Cardiothoracic increase.
Answer:

31. Asthma on leukotriene inhibitor <<increase cough & on exercise << admission on short acting β blocker then discharge <<
what is the drug on discharge?
A. Long acting β agonist and short acting β agonist as need-
B. Oral steroid once + short acting β agonist as need-
C. Inhaler steroid 2 time + short acting β agonist as need

Answer: a

56
The basis of treatment is with pre-exercise short-acting β2 -agonist administration.[1] a role also exists for long-acting β2 -agonists and
mast cell stabilizers. Anti-leukotriene drugs have been shown to be effective as well
Http://emedicine.medscape.com/article/1938228-treatment

32. Patient had 1.5 cm calcified lesion in the routine chest x-ray. He's symptomless. Next action will be.
Answer:
A. Observation
B. Percutaneous biopsy
C. Transbronchial biopsy
Answer: a
No symptoms + calcified lesion less than 3cm is almost always benign.
Http://www.clevelandclinicmeded.com/medicalpubs/diseasemanagement/hematology-oncology/pulmonary-nodules/

33. Rx of patient with lung Ca stage iiib came with sudden lower back pain?
A. MRI only
B. MRI with steroid
C. Radiotherapy
Answer: c or a
Http://www.uptodate.com/contents/non-small-cell-lung-cancer-treatment-stage-iv-cancer-beyond-the-
basics?Source=outline_link&view=text&anchor=h10#h10


34. Drug that cause white patches in mouth. What is the inhaler?
A. Ipratropium
B. Short acting beta2 agonist
C. Steroid
Answer: c

• Children on inhaled steroids also have increased incidence of oral candidiasis.

Reference: medscape: http://emedicine.medscape.com/article/969147-overview


35. Scenario about tb and the pt take 4 medication for tb he developed numbness in hand and feet
A. Rifmpacin
B. Inh
C. Ethambutol

Answer: b

36. Hemosiderin deposition in macrophage in lung in


A. CMV
B. Pneumocystic jervoci
C. Chronic lung infection
Answer c
The presence of hemosiderin laden macrophages indicates there was blood in the alveolar space which canbe caused by infection.
Http://www.ncbi.nlm.nih.gov/pmc/articles/pmc2610638/

37. Case of copd what will you see in bronchus


A. Hyper plastic mucous gland
B. Thinning in the basement membrane
C. C-dilated sac.

Answer: a

57
38. 3.5 cm lung tumor with paratracheal lymph nodes?
A. T1 n 0
B. T2 n1
C. T3 n2
Answer: b (not sure n2)
Lung ca
- t2a, n1, m0: the cancer has 1 or more of the following features:
~>
the main tumor is larger than 3 cm across but not larger than 5 cm.
The tumor has grown into a main bronchus, but is not within 2 cm of the carina (and it is not larger than 5 cm).
The tumor has grown into the visceral pleura (the membranes surrounding the lungs) and is not larger than 5 cm.
The tumor is partially clogging the airways (and is not larger than 5 cm).
The cancer has also spread to lymph nodes within the lung and/or around the area where the bronchus enters the lung (hilar lymph
nodes). These lymph nodes are on the same side as the cancer. It has not spread to distant sites.

- but here lymph nodes are in para trachea so n2


n2 = the cancer has also spread to lymph nodes around the carina (the point where the windpipe splits into the left and right bronchi)
or in the space between the lungs (mediastinum). Affected lymph nodes are on the same side as the main lung tumor. It has not spread
to distant sites.

Http://www.cancercenter.com/lung-cancer/stages/
http://www.cancer.org/cancer/lungcancer-non-smallcell/detailedguide/non-small-cell-lung-cancer-staging

39. Pt with acute asthma given drug that works by inhibition of phophodiestrase enzyme which drug was given?
A. Salmeterol
B. Beclomethasone
C. Aminophylline
Answer: c
Http://www.drugbank.ca/drugs/db01223

40. Pt with chest infection was treated with oral ab for 4 weeks later came complain from rt lung effusion what dx?
A. Parapneumonic effusion empyema
B. Tb
C. Lung ca
Answer: a
• Definition:
Pus in pleural space or an effusion with organisms seen on a gram stain or culture (e.g. Pleural fluid is grossly purulent)
• Positive culture is not required for diagnosis

• Etiology:
Contiguous spread from lung infection (most commonly anaerobes) or infection through chest wall (e.g. Trauma, surgery)
• Signs and symptoms:
Fever, pleuritic chest pain
• Treatment:
Antibiotic therapy for at least 4-6 wk (rarely effective alone)
Complete pleural drainage with chest tube
If loculated, more difficult to drain – may require surgical drainage with video-assisted thorascopic surgery (vats


41. Patient presented to the er with cough hemoptysis night sweats and malaise. What is most appropriate initial step in the
management?
A. Isolation in negative pressure room
B. Start anti tb
C. Give opd appointment after 2 weeks
Answer: a
58
42. Drug addicted, unconscious came with no gag reflex. What would you do?
A. Intubation
B. Gastric lavage
C. Give naloxone
Answer: a
Start with the abcs

43. Surgery is contraindicated for which lung ca?


A. Scc
B. Scl
C. Adenocarcinoma
Answer: b (small - oat - cell)

Sclc
Treatment — sclc is a disseminated disease in most patients at presentation and is very responsive to chemotherapy. Thus systemic
chemotherapy is an integral part of the initial treatment.
Patients with limited stage disease are primarily treated with a combination of chemotherapy and radiation therapy, since the addition
of radiation therapy has been shown to prolong survival compared with chemotherapy therapy alone
For patients with extensive stage sclc, chemotherapy alone is used as the initial therapy

Nsclc
Treatment — surgical resection offers the best opportunity for long-term survival and cure in patients with resectable nsclc. The
appropriateness of surgical resection of candidates with known or suspected nsclc includes preoperative staging and an assessment of
performance status with concurrent comorbidities and pulmonary function to allow prediction of postoperative function.

44. Bronchial asthma daily cs, daily pft type of ba ?


A. Mild persistent
B. Moderate
C. Severe

Answer: c


45. Pediatric patient complains of cough he controls it by leukotrienes when he visited the primary physician 6 month ago. Now
he develops cough for 4 days after exercise they give him albuterol the symptoms were relieved what you going to give him:
A. Leukotriene + short act b-2 agonist
B. Oral steroid + short act b-2 agonist
C. Long act b-2 agonist + short act b-2 agonist
Answer: a

46. What is the treatment for female pt with uncontrolled asthma (wake her up from sleeping at night) she is on short beta 2
agonist?
A. Long acting beta 2
B. Steroid
C. Theophylline
Answer: b next step management inhaled
Http://getasthmahelp.org/documents/gist-stepwise-approach.pdf



47. Female non smoker with nodule by CT found calcium and fat
A. Hamartona
B. Mystheoma

59
C. No adeno

Answer: A
Pulmonary hamartomas, the most common benign tumors of the lung, are the third most common cause of solitary pulmonary nodules.
It is composed of cartilage, connective tissue, muscle, fat, and bone. CT is far superior than x-ray in detecting intralesional fat and
calcification.
Http://emedicine.medscape.com/article/356271-overview
Http://radiopaedia.org/articles/pulmonary-hamartoma-1

48. Smoker with hilar mass what suspect?


A. Lymph node
B. Squamous cell Ca
C. Adenocarcinoma

Answer: B
Central tumors are generally squamous cell carcinomas (sccs) and most peripheral tumors are adenocarcinomas or large cell
carcinomas which can cause pleural effusion
Http://emedicine.medscape.com/article/279960-overview
Http://radiopaedia.org/articles/squamous-cell-carcinoma-of-the-lung


49. Pt present with pleuritic chest pain , examination shows decrease air entry in the right lower lung, attached chest x-ray
showing radiolucent right costophrenic angle, what is next ?
A. Ventilation perfusion scan
B. Needle decompression
C. Chest tube
Answer: isn’t this pneumothorax?
Following treatment recommendations in first episodes of primary spontaneous pneumothorax: simple observation in the case of
clinically stable patients with small pneumothoraces; lung reexpansion with a small-bore catheter or placement of a chest tube
attached to either a heimlich valve or a water seal device in the case of clinically stable patients with large pneumothoraces; and lung
reexpansion with a chest drain or small bore catheter attached to a water-seal device or applying suction of unstable patients with large
pneumothoraces
read more: http://www.atsjournals.org/doi/full/10.1164/rccm.200111-078oc#.v1_flvkdbbc

50. Anion gap of corrected na 138:


A. 26
B. 34
C. 44
Answer:?



51. Anioin gap in mmol/l ?
A. 3
B. 95
C. 123
Answer: question incomplete (if question is which is a normal value, answer is a)
Explanation: the anion gap is the difference between primary measured cations (sodium na+ and potassium k+) and the primary
measured anions (chloride cl- and bicarbonate hco3-) in serum. The reference range of the anion gap is 3-11 meq/l
Reference: http://emedicine.medscape.com/article/2087291-overview


52. Bronchial cancer mets to sypathatic plexus what is the sign?
A. Ptosis
B. Dilated pupil
answer: a

60
the answer is horner’s syndrome ptosis ,anhydrosis,miosis
Toronto notes neoplasm (r28)
http://www.uptodate.com/contents/superior-pulmonary-sulcus-pancoast-tumors

53. Lung cancer 3cm rt mediastinal ln (paratracheal) stage:


A. T1n1m
B. T1n2m
Answer:b
N1=ipsilatral peribronchial n2=ipsilateral mediastinal n3=contralateral mediastinal
http://www.radiologyassistant.nl/en/p42459cff38f02/lung-cancer-new-tnm.html

54. Old male with recurrent episodes of cough with sputum and hemoptysis
A. Bronchiectasis
B. Tb
Answer: a
Recurrence is a feature of bronchiectasis that classically manifests as cough and the daily production of mucopurulent and tenacious
sputum lasting months to years that presents with sputum and episodes of hemoptysis.
Http://cursoenarm.net/uptodate/contents/mobipreview.htm?9/24/9601

55. Patient came for routine checkup cxr shows a unilateral calcified nodule on the upper zone of his lung:
A. Scc
B. Hamartoma

Answer: b
Pulmonary hamartomas, the most common benign tumors of the lung, are the third most common cause of solitary pulmonary
nodules.
Reference: http://emedicine.medscape.com/article/356271-overview


56. Pt with lung nodule and high Ca& parathyroid
A. Hyperparathyroidism
B. Lung ca
Answer: b
Squamous cell carcinoma of the lung secreting parathyroid hormone–related peptide causing hypercalcemia.
Reference: http://emedicine.medscape.com/article/279960-workup?Src=refgatesrc1#c5

57. Positive ppd test but normal x-ray, what will you do?
A. Isoniazid for 6 months
B. Rifampin for 6 months

Answer: a

1. For a positive tb exposure and a positive ppd test (but no active disease), treatment is inh (isoniazid) only.

Reference: step up to medicine


Patients with positive ppd or interferon gamma release assay (igra) with no evidence of active disease (asymptomatic and normal x-
ray) should receive a 9-month therapy of isoniazid (inh) and vitamin b6.
Kaplan internal medicine 2013.
Http://www.uptodate.com/contents/treatment-of-latent-tuberculosis-infection-in-hiv-uninfected- adults?Source=see_link


58. Patient with small lung carcinoma (slc) undergoing chemotherapy. He developed …… lab values : low blood na, low urine
osmolality. The treatment?
A. Desmopressin
B. Nothing
Answer: a
61
Decreased urine osmolality in hyponatremic patients is observed in some patients with reset osmostat syndrome, when water intake
reduces the serum osmolality below the new threshold for adh release. Reset osmostat syndrome is a variant of the syndrome of
inappropriate adh secretion (siadh) which is a common paraneoplastic syndrome in slc. Siadh is managed by desmopressin.
Http://www.ncbi.nlm.nih.gov/pmc/articles/pmc100882/

59. Obstructive sleep apnea. Most effective treatment


A. Bmi less than 30
B. Cpap
Answer: b
Cpap is the standard treatment option for osa and generally can reverse this condition quickly with the appropriate titration of devices.
Reference: http://emedicine.medscape.com/article/295807-treatment

60. Hemosiderin laden in alveolar lavage?


A. Due to pcp(pnemocytits ji..
B. CMV

These hemosiderin-laden alveolar macrophages are characteristically found in bal fluid or lung biopsy from patients with diffuse
alveolar hemorrhage. Classically, diffuse alveolar hemorrhages are classified based on histologic appearance as capillaritis (wegener
granulomatosis, systemic lupus erythematosus, and propylthiouracil related), bland hemorrhage (coagulopathies, congestive heart
failure, and rapamycin related), diffuse alveolar damage (ards and crack cocaine inhalation), and miscellaneous conditions
(lymphangioleiomyomatosis and pulmonary capillary hemangiomatosis).

61. Pt. With moderate persistent bronchial asthma on beta agonist prn and low dose inhaled steroid comes with uncontrolled ba.
What you will add to the steroid?
A. Long acting beta agonist
B. Theophylline
Answer: a http://getasthmahelp.org/documents/gist-stepwise-approach.pdf

62. 20 year snoring ... He has enlarged tonsils treatment of obstructive sleep apnea.
A. Cpap.
B. Reduced weight.
Answer: missing information. But osa is an indication for tonsillectomy if enlarged.

63. According to the new classification of lung cancer, which of the following is considered carcinoma in-situ?
A. Adenocarcinoma less than 2 cm.
B. Atypical hyperplasia

Answer: A
Adenocarcinoma in situ (AIS) with no invasive features is a localized, small (≤3 cm) adenocarcinoma with growth restricted to a
noninvasive lepidic pattern and an absence of papillary or micropapillary patterns or intraalveolar tumor cells.
Reference: uptodate.

64. Most common cause of excessive sleepiness in the daytime is?


A. Narcolepsy
B. Obstructive Sleep Apnea (OSA)

Answer: B
The most common causes of excessive daytime sleepiness are sleep deprivation, obstructive sleep apnea, and sedating medications.
Reference: American Family Physician Journals.
If attack its narcolepsy

62
65. A male patient who is a smoker, developed symptoms? Cais High. CXR showed solitary nodule. What is the most likely
diagnosis?
A. Squamous Cell Carcinoma SCC
B. Adenocarcinoma

Answer: A
High Ca> Paraneoplastic of SCC.

66. Common cause of bronchiolitis?


A. Para influenza virus
B. RSV

Answer: B
Bronchiolitis is an acute inflammatory injury of the bronchioles that is usually caused by a viral infection (most commonly respiratory
syncytial virus and human metapneumovirus) Medscape

67. Old male with recurrent episodes of cough with sputum and hemoptysis:
A. Bronchiectasis(my answer)
B. Tb

68. Lung tumor with keratinization, what type of cells?


A. Epithelial
B. Squamous
Answer: b
Explanation: squamous cell carcinoma. Loose cohesive sheet of tumor cells and single tumor “strap cells” cells with keratinization are
present.
Link: http://www.ncbi.nlm.nih.gov/pmc/articles/pmc2872744/

69. Case of asthma sever, cough every week , he took nebulizer steroid , what is the next step of management ?
A. Add long acting b agonst
B. Ibrapritom

70. Copd patient was on oral steroid and there was improvment 17%in breathing , which medication will u put him on :
A. Theophthylin
B. Amitriptalin inhaler or oral .
Answer: choices incomplete and answer most likely missing
Explanation: systemic corticosteroids should be initiated after the first treatment of short-acting inhaled bronchodilators; beta-2
agonists (albuterol) are typically favoured as first-line as they function more rapidly than anticholinergic bronchodilators
(ipratropium).
Reference; http://bestpractice.bmj.com/best-practice/monograph/8/treatment/step-by-step.html

71. Old patient with squamous cell carcinoma of the lung present with lethargy for 2 months. Lab showed hyponatremia (na: 121)
(case of siadh), treatment?
A- normal saline
B- fluid restriction

A. Fluid restriction (since it is asymptomatic hyponatremia- no neurological manifestation)
Answer: severe symptoms (altered mental status, seizure, and coma) or sodium <125 mmol/l (<125 meq/l), divided into 2:
• Acute onset 48hrs or less:
o IV hypertonic saline
o In the er, start with 50 ml 3% saline iv, followed by 200 ml IV infusion over 4 to 6 hours
o There is a risk of central pontine myelinolysis (osmotic demyelination syndrome) in case of rapid correction
63
o Fluid restriction
o Treat underlying cause
o Furosemide: used in addition to hypertonic saline, especially if the patient is at risk for volume overload. It helps
to correct hyponatremia by increasing free water excretion.
• Chronic onset more than 48hrs or unknown
• IV hypertonic saline (same as above)
• IV vasopressin receptor antagonist → either conivaptan or tolvaptan
ü Conivaptan (a non-selective vasopressin receptor antagonist) + monitoring the patient for hypotension is a
must + common se is skin reaction at the site of iv
ü Tolvaptan (a selective v2 receptor antagonist)+ serum sodium should be checked at baseline and 8 hours
after the first dose + the drug should be discontinued immediately in patients with signs or symptoms of
liver injury (e.g., fatigue, anorexia, right upper abdominal discomfort, dark urine, jaundice, elevated LFTs).
Http://bestpractice.bmj.com/best-practice/monograph/196/treatment/details.html

72. An asthmatic patient is on inhaled corticosteroids. His asthma becomes more severe, what should you add?
Answer: laba
Https://www.nhlbi.nih.gov/files/docs/guidelines/asthma_qrg.pdf

73. Patient with chronic bronchitis (scenario write symptom productive cough for last 3 month per year through last 2 year) and
what is find in microscopy under histopathology?
Answer: chronic bronchitis is defined as a cough that occurs every day with sputum production that lasts for at least 3 months, 2 years
in a row.

74. Low ph, hco3, co2?


A. Compensated metabolic acidosis.

Answer: you need the exact numbers to determine if it is compensated or not

Reference: toronto note, pocket of medicine


64
75. Scenario of atypical pneumonia, causative organism ?
A. Mycoplasma pneumonea
Source: medscape

76. Cystic fibrosis mode of inheritance?

• Autosomal recessive condition predominantly affecting caucasians

Reference: step up to medicine

77. Pneumonia bilateral infiltration ?


A. Ciprofloxacine
Answer :?!
Bilateral infiltration usually caused by staph aureus

78. 7month present with respirtory sx ?? Bronchlitis ++


Answer: not clear

79. Pneumocyte type 1 features?


Answer: 95% of the alveolar surface
Type i alveolar cells:
Type i alveolar cells are squamous (giving more surface area to each cell) and cover approximately 90–95% of the alveolar surface.
Type i cells are involved in the process of gas exchange between the alveoli and blood. These cells are extremely thin (sometimes only

65
25 nm) – the electron microscope was needed to prove that all alveoli are covered with an epithelial lining. These cells need to be so
thin to be readily permeable for enabling an easy gas exchange between the alveoli and the blood.
Type ii alveolar cells:
Type ii alveolar cells cover a small fraction of the alveolar surface area. Their function is of major importance in the secretion of
pulmonary surfactant, which decreases the surface tension within the alveoli.


80. The question is missing a lot of information. Asthma with daily symptoms and frequent night time symptoms is classified as
severe.
Http://emedicine.medscape.com/article/296301-treatment#d8

81. Pt develop cough during exercise: which medication want to give her before exercise?
Answer: b2 agonist



82. Patient taking high dose of salicylate what to find?
A. Respiratory alkalosis & metabolic acidosis.
Answer: a
Medscape:

83. DM patient with abg values show ph 7.2 . Patient is hyperventilated. Why this happen?!
A. To get rid of co2
Answer: a
Since co2 is acidic, and the ph is acidic.. Getting rid of the co2 would increase the ph.

84. Status asthmaticus on drug inhibit cholinesterase what is the drug?


A. Ipratropium



85. Scenario about horner syndrome asking about the site of tumor:
A. On the lung apical .
Explanation: the common lesions that cause horner syndrome interfere with preganglionic fibers as they course through the upper
thorax. Pancoast tumor (tumor in the apex of the lung, most commonly squamous cell carcinoma) can cause horner syndrome.
Reference: http://emedicine.medscape.com/article/1220091-overview#a4

86. Pt came e hx of hf & pulmonary edema. Tx?!


A. Diuretics
Answer: a. Treat the cause and give o2 and IV furosemide and/or nitrates as needed; try noninvasive ventilatory assistance initially but
use tracheal intubation and assisted ventilation if necessary. (step up to medicine)
66
87. Easy q about cystic fibrosis "sweat test" ?
Sweat test is a diagnostic test of cf ( confirm ).
Sweat test and genetic test , it's caused by mutation in both copies of gene for the cystic fibrosis transmembrane conductance regulator
(cftr) protein .


88. Asthmatic controlled on albuterol prn, now she got pregnant,and she started to have daily symptoms & night ... ?
Answer: inhaled steroid + laba
Pregnant women manage asthma the same way nonpregnant women do. Like all people with asthma, pregnant women need to have an
asthma action plan to help them control inflammation and prevent and control asthma attacks.
Http://www.webmd.com/asthma/tc/asthma-during-pregnancy-topic-overview

89. Abg case of patient presented with vomiting?


Answer: metabolic alkalosis
Metabolic alkalosis is a primary increase in serum bicarbonate (hco3-) concentration. This occurs as a consequence of a loss of h+
from the body or a gain in hco3-. In its pure form, it manifests as alkalemia (ph >7.40).
Http://emedicine.medscape.com/article/243160-overview


90. Case of pt took nebulizer steroid , develop white patch on tongue ( moth) what is digenesis ?
A. Oral thrush


91. Lung cancer which stage:
A. Stage 1
Answer:a
Reference:http://cancerstaging.org/references-tools/quickreferences/documents/lungmedium.pdf

92. Non small cell lung cancer has 4 risk factors which are stage of the disease , condition of the patient and male sex
Question is not clear and choices incomplete, but male sex is a risk factor for non-small cell lung cancer
Reference: http://bestpractice.bmj.com/best-practice/monograph/1082/diagnosis/history-and-examination.html

93. Acid base disorder calculation?


Reference: http://fitsweb.uchc.edu/student/selectives/timurgraham/stepwise_approach.html

94. Patient with metabolic acidosis with high anion gap (aspirin toxicity)
Answer:
Activated charcoal and alkaline diuresis with extra kcl
Source: http://www.merckmanuals.com/professional/injuries;-poisoning/poisoning/aspirin-and-other-salicylate-poisoning

95. Regarding the previous case, what should be given ?


A. Misprostol
Answer: a

96. Case of surfactant deficiency?

97. Typical case of organophosphorus poisoning dx

98. Typical case of monoxide poisoning dx

99. Case of hypernatremia and hypokalemia and ask for how to replace them ?

67
100.RSV ttt
Http://www.mayoclinic.org/diseases-conditions/respiratory-syncytial-virus/basics/treatment/con-20022497

101. Pt with obstructive lung disease (FEV1), what is your best advice for him?
A. Stop smoking.
Answer:
All patients with COPD should be advised to quit smoking, educated about COPD, and given a yearly influenza vaccination, In
addition to pneumococcal polysaccharide vaccine.
Pharmacological management:
- For all patients with COPD, short-acting bronchodilator (eg, beta-agonist, anticholinergic agent) be prescribed for use as-needed for
relief of intermittent increases in dyspnea
- for patient insufficient to control symptoms or who have other high-risk predictors, including severe or very severe airflow
obstruction or two or more exacerbations in the previous year,
We prefer the long-acting inhaled anticholinergic agent to the twice daily long-acting beta agonists.
- For patients who continue to have symptoms or have repeated exacerbations despite an optimal long-acting inhaled
bronchodilator regimen,
We suggest adding an inhaled glucocorticoid .
- For symptomatic patients with GOLD Stage II, III, or IV COPD, we recommend pulmonary rehabilitation.
- long-term oxygen therapy in all patients with COPD who have chronic hypoxemia (Grade 1A).
Reference:
Http://www.uptodate.com/contents/management-of-stable-chronic-obstructive-pulmonary-disease#H40


102.Lung disease that causes clubbing?
Answer: A. Bronchiectasis (The only option mentioned)
Clubbing usually begins in the thumb and index fingers and is most often associated with pulmonary or cardiovascular diseases,
including lung cancer, interstitial pulmonary fibrosis, lung abscess, pulmonary tuberculosis, pulmonary lymphoma, congestive heart
failure, infective endocarditis, and cyanotic congenital heart disease. Less frequently, digital clubbing may occur in patients with
extrathoracic disease, including inflammatory bowel disease, liver cirrhosis, and gastrointestinal neoplasms.
Reference: Uptodate

103.A patient with lung cancer. Lab results: low PTH and High Calcium. What is the reason?
A. PTH related peptide for lung ca
Answer: A
Reference: kumar and clark’s

104.Patient with bronchiectasis. What else beside medical treatment can benefit this patient?
A. Chest physiotherapy
Answer: A
Reference: Toronto Notes

105.Best drug to decrease bronchial secretion in COPD?


A. Ipratropium
Answer: A
Reference: Master the board.

106.Most common cause of pneumonia? 2 times


Answer: Streptococcus pneumonia is by far the most common cause of typical bacterial pneumonia.
Reference: http://emedicine.medscape.com/article/300157-overview#a4

107.Pt have lesion in right upper lung look like calcium how to treat?
Answer:

68
The first step in the evaluation of a pulmonary nodule is to look for a prior x-ray. Finding the same pulmonary
nodule on an x-ray done years ago may save you from doing any further workup. If no prior x-ray is available,
then consider whether this patient is high or low risk for lung cancer.

In low-risk patients, <35 years of age and nonsmokers with calcified nodules, you may follow
the patient with chest x-rays or chest CT every 3 months for 2 years. Stop the follow-up if after 2 years
there is no growth.

High-risk patients >50 years of age with a smoking history and a nodule are likely to have bron-chogenic
cancer. The best diagnostic procedure is open-lung biopsy and removal of the nodule at the same time.

Reference : Kaplan Internal Medicine 2013

108.Asthmatic patient on monteleukast and bronchodilator, has dry cough every day came to ICU, what to give for long term?
Answer: high-dose inhaled corticosteroid plus a leukotriene receptor antagonist plus an oral corticosteroid. Consider omalizumab for
patients who have allergies.
Reference: Medscape

109.Bilateral pneumonia treatment


Answer: ??? Nothing specific found about bilateral pneumonia
Outpatient:
• Previously healthy, no antibiotic for 3 months – macrolide (azithromycin, clarithromycin) or doxycycline
• Comorbidity, use of antibiotic in 3 months – respiratory fluoroquine (levofloxacine, moxifloxacine)
Inpatient: respiratory fluoroquine (levofloxacine, moxifloxacine) or ceftriaxone + azithromycine
Reference: master the boards USMLE step 2, Toronto notes

110.Most common cause of obstructive sleep apnea ?


Answer:
Most patients have OSA because of a small upper airway. As the bones of the face and skull develop, some people develop a small
lower face, a small mouth, and a tongue that seems too large for the mouth. These features are genetically determined, which
explains why OSA tends to cluster in families. Obesity is another major factor. Tonsil enlargement can be an important cause,
especially in children.
Reference: Uptodate

111.The least cause of dyspnea is :


a. Chronic bronchitis
b. Emphysema
c. Metabolic acidosis

Answer: A
I thinck the question want to ask about the two types of COPD (blue bloter vs. Pink puffer)

69

Nephrology

70
1. High pco2, low ph ; (medicine)
A. Metabolic acidosis
B. Metabolic alkalosis
C. Respiratory acidosis
D. Respiratory alkalosis
Answer: c

2. Ph= 7.1, hco3= 18, co2= 25 dx:


A. Metabolic alkalosis
B. Metabolic acidosis
C. Respiratory alkalosis
D. Respuratory acidosis
Answer: b
Reference: http://lifeinthefastlane.com/investigations/acid-base/

3. Urine incontinence, bladder palpable during examination, what type of urine does the patient has?
A. Stress.
B. Overflow.
C. Reflux.
D. Urgency
Answer : b

4. First symptom of hypomagnesaemia?


A. Muscle paralysis?
B. Hypotension
C. Loss of deep muscle reflex
D. Respiratory depression
Answer: a
- Neuromuscular manifestations: muscular weakness, apathy, tremors, paraesthesia, tetany, vertical nystagmus and positive
chvostek and trousseau signs.
- Severe effects such as seizures, drowsiness, confusion and coma occur at magnesium concentrations <0·4 mm
- Cardiovascular manifestations: hypomagnesaemia has been associated with both atrial and ventricular arrhythmias
- Ecg: flattened t-waves, u-waves, prolonged qt interval and widened qrs complexes
- Metabolic effects: 1) hypokalemia is associated with hypomagnesaemia in up to 60% of cases. Due to underlying common
etiologies that cause magnesium and potassium losses and partly due to a specific disorder of renal potassium wasting as a result
of hypomagnesaemia. 2) hypocalcaemia is also commonly associated with hypomagnesaemia; hypomagnesaemia suppresses the
release of pth and also induces end-organ resistance to pth
- Http://www.medscape.com/viewarticle/753881_4

5. What is the most common cause of secondary hypertension?


A. Renal disease
B. Cushing
71
C. Conn
D. Pheochromocytoma
Answer: a
Depending on the age http://www.aafp.org/afp/2010/1215/p1471.html

6. Case with: ph 7.2 \ pco2: decreased below normal range \ bicarbonate decreased below normal range . Dx :
A. Compensated metabolic acidosis
B. Uncompensated metabolic acidosis
C. Compensated respiratory acidosis
D. Uncompensated metabolic acidosis
Answer: b? Partially abnormal ma?
If ph is normal, paco2 and hco3 are both abnormal = compensated
if ph is abnormal, paco2 and hco3 are both abnormal = partially compensated
if ph is abnormal, paco2 or hco3 is abnormal = uncompensated
Http://goo.gl/3gsaqp

7. Long scenario with urine analysis only which shows: normal urine ph high chloride bicarbonate low And others within normal
limit. What is your diagnosis?
A. Metabolic acidosis
B. Metabolic alkalosis
C. Respiratory alkalosis
D. Respiratory acidosis
Answer: most likely b


8. Characteristic signs of nephritic syndrome:
A. Hypertension
B. Hyperlipidemia
C. Hypoalbuminia
D. Edema
Answer: a
Nephritic syndrome symptoms: hematuria, proteinuria, hypertension, blurred vision, azotemia, oliguria.
- Hematuria: +++ blood – microscopic or macroscopic hematuria, red cell casts – distinguishing feature, form in nephrons &
indicate glomerular damage
- Haematuria occurs due to podocytes developing large pores which allows blood and protein to escape into the urine.
- 2 - proteinuria: ++ protein (small amount)
- 3 - hypertension: usually only mild
- 4 - low urine volume <300ml/day, due to reduced renal function.
Http://geekymedics.com/nephrotic-vs-nephritic-syndrome/


9. 68 years old male patient came with weight loss, hematuria, flank pain and flank mass. Which of the following imaging study
will confirm the diagnosis?
A. Us
B. Ct
C. Mri
D. Radionucliotide
Answer: b
Http://radiopaedia.org/articles/renal-cell-carcinoma-1


10. Renal or small cell cancer stage iii with bone pain what is the immediate action
A. MRI only
B. Radiotherapy
C. IV steroid and mri
D. No immediate action

72
Answer: a
• The most sensitive way of detecting bone metastases is by isotope bone scan.
• The main goals of management are: pain relief, preservation and restoration of function, skeletal stabilization, and local
tumour control (e.g. Relief of tumour impingement on normal structure).

reference: davidsons 22

11. Filling defect seen with acoustic shadow in the renal us dense echo:
A. Tumor
B. Uric acid stone
C. Blood clot
D. Sloughed papilla
Answer: b
Acoustic shadowing occurs when the sound wave encounters a very echo dense structure, nearly all of the sound is reflected,
resulting in an acoustic shadow such as stones. Http://emedicine.medscape.com/article/381993-overview


12. Urge incontinence principle management:
A. Medical
B. Surgical
C. Medical and surgical
D. Bladder training and physio..
Answer: a
Mainly lifestyle modification, physiotherapy and medications. Rarely surgical.
Merck manual

13. Renin secreted from ?


A. Renal afferent
B. Renal efferent
C. Distal tubules
D. Proximal tubule
Answer: a ( by juxtaglomerular cells in afferent arteriole )
Reference: usmle step 1

14. 50+ old patient ē HTN and proteinuria 4+, pyelography showed right kidney 14 cm, left kidney 7cm, arteriography showed left
renal artery stenosis. What to do?
A. CT angio
B. CT abdomen
C. Radiolucent something
D. Percutaneous renal biopsy

Answer: a
A. Renal arteriogram is the gold standard in diagnosing ras.

73
Reference: step up to medicine. Uptodate.


15. Uti case patient resistant to b lactams, sensitive to fluoroquinolones, chloramphenicol, aminoglycosides which drug is
contraindicated:
A. Gentamicin
B. Azithromycin
C. Lovcloxacillin
D. Chloramphenicol

Answer: a

Not a clear question and options!



16. To prevent recurrence of uti , what you prefer of the following circumstances ?
A. Decreased ph ,increased urea or urea(i did not remember),decreased urine osmolarity (i think)*
B. Decreased ph ,increase urea ,increased urine osmolarity
C. Increased ph ,increased urea or urea(i did not remember),decreased urine osmolarity
D. Increased ph ,decreased urea or urea(i did not remember),increased urine osmolarity

Answer: b

17. Patient on diuretics developed palpitation. Due to the disturbance of which of these electrolytes lead to his presentation?
A. Na
B. K
C. Cl
D. Ca

Answer: B
Hyperkalemia: usually asymptomatic but may develop nausea, palpitations, muscle weakness, muscle stiffness, paresthesias, areflexia,
ascending paralysis, and hypoventilation. Can be caused by K+-sparing diuretics such as Spironolactone, Amiloride and Triamterene.
Reference: Toronto Notes.

18. (long scenario) 55 year old known diabetic patient came for checkup. What is the earliest effect of Diabetes Mellitus on the
kidneys?
A. Hydronephrosis with ↑ protein excretion.
B. Hydronephrosis with ↓ protein excretion.
C. Sclerosis with ↑ protein excretion.
D. Sclerosis with ↓ protein excretion.

Answer: ?
Hyperfiltration with normoalbuminuria or ↑ protein excretion.
(I think they meant by “hydronephrosis” = hyperfiltration).

74
References:Toronto Notes + http://www.pathophys.org/ckd/ + http://emedicine.medscape.com/article/238946-overview#showall



19. Calculate anion gap
o Na: 135
o Cl: 100
o Hco3: 12
A. 23
B. 10
C. 6

Answer: a
The anion gap is the difference between primary measured cations (sodium na+and potassium k+) and the primary measured anions
(chloride cl- and bicarbonate hco3-) in serum.
Ag = na - (cl + hco3) or (na +k) – (cl + hco3)
The normal range value for the serum anion gap is 8-16 meq/l
Reference: http://emedicine.medscape.com/article/2087291-overview
Https://en.wikipedia.org/wiki/anion_gap


20. Most common sign in renal cancer.
A. Cachexia
B. Hematuria
C. Abdomen mass

Answer: b
Renal cell carcinoma (rcc) may remain clinically occult for most of its course. The classic triad of flank pain, hematuria, and flank
mass is uncommon (10%) and is indicative of advanced disease. Twenty-five to thirty percent of patients are asymptomatic, and
their renal cell carcinomas are found on incidental radiologic study.
The frequency of the individual components of the classic triad is as follows:
• Hematuria – 40%
• Flank pain – 40%
• A palpable mass in the flank or abdomen –25%
Reference: Medscape

21. Patient with pheochromocytoma and high catecholamine in urine. What’s the initial medical management?
A. ACEI
B. Aldosterone blocker
C. Phenoxybenzamine

Answer c
First goal in management is to control the blood pressure.
Phenoxybenzamine (alpha blockade) is the first to control blood pressure. Without alpha blockade, patients’ blood pressure can
significantly rise intra-operatively.
Reference: master the boards

75
22. Patient with pheochromocytoma and high catecholamine in urine initial medical management:
A. ACEI
B. Aldosterone blocker
C. Alpha blockers

Answer: c
All patients with pheochromocytoma need to undergo preoperative alpha-adrenergic blockade as the first line drug. After alpha
blocker is given we give beta blocker (control heart rate) for 2 – 3 days but never start beta blocker before the alpha blocker. If beta
blockade is started prematurely, unopposed alpha stimulation could precipitate a hypertensive crisis. Administer the last doses of oral
alpha and beta blockers on the morning of surgery. Surgical resection with early ligation of venous drainage is the treatment of choice
in pheochromocytoma
References: uptodate, Medscape

23. Patient with HTN taking medication c/o painful big toe what's the drug?!
A. Thiazide.
B. Mannitol.
C. Spironolactone.

Answer: a
Lippincott pharmacology:
• Thiazide can precipitate a gouty attack by increasing uric acid in predisposed individuals.
• Loop diuretics can have this effect too.

24. 50 yrs man diabetic, well controlled had colon cancer surgery ( coloectomy) they kept him on insulin and dextrose, after
surgery by 2 days he became irritable, in shock, his electrolyte ( na 129 ) ( k 3.2 ) urine and serum osmolality normal, what's
the dx:
A. Fluid overload.
B. Addison disease
C. Siadh

Answer: a

25. Patient drink antifreeze which contain ethylene glycol. What is the most likely complication he will have?
A. Rapid progression glomrernephrithis
B. Pyelonephritis
C. Atn
Answer: c
Ethylene glycol will produce oxalate crystals which will cause atn (kaplan internal medicine)

•siadh: hyponatremia, low serum osmolality, normal k+


• addison: hyponatremia, hyperkalemia
76
- fluid overload:

26. Patient with polyuria polydipsia on water deprivation test, low osmolality of urine ,,, not respond to desmopressin ,what is
the diagnosis ?
A. Central di
B. Nephrogenic di
C. Psychogenic polydipsia
Answer: b


27. Breast cancer metastasis to the lung before mastectomy pt is depressed , increase urination , thirsty all the time , lab
provided na low , urine osmo low?
A. Psych polydypsia
B. Siadh
C. Di

Answer: c
diabetes insipidus is defined as the passage of large volumes of dilute urine in 2 forms
Central: characterized by decreased secretion of antidiuretic hormone
Nephrogenic: characterized by decreased ability to concentrate urine because of adh resistance.
Reference: medscape http://emedicine.medscape.com/article/117648-overview



28. Anion gap mml/
A. 13
B. 28
C. 99
Answer: a

29. Case of a guy who has high phosphate. Question phosphate is elevated in which organ failure?
A. Liver
B. Kidney
C. Lung
Answer: b
Reference: http://emedicine.medscape.com/article/241185-overview#a4

30. Patient with vomiting and diarrhoea. What type of electrolyte imbalance?
A. Hypernatremia.
B. Hyperglycemia.
C. Hyperkalemia.
Answer: a
Ref.http://www.uptodate.com/contents/image?Imagekey=neph%2f69879&topickey=neph%2f2376&source=see_link&utdpopup=true

31. A patient with malar rash, arthritis, …… what is the diagnosis?


A. ITP
B. Lupus nephritis
C. Ra
Answer: b
The classic presentation of a triad of fever, joint pain, and rash in a woman of childbearing age should prompt investigation into the
diagnosis of sle. Patients may present with any of the following manifestations:
• Constitutional (eg, fatigue, fever, arthralgia, weight changes)
• Musculoskeletal (eg, arthralgia, arthropathy, myalgia, frank arthritis, avascular necrosis)
• Dermatologic (eg, malar rash, photosensitivity, discoid lupus)

77
• Renal (eg, acute or chronic renal failure, acute nephritic disease)
• Neuropsychiatric (eg, seizure, psychosis)
• Pulmonary (eg, pleurisy, pleural effusion, pneumonitis, pulmonary hypertension, interstitial lung disease)
• Gastrointestinal (eg, nausea, dyspepsia, abdominal pain)
• Cardiac (eg, pericarditis, myocarditis)
• Hematologic (eg, cytopenias such as leukopenia, lymphopenia, anemia, or thrombocytopenia)
Http://emedicine.medscape.com/article/332244-overview

32. What is true about polycystic kidney disease?


A. Autosomal recessive
B. X-linked
C. Autosomal dominant.
Answer: c
Autosomal dominant polycystic kidney disease (adpkd) is a multisystemic and progressive disorder characterized by cyst formation and
enlargement in the kidney (see the image below) and other organs (eg, liver, pancreas, spleen). Up to 50% of patients with adpkd require
renal replacement therapy by 60 years of age.
Http://emedicine.medscape.com/article/244907-overview

33. Which cells forming the filtration layer in the kidney:


A. Mesangial
B. Podocytes
C. Partial
Podocytes play an important role in glomerular function. Together with endothelial cells of the glomerular capillary loop and the
glomerular basement membrane they form a filtration barrier.

34. Treatment of urge incontinence ?


A. Medical
B. Surgical
C. Medical + surgical
Answer: a
Http://www.webmd.com/urinary-incontinence-oab/america-asks-11/urge


35. Patient with vomiting and diarrhea. What type of electrolyte imbalance?
A. Hypernatremia.
B. Hyperglycemia.
C. Hyperkalemia.

Answer: a
Explanation: both upper and lower gastrointestinal losses can result in hypernatremia when water intake is limited. Loss of gastric
secretions (due to vomiting or drainage) and upper gastrointestinal losses, which contain both gastric and small intestinal secretions,
have a sodium plus potassium concentration well below that in the plasma and will therefore promote the development of hypernatremia.
Similar considerations apply to osmotic diarrheas but not to secretory diarrheas, which have a sodium plus potassium concentration
similar to that in the plasma, the loss of which will not directly affect the serum sodium concentration
Reference http://cursoenarm.net/uptodate/contents/mobipreview.htm?32/2/32800#h6

36. Abdominal solid mass (renal i guess but not sure ) confirmed by
A. CT
B. MRI
C. US

Answer: a.
Contrast-enhanced CT scanning has become the imaging procedure of choice for diagnosis and staging of renal cell cancer and has
virtually replaced excretory urography and renal ultrasonography. Http://emedicine.medscape.com/article/281340-workup

78
37. Treatment of streptococcus glomerulonephritis in children with edema and htn?
A. High dose of antibiotic
B. Diuretic for edema
C. Diuretic for htn
Answer: b
Uptodate:
There is no specific therapy for psgn. Management is supportive and is focused on treating the clinical manifestations of the disease,
particularly complications due to volume overload. These include HTN and less commonly pulmonary edema. General measures
include na and water restriction and loop diuretics. Loop diuretics generally provide prompt diuresis with reduction of Blood
pressure and edema.


38. Patient presented with soda-color urine since one week, during examination there are congestion of throat with cervical
lymphadenopathy with fever. What's the cause?
A. Acute glomureties
B. IgAnephropathy
C. Acute cystitis
Answer: a
IgAnephropathy ~> gross hematuria usually appears simultaneously or within the first 48-72 hours after the infection begins;
persists less than 3 days; and, in about a third of patients, is accompanied by loin pain, presumably due to renal capsular swelling.
Acute gn ~> symptom onset is usually abrupt. In the setting of acute postinfectious glomerulonephritis (gn), a latent period of up to
3 weeks occurs before onset of symptoms. However, the latent period may vary; it is typically 1-2 weeks for postpharyngitis cases
and 2-4 weeks for cases of postdermal infection (ie, pyoderma). The onset of nephritis within 1-4 days of streptococcal infection
suggests preexisting renal disease.
Http://emedicine.medscape.com/article/239927-clinical#b1
http://emedicine.medscape.com/article/239278-clinical#b1

39. A scenario of a patient with HTN came with headache and anxiety and have 3 previous vistas of high blood pressure and
treated for it then we need to stop because of the patient had a hypotension what you will do:
A. 24 urine metamephrin (phenchromocytoma )
B. TSH
C. Acth
Answer: a


40. Patient drinks some material made from methanol used for freezing what is the complication?
A. Rapid progression glomerulonephritis
B. Pyelonephritis
C. Atn
Answer: c


41. A case of pyelonephritis, what is the next step?
A. Admit and give antibiotics
B. Do investigations
C. Give him antibiotics at home
Answer: treatment with fluids and oral antibiotics may be given on an outpatient basis if children are not vomiting and not markedly
ill so it depends on the case. It is prudent to order urinalysis (and urine culture in those with abnormal findings) in all febrile boys
younger than 6 months and febrile girls younger than 24 months with fever lasting more than 48 hours. Medscape


42. Child with nephrotic syndrome on steroid for 3-6wks or 6 months have vaccine ?
A. Give the vaccine
B. Stop the steroids
C. 3month and give appointment

79
43. Action of cytotoxic?
A. Il6
B. Il10
C. Tnf gamma

44. Pt drink some material made from ethanol use for freezing what the complication
A. Rapid proggesion glomrernephrithis
B. Pyeleyno nephritis
C. Atn
There is no clear or conclusive explanation of he direct effect of ethanol on kidney but in that q the ethanol used for freezing which
can induce vasoconstriction and cause ischemic atn


45. Patient with long hx of uncontrolled htn, he presented to you with headache and 160/90 Blood pressure , what you will see
in his kidneys:
A. Decrease sclerosis
B. Increase hyalinization of arterioles

Answer: b
Benign hypertensive arteriolar nephrosclerosis is characterized histologically by arteriolar hyalinosis caused by insudation of plasma
proteins and medial thickening caused by both hypertrophy and hyperplasia of vascular smooth muscle cells, tubular
atrophy, interstitial fibrosis, periglomerular fibrosis, hyaline casts, protein and blood in urine
Reference: merck manual, Wikipedia

Intimal thickening and luminal narrowing of the large and small renal arteries and the glomerular arterioles, and
Glomerulosclerosis: both focal global (involving the entire glomerulus) and focal segmental sclerosis
Reference: http://www.uptodate.com/contents/clinical-features-diagnosis-and-treatment-of-hypertensive-
nephrosclerosis?Source=preview&search=%2fcontents%2fsearch&anchor=h11268824#h2

46. Patient had throat infection 2 weeks ago was developed hematuria how to treat:
A. Corticosteroids
B. Thiazide
Answer: c
The diagnosis is post strep glomerulonephritis
During the acute phase of the disease, restrict salt and water. If significant edema or hypertension develops, administer diuretics.
Loop diuretics increase urinary output and consequently improve cardiovascular congestion and hypertension.
For hypertension not controlled by diuretics, usually calcium channel blockers or angiotensin-converting enzyme inhibitors are
useful. For malignant hypertension, intravenous nitroprusside or other parenteral agents are used.
Other features of therapy are as follows:
indications for dialysis include life-threatening hyperkalemia and clinical manifestations of uremia
restricting physical activity is appropriate in the first few days of the illness but is unnecessary once the patient feels well
steroids, immunosuppressive agents, and plasmapheresis are not generally indicated
Reference:
Http://emedicine.medscape.com/article/240337-treatment#d8

47. Patient on diuretics, diarrhea. What type of electrolyte imbalance?


A- Hypokalemia.
B- Hyperkalemia.

Answer: a
Explanation: hypokalemia is caused by many mechanisms one of which is increased excretion. Increases excretion could be by non-
renal losses such as diarrhea or by renal losses by administration of diuretic such as loop and thiazide diuretics.
Reference http://www.clevelandclinicmeded.com/medicalpubs/diseasemanagement/nephrology/hypokalemia-and-hyperkalemia/

80
48. Patient on diuretics, diarrhea. What type of electrolyte imbalance?
A. Hypokalemia.
B. Hyperkaliemia.
Answer: a
Ref.:http://www.clevelandclinicmeded.com/medicalpubs/diseasemanagement/nephrology/hypokalemia-and-hyperkalemia/

49. Patient missed his insulin injection. Most likely to be found in urine analysis?
A. Ketones.
B. Proteins.
Answer: a

50. Urology pt had papillary cancer removed biopsy shows removal of all tumor till muscle layer, what next?
A. Follow up with cystoscopy
B. Given something medication i think chemo and other two options
Answer:
For patients at low risk of recurrence following turbt, we recommend an immediate, single postoperative dose of chemotherapy. This
is considered sufficient in itself without the need for additional therapy, and bacillus calmette-guerin (bcg) is never given in this
setting. The most extensive data for intravesical chemotherapy are with mitomycin. Repeat urine cytology (particularly for high
grade cases) and cystoscopyare generally advised at three to six month intervals, depending on the number of tumor recurrences, for
the first four years and annually thereafter in the absence of tumor recurrence.
Ref. Uptodate


51. Post streptococcal glomerulonephritis. What is the treatment:
A. Steroid
B. Antibiotics
Answer: b
- Treatment is largely supportive, with management of fluid overload and hypertension with diuretics
- Most of cases resolve spontaneously (this is why biopsy is rarely needed)
- Antibioics should be given to eradicate the organism from the pharynx
- Kaplan usmle step2

52. Patient on diuretics developed diarrhea. What type of electrolyte imbalance?


A. Hypokalaemia.
B. Hyperkalaemia.
Answer: a

53. According to henoch schonlein purpura, which of the following carries bad prognosis :-
A. Renal failure
B. Hepatic failure
Answer: a
In adults, kidney involvement progresses to end-stage renal disease (esrd) more often than in children

54. Female patient swallow 1 liter of car anti-freeze water (ethylene glycol) what is going to happen?
A. Interstitial nephritis
B. End stage renal disease
Answer: can be b not sure if a is correct. Calcium oxalate crystals may form and accumulate in the renal cortex results in decreased
glomerular filtration and renal insufficiency. Oliguric or anuric renal failure is the result in the most severe cases and, although
permanent renal failure is rare, recovery of renal function may take up to two months.

81
Http://www.atsdr.cdc.gov/csem/csem.asp?Csem=12&po=10
http://www.aafp.org/afp/2002/0901/p807.html

55. Patient complaining of hematuria with wbc in urine and the culture is negative. Cystoscopy, revealed submucosal
hemorrhage what is the cause:
A. Cystolithiasis
B. Interstetial cystitis

Answer: b
Interstitial cystitis is a clinical syndrome characterized by daytime and nighttime urinary frequency, urgency, and pelvic pain of
unknown etiology. Interstitial cystitis has no clear etiology or pathophysiology.
Http://emedicine.medscape.com/article/2055505-overview

56. Long scenario about patient presented dry cough after being diagnosed with HTN what is the cause:
A. Furosemide
B. ACEI (they mentioned the drug name )

Answer: b
Angiotensin converting enzyme (ace) inhibitors are the treatment of choice in patients with hypertension, chronic kidney disease,
and proteinuria. Ace inhibitors reduce morbidity and mortality rates in patients with heart failure, patients with recent myocardial
infarctions, and patients with proteinuric renal disease. Ace inhibitors appear to act primarily through suppression of the renin-
angiotensin-aldosterone system.
Http://emedicine.medscape.com/article/241381-medication#5

57. The most common cause of renal failure is:


A. Diabetes mellitus
B. Hypertension
Answer: a
adults with diabetes or high blood pressure, or both have a higher risk of developing ckd than those without these diseases.
Approximately 1 of 3 adults with diabetes and 1 of 5 adults with high blood pressure has ckd.
Http://www.cdc.gov/diabetes/pubs/pdf/kidney_factsheet.pdf

58. Differential diagnosis of renal filling defects:


A. Tumor
B. Stones (show acoustic shadow on us, may show calcification)
Clot (history of severe hematuria, resorted in follow up


59. A patient with dyspnea, pallor, edema of legs and itching. What is the diagnosis?
A. Scabies
B. Renal failure

Answer: B

60. Case of DKA with metabolic acidosis. What is the early mechanism to restore blood ph?
A. Excretion of CO2 through lungs.
B. Excretion of lactic acid through kidneys.

Answer: A
Reference: https://en.wikipedia.org/wiki/Acid%E2%80%93base_homeostasis
Http://fitsweb.uchc.edu/student/selectives/timurgraham/Compensatory_responses_metabolic_acidosis.html

61. HTN pt with decrease gfr ;


82
A. Bilateral renal artery stenosis
B. DM nephropathy
I didn’t understand the question. Do they mean that bilateral renal artery stenosis causes HTN and decrease gfr?


62. A patient with signs and symptoms of renal and respiratory involvement. What is the diagnosis?
A. Glomerulonephritis
B. Wegener's granulomatosis

Answer: B?
Incomplete question but you should to keep one thing in your mind if they mentioned there is a Hx. Of sinusitis the diagnosis with be
100% Wegners
Reference: First Aid step 1
Reference: Master the boards


63. 40 year-old was missing. 3 days later, his colleagues found him in his home. He was thirsty and vomiting. High Ca<<<what is
the treatment?
A. Hydration

64. Patient has HTN control on his medications, developed albuminuria, what you should add to his HTN medications:
A. ACEI
Angiotensin converting enzyme (ace) inhibitors are the treatment of choice in patients with hypertension and chronic kidney disease or
proteinuria.
Source: http://emedicine.medscape.com/article/241381-medication
Http://emedicine.medscape.com/article/238158-treatment#d9

65. Ph.: 7.2 , hco3 : 25 , co2 : 60:


A. Respiratory acidosis


• Step 1

• Step 2→ anion gap= 2 x serum na – cl + bicarbonate. Normal{8-12} ~ 10


• Step 3 → third component i.e. Compensatory mechanism
∆ #$%&$ '()
= [> 2→ metabolic alkalosis/ 1-2 → metabolic acidosis/<1 → non- anion gap]
∆ *%+(,-&$(./

83
66. High k (hyperkalemia), ecg showed wide qrs, how to treat:
A. Give Ca gluconate
Answer: a
- Ca gluconate: reverse ecg changes through membrane stabilization (most emergent treatment)
- Sodium bicarbonate: shifts k ions into cells
- Insulin: drives k ions intracellular (takes 30-60 min)
- Glucose: to prevent hypoglycemia
- Beta agonist: shifting potassium into the intracellular compartment
- Loop diuretics (frusemide): enhances renal potassium excretion and thus lower serum levels
- Http://emedicine.medscape.com/article/240903-medication#1

67. A 55 years old female has htn, fluid overload and azotemia what is the diagnosis:
A. Tubular acidosis (was one of the choices)??
Answer: acute glomerulonephritis:
Acute gn is defined as the sudden onset of hematuria, proteinuria, and red blood cell (RBC) casts. This clinical picture is often
accompanied by hypertension, edema, azotemia (ie, decreased glomerular filtration rate [gfr]), and renal salt and water retention.
Http://emedicine.medscape.com/article/239278-overview



68. What’s the most common organism in uti?
Answer: no choices
The most common organism of uncomplicated cystitis is e. Coli.
Sources: https://yhdp.vn/uptodate/contents/mobipreview.htm?13/15/13561#h899949156

69. Patients with HTN developed s hypercalcimea. What is the treatment?


Answer: furosemide
Loop diuretics act by competing for the chloride site on the na-k-2cl cotransporter. Inhibiting sodium chloride reabsorption also inhibits
the backleak of potassium and the generation of the lumen-positive potential. As a result, calcium excretion rises, an effect that may be
exploited in the treatment of hypercalcemia in selected patients.
Http://www.uptodate.com/contents/diuretics-and-calcium-balance
Factors that can aggravate hypercalcemia, including thiazide diuretic and lithium carbonate therapy, volume depletion, prolonged bed
rest or inactivity, and a high calcium diet (>1000 mg/day).
Http://www.uptodate.com/contents/treatment-of-hypercalcemia?Source=see_link&sectionname=saline+hydration&anchor=h4#h4

70. Non opaque stone on radiography. What type of stone?


Answer: uric acid

71. Renal failure with azotemia?


A. Bilateral renal artery stenosis.
Answer: missing info


72. Diagnostic tests for pheochromocytoma include the following:
- plasma metanephrine testing: 96% sensitivity, 85% specificity.
- 24-hour urinary collection for catecholamines and metanephrines: 87.5% sensitivity, 99.7% specificity.
Http://emedicine.medscape.com/article/124059-overview

73. Drug induced hyperuricemia:


Hydrochlorothiazide and furosemide. Medscape. Http://www.medscape.com/viewarticle/823994_8

84
74. Dm, HTN developed microalbominuria what to give to prevent renal failure
Ace inhibitors and angiotensin receptor blockers (arbs). Bmj http://bestpractice.bmj.com/best-
practice/monograph/530/prevention/screening.html

75. Gfr 80 in this stage what happen?


Mesangial proliferation and thinking of basement membrane. Not complete question.

76. Pt had hx of infection 2 week later present with hematuria.


Answer: post strephematuria.

77. Boy hematuria +snhl his father has end stage renal disease and snhl wt diagnosis?
Answer: alport syndrome
Alport syndrome encompasses a group of inherited, heterogeneous disorders involving the basement membranes of the kidney and
frequently affecting the cochlea and eye as well.


78. Patient with bilateral flank pain for 6 months and there is gene 16 mutation , what is the disease
A. Adult polycystic kidney disease

Answer : a

o Adpkd (adult) type 1 on the short arm of chromosome 16, type 2 on the long arm of chromosome 4, type 3 no genomic locus is
assigned
o Arpkd (pedia) on the short arm of chromosome 6.

Reference: medscape.

79. Early sign of hypomagnesemia?


A. Loss of deep tendon reflex
Answer: the reflexes in hypomagnesemia are hyperactive

Neuromascular Cardiovascular Metabolic


A. Muscular weaknes • Non specific t wave changes.
B. Tremor ,seizure, • Prolonged qt and qu interval.
parasthesia, and • Repolarization alternans.
tetani • Premature ventricular
• Hypokalemia
C. +ve chvostek and contraction and
trousseau signs • Hypclacemia
monomorphic ventricular
D. Vertical and tachychardia
horizontal • Torsade de point
nastygmus • Vfib, enhance digitalis toxicity
Reference: medscape

80. Why is inulin used to measure gfr?


A. Freely filtered by glomeruli

Answer: a (similar to q)

85
A. Recall that all of the plasma that is filtered and only the plasma filtered is cleared of inulin so that if one were to measure
the clearance of inulin, it would equal the amount of plasma filtered in a minute, the glomerular filtration rate. Therefore, the
clearance of inulin is equal to the glomerular filtration rate, the volume of plasma filtered in one minute.
Reference: http://www.austincc.edu/emeyerth/clearancehtm.htm
Freely filtered by glomeruli; inulin is the most accurate substance to measure because it is a small, inert polysaccharide molecule that
readily passes through the glomeruli into the urine without being reabsorbed by the renal tubules. Britannica
http://global.britannica.com/topic/inulin-clearance

81. Calculate anion gap with corrected na 138


Answer:
Anion gap = na - ( cl + hco3 ) or (na + k) – (cl + hco3) normal range are varies depend on references (4- 14)
The na at the anion gap equation should be the measured na not corrected due to it will lead to false elevated calculation
Reference: pocket of medicine, medscape, clinical background

82. Polycystic kidney inheritance?


A. Dominant
Answer: autosomal dominant polycystic kidney disease (adpkd) is a multisystemic and progressive disorder characterized by cyst
formation and enlargement in the kidney and other organs (eg, liver, pancreas, spleen).
(http://emedicine.medscape.com/article/244907-overview)


83. Hydrochlothizaied fursemide what is the type of that something that is bind to ca in psudogout ?
- Pyrophosphate.
It is called calcium pyrophosphate.

84. Case of congenital adrenal hyperplasia?


A. Give steroid
Treatment of congenital adrenal hyperplasia: correct electrolyte & give glucocorticoids and mineralocorticoids if needed ( toronto 15
p13)

85. Patient with bilateral flank swelling dull move with shifting ?
A-ascitis
Answer:a
Reference: https://depts.washington.edu/physdx/liver/tech.html

86. Case of hyperosmolar state.


Article on hyperosmolar hyperglycemic state: http://emedicine.medscape.com/article/1914705-overview

87. Something about anion gap (causes)?


Mnemonic is karmel.
• K — ketoacidosis
• A — aspirin
• R — renal failure
• M — methanol
• E — ethylene glycol
• L — lactic acidosis
Further about anion gap:
Http://emedicine.medscape.com/article/2087291-overview#a2

88. Case about degree of dehydration?

86

Http://www.merckmanuals.com/professional/pediatrics/dehydration-and-fluid-therapy-in-children/dehydration-in-children

89. Patient diagnosed with papillary renal carcinoma >> ttt.


Papillary renal cell carcinoma (prcc) is one of the subtypes of renal cancer. This sub type may account 13-20% of all renal
cell cancer. There is slightly increased male predilection. As with other types of renal cell cancer, most are asymptomatic
and incidentally discovered.
Http://radiopaedia.org/articles/papillary-renal-cell-carcinoma

90. Anion gap if corrected na 138 >> choose the closest number to 20

91. Post infection the abx was formed (low c3)



92. Male had +ve semen abx , what is the cause:
exposure of ag of semen to the blood stream -as in trauma –

93. About chronic granulomatous disease!


94. Questions about bph tx "medication and surgical, and one about se!

95. Tx of sle pt with urti


96. Wbc given, pale pt > iron deficiency anemia

97. Boy referral due to having recurrent chest infections & has brother die at 6yrs as same chest infection sister normal all
immunoglobulins low, t cell function is good ?
A. X-linked agammaglobulinemia


98. Child given heparin blood +ftp the dz is >>> dic with thromboses

99. Case of renal artery stenosis (investigation)

100.Pt c/o hematuria and cough with saddle nose


A. Wegener granulomatosi . Medscape http://emedicine.medscape.com/article/332622-overview

101.A patient with glomerulonephritis developed hemoptysis. What is the most likely diagnosis?
A. Goodpasture syndrome.
87
Answer: A
Frank hemoptysis suggests Goodpasture syndrome (glomerulonephritis and pulmonary hemorrhage associated with anti GBM
antibody) but this also can be a prominent feature of systemic vasculitis. Reference: Diseases of the Kidney and Urinary Tract
textbook edited by Robert W. Schrier.
NB. Wegner’s was not included in the choices.

102.A patient presented with hemoptysis and signs of nephropathy. Biopsy of the lung showed presence of anti-GBM antibodies.
What is the most likely diagnosis?
A. Goodpasture syndrome.

Answer: A
Reference: http://emedicine.medscape.com/article/981258-overview

103.Histopathology of minimal change nephrotic syndrome?


The glomeruli appear normal on light microscopy in patients with and there are no complement or immunoglobulin deposits on
immunofluorescence microscopy. Glomerular size is usually normal by standard methods of light microscopy, although enlarged
glomeruli may be observed
The characteristic histologic lesion in MCD is diffuse effacement (also called "fusion") of the epithelial foot processes on electron
microscopy. More specifically, there is retraction, widening, and shortening of the foot processes. The spaces between flattened
podocyte foot processes are reduced in number and support the burden of plasma filtration, which may play a role in the excess
albumin load into the urinary space. The degree of effacement does not correlate with the degree of proteinuria. Foot processes regain
a normal appearance with remission of proteinuria
Reference: http://www.uptodate.com/contents/etiology-clinical-features-and-diagnosis-of-minimal-change-disease-in-
adults?Source=machinelearning&search=minimal+change+nephrotic+syndrome&selectedtitle=2~104&sectionrank=5&anchor=H2#H
58052618

104.Pt c/o of loin pain & hematuria and inherited autosomal dominant what the diagnosis?
A. Polycystic kidney
Answer: polycystic kidney disease.
Http://www.uptodate.com/contents/polycystic-kidney-disease-beyond-the-
basics?Source=outline_link&view=text&anchor=H1#H1
Http://emedicine.medscape.com/article/244907-overview
Http://emedicine.medscape.com/article/1958746-overview flank pain & hematuria differentials

105.Low ph, bicarb, co2?


A. Compensated metabolic acidosis

Answer: A
This image for revision

88
106.In addition to anti- HTN , what to advice ?
Answer: Restrict Na to 9mg ,Walking ( such meters)
Not sure about it since we do not have the other choices.
''Control of hypokalemia and hypertension in IHA can be achieved with sodium restriction (to < 2 g/day) and administration of
spironolactone or amiloride, but additional antihypertensives are often needed to achieve good control in this patient group''

107.Best investigation to measure GFR


Answer: 24 urine creatinine collection
Other options were irrelevant
''Inulin is seldom used in clinical testing, although it is used in research'', So the answer will be mostly 24 urine creatinine
measurement.

108.Young adult having episodic palpitation and fear and tightness. Btw the attack she feel fatigue? What investigation should
you order?
a. Urine catecholamine test

Answer: ?
• Pheochromocytoma is a nonmalignant lesion of the adrenal medulla autonomously overproducing catecholamine’s despite
a high blood pressure.
• The classic history of a patient with a pheochromocytoma includes spells characterized by headaches, palpitations, and
diaphoresis in association with severe hypertension. These 4 characteristics together are strongly suggestive of a
pheochromocytoma. In the absence of these 3 symptoms and hypertension, the diagnosis may be excluded.
• Best initial tests:
o High plasma and urinary catecholamine
o Plasma-free metanephrine and VMA
• Most accurate test:
o CT or MRI of the adrenal glands
o Metastatic disease is detected with an MIBG scan
Reference: Master the Boards & Medscape

89

Hematology

90
1. A patient presented with fatigue, palpitation, sob and pallor, hgb 9. Shown is the peripheral film. What is the type of anemia?

A. Megaloblastic anemia
B. Hypochromic microcytic
C. Sickle cell
D. G6pd

Answer: a
Note that RBCs are as large as the neutrophil and lymphocyte. Heinz bodies is in g6pd
Reference: https://labtestsonline.org/understanding/analytes/blood-smear/details/


2. Long standing trip, swelled ll, no pain, high d-dimer management?
A. Aspirin
B. Lmwh
C. Warfarin
D. Unfractioned heparin with warfarin

Answer: d (best combined???)


Admitted patients may be treated with a lmwh, fondaparinux, or unfractionated heparin (ufh). Warfarin 5 mg po daily is initiated and
overlapped for about 5 days until the international normalized ratio (inr) is therapeutic >2 for at least 24 hours.
Patients treated with lmwh or fondaparinux do not require monitoring of the aptt.
Platelets should be monitored. Heparin or lmwh should be discontinued if the platelet count falls below 75,000. Fondaparinux is not
associated with heparin-induced thrombocytopenia (hit).
Warfarin is long-term anticoagulant.
References: http://emedicine.medscape.com/article/1911303-treatment
Http://bestpractice.bmj.com/best-practice/monograph/70/treatment/step-by-step.html
Http://www.aafp.org/afp/2007/0401/p1031.html

3. Post streptococcal infection generalized petechia and plt =15 management?


A. Splenectomy
B. Cyclo
C. Viii
D. :ivig

Answer: d
9
It is a case of immune thrombocytopenic purpura (ITP). In adults, treatment is recommended for a platelet count <30×10 /l. The ash
recommends that if treatment is needed and corticosteroids are given, longer courses (eg, prednisone 1 mg/kg orally for 21 days
then tapered) are preferred over shorter courses of corticosteroids or ivig as first-line treatment. Ivig be used with corticosteroids in
patients who require a more rapid increase in platelet count. If corticosteroids are contraindicated, either ivig (initially, 1 g/kg in a
single dose) or IV rhig (in appropriate patients) may be used as a first-line treatment.
Reference: http://emedicine.medscape.com/article/202158-treatment

91
4. A patient was referred from cardiac unit due to severe decrease in platelets which were < 10,000. Patients is known to be
using heparin: what is the treatment:
A. Platelets transfusion.
B. Argatroban
C. Ivig
D. Steroid.

Answer: b
Explanation: there are two typer of heparin induces thrombocytopenia: type 1 which occurs in the 1st 2 days of therapy with platelets
normalizing with continuing therapy, and type 2 which is immune mediated and occurs 4-10 days of initiating therapy. Hit should be
suspected when platelets count decrease 50% of baseline even of nadir is still above 150x109
Management of hit type 2 is by first discontinuing heparin, then giving the patient argatroban, a direct thrombin inhibtor. Platelet
tranfusion should be avoided as it may increase the thrombogenic effect.
Reference: http://emedicine.medscape.com/article/1357846-treatment

5. Rheumatoid arthritis patient on treatment for 15y now came with anemia , the pt wasn't on any NSAIDs or asa what is the
type of anemia :
A. Macrocytic hyperchromic
B. Microcytic hypochromic
C. Normocytic normochromic
D. Macrocytic hypochromic

Answer: c
Ra causes anemia of chronic inflammation. This type of anemia is usually normocytic and normochromic unless if it was severe it
could be microcytic hypochromic
Reference: toronto notes


6. Long case about patient eat rice only with loss of coordination between upper and lower extremity with tongue and lip rash ,
vitamin loss suspected what is this Vitamin ?
A. B1
B. B3
C. B6
D. B12

Answer: a
thiamin deficiency is seen:
As beriberi is seen with patients who consume only polished rice, in chronic alcohol-dependent patients and in starved patients.
Patients with berberi present with a symmetrical polyneuropathy. The initial symptoms
are heaviness and stiffness of the legs, followed by weakness, numbness, and pins and needles. The ankle jerk reflexes are lost and
eventually all the signs of polyneuropathy that may involve the trunk and arms are found. Cerebral involvement occurs, producing
the picture of the wernicke–korsakoff syndrome.
Reference: kumar and clark’s clinical medicine

7. Scenario of a patient with malaria but no history of bleeding and give a peripheral smear of ring form of malaria what is the
type of malaria?
A. Plasmodium malariae
B. Plasmodium ovale
C. Plasmodium vivax
D. Plasmodium falciparum
Answer: d
Reference: http://www.malariasite.com/microscopic-tests/
Http://parasitologyillustrated.com/classes_of_parasites/protozoa/sporozoa/p_falciparum.html

8. Most abundant cells in peripheral smear?

92
A. Lymphocyte
B. Basophils
C. Neutrophil
D. Eosinophils
Answer: c
** neutrophils are the most abundant white blood cell, constituting 60-70% of the circulating leukocytes.
** basophils are the least abundant leukocyte.
Reference: http://www.histology-world.com/testbank/blood2a.htm

9. A vegetarian sle patient complaining of fatigue and tiredness. What will you find ?
A. Low iron low tibc
B. Low iron high tibc
C. High iron high tibc
D. High iron low tibc
Answer: a, if it mean anemia of chronic disease i think
Anemia is a common clinical finding in patients with systemic lupus erythematosus (sle).1-3 the most common form of anemia in these
patients is that of chronic disease (acd),1 however autoimmune hemolytic anemia (aha), iron deficiency anemia (ida), drug induced
myelotoxicity, and anemia of chronic renal failure are probably not uncommon.
Http://ard.bmj.com/content/59/3/217.full

10. What will you see in malaria rapid diagnostic test (rdt)?
A. Malaria antigens
B. Malaria antibodies
C. Malaria pigments
D. Parasite sexual
Answer: a
- Immunochromatographic tests detect the presence of malaria antigen or enzyme
- The main advantage of rdts is that they provide a means for rapid diagnosis, especially in health resource-limited areas where
microscopy is not available or reliable
- Http://bestpractice.bmj.com/best-practice/monograph/161/diagnosis/tests.html

11. A young male came back from visiting his family in sudan 2 weeks ago, now he presented in er complaining of severe headache,
fever and vomiting. Which of the following you will do immediately?
A. Blood culture
B. Stool culture
C. Peripheral blood smear
D. Lumbar puncture
Answer: c
- Malarial infection is suspected. Individuals are generally asymptomatic for 12 to 35 days but can commence symptoms as early
as 7 days (depending on parasite species) in most cases, the incubation period for p. Falciparum infection is about 12 to 14 days
(range 7 to 30 days); most infections due to p. Falciparum become clinically apparent within one month after exposure.
- Detection of parasites on giemsa-stained blood smears by light microscopy is the standard tool for diagnosis of malaria and
remains the most common onsite diagnostic method
o Source: https://yhdp.vn/uptodate/contents/mobipreview.htm?10/0/10248

12. Chronic myelogenous leukemia is associated with which translocation :


A. T (14; 18)
B. T ( 11;14)
C. T (8; 14)
D. T (9; 22).
Answer: d
Explanation: cml is associated with the fusion of two genes: bcr (on chromosome 22) and abl1 (on chromosome 9) resulting in the bcr-
abl1 fusion gene. This abnormal fusion typically results from a reciprocal translocation between chromosomes 9 and 22, t(9;22) that
gives rise to an abnormal chromosome 22 called the philadelphia (ph) chromosome.
Reference: http://cursoenarm.net/uptodate/contents/mobipreview.htm?33/25/34192 \
93
13. Case of pt with hemochromatosis what will be increased?
A. ……..
B. Cureloplasmin
C. Irrelevant
D. Ferittin
Answer: d

14. Long scenario with paragraph about patient who has low hemoglobin and macrocytic anemia. He treated now what does brown
line mean if the blue is hemoglobin
A. Reticulocyte
B. Hematocrit
C. RBCs
D. MCHc
Answer: look like picture in attachment. There’s no picture?

15. Patient with fatigue and pallor with blood values that show slightly low hemoglobin, wbc and platelets. Mcv and normal
reticulocyte are normal. Diagnosis?
A. Iron deficiency
B. Folate deficiency
C. Aplastic
D. Hypoplastic
Answer: c
Iron deficiency anemia will show low mcv, and folate deficiency will show high mcvc. Aplastic anemia shows normal mcv with a
decrease in all cell line but the reticulocyte count will be low
Toronto notes 2015


16. Patient with prostatic ca has 2 dvt how to prevent further dvt?
A. Short term lmwh followed by warfarin
B. Lmwh for 6 months
C. I think aspirin
D. Another long answer
Answer: b http://jco.ascopubs.org/content/25/34/5490.full

17. 75 years old male, asymptomatic, bm report: increased lymphocytes, immunohistochemestry: positive cd19, cd56. What is
the treatment?
A. No treatment
B. Rituximab + cvb
C. Rituximab + prednisolone
D. Cyclophosphomide
Answer:

18. 24 years old female, has 3 lan at the same side of diaphragm with no distant metastasis, diagnosed with hodgkin’s
lymphoma. What's the stage?
A. 4
B. 3
C. 2
D. 1
Answer: c

94
19. Pt with petechia and bruises labs normal except prolonged ptt due to which factor?
A. V
B. Vii
C. Viii
D. X

Answer: c
Aptt measures deficiencies in the intrinsic pathway
Http://emedicine.medscape.com/article/2085837-overview#a2


20. How to stop bleeding in vwd?
A. Fresh frozen plasma ❌
B. Vit. K
C. Platelets transfusion
D. Something irrelevant !!
Answer:
• desmopressin (ddavp®) is treatment of choice for type 1 vwd ƒ causes release of vwf and factor viii from endothelial cells ƒ
variable efficacy depending on disease type; tachyphylaxis occurs ƒ
• tranexamic acid (cyklokapron®, antifibrinolytic) to stabilize clot formation
• high-purity factor viii concentrate containing vwf (hemate p®) in select cases and type ƒ
• frozen plasma (fp) and cryopreciptate ~> avoid
• conjugated estrogens (increase vwf levels)
• platelet transfusions may be helpful in some patients with vwd (eg, type 3) to control bleeding that is refractory to other
therapies.
Answer : neither desmopressin nor recombinant vwf concentrate were in the choices
Http://emedicine.medscape.com/article/206996-treatment


21. Which of the following is associated with burkitt’s lymphoma?
A. EBV
B. Hiv
C. Coxsackie-virus
D. HBV
Answer: a
Burkitt lymphoma (bl) is an aggressive b-cell malignancy with endemic, sporadic and immunodeficiency-associated variants. It has
been known for many years that the fundamental transforming event in bl is the translocation of the myc gene, and the events that
bring about this translocation and those that allow cells to survive with the constitutive expression of myc have been the subject of
intense investigation. Epstein–barr virus (EBV) infection, malaria, immunodeficiency and spontaneous, somatic mutation can all
contribute to the origin and maintenance of this cancer and their mechanisms are the subject of this review.

95
Reference: http://www.ncbi.nlm.nih.gov/pmc/articles/pmc2095571/


22. Lab result show high aptt and bt (bleeding time), factor deficiency?
A. 8
B. 7
C. 9
D. 10
Answer: a


Aptt is prolonged in both hemophilia a (factor 8 deficiency) and hemophilia b (factor 9 deficiency) and bleeding time should be
normal in both. In vwf disease (factor 8 + vwf factors deficiency) both aptt and bleeding time are prolonged. However vwd is not
one of the options.
Reference: step up to medicine

23. Young sickler patient had hx of tiredness and fatigue within 10 hrs, drop in hb and palpable liver and spleen 6 cm below the
costal margin, had 3 previous similar episodes. What you will do:
A. Splenectomy
B. Start hydroxyurea
C. Reticulocyte count
D. Regular blood transfusions**********
Answer: a or d
If hemoglobin drop > than 2 or showing s/s of anemia: transfusion
If recurrence >3 times: splenectomy


24. Boy with elevated wbcs very high ?? Low hemoglobin low mcv low reticulocytes what is the diagnosis?
A. Iron deficiency
B. B thalassemia trait
C. Sickle
D. Anemia of chronic disease
Answer: d
If the wbc are high due to inflammation and chronic. Http://www.aafp.org/afp/2010/1101/p1117.html

25. Thalassemia hb f 40 hb a2 20
A. Alfa thalassemia
B. Beta thalassemia minor
C. Beta thalassemia major
D. Thalassemia intermediate
answer: b
96

Reference : https://yhdp.vn/uptodate/contents/mobipreview.htm?41/24/42380&utdpopup=true



26. Theoretically which of the following cancer will be prevented by vaccination?
A. All
B. Cml
C. Adult t cell leukemia
D. Mycosis fungoides
Answer: c
Atl/l is more common in adults who have been exposed to the human t-cell leukemia/lymphoma virus, type 1 (htlv-1). That’s why it
can theoretically be prevented by vaccination.
Refrence: http://www.cancer.ca/en/cancer-information/cancer-type/non-hodgkin-lymphoma/non-hodgkin-lymphoma/types-of-
nhl/adult-t-cell-leukemia-lymphoma/?Region=on


27. Constant defect in von willbrand :
A. Pt
B. B- ptt
C. Prolonged bleeding
D. Factor viii.
Answer: factor viii activity
reference: cecil medicine table in hemorrhagic dz.

28. Long case of boy bleeding epistaxis and ecchymosis with long lab results showing anemia thrombocytopenia and leukopenia,
what is the diagnosis:
A. Ida
B. Aplastic
C. Hypoplastic
D. Hemolytic

Answer: b
Aplastic anemia
A. Symptoms of anemia, thrombocytopenia, and/or infection.
B. Investigation (cbc): anemia or neutropenia or thrombocytopenia (any combination) ± pancytopenia. Decreased
reticulocytes (<1% of the total RBC count)
Reference: toronto notes; (p552)

97

29. Blood film attached, asking for diagnosis:
A. Leishmaniasis
B. Malaria
C. Lymphoma
D. Leukemia

Answer: according to the slide

30. How to monitor the response to iron treatment?


A. Ferritin
B. Hct
C. RBC
D. Reticulocyte count

Answer: D
Monitoring response:
a. Reticulocyte count will begin to increase after one week
Hb normalizes by 10 g/L per week (if no blood loss)
● Iron supplementation required for 4-6 mo to replenish stores.
Reference: Toronto notes

31. Asymptomatic patient, known case of chronic gastritis, has positive occult blood stool and his Hb=9. You will manage him by:
A. IM iron
B. Oral iron
C. Erythropoietin
D. Blood transfusion

Answer: A (IV iron would be more appropriate answer)

32. Elderly man on NSAIDs developed dyspepsia. Endoscopy showed gastritis. Labs showed iron deficiency anemia with Hb= 9.
What is the treatment?
A. IV iron
B. IM iron
C. Erythropoietin
D. Oral Iron
Answer: A
Oral ferrous sulfate associated with a significantly higher risk of GI side effects than IV iron. Acquired malabsorption for iron with
autoimmune atrophic gastritis or Helicobacter pylori infection
Reference: http://www.uptodate.com/contents/treatment-of-the-adult-with-iron-deficiency-anemia

33. A male patient presented with symptoms. Labs showed 80% blasts with 20% Aurer rodes. What is the diagnosis?
A. CML
B. AML
C. CLL
D. ALL

Answer: B
Auer rods are a hallmark of acute myeloid leukemia.

34. Old guy with lymph node enlargement and B-symptoms. What is the treatment?
A. CHOP-R

98
B. ABVD
C. Rituximab
D. CHOP

Answer: ? (It is not clear from the question whether it is HL or NHL)


B symptoms: fever, night sweats, weight loss
● NHL:
Local disease (stage Ia and Ila): local radiation and small dose/course of chemotherapy
Advanced disease (stage III and IV, any "B" symptoms): combination chemotherapy with CHOP and rituximab, an antibody
against CD20 (CHOP-R)
• HL:
Stage Ia and Ila: local radiation with a small course of chemotherapy
Stage III and IV or anyone with "B" symptoms: ABVD
*A = adriamycin (doxorubicin)
B = bleomycin V = vinblastine D = dacarbazine
Reference: Master The Board USMLE Step 2 CK

35. Which of the following can be found on smear in sickle cell disease?
A. Bite cells
B. Howell-Jolly bodies
C. Acanthocyte
D. Spherocyte

Answer: B
Reference: Master the Boards.

36. Most comman cause of death in sicke cell anima :


A. Aplastic crises
B. Seqseration crises
C. Acute chest syndrome
D. Parovirus b19



37. Sickle cell anemia false positive test because?
A. High protein level
B. Protein c
C. Protein d

Answer: a (high paraprotien like multiple myloma)


False positives may occur in patients with erythrocytosis, hyperglobulinemia, extreme leukocytosis or hyperlipidemia. Coarse
flocculation may occur in these samples due to elevated levels of total serum protein. These patient samples may be washed in
normal physiologic saline and centrifuged to minimize these problems.
False positives or false negatives may occur in patients with severe anemia hematocrit ≤ 15%.
False negatives may occur in infants under six months of age due to elevated levels of hemoglobin f.
False positives or false negatives may occur in patients with a recent blood transfusion.
Positive results may occur in patients with some rare sickling hemoglobin subtypes such as hemoglobin c harlem or hemoglobin c
georgetown.
References: http://education.questdiagnostics.com/faq/faq99
Https://www.streck.com/resources%5chematology%5csickledex%5c03_product_information%5c02_paper_sickledex_-
_screening_test_guidelines.pdf


38. Patient with osteoarthritis, you found that he developed anemia which type it would be:
A. Normochromic normocytic
B. Microcytic hypochromic (if with NSAID)

99
C. Macrocytic hyperchromic

Answer: a


39. Which type of anemia is associated with hyposplenism?
A. Sickle cell anemia
B. Thalassemia
C. Spherocitosis

Answer: a
Autosplenectomy is the physiological loss of spleen function (hyposplenism). It is associated with sickle cell anemia (chronic damage
to the spleen results in atrophy)
Reference: master the boards


40. Patient with past hx of hodgkin lymphoma, but cured completely. Presented with back pain examination and evaluation
show para-spinus edema and fluid collection -ve brucella titer and tuberculin test, what the cause?
A. Brucellosis
B. Breast cancer
C. Recurrent hodgkin lymphoma

Answer: b
Patients with hodgkin’s lymphoma who undergo chest radiotherapy have higher risk of developing breast cancer
Reference: http://www.ncbi.nlm.nih.gov/pmc/articles/pmc2409557/


41. Sickle cell anemia pt came complaining of cough & shortness of breath, there is splenomegaly, lab shows (wbc= low, hb= low,
retic= .04) what is the initial management of this case
A. Fluid & analgesia
B. Blood transfusion
C. Splenectomy

Answer: a
Http://bestpractice.bmj.com/best-practice/monograph/100/treatment/details.html

42. Pt with mild jaundice, hepatomegaly, fingers & toes parasthesia. Lab show (wbc= normal, b12=low, folate= normal, tbil=27,
dbil= 6)
A. Liver cirrhosis
B. Pernicious anemia
C. Folate def

Answer: b
Http://www.ncbi.nlm.nih.gov/pmc/articles/pmc4316289/
Http://bestpractice.bmj.com/best-practice/monograph/822/diagnosis/history-and-examination.html

43. Anemia+ neurological symptoms which Vitamin ?


A. B3
B. Folic acid
C. B12

Answer: c
Reference: step-up to medicine 3rd edition, page 330

100
44. Old age with lab results: hgb: low, lymphocyte: high, flow cytometry: different types of cd what is the treatment ?
A. Cyclophosphamide -
B. Rituximab +prednisolone -
C. Rituximab
Answer: ?
**the diagnosis is chronic lymphocytic leukemia

Reference: http://www.cancer.org/cancer/leukemia-chroniclymphocyticcll/detailedguide/leukemia-chronic-lymphocytic-treating-
treatment-by-risk-group
Http://www.oncolink.org/types/article.cfm?C=293&id=9590

45. 16 years old boy known case of sickle cell anemia presented to with painful right hip pain for several weeks (this was the
scenario and it was for several weeks ). What is the most likely diagnosis:
A. Avascular necrosis common in sicklers
B. Still’s disease
C. Tumor.
Osteomyelitis was not in the choices
Answer: avascular necrosis

46. Women (complain is not written). Her labs: high platelets. What’s the treatment?
A. Over-the-counter, low-dose aspirin to reduce blood clotting
B. Prescription medications to lower the risk of clotting or to reduce platelet production in the bone marrow
C. Platelet pheresis
Answer: not sure

47. Microcytic normochromic result in patient with pallor


A. Iron deficiency anemia
B. Hemolytic anemia
C. Folate deficiency
Answer: ????

48. Case of pt with malignancy , dr is planning to take the malignant cells and implant it in the pt dendritic cells to enhace immunity
against tumor cells what is this called :
A. Active immunotherapy
B. Passive immunotherapy
C. Weird name cant recall !!

49. Mcv high, ast high, no megaloblast cells:


A. B12 defe
B. Folate defeci
C. Alcohol abuse
Answer: c
Explanation:

101
50. Young patient presented with ptosis miosis depressed orbit and in the other eye there’s something he also has a neck mass
what is the most likely diagnosis:
A. Hodgkins lymphoma
B. Ewing's sarcoma
C. Wilms tumor
Answer: a

51. Factor vii (7) deficiency cause prolongation of :


A. Bleeding time
B. Pt
C. Aptt

Answer: b
Inherited factor vii (fvii) deficiency is a rare autosomal recessive hemorrhagic disorder. Clinical bleeding can widely vary and does not
always correlate with the level of factor vii coagulant activity measured in plasma. Factor vii is one of the vitamin k–dependent
coagulation factors synthesized in the liver. The prothrombin time (pt) is prolonged in factor vii (fvii) deficiency and the international
normalized ratio (inr) is elevated.
Http://emedicine.medscape.com/article/960592-workup


52. Iron overdose with bleeding per rectum
A. Lavage
B. IV anti dot
102
C. Chelating
Answer: c. Chelating. Treatment of a substantial ingestion is usually whole-bowel irrigation and chelation therapy with IV
deferoxamine. Merck manual
Http://www.merckmanuals.com/professional/injuries-poisoning/poisoning/iron-poisoning

53. How to stop bleeding in vwd?


A. Fresh frozen plasma
B. Vitamin. K
C. Platelets transfusion
Answer: desmopressin or factor viii concentrate. Step up to medicine

54. False positive sickling tests:


A. Hg c
B. Hg d
C. Hyperglobulinemia
Answer: c
False positives may occur in patients with erythrocytosis, hyperglobulinemia, extreme leukocytosis or hyperlipidemia. Coarse
flocculation may occur in these samples due to elevated levels of total serum protein. These patient samples may be washed in
normal physiologic saline and centrifuged to minimize these problems.false positives or false negatives may occur in patients
with severe anemia (<15% hematocrit).false negatives may occur in infants under six months of age due to elevated levels of
hemoglobin f.false positives or false negatives may occur in patients with a recent blood transfusion.

55. Long history about dic, lab showing fragmented RBC, low platelets . Which antibodies are target:
A. Cardiolipins
B. Adamts13
C. Glycoprotein

Answer: ?



56. Long term treatment of sickle cell is:
A. Folic acid
B. B- penicillin
C. Hydroxurea
Aanswer : a (depending on question) or c
Merck manual:
For long-term management the following interventions have reduced mortality, particularly during childhood:
Pneumococcal, haemophilus influenzae, influenza (inactivated, not live), and meningococcal vaccines
Early identification and treatment of serious bacterial infections
Prophylactic antibiotics, including continuous prophylaxis with oral penicillin from age 4 mo to 6 yr use of hydroxyurea and folate
supplementation supplemental folate, 1 mg po once/day, is usually prescribed.

57. Pt known case of g6pd low Hg what will u do?


A. Blood transfusion
B. Folic acid
C. Reassure
Answer: c
Most individuals with (g6pd) deficiency do not need treatment. Hemolysis is self-limited and often resolves in 8 to 14 days. Bmj
http://bestpractice.bmj.com/best-practice/monograph/704/treatment/details.html#expsec-3

58. Methotrexate overdose, treatment?


A. Folic acid
B. Folonic acid

103
C. Cobalamine
Answer: a reference: http://www.medscape.com/viewarticle/588229

59. Which type of anemia have high hemoglobin a2:


A. Scd
B. Thalassemia
C. Spherocytosis
Answer: b
Link: http://thalassemia.com/what-is-thal-beta.aspx#gsc.tab=0

60. Pt with anemia, high bilirubin, positive direct and indirect coombs
A. Photo of smear showing spherocytosis, what is the dx:
B. A-aiha
C. B-spherocytosis
Answer: a
Laboratory findings in patients with warm agglutinin aiha include hemolytic anemia of varying severity, a reticulocytosis in response
to the anemia, the presence of spherocytic red cells on the peripheral blood smear, and a positive direct antiglobulin (coombs) test.
These findings are discussed in detail below. An example of the baseline characteristics of 60 patients with warm agglutinin aiha, seen
in a french tertiary-care national referral center for adult autoimmune cytopenias is shown in the table (table 1) [16].

61. Patient on high dose aspirin present with melena endoscopy shows gastritis what to give?
A. Im iron
B. Oral iron (no IV in the choices)
C. In hx there is typical feature of hand foot syndrome:
Answer: sca

62. A 60-year-old male with history of lower back pain , constipation thirst low hemoglobin , low wbc , low plt , high Caand
lower spinal x-ray pic was attached
what is the next appropriate to be ordered:
A. Dexa
B. Protein electrophoresis
C. Parathyroid hormone
Answer: b
Multiple myeloma: m protein appears as a narrow spike in the gamma, beta, or alpha2 regions. Skeletal lesions (e.g., lytic lesions,
diffuse osteopenia, vertebral compression fractures) are present in 80% of patients. Anemia, pancytopenia, hypercalcemia, and renal
disease may be present.
Http://www.aafp.org/afp/2005/0101/p105.html

63. Iron deficiency anemia in 2 year old child Hg 9 what to give?


A. Oral
B. Iv
C. Transfusion
Answer: a
Oral iron therapy: is initially started at a dose of 3 mg/kg of elemental iron, given once or twice daily *. Parenteral iron
therapy: parenteral iron therapy should be reserved for patients with severe, persistent anemia who have proven intolerance to oral
supplements, malabsorption, or poor compliance to oral therapy. Iron dextran is the parenteral form most commonly used preparation
for pediatric patients *.
Blood transfusion: transfusion therapy is rarely necessary for severe ida, even with hemoglobin concentrations of 4 to
5 gm/dl. Transfusions should be reserved for patients in distress (heart rate greater than 160/min, respiratory rate greater
than 30/min, lethargy, not feeding well). Transfusions should be administered with caution to such patients, giving transfusion
volumes of 5 ml/kg over three to four hours to avoid inducing heart failure *.
Https://yhdp.vn/uptodate/contents/utd.htm?38/2/38949?Source=see_link *
Severe anemia: hemoglobin below 7.0 g/dl or hematocrit below 20%
http://www.who.int/nutrition/publications/micronutrients/guidelines_for_iron_supplementation.pdf

104
64. Pregnant patient on 32 weeks of gestation presented to antenatal care for the first time. She has symptoms of anemia with
orthopnea and pnd. On examination she is vitally stable with grade 3 pitting edema bilaterally labs shows low hemoglobin
low iron high ferritin low tibcwhat is the diagnosis
A. A-preeclampsia
B. Anemia
C. Heart disease
Answer: c
The patient has anemia of chronic disease that eventually leads to heart disease (pitting edema).


65. Aging amyloid where to find it ?
A. Heart
B. Kidney
C. Liver
Answer: A
Age-related (senile) systemic amyloidosis — Deposition of otherwise normal (wild-type) transthyretin in
myocardium and other sites may result in a form of amyloidosis that is referred to as systemic senile
amyloidosis (SSA), UpToDate

66. 10 years old patient presented with general fatigue and severe bone pain in hands and feet. What is the diagnosis?
A. Sickle cell anemia
B. Alpha thalassemia
C. Thalassemia major

Answer: a
Sickle cell disease (scd) usually manifests early in childhood. The most common clinical manifestation of SCD is vaso-occlusive
crisis. It may present as dactylitis (bilateral painful and swollen hands and/or feet in children.

Reference: medscape: http://emedicine.medscape.com/article/205926-clinical




67. Pt with starry sky pattern on biopsy > burkitt's
Wts the mutation:
A. C-myc gene ( rt answer)
B. Abl2
C. Bcr- abl
Answer: a
Http://emedicine.medscape.com/article/1447602-overview


68. Which of the following tests will you order to check iron therapy response?
A. Reticulocyte
B. Ferritin
C. Tibc

Answer: a
Response to iron therapy can be documented by an increase in reticulocytes 5-10 days after the initiation of iron therapy. The
hemoglobin concentration increases by about 1 g/dl weekly until normal values are restored. These responses are blunted in the
presence of sustained blood loss or coexistent factors that impair hemoglobin synthesis.
[http://emedicine.medscape.com/article/202333-treatment#d13]

69. Patient has been diagnosed with lymphoma in the past and has received full course of chemotherapy. Now complaining of
painless facial swelling, cough and flushing, what is the diagnosis?
A. Superior vena cava obstruction
105
B. Inferior vena cava obstruction
C. Some type of facial tumor

Answer: a.
Superior vena cava (svc) obstruction

Reference: davidsone 22.

70. Pt with past hx of hodgkin lymphoma but cured completely, presented with back pain. Examination and investigations
showed a paraspinal mass, edema and fluid collection, negative brucella titer and tuberculin test, what is the cause?
A. Brucellosis
B. Breast cancer
C. Recurrent hodgkin lymphoma

Answer: c
http://www.ncbi.nlm.nih.gov/pmc/articles/pmc3892516/


71. Female came from 18 hours flight and she feel leg pain what to give?
A. Warfarin
B. Lmwh
C. Unfractionated heparin and warfarin
Answer: c
reference: uptodate.

72. Prostate cancer patient with recurrent dvt. Best prophylaxis is:
A. Lmwh.
B. Unfractionated heparin.
C. Lmwh short-term therapy followed by warfarin.
Answer: a

73. Increase the hbf in sicklers, give:


A. Deferoxamine
B. Penicillamine
C. Folic acid

Answer:
hydroxyurea

106
74. A patient known case of anemia on medications. Later he came complaining of dark stools. What is the medication?
A. Ferrous sulphate
B. Folic acid
C. Iron dextran

Answer: A

75. A patient with decrease in factor V. The etiology is:


A. Inherited
B. Immune
C. Infection

Answer: A
The inheritance of factor V deficiency is autosomal recessive.
Reference: http://emedicine.medscape.com/article/209492-clinical#b5

76. Peptic ulcer patient with Anemia, what you will do regarding his anemia?
A. Oral iron supplement.
B. I.M iron.
C. Blood Transfusion

Answer: B
Reference : uptodate

77. What of the following present in in peripheral blood smear of Sickle cell disease patient?
A. Howell jolly bodies
B. Spherocytes and….
C. Acanthocyte

Answer: A. Howell-Jolly Bodies (hyposplenism : in Sickle Cell disease)
Reference: Tornoto Notes Hematology (H4)

78. 3-4 cases about IDA


Answer: iron deficiency anemia
A. Oral iron therapy: initial therapy, gastrointestinal side effects are extremely common and may result in poor adherence.
B. Parenteral iron: for those unresponsive to or intolerant of oral iron, for pt with IBD, gastric bypass surgery.
C. Blood transfusion is not recommended unless anemia is sever or the patient has cardiopulmonary disease.
(step up to medicine)


79. Sickle cell anemia came with hepatosplenomegaly and low platelets- hgb -wbc
A. Splenectomy
B. Blood transfusion

Answer: b
Hepatic or splenic sequestration is used acutely for the treatment of severe anemia that cannot be adequately compensated by
increased red cell production.
Splenectomy is often removed after a person has survived such a crisis to try and prevent another one.
References: http://www.ncbi.nlm.nih.gov/pubmedhealth/pmh0012048/
Http://emedicine.medscape.com/article/205926-overview
Http://www.bloodjournal.org/content/123/15/2302?Sso-checked=true


80. Pt has a family hx of hemochromatosis presented with abdominal pain and fatigue, what to check?

107
A. Ferritin .
B. Transferrin

Answer: b
High transferrin saturation is the earliest evidence of hemochromatosis; a value greater than 60% in men and 50% in women is
highly specific. It is more sensitive tests for detecting early hemochromatosis. Ferritin level is less sesnsitive. Genetic tests for the
c282y and h63d mutations is conducted to confirm the diagnosis or to discover asymptomatic patients.
Reference: http://emedicine.medscape.com/article/177216-workup#c7


81. Hodgkin lymphoma with no fibrosis and eosinophils, reed sternberg cells, histocytes. Which type of hl is this?
A. Mixed cellularity
B. Nodular sclerosis

Answer: a
Mixed cellularity — mixed cellularity hl (MCHl) is a heterogeneous category of classic hl with a diffuse or vaguely nodular growth
pattern without band-forming sclerosis or fibrosis. Fine interstitial fibrosis may be present, and classical diagnostic reed sternberg cells
are easily identified. Red sternberg cells are large with bilobate, double or multiple nuclei, and a large, eosinophilic nucleolus. The
background infiltrate is variable, but typically consists of eosinophils, neutrophils, histocytes, and plasma cells.
Reference: uptodate

82. A sickler with vaso-occlusive crisis, dehydrated with a hemoglobin level of 3.5. What is the next step in management?
A. PRBCs transfusion
B. Analgesia and IV fluids
Answer: b
Vaso-occlusive crisis is treated with vigorous intravenous hydration and analgesics.
Http://emedicine.medscape.com/article/205926-treatment#d9

83. Pic of peripheral blood smear showing microcytic hypochromic RBCs & the patient came with symptoms of anemia. Direct
coombs test was +ve, indirect coombs test was + ve , diagnosis?
A. Hereditary spherocytosis
B. Autoimmune hemolytic anemia (aiha).
Answer: b
First aid:
• Hereditary spherocytosis has a negative coombs test.
• Aiha has a positive direct coombs test.

84. Cause of low sickling solubility test:


A. Anemia,
B. Presence of hemoglobin c
Answer: b

85. History of infection, hb low, wbc high what is the investigation :


A. Bone marrow biopsy
B. Hb elect because high wbc is caused by the infection
Answer: b (sca)
Ref: http://emedicine.medscape.com/article/205926-workup#c1

86. What is the most helpful test in sickle cell disease?


A. Hemoglobin electrophoresis.
B. Blood film.
Answer: a

108
Hemoglobin electrophoresis differentiates individuals who are homozygous for hbs from those who are heterozygous. It establishes the
diagnosis of SCD by demonstrating a single band of hbs (in hbss) or hbs with another mutant hemoglobin in compound heterozygotes.
Http://emedicine.medscape.com/article/205926-clinical

87. Young patient presented with neck mass and itching. What is the diagnosis?
A. Hodgkin lymphoma
B. Lyme disease
Answer: a
- First aid usmle step 2ck.
- Https://en.wikipedia.org/wiki/hodgkin's_lymphoma

88. A patient came with weakness. What is the most likely deficiency?
A. Vitamin b1
B. Vitamin b3
Answer is a
- Vitamin b1 (thiamine) deficiency causes beriberi, which is characterized by severe burning dysesthesias, weakness and wasting
(distal more than proximal), trophic changes (shiny skin, hair loss), and acrodistal sensory loss in a graded fashion typical of dying-
back polyneuropathies.
- Vitamin b3 (niacin) deficiency causes pellagra, which is characterized by the 3 ds: (1) dermatitis, i.e., hyperkeratotic skin lesions,
particularly on hands, feet, face, and neck (sun-exposed regions); (2) diarrhea; and (3) dementia.
Source: http://emedicine.medscape.com/article/1171558-overview#a5



89. A young asymptomatic male came for routine checkup. Labs showed microcytic hypochromic anemia, what is the diagnosis?
A. Thalassemia trait (i chose it)
B. Iron def. Anemia (since he is young, not female, i excluded it)
It could be thalassemia because children who have untreated thalassemia major have ineffective erythropoiesis, decreased red cell
deformability, and enhanced clearance of defective red cells by macrophages (immune system cells). The result is a very hypermetabolic
bone marrow with thrombocytosis, leukocytosis and microcytic anemia in the young child prior to the enlargement of their spleen.

90. An HIV patient came with bowel obstruction… i think in ct there is mass then took biopsy from it & show some type of cell ( i
do not remember the name of cell). What is the diagnosis?
A. Hodgkin
B. Tuberculosis
Answer: a
Both high and intermediate grade non-hodgkin's lymphomas and hodgkin's lymphoma (hl) have an increased incidence in patients
infected with hiv.
Http://www.medscape.com/viewarticle/444346
Classic hodgkin disease (hd) accounts for about 95% of all cases of hodgkin disease in developed countries.the cancer cells in classic
hd are called reed-sternberg cells (after the 2 doctors who first described them). These cells are usually an abnormal type of b
lymphocyte. Reed-sternberg cells are much larger than normal lymphocytes and also look different from the cells of non-hodgkin
lymphomas and other cancers.
Http://www.cancer.org/cancer/hodgkindisease/detailedguide/hodgkin-disease-what-is-hodgkin-disease

91. Cervical ln enlargement, hepatosplenomegaly, circumoral edema, acute presentation..what is the diagnosis?
A. Lymphoma
B. Angioedema
Answer: ???

92. An elderly asymptomatic patient’s tests show 90% lymphoblast.. What to do for him?
A. Observation
109
B. Chemo
Answer: b
This is probably a case of all and the treatment uses multi-agent dose-intense chemotherapy regimens in induction, consolidation,
and maintenance phases.

93. Patient complains of fatigue and weakness. Lab results showed:


- Hemoglobin : decreased
- MCH : decreased
- RDW : 13
What is your dx :
A. Thalassemia
B. Iron defiency anemia
Answer: a
Probably thalassemia since the RDW is normal

94. Sca, no joint swelling complains of lower leg ?, fever


A. Osteomyelitis
B. Vaso-occlusive crisis
Answer: b due to the fact that it's a sca patient

95. Patient presented with anemia, thrombocytopenia, hypercalcemia, picture of vertebral column?, how to confirm the
diagnosis?
A. Genetic
B. Serum protein electrophoresis
Answer: b, dx = multiple myeloma, serum electrophoresis identifies m-protein in about 80 to 90% of patients.
Http://www.merckmanuals.com/professional/hematology-and-oncology/plasma-cell-disorders/multiple-myeloma

96. Pts take methotrexate for something present with tiredness with lab result high mcv
What to give?
A. Iron
B. Folic acid
Answer: b


97. Vwd lab findings ?
A. Prolonged bleeding time
B. Prolonged pt
Answer: a

98. Patient presented with heamarthrosis what is the defect:


A. Factor 8
B. Factor 7
Answer: a

99. 106. Burkitt's lymphoma associated with?


A. Epstein barr virus "EBV"
B. T lymphocyte infiltration

Answer: a

100. Pregnant patient with anemia, mcv high, what will you give her?
A. Iron
110
B. Foliate
Answer: b- foliate

101.Child had constipation after few days he develop bloody urine and lower abdominal pain
A. Autoimmune hemolytic anemia
B. Uti
Answer: a

102.Case of sickle cell anemia. What is most important on long term?


A. Penicilline
B. Hydroxyurea
Answer: a
Folic acis >>> lifelong
Penicillin >> for the age from 8 weeks to 5 years old
Hydrxyurea >>> for severe phenotypic and recurrent vaso-occlossive crisi

Refrence: step-up of medicine

103.Known case of SCD complaining from right shoulder pain xray show head humerus necrosis what's long term therapy
A. Hydroxyurea
B. Penicillin
Answer: a
The only drug currently approved by the us food and drug administration (fda) for the treatment of SCD is hydroxyurea. For frequent
and severe pain, long-term hydroxyurea is currently the accepted treatment.
Link: http://emedicine.medscape.com/article/205926-treatment#d8

104.What increase the absorption of iron?


A. Folic acid
B. Vitamin c
Answer: b
Ascorbic acid or vitamin c occurs naturally in vegetables and fruits, especially citrus. Ascorbic acid can also be synthesized for use in
supplements. Ascorbic acid enhances the absorption of nutrients such as iron. In studies about effects of ascorbic acid on iron
absorption, 100 milligrams of ascorbic acid increased iron absorption from a specific meal by 4.14 times.
Link: http://www.irondisorders.org/diet/

105.Polycythemia vera with blurred vision and headache what the cause of these sx?
A. Hypovolemia
B. Hyperviscosity
Answer: b
Symptoms of polycythemia vera (pv) are often insidious in onset, and they are often related to blood hyperviscosity secondary to a
marked increase in the cellular elements of blood. Subsequent sludging of blood flow and thrombosis lead to poor oxygen delivery,
with symptoms that include the following:
Headache, dizziness, vertigo, tinnitus, visual disturbances, angina pectoris, intermittent claudication.
Link: http://emedicine.medscape.com/article/205114-clinical

106.4 years old SCD w/ recurrent voc what is best management for future improvement?
A. Hydroxyurea
B. Multiple blood transfusions
Answer: a
Hydroxyurea decreases crises in patients with severe sickle cell disease
Https://www.cdc.gov/ncbddd/sicklecell/recommendations.html


111
107.The most common cause of hemarthrosis
A. ITP
B. Von willibrand disease

Answer: b

Reference: step-up to medicine (p 340-341)


Hemarothosis has a lot of causes but according to the choices von willibrand disease (coagulation disease) will be the best answer.
Hemarthrosis rarely happen with ITP (platelet disorder). Patients with hemophilia they mostly will experience hemarthrosis.

108.Generalized cervical lymphadenopathy + mild tenderness + low grade fever. What's the most likely diagnosis?
A. Small lymphocytic lymphoma (presented by generalized lymphadenopathy)
B. Hodgkin’s lymphoma

Answer: a
Hodgkins lymphoma presents with asymptomatic lymphadenopathy (above the diaphragm in 80% of patients) and it moves between
lymph node groups in an orderly fashion
Http://emedicine.medscape.com/article/201886-clinical
Non-hodgkin's lymphoma is much more common than hodgkin's disease.
And the incidence of non-hodgkin's lymphoma has been steadily increasing over the last decades unlike hodgkin’s which only
accounts for 1% of cancers in the united states and is declining in incidence.

Reference: http://www.medicinenet.com/hodgkins_and_non-hodgkins_lymphoma_comparison/article.htm

109.Hx of infection, hb low, wbc high, what is the investigation :


A. Bone marrow biopsy
B. Hb electrophoresis
Answer: a
This may be a case of hodgkin lymphoma but there are not enough information to make the diagnosis.

110.False positive sickle test ?


A. Hb c
B. Hb d
Answer: a
some say there was (protien) in the answers, if there's so it's the choice
false-positive results can be obtained if nucleated red blood cells are present in the peripheral blood or the patient has marked
hypergammaglobulinemia (such as in multiple myeloma).



111.Post-transfusion fever how to prevent?
A. Transfusion filter depleted leukocyte
B. Premedication with antupyretics

112
Answer :a (uptodate)

112.Elderly patient presented with typical symptoms of multiple myeloma. X ray showed lytic lesions, positive M protein and
hypercellular bone marrow. What other findings can be found in the blood?
A. Increase peripheral blood B cell
B. Rouleaux formation

Answer: B
Reference: Pubmed and Medscape
Http://emedicine.medscape.com/article/204369-workup#c6

113.A patient with Vitamin B 12 deficiency. What is the type of gastric cell that will be affected?
A. Chief cell
B. Parietal cell

Answer: B
Patient with pernicious anemia is usually directed against the intrinsic factor or parietal cell themselves. Parietal cell deficiency as it’s
responsible for intrinsic factor synthesis, which is required for vit. B12 to be absorbed in terminal ileum
“Autoimmune metaplastic atrophic gastritis – A major component of PA (pernicious anemia) is chronic atrophic gastritis, which is
associated with autoantibodies directed against gastric parietal cells in approximately 90 percent of patients with PA.”
Reference: b12-and-folate
deficiency?Source=machinelearning&search=megaloblastic+anemia&selectedtitle=3~91&sectionrank=1&anchor=H2125777#H2125
777

114.Hb F is present, Hb A2 is present also, what is the diagnosis?


A. Thalassemia minor.
B. Thalassemia major.

Answer: B (if Hb A was absent)

Beta-thalassemia trait: Hb A2 (Hb A2) values of 3.5% to 9% and Hb F is usually between 5% to 15%
Beta-thalassemia major: Hb F may be 30% to 90% or even more of the total hemoglobin
Reference: http://www.mayomedicallaboatories.com/test-catalog/Clinical+and+Interpretive/83341

115.A case of low platelets, low RBCs and low wbcs. What is the diagnosis?
A. Iron def anemia
B. Aplastic anemia

Answer: B
Aplastic anemia (AA) is characterized by diminished or absent hematopoietic precursors in the bone marrow, most often
due to injury to the pluripotent stem cell (Decrease RBCs, wbcs, Platelets).
Reference: Uptodate.

116.A known case of sickle cell disease presented with unilateral lower limb pain since (short period:acute). Vital signs:
Tachycardia, Fever 38.3. The range of movement is intact with no signs of inflammation over the limb. What is the diagnosis?
A. Vaso-occlusive Crisis
113
B. Osteomylitis

Answer: A
Findings secondary to vaso-occlusion.
A. Painful crises involving bone—bone infarction causes severe pain. This is the
Most common clinical manifestation.
• Bone pain usually involves multiple sites (e.g., tibia, humerus, femur). It may
Or may not be bilateral.
• The pain is self-limiting and usually lasts 2 to 7 days.
B. Hand–foot syndrome (dactylitis).
• Painful swelling of dorsa of hands and feet seen in infancy and early childhood
(usually 4 to 6 months).
• Often the first manifestation of sickle cell disease.
• Caused by avascular necrosis of the metacarpal and metatarsal bones.
Precipitated by infection , fever , dehydration , pregnancy and alcohol
Reference : step up to medicine

117.Sickle cell disease patient with multiple gall bladder stones. What is the Best thing to do?
A. Hydroxyurea
B. Cholysystectomy

Answer: B
If the patient does not have symptoms, no treatment is usually necessary. If there is recurrent or severe pain from gallstones, the
gallbladder may need to be removed. Minimally invasive procedures (using laparoscopy) reduce possible complications
Reference: https://umm.edu/health/medical/reports/articles/sickle-cell-disease

118.In hemolytic anemia which enzyme will be noticed?


A. Erythropoietin
B. Bilirubin

Answer: B
Standard blood studies for the workup of suspected hemolytic anemia include the following:
• Complete blood cell count
• Peripheral blood smear
• Serum lactate dehydrogenase (LDH) study
• Serum haptoglobin
• Indirect bilirubin
Reference : http://emedicine.medscape.com/article/201066-workup

119.Patient has splenomegaly and teardrop RBC on blood film, what is the diagnosis?
A. ITP
B. Myelofibrosis
Answer: b
Ref: http://www.pathologystudent.com/?P=607


120.Hemolytic anemia patient with schistiocyte normal aptt and pt what is management?
A. Acyclovir.
B. Antibiotic if case of hus tx supportive

121.Sickle cell anemia came with hepato splenomegaly And low platelets- HGB -WBC?
A. Splenectomy
B. Blood transfusion
Answer: A

114
122.Pt with anemia, high billirubin, positive direct and indirect coombs, Photo of samear shoing spherocytosis, wts the dx:
A. AIHA Autoimmune hemolytic anemia (rt answer since +ve coombs)
B. Spherocytosis
Answer : A


123.Sx of hydrop fetalis or HB h
A- normal 2 beta and abnormal 4 alfa
B- opposite of the above

Answer: A


124.Young patient presents with cough and chest pain. Chest x--ray showed lung infiltrates. Cbc: anemia with high reticulocytes
count (10%) and leukocytosis, what is the diagnosis?
A. Sickle cell anemia
Answer: a
Sickle cell anemia presenting with acute chest syndrome supported by clinical and radiological evidence

125.Sickle cell patient experienced pain in his hands (descriptions of vaso-occlusive crisis). What is the most probable
diagnosis……splenic sequestration
Answer:?? Need more details
Vasooclusive: because deoxygenated hemoglobin s (hbs) becomes semisolid, the most likely physiologic trigger of vaso-occlusive crises
is hypoxemia. This may be due to acute chest syndrome or accompany respiratory complications.
Read about sickle cell crisis
Http://emedicine.medscape.com/article/205926-clinical

126.What is the mechanism of action of heparin?


Answer: heparin binds to the enzyme inhibitor antithrombin iii (at)
Https://en.m.wikipedia.org/wiki/heparin

127.A young patient is continuously bleeding post operatively. Coagulation profile showed high aptt, otherwise normal, which
factor is deficient?
A. Factor viii
Answer: a
This patient has hemophilia a which characterized by factor viii deficiency. Those patients usually have normal pt and thrombin time,
fibrinogen, bleeding time. Some cases may have major hemorrhage after surgery or trauma
First aid usmle step 2ck.

128.A young patient complains of a sudden onset of dyspnea. Labs show decreased mcv, normal wbc, increased platelets.
Peripheral smear shows microcytic hypochromic anemia:
A. Iron deficiency anemia- thalassemia
The answer: is ida

129.Patient with multiple blood transfusion and jaundice + osmotic fragility test…….
Answer: spectrin-ankyrin binding deficiency (hereditary spherocytosis)
The osmotic fragility test (oft) is used to measure erythrocyte resistance to hemolysis while being exposed to varying levels of dilution
of a saline solution.conditions associated with increased osmotic fragility include the following:
• Hereditary spherocytosis
• Autoimmune spherocytosis
• Poisoning
• Severe burns

115
The following conditions are associated with decreased fragility:
• Thalassemias
• Iron deficiency anemia
• Sickle cell anemia
Http://emedicine.medscape.com/article/2085814-overview#a2

130.Most common presentation of hemophilia?


Patients with hemophilia bleed into tissues (eg, hemarthrosis, muscle hematomas, retroperitoneal hemorrhage).
Http://www.msdmanuals.com/professional/hematology-and-oncology/coagulation-disorders/hemophilia


131.Mediterranean blood disease?
Thalassemia (also known as mediterranean anemia, or cooley's anemia) is a genetic blood disease.

132.Case with CBC , lab results , increaseIgG, increase bun , creatinine, diagnosis : answer : multiple myeloma

133.In vwd bleeding time will be increased while pt is normal. Aptt may be increased due to concomitant decrease in factor viii.
Http://emedicine.medscape.com/article/206996-workup#c9

134.Case simple for iron deficiency anemia: with it's parameter?


Iron deficiency anemia
http://emedicine.medscape.com/article/202333-workup#showall

135.Pt present CBC and differential results are low


• Aplastic anemia. Bmj http://bestpractice.bmj.com/best-practice/monograph/93/diagnosis/guidelines.html

136.Post-surgical dvt what is the best treatment regarding to cost and effectiveness
1. Un-fractioned heparin
Rivaroxaban represents a cost-effective choice compared with lmwh/vka treatment, regardless of the required treatment duration.
Http://www.ncbi.nlm.nih.gov/pubmed/26074735

137. Pt presented with hemoytic anmia??


A. Increase uncongigated bilirubin
Http://bestpractice.bmj.com/best-practice/monograph/98/diagnosis/tests.html

116
138. Mechanism of aspirin?
Dose dependent:
Low doses (typically 75 to 81 mg/day) are sufficient to irreversibly acetylate serine 530 of cyclooxygenase (cox)-1. This effect
inhibits platelet generation of thromboxane a2, resulting in an antithrombotic effect.
Intermediate doses (650 mg to 4 g/day) inhibit cox-1 and cox-2, blocking prostaglandin (pg) production, and have analgesic and
antipyretic effects. Uptodate. Http://www.uptodate.com/contents/aspirin-mechanism-of-action-major-toxicities-and-use-in-rheumatic-
diseases#h3

139. Sickle cell disease with hip pain:


A. AVN . Step up to medicine

140.Old female live alone change her dietary habit what is the cause of anemia?
A. Fe deficiency


141.Sca with recurrent cholecystitis?
A. Cholecystectomy
Reference: http://emedicine.medscape.com/article/205926-treatment#d17

142.Woman (complaint). Lab: high platelets. Treatment?


A. Plateletpheresis
Best initial treatment hydroxyurea.


143.Child with hand and foot swelling and pain diagnosis?
Answer: sca
Http://emedicine.medscape.com/article/205926-overview

117
144.Pt with large tongue, anemia, numbness in feet. What is the best treatment?
Answer: Vitamin b12

145.About hemolytic anemia dx with peripheral blood smear shows microspherocytosis


Answer: hemolytic anemia

146.Which is feature of vwd?


Answer: prolonged bleeding time

147.Pt. With sever ida hg=10 range they put (120-something) what to do first?
A. Blood transfusion
Answer: ( no iron trial in other choices )

148.Yellow pt all normal only high indirect bilirubin (unconjugated) and alkaline phosphatase? What the diagnosis?
I think the most likely cause is a hemolytic disorder associated with bone infiltration such as thalassemia

149.Crescentic cell cause?


Answer: hemolytic anemia

150.Old lady living alone presents with anemia. What is the most likely cause?
A. B12 deficiency
Answer: a

151.Nhl staging (ann arbor) patient with b symptoms and lymph nodes from the neck to the para aortic with spleen involvement?
A. Cotswold modification of ann arbor staging system. Medscape http://emedicine.medscape.com/article/2007081-overview
Answer: III

118

152.Scenario of leukaemia with auer rods ? Aml

153.Patient with low hemoglobin and low mcv, which of the following will confirm the diagnosis?
A. Iron level and tibc

Answer: a
[https://arupconsult.com/sites/default/files/anemia%20testing%20algorithm.pdf]

154.Pt with iron deficiency anemia given iron supplements + lab hb dec, mcv dec, what you will give him?

Reference: uptodate

155.Leukemia bleeding is characterized by:


Multiple choices about abnormal bleeding test results.
119
Answer:?!

156.Sickle cell disease with hip pain?


Answer: AVN

Reference: step up to medicine

157.Splenectomy case, what vaccines should be given afterwards or prior to spleen removal?

Answer: against encapsulated organisms vaccinate


• Influenza vaccine each year to protect against seasonal flu.
• Tdap vaccine to protect against whooping cough and tetanus.
• Hib vaccine to protect against haemophilus influenzae type b (hib) if you were not previously vaccinated with the vaccine.
• Pneumococcal vaccines (both types) to protect against pneumonia and other pneumococcal disease.
• Meningococcal vaccines (both types) to protect against meningitis.
• Zoster vaccine to protect against shingles if you are 60 years and older.
• Hpv vaccine series to protect against human papillomavirus if you are a man up to age 21 or woman up to age 26.
• Mmr vaccine to protect against measles, mumps, and rubella.
• Varicella vaccine to protect against chickenpox.

Reference: cdc, http://www.surgicalcriticalcare.net/guidelines/splenectomy_vaccines.pdf

158.The best diagnosis tool for thalassemia is:

Answer : haemoglobin electrophoresis is the gold standard


Diagnosis of thalassemia:
• Evaluation for hemolytic anemia (microcytic hemolytic anemia, RDW normal, serum bilirubin, iron, and ferritin levels are
increased, RBC count is elevated relative to hb because the cells are very microcytic).
• Peripheral smear (diagnositic test), (nucleated erythroblasts, target cells, punctate and diffuse basophilia)
• Electrophoresis (elevation of hb a2 and hb f, hb h or bart’s fractions
• DNA testing (prenatal diagnosis)
• Bone marrow. (erythroid hyperplasia)
• X-rays (cortical thinning skull, widened diploic space, a sun-ray appearance of the trabeculae, and a granular or ground-glass
appearance)
• Long bone x-ray (cortical thinning, marrow space widening, and areas of osteoporosis)
• Vertebral x-ray (the bodies may have a granular or ground-glass appearance)
• The phalanges may appear rectangular or biconvex.

Reference: merck manual, uptodate

159.ITP case management:


120
Answer:
• Corticosteroids
• IV immune globulin—saturates the reticuloendothelial system binding sites for platelet-bound self-immunoglobulin, so there is
less platelet uptake and destruction by the spleen
• Splenectomy—induces remission in 70% to 80% of the cases of chronic ITP
• Platelet transfusions 1 u/10 kg—for life-threatening and serious hemorrhagic episodes. 1 u of platelets to increase count of a 70-
kg adult by 5-10,000/mm and an 18-kg child by 20,000/mm
• Two new drugs, romiplastim and eltrombopag, for splenectomy- resistant patients. Both work as thrombopoietin receptor
agonists to increase platelet production.
Reference: step-up of medicine, medscape

160.Patient with high aptt what mechanism of action of that drug?


" unfractionated heparin" antithrombin
Answer:
Low dose heparin : inactivates factor xa and inhibits conversion of prothrombin to thrombin
High dose heparin : inactivates factors ix , x , xi , xii and thrombin and inhibit conversion of fibrinogen to fibrin.
Antidote is protamine sulfate

161.Aspirin inhibit which product formation:


Answer: thromboaxan a2

162.Leukemia pt with 80 blast and Auer bodies in peripheral smear what type of leukemia?
Answer: Acute Myeloid Leukemia (AML).
Peripheral blood smear shows : circulating blasts with Auer rods (azurophilic granules) are pathognomonic for AML.
Bone marrow aspirate show : blast count >20%.
Reference: Toronto Notes Hematology (H38)

163.Case suggest of multiple myeloma what can you find ?


A- Bence jones protein
Answer :a
In multiple myeloma light chains are filtered at the glomerulus and appear as bence-jones proteins in the urine (monoclonal light
chains). The clinical features include:bone disease: pain (usually back), bony tenderness, pathologic fractures,anemia, infections ,n/v,
confusion, constipation, polyuria, polydipsia ,bleeding ,amyloidosis.

164.Treatment of acute myeloid leukemia:


Answer:

165.Elderly with high calcium and low back pain … :


A. Multiple myeloma
121
Incomplete question
Explanation: presenting symptoms of multiple myeloma (mm) include bone pain (most common, especially lumbar), pathologic
fractures, weakness, anemia, infection (often pneumococcal), hypercalcemia, spinal cord compression, or renal failure.
Reference: http://emedicine.medscape.com/article/204369-overview


166.Which type of anemia associated with chemotherapy and radiotherapy?
Answer: aplastic anemia
Http://www.mayoclinic.org/diseases-conditions/aplastic-anemia/basics/treatment/con-2001929


167.Vwb deficiency which will be affected ?
In the great majority of cases, vwd is an inherited condition. The vwf gene is located near the tip of the short arm of chromosome
12. The gene is composed of 52 exons and spans a total of 180kb of the human genome; therefore, it is similar in size to the fviii
gene. Expression of the vwf gene is restricted to megakaryocytes, endothelial cells, and, possibly, placental syncytiotrophoblasts. A
partial, nonfunctional duplication (pseudogene) is present on chromosome 22.
Vwf exists as a series of multimers varying in molecular weight between 0.5-kd (dimer) and 20 million kd (multimer). The building
blocks of multimers are dimers, which are held together by disulfide bonds located near the c-terminal end of each subunit



168.Picture of child with macular rash all over the body with hx of fever and carditis, what's the dx:
A. Kawasaki
Answer: a. Toronto:

169.A vegetation patient with lab showing microcytic hypochromic anemia what is the dx :
a. Iron deficiency

Answer: a

122
Another clue in this scenario is that she is a vegetarian
Reference: toronto notes


170.When to give both heparin and fresh frozen plasma?
Disseminated intravascular coagulation (dic): uncontrolled release of plasmin and thrombin leading to intravascular coagulation and
depletion of platelets, coagulation factors and fibrinogen this condition should be recognized early and treat underlying disorder..•
in hemorrhage: replacement of hemostatic elements with platelet transfusion, frozen plasma, cryoprecipitate• in thrombotic phase:
unfractionated heparin or lmwh
reference: toronto notes


171.2 cases of anaemia. (interpretation)
Https://www.arupconsult.com/algorithms/anemia.pdf

172.Thrombocytopenia and recurrent infection , male and uncle has the same condition ?
Answer: wiskott–aldrich syndrome


173.Purpura ( immmnue,, henoch purpura)
A. Lesion in mm

174.Infective rheumatic fever. ! Rash ,


A. Retuc count in

175.Hemolytic anemia:
A. Unconjugated bilirubin

176.Question about ( autoimmune hemolysis)


Destruction of red blood cells by autoantibodies may occur suddenly, or it may develop gradually. In some people, the destruction
may stop after a period of time. In other people, red blood cell destruction persists and becomes chronic. There are two main types
of autoimmune hemolytic anemia:
- warm antibody hemolytic anemia

123
- Cold antibody hemolytic anemia
Https://www.merckmanuals.com/home/blood-disorders/anemia/autoimmune-hemolytic-anemia

177.A patient presented with anemia and glossitis. He also complained of paresthesia
A. Pernicious anemia

Answer: a .
These features are caused by Vitamin b12 deficiency. The presentation of the patients include:confusion, delirium, dementia cranial
nerves (rare) ,optic atrophy cord (irreversible damage) ,subacute combined degeneration,decreased vibration sense, proprioception,
and 2-point discrimination,spastic weakness, hyperactive reflexes, peripheral neuropathy (variable reversibility).

178.Scenario about extremities numbness and stomatitis ?


A. Vitamin b12 deficiency
Question and choices incomplete
Explanation: subacute combined degeneration is due to vitamin b12 deficiency; affect the brain and spinal cord white matter. In early
stages, decreased position and vibratory sensation in the extremities, accompanied by mild to moderate weakness and hyporeflexia. In
later stages, spasticity, extensor plantar responses, greater loss of position and vibratory sensation in the lower extremities, and ataxia
emerge.
Reference: http://www.msdmanuals.com/professional/nutritional-disorders/vitamin-deficiency,-dependency,-and-toxicity/vitamin-b-
12

179.Long scenario about nonmegaloblastic macrocytic anemia ?


A. Alcohol consumption
Question and choices incomplete
Explanation: a history of alcohol abuse may be an important clue to the cause of the increased mean cell volume (mcv); long-term use
of alcohol may have a direct toxic effect on the bone marrow, causing macrocytosis. Alcohol may also cause anemia by causing folate
deficiency.
Reference: http://emedicine.medscape.com/article/203858-overview#a7

180.Case of iron deficiency anemia lab showed typical microcytic with high tibc , mechanism :
A. Decrease RBCs more than serum volume )

181.Two cases about anemia. (interpretation)

124
182.Peripheral blood smear about leukemia forgot it

183.Risk factors for thrombosis

184.Pt with pic of anemia and thrombocytopenia - blood film showed schistocytes what dx ?
Answer : DIC

125

185.Tttx of ITP
Answer:
Http://www.uptodate.com/contents/immune-thrombocytopenia-ITP-in-adults-initial-treatment-and-
prognosis?Source=search_result&search=ITP&selectedtitle=2~150

186.4 cases about IDA


Answer:
Http://www.uptodate.com/contents/causes-and-diagnosis-of-iron-deficiency-anemia-in-the-
adult?Source=search_result&search=IDA&selectedtitle=1~150

187.Case of fanconi syndrome


Answer:
Http://emedicine.medscape.com/article/981774-overview


188.Pt was anemic, being given blood transfusion. Developed fever, chills, burning at site of IV line, what to do:
A. Stop transfusion and give crystalloids
Answer : A
Stop transfusion as soon as reaction is suspected
Infuse normal saline via IV site using new tubing; aggressive fluid resuscitation is ordered to maximize renal perfusion.
Monitor vital signs and urine output
Examine blood bag to see if patient was the intended recipient.
Return donor blood back to the blood bank to determine whether the corrrect unit of blood was administered.
IV furosemide and low-dose dopamine may be ordered to increase renal perfusion (maintain urine output at 30-100 ml/hr).

If there is any suggestion (eg, clerical mistake, hypotension, pink plasma or urine) that an AHTR is possible, and even before
laboratory tests are available, generous fluid replacement with saline (100 to 200 ml/hour) to support a urine output above 100 to
200 ml/hour should be initiated immediately, in an attempt to prevent the development of acute oliguric renal failure. The beneficial
effect of urinary alkalinization in patients with marked hemoglobinuria is uncertain.
Vigorous supportive care is also important for the treatment of ahtrs. If there has been massive hemolysis and clinical or laboratory
signs of disseminated intravascular coagulation, cautious and early heparinization (10 units/kg per hour) for the next 12 to 24 hours
may be of value, although there are no recent studies on the effectiveness of this intervention.
A vasopressor, such as low-dose dopamine, may be required. If massive intravascular hemolysis has already occurred, hyperkalemia
is likely, and cardiac monitoring and acute hemodialysis may be required. Accordingly, the patient's renal function and coagulation
profile (eg, prothrombin time, partial thromboplastin time, fibrinogen, platelet count) should be monitored frequently.
126
189.Pt with large tongue, anemia, numbness in feet. What is the best treatment:
A- Vitamin B12
Answer: A


190.Patient with high Ca and low IgAIgm what is the diagnosis (this Q was already submitted by one of our colleagues but the
answer in the (gathered smle) had multiple myeloma and an explanation to a subtype of it. In the exam they offer you both
multiple myeloma and it's subtype as choices and you have to choose between the two

191.Multiple myeloma in the spine


(Histopath report)
Answer:
Dx of multiple myeloma requires the following:
• Bone marrow aspirate or biopsy showing that at least 10% of the cells are plasma cells or the presence of plasma cell tumor (
plasmacyte ) plus at least one of the following
• Evidence of the damage to the body by plasma cells ; bone damage , kidney failure , anemia or high calcium .
• Defection of one of the following ; > 60% plasma cells in bone marrow, free light chain ration of 100 or more, MRI showing
involvement of more than one lesion in the bone or bone marrow
Reference: http://www.uptodate.com/contents/multiple-myeloma-symptoms-diagnosis-and-staging-beyond-the-
basics?Source=outline_link&view=text&anchor=H12#H12

192.Chemotherapy can case which type of anemia?


A. Aplastic anemia
Answer: A
It will cause bone marrow suppression.

193.Old lady living alone for 5 years. She has memory problem and looks pale.
A. B12 deficiency
Answer: A
This patient is “pale”, this is a hint for anemia, vit-B12 deficiency can cause memory problems plus anemia.
That’s why if you have old patient with memory problems, before you say he has dementia, you should exclude: Hypothyroidism and
vit-B12 deficiency.
Reference: Step-up to Medicine.

194.Patient on warfarin 7 mg presented with melena. INR was very high. What will you do?

Answer:?
The treatment of excessive anticoagulation is based on the level of the INR, the presence or absence of bleeding, and clinical
circumstances. Reversal of Warfarin sodium anticoagulation may be obtained by discontinuing Warfarin sodium therapy and, if
necessary, by administration of oral or parenteral vitamin K1.
Reference: http://www.drugs.com/pro/warfarin.html

127
Reference: Toronto Notes.

195.Mediterranean blood disease


A. Thalassemia
Answer: A
Reference: Toronto Notes

196.What is the most common symptom of factor 8 deficiency?


A. Hemarthrosis
Answer: A
In severe hemophilia A, spontaneous joint or deep-muscle bleeding is the most frequent symptom.
Reference: http://www.ncbi.nlm.nih.gov/books/NBK1404/

197.Osmotic fragility test.


A. Hereditary spherocytosis
Answer: A

198.What type of anemia is associated with rheumatoid arthritis?


A. Normocytic normochromic
Answer: A

199.70 years old man complaining of back pain. Calcium: High, iga,igm and igg: Low (i.e. Three types of immunoglobulins are low).
What is the diagnosis? (They didn’t give any other details)
Answer: ?
Multiple myeloma
Reference: http://www.cancernetwork.com/hematologic-malignancies/non-secretory-myeloma-clinician%E2%80%99s-
guide#sthash.nmvyh621.dpuf
Monoclonal Gammopathy of Unknown Significance (MGUS) is the least aggressive subclass of plasma cell dyscrasia. Patients
have only a small abnormal protein spike (<3.0 gm/dl and <2.0 gm/dl forIgGand IgAand igm, respectively), minimal or no bone
marrow involvement (<10% plasma cells), no bony involvement, normal blood counts and usually normal levels of unaffected
antibodies. The urine is usually free of monoclonal protein, however, it is not unusual to find small amounts of monoclonal light
chains in the urine of MGUS patients. Patients with MGUS have a 20-25% chance of developing multiple myeloma or a related
lymphoproliferative disorder. The remainder lead a normal life. Reference: https://www.clevelandclinic.org/myeloma/mm-pt.htm

200.A patient working in a new place up the hill. Recently, he had raised hemoglobin. What is the reason?
A. Due to increased production of erythropoietin.
Answer: A

201.Read string cells


A. Hodgkin lymphoma
Answer: A
The histological hallmark of HL is the presence of Reed–Sternberg cells, large malignant lymphoid cells of B cell origin.
Reference: Davidson's Principles and Practice of Medicine

202.Sickle Cell Disease (SCD) patient presents with bloody urine. What is the cause?
A. Recurrent UTI
Answer: ?
Typically, the hematuria is mild and self-limited in SCD. As a rule, the hematuria originates from the left kidney; this has been
attributed to the greater length of the left renal vein and compression of the left renal vein between the aorta and superior
mesenteric artery (ie, the nutcracker phenomenon).
Hematuria can also be secondary to papillary necrosis or renal medullary carcinoma.
It can result from other problems such as urinary infections, renal stones or glomerulonephritis

128
Reference: http://emedicine.medscape.com/article/1957952-overview#a7

203.Which bleeding disorder presents with prolonged BT & aptt?


Answer: ?
Von Willebrand Disease (vwd) presents with increased Bleeding Time (BT) and elevated PTT.

204.40 years old man, Hb: low, MCV: high, No megaloblast. What is the most likely diagnosis?
A. Alcoholic anemia
Answer: A
Non-megaloblastic ddx: Liver disease, Alcoholism, Reticulocytosis, Hypothyroidism and Myelodysplasia.
Reference: Toronto Notes.

205.Sickle cell disease and joint pain? 2 times


o Due to avascular necrosis
Answer: ?
Not clear but Pain can be mild or severe and usually develops gradually. Pain associated with avascular necrosis of the hip may be
focused in the groin, thigh or buttock. In addition to the hip, the areas likely to be affected are the shoulder, knee, hand and foot. Some
people develop avascular necrosis bilaterally — for example, in both hips or in both knees.
Reference: http://www.mayoclinic.org/diseases-conditions/avascular-necrosis/basics/symptoms/con-20025517

206.Spoon shaped nail?


Answer: iron deficiency anemia

207.Elderly came with leukocytosis... What supports the diagnosis of CML


Answer: The diagnosis of CML is suspected based on the results of a simple blood test The test may show an abnormally high white
blood cell count in blood samples examined under a microscope, less mature white blood cells, normally found only in bone
marrow, are seen
Ref: Medical council of canada exam
On CML---- increase WBC and decrease Leukocyte alkaline phosphatase
On Leukemoid reaction -----increase WBC and Increase Leukocyte alkaline phosphatase

208.Best diagnostic tool for thalassemia is ….?


Answer: electrophoresis if asked about thalassemia in general. However if asked specifically about Alpha or Beta thalassemia is
different. Beta is diagnosed best with electrophoresis, but Alpha is diagnosed best with DNA test- polymerase chain reaction (PCR ).
Reference: Toronto notes 2015 page (H20)
Http://emedicine.medscape.com/article/206490-workup#c7

209.What factor causes thrombosis?

129
Answer: The formation of blood clots in the veins is related to three abnormalities commonly known as “Virchow’s triad”. The
processes of Virchow’s triad include the following: Damage to blood vessels (endothelial injury), Excessive clotting ability
(hypercoagulability) and Pooling of blood (stasis) - disturbed flow.

Table 1. Inherited Causes of Blood Clots


Increased Levels of Procoagulants Decreased Levels of Abnormal Fibrinolysis Other Inherited
Anticoagulants Causes
Factor V Leiden mutation or Antithrombin Plasminogen Deficiency Paroxysmal Nocturnal
activated protein C resistance* Hemoglobinemia
Prothrombin 20210 mutation Protein C Decreased Levels of Tissue
Plasminogen Activator (t-
PA)
Hyperhomocysteinemia Protein S Increased Levels of
Plasminogen Activator
Inhibitor-1 (PAI-1)
FVIII, FIX, FXI, FVII, VWF Thrombomodulin Elevated Thrombin-
Activatable Fibrinolysis
Inhibitor (TAFI)
Heparin Cofactor II
Tissue Factor Pathway
Inhibitor (TFPI)
*The Factor V Leiden mutation does not result in increased FV levels but a resistance to the anticoagulant action
of activated protein C.

Http://www.ihtc.org/patient/blood-disorders/clotting-disorders/thrombosis/

210.What medication dissolve thrombus?


Answer: Dissolution occurs when the fibrinolytic mechanisms break up the thrombus and blood flow is restored to the vessel. With the
usage of these drug there is a risk of hemorrhage. 2 categories of drugs:
• Fibrin-specific agents, which include alteplase (tpa), reteplase (recombinant plasminogen activator [r-PA]), and tenecteplase,
produce limited plasminogen conversion in the absence of fibrin.
• Non–fibrin-specific agents (eg, streptokinase) catalyze systemic fibrinolysis. Streptokinase is indicated for the treatment of AMI,
acute massive pulmonary embolism (PE), deep vein thrombosis (DVT), arterial thrombosis, and occluded arteriovenous cannulae.
• Fibrinolytic agents can be administered systematically or can be delivered directly into the area of the thrombus. Systemic delivery
is used for treatment of AMI, acute ischemic stroke (AIS), and most cases of acute massive PE. Peripheral arterial thrombi and
thrombi in the proximal deep veins of the leg are most often treated via a catheter-directed approach.
Reference: http://emedicine.medscape.com/article/811234-overview#a2
Https://en.wikipedia.org/wiki/Thrombus




211.What the scientific term of chewing ice?
• Pagophagia
Answer: A, is a form of the disorder pica involving the compulsive consumption of ice or iced drinks.
Link: http://www.mayoclinic.org/diseases-conditions/iron-deficiency-anemia/expert-answers/chewing-ice/faq-20057982


212.Pt with high aptt, what mechanism of action of that drug?
• " unfractionated heparin" Antithrombin
Answer:
Medscape: Unfractionated Heparin = “Its anticoagulation effect is mediated by the activation of antithrombin III, which then
inactivates with relatively equal potency the coagulation enzymes thrombin (factor iia) and factor Xa. Other antithrombotic effects
include inhibition of platelet aggregation and additional antithrombin III-independent mechanisms. The partial thromboplastin time
may be elevated because of inactivation of thrombin.

130


213.Case of Fanconi syndrome
Answer:
A type of RTA a disease of the proximal renal tubules in which glucose, AA, uric acid, P and bicarbonate are passed into the urine,
instead of being reabsorbed. It may be inherited, or caused by drugs or heavy metals.
Clinical features of proximal renal tubular acidosis are:
• Polyuria, polydipsia and dehydration
• Hypophosphatemic rickets (in children) and osteomalacia (in adults)
• Growth failure
• Acidosis
• Hypokalemia
• Hyperchloremia
Other features of the generalized proximal tubular dysfunction of the Fanconi syndrome are:
• Hypophosphatemia/phosphaturia
• Glycosuria
• Proteinuria/aminoaciduria
• Hyperuricosuria
- treatment :
Mainly consists of the replacement of substances lost in the urine. Mainly fluids and electrolytes.
• Dehydration due to polyuria must be prevented by allowing free access to water; treat dehydration with either oral or
parenteral solutions.
• Metabolic acidosis due to the loss of bicarbonate is corrected by 3-10 mg/kg/d of sodium bicarbonate in divided doses.
• Diuretic, as 1-3 mg/kg/d of hydrochlorothiazide, may be necessary to avoid volume expansion, need to augment potassium
supplementation in the form of potassium bicarbonate, citrate, or acetate.
a. Phosphate and vitamin D supplementation are necessary, 1-3 g/d of supplemental phosphate
Https://en.wikipedia.org/wiki/Fanconi_syndrome
Http://emedicine.medscape.com/article/981774-treatment#d6



214.Case of multiple myeloma
Answer:
-cancer of plasma cells in the bone marrow.
-Criteria for diagnosis:
- A bone marrow aspirate or biopsy showing that at least 10% of the cells are plasma cells or the presence of a plasma cell tumor
(called a plasmacytoma), plus at least one of the following two features:
- Evidence of damage to the body as a result of the plasma cell growth, as severe bone damage, kidney failure, anemia, or high
calcium in the blood, and/or
- Detection of one of the following findings: ≥60% plasma cells in the bone marrow; free light chain ratio of 100 or more (provided
involved FLC level is at least 100 mg/L); or MRI showing more than one lesion (involving bone or bone marrow).
Variety of symptoms:
-bone pain in the back or chest, or less commonly, the arms and legs. The pain is usually triggered by movement and is absent at
night, except when changing positions, The bone loss and erosions can lead to osteoporosis and fractures “pathological fracture”
-High blood calcium levels : hyperca symptoms loss of appetite, nausea, vomiting, frequent urination, increased thirst, constipation,
weakness, confusion, stupor, or coma.
-Anemia: paleness, weakness, and fatigue.
-Impaired kidney function: Occasionally, kidney failure is the first sign of MM.
-hyperviscosity syndrome The symptoms may include bleeding from the nose and mouth, blurred vision, neurologic symptoms, and
heart failure.
-Neurologic symptoms radiculopathy causing numbness or tingling, pain, or muscle weakness.
- Generalized symptoms — The generalized symptoms of MM include an increased susceptibility to infections (especially during
chemotherapy) and weight loss.
Dx: Blood and urine tests for monoclonal protein , Bone marrow examination (aspiration and biopsy)
Http://www.uptodate.com/contents/multiple-myeloma-symptoms-diagnosis-and-staging-beyond-the-basics#H3498669556

131
215.Patient with prolonged PT and PTT 3

Answer:
Http://www.practical-haemostasis.com/Screening%20Tests/aptt.html

216.Case of multiple myeloma


Myeloma is an abnormal proliferation of plasma cells. These plasma cells are unregulated in their production of useless
immunoglobulin that is usuallyIgGor iga.IgMis a separate disease called Waldenstrom macroglobulinemia. These immunoglobulins
do not fight infection but clog up the kidney. The most frequent presentation of MM is with bone pain caused by a fracture occurring
under normal use.. Also may present with Hyperuricemia ,anemia and renal failure .
Diagnostic Testing :
· Skeletal survey to detect punched out osteolytic lesions. (Osteoblastic lesions suggest metastatic prostate cancer.)
· Serum protein electrophoresis (SPEP):You are looking for elevated levels of monoclonal antibody (usually igg). 20% are iga.
· Urine protein electrophoresis (UPEP):Detects Bence-Jones protein.
· Peripheral smear:Shows “rouleaux” formation of blood cells.
-The single most specific test is the bone marrow biopsy,which detects high numbers of plasma cells (10 percent).
Treatment :
Treat with melphalan and steroids. Thalidomide, lenalidomide, or bortezomibmay be added The most effective therapy is an
autologous stem cell bone marrow transplantation.
Reference : master the boards usmle .

217.How to differentiate between hypersplenism and aplastic anemia?


Answer:
• Aplastic anemia is characterized by diminished or absent hematopoietic precursors in the bone marrow “empty marrow”,
most often due to injury to the pluripotent stem cell. Major causes: idiopathic, cytotoxic drug & radiation, drug reaction
(“Anticonvulsants”: carbamazepine Antibiotics: sulfonamides, chloramphenicol, “NSAIDs” phenylbutazone, indomethacin,
“anti thyroid” methimazole, propylthiouracil), Toxic chemicals, viral infections (EBV, HIV, other herpes), Immune disorders.
• Hypersplenism is a condition in which the spleen becomes increasingly active and then rapidly removes the blood cells. It
can result from any cause of splenomegaly. It is most common with splenomegaly secondary to portal hypertension and
hematological disorders. The bone marrow in case of hypersplenism is cellular unlike in aplastic anemia.
Reference: http://www.researchgate.net/publication/237841754_Hypersplenism_Review_article


218.ITP case management >> IVIG, prednisolone


132

219.High mcv + ast which type of anemia.
Answer: could be both folic acid and vitamin b12 deficiency. Both answers were provided.
Serum total homocysteine levels are almost always elevated in patients with folate deficiency because folate is required in the
remethylation step that converts homocysteine to methionine.[16] serum methylmalonic acid and homocysteine levels are
increased early in vitamin b-12 deficiency, even before hematologic manifestations or decreases in b-12 levels are noted.[17]
http://emedicine.medscape.com/article/203858-overview


220.A male patient from guinea with a neck mass the biopsy showed starry appearance
A. Burkitt lymphoma
Answer:a
Burkitt lymphoma, or small noncleaved cell lymphoma, is a highly aggressive b-cell non-hodgkin lymphoma.under the microscope,
the hallmark of burkitt lymphoma (bl) is the presence of a "starry sky" appearance.
Reference:http://emedicine.medscape.com/article/1447602-overview#a5


221.Pt with past hx of Hodgkin lymphoma, cured completely. Presented with back pain
Answer

222.What factor cause thrombosis ?
answer :
Risk Factors for VTE:
• Stasis
• Age > 40
• Immobility
• CHF
• Stroke
• Paralysis
• Spinal Cord injury
• Hyperviscosity
• Polycythemia
• Severe COPD
• Anesthesia
• Obesity
• Varicose Veins
• Hypercoagulability
• Cancer
• High estrogen states
• Inflammatory Bowel
• Nephrotic Syndrome
• Sepsis
• Smoking
• Pregnancy
• Thrombophilia
• Endothelial Damage
• Surgery
• Prior VTE
• Central lines
• Trauma
Reference: Anderson FA Jr. & Wheeler HB. Clin Chest Med 1995;16:235.




133


Neurology

134
1. Patients with epilepsy, which of the following receptors most likely is stimulated?
A. Protein g
B. Glutamate
C. Serotonin
D. Kinase
Answer: b
Glutamate is the major excitatory neurotransmitter in the brain. The release of glutamate causes an epsp in the postsynaptic neuron
by activating the glutaminergic receptors ampa/kainate and nmda and the metabotropic receptor.
Reference: http://emedicine.medscape.com/article/1184846-overview#a3


2. What is the most accurate test for carpal tunnel syndrome?

A. Tinel
B. Compression test
C. Durkan's carpal test
D. Phallens test

Answer: all the resources i found were clinical studies with various numbers.
Specific tests might be as the following order:
• 1-''two point discrimination <6 mm with caliper (33% sensitive, 100% specific).
• Flick test (93% sensitive, 96% specific)
• Durkan's test (sensitivity from 87% to 91% and its specificity from 90% to 95%)
• 4-tinel's sign (44-70% sensitive, 94% specific)
• Carpal compression test (90% sensitive, 90% specific)
• Phalen's maneuver (70-80% sensitive, 80% specific).
So, from the choices above, most likely it is durkan test or tinnel
Resource http://fpnotebook.com/ortho/wrist/crpltnlsyndrm.htm
& https://en.wikipedia.org/wiki/durkan%27s_test

3. Patient has parotitis complains of pain with eating that radiate to the ear , with nerve transmit pain with eating?
A. 8
B. 9
C. 10
D. 7
Answer: Crinial nerve 5 (Trigeminal) >> Mandbular branceh (V3) >>> auriculotemporal
“The pain produced can be referred to the external ear. This is because the auriculotemporal nerve provides
sensory innervation to the parotid gland and the external ear.:

4. Patient complain of headache (worst headache in life) csf result (blood)


A. Brain abscess
B. Brain infection
C. Epidural hematoma.
D. Ruptured berry aneurism.

Answer: d. Ruptured berry aneurism. This is subarachnoid hemorrhage and it’s classically description as “the worst headache of my
life” and one of its causes is berry aneurism rupture. Step up to medicine

5. Patient with increase in icp, complains of vomiting, tinnitus, nausea, headache and blurred vision, the doctor orders him a ct
scan, what the cranial nerve will discover to know the diagnosis before doing imaging study?
A. Orbital
B. Facial
135
C. Trochlear
D. Occulomotor

Answer: ?
Uptodate: https://yhdp.vn/uptodate/contents/mobipreview.htm?32/39/33402
Clinical manifestations: signs include cn vi palsies (abducens), papilledema secondary to impaired axonal transport and congestion,
spontaneous periorbital bruising and a triad of bradycardia, respiratory depression, and hypertension (cushing's triad, sometimes called
cushing's reflex or cushing's response).

6. After a motor vehicle accident, a patient can not bring the spoon to feed himself. Which cerebral lobe is affected?
A. Temporal
B. Parietal
C. Occipital
D. Cerebellum
Answer: d
The cerebellum: is principally concerned with balance and the regulation of posture, muscle tone and muscular co-ordination.
Frontal lobe: a-the motor cortex: the primary motor area. It receives afferents from the premotor cortex, thalamus and cerebellum and
is concerned with voluntary movements. B-the premotor cortex. *clinical anatomy applied anatomy for students and junior doctors.

7. A seizing mother was given phenobarbital. How will you advise her regarding her lactation?
A. Stop immediately lactation
B. Wait for 8 hours after taking the drug then lactate
C. Stop lactating gradually over 3 weeks
D. Continue lactation
Answer:

8. 32 years old male presenting with left sided headache associated with nausea, vomiting, photophobia, aura, lasting for 12
hours occurring 4-5 times per month, what's your best modality of investigation?
A. Cbc
B. Esr
C. Precise history and physical examination.
D. Mri
Answer: c

9. Which nerve carries the referred pain of the parotid to the ear?
A. Vagus
B. Facial
C. Auriculotemporal
D. Trigeminal
Answer: C

Answer: Crinial nerve 5 (Trigeminal) >> Mandbular branceh (V3) >>> auriculotemporal
“The pain produced can be referred to the external ear. This is because the auriculotemporal nerve provides
sensory innervation to the parotid gland and the external ear.:

10. Old male hemiplegia, ct showed ischemia, examination normal except for hemiplegia, now he is stable doing physiotherapy.
Taking hydrochlorothiazide. 2 yrs ago had a history of acute gastric ulcer...
What to give him:
A. Do nothing
B. Tpa
C. Aspirin
D. Warfarin
Answer: c

136
Tpa: time to therapy for acute ischemic stroke management is 3 - 4.5 hours. Contraindication to tpa …
aspirin: aha/asa guidelines recommend giving aspirin, 325 mg orally, within 24-48 hours of ischemic stroke onset. Warfarin:
currently, data are inadequate to justify the routine use of heparin or other anticoagulants in the acute management of ischemic stroke.
Http://emedicine.medscape.com/article/1916852-treatment#showall

11. Which antiviral used in parkinsonism:


A. Ribiverin
B. Zuvedamin
C. Gancyclovir
D. Amantadine
Answer: d

12. Unwanted side effect of anticholinergics?


A. Diarrhea
B. Blurred vision
C. Excessive salivation
D. Urinary incontinence
Answer: b
According to medscape, remember common signs and symptoms with the mnemonic, "red as a beet, dry as a bone, blind as a bat, mad
as a hatter, hot as a hare, and full as a flask." the mnemonic refers to the symptoms of flushing, dry skin and mucous membranes,
mydriasis with loss of accommodation, altered mental status, fever, and urinary retention, respectively.
Http://emedicine.medscape.com/article/812644-clinical#showall

13. Mainstay treatment for parkinson’s disease?


A. Dopamine agonists
B. Mao inhibitors
C. Anticholinergics
D. Levodopa
Answer: d
Dopamine agonists provide symptomatic benefit that is comparable to that with levodopa/carbidopa in early disease, but these agents
lack sufficient efficacy in more advanced disease. They are commonly reserved for younger individuals (< 65-70 years) who are
cognitively intact. Mao-b inhibitors, such as selegiline and rasagiline, may be used for early symptomatic treatment of parkinson
disease. These medications provide mild symptomatic benefit & have excellent adverse effect profiles. Anticholinergic agents: are
not first-line drugs, because of their limited efficacy and the possibility of neuropsychiatric side effects. Levodopa, coupled with a
peripheral decarboxylase inhibitor such as carbidopa, remains the standard of symptomatic treatment for parkinson disease. It provides
the greatest antiparkinsonian benefit with the fewest adverse effects in the short term.

14. Which of the following drugs causes hair growth:


A. Phenytoin
B. Phenobarbital
C. Valproic acid
D. Carbamazepine
Answer: both a&d may cause hair growth.

15. Patients with epilepsy, which of the following receptors most likely is stimulated?
A. Protein g
B. Glutamate
C. Serotonin
D. Kinase
Answer: - glutamate
Https://en.wikipedia.org/wiki/glutamate_receptor#seizures

16. Adolescent with fever, headache, meninges after swimming in a river; causative organism

137
A. Streptococcus
B. H. Influenza
C. N. Meningitides
D. Naegleria fowelri
Answer: d
It is a rare deadly infection of the meninges by that amoeba.
Http://www.cdc.gov/parasites/naegleria/

17. Side effect of anticholinergic. Hot as a hare: increased body temperature


A. Blind as a bat: mydriasis (dilated pupils
B. Dry as a bone: dry mouth, dry eyes, decreased sweat
C. Red as a beet: flushed face
D. Mad as a hatter: delirium

18. Case of meningitis caused by meningococcal type b. Which of the following can decrease risk of spreading of the infection:
A. Do nothing
B. Give ceftriaxone or cefotaxime to decrease risk of spreading from nasal mucosa
C. Give meningiooccal vaccine for pt & contact
D. Isolation of all contact for 4 weeks

19. Loss of sense of smell which lobe affected?


A. Frontal
B. Occipital
C. Parietal
D. Temporal
Answer: d?
Temporal lobe—senses of smell and sound, as well as processing of complex stimuli like faces and scenes
Frontal lobe—conscious thought; damage can result in mood changes, social differences, etc. The frontal lobes are the most
uniquely human of all the brain structures.
Parietal lobe—plays important roles in integrating sensory information from various senses, and in the manipulation of objects;
portions of the parietal lobe are involved with visuospatial processing
Occipital lobe—sense of sight; lesions can produce hallucinations
Limbic system—emotion, memory, emotional expression, resolve conflict from frustration. "odors often trigger emotional reactions
and memories" (elaine & katja, 2015).
Insula—"island inside" pain, taste, hunger, visceral functions, social emotions, time perception and awareness "connects to the
cortex and the limbic systems" (blanc et. Al., 2014).

- The olfactory cortex is located on the base of the frontal lobe and medial aspect of the temporal
lobe

20. A patient with tuberculosis on medication for 3 months. He developed pins and needles sensation of his lower limbs.
Deficiency of which of the following caused his symptoms?
A. Niacin
B. Folic acid
C. Iron
D. Pyridoxine (B6)

Answer: D
Vitamin B6 (pyridoxine) supplementation during isoniazid (INH) therapy is necessary in some patients to prevent the development of
peripheral neuropathy.
Reference: Pubmed.

21. A girl with migraine. She doesn’t want to take prophylactic medicine.
A. Biofeedback
138
B. Sumatriptan
C. Ergotamine
D. Propranolol

Answer: A

22. College student have meningitis. What to do as a prophylaxis to dorm friends next?
A. Isolate all contacts for 4 weeks
B. Immunize all contacts
C. Give antibiotic ( Penicillin and other similar antibiotics) – exact sentence was written
D. Do nothing

Answer: d
- “Close contacts” means those who have major respiratory fluid contact, such as
Household contacts, kissing, or sharing cigarettes or eating utensils.
- Routine school and work contacts are not close contacts. Sitting in class with
Someone with Neisseria infection does not make them a close contact Reference: master the board

Close contacts — Chemoprophylaxis is indicated in close contacts of patients with meningococcal infection and should be given as
early as possible following the exposure. Although "close contact" has not been clearly defined, it generally refers to individuals who
have had prolonged (>8 hours) contact while in close proximity (<3 ft) to the patient or who have been directly exposed to the patient's
oral secretions during the seven days before the onset of the patient's symptoms and until 24 hours after initiation of appropriate
antibiotic therapy
Close contacts may include individuals exposed in the following ways
●Household members, roommates, intimate contacts, contacts at a childcare center, young adults exposed in dormitories, military
recruits exposed in training centers
●Travelers who had direct contact with respiratory secretions from an index patient or who were seated directly next to an index
patient on a prolonged flight (ie, one lasting ≥8 hours)
●Individuals who have been exposed to oral secretions (eg, intimate kissing, mouth-to-mouth resuscitation, endotracheal intubation, or
endotracheal tube management)
Prophylaxis is not indicated if exposure to the index case is brief. This includes the majority of healthcare workers unless there is
direct exposure to respiratory secretions (as with suctioning or intubation).

Secondary cases usually occur within 10 days of the primary case,

chemoprophylaxis administered >14 days after exposure is not recommended by the CDC.

23. Old patient presented with agitation, urinary incontinence, confusion and impaired short-term memory. Long-term memory
is intact. CT shows temporal and hippocampal atrophy. Which of the following genes might be affected?
A. 13
B. 15
C. 18
D. X

Answer: 19?
Alzheimer’s Disease:
Early-onset, (age 30 to 60): 3 major genes for autosomal dominant AD have been identified: amyloid precursor protein (chromosome
21), presenilin 1 (chromosome 14) and presenilin 2 (chromosome 1).
139
Late-onset (mid-60s and later): The E4 polymorphism of apolipoprotein E is a susceptibility genotype (E2 is protective). The APOE
gene is located on the long (q) arm of chromosome 19.
Reference: Toronto Notes

24. A patient with high ICP (I think it was due to a brain tumor), which of the following nerves is most likely to be affected?
A. Trochlear
B. 6th cranial
C. Optic
D. Facial

Answer: B
Increase ICP leads to CN III and VI deficient.
Papilledema >>> optic nerve
Reference: First Aid USMLE 2.

25. A 72 y male disoriented and hallucinating and disorganized thinking had aorto popletial graft and symptom fluctuates in the 2
days what the cause?
A. Multi infraction demntia
B. Mania
C. Demensia
D. Delirium

Answer: D (post operative delirium)
Delirium is a common but often undiagnosed complication in the elderly following a major operation. There are multiple risk factors
for developing postoperative delirium. Risk factors can be separated into patient-specific and operation-specific risk factors.
Established patient-specific risk factors for the development of delirium following noncardiac operations include pre-existing
dementia(strongest factor), older age, functional impairment, greater co-morbidities, and psycopathological symptoms. Operation-
specific risk factors for the development of postoperative delirium are based on the degree of operative stress. For example, low
operative stress procedures such as cataract surgery result in delirium in 4% of cases in comparison to high surgical risk procedures
such as vascular operations.
Http://www.ncbi.nlm.nih.gov/pmc/articles/PMC2546478/

26. Hempligic pt come with some thing in the eye( may be nystamius or some thing )where is the lesion in the brain :
A. Pons
B. Medulla oblangta
C. Interna capsule
D. Midbrain


27. Pt with absence seizure the doctor wants to start him on sodium valproate, what test should be done before starting the tx?
A. Liver function
B. Creatinine
C. Urea

Answer: a (hepatotoxicity & teratogenicity)
Side effects of vpa include nausea, vomiting, hair loss, easy bruising, and tremor. Vpa is associated with weight gain, obesity, insulin
resistance, and the metabolic syndrome. Vpa can also cause thrombocytopenia and other coagulation disturbances and has also
been associated with subclinical hypothyroidism with mild to moderate elevations in thyrotropin (TSH ) levels. Vpa has also been
linked to the polycystic ovarian syndrome.
Vpa-exposure in utero is associated with major malformations and other adverse effects, including neurodevelopmental
abnormalities. Vpa should be avoided in pregnancy when possible
Approximately 5 to 10 percent of patients develop ALT elevations during long term vpa therapy; most of the time these
abnormalities are asymptomatic and can even resolve with continuation of the drug. In addition, there are more serious forms of
hepatotoxicity that can occur with vpa:

140
The fda recommends checking LFTs prior to initiating treatment and at frequent intervals thereafter, especially during the first six
months
Reference: uptodate, toronto notes page (n54)
Http://www.drugs.com/sfx/valproic-acid-side-effects.html
Http://www.epilepsy.com/medications/valproic-acid
Http://reference.medscape.com/drug/depakene-stavzor-valproic-acid-343024#5

28. Pt had head trauma, he has nausea, vomiting and decreased level of consciousness, icp was suspected and ct was arranged,
what cranial nerve examination can confirm the dx?
A. Optic
B. Oculomotor
C. Trochlear
Answer: b
Uptodate: “ global symptoms of elevated icp include headache, which is probably mediated via the pain fibers of cranial nerve (cn) v
in the dura and blood vessels, depressed global consciousness due to either the local effect of mass lesions or pressure on the
midbrain reticular formation, and vomiting.

29. Most common intracranial tumor in adults?


A. Hemangioblastoma
B. Ependymoma
C. Schwannoma

Answer: b
Gliomas are the most prevalent type of adult primary brain tumor, accounting for 78 percent of malignant brain tumors.
Meningiomas are the most common benign intracranial tumors.
Pituitary adenomas are the most common intracranial tumors after gliomas, meningiomas and schwannomas. So, the most common
brain tumors are (in sequence)
• Glioma
• Meningioma
• Schwannoma
• Pituitary adenoma
Gliomas include: astrocytoma, ependymoma, glioblastoma multiform, medulloblastoma, and oligodendroglioma.
Most common in children: medulloblastoma
Reference: american association of neurological surgeons
http://www.aans.org/patient%20information/conditions%20and%20treatments/brain%20tumors.aspx


30. Most common risk factor for stroke:
A. Htn
B. Atrial fibrillation
C. Ldl

Answer: a
Reference: step-up of medicine

31. Case of migraine what next step:


A. Mri
B. Blood culture
C. More history and physical ex
Answer: c

32. Patient complain of headache band like distribution ?


A. Cluster
B. Migraine

141
C. Tension
Answer: c

33. Best anticonvulsant for treatment of trigeminal neuralgia;


A. Phynetoin
B. Carbamezabin
C. Ethuximab
Answer: b
Http://emedicine.medscape.com/article/1145144-treatment

34. Which one risky (more) to stroke?


A. 55 , male, HTN , obese
B. 50, male , dm, hypercholesterolemia
C. 22,female,smoker
Answer: a
Risk factors for stroke are:
• The most important risk factors are age and htn. Others include smoking, dm, hyperlipidemia, atrial fibrillation, coronary artery
disease, family history of stroke, previous stroke/tia, and carotid bruits.
• In younger patients, risk factors include oral contraceptive use, hypercoagulable states (e.g., protein c and s deficiencies,
antiphospholipid antibody syndrome), vasoconstrictive drug use (e.g., cocaine, amphetamines), polycythemia vera, and sickle cell
disease.

35. Women brought her father had dementia of recent events what will you do?
A. Refer to geriatric
B. Give him antipsycotic
C. Measure iq
Answer: a
Explanation: no reference for explanation.

36. Patient admitted to hospital with headache, nausea and vomiting (signs of increased icp ) what you find in eye examination ?
A. Papiledema
B. Central retinal artery ischemia
C. Glaucoma
Answer: a

37. Patient presented with status epilepticus, lorazepam was given, however the patient didn’t improve. What is the most
appropriate drug to be given?
A. IV phenobarbital
B. IV phenytoin
C. Oral carbamazepine

Answer: b
The first line in managing status epilepticus are benzodiazepines, second line is phenytoin and third line is phenobarbital.
Http://bestpractice.bmj.com/best-practice/monograph/464/treatment/details.html

38. Amyotrophic lateral sclerosis which horn is infected>>


A. Anterior horn
B. Lateral horn
C. Posterior horn

Answer: a.

142
Anterior horn cell. Als: a disorder affecting the anterior horn cells and corticospinal tracts at many levels. Corticobulbar involvement
is common as well. Step up to medicine.

39. Type of cerebral palsy lower paralysis more than upper?


A. Diplagia
B. Quadriplagia
C. Hemiplagia
Answer: a

40. Best strategy to decrease stroke among public:


A. Anti smoking campaign
B. Free blood lipid screening
C. Blood pressure screening
Answer: c
Hypertension is the most important modifiable risk factor for stroke and intracerebral hemorrhage (ich), and the risk of stroke
increases progressively with increasing blood pressure, independent of other factors. Regular blood pressure screening and a
combination of behavioral lifestyle modification and drug therapy are recommended to achieve these goals. Studies on the
comparative benefits of specific classes of antihypertensive agents have not shown definitive results. In patients who have
hypertension with diabetes or renal disease, the blood pressure goal is < 130/80 mm hg

41. Cavernous sinus thromboembolism. Which sinus is more common?


A. Ophthalmic
B. Maxillary
C. Sphenoid
Answer: c
Most commonly nasal furuncle then ethmoid and sphenoid sinuses.

CST is more commonly seen with sphenoid and ethmoid and to a lesser degree with frontal sinusitis.
The cavernous sinuses receive venous blood from the facial veins (via the superior and inferior ophthalmic veins) as well as the
sphenoid and middle cerebral veins.

42. Young male had a history of two seizures in the last 30 minutes, no history of diabetes no history of head trauma, at initial
presentation had, another seizure. What is the best to give him now?
A. Phenytoin
B. Diazepam
C. Phenobarbital
Answer: b
Benzodiazepines are the preferred drug class for the initial treatment of se. Lorazepam, when available, is thought to be the most
effective of the benzodiazepines and has a longer seizure half-life than diazepam. Phenytoin or fosphenytoin (cerebyx) is the next drug
to be administered when a second drug is needed. Failure to respond to optimal benzodiazepine and phenytoin loading operationally
defines refractory se.
Http://emedicine.medscape.com/article/1609294-overview#a11

43. A 30-year-old male presented with severe headache, never had like this headache before, photophobia, neck rigidity.
Intracranial hemorrhage was suspected. Where is the most likely site of this hemorrhage?
A. Intracerebral
B. Subarachnoid
C. Epidural
Answer: b
Symptoms of subarachnoid hemorrhage include; headache described as “worst headache of my life” (up to 97% sensitive, 12-25%
specific), n/v, photophobia, meningismus (neck pain/stiffness, positive kernig’s and brudzinski’s sign), decreased loc. Toronto notes
2015

143
44. Lt. Hemiplagia, MRI at t2 show hyperdense area in the rt. Side of the brain. Which of the following will worsening the pt
prognosis?
A. blood glucose < 6.5
B. Blood pressure < 140
C. Incomplete mcq

45. Patient with exophthalmos and swollen lids and you can feel its pulse, TFT normal. Whats ur diagnosis.
A. Hyperthyroid
B. Cellulitis
C. Cavernous sinus thrombosis
Answer c
Reference: http://emedicine.medscape.com/article/791704-clinical#showall
It can be b according to other info if provided in the q

46. Clear case of absence seizure then asked what is going to happen to this patient if given fentanyl :
A. Glutamate receptors activation
B. Seizure activity due to toxic neurotransmitters release
C. Demyelination.
Answer: b
Epidural fentanyl is local anesthetic : moa due to its high lipid solubility it rapidly binds dorsal horn receptors in the spinal cord. G-
protein coupled receptors & inhibit adenylate cyclase
Nb: its not related to absence seizure.
fentanyl-induced epileptiform activity on the electrocorticogram

47. Teacher who suffer from headache on temporal side , ct normal blood pressure normal he was taking paracetamol but not
improve well , what can you give ?
A. Aspirin
B. Ibuprofen
C. Triptans
Answer: c (for treatment of migraine and cluster headache)
If the scenario goes with tension type; give ibuprofen. If it goes with temporal arteritis; give steroid with aspirin. If it goes with
migraine; give triptans.

48. Medical student diagnosed as meningitis, what you do for him?


A. Start antibiotics
B. Isolate him for 4 weeks
C. Give him influenza vaccine for his colleagues

Answer: a
bacterial meningitis must be the first and foremost consideration in the differential diagnosis of patients with headache, neck stiffness,
fever, and altered mental status. Acute bacterial meningitis is a medical emergency, and delays in instituting effective antimicrobial
therapy result in increased morbidity and mortality.
[http://emedicine.medscape.com/article/232915-workup]

49. Female complaining of hair growth no seizure for 7 years wants to stop the drug?
A. Stop the drug after 6 months (tapering)
B. Continue the drug indifinitly
C. Continue till 10 years

Answer: a

50. Gingival hypertrophy side effect


A. Carbamazepine
B. Phenytoin
C. Phenobarbital
144
Answer: b
• Gastrointestinal side effects including gingival hyperplasia (in as many as 50% of treated patients) have been reported.
The gingival hyperplasia associated with phenytoin is occasionally severe enough to merit surgical removal.

Reference: http://www.drugs.com/sfx/phenytoin-side-effects.html


51. Reversible cause of stroke?
A. Hypertension
B. Obesity
C. Smoking
Answer: a

52. How you can confirm Brain stem death?


A. Absence of doll eye
B. Active coughing & gag reflex
C. One inactive pupil

Answer: A
The process for brain death certification includes “3- Performance of a complete neurological examination. Components of a
complete neurological examination are: iii. Absent corneal, oculocephalic (doll's eye ), cough and gag reflexes. The corneal reflex
may be altered as a result of facial weakness.” Negative vestibule-ocular reflex "doll's eyes" would stay fixed midorbit, and having
negative "doll's eyes" is therefore a sign that a comatose patient's brainstem is functionally not intact.
Reference: http://www.ncbi.nlm.nih.gov/pmc/articles/PMC2772257/
Https://en.wikipedia.org/wiki/Vestibulo%E2%80%93ocular_reflex

53. Case of Absence seizure What is best treatment for that? " 4 times
A. Phonation
B. Carbazapin
C. Ethosuximide

Answer: C
Ref: Only 2 first-line aeds have approval from the US Food and Drug Administration (FDA) to be indicated for absence seizures:
ethosuximide (Zarontin) and valproic acid (Depakene, Depacon). Ethosuximide has efficacy for absence seizures only and valproic
acid has efficacy for absence, generalized tonic-clonic, and myoclonic seizures.
Ethosuximide (Zarontin) is effective only against absence seizures.
Reference: http://reference.medscape.com/article/1183858-overview#a9

54. 45 years old with proximal muscle weakness, tongue fasciculation, with history of recurrent aspiration pneumonia. What is
the diagnosis?
A. Mononeuropathy
B. Myasthenia gravis
C. Mysthenic syndrome

Answer: B. Myasthenia Gravis


Reference: http://m.jnnp.bmj.com/content/73/6/766.full

55. 23 y girl come e one eye blindness+past hx 2 year back headech + diplopia ...ect best investigation?
A. Mri
B. Ct
C. Cbc

145
Multiple sclerosis is probable and MRI is the probable answer


56. Bacterial meningitis in lp:
A. Decrease glucose and increase protein
B. Increase glucose and decrease protein

Answer: a

57. A women with neck pain after the pain there is tingling and shock like waves and weakness ,what is diagnosis
A. Whiplash injury
B. Brachial plexus injury
Answer:?


58. Lady with headache " worst headache in her life, neck stiffness, blood in
Csf:
A. Epidural hemorrhage
B. Ruptured aneurysm
Answer: b
Subarachnoid hemorrhage is sudden bleeding into the subarachnoid space. The most common cause of spontaneous bleeding is a
ruptured aneurysm. Symptoms include sudden, severe headache, usually with loss or impairment of consciousness. Secondary
vasospasm (causing focal brain ischemia), meningismus, and hydrocephalus (causing persistent headache and obtundation) are common.
Diagnosis is by ct or mri; if neuroimaging is normal, diagnosis is by csf analysis. Treatment is with supportive measures and
neurosurgery or endovascular measures, preferably in a comprehensive stroke center.

59. 16-year-old came with headache band like, behind eye throbbing. Stressful life
A. Migraine
B. Tension

Answer: b http://emedicine.medscape.com/article/792384-clinical

60. Patient after motor vehicle accident suspected increase icp. What cranial nerve to check for?
A. Optic
B. Trochlear
Answer: b
The sixth cranial nerve as at risk of injury due to stretch which might cause diplopia
146
Http://www.ajol.info/index.php/cme/article/download/87803/77457

61. Guillain-barre syndrome?


A. Ascending paralysis more in the legs
B. Descending asymmetrical paralysis

Answer: a
Davidson’s medicine page 1229: distal paraesthesia and limb pain (often severe) precede a rapidly ascending muscle weakness, from
lower to upper limbs, more marked proximally than distally. ß proximally in the limb

62. Vegetarian with numbness and socks and gloves distribution:


A. B6 deficiency
B. B12 deficiency
Answer: b

63. Most common brain tumor:


A. Epindemoma
B. Astrocytoma
Answer:
The option was glioma types : asrrosytoma and glioblastoma multiform the most common.

64. Glove nuropathy cause?


A. Vitamin b12
B. Vitamin b6
Answer: a

Presentations vary greatly among patients. The symmetric glove-and-stocking paresthesias, or tingling in the distal aspect of the toes,
numbness, coldness, a pins-and-needles feeling, and occasional feelings of swelling or constriction, are slowly progressive and
insidious. Symptoms progress up the legs, occasionally affect the fingers, and culminate in weakness and spasticity.
Link: http://emedicine.medscape.com/article/1171558-clinical

65. Gillian barre syndrome, distal progresses paralysis upper and lower limb what will you find on csf?
A. Increase protein
B. Decrease glucose
Answer: a

Lumbar puncture
Most, but not all, patients with gbs have an elevated csf protein level (>400 mg/l), with normal csf cell counts. Elevated or rising
protein levels on serial lumbar punctures and 10 or fewer mononuclear cells/mm3 strongly support the diagnosis.

66. Baby after operation has loss of gag reflex in left side, normal uveal movement. What nerve injured?
A. Glossopharyngeal
B. Vagus
Answer: a
The sensory limb is mediated predominantly by cn ix (glossopharyngeal nerve)
The motor limb by cn x (vagus nerve). In unilateral (one-sided) glossopharyngeal nerve (cn ix- sensory component) damage, there will
be no gag response when touching the pharyngeal wall on the same side of the damaged nerve. With one-sided vagal nerve (cn x-
motor component) damage, the soft palate will elevate and pull toward the intact side regardless of the side of the pharynx that is
touched. This is because the sensory component is intact on both sides, but only the motor nerves supplying one side of the soft
palatine and pharyngeal muscles is working, therefore the contraction of the muscles in the reflex is asymmetrical. If both cn ix and x
are damaged on one side (not uncommon), stimulation of the normal side elicits only a unilateral response, with deviation of the soft
palate to that side; no consensual response is seen. Touching the damaged side produces no response at all.
147
67. Old patient with recent memory loss and poor self care and social withdrawal, what to give him ?
A. Neostigmen
B. Rivastigmen

Answer: b
The treatment for mild to moderate alzheimer’s disease is central cholinesterase inhibition. We chose rivastagmine because it crosses
the blood-brain barrier while neostigmine doesn’t.
Http://emedicine.medscape.com/article/1134817-treatment#d10

68. Most common neurosurgical tumor?


A. Astrocytoma.
B. Medullablastoma.

Answer:
gliomas (principally astrocytoma) account for approximately 30% and meningioma 35% to 40% of symptomatic primary brain
tumors.

reference : pic and uptodate

69. A pt with meningitis, which combination of antibiotic should be given ?


A. Vancomycin + ampicillin
B. Amoxicillin + gentamycin
Answer: a

According to age

Reference: step-up to medicine 4th edition

148
70. Patient with seizure, what is the treatment
A. Phenobarbitone
B. Phenytoin
Answer: depend on the type of seizure
toronto:
antiepileptic drugs:
• Generalized-onset and partial-onset seizures: felbamate, lamotrigine, levetiracetam, refinamide, topiramate, valproate, zonisamide
• Partial seizures (simple partial, complex partial, and secondarily generalized seizures): carbamazepine, gabapentin, lacosamide,
oxcarbazepine, phenobarbital, phenytoin, pregabalin, primidone, tiagabine, vigabatrin (note: these drugs may exacerbate
generalized seizures)
• Absence seizures: ethosuximide

71. A patient with myasthenia gravis presents with myasthenic crisis. What is the cause?
A. Anticholinesterases
B. Pyridostigmine

Answer: ?
Numerous medications may exacerbate MG, including quinidine, procainamide, β-adrenergic antagonists, calcium channel
antagonists (verapamil, nifedipine, felodipine), magnesium, antibiotics (ampicillin, gentamicin, streptomycin, polymyxin,
ciprofloxacin, erythromycin), phenytoin, gabapentin, methimazole, α-interferon and contrast media. Reference: Pubmed

Myasthenia gravis (MG) is a disorder of muscular weakness from the


production of antibodies against acetylcholine receptors at the
neuromuscular junction.
Best initial treatment of myasthenia gravis: Neostigmine or pyridostigmine
(anticholinesterase agents / acetylcholinesterase inhibitors). These are longer acting
versions of edrophonium.
Acute myasthenic crisis presents with severe, overwhelming disease with profound
weakness or respiratory involvement. It is treated with IVIG or
plasmapheresis. Reference: Master the boards



72. A young female experiencing symptoms of optic neuritis and other neuro symptoms (case of ms) what is the best diagnostic
test:
Answer: (mri)
Multiple sclerosis (ms) is diagnosed on the basis of clinical findings and supporting evidence from ancillary tests, such as magnetic
resonance imaging (mri) of the brain and spinal cord and cerebrospinal fluid examination.
Http://emedicine.medscape.com/article/1146199-workup

73. A question about huntington disease:


You can read more about huntington disease here:
Https://yhdp.vn/uptodate/contents/mobipreview.htm?29/8/29832
Https://yhdp.vn/uptodate/contents/mobipreview.htm?38/29/39385


74. Case of migraine headache.
Migraines: http://emedicine.medscape.com/article/1142556-overview

75. What is most risk factor for stroke?


Modifiable: • high blood pressure. This is the single most important risk factor for stroke because it’s the no. 1 cause of stroke. •
tobacco use. • diabetes. • high cholesterol. • physical inactivity and obesity. • carotid or other artery disease. • transient ischemic
attacks (tias). • atrial fibrillation (afib) or other heart disease. • certain blood disorders e.g.: a high red blood cell count, sickle cell
anemia. • excessive alcohol intake. • illegal drug use.
Non-modifiable: • increasing age. • gender. • heredity and race. • prior stroke.
Https://www.heart.org/idc/groups/stroke-public/@wcm/@hcm/documents/downloadable/ucm_309713.pdf
149
76. Gillian barre associated with ?
A. Cranial nerve involvement
Answer: a
Gbs http://emedicine.medscape.com/article/315632-overview

77. Neurofibromatosis genome:


The nf1 gene is cytogenetically located on the long (q) arm of chromosome 17, at band 11.2 (17q11.2). Medscape
http://emedicine.medscape.com/article/950151-overview

78. Pt had hx of AFib and prior stroke, how to prevent this patient from second stroke?
A. Warfarin

79. Epilepsy stimulated by >>glutamate

80. 65 years old male, presenting with peripheral neuropathy which progressed to weakness (sub-acute combined
degeneration), labs shows macrocytic anemia, diagnosis?
A. Vitamin b12 deficiency

Vitamin b 12 deficiency presentation beside the anemia symptoms (megaloblastic macrocytic anemia), they present with neurological
changes — classic picture of sub-acute combined degeneration of the dorsal and lateral spinal columns. This lesion, specific for
Vitamin b12 deficiency, is due to a defect in myelin formation. The neuropathy is symmetrical and affects the legs more than the
arms. It begins with paresthesia and ataxia associated with loss of vibration and position sense, and can progress to severe weakness,
spasticity, clonus, paraplegia, and even fecal and urinary incontinence. Other symptoms include: axonal degeneration of peripheral
nerves and cns symptoms including memory loss, irritability, and dementia.

81. Old lady with forgetfulness and numbness


A. B12 deficiency


82. Wernicke's area pathology leads to:
Damage caused to wernicke's area results in receptive, fluent aphasia. This means that the person with aphasia will be able to fluently
connect words, but the phrases will lack meaning. This is unlike non-fluent aphasia, in which the person will use meaningful words,
but in a non-fluent, telegraphic manner.

83. Blindness. Numbness that comes and goes (multiple sclerosis). Diagnostic test
A. Mri
Reference: http://emedicine.medscape.com/article/1146199-workup#c10

84. Case of a lady presented with inability to blink the eye, unsteady gait, something, she had brain tumor in the ct it involve
perventrcular, part of brain i don't remember and cord. Tumor type?
Answer: missing info


85. Brain tumors
http://www.disabled-world.com/health/cancer/brain/

86. Case of old patient with signs and symptoms of parkinson disease, where is the lesion?
Answer: basal ganglia, mainly substantia nigra

150
The dopaminergic tract is predominantly affected in parkinson disease, and histologically, it is characterised by nigrostriatal
dopaminergic degeneration leading to neuronal loss in the substantia nigra pars compacta, most conspicuous in the ventrolateral tier of
neurons. A number of other regions including parts of the basal ganglia, brainstem, autonomic nervous system and cerebral cortex.
Link: http://radiopaedia.org/articles/parkinson-disease-1

87. Child after hx of urti developed ascending muscle weakness?


A. Guillian-barre syndrome
Answer: a
Link: http://emedicine.medscape.com/article/315632-clinical


88. Brest feeding mother k/c of seziure on phenytoin ask about brest feeding:
Answer: regarding to nuropedia consultant : answer either to stop feeding which better or change to another antiepileptic drug not
excreted in milk both answer right

89. Which nerve is responsible for muscle that causes tongue movement?
Answer: hypoglossal


90. On the eye type of headache:
Answer: clusters headache

91. Antidote of opiate:


A. Naloxone
Answer: a

92. Multiple sclerosis present to er. What to give ?


Medical management goals that are sometimes achievable in the emergency department are to relieve symptoms and to ameliorate
risk factors associated with an acute exacerbation. In patients with fulminant ms or disseminating acute encephalitis, management
involves the following:
- stabilize acute life-threatening conditions
- initiate supportive care and seizure precautions
- monitor for increasing intracranial pressure
consider intravenous steroids, IV immunoglobulin (ivig), or emergent plasmapheresis. One study suggested that plasmapheresis
may be superior to IV steroids in patients with acute fulminant ms, the 2011 american academy of neurology (aan) plasmapheresis
guideline update states that plasmapheresis is possibly effective and may be considered in acute fulminant demyelinating cns
disease.
Identification and control of known precipitants of ms exacerbation include the following:
aggressively treat infections with antibiotics
in patients with a fever, normalize the body temperature with antipyretics, as even small increases in temperature can strongly
affect conduction through partially demyelinated fibers
provide urinary drainage and skin care, as appropriate.
Reference: medscape

93. Case of gbs. What you will find in lp?


Answer: high protein
Guillain-barre syndrome (diagnosis):
a. Csf analysis—elevated protein, but normal cell count
b. Electrodiagnostic studies—decreased motor nerve conduction velocity
Reference: step up to medicine

151
94. Cafe au lait spots, to diagnose neurofibromatosis:
A. Presence of axillary freckling
Answer: a
toronto:
Diagnosis of nf-1 requires 2 or more of:
• ≥6 café-au-lait spots (>5 mm if prepubertal, >1.5 cm if postpubertal)
• ≥2 neurofibromas of any type or one plexiform neurofibroma
• ≥2 lisch nodules (hamartomas of the iris)
• Optic glioma
• Freckling in the axillary or inguinal region
• A distinctive bony lesion (e.g. Sphenoid dysplasia, cortical thinning of long bones) ƒ
• First degree relative with confirmed nf-1

95. Epileptic patient with gingival bleeding and white gain which medication:
A. Phenytoin
Answer a
Http://reference.medscape.com/drug/dilantin-phenytek-phenytoin-343019#5
Http://emedicine.medscape.com/article/1076264-overview#showall

96. Patient taking carbamazepine developed generalized rash and peeling of epidermis:
Answer steven johnson syndrome
Http://emedicine.medscape.com/article/1197450-overview#a5



97. Best treatment of trigeminal neuralgia?
Toronto:

98. 70 Year-old female brought to your clinic by her daughter . The daughter said her mother's memory deteriorated in the last 2
years . She can dress her self but with difficulty , she can cook for herself but sometimes leave the oven on ,,,,,,,,,what's the
management ?
A. Refer her to geriatric clinic.

99. Old patient with alzheimer's dementia, became agitated and have hallucinations and delusions. What is the appropriate drug
in his case ?
A. Haloperido

100.Ascending paralysis with areflexia post urti


A. Gullen barre syndrome (gbs)
Answer:
• Inflammatory demyelinating polyneuropathy, that primarily affects motor nerves tract.
• Usually proceeded by a viral or mycoplasma infection or upper respirator or gi tract. Common infection may include
campylobacter jejune, CMV , hepatitis, and hiv.
rd
• Reference: step up to medicine 3 edition.pg 223


101.Alzheimer disease or lowes bodies ct brain changes ?

152
The initial criteria for ct scan diagnosis of alzheimer disease includes diffuse cerebral atrophy with enlargement of the cortical sulci
and increased size of the ventricles. A multitude of studies indicated that cerebral atrophy is significantly eater in patients with
alzheimer disease than in patients who are aging without alzheimer disease.
Ref: http://emedicine.medscape.com/article/336281-overview#a2

102.Definition of status epilepticus?


Status epilepticus (se) is a common, life-threatening neurologic disorder that is essentially an acute, prolonged epileptic crisis. In
patients with known epilepsy, the most common cause is a change in medication. Most seizures terminate spontaneously.
Http://emedicine.medscape.com/article/1164462-overview

103.Clinical dementia ?
Dementia is a pattern of mental decline caused by different diseases or conditions. Most commonly, dementia occurs when brain nerve
cells (neurons) die, and connections between neurons are interrupted. These disruptions have a variety of causes and usually cannot be
reversed. Alzheimer's disease causes over 60% of all dementias. Vascular disease, such as stroke, is the second most common cause.
In rare cases, dementia is caused by a treatable condition, and it may be partially or entirely reversed if the condition is diagnosed and
treated early.
Http://www.drugs.com/health-guide/dementia.html

104.Brain ventricles anatomy


The neural canal dilates within the prosencephalon, leading to the formation of the lateral ventricles and third ventricle. The cavity of
the mesencephalon forms the cerebral aqueduct. The dilation of the neural canal within the rhombencephalon forms the fourth
ventricle.
The lateral ventricles communicate with the third ventricle through interventricular foramens, and the third ventricle communicates
with the fourth ventricle through the cerebral aqueduct.
Http://emedicine.medscape.com/article/1923254-overview#a2


105.What is a type of headache that a patient who have stress at school had:
A. Tension
Answer:a
Tension headache never happens during sleep, gradual over 24 h. The site is posterior/occipital.
Increases with stressors. Treated by modifying stressor(s), local measures, NSAIDs, tricyclic antidepressants.
Reference: toronto notes


106.Most common brain cancer?
A. Astrocytoma
Explanation: astrocytoma type of glioma which is the most common brain tumors
Reference: http://www.neurosurgery.pitt.edu/centers-excellence/neurosurgical-oncology/brain-and-brain-tumors/types-brain-
tumors

107.Sign in duchene muscular dystrophy?


A. Growers sign

108.Case of nuroparlysis and numbness in all extermity .

109.Duchenne muscular dystrophy:


Answer: proximal muscle wasting

153
110.Patient came with severe pain involving the forehead to nose (trigeminal distribution) what is the diagnosis?
A. Trigeminal neuralgia

Answer: A
• Trigeminal neuralgia is an idiopathic disorder of the fifth cranial nerve resulting in severe, overwhelming pain in the face.
Attacks of pain can be precipitated by chewing, touching the face, or pronouncing certain words in which the tongue strikes
the back of the front teeth. Patients describe the pain as feeling as if a knife is being stuck into the face.
• There is no specific diagnostic test.
• Treat with oxcarbazepine or carbamazepine. Baclofen and lamotrigine have also been effective. If medications do not
control the pain, gamma knife surgery or surgical decompression can be curative.
Reference: Master the Boards

111.B6 & B12 deficiency


Answer: ??
B6 deficiency: Overt deficiencies of vitamin B6 are probably rare. Marginal deficiencies may be more common, manifested as
nonspecific stomatitis, glossitis, cheilosis, irritability, confusion, and depression.
The classic clinical picture of cobalamin deficiency (Vitamin B12), mentally sluggish, shiny tongue (atrophic glossitis), and a
shuffling broad-based gait. Hematologic changes (e.g., macrocytic anemia with oval macrocytes and increased neutrophil
lobulation) and neurologic abnormalities classic picture of subacute combined degeneration of the dorsal (posterior) and lateral
spinal columns, neuropathy which is symmetrical and affects the legs more than the arms. It begins with paresthesias and ataxia
associated with loss of vibration and position sense, and can progress to severe weakness, spasticity, clonus, paraplegia, and
even fecal and urinary incontinence.
Reference: Uptodate

112.A patient with facial nerve involvement. Presents with loss of taste sensation of the anterior 2/3 of tongue. There is loss of
function of stapedius as well. At what point is the injury?
*options includes different points of facial nerve course
Answer: ?
Facial canal between geniculate ganglion and nerve to stapedius muscle.
Reference: http://www.ncbi.nlm.nih.gov/books/NBK385/

113.Postherpetic neuralgia treatment


A. Antiviral
Answer: TCA (Amitriptyline or Nortiptyline), Antiviral aims to shorten the clinical course, prevent complications, prevent the
development of latency and/or subsequent recurrences, decrease transmission, and eliminate established latency. (Medscape)

154
Reference: Master the board

114.What the investigation should be done before lumbar puncture?


A. Platelets
Answer: A
We recommend NOT performing an LP in patients with coagulation defects who are actively bleeding, have severe thrombocytopenia
(eg, platelet counts <50,000 to 80,000/µl), or an INR >1.4, without correcting the underlying abnormalities.
Reference: uptodate

115.In Guillain-Barré syndrome, which cell is affected?


Answer: Schwann cells
Guillain-Barré syndrome; Rapidly progressive limb weakness that ascends following GI/ upper respiratory infection.
Reference: http://www.uptodate.com/contents/pathogenesis-of-guillain-barre-syndrome

116.Patient is seizure sence 35 min, he take diazepam i.v but nor effective , what will you do ? Frn
Answer:
Phenyton
In the initial therapy phase, a benzodiazepine (specifically IM midazolam, IV lorazepam, or IV diazepam) is recommended as
initial therapy.
In the second phase, options include IV fosphenytoin, valproic acid, or levetiracetam. If none of these is available, IV
phenobarbital is a reasonable alternative.
In the third phase, if a patient experiences 40+ minutes of seizure activity, treatment considerations should include repeating
second-line therapy or anesthetic doses of thiopental, midazolam, pentobarbital, or propofol
Http://emedicine.medscape.com/article/1164462-treatment
Http://www.ncbi.nlm.nih.gov/pmc/articles/PMC2824929/


117.CLUSTER HEADACHE TREATED BY:
A- Oxygen
Answer: A
Pharmacologic management of cluster headache (CH) may be classified into 2 general approaches as follows:
Abortive/symptomatic (eg, oxygen, triptans, ergot alkaloids, and anesthetics)
Preventive/prophylactic (eg, calcium channel blockers, mood stabilizers, and anticonvulsants)
Reference: http://emedicine.medscape.com/article/1142459-treatment

118.Patient is seizure sence 35 mint, he take diazepam I.V but not effective , what will you do ? “ Phenytoin is not in the choices ”
Answer:

155
Seizures occurring continuously for at least 30 minutes, or two or more seizures occurring without full recovery of consciousness
between attacks.
First-line therapy = benzodizepines—diazepam, lorazepam, or midazolam.
- Lorazepam has a relatively longer duration of seizure suppression.
Second-line therapy = phenytoin and/or phenobarbital
-Phenytoin: Rapid administration may cause hypotension and cardiac dysrhythmias this may be avoided with fosphenytoin .
- Phenobarbital: Anticipate sedation, respiratory depression and hypotension.
Drug-induced coma (pentobarbital, midazolam, propofol) or general anesthesia, if resistant to above
Reference : first aid of emergency


119.A pateint with difficulty in swallowing she has frontal baldness and cataract (other symptoms that i can't remember) her
mother has the same condition
Myotonic muscular dystrophy
Answer: a
((first aid for the usmle step 1 ck 2014,p89 ))


120.3rd Cranial palsy ?
Answer: Out and down

121.Calculate gcs?

156

Infectious disease



















157
1. Ksa have implemented strong regulations regarding worker health cares, which of the following diseases if the worker had,
he can't work? Repeated in family medicine
A. HBV
B. Hcv
C. Hiv
D. All of the above

Answer: d

2. After appendectomy a patient got abdominal infection by enterococcus faecium. He is allergic to penicillin what are you
going to give him:
A. Ceftriaxone
B. Vancomycin
C. Cefotaxim
D. Tmp-smx or metronidazole (not sure which one of them was mentioned)
Answer: b
Fortunately, there are a number of antibiotics with broader spectrum and high efficacy that can be used as alternatives to penicillin.
The chances of cross reactivity between penicillins and antibiotics of other groups are rare. Therefore, other antibiotics can be used
safely in patients with history of penicillin allergy. However, the choice of alternative antibiotic depends upon the kind of infection
that needs to be treated.
Http://medlicker.com/880-allergy-to-penicillin-alternative-antibiotics

Vancomycin should be used in patients with a penicillin allergy or infections with strains that have high-level penicillin resistance due
to altered PBPs.
Reference; emedicnie

3. Which vaccine is contraindicated in HIV patient
A. Opv
B. Varicella
C. Mmr
D. HBV
Answer: a ** the live, attenuated oral polio vaccine (opv) is not recommended for persons with hiv. Reference :
http://hivinsite.ucsf.edu/insite?Page=kb-03-01-08

4. A patient has symptoms of infectious mononucleosis. Monospote test was positive. After 8 days, he suddenly complained of
acute abdominal pain, decrease Blood pressure . What will you do?
A. Antibiotic
B. Fluid resuscitation, urgent ct abdomen
C. Urgent gasto.
D. Barium enema
Answer: b

5. Patient has uti organism grows on both antiseptic & detergent?


A. E.coli
B. Proteus
C. Pseudomonas
D. Staph areus or strep
Answer: c, pseudomonas is associated with nosocomial infections.
Http://www.soeagra.com/abr/december%202011/9.pdf

6. Methicillin-resistance, sensitive to chloramphenicol and aminoglycosides, what not to give?


A. Gentamycin
B. Azithromycin
C. Chloramphenicol

158
D. Vancomycin
Answer: b
mrsa is most commonly resistant to:
beta-lactams (penicillins and cephalosporins), fluoroquinolones (e.g., levofloxacin), macrolides (e.g., erythromycin, azithromycin)
Mrsa can usually be treated with “last-resort” antibiotics: clindamycin, vancomycin, linezolid and daptomycin
http://www.pewtrusts.org/en/research-and-analysis/issue-briefs/2012/04/03/mrsa-a-deadly-pathogen-with-fewer-and-fewer-treatment-
options

7. Immune deficient patient what vaccine could be given:


A. Meseals
B. Rubella
C. Pneumococcal
D. Varicella
Answer: c
https://www.cdc.gov/vaccines/pubs/pinkbook/downloads/appendices/a/immuno-table.pdf

8. (long scenario) report {single strand RNA)?


A. Hepatitis a
B. Hepatitis b
C. Hepatitis c
D. Hepatitis d.
Answer: scenario is missing information to decide between option a and option c
The structure of hepatitis viruses are as follow:
Hepatitis a virus (non-enveloped single-stranded RNA)
Hepatitis b virus (enveloped double-stranded dna)
Hepatitis c virus (enveloped single-stranded RNA)
Hepatitis d virus (envelop from HBV, single-stranded RNA)
Hepatitis e virus (non-enveloped single-stranded RNA)
Link: http://www.ncbi.nlm.nih.gov/books/nbk7864/

9. A patient presented with (meningitis symptoms) and history of swimming in the river, what is the organism?
A. S. Pneumoniae
B. H. Influenzae
C. N. Meningiditis
D. Naegleria fowleri
Answer is d
Primary amebic meningoencephalitis (pam) is a very rare form of parasitic meningitis that causes a brain infection that is usually fatal.
Pam is caused by the microscopic ameba (a single-celled living organism). Naegleria fowleri when water containing the ameba enters
the body through the nose.
Source: http://www.cdc.gov/meningitis/amebic.html
It is commonly found in warm fresh water (e.g. Lakes, rivers, and hot springs) and soil. Naegleria fowleri usually infects people when
contaminated water enters the body through the nose.
Http://www.cdc.gov/parasites/naegleria/
• Naegleria fowleri (parasite) lives in fresh water such as lakes, ponds and poorly maintained swimming pools, and can
cause meningitis
• S. Pneumonia: is the most common cause of meningitis for all patients beyond neonatal period
• H. Influenza: in the past it was the common cause of meningitis in children. After haemophilus type b vaccine this
organism is markedly reduced
• N. Meningitis: spread by respiratory droplets, and it is the most common cause in adolescent
Kaplan usmle step2

10. Enteric fever resistanant to chloramphenicol, what's next ?


A. Double chloramphenicol
B. Ciprofloxacin alone

159
C. Add ciprofloxacin
D. Im ceftriaxone
Answer: d?
In general, preferred antibiotics include ceftriaxone 1 g im or IV q 12 h for 14 days and various fluoroquinolones (eg, ciprofloxacin 500
mg po bid for 10 to 14 days, levofloxacin 500 mg po or IV once/day for 14 days, moxifloxacin 400 mg po or IV once/day for 14 days).
Chloramphenicol 500 mg po or IV q 6 h is still widely used, but resistance is increasing.

11. A family who ate at a restaurants come to the er with diarrhea (…sing of food poising). After 24 hours all of them improved.
Their cultures showed gram positive spore with oxidative +ve what the organism??
A. Salmonella typhi
B. Staphylococcus aureus
C. Shigella
D. Bacillus
Answer: d
Spore-forming pathogenic bacteria in ready-to-eat food. Bacillus cereus and clostridium perfringens are examples of pathogenic
bacteria that can exist in both spore and vegetative forms. The thick-walled structure of the spore is resistant to heat and it can help
the pathogenic bacteria survive the heat of cooking.
Http://www.cfs.gov.hk/english/multimedia/multimedia_pub/multimedia_pub_fsf_25_02.html

12. Patient with ventilator associated pneumonia. Culture showed lactose non-fermenting, gram negative motile bacilli
producing greenish colony and oxidase positive. What is the organism?
A. Coli
B. Pseudomonas
C. Klebsiella
D. Proteus
Answer: b

13. Giardia diagnostic test is?


A. 3 stool parasite
B. 3 stool cultures
C. Concentration test
D. Stool microassay.
Answer: d
Http://www.cdc.gov/parasites/giardia/diagnosis.html

stool microscopy
This remains the most definitive investigation.
Inexpensive, but requires microscopic expertise and specialised laboratories.
Three specimens from different days should be examined.
Some authorities have issued specific guidelines for collection of stool samples.
BMJ

14. Patient diagnosed with rheumatic heart disease and mitral regurge. What is the treatment of choice?
A. Oral penicillin and aspirin (i choose this one)
B. Im penicillin once every month
C. Corticosteroids
D. IV penicillin
Answer: a
Oral (po) penicillin v remains the drug of choice for treatment of gabhs pharyngitis. Aspirin in anti-inflammatory doses effectively
reduces all manifestations of the disease except chorea, and the response is typically dramatic.

15. What is the chemoprophylaxis for vibrio cholera in pandemic area?


A. Penicillin

160
B. Cephalexin
C. Tetracycline
D. Erythromycin
Answer: c. Tetracycline. Cdc http://www.cdc.gov/cholera/treatment/antibiotic-treatment.html

Doxycycline is recommended as first-line treatment for adults, while azithromycin is recommended as first-line
treatment for children and pregnant women.

16. What is the prevention method for food poisoning?


A. Prolong heating and re cooking
B. Antibiotic
C. Prolong mastication
D. No peeled fruit in choices
Answer: a

17. Patient with end stage liver disease, on central venous line and septic. His blood culture showed budding yeast. What anti-
fungal is appropriate at this stage?
A. Cuspofungin
B. Flucytosine,
C. Fluconazole
D. Itraconazole
Answer a
Cuspofungin is used for systemic candiasis that could be caused by indwelling devices. Flucoenazol is used for cryptococcal meningitis,
disseminated histoplasmosis and cocccidiodomycoses. Itraconazole is used for dermatophytes mucocautenous candiasis and seborrheic
dermatitis. Flucytosine is a weak agent that’s rarely used alone.

18. What is the definitive test for diagnosing tb?


A. Ppd
B. Sputum culture
C. Blood culture
D. Serum specific antigen or breathing test urease
Answer: b
Sputum studies (sputum acid-fast testing): definitive diagnosis is made by sputum culture—growth of m. Tuberculosis, must obtain
three morning sputum specimens—culture takes 4 to 8 weeks.
Cxr: classic findings are upper lobe infiltrates with cavitation. Other possible findings: pleural effusion, ghon complex and ranke
complex: evidence of healed primary tb.
-tuberculin skin test (ppd test) is a screening test to detect those who may have been exposed to tb. It is not for diagnosis of active tb.

Chest x-r ay. Classic findings are upper lobe infiltrates with cavitations.
Sputum studies (sputum acid-fast testing). Definitive diagnosis is made by sputum culture—growth of m. Tuberculosis. obtain three
morning sputum specimens—culture takes 4 to 8 weeks.
PCR .
Finding of afb on microscopic examination, but this is not definitive.
Tuberculin skin test (ppd test). A screening test, not for diagnosis. positive if induration ³15 mm in patients with no risk factors.
health care workers, nursing home residents, close contact of someone with tb, alcoholics, diabetics, 10 mm of induration is
considered positive. hiv, steroid users, organ transplant recipients, close contacts of those with active tb, or those with radiographic
evidence of primary tb, induration of 5 mm is positive.
Quantiferon-tb gold test.
Reference: step-up of medicine

19. Female with uti most common cause?


A. Napkin use
B. Cab for contraception
C. Back to front cleaning

161
D. Using of pad
Answer: c

20. HIV patient ... (symptoms of intestinal obstruction) did intestinal resection. They found tumor white in color nearly encircling
the wall. What is the tumor?
A. Hodgkin’s
B. Non hodgkin’s
C. Adenocarcinoma
D. Plasmacytoma
Answer: c
All blood cancers except adenocarcinoma

21. Pneumococcal vaccine conjugate 13?


A. Active vaccine
B. Live attunuated vacine
C. Conjugated (conjugated polysaccharide)
D. Inactive vaccine
Answer: c
Pneumococcal conjugate vaccine (pcv, initially marketed as a 7-valent vaccine, pcv7 [prevnar or prevnar 7], now replaced by pcv13
[prevnar 13]) consists of capsular polysaccharides from the 13 most common types that cause disease, covalently linked to a nontoxic
protein that is nearly identical to diphtheria toxin. This covalent linking to a protein renders the polysaccharide antigenic in infants and
toddlers.

22. Shigela spices treatment:


A. Metrondozole
B. Azthromycin
C. Amoxicillin
D. Ceftrixone
Answer: d
we suggest treatment of shigellosis in adults with a fluoroquinolone. we suggest treatment of shigellosis with a third-generation
cephalosporin. azithromycin and trimethoprim-sulfamethoxazole.

23. What transmitted infection will require relieve from work in health care:
A. HBV
B. HBV
C. HAV
D. HIV
Not sure about the answer
In general, three conditions are necessary for health-care personnel to pose a risk for bloodborne virus transmission to patients. First,
the health-care provider must be sufficiently viremic (i.e., have infectious virus circulating in the bloodstream). Second, the health-
care provider must have an injury (e.g., a puncture wound) or a condition (e.g., nonintact skin) that allows exposure to his/her blood or
other infectious body fluids. Third, the provider's blood or infectious body fluid must come in direct contact with a patient's wound,
traumatized tissue, mucous membranes, or similar portal of entry during an exposure-prone procedure. The vast majority of HIV and
HBV-infected health-care personnel pose no risk for patients because they do not perform activities in which both the second and third
conditions are met.

All healthcare workers have to go through standard healthcare clearance when applying for a medical post or training. This involves
being tested for tb, being offered a hepatitis b vaccination, and being offered tests for HIV and hepatitis c. The HIV and hepatitis c
tests are not compulsory. A positive HIV diagnosis, or declining an HIV test should not affect the employment or training of
healthcare workers who will not perform epps (exposure prone procedures). Healthcare workers who are applying for a post or
training which does involve epps have to go through additional healthcare clearance. This means testing negative for hiv, hepatitis b
and hepatitis c.

24. Patient presented with oropharengeal maculopapular rash plus rash in palm and soles?

162
A. CMV
B. EBV
C. Vaccine virus
D. Coxsackievirus virus
Answer: d
This seems to be a case of hand-foot-mouth disease, in which the patient has eruptions starting on the back of the throat and may spread
to the hands and soles. It usually affects children but it can also affect adults.
Reference: http://emedicine.medscape.com/article/218402-clinical

25. Most cases of cerebral edema were found to be associated with which of the following organisms?
A. Plasmodium malaria
B. P. Ovale
C. P. Vivax
D. P. Falciparum
Answer: d
Most observations of the pathophysiology of disease come from postmortem observations of plasmodium falciparum (pf) infections,
which are thought to account for the vast majority of cm cases, and show a common feature of vascular sequestration of infected
erythrocytes (ie) in the brain.

26. HIV attacks?


A. B cells
B. T cytotoxic cells
C. Macrophages
D. T helper cells
Answer: d
HIV produces cellular immune deficiency characterized by the depletion of helper t lymphocytes
(cd4+ cells). Http://emedicine.medscape.com/article/211316-overview#a3

27. Patient came from africa develops symptoms "i don't remember all of them but i’m sure he had arthralgia" the vitals normal
"no fever" what is the diagnosis:
A. Yellow fever
B. Ebola
C. Lassa fever
D. Chikungunya fever
Answer: d
All of the options come with high fever but chikungunya is characterized by an abrupt onset of fever frequently accompanied by
joint pain. The joint pain is often very debilitating, but usually lasts for a few days or may be prolonged to weeks. Most patients
recover fully, but in some cases joint pain may persist for several months, or even years. The disease occurs in africa, asia and the
indian subcontinent. Http://www.who.int/mediacentre/factsheets/fs327/en/


28. Gram negative bacteria oxidase, non lactose fermenter which of the following the best is antimicrobial ?
1) Ceftriaxone
2) Cefipime
3) Ciprofloxacin
4) Smz –tmp (bactrem)

Answer: c
Fluoroquinolones are the only class of antibiotics which has an oral formulation that is reliably active against P. aeruginosa.

29. Patient came from africa with fever , myalgia and arthralgia ?
A. Ebola fever
B. Lassa fever
C. Yellow fever
D. Malaria

163
Answer: d

30. Fever, headache, with macule, papule, pustules & vesicles over the face, scalp & the trunk ; what is the causative organism ?
A. Herpes type 6 (roseola)
B. CMV
C. Herpes zoster
D. EBV
Answer: c?? Age in infancy and any missing symptoms mainly an episode of seizure might change the diagnosis to a
Http://emedicine.medscape.com/article/1132465-overview

31. What is the mechanism of prophylactic antiviral that given against flu?
A. DNA polymerase 2.
B. RNA1
C. RNA2 ??
D. RNA 3
Answer:

32. HIV patient come with diffuse pustule in skin an mouth, management is :
A. Topical steroid
B. Oral ab
C. Topical ab
D. Chemo & radiotherapy
Answer: a

33. Patient is presented with hand cellulitis and red streaks in the hand and tender axillary
lymphadenopathy. This condition is more likely to be associated with:
A. Malignancy
B. Pyoderma
C. Neuropathy
D. Lymphangitis
Answer : d
Lymphangitis is defined as an inflammation of the lymphatic channels that occurs as a result of infection at a site distal to the channel
O/e
Erythematous and irregular linear streaks extend from the primary infection site toward draining regional nodes. These streaks may be
tender and warm.
Reference: http://emedicine.medscape.com/article/966003-clinical#showall

34. Patient with ventilator associated pneumonia. Culture showed lactose non-fermenting, gram negative motile bacilli
producing greenish colony + Oxidase positive. What is the organism?
A. Haemophilus influenza
B. Streptococcus pneumoniae
C. Klebsiella or other gram negative bacteria
D. Pseudomonas aeruginosa
Answer: D

164
Reference: http://www.columbia.edu/itc/hs/medical/pathophys/id/2008/utignr.pdf

35. What is the optimal duration of antibiotic treatment in strep throat?


A. 3 days
B. 5 days
C. 7 days
D. 10 days

Answer: D
Penicillin is the treatment of choice for strep throat. It is usually given in pill or liquid form, two to four times per day for 10 days.
Reference: uptodate.

36. Young adult came complaining of painless penile ulcer, what is the appropriate investigation to do?
A. Blood culture.
B. Excisional biopsy.
C. Swab culture and urinalysis.
D. Darkfield microscope.
E. CBC & ESR.

Answer: D
Primary syphilis usually begins with a single, painless, well-demarcated ulcer (chancre) with a clean base and indurated border.
Treponema pallidum is identified on darkfield microscopy or direct fluorescent antibody testing of a chancre or lymph node aspirate.
Reference: http://www.aafp.org/afp/2012/0201/p254.html

37. A patient was diagnosed with enteric fever. What is the presentation that he will have?
A. Confusion (or other CNS problems)
B. Maculopapular rash
C. Nausea, vomiting and loose stools
D. Abdominal pain, headache, fever( newly added)

Answer: D
Signs and symptoms of Salmonella typhi (aka Enteric Fever, Typhoid) include "Rose spot" rash (on anterior thorax, upper abdomen),
sustained fever 39° to 40° C (103° to 104° F), abdominal pain precedes diarrhea/ constipation, headache, loss of appetite and cough.
Toronto Notes. If not treated the patient may become delirious. Reference: Mayoclinic

38. A patient with signs of TB. What vaccination you would give to his family?
A. MMR
B. Dtap
C. BCG
D. Polio
165
Answer: C
Reference: kumar and clark’s

39. Enteric fever best diagnosed in the first week of presentation by?
A. Blood culture
B. Stool culture
C. Multiple something?
D. Bone marrow>>not sure

Answer: A
Blood, intestinal secretions (vomitus or duodenal aspirate), and stool culture results are positive for S typhi in approximately 85%-90%
of patients with typhoid fever who present within the first week of onset.
Http://emedicine.medscape.com/article/231135-workup

40. Fever, headache, with macules, papules, pustules & vesicles over the face, scalp & the trunk ; what is the causative organism
?
A. Herpes type 6
B. CMV
C. Herpes zoster
D. EBV

41. What is the cause to use every year influenza vaccine ?


A. Resistance of antimicrobial
B. New antigen
C. Different type of transmission
D. Drift .. So could be new antigen .

The rational of giving the vaccine yearly is related to the high mutagenicity of the influenza viruses. Influenza virus is remarkable for
its high rate of mutation, compromising the ability of the immune system to protect against new variants. As a consequence, new
vaccines are produced each year to match circulating viruses. Currently, vaccine production takes, on average, six months from the
selection of seed strains to the final vaccine product. The decision of which influenza antigens to include in the vaccines is made in
advance of the influenza season and is based upon global surveillance of influenza viruses circulating at the end of the prior influenza
season. As a result, rarely there are mismatches between the vaccine strain and the circulating strain that result in reduced efficacy of
the vaccine.

42. Dm, hypothyroid, irregular menses female, present with recurrent itching & white adherent oral plaque , +ve mantux test ,
she was exposed to tb 4 years ago, immunoglobulin, wbc, RBCs all are normal ; dx ;
A. Chronic granulomatous disease
B. Chronic candidiasis
C. Digeorge syndrome
D. Hyperglobulinemia ( or hypo i don't remember )
Answer: risk factors for candida albicans are: dm,broad sprectrum antibiotic use, pregnancy, conrticosteroids, hiv, ocp use, increase
frequency of intercourse.reference: step2ck 8th edition pg 333

43. Old pt came to er 4 w of fever cough night sweat... (clear t.b symtome) immeditly do !??
A. Put pt in negtive pressure
B. Give anti t.b drug
C. Sputum culture
D. Chest x ray

44. Healthy pt with no symptoms x ray is normal, has negative hx of tuberculin test, now has positive test ..?
A. Reassure
166
B. Give rifampicin and iso
C. Give iso for 6 months

Answer: most likely c more specific
6-month or 9-month isoniazid daily,
3-month rifapentine plus isoniazid weekly,
3 or 4-month isoniazid plus rifampicin daily,
3 or 4-month rifampicin alone daily.
References: http://www.cdc.gov/tb/topic/treatment/ltbi.htm
Http://www.who.int/tb/challenges/ltbi/en/

45. Which part of bone is firstly affected in hematogenous osteomyelitis?


A. Epiphysis
B. Metaphysis
C. Diaphysis

Answer:
In children: metaphysis
Reference: http://www.uptodate.com/contents/hematogenous-osteomyelitis-in-children-epidemiology-pathogenesis-and-
microbiology?Source=search_result&search=hematogenous+osteomyelitis&selectedtitle=1%7e30
In adults: diaphysis
Reference: http://www.uptodate.com/contents/hematogenous-osteomyelitis-in-
adults?Source=search_result&search=hematogenous+osteomyelitis&selectedtitle=3%7e30

46. HIV patient, what test confirm it?


A. PCR
B. Western blot
C. Elisa

Answer: b
The best initial test for HIV is the elisa test. This is confirmed with western blot testing. Infected infants are diagnosed with PCR or
viral culture.
Reference: master the boards usmle step 2


47. HIV presents commonly with?
A. Opportunistic infection
B. Chronic diarrhea
C. Generalized lymphadenopathy

Answer: a
The patient may present with signs and symptoms of any of the stages of HIV infection. Acute seroconversion manifests as a flulike
illness, consisting of fever, malaise, and a generalized rash. The asymptomatic phase is generally benign. Generalized
lymphadenopathy is common and may be a presenting symptom.
Reference: http://emedicine.medscape.com/article/211316-clinical
Early symptomatic HIV infection includes persistent generalized lymphadenopathy, often the earliest symptom of primary HIV
infection. The most common findings include fever and chills, lymphadenopathy, pharyngitis, anemic pallor, and rash. (flu-like
symptoms)
Reference: http://reference.medscape.com/article/211873-overview#a4

48. Parasite in soil contamination:


A. Tenia saginata
B. Ascaris
C. Bancrofti
Answer: b

167
Soil-transmitted helminths refer to the intestinal worms infecting humans that are transmitted through contaminated soil ("helminth"
means parasitic worm): ascaris lumbricoides (ascaris), whipworm (trichuris trichiura), and hookworm (anclostoma
duodenale andnecator americanus).
Reference: cdc

49. Ppd was +ve , to prevent false +ve , what to do?!


A. Repeat it.
B. Do x-ray.
C. Do mantoux test.

Answer: b
Resource: previously written!

• Interferon gamma release assay (igra) after six weeks and repeat the mantoux test to increase the sensitivity (to reduce
false negative results).
• Since the quantiferon®-tb gold (qft-g) blood-based test does not cross-react with bcg, this test is particularly useful for
testing individuals with a history of bcg vaccination • patients who have a positive tst reaction should undergo clinical
evaluation, including a chest x-ray (cxr) to rule out tb. If the ini- tial cxr is normal, repeated ones are not indicated unless
the individual develops signs or symp- toms of tb. Tst-positive individuals should be started on treatment for ltbi according
to the guidelines in
• Qft-g can yield cost savings in terms of medical staff time—both by elimination of a second patient visit for test
interpretation and by the elimination of common false-positive results, which typically involve both unnecessary follow-up
testing and treatment for ltbi.

50. What case infection by food with soil contaminated?


A. Ascaris
B. Tinae
C. Schistosoma
Answer: a

51. Patient presented with unilateral eye swelling with purple skin discoloration, what is the organism?
A. Staph.aures
B. B hemolytic strep
C. Haemophilus influenzae
Answer: b

52. Patient has been treated four times by sulfonamide antibiotics and there were lesions in the glans penis and scrotum each
time he was using that drugs ,what is the best explanation of this lesion ?
A. Vesicle
B. Patchy red nodule
C. Redness with ulcers & blister
Answer: c
Read about balanitis
Http://www.hopkinsmedicine.org/healthlibrary/conditions/dermatology/drug_rashes_85,p00280/
Http://emedicine.medscape.com/article/777026-clinical#b5

53. Transmitted of parasite by ingestion of undercooked meat ?


A. Schistosoma
B. Tinea saginata
C. Tenia solium
Answer: b
Tenia solium is more associated with pork

168
Schistosoma infection occurs when skin comes in contact with contaminated freshwater in which certain types of snails that carry the
parasite are living.

54. Fever, malaise, maculopapular rash over the body & behind the ear; causative organism
A. Rubella
B. Measles
C. Mumps
Answer:
Measles: distribution: starts at hairline and spreads downwards with sparing of palms and soles
Rubella: distribution: starts on face and spreads to neck and trunk
Roseola: distribution: starts at the neck and trunk and spreads to the face and extremities
Http://www.healthline.com/health/measles

55. Enteric fever presentation;


A. Fever, headache, abdominal pain
B. Nausea vomiting abdominal pain
C. Something with diarrhea
Answer: tyohiod or enteric fever is an infection salmonella typhi bacterium. Incubation period of 7-14, present with fever, abdominal
pain and red spot on the abdomen ( pathognomic) diarrhea is not a common symptom.
Http://emedicine.medscape.com/article/231135-clinical

56. Patient c/o fever ,productive cough , x-ray show rt lung opacification and oblitration of rt costophrenic angle what u will find
on examination?
A. Crepitations on both lungs
B. Absent of vesicular breathing sounds of rt side
C. Presence of bronchial breathing

Answer: c
Explanation: pneumonia may present with evidence of consolidation (dullness to percussion, bronchial breath sounds, crackles)
Reference: toronto notes 2015, page id8, infectious diseases

57. Ppd was +ve , to prevent false +ve , what to do?!


A. Repeat it
B. Do x-ray
C. Do mantoux test

169
58. A nurse with pneumonia they gave different type of penicillin.
A. Cloxacillin
B. Amoxicillin
C. Pepracillin i
Answer: c

Treatment of hospital-acquired pneumonia is tailored toward gram-negative rods (any of the following three are appropriate):
• Cephalosporins with pseudomonal coverage: ceftazidime or cefepime
• Carbapenems: imipenem
• Piperacillin/tazobactam
reference: step up to medicine


59. Diabetic with painful back swelling has multiple discharges:
A. Cellulitis
B. Lymphoid
C. Something lymphadenitis carbuncle
Answer: c

Reference: http://www.merckmanuals.com/professional/dermatologic-disorders/bacterial-skin-infections/furuncles-and-carbuncles
Http://patient.info/doctor/boils-and-carbuncles

60. What is the most common organism in whooping cough ?


A. Bartonella
B. Pertussis
C. H.infeluanza
Answer: b

• Pertussis, a respiratory illness commonly known as whooping cough, is a very contagious disease caused by a type of
bacteria called bordetella pertussis

Reference: http://emedicine.medscape.com/article/967268-overview
Http://www.cdc.gov/pertussis/about/causes-transmission.html

170
61. Picture of skin with chickenpox, with history of child with malaise and fatigue followed by single macule then spread all over
the body including the face, what is the treatment:
A. Antibiotics
B. Antiseptic
C. Acyclovir
Answer: c
Antiviral medications are recommended for people with chickenpox who are more likely to develop serious disease including
* otherwise healthy people older than 12 years of age
* people with chronic skin or lung disease
* people receiving steroid therapy
* some groups of pregnant women
acyclovir, an antiviral medication, is licensed for treatment of chickenpox



62. A patient with a central line developed an infection. What is the most common cause of infections?
A. Skin opening
B. Drug administration
C. Contamination from staff hand
Answer is a
The most common source of intravascular catheter related infections is colonization of the intra-cutaneous and intravascular portions of
the catheter by microorganisms from the patient's skin. A number of studies have found a strong correlation between heavy skin
colonization and both catheter colonization and subsequent catheter-related infection. Microorganisms gain access to the catheter wound
and migrate along the catheter-subcutaneous tract into the fibrin sheath that surrounds intravascular catheters.
Source: https://yhdp.vn/uptodate/contents/utd.htm?36/37/37464?Source=see_link

63. Cervical infection can enter to superior-mediastinum through:


A. Retropharyngeal space
B. Para-pharyngeal space
C. Carotid sheath
Answer: a
The “danger space” is a potential space that lies posterior to the retropharyngeal space, it is bound by the alar fascia anteriorly and the
prevertebral fascia posteriorly. It extends from the base of the skull and descends freely through the entire superior and posterior
mediastinum to the level of the diaphragm (t1 to t2) where the two fascial layers fuse. Thus, the danger space provides the most important
anatomic route for contiguous spread between the neck and the chest e.g., descending necrotizing mediastinitis Source:
https://yhdp.vn/uptodate/contents/mobipreview.htm?35/45/36569
Http://emedicine.medscape.com/article/784277-overview#a5

171



64. Known case of HIV , have several problems on iris ,including the word"necrotizing"
Wt the cause?
A. Hiv
B. Cytomegalovirus
C. Toxoplasma
Answer: most likely b

Explanation: epidemiologic data regarding iris are variable and depend largely on the incidence and types of infections that patients
have at the time of initiation of arv therapy. In the united states, retrospective studies have reported iris in 63% of hiv-infected patients
who had inactive CMV retinitis at the time of initiation of arv therapy3 and 30% to 34% of those with inactive cryptococcus.4,5 similar
rates have been found retrospectively in 30% and 31% of hiv-infected patients with mycobacterium tuberculosisand m. Avium
complex (mac), respectively.5 however, retrospective studies may overestimate the incidence of iris. A prospective actg study, actg
a5164, reported a rate of 7.6%6; however, this rate may have been low because most of the reported opportunistic infections (ois) were
pcp. Steroid treatment for pcp may have mitigated iris-related symptoms and reduced the number of iris diagnoses in the study.
Link: http://www.hivguidelines.org/clinical-guidelines/adults/immune-reconstitution-inflammatory-syndrome-iris-in-hiv-infected-
patients/
Http://www.aidsinfonet.org/fact_sheets/view/483

65. Patient have a chronic liver disease and you found a fungal infection which drug you will use :
A. Floconazole
B. Itraconazole
C. Amphotricin b

66. An asymptomatic patient had a positive tb skin test. What will you do?
A. Reassurance
B. Isoniazid for 6 month
C. Rifampin and isoniazid 6 month
Answer: b
172
Patients with a clinically significant result on tuberculin skin testing or a positive interferon-gamma release assay (igra) result should
receive a course of therapy for latent tb, once active infection and disease are ruled out. Recommended regimens for latent tb
published by the us centers for disease control and prevention (cdc) are as follows:
• Isoniazid 300 mg - daily for 9 months
• Isoniazid 900 mg - twice weekly for 9 months (administered as dot)
• Isoniazid 300 mg - daily for 6 months (should not be used in patients with fibrotic lesions on chest radiography, patients with HIV
infection, or children)
• Isoniazid 900 mg - twice weekly for 6 months (administered as dot; should not be used in patients with fibrotic lesions on chest
radiography, patients with HIV infection, or children)
• Rifampin 600 mg - daily for 4 months
• Rifapentine 900 mg plus isoniazid 900 mg - once-weekly for 12 weeks (administered as dot)
• No longer recommended - rifampin plus pyrazinamide daily for 2 months (increased liver toxicity)
Http://emedicine.medscape.com/article/230802-treatment#d13

67. A 12 year-old’s x-ray showed bilateral lower lung infiltration. What is the treatment?
A. Ciprofloxacin
B. Azithromycin
C. Pincillen
Answer: b repeated without options

68. A patient is taking amoxicillin for his pneumonia and is found not sufficient. What will you add?
A. Vancomycin
B. Erythromycin
C. Azithromycin
Answer is: a or c (not sure)
Vancomycin is used to treat cap in patients < 60 years with the organisms (s. Pnuemoniae, mycoplasma, chlamydia, legionella).
Macrolides (azithromycin, clarithromycin) will cover all these organisms and are first-line treatment.
Source: step-up to medicine “page 364”

69. A patient complained of neck pain and occipital headache. No history of trauma. There is a limitation in neck movement on
examination, weakness in upper shoulder? What is the diagnosis?
A. Cervical spondylosis
B. Cannot remember other choices
C. Meningitis
Answer: c

70. A 25 year-old teacher is complaining of abdominal pain and fatigue. On exam, there was icting , palpable liver 1 cm. Also, 2
student complained of the same symptoms. What is the most likely diagnosis?
A. Hav
B. HBV
C. HBV.
Answer: a
HAV hepatitis has prodrome of mild flu-like symptoms (anorexia, nausea, fatigue, malaise and joint pain) preceding the jaundice. This
can progress to the icteric phase with dark urine (appears first). Pale stools (not always). Jaundice occurring in 70-85% of adults with
acute HAV infection. Abdominal pain occurring in 40% of patients. Itch or pruritus (usually with jaundice but can occur without).
Arthralgias and skin rash. These occur less often (lower limbs and with a vasculitic appearance).spread is normally by the faecal-oral
route although there are occasional outbreaks through food sources.
Http://patient.info/doctor/hepatitis-a-pro

71. Rota confirmatory diagnosis


A. Serum antibody
B. Stool antigen
C. Stool leukocyte

173
Answer: b http://www.cdc.gov/rotavirus/clinical.html
Direct immune-based assays of stool and polymerase chain reaction (PCR ) techniques have been employed most frequently to make
the diagnosis of rotavirus. Assays generally detect the rotavirus group antigen present on vp6.

72. Indonesian lady, suspected tb. Best test?


A. Sputum culture
B. PCR
C. Cxr
Answer: c
Chest x-ray is the first line diagnostic modality for tb. It is almost always abnormal in immunocompetent individuals. Typically
presents as fibronodular opacities in upper lobes with or without cavitation. Http://bestpractice.bmj.com/best-
practice/monograph/165/diagnosis/tests.html


73. Young female came with jaundice disoriented, tremor. Husband has HBV
Lab -ve. LFTslightly elevated what lab to order next ?
A. Serum copper
B. Serum level of ceruplasmen
C. HBV core

Answer: b
Jaundice, tremors, and elevated LFTs are indicative of wilson’s disease. Wilson’s disease can present with hepatic or neurologic
symptoms or both. The next step in diagnosis should be to obtain urine copper or serum ceruplasmin which will be low. The gold
standard for diagnosis is liver biopsy. Http://bestpractice.bmj.com/best-practice/monograph/427/diagnosis/step-by-step.html

74. Giardiasis treatment


A. Metrindazole
B. Parpomycine
C. Ciprofloxacin
Answer: a

75. Patient in college diagnosed with meningitis proved and she receiving treatment
How to prevent the spread to the roommate?
A. Give them pencilin and other antibiotics
B. Isolate for 4 weeks
C. Do nasal swab and now the affected
Answer: a (no rafmpicin or ciprofloxacin or ceftrixon in the options)

76. Meningitis case fever, headache, nuchal rigidity and rash (pic) what is the most complication?
A. Deafness
B. Seizure
C. Focal neurological defect.

Answer: a. Deafness. Some of the most common complications associated with meningitis are: hearing loss, which may be partial or
total.recurrent seizures (epilepsy), problems with memory and concentration, learning difficulties and behavioural problems, vision
loss, which may be partial or total, loss of limbs, kidney problems.
Nhs http://www.nhs.uk/conditions/meningitis/pages/complications.aspx

77. Patient on anti tb medication complaining of numbness and paresthesia. What is the treatment?
A. Pyridoxine
B. Iron
C. Thiamine
Answer: a

174
Isoniazid: patients at risk of peripheral neuropathy, should additionally receive pyridoxine (b6), 10 mg daily. Where the standard of
health in the community is low, pyridoxine should be offered routinely. For established peripheral neuropathy, pyridoxine should be
given at a larger dose of 50–75 mg daily. Pyridoxine supplementation is recommended for all pregnant (or breastfeeding) women
taking isoniazid.

78. Which of the following is associated with animal bites?


A. Pasturella multicodi
B. Pseudomonas
C. Eikenella
Answer: a
Pasturella multicodi exists as a commensal in the upper respiratory tracts of many livestock, poultry, and domestic pet species, especially
cats and dogs. In fact, p multocida infection in humans is often associated with an animal bite, scratch, or lick. Eikenalla is part of the
hacek organisms which are gram-negative coccobacilli that exist asca oropharyngeal commensals associated with infective endocarditis.
Http://emedicine.medscape.com/article/224920-overview
Http://emedicine.medscape.com/article/218158-overview

79. Hsv- 2 most appropriate treatments?


A. Acyclovir
B. Lamivudine
C. Ribavirin
Answer: a
Reference: http://emedicine.medscape.com/article/218580-medication#2

80. Patient has been bitten by a cat, what is the organism that has been transmitted?
A. Staphylococcus aureus
B. Pasteurella multocida
C. Streptoccus pneumonia
Answer: b
Cat bite-associated infections, are polymicrobial, with a mix of aerobes and anaerobes. Common aerobic pathogens in cat bites include
streptococcus species (including streptococcus pyogenes), staphylococcus species, especially s. Aureus and moraxella. Pasteurella
multocida, a small (0.2–2.0 µm) facultatively anaerobic, gram-negative, nonmotile, non-spore-forming, pleomorphic coccobacillus is
the most common organism isolated in cat bites. Reference: http://www.medscape.com/viewarticle/739023_5

81. Typical history of pneumonia, x-ray was done, lower lobe consolidation was found. Culture shows gram-positive cocci
arranged in clusters. Catalase and coagulase were positive. What is the most appropriate ab for this infection?
A. Oxacillin
B. Penicillin g
C. Amoxicillin
Answer: a

82. IV drug abuser has HIV +ve. Presented with oral thrush and symptoms of pneumonia. X-ray and bronchoalveolar lavage was
done. After staining with silver stain, pneumocystis jiroveci was found. What is the most likely predictor of her HIV infection?
A. Pneumocystis jiroveci infection
B. Iv drug use
C. Candida
Answer: a
Pneumocystis jiroveci pneumonia (pjp), formerly known as pneumocystis cariniipneumonia (pcp), is the most common opportunistic
infection in persons with HIV infection.

175
83. Patient with secondary syphilis was treated with penicillin. 2 hours following the first dose he developed fever myalgia and
malaise. What is your management?
A. Epinephrine
B. Antihistamine
C. Symptomatic management with paracetamol
Answer: c


84. Child with diarrhea. What you recommend the mother to do?
A. Oral hydration
B. Iv hydration
C. Antidiarrheal medication
Answer: a

nd
85. History of arthritis on (cephalexin). Culture is gram+ cocci that’s resistant to (cefzil), a 2 generation cephalosporin. What will
you do?
A. Prescribe vancomycin
B. Stop the drug
C. Continue same drug
Answer: a
Explanation: the patient most likely has mrsa infection. Septic arthritis due to mrsa should be treated with vancomycin.
Reference http://cursoenarm.net/uptodate/contents/mobipreview.htm?32/63/33776#h18

86. Pt on cloxacillin for staph micro reported it is resistant to one of the cephalosporins what to do:
A. Continue cloxacillin
B. Start vancomycin
C. Stop antibiotics
Answer: b
Read about viral hemorrhagic disease http://emedicine.medscape.com/article/830594-overview

87. Child came back with his family from africa developed fever, n&v, abnormal reflexes, stiff legs, unable to raise legs and neck
when he is supine cns analysis nl what is the most likely cause?
A. Corona virus
B. Poliovirus
C. CMV
Answer: b

88. Which of the following transmitted through un cooked meat ?


A. Entameba coli
B. Entamebea.h
C. Teania

Answer: c
(important foodborne parasites: trichnella (undercooked pork), tinea saginata (undercooked beef), tinea solium (undercooked pork),
toxoplasma gondii (undercooked meat).)
Refernce: cdc

89. Male what is type of dysuria after urination bladder still full and palpable
Maculopapular rash on face and some inside buccal white?
A. Mumps
B. Herpes zoster
C. Measles
Answer: herpes simplex virus

176
Clinical features in men: herpetic vesicles appear in the glans penis, the prepuce, the shaft of the penis, and sometimes on the scrotum,
thighs, and buttocks. In dry areas, the lesions progress to pustules and then encrust. Herpetic urethritis occurs in 30%-40% of affected
men and is characterized by severe dysuria and mucoid discharge. The perianal area and rectum may be involved in persons who
engage in anal intercourse, resulting in herpetic proctitis.
Reference: medscape

90. Varicella vaccine in adult.


A. Dose once
B. Doses 4 weeks
C. 2 doses 6 weeks
Answer: b (2 doses at least 4 weeks apart)

91. Aids patient 34 w pregnant her cd count dropped to 200 what will u do:
A. Book for cs
B. Cs when spontaneous labor
C. Vaginal delivery
Answer: i think a
Http://www.uptodate.com/contents/hiv-and-pregnancy-beyond-the-basics
Https://aidsinfo.nih.gov/contentfiles/hivandpregnancy_fs_en.pdf

92. A married woman was diagnosed with uti. Urine culture revealed staphylococcus saprophyticus. What you should ask this
patient about?
A. Use of condom and spermicides.
B. Alcohol consumption.
C. Fecal incontinence.

Answer: a
honeymoon cystitis. Sexual activity increases the risk of s. Saprophyticus uti because bacteria are displaced from the normal flora of
the vagina and perineum into the urethra. Most cases occur within 24 hours of intercourse.

93. Patient is allergic to sulfa drugs and penicillin and shellfish. She has uti what antibiotic you will give?
A. Nitrofurantoin
B. Trimethoprim sulfamethoxazole
C. Amoxicillin

Answer: a

94. When does acute episode of rheumatic fever occur?


A. In case of pharyngitis
B. When bacteria invade joints
C. When bacteria coat the myocardium
Answer : a
[1, 2, 3, 4]
Rheumatic fever follows pharyngeal infection with rheumatogenic group a streptococci. the risk of developing rheumatic
[1]
fever after an episode of streptococcal pharyngitis has been estimated at 0.3-3%.
Http://emedicine.medscape.com/article/236582-overview#a5


95. Dog bite, what is the most common organism ?
A. Rabies
B. Yersinia pestis ( rodent - plaque )
C. Toxoplasma gondii ( cat )

Answer: a

177
Pasteurella ( 50% )
Strept 46% , staph 46%
Neiserria 32%
Corynebasterium 12%
Bite wound infections are usually polymicrobial, with a mix of animal oral flora, recipient skin flora
And environmental organisms. The most common pathogens in dog bites arepasteurella spp. (both
Pasteurella multocida and pasteurella canis), staphylococcus and streptococcusspp., and the
Fastidious gram-negative rod capnocytophaga canimorsus. The dog is the most common transmitter
Of rabies to humans worldwide, with greater than 95% of reported cases being due to these animals.
Reference:
Http://www.medscape.com/viewarticle/739023_4

96. HIV patient, presented with SOB and productive cough.


Lung biopsy showed soap bubble like intra-alveolar lesions with exudates, small cyst, stained by silver stain.
A. Pneumocystis jiroveci
B. Aspergillus fumigatus
C. Cryptococcus neoformans

Answer: A
Pneumocystis jiroveci (previously named P. Carinii), is the most common opportunistic infection in patients with AIDS, and it is an
increasing cause of disease in other immunosuppressed persons. It cannot be cultured and most clinical laboratories rely on
microscopic examination of stained material from the respiratory tract. The most widely used stain techniques are those that stain the
cyst wall, such as Gomori methenamine silver (GMS), toluidine blue, and certain fluorescent brighteners (FB).
Reference: American Journal of Clinical Pathology (AJCP).
One of the special stains is silver stain (fungi. Pneumocystis carinii) for HIV/ immunocompromised patients.
Reference: Step Up to Medicine.

Cryptococcus - Diagnosis - M/E SOAP BUBBLE like organism (silver stain, mucicarmine and PAS), India Ink negatie staining,
LATEX AGGLUTINATION test for antibodies against capsular polysaccharides; CD4 < 100

97. Patient developed nausea and vomiting then developed cranial nerve palsies then bilateral symmetrical progressive LL
paralysis. What is the most likely diagnosis?
A. Tetanus
B. Botulism
C. Lead poisoning

Answer: B
Botulism is an acute neurologic disorder that causes potentially life-threatening neuroparalysis due to a neurotoxin produced by
Clostridium botulinum. The 3 main clinical presentations: Infant botulism, Foodborne botulism and Wound botulism.
Signs and Symptoms:
Occurs 6-48 h after ingestion
Difficulty with convergence, ptosis, paralysis of extraocular muscles
● Dilated, poorly reactive pupils
● Other autonomic dysfunction: jaw weakness, dysarthria, dysphagia.
Spreads to trunk and limbs
a. Abdominal cramps with nausea and vomiting
Symmetric weakness with paralysis and absent/decreased deep tendon reflexes
Anticholinergic symptoms: dry mouth, constipation, urinary retention
● Rarely respiratory distress, potentially advancing to respiratory failure
Reference: Toronto Notes and Medscape.

98. Child – parents Have TB - You read a PPD result after 48 hours. It showed 10 mm induration. What does that indicate?
A. Negative

178
B. Weakly positive
C. Strongly positive

Answer: C
The person's medical risk factors determine the size of induration the result is positive (5 mm, 10 mm, or 15 mm).
● Five mm or more is positive in:
● HIV-positive person
● Recent contacts of active tuberculosis cases
● Persons with nodular or fibrotic changes on Chest X-ray consistent with old healed TB
Organ transplant recipients and other immunosuppressed patients who are on cytotoxic immune-suppressive agents such as
cyclophosphamide or methotrexate.
● Patients on long term systemic corticosteroid therapy (> than six weeks) and those on a dose of prednisone ≥ 15 mg/day or
equivalent.
● End stage renal disease
● Ten mm or more is positive in:
● Recent arrivals (less than five years) from high-prevalence countries
● Injectable drug users
● Residents and employees of high-risk congregate settings (e.g., prisons, nursing homes, hospitals, etc.)
● Mycobacteriology lab personnel
Persons with clinical conditions that place them at high risk (e.g., diabetes, prolonged corticosteroid therapy, leukemia, end-
stage renal disease, chronic malabsorption syndromes, low body weight, etc.)
● Children less than four years of age, or children and adolescents exposed to adults in high-risk categories
Infants, children, and adolescents exposed to adults in high-risk categories
Fifteen mm or more is positive in:
Persons with no known risk factors for TB. (Reactions larger than 15 mm are unlikely to be due to previous BCG vaccination or
exposure to environmental mycobacteria).
Reference: Pubmed

99. Patient with oral ulcers. Culture showed herpesvirus.


A. HSV2
B. VSV
C. HSV1

Answer: C

100.Old patient with poor control of DM1, complaining of SOB and hemoptysis. X-ray shows lung consolidation. Culture show non
septa fungal hypha. What is the diagnosis?
A. Aspiragillus
B. Candida
C. Zymgomycetes

Answer: c
Opportunistic infection especially associated with diabetes; other predisposing factors are neutropenia, corticosteroid therapy, iron
overload and mucocutaneous trauma.
Large, non-septa hyphae with 90 degree angle branching and non-parallel walls, angioinvasive causing tissue necrosis and hemorrhage

Reference: http://www.pathologyoutlines.com/topic/lungnontumormucor.html

101.20yrs old gentleman came with blocked nose and urt symptoms, followed by swelling eyes and redness what's the possible
answer :

179
A. Meningiococcal conjunctivitis
B. Chlamedia conjunctivitis
C. Parainfluenza virus

102.Man in acetaminophen drink 2 glass of win every week LFT(high) and bilirubin is high dx:
A. Alcoholic hepatitis ??
B. Drug induced hepatitis
C. Wipple

103.Patient have history of meningitis befour 4 week , come againe to hospital , what is most increase of his lap
A. Protien ?? "not sure"
B. Lekocytosis
C. Glocose

104.Patient has blood transfusion from kenya and had anal infection. What´s the most likely diagnosis?
A. HBV
B. Syphilis
C. Answer: ???

105.Action of cytotoxic?
A. Il6
B. Il10
C. Tnf gamma


106.Long case about adult come to er complaing of diarrhea , have weekness in body and fatige , suddenly he fall down ,
hypertension when he come was 120 / 80 now it is 90 / 60 , what is the cause ?
A. Extracelular voluim loss
B. Intracellular fluid loss !?
C. Intracellular glocse loss


107.Std case with culutre gram -ve diplocci , wt is organism ?
A. N . Gonnorhra
B. Chalmyida
C. E.coli


108.Enteric fever resistance to chloramphenicol, what should you do next ?
A. Double chloramphenicol
B. Clindamycin alone
C. Add clindamycin
Answer: ??
Explanation: drugs of choice for the treatment of typhoid fever in adults include : a fluoroquinolone such as ciprofloxacin or
ofloxacin (400 mg twice daily)/ the fluoroquinolones should not be used as a first-line treatment for typhoid fever in patients from
south asia or other regions with high rates of fluoroquinolone resistance unless antibiotic susceptibility data demonstrate
fluoroquinolone or nalidixic acid sensitivity or beta-lactam such as ceftriaxone or cefixime. Reference:
http://cursoenarm.net/uptodate/contents/mobipreview.htm?37/1/37904

109.4w back pt had mycoplasma pneumonia what will be very high in lp?
A. Protein
B. Wbc
180
C. Glucose
Mycoplasma pneumonia increase light chain protienIgMthats why those patient

110.Old lady with sharp chest pain and fever diagnosed with pericarditis what will you do to dx the case , most accurate test is :
(cardio)
A. Acid fact stain
B. Pericardial biopsy(my answer)
C. Pleural aspiration

111.A man came from Africa with some symptoms. Vital signs were provided. What is the diagnosis?
A. Yellow fever
B. Ebola
C. Lassa fever

Answer: ?

Reference: http://lectures.shanyar.com/3rd_Stage/Medicine/Dr._Muhammad_Shaikhani/5._Viral_Hemorrhagic_Fevers.pdf


112.Rhinorrhea, cough and conjunctivitis etiology?
A. Rhinovirus
B. Adenovirus

Answer: b
The most common cause of rhinorrhea and sinusitis is rhinovirus., but since there is also conjunctivitis, then adenovirus is more
appropriate.
Reference: http://emedicine.medscape.com/article/302460-clinical



113.Increase of which of the following prevalence cause reactivation of tb in developed countries? Repeated in family medicine
A. DM
B. Hiv

Answer: b
Who & uptodate: according to data from the united states national tb surveillance system for 1993 to 2005, 7.7 percent of tb
patients were HIV infected; greater than 80 percent of tb patients in the united states received HIV testing. HIV infection rates in tb
patients were highest among injection drug users, homeless persons, correctional facility inmates, and alcoholics (35, 22, 16, and 15
percent, respectively)
The coinfection with m tuberculosis and HIV has profound epidemiologic implications worldwide. From one perspective, tb has
reemerged in some countries, such as the united states, in association with multiple factors, of which HIV infection is one of the

most relevant.

181
References:
Http://hivinsite.ucsf.edu/insite?Page=kb-05-01-06
Http://www.ncbi.nlm.nih.gov/pmc/articles/pmc3276831/
Http://www.medscape.com/viewarticle/443137_2


114.Woman with recurrent uti, what is the cause?
A. Because she cleans herself of the from anus to vulva
B. Using of cervical cap for contraception

Answer: b
Note: cervical cap usage require spermicidal cream with it, & since spermicide is a risk for recurrent uti according to the uptodate

115. Pt in icu on ventilator, develop yeast infection what is rx?


A. Fluconazole
B. Itraconazole

Answer: fluconazole
(uptodate: “the most common antifungal agents used currently for the treatment of candidemia are fluconazole and the
echinocandins (caspofungin, micafungin, anidulafungin). Formulations of amphotericin b are given less often due to the risk of
toxicity. Both the echinocandins and the azoles are better tolerated than amphotericin b formulations.”
& oxford journals: “overall, fluconazole-susceptible candida albicans remains the most common species causing candidaemia in icu
patients.”)


116.Long scenario, bloody diarrhea and RBCs in urine after 7 days hx of food posing, rx?
A. Steriod
B. Antibiotic

Answer: conservative
This is e coli: hus


117. Which marker indicates chronic hepatitis b?
A. Igm
B. Hep b s antibody

Answer:IgManti-hbc
Hepatitis b surface antigen (hbs ag) indicates that a person is infected with the hep b virus. It is detectable as early as 2 weeks and is
seen in both acute and chronic infection. It persists in chronic infection regardless of the presence of symptoms.
Hepatitis b surface antigen antibody (anti-hbs) indicates immunity against hep b due to either vaccination on recovery from previous
infection. Not present in acute or chronic infection.
Hepatitis b core antigen antibody (anti-hbc) indicates that a person may have been exposed to the hep b virus but it does not indicate
immunity or protection. It is present in both acute and chronic infection.
So how is chronic infection diagnosed? The diagnosis of chronic HBV infection is based on persistence of hbs ag for more than six
months +IgGanti-hbc is positive, whileIgManti-hbc is negative.
References: step up to medicine + this cool cdc pdf: https://www.cdc.gov/hepatitis/HBV/pdfs/serologicchartv8.pdf

118.Treatment of trigeminal neuralgia?


A. Prednisiolone
B. Naloxone
Answer :carbamazepine and oxcarbazepine are considered first-line therapy in trigeminal neuralgia (tn)
http://emedicine.medscape.com/article/1145144-treatment#d9


182
119.67 years patient complain of progressive weakness for 1 year. Things are falling from his hand. On examination there is wide
gate, +ve hoffman & babinski sign?
A. Cervical canal stenosis
B. Spinal cord tumor?
Answer: b

120.Patient present with itchy foot, on examination there is linear lesions with red dots at the end of the lines, diagnosis;
A. Lintego ?
B. Scabies
Answer: b
Burrows are a pathognomonic sign and represent the intraepidermal tunnel created by the moving female mite. They appear as
serpiginous, grayish, threadlike elevations in the superficial epidermis.
Http://emedicine.medscape.com/article/1109204-clinical

121.Case of herpes type 1, what to give?


A. Oral antiviral
B. Topical steroids
Answer: a
Reference: overall, medical treatment of herpes simplex virus (hsv) infection is centered around specific antiviral treatment.
(http://emedicine.medscape.com/article/218580-treatment)



122.Which of the following organisms is seen in patients with chronic granulomatous disease?
A. Cl. Difficle
B. Staph aurues
Answer: b

123.There is tb outbreak in a region, however you tested one patient and you found him to be negative for tb, what are you going
to give him:
A. Bacillus calmette-guérin vaccine
B. Rifampin chemoprophylaxis
Answer:b

124.Mouth ulcer :
A. Hsv 1
B. Hsv2
Answer:a
Most often, hsv-1 causes gingivostomatitis, herpes labialis, and herpes keratitis. Hsv-2 usually causes genital lesions.
Http://www.merckmanuals.com/professional/infectious-diseases/herpesviruses/herpes-simplex-virus-hsv-infections



125.Ie most common organism:
A. S. Aureus
B. S.viridins

Reference: medscape table 1 http://emedicine.medscape.com/article/216650-overview#a4

183
126.Vesicles on the eye and forehead?
A. Herpes zoster ophthalmicus.
B. Herpesvirus ophthalmic.
Answer: a

127.Patient with infection. Culture and sensitivity showed methicillin sensitive organisms, which antibiotic you will select :
A. Piperacillin
B. Oxacillin
Answer: b
Serious staphylococcal infections require treatment with parenteral penicillinase-resistant penicillin (e.g. Nafcillin, oxacillin)

128.Gram positive organisms in cluster? What will be positive?


A. Coagulase
B. Oxidase
Answer: a

129.A breast abscess showed gram positive organisms. What will u do?
A. Coagulase
B. Oxidase
Answer: a
S. Aures is the most common organism causing breast abscess, and coagulase test is used to differentiate coagulase +ve staphylococcus
aureus from coagulase -ve staphylococcus.

130.61yo male patient with bilateral lung base infiltrate (x-ray chest) cough, diarrhea, temp. 38.7c (long scenario),what is the
microorganism?
A. Legionella pneumonia
B. Mycoplasma pneumonia
Answer: could be a or b.
Both are atypical community-acquired pneumonia, but x-ray direct more toward a.
Legionella pneumonia usually cause a patchy, localized infiltrate in the lower lobes.
Http://emedicine.medscape.com/article/234240-overview#a6
http://emedicine.medscape.com/article/363083-overview#a2

131.Patient with positive ppd, never was +ve before, no x-ray findings, what is next step?
A. Isoniazide and rifampin 6 months
B. Rifampin 3 months
Answer: b, close to one of cdc regimens
it's latent tb. Http://www.cdc.gov/tb/topic/basics/tbinfectiondisease.htm
(cdc) recommend regimens:

184
Isoniazid 300 mg – daily for 9 months
isoniazid 900 mg – twice weekly for 9 months
isoniazid 300 mg – daily for 6 months (should not be used in patients with fibrotic lesions on chest radiography, patients with HIV
infection, or children)
isoniazid 900 mg – twice weekly for 6 months (should not be used in patients with fibrotic lesions on chest radiography, patients with
HIV infection, or children)
rifampin 600 mg – daily for 4 months
rifapentine 900 mg plus isoniazid 900 mg – once-weekly for 12 weeks

132.Why patient will be in a risk of neisseria infection defect in which of the following:
A. Classical complement pathway
B. Final lytic complement pathway
Http://emedicine.medscape.com/article/135478-clinical

133.A patient showed antibodies to toxoplasma (IgM) .. How to confirm diagnosis ?


A. Double of igm
B. DetectIgGfor toxoplasma
Answer: ?

134.Trypanosoma cruzi act by mutation on what gene?


A. Myocin
B. Tryptomyocin t
Answer: not sure

135.A patient came from a trip suffered an infection for which he took amoxicillin. After that he developed rash and
lymphadenopathy. What investigation will you ask for this patient?
A. EBV
B. Brucellosis
Answer: a, EBV infectious mononeucleosis.
EBV is transmitted via intimate contact with body secretions, primarily oropharyngeal secretions. EBV infects the b cells in the
oropharyngeal epithelium. The organism may also be shed from the uterine cervix, implicating the role of genital transmission in some
cases. On rare occasion, EBV is spread via blood transfusion.
Http://emedicine.medscape.com/article/222040-overview

136.Csf lp shows +IgGand irregular band under the gel?


A. Ms
B. Spinal tumor
Answer: a
igg is usually increased and correlate with disease severity. OligoclonalIgGbands can usually be detected by electrophoresis of csf.
Http://www.merckmanuals.com/professional/neurologic-disorders/demyelinating-disorders/multiple-sclerosis-ms#v1045134

137.Patient diagnosed as mg came to er with weakness & severe fatigability she is on pryditostigmine what initial step you do?
A. Add other drug
B. Plasmophresis
Answer: b, plasmapheresis has been found to be an effective short-term treatment of acute exacerbations of myasthenia gravis.
Http://emedicine.medscape.com/article/793136-overview#a10



138.Painless genital ulcer + lymph nodes enlargement
A. Syphilis
B. Secondary syphilis

185
Answer: a

139.Recurrent mouth and genital ulcers with arthralgia:


A. Systemic herpes infection
B. Syphilis
Answer: b
Rheumatic syndromes, including arthralgia, inflammatory arthritis, and neuropathic arthritis, may occur during any stage of congenital
or acquired syphilis. Also, the patient acquired syphilis through oral sex, a syphilitic chancre may be noted in the oral cavity on the
buccal mucosa, tongue, or lips. These syphilitic lesions can be misdiagnosed initially as herpetic or aphthous ulcers.
Http://www.ncbi.nlm.nih.gov/pubmed/2246956
Http://cursoenarm.net/uptodate/contents/mobipreview.htm?12/44/12993#h12

140.What is the best treatment for chlamydia ?


A. Doxycyclin
B. Azithromycin
Answer: ???
Treatment of genitourinary chlamydial infection is clearly indicated when the infection is diagnosed or suspected. Chlamydiae are
susceptible to antibiotics that interfere with DNA and protein synthesis, including tetracyclines, macrolides, and quinolones.[47] cdc
recommends azithromycin and doxycycline as first-line drugs for the treatment of chlamydial infection.
Http://emedicine.medscape.com/article/214823-treatment#d8

141.What is the treatment of acute active rheumatic fever?


A. Monthly im pencilin
B. Amoxicillin q 6 hours
Answer: b

142.Patient with history of infective endocarditis planned for dental procedure:


A. Amoxicillin 1 hour before procedure.
B. Amoxicillin after procedure
Answer: a
Amoxicillin single dose 30-60 min prior, clindamycin if penicillin allergy

143.Patient with fever, night sweat, weight loss and unilateral supraclavicular ln which was firm and 3cm:
A. Ct
B. Biopsy
Answer: a
144.Diabetic patients presents with dark color nasal discharge. What is the most likely diagnosis?
A. Cryptococcosis
B. Zygomycosis
186

Answer: b
Mucormycosis is a group of mould infections caused by fungi in the class previously known as zygomycetes, now re-named
glomeromycetes.

145.How to treat water from entamoeba histolytica?


A. Boiling
B. Chloride
Answer: a

146.Organism that grow in antiseptic area?


A. Pseudomonas
B. Proteus
Answer: a
Gram-negative bacteria that show a high level of resistance to many antiseptics and disinfectants include p. Aeruginosa, burkholderia
cepacia, proteus spp., and providencia stuartii. The outer membrane of p. Aeruginosa is responsible for its high resistance.

147.Fever with spot in molar tooth?


A. Measles
B. Rubella
Answer: a
Koplik spots (also koplik's sign) are a prodromic viral enanthem of measles manifesting two to three days before the measles rash
itself. They are characterized as clustered, white lesions on the buccal mucosa (opposite the upper 1st & 2nd molars) and are
pathognomonic for measles. Reference: http://emedicine.medscape.com/article/966220-clinical

148.Trichomonas vaginalis usually presents with.


A. Lack of irritation
B. Greenish foul smelling discharge
Answer: b
The diagnosis is made by observation of the following features (table i):
A foul-smelling frothy discharge (present in 35% of cases
Vaginal ph >4.5 (70% of cases)
Punctate cervical microhemorrhages (25%)
Motile trichomonads on wet mount (50%-75%)
Link: http://www.medscape.com/viewarticle/719240_4

149.Mother with HBV ag gave birth what to give for child


A. Give vaccin and igg.
B. GiveIgGonly
Answer: a

187
Link: http://www.aafp.org/afp/2010/1115/p1225.html

150.19-year-old boy after unprotected sex he developed generalized rash involving hand and feet
A. Chlamydia
B. Syphilis
Answer: b
Link: http://www.acog.org/patients/faqs/gonorrhea-chlamydia-and-syphilis


151.Married male has history of unprotected sex, days later he had purulent discharge with gram negative intracellular
diplococci?
A. Non gonococcal urethritis
B. Gonococcal urethritis
Answer: b


152.Pt with ulcer on penis?
A. Chrocoid
B. Syphilis
See: infectious agents affecting the male genitalia http://emedicine.medscape.com/article/455021-overview#a5

153.What is the organism gram -ve bacilli, non-lactose fermenter, oxidase +ve
A. A-pseudomonas
B. B-proteus
Answer: a

154.Staph saprifitucus vaginal infection, what’s a risk factor for it:


A-septicides in condoms
B-douching habbits

Not sure about this but staphylococcus saprophyticus is normal flora in the vagina therefore it doesn’t usually cause vaginitis, however
it may commonly cause cystitis. S. Saprophyticus causes 10–20% of urinary tract infections (utis). In females 17–27 years old, it is the
second-most common cause of community-acquired utis, after escherichia coli.[5] sexual activity increases the risk of s. Saprophyticus
utis because bacteria are displaced from the normal flora of the vagina and perineum into the urethra.[3] most cases occur within 24
hours of sex,[3] earning this infection the nickname "honeymoon cystitis".

155.Child with itchy scalp and scales, other classmates affected, dx:
A. A-tinea capitus

188
B. B-scabies
Answer: a
Clinical manifestations — tinea capitis may present with a variety of clinical features. There are five major clinical variants of tinea
capitis as well as an asymptomatic carrier state. The causative organism and the host immune response to infection influence the
clinical presentation.
Clinical variants — the most common clinical manifestations of tinea capitis are the development of scaly patches with alopecia and
patches of alopecia with visible black dots. Diffuse scalp scaling with subtle hair loss, kerion, and favus are additional clinical
variants. Pruritus is a common associated symptom in the various presentations of tinea capitis.

156.Pt with +ve ppd for the first time and -ve cxr. No signs or symptoms of tb, wt to do:
1. A-reassure
2. B-inz for 6 mns
Answer: b

Adults
Isoniazid* Standard regimen:
300 mg po daily for nine months
Alternate regimens:
300 mg po daily for six months
900 mg po twice weekly for nine months
900 mg po twice weekly for six months
Isoniazid and rifapentine◊ Isoniazid (orally once weekly for 12 doses, given by direct observation)
15 mg/kg, rounded up to the nearest 50 or 100 mg; 900 mg maximum
Rifapentine (orally once weekly for three months, given by direct observation)
Rifampin 600 mg po daily for four months
Isoniazid and rifampin§ Isoniazid 300 mg po daily for three months
Rifampin 600 mg po daily for three months

157.Patient in icu with central and peripheral line developed fungal infection what to give:
A. Fluconazole
B. Caspofungin
Answer: b
158.Mycobacterium tuberculosis test :
A. Ifn-γ
B. Ifn- γ slow release assay (igra)
Answer: b
Explanation: igra is a blood test equal in significance to ppd to exclude ppd exposure.
Reference master the board step 2 ck

159.How to diagnose giardia lambila?


A. Three consecutive stool analysis
B. Three separate stool analysis
Answer: b

160.35 Year-old female, used ampicillin 1week ago for uti, now presented with hx of fever, hypotension and tachycardia:
A. Pseudomemparenous colitis
B. Toxic mega colon
Answer: i think it's sign of septic shock (intern’s answer)

161.Fever and cough then facial nerve then loss of reflexes?


A. Tetanus

189
B. Botulism

Answer: b
If guillain-barre syndrome was one of the choices it would be the answer.
Signs and symptoms of botulism:
• Occur 6-48 h after ingestion of clostridium botulinum from soil
• Difficulty with convergence, ptosis, paralysis of extraocular muscles.
• Dilated, poorly reactive pupils
• Jaw weakness, dysarthria, dysphagia.
• Spreads to trunk and limbs
• Abdominal cramps with n/v.
• Symmetric weakness with paralysis and absent/decreased deep tendon reflexes
• Anticholinergic symptoms: dry mouth, constipation, urinary retention
• Rarely respiratory distress.
Pattern of paresis often starts with gi symptoms (constipation, early satiety), then paresis of extraocular muscles, then dysphagia, then
limbs/respiratory involvement; all associated with dry mouth
Reference: toronto note.

162.Patient with fever, pre-auricular swelling, (description of the swelling was provided), what is the diagnosis?
A. Mumps
B. Pre-auricular lymphadenopathy

Answer: b
Mumps parotitis is usually bilateral but may be unilateral. Pain while chewing or swallowing, especially while swallowing acidic
liquids such as vinegar or citrus juice. Its swelling beyond the parotid in front of and below the ear. The skin over the glands may
become tense and shiny.
Reference: merck manual.

163.Which of the following is major criteria of rheumatic fever :


A- fever
B- subcutaneous nodule
Answer:b

164.Old man with dm, has redness in calf area, raised and painful, tender:
A. Cellulitis
B. Diabetic neuropathy
Answer: a
• Cellulitis is acute bacterial infection of the skin and subcutaneous tissue most often caused by streptococci or
staphylococci. Symptoms and signs are pain, rapidly spreading erythema, and edema; fever may occur, and regional
lymph nodes may enlarge. Diagnosis is by appearance; cultures are sometimes helpful, but awaiting these results should
not delay empiric therapy. Treatment is with antibiotics. Prognosis is excellent with timely treatment.
Reference: merck manual

190
165.A farmer presented with 2 weeks history of fever, headache and one more symptom. What is the most likely diagnosis?
A. Brucellosis
B. Meningitis

Answer: A
The trick in the duration, bacterial meningitis usually present over several hours ( Cryptococcal meningitis may be present for several
weeks - but it's not one of the options). In the other hand, brucellosis may show up anytime from a few days to a few months after the
person get infected.
Brucellosis is an infectious disease. People can get the disease when they are in contact with infected animals or animal products
contaminated with the bacteria. Animals that are most commonly infected include sheep, cattle, goats, pigs, and dogs, among others.
Signs and symptoms include fever, headache, sweats, malaise, anorexia pain in muscles, joint, and/or back, fatigue, …
Reference: Centers for Disease Control and Prevention

166.Patient with honeymoon cystitis. Which of following could be the organism?


A. Staph. Saprophticous
B. Coli

Answer: b
Honeymoon cystitis is a bladder infection that results from sexual activity. It occurs when the bacteria travels in the upward direction
from the urethra to the bladder.
Reference http://www.m.webmd.com/a-to-z-guides/tc/understanding-bladder-infections-basic-information?Page=2
Http://www.urineinfection.net/honeymoon-cystitis-symptoms-and-relief/

The microbial spectrum of uncomplicated cystitis and pyelonephritis in women consists mainly of Escherichia coli (75 to 95 percent),


167.Positive culture of budding yeast in urine what is the management?
A. Flucanazole
B. Caspofungin

Answer: A
Vulvovaginal candidiasis (VVC) can be managed with either topical antifungal agents or a single dose of oral fluconazole (150 mg).
Candida cystitis in noncatheterized patients should be treated with fluconazole at 200 mg/d orally for at least 10-14 days. For fungi
resistant to fluconazole, amphotericin B is recommended at dose of 0.3 to 0.6 mg/kg IV once/ day for 2 wk for cystitis and 0.5 to 0.7
mg/kg IV once/day for 2 wk for pyelonephritis.
Http://www.merckmanuals.com/professional/genitourinary-disorders/urinary-tract-infections-uti/fungal-urinary-tract-infections
Http://emedicine.medscape.com/article/213853-treatment

168.Pt in ICU on ventilator develop yeast infection what is the treatment?


A. Fluconazole
B. Itraconazole

Answer:A
Antifungal drugs for the treatment of candidiasis include polyenes (amphotericin B and triazoles-fluconazole, itraconazole). The main
adverse events of Amphotericin B include nephrotoxicity, hepatotoxicity, anemia, and thrombocytopenia while triazoles have many
drug interaction. For Aspergillosis although amphotericin B, azoles and echinocandins may be used for the treatment of Aspergillosis,
voriconazole is considered as the first line treatment.
Http://www.ncbi.nlm.nih.gov/pmc/articles/PMC4419464/
Http://emedicine.medscape.com/article/213853-treatment


169.Patient with hemoptysis, at first it was blood tinged then it appeared bright red blood what should the next investigation be?
A. Chest x-ray
B. Ppd

191
170.There is tb outbreak in a region, however you tested one patient and you found him to be negative for tb, what are you going
to give him:
A. Bacillus calmette-guérin vaccine
B. Rifampin chemoprophylaxis
Answer:a
Http://www.cdc.gov/tb/publications/factsheets/prevention/bcg.htm

171.Scenario of diarrhea. Choices about which type of clostridium:


A. Difficile
B. Tetani



172.Treatment of enteric fever :
A. Ciprofloxacin
B. Chloramphenicol

173.Obese, acid reflux symptoms, 2week diagnosed with rf, he is on asprin rx??
A. Metlozam
B. Cinitidine


174.Patient came from africa:
A. Ebola
B. Hiv
Answer: ??

175.Trypanoma cruzi, act by mutaion on what gen.?


A. Myocin
B. Tryptomyocin t

176.Patient transfers blood from kenya and had anal infection…?


A. HBV
B. Syphilis
Most probably is syphilis (their original answer)
Http://www.uptodate.com/contents/blood-donor-screening-laboratory-testing?Source=machinelearning&search=transfusion-
transmitted+infection&selectedtitle=2~34&sectionrank=1&anchor=h8#h22112077
http://www.uptodate.com/contents/pathogenesis-clinical-manifestations-and-treatment-of-early-
syphilis?Source=machinelearning&search=syphilis&selectedtitle=1~150&sectionrank=1&anchor=h10#h10

177.78 year old painless genital ulcer and lymph nodes enlargement:
A. Primary syphilis
B. Secondary syphilis
Answer: not sure
Explanation:
Primary syphilis: chancer that begins as single painless papule as site of inoculation 2-3 weeks after initial exposure. May be
accompanied with bilateral painless rubbery adenopathy
Secondary syphilis: rash may be macular, maculopapular or pustular and may involve whole body or palms or sole. May be associated
with flu-like symptoms, generalized lymphadenopathy and temporary patchy alopecia
Reference: the johns hopkins internal medicine board reviews book

192
178.Boy with sever abdominal pain, guarding, CT show, gas in the wall of intestine
Undercooked meet infected with:
A. Entero Coli
B. Entamiba histolitica
Answer: ?
Organisms that can be found in raw meat:
Beaf: E. Coli O157:H7, Salmonella, Shigella, Staphylococcus aureus and Listeria monocytogenes
Poultry: Salmonella and Campylobacter
Shellfish: Vibrio gastroenteritis, Salmonellas, Plesiomonas shigelloides, Staphylococcus and Bacillus cereus

179.Human papilloma virus cause cervical cancer: (Infectious – Medicine )


16,
B. 18
Answer: both A&B
HPV genotypes can be broadly split into “high-risk” (16, 18, 31, 33, 35, 39, 45, 51, 52, 56, 58, 59 and 68) and “low-risk” (6, 11, 40,
42, 43, 44, 53, 54, 61, 72, 73 and 81) based upon their phenotypic association with the development of cervical cancer. Types 16 and
18 are the most commonly isolated HPV types in cervical cancer with type 16 found in approximately 50 percent of patients.
Reference : https://yhdp.vn/uptodate/contents/mobipreview.htm?25/53/26455

180.A LONG scenario about a patient with high temperature, high WBC, low RBC
And low PH. What are you going to do?
A. Bone marrow aspiration
B. Cultures
Answer : B

181.A case of a patient diagnosied to have cutenous lachmenia or bghlabar/ bhagdad type which type of lachmenia?
A. Kalazar
B. Donavan
And 2 more type of lachmenia

Answer: TROPICA
There are 3 main forms of leishmaniases – visceral (also known as kala-azar and the most serious form of the disease), cutaneous
(the most common), and mucocutaneous. Species that cause cutaneous are L donovani (can cause visceral leishmaniases), L tropica ,
L aethiopica , L major , L infantum. So, L.tropica could be most accurate.
Http://www.cdc.gov/parasites/leishmaniasis/health_professionals/index.html#vl
Http://emedicine.medscape.com/article/220298-overview#a4


182.Long scenario about pt coming from Africa 3 wks ago with fever no other +ve points, What is the Dx?
A. Ebola
B. Yellow fever

Answer:
Yellow fever incubation period 3-6 days and in this scenario 3 weeks so we exclude it
Ebola from 2 to 21 days so it could be
If malaria present most likely 6–30 days (98% onset within 3 months of travel)

DISEASE USUAL INCUBATION PERIOD DISTRIBUTION


(RANGE)

Incubation <14 days

Chikungunya 2–4 days (1–14 days) Tropics, subtropics


193
Dengue 4–8 days (3–14 days) Topics, subtropics

Encephalitis, arboviral (Japanese encephalitis, 3–14 days (1–20 days) Specific agents vary by region
tickborne encephalitis, West Nile virus, other)

Enteric fever 7–18 days (3–60 days) Especially in Indian subcontinent

Acute HIV 10–28 days (10 days to 6 weeks) Worldwide

Influenza 1–3 days Worldwide, can also be acquired


while traveling

Legionellosis 5–6 days (2–10 days) Widespread

Leptospirosis 7–12 days (2–26 days) Widespread, most common in


tropical areas

Malaria, Plasmodium falciparum 6–30 days (98% onset within 3 months Tropics, subtropics
of travel)

Malaria, P. Vivax 8 days to 12 months (almost half have Widespread in tropics and
onset >30 days after completion of subtropics
travel)

Spotted-fever rickettsiae Few days to 2–3 weeks Causative species vary by region

Incubation 14 Days to 6 Weeks

Encephalitis, arboviral; enteric fever; acute HIV; See above incubation periods for See above distribution for relevant
leptospirosis; malaria relevant diseases diseases

Amebic liver abscess Weeks to months Most common in developing


countries

Hepatitis A 28–30 days (15–50 days) Most common in developing


countries

Hepatitis E 26–42 days (2–9 weeks) Widespread

Acute schistosomiasis (Katayama syndrome) 4–8 weeks Most common in sub-Saharan


Africa
Table 5-03. Common infections, by incubation period

183.A case of a patient diagnosed to have cutaneous leishmania or baghdad boil which type of leishmania ?
A. Kalazar
B. Donavan

Answers is

194
=

Cutaneous leishmaniasis (also known as oriental sore, baghdad boil)i s the most common form of leishmaniasis affecting humans
caused by (e.g., L. Major and L.tropica)

Visceral leishmaniasis (VL), also known as kala-azar caused by (e.g., L. Infantum and L. Donovani)




184.Case scenario, then they prescribe organism,,, gram -ve (non lactose fermenter) oxidase test -ve , urease test,,,, then they
asked about the antibiotic:
Diagnosis of one of these bacteria:
• Samonella typhi
• Shigella dysenteriae
• Proteus mirabilis
• Yersinia pestis
Shigella, yersinia: fluoroquinolone
Salmonella typhi: empiric treatment with ceftriaxone or azithromycin, fluoroquinolone resistance is increasing


185.A scenario of a boy who developed aplastic anemia after receiving an antibiotic. What is the antibiotic?
A. Tetracycline

186.Bacteria grow in antiseptic and detergent


A. E.coli
Answer: klebsiella
All the bacterial contaminants of these antimicrobial substances were klebsiellaspecies of which all k. Pneumonia species were multidrug
Http://pubs.sciepub.com/ajnr/2/3/3/

195
187.Young pt wt recurrent bacterial infections (I believe encapsulated organisms), hx of arthritis treated with IVIG, lymph node
biopsy (central, depletion) In invx ) Immunegloulin within normal range , low lymphocytes on CBC ) >>> what is the disease (
not the Dx the pathophysiology of the disease adenosine …… ( SCID )
Answer:


188.Treatment of type of bacteria " fragilis"

Bacteroides species are anaerobic bacteria that are predominant components of the bacterial florae of mucous membrane. These bacteria
are resistant to penicillins, mostly through the production of beta-lactamase. They are part of the normal gi florae and predominate in
intra-abdominal infections and infections that originate from those florae (eg, perirectal abscesses, decubitus ulcers). In general, b.
Fragilis is susceptible to metronidazole, carbapenems, tigecycline, beta-lactam/, and certain antimicrobials of the cephamycin class,
including cefoxitin. The bacteria have inherent high-level resistance to penicillin. Production of beta lactamase appears to be the main
mechanism of antibiotic resistance in b. Fragilis. Clindamycin is no longer recommended as the first-line agent for b. Fragilis due to
emerging high-level resistance.
Https://en.wikipedia.org/wiki/bacteroides_fragilis#cite_note-mandell-12
Http://emedicine.medscape.com/article/233339-treatment

189.Red lesion on the back of the hand with previous prick sensation and fever. What is the diagnosis?
A. Cellulitis
Answer: ??

190.Infectious mononucleosis test….


Answer: monospot.
Monospot® test: horse red blood cells agglutinate on exposure to heterophile antibodies. The monospot® uses this in conjunction with
the principle of the davidsohn differential test. Sensitivity and specificity for monospot® are 70-90% and 100% respectivel
Http://patient.info/doctor/infectious-mononucleosis
Http://emedicine.medscape.com/article/222040-workup

191.Side effect of vancomycin?


Injection: Oral:
>10%: Cardiovascular: Hypotension (accompanied by >10%: Gastrointestinal: Abdominal pain, dysgeusia (with oral
flushing) solution), nausea
Hypersensitivity: Red neck syndrome (infusion rate-related) 1% to 10%: Cardiovascular: Peripheral edema
1% to 10%: Cardiovascular: Local phlebitis Central nervous system: Fatigue, headache
Central nervous system: Chills, drug fever Gastrointestinal: Diarrhea, flatulence, vomiting
Dermatologic: Skin rash Genitourinary: Urinary tract infection
Hematologic & oncologic: Eosinophilia, neutropenia Neuromuscular & skeletal: Back pain
(reversible) Miscellaneous: Fever
<1% DRESS syndrome (drug rash with eosinophilia and <1%: Increased serum creatinine, interstitial nephritis,
systemic symptoms), ototoxicity, renal failure, Stevens- ototoxicity, renal failure, renal insufficiency,
Johnson syndrome, thrombocytopenia, vasculitis thrombocytopenia, vasculitis

192.A farmer was injured with a flower for which he was treated. What is the side effect of his treatment?
Answer: no choices
- Farmers and gardeners are commonly affected by a fungal infection called sporotrichosis. The organism causing this
infection is sporothrix schenckii, which is naturally found in soil, hay, and plants.
- Clinical experience has shown that itraconazole (200 mg/day orally) is the treatment of choice for patients with most
localized forms of sporotrichosis, and amphotericin b (3 to 5 mg/day iv) is the preferred treatment for patients who are
severely ill
196
- Ses of itraconazole are: hearing loss, elevated liver enzymes, hepatotoxicity, and heart failure.
Source: https://yhdp.vn/uptodate/contents/mobipreview.htm?34/0/34822
rose gardener's disease


193.An end stage renal disease patient is admitted to the icu. He develop fever. Blood culture showed yeast. What is the treatment?
Answer: prompt administration of azoles (e.g., fluconazole) or echinocandins (e.g., anidulafungin, caspofungin), depending on the
patient’s clinical picture, co-morbidities and blood culture. The most appropriate drug is fluconazole. Amphotericin b is not used due
to nephrotoxicity.
Review of antifungal medications: http://www.medscape.com/viewarticle/775172_4

194.What is the definitive test for visceral leishmania?


Answer: bone marrow aspiration
Leishmaniasis is a disease caused by an intracellular protozoan parasite (genus leishmania) transmitted by the bite of a female
phlebotomine sandfly.
Laboratory diagnosis of leishmaniasis can include the following:
• Isolation, visualization, and culturing of the parasite from infected tissue
• Serologic detection of antibodies to recombinant k39 antigen
• Polymerase chain reaction (PCR ) assay for sensitive, rapid diagnosis of leishmania species
Other tests that may be considered include the following:
• CBC count, coagulation studies, liver function tests, peripheral blood smear
• Measurements of lipase, amylase, gamma globulin, and albumin
• Leishmanin (montenegro) skin testing (lst) (not fda approved in the united states)
Http://emedicine.medscape.com/article/220298-overview

195.Typical presentation of coxsackie b virus.


Symptoms of infection with viruses in the coxsackie b grouping include fever, headache, sore throat, gastrointestinal distress, as
well as chest and muscle pain. This presentation is known as pleurodynia or bornholm disease in many areas. In some cases, viruses
in the coxsackie b family progress to myocarditis or pericarditis, which can result in permanent heart damage or death.

196.Antiviral inhaled how it work (DNA polymerase-nurnd....-prots...)


Inhibits viral neuraminidases; stops release of virus from cells & prevents virus from crossing mucous lining of respiratory tract
Http://reference.medscape.com/drug/relenza-zanamivir-342636
Zanamivir (rx)relenza:
Http://reference.medscape.com/drug/relenza-zanamivir-342636#10

197.A patient is having sore throat, gingivitis and papule and pustule with erythematous base what is diagnosis?
Answer: no choices
Possible answer: hsv-1 infection
Hsv is the most common viral infection of the oral/facial area. It has two subtypes: type 1, which affects the oral cavity; and type 2,
which affects the genitals. Primary herpetic gingivo-stomatitis is most commonly observed in children from 7 months to 4 years of age
but can also be found in adolescents or young adults. The primary infection may be asymptomatic but can manifest as severe gingivo-
stomatitis, in which the gingiva are painful, inflamed and ulcerated.
Read more about the differentials of gingivitis in the following link: http://cdn.intechopen.com/pdfs/20292/intech-
etiology_of_gingivitis.pdf

198.An asymptomatic man results showed a positive ppd skin test. What will you do next?
Answer: chest x ray if negative inh for 9 m
Chest radiographs may show a patchy or nodular infiltrate. Tb may be found in any part of the lung, but upper lobe involvement
is most common.
Http://emedicine.medscape.com/article/230802-workup#c12

197
199. An x-ray of a patient showed upper lung lobe lesion with calcium stone. What is the most likely organism:
A. Proteus
Answer: a
- It is anaerobic gram-negative bacteria from enterobacteriaceae family
- It has the ability to produce high levels of urease, which hydrolyzes urea to ammonia (nh3), which makes the urine more
alkaline
- Increased alkalinity can lead to the formation of crystals of struvite, calcium carbonate, and/orapatite, which can result
in kidney stones
- Proteus species can also cause wound infections, septicemia, and pneumonia, mostly in hospitalized patients
- Cxr can shows large, thick walled upper lobe abscess with an air fluid level, and associated with infiltrates
(proteus pneumonia)
- In general treatment is through antibiotics such as b-lactams.




200.What is type of pneumococcal vaccine pcv?

201.What is the treatment of gonorrhea in general ?


Recommended regimen: ceftriaxone 250 mg im in a single dose plus azithromycin 1g orally in a single dose – cdc

202.Which virus cause valvular cancer ?


Data suggest that human papillomavirus (hpv) may be a cause of some vulvar malignancies
Http://emedicine.medscape.com/article/264898-overview

203.A patient with trichomonas is completely asymptomatic. When to treat?


Answer: treat the patient and partner even if they are asymptomatic..
Treatment should be instituted immediately and, whenever possible, in conjunction with all sexual partners. 5-nitroimidazole drugs are
used for the treatment of trichomoniasis, single-dose therapy of metronidazole.
Http://emedicine.medscape.com/article/230617-overview

204.Which antibiotic give for shigella gastroenteritis in pediatrics?


A. Ampicillin
Antibiotic treatment may shorten illness duration and shedding but does not prevent complications. Most mild infections will recover
without antibiotics. Moderate to severe cases should be treated with antibiotics. Ampicillin is preferred for drug-sensitive strains. For
ampicillin-resistant strains or in cases of penicillin allergy, trimethoprim-sulfamethoxazole is the drug of choice, although resistance
does occur. Fluoroquinolones may be considered in patients with highly resistant organisms.
Http://emedicine.medscape.com/article/176400-medication

198
205.Food poisoning > shigella -- what ttt
Source of shigella is food/water; associated with overcrowding. Symptoms are fever, nausea, vomiting, severe bloody diarrhea,
abdominal pain (risk of hus). Treatment is hydration; self-limited; ciprofloxacin, tmp-smx in severe cases. (step up to step 2 ck)

206.Salmonella gastroenteritis which ab??


Salmonella gastroenteritis is usually a self-limiting disease. Fluid and electrolyte replacement may be indicated in severe cases. Because
antibiotics do not appear to shorten the duration of symptoms and may actually prolong the duration of convalescent carriage, they are
not routinely used to treat uncomplicated nontyphoidal salmonella gastroenteritis. Current recommendations are that antibiotics be
reserved for patients with severe disease or patients who are at a high risk for invasive disease
Azithromycin is likely to be the preferred empirical treatment, often given together with ceftriaxone, in developed countries where
chloramphenicol is usually reserved for life-threatening situations, for which no alternatives are available, and physicians are reluctant
to use fluoroquinolones in children and lack easy access to gatifloxacin.
Http://emedicine.medscape.com/article/228174-treatment

207.Picture of parasite or fungal ( not sure ) and patient complain of diarrhea ( blood and mucous):
A. Metronidazole

208.Treatment of enteric fever?


A. Ciprofloxacin
Answer: a

209.Male patient has diarrhea after he came back from indonesia and ate from their food, what is the organism?
A. Answer: traveler’s diarrhea e. Coli.

210.Most common organism of infective endocarditis?


(no choices listed)
Answer: (varies between references). Uptodate, medscape, goldman cecil medicine:
StaphylococcalIEis substantially more common in health care associated ie. Streptococci remain common causes of community
acquired ie. In a cohort study: s. Aureus 31% - viridans 17%
Native valve:
Community dwelling patients: streptococcus viridans. Patients with health care contact or injection drug use: s.aureus
Prosthetic valve:
Early within 2m: s. Aureus and coagulase negative staphylococci. Followed by gram neg bacilli and fungi
Late > 2m: streptococci and s. Aureus. Followed by coagulase negative staphylococci and enterococci.
Overall, s. Aureus is the most common cause of ie, including prosthetic valve endocarditis, acute ie, and iv drug abuse ie.
Http://emedicine.medscape.com/article/216650-overview#a4

211.Duke criteria?
(no choices listed)
Answer:

199

212.Patient with flushing face & neck, which antibiotic she's using:
A. Answer: vancomycin (called "red neck" or "red man syndrome") (rxlist) http://www.rxlist.com/vancomycin-injection-side-
effects-drug-center.htm

213.Case of infectious mononucleosis ( diagnosis )bmj http://bestpractice.bmj.com/best-


practice/monograph/123/diagnosis/tests.html

214. Diagnosis of pertussis:

PCR or culture of a nasopharyngeal aspirate or swab from the posterior nasopharynx. Bmj http://bestpractice.bmj.com/best-
practice/monograph/682/diagnosis/tests.html

200
215. Indole positive bacteria:
Aeromonas hydrophila, aeromonas punctata, bacillus alvei,edwardsiella sp., escherichia coli, flavobacterium sp., haemophilus
influenzae, klebsiella oxytoca, proteus sp. (not p. Mirabilis and p. Penneri), plesiomonas shigelloides,pasteurella multocida,
pasteurella pneumotropica, enterococcus faecalis, and vibrio sp. Microbiologyinfo http://www.microbiologyinfo.com/indole-test-
principle-reagents-procedure-result-interpretation-and-limitations/

216.Patient in icu, esrd develops fever culture show yeast on blood what is the?
Answer: it is candidemia

217.Best method to prevent food poisoning


Answer: thoroughly cooked food and rewarm
The four c’s of prevention: cleaning, cooking, chilling & cross contamination (avoiding it) http://www.nhs.uk/conditions/food-
poisoning/pages/prevention.aspx

218.Patient with diarrhea and cx-ray showing bilateral infiltrates (pneumonia). Which organism responsible?
Answer: legionella : the patient is having atypical pneumonia.
Reference: https://www.nlm.nih.gov/medlineplus/ency/article/000079.htm

219.Recurrent viral & fungal infection cell deficient?


1. T cell
Answer: a, link : http://www.antimicrobe.org/e33.asp

220.Child came with rhinorrhea, cough, respiratory distress, which vaccine can prevent this disease?
A. Haemophilus influenzae serotype b (hib)

221.Know about hsv1and hsv2? What is the difference?


Herpes simplex viruses are ubiquitous, host-adapted pathogens that cause a wide variety of disease states. Two types exist: herpes
simplex virus type 1 (hsv-1) and type 2 (hsv-2). Both are closely related but differ in epidemiology. Hsv-1 is traditionally
associated with orofacial disease (see the image below), while hsv-2 is traditionally associated with genital disease; however,
lesion location is not necessarily indicative of viral type.

222.Patient eat mushroom came with nausea vomiting .abdominal pain what the mechanism of this?
[2]
Possible toxins: alpha-amanitin: α-amanitin is an inhibitor of RNApolymerase ii and iii. this mechanism makes it a deadly
toxin. For 6–12 hours, there are no symptoms. This is followed by a period of gastrointestinal upset (vomiting and profuse,
[22]
watery diarrhea). This stage is caused primarily by the phallotoxins and typically lasts 24 hours. At the end of this second
stage is when severe liver damage begins. The damage may continue for another 2–3 days. Kidney damage can also occur. Some
[23]
patients will require a liver transplant. amatoxins are found in some mushrooms in the genus amanita, but are also found in
[10]
some species of galerina and lepiota. overall, mortality is between 10 and 15 percent.
Muscarine: muscarine stimulates the muscarinic receptors of the nerves and muscles. Symptoms include sweating, salivation,
tears, blurred vision, palpitations, and, in high doses, respiratory failure.[29] muscarine is found in mushrooms of the genus
omphalotus, notably the jack o' lantern mushrooms. It is also found in a. Muscaria, although it is now known that the main effect
of this mushroom is caused by ibotenic acid.

201
223.Pt with red points in palms and hands, buttocks, fever. Turned into vesicles
A. -herpes simplex

224.Pt travelled, came back with large watery diarrhea, dx:


A. The more likely answer is traveler’s diarrhea caused by e. Coli

225.Bacteroids in gunshot wound abdomen, what antibiotics:


Penicillin g is still the drug of choice against most non–beta-lactamase–producing agnb. However, in addition to the b fragilis group,
which is resistant to penicillin, other agnb show increased resistance.

The combination of beta-lactamase inhibitors (eg, clavulanic acid, sulbactam, tazobactam) with a beta-lactam antibiotic (eg,
ampicillin, amoxicillin, ticarcillin, piperacillin) can overcome these beta-lactamase–producing agnb.
In high concentrations, ticarcillin, piperacillin, and mezlocillin have good activity against gram-negative enteric bacilli and most
anaerobes cefoxitin is the most effective cephalosporin against the b fragilis group, although 5-15% may be resistant.
Carbapenems these agents, including imipenem, meropenem, doripenem, and ertapenem have excellent activity against a broad
spectrum of aerobic and anaerobic bacteria.

226.N. Meningitis meningococcemia, prophylaxis for family:


Answer: rifampin

227.Question about syphilis ulcer. How will you diagnose?


Answer: dark field microscopy
Explanation: primary syphilis produce painless that is positive under darkfield microscopy for spirochete
Reference kaplan ob/gyne usmle step2

228.Most common organism of infective endocarditis?


A. Staphylococcus aureus
Answer: overall, s. Aureus is the most common causative organism in infective endocarditisreference toronto notes 2015

229.College guy with meningococcemia how to prevent spread:



Isolation for 4 weeks . Chemoprophylaxis seems to be most appropriate especially since classically the case arises in a college student
living in a dorm. Rifampin is commonly used for meningococcal prophylaxis of household contacts in the united states, where one
third of the prevalent strains are sulfadiazine resistant. A 2-day course of rifampin is recommended. A single dose of ciprofloxacin has
been found to provide an effective alternative to rifampin for the eradication of meningococcal carriage in adults.

230.What the agar used for mycobacterium?


Culture of mycobacteria:
There are three types of traditional culture media: egg based (lowenstein-jensen), agar based (middlebrook 7h10 or 7h11), and liquid
(middlebrook 7h12 and other commercially available broths). Growth in liquid media is faster (one to three weeks) than growth on
solid media (three to eight weeks). Growth tends to be slightly better on egg-based medium, but growth is more rapid on agar medium.
Agar medium permits examination of colony morphology and detection of mixed cultures.

231.New born with meningitis organism + cocci??


Etiology of neonatal meningitis:

202
Group b streptococci (gbs) are the most commonly identified causes of bacterial meningitis, implicated in roughly 50% of all cases.
Escherichia coli accounts for another 20%. Thus, identification and treatment of maternal genitourinary infections is an important
prevention strategy.[5] listeria monocytogenes is the third most common pathogen, accounting for 5-10% of cases; it is unique in that it
exhibits transplacental transmission.


232.Diagnosis of giardiasis:
Diagnosis — tools for diagnosis of giardiasis include antigen detection assays and stool examination. In areas where available, antigen
detection tests are preferred over stool examination.
Antigen detection assays — a number of immunoassays using antibodies against cyst or trophozoite antigens have been developed
for stool analysis. Available kits include direct immunofluorescent assays (dfa) that use fluorescein-tagged monoclonal antibodies,
immunochromatographic assays, and enzyme-linked immunosorbent assays (elisas). In general, these methods have greater sensitivity
and faster turn-around time than conventional stool microscopy methods.

233.Wet cutaneous leishmania transmitted by?


Answer: sand fly
Leishmaniasis is caused by infection with leishmania parasites, which are spread by the bite of phlebotomine sand flies. There are
several different forms of leishmaniasis in people. The most common forms are cutaneous leishmaniasis, which causes skin sores.
Http://www.cdc.gov/parasites/leishmaniasis/

234.DM with uti, culture shown, antiseptic ? Psedoumonas.


True. Pseudomonas is known to persist in hospital solutions and antiseptics.

235.Prophylactic of rheumatic disease normal pt ?


Answer: pencillin

236.Q about pt having some infection and having allergy from penicillin. What other medications will you give?
Answer: ??
1. What about other types of antibiotics (alternatives)?
Tetracyclines (e.g. Doxycycline), quinolones (e.g. Ciprofloxacin), macrolides (e.g. Clarithromycin), aminoglycosides (e.g.
Gentamicin) and glycopeptides (e.g. Vancomycin) are all unrelated to penicillins and are safe to use in the penicillin
allergic patient.
Reference: http://www.nhstaysideadtc.scot.nhs.uk/antibiotic%20site/penhypers.htm

237.Syphilis case, what do you give?


Answer: penicillin benzathine (aka penicillin g)
• Benzathine penicillin g (one dose im) is the preferred agent. If the patient is allergic to penicillin, give oral antibiotics
(doxycycline, tetracycline) for 2 weeks.

Reference: step up to medicine

238. Which of the following antibiotics is DNA gyrase and it works on what organism?
Answer: fluroquinolon = pseudomonus

203
239.Fever, flu, then developed rashes, which virus ?
a.Measles


240.A patient diagnosed with n. Gonorrhea what other infection you should look for ?
Answer : chlamydia
(master the board)
Also: http://emedicine.medscape.com/article/218059-overview

241.A patient given yellow fever vaccine, started vomiting and sob.. Can’t remember the rest of q.
Answer: http://www.nhs.uk/conditions/yellow-fever/pages/prevention.aspx



242.A scenario about a patient who deals with flowers and got pricked by rose thorn, presented with redness at the prick site.
Which of the following is the cause:
Answer: sporothrix schenckii . Http://botit.botany.wisc.edu/toms_fungi/feb2003.html

243.Mechanism of action of inhaled antivirus?


Answer: block the ion channel function of the m2 protein of influenza a virus , thus interfering with corresponding specific steps in
the viral life cycle. The neuraminidase inhibitors are novel drugs, designed on the basis of the three-dimensional structure of the
influenza a and b neuraminidase the mechanism by which antiviral drugs interrupt the replicative cycle of influenza is illustrated. M2
inhibitors prevent the m2-mediated acidification of the interior of the virus while it resides in endosomes and the subsequent uncoating
of the viral genome, thus inhibiting viral replication. Neuraminidase inhibitors (nais) prevent cleavage of sialic acid residues and thus
newly formed virus cannot be released from the cell surface to infect adjacent cells; also, virus particles remain associated to one
another.

244.Female complain of malaise rash all over the body sparing the face:
A. Syphilis.

204
Answer: a. Secondary syphilis (can resolve spontaneously) ƒ 2-6 mo after initial infection ƒ nonspecific symptoms: malaise, anorexia,
headache, diffuse lymphadenopathy ƒ generalized maculopapular rash: palms, soles, trunk, limbs ƒ condylomata lata: anogenital,
broad-based fleshy gray lesions ƒ serological tests usually positive except the face, this sx after unprotected sex. Dx?!

245.Scenario about herpes zoster ,, they asked about treatment.


Toronto:
Herpes zoster management:
Compress with normal saline, burow’s, or betadine solution.
Analgesics (NSAIDs, amitriptyline)
Famciclovir or valacyclovir or acyclovir for 7 d; must initiate within 72 h to be of benefit; iv acyclovir for ophthalmic or disseminated
involvement.
Gabapentin 300-600 mg po tid for post-herpetic neuralgia.



246.Abscess in which area in neck will extend to mediastinum
A. Retropharyngeal

247.Parasite transmitted by meat?


A. Trichinosis

248.Most common cause of hepatitis a?


Hepatitis a is usually spread when the hepatitis a virus is taken in by mouth from contact with objects, food, or drinks contaminated by
the feces. Http://www.cdc.gov/hepatitis/hav/afaq.htm

249.Most common infectious disease in pre-employment ex?


A. HBV

250.Known vulvar damage, undergoing tooth extraction, what is the organism :


A. Strep viridians
Explanation: most subacute disease caused by s viridians infection is related to dental disease. Most cases are not caused by dental
procedures but by transient bacteremias secondary to gingivitis. In 85% of patients, symptoms of endocarditis appear within 2 weeks
of dental or other procedures.
Reference: http://emedicine.medscape.com/article/216650-clinical


251.Student had a meningitis what should you do for other students?
A. Antibiotics prophylaxis
Answer:a
Http://www.medscape.com/viewarticle/451596_4

252.Hepatitis prevented by vaccine:


Answer: hepatitis b.
Plasma-derived and recombinant HBV vaccines use hbsag to stimulate the production of anti-hbs in noninfected individuals. The
vaccines are highly effective, with a greater than 95% rate of seroconversion. Vaccine administration is recommended for all infants
as part of the usual immunization schedule, as well as for adults at high risk of infection (eg, those receiving dialysis and healthcare
workers).
Http://emedicine.medscape.com/article/775507-treatment#d15


253.Viral gastroenteritis prevented by which vaccine

205
Answer: rota vaccine . Viruses are responsible for a significant percentage of gastroenteritis cases affecting patients of all ages. Viral
gastroenteritis ranges from a self-limited watery diarrheal illness (usually < 1 wk) associated with symptoms of nausea, vomiting,
anorexia, malaise, or fever, to severe dehydration resulting in hospitalization or even death.
Http://emedicine.medscape.com/article/176515-followup
Rotavirus vaccine. A vaccine that helps protect children against gastroenteritis caused by the rotavirus is now part of the routine
childhood vaccination schedule. This vaccine is given as a liquid that is dropped into a baby's mouth. It is given in two doses, with the
first given at two months and another at three months.
Two possible side effects of the vaccine are diarrhoea and irritability, but these are usually mild and short-lived.
Http://www.nhs.uk/Conditions/Rotavirus-gastroenteritis/Pages/Prevention.aspx

254.Women not oriented and confused her husband have HBV ig titer is -ve direct bilirobin is high (55) what will you do to
confirm dx:
A. Check for anti Hep B core antibody (igm)

Answer: a
Hepatitis b surface antigen (hbsag) and hepatitis b e antigen (hbeag) (marker of infectivity) are the first markers that can be identified
in the serum in acute disease. Hepatitis b core antibody (anti-hbc) immunoglobulin m (igm) follows.
Reference: http://emedicine.medscape.com/article/177632-workup#c7


255.True fever vs factitious fever :
Refers to a condition in which the patient has an elevated temperature (fever) but despite investigations by a physician no explanation
has been found( it should for a week inpatient and a 3 wks. Outpatients). Https://en.wikipedia.org/wiki/fever_of_unknown_origin

256.Long case most probably leishmania disease. What is the treatment for this case
Answer: pentamidine

257.Differentiate between antigenic “drift” and “shift”


Http://www.cdc.gov/flu/about/viruses/change.htm



258.HIV patient has velvety skin rash :
A. Kaposi sarcoma
Explanation: kaposi sarcoma is a multicentric vascular tumor caused by herpesvirus type 8. It is the most common aids associated
cancer, and may be the first aids presentation. Presents as cutaneous lesions are asymptomatic purple, pink, or red macules that may
coalesce into blue-violet to black plaques and nodules.
Reference: http://www.msdmanuals.com/professional/dermatologic-disorders/cancers-of-the-skin/kaposi-sarcoma

259.Fever, diarrhea bloody and stool microscopic showed ( pic ) based on the organism above which drug supposed to be given ?
Answer: metronidazole
Clostridium difficile pseudomembranous colitis results from a disturbance of the normal bacterial flora of the colon, colonization by c
difficile, and the release of toxins that cause mucosal inflammation and damage. Antibiotic therapy is the key factor that alters the
colonic flora. C difficile infection (cdi) occurs primarily in hospitalized patients. Http://emedicine.medscape.com/article/186458-
overview

260.After appendectomy he has lung abscess which antibiotic you will choose:
Most abscesses develop secondary to aspiration and are caused by anaerobes. Clindamycin covers anaerobes.

261.Best way to prevent human brucellosis?


A. Pasteurization of milk products
Refrence: cdc
206
Http://www.cdc.gov/brucellosis/prevention/index.html

262.Tb medication causing red urine :


A. Rifampin
Answer; a, reference ; http://aippg.net/threads/a-patient-with-tuberculosis-develops-bright-orange-red-urine.82781/

263.Scenario of a child after urti there is a history of bleeding after brushing his tooth lab :low platelets. Bone marrow:
megakaryocyte what is the treatment?
Prednisolone i couldn't remember the other choices
Recommended general approach for children with acute immune thrombocytopenic purpurafor initial (induction) treatment, in patients
with a platelet count of 20-30 × 109/l [20-30 × 103/µl] and/or mucocutaneous bleeding), one regimen is prednisone 4-8 mg/kg/d with
the intent of a rapid and complete taper after 7-10 days or when the platelet count reaches 50 × 109/l (50 × 103/µl),
Reference : for diagnosis http://emedicine.medscape.com/article/202158-clinical
For treatment http://emedicine.medscape.com/article/202158-medication

264.Reiter's syndrome
** the classic triad of arthritis, urethritis and conjunctivitis
Reference : http://www.aafp.org/afp/1999/0801/p499.html
Http://www.fpnotebook.com/rheum/spondylitis/rtrssyndrm.htm

265.Patient with history urethral discharge culture (negative )and severe unilateral knee pain what is the diagnosis ?

266.Croup case what will you hear by pulmonary auscultation


a. inspiratory stridor,
Http://www.msdmanuals.com/professional/pediatrics/respiratory-disorders-in-young-children/croup

267.Review conjunctivitis

268. Taenia saginata


Taeniasis in humans is a parasitic infection caused by the tapeworm species taenia saginata (beef tapeworm), taenia solium (pork
tapeworm), and taenia asiatica (asian tapeworm). Humans can become infected with these tapeworms by eating raw or undercooked
beef (t. Saginata) or pork (t. Solium and t. Asiatica). People with taeniasis may not know they have a tapeworm infection because
symptoms are usually mild or nonexistent.
Rx: treated with praziquantel
Ref. Cdc

269.Elderly with back bone pain, bone biopsy positive acid fast bacilli ,the organism?
Answer: mycobacterium tuberculosis

270.Where do latent herpes virus stay?


A. Macrophages.
Answer: sensory neuron?

271.How can prevent dust mite?


Http://www.aafa.org/page/dust-mite-allergy.aspx

207
272.Side effect of the medication used to treat infection (you saw donovan body):
A. Answer: no choices.
Donovan bodies or klebsiella granulomatis are intracellular gram-ve rod organisms seen on ulcer smear in individuals with the
infection: granuloma inguinale.
Azithromycin (1st line tx) se: diarrhea, n/v, abdominal pain
Source: http://reference.medscape.com/drug/zithromax-zmax-azithromycin-342523#4
Donovan bodies (or leishman-donovan bodies) can also refer to the organisms leishmania donovani seen on blood or bone marrow
smear. It is found in patients with visceral leishmaniasis.
Liposomal amphotericin b (1st line tx in vl) se: fever, chills, n/v, hypokalemia, hypomagnesemia, nephrotoxicity
Source: http://reference.medscape.com/drug/ambisome-amphotericin-b-liposomal-999576#4

273.What the organism for someone sky with diver ?

274.Type produce inside cell( DNA , RNA...)

275.Which antibiotic is used in the treatment of salmonella?

The appropriate antibiotic from the best to the alternative


Ciprofloxacin>>>azithromycin >>>> ceftriaxone, >>>> trimethoprim/sulfamethoxazole, >>>> amoxicillin
Ref. BMJ

276.A patient had repeated attacks of reddish rash and plaques in mouth. What is the diagnosis?
A. Answer: candidiasis?

Pseudomembranous candidiasis. The typical adherent white plaques may be removed by wiping firmly with a tongue blade or gauze

277.Pt had constpation +tender abd +wbc high +fever+ s.s of uti....rx?
a) Antibiotic (which one?)
b) Steroid

278.Hypopigmentation and loss of sensation in forearm with ulnar nerve thickness?


A. Leprosy (Hansen disease)
Answer: Leprosy is a chronic infectious disease caused by Mycobacterium leprae, an acid-fast, rod-shaped bacillus. The disease mainly
affects the skin, the peripheral nerves, mucosa of the upper respiratory tract, and the eyes.
Caused by Mycobacterium leprae, an acid-fast bacillus that likes cool temperatures (infects skin
and superficial nerves “glove and stocking” loss of sensation) and cannot be grown in vitro.
Hansen disease has 2 forms:
ƒ Lepromatous —presents diffusely over the skin, with leonine (lion-like) facies, and is
communicable; characterized by low cell-mediated immunity with a humoral Th2 response.
Lepromatous form can be lethal.
ƒ Tuberculoid —limited to a few hypoesthetic, hairless skin plaques; characterized by high cellmediated
immunity with a largely Th1-type immune response.
Treatment: dapsone and rifampin for tuberculoid form; clofazimine is added for lepromatous form.

208
279.N. Meningitis meningococcemia, prophylaxis for family:
A. Rifampin
Answer: Deterrence and prevention of meningococcal meningitis can be achieved by either immunoprophylaxis or chemoprophylaxis.
Rifampin, quinolones, and ceftriaxone are the antimicrobials that are used to eradicate meningococci from the nasopharynx.
(http://emedicine.medscape.com/article/1165557-treatment#d11)


280.Condition which increasing the incidence of TB recurrence?
A. HIV.

281.What are the findings in CSF analysis that determines “bacterial” cause of meningitis?

Answer:
• Appearance: Clear, cloudy, or purulent
• Opening pressure: Elevated (>25 cm H 2 O)
• WBC count: >100 cells/µl (>90% PMN)
• Glucose level: Low (< 40% of serum glucose)
• Protein level: Elevated (>50 mg/dl)

282.Patient with watery diarrhea.

Answer: NOT SURE


A. Acute diarrhea most probably is due to GI infection.
B. Most common cause of acute watery diarrhea (gastroenteritis) worldwide is viral (rotavirus)
Reference: http://www.medicinenet.com/rotavirus/article.htm

283.Treatment of HBV?
Answer: ?
Interferone + ribavirin (Kumar)
Patients with acute hepatitis C virus (HBV) infection appear to have an excellent chance of responding to 6 months of standard
therapy with interferon (IFN). Reference:http://emedicine.medscape.com/article/177792-treatment

284.Schistosoma Tx?

209
A. Praziquantel

Answer: A
Praziquantel and steroid
Oral corticosteroids are the mainstay of treatment to reduce immunological reaction severity and are given until the worst symptoms
resolve. prednisolone should be started 1 day before praziquantel treatment.
. Reference: BMJ

285.Patient on central cath developed infection. Blood gram stain shows budding yeast.
A. Fluconazole

Answer: A
Reference: Toronto notes

286.The most specific/ best diagnostic test for syphilis?


a. Fluorescent treponemal antibody absorption (FTA-ABS)

Answer: A
• Nontreponemal testing is commonly used for serologic screening for syphilis, which include VDRL, RPR, and TRUST.
(These tests must be confirmed with specific treponemal testing since false positive nontreponemal tests can occur.
Nontreponemal testing is performed initially due to its low cost.)
• Specific treponemal tests include FTA-ABS, MHA-TP, TP-PA, and TP-EIA. Reference: uptodate

287.Signs and symptoms of 1st syphilis?


A. Painless genital ulcer

Answer: A
Reference: http://www.cdc.gov/std/syphilis/stdfact-syphilis-detailed.htm

288.Patient with needle sensation after TB drug.


A. Isoniazid

Answer: A

289.40 years old man presents with persistent lesion on the forearm that started 3 months
ago. He came from a deserted area that has sandfly infestation. Giemsa stain showed
Donovan bodies inside and outside monocytes. What will you use to treat him?
A. Miltefosine

Answer: A?
This patient has leishmaniasis.
• Uncomplicated cutaneous leishmaniasis: Topical application of paromomycin or
Intralesional antimony (Sodium stibogluconate, Meglumine antimoniate).
• Complicated cutaneous leishmaniasis: Oral systemic therapy; include azoles and miltefosine
• Agents with activity against visceral leishmaniasis (VL) include amphotericin B, pentavalent antimonial drugs, Paromomycin (a
parenteral aminoglycoside), Miltefosine (the first oral drug for treatment of VL)
Reference: http://www.uptodate.com/contents/treatment-of-cutaneous-leishmaniasis
Additional information ; Topical paromomycin has been shown to be effective against cutaneous leishmaniasis caused by L major ( in
Iraq ) and L mexicana.
While Oral miltefosine is used for Treatment of New World cutaneous leishmaniasis.
Http://emedicine.medscape.com/article/220298-treatment#d9

290.(Long scenario) Patient with fever, lymphadenopathy and hepatosplenomegaly…


210
Answer: ? (Case of Mononucleosis)
See: http://emedicine.medscape.com/article/222040-overview

291.Chlamydia ttt?
Answer: Azithromycin (oral one dose) or doxycycline (oral for 7 days ) + treat all sexuall parners
Reference : step up to medicine

292.Ebola very clear ?


Answer:
Reference: http://www.who.int/mediacentre/factsheets/fs103/en/

293.Parasite infection in under cooked food


Answer: Trichinellosis/Trichinosis
Infection is initiated by ingestion of viable larvae in raw or undercooked meat. Digestive action liberates the larvae.
Reference: http://emedicine.medscape.com/article/787591-overview#a5

294.Cholera vibro antibiotic


A. Doxycycline
Answer: Doxycycline inhibits protein synthesis and, thus, bacterial growth by binding to 30S and possibly 50S ribosomal subunits of
susceptible bacteria.
In areas of known tetracycline resistance, therapeutic options include ciprofloxacin and erythromycin. Strains resistant to
ciprofloxacin have been reported from Calcutta, India.
Pharmacotherapy plays a secondary role in the management of cholera; fluid replacement is primary.
Reference http://emedicine.medscape.com/article/962643-medication#2
The antibiotic options for cholera include macrolides, fluoroquinolones, and tetracyclines. The choice between
them should be based on availability and local resistance patterns.

295.Female pt diagnosed with IBD on ceftriaxon with no benefit wt is the organism


A. Herpes
Can't recall others
Answer: Pseudomonas aeruginosa
Ceftriaxone does not have useful activity against Pseudomonas aeruginosa.It is generally not active against Enterobacter species, and
its use should be avoided in the treatment of Enterobacter infections even if the isolate appears susceptible because of the emergence
of resistance.Some organisms, such as Citrobacter, Providencia, and Serratia, have the ability to become resistant through the
development of cephalosporinases (these enzymes hydrolyze cephalosporins and render them inactive)



296.Monospot test +ve what is the diagnosis?
A. Infectous mononeclosis

211
Answer: A
Epstein-Barr virus (EBV) infectious mononucleosis has been recognized as a clinical syndrome consisting of fever, pharyngitis, and
adenopathy. Heterophile test antibodies are sensitive and specific for EBV heterophile antibodies, they are present in peak levels 2-6
weeks after primary EBV infection, and they may remain positive in low levels for up to a year.The latex agglutination assay, which is
the basis of the Monospot test using horse RBCs, is highly specific. Sensitivity is 85%, and specificity is 100%
Http://emedicine.medscape.com/article/222040-workup
Https://www.nlm.nih.gov/medlineplus/ency/article/003454.htm



297.How to dx hepatitis B?
Answer:
Hbsag, hepatitis B core antibody (anti-hbc), and hepatitis B surface antibody (anti-hbs)



Http://62.210.217.55/humhub-1.0.1/space/page/hepatitis-b-blood-test-results-interpretation
Http://www.hepatitisaustralia.com/hepatitis-b-facts/
Https://labtestsonline.org/understanding/analytes/hepatitis-b/tab/test/



298.Pt with heart valve lesion developed endocarditis, what is the organism?

Answer: Streptococcus viridans
Native valve: Streptococci account for 50%–80% of IE cases ( Streptococcus viridans).
Prosthetic valve:

212
• Early-onset (within 2 months after surgery): Staphylococcus aureus and Staphylococcus epidermidis are the most common.
• Late-onset (more than 2 months of surgery): streptococci is more common.
Staphylococcus aureus is the most common in intravenous drug abusers (ivdas) (most commonly affecting tricuspid valve).
Http://emedicine.medscape.com/article/216650-overview#a4
Http://www.ncbi.nlm.nih.gov/books/NBK2208/
Step up to medicine book (page 47)



299.Infection in venous lines? With needle insertion. (Something like that)

Answer: catheter related blood stream infection (CRBSI), central line associated blood stream infection (CLABSI)
Gram-positive organisms currently account for ~ 50–60% of nosocomial bacteremic events. Staphylococcus epidermidis( coagulase
negative staphylococci) is the most common gram-positive organism isolated from blood (~ 30% of isolates) and accounts for the
majority of infections that are associated with an intravascular catheter. Staphylococcus aureus also causes a significant number of
bloodstream and intravascular catheter. Enterococci have become a significant cause of bacteremia in surgical patients and have
been isolated increasingly from patients with burns or multiple injuries
Gram-negative bacteria account for ~ 30% of all episodes of bacteremia at most institutions.
The mortality associated with gram-negative bacteremia in normal individuals is ~ 10% and may exceed 50% in
immunocompromised patients. The most common causative microbes include Escherichia coli, Klebsiella pneumoniae, Pseudomonas
aeruginosa, Enterobacter aerogenes and cloacae, although a vast array of organisms can be responsible.
Candida most common in yeast category.
In pathogenesis: there are 4 routes for contamination, the most common route of infection for short-term catheters is migration of
skin organisms at the insertion site into the cutaneous catheter tract and along the surface of the catheter with colonization of the
catheter tip.
Http://www.ncbi.nlm.nih.gov/books/NBK7008/
Http://www.cdc.gov/hicpac/BSI/04-bsi-background-info-2011.html
Http://www.ncbi.nlm.nih.gov/pmc/articles/PMC3805442/
Http://lifeinthefastlane.com/ccc/central-line-infections/


300.Patient with lymphadenopathy , splenomegaly and fever, what is the diagnosis?
A. Infectious mononucleosis
Answer:A
Infectious mononucleosis by EBV , early sign is a fever, lymphadenopathy. Later finding include hepatomegaly and splenomegaly.
Reference: medscape


301.Gastoentritis ddx
Answer:
DIFFERENTIAL DIAGNOSIS — The differential diagnosis of acute viral gastroenteritis includes other causes (infectious and non-
infectious) of acute diarrhea. Diarrhea that lasts over a week in an individual with a history of travel, hiking, or oral-anal sexual
activity should prompt evaluation for protozoa such as Giardia and cryptosporidium. Recent antibiotic use or hospitalization should
prompt consideration of Clostridium difficile infection. Common foodborne illnesses (eg, Staphylococcus aureus) need to be
considered, particularly when the incubation period is shorter than is typical for viral illness (ie, within 8 to 16 hours). The presence
of alarm symptoms or signs should prompt further investigation for an alternate diagnosis.
Causes of chronic diarrhea that may less commonly masquerade as acute viral gastroenteritis include: colorectal cancer, irritable
bowel syndrome, inflammatory bowel disease, microscopic colitis, malabsorption syndromes, post-cholecystectomy related
diarrhea, medication-induced diarrhea, laxative abuse, and chronic infections. Patients with acute viral gastroenteritis may also
present with isolated vomiting without prominent diarrhea. Clinicians should consider adverse effects of medications and acute
vestibular disorders in the differential diagnosis of these patients.
Uptodate
302.Splenctomy what vaccine he should took ?
Answer: pneumococcus, Haemophilus influenza type B (Hib), and meningococcus
''People without a functional spleen have certain vaccinations to reduce the risk of sepsis. Although these vaccines do not prevent all
infections with pneumococcus, Haemophilus influenzaetype B (Hib), and meningococcus, they can protect most people and reduce the
severity of infection in others''
Reference:
213
Http://www.uptodate.com/contents/preventing-severe-infection-after-splenectomy-beyond-the-basics

303.Mycobacterium tuberculosis , that is the best culture media ?


A. Blood agar
Answer: löwenstein Jensen agar
I think the blood agar is not the correct answer
Löwenstein–Jensen medium is a growth medium especially used for culture of Mycobacterium, notably Mycobacterium tuberculosis.
Also egg based media was used so I don’t know the correct answer without the choices.
References: http://www.microbiologyinpictures.com/mycobacterium-tuberculosis.php


Rheumatology



















214

1. How are you going to manage a HTN patient with acute gout?
NSAID
A. Allopurinol
B. Colchicine
C. Intra articular steroid
Answer: d
The treatment of acute gout is NSAIDs: high dose, then taper as symptoms improve but
corticosteroids (ia, oral, or intra-muscular) if renal, cardiovascular, or gi disease and/or if NSAIDs contraindicated or failed.
Reference:toronto notes


2. High density bone in dexamethasone with scheduled total hip replacement
A. Osteoporosis
B. Osteomalacia
C. Osteoarthritis
D. Paget disease
Answer: d
It could be paget disease. However, steroids have no role in its treatment except in case of developing pseudomalignancies. . Another
possibility is osteopetrosis (marble bone disease) which also has increased bone density & steroids are used to enhance bone resorption
and improve the anemia. Both commonly affect pelvis and increase its risk of fracture. So it could’ve been in the answers.
Http://www.aafp.org/afp/2002/0515/p2069.html
Http://emedicine.medscape.com/article/123968-medication

3. Most specific sign of osteropreosis?


A. Increase alp
B. Increase calcium
C. Calcification
D. Decrease bone matrix
Answer: d
4. Bechet disease is more common in:
A. Children
B. Young men
C. Old women
D. Equal in men & women
Answer:
The sex prevalence varies by country. In the middle east, behçet disease is more common among males, with male-to-female ratios of
3.8:1 (israel), 5.3:1 (egypt), and 3.4:1 (turkey). In germany, japan, and brazil, the disease is slightly more common in females. In the
united states, behçet disease is more common in females (5:1 female-to-male ratio)
Link: http://emedicine.medscape.com/article/329099-overview#a7

5. Female patient with skin thickening over the forearm, with raynaud’s phenomenon. [case of scleroderma] which of the
following would help you the most?
A. Antiscleroderma - scl 70
B. Anti-centromere
C. Ss-a
D. Ss-b
Answer: b
This case is showing limited disease. Symptoms of limited disease include (skin sclerosis is restricted to the hands, the distal forearm,
face and neck. And generally have prominent vascular manifestations, including severe raynaud phenomenon and cutaneous
telangiectasia). So in limited disease anti centromere would help the most. - uptodate

6. Female with arthritis and butterfly rash on the face. Ana is positive. What other marker you would order?
A. Anti double stranded DNA
B. Anti centromere

215
C. Ss-a
D. Ss-b
Answer: a

7. 61 y female known case of osteoarthritis, came for regular checkup, not taking ca supplements nor high ca diet, she is a high
risk of osteoporosis what is the best initial thing before deciding the appropriate mx ?
A. Dexa scan
B. Oral ca, Vitamin d, biaphpsphonate
C. TSH , ca, Vitamin d
D. Intranasal calcitonin, ca, Vitamin d
Answer: a

8. The first initial treatment of osteoarthritis in middle-aged patient is:


A. Intra articular steroids.
B. Oral steroids.
C. Stairs exercise
D. Muscle strengthening.

Answer: d
Optimally, patients should receive a combination of non-pharmacologic and pharmacologic treatment.
• Non-pharmacologic: patient education, heat and cold, weight loss, exercise, physical therapy, occupational therapy,
unloading in certain joints (eg, knee, hip).
• Pharmacologic: acetaminophen/NSAIDs.
Reference: medscape: http://emedicine.medscape.com/article/330487-treatment#d1



9. Long scenario about a patient who presented with gout. Inhibition of which enzyme will treat this disease?
A. PRPP synthase
B. Adenosine deaminase
C. Xanthine oxidase
D. Orotate phosphoribosyltransferase

216
Answer: C
The enzyme xanthine oxidase catalyses the oxidation of hypoxanthine to xanthine and then to uric acid, which plays a crucial role in
gout. Reference: Pubmed.
That’s why we give Xanthine Oxidase inhibitors such as Allopurinol.

10. Which of the following is the most specific for Rheumatoid arthritis?
A. HLA-DR4.
B. Rheumatoid factor.
C. CRP.
D. Anti cyclic citrullinated peptide. (Anti-CCP)

Answer: D
Anti CCP is the most specific for RA
Reference: http://qjmed.oxfordjournals.org/content/100/4/193



11. Pt with 1st metatarsal joint pain, redness and erythema. High temperatures? What is the cause?
A. Staph aureus
B. Na monourate crystal
C. Ca pyrophosphate crystal

Answer: b
Came with gout
This joint ,podagra, is typical for gout(sodium monourate crystals) though it is clinically indistinguishable from pseudogout (this is
why it is pseudo)
I think b is correct. Acute gout can come with high grade fever
Reference: http://worldwidescience.org/topicpages/a/acute+gouty+arthritis.html
Monosodium urate monohydrate crystals cause gout.
Calcium pyrophosphate crystals cause psuedogout.
The scenario describes podagra, which is spontaneous onset of excruciating pain, edema, and inflammation in the metatarsal-
phalangeal joint of the great toe
Podagra is the initial joint manifestation in 50% of gout cases; eventually, it is involved in 90% of cases.
Psuedogout usually manifests in large joints like the knee.
Reference: Medscape

12. How are you going to manage a patient with a long history of big toe pain with –ve bifrengt [?] Crystals what is the
management
Colchicine
A. Allopurinol
B. NSAID

Answer: c



13. Which of the following anti osteoporotic agent cause osteoclast purse [?]?
Alendronate
A. Risedronate
B. Ibandronate

Answer:?
These medications are bisphosphonates. They inhibits osteoclastic bone resorption and promotes renal excretion of calcium. There
side effects include gi,msk pain, headache, osteonecrosis of the jaw and atypical femoral shaft fractures .
Reference: toronto notes

217


14. Dermatomyositis associated with :
A. generlized morbiliform eruption
B. distal muscle weakness
C. malignancy
Answer: c
Reference: http://www.mayoclinic.org/diseases-conditions/dermatomyositis/basics/complications/con-20020727



15. Man with osteoarthritis initial management
A. Intra-articular corticosteroid injections
B. Exercise to increase the strength of thigh muscle
C. Opioid
Answer: b
Local analgesics (e.g., capsaicin, methylsalicylate cream, or topical NSAIDs) should be used as first-line therapy. In addition to exercise
programmes, physiotherapy and occupational therapy and quadriceps-strengthening exercises
Http://bestpractice.bmj.com/best-practice/monograph/192/treatment/details.html

16. When diagnosing patient with sle, what's most important:


A. Age
B. Sex
C. Joint affection
Answer: c
Although more than 90% of cases of sle occur in women, gender and age are not as important as “joint affection” because it is part of
the diagnostic criteria of sle
Sle criteria: http://www.rheumtutor.com/2012-slicc-sle-criteria/

17. A patient complains of unilateral knee swelling and pain. Knee tap shows: cloudy yellow color, mucoid, wbc - pmn 15 (normal
less than 200), lymphocytes 80%. What is the most likely diagnosis?
A. Gout
B. Septic arthritis,
C. Rheumatoid arthritis
Answer:




18. Something about prognosis of sle?
A. Sex
B. Arthritis
C. Nephritis

218
Answer: c

19. 50 yo w painless loss of vision in one eye, with headache and pain when touching the hair on the same side. What is next?
A. topical steroids
B. oral steroids
C. brain ct
Answer: b
This appears to be a presentation of giant cell arteritis. Symptoms include:
Headache — also common in the presentation of gca is headache, which occurs in more than two-thirds of patients [15,19]. The
quality of headache in gca, apart from the occasional specific complaint of tenderness of the scalp to touch, has no defining
characteristics.
Jaw claudication — nearly one-half of gca patients experience jaw claudication. In some cases, a trismus-like symptom occurs rather
than fatigue of the muscles of mastication. Two striking features of jaw claudication are its rapid onset after the start of chewing and
the ensuing severity of pain. Patients seldom recognize the significance of symptoms of jaw claudication and must be questioned
directly about this symptom
Transient visual loss (amaurosis fugax) — transient monocular (and, rarely, binocular) impairment of vision can be an early
manifestation of gca. With transient monocular visual loss (tmvl), affected patients typically note an abrupt partial field defect or
temporary curtain effect in the field of vision of one eye.
Treatment:
Glucocorticoid treatment should be instituted promptly once the diagnosis of gca is suspected strongly, often even before it is
confirmed. For patients in whom the diagnostic suspicion of gca is high, especially those with recent or threatened vascular
complications such as visual loss, therapy should be started immediately. A temporal artery biopsy should be obtained as soon as
possible, but treatment should not be withheld while awaiting the performance or the results of the biopsy.
If gca is not complicated by symptoms or signs of ischemic organ damage (eg, visual loss), we suggest an initial dose of
glucocorticoid equivalent to 40 to 60 mg of prednisone in a single dose. IV steroid if there is a visual loss.

20. Case of gout the q is what case the crystal in the joint:
A. Purine
B. Amino acid
C. Fatty acid
Answer: a
Http://ccdalmatians.com.au/product/urate-crystals-stones/
Uric acid, the final product of purine metabolism, is a weak acid that circulates as the deprotonated urate anion under physiologic
conditions, and combines with sodium ions to form monosodium urate (msu). Msu crystals are known to have a triclinic structure; in
which stacked sheets of purine rings form the needle-shaped crystals that are observed
microscopically. Http://www.ncbi.nlm.nih.gov/pmc/articles/pmc3975080/
Many enzymes are involved in the conversion of two purine nucleic acids, adenine and guanine, to uric acid.
Http://www.sciencedirect.com/science/article/pii/s0167527315303429

Answer: Overproduction accounts for only a minority of patients presenting with hyperuricemia. The causes for hyperuricemia in
overproducers may be either exogenous (diet rich in purines) or endogenous (increased purine nucleotide breakdown). A small
percentage of overproducers have enzymatic defects that account for their hyperuricemia. These include a complete deficiency of
hypoxanthine guanine phosphoribosyltransferase (HGPRT) as in Lesch-Nyhan syndrome, partial deficiency of HGPRT (Kelley-
Seegmiller syndrome), and increased production of 5-phospho-alpha-d-ribosyl pyrophosphate (PRPP) activity. Accelerated purine
degradation can result from rapid cell proliferation and turnover (blast crisis of leukemias) or from cell death (rhabdomyolysis,
cytotoxic therapy). Glycogenoses types III, IV, and VII can result in hyperuricemia from excessive degradation of skeletal muscle
ATP.
Reference: http://emedicine.medscape.com/article/241767-overview#a5

21. A patient devolved gout. Which of the following could possibly cause it?
A. Thiazide
B. Hypertension
C. Congestive heart failure
Answer: a
Explanation: gout is caused by defect in urate metabolism which could be due to:

219
Over-production: idiopathic, increased turnover of cells (cancer, hemolysis, psoriasis, chemotherapy) and enzyme deficiencies (lesch-
nyhan syndrome, glycogen storage disease).
Under-excretion: renal insufficiency, keto/lactic acidosis and thiazide & aspirin
Reference master the board step 2 ck

22. Old man with multiple fractures during the last 2 years, he is k/c of osteoporosis. What should you give him at this stage?
A- alendronate.
B- vitamin d.
C- estrogen.
Answer: a- alendronate.

23. A patient with gout. What drug should be avoided?


A. Thiazide
B. Furosemide
C. ACEI

Answer: A
Hyperuricemia is a relatively common finding in patients treated with a loop or thiazide diuretic and may, over a period of time, lead
to gouty arthritis. Furosemide can elevate uric acid levels as well. Reference: uptodate.

24. 60 years old man with a fractured thoracic vertebrae. T Score = -2.6. What is the diagnosis?
A. Osteoporosis
B. Established osteoporosis
C. Osteopenia

Answer: B
Dexa scan interpretation:
T-score Diagnosis

>–1.0 Normal

<–1.0, >–2.5 Osteopenia

<–2.5 Osteoporosis

<–2.5 plus fragility fractures Severe/ Established osteoporosis




25. 70 years old smoker woman with low vit-D and osteoporosis. Which of the following has the highest risk for osteoporosis?
A. Smoking
B. Age
C. Vitamin D

Answer: B
Increasing age is an important risk factor in both men and women, since bones become less dense and weaker with age.
Reference: http://www.medscape.com/viewarticle/410461_3

26. The most risk factor for osteoporosis is:


a. Age

220
b. Sex
c. Decrease oral intak of calcium and vit D
d. smoking
Answer is :

27. (long scenario) adult with right toe tenderness for 1 week and he have ear pain also (investigation included). What is the best
treatment for him?
A. Cortisone.
B. NSAIDs.
C. Allopurinol.

Answer: B
(For acute attack. Allopurinol can be given after attack resolution)
Treatment of acute gout attacks does not differ substantially in patients with or without clinically apparent tophi, although the presence
of tophi is an indication for the initiation of long-term urate-lowering therapy after attack resolution to prevent or reverse chronic
gouty arthropathy.
Reference:
Http://emedicine.medscape.com/article/329958-treatment#showall
Http://www.uptodate.com/contents/treatment-of-acute-gout

28. Rheumatoid arthritis with the loss of bones in joint? What is the cause?
A. Substance released by synovial cell
B. Synovial fluid pressure
C. Something prostaglandin?

Answer: A
The bone loss is most pronounced in areas immediately surrounding the affected joints. Bone is composed primarily of type I
collagen, bony destruction is a characteristic of RA. This process is primarily driven by the activation of osteoclasts. Osteoclasts
differentiate under the influence of cytokines especially the interaction of RANK with its ligand. The expression of these are driven by
cytokines including TNF and IL1, as well as other cytokines including IL-17. There may also be a contribution to bony destruction
from mediators derived from activated synovial cells.
Http://www.hopkinsarthritis.org/arthritis-info/rheumatoid-arthritis/ra-pathophysiology-2/
Http://arthritis-research.biomedcentral.com/articles/10.1186/ar2337

29. Case of arthritis in first metatarsal joint ?Dx‫ ؟‬sumptoms were pointing to gout
A. Gout ..
B. Ra..
C. Osteoarthritis ..


30. Giant cell arteritis sequelae:
A. Complete loss of vision
B. Destructive arthritis
Answer: a
If left untreated may lead to permanent vision loss
Reference: kaplan internal medicine

31. Adolescent male with swelled parotid and salivary gland with dry eye and dry mouth, labs hla, ana and rf are positive, which
of the following is appropriate treatment?
A. Physostegmine
B. Artificial eye and saliva drops

Answer: b

32. Temporal arteritis, confirmatory dx?
A. Biopsy temporal muscle
B. Biopsy temporal artery

221
Answer b
Superficial temporal artery biopsy (tab) is the criterion standard for making a diagnosis of temporal arteritis. Tab should be obtained
almost without exception in patients in whom gca is suspected clinically.



33. 60 years old female with distal phalangeal joint swelling and shoulder pain and knee pain x ray showed narrow joint space
and osteophytes:
A. Rheumatoid arthritis
B. Osteoarthritis
Answer : b toronto notes

34. A clear case of septic arthritis - high wbc high esr?


A. Aspiration
B. Aso
Answer: a definitive diagnosis uptodate



35. Patient with rheumatic fever on asprin developed heart burn what you will add
A. Misoprostol
B. Durg from ( h2 blocker )
Answer: a
Treatment of NSAID induced ulcer: combine NSAID with PPIor misoprostol ( torrento 2015)

36. Patient is diagnosed with rheumatoid arthritis and is on aspirin he developed symptoms of heartburn which is relieved with
antacids what will you give him:
A. Mesoprostol
B. H2 blocker
Answer: a
Ongoing medication for peptic ulcer specifically indicated for non-steroidal anti-inflammatory drug (NSAID)-associated ulcers only
Http://bestpractice.bmj.com/best-practice/monograph/80/treatment/details.html

37. Patient presented with knee swelling and pain they did x ray and aspiration and found negative birefringence needlelike
what are you going to discharge the patient with:
A. Allopurinol
B. NSAID
Answer: b

38. Morning stiffness , pain and weakness in proximal muscles , difficulty in getting out of setting position :
A. Polymyalgia reuhmatica
B. Polymyositis

Answer: b
Explanation: polymositis is progressive symmetrical proximal muscle weakness (shoulder and hip) developing over weeks to months
(difficulty lifting head off pillow, arising from chair, climbing stairs). It is a systemic disease" symptoms may include the following:
morning stiffness - fatigue - anorexia - fever - weight loss
Reference: step up to medicine – 3rd editon, page 259
Http://emedicine.medscape.com/article/335925-clinical

39. Headache when combing hair, tender vessel on temporal area, sudden loss of vision or clouding in one eye. What next action?
222
A. Oral prednisolone for 3 months
B. Immediate cortisone eye drops\
Answer: a
Treatment with steroid should be started as soon as the diagnosis of temporal arteritis (ta) is suspected even before confirmation. A
biopsy of temporal artery should be obtained immediately. However, if the biopsy did not show evidence of ta despite high clinical
suspicion, steroid therapy should be continued anyways.
Http://cursoenarm.net/uptodate/contents/mobipreview.htm?20/11/20656v


40. Polymyalgia rheumatica. Aid dx ?
A. Proximal muscle weaknesses
B. Proximal muscle tenderness
Answer: b

41. Old man with joint pain that worsens on movement. X-ray of wrist shows narrow joints in small joints. What's the dx?
A. Osteoarthritis oa
B. Rheumatoid arthritis ra
Answer: a
Radiographs help distinguish rheumatoid arthritis from osteoarthritis. OAis characterized by narrowing of the joint space due to cartilage
loss and osteophytes due to bone remodeling, but not erosions or cysts which are characteristics of ra.
Http://cursoenarm.net/uptodate/contents/mobipreview.htm?20/22/20833#h8594869

42. A case of t score of -3.5 what is the diagnosis:


A. Osteopenia
B. Osteoporosis

Answer: b
-1 and above: normal.
Between -1 and -2.5: osteopenia
-2.5 and below: osteoporosis.

43. What’s the management of septic arthritis?


A. Aspiration and antibiotic
B. Iv antibiotic
Answer: a
Reference: http://cursoenarm.net/uptodate/contents/mobipreview.htm?32/63/33776#h21

44. A patient who is medically free presented with swollen knee, no tenderness or redness. What is the best next step?
A. Mri
B. X-ray
Answer: b
Explanation: the use of plain radiographs is often necessary to assess a swollen knee. In an acute injury, selecting the appropriate
radiographic series is critical. The most common views are the anteroposterior (ap), lateral and axial patellar images to assess for fracture,
dislocation and effusion.
Reference: http://www.aafp.org/afp/2000/0415/p2391.html

45. In gout what substance would be high?


A. Prpp
B. Xanthine
Answer: a
There are at least three different inherited defects that lead to early development of severe hyperuricemia and gout: glucose-6-
phosphatase (gene symbol: g6pt) deficiency; severe and partial hypoxanthine-guanine phosphoribosyltransferase (hgprt, gene symbol:
hprt) deficiency; and elevated 5'-phosphoribosyl-1'-pyrophosphate synthetase (prpp synthetase) activity

223
Http://themedicalbiochemistrypage.org/gout.php
- xanthine oxidase inhibitor allopurinol is used in the treatment of gout.

46. Old lady with osteoarthritis and risk for osteoporosis , what you will give?
A. Calcium, TSH , dihydroxy Vitamin d.
B. Bisphosphonate, Vitamin d , calcium.

Answer: b
Guidelines from the american association of clinical endocrinologists (aace), published in 2010, include the following
recommendations for choosing drugs to treat osteoporosis[11] :
First-line agents: alendronate, risedronate, zoledronic acid, denosumab
Second-line agent: ibandronate
Second- or third-line agent: raloxifene
Last-line agent: calcitonin
[http://emedicine.medscape.com/article/330598-treatment#d8]

47. Bilateral shoulder and hip stiffness and pain what is dx :


A- polymyalgia rheumatica
B- OA

Answer: a
Polymyalgia rheumatica (pmr) is characterized by aching and morning stiffness in the shoulders, hip girdle, neck, and torso in patients
over the age of 50
Reference: uptodate http://goo.gl/zzpjli

48. Pt with pic of osteoarthritis on examination there is nodule in the tips of the finger the name of the finding?
A. Heberden
B. Boutchared
Answer: a
note:
OApip nodules- boutchard (the p/b sound )
OAdip nodules: heberden.
Reference: uptodate

49. Patient with OAwhich type of exercise is the best :


A- high repitition and .........
B- low repition and .......
Answer: a? Not enough info

50. A patient presented with hip and shoulder pain. ESR: high (Polymyalgia Rheumatica case). In addition to these symptoms,
what else can be there in the history?
A. Proximal muscle weakness.
B. Proximal muscle Tenderness.

Answer: B
Muscle weakness is not a feature of PMR.
Reference: http://emedicine.medscape.com/article/330815-clinical

Signs and Symptoms (Toronto Note )


• constitutional symptoms prominent (fever, weight loss, malaise)
• pain and stiffness of symmetrical proximal muscles (neck, shoulder and hip girdles, thighs)
• gel phenomenon (stiffness after prolonged inactivity)
• physical exam reveals tender muscles, but no weakness or atrophy

51. A patient with right knee osteoarthritis, presented with swelling of the right knee. On examination the right knee is swollen
with no change in temperature. What will you do?
224
A. Aspiration of the knee fluid
B. Bilateral Knee X-Ray and Ibuprofen.

Answer: B
Because there is no change in temperature.
It’s normal for osteophyte to cause swelling of the knee in OA

52. How would you manage aggressive ra:


A. Methotrexate
B. Cyclosporine
Answer:a
Methotrexate is the gold standard and is first-line unless contraindicated. The management of ra should starts with disease modifying
anti-rheumatic drugs (dmards) as soon as possible.including: hydroxychloroquine, sulfasalazine,methotrexate, leflunomide. Note that
cyclosporine is one of dmards but it is not commonly used. Surgical therapy is indicated for structural joint damage.
Reference: toronto notes

53. Treatment of acute gout :


A. Indomethacin
B. Indomethacin is commonly used if patient has contraindication to it or other nsid colchcen could be use and at the end if
there is no response steroid is the final path

54. Pseudogout What type of calcium crystals?


A. Carbonate
B. Pyrophosphate
Answer : B
Https://yhdp.vn/uptodate/contents/mobipreview.htm?17/45/18134
Http://www.mayoclinic.org/diseases-conditions/pseudogout/basics/definition/con-20028152
Answer: PSEUDOGOUT DIAGNOSIS — A healthcare provider can confirm or rule out a diagnosis of pseudogout by performing an
examination and tests. In many patients, a sample of joint fluid is obtained in order to determine whether calcium pyrophosphate
dihydrate (CPP) crystals are present and to exclude arthritis due to other causes, such as gout or joint infection.
Ref: uptodate


55. Behcet's disease:
A. Vasculitis << ulcer ...test
Answer:a
Behçet disease is a rare vasculitic disorder that is characterized by a triple-symptom complex of recurrent oral aphthous ulcers, genital
ulcers, and uveitis.
Reference: http://emedicine.medscape.com/article/329099-overview#showall

56. Long scenario about ra and he give lab result he q was what is the type of anemia in ra patient ?
Answer: normocytic normochromic ( ddx: iron deficiecy anemia in 50 to 70 ٪ )
Uptodate:
Http://www.uptodate.com/contents/hematologic-manifestations-of-rheumatoid-
arthritis?Source=outline_link&view=text&anchor=h3#h3

57. Patient known case of behcet's disease came with arthritis resistant to steroid what to do:
A. Colchicine
Answer: a ?
- Lesions resistant to local measures may require systemic treatment with colchicine, or an oral corticosteroid or other
immunosuppresant agent
- Tnf-alpha inhibitors: helpful in mild behcet's syndrome resistant to corticosteroids and oral immunosuppressants
- Http://bestpractice.bmj.com/best-practice/monograph/376/treatment/details.html

225
58. Patient with behçet syndrome not responding to steroid wash what to give?
A. Azathiiprine
Answer: a

● Colchicine – 1 to 2 mg/day, administered in divided doses.


● NSAIDs
●Glucocorticoids – Joint complaints not controlled by colchicine may require low-dose glucocorticoids.
●Azathioprine – The primary use is as a glucocorticoid-sparing agent.
●Methotrexate has not been well-studied.

59. Man with features of temporal arteritis, biopsy showed giant cells. What is the most concern problem?
A. Loss of vision
Reference: http://emedicine.medscape.com/article/332483-overview

60. Patient with ankylosing spondylitis associated with ?


A. Uveitis
Answer: a
Acute anterior uveitis is the most common extra-articular feature of as, occurring in 25%-40% of patients at some time in the course of
their disease.
Link: http://www.medscape.org/viewarticle/544971_2

Ankylosing spondylitis (A)s:


• Apical lung fibrosis
• Anterior uveitis
• Aortic regurgitation
• Achilles tendonitis
• AV node block
• Amyloidosis (kidney)
• Autoimmune bowel disease (UC)
• Atlanto-axial subluxation


61. Headache unilateral. Sample taken from temporal artery showed.. This patient at risk of....
Answer: vision loss due to temporal arteritis.

62. Polymyalgia rhumatica question, what supports the dx


There is no pathognomonic test or established diagnostic criteria for polymyalgia rheumatica (pmr). We use the presence of all of
the following empirically formulated criteria for the clinical diagnosis of pmr, in whom another disease to explain the findings is
not present [20,45-47]:
●age 50 years or older at disease onset.
●proximally and bilaterally distributed aching and morning stiffness (lasting at least 30 minutes or more) persisting for at least
two weeks. The stiffness should involve at least two of the following three areas: neck or torso, shoulders or proximal regions of
the arms, and hips or proximal aspects of the thighs.
●erythrocyte sedimentation rate (esr) (westergren) ≥40 mm/h.
●rapid resolution of symptoms with low-dose glucocorticoids. Symptoms are generally 50 to 70 percent better within three days
in patients with pmr started on prednisone at a dose of 10 to 20 mg/day. The lack of response to initial therapy strongly suggests
an alternative diagnosis.

63. A patient has osteoporosis on ca and Vitamin d but no improvement why?


A. Hypomagnesemia
Answer: a. Very low mg prevents release of pth and causes unresponsive hypocalcemia but not related to osteoporosis
226
64. Pt comes with oral and genital ulcer ...dx?
A) bechet disease
Answer: a

65. Case of early osteoporosis. How you'll manage?


Bisphosphonates inhibit bone resorption and are first-line treatment.
• They decrease osteoclastic activity (via binding to hydroxyapatite) and decrease the risk of fractures.
• Oral bisphosphonates (alendronate, risedronate) are preferred in most patients
• Side effects include reflux, esophageal irritation, and ulceration.
• • if patient cannot tolerate oral bisphosphonates, iv bisphosphonates (iv zoledronic acid)
Reference: step up to medicine

66. Pt does not complain of anything ,, has sudden knee swelling ? What is the best thing to do ?
Answer: xray
x-ray if no symptoms
aspiration if septic arthritis
MRI to check meniscus

67. Patient does not complain of anything, has sudden knee swelling? What is the best thing to do?

Answer:
Treatment will depend on the cause of the swollen knee, but the most common ways to reduce the swelling are:
• Ice: to slow down the blood flow and therefore reduce swelling and pain.
• Compression: tubigrip and knee braces can be used to provide compression to the knee which helps reduce swelling.
• Medication: non-steroidal anti-inflammatories e.g. Ibuprofen may be prescribed to reduce the knee swelling and pain
• Knee aspiration
• Steroid injections: reducing inflammation and pain
• Elevation: higher than the level of the heart can help treat a swollen by gravity.
• Rest

68. Young boy with pain in his knee, aspiration of fluid reveal yellowish and turbid appearance, diagnosis?
A. Septic arthritis
Answer: infected joint fluid is typically yellow-green due to elevated levels of nucleated cells, and the cell count is usually markedly
elevated, demonstrating a predominance of polymorphonuclear leukocytes.
(http://emedicine.medscape.com/article/236299-workup#c8)

69. Treatment of rheumatic fever?


A. Penicillin and high dose of aspirin
Answer:
A regiment of antibiotic and anti-inflammatory. Penicillin and aspirin or prednisone is severe cases. Erythromycin or sulfadiazine
may be used in patients who are allergic to penicillin.
Http://emedicine.medscape.com/article/236582-medication#2

70. Ttt of polymyalgia rheumatica?


A. Answer: corticosteroid
Oral corticosteroids are the first line of treatment
Reference: http://emedicine.medscape.com/article/330815-medication

227
71. Patient symptoms of rheumatic fever with heart involvement and history of sore throat was not treated.. Treatment ?
A. Aspirin and steroid



72. Treatment for rheumatoid arthritis uses (something about macrophages and tnf and il) what can recur in patients using the
treatment mechanism?
a. TB
Hepatitis B and C status, purified protein derivative (PPD), FBC, and LFTs need to be checked before starting biological agents
BMJ

73. Pt numbness in rt arm +feel electric shock dx ?


B. Cervical disc prolapse or spondylitis

74. Patient with symptoms of gout , which medication will help :


A. Inhibit the xanthien oxidase
Explanation: allopurinol (a xanthine oxidase inhibitor, decreases uric acid synthesis)—if the 24 hour urine uric acid is >800 mg/day,
this indicates overproduction. Never give this for acute gout; it makes it worse. Drugs used for acute gout include NSAIDs and
colchicine.
Reference: step up to medicine 3rd edition, page 256,257



75. Management of septic arthritis mortality risk?



76. Definition of leiomyoma?
Answer : is a benign smooth muscle neoplasm that very rarely becomes cancer.
Https://en.wikipedia.org/wiki/leiomyoma


77. Major jones criteria of rheumatic fever?
Major diagnostic criteria:
Carditis, polyarthritis, chorea, erythema marginatum, subcutaneous nodules
Http://emedicine.medscape.com/article/236582-clinical
Http://reference.medscape.com/calculator/jones-criteria-diagnosis-rheumatic

78. Patient with arthritis and rash on the face. ANA is positive. What should you do next?
A. Anti DNA

Answer: A
Reference: http://www.hopkinslupus.org/lupus-tests/lupus-blood-tests/

79. A patient with SLE with rash on her cheeks, etc. What will you advise her?
A. Avoid sun exposure as much as she can
Answer: A
Reference: Medical diagnosis and Management by Mohammad Inam Danish

228
80. Characteristic finding in Behçet disease?
Answer: ?
Leukocytoclastic vasculitis, multi-system disorder presenting with ocular involvement (uveitis), recurrent oral and genital ulceration,
venous thrombosis, skin and joint involvement, more common in Mediterranean and Asia, average age 30s, M>F
Reference: Toronto Notes

81. Raynaud phenomenon. Which antibody will be positive?


• Anti-SCL-70
Answer: A
If it’s scleroderma case the answer will be A
ANA is ordered for Raynaud phenomenon, although it's not specific. The anti-centromere antibodies are ordered for CREST
syndrome. (R for Raynaud)
Reference: http://emedicine.medscape.com/article/1064663-workup

82. Henoch schonlein purpura:


• IgA Vasculitis
Answer: A

83. What is the treatment of choice for Kawasaki?


Answer: ?
Intravenous Immunoglobulins (IVIG) is mainstay of treatment + Aspirin
Reference: canadaqbank


84. Patient 3 weeks after URTI develop rash, knee pain, and hematuria. What' the Dx?
A- Henoch-Schönlein purpura ( specific type of hypersensitivity vasculitis)
Answer: A
Henoch-Schönlein purpura “IgAvasculitis” characterized by a tetrad of clinical manifestations:
●Palpable purpura in patients with neither thrombocytopenia nor coagulopathy 95-100%
●Arthritis/arthralgia “especially involving the knees and ankles” 60-84%
●Abdominal pain “usually diffuse, with acute-onset” 35-85%
●Renal disease “proteinuria, hematuria”
In one half to two thirds of children, an upper respiratory tract infection (URTI) precedes the clinical onset of HSP by 1-3 weeks.
Http://www.uptodate.com/contents/henoch-schonlein-purpura-immunoglobulin-a-vasculitis-clinical-manifestations-and-diagnosis



85. Polymyalgia Rheumatica question, what supports the diagnosis?
Pain and stiffness in shoulders and hips, often with fever, malaise, weight loss. Does not cause muscular weakness. More common in
women > 50 years old; associated with temporal (giant cell) arteritis.
Answer: The symptoms include pain and stiffness of the shoulder and hip girdle. Stiffness after periods of rest (gel phenomenon) as
well as morning stiffness of more than 1 hour typically occurs. Low-grade fever and weight loss. Malaise, fatigue, and depression.Muscle
weakness is not a feature of PMR
FINDINGS :high ESR, high CRP.
(http://emedicine.medscape.com/article/330815-clinical)



86. Typical symptoms of RA, wts the dx.
A. RA
Answer: Rheumatoid arthritis is a chronic inflammatory disorder autoimmune disorder that typically affects the small joints in your
hands and feet on both sides of your body.
Symptoms: tender swelling joint, morning stiffness last for hours, rhomatoid nodules, fever .
Complication: osteoporosis ,carpal tunel syndrom, heart proplem,lung diseas.
Blood test: high CRP, anti-cyclic citrullinate d peptide (anti-CCP) antibodies.

229
Medication:NSAIDs. Side effects ringing in your ears, stomach irritation, heart problems, and liver and kidney damage.
Steroids. Side effects may include thinning of bones, weight gain and diabetes.
Disease-modifying antirheumatic drugs (dmards).. Include methotrexate (Trexall), leflunomide (Arava), hydroxychloroquine
(Plaquenil) and sulfasalazine (Azulfidine).
Side effects vary but may include liver damage, bone marrow suppression and severe lung infections.

87. Pt with PIP, DIP swollen and painful. Tests show high uric acid. Wts the treatment:
A-allopuranol
Answer: A

88. A patient with symmetrical joint involvement, complaining of morning stiffness that is relieved with movement. MCP and PIP
are involved as well. What is the diagnosis?
• RA
Answer: A

Signs and Symptoms (Toroto Note)


• variable course of exacerbations and remissions
• morning stiffness >1 h, improves with use, increases with rest
• may have joint pain with activity
• symmetric joint involvement
• joint swelling, tender joints
• constitutional symptoms: profound fatigue; rarely myalgia or weight loss
ƒ signs of mechanical joint damage: loss of motion, instability, deformity, crepitus, joint
deformities
Š swan neck deformity, boutonni.re deformity
Š ulnar deviation of MCP, radial deviation of wrist joint
Š hammer toe, mallet toe, claw toe
Š flexion contractures
ƒ atlanto-axial and subaxial subluxation
Š C-spine instability
Š neurological impingement (long tract signs)
Š difficult/dangerous intubation: risk of worsening subluxation and damage to spinal cord
ƒ limited shoulder mobility, spontaneous tears of the rotator cuff leading to chronic spasm
ƒ tenosynovitis may cause rupture of tendons
ƒ carpal tunnel syndrome
ƒ ruptured Baker’s cyst; presentation similar to acute DVT

89. Symmetric joint pain and swelling worse at morning. Dx?


Answer: rheumatoid artheritis
Reference: http://www.webmd.com/rheumatoid-arthritis/guide/diagnosing-ra

90. Chronic Gout >allopuriol


230
Answer: Allopurinol inhibition of uric acid production is in large part due to inhibition of xanthine oxidase (xanthine dehydrogenase)
by both the native drug and the active metabolite oxypurinol. Allopurinol and oxypurinol are pyrazolo-pyrimidine analogs of the
purine bases hypoxanthine and xanthine, respectively
Ref :uptodate

91. 2 questions has the same idea with the same options about polymyalgia rheumatica (they mention that ESR was high)
Answer: PMR
Go and read: http://emedicine.medscape.com/article/330815-overview

92. Case of rheumatoid arthritis.

Answer: ??
• Rheumatoid arthritis is a chronic inflammatory condition. Its symptoms develop gradually and may include joint pain,
stiffness, and swelling. The condition can affect many tissues throughout the body, but the joints are usually most severely
affected. The cause of rheumatoid arthritis is unknown.
• A person with well-established rheumatoid arthritis typically has or has had at least several of the following:
o Morning stiffness that lasts at least one hour (> 30 minutes) and that has been present for at least six weeks
o Swelling of three or more joints for at least six weeks
o Swelling of the wrist, hand, or finger joints for at least six weeks
o Swelling of the same joints on both sides of the body
o Changes in hand x-rays that are characteristic of rheumatoid arthritis
o Rheumatoid nodules of the skin
o Blood test positive for rheumatoid factor and/or anti-citrullinated peptide/protein antibodies (ACPA)
• Not all of these features are present in people with early RA, and these problems may be present in some people with other
rheumatic conditions.
• In some cases, it may be necessary to monitor the condition over time before a diagnosis of rheumatoid arthritis can be
made with certainty.
• Laboratory tests — Laboratory tests help to confirm the presence of rheumatoid arthritis, to differentiate it from other
conditions, and to predict the likely course of the condition and its response to treatment.
o Rheumatoid factor — An antibody called rheumatoid factor is present in the blood of 70 to 80 percent of people
with rheumatoid arthritis. However, rheumatoid factor is also found in people with other types of rheumatic
disease and in a small number of healthy individuals.
o Anti-citrullinated peptide/protein antibody test — Blood tests for ACPA are more specific than rheumatoid factor
for diagnosing rheumatoid arthritis. Anti-ACPA antibody tests may be positive very early in the course of disease.
The test is positive in most patients with rheumatoid arthritis.
Reference: Uptodate

93. Case of juvenile rheumatoid/idiopathic arthritis treatment.

a. Paracetamol
b. Asprin

Answer: b

94. a patient came to you complaining of morning stiffness of the pip and dip that decreases and goes away with activity. On
x-ray you observed a bone growth. What is the name of that growth ?
A. Heberden node
Answer: ? (depends on the x-ray given)
It is a case of arthritis.

231

Osteoarthritis hand
● Cartilage loss with narrowing of interphalangeal joints
● B: bouchard nodes (osteophytes proximal interphalangeal joints)
● H: heberden nodes (osteophytes distal interphalangeal joints)

95. boutonniere deformity:
A. Flexion of pip joint & hyperextension of dip
Answer: a

232


Endocrinology

233

1. Q pt k/c of DM with uncontrolled blood sugar with figure shows high at the 6am wt to do?
A. ↑ long acting night insulin
B. ↑ short acting night insulin
C. ↑ long acting morning in...
D. ↑ short acting morning in…

Answer: a
Reference: by senior medical resident



2. Hypothyroidism patient on dose of thyroxin 75 mg. Missed the dose 2 days because he does not have the drug, lab result
show high TSH and normal t4. What dose should be taken?
A. 25
B. 50
C. 75
D. 100

Answer: c (D)

I thick we should increase the dose to 100 due to high TSH, missing the dose for 2 days is not expected to change the TSH. Normally
we see the effect of medication within 6 weeks for TSH and witin 2-4 weeks for T4. The half life of thyroxin is 1 week. Our monitoring
lab is TSH not the T4 nor T3.


3. Patient with dka he start to breath rapidly to buffer his acidosis "to get rid of" :
A. Oxygen
B. Carbon monoxide
C. Carbon dioxide
D. Nitrogen

Answer: c


4. Patient dx with dm1 which will confirm the dx:
A. Dr3
B. Dr4 ??
C. Dr7
D. Dr5
According to best practice and medscape, both dr3/dr4 are considered as risk factors (approximately 95% of patients with type 1 DM
have either hla-dr3 or hla-dr4) and are not used in diagnosing type 1 dm, but instead, insulin and c-peptide levels and
immune markers (eg, glutamic acid decarboxylase [gad] autoantibodies) are used.
References:
Http://emedicine.medscape.com/article/117739-overview#showall
Http://bestpractice.bmj.com/best-practice/monograph/25/diagnosis/tests.html
Medicine

5. Early sign of puberty in males?

234
A. Hair in the face
B. Hair in the gentiles
C. Hoarseness of voice
D. Enlarged penis
Answer: D

Females Males
• onset: age 8-13 yr old (may start as early as 7 yr in girls of African • onset: age 9-14 yr old
descent) • usual sequence
• usual sequence 1. testicular enlargement
1. thelarche: breast budding 2. penile enlargement
2. pubarche: axillary hair, body odor, mild acne 3. pubarche: axillary and facial hair, body odor, mild acne
3. growth spurt 4. growth spurt: occurs later in boys
4. menarche: mean age 12.5 yr; indicates that growth spurt is almost • early puberty is uncommon (rule out organic causes), late puberty is
complete; menses may be irregular in duration and length of cycle common and often constitutional
• early puberty is common and often constitutional, late puberty is rare • gynecomastia (transient development of breast tissue) is a common
(rule out organic causes) self-limited condition seen in 50% of males during puberty (but any
discharge from nipple or fixed mass should be investigated)
Toronto Note

6. Difference between type 1 and type 2 dm?


A. CPR
B. Level of insulin
C. Insulin growth factor
D. Islet of b cell
Answer: b
DM 1: beta cells in pancreas are attacked by the body's own immune system, therefore reducing insulin production, leading to elevated
blood glucose. Insulin is not produced or is produced in insufficient amounts
DM 2: persistently high intakes of dietary sugars lead to excess demands on insulin production, which leads to insulin resistance over
time. Receptor cells that have become less sensitive (resistant) to insulin are unable to remove glucose from the blood, leading to higher
blood glucose and greater demands on insulin production.

7. Pt for annual checkup everything is normal. Normal blood glucose. What should be the next checkup?
A. 3 month
B. 6 month
C. 12 month
D. 36 month
Answer: d
The american diabetes association (ada) recommends testing at three-year intervals for diabetes or prediabetes in all adults with bmi
≥25 kg/m2 and one or more additional risk factors for diabetes using either a1c, fasting plasma glucose, or 2-hour ogtt [2]. In
individuals without risk factors, the ada recommends that testing begin at age 45 years.

8. A 30-year-old male, diabetic otherwise healthy with persistent one hour post prandial hyperglycemia after lunch and dinner:
A. Milk
B. Meat
C. Diet cola
D. Potato
Answer: d
the glycemic index of a food refers to the speed with which the food raises blood glucose level.
Http://static.diabetesselfmanagement.com/pdfs/pdf_2102.pdf

9. Diabetic and hypertensive, was given a drug then experienced cough, improved after changing the drug to
hydrochlorothiazide what to add:
A. Atenolol
B. Amlodipine
C. Losartan

235
D. ACEI (i forgot the name)
Answer: c

10. Myasthenia gravis on neostigmine and case worsen?


A. Add more neostigmine
B. Add revistagmine
C. Add azthropitine
D. Plasmapheresis or hemodialysis
Answer: ??
pyridostigimine is preferred over neostigmine. Due to CNS permability
the answer depends on the scenario, if MG crisis answer is D

11. What is maximum normal 2 h postprandial blood glucose?


A. 8
B. 9
C. 10
D. 11
Answer: a

12. Cushing case which skin manifestation is associated with it


A. Vitiligo
B. Telangiectasia
C. Acropathy
D. Something derma

Answer: b
Skin manifestations of cushing:
Easy bruising , purple striae
Telangiectatic cheeks
Fragile skin and poor wound healing
Acne and hirsutism
Reference: http://www.dermnetnz.org/systemic/cushings.html

13. Elderly patient presented with chest pain palpitation and SVT with normal vital signs including o2 sat. Air entry was bilateral
which of the following is most helpful in establishing the diagnosis:
A. Pulmonary arteriography
B. Chest CT
C. TSH
D. Brain mri

Answer: c

14. Patient with hard palpable nodule in left thyroid. Fna taken and show amorphous intercellular substance & by special stain
there is amyloid deposition.
A. NHL.
B. Follicular cancer.
C. Papillary cancer
D. Medullary carcinoma
Answer: d

15. Case of hyperthyroidism clear but with bradycardia


A. Hypothyroid
B. Hyperthyroidism
C. Goiter
236
D. Neoplasm
Answer: b

16. What is the preferable imaging to diagnose Pituitary Microadenoma?


A. Contrast with enhanced brain CT
B. Brain CT
C. Contrast with enhanced brain MRI
D. Unenhanced brain MRI

Answer: C and D are correct


Reference: uptodate , Usmle 2 first aid

17. Yrs old girl her parent have dyslipedimia and she denies having dyslipidemia. Lpid profile
showing high triglyc, cholestrol, high LDL, low HDL what is the best to check next ?
A. GH
B. TSH
C. FSH
D. ACTH

Answer: b
TSH to rule out hypothyroidism
Reference: http://www.uptodate.com/contents/lipid-abnormalities-in-thyroid-disease
Tests for secondary causes of dyslipidemia—including measurements of fasting glucose, liver enzymes, creatinine,
thyroid-stimulating hormone (TSH), and urinary protein—should be done in most patients with newly diagnosed
dyslipidemia and when a component of the lipid profile has inexplicably changed for the worse.
Mearckmanual.

18. Se female irritable + loss of wt+ bradycardia?


A. Hypothyroid
B. Hyperthyroid
C. Nodule
D. Thyroid cancer ( i chose this because hyperthyroidism comes with afib) < their original answer
Reference: toronto notes


19. Diabetic patient who is allergic to sulfa drugs, on metformin but it’s not controlled. What will you add to control his diabetes?
A. Chlorpropamide
B. Glyburide
C. Rosiglitazone

Answer: c or gliptin?
Gliptin (dpp-4 inhibitor) is a better choice (answered by an endocrine consultant).
Chlorpropamide (sulfonylurea)
Glyburide (sulfonylurea)
Rosiglitazone (thiazolidinedione)

20. One small thyroid nodule, we do investigation and its iodine uptake increases, what is the best treatment?
A. Conservative
B. Antithyroid drug
C. Iodine radiotherapy

Answer: a if it is (hot + small and the patient is asymptomatic).


If the hot nodule is large in size or causing clinical hyperthyroidism, then treatment is started. Treatment may be by surgery
(especially when there is compressive symptoms) or radioactive ablation
Reference: http://www.mythyroid.com/hotnodule.html

237
• Any nodule >1 cm should undergo FNA
• If a thyroid nodule isn't cancerous, not functional,
1- Watchful waiting with regular investigations and biopsy if need it
2- Thyroid hormone suppression therapy (levothyroxine)
3- Surgery. if it's so large that it makes it hard to breathe or swallow.

• Treating nodules that cause hyperthyroidis
o Radioactive iodine.
o Anti-thyroid medications
o Surgery

21. 70 years old smoker woman with low vit-d and osteoporosis. Which of the following has the highest risk for osteoporosis?
A. Smoking
B. Age
C. Vitamin d

Answer: b


22. Patient k/c of DM on glipizide want to go for elective surgery. You want to control his blood sugar during the surgery, what to
add?
A. Insulin
B. Metformin
C. Sulfonylurea

Answer: a
Patients with type 2 diabetes that take oral hypoglycemic drugs or non-insulin injectable are advised to hold their oral hypoglycemic
and noninsulin injectable drugs on the morning of surgery. For patients who develop hyperglycemia, supplemental short or rapid-
acting insulin may be administered subcutaneously (typically every six hours), based on frequently (every one to two hours)
measured glucose levels, which are often obtained on capillary "fingerstick" samples.
Reference: http://www.uptodate.com/contents/perioperative-management-of-blood-glucose-in-adults-with-diabetes-
mellitus?Source=preview&search=%2fcontents%2fsearch&anchor=h20#h20

238


23. Patient with 3 reading of high Blood pressure . All investigations are normal except high na. What’s the diagnosis?
A. Primary hyperaldosteronism
B. Essential hypertension
C. Secondary hypertension

Answer: a
Mild serum hypernatremia in the 143-147 meq/l range and mild hypomagnesemia from renal magnesium wasting are other
associated biochemical findings in established primary aldosteronism.
Reference: http://emedicine.medscape.com/article/127080-workup#c8


24. DM pt . Which of the following increase risk of retinopathy :
A. HTN + obesity
B. HTN + smoking
C. Dylipidemia + obesity

Answer: B
Anyone who has diabetes can develop diabetic retinopathy. Risk of developing the eye condition can increase
as a result of:
o Duration of diabetes — the longer you have diabetes, the greater your risk of developing diabetic
retinopathy
o Poor control of your blood sugar level
o High blood pressure
o High cholesterol
o Pregnancy
o Tobacco use
o Being black, Hispanic or Native American

Reference; MyoClinic
25. Which of the following drug may presenting with hypoglycemic attack?
A. Metformin
B. Glitazone
C. Glyburide
Answer: c
Http://www.webmd.com/diabetes/guide/diabetes-hypoglycemia#2

26. Patient on metformin, lab values given with low ph, normal urine osmolality, no ketones ;
A. DKA
B. Hyperismolar hyperglycemia
C. Lactic acidosis
Answer: c
Metformin side effects: lactic acidosis, it is rare but dengerous. Http://reference.medscape.com/drug/glucophage-metformin-342717#4
Long term treatment to avoid dependence

27. Controlled DM what make an abnormal post prandial glucose


A. Diet cola
B. Meat
C. Potato
Answer: c

239
28. What is the optimal treatment of osteoporosis?
A. Calcium, vitamin d and bisphosphonate
B. Vitamin d and calcium
C. Calcium and bisphosphonate
Answer: a
The goals of treatment are to prevent fractures, decrease pain, and maintain function which is achieved by this combination that mainly
helps preserving and building bone mass.
Http://www.merckmanuals.com/professional/musculoskeletal-and-connective-tissue-disorders/osteoporosis/osteoporosis#v906887

29. Patient heard about the new anti-DM medication “incretin” on tv, asking about the mechanism of action of the new drug?
A. Inhibit liver gluconeogenesis
B. Increases the insulin secretion
C. Decreases insulin resistance
Answer: b
The incretins are peptide hormones secreted from the gut in response to food. They increase the secretion of insulin. The incretin
response is reduced in patients with type 2 diabetes, so drugs acting on incretins may improve glycaemic control.
The two most important incretin hormones are called glucagon-like peptide-1 (glp-1) and glucose-dependent insulin tropic polypeptide
(gip).
http://www.diabetesselfmanagement.com/diabetes-resources/definitions/incretin-hormone/

30. 46 Year-old came to the clinic with her 1w old baby, complaining of (symptoms of hyperthyroidism recently developed):
A. Hypothyroidism
B. Postpartum thyroiditis
C. Thyroid tumor (i can't remember the other choices)
Answer: b
Postpartum thyroiditis (ppt) reportedly affects 4-10% of women. Ppatient is an autoimmune thyroid disease that occurs during the first
year after delivery. Women with ppatient present with transient thyrotoxicosis, hypothyroidism, or transient thyrotoxicosis followed
by hypothyroidism.
Link: http://emedicine.medscape.com/article/261913-overview

31. 10 or 12 yo girl diabetic for 3 years. Which of the following should be checked annually?
A. Ophthalmoscopy
B. Celiac test
C. Kidney ultrasound
Answer: a

32. Patient has DM & HTN (i think on captopril which induce dry cough) present with mild ankle edema and positive protein in
urine. What will you give instead of ace inhibitors? (not sure about this question)
A. ARB
B. Thiazide
C. CCB
Answer: a

33. 30yo with high cholesterol what’s the most common gene defect.
A. Apo cii
B. Apo b100
C. Ldl receptor (rt)
Answer: c
Familial hypercholesterolemia (fh) is a genetically modulated clinical syndrome in which the phenotype is characterized by a high low
density lipoprotein cholesterol (ldl-c) level from birth, a propensity to tendon xanthomata, and early onset coronary heart disease
(chd). In its most common form, fh is a monogenic, autosomal dominant disorder caused by defects in the gene that encode for the apo
b/e (ldl) receptor.

240
34. Diabetes mellitus type 1 patients express what mutation:
A. HLA-DR4
B. HLA-DR 6
C. HLA-DR 7
Answer: a
Reference: http://www.merckmanuals.com/professional/endocrine-and-metabolic-disorders/diabetes-mellitus-and-disorders-of-
carbohydrate-metabolism/diabetes-mellitus-dm


35. Diabetic with painful back swelling has multiple discharges:
A. Cellulitis
B. Lymphoid
C. Something lymphadenitis carbuncle
Answer: a
Patients with uncontrolled diabetes are predisposed to multiple bacterial infections like abscesses, carbuncles, chronic balanitis, and
diabetic foot

36. Patient with swelling in the thyroid, she is euthyroid what will you do ?
A. Thyroid lobectomy
B. Fna biopsy
C. Excisional biopsy
Answer: b

37. Patient with hba1c within pre diabetic range .when to repeat the test ? –
A. 3months
B. 6 months
C. 1 year
Answer: c.
An A1C level ≥6.5 percent be used to diagnose diabetes. – uptodate
Those who are normal should be re-screened every 3 years.

38. Hypertension, high k in urine, high serum na, low serum k.


A. Cushing
B. Conn's syndrome
C. Hypothyroidism

Answer: b

Primary aldosteronism (conn's syndrome) is aldosteronism caused by autonomous production of aldosterone by the adrenal cortex
(due to hyperplasia, adenoma, or carcinoma). Symptoms and signs include hypernatremia and hypervolemia episodic weakness,
elevated Blood pressure , and hypokalemia.
Reference: merck manual: http://www.merckmanuals.com/professional/endocrine-and-metabolic-disorders/adrenal-
disorders/primary-aldosteronism

39. In dka which ketone is predominantly found in urine?


A. Acetone
B. Acetoacetate
C. Beta-hydroxybutyrate

Answer: b, dipstick detect acetoacetate but not beta-hydroxybutyrate


References
Http://fitsweb.uchc.edu/student/selectives/timurgraham/ketoacidosis_dka.html
Http://patient.info/doctor/urine-dipstick-analysis

241
40. Female presents with weight loss …. What clinical finding you will see?
A. Buffalo hump
B. Skin hyperpigmentations
C. Cutaneous stria
Answer : b (addison disease)

41. Why DM decrease wound healing
A. Decrease immunity
B. Increase bacteria due to increase glucose
C. Decrease phagocytosis
Answer: c i think the best answer is a.

Uptodate: over 100 known cytologic factors contribute to impaired wound healing in patients with diabetes.these include decreased or
impaired growth factor production, angiogenic response, macrophage function, collagen accumulation, epidermal barrier function,
quantity of granulation tissue, keratinocyte and fibroblast migration and proliferation, number of epidermal nerves, bone healing, and
abnormal balance between the accumulation of extracellular matrix components and their remodeling by matrix metalloproteinases.
Diabetes is a particularly important risk factor for the development of chronic wounds because it is associated with vasculopathy,
neuropathy and immunopathy.

42. Case a guy has high phosphate question organ phosphate elevated which organ?
A. Liver
B. Kidney
C. Lung
Answer:

43. What causes polyuria in DM?


A. Increase glucose in urine
B. Increase glucose in serum
C. Increase ketones in serum

Answer: A

44. 52 years old woman recently diagnosed to have DM, came with high ketones and
hyperglycemia. She was treated for it but she developed DKA again. The doctor is confused
whether she has type 1 or 2 DM. What test should be ordered?
A. Insulin
B. Hba1c
C. C-peptide

Answer: C
C-peptide is commonly used in preference to insulin measurement when assessing β-cell function in clinical practice. In patients on
insulin, C-peptide measurement must be used as exogenous insulin will be detected by insulin assays.
Reference: http://www.ncbi.nlm.nih.gov/pmc/articles/PMC3748788/

45. What is the best anti-diabetic regime that resembles the normal physiology?
A. Lispro & glargine

242
B. NPH & glargine
C. Lispro & NPH

Answer: A
Glargine is better than NPH

Single injection of insulin glargine leads to a smooth 24-hour time–action profile with no undesirable pronounced peaks of activity. In
clinical trials, this profile has been associated with at least equivalent, if not better, glycemic control than other traditional basal
insulins and a significantly lower rate of overall and nocturnal hypoglycemia.
Reference: http://www.ncbi.nlm.nih.gov/pmc/articles/PMC1993975/

46. A patient with signs and symptoms of Cushing syndrome. What is the best next investigation?
A. Brain MRI
B. Adrenal scan
C. Adrenal MRI

Answer: A
In adults, 80% of CS is due to ACTH-dependent causes and 20% due to adrenal causes. Since the majority of patients with ACTH-
secreting tumors have a pituitary lesion (often very small), a MRI of the pituitary gland with gadolinium enhancement is always the
initial approach.
References:
Www.ncbi.nlm.nih.gov/pubmed/18209870
Https://csrf.net/understanding-cushings/diagnostic-testing/

47. Patient with DM and HTN controlled on ace , frusemide, spironolactone ,, elctrolyte balance is normal what is the action
should do:
A. Stop frusemide
B. Stop spironolactone
C. Add digoxin.

48. 25 Year-oldwoman came to the clinic with her 6 weeks old baby, complaining of irritability, weight loss, and inability to
sleep?
A. Post partum thyroditis *
B. Hyperthyroidism
C. Hashimoto thyroditis
Reference: toronto notes

243
49. Response glycolysis?
A. Hyxokinase
B. Phsophphrktokinase
C. Pyrovate kienzyme nase



50. Acetone + high glucose?
A. Dka
B. Hyperosmolar

Answer: a
Diabetic ketoacidosis is typically characterized by hyperglycemia over 250 mg/dl, a bicarbonate level less than 18 meq/l, and a ph less
than 7.30, with ketonemia and ketonuria.
Reference: http://emedicine.medscape.com/article/118361-workup


51. 70 years old female patient with osteoporosis what is the treatment?
A. Estrogen
B. Biophosphate

Answer: b
Diagnosed by dexa. Bisphosphonates are first-line drug therapy.
References: 1st aid page (320) +
Http://www.msdmanuals.com/professional/musculoskeletal-and-connective-tissue-disorders/osteoporosis/osteoporosis
Bisphosphonates (e.g. Alendronate) is first line
HRT: second-line treatment (unless for vasomotor instability as well)
reference: toronto notes 2015 p 516

52. Patient is concerned about osteoporosis as her mother had it, what you will do?
A. Give Vitamin d, calcium
B. Give estrogen postmenopausal

Answer: a
Prevention and treatment of osteoporosis by UpToDate:
o Calcium and vitamin D (premenopaus or young man (less than 70 year-old) 1000 mg Ca + 600 IU vit D per day,
postmenopaus women or elderly men 1200 mg Ca + 800 IU vit D)
o Exercise should be done for at least 30 minutes three times per week.
o Stopping smoking
o Some medications can cause bone thinning. Such as glucocorticoid, heparin, phenytoin, carbamazepine, primidone, and
phenobarbital.
For treatment:
o Alendronate or risedronate are recommended to TREAT women after menopause who have osteoporosis.

244
o Zoledronic acid (given once a year IV), for patients who cannot tolerate oral bisphosphonates, or who have difficulty taking
the medication, including an inability to sit upright for 30 to 60 minutes.
o Raloxifene is generally less effective than other medications and is usually recommended for women who cannot tolerate
or are not candidates for any bisphosphonates or for postmenopausal women with osteoporosis who are also at high risk
for invasive breast cancer.
o Denosumab is another option for patients who are intolerant or unresponsive to bisphosphonates.
o Parathyroid hormone (PTH) teriparatide
o Hormone replacement (eg, estrogen, progesterone) is NOT usually recommended to prevent osteoporosis in women after
menopause.


53. Female patient with high prolactin, what to exclude?
A. Thyroid disease
B. Pituitary tumors

Answer: b
Causes of hyperprolactinemia:
- Prolactinoma
o Most common cause of hyperprolactinemia
o Most common type of pituitary adenoma (up to 40%)
- Medications (e.g., psychiatric medications, h2 blockers, metoclopramide, verapamil, Estrogen)
- Pregnancy
- Renal failure
- Hypothyroidism
Reference: step up to medicine

54. DM patient with dka what to give?


A. 1l of normal saline.
B. Dextrose 50% followed by insulin.
Answer: a
Explanation: managing DKA in an ICU during the first 24-48 hours always is advisable. When treating patients with DKA, the
following points must be considered and closely monitored:
• Correction of fluid loss with intravenous fluids (first thing to be start with)
• Correction of hyperglycemia with insulin
• Correction of electrolyte disturbances, particularly potassium loss
• Correction of acid-base balance
• Treatment of concurrent infection, if present
Reference: http://emedicine.medscape.com/article/118361-treatment

• Bicarbonate rarely used (only if ph is <6.9)

55. Diabetic man with uncontrolled blood sugar he is on metformin and pramlintide, what drug you will add or change ( i don't
remember the exact question )
A. Repaglinide
B. Glitazone
Answer:

56. Thyroid move with swallowing because:


A. Pretracheal fscia
B. Carotid sheath
Answer: A
Condensation of the pretracheal fascia called the posterior suspensory ligament of berry. The thyroid gland is invested in a
sheath derived from the pretracheal fascia. so the thyroid follows the movements of the larynx during swallowing. Any
pathological swelling of the thyroid will move upwards with swallowing, distinguishing it from a mass in some other part of
the neck.
245



57. Patient presented with acne , upper body obesity and hypertension ,what is the test to diagnose this patient ?
A. Anabolic stress test
B. Urine metanephrines
Answer: b
Not complete question: might be a case of phaeochromocytoma with cushing's syndrome and first test to do is 24hrs urine
metanephrines
Or may be a cushing’s syndrome case

58. What’s the difference between type 1 and type 2 dm?


A. Endogenous insulin
B. Weight
Answer: a




59. A young female patient complains of heat intolerance and excessive sweating. On examination, neck swelling is felt. TSH normal
and fna done but not conclusive. What is your next step :
A. Repeat fna
B. Thyroidectomy
Answer: A
Nondiagnostic FNA (risk of cancer 1- 4%) — A nondiagnostic biopsy is cytologically inadequate. We should repeat the FNA in 4 to 6
weeks, using ultrasound guidance if not used for the first FNA.
If repeated ultrasound-guided aspirations are nondiagnostic, ultrasound-guided core needle biopsy should be considered.
Surgical excision, especially for larger solid nodules with sonographically suspicious features, or observation, for smaller partially
cystic nodules, are reasonable options for repeatedly nondiagnostic biopsies. If nodule is growth diagnostic surgery is needed.
UpToDate

60. Female DM on metformin and sulfonylurea can’t tolerate them what you will do?
A. Insulin
B. Glitazone
Answer: a ?

246
61. A history of missed insulin was approved from the pt, what do you suspect to find in urine?
A. Ketone
B. Protein
Answer: a
Http://emedicine.medscape.com/article/118361-workup#c10

62. Patient presented with severe vomiting. His labs showed hypocalcaemia. What is your management?
A. Furosemide
B. Hydration
Answer:
If hypo! Iv ca gluconate
If hyper iv fluid


63. Hx and investigation all normal exce hyponatremia and crepitation on ex. What would you give?
A. Iv ns
B. Furosemide
Appears to be SIADH due to pulmonary disease. So the most appropriate answer is probably treat the underline cause.
o Hypertonic salin IV, if there is severe neurologicl symptoms or coma.
o Fluid restriction.
o Furosomid, may be used in addition to hypertonic saline, especially if the patient is at risk for volume overload.
o Conivaptan is a non-selective vasopressin receptor antagonist and affects both V1 and V2 receptors, or, Tolvaptan a
selective V2 receptor antagonist. We use these If the patient is intolerant to fluid restriction.

64. Consultation .patient with hypercholsterima. What best food to avoid?


A. Organ meats
B. Egg white (protein)

247
Answer: a
Eating too much saturated fat increases cholesterol levels. This is why it is best to limit the amount of foods we eat that are high in
saturated fats such as:
- Butter
- Ghee
- Hard margarines
- Lard, dripping and goose fat
- Fatty meat and meat products such as sausages
- Full fat cheese, milk, cream and yogurt
- Coconut and palm oils and coconut cream
Additionally, many foods such as milk chocolate, toffee, cakes, puddings, pastries, pies and rich biscuits, which are made from the list
above can also increase cholesterol.

65. Male is a k/c hyperthyroidism on medication, has infection. His labs: low neutrophil.
What drug has these se?
A. Methimazole
B. Iodate
Answer: a
Http://www.uptodate.com/contents/antithyroid-drugs-beyond-the-basics?Source=outline_link&view=text

66. Diabetic patient went for surgery; he was given insulin and dextrose. Then developed neurological symptoms (low na). What
is the mechanism?
A. Water overload.
B. Siadh.
Answer: incomplete q the two pictures explain the possible answers

Reference: pics and toronto note

67. DM on metformin and gliclazide .. She is not well controlled .. Which drug u will add ?
A. Acarbose
B. Pioglitazone
Answer: b

68. A patient presented with high parathyroid hormone and high calcium what is the diagnosis?
A. Primary parathyroid
B. Secondary parathyroid

Answer: A
High ca + high PTH = primary hyperparathyroid
Reference: medlineplus medical encyclopedia

248
69. Patient with type 1 DM, what's the diagnostic antibody?
A. Glutamic Acid Decarboxylase (GAD65)
B. Others antibodies not related to DM1

Answer: A
Reference: Islet-cell (IA2), anti-GAD65, zinc transporter 8 (ZnT8). and anti-insulin autoantibodies can be present in early type 1 but
not type 2 DM. Measurements of IA2 autoantibodies within 6 months of diagnosis can help differentiate between type 1 and type 2
DM. These titers decrease after 6 months. Anti-GAD65 antibodies can be present at diagnosis of type 1 DM and are persistently
positive over time
Http://emedicine.medscape.com/article/117739-workup#c8

70. Which dietary supply prevent cancer


A. Fibers
B. Vitamin d

- Fiber (this is the only well know answer)


- Resistant starch
- Folic acid and folate (this is the chois of SLE made easy)
- Vitamin B6 (pyridoxine)
- Calcium and dairy products (This is the choice of BMJ best practice)
- Vitamin D (this is some refrence choice)
- Magnesium
- Garlic
- Fish consumption

Aspitin is well known drugs prevent colorectal cancer.

71. Bond between t-RNA and mRNA.


A. Answer: hydrogen
Http://depts.washington.edu/hhmibio/translationstudyguide.pdf

72. Symptoms of hyperthyroid wh present ?


A. Retraction led
B. Prietabial myxedema

Answer: A

73. X-ray of lower back , inv shows increase calicium , what to do :
A. Dexa
B. Parathyroid
Answer: b
Explanation: clinical syndrome of primary hyperparathyroidism can be easily remembered as bones, renal stones, abdominal groans,
and psychic moans. Hypercalcemia should be documented on more than one occasion before a diagnostic workup is undertaken.
Testing of the intact parathyroid hormone level is the core of the diagnosis. Reference:
http://emedicine.medscape.com/article/127351-overview#a1

74. Sore throat and pancytopenia in women taking antithyroid , which one ?
A. Methimazole
B. Propranolol
Answer: Methimazole .


75. Patient has osteoporosis on ca supplement and vitamin d, but has no improvement why?
A. Hypomagnesemia
Answer: a

249
- Magnesium deficiency contributes to osteoporosis directly by acting on crystal formation and on bone cells and indirectly by
impacting on the secretion and the activity of parathyroid hormone and by promoting low grade inflammation
- Http://www.mdpi.com/2072-6643/5/8/3022/htm

76. Pharmacodynamics of metformin, stimulation or inhibition of which enzyme?


Answer: metformin activate ampk and inhibit glycerophosphate dehydrogenase
Metformin decreases blood glucose levels by 3 mechanisms:
- Decreasing hepatic glucose production (gluconeogenesis).
- Decreasing intestinal absorption of glucose.
- Improving insulin sensitivity by increasing peripheral glucose uptake and utilization.
Metformin works by activating an enzyme called amp-activated protein kinase (ampk), that in turn signals the liver to produce
less glucose (inhibits gluconeogenesis) and encourages cells to store glucose from the bloodstream thus lowering blood glucose levels.
Also, metformin non-competitively inhibits the redox shuttle enzyme mitochondrial glycerophosphate dehydrogenase, resulting in an
altered hepatocellular redox state, reduced conversion of lactate and glycerol to glucose, and decreased hepatic gluconeogenesis.

77. In dka what is found in urine:


Glucose and ketones (acetoacetate).
Medscape. Http://emedicine.medscape.com/article/118361-workup

78. The cause of fracture in osteoporosis is due to:


Answer: vitamin d deficiency
Vitamin deficiency is not a direct cause rather it is a risk factor that predisposes to fractures due to its effect on bone mineralization and
its neuromuscular support preventing falls.
Http://cursoenarm.net/uptodate/contents/mobipreview.htm?32/55/33649?Source=see_link
250
Http://emedicine.medscape.com/article/330598-overview#a3

79. Male got sting and ignored, develop sob + itching when he went to hospital, collapsed at the taxi, what to give?
Answer: epinephrine
Reference: http://emedicine.medscape.com/article/769067-treatment

80. Osteoporosis patient had bone density scan dxa (-2)?


T score — this number shows the amount of bone you have compared with a young adult of the same gender with peak bone
mass. A score above -1 is considered normal. A score between -1 and -2.5 is classified as osteopenia (low bone mass). A score
below -2.5 is defined as osteoporosis. The t score is used to estimate your risk of developing a fracture.

Reference: http://www.radiologyinfo.org/en/info.cfm?Pg=dexa

a. Z-score, the number of standard deviations above or below the mean for the patient's age, sex and ethnicity
b. T-score, the number of standard deviations above or below the mean for a healthy 30-year-old adult of the same sex and
ethnicity as the patient

81. Rapid treatment of hyperthyroid


A. Beta blockers
Answer:a
• β-blockers often necessary for symptomatic treatment prior to definitive therapy (tornto note)

82. DM patient presented with hx of itching and rash like lesions with white center in inguinal region but with sparing of the
folds (picture) what is the cause??
A. Candida
Answer: a
Candidiasis or dermatophytosis may underlie pruritus in diabetic patients. Anogenital pruritus is often caused by candidiasis in
diabetic patients
Link: http://www.idb.hr/diabetologia/02no3-2.pdf

83. Man his wife diagnosed with osteoporosis, he has history of recurrent renal stone, labs showed can’t remember?!! What is
the diagnosis?

May be hyperparathyroidism lead to osteoporosis with hypercalcemia lead to kidney stone (the effect of PTH).

84. Obese patient diagnosed with diabetes type 2, in addition to lifestyle modification which drug you want to add?
Answer: metformin

85. Patient with hx of dyslipidemia on statins lipid profile is normal which drug would you add to the medications?
They mentioned the other medications of dyslipidemia
Not sure i understand this

86. Drug cause hypoglacemia?


And they mentioned some types of antideibetics, i think sulfynelaurea.

87. Patient with high cholesterol on treatment, has muscles aches what was she given:
Answer: statins

88. Patient diabetic for years and was just dx as htn, what to give:
Answer: ACEI best
251
Thus, our overall approach in diabetic patients who require antihypertensive therapy is as follows:

In patients with severely increased albuminuria (formerly called "macroalbuminuria"), we treat with an ACE inhibitor or an ARB. We
also use these drugs in patients with moderately increased albuminuria (formerly called "microalbuminuria").

89. Calculation of bmi and what degree pf obesity


• Underweight – bmi <18.5 .
• Normal weight – bmi ≥18.5 to 24.9.
• Overweight – bmi ≥25.0 to 29.9.
• Obesity – bmi ≥30.
• Obesity class i – bmi of 30.0 to 34.9.
• Obesity class ii – bmi of 35.0 to 39.9.
• Obesity class iii – bmi ≥40 kg/m2. This type of obesity is also referred to as severe, extreme, or massive or morbide obesity.
Source: http://www.uptodate.com/contents/obesity-in-adults-prevalence-screening-and-
evaluation?Source=search_result&search=bmi&selectedtitle=1%7e150#h7

90. Patient diagnosed as HTN and started meds came back with high glucose. Wt was he given:
Answer: thiazides

Thiazide diuretics
Mode of action
Inhibition of sodium and chloride reabsorption in the distal tubule of the kidney, resulting in increased urinary excretion of sodium
and water. Direct arteriolar vasodilation.
Contraindications
Known hypersensitivity to thiazides or other sulfonamides. Anuria.
Adverse effects
Electrolyte abnormalities, including hypokalemia, hypomagnesemia, hyponatremia, and hypercalcemia may occur. Elevated blood
glucose levels have also been reported. Hyperuricemia is possible, therefore use with caution in patients who suffer from gout.
Arrhythmias may be precipitated secondary to electrolyte abnormalities. Hyperlipidemia (increase in total cholesterol, triglycerides,
and ldl cholesterol) has occurred. Dermatologic side effects include photosensitivity and an SLE-like syndrome.
Source: http://www.medscape.com/viewarticle/421426_1

91. Patient diagnosed DM and compliant on medication but he complained of hypoglycemia more than 3 times. Cause?
Answer: honeymoon phase
Http://www.diabetes.co.uk/blood-glucose/honeymoon-phase.html

92. Primary hyperparathyroidism what will be the lab results:


a. High PTH, hypercalcemia, hypophosphatemia, hypercalciuria, and urine camp is elevated.
b. Chloride/phosphorus ratio of >33 is diagnostic of primary hyperparathyroidism (33-to-1 rule). Chloride is high secondary
to renal bicarbonate wasting (direct effect of PTH).
Reference: step up to medicine

93. Papillary thyroid ca mostly associated with which of the following ?


A. Hürthle cell

Answer: a
most common thyroid malignancy (80%)
radiation exposure
lymphatic spread to cervical nodes

252
prognosis: excellent- slow growth. 20 years survival
tx: surgery ressection or radiotherapy with to I131 if mts.
Histology: papillary pattern, psammoma bodies, clear nuclei “orphan annie eyes nuclei”, intranuclear cytoplasmatic inclusions.

Medscape:
another subclassification of hürthle cell neoplasms has been proposed, namely the papillary variant of hürthle cell cancer (ie, hürthle
cell papillary thyroid carcinoma), in addition to hürthle cell cancer and adenoma. Clinically, tumors in this group tend to behave like
papillary thyroid carcinoma; however, they are more indolent, with a propensity for lymph node metastasis rather than hematogenous
spread. Hürthle cell carcinoma is a variant of papillary cell carcinoma.

94. Patient with constipation increase weight, thinning of hair ?


A. Hypothyroidism
Answer: a


95. Diabetic patient, what type of carbohydrates is recommended?

Answer: polysaccharide (complex)


Reference: dietitian

96. Earliest sign of hypophosphatemia


• Symptoms of hypophosphatemia are nonspecific and highly dependent on cause, duration, and severity. Mild
hypophosphatemia (ie, 2-2.5 mg/dl), whether acute or chronic, is generally asymptomatic. Patients with severe and/or
chronic hypophosphatemia are more likely to be symptomatic. Weakness, bone pain, rhabdomyolysis, and altered mental
status are the most common presenting features of persons with symptomatic hypophosphatemia.
Reference: medscape: http://emedicine.medscape.com/article/242280-clinical

97. Which type of thyroid cancer will responds to radioactive therapy?

• Papillary (as adjuvant), follicular, and anaplastic carcinoma respond to radioactive therapy. (for recurrent or metastatic
cancer)

Reference: step up to medicine.




98. Old with osteoporosis risk scenario, has spine compressive fracture, what to give her to protect?
A. Bisphosphonate, calcium, Vitamin d other
Answer:
Http://www.m.webmd.com/a-to-z-guides/spinal-compression-fractures-preventing


99. Follow up of blood lipids and follow up of blood sugar.
Answer:
Http://www.e-mercy.com/images/cholesterol-metabolism/research/2/guidelines-to-interpret-results-from-health-screening.pdf
- Obtaining a fasting lipid profile is recommended for all adults aged ≥20 years; this can be repeated every 5 years.
- DM screen oportonisitic and if normal repeated every 3 years.

100.High triglyceride what to add to statins?


Answer: Fibrate
HMG COA reductase inhibitor medications (statins) are most effective in lowering LDL cholesterol, mildly effective in increasing
HDL cholesterol, and mildly effective in lowering triglycerides.
Fibric acid such as Gemfibrozil and Fenofibrate are most effective in lowering triglycerides, effective in increasing HDL, and
minimally effective in lowering LDL levels.
Nicotinic acid (niacin), is most effective in increasing HDL, effective in lowering triglycerides, and mildly to moderately effective in
lowering LDL levels

253
Http://www.emedicinehealth.com/script/main/mobileart-emh.asp?Articlekey=114550&page=7

101.Patient with fracture, constipation and abdominal pain, high serum ca:
A. Hyperparathyroidism
Answer: a
Source: master the boards usmle step 2 ck

102.Patient use anti-hyperlipidemia, liver enzymes normal, creatin kinase(ck) high:


A. Statins
One of the side effects of statins is myositis which raise ck, but patietns are not followed for that. They who use it are followed
regularly with LFTto detect any liver damage early which is a common se of the drug. (case files family medicine hyperlipidemia
chapter)

103.Which of the following drug from biphosphonate work as atp analog to suppers osteoclasts? All the medication given end
with dronate ...........
(answer: clodronate + etidronate+ tiludronate )

104.DM patient on medication present with weakness & dizziness, what the drug he is taking?!
A. Sulfonylurea.
Answer: a
Medscape

105.Hormone that helps in increase body sensitivity to insulin:


A. Leptin.
Answer: a


106.Cushing syndrome with fracture, because of :
A. Osteoporosis
Explanation: signs of cushing : central obesity, round face, supraclavicular and dorsal fat pads, facial plethora, proximal muscle
wasting, purple abdominal striae, skin atrophy, acanthosis nigricans, htn, hyperglycemia, osteoporosis, pathologic fractures
Reference: toronto notes 2015, page e33, endocrinology

107.An asthmatic patient with no hx of DM in the family, now his bs is within normal range when to check again:
A. 3 years if the patient did not have any risk factor

254
Reference: http://guidelines.diabetes.ca/browse/chapter4


108.Vitamine c deficiency which affected in stages of wound healing ?
A. Collagen synthesis
Vitamin c is important in the wound repair process, facilitating the building of collagen in the wound, which forms the framework for
the building of new tissue .
Answer:a
Reference: http://www.surgerysupplements.com/the-role-of-vitamin-c-in-wound-healing/
Reference : http://emedicine.medscape.com/article/1173204-clinical

109.Administering pyridoxine and supportive care?


Pyridoxine used as vitamin b6 dietary supplement. The classic clinical syndrome for vitamin b6 deficiency is a seborrhoeic dermatitis-
like eruption, atrophic glossitis with ulceration, angular cheilitis, conjunctivitis,intertrigo, and neurologic symptoms of somnolence,
confusion, and neuropathy. The elderly and alcoholics have an increased risk of vitamin b6 deficiency, as well as other micronutrient
deficiencies.



110.Hypoglycemia is a side effect of which medication?
Answer is: no choices + no sulfonylurea in options
Su is the most common oral anti-diabetic drug to cause hypoglycemia.
Metformin has very low-risk se of hypoglycemia.
Any 2 oral anti-diabetic agents combination can cause hypoglycemia.
Source: http://www.medscape.com/viewarticle/722513_3

111.Case of hypothyroidism

Box 2 Causes of Hypothyroidism

PRIMARY

Agenesis

255
Gland destruction

● Surgical removal

● Irradiation (therapeutic radioiodine, external irradiation)

● Autoimmune disease (Hashimoto’s, atrophic thyroiditis)

● Idiopathic atrophy

● Infiltrative process

Inhibition of thyroid hormone synthesis and release

● Iodine deficiency

● Excess iodide in susceptible persons

● Drugs: interferon alpha, lithium, amiodarone, aminoglutethimide, thalidomide, sunitinib, sorafenib

● Inherited enzyme defects

Transient

● After surgery or therapeutic radioiodine

● Postpartum

● Thyroiditis

SECONDARY

Hypothalamic disease

Pituitary disease

● Genetic forms of pituitary hormone deficiencies

● Infiltrative disorders

● Sheehan’s syndrome

● Surgery or irradiation

● Trauma

256
● Tumors

Resistance to thyroid hormone

112.Most significant risk factor for osteoprosis?


A. Advanced age

113.Pheochromocytoma what is the investigation :


A. Catecholamine in urine
Answer: a
It is are catecholamine secreting tumor derived from chromaffin cells of the sympathetic system.it has a classic triad
(pheochromocytoma )palpitations headache episodic sweating. Urine catecholamines will show increased catecholamine metabolites
(metanephrines) and free catecholamine. Plasma metanephrines if available is the most sensitive test. Reference: toronto notes

114.Vitamin b3 deficiency.
Pellagra is a vitamin deficiency disease most frequently caused by a chronic lack of niacin (vitamin b3) in the diet. Pellagra is
classically described by "the three ds": diarrhea, dermatitis, and dementia.


115.Mechanism of action of propthyrocil (ttt of hyperthyroidsm):
Answer: inhibity thyroperoxsidase enzyme centrally and inhibit conversion of t4 to active t3 peripherally. Medcine. Kaplan
medicine endocrine chapter

116.Young patient has hypertension, high Na and low K. What is the treatment?
• Spironolactone

Answer: A
This patient has hyperaldosteronism.
Spironolactone is a potassium sparing diuretic.

117.Case of polyuria, polydipsia and weight loss. Na: ?. What is the diagnosis?
• DI

Answer: A (Depends on the case)


Diabetes Insipidus (DI) clinical features: Passage of large volumes of dilute urine, polydipsia, dehydration; hypernatremia can develop
with lack of access to water or impaired thirst mechanism.
Reference: Toronto Notes.

118.Pheochromocytoma ..what is the initial tx ?


Alpha antagonist.
Answer: A
Alpha adrenergic blocker to control hypertension follow by beta blocker to control tachycardia never give BB first ( unopposed
action lead to refractory hypertension.
Reference: USMLE step 2 first aid +step up to medicine

119.Young boy presented with abdominal pain and vomiting

Answer: DKA?
Very short question, not enough info, can’t decide but the answer depends on the clinical scenario.
257
Common causes of abdominal pain include constipation, gastrointestinal (GI) infections, infections outside of the GI tract, and
colic.
Less common GI conditions (i.e., inflammatory bowel disease, pancreatitis, cholecystitis, intra-abdominal abscess, dietary milk
protein allergy, malabsorption, and Meckel's diverticulum)
Causes of abdominal pain outside GI tract include: diabetic ketoacidosis, painful crisis with sickle syndromes, Henoch Schönlein
purpura (IgAvasculitis), tumors, urolithiasis, ovarian torsion, testicular torsion, and some toxic ingestions.
The answer “DKA” was written by the person who provided the question, so it’s possibly the right answer.
Reference: http://www.uptodate.com/contents/causes-of-acute-abdominal-pain-in-children-and-
adolescents?Source=preview&search=%2Fcontents%2Fsearch&anchor=H30#H30
Please read here for more specific classification according to age:
Http://www.uptodate.com/contents/image?Imagekey=EM/65488&topickey=EM%2F6454&source=outline_link&search=%2
Fcontents%2Fsearch&utdpopup=true

258
Gastroenterology

259
1. Patient c/o liver cirrhosis and ascites now he c/o weight loss, what should u do ?
A. cea
B. ca125,
C. abdominal us
D. alpha-fetoprotien

Answer: d
Explanation: cirrhosis is a risk factor for hepatocellular carcinoma. When elevated, the AFP is 75-91% specific, and values greater
than 400 ng/ml are generally considered diagnostic of HCC. Ultrasonographic identification of HCC can be difficult in the
background of regenerative nodules in the cirrhotic liver. CEA is mainly for colon cancer. Ca125 for ovarian cancer.
Reference: http://emedicine.medscape.com/article/197319-workup#c1

2. Patient with history of gastric ulcer. Which of the following are used for pain control does not cause gastric irritation?
A. Aspirin
B. Ibuprofen
C. Celecoxib
D. Indomethacin

Answer: c
Selective COX-2 NSAID


3. Migratory thrombophlebitis is associated with which of the following:
A. Chronic pancreatitis
B. Acute pancreatitis
C. Pancreatic malignancy
D. Pancreatic cyst

Answer: c
Trousseau sign can be an early sign of gastric or pancreatic cancer, typically appearing months to years before the tumor would be
otherwise detected.

4. What hepatitis can be prevented by vaccination?


A. A
B. B
C. D
D. E
Answer: B and A (most probably B)

5. Diagnostic peritoneal lavage indicated in :


A. All patient with rta
B. Patient with hypotension and distended abdomen
C. Patient with abdominal pain
D. Patient with sever head trauma
Answer: b

260
6. A patient complains from dysphagia and feel of foreign body sensation. What is the most likely diagnosis?
A. Achalasia
B. Laryngeal cancer
C. Globus pharyngis
D. Esophageal cancer
Answer: d
Explanation:
• Esophageal cancer: dysphagia is most common presenting symptom; it is progressive first to solids then liquids. Second most
common presenting symptom is weight loss. Patients also complain of foreign body sensation
• Achalasia: dysphagia equal to solids and liquids. Patients tends to eat slowly and drink a lot of water to wash down food.
• Globus: persistent or intermittent non-painful sensation of a lump or foreign body in the throat. However, it frequently improves
with eating and is generally unaccompanied by dysphagia or odynophagia.
Step up to medicine
Http://www.hopkinsmedicine.org/gastroenterology_hepatology/_pdfs/esophagus_stomach/esophageal_cancer.pdf
Http://www.ncbi.nlm.nih.gov/pmc/articles/pmc3360444/

7. Patient was screened for hemochromatosis by ALT & AST what another test can be used also:
A. Glucose
B. Ferritin
C. Creatinine level
D. Ceruplasmin
Answer: b
Reference: http://emedicine.medscape.com/article/177216-workup#c1

8. Scenario, best site to insert the needle for liver biopsy?


A. 6th intercostal space
B. 7th intercostal space
th
C. 10 intercostal space
D. Subcostal space.

Answer: b
The biopsy site is usually located in the seventh or eighth intercostal space in the mid axillary line. Reference:
http://emedicine.medscape.com/article/149684-technique

9. Young male complaining of epigastric discomfort, he tried over-the-counter (otc) medication to relieve this discomfort, he
noticed some improvement but experienced constipation.
Which otc drug most likely he tried ?
A. Calcium carbonate
B. Sodium carbonate
C. Aluminum hydroxide
D. Something aluminum
Answer: c



10. A 40-year-old male, 2 years history of difficulty swallowing and lump sensation in throat, excessive salivation, intermittent
hoarseness, weight loss:
A. Achalasia
B. Scleroderma
C. Diffuse esophageal spasm
D. Cricopharyngeal dysfunction
Answer: ??
Achalasia: dysphagia for solids and liquids is the primary clinical feature of achalasia. Weight loss, regurgitation, chest pain,
heartburn, globus sensation (a lump in the throat) & hiccups. Https://yhdp.vn/uptodate/contents/mobipreview.htm?14/26/14759
261
Scleroderma:GI findings in systemic sclerosis include the following: gastroesophageal reflux (may lead to hoarseness, dysphagia
and aspiration pneumonia), dyspepsia, early satiety, malnutrition from inadequate caloric intake.
Http://emedicine.medscape.com/article/331864-clinical - showall
Esophageal spasm: noncardiac chest, globus sensation, dysphagia, regurgitation &
heartburn http://emedicine.medscape.com/article/174975-clinical - showall
Cricopharyngeal dysfunction: progressive dysphagia, coughing, choking, drooling, and regurgitation when swallowing liquids or
solid food
Https://yhdp.vn/uptodate/contents/utd.htm?1/63/2037?Source=see_link#h9

11. Man present with epigastric pain for 3 months worse after eat diagnosed as hpylori triple treatment given to him what is the
best indicator for his improvement
A. A-endoscope
B. Ph
C. Blood test for h pylori
D. Clinical improvement
Answer: d or urea breath test or fecal antigen test

12. Patient with abdominal pain, nausea, vomiting, wt loss. On examination: palpable mass. What is your action?
A. PPI
B. Follow up
C. Urgent referral
D. Ultrasound

Answer: c
This may be a case of gastric cancer which needs to be confirmed by upper gi endoscopy
Http://emedicine.medscape.com/article/278744-workup

13. Asymptomatic patient with positive HBV antigen?


A. Acute hepatitis
B. Chronic hepatitis
C. Active carrier
D. Non active carrier

Answer: d
If the hbsag is positive and the liver enzymes are normal it can be either hbeag negative chronic hepatitis of inactive carrier status. The
only way to differentiate is by follow up.

Reference: https://www.cdc.gov/hepatitis/HBV/pdfs/serologicchartv8.pdf
262
Http://www.bjmp.org/content/hbsag-carriers-normal-alt-levels-healthy-carriers-or-true-patients

• Appears 2 to 10 weeks after exposure to HBV. In self-limiting acute HBV


infection, HBsAg usually becomes undetectable after 4 to 6 months of infection.
Persistence of HBsAg for >6 months implies chronic HBV infection

Answer: a
• Both acute and chronic could be asymptomatic.
• Hepatitis b surface antigen (hbsag)
• Present in acute or chronic infection
• Detectable as early as 1 to 2 weeks after infection
• • it persists in chronic hepatitis regardless of whether symptoms are present. If virus is cleared, then hbsag is undetectable.
Reference: step up to medicine

14. What is the common disease to make the patient retire in ksa?
A. HBV
B. HBC
C. HIV
D. HAV

Answer: A
HBV most common according to study
HCV was an answer of some intern took the exam but not sure if correct or not
HIV is an answer of ID fellow
Good Luck !!

15. Lady come to the clinic to regular check up with everything is normal with table of direct and indirect bilirubin (within the
normal range) what is your diagnosis:
A. Rotor syndrome
B. Crigler najjar
C. Dubin johnson
D. Gilbert syndrome
Answer:

16. A patient with heartburn taking antacids. She had rheumatic fever 1 week ago and was started on aspirin. What side effect
she can develop?
A. Constipation
B. Diarrhea
C. Dry mouth
D. Galactorrhea
Meaning of q is se of which drug!! Antacid or aspirin??
Answer: aspirin se: conditions of excess stomach acid secretion, nausea, vomiting, heartburn, irritation of the stomach (cramps)
Antacid se: cause nausea, constipation, diarrhea, or headache. Diarrhea is more common with this product than constipation
http://www.webmd.com/drugs/2/drug-76860-769/antacid-oral/aluminum-magnesiumantacid-simethicone-oral/details#side-effects
263


17. What is the percentage of complete recovery from HBV?
A. 20%
B. 40%
C. 60%
D. 80%

Answer: d (80%)

18. A patient was diagnosed with enteric fever. What is the presentation that he will have?
A. Confusion (or other cns problems)
B. Maculopapular rash
C. Nausea, vomiting and loose stools
D. Abdominal pain

Answer: d

Over the course of the first week of illness, the notorious gastrointestinal manifestations of the disease develop. These include
diffuse abdominal pain and tenderness and, in some cases, fierce colicky right upper quadrant pain. Monocytic infiltration inflames
peyer patches and narrows the bowel lumen, causing constipation that lasts the duration of the illness. The individual then develops
a dry cough, dull frontal headache, delirium, and an increasingly stuporous malaise.

Reference: medscape: http://emedicine.medscape.com/article/231135-clinical



19. Most common hepatitis is:
A. HBV
B. HBV
C. HDV
D. HEV

Answer: a
reference: medscape

20. 40 years old patient complaining of weight loss, nausea, lethargy and jaundice. When he was asked about similar attack in
the past, he mentioned 4 episodes during the past two years. What is the most likely diagnosis?
A. Acute pancreatitis.
B. Cancer head of pancreas.
C. Peptic ulcer disease.
D. Chronic pancreatitis

Answer: B
The most characteristic sign of pancreatic carcinoma of the head of the pancreas is painless obstructive jaundice.
Reference:http://emedicine.medscape.com/article/280605-clinical

21. 27 years old smoker who was studying in a foreign country for two years and lived in a student housing. He returned home
two months ago. The patient complains of 4 day mid epigastric pain, what is the most likely diagnosis?
A. Viral Hepatitis.
B. H. Pylori infection.
C. Acute Pancreatitis.
D. Myocardial infarction.

Answer: B

264
Epigastric pain is the most common symptom of both gastric and duodenal ulcers. It is characterized by a gnawing or burning
sensation and occurs after meals—classically, shortly after meals with gastric ulcer and 2-3 hours afterward with duodenal ulcer.
Pain radiates to back with pancreatitis.
Reference: http://emedicine.medscape.com/article/181753-overview

22. Which disease involves antibiotic in treatment regimen?


A. Crohn’s
B. Ulcerative colitis
C. Celiac
D. Whipple

Answer: D
Penicillin or ceftriaxone, 2-4 weeks as intial followed by maintenance therapy with oral Trimethoprim-Sulfamethoxazole for
one year.
Reference: UpToDate

23. What is the best treatement for traveler diarrhea? 3 times


A. Ciprofloxacin
B. Cipro
C. Amoxycillin
D. Metronodazole

Answer: A
The most common organism is enterotoxigenic E. Coli and it is treated with fluoroquinolones (e.g. Ciprofloxacin).
Reference: Uptodate.
Initial treatment: rehydration, the Abx only will decrease the duration of symptoms;
1st line: antibiotics include fluoroquinolones, such as ciprofloxacin or levofloxacin or ofloxacin
2nd line: azithromycin. 3rd line: Rifaximin.

24. H.pylori infection cause antral gastritis :h.pylori stimulate which cell?
A. Chief cell
A. Gastrin
B. Parietal
C. Mucous

Stimulate G cell to produce gastrin Inhibit D cell which secrete prostaglandin


25. Young boy presented with diarrhea which is sometime bloody, weight loss, arthritis, anemia. The most likely diagnosis is?
A. Crohn’s disease
B. Uc
C. Celiac disease
Answer: B (most propaply due to clear complint of bleeding by rectal and diarrhea)
Both crohn & uc cause the same symptoms (bleeding is more in uc) while rheumatological symptoms more common in CD.
Celiac will cause steatorrhea (step up)

265
26. Patient has whitish elevated patchy lesion over the dorsal surface of the tongue, it does not remove after scrubbing the
lesion , what is the most likely diagnosis ?
A. Dysplasia
B. Neurofibroma
C. Foreign body
Answer:

27. Oral ulcers with intermittent chronic bloody diarrhea...


A. Celiac disease
B. Uc
C. Crohn's disease

Answer: c
Http://emedicine.medscape.com/article/172940-clinical

28. Alcoholic patient presented with ruq pain, jaundice , spleenomegaly , ascites ,and caput medusa
A. Pht
B. Hepatitis
C. Budd-chiari syndrome
Answer: c

The clinical presentation of budd-chiari syndrome (resemble those of cirrhosis)—hepatomegaly, ascites, abdominal pain (ruq),
jaundice, variceal bleeding. This disease is caused by occlusion of hepatic venous output, which leads to hepatic congestion and
subsequent microvascular ischemia.
Note: if cirrhosis or alcoholic liver disease was mentioned in the choices it is more correct than budd-chiari syndrome
Reference: step up to medicine

266
29. Origin from cadal part of foregut and cephalic of midgut
A. Esophagus
B. Duodenum
C. Ileum
Answer: b
the duodenum develops from the caudal portion of the foregut and cranial portion of the midgut.
Http://www.chronolab.com/atlas/embryo/duodenum.htm

30. Patient complains of acute sub mandibular pain which comes after food. He has history of similar episodes with secretions.
What is the most likely diagnosis?
A. Calaculs in submandibular gland duct (wharton duct)
B. Parotid cancer
C. Parotitis
Answer: a
Salivary duct stones present as pain and swelling in the involved area. It is usually aggravated by eating or anticipation of food. Parotid
cancer’s most common presentation is painless swelling. Parotitis presents as progressive painful swelling with fever.
Http://cursoenarm.net/uptodate/contents/mobipreview.htm?7/20/7488
Http://emedicine.medscape.com/article/882461-clinical
Http://emedicine.medscape.com/article/1289616-overview#a2

31. 19 years old female, presenting with abdominal pain, diarrhea, bloating, improved with defecation, diagnosis?
A. Ibd
B. Ibs
C. Celiac
Answer: b
Irritable bowel syndrome (ibs) is a functional gi disorder characterized by abdominal pain that commonly relived by defecation,
abdominal distension and altered bowel habits in the absence of a specific and unique organic pathology.
The Rome criteria require the presence of abdominal pain or discomfort for at least 3 days/mo in the last 3 mo along with ≥ 2 of
the following:

• Improvement with defecation


• Change in frequency of defecation
• Change in consistency of stool

32. When does the jaundice occurs in gilbert syndrome?


A. Pregnancy
B. Inter current illness
C. Others (no stress in options.)

Answer:
Some conditions and situations that can increase bilirubin levels, and thereby jaundice, in people with Gilbert's syndrome include:
- Illness, such as a cold or the flu
- Fasting or eating a very low-calorie diet
- Dehydration
- Menstruation
- Stress
- Strenuous exercise
- Lack of sleep

Gilbert’s syndrome is not affected by the pregnancy

33. Patient on NSAID with signs of ulcer what you will do?
A. Triple therapy
B. Urea breath test
C. Immediate endoscope

267
Answer: b
Reference: http://www.aafp.org/afp/2007/1001/p1005.html
Algorithm for the treatment of peptic ulcer disease:
*—alarm symptoms include evidence of bleeding (e.g., anemia, heme-positive stool, melena), perforation
(e.g., severe pain), obstruction (e.g., vomiting), and malignancy (e.g., weight loss, anorexia).

(egd = esophagogastroduodenoscopy; PPI= proton pump inhibitor; NSAID = nonsteroidal anti-inflammatory drug.)

34. Hemoangioma with high heart output. What other site will be affected?
A- Lung.
B- Spleen
C- Kidney.
Not sure i think the best answer would be liver

Giant cutaneous hemangiomas — giant cutaneous hemangiomas can also promote the development of high-output failure.
Hemangiomas are the most common tumors of infancy. 50% percent of cutaneous lesions are present at birth; the remainder usually
surface by two months of age. In rare cases, high-flow arteriovenous shunting in giant or multiple cutaneous hemangiomas can lead to
the development of high-output failure

Multifocal hepatic hemangiomas most commonly occur in the presence of multiple skin hemangiomas and are probably most often
asymptomatic, in which case they can be managed with observation with or without serial imaging.
Rarely, multifocal hepatic hemangiomas can have large vessel shunts that result in heart failure, which can be treated with
pharmacotherapy or embolization if necessary.

Diffuse liver hemangiomas, which may occur in the absence of skin hemangiomas, cause massive hepatomegaly with abdominal
compartment syndrome, impaired ventilation, impaired venous return, and renal vein compression, and are associated with a high
mortality rate

35. 30 years with chronic diarrhea and ataxia and abnormal movements. Jejunal biopsy showed: t. Wheppli infection
management?
A- short term antibiotics
B- steroids
C- long term antibiotics.
Answer: c
Spectrum of disease — the spectrum of clinical findings due to t. Whipplei infection is wide.

268
joint symptoms, chronic diarrhea, malabsorption, and weight loss. The joint symptoms preceding the others by many years, so not all
symptoms may be manifest at the time of presentation. Isolated involvement of other organs, most prominently the central nervous
system and heart valves, can also occur in the absence of the classic findings of whipple’s disease.
Treatment rationale:
Management — Penicillin or ceftriaxone, 2-4 weeks as intial followed by maintenance therapy with oral Trimethoprim-
Sulfamethoxazole for one year.

36. Dietary supplementation prevent colon cancer:


A. Folic acid
B. Vitamin d
C. Vitamin c
Answer: b
Cohort studies and meta-analyses provide evidence on the benefits of circulating, diet-derived and supplemented vitamin d and
calcium. On the basis of current evidence one could suggest intake of vitamin d.
Http://www.ncbi.nlm.nih.gov/pmc/articles/pmc3819783/
- a review of nutrition and cancer by the world cancer research fund and the american institute of cancer research concluded that:
“there is enough evidence to conclude that milk may protect against colorectal cancer.”
- folic acid: http://www.nature.com/articles/srep12044

37. A 30-year-old female, history of constipation, strains when she passes stool.
What muscle should be relaxed?
A. Coccygeus
B. Pubococcygeus
C. Puborectalis
Answer: c
the levator ani is a broad sheet of muscle. It is composed of three separate paired muscles, called
the pubococcygeus, puborectalis and iliococcygeus.
Puborectalis main function is to maintain fecal continence – during defecation this muscle relaxes.
Http://teachmeanatomy.info/pelvis/muscles/pelvic-floor/

38. 25 years' teacher, complaining of abdominal pain, fatigue, on exam there was icting, palpable liver 1cm, also 2 student
complaining from same complain?
A. HAV
B. HBV
C. HBV
Answer: a

39. What vitamin increases metabolism or absorption of iron?


A. Vitamin c
B. Vitamin b
C. Vitamin d
Answer: a
Explanation: try to take iron supplement with vitamin c (for example, a glass of orange juice) to increase absorption.
Reference https://my.clevelandclinic.org/health/diseases_conditions/hic_anemia/hic_oral_iron_supplementation

40. 70 years old male presented with recent tiredness and dizziness, he gives history of change of his bowel habit since awhile, in
that he change his diet to gaurge (something i don't know type of food) on lap he had hypochromic microcytic anemia and
iron deficiency. What could be the cause:
A. Change in his bowel habit
B. Change in diet
C. His age
Answer: a ??

269
41. Patient with HBA after 3 weeks we take biopsy, what is show?
A. Normal architecture
B. Fibrosis something
C. Another something

Answer: b
May be the question mean HBV…
"ground-glass" cells seen in approximately 50-75% of livers affected by chronic HBV infection which may progress to fibrosis in
severe disease.
Reference: http://emedicine.medscape.com/article/177632-workup#c9

42. Man came with loose stools, history of loose stools before, its watery with mucus not containing blood (forget the other
details ) what is your diagnosis:
A. Ibs
B. Crohn's disease
C. Ulcerative colitis

Answer : a
Unlike ibd, ibs does not cause inflammation, ulcers or other damage to the bowel which can cause bleeding. Symptoms of ibs may
include crampy pain, bloating, gas, mucus in the stool, diarrhea and constipation.
Reference: http://www.webmd.com/ibs/ibd-versus-ibs

43. Patient came with history of alternating bowel habits, sometimes the stool is loose and sometimes suffer from constipation
for weeks, what’s the diagnosis :
A. Ibs
B. Crohn’s disease
C. Ulcerative colitis

Answer: a
Clinical features of IBS
1. Change in frequency/consistency of stool—diarrhea, constipation (or alternating
diarrhea and constipation)
2. Cramping abdominal pain (relieved by defecation)—location varies widely, but
sigmoid colon is the common location of pain
3. Bloating or feeling of abdominal distention

Step up of Medicince & Toronto Note



44. Abdominal pain and fever, then constipation then diarrhea. Culture showed gram –ve rod, non-lactose fermenting, oxidase -
ve organism that produces hydrous sulphate. Most appropriate treatment?
(salmonella typhi)
A. 50s subunit
B. DNA gyrase inhibitor
270
C. Transpeptidase

Answer: b fluoroquinolones is the drug of choice for treating typhoid fever.


Reference
• Peer review
• Medscape but not very clear there is a lot of details regarding resistance.

45. Mountain climber who has hypoxia, which of the following liver zones is most affected by hypoxia
A. Central of acini zone ii
B. Peripheral of acini zone ii
C. Sinusoidal

Answer : ?
o Acinus is a diamond-shaped; divided into zone I (periportal), zone II (transition zone), and zone III
(pericentral).
o The periportal zone I is nearest to the entering vascular supply and receives the most oxygenated blood,
making it least sensitive to ischemic injury while making it very susceptible to viral hepatitis.
o Zone III has the poorest oxygenation, and will be most affected during a time of ischemia.
o Zone I is specialized for oxidative liver functions such as gluconeogenesis, β-oxidation of fatty
acids and cholesterol synthesis.
o Zone III is more important for glycolysis, lipogenesis and cytochrome P-450-based drug detoxification.
o
Zone III is most sensitive to acetaminophen toxicity.
o Zone I most affected in hemochromatosis
o Zone II necrosis in yellow fever.

46. Best prophylactic against traveller's diarrhea? Repeated in family medicine
A. Fresh fruit and vegetables
B. Peeled fruit
C. Daily antibiotic
D. Drinks with rice

Answer: B
Uptodate: “basic advice for travelers to moderate or high-risk regions for travelers' diarrhea includes eating only food that has been
thoroughly cooked and served hot, fruits that the traveler peels just prior to eating, and pasteurized dairy products. Beverages
should be bottled or disinfected. Bottled drinks should be requested without ice and should be drunk from the bottle with a straw
rather than from a glass. Hot tea and coffee are usually safe alternatives to boiled water.”
“although antibiotics and other agents (namely bismuth salicylate) are effective in reducing the rate of travelers' diarrhea for
individuals traveling from resource-rich to resource-poor areas, we do not routinely recommend chemoprophylaxis. Use of daily
antibiotics is expensive, has potential side effects, can wipe out normal gastrointestinal flora that may be beneficial, and can
promote bacterial resistance.”
“however, chemoprophylaxis may be a reasonable approach in the setting of an underlying medical condition that would increase
the risk of complications from diarrhea or would be severely exacerbated by dehydration from diarrhea such that the benefits of
using antibiotic prophylaxis outweigh its risks. Such situations include known severe inflammatory bowel disease that could be
exacerbated by an episode of infectious diarrhea; severe vascular, cardiac, or renal disease that would be seriously compromised by
dehydration; or a severe immunocompromised state, such as advanced HIV disease or after a complicated organ transplant.
If prophylaxis is administered, the options include:
●ciprofloxacin — 500 mg once daily
●norfloxacin (not available in the us) — 400 mg once daily
●rifaximin — 200 mg once or twice daily
●bismuth subsalicylate — two tablets chewed four times daily

47. Patient with Crohn's disease, what is the most relevant and associated with CD?
A. Positive Family History
B. Smoking
C. Alcohol
271
Answer: A
Positive family history is the largest independent risk factor for CD and UC
Reference: (Kumar)


48. Women coming with elevated indirect bilirubin:
A. Rotor syndrome
B. Crigler Najjar
C. Dubin Johnson

Answer: B
Differential diagnosis:
- Unconjugated (indirect) hyperbilirubinemia: Hemolytic, physiologic (newborns), Crigler-Najjar, Gilbert syndrome
- Conjugated (direct) hyperbilirubinemia:
1- Biliary tract obstruction: gallstones, cholangiocarcinoma, pancreatic or liver cancer, liver fluke.
2- Biliary tract disease: primary sclerosing cholangitis and primary biliary cirrhosis
3- Excretion defect: Dubin-Johnson syndrome, Rotor syndrome.
- Mixed (direct and indirect) hyperbilirubinemia: Hepatitis, cirrhosis.


49. First sign of portal HTN?
A. Spleenomegaly
B. Hepatomegaly
C. Ascites

Answer : A
Non of the sources mention any of the above options as being the first sign. Ascites is always mentioned first before splenomegaly in
signs and symptoms section and step up books state that portal hypertension can’t happen without ascites. However, some sources
stated that splenomegaly is the most cardinal sign of portal hypertension.
Old sources: Davidson’s Principles and Practice of Medicine.
New sources:
Https://dundeemedstudentnotes.wordpress.com/2014/06/13/portal-hypertension-and-oesophageal-varices/
Http://www.score95.com/blog/blog/usmle-portal-hypertension/

50. Asymptomatic 40 y female patient, direct bilirubin 5, indirect 9, what is your diagnosis?
A. Gilbert disease
B. Rotor syndrome
C. Dubin-johnson syndrome
Answer: A
patient is asymptomatic and the indirect (unconjegated) bilirubin is higher than direct (conjugated). Hereditary or inborn
metabolic disorders may cause unconjugated or conjugated hyperbilirubinemia.
Unconjugated: gilbert syndrome, crigler-najjar syndrome, and primary shunt hyperbilirubinemia. Conjugated: dubin-johnson
syndrome and rotor syndrome
http://www.clevelandclinicmeded.com/medicalpubs/diseasemanagement/hepatology/guide-to-common-liver-tests/
Http://www.merckmanuals.com/professional/hepatic-and-biliary-disorders/approach-to-the-patient-with-liver-disease/inborn-
metabolic-disorders-causing-hyperbilirubinemia#v897839

51. Patient with mild to moderate inflammatory bowel disease, what is the mainstay of treatment?
A. Surgery
B. Antibiotics
Answer:
The first step in medication therapy for ibd is usually aminosalicylates with greater efficacy ulcerative colitis than for crohn disease.
For crohn disease, metronidazole or ciprofloxacin is occasionally used.
Second step is often corticosteroids (oral prednisone), which tend to provide rapid relief of symptoms and a significant decrease in
inflammation.
272
The immune-modifying agents are step III drugs and are used if corticosteroids fail or are required for prolonged periods. Anti-TNF
monoclonal antibody therapies are also step III drugs that are effective in both crohn disease and ulcerative colitis.
Consider surgical intervention for ulcerative colitis patients in whom medical therapy fails.


Http://emedicine.medscape.com/article/179037-treatment

52. Child ate a number of iron tablets presented with severe symptoms including constipation and bloody stool nausea and
vomiting and drowsiness how would you treat him:
A. Iv deferoxamie.
B. Dialysis
Answer: A
Chelation is the mainstay of therapy. It is indicated for serum iron levels >350 mcg/dl with evidence of toxicity or >500 mcg/dl
regardless of signs or symptoms.
Http://emedicine.medscape.com/article/815213-treatment


53. Obese, acid reflux symptoms ,2week diagnosed with rf ,he is on asprin rx??
A. Metlozam
B. Cinitidine
Reference: http://emedicine.medscape.com/article/236582-overview

o In patients with mild and intermittent (less than two episodes per week) symptoms, low-dose histamine 2 receptor
antagonists as needed (PRN)
o If symptoms persist, we discontinue H2RAs and initiate once-daily proton pump inhibitors (PPIs).

54. A patient presented with bilateral lower limb edema, distended abdomen and palmer erythema. On examination, small vessels
appear on abdomen. What’s the diagnosis?
A. Liver cirrhosis
B. Heart failure
Answer is a
Lower limb edema and distended abdomen (ascites) are physical signs of portal hypertension that can happen in both right-sided hr and
liver cirrhosis, while palmar erythema and small vessels on the abdomen (spider telangiectasia) are some of the stigmata of liver disease.
Source: http://www.turner-white.com/pdf/hp_jul03_stigmata.pdf

55. 12 -years with hepatic failure admitted to icu, his skin was yellow . Now become green in color , what does this indicates :( not
mentioned obstructive jaundice in the choices!)
A. Oxidation of bilirubin.
B. Impending death
Answer: a

273
Bilirubin is created by the activity of biliverdin reductase on biliverdin, a green tetrapyrrolic bile pigment that is also a product of heme
catabolism. Bilirubin, when oxidized, reverts to become biliverdin once again.

56. Patient with right lower quadrant pain and swelling and loss of weight. Colonoscopy done showed mass in right lower quadrant.
What is the diagnosis?
A. Appendic tumor
B. Cecum tumor
Answer: b

57. Case about ibd which cell type responsible about ulceration in intestine?
A. B cell
B. T cell
Answer: b
Cytokines differentiate lymphocytes into different types of T cells. Helper T cells, type 1 (Th-1), are associated principally with crohn
disease, whereas Th-2 cells are associated principally with ulcerative colitis. The immune response disrupts the intestinal mucosa and
leads to a chronic inflammatory process.
Link: http://emedicine.medscape.com/article/179037-overview#a3

58. Which of the following lead to dilatation of aorta?


A) achalasia
B) barrett
Answer??
I suspect this is dilation on the esophagus not the aorta
The diagnosis of achalasia is established by the presence of aperistalsis in the distal two-thirds of the esophagus and incomplete lower
esophageal sphincter (les) relaxation on manometry. In patients with typical achalasia symptoms (dysphagia to solids and liquids and
regurgitation of bland undigested food or saliva) and equivocal manometric findings, the diagnosis is supported by aperistalsis,
dilation of the esophagus, bird beak appearance, and poor emptying on barium esophagram.
Achalasia should be suspected in patients with dysphagia to solids and liquids and in those with regurgitation unresponsive to
a trial of proton pump inhibitor (ppi) therapy for four weeks. Manometry is required to establish the diagnosis of achalasia. In patients
with equivocal motility testing, barium esophagram should be performed to assess esophageal emptying and esophagogastric junction
morphology. Endoscopic evaluation should be performed in patients with suspected achalasia to exclude a malignancy at the
esophagogastric junction that can mimic primary achalasia

59. Tenia coli which layer


A) adventitia
B) musculosa
Answer: muscularis externa

60. Female itching high alkaline phosphatase +ve antimitochondrial antibody


A) primary biliary cirrhosis
B) sclerosing cholangitis
Answer: a

61. Best treatment for primary biliary cirrhosis is:


A. Azathioprine
B. Urdox

Answer: b
Ursodeoxycholic acid (UDCA) is the major medication used to slow the progression of the disease.
Http://emedicine.medscape.com/article/171117-medication#showall

274
62. What type of cell is decreased in uc?
A. Enterocyte
B. Goblet cell
Answer: b
Uptodate: the biopsy features suggestive of ulcerative colitis include crypt abscesses, crybranching, shortening and disarray, and
cryatrophy. Epithelial cell abnormalities including mucin depletion and paneth cell metaplasia may be seen.
Https://yhdp.vn/uptodate/contents/mobipreview.htm?11/7/11386#h596557081
Medscape, biopsy and microscopic features: http://emedicine.medscape.com/article/2005396-overview#a8

63. Long scenario, bloody diarrhea and RBC in urine after 7 days hx of food poisoning, rx?

A. Steroid
B. Antibiotic
Answer: b ??
Hus etiology:
1. Diarrhea positive hus: 90% of pediatric hus from e. Coli o157:h7, shIgAtoxin, or verotoxin.
2. Diarrhea negative hus: other bacteria, viruses, familial, drugs, familial/genetic.

Reference: toronto note, step-up of medicine


Treatment of HUS
I. Therapy is primarily supportive
a. Fluid and electrolyte management; dialysis is indicated for refractory
acidosis, hyperkalemia, fluid overload, and/or uremia
b. HTN control
c. Seizure control
d. RBC transfusion to keep hemoglobin > 7 gldL
e. Consultation with nephrologist or hematologist

64. Kwashiorkor disease is due to which kind of diet?


A. High carb, low protein diet
B. Low carb, high protein

275
Answer: a

• Kwashiorkor is a severe form of malnutrition, caused by a deficiency in dietary protein

Reference: wikipedia: https://en.wikipedia.org/wiki/kwashiorkor





65. Hereditary chronic pancreatitis is:
A. Autosomal dominant
B. Polygene
Answer: a
(uptodate) and (pubmed) thereafter, hereditary chronic pancreatitis (hcp) was defined as an autosomal dominant disease with a
penetrance of approximately 80%.

66. If the patient has hbsag +ve andIgM+ve what do you treat the patient with?
A. Interferon
B. Lamivudine
Answer: b,
Toronto and UpToDate: treatment options: tenofovir, entacavir, lamivudine, adefovir, interferon,
medscape: interferon-a and etacavir ~> 1st line agents

67. What is the oncogene for pancreatic cancer?


A. K-ras
B. Myc

Answer: A
Genetic alterations can be detected at different levels. These alterations include oncogene mutations (most commonly, K-ras
mutations, which occur in 75% to more than 95% of pancreatic cancer tissues), tumour suppressor genes alterations (mainly, p53, p16,
DCC, etc.), overexpression of growth factors (such as EGF, TGF alpha, TGF beta 1-3, afgf, btgf, etc.) And their receptors (i.e., EGF
receptor, TGF beta receptor I-III, etc.).
Reference:http://www.ncbi.nlm.nih.gov/pubmed/1964102 http://www.ncbi.nlm.nih.gov/pubmed/10660490

68. During colonoscopy of patient who has fragile thin surface of the colon with multiple blood dots , there was no previous
chronic disease , what is the most likely diagnosis ?
A. Ulcerative colitis
B. Mesentric ischemia
Answer: A
Aphthous ulcer:
Ibd
It is one of the main manifestations of crohns disease usually in addition to prolonged diarrhea with abdominal pain, weight loss, anal
skin tags, and fistulae. Http://emedicine.medscape.com/article/172940-clinical#b3
First aid:

69. 40 y male 6m hx of diarrhea mix e blood +wt loss+ no family hx of inflammatory dis then patient had intermittent fever ..in
sigmoid scope see fragile mucosa of rectum and spot blood ..dx? Bacterial dysntria
A. Uc
B. Ischmic colitis

70. 16 years old female . Fever and Chronic diarrhea for 10 months Post meal para umbilical pain Sometimes blood mixed with
stool
A-crohn
B- chronic pancreatitis

276
Answer: A


71. Kayser flescher ring what treatment?
A. Penicillamine
B. Deferoxamine
Answer: A
Treatment of wilson disease
4 drugs are available:
• Penicillamine chelates copper, poorly tolerated
• Trientine chelates copper zinc impairs copper excretion in stool/decreases copper absorption from gut tetrathiomolybdate
preferred if neurological involvement
• Liver transplant in severe cases

72. Old man started a year ago on medication for peptic ulcer developed gynecomastia what is the drug:
A. Cimetidine
Answer: a
- Https://en.wikipedia.org/wiki/cimetidine
- Http://reference.medscape.com/drug/tagamet-cimetidine-341984#4

73. Patient 24 hours with diarrhea and vomiting. Blood pressure supine 120/80, Blood pressure . 80/40.what is the cause?
Answer: decrease intravascular
Dehydration (low fluid volume in the body) causes orthostatic hypotension other causes:
http://my.clevelandclinic.org/health/diseases_conditions/hic_orthostatic_hypotension

74. Case of acetaminophen toxicity, what’s the sequels of liver damage?


Answer: see image

75. Patient with history of GERD, required aspirin for the treatment of rheumatoid
Arthritis, what medication you’re going to add next?
Answer: misoprostol

76. Diarrhea in a kid what's the mechanism ( decreased absorption , membranous coating )
Answer: ???

77. Treatment of plummer-vinson syndrome?

277
Dysphagia, upper esophageal webs, and iron deficiency anemia = plummer-vinson syndrome (pvs). Iron replacement to correct the
anemia it may improve the dysphagia. Those with significant and long-standing dysphagia usually require mechanical dilation e.g.
Endoscopy.

78. Peptic ulcer disease Rx the kid is on PPI and metronidazole what to add?
o Quadruple therapy
§ PPI
And --
§ Bismuth subsalicylate: 525 mg orally four times daily
And --
§ Metronidazole: 500 mg orally four times daily
And --
§ Tetracycline: 500 mg orally four times daily
Or
o Sequential therapy
§ PPI
-- and --
§ Amoxicillin: 1000 mg orally twice daily on days 1-5
-- and --
§ Clarithromycin: 500 mg orally twice daily on days 6-10
Bmj http://bestpractice.bmj.com/best-practice/monograph/80/treatment.html

79. Antacid on of its side effects is constipation


A. Aluminum hydroxide.
Http://www.rxlist.com/antacids-page5/drugs-condition.htm

80. The cause of crohn’s disease:


A. Idiopathic
Answer: a

81. Patient AST very high, the cause?
Answer: alcohol

82. Treatment of acute hepatitis C?


A. Interferon
Answer: a
Uptodate: Interferon-based therapies are likely the only option for treatment of acute HCV infection available for most patients.

o Supportive care
o No treatments attenuate acute viral hepatitis, including hepatitis C.
o Follow patients for 6 mo to allow spontaneous clearance and then treat those who have persistent viremia (ie, chronic
hepatitis C).
o Alcohol should be avoided because it can increase liver damage.
o Most patients may safely return to work after jaundice resolves, even if AST or ALT levels are slightly elevated.
o For cholestatic hepatitis, cholestyramine 8 g po once/day or bid can relieve itching.

83. Pt with dysphagia decrease manometry in pharynx and upper esophagus what is the diagnosis?
No options for this one.
Possible causes: mysthenia graves, stoke, or dermatomyositis, parkinson…etc

84. Child with jaundice and kieser ring in iris.


Answer: wilson disease.
278
85. Mode of inheritance for wilson's disease answer:

autosomal recessive

86. Most common infectious disease among medical staff in saudi arabia
answer: HBV

87. Oral leukoplakia that can’t be swiped off. Most likely to be?
A. Dysplasia

Answer : ?? It’s hyperplasia of squamous cells >> dysplasia >> carcinoma in situ

88. Enterococcus faecalis antibiotic and the patient is allergic to ampicillin, what to give?
A. Vancomycin
Answer: a
Reference: uptodate.

89. Patient came with cough and she takes an anti-cholesterol medication (statins), she started it 3 weeks ago, what should the
doctor monitor?
A. Liver function test
Answer: a
LFT should be carried out before and within 4-6 weeks of starting statin therapy. Thereafter at intervals of 6 months to 1 yr.
• If satisfactory lipid control and no evidence of adverse effects then review again at 4-6 months, then 6-12 monthly.
• If unsatisfactory lipid control then measurements should be repeated 6 weeks after dosage adjustments are made until the desired
lipid concentrations are achieved.
• Nice state that LFTs only need to be measured on three occasions:
• Baseline liver enzymes.
• Within 3 months of starting treatment and at 12 months.
• Treatment should be discontinued if serum transaminase concentrations rise to, and persist at, 3x normal range.

90. Triple therapy for gastric ulcer in 10 yo boy?


Answer: triple therapy regimens for h pylori consist of a ppi, amoxicillin, and clarithromycin for 7-14 days.

91. Pt with bilateral flank swelling dull move with shifting


A. Ascitis

92. Prevents stomach ulcers caused by nonsteroidal anti-inflammatory drugs (NSAIDs).


A. Misoprostol or PPI

Source: https://en.wikipedia.org/wiki/misoprostol

93. Clear history about GERD, they want diagnosis.


First aid:
Primarily a clinical diagnosis. History of:
- heartburn that commonly occurs 30-90 minutes after a meal, worsens with reclining, and often improves with antacids, sitting, or
standing. Also, sour taste, a sensation of a lump in the throat, unexplained cough, morning hoarseness, and chest pain mimicking
CAD.

279
94. Patient came complain of heartburn after taking med. He is takin- is. What the med?!
The answer is one of bisphonate (ends with -onate)

95. Pt diagnosed to have panacinar emphysema, splenomegaly & liver disease. What is the underlying cause :
Answer: alpha 1 antitrypsin deficiency

96. Female using NSAID for her dysmenorrheal developed epigastric pain , most likely dx :
A. Gastritis
Explanation: drugs like NSAIDs are known to cause gastritis. The major mechanism of injury is the reduction in prostaglandin
synthesis. Prostaglandins are chemicals responsible for maintaining mechanisms that result in the protection of the mucosa from the
injurious effects of the gastric acid.
Reference: http://emedicine.medscape.com/article/175909-overview#a4

97. Case of hepatic failure on medication for treatment what the cause of change jaundice from yellow to green yellow ??

98. Patient treated for duodenal ulcer. Now complains of breast enlargement and decrease sexual desire. Which drug?
Answer :
Ranitidine (h2 blocker)
Ref./ http://www.drugs.com/sfx/ranitidine-side-effects.html

99. Young girl with jaundice ( and i think liver failure) her color turned from yellow to green, why?

Oxidation

100.I had 3 qs about ibd manifestation::


Http://emedicine.medscape.com/article/179037-overview
Inflammatory bowel disease (ibd) is an idiopathic disease caused by a dysregulated immune response to host intestinal microflora. The
two major types of inflammatory bowel disease are ulcerative colitis (uc), which is limited to the colon, and crohn disease (cd), which
can affect any segment of the gastrointestinal tract from the mouth to the anus, involves "skip lesions," and is transmural. There is a
genetic predisposition for ibd, and patients with this condition are more prone to the development of malignancy.

101.Case of painless jaundice & the answer is “gilbert’s syndrome”.


Gilbert syndrome (also called familial non-hemolytic jaundice) has no signs or symptoms other than mild jaundice and is discovered
only when routine blood tests reveal elevated unconjugated bilirubin levels. You can read more about gilbert’s syndrome here:
https://yhdp.vn/uptodate/contents/mobipreview.htm?41/23/42359

102.Antacid mechanism of action_


Antacids aluminum salts and magnesium antacids are contact; they decrease the acidity of gastric secretion, and by their buffer by
direct neutralization of the hydrochloric acid present in the stomach.
Http://www.rayur.com/antacids-definition-mechanism-objectives-indications-adverse-precautions-and-risk.html


103.Patient with epigastric pain that decrease by food intake what is the organism?
A. Hilcobacter pylori
Answer:a
This is a case of duodenal ulcers(epigastric pain; may localize to tip of xiphoid,burning,develops 1-3 h after meals and relieved by
eating and antacids). The most common causes of ulcer are(h.pylori infection,idiopathic, NSAIDs, stress induced and ze
syndrome)
reference: toronto notes.

280
104.Long scenario about patient with peptic ulcer disease you give him PPI+ amoxicillin , what can you add to them ?
A. Clarithromycin
Answer: a
To eradicate h.pylori there are 2 regimens of therapy:
Triple therapy for 7-14 d (hp-pac®): PPIbid (e.g. Lansoprazole 30 mg bid) + amoxicillin 1 g bid + clarithromycin 500 mg bid (the
regimen in the question)
Quadruple therapy for 10-14 d: PPIbid + bismuth 525 mg qid + tetracycline 500 mg qid + metronidazole 250 mg qid
Reference: toronto notes

105.Which of the following is contraindicated to do gastric lavage


A. Drain cleaning solution
Answer: a (their original answer)
Contraindications to gastric lavage include ingestion of corrosives and most hydrocarbons, patients with depressed gag reflexes who
are not intubated and clinically insignificant ingestions.
Bleeding diathesis

106.College student complains of generalized pain relieved by defecation. No blood or mucus. Diagnosis?
A. Ibs.
Answer: a
Diagnostic criteria: 3d in 3m of episodic abdominal discomfort that is two or more of 1- relieved by defecation, 2-change in stool
frequency or consistency 3- change in stool appearance?


107.70 Year-old female brought to ur clinic by her daughter . The daughter said her mother's memory deteriorated in the last 2
years . She can dress her self but e difficulty , she can cook for herself but sometimes leave the oven on ,,,,,,,,,what's the
management ?!
A. Refer her to geriatric clinic

What gene is related to Celiac Disease?



Answer: ?
HLA-DQ2 (chromosome 6) found in 80-90% of patients compared with 20% in general
population; also associated with HLA-DQ8.
Reference: Toronto Notes and http://emedicine.medscape.com/article/171805-overview#a6

108.Which one of the following can cause liver cirrhosis?


Answer:?
Fatty liver (alcohol, metabolic syndrome)
Chronic viral hepatitis (B, B+D, C; not A or E)
Autoimmune hepatitis
Hemochromatosis
Α1-antitrypsin deficiency
Primary biliary cirrhosis
Chronic hepatic congestion
- Cardiac cirrhosis (chronic right heart failure, constrictive pericarditis)
- Hepatic vein thrombosis (Budd-Chiari),
- Idiopathic
- Rare: Wilson’s disease, Gaucher’s disease . Reference: Toronto Notes.

109.Celiac disease affects which of the following locations?


A. Distal SB

281
Answer:?
Affect the upper small bowel (Duodenum and proximal jejunum) more than lower small bowel.
Reference: webmed website and digestive disease center

110.A patient with chronic retrosternal pain, cough and metallic taste in mouth. What is the most likely diagnosis?
A. GERD

Answer: A
“Heartburn”(pyrosis) and acid regurgitation (together are 80% sensitive and specific for reflux) ± sensation of a lump in the throat
(bolus sensation), coughing, chest pain, and wheezing. Usually a clinical diagnosis based on symptom history and relief following a
trial of pharmacotherapy (proton pump inhibitor (PPI): symptom relief 80% sensitive for reflux).
NB: Ambulatory 24-hour ph monitoring is the criterion standard in establishing a diagnosis of GERD, with a sensitivity of 96% and a
specificity of 95%. Ppis are the most effective therapy.
Reference: Toronto Notes and http://emedicine.medscape.com/article/176595-clinical

111.Pyoderma gangrenosum is associated with which one of these diseases?


A. Ulcerative colitis.
Answer: A
PG is more common in UC (5–12%) than CD (1–2%).
Reference: http://www.ncbi.nlm.nih.gov/pmc/articles/PMC3273725/table/T1/

112.Scenario pt. Came with melena and bleeding take NSAID for 3 weeks, what is the cause ?
Answer: NSAID use is associated with an increased risk of gastric or duodenal ulcer by inhibition of prostaglandins that leads to
symptomatic chronic ulceration ,also inhibit platelet function, and their use has been associated with GI bleeding from
throughout the GI tract.
Reference: http://www.uwgi.org/guidelines/ch_07/ch07txt.htm

113.Celiac pt what is safe for him?


A.Rice
Answer : A. Rice is safe for celiac patients
Http://wheat.pw.usda.gov/ggpages/topics/Celiac.vs.grains.html
Https://www.nlm.nih.gov/medlineplus/ency/article/002443.htm

114.Primary biliary cirrhosis options were about pathophysiology.


Answer:
Intrahepatic - T lymphocyte mediated attack on small intralobular duct
Go and read: http://emedicine.medscape.com/article/171117-overview#a6

282


Immunology &
Allergy











283
1. Least harmful vaccine in immunocompromised?
A. Bcg
B. Mumps
C. Measles
D. Pneumococcal
Answer: d
A,b,c are all live vaccines. Pneumococcal vaccines is a subunit vaccine.
Http://www.historyofvaccines.org/content/articles/different-types-vaccines


2. Dm, hypothyroid, irregular menses female, present withe recurrent itching & white adherent oral plaque, +ve mantux test,
she was exposed to tb 4 years ago, immunoglobulin, wbc, RBCs all are normal; diagnosis;
A. Chronic granulomatous disease
B. Chronic mucocotanous candidiasis
C. Digeorge syndrome
D. Hyperglobulinemia ( or hypo i don't remember )
Answer: b
Chronic granulomatous: leuckosytosis <charestersitc>, levels of the 3 major classes of immunoglobulins, immunoglobulin g,
immunoglobulin m, and immunoglobulin a, are increased. Immunoglobulin e levels are increased or in the reference range.

3. Patient got rapid swelling response after a bee sting what type of hypersensitivity ?
A. 1
B. 2
C. 3
D. 4

Answer: a

• Pathology is similar to other immunoglobulin e (ige)–mediated allergic reactions.

Reference: medscape: http://emedicine.medscape.com/article/768764-overview#a5




4. Which vaccine you will give to immunocompromised?
A. Recom. HBV
B. Sabin
C. Salk
D. Bcg

284

o Killed or inactivated vaccines do not represent a danger to immunocompromised persons and generally should be
administered as recommended for healthy persons.
o Steroid therapy usually does not contraindicate administration of live-virus vaccines when such therapy is short term (less
than 2 weeks)
o 2 mg/kg of body weight or a total of 20 mg/day of prednisone is immunesuppretive dose and given live vaccine is
questionable.
o Physicians should wait at least 3 months after discontinuation of steroid before administering a live-virus vaccine to
patients who have received high-dose,
o MMR can be given to HIV patient.
o When elective splenectomy is planned, vaccination with pneumococcal, meningococcal, and Hib vaccines should precede
surgery by at least 2 weeks, if possible.
o BCG vaccination is not recommended for HIV.
o Yellow fever vaccine is a risk of encephalitis.
o Hib vaccine to HIV-infected persons should take into consideration the individual patient's risk of Hib disease and the
effectiveness of the vaccine for these persons.
o Persons who received chemotherapy and will be given live-virus vaccines, should be at least 3 months after stopped the
chemotherapy or at least 2 weeks before start chemotherapty.
o Vaccination during chemotherapy or radiation therapy should be avoided because antibody responses are suboptimal.


5. What type of vaccine is the pneumococcal conjugate vaccine (pcv13)?
A. Live
B. Conjugate
C. Inactive
Answer: b reference: http://vk.ovg.ox.ac.uk/pcv

6. Most common cause of central line infection


A. During the insertion
B. Migration of bacteria from other site
C. While giving the medication through the line
Answer: a.
Reference: http://www.uptodate.com/contents/epidemiology-pathogenesis-and-microbiology-of-intravascular-catheter-infections

7. Like mump senario :"can't remember "
A. Sjögren
B. Hypersensitvity vasculitis
285
C. Autoimmune....
Answer: a
Sjögren syndrome is a systemic chronic inflammatory disorder characterized by lymphocytic infiltrates in exocrine organs. Recurrent
swelling of the parotid glands which is similar to mumps. Http://emedicine.medscape.com/article/332125-clinical#b3

8. Bee sting since 18 hrs. With swelling and redness, what will you do?
A. Antihistamines
B. Steroids
C. Observe

Answer: a
• Local reactions can be life threatening if swelling occludes the airway. Initiate invasive measures to secure the airway if
this occurs. Otherwise, the following local care measures suffice:
• Provide supplemental oxygen
• Diphenhydramine limits the size of the local reaction.
• Clean the wound and remove the stinger if present.
• Apply ice or cool packs.
• Elevate the extremity to limit edema.

• Treatment should include an initial intravenous (iv) bolus of 10-20 ml/kg isotonic crystalloids in addition to
diphenhydramine and epinephrine.
If the patient has not removed the stinger, it should be removed as soon as possible by the first caregiver on the scene.
Delay increases venom load, so the fastest removal technique is the best. Pinching and traction is an acceptable technique.
Reference: medscape: http://emedicine.medscape.com/article/768764-treatment


9. Clear presentation of sjogren syndrome, asking about the complication:
A. Pulmonary fibrosis
B. Malabsorption
C. Lymphocytic tissue infiltration
Answer: c
Morbidity associated with sjögren syndrome is mainly associated with the gradually decreased function of exocrine organs, which
become infiltrated with lymphocytes.
Reference: http://emedicine.medscape.com/article/332125-overview#a7

10. Man got a bee sting then his wife trying look for the epinephrine what it gonna inhibit?
A. Leukotriene release from macrophages
B. Cross reactivity with the cardiac..
C. Inhibit immunocomplex formation

Answer: (inhibit histamine and tryptase, missed option)


Epinephrine is the drug of choice for anaphylaxis. It stimulates both the beta-and alpha-adrenergic receptors and inhibits further
mediator release from mast cells and basophils. It inhibits histamine, the proteases tryptase and chymase.
References: https://quizlet.com/8362963/immuno-block-4-practice-exam-flash-cards/
Http://www.worldallergy.org/professional/allergic_diseases_center/anaphylaxis/anaphylaxissynopsis.php
Http://www.ncbi.nlm.nih.gov/pmc/articles/pmc3343118/


11. Patient can't take bcg vaccine because he deficiency in?
A. Il
B. Tnf gama
C. Ifnγ

Answer: c
Complete il-12p40 and il-12rβ1 deficiencies and partial ifnγr1 and ifnγr2 deficiencies generally predispose the patient to curable
infections. Complete ifnγr1 and ifnγr2 deficiencies predispose the patient to overwhelming infection

286
12. BCG vaccine is contraindicated in which of the following deficiencies?
A. IL
B. TNF
C. INF

Answer: C
Reference:https://books.google.com.sa/books?Id=mrta5qwtc7ac&pg=PA459&lpg=PA459&dq#v=onepage&q=BCG%20contraindica
ted&f=false P: 459


13. Which of the following organisms is seen in patients with chronic granulomatous disease?
A. Cl. Difficle
B. Staph aureus
Answer: b
Cgd is a primary immunodeficiency that affects phagocytes of the innate immune system and leads to recurrent or persistent
intracellular bacterial and fungal infections
Most infections in cgd are caused by staphylococcus aureus.
Reference: http://emedicine.medscape.com/article/1116022-clinical#showall

14. Pt with allergy to penicillin. When he receive the medication he develop sob, urtecaria &….. . What does this called?
A. Asthma
B. Anaphylactic
Answer: b

15. Long scenario where the investigation shows highIgG:


A. Monoclonal something
B. Mutiple myeloma .
Answer: b (question incomplete)
Explanation: multiple myeloma diagnosis: protein electrophoresis: monoclonal spike due to a malignant clone of plasma cells
synthesizing a single ig (usually igg) called a monoclonal protein (m-protein). The 1st choice is likely monoclonal gammopathy of
unknown significance, diagnosed by the presence of a serum monoclonal protein (m protein) at a concentration <3.0 g/dl. And <10%
plasma cells in bone marrow
References: step up to medicine 3rd edition, page 351
Toronto notes 2015, page h51
Reference: http://emedicine.medscape.com/article/966003-overview


16. What vaccine is contraindicated in immunocompromised (HIV or chemotherapy) patients?
Live attenuated vaccines( measles, mumps, dpt..)
Reference: http://www.cdc.gov/vaccines/schedules/hcp/imz/adult-contraindications-shell.html

17. Which vaccine is least harmful to immunocompromised patients?


A. Pneumococcal vaccine
Reference: https://www.cdc.gov/mmwr/preview/mmwrhtml/00023141.htm

18. Allergy to peanut butter what is the mechanism ?


(not written from the original writer, but it is type i ige mediated hypersensitivity, initial sensitization with the ag passing through the
gi tract)

19. Pt with recurrent oral and genital ulcers and arthritis when the pt taken some type of im vaccine develop sterile abscess at
the site of injection. What is the most likely dx.?

287
Answer: behcet disease

Behçet disease is characterized by a triple-symptom complex of recurrent oral aphthous ulcers, genital ulcers, and uveitis.

Reference: medscape: http://emedicine.medscape.com/article/329099-overview

20. -x-linked a gamma globulinemia cd 19 and 20 cd40 mutation

21. Case of lactose intolerance


Answer: http://www.medicinenet.com/script/main/mobileart.asp?Articlekey=7809


22. Peanut allergy mechanism of action


23. Case of multiple myeloma.
Answer: http://emedicine.medscape.com/article/204369-overview#a1

24. A child present with s&s of leukemia with calla +ve? Dx?
A. All

25. What of the following is poor prognosis? _ high ig m -low ig a -high ca, should be beta 2 micro globin


26. Case of guillain barre syndrome ..clear. (hx of diarrhea 3w past then devolp asending symetrical ll parlysis... )


27. Case of sle .. Rash type: * case of ?


28. Scenario about a patient with bone lytic lesions and lab result what is the disease
• Paget’s disease
Answer:a
It is a metabolic disease characterized by excessive bone destruction and repair. Patients usually are asymptomatic. Severe bone pain
is one of the presenting complaint and skeletal deformities.in cases of skull involvement: headaches, increased hat size, deafness.
Increased warmth over involved bones. Patients will have high levels of alp and urinary hydroxyproline. Ca and po are normal(ca
could be elevated).

288
Reference: toronto notes.

Bullet points

289
• Read about epstien bar virus
• Read about kawasaki disease
• Read about mononucleosis

• Corona virus
Http://www.uptodate.com/contents/middle-east-respiratory-syndrome-coronavirus
• Ebola virus
Http://www.uptodate.com/contents/clinical-manifestations-and-diagnosis-of-ebola-virus-disease
• Alport disease
Hereditary nephritis (alport’s disease): x-linked nephritis often associated with sensorineural hearing loss; proteinuria <2
g/d
• Dic
• Addison disease.
• Cardiomyopathy
• Iron deficiency with koilonychia.
• Types of bacteria and antibiotics
• Siadh
• Osteoarthritis
• Guillain barre syndrome
• Liver cirrhosis
• HBV
• Juvenile rheumatoid arthritis
• Chronic pain
• Bilateral pneumonia
• Hyperthyroidism
• Read about a thalassemia.
• Q about cohort study
• Child fell from 10 meter height with bleeding from his ears, where is the injury?
• Gcs of a 10 yrs child, continuously crying and screaming not responding to his name, doesn't move but respond to pain by
...'i forgot the scenario'
• 11 b. 10 c. 9 d. 8
• Pt took her anti osteoporosis drug then had a sever retrosternal pain.
• Don’t remember exact drugs, but belongs to bisphosphonate.
• Goodposture syndrome: types of gn
• 2nd index finger pain with numbness of hand: tendon injury
• Oral hyperglycemic drugs case hypoglycemia
• Lipid lowering agent can cause sever muscle weakness
• Best muscle relaxing agent
• Pt on hypoglycemic agent with poor control blood sugar have sulpha drug allergy <<< which medication you give
• The most common disorders difrantional sex in boys?
• Case of tonsillitis <<< antibiotic
• Most common stain blood nipple discharge
• Drug maintenance in bipolar
• Drugs in several pre-eclampsia to prevent eclampsia seizure
• Picture of ectotropion
• Picture of senlle chart how many meters pt stand >>>>6 m
• Direction of im injection related to sciatic nerve? Safest to use the upper outer quadrant.
• Presentation of posterior hip dislocation? The limb is shortened and internally rotated
• Treatment for severe depression?
• Sensrion , valgus test positive which ligament will be affected ? Collateral ligament
• Child with meningitis treated with antibiotics see report what is the cause
• Agrranulocytoma >>> carmphicanol treatment
• Cavernous sinus thrombosis which vein drainage? Maxillary, ophthalmic
• Farmer with lesion protruding keratin? Keratosanthoma.

290
• Highest risk of stroke? Htn.
• Child with gradual developing of cyanosis and ejection systolic murmur of left upper sternal border? Tof.
• Bacteria associated with ventilator- pneumonia? Pseudomonas
• Child with strabismus, treatment? Glasses
• Depigmented lesion? Vitiligo
• Child with morning stiffness and joint pain of wrist and ankle? Juvenile rheumatoid arthritis
• Investigation for bone density? Dxa scan.
• Retired man used to work in factory with sensorineural hearing loss? Noise related
• Best lab for patient with hemochromatosis? Ferriten
• Older patient with memory loss and change of behavior? Alzheimer
• Patient with pinpoint pupil antidote? Nalxone.
• Patient with dilated pupil tachycardia cause? Sympathomimetic
• Girl with premenstrual syndrome treatment? Ssri.
• Patient with anemia, thrombocytopenia, splenomegaly and hepatomegaly investigation? Bone marrow biopsy
• Pregnant third trimester with bright red blood? Placenta previa.
• Patient history of travel c/o bloody diarrhea? Amebiasis
• Sickle cell patient with hip pain? Avascular necrosis.
• Patient with foot pain? Planter fasciits
• Patient involved in trauma with signs of increase icp which nerve you should examine? Optic
• Older patient lifting heavy object with mass not reaching the scrotum? Direct inguinal hernia
• Pediatric boy 14 years with follow up asymptomatic by ex arrhythmia and (pic of ecg which show irregular rate and t
elevation!! What's dx:
• Rx of alzheimer disease?
• Mass in upper part of kidney what's the gene
• Muscles of knee
• Embryology >> origin of portion smooth muscle in right atrium or ventricle ..
• Patient complain of pain swelling of big toe what's the dx > gout
• HTN with Blood pressure h what's the drug of htn
• Gene in pt breast Ca> brca
• What's the anion gap na= 138
• Which ligament when you do epidural anaesthesia
• Women fully dilated 0 station, for breech delivery with head extension what's the management > >
• Migraine
• Metabolic acidosis
• Campaign to prevent stroke: Blood pressure control
• 2 qs about croup
• Gout: xanthine oxidase
• Tb. Numbness: pyridoxine
• Numbness. What drug: isoniazid
• Q about the muscles of the mandible.
• Pt with urti symptoms when he coughed he see something i forget: retina pressure
• Pic of dendriatic herpatitis:
• Best description of case-control study.
• Relative risk equation
• Pt with depigmentation and problems in her eyes (i can't recall them) what can be associated with it? Renal
• Pregnant lady twins a ciphalic b breach

• Pregnant bleeding from every where after delivery >>> dic


• Patient presented with pneumonia symptoms for 2 weeks i think the gram stain negative: mycoplasma pneumonia
• Case about rh –ve women married a rh +ve man, their first child was rh +ve like the father, now the wife is pregnant for
the second time,
• Not sure about the question, or what they were asking about, may about the frequency of monitoring, and the titers.
• 11 yrs old female with jaundice and splenomegaly?
• (sca, spherositosis) both present with splenomegaly and jaundice!
• Child with left sided abdominal mass, and other features, dx? Wilms tumor.

291
• Small fluid discharge <<<< ‫ اﺳﻢ ﺗﯿﺴﺖ ﺑﯿﺒﺮ ﺗﺤﻮل ﻟﻮﻧﮭﺎ ازرق‬
• Embryologic origin of follicle cells in the ovaries
• Pt with diarrhea and mild dehydration shows very enlarge clitoris? She received ors what else? Hydrocortisone
• Which pathology does echoccus granulosus?
• Child came with swelling in the scalp with loss of hair?
• Something sebum
• G1p0 accidently discovered fibroid on us what best advice?
• If it is asymptomatic have no effect
• Gene in DM 2 <<<<
• Characteristic of sca in peripheral blood smear
• What best treatment for mania with advanced liver disease? May be lithium
• What breast cancer present bilateral
• 14y girl had sexual contact in mid-cycle and she concerned about pregnancy what are you tell her:
• Pregnancy rate 5%
• Douching
• Neck pain radiating inter-scapular and numbness
• Vertebral collapse (my answer)
• Polymylagia rheumatic
• Pt with hyperthyroidism on treatment. C/o recurrent infections. What is the drug
o Methamizole
• Pt present with sx of brochictasis what is the best advice?
o Stop smoking
• Pt present with rash and had hx of infection >ITP or hsp
• Pt has thyroid nodule we did TSH the result eutheroid>>fna
• Constrictor patient complain>>lateral epicondilitis
• 29-case of opacification in lense >>cataract (sure)
• 31-athelete after exercise presented with hypopegminted lesion tx >>(tinea versicolor) >>selinum
• Topical antifungal medications containing selenium sulfide are often recommended
• Pt i didnt remember the case but culture result gram negative ,motile urease and oxidase negative what is the best
treatment ?? Salmonella tx(bed rest if u don't find it choose ceftraixone i think!!
• Pt in pain, difficult to communicate with him how to asses his pain ?
o Numerical scale
o Face scale( my ans )
• Increase effect of analgesia? Metoclopramide (my ans)
• Question about tt of alzahimer
• Another question about alzahimer drug that is hepatotoxic
• A drug to treat DM named incritin what's the moa of it
• Glipizid moa
• Right peritonsilar abscess and fever
o Acute quincy ++
o Retropharyngeal abscess( complication)
• Patient c/o decrees vision and another hx in the eye also had a mass in the neck
o Neuroblastoma
o Wilm's
• Mammogram can detect breast cancer how many years prior to detection by self-examination?
o 1 - 2 -3 -4
• A patient took a drug that cause pinpoint pupil
o Opioids
• 2 question about the anti dot of the previos answer
• Naloxone++✅
• There were many questions about kidney changes in DM and moa of DM drugs
• Silver crust at hair line and nasal what is ttt ?
• HTN patient with renal failure ? Name ttt of htn
• Cyanotic heart disease admitted to er with convulsion what medication induce ? Digoxin(rare symptom of toxicity-
medscape)
• HTN patient with renal what mechanism in kidney with HTN ?
• Witch of them consider ( produce cyanosis )
292
• All age in options 10 year .. 11 year one options :18 month with VSD muscular !! Not confirm
• Read about embryology problem about cleft palate ?
• Bee sting lead erythema - swelling what ttt : 2 q with diffrent options
• Http://www.mayoclinic.org/diseases-conditions/bee-stings/basics/treatment/con-20034120
• Women with stress pick of hair ? What is disorder .. !!
• Another about disorder ? Read !!
• Read the algorithm of bacteria
• Read type of pregnancy ?
• DM 1 loss conscious in school ? Best management ? Admitted to hospital !! No option for glucagon
• Read sexual infections !! May 6 q
• Read ttt of hurshpring disease and see picture for x-ray 2 q about ttt
• Neonate with duodenal obstruction ? What the sign ? Bubble !!! Not confirm
• Read about anterior ant posterior dislocation ( shoulder and hip )
• What medication lead to convulsion ?Ssri - anti depressant read about
• What medication lead to HTN crisis ?Ssri - anti depressant read about !!
• Read about glucoma ttt .. And congenital
• Read about antibiotic and mechanism of action 2 q
• Related hernia to cord ? Anatomy
• Emergency treatment for mi ?
• Elevated of direct bilirubin with ast and ALT ? Hepatitis
• Elevated direct and indirect with ast ?
• Elevated total bilirubin with alp and ast or ALT not cnfirm ? Diagnosis ( no obs jaundice or any liver disease ) .. Gb
stone in option !!
• Tear eye - red conjuntiva ? Diagnosis ( no infection in option ) keratitis - uveitis .. Others
• Pregnant lady developed edema and hypertension:
o Mgso4
• Limping child , painful left hip ( x ray given )
o Slipped capital femoral epiphysis ( 10 y and above)
o Pethe's disease .(4-8 y)
• Diagnosis of migraine ( straightforward )
• Same weight for 6 months , amenorrhea :eating disorder ?
• Diagnosis of penile painless ulcer :
o Darkfield microscopy
• Itching in lower limbs , otherwise normal :
o Tinea ?Scabies ?
• Well circumscribed lesion on erythamatous base , arthritis :
o Rhumatological disease
• Vesicles highly suspected round worms:
o Ascaris
o Tenea saginata
• Aids, retinitis :
o Cytomegalovirus
• Case of uti methicillin sensitive : cloxacillin ?
• Question about splenic sequestration ( with lobar infiltrate in lung )
• New classification of lung ca : adenomatous mass less than 2 cm ?✅
• Case of lymphoma , treatment :
o Chop r
o Abvd
• Hemolytic anemia, coombs’ positive, what type of hypersensitivity: type 2
• Lung infection :iv ceftriaxone ?
• EBV pharyngitis what is ttt :iv acyclovir ?
• About latent sle
• Mother close to delivery developed respiratory symptoms + fetal distress :
o Amniotic fluid embolism
• Thyroid disease associated with papillary ca :hashimoto ?✅ reidle thyroditis ?
• Soldier walks 1000miles developed pain on foot :spring ligament
• How poliovirus vaccine works ?
• Kid with bilateral abscess of inguinal nodes what causes this condition ?Chronic granulmoatous disease
293
• DM pregnant with hx of fetal death before delivery now she's 32 weeks pregnant with a new baby what to do? Deliver
her immediately wait until 36 weeks, weekly biophysical profile or fetal heart rate testing can be combined with
maternal kick counts in the third trimester. For patients who have experienced earlier loss, frequent ultrasound is
reassuring
• Scalp lesion (derma) what is the treatment?

• Man with low back pain and lytic lesion biopsy shows schistocytes and giant cells what is the diagnosis ?A- giant cell
tumor of bone
• Boy goes camping when he's back he came with constitutional symptoms lymph nodes enlargement and took penicillin
developed rash what does he havei chose mononucleosis i think it's right
• 3 years old with a father known to have pulmonary tb his ppd 10mm what does he have ?
o Strong +ve
o Week +ve
• Hf patient what to order next ?
• Thyroid nodule rx in the right side onl
• Anemia in kids what's the management ?
• Growth chart for a kid with
o Low weight and length what does he have ?
o Familial short stature or normal or what
• Research questions very difficult relative risk, prevalence
o Most accurate test etc
• Mers cov > 66:1
• Lady with a mass in left upper quadrant of the breast, aspiration was yellow fluid without masses, dx?
o Phyllodem, normal tissue variant, ..."no simple cyst"
• 1000-20, 5000-6, or? 2.0
• Ionizing radiation effect on DNA? Pyrmedine ..., deamination, deputination (no oxidization)
• Inflammatory bowel disease
• Gram -ve pneumonia

294
2/5
SMLE
KSAU-HS
Question Bank
1.10 Edition
Surgery Questions

This is an accumulative effort from King Saud bin Abdulaziz University for Health Sciences (2016-17/
Batch 9) interns to organize and answer what have been collected previously from SMLE Q Bank
2015-16

We would like to acknowledge:


- King Saud bin Abdulaziz University for Health Sciences (2016-17/Batch 9) interns for their huge efforts in
accomplishing this project
- SMLE Q Bank Group
‫ مدونة طالب طب سعودي‬-

‫جهد بشري قابل للخطأ والصواب‬


For any comments, kindly contact us at
SMLE2016.17@gmail.com

Highlighted in yellow are queried questions


Highlighted in red are some repeated questions

2
Surgery

Table of Contents
General Surgery ........................................................................................................................................... 4
Plastic Surgery ........................................................................................................................................... 69
Pediatrics Surgery ...................................................................................................................................... 76
Urology ..................................................................................................................................................... 79
ENT ........................................................................................................................................................... 99
Ophthalmology.........................................................................................................................................125
Orthopaedics ............................................................................................................................................160
Anesthesia ...............................................................................................................................................215
Extra information .....................................................................................................................................221

3
General Surgery

4
1. Internal iliac injured which is affected

A. Anterior thigh muscle


B. Ovary
C. Bladder
D. Anterior abdomen
Answer: C
The arterial blood supply of the bladder arrives primarily via the internal iliac (hypogastric) arteries. These branch into the umbilical
artery, which supplies several superior vesicle branches, and inferior vesical arteries, which come as direct internal iliac branches in
males or from the vaginal arteries in females.
(http://emedicine.medscape.com/article/1949017-overview#a2)

2. Case Necrotizing fasciitis treatment?

A. Imipenem & metronidazole


B. Ampicillin & gentamicin
C. Pipracillin & tazobactam
D. Penicillin and smth

Answer: C
Necrotizing fasciitis is a rapidly progressive inflammatory infection of the fascia, with secondary necrosis of the subcutaneous
tissues.
For treatment refer to:
(http://emedicine.medscape.com/article/2012058-overview)

3. MVA when to do diagnostic peritoneal lavage:


A. Hypotensive patients.
B. All MVA patients.
C. Unconscious patients with severe head injury.
D. Conscious patients with abdomen pain.
Answer: A
DPL can be used to evaluate both blunt and penetrating abdominal trauma in patients who are hemodynamically unstable or who
require urgent surgical intervention for associated extra-abdominal injuries. DPL can rapidly confirm or exclude the presence of
intraperitoneal hemorrhage. Thus, the patient with a closed head injury, the unstable patient who has been in a motor vehicle
accident, or the patient with a pelvic fracture and potential retroperitoneal hemorrhage can be appropriately triaged to emergency
laparotomy.
Http://emedicine.medscape.com/article/82888-overview

4. Patient with BPH best investigation:


A. Cystoscopy
B. Annual renal function
C. Annual prostate antigen

5
D. Beta-blocker
Answer : C
Source: http://emedicine.medscape.com/article/437359-workup
And Toronto notes 2014 FM17

5. A patient undergone for a major surgery, a days later, his surgical wound was infected, what is the most likely source of
infection:
A. The patient visitor at visiting time.
B. The staff hands during examination and dressing.
C. The pressure dressing gauze.
D. The dressing tools and devices
Answer: B

6. A child falls down the stair and his head hit the floor. On examination, he was alert and oriented, not having any neurological
deficits, he had only bleeding from his right ear, on further ear exam, the eardrum was obviously rupatient ured. What is the
most likely the source of the bleeding?
A. Subdural Hemorrhage.
B. Skull base Fracture.
C. Subarachnoid hemorrhage.
D. Other choices I can not remember.
Answer: B

7. A child suddenly hit a hard object, he sustained a forehead wound, where will this wound be drained?
A. Pre auricle lymph nodes.
B. Retro auricle lymph nodes.
C. Occipital lymph nodes.
D. Sub-mental lymph nodes.
Answer: A
Forehead drain in pre auricle ( parotid)

8. During nissen fundoplication the surgeon injure posterior vagal trunk. Which of the following structure will be affected
A. Esophagus
B. Jejunum
C. Descending colon
D. Urinary bladder

6
Answer: A
9. Which of the following anaphylactic shock appears initially then disappear?
A. Patchy
B. Papule
C. Macule or nodule (I can not remember )
D. Whale* ?? (not clear choice)

Answer:

10. Sickle cell patient with recurrent RUQ pain with elevated pancreatic exam ,During U/S there was multiple gall stone (I think 7
gall stones), The largest one was2.5 cm when you did ERCP there was no stone in billiary tree , what is your management ?
Antibiotic
Answer: Cholecytectomy

11. What is the most common presentation of lower intestinal obstruction ?


A. Alternating diarrhea and constipation
B. Absolute constipation
C. Vomiting
D. Abdominal distention
Answer: D the answer should be colicky abdominal pain
Http://bestpractice.bmj.com/best-practice/monograph/877/diagnosis/history-and-examination.html

12. Newborn baby complains of (many signs that indicate distress likes tachypnea). Patient looks unwell on examination.
Auscultation of the left side of chest shows no breath sound and the heart sound heard in right side. What is the most likely
diagnosis?
A. Situs inversus
B. Pneumothorax
C. Bowl hernia
D. Another not related answer
Answer: C.
Infants with congenital diaphragmatic hernia (CDH) most often present with respiratory distress in the first few hours or days of life.
Physical findings include a barrel-shaped chest, a scaphoid-appearing abdomen (because of loss of the abdominal contents into the
chest), and absence of breath sounds on the ipsilateral side. In patients with a left-sided CDH, the heartbeat is displaced to the right
because of a shift in the mediastinum.
Http://cursoenarm.net/UPTODATE/contents/mobipreview.htm?31/57/32657

13. Routine breast self-exam advice:


A. Daily
B. Weekly
C. Monthly
D. Yearly
Answer: C, New Evidence: Not Advised
Http://www.uspreventiveservicestaskforce.org/Page/Document/updatesummaryfinal/breast-cancer-screening

7
14. Elderly bedridden has an ulcer 4*5 reaching fascia and muscle, what is the stage?
A- Stage 1
B- Stage 2
C- Stage 3
D- Stage 4
Answer: D

15. What is the recommended management for a patient with carotid bruit with occlusion 60% of the left carotid artery?
A. Aspirin daily
B. Angiography
C. Carotid endarterectomy
D. None of the above
Answer: A
We recommend intensive medical therapy using all available risk reduction strategies for patients with asymptomatic carotid
atherosclerosis. Currently viable strategies include statin therapy, antiplatelet therapy, blood pressure control, and lifestyle
modification consisting of smoking cessation, limited alcohol consumption, weight control, regular aerobic physical activity, and a
Mediterranean diet.
We suggest CEA (see 'Carotid endarterectomy' below) plus intensive medical therapy, rather than intensive medical treatment
alone, for medically stable patients who have a life expectancy of at least five years and a high grade (≥80 percent) asymptomatic
carotid atherosclerotic stenosis at baseline or progression to ≥80 percent stenosis despite intensive medical therapy while under
observation, provided the combined perioperative risk of stroke and death is less than 3 percent for the surgeon and center.

16. 5-year-old with blunt abdominal trauma, Upper GI series showed coil spin in 2nd 3rd duodenal and high amylase, How to
manage:
A) Laparotomy and hematoma evacuation
B) Gastroenterostomy
C) Bowel rest
D) CT guided needle for hematoma extraction
Answer: C
17. 18 years old boy play basketball he came with abdominal pain without any injury in match physical exam was tenderness in
Periumbilical what you next:
A. Chest x-ray
B. Abdominal CT
C. 24 recheck
D. Kidney US

ANSWER: B
- In Acute abdomen: symptoms that suggest surgical or emergent conditions include fever, protracted vomiting, syncope or
presyncope, and evidence of gastrointestinal tract blood loss.
- CT scan should be used after other investigations (CBC- Electrolytes- Amylase- Lipase- LFT- ECG)
- FIRST AID: SURGERY Clerkship

18. Presentation of DiGeorge syndrome


A. Cervical cyst
B. Ectopic thymus
C. Ectopic parathyroid
D. Fistula
Answer: B
- Typically results from a deletion in chromosome 22, which disrupts the development of the pharyngeal arches and pouches,
and may also cause neurological, immunological, endocrinological, or cognitive deficits.

8
- The classical presentation is a triad of cardiac anomalies, hypoplastic thymus, and hypocalcaemia (resulting from
parathyroid hypoplasia).
- An absent thymus or one in an aberrant location may be noted on chest radiographs and CT scans.
- Http://bestpractice.bmj.com/best-practice/monograph/947/highlights/summary.html

19. A man who is post-cholecystectomy, now complaining of unilateral parotid swelling. He has history of mumps. Facial nerve
intact, no decrease in salivation. Lab shows cloudy saliva, and saliva culture in negative.
A. Sarcoidosis Granuloma
B. Sialadenitis
C. Parotid cancer
D. Mumps
E. Sjogren syndrome

Answer is B
Sialadenitis (inflammation of the salivary glands), or in this case bacterial parotitis occurs in postoperative patients or other
severely ill patients who become dehydrated.
Mumps (viral parotitis) happen in unvaccinated children. Sjogren syndrome affects post-menopausal women; it causes
chronic parotitis, dry eyes, and dry mouth.
Source: http://emedicine.medscape.com/article/882461-overview

20. HIV patient presented with (symptoms of intestinal obstruction); did intestinal resection. The tumor found was white in
color, nearly encircling the wall. What is the tumor?
A. Hodgkin
B. Non-Hodgkin
C. Adenocarcinoma
D. Plasmacytoma

Answer is B
The most common malignancies in HIV patients are Kaposi sarcoma followed by Non-Hodgkin’s lymphoma. Non-Hodgkin’s
in HIV patients is almost always extra-nodal, and one-third occurs in the GI tract.
Source: http://hivinsite.ucsf.edu/insite?Page=kb-04-01-11#S9X
Http://www.medscape.com/viewarticle/734593

21. Child after trauma in perineum region showed urine extravasation


A. Penile urethra
B. Prostate urethra
C. Ureter
D. Bladder

9
22. Women traveled 18 hrs after landing She couldn't put her feet back on the shoes The best Diagnosis test ?

A. MRI pelvis
B. CT pelvis &abdomen
C. CT abdomen
D. Compression CT

Answer: D

23. Patient after 2 weeks post MI c/o unilatral leg swelling, pale, loss of hair other leg is normal. Diagnosis?
A- acute arterial emboli
B- acute arterial thrombus
C- DVT
D- disecting artrey

Answer: A
Explanation: It is often difficult to distinguish an embolus from a thrombosis, but embolic occlusions should be suspected in patients
with the following features: 1) acute onset, where the patient is often able to accurately time the moment of the event; 2) a history
of embolism; 3) a known embolic source, such as cardiac arrhythmias; 4) no prior history of intermittent claudication; and 5) normal
pulse and Doppler examination in the unaffected limb.
Reference: http://www.medscape.com/viewarticle/431272_2

24. Dense echo, acoustic shadow:


A-Tumor
B-Uric acid stone
C-Blood clot
D-Sloughed papilla
Answer: B
Reference: https://quizlet.com/60348762/nephro-flash-cards/ question number 7
Http://radiopaedia.org/articles/urolithiasis http://emedicine.medscape.com/article/983759-workup#c5

25. Patient with swelling in the thyroid, She is euthyroid what will you do ?
A. Thyroid lobectomy
B. FNA
C. Biopsy
D. Excisional biopsy

Answer: B

If the serum TSH concentration is normal or elevated, and the nodule meets criteria for sampling, then FNA biopsy is indicated.

Http://www.uptodate.com/contents/diagnostic-approach-to-and-treatment-of-thyroid-
nodules?Source=outline_link&view=text&anchor=H11#H11

10
26. Patient was skating on the stair, then he was falling with a saddle trauma, he developed penile, scrotal & lower abdominal
swelling with hematuria, which organ was affected?
A. Bladder
B. Urethra
C. Testes
D. Prostate

Answer: B
In this injury a male patient falls forcefully with the legs apart on something hard. Some examples of this are falling upon the
crossbar of a bicycle, a railing, or being bucked onto the horn of a saddle. In this injury, the urethra is squeezed against the underside
of the pubic bone and forcefully divided in two

Http://healthcare.utah.edu/urology/conditions/traumatic-urologic-injuries.php

27. Patient presented with cervical mass not in the midline and move with swallowing, what is diagnosis:
A. Thyroid nodule
B. Thyroglossal cyst
C. Brachial plexus
D. Cervical lymph nodes

Answer : A

Assessment of the mass with swallowing is important as movement from swallowing suggests a lesion in the thyroid gland or a
thyroglossal cyst . The latter also elevates with tongue protrusion .

Http://www.ajol.info/index.php/cme/article/download/43974/27491

28. Patient presented with cervical swelling associated with weight loss, night sweating and fever but no respiratory or
neurological symptoms, he was diagnosed as non hodgkin's lymphoma. On CT scan, there are paraaortic and inguinal lymph
nodes enlargement
what is the stage:
A. IB
B. IIB
C. IIIBS
D. IIIIBD
Answer :C

http://www.uptodate.com/contents/image?Imagekey=HEME%2F97479&topickey=HEME%2F4696&source=see_link
Http://www.cancer.org/cancer/non-hodgkinlymphoma/detailedguide/non-hodgkin-lymphoma-staging

29. Patient came with symptoms of increase IC pressure (confusion , nausea , vomiting), before doing Head CT what is the cranial
nerve examination will support the DX.?
A. Optic
B. Oculomotor
C. Facial
D. Vestibulococlar
Answer: A

papilledema is often used broadly to denote a swollen optic nerve head, the term papilledema should be reserved for optic disc
swelling that is due to raised intracranial pressure.

11
Http://www.uptodate.com/contents/overview-and-differential-diagnosis-of-
papilledema?Source=outline_link&view=text&anchor=H6#H6

30. What’s the medication that could increase pain in acute cholecystitis?
A. Acetaminophen
B. Propoxyphene
C. Meperidine
D. Morphine (increases the pressure at sphincter of Oddi)
Answer: D
Http://www.meb.uni-bonn.de/dtc/primsurg/docbook/html/x3982.html
31. Lesion that is volcano-like on a 70 year old farmer’s hand?
A. Basal cell carcinoma
B. Squamous cell carcinoma
C. Melanoma
D. Leishmania
Answer: B
Keratoacanthoma is a Squamous cell carcinoma associated with sun exposure and usually present on the hand
Http://www.skinsight.com/adult/keratoacanthoma.htm

32. Theoretically if there could be a vaccine that could prevent an oncogenic disease, it would be?
A. AML
B. ALL
C. Adult t-cell lymphoma/leukemia
D. Mycosis fungoides
Answer: C

33. Thoracentesis in mid axillary line


A. 4th intercostal space
B. 5th intercostal space
C. Intercostal space 8
D. Between 7
Answer: D
According to Medscape it’s between 7-9 ribs, so I believe D is the right answer but it wasn’t written completely.
Http://emedicine.medscape.com/article/80640-overview#a3

34. X of ulcerative colitis when will do colonoscopy?


A. 1 y
B. 6 m
C. 8y after dx
D. Barem enema 1 y
Answer: C -First Aid
35. Patient has high Alkaline Phosphatase, in order to confirm the hepatic origin of this elevation the doctor ordered one more
test. What is it?

A- Lactate dehydrogenase.
B- Creatinine kinase
C- Gamma-glutamyl transpeptidase
D-Lipase

12
Answer: C
The gamma-glutamyl transferase (GGT) test may be used to determine the cause of elevated alkaline
Phosphatase (ALP).
Both ALP and GGT are elevated in disease of the bile ducts and in some liver diseases, but only ALP
Will be elevated in bone disease. Therefore, if the GGT level is normal in a person with a high ALP, the
Cause of the elevated ALP is most likely bone disease.

Reference: https://labtestsonline.org/understanding/analytes/ggt/tab/test/

36. Old patient that presented with abdominal pain from time to time starts in the left mid abdomen radiates to the back,
whenever he have the attacks, he lies down on that side and bend his body position like a baby, no vomiting, diarrhea or wt
loss what is the diagnosis:
A- Duodenal ulcer
B- Gastric ulcer
C- Chronic pancreatitis
D- Mesenteric thrombosis

Answer: C
Pancreatitis
- Clinically, the patient experiences intermittent attacks of severe pain, often in the midabdomen or left upper
abdomen and occasionally radiating in a bandlike fashion or localized to the midback.
- Other symptoms associated with chronic pancreatitis include diarrhea and weight loss.
- During an attack, patients may assume a characteristic position in an attempt to relieve their abdominal pain
(eg, lying on the left side, flexing the spine and drawing the knees up toward the chest).
Reference: Medscape: http://emedicine.medscape.com/article/181554-clinical#b1

37. A patient was not able to move her arm above shoulder after breast CA surgery that involved axillary dissection.
What is the nerve injured?
A. Long thoracic nerve
B. Rotator cuff
C. Supraspinatus
D. Brachial plexopathy

Answer: A
We have four nerves must the surgeon be aware of during an axillary dissection:
Long thoracic nerve
Thoracodorsal nerve
Medial pectoral nerve
Lateral pectoral nerve
th
Reference: Surgical Recall, 6 edition

38. A 32 years-old alcoholic male patient is brought to the emergency department with the history of vomiting large amount of
bright red blood. Physical examination revealed splenomegaly and ascites. Which of the following is the most likely source of
bleeding?
A. Duodenal ulcer.
B. Proton pump.
C. Esophageal varices.
D. Gastric cancer.

13
Answer: C
Reference: Toronto Notes

39. 23 years old woman with cyclic bilateral nodularity in her breast since 6 months. On examination there is tender 3 cm mobile
subareolar mass on her right breast. What will you do next?
A. FNA with cytology
B. Mammogram
C. Biopsy
D. Follow up for next cycle
E. Observation

Answer: C

Reference: https://books.google.com.sa/books?Id=rkt3hwnsbp4c&pg=PT1027&dq=#v=onepage&q&f=false

40. A patient with diaphragmatic hernia and you want to cut the phrenic nerve. Where can you do that?
A. Anterior to scalenus anterior
B. Anterior to scalenus medius
C. Posterior to scalenus anterior
D. Posterior to scalenus medius

Answer: A

14
41. Which of the following suggest ductal papilloma on breast examination?
A. Blood stain on nipple
B. Hemorrhagic discharge
C. Serous discharge
D. Pus from nipple

Answer: A

42. A patient came to your clinic with inflammation, redness and swelling around the nail. You gave 1 week course of augmentin
but of no use. What should be done? (Picture was also attached)
A. Give augmentin for one more week
B. Incision and drainage
C. Warm soaks
D. Splint

Answer: B If an abscess has developed, incision and drainage must be performed. Reference:
http://emedicine.medscape.com/article/1106062-overview

43. You need to put cannula in the great saphenous vein. Where can you find
its branch?
A. In front of medial malleolus
B. In front of lateral malleolus
C. Behind medial malleolus
D. Behind lateral malleolus

Answer: A
The great saphenous vein runs anteriorly to the medial malleolus. Reference:
http://www.umed.lodz.pl/anestezja/dokumenty/injections.pdf

44. A patient presented with hard mass on the outer upper area of the breast. Which lymph node you have to examine?
A. Posterior axillary
B. Anterior axillary

15
C. Lateral axillary
D. Medial axillary

Answer: B
Anterior (pectoral) group: from the lateral quadrants of the breast. Reference:
https://www.dartmouth.edu/~humananatomy/part_2/chapter_7.html#chpt_7_lymphatic

45. A patient presented with lymph node enlargement on the horizontal line of the inguinal ligament. What is the structure that
you must examine?
A. Anal canal
B. Gluteal folds
C. Anterior medial leg
D. Anterior medial thigh

Answer: A
Superficial inguinal lymph nodes: Anal canal (below pectinate line), skin below umbilicus (except popliteal territory), scrotum.
Reference: FA USMLE step1

46. A patient with a stabbed wound to the Gluteus. Examination: The patient tilt to the unaffected side while walking. Which
nerve is affected?
A. Femoral N
B. Obturator N
C. Superior Gluteal N
D. Inferior Gluteal N
E. Peroneal N

Answer: C
Superior gluteal nerve is a nerve that originates in the pelvis and supplies the gluteus medius, the gluteus minimus (abductor
muscles), and the tensor fasciae latae muscles. (Trendelenburg gait)

47. A patient had appendectomy. The artery that supplies the appendix is a branch from which of the following?
A. Superior epigastric Artery
B. Inferior epigastric Artery
C. Superior Mesenteric Artery
D. Inferior Mesenteric Artery

Answer: C
The appendicular artery (appendiceal artery) is a terminal branch of the ileocolic artery which is a branch of Superior mesenteric
th
artery. Reference: Surgical Recall, 6 edition

16
48. Patient have appendectomy what is the most common infection will be come after operation?
A. B. Fragilis
B. Staph aureus
C. Shigella
D. Pseudomonas

Answer: A
Enterococcus faecalis, Escherichia coli and B. Fragilis are common pathogens in Surgical Site Infections after clean contaminated
surgery.
Reference:
Https://books.google.com.sa/books?Id=n9y58-950vyc&pg=PA543&lpg=PA543&dq#v=onepage&q&f=false
Http://www.cdc.gov/hicpac/SSI/table7-8-9-10-SSI.html
49. During a cholecystectomy, there was an injury to the cystic artery. The surgeon applied pressure on the free margin of the
lesser omentum. What is the name of the artery compressed in this maneuver?
A. The right gastric artery
B. Coeliac
C. Hepatic
D. Splenic
E. Gastro-epiploic/duodenal

Answer: C

17
Reference: Farquharson's Textbook of Operative General Surgery, 10th Edition and Wikipedia.

50. 30 years pt with cyclic breast pain and tenderness ..pain is relieved after the cycle..
By examination there is periaerular nodular mass 3cm ..what is invx?
A. FNA
B. mammogram followed by US ( newly added )
C. u/s
D. follow her to next period CT scan ( newly added )
Answer: U/S

The first line of investigation is imaging not cytology nor tissue biopsy, So that excludes FNA.
Because the pt is young (30 yr old) U/S is more sensitive. Because of the tissue density.
Refereence: Baily & Love

51. Chronic alcoholic wake up at 3 am with abdominal pain after a little while the pain generalized all over the abdomen and is
severely exacerbated with movement:
A. Peptic ulcer perforation
B. Cholecystitis
C. Diverticulitis
D. Pancreatitis

Answer : D
-Alcohol do NOT cause ulcers. They delay the healing of ulcers
-Alcohol is a major cause of acute and chronic pancreatitis
Refrence: https://www.pancreasfoundation.org/patient-information/acute-pancreatitis/acute-pancreatitis-diagnosis-and-
treatment/

52. 18-year-old healthy male was playing baseball and suddenly he felt abdominal pain. On examination he has para-umbilical
mass. His vital signs are: BP 100/76, RR 30, HR 100, O2 sat is 95% with 100% oxygen mask. What is your next step in the
management?
A. Abdominal US.
B. CT scan.
C. Erect Chest x-ray.
D. Reassure and send home.

Answer : C ?
Intestinal obstruction due to hernia
Erect chest xray looking for air fluid level
Reference : Toronto notes

53. If there was inferior mesenteric artery thrombosis. Which artery will not be affected!
A. Descending colon *IMA
B. Sigmoid *IMA
C. Splenic *IMA
D. Cecum *SMA
Answer: D
Referenec: http://radiopaedia.org/articles/inferior-mesenteric-artery

18
54. Bilateral breast cancer is associated with :
A. Paget disease
B. Lobular carcinoma
C. Mucinus carcinoma
D. Ductal carcinoma
Answer: B
Invasive Lobular carcinoma is the second most common cause of breast cancer. Complication of ILC in addition to invading the
breast tissue is that it can spread to the lymph nodes, the other breast and possibly to other areas of the body.
Reference: http://www.breastcancer.org/symptoms/types/ilc

55. What is the best drug given to prevent postoperative thromboembolism?


A. LMW heparine
B. UF heparine
C. Warfarin
D. Enoxaparin
Answer: D

Reference: https://www.surgeons.org/media/19373/VTE_Poster.pdf
19
56. Patient having Fecal incontinence, which level of the spinal cord is affected:
A. Above C2
B. Below C2
C. Above T12
D. Below T12

Answer: D
Pudendal nerve (s2-4) is responsible for innervation to pelvic floor muscles and external anal sphincter
Reference: http://www.nafc.org/spinal-cord/

57. patient post lap choly develope sob cough respiratory distress What's dx
A. PE
B. Stroke
C. Pneumonia
D. Atelectasis.

Answer: D
It is a common pulmonary complication in patients following thoracic and upper abdominal procedures. General anesthesia and
surgical manipulation lead to atelectasis by causing diaphragmatic dysfunction and diminished surfactant activity.
PE is one of the correct options if the surgery is prolonged like in joint replacements (so unlikely after lap chole) and usually present
5-7 days after surgery, unlike atelectasis which develop one day after surgery.
Reference:
http://emedicine.medscape.com/article/296468-overview.

58. 17 years old boy with acute appendicitis, appendectomy was done. If you take it under microscope, which of the following
will be found?
A. Neutrophils in muscularis propria.
B. Mucus filled lumen.
C. Neoplastic tumor at the tip.

Answer: A
Microscopy demonstrates neutrophil infiltrate of the mucosal and muscularis layers extending into the lumen.
Reference: http://emedicine.medscape.com/article/773895-workup#c20
Neutrophils extend into and through the wall of the appendix in a case of acute appendicitis. Clinically, the patient often presents
with right lower quadrant abdominal pain. Rebound tenderness of the right lower quadrant is often noted on physical examination,

20
as well as positive obturator or psoas sign. An elevated WBC count is usually present.
Http://library.med.utah.edu/webpath/GIHTML/GI058.html

59. mass in neck in cervicle the best intial test?


A. Laryngioscope
B. CT
C. FNA
D. Biopsy

Answer: C
Reference: https://www.med.unc.edu/surgery/education/files/articles/Neck%20Mass.pdf

60. Case of female with Hx of gallstones in the gallbladder she had 2 Attack which relived her attack by analgesia:
In the CT report: Low density 0.3 cm calculi in the gallbladder. What you will prescribe:
A. Imitadiben (lipid lowering agent )
B. Simvastarin
C. Cholydoxcoic
D. Urodoxcholic

Answer: D
Explanation: For patients who decline surgery or who are at high surgical risk (eg, because of concomitant medical disorders or
advanced age), gallbladder stones can sometimes be dissolved by ingesting bile acids orally for many months. Ursodeoxycholic acid
dissolves 80% of tiny stones < 0.5 cm in diameter within 6 mo.
Reference: http://www.merckmanuals.com/professional/hepatic-and-biliary-disorders/gallbladder-and-bile-duct-
disorders/cholelithiasis

21
61. Self-breast examination decrease breast cancer by years:
A. One year.
B. Two years.
C. Three years.
D. Four years.
Answer: ?

62. Colon Cancer stage B2 means?


A. Mets to LN
B. No LN involvement
Answer: B

A- invasion of submucosa
B1- invasion of muscularis propria
B2- invasion through wall
C- positive lymph node
D- Distant metastasis

Surgical recall

63. Most specific predictor of local recurrence in breast cancer.


A. size.
B. ln mets. lymph node number
C. estrogen receptors
D. progest receptors
Answer: B
Several clinical and histopathologic factors, such as young age and presence of ductal carcinoma in situ, are known to be predictors
for local recurrence after breast conserving therapy (BCT). After mastectomy, lymph node status and tumor size are dominant risk
factors for local recurrence.
Link: https://www.researchgate.net/publication/221884330_Predictive_Factors_for_Local_Recurrence_in_Breast_Cancer

22
64. VMA pateint hemodynamic collapse what is warning sign can present when he collapse?
A. Hypotension
B. Rised jvp
C. Deviation of trachea
D. Resistance of ventlitor

65. Post RTA in ICU present with significant blood loss hypotension now in multi organ failure what is the most organ causing
other organ failure?
A. Heart
B. Lung
C. Kidney
D. Liver

66. Scenario of pt come with fractured rub what is your next action
A. Echo and cadilogy consltant
B. X – ray
C. Picardiocynteysis more info is needed

67. Best diagnostic method for diagnosing breast cyst is:


A. US
B. Mammogram
C. CT
Answer: A

68. Patient came to u her mother and her sister diagnosed with breast cancer, came for counseling, what to do now?
A. BRCA?
B. MRI
C. PET scan

Answer: US and mammogram if present + genetic counseling

69. Male after prostatectomy through venous plexus the metastasis will go to:
A. Skull
B. Vertebral body
C. Lung
Answer: B
Prostatic venous plexus drains into the internal iliac vein which connects with the vertebral venous plexus, this is thought to be the
route of bone metastasis of prostate cancer.

70. Psychiatric patient swallowed 2 safety pins, found on duodenum what to do?
A. Immediate laparotomy
B. Admit and observe
C. Charcoal

Answer: B

23
71. Female with right upper abdomen pain and fever no jaundice.. What is the management :
A. Emergent surgery.
B. IV fluid and antibiotics.
C. Discharge
Answer: A
Definitive treatment: Surgery within 3 days.
Initial management: IV fluids and Antibiotic.
Source: Toronto Notes 2014

72. Patient with abdominal trauma, after stabilization CT was done and there is splenic injury grade 1. How you will manage?
A. Conservative in surgical ward
B. Conservative in ICU AV.
C. Immediate laparotomy AW. Splenectomy

Answer: B
The diagnosis is confirmed with CT in stable patients and with bedside (point of care) ultrasonography or exploratory laparotomy in
unstable patients.
Patients who remain stable are transferred from the ICU after 12 to 48 h of observation, depending on the severity of their other
injuries, and are discharged after becoming mobile and tolerating diet. Http://www.msdmanuals.com/professional/injuries-
poisoning/abdominal-trauma/splenic-injury

73. A patient was stabbed in his abdomen, o/e he’s vitally stable, and some of the mesentery is out. What will u do
A. Exploratory laparotomy
B. Wound exploration
C. Observe
Answer: A

By definition, an exploratory laparotomy is a laparotomy performed with the objective of obtaining information that is not available
via clinical diagnostic methods. It is usually performed in patients with acute or unexplained abdominal pain, in patients who have
sustained abdominal trauma, and occasionally for staging in patients with a malignancy.
Http://emedicine.medscape.com/article/1829835-overview
74. First degree spleen injury?

o Diagnosis
Imaging (CT or ultrasonography)

24
The diagnosis is confirmed with CT in stable patients and with bedside (point of care) ultrasonography or
exploratory laparotomy in unstable patients.
o Treatment
Observation
Angioembolization
Sometimes surgical repair or splenectomy
Hemodynamically stable patients with low-grade (I to III) blunt or penetrating splenic injuries without any evidence for other
intra-abdominal injuries, active contrast extravasation, or a blush on CT, may be initially observed safely .

- Hemodynamically unstable – Based upon ATLS principles, the hemodynamically unstable trauma patient with a positive FAST
scan or DPA/DPL requires emergent abdominal exploration to determine the source of intraperitoneal hemorrhage .

Reference: http://www.uptodate.com/contents/management-of-splenic-injury-in-the-adult-trauma-
patient?Source=outline_link&view=text&anchor=H9#H9

75. You performed a pudendal nerve block on a woman in labor, which of the following structures will be fully sensitive and not
blocked by the anesthesia?
A. Perineal body
B. Urogenital diaphragm
C. Rectum
Answer: C perineum, vulva, scrotum/vagina those are blocked.
UpToDate

76. Surgeon take graft from rectus muscle which artery should be dissected?
Other version:
(In patient post mastectomy they do for her reconstruction from the rectus muscle what is the vessels may be injured or effected)
A. Superior epigastric
B. Inferior epigastric
C. Superficial epigastric
Answer B The deep superior epigastric vessels are not used as the pedicle for the free flap because it is of smaller caliber than the
inferior vessels and a greater amount of skin can be harvested with the inferior system. Medscape

77. Child with history of comminuted clavicular # due to MVA , treatment :


A. Arm sling
B. Close reduction
C. ORIF (open reduction internal fixation)
Answer: C.
Http://www.orthobullets.com/trauma/1011/clavicle-fractures

78. Case of trauma patient unconscious e abdomen distension what initial treatment:
A. CT
B. IV fluid
C. Fast
Answer: B
In hemorrhagic shock, in trauma patients, first start with IV fluids to stabilize the patient. Kaplan surgery trauma chapter

79. Nerve supply to tensor tympani and stapedius:


A. Trigeminal and facial

25
B. Facial and auditory
C. Trigeminal and ....
Answer: A
Tensor tympani is innervated by the mandibular division of the trigeminal nerve (V), through the trigeminal ganglion. Stapedius is
innervated by the tympanic branch of the 7th nerve. Http://www.dizziness-and-balance.com/anatomy/ear/ema.html

80. Headach , pain in Rt eye ,Halos around light :


A. Hyphema
B. Digoxin intoxication
C. Acute angle Glucoma
It is because of pain and halos. They do happen in digoxin toxicity but without pain and bilateral.
Http://emedicine.medscape.com/article/798811-clinical

81. Liver biopsy :


A. Midaxillary at 6th intercostal space
B. Midaxillary at 7th intercostal space
th
C. Midaxillary at 10
Answer: B
Http://emedicine.medscape.com/article/149684-technique

82. Low intestinal obstruction, 1st symptom?


A. Constipation
B. Flatus
C. Distention

Answer: A,C
Mont Reid:
• Distal Intestinal Obstruction: Obstipation\constipation and distention may occur before feculent emesis. A cramping pain
referred to lower abdomen. Obstipation is characteristic of complete obstruction.
• Proximal Intestinal Obstruction: bilious emesis early in course, minimal distention. May still have bowel movements, pass
gas while moving bowels distal to obstruction. A cramping pain referred to periumbilical region.

26
83. Patient suffered a Road Traffic Accident came to ER after 30 mins. On examination, in the chest there's sound bilateral +
distended neck. Diagnosis?
A. Hemothorax.
B. Pneumothorax.
C. Cardiac Tamponade.

Answer: A

Toronto:
Becks triad: Hypotension, Distant Heart Sound, Distended Neck Veins.

84. 70 years old man with back pain & urinary symptom. Lab shows PSA= 40, ALP= high, LFT= normal
A. Benign prostatic hypertrophy (BPH)
B. Prostatic cancer with metastasis
C. Renal cell cancer

Answer: B
A normal PSA value ranges from 0 to 4 micrograms/L, in BPH the PSA is slightly elevated while in late stage malignancy it’s more
than 20 micrograms/L and signs of metastasis will appear above this threshold (in this case bone metastasis)
A positive prostate biopsy is the most sensitive and specific test to differentiate between prostate cancer and BPH.
Http://bestpractice.bmj.com/best-practice/monograph/254/diagnosis/tests.html
Http://bestpractice.bmj.com/best-practice/monograph/254/treatment/step-by-step.html

85. There was injury to the head exactly to the jagular foramen ,so which of the following muscle is intact ? ‫يعني كل العضالت مصابه‬
‫ ماعدا ؟ في األسئله حقتهم هنا جايبن االكسبت لكن بطريقه‬EXCEPT ‫ويقولون الهيئه مايجيبون الـ‬

A.Staylopharygeus
B. Styloglossus
C. Sternocleidomastoid
Answer: B

86. Loss of taste sensation of the anterior 2/3 of the one side of the tongue and decreased hearing on the ear on the same side ,
so where is the level of the injury of the facial nerve ?
A. After the nerve to stapidial muscle
B. Proximal to greater petrosal nerve
C. Distal to greater petrosal nerve

Answer: C

87. Post op prophylactic against DVT?


A. LMWH
B. Unfractionated heparin
C. Warfarin

Answer: “B” if Moderate risk patient, “A\B” if high risk patient.

Toronto:

27
88. MVA unconscious patient in ER, with difficulty in ventilation with ambubag-mask what you will you do?
A. Exaggerate jaw thrust
B. More head tilt
C. Go directly for intubation
Answer: Most likely C

89. Young female noticed a mass 1-week after her menstrual cycle. It persists for three cycles; the mass is rubbery and mobile.
Diagnosis?
A. Fibroadenoma
B. Fibrocystic changes
C. Duct ectasia
Answer: A
Http://emedicine.medscape.com/article/345779-overview

90. Patient had a RTA on x-ray there was multiple forehead fractures, and there is nasal discharge, what nerve is affected?
A) Optic
B) Olfactory
C) Ophthalmic

Not sure see below

The most common anatomic sites of spontaneous cerebrospinal fluid (CSF) leaks are the areas of congenital weakness of the
anterior cranial fossa and areas related to the type of surgery performed. The lateral lamella of the cribriform plate appears to be
involved in approximately 40% of the cases, whereas a defect in the region of the fontal sinus is detected 15% of the time. The sella
turcica and sphenoid sinus are involved in 15% of the cases as well.
Common sites of injury secondary to endoscopic sinus surgery include the lateral lamella of the cribriform plate and the posterior
ethmoid roof near the anterior and medial sphenoid wall.

Cerebrospinal fluid rhinorrhea in the acute phase after trauma has been reported in as many as 39% of the patients with skull base
[104]
fractures. Patients present with a variety of symptoms depending on the acuteness of the event. In the acute phase following the
traumatic event, patients may present with epistaxis, nasal discharge, periorbital ecchymosis, chemosis, oculomotor impairment,
anosmia, motor deficit, open-head injury with CSF leakage, loss of vision, cranial nerve deficits (most frequently, first–third and
[104,118]
fifth–seventh cranial nerve injuries), meningitis, and pneumocephalus. In the chronic phase, patients may present with
[59] [23]
recurrent meningitis, intermittent nasal discharge, headaches, salty or sweet taste in the retropharyngeal space, hyposmia, and
brain abscess.
91. Young boy presented with diarrhea sometimes bloody, Weight loss, arthritis, anemia the diagnosis is:
A. Crohns
B. UC
C. Celiac
28
Answer: b
Rectal bleeding is the hallmark feature of UC, however diarrhea may be present if more than the rectum is involved. Extra-intestinal
manifestation: Erythema nodosum, Peripheral arthritis Ankylosing spondyliti – Toronto notes 2015

92. Which of the following types has the highest risk for developing strangulated hernia?
A) Direct inguinal
B) Indirect inguinal
C) Femoral
Answer: C
The rates of strangulation were 22 and 45 percent at 3 and 21 months, respectively, for femoral hernias, compared with 2.8 and
4.5 percent for inguinal hernias. Http://www.uptodate.com/contents/overview-of-treatment-for-inguinal-and-femoral-hernia-in-adults

93. Pt. With high intestinal obstruction, what will be the symptom?
A) Absolute constipation
B) Diarrhea
C) Change of habit
Answer: should be vomiting, if it's not in options it maybe B
Nausea, vomiting - Associated more with proximal obstructions
Diarrhea - An early finding. Constipation - A late finding, as evidenced by the absence of flatus or bowel movements.
Http://emedicine.medscape.com/article/774140-clinical
Small gut obstruction produces effects which differ according to the level at which it occurs. The higher the obstruction the
earlier and the worse the patient's vomiting, and the greater the threat to his life from electrolyte imbalance but the less his
distension. Conversely, the lower the obstruction the greater his distension, the greater his pain, and the later he starts to
vomit. Http://www.meb.uni-bonn.de/dtc/primsurg/docbook/html/x3146.html

94. Pt. Comes with sign of intestinal obs and decrease pr may be absent of bowel sound Dx?
A) Pancreatitis
B) Cholecystitis
C) Myocardial infarction
Answer: A
Pancreatitis: Abdominal tenderness, muscular guarding (68%), and distention (65%) are observed in most patients; bowel
sounds are often diminished or absent because of gastric and transverse colonic ileus.

95. Patient present with hypotension his phosphate in normal level after one day her phosphate level decrease. What organ
damage:
A- liver
B- kidney
C- lung
Answer: B

96. During adrenalectomy you injured structure passing anterior to it:


A. Inferior vena cava
B. Duodenum
C. Rectum

ANSWER: A
- Anterior to right Supra renal Gland is: part of the right lobe of the liver + Inferior vena cava
- Anterior to the Left supra renal gland is: part of the stomach and Pancreas
- Part of diaphragm is posterior to both glands
- GRAY'S ANATOMY FOR STUDENTS

29
97. Women with mastitis:
A. Stop breast feeding
B. Clean nipple with alcohol
C. Surgical drainage

Answer: question and choices are not complete


- Mastitis is a cellulitis of the periglandular breast tissue
- Caused by nipple trauma from breastfeeding coupled with the introduction of bacteria, usually Staphylococcus aureus, from
the infant’s pharynx into the nipple ducts.
- Symptoms often begin 2–4 weeks postpartum
- Symptoms are usually unilateral and include the following: Breast tenderness. \ Palpable mass \ Erythema, edema, warmth,
and possible purulent nipple drainage.
- Treatment:
1- Continued breastfeeding to prevent the accumulation of infected material
2- Oral antibiotics effective against penicillin-resistant staphylococci (dicloxacillin, cephalexin, amoxicillin/clavulanate,
azithromycin, clindamycin).
3- If not clinical improvement within 48–72 hours, evaluate with breast ultrasonography to assess for abscess. If present, treat
with incision and drainage.

- First Aid Step 2CK


98. Abdominal solid mass (renal I guess but not sure) confirmed by:
A. CT
B. MRI
C. US

ANSWER: A
- Both US and CT usually demonstrate the organ from which a mass arises, But CT is more accurate than US
- MRI may be used to evaluate complex lesions not definitely characterized by US or CT. MRI excels in specifically
characterizing fat, protein, fluid, blood products, vascularized tissue, and metal.
- Https://acsearch.acr.org/docs/69473/Narrative/

99. Patient male 50s i think with chronic epigastric abd pain , last wk lost 7 kg +ve stool occult blood
A. Chronic pancreatitis
B. Chronic cholecystitis
C. Chronic gastritis
Answer: i think due to significant wt loss should think about malignencey .. But the case is not clear .

100.Male with right lower hypochondrium pain , in surgical exploration they found inflamed appendix , what is the congenital
malformation responsible for this :
A- reverse rotation
B- non rotation
C- behind liver ..

Answer: B
Explanation: Intestinal malrotation, also known as intestinal nonrotation or incomplete rotation, refers to any variation in this
rotation and fixation of the GI tract during development. Appendectomy is performed during operation for malrotation because the
normal anatomical placement of the appendix is disrupted.
Reference: http://emedicine.medscape.com/article/930313-overview

30
101.Elderly with acute urine retention:
A- transurethral prostatectomy
B- partial prostatectomy
C- foley catheter and urine culture
Answer: C
Explanation: The overriding therapeutic goal is reestablishment of urinary flow. Before specific therapy for obstruction is initiated,
the life-threatening complications of obstructive uropathy must be investigated and treatment started. Once urinary obstruction is
under consideration, a transurethral bladder catheter should be placed
Reference: http://emedicine.medscape.com/article/778456-overview

102.Old patient c/o increase urination at night and weak urine stream. Lab result show PSA 1 ? What is your diagnosis?
A- Prostitis
B- BPH
C- prostatic cancer

Answer: B

Explanation: usually fever, lower back or suprapubic pain, or tender rectal exam is more consistent with prostitis. In BPH, Symptoms
include frequency, urgency, and nocturia due to incomplete emptying, In addition to hesitancy and intermittency. If the PSA level is
> 4 ng/ml, then a transrectal biopsy is recommended to exclude prostate cancer.
Reference: http://www.merckmanuals.com/professional/genitourinary-disorders/benign-prostate-disease/benign-prostatic-
hyperplasia-bph
103.Male patient came to ER with stab wound and hypotension what is your next step?
A. Fresh frozen plasm
B. IV ringer lactate
C. Packed RBC

Answer: B
Explanation: According to ATLS guidelines, trauma patients are managed by following ABC (Airway, breathing & circulation) in
primary survey. In Circulation, apply direct pressure to any bleeding wound, establish 2 16 gauge IV line in each antecubital fossa
then administer isotonic fluid either normal saline or ringers lactate with 3:1 ratio (fluid to blood loss ratio) & start with 2L initially.
Only if the patient remains unstable packed RBC is considered, and the ratio is 1:1.
Reference: First Aid Step 2Ck.

104.Characteristic of perforated duodenal ulcer?


A. Mid-epigastric pain
B. Steatorrhea
C. Melena

Answer: A
• Symptoms include sudden, severe abdominal pain, a rapid heartbeat, and a low body temperature. Pain may radiate to one
or both shoulders, and the abdomen may become rigid. The abdominal pain is usually sudden, sometimes producing
collapse or syncope. Localization is usually epigastric, but it quickly becomes generalized.
Reference: Uptodate

105.If a breast lump felt by self-exam for how many years it's there?
A. 1
B. 2
C. 4

Answer: C
31
Source: No clear source. References generally indicate that it is between 2 and 5 years. Mammograms can detect breast CA 2 years
before being felt as a lump.
106.Which vitamin can prevent or reduce risk of colorectal cancer? 3 times
A. Folic acid
B. Vit.D
C. Vit.E
Answer: B
Reference: UpToDate
Http://www.uptodate.com/contents/colorectal-cancer-epidemiology-risk-factors-and-protective-
factors?Source=outline_link&view=text&anchor=H19#H19
107.decrease colorectal cancer
A. Vitamin
B. Vitamin E
C. Folic acid
Answer:
- Folic acid decrease the risk of colorectal cancer .
- Vit D act as inhibtor of colorectal cancer by influence the intiation and progrestion .
-also, "higher intake of vitamin B6 was associated with lower risk of colorectal cancer"

Reference:http://www.hopkinscoloncancercenter.org/CMS/CMS_Page.aspx?Currentudv=59&CMS_Page_ID=1293D614-71B1-4A5A-
8CFD-7BF8760295FA

Vit B6
Medscape
Http://www.medscape.org/viewarticle/506337
uptodate
http://www.uptodate.com/contents/colorectal-cancer-epidemiology-risk-factors-and-protective-
factors?Source=outline_link&view=text&anchor=H20#H20

108.A patient with achalasia. Repeated attempt of balloon expansion relapses again. What is the management?
A. Stent insertion
B. Myotomy
C. Nasogastric tube
Answer: B
Reference: Surgical Recall, 6th edition
109.24 years old girl came to you with a painless mobile breast mas that does not change with menstrual cycle. It started
increasing since 3 months. Now it’s 3 cm in size. What is the most likely cause?
A. Fibroadenoma
B. Fat tissue
C. Cyst

Answer: A
➢Fibrocystic: changes with menstrual cycles (hormones), lumps in both breasts
➢Fibroadenomas. These are the most common benign tumors. They are solid, round, rubbery lumps that move freely. They’re
usually painless.
➢Simple cysts: Simple cysts are fluid-filled sacs that usually happen in both breasts. There can be one or many. They can vary in
size. Tenderness and size often change with menstrual cycle.
➢Intraductal papillomas: These are small, wart-like growths in the lining of the mammary duct near the nipple. They usually affect
women who are 45 to 50. They can cause bleeding from the nipple.

32
110.45 years old female presented with neck swelling and anxiety. On examination the swelling is moving with swallowing and
lateral to midline. What is the most likely diagnosis?
A. Thyroglossal cyst
B. Branchial cyst
C. Thyroid nodule
Answer: C
th
Reference: Surgical Recall, 6 edition

111.A patient received clindamycin before surgery. 3rd day post-op patient developed watery diarrhea. What test will you order?
A. Stool ova and parasite
B. EIA for C.difficile
C. Viral

Answer: B The primary risk factor for C difficile colitis is previous exposure to antibiotics; the most commonly implicated agents
include the cephalosporins (especially second and third generation), the fluoroquinolones, ampicillin/amoxicillin, and clindamycin
Reference: http://emedicine.medscape.com/article/186458-overview#a5

112.(long scenario) lactating women 10 days after delivery complaining of fever and rigors. On examination: tender left breast
and nodules in upper outer area (investigations result included). What is the most likely diagnosis?
A. Postpartum sepsis.
B. Breast abscess.
C. Inflammatory breast cancer.
Answer: B
Postpartum fever is defined as a temperature greater than 38.0°C on any 2 of the first 10 days following delivery exclusive of the first
24 hours. (which is not clearly met here).
Reference: http://emedicine.medscape.com/article/796892-overview#showall

113.Patient have blunt in right 4 intercostal space. Which of the following is most likely affect affected?
A. Upper lobe of lung
B. Lower lobe of lung
C. Horizontal

Answer: C
The horizontal fissure arise from the right oblique fissure and follow the fourth intercostal space from the sternum until it meets the
oblique fissure as it crosses right 5th rib. Reference: http://radiopaedia.org/articles/horizontal-fissure

114.(long scenario) man with solid thyroid nodule, what is the most appropriate thing to do?
A. Incisional biopsy.
B. Excisional biopsy.
C. Fine needle aspiration.

Answer: C
Fine-needle aspiration biopsy is used for definitive diagnosis. FNAB has attracted much attention in the adult population in the
evaluation of thyroid nodules. Reference: http://emedicine.medscape.com/article/924550-workup#c5

115.indirect hernia related to spermatic cord ?


A. Anterio medial
B. Posterio medial
C. Something another something
Answer: A

33
http://fitsweb.uchc.edu/student/selectives/Luzietti/hernia_inguinal_indirect.htm

116.Patient came with history of upper abdominal pain, the investigation showed high amylase levels, your diagnosis will be:

A. Acute pancreatitis.
B. Gastric ulcer.
C. Acute cholecystitis.

Answer is: A
Amylase is an enzyme produced by your pancreas and salivary glands.
The pancreas can sometimes become damaged or inflamed, which causes it to produce too much or too little amylase. An abnormal
amount of amylase in your body may be a sign of a pancreatic disorder.
Reference: http://www.healthline.com/health/amylase-blood
http://bestpractice.bmj.com/best-practice/monograph/66/diagnosis/tests.html

117.which of the following has strong association for colorectal cancer:

A. Familial Adenomatous Polyposis (FAP)


B. Ulcerative colitis
C. Crohn’s disease

Answer is: A
More than 95% of people with FAP will have multiple colon polyps by age 35. If FAP is not recognized and treated, there is almost a
100% chance that a person will develop colorectal cancer.
Reference: http://www.cancer.net/cancer-types/familial-adenomatous-polyposis
118.Patient has deep ulcer with erythema on the sole of the foot, he is a known case of uncontrolled DM. What is the treatment?
A. Give oral antibiotics
B. Admit the patient for debridement and deep culture
C. Perform superficial culture

Answer: B
- Adequate debridement, proper local wound care (debridement and dressings), redistribution of pressure on the ulcer by
mechanical off-loading, and control of infection and ischemia (when present) are important components of treatment for all ulcers,
regardless of stage and depth.

- For most patients with diabetic foot ulcers, we suggest surgical (sharp) debridement rather than another method .
Reference: http://www.uptodate.com/contents/management-of-diabetic-foot-
ulcers?Source=outline_link&view=text&anchor=H28#H28

119.Patient did gastrectomy and need to take vitamin B12 for life ,which cells are responsible: (Repeated in Basic Science)
A. Goblet cells
B. Chief cells
C) Parietal cells
Answer: C
Parietal cells are responsible for the secretion of intrinsic factor. Under normal circumstances intrinsic factor binds to vitamin B12
and assists with the absorption of this vitamin in the lower portion of the small bowel. When vitamin B12 is poorly absorbed, anemia
and, in some cases, poor nerve function can occur.
Reference: http://my.clevelandclinic.org/health/diseases_conditions/post-gastrectomy-syndrome-overview

34
120.Patient underwent supradrelenectomy, you are worried not to injure which organ:
A. Inferior vena cava
B. Kidney
C. Appendix
Answer: B
On the left side, the spleen is the most commonly, The tail of the pancreas and the blood vessels supplying the kidney are also at
risk.
On the right side, the liver and duodenum (first part of the small intestine) are also at risk of injury.
Http://m.endocrinediseases.org/adrenal/surgery_complications.shtml

121.Case of RLQ pain and mass, what is Tx :


A. Conservative.
B. Surgery.
C. Antibiotics.
Answer: C , Inflamed Appendicular mass is either a phlegmon or an abscess , both are treated non surgically by antibiotics ( in some
abscess cases by US guided drainage ).

122.gastrectomy which one will be defecient


A. Lipase
B. Pepsin
C. Trypsin
Answer: B
Reference: http://rfwdata.net/USMLEQBANK/block17questions.html

123.painful axillary lump with tender and erythematous with black head papule and large pores?
A. Local antibiotic
B. Oral antibiotic
C. Surgery

Answer: C
It is most probably epidermoid cyst , if not infected it will resolve spontaneously but recurrence in high , if fluctuating I and D is
indicated . It's removed via simple excision or incision with removal of the cyst and cyst wall though the surgical defect.
Reference ; up to date, medscape
http://www.uptodate.com/contents/overview-of-benign-lesions-of-the-
skin?Source=outline_link&view=text&anchor=H1101420445#H1101420445
Http://emedicine.medscape.com/article/1061582-treatment#d8

124.Single thyroid nodule iodine sensitive. Best management?


A. Lobectomy
B. Radio iodine therapy
C. Anti thyroid drug
Answer: B
Hot nodules secrete thyroid and should be ablated using radioactive iodine or surgery

35
Reference :american family physician
125.Pt co of infected wound ulcer ,intact pulse ,no improve by AB & pt known uncontrolled DM2 •
A. Amputation
B. Antibiotics
C. Surgical debridement

Answer: B
A good clinical response for mild to moderate infections can be expected in 80%–90% of appropriately treated patients [10, 50] and,
for deeper or more extensive infections, in 50%–60% [64, 86]. When infection involves deep soft-tissue structures or bone, more
thorough debridement is usually needed. Bone resections or partial amputations are required in about two-thirds of this patient
group. Most of these amputations can be foot sparing, and long-term control of infection is achieved in >80% of cases. Infection
recurs in 20%–30% of patients, many of whom have underlying osteomyelitis. Factors that predict healing include the absence of
exposed bone, a palpable popliteal pulse, toe pressure of >45 mm Hg or an ankle pressure of >80 mm Hg, and a peripheral WBC
count of <12,000/mm3 [19]. The presence of edema or atherosclerotic cardiovascular disease increases the likelihood of
amputation. Amputation may be more often required for patients with combined soft-tissue and bone infection than for patients
with either type of infection alone [86]. Patients who have had one infection are at substantial risk of having another within a few
years; thus, educating them about prevention techniques and about prompt consultation when foot problems occur is critical.
Reference: http://cid.oxfordjournals.org/content/39/Supplement_2/S104.full

126.case head trauma on parietal lobe subdural hematoma which artery is injured ?
A. Superficial temporal .
B. Mid cerebral
C. Rt.cerebral .......

Answer: B

The cortical branches of the MCA supply the lateral surface of the hemisphere, except for the medial part of the frontal and the
parietal lobe (anterior cerebral artery), and the inferior part of the temporal lobe (posterior cerebral artery)
http://www.radiologyassistant.nl/en/p484b8328cb6b2/brain-ischemia-vascular-territories.html
36
127.pt noticed lump for three months the mass freely mobile no discharge not related to menstrual?
A. Cystic
B. Fibroadenoma
C. Ductal papilloma

Answer: B
Reference: http://www.mayoclinic.org/diseases-conditions/fibroadenoma/basics/definition/con-20032223

128.old patient presented with congested neck veins bilateral upper limb swelling tumor in the right upper lung which part it's
compressing?
A. Superior mediastinal
B. Anterior mediastinal
C. Posterior mediastinal
Answer: A
This patient has superior vena cava (SVC) syndrome due to lung mass compression
The SVC is located in the anterior right superior mediastinum
Reference: http://www.ncbi.nlm.nih.gov/pmc/articles/PMC4093359/

129.Best prognostic indicator for breast cancer?


A. Estrogen receptor
B. Stage and grade
C. Involvement of axillary LN
Answer: C
The most significant prognostic indicator for patients with early-stage breast cancer is the presence or absence of axillary lymph
node involvement.
http://theoncologist.alphamedpress.org/content/9/6/606.full

130.Child with history of severe pain in testis, no history of trauma what is the best next step?
A. Surgery consultation
B. Urine culture and analysis
C. Analgesics and ice

Answer: b (not sure)


Reference : http://www.aafp.org/afp/1999/0215/p817.html

37
131.Newborn with tender bilateral scrotal swelling what is the next investigation :
A. Illumination
B. Wait till patient cough or cry
C. Watch voiding

Answer: ?

132.Best imaging to visualize breast mass:


A) Mammogram
B) US
C) CT
Answer: A
I looked it up and got confused but it is either Mammogram or breast MRI. According to this article which reviewed the literature, it
is mammogram.
Reference: http://citeseerx.ist.psu.edu/viewdoc/download?Doi=10.1.1.409.2944&rep=rep1&type=pdf (last paragraph in the
conclusion part)

133.Infant periumbilical hernia, what you will do:


A. Put plastic in mid abdomen
B. Reassurance
C. Do hernia repair before start school
answer: b
Umbilical hernias: Most umbilical hernias do not require surgical repair because they close by their own. Surgery is indicated when it
doesn’t close by the age of 5 or when it’s painful.
Http://emedicine.medscape.com/article/932680-treatment
Http://www.mayoclinic.org/diseases-conditions/umbilical-hernia/basics/treatment/con-20025630

134.48 yrs women, without any family history of Breast CA had Normal mammogram when to repeat it?
A. 2 yrs
B. 3 yrs
C. 5 yrs
Answer : ?
Annual screening for some women annual screening for some women (eg, premenopausal)
http://www.uptodate.com/contents/screening-for-breast-cancer-strategies-and-
recommendations?Source=outline_link&view=text&anchor=H79986#H79986

Women older than 45 with average risk should get mammogram every year!!
Http://www.cancer.org/cancer/breastcancer/moreinformation/breastcancerearlydetection/breast-cancer-early-detection-acs-recs

135.Testicular cancer with high AFP and BHCG


A. Seminoma (high beta-hcg)
B. Embryonal (high AFP+ beta-hcg )
C. Teratoma
Answer: B
Elevations in the serum concentration of AFP or beta-hcg cannot be attributed to teratomatous elements.
AFP: Elevation of serum AFP is seen in 40% to 60% of men with nonseminomas. Seminomas do not produce AFP.

38
Beta-hcg: Elevation of the beta subunit of hcg is found in approximately 14% of the patients with stage I pure seminoma
Approximately 40% to 60% of men with nonseminomas have an elevated serum beta-hcg.
Http://www.cancer.gov/types/testicular/hp/testicular-treatment-pdq
Http://www.uptodate.com/contents/anatomy-and-pathology-of-testicular-
tumors?Source=outline_link&view=text&anchor=H11#H11

136.Baby bout by his mother with abdominal distention, bad smell vomiting, history of delayed passege of muconime “after 3
days after using enema’ rectal examination showed empty rectum (typical history of hirschsprung's disease with pic of
abdominal x ray showing multiple air-fluid levels). And the doctors start to decompress his stomach. What is the definitive
treatment?
A. Leveling Colostomy
B. Total colectomy
C. Antibiotic
Answer: abdominoperineal pull-through in two or three stages, in which patients initially underwent a diverting colostomy
http://www.uptodate.com/contents/congenital-aganglionic-megacolon-hirschsprung-
disease?Source=outline_link&view=text&anchor=H13#H13

137.A woman has lichen sclerosis which cancer she will get:
A. Squamous cell carcinoma
B. Adenocarcinoma carcinoma
C. Adenosquamous carcinoma

Answer : A
Lichen sclerosus of anogenital sites is associated with an increased risk of vulval, penile or anal cancer (squamous cell carcinoma,
SCC)
Http://www.dermnetnz.org/immune/lichen-sclerosus.html

138.Question about bladder cancer after they removed what you will do next
A. Intravesical medication
B. Observe and serial assessment
C. And 2 another choices regarding treatment
Answer: ? Depend on stage

139.Surgery is contraindicated for which lung CA?


A. SCC
B. SCL
C. Adenocarcinoma
Answer: ?
I think B refers to small cell lung cancer (SCLC) which is the right answer.

140.Para follicular cell neoplasm? Which tumor?


A. Papilary thyroid cancer
B. Follicular thyriod ca
C. Others
Answer: Medullary ca
Medullary thyroid cancer originates from the parafollicular cells (also called C cells) of the thyroid.
Http://www.endocrineweb.com/conditions/thyroid-cancer/thyroid-cancer-medullary-cancer

141.Resected end of small bowel and beginning of large colon >> deficiency in?
A. Fa

39
B. B12✔
C. Uc-..

Answer: B
Vitamin B12 is readily absorbed in the last part of the small intestine (ileum)
Http://www.merckmanuals.com/home/disorders-of-nutrition/vitamins/vitamin-b-12

142.Patient came with loose stools, history of loose stools before ,, Its watery like with mucous not containing blood ( forget the
other details ) what is you diagnosis:
A. IBS
B. Crohn’s disease
C. Ulcerative colitis
Answer: A – Kaplan

143.And half years old uncircumcised boy, toilet Trained , came with his parent with UTI
What is the best method to obtain urine culture?
A. Catheter
B. Clean catch
C. Suprapubic tap
Answer: B

A midstream, clean-catch specimen may be obtained from children who have urinary control (toilet trained). In the infant or child
unable to void on request, the specimen for culture should be obtained by suprapubic aspiration or urethral catheterization.

144.Old patient in 60s of age smoker presented with history of sore throat hoarseness of voices for 3 month with enlarged
cervical lymph nodes: on examination the was large mass in the throat ( the history was highly suggestive of larynx Ca ) what
is the most probably your action :
A. Saline water gargles
B. Analgesia and antibiotic
C. Surgery and radiotherapy
Answer: C

145.For LP you will insert the needle in:


A. L1- L2
B. L3 - L4
C. L5 - S1
Answer: B, L3-L4

146.Female with breast tender nodule 3*4 in the upper outer quadrant , what is the Next step?
A. FNA and cytology
B. Follow up
C. Reassurance
Answer: A - Surgical recall

147.Patient presented with dysphasia, he has hoarseness and excessive salivation and feels hbat there is a lump in his throat,
what is the dx?
A. Achalisia
B. Diffuse esophageal spasm
C. Cricopharengeal dysfunction
Answer:

40
Most probable esophagus ca
Http://emedicine.medscape.com/article/277930-clinical

148.Obstruction: abdominal pain and vomiting, HD stable, on x-ray dilated loops and air in rectum, what will you do?
A. Rectal decompression and IV antibiotics
B. Nasogastric tube and IV NS/Antibiotic
C. Nasogastric tube and …
D. (last option I think was observe)
- (when you have air/gas in rectum + dilated loops > think paralytic ileus)
Answer: c if (NGT and IV saline)
most cases of postoperative ileus resolve with watchful waiting and supportive treatment. Patients should receive intravenous
hydration. For patients with vomiting and distention, use of a nasogastric tube provides symptomatic relief; however, no studies in
the literature support the use of nasogastric tubes to facilitate resolution of ileus. Long intestinal tubes have no benefit over
nasogastric tubes
Http://emedicine.medscape.com/article/2242141-treatment#d6

149.Case of diffuse abdominal pain, X-ray show dilated loops ?


A. Acute pancreatitis ?
B. Acute cholecystitis
C. Perforated duodenal ulcer
Answer: Acute pancreatitis may cause paralytic ileus.????
The most commonly recognized radiologic signs associated with acute pancreatitis include the following: 1) Air in the duodenal C-
loop, 2) The sentinel loop sign, which represents a focal dilated proximal jejunal loop in the left upper quadrant, and 3) The colon
cutoff sign, which represents distention of the colon to the transverse colon with a paucity of gas distal to the splenic flexure
Medscape http://emedicine.medscape.com/article/371613-overview#a2

150.Swilling test:
A. Torsion test
B. Cancer test
C. Orcico epidermitis
Answer:?

151.Lady with a mass in left upper quadrant of the breast, aspiration was yellow fluid without masses, Dx?
A. Phyllodem
B. ormal tissue variant
C. ..."no simple cyst"

152.A case of painful nodules in groin area and pain relieved after punching and discharge came out of it:
A. Hidradenitis suppurativa.
B. Furunculus
Answer: A http://dermnetnz.org/acne/hidradenitis-suppurativa.html
153.At inguinal canal base, what artery you can find?
A. Iliac
B. Femoral

Answer : if they ask about artery at the base i think it’s femoral , but if the ask about which artery pass through the canal the
answer will be 3 arteries: artery to vas deferens (or ductus deferens), testicular artery, cremasteric artery. But if they just asked

41
about the base of the canal the answer will be >> The Canal Boundaries :The anterior wall is formed by the aponeurosis of the
external oblique, and reinforced by the internal oblique muscle laterally.The posterior wall is formed by the transversalis fascia.The
roof is formed by the transversalis fascia, internal oblique and transversus abdominis.The floor is formed by the inguinal ligament
(a ‘rolled up’ portion of the external oblique aponeurosis) and thickened medially by the lacunar ligament
Http://teachmeanatomy.info/abdomen/areas/the-inguinal-canal/

154.Bacteroides in an abdomen gunshot wound, what antibiotics are most appropriate?


A. Clindamycin

B. Others options
Answer:
No specific agent is recommended for penetrating abdominal traumas, but it may be a single agent with beta-lactam coverage or
combination therapy with an aminoglycoside and clindamycin or metronidazole. Antibiotics include cefotetan, metronidazole
hydrochloride, gentamicin sulfate, vancomycin hydrochloride, & ampicillin sodium-sulbactam sodium.
Http://emedicine.medscape.com/article/2036859-medication#showall

155.40 years old female with 3 months mass 2.5 cm firm, mammogram and US normal what to do ?
A. Follow up after two cycles
B. Obtain biopsy
Answer: B
Source: answered by GS resident.

156.39 y.old p3+0 complete her family , history of left ovary and endometrial ablation ,complain of dysmenorrhea,..,... Now
another ovarian cyst 6-7cm Managment:
A. Hysterectomy+oophorectomy
B. Remove cyst+ablation affected endometrial
Answer

157.patient presents with signs of infectious mononucleosis with enlarged lymph nodes, when you were palpating the spleen he
became cold and pale. what to do next?
A. urgent gastroscope
B. fluids, urgent ct and abx

Answer: B
This seems like the presentation of splenic rupture, the patient should be stabilized and CT can be done for grading in the stable
patient or FAST scan in the unstable patient.
http://emedicine.medscape.com/article/432823-treatment

158.Why we use CT scan in trauma patient?


A. Can detect retroperitoneum hemorrhage.
B. You can see the hemorrhage with contrast

Answer: A
Medscape:
CT scans of the abdomen and pelvis usually are performed together, using both IV and oral contrast. Use this study to
identify injuries to abdominal and pelvic organs and to identify bleeding in the retroperitoneum and pelvis

159.Present to ER with severe headache and LP showed blood :


A. Ruptured berry aneurysm,
B. Epidural hematoma
Answer: A

42
160.12 years old football player presented to the ER with nausea and vomiting and tender scrotal swelling , what is the
management ?
A. Urology referral
B. Immediate surgery
Answer: B

161.Patient has got stab wound at the middle of posterior border of sternocleidomastoid muscle , afterthat the patient cannot
elevate his arm above his head , what is the injured nerve ?
A. Axillary nerve
B. Long thoracic nerve

Answer: A

162.Patient with enlargement of the urethra , what you will do ?


A. Annually Renal function test
B. Annually Prostatic specific antigen (PSA)

Answer: B
Patients with BPH who are on watchful waiting or are treated with medical or surgical therapy should undergo monitoring of
their clinical symptoms with intermittent administration of the International Prostate Symptom Score (IPSS). This is an accurate
and reliable way to monitor improvement or progression of symptoms in patients with BPH.
Patients between 40 and 75 years of age may undergo (after discussing potential risks and benefits with their physician) annual
screening for prostate cancer with a digital rectal examination and a serum PSA
Http://bestpractice.bmj.com/best-practice/monograph/208/follow-up/recommendations.html
163.41-year-old patient with recurrent episodes of acute cholecystitis. Ultrasound showed small stones, you will give her?
A. Ezetimibe
B. Fibramait << No idea what this is. Maybe the mean fibrinate!

Answer: Not sure


Explanation: Management of acute cholecystitis starts with supportive care; that is hospital admission, IV hydration and correction
of any electrolyte imbalance. Analgesia often by opioid is administered, however, ketorolac (NSAID) may be enough to relieved
biliary colic. Usually, acute cholecystitis is accompanied with an infection most commonly E.coli for which antibiotics should be
prescribed. After that a surgical definitive treatment is decided. Prevention measurements include use of NSAIDS which can produce
effective analgesia for biliary colic, and may favorably alter its natural history
Http://cursoenarm.net/UPTODATE/contents/mobipreview.htm?11/26/11695#H8

164.MVA victim presented to ER with hypotension, given crystolloid his BP return to normal, and chest tube was inserted. What
to do next?
A. Re-examine the chest
B. ABG
Answer: A

165.You are palpating artery between symphysis puppies and anterior superior spine
A. Femoral artery
B. Internal iliac

ANSWER: A
- Femoral Artery is a large artery in the thigh and the main arterial supply to the lower limb
- It lies midway between the anterior superior iliac spine and the symphysis pubis.
- Https://en.wikipedia.org/wiki/Femoral_artery

43
166.Patient with RLQ pain and swelling, weight loss, colonoscopy done showed mass in RLQ, what is the diagnosis?
A. Appendix tumor
B. Cecum tumor

Answer is B
Colonoscopy is the most accurate and versatile diagnostic test for colorectal tumors/cancers.
Appendiceal abnormalities are infrequently seen on colonoscopy and rarely yield a diagnostic biopsy in patients with
appendiceal carcinoma.
Source: http://www.ncbi.nlm.nih.gov/pubmed/19089515

167.When u operate for aneurism


A. More 5.5 diabetic thoracic aorta
B. More 5.5 abdominal atherosclerosis
Thoracic Aortic Aneurysm: elective repair of ascending aneurysms at 5.5 cm and descending aneurysms at 6.5 cm for patients
without any familial disorders such as Marfan syndrome. Patients with Marfan syndrome or familial aneurysms should undergo
earlier repair, when the ascending aorta grows to 5.0 cm or the descending aorta grows to 6.0 cm.
Http://emedicine.medscape.com/article/424904-treatment
Abdominal Aortic Aneurysm: Most people with an aneurysm less than 4.0 cm (1.6 inches) in diameter are advised not to have
immediate surgery, but rather to follow the aneurysm over time; this is known as watchful waiting. Usually, this involves an
ultrasound examination of the abdomen every six months to three years, depending on the size of the aneurysm.On the other hand,
most patients with an asymptomatic aneurysm greater than 5.5 cm (2.2 inches) in diameter or that expands more than 0.5 cm within
a six-month period are advised to have repair.
Http://www.uptodate.com/contents/abdominal-aortic-aneurysm-beyond-the-basics

168.Abdominal pain after lifting heavy object + with defect in abdominal wall, what is the diagnosis?
A) direct inguinal hernia
B) indirect inguinal hernia
Answer: A
Direct: The direct inguinal hernia enters through a weak point in the fascia of the abdominal wall.

Indirect: An indirect inguinal hernia results from the failure of embryonic closure of the deep inguinal ring after the testicle has
passed through it.

169.Cause of pruritus ani?


A) Hemorrhoids
B) perianal abscess
Answer: Not sure - Question not clear enough.

44
Explanation: Merck manual - No age is mentioned though according to group ages the following are the most common cause of anal
pruritis.

170.Can't recall the senior very well but it was symptoms and sign of pancreatic disease that caused thrombophlebitis ?
A- acute pancreatitis
B- pancreatic tumor

Answer: B
Explanation: Migratory thrombophlebitis (ie, Trousseau sign) and venous thrombosis also occur with higher frequency in patients
with pancreatic cancer and may be the first presentation.
Reference: http://emedicine.medscape.com/article/280605-clinical#b1

171.Patient with Calcium renal stones so was given a diuretic to lower the Calcium then he developed Gout ?
A- Hydrochlorothiazide
B- Furosemide

Answer: A
Explanation: Both thiazide and loop diuretics cause hyperuricemia which may lead to
Gout. However, thiazide diuretics decrease calcium urinary excretion, while loop diuretics like furosemide increase calcium urinary
excretion. In calcium renal stones, thiazides are used due to their effect on lowering urine calcium.

45
References: http://www.uptodate.com/contents/diuretic-induced-hyperuricemia-and-gout
Http://www.msdmanuals.com/home/kidney-and-urinary-tract-disorders/stones-in-the-urinary-tract/stones-in-the-urinary-tract
172.Which of these patient peritoneal lavage is indicated?
A- severe head trauma,
B- hypotensive patient with abd distention

Answer: B
Explanation: DPL can be used to evaluate both blunt and penetrating abdominal trauma in patients who are hemodynamically
unstable or who require urgent surgical intervention for associated extra-abdominal injuries. DPL can rapidly confirm or exclude the
presence of intraperitoneal hemorrhage.
Reference: http://emedicine.medscape.com/article/82888-overview

173.Bladder cancer after they removed what you will do next:


A. Intravesical medication
B. Observe and serial assesment
C. And 2 another choices regarding treatment

174.13 Y/O child limping & pain, radiological finding destructed femur head, high WBC, diagnosis:
A) Septic arthritis of the hip
B) legg calve perthes
Answer: A
WBC is normal in legg calve perthes disease. Septic arthritis causing painful joint with fever & high WBC. Radiologic features include:
X-rays may be normal in the very early stage of the disease
Joint effusion may be seen
Juxta-articular osteoporosis due to hyperaemia
Narrowing of the joint space due cartilage destruction in the acute phase
Destruction of the subchondral bone on both sides of a joint
If left untreated, reactive juxta-articular sclerosis and, in severe cases, ankylosis will develop
Http://radiopaedia.org/articles/septic-arthritis

175.Child with sudden abdominal pain bloody diarrhea, management?


A) Radio decompression
B) Immediate surgery
Answer: According to the age of pt, for patient from 5 months to 3 years, most common cause of bloody stool is intussusception and
they usually respond to non-operative management by enema decompression.
Http://emedicine.medscape.com/article/930708-treatment#d1
Intestinal malrotation is suspected with the sudden onset of melena in combination with bilious emesis in a previously healthy, non
distended baby.

Bloody diarrhea and signs of obstruction suggest volvulus, intussusception, or necrotizing enterocolitis, particularly in premature
infants. Acute bloody diarrhea should be considered a medical emergency.
Http://emedicine.medscape.com/article/1955984-clinical#b1

Pediatric surgeon: The only definitive treatment for malrotation is surgical in nature
Http://emedicine.medscape.com/article/930313-treatment#d7

176.Child with yellow and cavities in his teeth, what to give?

46
A) Antiseptic mouthwash
B) Fluoride (found in tooth paste)
Answer: B

177.Case of trauma patient unconscious e abdomen distension what intial treatment :


A. CT
B. IV fluid
C. FAST
Answer: A
Resuscitation procedures will begin simultaneously with the assessment involved in the primary survey.
Reference: http://patient.info/doctor/trauma-assessment

178.Where is the common site of Meckel’s diverticulum?


A. Duodenum
B. Ileum
Answer : B

Meckel’s Diverticulum Most common remnant of vitelline duct that connects yolk sac with primitive midgut .Present most
frequently during the first 5yr of life
Reference: Toronto notes.
179.Old patient with Back pain, dysuria , frequency what is the next investigation:
A. PSA
B. ALP

Answer:A
The approach to dysuria is focused History and P/E to determine cause, any d/c (urethral, vaginal, cervical) should be sent for
gonococcus/chlamydia testing; U/A and urine C&S ± imaging of urinary tract. In this case the age and the back pain are suspicious of
malignancy thus PSA should be measured.
Reference:Toronto notes

180.Old patient developed mass in the lung (CXR was attached)


a. SCC
b. Adenocarcinoma
Answer: it depend on CXR and Hx
If it is arise in the peripheral, areas of the lungs. They also have a tendency to spread to the lymph nodes and beyond. Multiple sites
in the lungs and spreads along the preexisting alveolar walls. It may also look like pneumonia on a chest X-ray. And is very common
in non-smoking women and in the Asian population. Adenocarcinoma is more common than SCC which arise in the central chest
area in the bronchi. This type of lung cancer most often stays within the lung, spreads to lymph nodes, and grows quite large,
forming a cavity.
Http://www.webmd.com/lung-cancer/guide/lung-cancer-types

181.16 years old female. Fever and Chronic diarrhea for 10 months Post meal para umbilical pain Sometimes blood mixed with
stool?
A. Crohn’s
B. Chronic pancreatitis

Answer: A
Crohn's disease Symptoms include watery diarrhea, abdominal pain, fever and weight loss.
Http://www.webmd.com/ibd-crohns-disease/crohns-disease/crohns-disease

47
182.Intra-abdominal bleeding can lead to:
A. Hypovolemia
B. Dehydration
Answer: A

Rapid volume repletion is indicated in patients with severe hypovolemia (ie, decreased peripheral perfusion, as indicated by delayed
capillary refill and cool mottled extremities) or hypovolemic shock (ie, severe hypovolemia plus hypotension)

http://www.uptodate.com/contents/treatment-of-severe-hypovolemia-or-hypovolemic-shock-in-
adults?Source=outline_link&view=text&anchor=H1#H1

183.Patient with abscess in his forehead. Where can you find a palpable LN ?
A. Mastoid
B. Superficial parotid
Answer: Parotid gland drain most of the forehead EXCEPT middle part which is drained by submandibular, but many resources are
saying it’s pre auricular!

184.Orchitis case, pain in scrotum with urine +ve culture, what’s the treatment?
A. IV cectriaxone + Doxycycline
B. IV amoxicillin + clavulanate
Answer: A

Acute epididymitis most likely caused by enteric organisms (eg, men with a negative gram stain or nucleic acid amplification test for
N. Gonorrhoeae or men with epididymitis after urinary-tract instrumentation procedures) may be treated with fluoroquinolones
alone

Acute epididymitis most likely caused by sexually transmitted chlamydia and gonorrhea should be treated with ceftriaxone plus
doxycycline /azithromycin .

Http://www.uptodate.com/contents/evaluation-of-the-acute-scrotum-in-
adults?Source=outline_link&view=text&anchor=H2012423711#H12

185.Thyroid disease associated with papillary cancer?


A. Hashimoto
B. Riedel thyroiditis
Answer: A
Http://www.thyroid.org/patient-thyroid-information/ct-for-patients/vol-6-issue-7/vol-6-issue-7-p-6/

186.Female patient present with laughing and coughing passing out urine O/E there is swelling in labia majora ( stress
incontinence
A. Cystocele
B. Urethrocele
Answer: cystocele
Can’t find any clue to go with each one!

187.Case of painful nodules in groin area and pain relieved after punching and discharge came out of it?
A. Hidradenitis suppurativa
B. Frunculs
Answer: B
Http://www.merckmanuals.com/professional/dermatologic-disorders/bacterial-skin-infections/furuncles-and-carbuncles

48
188.Hereditary chronic pancreatitis is?
A. Autosomal dominant
B. Polygene ?
Answer: A
Ø source: http://www.ncbi.nlm.nih.gov/pmc/articles/PMC177456/

189.Patient did IVP shows filling defect then do us shows hypoecnecnty ?


A. Uric acid stone
B. Papillary
Answer: B

ANY STONE IS hyperechoic


But papillary necrosis is hypo

190.M patient went for surgery; he was given insulin and dextrose. Then developed neurological symptoms (Low Na). What is the
mechanism?
A. Water overload.
B. SIADH.
Answer A?

191.Patient did routine X Ray, they found 2*3 cm nodules that has calcified center, it was in the upper left side of lung he is
asymptomatic, what is the next step?
A. Follow up and repeat x Ray
B. Biopsy the lesion
Answer: A – Kaplan

192.patient for surgery known case of DM2 on glimepiride you will shift patient to which drug during and after surgery?
A. insulin
B. metformin

Answer: On the morning of surgery hold oral hypoglycemic agent because Sulfonylureas will increase the risk for hypoglycemia, and
metformin will increase the risk for lactic acidosis.
If the patient develop hyperglycemia give SC insulin, and correction insulin is given until the patient is eating and either can resume
oral agent.

Reference : uptodate "perioperative management of blood glucose in adult with DM"

193.16 years old female complaining of abdominal pain for 2 days. It started in the periumbilical area then it radiated to the right
lower quadrant. It is associated with anorexia. On examination, right lower mass was felt. What is the management?
A. Surgery immediately
B. Conservative
*Antibiotics weren’t mentioned in the choices.
Answer: ?
Appendectomy remains the only curative treatment of appendicitis, but management of patients with an appendiceal mass can
usually be divided into the following 3 treatment categories:
● Patients with a phlegmon or a small abscess: After intravenous (IV) antibiotic therapy, an interval appendectomy can be
performed 4-6 weeks later.
● Patients with a larger well-defined abscess: After percutaneous drainage with IV antibiotics is performed, the patient can be
discharged with the catheter in place. Interval appendectomy can be performed after the fistula is closed.
49
● Patients with a multicompartmental abscess: These patients require early surgical drainage.
Reference: Medscape.
194.A mass at the middle of the Esophagus:
A. Adenocarcinoma
B. Squamous

Answer: B
We have two types of Esophageal CA:
● Adenocarcinoma usually at the GE junction, due to chronic reflux that cause dysplasia.
● Squamous cell carcinoma in most of the esophagus
th
Reference: Surgical Recall, 6 edition

195.40 years old woman with no pain but you noticed jaundice. She has high direct bilirubin and high ALT what is your
diagnosis?
A. Gilbert’s disease
B. Biliary stone

Answer: B
- Gilbert’s disease classified as a prehepatic (associated with high indirect bilirubin)
- Biliary stone (It can be the answer to this Q, exclude other ddx first): asymptomatic (80%), Biliary colic (10 - 25%, painfull).
- Other ddx: pancreatic head CA, Cholangiocarcinoma and primary biliary cirrhosis
Choledocholithiasis (Stone in common bile duct)
- Sign and symptoms: Epigastric or RUQ pain and tenderness + Jaundice + Cholangitis OR recurrent attack of acute
PANCREATITIS
- Labs: Increased ALP, LFT and total + DIRECT bilirubin
- ERCP: Gold standard for diagnosis of CBD stones

Choleclithiasis:
- In Examination: Tenderness to palpation in the RUQ/epigastric area (Most common). In acute cholecystitis, Murphy’s sign
with inspiratory arrest when palpating the gallbladder fossa. Fever suggests a complication of cholelithiasis such as
cholecystitis, while jaundice tends to accompany cholangitis or pancreatitis.
- Labs: alkaline phosphatase and bilirubin elevations
- The initial test of choice in all patients with suspected biliary pain is abdominal ultrasound: The classic findings include
(Acoustic Shadow "headlight" + movement of the stone with patient repositioning + Gallblader thickening > 4mm +
Pericholecystic fluid)

- Http://bestpractice.bmj.com/best-practice/monograph/873/diagnosis/step-by-step.html
- FIRST AID: SURGERY Clerkship

196.Patient underwent orthopedic surgery. 2 days after the surgery he started to complain of swelling, erythema and tenderness
at the site of surgery. Which organism can cause the wound infection?
A. Clostridium perfringens
B. Other choices included different subtypes of clostridium

Answer: A
C. Perfringens is the only Clostridium species can cause wound infection (Gas gangrene; A medical emergency). Reference:
http://www.ncbi.nlm.nih.gov/books/NBK8219/

197.In appendectomy. Which artery you have to pay attention to during the procedure?
Answer: ?
A. Gonadal vessels
50
B. Iliac artery/vein
One must be cognizant of anatomical structures in the right lower quadrant, in order to avoid injuries during an appendectomy.
These include the right ureter, gonadal vessels, iliac artery/vein,psoas muscle, ileum and cecum.
Reference: http://www.eaes-eur.org/getmedia/7994a09c-344d-4c5e-81d5-4b9a75b8304b/Grantcharov_Appendectomy.pdf

198.What is the cause of skin dimpling in breast cancer?


A. Cooper ligament
B. Lactiferous duct

Answer: A. Tumor involvement of Cooper’s ligaments and subsequent traction on ligaments pull skin inward. Reference: Surgical
th
Recall, 6 edition
199.Pheochromocytoma, associated with which neoplasm?
A. Insulinoma
B. Thyroid cancer

Answer: B
- Pheochromocytoma is associated with Medullary thyroid carcinoma , hyperparathyrodism .
- Pheochromocytoma is a type of MEN IIA ( multiple endocrine neoplasia )
Reference: Step-up medicine P 185 .

200.In thyroidectomy which nerve injury lead to hoarseness of voice?


A. Recurrent laryngeal
B. Right vagus

Answer :A
Recurrent laryngeal nerve: Unilateral damage results in a hoarse voice.
Bilateral damage presents as laryngeal obstruction after surgery and can be a surgical emergency: an emergency tracheostomy may
be needed.
Refrence: http://emedicine.medscape.com/article/852184-overview#a4

201.Patient after RTA had spleen injury, he underwent surgical exploration and splenectomy. Which of the following should be
given?
A. Some kind of antibiotic.
B. Meningiococcal vaccine.

Answer: B
There are multiple vaccines that should be given to a patient if not already vaccinated after a splenectomy, some of which are:
1- Pneumococcus
2- Haemophilus influenzae type b (Hib
3- Meningococcal
4- Flu (influenza).
Reference: http://patient.info/health/preventing-infection-after-splenectomy-or-if-you-do-not-have-a-working-spleen

202.Abdominal radiological investigation showed the inner and outer surface of intestine , what the patient has ?
A) Perforation
B) Obstruction

Answer: A
Normally on X-ray only the inner wall of the bowel is visible. If there is pneumoperitoneum (Gas under the diaphragm) both sides of
the bowel wall may be visible. (Rigler's/double wall sign)

51
Reference:
http://www.radiologymasterclass.co.uk/tutorials/abdo/abdomen_x-
ray_abnormalities/pathology_bowel_gas_perforation
203.What is the most common type of gallstones?
A. Cholesterol gallstones
B. Bile pigmented or Pigmented gallstones
Answer : A
Reference: http://www.mayoclinic.org/diseases-
conditions/gallstones/basics/causes/con-20020461

204.Best imaging for cyst in the breast


A. US
B. Mammogram
Answer: A
- US is used to differentiate between solid and cystic lesion .
- Mammogram visualize large cyst but not small microcystic lesion .
Reference :-
- Http://www.uptodate.com/contents/breast-cysts-clinical-manifestations-diagnosis-and-
management?Source=outline_link&view=text&anchor=H91862703#H91862703
- Http://www.mayoclinic.org/diseases-conditions/breast-cysts/basics/tests-diagnosis/con-20032264

205.A patient presented with splenic injury after abdominal trauma. The surgeon decided to embolize the splenic artery. Which
of the following will be compromised?
A. Stomach fundus
B. Splenic flexure

Answer : A
Reference: http://radiopaedia.org/articles/splenic-artery

52
206.Ligament in inguinal canal?
A. A.Broad
B. B.Round

Answer: B

207.Perianal itching. Most likely?


A. Perianal abscess
B. Hemorrhoid
Answer:B
Hemorrhoids, which cause painful swelling of blood vessels in the anal area, can cause itching.
References:
Http://www.emedicinehealth.com/script/main/mobileart-emh.asp?Articlekey=58920&page=2
Http://www.emedicinehealth.com/script/main/mobileart-emh.asp?Articlekey=58920&page=2

208.Patient history of trauma and right femur fracture with blood loss which is accepted change ?
A. Decrease coronary blood flow
B. Decrease venous capacity

Answer: The cardiovascular system initially responds to hypovolemic shock by increasing the heart rate, increasing myocardial
contractility, and constricting peripheral blood vessels. This response occurs secondary to an increased release of norepinephrine
and decreased baseline vagal tone (regulated by the baroreceptors in the carotid arch, aortic arch, left atrium, and pulmonary
vessels).

209.Patient developed parotid gland swelling after cholecystectomy, what is the cause?
A. Bacterial sialadenitis
B. Immunity

Answer:
Explanation: Swelling of the parotid gland post operatively “called anesthesia mumps” is almost always painless and it resolves
spontaneously over a period of hours or days. The exact cause is not known.

210.Patient came to ER with multiple fractures, no loss of consciousness with low blood pressure, tachycardia, normal RR, O2
saturation = 95%. What is the most appropriate next step?
A. IV fluid
B. Save airway
Answer: B
In accordance with the ATLS protocol the airway has to be assessed first and made sure to be patent before assessing the circulation.
Reference: http://www.who.int/violence_injury_prevention/publications/services/en/guidelines_traumacare.pdf
Management of the trauma patient: primary survey ABCDEs. Airway (with maintenance of C spine): if the patient is alert and
answers the questions with a clear voice, the airway is intact. If not secure, rapid sequence endotracheal intubation or definitive
surgical airway should be established. (Mont Reid surgical handbook)

211.In mastectomy what you will cut? Pectorals major muscle


Answer
Total mastectomy: remove entire brest with nipple.
Preventive mastectomy: is prophylactic, same total mastectomy

53
Partial mastectomy: it’s is a breast-conserving method in which the tumor and the tissue surrounding it are all that’s removed.
Radical mastectomy: is the complete removal of the breast, including the nipple, the overlying skin, the muscles beneath the breast,
and the lymph nodes.
Modified radical mastectomy: entire breast is removed as well as the underarm lymph nodes. But chest muscles are left intact. The
skin covering the chest wall may or may not be left intact. The procedure may be followed with breast reconstruction.
(http://www.webmd.com/breast-cancer/mastectomy?page=2#2)

212.drug to delay surgery?


A. Nefidipine

213.elderly heavy smoker with pain on walking and o/e loss of hair and pallor of feet whats dx?
Answer: pvd
Ref: http://www.healthline.com/health/peripheral-vascular-disease#overview1

214.Trauma with forearm streak … long scenario , how to manage :


A. Oral antibiotic
Question and choices incomplete
Explanation: patient most likely has lymphangitis. A history of minor trauma to an area of skin distal to the site of infection is often
elicited in patients with lymphangitis. Upon clinical examination, erythematous and irregular linear streaks extend from the primary
infection site toward draining regional nodes. These streaks may be tender and warm. Patients with lymphangitis should be treated
with an appropriate antimicrobial agent.

215.Papillary thyroid ca mostly associated with which of the following


A. Hurthle cell
Http://www.nature.com/modpathol/journal/v24/n2s/full/modpathol2010129a.html

216.Name of triangle between hyoid bone and two anterior belly of diagastric?
A. Submental triangle
Submental Triangle: between the anterior belly of the digastric, superior to the hyoid bone, and the midline of the neck

217.Femoral vein relation to the femoral artery?


Answer: Medial

The well-described NAVEL mnemonic has been used to help medical students and surgical trainees remember the relationship of the
femoral sheath structures in a lateral-to-medial direction: femoral N erve, femoral A rtery, femoral V ein, E mpty space, and L
ymphatics.
Source: http://emedicine.medscape.com/article/1923032-overview#a2

218.Patient underwent right radical mastectomy. Which of the following will be affected post operatively?
A) Right arm adduction
Answer: A - pec. Major action
Radical mastectomy is a surgical procedure in which the breast, underlying chest muscle (including pectoralis major and pectoralis
minor), and lymph nodes of the axilla are removed as a treatment for breast cancer.
Http://thewellnessdigest.com/pectoralis-major-anatomy-origin-insertion-actions-innervation/

54
219.Fractured pelvis, injury to gonadal artery, what organ affected? (not sure of the Q)
A. Ovary

Answer: the pelvis forms one major ring and two smaller rings of bone that support and protect the bladder, intestines and rectum.
Organ pelvic protect (Bladder, lower colon, lymph nodes, uterus, and vagina

220.64 years male with medial enlarged prostate, all labs normal, what to do?

Http://emedicine.medscape.com/article/437359-treatment

221.Spontaneous lips swelling?

Answer: Swollen lips may have a range of different causes, from mild conditions to more serious disorders, including infections,
allergy, inflammation, injuries or an underlying medical condition.

222.Patient with bilateral abdominal mass:


Polycystic kidney disease.
Answer: A
Source: http://emedicine.medscape.com/article/244907-overview

223.A patient was injured and now has scrotal pain, what’s the most likely diagnosis (was playing football)
A. Testicular torsion
Answer:A http://www.uptodate.com/contents/evaluation-of-the-acute-scrotum-in-
adults?Source=machinelearning&search=scrotal+pain&selectedtitle=1~25&sectionrank=1&anchor=H671410222#H2012423711

224.What is the cause of death due to flames fire:


A. Smoke inhalation injuries
Answer: A
Source: http://www.nfpa.org/press-room/reporters-guide-to-fire-and-nfpa/consequences-of-fire

225.Dislocation of temporomandibular joint, doctor try to hyper extend the jaw to relocate it. What is the muscle that elevate the
jaw?
A. Temporalis

226.A victim of RTA present with multi trauma . There is ecchymosis around the eye & subconjunctival haemorrhage. During
examination the patient was unable to look superiorly.
A. Blunt trauma to the eye

Answer: Read about orbital fracture


Key diagnostic factor is diplopia on upward gaze, the most common clinical signs seen with adult orbital fractures are periorbital
ecchymoses and subconjunctival hemorrhages
Http://bestpractice.bmj.com/best-practice/monograph/1172/diagnosis/history-and-examination.html
Http://www.ncbi.nlm.nih.gov/pmc/articles/PMC3324224/
227.Cause of abnormality in breast cancer .. dimpling skin. Ligament affected?

Retraction phenomena (concave changes of the skin of the breast) range from a small area of skin flattening in the vicinity of an
underlying tumor or area of thickening, to shrinkage of most of the skin of the breast. They usually result from shortening of
Cooper's ligaments due to fibrosis. Carcinoma is the most common cause and is usually associated with a distinct mass or very
subtle underlying thickening in the breast tissue.

55
228.Pancreatitis chronic vs. Acute

229.What is the name of incision in open cholecystectomy?

Answer: Kocher (subcostal)

230.What is the lymphatic drainage of upper lateral part of breast?

Answer: no choices
Possible answer: the lateral quadrant drains into anterior axillary or pectoral LN (75%)

56
231.What is the most sensitive and specific to diagnose and localize bowl obs ?
CT
CT is appropriate for further evaluation of patients with suspected intestinal obstruction in whom clinical examination and
radiography do not yield a definitive diagnosis. CT is sensitive for detection of high-grade obstruction (up to 90 percent in some
series), and has the additional benefit of defining the cause and level of obstruction in most patients. In addition, CT can identify
emergent causes of intestinal obstruction, such as volvulus or intestinal strangulation.
Http://www.aafp.org/afp/2011/0115/p159.html

232.Most accurate test for acute colysctitis?

The American College of Radiology (ACR) makes the following imaging recommendations:
Ultrasonography is the preferred initial imaging test for the diagnosis of acute cholecystitis; scintigraphy is the preferred
alternative
CT is a secondary imaging test that can identify extrabiliary disorders and complications of acute cholecystitis
CT with intravenous (IV) contrast is useful in diagnosing acute cholecystitis in patients with nonspecific abdominal pain
MRI, often with IV gadolinium-based contrast medium, is also a possible secondary choice for confirming a diagnosis of acute
cholecystitis
MRI without contrast is useful for eliminating radiation exposure in pregnant women when ultrasonography has not yielded a clear
diagnosis
Contrast agents should not be used in patients on dialysis unless absolutely necessary
Http://emedicine.medscape.com/article/171886-overview

233.Sites of Displaced appendix ?


Read about appendicitis:
Appendicitis is defined as an inflammation of the inner lining of the vermiform appendix that spreads to its other parts. The
appendix has no fixed position. It originates 1.7-2.5 cm below the terminal ileum, either in a dorsomedial location (most common)
57
from the cecal fundus, directly beside the ileal orifice, or as a funnel-shaped opening (2-3% of patients). The appendix has a
retroperitoneal location in 65% of patients and may descend into the iliac fossa in 31%
Http://emedicine.medscape.com/article/773895-overview#a3

234.Patient had splenic trauma and splenectomy planned, if the surgeon ligate the splenic artery high up what structure would be
affected?
Answer: Stomach fundus not sure
The splenic artery supplies the spleen and substantial portions of the stomach and pancreas (Fig 1) (1). The splenic artery courses
superior and anterior to the splenic vein, along the superior edge of the pancreas. Near the splenic hilum, the artery usually divides
into superior and inferior
Terminal branches, and each branch further divides into four to six segmental intrasplenic branches.
Http://pubs.rsna.org/doi/full/10.1148/rg.25si055504

235.Post surgical prophylaxis ?


Answer: LMWH
Heparin has been successfully used in the prophylaxis and treatment of venous and arterial thrombotic disorders. When compared
with unfractionated heparin, lmwhs have fewer serious side effects, such as heparin-induced osteopenia and heparin-induced
thrombocytopenia (HIT).[9] In contrast to UFH, the lmwhs appear not to be inactivated by platelet factor 4. After subcutaneous
injection, lmwhs also exhibit less plasma protein binding (and therefore greater bioavailability) than UFH.
Http://www.medscape.com/viewarticle/437218

236.Diabetic patient developed foot ulcer, diabetes role for this non healing?
Answer: Inhibit phagocytosis
Occur as a result of various factors, such as mechanical changes in conformation of the bony architecture of the foot, peripheral
neuropathy, and atherosclerotic peripheral arterial disease, all of which occur with higher frequency and intensity in the diabetic
population.
Http://emedicine.medscape.com/article/460282-overview

237.Patient with groin abscess after aspirate there was multiple cells ..... Cause?
Immunodefieciency - C5 --)

238.Less complicated of NSAID drugs in PUD?

239.Elderly with abdominal tenderness and dilated bowel loops?


Small bowel obstruction?

240.Patient treated for duodenal ulcer. Now complains of breast enlargement and decrease sexual desire. Which drug?
Answer: cimetidine ?
In men, gynecomastia has been reported. During post marketing surveillance in the 1980s, cases of male sexual dysfunction were
also reported.

241.Acute cholecystitis management?


In acute cholecystitis, the initial treatment includes bowel rest, intravenous hydration, correction of electrolyte abnormalities,
analgesia, and intravenous antibiotics.

242.A question about the artery you see on lateral of bowel in hernia operation?
Answer: Inferior epigastric artery (medial to inguinal ring lateral to bowel)

58
Vessels regularly found during inguinal hernia repairs are the superficial circumflex iliac, superficial epigastric, and external
pudendal arteries, which arise from the proximal femoral artery and course superiorly. The inferior epigastric artery and vein
run medially and cephalad in the preperitoneal fat near the caudad margin of the internal inguinal ring.
Link:
Http://emedicine.medscape.com/article/189563-overview

243.A patient with indirect inguinal hernia. What artery runs medial to it?
A. Inferior epigastric artery
Answer: A
Reference: Surgical Recall, 6th edition

244.A patient presented with RLQ pain, guarding, tenderness and positive obturator sign. What is the most likely diagnosis?
A. Appendicitis.

Answer: A Obturator sign: Pain upon internal rotation of the leg with the hip and knee flexed; seen in patients with pelvic
appendicitis.
th
Reference: Surgical Recall, 6 edition

Explanation: The classic history of anorexia and periumbilical pain followed by nausea, right lower quadrant (RLQ) pain, and
vomiting. Physical exam: Rebound tenderness, pain on percussion, rigidity, and guarding: Most specific finding.
Reference: http://emedicine.medscape.com/article/773895-overview

245.Typical scenario for appendicitis , mechanism: (GS)


Fecal impaction on the app Lumen

Answer:
Appendiceal obstruction has been proposed as the primary cause of appendicitis [7,11-14]. Obstruction is frequently implicated but
not always required for the development of appendicitis.Appendiceal obstruction may be caused by fecaliths (hard fecal
masses), calculi, lymphoid hyperplasia, infectious processes, and benign or malignant tumors. However, some patients
with a fecalith have a histologically normal appendix and the majority of patients with appendicitis do not have a fecalith.
Refrence: http://cursoenarm.net/UPTODATE/contents/mobipreview.htm?16/55/17265

246.Patient with right hypochondrial pain and US showed echogenic shadow ?


Answer : Gallstones (Cholelithiasis) ,
Stone in the gallbladder neck with typical acoustic shadow.
Although it originally referred to ultrasonographic findings of echogenic, nonshadowing, microscopic material within the gallbladder,
the term biliary sludge currently indicates a precipitate of microcrystals occurring in bile with high mucous content. Sludge may
contain microliths. Milk of calcium bile, a calcium carbonate precipitate opaque on plain radiographs, may coexist with cholelithiasis.
Ref: http://emedicine.medscape.com/article/366246-overview#a1

247.The surgeon has documented if the infant has a constant infantile esotropia exceeding 12 PD, surgical realignment should be
performed.
Http://emedicine.medscape.com/article/1198876-treatment#d6

248.Pt. Perform surgery to stomach that rapped her around the esophagus which nerve will be injured:
A) Vagal
Answer: A
Reference: http://www.medscape.com/viewarticle/804147_4
59
249.Patient with metastatic breast cancer presented with SOB , distended neck vain ,apex beat not palpable Distal heart sounds ?
BP 70/... The best step to confirm the Diagnosis is?
IV fursmide IV fluid & urgent echo :::::::::::::::::::::::::::::
Answer : convensional SV Cavography .
Inisial >> doplex US

250.Acute scrotal pain


Reference: http://www.rch.org.au/clinicalguide/guideline_index/Acute_Scrotal_Pain_or_Swelling/ + Toronto notes.

251.Case with leg pain while walking, loss of hair in leg , nail changes , what is the cause , The cause is going to be a peripheral
vascular cause

Reference: Toronto notes.

60
252.Patient with +ve Cullen sign and Grey-Turner sign what is the diagnosis?
A.Acute pancreatitis
Answer:A

Reference:Toronto notes

253.Question about thyroid cancer , i remember that i choose hurthle cell adenoma but not sure if it’s the correct answer or not
hurthle cell adenoma:

Hürthle cell carcinoma of the thyroid gland is an


unusual and relatively rare type of differentiated
thyroid cancer.The cytologic features for Hürthle
cell neoplasms are hypercellularity, with a
predominance of Hürthle cells(large and polygonal
in shape, with indistinct cell borders) ,no
lymphocytes, and scanty or absent colloid.

Reference:
http://emedicine.medscape.com/article/279462-
overview+Toronto notes

254.Patient with appendicitis but it presented as right upper quadrant pain. Cause?

61
A. Subhepatic appendix

Answer: A
Ascending subhepatic appendicitis presenting with right upper abdominal pain may be clinically indistinguishable from acute
pathology in the gallbladder, liver, biliary tree, right kidney and right urinary tract
http://www.sciencedirect.com/science/article/pii/S0378603X12000538

255.Patient present with Right subcostal area pain ... And you do surgery and you find inflamed appendix?
Answer: sub-hepatic appendix
http://www.sciencedirect.com/science/article/pii/S0378603X12000538

256.Forceps forgotten inside patient after surgery, what will you do?
Answer : tell the patient what you found and refer him to OR

257.Patient with osteoid sarcoma. Then he started having pelvic pain. Cartilage and bone tumor in pelvic bone on imaging.
Cause?
a. Chondreoblastic Osteoma
Answer : Osteochondroma or Enchondroma ?

Http://www.uptodate.com/contents/chondrosarcoma?Source=outline_link&view=text&anchor=H2#H2

258.About patient with breast cancer on something and having bleeding on routine check 2x3 cm then became 3x6cm what
suspect: type of cancer? "
Inflammatory breast cancer is a rare and very aggressive disease in which cancer cells block lymph vessels in the skin of the breast.
This type of breast cancer is called “inflammatory” because the breast often looks swollen and red, or inflamed.
Inflammatory breast cancer is rare, accounting for 1 to 5 percent of all breast cancers diagnosed in the United States. Most
inflammatory breast cancers are invasive ductal carcinomas, which means they developed from cells that line the milk ducts of the
breast and then spread beyond the ducts.
Http://www.cancer.gov/types/breast/ibc-fact-sheet

259.After appendectomy he has lung abscess which antibiotic you will choose
(read about it)
Answer: ?

260.Auer rods indicate?


a. Acute myeloid leukemia
Answer: A
Ø source: http://www.pathologystudent.com/?P=4868

261.Microsatellite instability?
a. Colorectal cancer
Answer: A
Http://www.ncbi.nlm.nih.gov/pmc/articles/PMC3037515/

262.Female came for breast screening, her mother and sis had breast ca at 45-48 year

62
What will u do now?
a. BRAC1 Gene Screen

Answer: ?
BRCA 1 And mammogram

Http://ww5.komen.org/breastcancer/breastcancerscreeningforwomenathigherrisk.html
263.Main difference between chron's , ulcerative and IBS should be known
Answer:

264.Patient had trauma, presented with tachycardia, bradychpnea and hypotension, what is the first thing you will do?
a. Needle thoracotomy

Answer: A (tension pneumothorax)


“Classical management of tension pneumothorax is emergent chest decompression with needle thoracostomy. A 14-16G
intravenous cannula is inserted into the second rib space in the mid-clavicular line.
Http://www.trauma.org/archive/thoracic/chesttension.html

265.Patient presented to you with Rt scrotal swelling, he said that it is increasing in size day after day , on examination , it is not
transluminate , what will you do ?
a. Reassure him and get surgical opinion
Answer: A

266.High fat and low fiber diet increases the risk of which of the following cancers?
a. Colorectal cancer
Answer: A

267.Most common parotid gland malignancy?


ANSWER: Mucoepedermoid carcinoma
Medscape

63
268.Staging of lung cancer, tracheal nodes were involved?
Answer : if lung cancer reash trachea it’s T4
N1 Metastasis in ipsilateral peribronchial and/or ipsilateral hilar lymph nodes and intrapulmonary nodes, including involvement
by direct extension
missing data
http://www.uptodate.com/contents/image?Imagekey=ONC%2F80099&topickey=NEPH%2F3840&source=see_link

269.The most common cause of breast bloody discharge:


A. Ductal papilloma
Answer: A
Reference: Surgical Recall, 6th edition

270.Patient came with abdominal swelling and weak abdominal wall. What’s the diagnosis?
A. Direct inguinal hernia

Answer: A

271.A patient presented complaining of dark stool and vomiting blood. He is stable now. What is your investigation?
A. Upper GI endoscope.

Answer: A
Mostly from upper GIT bleeding

272.What is the best study to determine cystic breast lesion?


Answer: ?
US is the best way to evaluate cystic lesions. Reference: http://breast-cancer.ca/ult-bens/
273.What is the most common site of Meckel’s diverticulum?
A. Lower ileum

Answer: A
Mostly Meckel diverticula are found Within 2 feet (100cm) of the ileocecal valve on
th
The antimesenteric border of the bowel. Reference: Surgical Recall, 6 edition

274.What is the most common site of breast mass?


A. Superior and lateral

Answer: A
Approximately one half of breast cancers develop in the upper outer quadrants.
th
Reference: Surgical Recall, 6 edition

275.Patient had Upper GI bleeding. What is the diagnosis?


A. Zollinger ellison syndrome

Answer: Missing a lot of info


Read about Upper GI bleeding. Zollinger ellison syndrome is a condition in which a gastrin-secreting tumor or hyperplasia of the islet
cells in the pancreas causes overproduction of gastric acid, resulting in recurrent peptic ulcers.

276.What is the common presentation of perforated duodenal ulcer?

64
Answer: ?
The most characteristic symptom is the suddenness of the onset of epigastric pain. The pain rapidly becomes generalized although
occasionally it radiates to the right lower quadrant (Valentino’s sign).
Reference: http://www.ncbi.nlm.nih.gov/books/NBK6926/
277.17-year-old complaining of abdominal pain that shifts to the right iliac fossa. What is the most likely diagnosis?
Answer: ?
Appendicitis
Features of the abdominal pain are as follows:
● Typically begins as periumbilical or epigastric pain, then migrates to the RLQ
● Patients usually lie down, flex their hips, and draw their knees up to reduce movements and to avoid worsening their pain
Reference: http://emedicine.medscape.com/article/773895-overview

278.A female patient who has high risk for breast cancer. Which gene is responsible for that?
Answer: ?
BRCA1
The family history characteristics that suggest increased risk of cancer are summarized as follows:
● Two or more relatives with breast or ovarian cancer
● Breast cancer occurring in an affected relative younger than 50 years
● Relatives with both breast cancer and ovarian cancer
● One or more relatives with two cancers (breast and ovarian cancer or 2 independent breast cancers)
● Male relatives with breast cancer
● BRCA1 and BRCA2 mutations
● Ataxia telangiectasia heterozygotes (quadrupled risk)
● Ashkenazi Jewish descent (doubled risk)
Reference: http://emedicine.medscape.com/article/1947145-overview#a6

279.Prevent of hemorrhoid:
A. High fiber diets
Answer: A
The best way to prevent hemorrhoids is to keep your stools soft, so they pass easily. To prevent hemorrhoids and reduce symptoms
of hemorrhoids, follow these tips:
Eat high-fiber foods. Eat more fruits, vegetables and whole grains. ...
Drink plenty of fluids. ...
Consider fiber supplements. ...
Don't strain. ...
Go as soon as you feel the urge. ...
Exercise. ...
Avoid long periods of sitting.
Reference: http://www.mayoclinic.org/diseases-conditions/hemorrhoids/basics/prevention/con-20029852

280.which thyroid cancer have the best prognosis?


Answer: Papillary cancer. Survival rate 98% at 10 yr Reference: Toronto Notes 2015

281.Which of the following protect against some cancers?


A. Fibrate

Reference : http://www.uptodate.com/contents/cancer-prevention?Source=outline_link&view=text&anchor=H11#H11

65
282.during laparoscopic surgery of inguinal hernia, you find artery superficial going upward?

A. Inferior epigastric artery.


Answer: Inferior Epigastric vessels
Great care must be exercised as the dissection approaches the iliac vessels. In addition, obturator vessels
often cross the dissection planes and may need to be clipped and divided.
The inferior epigastric vessels are identified, and dissection lateral to the vessels leads to the space of Bogros, the cord structures,
and indirect hernias (see the video below). The proper plane of dissection is between the transversalis fascia and the peritoneum.
This is identified by retracting the inferior epigastric vessels upward against the rectus muscle. A plane containing areolar tissue is
identified, and this plane is dissected toward the pelvic sidewall
Referance : medscape
http://emedicine.medscape.com/article/1534321-overview

283.Case Symptoms of cholecystitis what best initial modality ?


A. U/S

Answer: A
Sonography is the preferred initial imaging test for the diagnosis of acute cholecystitis, and scintigraphy is the preferred alternative.
CT is a secondary imaging test that can identify extrabiliary disorders and complications of acute cholecystitis, such as gangrene, gas
formation, and perforation.
Reference:
Http://emedicine.medscape.com/article/171886-workup#c7
284.Post appendectomy, penicillin resistance? What antibiotic use?

Answer: Gentamicin
Gentamicin is not the drug of choice in cases of appendectomy, but consider using this drug if penicillins or other less toxic drugs are
contraindicated.
Reference: http://emedicine.medscape.com/article/773895-medication#4
Answer: Broad-spectrum gram-negative and anaerobic coverage is indicated .Penicillin-allergic patients should avoid beta-
lactamase type antibiotics and cephalosporins. Carbapenems are a good option in these patients.

Reference: http://emedicine.medscape.com/article/773895-treatment#d10

NONPERFORATED APPENDICITIS :
Postoperative management — With both the open and laparoscopic approaches, most patients are discharged within 24 to
48 hours of surgery. Patients may be started on a clear liquid diet post-operatively and advanced to regular diet as tolerated.
Antibiotics are not required postoperatively in nonperforated appendicitis.

PERFORATED APPENDICITIS :
Postoperative management — Postoperatively, these patients often have an ileus, and diet should only be advanced as the
clinical situation warrants. Patients may be discharged once they tolerate a regular diet, usually in five to seven days. The duration of
antibiotic therapy in such patients is discussed elsewhere. (See "Anaerobic bacterial infections", section on 'Antibiotic treatment'.)

Reference: http://www.uptodate.com/contents/management-of-acute-appendicitis-in-adults

285.mass in the neck in cervical area, the best initial test?

Answer: CT
Reference:
Http://www.uptodate.com/contents/evaluation-of-a-neck-mass?Source=outline_link&view=text&anchor=H13#H13

66
286.30 years old female dancer with breast mass that disappeared by aspiration ?
Answer: Complete disappearance of the breast mass after aspiration of non bloody fluid , indicate simple cyst
References : pubmed & medscape
http://www.medscape.org/viewarticle/718139
Http://www.ncbi.nlm.nih.gov/pmc/articles/PMC2855917/#!Po=15.7407

287. Sign of hyperthyroidism with nodule in lateral to mid line


Answer: First rule any thyroid nodule not showing or changing hormone activity and signs it is most likely malignant nodule
Hyperthyroidism signs and symptoms:
Nervousness
Anxiety
Increased perspiration
Heat intolerance
Hyperactivity
Palpitations
Reference: Medscape

288.MVA diagnosed with fracture of base of skull and injury to nerve pass through jugular foramen

Answer: Cranial nerves 9, 10, and 11


Vernet syndrome or jugular foramen syndrome is involvement of the IX, X, and XI cranial nerves with basal skull fracture. Patients
present with difficulty in phonation and aspiration and ipsilateral motor paralysis of the vocal cord, soft palate (curtain sign),
superior pharyngeal constrictor, sternocleidomastoid, and trapezius.
Reference : http://emedicine.medscape.com/article/248108-overview#showall

289.In splenectomy, what organ might be affected?


Answer: Atelectasia of the lower lung +injury of surrounding structure eg. gastric wall , tail of pancreas
((Toronto notes,2015,GS52 ))

290.Female c/o clear fluid from nipple. Breast examination was normal. Brain MRI: pituitary mass. This mass secrets?
A. Prolactin
Answer: A

291.Male patient complain of dysphagia for solid and liquid, radiograph shows dilated esophagus with tapering at end, what is
the diagnosis?
A. Spasm
B. Cancer
C. Achalasia

Answer: C, dysphagia (which is for both solids and liquids, in comparison to dysphagia for solids only in cases of esophageal
carcinoma).A barium swallow of dilated esophagus with tapering at end (bird-beak sign or rat-tail sign) is classical in primary
achalasia, can also occur in pseudoachalasia due to a gastric tumour involving the gastro-oseophageal junction.
Http://radiopaedia.org/articles/achalasia

292.A patient with a family history of multiple sudden cardiac death patientt has Marfan's syndrome features. What might be the
cause of death?
A. Ruptured aortic aneurysm
Answer: A
((First Aid for the USMLE Step 1 CK 2104,p 284 ))

67
293.Tamoxifen SE or Complications?
Answer:
Commonly reported side effects of tamoxifen include: nausea, weight loss, amenorrhea, vaginal discharge, hot flash, fluid
retention, vaginal hemorrhage, and skin changes. Other side effects include: sepsis, infection, constipation, weight gain, alopecia,
menstrual disease, vomiting, ostealgia, increased serum aspartate aminotransferase, cough, oligomenorrhea, edema, and diarrhea.
See below for a comprehensive list of adverse effects. Drug.com https://www.drugs.com/tamoxifen.html

294.Stap wound in the abdomen with bacteroid fragilis what is the best antibiotic to use …
A. ampicillin

295.Patient with BPH, what will you give?


Answer: ?
It depends on the case and the which type of treatment (watchful waiting, medical, surgical)
If it’s about medication you might check the following link
Http://www.urologyhealth.org/urologic-conditions/benign-prostatic-hyperplasia-%28bph%29/treatment

296.her-neu gene amplification


Breast cancers with her2 gene amplification or her2 protein overexpression are called her2-positive in the pathology report. Her2-
positive breast cancers tend to grow faster and are more likely to spread and come back compared to her2-negative breast cancers.
Treatments available specifically for her2-positive breast cancer: herceptin (chemical name: trastuzumab), which works against her2-
positive breast cancers by blocking the ability of the cancer cells to receive chemical signals that tell the cells to grow.
Http://www.breastcancer.org/symptoms/diagnosis/her2

297.BRCA
BRCA1 and BRCA2 are human genes that produce tumor suppressor proteins. These proteins help repair damaged dna and,
therefore, play a role in ensuring the stability of the cell’s genetic material. When either of these genes is mutated, or altered, such
that its protein product either is not made or does not function correctly, DNA damage may not be repaired properly. As a result,
cells are more likely to develop additional genetic alterations that can lead to cancer.
http://www.cancer.gov/about-cancer/causes-prevention/genetics/brca-fact-sheet

298.Hiatal hernia .. During surgery, where is nerve position?

Read about:
• Fertility Q
• Case of cystitis.
• Case about renal colic
• Main difference between ( crohn's , ulcerative and IBS )
• Dumping syndrome post surgery
• Hernia (especially anatomical structures)
• Renal artery stenosis
• Gallbladder stone
• Big arterier and their branches with their supply.
• *dermatomes and nerve supply of upper and lower limb.
• *acute cystitis
• * Margin of neck Quadrants
VAN:
V- Vein lateral
A- Artery middle
N- Nerve medial

68
Plastic Surgery

69
1. Body surface area rule of?
A. 9s
B. 8
C. 7
Answer: A

2. Burn tanning bed swelling red blisters?


A. Grade 1
B. Grade 2
C. Grade 3
Answer: B

3. Child presented with burn in the upper right extremity with bluster what is the degree of the burn
A. 2nd degree more than 15 %
B. 2nd degree less than 15 %
C. 3rd degree more than 15 %
D. 3rd degree less than 15 %
Answer: B

4. Median nerve injury leads to:


A. Complete claw hand
B. Ape hand
C. Partial claw
D. Ulnar Claw

Answer: B
Reference: Textbook of Anatomy - Upper Limb and Thorax

5. Long thoracic nerve damage with winging of scapula, this nerve arises from which part of the brachial plexus?
A. Upper trunk.
B. Posterior cord.
C. Medial cord.
D. Root

Answer: D
Root: C5, C6, C7 http://www.orthobullets.com/anatomy/10135/long-thoracic-nerve

6. In which position will you apply a splint in patients with carpal tunnel syndrome?
A. Dorsiflexion.
B. Plantarflexion
C. Extension
D. Abduction

Answer: A
The ideal position of the wrist is close to neutral with 2° + 9 of dorsiflexion and 2° + 6 ulnar deviation to prevent prolonged flexion or
extension that prevents any raise in the pressure.
Reference: Occupational Therapy for Physical Dysfunction.

7. A patient with thenar atrophy but with no numbness. What is the nerve affected?
A. Axillary
B. Ulnar

70
C. Median (Anterior interosseous nerve)
D. Radial

Answer: C
Thenar atrophy is an eroding of muscle tissue, which can impair control over the thumb and leave the hand disfigured. The problem
can be a complication of several different conditions and disorders, including carpal tunnel syndrome, acromegaly, or direct trauma
to the wrist or thumb.

8. Pt with carpal tunnel what is the diagnostic test:


A. MRI
B. CT
C. X-ray
D. Nerve conduction
Answer: D
Electrophysiological tests (nerve conductive study). Electrical testing of median nerve function is often done to help confirm the
diagnosis.
Reference: http://orthoinfo.aaos.org/topic.cfm?Topic=a00005

9. Artistes cannot abduct his fingers; which muscle is affected?


A. Thenar muscle
B. Anterior interossei
C. Posterior interossei

ANSWER: C Dorsal (Posterior Interossei)

- The palmar interosseous muscles adduct the fingers towards the middle finger. This is in contrast to the dorsal interossei,
which abduct the fingers away from the middle finger.
- SURGERY Clerkship
- Https://en.wikipedia.org/wiki/Palmar_interossei_muscles
- Thenar muscles are 4 muscles that are responsible for fine movements of the thumb; they are supplied by the median &
ulnar nerve.
Palmar (anterior) interossei adduct the fingers at the MCP joint; they are supplied by the ulnar nerve.
Dorsal (posterior) interossei abduct the fingers at the MCP joint; they are supplied by the ulnar nerve.
Source: http://teachmeanatomy.info/upper-limb/muscles/hand/
10. Carpal tunnel syndrome. Pt can't work and write. Which muscles affected?
A- Theaner
B- Interossei palmar muscle
C- Interossei dorsal muscle
Answer: A

11. Pronator teres syndrome, which nerve is entrapped:


A. Ulnar
B. Radial
C. Median

Answer: C
Reference: https://www.youtube.com/watch?V=zqho1dzqtty
Pronator teres syndrome is a compression neuropathy of the median nerve at the elbow. It is rare compared to compression at the
wrist (carpal tunnel syndrome) Reference: Wikipedia.

71
12. 40 years old male patient work on the computer a lot. After that get tingles around lateral aspect of the hand. What is the
most accurate test??????????
A. Tinel’s sign
B. Phalen Test
C. Durkan's carpal test

Answer: C
Durkan: (Also known as carpal tunnel compression test) sensitivity: 89% and specificity: 96%. It is considered by some to be a better
provocative test than tinel and phalen tests.
Reference:https://books.google.com.sa/books?Id=d4mqcgaaqbaj&pg=PA110&dq=tinel+phalen+compression&hl=en&sa=X&redir_e
sc=y#v=onepage&q&f=false

13. Diagnosis of carpal tunnel is by?


A. Compression test
B. Kats test
C. Phalen's test

Answer: A
Electrodiagnostic testing, primarily with nerve conduction studies (NCS), sometimes supplemented with needle electromyography
(EMG), is a standard part of the evaluation for CTS because it has a high sensitivity and specificity for confirming the diagnosis.

14. Which indicate how long open wound ..in inflammatory process & how can you tell when it has healed ?
A. Coverd by eshar
B. Covered by epithelium
C. Neovascularization

Answer: B
Reference: http://www.uptodate.com/contents/wound-healing-and-risk-factors-for-non-
healing?Source=outline_link&view=text&anchor=H1070222553#H1070222553
http://emedicine.medscape.com/article/1298129-overview#a5

15. Trauma to shoulder PT can’t raise hand the injury is in:


A. Posterior cord of biracial plexus
B. Medial cord
C. Lateral cord

Answer: A
The axillary nerve is a branch of the posterior cord of the brachial plexus

Http://emedicine.medscape.com/article/1877731-overview#a2

16. Patient after tan bath presented with pain and swelling in chest and back:
A. 1st degree burn
B. 2nd degree
C. 3rd degree
Answer: A
17. Loss of adduction of fingers caused by injury to:
A- ulnar
B- median

Answer: A

72
Ulnar nerve supplies all the intrinsic muscles of the hand except the first two lumbricals and muscles of thenar eminence
[http://emedicine.medscape.com/article/1243669-overview]

18. Patient had injury to the ulnar nerve?


A. Complete claw hand
B. Partial claw

Answer: B
➢Effects of the ulnar nerve injury: ulnar claw hand (Partial claw hand) and hollowing of skin in the first web space on dorsal aspect
of hand.
➢Complete claw hand results from combined lesions of the median and ulnar

Reference: Textbook of Anatomy - Upper Limb and Thorax


19. Patient with chronic osteomyelitis of her leg and there is sinus draining. She has cancer of skin near the sinus which has
eosinophilic inclusions. What is the type of cancer?
a. Squamous cell carcinoma.
b. Basal cell carcinoma.

Answer: A
De novo squamous cell carcinoma emerge in the setting of long-standing ulcers, burn scars, or osteomyelitis. Reference: Journal of
Skin Cancer.

20. Young patient with pain in forearm worsening day by day, x-ray shows "onion peel" best investigation?
A. MRI??

73
ANSWER:
- “onion skinning” is classic sign for Ewing sarcoma
- The diagnostic work-up is usually initiated with a plain radiograph of the affected area
- MRI is preferred in most cases because of its superior definition of tumor size, local intraosseous and extraosseous extent,
and the relationship of the tumor to fascial planes, vessels, nerves, and organs
- Https://yhdp.vn/uptodate/contents/mobipreview.htm?36/56/37769#H6

21. Patient was burn with boiling water after that he develop blister and blanch skin with pressure what is the degree of this
burn?

Answer: second degree burn


Second-degree burns (also known as partial thickness burns) involve the epidermis and part of the dermis layer of skin. The burn site
appears red, blistered, and may be swollen and painful.
Http://www.stanfordchildrens.org/en/topic/default?Id=second-degree-burns-partial-thickness-burns-90-P01757

22. Case of burn (Know how to calculate the TBSA and how to determine the degree of burn).

Reference: Toronto Notes

74
23. Case of burn, how much fluids you will give in the 1st 8 hours?
Answer: ½ of fluids
Parkland formula: 4 ml * kg * TBSA%
Reference: Toronto notes

24. Case of burn (anterior trunk, upper and lower limb), wt. 75, measure the IV fluid this patient need?
Answer: 72% of body is burned = 21.6 L Fluid Requirements, 1st 24 Hours = 10.8L
Fluid Requirements, 1st 8 Hours (1/2 of Total)
Parkland formula: Initial 24 hours: Ringer’s lactate (RL) solution 4 ml/kg/% burn for adults and 3 ml/kg/% burn for children. RL
solution is added for maintenance for children:
● 4 ml/kg/hour for children weighing 0–10 kg
● 40 ml/hour +2 ml/hour for children weighing 10–20 kg
● 60 ml/hour + 1 ml/kg/hour for children weighing 20 kg or higher
This formula recommends no colloid in the initial 24 hours.
Next 24 hours: Colloids given as 20–60% of calculated plasma volume. No crystalloids. Glucose in water is added in amounts
required to maintain a urinary output of 0.5–1 ml/hour in adults and 1 ml/hour in children.
Link: http://www.ncbi.nlm.nih.gov/pmc/articles/PMC3038406/

Fluid Requirements = 4ml x TBSA burned (%) x Weight (kg)


Give 1/2 of total requirements in 1st 8 hours, then give 2nd half over next 16 hours.

25. Patient complaining of pain in the 2nd digit. Which tendon is affected?
Answer: Extensor indices

26. Something about post facial surgery, patient developed some neurological
symptom, which branch is effected options were branches of the main nerves?
Answer: Sensory or Motor?? Q is messing imp info

27. Generalized body scaling? Surface area affected (70-60-50-90)

75
Pediatrics Surgery

76
1. Child was crying since 4 hours ago. He has fever and vomiting. On examination there was a mass on the RUQ. What is your
diagnosis?
A. Appendicitis
B. Gastroenteritis
C. Pancreatitis
D. Intussusception

Answer: D

2. Case scenario of intussusception, what is the gold standard investigation?


A. Abdominal CT
B. CXR
C. Barium study
D. Abdominal X-ray
Answer: C

3. 2 years old with crampy abdominal pain, bleeding per rectum. What is your next step?
a. Barium enema

4. Which one of these layers is involved in chagas and hirschsprung disease?


A. Muscularis externa
B. Mucosa
C. Submucosa

Answer: A
Hirschsprung disease (congenital megacolon) and Chagas disease have different etiologies, but both inhibit intestinal motility by
affecting the myenteric (Auerbach's) plexus located between the layers of the muscularis externa.
Reference: Berkeley coursehero.

5. 7 years old boy brought by his parents, he has nausea, severe vomiting for 20 minutes and now semi comatosed. The parents
mentioned that he has same episode two weeks ago for 5 minutes without deterioration in consciousness. On examination
there is right testicular mass that does not transilluminate with light. What is the best action to do ?
A. Radiology.
B. ESR.
C. Surgical exploration.

Answer: C
The most likely diagnosis is strangulated hernia
Once the diagnosis of a hernia is made, surgical repair (a herniotomy) will be performed.
Reference: http://www.rch.org.au/kidsinfo/fact_sheets/Inguinal_hernia/

6. One month Baby come with abdominal distension and constipation since birth what is the next step:
A. X-ray
B. Biopsy
C. Rectal manometry

ANSWER: A
Plain abdominal x-ray should always be performed initially for neonate with intestinal obstruction. The presence of air-fluid levels is
evidence of obstruction, but is non-specific. Rectal Biopsy is a gold standard for definitive diagnosis.

Http://bestpractice.bmj.com/best-practice/monograph/750/diagnosis/step-by-step.html

77
7. Patient came to you with history of ingestion of 2 safety pins what's your action? They didn't specify where.
A. Surgery
B. Follow up with x-ray
Answer: B
Objects with sharp edges or points present a special problem because of the possibility for erosion or perforation. These include
pins, needles, tacks, razor blades, pieces of glass, or open safety pins. Children who have swallowed such objects should be vigilantly
observed.
Medscape

8. 12 years old male with undescended testes. The surgeon excised it. What is the most likely histology?
A. Malignant transformation
B. Normal histology
Answer: B
6 % of testicular tumors develop in patients with a history of cryptorchidism (undescended testes).
th
Reference: Surgical Recall, 6 edition

9. child with blue dot in testis and painful mass in inguinal area?
A. Incarcerated hernia
B. testicular appendix torsion
Answer: B

10. pic of bowel obstruction with baby 2 week what is the treatment?
A. primary surgery
B. leavler surgery
Answer: A

11. mass moving with tongue protrusion?


A. Thyroglossal cyst
Answer: A

12. Treatment of hirschsprung disease


A. Surgical repair
Answer: A
th
Reference: Surgical Recall, 6 edition

Answer: A (It’s a case of intussusception, there were more details in the history)

13. Child was diagnosed to have meckel's diverticulae. What is the best diagnostic test?
Answer: ?
Meckel’s scan (technetium-99m pertechnetate scanning)
Meckel's scan has a sensitivity of 85 to 97 percent in pediatric patients
Reference: Uptodate

14. Case: battery lodged at esophagus what to do:


A. Urgent endoscopy?

15. Meckel’s diverticulum?

Http://emedicine.medscape.com/article/931229-overview
Meckel diverticulum (also referred to as Meckel's diverticulum) is the most common congenital abnormality of the small intestine; it
is caused by an incomplete obliteration of the vitelline duct (ie, omphalomesenteric duct).

78
Urology

79
1. Patient came with Lt flank pain radiates to groin no fever no change in urine color or amount no sign other than tenderness
this pain most likely:
A. Ureteric stone
B. Renal colic
C. Diverticulitis

Answer : B
The site of obstruction determines the location of pain. Upper ureteral or renal pelvic lesions lead to flank pain or tenderness,
whereas lower ureteral obstruction causes pain that may radiate to the ipsilateral testicle or labia.
Costovertebral angle tenderness on the affected side is common.
Reference: http://www.merckmanuals.com/home/kidney-and-urinary-tract-disorders/symptoms-of-kidney-and-urinary-tract-
disorders/flank-pain

The majority of renal calculi contain calcium. The pain generated by renal colic is primarily caused by dilation, stretching, and spasm
because of the acute ureteral obstruction. The classic presentation for a patient with acute renal colic is the sudden onset of severe
pain originating in the flank and radiating inferiorly and anteriorly; at least 50% of patients will also have nausea and vomiting.

Http://emedicine.medscape.com/article/437096-overview

2. 60 years old male presenting with flank pain, paraspinal mass, hematuria, weight loss O/E palpable firm mass in the lumbar
area, BP 150/90 What is the best initial investigation? (Clear case of RCC, with the RCC triad)?
A. US
B. CT
C. MRI
D. Radionucliotide

Answer: B- CT
Renal cell carcinoma:
The classic triad of RCC (flank pain, hematuria, and a palpable abdominal renal mass) occurs in at most 9 percent of patients; when
present, it strongly suggests locally advanced disease.
Radiographic testing — The usual first test is abdominal computed tomography (CT) or occasionally abdominal ultrasound.

Http://emedicine.medscape.com/article/281340-clinical
Medscape: Contrast-enhanced CT scanning has become the imaging procedure of choice for diagnosis and staging of renal cell
cancer and has virtually replaced excretory urography and renal ultrasonography.

3. A 50-year-old male diabetic and hypertensive, unable to maintain an erection, started on Phosphodiesterase type 5 inhibitors
(PDE5Is)
Which one of the following drug classes should be avoided?
A) Steroids
B) Antibiotics
C) Narcotics
D) Nitrates
Answer: D
PDE5Is prevent the breakdown of cgmp. Nitric oxide donors (ie, nitrates) increase the production of cgmp. Because both PDE5Is and
nitrates increase cgmp, coadministration can generate excess accumulation of cgmp and can trigger marked vasodilation and severe
hypotension.
Link: http://circ.ahajournals.org/content/122/1/88.full

4. Urge incontinence what is best to do?


A) medical

80
B) Surgical
C) Medical and surgical
D) Behavioral and physiotherapy of bladder
Answer: D
INITIAL TREATMENT — Initial treatment includes lifestyle modifications and pelvic floor muscle exercise for all patients with urinary
incontinence (stress, urgency, or mixed), along with bladder training in women with urgency incontinence and for some women with
stress incontinence
We typically treat with these conservative therapies for six weeks before considering subsequent therapies.

Lifestyle modification — We suggest weight loss to improve symptoms of urinary incontinence in obese women. Other lifestyle
modifications generally focus on other dietary changes. These have been less studied but are often helpful to alleviate symptoms.

Weight loss – Obesity is a known risk factor for urinary incontinence, and weight loss in obese women appears to improve
symptoms of urinary incontinence
Dietary changes – Some beverages may exacerbate symptoms of urinary incontinence. We ask patients to reduce
consumption of alcoholic, caffeinated, and carbonated beverages
Constipation – Constipation can exacerbate urinary incontinence and increase the risks of urinary retention [17].
Constipation should be managed and avoided when possible
Smoking cessation – Smoking has been associated with an increased risk for urinary incontinence [16,18]. However, no
studies have evaluated whether smoking cessation decreases urinary incontinence.
Pelvic floor muscle exercises (Kegel exercises) — We suggest pelvic floor muscle exercises for all women with urinary incontinence.
In women who are able to isolate their pelvic floor muscles to stop urine flow, verbal instruction on timing and frequency of exercise
is usually sufficient. For those with difficulty identifying the proper muscles, supplemental modalities can help women to perform
these exercises properly.

Bladder training — Bladder training is most effective for women with urgency incontinence [21]. Some women who have stress
incontinence only at higher bladder volumes may also benefit from the timed voiding component to keep bladder volumes below
that where stress incontinence occurs. Bladder training and pelvic muscle exercises are often used in combination.
Bladder training starts with timed voiding. Patients should keep a voiding diary to identify their shortest voiding interval. They are
then instructed to void by the clock at regular intervals using the shortest interval between voids identified on their voiding diary as
the initial voiding interval. Urgency between voiding is controlled with either distraction or relaxation techniques (eg, performing
mental math, deep breathing, or by quick contractions of the pelvic floor muscles "quick flicks") [4]. When the patient can go two
days without leakage, the time between scheduled voids is increased. The intervals are gradually increased until the patient is
voiding every three to four hours without urinary incontinence or frequent urgency.

URGENCY INCONTINENCE/OVERACTIVE BLADDER — If initial treatment of urgency, urgency-predominant mixed urinary


incontinence, or overactive bladder (OAB) with incontinence is ineffective, then we suggest a trial of pharmacologic therapy. Women
who fail initial and pharmacologic therapy should be referred to a specialist to consider other options

Pharmacologic therapy — Antimuscarinics are appropriate for women with urgency incontinence symptoms who do not experience
enough improvement with initial treatments. For women who cannot tolerate or do not have sufficient improvement on
antimuscarinics, beta-adrenergic therapy (mirabegron) may be an option. The combination of medication with behavioral therapy is
more effective than either alone, but must be balanced against costs and side effects

5. 46 y. O. Woman with urine incontinence that makes her wake up at night but she urinates herself before going to the
bathroom, what type of incontinence is this?
A. Urge
B. Overflow
C. Stress
Answer: A

81
6. Patient male with urine incontinence, go to the bathroom many times with urine retention. On examination, the bladder was
palpable, what is the incontinence type?
A- Urge
B- Stress
C- Mixed
D- Overflow

Answer: D
Explanation: Overflow incontinence is dribbling of urine from an overly full bladder. Volume is usually small, but leaks may be
constant. Overflow incontinence is the 2nd most common type of incontinence in men. Obstruction leads to a chronically
overdistended bladder, which loses its ability to contract; then it does not empty completely, resulting in overflow.

Reference: https://www.msdmanuals.com/professional/genitourinary-disorders/voiding-disorders/urinary-incontinence-in-adults
7. urge and stress incontinence treatment.

Reference: Toronto Notes.

Stress Incontinence:
With this type, urine leaks due to weakened pelvic floor muscles and tissues. It can happen when pressure on your bladder
increases -- such as when you exercise, laugh, sneeze, or cough.
Urge Incontinence:
This is also called overactive bladder (OAB). With this type, you have an urgent need to go to the bathroom and may not get
there in time.
Overflow Incontinence:
If you can't empty your bladder, you may have overflow incontinence. This means you may dribble urine.
82
8. Urine retention and bilateral hydronephrosis ?
A. Urethral Meatal..
B. Bladder cancer
C. Prostatic cancer
D. Pelvic tumor
Answer: C

Obstruction at or distal to the renal pelvis causes diffuse caliectasis or hydronephrosis. Ureteral obstruction may result from stones,
transitional cell carcinoma, external compression (tumors, enlarged lymph nodes, retroperitoneal fibrosis), blood clots, and fungus
balls. When obstruction has been relieved with a stent, obstruction of the stent can lead to recurrent hydronephrosis.
Bladder tumors can obstruct one or both ureters or ureteral orifices. The most common causes of bilateral obstruction include
bladder outlet obstruction (from prostatic enlargement or posterior urethral valves) and neurogenic bladder. In women, uterine,
cervical, or ovarian cancer should be considered when unilateral or bilateral hydronephrosis is detected.

9. X-ray showed opacity at L2 or L4. Stone at what level:


A- Ureteric
B- Minor calyces
C- Major calyces
D- Renal pelvis
Answer: A
The kidneys are paired retroperitoneal structures that are normally located between the transverse processes of T12-L3 vertebrae,
with the left kidney typically somewhat more superior in position than the right.
The ureter begins its descent to the bladder by running along the medial aspect of the psoas muscle. Here, the ureter lies anteriorly
and slightly medial to the tips of the L2-L5 transverse processes.
It enters the pelvis anteriorly to the sacroiliac joint at the bifurcation of the common iliac vessels (at the pelvic brim) and then
courses anteriorly to the internal iliac artery down the lateral pelvic sidewall.
At the level of the ischial spine it turns forward and medially to enter the posterolateral wall of the bladder, where it runs an oblique
1-2 cm course, before opening into the bladder at the internal ureteric orifice

10. Elderly male, presenting with fever (38.5), dysuria, exam was normal EXCEPT DRE which showed tender enlarged prostate, no
penile secretions, no flank pain, hemodynamically stable, normal level of consciousness. Labs: UA showed 20 WBC +ve citrate
only. Diagnosis?
A- Pyelonephritis
B- Acute Prostatitis
C- Cystitis
D- Gonorrhea
Answer: B

Acute prostatitis clinical presentation and diagnosis: “The clinical presentation of acute prostatitis is generally not subtle. Patients
are typically acutely ill, with spiking fever, chills, malaise, myalgia, dysuria, irritative urinary symptoms (frequency, urgency, urge
incontinence), pelvic or perineal pain, and cloudy urine. Men may also complain of pain at the tip of the penis. Swelling of the
acutely inflamed prostate can cause voiding symptoms, ranging from dribbling and hesitancy to acute urinary retention. Rarely,
patients lack these local symptoms and present instead with constitutional symptoms or a flu-like illness.
On exam, the prostate is often warm, firm, edematous, and exquisitely tender. Common laboratory findings include
peripheral leukocytosis, pyuria, bacteriuria, and, occasionally, positive blood cultures. Inflammatory markers (erythrocyte
sedimentation rate, C-reactive protein) are elevated in most cases. Inflammation of the prostate can also lead to an elevated serum
prostate specific antigen (PSA) level. Thus, if serum PSA testing for prostate cancer screening is planned, it should be deferred for
one month following resolution of acute prostatitis.

Explanation: boggy prostate is prostatitis. Stony hard with obliteration of median sulcus indicates prostate cancer. Rubbery prostate
with uniform enlargement is BPH.

83
Reference First aid USMLE step 1

11. Which of the following renal stones is associated with infection and alkaline urine?
A. Calcium oxalate.
B. Uric acid.
C. Cysteine.
D. Struvite.
Answer: D
Magnesium ammonium phosphate (MAP)
Alkaline urinary ph due to infection with urea-splitting organisms precipitates MAP
Proteus, Pseudomonas, Providencia, Klebsiella, Mycoplasma, Serratia, S. Aureus, NOT E. Coli
Perpetuate UTI because stone itself harbours organism, therefore must remove stone to cure
Infection stone and all foreign bodies must be cleared to avoid recurrence associated with staghorn
Calculi.
Reference: Toronto Notes
Reference: First aid:

Child or neonate diagnosed with UTI which next step before treatment?
A- Urethral culture
B- Cystoscopy
C- Catheter drainage
D- Renal US

Answer: D (if already the patient diagnosed with UA and culture, and the patient is febrile)
- Diagnosis established if urinalysis suggests infection AND if ≥50,000 colony-forming units per of ml a
uropathogen cultured.
- Renal and bladder U/S for all febrile infants with utis looking for anatomical abnormalities, hydronephrosis,
abscess.
Reference:
- Toronto Notes (P 1088)
- Medscape: http://emedicine.medscape.com/article/969643-workup

84
12. You did rectal examination to someone and found there is a mass in front of anterior rectal wall which was diagnosed as
adenoma. What is the location of this neoplasm?
A. Anterior
B. Posterior
C. Lateral
D. Median

Answer: B
The prostate is divided into lobes:
● The anterior lobe is the portion of the gland that lies in front of the urethra. It contains no
glandular tissue but is made up completely of fibromuscular tissue.
● The median or middle lobe is situated between the two ejaculatory ducts and the urethra.
● The lateral lobes make up the main mass of the prostate. They are divided into a right and left lobe
and are separated by the prostatic urethra.
● The posterior lobe is the medial part of the lateral lobes and can be palpated through the rectum
during digital rectal exam (DRE).
Reference: http://www.cdc.gov/cancer/npcr/pdf/abstracting/prostate.pdf

13. A patient had prostate CA and it was removed. Now he presented with metastasis. What is the most probable location?
A. Scalp
B. Brain
C. Bone
D. Lung

Answer: C
The commonest site of metastasis for prostate CA are: Osteoblastic bony lesions, lung, liver, adrenal.
th
Reference: Surgical Recall, 6 edition.
Also “Two triangle Rule that love bone. Thyroid, Breast. Kidneys and Prostate“

14. Patient is having infertility came to infertility clinic. He is complaining of testicular pain. On examination the testicle is
swelled like a bag of worms. What is the diagnosis?
A. Varicocele.
B. Spermatocele.
C. Hydrocele.
D. Testicular cancer.
Answer: A
Palpating a varicocele can be likened to feeling a bag of worms
A varicocele is an enlargement of the veins within the scrotum. A varicocele is similar to a varicose vein that can occur
in the leg.
Varicoceles are a common cause of low sperm production and decreased sperm quality, which can cause infertility
Reference: http://www.mayoclinic.org/diseases-conditions/varicocele/basics/tests-diagnosis/con-20024164

85
Explanation: Varicocele is scrotal varicose veins and is the most common cause of scrotal enlargement in adult males.
Reference Master the board step 2CK

15. Women with dysuria routine urine analysis show epithelial cell diagnosis, What is the source of these cells?
A. Cervical erosion
B. Chlamydia urethritis
C. Vaginal contamination
D. Renal Stone

Answer: C
Women suffering from vaginal infections caused by bacteria may show large amounts of squamous epithelial cells in their urine.
These cells mostly come with bacteria attached and are referred to as clue cells.
Reference: http://www.healthcare-online.org/Epithelial-Cells-in-Urine.html

16. Case of BPH, while prostatectomy, the doctor injured prostatic nerve, this patient will have :
A. Erection inability
B. Urine incontinence
C. Stress incontinence
D. Stool incontinence
Answer: A
Erectile Dysfunction then Retrograde ejaculation. (Answered by a urologist).
17. Stupid question; patient present with flank pain, fever, hematuria, how to prevent the condition?
A. Vit D
B. Calcium
C. Bisphosphonate
D. mannose?
Answer:
If the question is about renal stone we need to control their levels by hydrochlothiazide, which reduces ca in urine.

18. A patient is complaining of recurrent urinary tract infection. Ultrasound revealed kidney stones…. Organism with swarming
motility, more details I don’t remember, what is the most likely organism?
A. Proteus mirabilis
B. Pseudomonas
C. E coli
Answer: A
Struvite stones account for 15% of renal calculi. They are associated with chronic urinary tract infection (UTI) with gram-
negative, urease-positive organisms that split urea into ammonia, which then combines with phosphate and magnesium to
crystalize into a calculus. Usual organisms include Proteus, Pseudomonas, and Klebsiella species
Http://emedicine.medscape.com/article/437096-overview#a5

19. Patient with Rt kidney 14 Cm and left kidney 7 cm .. Arteriography: renal artery stenosis,what to do
A. CT angio
B. Ct abdomen
C. Biopsy
Answer : ??

20. Long Scenario about old male bedridden on foly's catheter he developed Gram -ve bacteria what is the organism
A. E.coli
B. Pseudomonas aergonsa
C. Strep. Puomonia

86
Answer: a
Enteric pathogens (eg, Escherichiacoli) are most commonly responsible
( http://emedicine.medscape.com/article/2040035-overview )
21. Patient with recurrent UTI. Bilateral hydronephrosis. Renal biopsy showed multiple cysts with some description, what is the
diagnosis?
A-Renal cell carcinoma.
B-Polycystic kidney disease.
C-Renal dysplasia.

Answer: B
Approximately 30 to 50 percent of patients with autosomal dominant polycystic kidney disease (ADPKD) will have a urinary tract
infection (UTI) during their lifetime.

Reference: http://www.uptodate.com/contents/urinary-tract-infection-in-autosomal-dominant-polycystic-kidney-disease

22. Patient with adrenal mass on examination. What is the next step before you do FNA?
A. Renin/aldosterone ratio
B. Dexamethasone stress test
C. Free metanephrines

Answer: C
Pheochromocytoma should always be suspected to avoid the risk of lethal hypertensive crises, especially during biopsy or surgery.
Plasma-free metanephrines provide the best test for excluding or confirming pheochromocytoma.
Reference: http://www.medscape.com/viewarticle/442383_3

23. A case of pyelonephritis, what is the next step?


A. Admit and give antibiotics
B. Do investigations
C. Give him antibiotics at home
Answer: scenario not clear.
Diagnosis is usually suggested by the history and physical examination and supported by urinalysis results. Imaging studies are rarely
indicated for the diagnosis of acute pyelonephritis in an adult who presents with typical signs and symptoms. Ambulatory younger
women may be treated in an outpatients setting, pregnant and severely ill patients must be admitted.

24. Man done vasectomy, change his mind and want to reproduce again, they found antisperm antibodies what is the cause?

A. Antigen release or something like that?


B. Cross reactivity with viral infection
C. Inappropriate response of MHC II to antigen presenting cell

Answer: B
Causes of antisperm antibodies are: Trauma, chemical injury, and infection
The above mentioned causes lead to the formation of antisperm antibodies because they lead to the breakage of the blood-testis
barrier, when this barrier is breached, sperm antigens escape from their immunologically protected environment and come in direct
contact with blood elements that launch the antisperm antibodies.
Reference: http://www.newyorkfertilityservices.com/causes-of-infertility/anti-sperm-antibodies/
25. Patient is having unilateral testicular pain and fever. What is the diagnosis?
A. Epidedmo-orchitis.
B. Prostatitis.
C. Testicular torsion.

87
Answer: A
- Epididymitis and epididymo-orchitis are usually caused by a bacterial infection from surgery, urinary catheter, utis, or STD.
Sometimes there is no infection of any kind
- Symptoms include swelling and tenderness, pain, hydrocele and fever.
- Diagnosed by physical examination, urinalysis, and sometimes Doppler ultrasonography
Reference: http://www.merckmanuals.com/home/men-s-health-issues/penile-and-testicular-disorders/epididymitis-and-
epididymo-orchitis

26. Testicular lymph node drainage? ( I am not sure is it testicular or scrotum )


A. Left aortic lymph node
B. Superficial inguinal lymph node
C. Deep inguinal lymph node

Answer: A
- Testis: to para-aortic lymph nodes
Lymphatic drainage of the testis follows the testicular vessels (in the spermatic cord) to the right and left lumbar (caval/aortic) and
preaortic lymph nodes at the second lumbar level.
- scrotal: to superficial inguinal lymph nodes.
Reference: http://teachmeanatomy.info/pelvis/the-male-reproductive-system/testes-epididymis/

27. Best screening method for prostate cancer?


A. Digital rectal examination
B. Cytology for prostate cancer
C. PSA

Answer: C
Digital rectal exam is not as effective as PSA
Reference: http://www.uptodate.com/contents/prostate-cancer-screening-beyond-the-basics

28. 65 patient of difficulty in starting urine and dribbling >PR is normal >PSA is normal >US show medial lobe enlargement = how
to approach
A. Annual Renal function test
B. Repetitive PSA
C. Cystoscope
Answer: C

29. Old patient with back pain and difficulty in urination.


Investigations:
liver is normal
High AlP
High PSA
What's the diagnosis?
A. Prostaitis
B. Prostate cancer
C. bPH
Answer: B (Prostate cancer with bone metastasis most likely)

Explanation: With a PSA level of 4-10 ng/ml, the likelihood of finding prostate cancer is about 25%; with a level above 10 ng/ml, the
likelihood is much higher. Skeletal manifestations are common in metastatic prostate cancer. Elevated alkaline phosphatase aids in
the diagnosis of metastatic bone disease.
Reference: http://emedicine.medscape.com/article/1967731-overview

88
30. Rx uncomplicated 5 yrs cystitis
A. IM Ceftriaxone
B. IV copra
C. Oral….
Answer: C
Children with cystitis usually do not require special medical care other than appropriate antibiotic therapy. A 4-day course of an oral
antibiotic agent is recommended for the treatment of cystitis. Antibiotic agents used include Sulfamethoxazole and trimethoprim
(SMZ-TMP), Amoxicillin and clavulanic acid, Cephalexin, Cefixime, Cefpodoxime, Nitrofurantoin.
Reference: http://emedicine.medscape.com/article/969643-treatment#d12

31. Prostate cancer marker:


A. Carcinoembryonic antigen
B. Alpha fetoprotein
C. Acid phosphatase
Answer: C
Explanation: In this questions C is the right answer. However in general prostate specific antiget (PSA) is a better answer if given in
the answers since acid phosphatase is an old marker that is now widely replaces by PSA
Reference http://www.ncbi.nlm.nih.gov/pubmed/2431533

32. Abdominal solid mass (renal I guess but not sure ) confirmed by?
A. CT
B. MRI
C. US
Answer: C
US detect renal mass, CT for perforation ((Step-Up to Medicine, 4E,p300))

33. To decrease UTI recurrence?


A - decrease urea, decrease PH, increase osmolarity
B - increase urea, decrease PH, increase osmolarity
Answer : B

Biochemical properties are normally important in making bacterial survival difficult in urine: acid ph, high urea content, and high
osmolality.
Https://www.auanet.org/education/adult-uti.cfm

34. Treat HTN and BPH??


A. Atenolol
B. Prazosin
Alpha blockers: These medications relax the muscle of the prostate and bladder neck, which allows urine to flow more easily. There
are at least five medications in this category: terazosin (Hytrin), doxazosin (Cardura), tamsulosin (Flomax), alfuzosin (Uroxatral), and
silodosin (Rapaflo). Terazosin and doxazosin were initially developed to treat high blood pressure, but were later found to be useful
for men with BPH.
Http://www.uptodate.com/contents/benign-prostatic-hyperplasia-bph-beyond-the-basics

35. A patient who has hematuria was diagnosed with renal stones. What is the most likely organism?
A. Schistosoma haematobium
B. E coli
Answer: B

89
36. Which testicular tumor is radio sensitive?
A- Seminoma
B- Teratology
Answer: A
Seminoma is extremely radiosensitive, and low-stage disease is treated with radiation to the inguinal and retroperitoneal areas. Men
with nonseminomatous germ cell tumors of the testis can be managed with observation, chemotherapy or RPLND.
Link: http://www.aafp.org/afp/1999/0501/p2539.html

37. Patient with back pain and signs of retention, what to do first?
A. PSA
B. Urodynamic

ACUTE MANAGEMENT — The initial management of acute urinary retention (AUR) is prompatient bladder decompression by
catheterization.
Options for bladder decompression — Bladder decompression can be accomplished with urethral or suprapubic catheterization.
There are no uniform guidelines for bladder decompression. Most patients will have an initial attempatient at urethral
catheterization.
Urethral catheterization — An initial attempatient at urethral catheterization is appropriate for most patients, particularly in
patients for whom AUR is expected to resolve (eg, patients with urinary tract infections or AUR secondary to medication effect).
Urethral catheterization is contraindicated in patients who have had recent urologic surgery (eg, radical prostatectomy or urethral
reconstruction) and these patients should have suprapubic catheterization. Although there is a theoretical risk to placement of a
urethral catheter in the setting of acute bacterial prostatitis, these patients may have an attempatient at gentle urethral
catheterization by an experienced clinician.

Indwelling catheter — A 14 to 18 gauge French catheter should be inserted as first-line therapy in most patients with AUR [15].
Some patients may have an obstruction that does not readily allow passage of the catheter. If the patient has had a prior
transurethral procedure (eg, transurethral resection of the prostate), a partially obstructing urethral or prostatic scar may be
present. In this case, the obstruction may be bypassed by downsizing the catheter to a 10 or 12 gauge French indwelling catheter.

Suprapubic catheter — Placement of a suprapubic (SP) catheter is sometimes necessary in patients who have contraindications to or
fail urethral catheterization (eg, those with recent urologic surgery, acute prostatitis, urethral stricture disease, severe benign
prostatic hyperplasia (BPH), or other anatomic abnormalities)

38. Patient has difficulty to urinate and back pain. High prostatic enzyme. Most likely diagnosis?
A. BPH
B. Prostatic cancer
Answer: B

Symptoms — Most men with early stage prostate cancer have no symptoms attributable to the cancer.
Urinary frequency, urgency, nocturia, and hesitancy are seen commonly but are usually related to a concomitant benign
prostate enlargement.
Hematuria and hematospermia are uncommon presentations of prostate cancer but their presence in older men should
prompatient consideration of prostate cancer in the differential diagnosis. These symptoms are also present in men with
benign prostatic hyperplasia (BPH) and are more likely to be caused by BPH than cancer.
Bone pain may be the presenting symptom in men with metastatic disease but an initial diagnosis when bone metastases
are present has become unusual
Men with abnormal prostate exams (nodules, induration, or asymmetry) should be referred to a urologist for a prostate biopsy, with
a the histologic diagnosis based upon tissue obtained from the biopsy. A prostate biopsy may also be indicated based upon
abnormal PSA values.

90
39. 2 YO with scrotal cyst?
A. Spermatocele
B. Hydrocele
Answer: B

40. Prostatic ca w/ back pain investigation?


A. Back sincitograph
B. Back CT
Answer: B
He has mets so it should be CT Toronto notes
Http://emedicine.medscape.com/article/379996-overview#showall

41. Old male patient presented with urgency and frequncy symptoms whats the first thing that u will do ?
A- PSA
B- transrectal US
Answer: ???

Lower urinary tract symptoms may be divided into voiding and storage, and men may present with a combination of the two
symptom groups. Voiding symptoms include weak stream, hesitancy, and incomplete emptying or straining and are usually due to
enlargement of the prostate gland. Storage symptoms include frequency, urgency and nocturia and may be due to detrusor
overactivity.
Http://www.racgp.org.au/download/documents/AFP/2011/October/201110arianayagam.pdf

42. What is the most common part of urethra could be injured during catheterization?
A. Membranous urethra
B. Prostatic urethra
C. Penile urethra
Answer A (narrowest part)
Urethral injury due to trauma is a common complication associated with Foley catheters especially if the catheter is inserted
forcefully or roughly. A significant urethral injury can even cause death. (http://patientmodesty.org/urinarycatheterizationrisks.aspx)
^Urology Resident

43. Child develop straddle injury after playing with bicycle presented with scrotal and penile hematoma “I think” which structure
would probably be injured:
A. Penile urethra
B. Prostatic urethra
C. Membranous urethra
Answer: C

^Urology Resident

The membranous urethra is prone to injury from pelvic fracture. The bulbar urethra is susceptible to blunt force injuries because of
its path along the perineum. Straddle-type injuries from falls or kicks to the perineal area can result in bulbar trauma. Conversely,
the penile urethra is less likely to be injured from external violence because of its mobility, but iatrogenic injury from catheterization
or manipulation can occur.
Reference: http://emedicine.medscape.com/article/451797-overview#a11

91
44. Cancer associated with Smoking?
A. Bladder
B. Colorectal
th
Answer: A. Reference: Surgical Recall, 6 edition

45. Old patient with COPD, DM and now he’s diagnosed to have BPH, what medication will you use?
A. B-Blocker
B. Terazosin

Answer: B
Terazosin is an alpha blocker
46. The narrowest part of male urethra is?
A. Membranous urethra.
B. Penile urethra.
Answer: A
membranous urethra (1 cm long): passes through the urogenital diaphragm, surrounded by sphincter urethra the shortest and
narrowest portion.
Reference:
Http://radiopaedia.org/articles/male-urethra

47. bladder ca surgical removed and the area of cancer now clear and no invasion of the muscle
A. Follow up with cystoscopy and biopsy
B. Intravesical chemotherapy..

Answer: B
Reference : uptodate + toronto notes
http://www.uptodate.com/contents/overview-of-the-initial-approach-and-management-of-urothelial-bladder-
cancer?Source=outline_link&view=text&anchor=H3#H3

48. Patient with hematuria diagnosed bladder cancer did resection what is the next step ?
A. BCG intrbladder
B. Mitomycin-C

Answer: B
Within the first 24 hours, a single intravesical instillation of (40 mg in 20 ml of saline) has been shown to reduce the frequency of
tumor recurrence.
Reference: http://emedicine.medscape.com/article/1951622-overview#a4

49. 3 years old boy with UTI,, what you’ll do before starting treatment (Ascending urethrogram was not an option)
A. -US
B. -culture urethra !!

Answer: (the answer should be: send urine culture)


-Urine culture should be done beside urine analysis , but do not delay antibiotic.
- Renal ultrasonography should be considered for any child with a first febrile UTI in whom good follow-up cannot be
ensured.
- Blood cultures are not recommended .
Source :
http://emedicine.medscape.com/article/969643-workup

92
Toronto note :
U/s in recommended for all febrile "infants" with UTI
uptodate : US is done if :
• Child with recurrent UTI at any age
• Children younger than 2 yrs with a first febrile UTI
• Children who dont respond as expected to appropreate ttt .
• Children of any age with with a UTI who have family history of renal or urologic disease , poor growth or HTN.

50. Non-invasive papillary tumor of the bladder, size 2 cm ( more details ). What is the next step?
A. Intra-vesical BCG
B. Radical
Answer: A
Explanation: non-invasive papillary carcinoma of the bladder is staged Ta. The management of which is intra-vesicle BCG.
Reference: http://emedicine.medscape.com/article/438262-treatment#d10

51. Why to use inulin to estimate GFR? Active biological


A. Passive free in glumurs
Inulin is useful as an indicator of GFR because the kidneys handle it in a unique way. Unlike most other substances in the blood,
inulin is neither reabsorbed into the blood after filtration nor secreted through peritubular capillaries. Thus, the amount of inulin
cleared through the urine is indicative of the amount of plasma filtered by the body’s glomeruli.
Https://www.khanacademy.org/test-prep/mcat/biological-sciences-practice/biological-sciences-practice-tut/e/measuring-
glomerular-filtration-rate-of-the-kidneys-with-inulin
st
+ Davidson, 21 edition page 464

52. Patient on chemotherapy Prevention of hemorrhagic cystitis:


Answer: Mesna
The best treatment for HC is prevention. Effective prophylactic regimens include continuous bladder irrigation, suprahydration, and
parenteral or oral mesna (2-mercaptoethane sodium sulfonate, a sulfhydryl compound). However, once HC is established, the
optimal treatment depends upon the degree or grade of hematuria

53. Patient presented with non-traumatic acute urinary retention. How you will treat the patient?
a. Foley catheter

Answer: A
Reference: UpToDate

54. Pt presented with urethral injury (already established) with blood coming out from the meatus, what will you do for him?
A. Insert suprapubic catheter.
Answer: ?
Rectal and pelvic examinations are of utmost importance during the initial evaluation to rule out the presence of an open fracture.
Blood in the vaginal vault or in the rectum should raise the level of suspicion for an open injury. Palpable bony spicules within the
rectum or vagina may be present indicating an open injury. A high-riding prostate may also be detected on rectal examination,
indicating the presence of a periurethral or periprosthetic hematoma occurring secondary to genitourinary injury >> Pelvic
stabilization for patients with pelvic fractures who are hemodynamically unstable. Reference:
http://www.sportssurgerynewyork.com/articles/diagnosis-management-pelvic-fractures.pdf

In any male patient with suggestive symptoms or signs urethral injury, the diagnosis is confirmed by retrograde urethrography. This
procedure should always precede catheterization. Urethral catheterization in a male with an undetected significant urethral injury

93
may potentiate urethral disruption (eg, convert a partial disruption to a complete disruption). Female patients require prompt
cystoscopy.

55. Elderly patient is having bilateral Hydronephrosis:


A. BPH

Answer: A There are a lot of causes check the link.


Reference: http://emedicine.medscape.com/article/436259-clinical#b5

56. A man presented with pain and mass in the flank. What is the diagnosis?
Answer: ?
DD of flank pain and mass:
● Renal abscesses: patient have pain because inflammation and edema produce stretch of renal capsule.
Sometimes, a flank mass may be palpable.
● Polycystic kidney disease.
● Renal vein thrombosis.
● Renal cell carcinoma (pain, mass and hematuria)
Reference: Medscape + rightdiagnosis

57. Pt with testicular mass after excision you took a sample and see it under microscope it showed (I can't recall) what is the type
of malignancy...? - seminoma - yolk sac -???
Answer: no enough information.
• Histology:
Seminoma
Large cells in lobules with watery cytoplasm
"fried egg" analogous to dysgerminoma of the ovary
Embryonal carcinoma
Glandular/papillary
Yolk sac (endodermal sinus) tumor
Yellow, mucinous Schiller-Duval bodies resemble primitive glomeruli
Choriocarcinoma
Disordered syncytiotrophoblastic and cytotrophoblastic elements
Teratoma
Contain three tissue types
Leydig cell
Contains Reinke crystals
Reference: usmlebullets.com
58. One cell type testicular mass, what is the diagnosis?
A. Seminoma

94
Answer: ?

Most common urologic diagnosis in men <50 yr. Prevalence 2-12%

* KEEPS Klebsiella sp.


E. Coli (90%), other Gram-negatives Enterococci
Proteus mirabilis, Pseudomonas
S. Saprophyticus

95
Reference: Toronto Notes

59. Renal artery stenosis, Angie was done , what is next ?


Since the renal artery stenosis is already detected by the angiography, there is no role for CT scan further this point. The next
step is to differentiate between atherosclerosis & the fibromuscular dysplasia since the management of them differ.
There is similar question with ( renal artery biopsy ) was one of the choices. I think it is the proper next step.
Angioplasty with stenting?

When renal function is normal or nearly normal, specialists recommend revascularization for prevention of renal
insufficiency if the patient meets the following criteria:
• The degree of stenosis is more than 80-85%
• The degree of stenosis is 50-80%, and captopril-enhanced scintigraphy demonstrates an activation of intrarenal renal
artery stenosis
Reference: Medscape

60. Pt presented to you with Rt scrotal swelling, he said that it is increasing in size day after day, on examination , it is not trans-
illuminate , what will you do ?
A. Reassure him and get surgical opinion

Answer: the question is missing important info


Differential diagnosis in this case can be hematocele or testicular cancer and both of them need further evaluation by US
and depending on the results further investigations may be ordered.
Reference: https://www.merckmanuals.com/professional/genitourinary-disorders/symptoms-of-genitourinary-disorders/painless-
scrotal-mass

61. scrotal pain with dysuria


Answer: It is mostly a case of epididymo-orchitis:
1- it is's caused by STD ( gonorrhea or chlamydia ), treat with Ceftriaxone+ Azithromycin, OR with doxycycline .
2- if from urinary pathogens, fluoroquinolones are the preferred antibiotic, as they have excellent penetration into the
testes. (Trimethoprim-sulfamethoxazole OR Amoxicillin-clavulanate can be used)

Http://emedicine.medscape.com/article/2018356-overview

62. gene mutation of suprarenal tumor?

The Li-Fraumeni syndrome is caused by inherited mutations that inactivate the p53 tumor suppressor gene. This syndrome causes
few cases of adrenal cancer in adults (1 of every 20), but is often the cause of adrenal cancer in children. In fact, about 8 of every 10
cases of adrenal cancer in children are caused by Li-Fraumeni syndrome. Many other adrenal cancers have also been found to have
abnormal p53 genes that were acquired after birth (not inherited).
Http://www.cancer.org/cancer/adrenalcorticalcancer/detailedguide/adrenal-cortical-cancer-what-causes
Mutations in the VHL gene cause von Hippel-Lindau syndrome.
Http://ghr.nlm.nih.gov/condition/von-hippel-lindau-syndrome
Many different syndromes

96
63. Patient diagnose with renal cell carcinoma which gene do u think have mutation?
A. PKD
B. PKHD-1
C. VHL

Answer: C
Referance : http://emedicine.medscape.com/article/281340-overview#a3

64. Kidney tumor oncogene?


A. P53
Answer:
-MET proto oncogene responsible for hereditary papillary renal cell cancer
Ref : PubMed
http://www.ncbi.nlm.nih.gov/pmc/articles/PMC2929006/

-Alteration of p3 chromosome result in alteration of either tumor suppressor gene ( VHL , TSC ) or oncogene ( MET )
Ref : Medscape
http://emedicine.medscape.com/article/281340-overview#a2

65. A patient presented with trauma on the right side of abdomen, he then developed abdominal swelling and hematuria?
A. Wilms tumor
Explanation: Wilms tumor is the most common primary renal neoplasm of childhood & most common abdominal mass in children.
80% with unilateral abdominal mass. They may also develop hypertension, gross hematuria, abdominal pain and vomiting.
Reference Toronto notes

66. IHD or (DM + HTN) with BPH ? Medication? The choices were: 2 types of ACE 1 Beta Blocker 1 choice of I do not know the
optimal Rx is alpha blocker (prazocin etc).

Answer: Alpha blockers are preferred for such a scenario (HTN & BPH) (uptodate, Medscaper…etc). In practice, a “prostate”
selective alpha blocker like Tamsulsin is used for BPH along with 1st line Antihypertensive drugs for HTN (ACE/ARB, CCB, Thiazide.) If
no Alpha blockers in the answers, then avoid BB because they mask the symptoms of DM. ACEI would be the first choice especially
that they are first line in the new HTN guidelines (JNC8) plus the fact that they are useful for diabetic nephropathy, and they are not
diuretics (BPH).
67. Case of urine incontinence. What mechanism of action of oxybutynin?
Competitive antagonist of acetylcholine at postganglionic -muscarinic receptors

68. Patient diagnosed with papillary renal carcinoma, treatment?


Http://emedicine.medscape.com/article/281340-treatment

69. Patient with bladder cancer post-surgery what to inject?


BCG (bacillus Calmette-Guérin)
Reference: http://emedicine.medscape.com/article/1950803-overview

70. Man done vasectomy, change his mind want to reproduce again, they found antisperm antibodies what is the cause?
A. Antigen release or something like that?
B. Cross reactivity with viral infection
C. Inappropriate response of MHC II to antigen presenting cell
Answer: A
Antigen releasing
97
Urology resident
Http://www.medscape.com/viewarticle/571266_2

71. Ureteral stone what is the radiological study to detect it?


A. Abdominal ultrasound
B. CT with contrast
C. CT without contrast
There was No x-ray in options
Answer: C
Bailyand love
Http://radiopaedia.org/articles/ureteric-calculi
Refrence: http://www.merckmanuals.com/home/kidney-and-urinary-tract-disorders/stones-in-the-urinary-tract/stones-in-the-
urinary-tract

72. Left side pain from lion to groin Dx:


A) Renal stone
B) Acute app
C) Acute pancreatitis
D) Duodenal ulcer
Answer: A
Reference: http://emedicine.medscape.com/article/437096-overview

73. Enuresis MCQ.

74. Vesicoureteral reflux, read about it.

98
ENT

99
1. Young female came to your office complaining of swelling in front of her parotid gland. No tenderness, no secretion coming
out, what is the most likely diagnosis?
A. Parotid ca
B. Sialadenitis
C. Mumps
D. LN enlargement

Answer: a
The most common presentation of parotid ca. Is a painless, asymptomatic mass; >80% of patients present because of a mass in the
posterior cheek region. Some people infected with the mumps virus have either no signs or symptoms or very mild ones. When signs
and symptoms do develop, they usually appear about two to three weeks after exposure to the virus and may include: swollen,
painful salivary glands on one or both sides of your face (parotitis).
Reference: medscape and myoclinic

2. How to assess hearing in adult:


A. Recording music
B. Recording dog parking
C. Sound of watch
D. Paper flabbing
Answer: c
Reference: aafm

3. Which part of the ear is responsible of angular acceleration:


A. Saccula
B. Macula
C. Organ of corti
D. Semilunar canal
Answer: d
http://neuroscience.uth.tmc.edu/s2/chapter10.html

The anterior and posterior semicircular canals are oriented vertically at right angles to each other. The lateral semicircular canal is
about 30-degree angle from the horizontal plane. The orientations of the canals cause a different canal to be stimulated by rotation
of the head in different planes. Thus, the horizontal canal detects angular acceleration of the head (such as when you spin in a
rotating chair), while the superior and posterior canals detect vertical head movements (such as when you bend forward to pick
something up from the floor).

4. What is the most sensitive part to the linear acceleration?


A. Semicircular canal.
B. Organ of corti.
C. Utricle.
D. Saccule.
Answer: c
Explanation: the otolith organs sense gravity and linear acceleration such as from due to initiation of movement in a straight line.
Persons or animals without otolith organs are imbalanced.. A set of hair cells are coupled to a mass of stones. When the stones

100
accelerate, with respect to the hairs, they exert a shearing force on the hairs. This force is detected by the hair cells and sent to the
brain via branches of the vestibular nerve. The utricle sends input to the brain via the superior division of the nerve, and the saccule,
via the inferior division. There is considerably more complexity to the organization of the utricle and saccule, including different
types of hair cells and detail to the sensory macule (patch of sensory cells) that we have omitted.
The otolithic organs sense motion according to their orientation. The utricle is largely horizontal in the head, and largely registers
accelerations acting in the horizontal plane of the head (called the axial plane by radiologists). The saccule is largely vertical, actually
parasagittal, in the head, and registers accelerations in the vertical plane (called parasaggital or coronal plane).
Otolithic organs= the saccule and utricle >> for linear acceleration
Reference: http://www.tchain.com/otoneurology/disorders/bppv/otoliths.html + medscape

5. Pregnant 32 year old came with hearing loss, her mother same condition what is the cause?!
A. Acoustic neuroma
B. Presb
C. Perforated tm
D. Otosclerosis
Answer: d?
Answer a is not supported with other presenting symptoms such as vertigo or tinnitus. Answer b is unlikely because of the patient’s
age. Answer c no risk factors such as recurrent otitis media to support this choice. Otosclerosis: fusion of stapes footplate to oval
window so it can't vibrate. It’s ad, more common in female; progress during pregnancy ( hormone responsive)
Reference: toronto note

6. 6 years old female underwent tonsillectomy and started bleeding 24 hours after surgery. You examined the surgical site and
found no abnormality. Her platelet count, bleeding time, platelet function, pt, aptt and clotting time are all normal, what test
confirms her diagnosis:
A. Fibrinogen
B. Plasminogen
C. Clot lysis test
D. Vwf
Answer: c

7. 17 years old male presented with hearing loss in the left ear (picture of conductive hearing loss audiogram). What is your
diagnosis?
A. Presbycusis
B. Otosclerosis
C. Interosseous
D. Otitis media

Answer: b
Explanation: otosclerosis is traditionally diagnosed by characteristic clinical findings, which include progressive conductive hearing
loss, a normal tympanic membrane, and no evidence of middle ear inflammation.

101
8. Patient with nasal sx. What is the "best" treatment for it ?
A. Steroid
B. Antihistamine.
C. decongestants
D. Ab

Answer: a
Explanation:
Prevention with avoidance of the precipitating allergen:
• Close the windows and use air conditioning to avoid pollen.
• Get rid of animals to which the patient is allergic.
• Cover mattresses and pillows.
• Use air purifiers and dust filters.
Intranasal corticosteroid sprays
antihistamines: loratidine, clemastine, fexofenadine, brompheniramine
intranasal anticholinergic medications: ipratropium
desensitization to allergens that cannot be avoided

Reference: master the board


9. How to examine child ear? Pull pinna?
A. Inf backwards
B. Sup backward
C. Inf forward
D. Sup forward
Answer a?
B for adult
Explanation:
For children above 3 years: sup backward
For children under 3 years: inf backwards
Reference: http://www.atitesting.com/ati_next_gen/skillsmodules/content/physical-assessment-
child/equipment/ap_ear_nose_throat.html

10. Patient in his work expose to noise what type of hearing problem may has:
A. Conductive hearing loss
102
B. Non sensory hearing loss
C. Sensory hearing loss
D. Vomiting
Answer: c sensorineural hearing loss (noise induced snhl)
11. Picture of tonsillitis, what is the best drug for treatment?
A. Ceftriaxone.
B. Acyclovir.
C. Ampicillin.
D. Doxycycline.
answer: ?
Explanation: the best is penicillin or amoxicillin; penicillin allergic patient are treated with cephalexin if the reaction only rash , if
allergy is anaphylactic use clindamycin or a macrolides
Reference: master the board step 2 CK

12. Painful ear, fever with tm having hemorrhagic vesicles on pe. What’s the cause?
A. Otitis externa
B. Fungal
C. Streptococcus pyogenes
D. Pseudomonas
Answer :
This is called myringitis
(particularly streptococcus pneumoniae), or mycoplasma otitis media.
http://www.merckmanuals.com/home/ear,-nose,-and-throat-disorders/middle-ear-disorders/infectious-myringitis
Myringitis is a form of acute otitis media in which vesicles develop on the tympanic membrane>>> most common cause of acute
otitis media is streptococcus pneumonia. However, it is not hemorrhagic.
This article explained bollous hemorrhagic myringitis as a form of otitis externa, hence, the etiologic organism could be the same
(pseudomonas) (http://oto.sagepub.com/content/148/2/347.extract )
While this article state streptococcus pneumoniae being a possible organism for hemorrhagic vesicles
(http://www.sbccp.org.br/arquivos/oto-2013-02-bullous-hemorrhagic.pdf)
13. Young patient presented with fever, vertigo, nystagmus no hearing loss, for the past 2 days, cerebellum signs are present,
what is the diagnosis?
A. Central vertigo.
B. …… neuritis.
C. Menieres.
D. Bppv.
Answer from medscape : a- central vertigo .

14. 3 weeks post antibiotic treatment of acute otitis media reveals non erythematous tympanic membrane which doesn't move
with insufflation with fluid collection, what to do next?
A. Antibiotic
B. Watchful waiting
C. Decongestant
D. Anti-histamine
Answer: b
Otitis media with effusion usually resolve spontaneously within 3 months
http://patient.info/doctor/otitis-media-with-effusion

according to Toronto notes this is an “otitis media with effusion” the treatment for which is expectant waiting for resolution of the
effusion. No clinical evidence supports the use of antibiotics.

103
15. Most sensitive test for maxillary sinusitis?
A. CT
B. US
C. Transillumination
D. X-Ray

Answer: a
Explanation: ct scanning is the preferred imaging method for rhinosinusitis. A screening sinus ct scan is adequate for diagnosis and
less expensive than other methods but is necessary only in cases of treatment failure or chronic rhinosinusitis.
Reference: http://emedicine.medscape.com/article/232670-workup#c13

16. Scenario consistent with sialadenitis (the pain in the submandibular area ) they are asking about the location of obstruction :
A. Submandibular
B. Sublingual
C. Parotid
D. Submental
Answer:a
Ductal stone (mainly hydroxyapatite) in adults, sand/sludge in children, leading to chronic sialadenitis .80% in submandibular gland,
<20% in parotid gland, ~1% in sublingual gland. The clinical presentation pain and tenderness over involved gland ,intermittent
swelling related to meals and digital palpation reveals presence of calculus.

17. Patient with thyroid nodule and tortious dilated artery that compress the external laryngeal nerve, which of the following
will be affected
A. Tone of the voice
B. Lateral adduction of vocal cord
C. Abduction of vocal cord
D. Sensation above the vocal cord

Answer: a

18. Child with ear secretion with defenses and move pinna what is diagnosis ?
A. Otitis media
B. Cholesteatoma
C. Otitis externa
D. Foreign body
Answer: c

19. Man complaining of ear pain. On examination he feels pain when moving ear pinna and there is erythema of auditory canal
and normal tympanic membrane. What is the most likely diagnosis?
A. Otitis externa.
B. Otitis media.
C. Squamous cell carcinoma.

Answer: a
Explanation: Clinical features:
acute:
Pain aggravated by movement of auricle (traction of pinna or pressure over tragus)
Otorrhea (sticky yellow purulent discharge)
Conductive hearing loss aural fullness 2o to obstruction of external canal by swelling andpurulent debris

104
Posterior auricular lymphadenopathy
Complicated OE exists if the pinna and/or the periauricular soft tissues are erythematous and
Swollen

Reference: toronto notes 2015

20. Otalgia, fever, sore throat? What is the nerve that refers ear pain?
A. Glossopharyngeal
B. Vagus
C. Sphenopalatine
D. Nasopalatine
Answer : a

This is referred otalgia. The general ear region has a sensory innervation provided by four cranial nerves and two spinal segments.
Hence, pathology in other "non-ear" parts of the body innervated by these neural pathways may refer pain to the ear. These general
pathways are:
Via trigeminal nerve [cranial nerve v]. Rarely, trigeminal neuralgia can cause earaches. Oral cavity carcinoma can also cause
referred ear pain via this pathway.
Via facial nerve [cranial nerve vii]. This can come from the teeth, the temporomandibular joint (due to its close relation to the
ear canal), or the parotid gland.
Via glossopharyngeal nerve [cranial nerve ix]. This comes from the oropharynx, and can be due to pharyngitis, pharyngeal
ulceration, tonsillitis, or to carcinoma of the oropharynx (base of tongue, soft palate, pharyngeal wall, tonsils).
Via vagus nerve [cranial nerve x]. This can arise from the laryngopharynx in carcinoma of this area, or from
the esophagus in gerd.
Via the second and third spinal segments, c2 and c3.

21. Nerve can cause problems in eye and ear ?


A. Oclouomotor 3
B. Trochlear 4
C. Facial 7
D. Abducent 6
Answer: c
Motor innervation to 1) stapedius muscle (adjusts stapedius bone in response to loud sounds) 2) posterior belly of digastric muscle
3) muscles of facial expression (buccinator, platysma, occipital muscles)
Innervation of 1) lacrimal gland (tearing of the eye) 2) sublingual glands (salivation) 3) submandibular gland (salivation) 4) mucous
membranes of nose.

22. Patient developed imbalance, tinnitus and decrease hearing what drug he took:
A. ...mycin (this is most probably the correct answer as this suffix is probably denoting an aminoglycoside)
A. Isoniazid
B. Ethambutol
C. Pyramedizine
Ototoxic medications:
• Aminoglycosides. Aminoglycoside antibiotics (e.g., kanamycin, neomycin, amikacin, streptomycin, gentamicin) exhibit
[3,4]
cochleotoxicity but also affect the stria vascularis, causing vestibular problems. they produce damage through the
ability to generate free radicals in the inner ear.
• Loop diuretics. Loop diuretics (e.g., furosemide, ethacrynic acid, bumetanide) affect the potassium gradient of the stria
[2,3]
vascularis, as well as the electrical potential of the endocochlear structure. these medications produce tinnitus and

105
hearing loss. The hearing loss may be perceptible to patients or may be apparent only with audiometric testing. Their
[12]
toxicity is dose-related. thus, ototoxicity is more likely when the patient receives a rapid infusion of injectable loop
diuretics in renal failure, which allows the medications to accumulate. Furosemide-related ototoxicity is usually
reversible but may be permanent in rare instances (e.g., in patients with renal failure).
• Cisplatin affects the cochlea and stria vascularis through its ability to generate free radicals within the inner ear.
• Salicylates impact the cochlea. In high doses, they cause tinnitus and loss of hearing; both are usually seen only with
[7]
higher doses and regress upon discontinuation in most instances. the relationship between salicylate serum
concentrations and the level of hearing loss is linear.

23. Lump in throat barium enema and endoscopy normal?


A. Squamous cancer
B. GERD
C. Peptic ulcer
D. Pharyngeal Globus
Answer: D

Globus sensation is a functional esophageal disorder characterized by a sensation of a lump or foreign body in the throat. Globus
sensation has also been referred to as globus pharyngeus and the misnomer "globus hystericus." it is characterized by a sensation of
a lump, retained food bolus, or tightness in the throat that is not due to an underlying structural lesion, gastroesophageal reflux
disease, mucosal abnormality, or an esophageal motility disorder.
Patients have a sense of a lump, a retained food bolus, or tightness in the throat. Globus sensation is not painful and is typically
worse when swallowing saliva (dry swallow) and less noticeable when swallowing solids or liquids. In approximately 70 percent of
patients, globus symptoms are intermittent.

Http://www.uptodate.com/contents/globus-sensation?Source=outline_link&view=text&anchor=H480081804#H480081804

24. Man was has cholecystectomy now complaining of unilateral parotid swelling ,he has hx of mumps, facial nerve intact ,no
decrease in salivation ,lab was show cloudy saliva?
A. Sarcoidosis granuloma
B. Sialadenitis
C. Parotid cancer
D. Mumps
Answer: b
Patients typically present with erythema over the area, pain, tenderness upon palpation, and swelling. Frank cellulitis and induration
of adjacent soft tissues may be present. Purulent material may be observed being expressed from the wharton duct, particularly
upon milking the gland.
The most common organism is staphylococcus aureus. Other bacterial organisms include streptococcus viridans, haemophilus
influenzae, streptococcus pyogenes, and escherichia coli. The infection is often the result of dehydration with overgrowth of the oral
flora. The most common causes are postoperative dehydration, radiation therapy, and immunosuppression (eg, diabetes mellitus,
organ transplant, chemotherapy, human immunodeficiency virus). .medscape

25. Elderly patient with sudden onset hearing loss, loss of balance, and loss of eye blinking, where is the tumor?
A. Foramen ovale
B. Foramen secundum
C. Acoustic
Answer: C, acoustic neuroma or called schwannoma
Symptoms of acoustic neuroma: unilateral hearing loss, tinnitus, facial numbness and weakness due to compression on trigeminal
and facial nerve and imbalance. Source: bmj best practice.
http://www.nidcd.nih.gov/health/hearing/pages/acoustic_neuroma.aspx

106
26. Patient with an urti with hoarseness and painful swallowing, where is the site of infection
A. Larynx
B. Bronchiole
C. Trachea
Answer: larynx:

27. Most common or specific sings for otitis media:


A. Pain
B. Hearing loss
C. Discharge
Answer: a
Most common: pain. Most sensitive: immobility
Reference: master the board , uqu

28. Prevent symptoms of allergy to pollens and dust by using what:


A. Terbutaline
B. Montelukast
C. Ipratropium bromide
answer: b
Reference: toronto note

http://www.aafp.org/afp/2010/0615/p1440.html

107
29. Patient have sleep apnea & on pacemaker not responding to cpap (continuous positive airway pressure) what you do ?
A. Nose surgery
B. Chest surgey
C. Do cpap again
Answer: c?

30. 55 years with unilateral nasal chronic obstruction and epistaxis what you suspect
A. Polyp
B. Malignancy
C. Fibroangioma
Answer: b

31. A person fell and developed bruise and laceration over the nose. X-ray shows no fracture or displacement, what to do?
A. Ask for ct
B. Refer to ent
C. Nasal packing
Answer: c

32. 18 years old ear examination is normal and there is audiogram picture, what's the diagnosis?
A. Presbycusis
B. Otosclerosis
C. Otitis media
Answer : source of diagram: toronto notes.

33. A 35-year-old smoking man has white plaques in his mouth. What will you do?
A. Excisional biopsy
B. Close follow up
C. ?Lymph node
Answer: a , to roll out cancer

108
Leukoplakia — oral leukoplakia is a precancerous lesion that presents as white patches or plaques of the oral mucosa ( picture 2a-
c ). It represents hyperplasia of the squamous epithelium, which is believed to be an early step in the transformation of clonally
independent premalignant lesions from hyperplasia, to dysplasia, to carcinoma in situ, to invasive malignant lesions.
Risk factors for oral leukoplakia are similar to that of squamous cell carcinoma; it is particularly common in smokeless tobacco users
[ 7 ]. Any indurated areas should be biopsied to rule out carcinomatous changes.
https://yhdp.vn/uptodate/contents/mobipreview.htm?25/11/25786

34. Unilateral obstruction nose then foul smell, nasal discharge , erosin of vestible , normal in exam of nose
A. X ray head and chest
B. Nasal exam in ga
C. Antiboitic
Answer: this is a case of rhinosinusitis
http://www.uptodate.com/contents/acute-sinusitis-and-rhinosinusitis-in-adults-treatment

35. Patient presention with hearing loss , dizziness , vertigo what are you going to do to locate the lesion ?
A. Audiogram
B. MRI cerebellopontine
C. Ct of temporal area
Answer: a

36. Most common cause of tinnitus is:


A. Hypertension
B. Chronic use of salicylate
C. Sensorineural hearing loss (SNHL) – Got 100% in ENT
Answer: C
Toronto
Subjective tinnitus ( only heard by the patient ):
common otologic cause is: presbycusis. Common drug cause: asa
Objective tinnitus ( can be heard by others ):
benign intracranial htn
Reference: toronto note.
http://www.webmd.com/a-to-z-guides/understanding-tinnitus-basics

37. Nasal bleeding, first thing to do:


A. Balloon
B. Press on soft part of the nose
C. Press the carotid
Answer: b
http://cursoenarm.net/uptodate/contents/mobipreview.htm?12/45/13010

38. Patient has congestestion in tonsils and paratonsillar and c/o pain and cann't open mouth ?
A. Quinsy
B. Peritonsillar abscess
C. Tonsillitis
Answer: a or b (both are the same)
Explanation: the inability to open his mouth is trismus, which along with uvular deviation and dysphonia are the triad for
peritonsillar abscess (quinsy). The paratonsillar swelling also supports the diagnosis. There is most likely a mistake when writing the
choices since both a and b are the same.
Reference: toronto notes 2015, page ot42, otolaryngology

109
39. Young patient with conducting hearing loss and semicircular canal dehiscence
A. Glue ear
B. Otosclerosis
C. Tympanosclerosis

40. Nerve supply to tensor tympani and stapedius :


A. Trigeminal and facial
B. Facial and auditory
C. Trigeminal and ....

Answer :a
The stapedius is a muscle with a high oxidative capacity. It is innervated by the stapedial branch of the facial nerve (7th nerve). The
tensor tympani is innervated by a branch of the mandibular division of cn v, via the otic ganglion. While the nerve passes through
the ganglion, it does not synapse there.
Reference: http://www.dizziness-and-
balance.com/disorders/hearing/tinnitus/tensor%20tympani%20and%20stapedius%20myoclonus%20tinnitus.html

41. Patient came with green discharge nasal, congestion, 3 time took ab no benefit >> what is new tx?
A. Abx
B. Histamine
C. Steroid
Pharmacologic management: patients are often successfully treated with oral antihistamines, decongestants, or both; regular use of
an intranasal steroid spray may be more appropriate for patients with chronic symptoms.
Reference: http://emedicine.medscape.com/article/134825-overview

42. Allergic rhinitis symptoms caused by chemicals produced from?


A. Mast cells
B. Nk cells
C. Cd4
Answer: a
The mediators that are immediately released include histamine, tryptase, chymase, kinins, and heparin. The mast cells quickly
synthesize other mediators, including leukotrienes and prostaglandin d2.
Reference: http://emedicine.medscape.com/article/134825-overview#a5

43. Neck infection can spread to the mediastinum through:


A. Carotid artery
B. Para-pharyngeal space
C. Retropharyngeal space
Answer: c

44. Male patient presented with history of unilateral nasal obstruction and epistaxis. Ct scan done and shows soft tissue filling
the whole nose in that side. What is the cause of these symptoms?
A. Nasal polyps
B. Angiofibroma
C. Maligenant tumour
Answer: b
Reference: http://emedicine.medscape.com/article/872580-overview#a9
45. Patient with injury to accessory nerve which triangle injured?
A. Sub mental
110
B. Sub mandibular
C. Posterior
Answer: c

46. Swelling between ear and base of jaw with inflammation, what is the complication of it?
A. Facial n paralysis
B. Hearing loss
C. Encephalitis
Answer: c
CNS involvement is the most common extra salivary complication of mumps. Http://reference.medscape.com/article/966678-
overview#showall

47. Case of auricular hematoma (4 cm) came to ER. What's your initial management?
A. Give oral antibiotic and discharge with clinic follow up.
B. Incision and drainage and give oral antibiotic
C. Admit the patient and start iv antibiotic
Answer: b
-uptodate:
<2 cm = aspiration
> or 2 = i&d
Incision and drainage: a small (5 mm) incision is often all that is necessary. Gently separate the skin and perichondrium from the
hematoma and cartilage and completely express or suction out the hematoma. Be careful not to damage the perichondrium. If a
drain is placed, the patient should always be given antibiotics upon discharge. Http://emedicine.medscape.com/article/82793-
overview#showall

48. Old patient with epistaxis when he went from outside to home in winter “cold to warm place" what you going to give him:
A. Nasal steroid
B. Send home
C. Ephedrine nasal spray "not sure about this option"
Answer:
Hot and dry environments. The effects of such environments can be mitigated by using humidifiers, better thermostatic control,
saline spray, and antibiotic ointment on the kiesselbach area.
http://emedicine.medscape.com/article/863220-overview#a5

49. Case scenario dizziness vertigo progressive hearing loss what to see in ct?
A. Hemorrhage
B. Stroke
111
C. Normal CT
Answer: c
Ménière’s disease describes a set of episodic symptoms including vertigo (attacks of a spinning sensation), hearing loss, tinnitus (a
roaring, buzzing, or ringing sound in the ear), and a sensation of fullness in the affected ear.

50. pt with unilateral hearing loss , tinnitus, ..how to confirm the diagnosis :
A. tympanometry
B. tympanogram
C. MRI of cerbellopintine angel
Answer: C
51. adolescent with epistaxis for 10 min with no Hx of trauma, on examination: there was posterior oozing, Rx?
A. Conservative
B. posterior tampon
C. constrictive spray
Answer: B

52. Patient with recurrent episodes of vertigo, she feels of ear fullness , snhl and nausea , what is the diagnosis ?
A. Menere's diseases
B. Benign progressive vertigo
Answer: a
Diagnostic criteria for menere’s disease: 2 spontaneous episodes of rotational vertigo ≥20 min, audiometric confirmation of snhl and
tinnitus and/or aural fullness. (must have all three). Source: toronto notes.
http://www.nidcd.nih.gov/health/balance/pages/meniere.aspx

Meniere's disease is a disorder of the inner ear that causes episodes in which you feel as if you're spinning (vertigo), and you have
fluctuating hearing loss with a progressive, ultimately permanent loss of hearing, ringing in the ear (tinnitus), and sometimes a
feeling of fullness or pressure in your ear. In most cases, meniere's disease affects only one ear.
http://www.mayoclinic.org/diseases-conditions/menieres-disease/basics/definition/con-20028251

53. 40 years old male, complaining of sudden dizziness attack for 30-40 min, associated with nausea, vomiting, unilateral tinnitus
and hearing loss.
A. Acoustic neuroma
B. Neuritis
C. Meniere disease

Answer: c
Explanation: meniere’s disease: a cause of recurrent vertigo with auditory symptoms more common among females.
Hx/pe: presents with recurrent episodes of severe vertigo, hearing loss, tinnitus, or ear fullness, often lasting hours to days. Nausea
and vomiting are typical. Patients progressively lose low-frequency hearing over years and may become deaf on the affected side.
Reference: 3rd edition uqu > ent > q 104.

112
54. Which of the following medication causes vertigo?
A. Ethambutol
B. Streptomycin

Answer: b
Source: mercks manual
http://www.drugs.com/sfx/streptomycin-side-effects.html

55. Rhinorrhea, cough and conjunctivitis etiology ?


A. Rhinovirus
B. Adenovirus
Answer: adenovirus (conjunctivitis gave it away)

56. Patient came to ER with inability to breath from one nostrils and on examination the nostril is edematous and swollen what's
the initial management?
A. Local decongestant
B. Steroid
Answer: b

57. Year old man with chronic unilateral nasal obstruction, imaging shows flushy mas1
A. Polyp
B. Chronic forging body effect
Answer: polyp

58. Child complaining of painless unilateral hearing loss. On examination, tympanic membrane was opaque (erythematous).
A. Serous otitis media
B. Acute otitis media

Answer: a or b?
Explanation: answer: a there is no fever or pain, so most likely not infection. And this usually goes with otitis media with effusion.
Answer : b. Serous otitis media is a form of chronic otitis media ( according to toronto notes ) in which the tympanic membrane is
perforated while erythematous tympanic membrane is a diagnostic factor for aom.
Reference: http://emedicine.medscape.com/article/858990-overview
Aafm

History Physical findings Suggested cause of chl

Sudden painless Cerumen Complete canal occlusion

Sudden painful Narrow canal with debris Oe

Normal canal with red, immobile tm Com

Gradual painless Normal tm Otosclerosis

Reddish- blue pulsatile mass behind intact tm Glomus tumor

Perforated tm Cholesteatoma

113
59. Child with croup what is the best initial investigation to diagnosis5.
A. Chest x-ray
B. Pharyngeal swab
Answer:
Chest x ray ….(diagnosis of croup is made clinically. Cxr is needed in atypical presentation which will show “steeple sign “ from
subglotic narrowing ). Source: toronto notes.
60. Man working in factory, loud noises, his father and grandfather worked there. Hearing loss both of them. What to tell him?
A. Use cover protection for ear
B. Familial hearing loss

Answer: A
This is most likely a case of noise induced hearing loss.
http://emedicine.medscape.com/article/857813-overview

61. Indications of tonsillectomy? " recurrence not included in the choices "
A. Sleep apnea syndrome
B. Obesity

Answer: a
http://reference.medscape.com/article/872119-overview#a10

62. This a case of otitis media with effusion child with middle ear fluid and bulging in ear drum the rx?
A- Tympanostomy
B- Grommet insertion
Answer: • surgery: myringotomy ± ventilation tubes ± adenoidectomy (if enlarged or on insertion of second set of tubes after first
set falls out)
• ventilation tubes to equalize pressure and drain ear
Reference: toronto note

63. Case of serous otitis media with effusion , how to relieve symptoms (similar to the above q)?
A. Myringotomy

114
B. Grommet insertion
Answer: a
Reference: toronto note
In illustrated textbook of pediatric, it is mentioned that if conservative measures didn’t work (which is expectant management for
three months) insertion of ventilation tubes is indicated. If the ventilation tubes failed then insertion of ventilation tubes with
adjuvant adenoidectomy is indicated.
In mercks manual, myringotomy with tympanostomy tube insertion is indicated if no improvement occurs within 1-3 months.
Typmanostomy tubes are suggested at 6-12 months of continued bilateral ome or 4 months with bilateral hearing loss.

64. Case of otitis media what is complication:


A. Encephalitis
B. Hearing loss
answer: b
Extracranial: hearing loss, speech delay, ™ perforation, extension of suppurative process to adjacent structures ( mastoiditis,
petrositis, labyrinthitis ), cholesteatoma, facial nerve palsy, ossicular necrosis, vestibular dysfunction.
Intracranial: meningitis, epidural and brain abscess, subdural empyema, lateral and cavernous sinus thrombosis, carotid artery
thrombosis
Others: sigmoid sinus thrombophlebitis.
Source: toronto notes.

65. Runny nose and unilateral nasal obstruction for more than one year, on exam the inferior turbinates were swollen. What’s
the most likely diagnosis ?
A. Allergic rhinitis
B. Sinusitis
Answer : a?
Turbinates swollen : allergic rhinitis

66. 40 years old man presents with decreased hearing but his own sound feels louder than before to him. What is diagnosis?
A. Presbycusis
B. Otitis media
Answer: b, but if there is osteosclerosis it will be more appropriate.
Explanation: the presentation is most likely conductive hearing loss, which is can be caused by otitis media but not presbcusis.
67. Scenario about adult male complaining of severe ear pain with discharge inflamed tympanic membrane ( missed some
points) what is the diagnosis ?
A. Otitis externa
B. Cholesteatoma
Answer: b
The majority (98%) of patients with cholesteatoma have ear discharge or hearing loss or both in the affected ear.
Both can present with these symptoms, however, more accurate diagnosis can be made based on the smell of discharge and more
information on the appearance of the auditory canal and the tympanic membrane.
If the discharge is smelly and attic crust is visualized in retraction pockets or white mass behind intact tympanic membrane this is
likely to be cholesteatoma.
https://en.wikipedia.org/wiki/cholesteatoma
O.e is not associated with tympanic membrane infection

115
68. A patient has fronciosis in the left side of the nose, then he developed orbital edema. Which vein will carry this effect ?
A. Maxillary
B. Ophthalmic
Answer: ophthalmic vein is the most accurate answer, if it not one of choices then “ethmoidal “, if both are not present choose:
ophthalmic a.
Ophthalmic vein. Source: lippincott's illustrated q&a review of anatomy and embryology.

69. Child c/o unilatral nasal foul smelling discharge for two weeks ,what is your treatment?
A. X-ray of the head
B. Antibiotics
Answer: choices incomplete and answer probably missing. If not, answer is a
Explanation: based on symptoms child has foreign body, and between the two choices a is more suitable, even though imaging is not
usually done according to medscape and only occasionally done according to lecture notes.
References: http://emedicine.medscape.com/article/763767-overview#a5
- Lecture notes: diseases of the ear, nose and throat, 11 edition – page 191

70. scenario of patient with mastoiditis what is the proper antibiotic:


A. Amoxicillin
B. Azythro
Answer:?
Vancomycine and ceftricaxione are recommended until the culture become available.
Reference: http://emedicine.medscape.com/article/2056657-treatment

71. Case about deafness and paralysis of tympanic muscle and other muscle i can not recall it
A. Trigeminal
B. Facial
The q not clear mean the tensor tympani is a muscle within the ear,innervation of the tensor tympani is from the tensor tympani
nerve, a branch of the mandibular division of the trigeminal nerve
https://en.wikipedia.org/wiki/tensor_tympani_muscle
If the q mean tympanic membrane so, the nerve is facial n.

72. Patient after tooth extraction he feel numbness in his left ant tongue, what nerve is sensory supply to this area :
A. lingual ( right answer)
B. Infralvoular
the lingual nerve is a branch of the mandibular division of the trigeminal nerve (cn v3), which supplies sensory innervation to
the tongue. It also carries fibers from the facial nerve, which return taste information from the anterior two thirds of the tongue, via
the chorda tympani.

73. Child with signs and symptoms of acute otitis media what treatment will you give?
A. All answers were antibiotics
B. Amoxicillin
Answer: b.
The recommendations support the use of amoxicillin as the first-line antimicrobial agent of choice in patients with aom.
Toronto notes 2015

74. Patient with history of pinna pain with discharge from the ear, best mx?
A. Oral feuroxim
B. Topical neomycin
Answer: b
Sources: http://emedicine.medscape.com/article/994550-overview

116
75. Child has an URTI then has ear pain, Rinne test is negative but Weber test show the sound is louder in the affected area
A. otitis media
B. Mastoiditis
Answer: a

76. Patient with submandibular mass that increase in size with eating. Diagnosis?
A. Acute sialadenitis
B. Canal calculi
Answer: b
Acute sialadenitis is an acute inflammation of a salivary gland. Patients typically present with erythema over the area, pain,
tenderness upon palpation, and swelling. Frank cellulitis and induration of adjacent soft tissues may be present. Purulent discharge
upon milking gland. Http://emedicine.medscape.com/article/882358-clinical#showall canal calculi (sialolithiasis): more than 80%
occur in the submandibular gland or its duct, 6% in the parotid gland and 2% in the sublingual gland or minor salivary glands.
Sialolithiasis typically causes pain and swelling of the involved salivary gland by obstructing the food related surge of salivary
secretion. Http://www.nature.com/bdj/journal/v193/n2/full/4801491a.html

salivary gland problems that cause clinical symptoms include:


Obstruction: obstruction to the flow of saliva most commonly occurs in the parotid and submandibular glands, usually because
stones have formed. Symptoms typically occur when eating. Saliva production starts to flow, but cannot exit the ductal system,
leading to swelling of the involved gland and significant pain, sometimes with an infection. Unless stones totally obstruct saliva flow,
the major glands will swell during eating and then gradually subside after eating, only to enlarge again at the next meal. Infection
can develop in the pool of blocked saliva, leading to more severe pain and swelling in the glands. If untreated for a long time, the
glands may become abscessed.
Refrence: http://www.entnet.org/content/salivary-glands

77. What is the most common site of malignancy in paranasal sinus?


A. Maxillary
117
Answer: a
Reference: 3rd edition uqu > ent > q 92 + medscape.
Reference: http://emedicine.medscape.com/article/847189-overview#showall

Comment about the question: the presentation is most likely related to otitis media with effusion, however, the name serous otitis
media is given to a condition where there is serous drainage from the ear.

78. Post tonsillectomy patient has loss of taste of the posterior 1/3 of tongue. What is the nerve injured?
A. Glossopharyngeal nerve injury
Answer: a
Reference: toronto notes 2015

79. In presbycusis (snh loss). Which structure in the inner ear gets affected?
A. Hair cells.
Answer: a
Presbycusis casuses hair cell degeneration.
Reference: toronto notes 2015

80. Old guy can’t tolerate loud noises which nerve is affected?
Answer: facial nerve (due to loss of protective muscle (stapedius))
Reference: toronto notes 2015

81. A patient with chronic tonsillitis complaining of fever, chills, left throat pain and uvula deviation. What is the diagnosis?
Answer: peritonsillar abscess
Reference: toronto notes 2015

82. "otitis media case + child with acute ear pain , redness and bulging of tympanic membrane, whats the diagnosis ?
A. Acute otitis media
Answer: a
Explanation: clinical features:
• triad of otalgia, fever (especially in younger children), and conductive hearing loss
• rarely tinnitus, vertigo, and/or facial nerve paralysis
• otorrhea if tympanic membrane perforated
• infants/toddlers ƒ ear-tugging (this alone is not a good indicator of pathology) ƒ hearing loss, balance disturbances (rare) ƒ
irritable, poor sleeping ƒ vomiting and diarrhea ƒ anorexia
• otoscopy of tm ƒ hyperemia ƒ bulging, pus may be seen behind tm ƒ loss of landmarks: handle and long process of malleus not
visible
Reference: toronto note 2015

83. The most common cause of otitis media in all age group ?
Answer: streptococcus pneumonia, haemophilus influenzae, moraxella catarrhalis.
Reference: aafm

84. Best treatment for allergic rhinitis


Answer: ?
The management of allergic rhinitis consists of 3 major categories of treatment, (1) environmental control measures and allergen
avoidance, (2) pharmacological management, and (3) immunotherapy.

118
Glucocorticoid nasal sprays are presently the most effective single-agent maintenance therapy for allergic rhinitis
Answer of one who got 100 % in ent
Reference: http://www.uptodate.com/contents/pharmacotherapy-of-allergic-rhinitis#h25107621

85. Nasal obstruction, rhinorrhea, pale swelling, sneezing and tearing eyes, diagnosis ?
Answer: allergic rhinitis
Explanation:
Clinical features
• nasal: obstruction with pruritus, sneezing
• clear rhinorrhea (containing increased eosinophils)
• itching of eyes with tearing
• frontal headache and pressure
• mucosa: swollen, pale, “boggy”
Reference: Toronto notes 2015

86. Case of boy recently bought a cat developed allergy ?


Answer: allergic rhinitis?

87. What causes mixed hearing loss?


Answer: causes of mixed hearing loss vary wildly. Typically, the sensorineural hearing loss is already present and the conductive
hearing loss develops later and for an unrelated reason. Very rarely, a conductive hearing loss can cause a sensorineural hearing
loss. Some diseases that might present with mixed type hearing loss: chronic otitis media, otosclerosis, ear trauma, barotrauma and
head trauma.
http://www.coastalhearingcenters.com/mixed-hearing-loss/

88. Patient complaining of euphonia & doing laryngoscope it was normal & ask him to coughing he is cough what diagnosis?
A. functional euphonia
Answer: a
89. Intracranial growth compressing the maxillary artery. Which one of the following is affected?
Answer: ?
Explanation: the maxillary artery can be defined as one of the continuations of the external carotid artery, and distributes the blood
flow to the upper (maxilla) and lower (mandible) jaw bones, deep facial areas, cerebral dura mater and the nasal cavity. Hence it is
considered a blood vessel which supports both hard and soft tissues in the maxillofacial region.
Nb know the anatomy and branches.

90. 60-year-old male with unilateral parotid swelling tender but no facial nerve dysfunction?
Answer: ?
Explanation: if there is fever along with the tenderness the answer would be bacterial parotitis , if no systematic sx usually there will
be no tenderness where the answer would be benign pleomorphic adenoma , facial nerve involvement would rise the suspicion
toward malignancy .
Reference: medscape

91. Sore throat, palpable neck mass, hoarseness voice, hearing loss ask about tx?
A. Surgery and radiotherapy
(i thought about nasopharyngeal ca) from toronto note:

119
92. Most common cause of deafness in children is
Recurrent otitis media is the most common conductive cause of hearing loss
Http://www.asha.org/public/hearing/causes-of-hearing-loss-in-children/

93. Bleeding in posterior inferior area of nasal septum. Which vessel affected?
Answer: sphenopalatine artery
Supply to that area: external carotid > internal maxillary > sphenopalatine artery > nasopalatine.
Source: toronto notes.

94. Otitis media pt. Not taking antibiotic probably then after week came back by tenderness and swelling behind the ear,
diagnosis?
A. Acute mastoiditis
Answer: a

95. Thick white plaques in throat associated with gingivitis , diagnosis ?


Answer: could be fungal infection ? ????

96. Neck mass; pathology report: ki67 +ve what else you expect
A. Bcl2 over expression
Answer: Burkitt lymphoma
97. Boy with unilateral nasal obstruction, foul smelling diagnosis ?
A. Foreign body
Answer: a
98. Red erythematous epiglottis...causative organism
Answer: case of epiglottitis
H.influenzae tybe b

99. Hemorrhagic vesicles on tympanic membrane


Answer: bullous myringitis
http://www.msdmanuals.com/professional/ear,-nose,-and-throat-disorders/middle-ear-and-tympanic-membrane-
disorders/myringitis

120
100.Baby come with ear pain and discharge, in examination erythema and edema in the ear canal what is the diagnosis?
A. Otitis externa
Answer: a
Source: http://emedicine.medscape.com/article/994550-overview

101.Management of cholesteatoma?
http://emedicine.medscape.com/article/860080-overview

102.Chronic om with green discharge oxidase +ve organism ? (ent)


A. Pseudomonas
Answer is a. Pseudomonas which is an oxidase positive gram negative rod. (first aid step 1)
Questions about menires disease:
Ménière disease is a disorder of the inner ear that is also known as idiopathic endolymphatic hydrops. Endolymphatic hydrops refers
to a condition of increased hydraulic pressure within the inner ear endolymphatic system. Excess pressure accumulation in the
endolymph can cause a tetrad of symptoms: (1) fluctuating hearing loss, (2) occasional episodic vertigo (usually a spinning sensation,
sometimes violent), (3) tinnitus or ringing in the ears (usually low-tone roaring), and (4) aural fullness (eg, pressure, discomfort,
fullness sensation in the ears).
http://emedicine.medscape.com/article/1159069-overview

103..in case of external laryngeal nerve injury the ability to produce pitched sounds is then impaired along with easy voice
fatiguability

104.Loss of sensation on jaw line.


th
If asking about affected nerve, then it would be “mandibular branch” of the 5 cranial nerve, trigeminal nerve.

105.Long case ask u for languages assessment for 3 y child?

106.Old age with recurrent parotid enlarge with tender and erythema?
http://emedicine.medscape.com/article/882461-overview

107.The nerve that supply the largest part of tongue is ?


Answer: hypoglossal nerve

121
Supply the anterior 2/3 of the tongue
Sensory = anterior 2/3: lingual from trigeminal while taste by chora tympani from facial
Posterior 1/3: glossopharyngeal nerve (ix)
Motor innervation: - cn xii (hypoglossal) except palatoglossus muscle cn x (vagus)
http://medicalopedia.org/2695/nerve-supply-of-tongue/

108.Loss of sensation over maxilla and mandible, which nerve ?


Answer: trigeminal
The trigeminal nerve is the largest and most complex of the 12 cranial nerves (cns). It supplies sensations to the face, mucous
membranes, and other structures of the head. It is the motor nerve for the muscles of mastication and contains proprioceptive
fibers.
http://emedicine.medscape.com/article/1873373-overview

109.Overuse of vasoconistrictor nasal drops ?


Answer: rebound phenomena
Rhinitis medicamentosa (rm), also known as rebound rhinitis or chemical rhinitis, is a condition characterized by nasal congestion
without rhinorrhea or sneezing that is triggered by the use of topical vasoconstrictive medications for more than 4-6 days
http://emedicine.medscape.com/article/995056-overview

110.Patient with neck pain and headache in occipital area for months ,, there is a lomitation in movrmement ? What is

111.3rd branch of maxillary artery :


Spenopalatine ??
Answer: sphenopalatine artery is from third portion.

The maxillary artery, the larger of the two terminal branches of the external carotid artery, arises behind the neck of the mandible,
and is at first imbedded in the substance of the parotid gland; it passes forward between the ramus of the mandible and the
sphenomandibular ligament, and then runs, either superficial or deep to the lateral pterygoid muscle, to the pterygopalatine fossa.
It supplies the deep structures of the face, and may be divided into mandibular, pterygoid, and pterygopalatine portions.

122
112.Nerve responsible for the tonsils?
Answer: glossopharyngeal nerve (cn ix) & lesser palatine branch of sphenopalatine ganglion (branch of cn ix) explanation: nerve
supply to the tonsil is from the glossopharyngeal nerve.
nerve supply of tonsils are innervated via tonsillar branches of the maxillary nerve and the glossopharyngeal nerve
Link: http://www.drtbalu.co.in/tonsil.html
http://emedicine.medscape.com/article/1899367-overview#a2

113.Nerve responsible for the sinuses?


Answer: maxillary artery & its branches
Explanation:the maxillary sinus is supplied by branches of the internal maxillary artery, which include the infraorbital, alveolar,
greater palatine, and sphenopalatine arteries. It is innervated by branches of the second division of the trigeminal nerve, the
infraorbital nerve, and the greater palatine nerves
Link: http://emedicine.medscape.com/article/1899145-overview#a2

114.Case of serous otitis media with effusion , how to relieve symptoms ?


st
Confirmed aom :1 : oral or rectal analgesia plus antibiotic therapy. If unresponsive to antibiotics or with severe pain :
tympanocentesis .bmj http://bestpractice.bmj.com/best-practice/evidence/intervention/0301/0/sr-0301-i8.html

115.50 years old patient complaining of unilateral nasal obstruction associated with epistaxis, ct done showing ............, what's
the diagnosis:
A. Malignancy (my answer, i got 100% in ENT)

116.A child was diagnosed with otitis media. The isolated organism was streptococcus pyogenes. He is currently on amoxicillin,
what would you add if there is no enough response?
Answer: clavulanic acid
st
Amoxicillin is the first line therapy for om. If the 1 line therapy fails, which is defined as at least 3 days of treatment with high dose
amoxicillin, a second line treatment is to be administered. The recommendations suggested administering the following three
preparations: high-dose oral amoxicillin-clavulanate, oral cefuroxime and intramuscular ceftriaxone

117.32 years old female, presenting with decrease in hearing, her mother lost hearing in her 30s. Diagnosis ?
A. Acoustic neuroma

An acoustic neuroma, also called a vestibular schwannoma, is a benign slow-growing tumor affecting the vestibulo-cochlear nerve.
95% of acoustic neuroma (an) are unilateral. Initial symptoms include unilateral hearing loss, balance disturbances or vertigo,
tinnitus, and a feeling of fullness in the ear. Most unilateral vestibular schwannomas are not hereditary and occur sporadically.
Bilateral vestibular schwannomas affect both hearing nerves and are hereditary usually associated with a genetic disorder called
neurofibromatosis type 2 (nf2).

118.Patient came with hearing loss, her mother has history of the same, what is the diagnosis ?
Most likely inheritance pattern is autosomal dominant as only the mother is affected
Approximately 50% of all cases of congenital deafness are genetic. Approximately 70% of cases of hereditary deafness are
nonsyndromic, and the remaining 30% are syndromic, associated with specific deformities or medical problems. Of nonsyndromic
hearing losses, 75-85% are inherited in an autosomal recessive pattern, 15-20% are inherited in an autosomal dominant pattern, and
1-3% are inherited in an x-linked pattern.

Waardenburg syndrome

123
waardenburg syndrome is the most common cause of autosomal dominant syndromic hearing loss. The syndrome includes dystopia
canthorum, a broad nasal root, confluence of the medial eyebrows, heterochromia irides, a white forelock, and bilateral or unilateral
sensorineural hearing loss

Branchio-oto-renal syndrome
Branchio-oto-renal syndrome is the second most common cause of autosomal dominant syndromic hl. This condition manifests as
renal abnormalities, preauricular pits, deformed auricles, and lateral branchial cysts. The hearing loss may be conductive, snhl, or
mixed.
Neurofibromatosis type 2
Neurofibromatosis type 2 (nf2) is associated with vestibular schwannomas, meningiomas, ependymomas, juvenile cataracts, and
other intracranial and spinal tumors. The gene for nf2 has been mapped to chromosome 22q12.2 and is thought to be a tumor-
suppressor gene. It has about 50% penetrance. In the wishart type of nf2, the disease manifests in childhood or early adulthood.

119.Where we can palpate the facial nerve


Answer:
Link: http://teachmeanatomy.info/head/cranial-nerves/facial-nerve/
http://emedicine.medscape.com/article/835286-overview
120.Pt with epistaxis (they describe the site and asked about the artery affected) Vague description.

Kiesselbach's plexus, in Little's area, is a region in the anteroinferior part of the nasal septum where four arteries
anastomose to form a vascular plexus of that name. The arteries are
• Anterior ethmoidal artery (from the ophthalmic artery)
• Sphenopalatine artery (terminal branch of the maxillary artery)
• Greater palatine artery (from the maxillary artery)
• Septal branch of the superior labial artery (from the facial artery)
• Posterior ethmoidal artery
Reference: Wikipedia: https://en.wikipedia.org/wiki/Kiesselbach's_plexus

121.Submental pain, tender, palpable---- ? Submental calculi (sialolithiasis)

122.What is the most common malignant sinus tumor?


Answer: Maxillary
Approximately 60-70% of sinonasal malignancies (SNM) occur in the maxillary sinus and 20-30% occur in the nasal cavity itself. An
estimated 10-15% occur in the ethmoid air cells (sinuses), with the remaining minority of neoplasms found in the frontal and
sphenoid sinuses http://emedicine.medscape.com/article/847189-overview#a6

123.medication for resistant hiccups?


Answer:
Chlorpromazine is the most thoroughly studied and appears to be the drug of choice.
Reference: medscape http://emedicine.medscape.com/article/775746-medication
Similar question in another exam with different choices:

124.gradual hearing loss in a 17 yo male with normal tympanic membrne.


Answer if unilateral: acoustic neuroma, if with noise exposure and bilateral: noise induced
Ref: http://www.aafp.org/afp/2003/0915/p1125.html

124
Ophthalmology

1. Acute closed angle glaucoma what drug is contradicted?


A. Timolol
125
B. Pilocarpine
C. Acetazolamide
D. Cimetidine
Answer: D
Antihistamines. H2 receptor agonists (cimetidine), adrenergic agonists. Epinephrine, ipratropium bromide,
etc https://www.reviewofoptometry.com/ce/meds-that-dont-mix-with-glaucoma-patients
Timolol: a synthetic compound that acts as a beta blocker and is used to treat hypertension, migraines, and glaucoma.)
[2]
Pilocarpine: It is a non-selective muscarinic receptor agonist in the parasympathetic nervous system, which acts
[3]
therapeutically at the muscarinic acetylcholine receptor M3 due to its topical application, e.g., in glaucoma and xerostomia.
The answer is probably going to be an anticholinergic agent such as:
Anticholinergics include the following:
. Atropine, scopolamine
. Glycopyrrolate
. Benztropine, trihexyphenidyl
Precipitating factors include drugs (ie, sympathomimetics, anticholinergics, antidepressants [SSRIs], anticonvulsants, sulfonamides,
cocaine, botulinum toxin), dim light, and rapid correction of hyperglycemia.

2. 24 years old female newly diagnosed type 2 DM, she is wearing glasses for 10 years, how frequent she should follow with
ophthalmologist?
A - 6 months
B - 12 months
C - 5 years
D - 10 years
Answer: B
Evidence:
- For type 1 diabetic: retinal screening annually beginning 5 years after onset of diabetes, general not before onset of puberty.
- For type 2 diabetic: screening at the time of diagnosis then annually.
rd
Reference: 3 Edition UQU > Ophthalmology
3. Cavernous sinus aneurysm leading to diplopia & blurred vision, what nerve affected?
A - Trochlear
B - Abducens
C - Optic
D - Ophthalmic (V1)

Answer: B
Evidence: Cavernous sinus syndrome presents with variable ophthalmoplegia, decrease corneal sensation, Horner syndrome and
occasional decreased maxillary sensation. 2° to pituitary tumour mass effect, carotid-cavernous fistula, or cavernous sinus
thrombosis related to infection. CN VI is most susceptible to injury. Reference: FA USMLE step 1

4. What is the function of superior rectus muscle? (answer not known)


A - in down
B - out down
C - up in
D - up out
Answer: ? | Reference: Gray’s anatomy
o The superior rectus is an extraocular muscle that attaches to the top of the eye. It moves the eye upward.
American Association for Pediatric Ophthalmology and Strabismus
(Contraction of the superior rectus results in elevation, intorsion, and adduction of the eye)
Reference: http://www.yale.edu/cnerves/cn3/cn3_3.html
5. patient complaint of red eye and decreased vision in left eye since 3 days, in examination there are ciliary flush, not fully
reactive pupil and floaters in anterior chamber. What is the diagnosis:
126
A - Uveitis
B - Glaucoma
C - Retinitis
D - Keratitis

Answer: A | Reference: http://www.merckmanuals.com/professional/eye-disorders/uveitis-and-related-disorders/overview-of-


uveitis
6. Which of the following cause loss of colour vision?
A - acute angle closure glaucoma
B - mature cataract.
C - severe iridocyclitis.
D - optic atrophy.

Answer: D (Loss of colour of vision out of proportion to the loss of visual acuity is specific to optic nerve pathology)
Reference: http://www.uptodate.com/contents/optic-neuritis-pathophysiology-clinical-features-and-diagnosis

7. Patient had laser for refractive correction. One of post-op complications of this procedure is dryness of the eye. How to
prevent this dryness?
A - blockage of lacrimal duct
B - blockage of lacrimal canal
C - blockage of punctum
D - blockage of lacrimal sac

Answer: C | Reference: http://eyewiki.aao.org/Dry_Eyes_After_Laser_in_situ_Keratomileusis_(LASIK)#Therapeutic_Options

8. Patient wakes up with mucopurulent discharge and red eye. (other version: mucopurulent discharge, sticky eyes upon waking
up) What is dx?
A - viral.
B - bacterial (other version: bacterial conjunctivitis)
C - allergy
D - hypersensitive

Explanation: From the presentation, most likely Bacterial conjunctivitis.


Discharge: More purulent than in viral conjunctivitis, with more mattering (generally white or yellow mucous discharge [green in
some cases]) of the eyelid margins and greater associated difficulty prying the eyelids open following sleep; patients may report
waking up with their eyes “glued” shut.
Reference: http://emedicine.medscape.com/article/1191730-overview

Answer: B | Reference: http://www.medscape.com/viewarticle/522242


9. Which layer removes excessive water from cornea:
A - stroma
B - tear film
C - endothelium
D - epithelium

Answer: C | Reference: http://www.ncbi.nlm.nih.gov/pubmed/7122038

10. 20 y.o had trauma to left eye, he has corneal ulcer, photophobia, and pain, how do you manage?
A - debridement with Burr and systemic antibiotics.
127
B - cotton swab debridement and local steroid.
C - irrigation, systemic antibiotics and cycloplegia.
D - local antibiotics, cycloplegia and referral.

Answer: D | Reference: Toronto notes (Culture prior to treatment, & topical antibiotics every hour)
Evidence: Bacterial ulcer is most common cause in contact lens wearer. Acanthamoeba ulcer is due to swimming while wearing
contact lens. Fungal ulcer is due to trauma with vegetable material. Treatment for corneal ulcers, regardless of cause, begins with
Antibiotics. Source; merck’s manual.
References: Toronto note; https://www.merckmanuals.com/professional/eye-disorders/corneal-disorders/corneal-ulcer

11. patient with bilateral eye myopia each 2 month change his glasses (other version: Long scenario about patient with eye
problems, he has myopia, bilateral decreased vision, glasses were advised, after month he became not satisfied, there is
refractive error and astigmatism:):
A - Keratoconus
B - Keratomalacia
C - Keratoglobus
D - Keratectasia

Answer: A

12. A young male complaining of redness and discomfort in his eyes during the exam time, his visual acuity is 6/6 in both eyes
unaided. Both the anterior and posterior chambers are normal. What is the most likely diagnosis: (answer not sure of)
A - Myopia
B - Anisometropia
C - Astigmatism
D - Hyperopia

Answer: D?

13. patient wakes up from sleep with watery discharge from RT eye with redness. everything else is normal. What is the Tx?
A - topical steroid
B - topical Antihistamine
C - topical antibiotics
D - reassurance

Answer: D? (answer not sure of) [reassurance? (viral) or (eye dryness)]

14. Picture of eyelid swelled there is pain and increased tearing. what is the most likely sign? (answer not known)
A - decrease vision
B - discharge
C - uveitis
D - scleritis

Answer: ? eyelid swelling is not a specific sign for any of the choice
Reference: http://www.webmd.boots.com/eye-health/guide/swollen-eyes

15. Pic of eye ... mother got tear in her eye by her daughter fingernail. . Treatment is?
A. Antibiotics drops
B. Antibiotics ointment

128
C. Steroids drops
D. Steroids ointment
Answer: Antibiotic ointment.
Explanation: Corneal Abrasion antibiotic ointment. Ophthalmic corticosteroids tend to promote the growth of fungi and reactivation
of herpes simplex virus and are contraindicated.
Reference: http://www.merckmanuals.com/professional/injuries-poisoning/eye-trauma/corneal-abrasions-and-foreign-bodies

16. Treatment of orbital pseudo tumor?


A. Radiation
B. Antibiotics
C. Surgical excision
D. Systemic steroid
Answer: D (mainstay therapy of orbital pseudotumor is Systemic Steroid)
Explanation: Orbital inflammatory disease (pseudotumor) usually is treated medically with systemic steroids.
Reference: http://emedicine.medscape.com/article/1218892-treatment

17. Pt. Came complain of foreign body sensation in his eye after splash something on him.. after removing what should you give
him?
A) Antibiotics oral
B) Antibiotics topical
C) Steroids oral
D) Steroids topical
Answer: B
Reference: http://emedicine.medscape.com/article/82717-overview#a5

18. Patient presenting with red eye and ciliary flush, acute painful vision loss in his right eye, and headache.His right eye had
cloudy oedematous cornea, his left eye showed superior temporal rim cupping of the disc. Tonometer showed high IOP in the
right eye, and normal IOP in left eye. Which of the following is most likely explanation for his presentation?
a) bilateral glaucoma.
b) uveitis in right eye and congenital anomaly in left eye
c) macular degeneration in right eye.
d) retinal detachment in right eye.
Answer: A

19. Best distance between "Snell Chart" & the patient?


A) 3 Meters
B) 4 Meters
C) 6 Meters
D) 9 Meters
Answer: C
Reference: Toronto Notes 2015

20. you did examination on elderly Hypertensive man you find increase in cupping he is not complaining of anything what is the
cause of cupping?
A) Acute angle glaucoma
B) HTN
C) DM
D) Retinal detach.
Answer: incomplete
(Acute angle glaucoma painful; retinal detach curtain like vision loss or floaters)

129
21. Painful mass in medial side of eye to the nose, what is the management?
A. Oral antibiotic
B. Topical antibiotic
C. Topical corticosteroid
D. Surgical drainage

Answer: B
If it's Hordeolum (Stye) which is Painful infectious inflammation, the treatment is topical antibiotics (ex. Erythromycin), but if it's
chalazion which is Painless granulomatous inflammation, the treatment is incision and curettage or intrachalazion steroid

22. A picture of an eye with fluorescent, what is the Dx?


A. Bacterial keratitis
B. Fungal keratitis
C. Herpetic keratitis
D. Amoebic keratitis
Answer: C
Lecture notes Ophthalmology: The pathognomonic appearance is of a dendritic ulcer.

Figure 7.6 A dendritic ulcer seen in herpes simplex infection.

23. Patient complains of dry eyes, Doctor prescribed eye drops:


A. 2 drops in lower fornix
B. 1 in lower fornix
C. 2 in upper fornix
D. 1 in upper fornix
Answer: B
Instructions for patients using eye drops or ointments
Eye drops:
Wash hands before and after using drops.
Shake the bottle.
Instil in the lower conjunctival fornix and ideally keep the eye closed for 1-2 minutes after application.
Only one drop is needed per dose.
Minimise systemic absorption and adverse effects by closing eyes after administering eye drops, gently but firmly pressing the tear
duct against the nose for at least one minute, and then removing excess solution with absorbent tissue.
http://patient.info/doctor/eye-drugs-prescribing-and-administering
Only one drop of each drug is required
https://www.nursingtimes.net/download?ac=1289059
24. Hemiplegic patient comes with abnormality in the eye; may be nystagmus, where is the lesion in the brain?
A. Pons
130
B. Medullar oblongata
C. Internal capsule
D. Midbrain
Answer: B

25. Man got new glasses, after one week he cannot see well by the new glass , after examination change the glass and referred
to ophthalmologist.. What is the cause of change the old glass?
A. Glucose in lens
B. Cataract
C. Glaucoma
D. keratitis
Answer : A
From ophthalmic consultant

26. If you are examining the right eye by using with light reflex and there was no changing of pupil of the right eye but there was
changing in pupil of the left eye(consensual) , so where is the lesion ?
A. Rt optic nerve
B. Lt optic nerce
C. Rt occulomotor nerve
D. Lt occulomotor nerve
Answer: C
Afferent optic nerve when light shone on 1 eye both eyes are stimulated
Efferent oculomotor nerve constrict the ipsilateral pupil

http://www.fpnotebook.com/eye/exam/PplryLghtRflx.htm

27. pic of eye with fluorescence (it shows dendritic herpes ulcer) what to give :
A- antiviral
B- antifungal
C- topical antiviral
D- topical antifungal

Answer: C
Explanation: treatment of herpitic keratitis: topical antiviral such as trifluridine, consider systemic antiviral such as acyclovir.
Dendritic debridement. And NO STEROIDS initially – may exacerbate condition.
Reference: Toronto Notes 2015, page OP19, ophthalmology

28. patient with photophobia and tearing got injured by his brother finger nail what is the treatment:
A. steriod ointment
B. steriod drop
C. Antiviral
D. fitting contact lens

Answer: C
Corneal abrasion is probably the most common eye injury and perhaps one of the most neglected. It occurs because of a disruption
in the integrity of the corneal epithelium or because the corneal surface scraped away or denuded as a result of physical external
forces.
http://emedicine.medscape.com/article/1195402-medication#5
29. Pt. complaining of floaters, flashes and hx of DM and HTN what is the cause
A. DM
B. HTN
131
C. Retinal detachment
D. Cataract
Answer: c. retinal detachment. Symptoms supported by Hx of DM. Mayoclinic http://www.mayoclinic.org/diseases-
conditions/retinal-detachment/symptoms-causes/dxc-20197292

30. Type of Glaucoma you will do Trabeculectomy for?


A. ACAG,
B. COAG,
C. CCAG,
D. Congenital
Answer: B
The indications for trabeculectomy are as follows: Primary open-angle glaucoma, Primary angle-closure glaucoma not
responsive to iridotomy and/or iridoplasty, Secondary open-angle glaucoma, Secondary angle-closure glaucoma, Childhood
glaucomas. Medscape http://emedicine.medscape.com/article/1844332-overview

31. Boy scratched his mother's eye, inflammation developed, what to do;
A. Steroid ointment
B. Steroid drops
C. Antiviral
D. Contact eye
Corneal Abrasion: Topical ABx, Topical NSAIDS, Most cases resolve with no treatment.

32. Foreign body sensation after exposure to (something). He treated now what to give
A. oral antibiotics
B. oral steroid
C. local antibiotics
Answer:

33. Patient got a swelling under his eyebrow to which lymph node does it drain?
A. parotid
B. submandibular
C. sub mental
D. Cervical
Answer: A
Reference: http://teachmeanatomy.info/neck/vessels/lymphatics/

34. Patient with progressive decrease of vision, His vision acuity is about 8/20, (and he had something -8 but I did not remember
what is that) and virtuous atrophy. What is the type of myopia in this patient?
A. Physiological
B. Pathological
C. Curvature
D. Index
Could not find an answer

35. When examine Trochlear nerve we will said to patient to move his eye
A- Medial upward
B- Medial downward
C- Lateral upward
D- Lateral downward

Answer: D
132
The trochlear nerve is tested by examining the action of its muscle; the superior oblique. When acting on its own this
muscle depresses and abducts the eyeball.

Reference: Wikipedia: https://en.wikipedia.org/wiki/Trochlear_nerve#Examination

36. A known case COPD and DM, diagnosed to have primary open angle glaucoma. What is the optimal treatment for glaucoma?
A - Topical Timolol
B - Topical Carteolol
C - Systemic Acetazolamide
Answer: C

37. Pt came to ER with acut eye pain and headache , nausea and vomiting case of acute glaucoma?
A. IV acetazolamide + pilocarpi drop
B. Oral acetazolamide
C. Topical acetazolmaid and topical pilocarpine
Answer: A

38. patient with unilateral painful vision loss. What is the cause?
A - papilledema
B - toxic amblyopia
C - optic neuritis
th
Answer: C | Reference: Lecture note on ophthalmology, 9 edition
39. A patient with eye movement abnormalities and ptosis. What is the nerve is involved?
rd
A-3
th
B-4
th
C-6
Answer: A
Evidence:
rd
- Oculomotor (3 cranial nerve): Eye movement (SR, IR, MR, IO), pupillary constriction (sphincter pupillae: Edinger-Westphal
nucleus, muscarinic receptors), accommodation, eyelid opening (levator palpebrae). Loss of levator palpebrae function ptosis
th
- Trochlear (4 cranial nerve): Eye movement (SO)
th
- Abducens (6 cranial nerve): Eye movement (LR)
Reference: FA USMLE step 1

40. Patient came for annual check-up & found to have cupping of disk. What is the diagnosis?
A - Retinal detachment
B - Diabetic retinopathy
C - Chronic open angle glaucoma

Answer: C | Reference: http://www.glaucoma.org/treatment/optic-nerve-cupping.php

41. Adult complaining of pain when moving the eye, fundoscopy reveal optic disc swelling. What is the most likely diagnosis?
A - Optic neuritis.
B - Central vein occlusion.
C - Central artery occlusion.
Answer: A (Central vein and artery present with painless)
42. A patient presented with pterygium. What is the possible complication?
A - Corneal scarring
B - Corneal perforation

133
C - Strabismus

Answer: A | Reference: http://eyewiki.aao.org/Pterygium#Complications

43. a 47-year-old man with HTN & DM, past history of treatment of mycobacterial infection. He gave history of blurred and
decreased vision. On examination there is a flame shape on the retina, cotton wool spots and macular oedema (Other
investigations were included as well). What is the cause?
A - Retinal vein occlusion.
B - Ethambutol.
C - DM.

Answer: C
Evidence: Diabetic retinopathy is common in patients with chronic poor glycaemic control. Features include:
Microaneurysms (The earliest clinical sign of diabetic retinopathy), Dot and blot haemorrhages, Flame-shaped haemorrhages,
Retinal oedema and hard exudates, Cotton-wool spots, Venous loops and venous beading [Their occurrence is the most significant
predictor of progression to proliferative diabetic retinopathy (PDR)], & Macular oedema (Leading cause of visual impairment in
patients with diabetes) | Reference: http://emedicine.medscape.com/article/1225122-overview

44. Pic about periorbital swelling, what other symptoms could the patient have?
A - discharge
B - ↓ vision
C - uveitis

Answer: B (decrease vision if they mean (eye cellulitis) in this scenario)


Evidence: Symptoms and signs of orbital cellulitis include swelling and redness of the eyelid and surrounding soft tissues,
conjunctival hyperemia and chemosis, decreased ocular motility, pain with eye movements, decreased visual acuity, and proptosis
caused by orbital swelling
Reference: http://www.merckmanuals.com/professional/eye-disorders/orbital-diseases/preseptal-and-orbital-cellulitis

45. long case of eye trauma then repair then developed most likely endophthalmitis done enucleation, found in the uvea
lymphocyte and CD4 I think, what's is the cause? (Repeated)
A - cross reactivity
B - activation of lymphocytes
C - antigen release

Answer: C (antigen release)

46. Case of patient with progressive painless visual loss -8 6/18 is it:
A - Physiological
B - Pathological
C - Curvature Index alized area sta
Answer: B (pathological which could be keratoconus)
47. when to refer conjunctivitis to ophthalmologist:
A - Photophobia
B - Bilateral conjunctivitis
C - Mucopurulent discharge

Answer: A
Evidence: Ophthalmologist referral is indicated for any patient with conjunctivitis presented with one of the following Sx:

134
eye tenderness, Difficult seeing clearly, Difficult keeping the eyes open or sensitivity to light, Severe headache with nausea, Recent
trauma to the eye, Use of contact lenses. Reference: http://www.uptodate.com/contents/conjunctivitis-pinkeye-beyond-the
basics?source=outline_link&view=text&anchor=H8#H8

48. Patient had eye pain and photophobia upon examination by slit lamp there was circumcorneal congestion, keratic
precipitates and posterior corneal opacity. what is the diagnosis:
A - anterior uveitis
B - acute keratitis
C - acute angle glaucoma

Answer: A | Reference: http://emedicine.medscape.com/article/798323-clinical#b4

49. cup picture attached asking for the diagnosis: (no stem of Q, answer not known)
A - Glaucomatous
B - Papillitis
C - Diabetic retinopathy

Answer: see picture of each disease.

50. Child has red eye and tearing since birth:


A - Congenital glaucoma
B - herpes keratitis
C - bacterial conjunctivitis

Answer: A | Reference: Ophthalmology Consultant

51. Distichiasis?
A. Malrotation of eye lashes
B. Pigmentation of eyelashes
C. Extra row of eyelashes
Answer: C
Explanation: defined as the abnormal growth of lashes from the orifices of the meibomian glands on the posterior lamella of the
tarsal plate (see following image).

Reference: http://emedicine.medscape.com/article/1212908-overview

52. Patient with vesicles in forehead and supraorbital region for one day, what will you do?
A. Antiviral
B. Antiviral and refer to ophthalmologist.
C. Reassure
Answer: B
135
Explanation: The cause most likely is Herpes zoster ophthalmicus, it presents with dermatomal forehead rash and painful
inflammation of all the tissues of the anterior and, rarely, posterior structures of the eye.
Reference: http://www.merckmanuals.com/professional/eye-disorders/corneal-disorders/herpes-zoster-ophthalmicus

53. Best to diagnosis corneal abrasion?


a) Fluorescein eye test
b) Slit lamp test
c) Others
Answer: A

54. Lower visual field loss with flashes and other symptoms, what is the Dx?
A. HTN
B. DM
C. Retinal detachment
Answer: C

55. Patient came with feeling of foreign body sensation after removing fly from eye. What to give him?
A. Topical antibiotics
B. Oral antibiotics
C. Topical steroids

Answer: A
this sounds like foreign body induced corneal abrasion which should be treated by removing the foreign body then topical antibiotics
(erythromycin)
http://bestpractice.bmj.com/best-practice/monograph/500/treatment/details.html

56. Pic of eye with dendritic lesion Stained by fluorescein.. Management?


A) Antiviral
B) Antifungal
C) Others
Answer: A
Explanation: Most likely caused by Herpes Simplex virus, treat with Antiviral.
Reference: http://bestpractice.bmj.com/best-practice/monograph/561/treatment/details.html#expsec-13

57. child with eye itching for one month (no other symptoms in scenario), and have Hx of asthma, what's the Dx?
A. Bacterial conjunctivitis
B. Viral conjunctivitis
C. Venral conjunctivitis
Answer: C
Vernal conjunctivitis is long-term (chronic) swelling (inflammation) of the outer lining of the eyes. It is due to an allergic reaction.
often occurs in people with a strong family history of allergies. These may include allergic rhinitis, asthma, and eczema
https://www.nlm.nih.gov/medlineplus/ency/article/001390.htm

58. High myopia tx


A) leser ...-
B) refractory ...
C) lens - ...
Answer: phakic Intraocular lens?

136
59. Case of patient having irritation in the eyes with watery discharge. ..what usually associated with it
A) Enotropia
B) Ectropia
C) Strabismus
Answer: A

60. Patient around 50 y with unilateral eye discoloration they give discretion of something in the sclera I think! What you’ll tell
him! I DON” T KNOW
A) he’s in high risk of visual loss
B) this is a manifestation of systemic disease
C) vitamin A deficiency
Answer: Incomplete, I think it is bitot spot and is associated with vit. A deficiency

61. Increase ICP, with bilateral impaired abduction;


A. Oculomotor
B. Abducens
C. Trochlear

Answer: B
affects the lateral rectus muscle, impairing eye abduction. The eye may be slightly adducted when the patient looks straight ahead.
http://www.merckmanuals.com/professional/neurologic-disorders/neuro-ophthalmologic-and-cranial-nerve-disorders/sixth-cranial-
nerve-palsy

62. Child opaque lens with signs of inflammation?


A. Cataract
B. Neuroblastoma
C. Retinoblastoma
Answer: C

63. Patient with exophthalmos and swollen lids and you can feel its pulse, TFT normal. What’s your diagnosis?
A. Hyperthyroid
B. Cellulitis
C. Cavernous sinus thrombosis
Answer: C
source: The wills eye manual 6th edition

64. Post cataract surgery ciliary injection & fever, normal extra ocular movements;
A. Endophthalmitis
B. Panophthalmitis
C. Cellulitis

Answer: A
Endophthalmitis: after one-day vs Sterile postoperative inflammation: in same day
Cellulitis: decrees range of movement
http://emedicine.medscape.com/article/1201260-overview

65. Patient has a cat and then she C/O eye pain and erythema in the eye?
A. Allergic conjunctivitis
B. Contact dermatitis
137
C. Bacterial conjunctivitis
Answer: A

66. During examination of the eye of uncontrolled diabetic patient ,what you may find ?
A. Central Retinal artery ischemia (I think)*
B. Glaucoma
C. Papiledema
Answer: B
http://www.glaucoma.org/glaucoma/diabetes-and-your-eyesight.php
Central Retinal artery ischemia patients might have history of atrial fibrillation, endocarditis, coagulopathies, atherosclerotic
disease, hypercoagulable state

67. At day care center 10 out of 50 had red eye in the 1st week, another 30 develop same condition in the next 2 nd wk ,,,what's
the attack rate?!
A. 40%
B. 60%
C. 80%

Answer: C

68. Pic of congested eye, history of URTI one week ago , eye has watery discharge not purulent , what will support what is the
Diagnosis: (What is the most likely you will find ?)
a. Papilla
b. Follicles
c. Papilloma
Answer: B
Explanation: Follicles usually seen in viral and chlamydial infections; Symptoms make viral conjunctivitis most likely. Patients with
adenoviral conjunctivitis may give a history of recent exposure to an individual with red eye at home, school, or work, or they may
have a history of recent symptoms of an upper respiratory tract infection.
Reference: http://emedicine.medscape.com/article/1191370-clinical
Toronto Notes 2015, page OP15, ophthalmology

69. A passenger on a bus sustains an injury to his eye with the newspaper of a neighboring person. His vision becomes blurry and
he develops redness and pain in his eye.
A. Abrasion
B. Laceration
C. Infiltration
Answer: a
Corneal abrasion is the most common type of injury involving the cornea. Contact with dust, dirt, sand, wood shavings, metal
particles or even an edge of a piece of paper can scratch or cut the cornea. Because the cornea is extremely sensitive, abrasions can
be painful. In case of such an injury, prompt medical care should be obtained.
http://emedicine.medscape.com/article/1195402-overview
70. Retinoblastoma affect the vision MRI show intact optic N (something like that) Ttt?
A. Chemotherapy
B. Steroid
C. Removal of the eye

Answer: A

138
Chemoreduction — Since most retinoblastomas are large at the time of presentation, chemoreduction is often used to reduce
tumor volume, which enhances the success of local therapies. Chemoreduction has become a critical component of the
initial treatment of retinoblastoma and has improved the ocular salvage rate [69,106-111]. The most common
chemoreduction regimen contains carboplatin, vincristine, and etoposide given approximately every four weeks.
FYI: Enucleation (removal of the eye) usually is indicated for large tumors (>50 percent of globe volume) with no visual potential,
blind, painful eyes, and/or tumors that extend into the optic nerve
Refrence: http://cursoenarm.net/UPTODATE/contents/mobipreview.htm?28/14/28897

71. Esotropia 25 degree, initial management:


A. Orthoptic
B. Exercise prism
C. Medial muscle recession
Answer: C
In patients with acquired esotropia, surgery is indicated when the deviation is greater than 15 PD and stable.
http://emedicine.medscape.com/article/1198784-treatment#d6

72. Patient with tumor affect eye what is the cause ?


C. Ewing sarcoma
D. Neuroplastoma
E. Nephroblastoma
Answer:B

Metastasis is most commonly from breast and lung in adults, neuroblastoma in children. Usually infiltrate the choroid, but may also
affect the optic nerve or extraocular muscles. Patients present with decreased or distorted vision, irregularly shaped pupil, iritis,
hyphema.
Reference: Toronto notes

73. patient c/o red eye and tearing but there is no itching ,hx of trauma , what u will do ?
A- Give him topical AB
B- topical corticosteroid
C- reassure
Answer: A (C is also possible)
Explanation: Since there is history of trauma, along with redness and tearing, patient most likely has corneal abrasion. Treatment:
topical antibiotic, consider topical NSAID, cycloplegic, consider patch (if no contact lenses)
Most abrasions clear spontaneously within 24-48 h (so, answer C is possible)
Reference: Toronto Notes 2015, page OP17, ophthalmology

74. women came complain of eye pain case her daughter scratch her eye with her nail , there is picture of (i think slit lamp shows
abrasion on cornea), what to give?
A. drop steroid
B. ointment steroid
C. contact lens
answer: A, antibiotic, eye drops- NSAIDS
Corneal abrasions heal with time. Prophylactic topical antibiotics are given in patients with abrasions from contact lenses.
Traditionally, topical antibiotics were used for prophylaxis even in noninfected corneal abrasions not related to contact lenses, but
this practice has been called into question. Some ophthalmologists advocate the use of diclofenac (Voltaren) or ketorolac (Acular)
drops (NSAIDS).
Medscape: http://emedicine.medscape.com/article/1195402-overview

75. Picture with history of eye inflammation, no ophthalmoplegia :


139
A. Chalazion
B. Stye
C. Orbital cellulitis
Answer: A or B according to the picture and scenario

Answer: see pictures to match it with the description.


Explanation:
- chalazion is noninfectious obstruction of a meibomian gland causing extravasation of irritating lipid material in the eyelid soft
tissues with focal secondary granulomatous inflammation. Disorders that cause abnormally thick meibomian gland secretions
(eg, meibomian gland dysfunction, acne rosacea) increase the risk of meibomian gland obstruction.
- Hordeolum (stye) is an acute, localized swelling of the eyelid that may be external or internal and usually is a pyogenic (typically
staphylococcal) infection or abscess. Most hordeola are external and result from obstruction and infection of an eyelash follicle
and adjacent glands of Zeis or Moll glands. Follicle obstruction may be associated with blepharitis.

Reference: Merck’s manual. Pic from Google

76. Retinal vessels displaced, old age come with headache ?


A. Acute angle glaucoma
B. Diabetic retinopathy
C. Macular degeneration
(No open angle glaucoma in choices)
Answer: might be A, headache goes more with that!
140
77. The most presenting signs of retinoblastoma:
A. Leukocoria
B. Nystagmus
C. Strabismus
Answer: A
Reference: http://emedicine.medscape.com/article/1222849-clinical#b4
The most common presenting sign in Fundoscopy is white pupillary reflex (Leukocoria)

78. Elderly female, came with right eye pain, vomiting...


A. Hyperthyroid
B. Acute angle closure glaucoma
C. Hyphema
Answer: B
Acute angle closure glaucoma: Red, Painful eye, unilateral. Decreased visual acuity, blurred vision due to corneal edema. “Halos
around light”. N&V, abdominal pain. Fixed mid-dilated pupil. Management: laser iridotomy. – Toronto Notes Optha
Hyphema: http://emedicine.medscape.com/article/1190165-overview#showall

79. Pt. with controlled DM & HTN see flashes and loss part of visual field?
A. Retinal detachment
B. Diabetic retinopathy
C. Hypertensive retinopathy
Answer: A
Initial symptoms commonly include the sensation of a flashing light (photopsia) related to retinal traction and often accompanied by
a shower of floaters, shadow in the peripheral visual field, and vision loss.
http://emedicine.medscape.com/article/798501-clinical
80. SCD patient comes with sudden painless loss of vision cloudy retina and cherry red spot finding the macula. What is your
diagnosis?
A. Retinal artery occlusion
B. Retinal vein occlusion
C. Retinal detachment
Answer: A
Typical funduscopic findings of a pale retina with a cherry red macula (ie, the cherry red spot) result from obstruction of blood flow
to the retina from the retinal artery, causing pallor, and continued supply of blood to the choroid from the ciliary artery, resulting in
a bright red coloration at the thinnest part of the retina (ie, macula).
For more reading:
http://emedicine.medscape.com/article/799119-clinical - b5
- Coagulopathies from sickle cell anemia or antiphospholipid antibodies are common etiologies for central Retinal artery occlusion in
patients younger than 30 years

81. Eye surgery for high myopia in one eye:


A. LASIK
B. IOL
C. PRK
*Most likely answer: Phakic intraocular contact lenses
http://emedicine.medscape.com/article/1221828-overview#a9
http://emedicine.medscape.com/article/1221604-overview#a8
(In the above links you will find indications for both LASIK and PRK but not sure what the answer is.)
141
82. Iris neovascularization caused by? (missing options)
A. Non-proliferative diabetic retinopathy.
B. Central retinal vein occlusion

Answer: B
Evidence:
- The three most common causes are: DM, Central retinal vein occlusion and carotid artery obstructive disease.
Reference: http://www.aao.org/eyenet/article/diagnosis-treatment-of-neovascular-glaucoma?julyaugust-2006

- Abnormal iris blood vessels (neovascularization) may obstruct the angle (secondary glaucoma) and cause the iris to adhere to the
peripheral cornea, closing the angle (rubeosis iridis). This may accompany proliferative diabetic retinopathy or central retinal vein
occlusion due to the forward diffusion of vasoproliferative factors from the ischaemic retina.
th
Reference: Lecture note on ophthalmology, 9 edition

83. commonest cause of iris neovascularization is: (missing options)


A. DM
B. HTN

Answer: A

84. Corneal epithelial defects (CED), in addition to this presentation, what else this patient might have? (missing options)
A. Photophobia
B. Visual loss

Answer: A | Reference: http://eyewiki.aao.org/Corneal_Epithelial_Defect#Management

85. Long term topical steroid drops can cause: (missing options)
A. Glaucoma.
B. posteior subcapsular cataract

Answer: A | Reference: http://www.ncbi.nlm.nih.gov/pmc/articles/PMC4228634/table/T1/

86. Trabeculectomy is an operation used for which of the following conditions? (missing options)
A. Open angle glaucoma
B. Closed angle glaucoma

Answer: A
Evidence: Trabeculectomy is effective for chronic angle-closure glaucoma. However, compared to primary open-angle glaucoma, any
aqueous-draining procedure in an eye with a shallow anterior chamber and a chronic closed angle poses the risk of further
shallowing the anterior chamber or precipitating malignant glaucoma.
Reference: Toronto notes; http://emedicine.medscape.com/article/1205154-treatment#showall

142
87. A patient with painful swelling (dacryocystitis). What is the best management? (missing options, answer not known)
A. Oral antibiotic
B. Drain surgically

Answer: ?
Evidence: In general, dacryocystitis is a surgical disease. Surgical success rates in the treatment of dacryocystitis are approximately
95%. Acute cases are best treated surgically after the infection has subsided with adequate antibiotic therapy.
For acute dacryocystitis, an external dacryocystorhinostomy is preferred after several days of initiating antibiotic therapy.
Rarely, dacryocystorhinostomy must be performed during the acute phase of the infection to facilitate clearing of the infection.
Reference: Medscape.

88. Father came with his 6 years old daughter to the clinic, she has esotropia. What will you do? (missing options, no answer)
A. Glasses.
B. Surgery.
Answer: ? [Nonsurgical treatments include patching, correction of full hyperopic refractive error (glasses) then if not improved
surgery] | Reference: http://eyewiki.aao.org/Accommodative_Esotropia#General_treatment

89. (pic) Eye lid inflammation what most associated symptoms: (missing options)
A. secretion
B. vision loss

Answer: A | Reference: http://emedicine.medscape.com/article/1211763-clinical


90. stage of hypertensive or increase IOP, ask about third stage optic change? (missing details of Q, missing options, no answer)
A. Papilloedema
B. nipping of vein

Answer: ?
Evidence:
Grade I: mild arteriolar narrowing
Grade II: arteriovenous crossing, AV nicking
Grade III: retinal haemorrhage, exudate, cotton wool
Grade IV: papilloedema
Reference: http://www.retinalphysician.com/articleviewer.aspx?articleID=109106

91. Bacterial keratitis occurs with contact lenses microorganism: (missing options)
A. S. Aurea (in general)
B. Pseudomonas Aeruginosa (contact lens wearer)

Answer: B | Reference: http://www.ncbi.nlm.nih.gov/pubmed/3942549/ ; http://www.aao.org/eye-health/diseases/what-is-


bacterial-keratitis

92. Painless loss of vision in right eye with headache. First thing to do? (missing options)
A. Prednisolone
B. CT

Answer: A (Prednisolone)

143
93. Type of gonorrhoea discharge in eye: (missing options)
A. Purulent
B. Mucopurulent if chlamydia

Answer: A
94. Conjunctivitis patient with bottom eyelash turned inwards? (missing options, answer not clear from options)
A. Ectropian
B. Entropian

Answer: ? (eyelashes turned inward: trichiasis; Lid margin turned inward: entropion)
Reference: http://emedicine.medscape.com/article/1212456-overview

95. A patient who presented with ciliary flush, bilateral eye redness and pain. Examination revealed keratic precipitate and
presence of cells in the anterior chambers (classical scenario of uveitis) what is the treatment:
A. systemic corticosteroid
B. cyclopentolate with topical steroids
Answer: B
Source: Toronto notes 2014 OP20

96. Patient struck her eyes by her finger and came with burning pain and crying, what is the suspected symptom or
complications:
A. Blindness
B. Photophobia

Answer: B (missing options)


Evidence:
- symptoms associated with corneal abrasion are: pain, blurred vision, tearing and photophobia.
- complication include: bacterial keratitis, corneal ulcers, traumatic iritis, and recurrent erosion syndrome.

97. How you test trochlear nerve :


A. Adduction and downward
B. Abduction and downward

Answer: A
http://teachmeanatomy.info/head/cranial-nerves/trochlear-nerve/

98. Patient presented with dilated pupils, nausea vomiting and abdominal pain. What is the most likely diagnosis?
A. digoxin toxicity
B. angle glaucoma

Answer: B
classic presentation of Acute angle closure glaucoma.
http://emedicine.medscape.com/article/798811-clinical

99. An old man presents after a crash with blurred vision, on o/c: anterior chamber clouded or something ...what causes these
symptoms?
A. Retinal detachment
B. Cataract.
Answer: B
144
100.Case of acute glaucoma, what is the treatment?
A. Acetazolamide
B. Timolol
Answer: A
Explanation: Carbonic anhydrase inhibitors decrease aqueous humour formation and are used commonly as first-line therapy
(generally managed based on the cause and initially will be both topical and systemic treatment.)
Reference: http://bestpractice.bmj.com/best-practice/monograph/372/treatment.html

101.He was playing tennis and he get a force on his eye .. There was a bleeding in anterior chamber of the eye .. Which of the
following you must exclude first?'
A) Presence of Foreign body
B) sorry I forget the other answer
Answer: increase in intraocular pressure
Reference: http://www.healthline.com/health/hyphema#Overview1

102.How to prevent eye infection?


A. eye patches
B. hand hygiene
Answer: B
Generally avoiding close contact with infected individual and their eye secretions, as they are highly contagious in case of
bacterial and viral conjunctivitis.
Infected individual are advised of frequent hand washing, avoiding sharing towels and linens, avoiding public pools.
Workers and students often are excused from work or school during the first several days of treatment to decrease the
possibility of spread.
Source: http://emedicine.medscape.com/article/1191730-treatment#d11

103.lateral movement of one eye of a child when you close the other eye?
A. squint (strabismus)
B. nystagmus
Answer: A.
Strabismus is misalignment of the eyes, which causes deviation from the parallelism of normal gaze.
By the direction of the squinting (turning) eye:
An eye that turns inwards is called an esotropia.
An eye that turns outwards is called an exotropia.
An eye that turns upwards is called a hypertropia.
An eye that turns downwards is called a hypotropia.
104.patient presented with foreign body sensation in his eyes, itchiness and grittiness. (not sure if mentioned watery discharge or
no). he is taking (a drug) and antihistamine. Also he mentioned that he sit in front the screen for six hours daily. What is the
cause of his disease? ( the scenario was not very clear whither it is allergic or just simple dryness)
A. mast cell degranulation and histamine release.
B. Corneal dryness
Answer: b
http://emedicine.medscape.com/article/1210417-overview

105.sudden onset of proptosis limited eye movement pain swelling vision is normal
A. orbital cellulitis,
B. cavernous sinus thrombosis
Answer: A
145
Toronto note 2015

106.pic of eye (there is proptosis and redness of upper eyelid on one eye) what is diagnosis;
A. Orbital cellulitis
B. Chalazion
Answer: A
The question has missing details like the presence of fever, pain on movement but in general Proptosis and ophthalmoplegia are the
cardinal signs of orbital cellulitis.
Reference: http://emedicine.medscape.com/article/1217858-clinical#showall

th
Reference: Lecture note on ophthalmology, 9 edition

107.Eye picture red conjunctiva with white ring around cornea


A. Vernal conjunctivitis
B. Chlamydia
answer: A
VKC is characterized by symptoms consisting of severe itching, photophobia, foreign body sensation, mucous discharge (often
described as “ropy”), blepharospasm, and blurring of vision
Classic signs of allergic conjunctivitis include injection of the conjunctival vessels as well as varying degrees of chemosis (conjunctival
edema) and eyelid edema. The conjunctiva often has a milky appearance due to obscuration of superficial blood vessels by edema
within the substantia propria of the conjunctiva.
http://emedicine.medscape.com/article/1191467-overview

146
108.Boy has a cat developed itching for a month with Red eye and watery with discharge No lymphadenopathy and general exam
normal:
A. Cat scratch Dermtitis Allergic
B. Conjunctivitis
Answer: B
reference: http://emedicine.medscape.com/article/1191467-clinical

109.what is the diagnosis of DM patient with history of sudden unilateral vision loss pupil afferent affected -retinal hemorrhages
and macular edema?
A. retinal artery occlusion -
B. retinal vein occlusion
answer:b
reference : https://quizlet.com/20317777/ophthalmology-flash-cards/

110.patient had red eyes, tearing, change in pupil shape didn't improve with patch:
A. keratitis
B. glaucoma
Answer:?

111.HIV patient for eye exam. Upon examination he’s shown to have cotton wool spot appearance. (Other details of eye exam
are missing) Which virus may cause this?
A. HIV
B. CMV
Answer: B
CMV retinitis is the most common intra-ocular infection in HIV-infected patients.
Source: http://www.icoph.org/med/ppt/hiv.pdf http://emedicine.medscape.com/article/1227228-overview

112.Patient with Post cataract surgery he came with Painful eye , yellow, red reflux , muscle intact what is 1DDx?
A. Enophalamitis
B. Cellulitis
Bacterial endophthalmitis (see the image below) is an inflammatory reaction of the intraocular fluids or tissues caused by microbial
organisms. On physical examination, general findings in bacteria endophthalmitis are as follows: Hypopyon, Loss of red reflex …etc
http://emedicine.medscape.com/article/1201134-overview

113.56 male patient came with Pterygium , what you will tell him ?
A. Pre malignant lesion
B. Affect his vision
Answer: b
fibro vascular triangular encroachment of epithelial tissue onto the cornea, usually nasally
may induce astigmatism, decrease vision / torento note 2015
http://emedicine.medscape.com/article/1192527-overview

147
114.severe acute onset eye pain fixed pupil hazy cornea , redness o think , what u will give :
A. IV Acetazolamide
B. Topical Pilocarpine
Acute Angle-Closure Glaucoma 
Look for the sudden onset of an extremely painful, red eye that is hard to palpation. Walking into
a dark room can precipitate pain because of pupilary dilation. The cornea is described as “steamy” and the pupil does not react to
light because it is stuck. The cup-to-disc ratio is greater than the normal 0.3. The diagnosis is confirmed with tonometry. Treat with:
· Intravenous acetazolamide, Intravenous mannitol , Pilocarpine, beta blockers, and apraclonidine to constrict the pupil and

enhance drainage , Laser iridotomy (Master the boards)

115.Case with red eyes with watery discharge, there is history of recurrent of similar attack, which can aggravate this condition:
A. Pollens exposure
B. Ultraviolet exposure
Answer: A, allergic conjunctivitis

116.Headache, pain in Rt eye ,Halos around light :


A. Hyphema
B. digoxin toxicity
Answer: Glaucoma or Cataracts - Not mentioned in the options/Scenario missing details
Explanation: Halos are troublesome, bright circles of light that surround headlights and other light sources.Halos Around Lights may
be associated with:
● Cataract
● Fuchs' Dystrophy
● Glaucoma
● Kerataconus
Link: http://www.aao.org/eye-health/symptoms/halos-around-lights
http://www.webmd.com/eye-health/halos-and-glare-causes-prevention-treatment

117.Which of the following can cause painful loss of vision?


A. Retinal detachment
B. Acute angle glaucoma
Answer: B
Acute angle glaucoma presents with a red painful eye which can progress to irreversible loss of vision within hours if left untreated.
Retinal detachment presents as sudden onset flashes of lights and floaters followed by peripheral field loss (curtain of blackness)
and loss of central vision if macula was off.
Toronto notes 2015

148
118.Patients complains of unilateral eye pain, redness and photophobia. What is the most likely diagnosis?
A. Keratitis
B. Uveitis
Answer: B
Acute iritis Keratitis
Discharge No Profuse tearing
Pain ++ (tender globe) ++ (on blinking)
Photophobia +++ Varies
Blurred vision ++ ++
Pupil Smaller Same or smaller
Injection Ciliary flush Diffuse
Cornea Kertic precipitates Infiltrate, edema, epithelial defects
Intraocular pressure Varies Normal or increased
Anterior chamber +++ cells and flares Cells and flares or normal
Other Posterior synechiae
Toronto notes 2015

119.Neovascularization on the retina, caused by?


A. DM proliferative Retinopathy
B. HTN
Answer: A
Reference: http://emedicine.medscape.com/article/1225122-overview#a3

120.Diabetic controlled on medication developed (eye down and out) dropped upper eyelid limited movement of eye medially
and up and down.
A. Right oculomotor palsy
B. Right facial palsy
Answer: A
http://emedicine.medscape.com/article/1198462-overview

121.Picture of Snellen's test. What is the visual level of this patient?


Answer: ? (missing details of Q, missing options, answer not known)
e.g. A patient can read until the 3rd line, the visual acuity is 20/70.

122.Lacrimal gland tumour lead to proptosis in which direction? (missing options)

149
A - Down\in
Answer: A
Evidence:
- Maxillary sinus growth: Superior
- Lacrimal gland tumour: Inferomedial
- Frontal or ethmoidal sinus lesion: Inferolateral
Reference: Textbook of Ophthalmology

123.What is the treatment of chronic ptosis? (missing options, answer not known)
Answer: ? | Reference: http://eyewiki.aao.org/Aponeurotic_ptosis#General_treatment

124.Corneal epithelial defects (CED) (Another term of corneal abrasion or ulcer) due to fingernail injury. What is the treatment?
A - Double patch. (missing options)
Answer: A | Reference: http://eyewiki.aao.org/Corneal_Epithelial_Defect#Management & an intern who had the same Q in his
exam and got the full mark in ophthalmology section

125.How to administer eye drops and ointments? (missing options)


A - One drop in the lower fornix
Answer: A | References: http://patient.info/doctor/eye-drugs-prescribing-and-administering ;
https://www.nursingtimes.net/download?ac=1289059

126.A patient came with drooling and lacrimation (cholinergic reaction). What to give? (missing options, answer not known)
- Physostigmine
Answer: ? (No atropine in the choices; Physostigmine is NOT the right answer, it is an acetylcholinesterase inhibitor (It can be the
cause of the cholinergic reaction)

127.A patient with follicular keratosis. What will you check? (missing options)
A - Eyes
Answer: A (Because of Vit. A deficiency; Keratosis pilaris is another name of follicular keratosis)
Reference: http://disorders.eyes.arizona.edu/category/clinical-features/keratosis-pilaris

128.A patient with increased ICP. What will you check? (missing options)
A - Papilledema
Answer: A

129.Eye drops that is contraindicated in acute closed angle glaucoma? (missing options, answer not known)
Answer: ? (α1-agonists (Epinephrine) because of Mydriasis side effect) | Reference: FA USMLE step 1
Explanation: The pathophysiology of drug-induced angle-closure glaucoma is usually increased pupillary block (ie, increased iris-lens
contact at the pupillary border) from pupillary dilation. The classes of medications that have the potential to induce angle closure
are topical anticholinergic or sympathomimetic dilating drops, tricyclic antidepressants, monoamine oxidase inhibitors,
antihistamines, antiparkinsonian drugs, antipsychotic medications, and antispasmolytic agents.

130.A male patient presented with breast enlargement and decreased sexual desire. Labs: Hyperprolactinemia. What visual
deficiency this patient might have? (missing options)
A - Bitemporal hemianopia
Answer: A

131.How to test visual acuity? (missing options)

150
A - Snellen chart
Answer: A | Reference: Toronto notes

132.A patient comes with signs and symptoms of glaucoma. What is treatment? (missing options, answer not known)
Answer: ? (IV acetazolamide, topical pilocarpin and B blocker) | Reference: UQU; Toronto notes

It depends on the type of Glaucoma, whether Open or Close Angle Glaucoma. In any case, the goal of treatment is reduction of the
pressure before it causes progressive loss of vision.
Major drug classes for medical treatment: alpha-agonists, beta-blockers, carbonic anhydrase inhibitors, miotic agents, and
prostaglandin analogs. Also surgical options include trabeculectomy or aqueous shunt which is most commonly used.
http://emedicine.medscape.com/article/1206147-overview

133.Diabetic patient with high cup to disc ratio. What is the cause? (missing options, answer not known)
Answer: ?
Evidence: Patients with DM were previously thought to have a greater risk of developing primary chronic glaucoma with loss of
visual field. However, more recent papers suggest that DM is not a greater risk factor, but simply that glaucoma was found more
readily. Patients with PDR are at risk of developing secondary glaucoma, particularly neovascular (rubeotic) glaucoma. Rubeosis iridis
is the growth of new vessels on the iris in eyes with advanced retinal ischaemia. Rubeosis – neovascularization of iris (NVI) may
induce a severe form of intractable glaucoma with growth of new vessels in the anterior chamber angle (NVA). If uncontrolled, NVA
leads to closure of the aqueous fluid drainage route in the anterior chamber angle of the eye by fibrovascular tissue.
Reference: http://www.icoph.org/dynamic/attachments/taskforce_documents/2012-sci-
267_diabetic_retinopathy_guidelines_december_2012.pdf

134.(pic) Eye with entropian: (no stem of Q, missing options)


Answer: Entropion is a malposition resulting in inversion of the eyelid margin.

151
135.orbital cellulitis 2 times: (no stem of Q, missing options)
Answer: Orbital cellulitis and preseptal cellulitis are the major infections of the ocular adnexal and orbital tissues. Orbital cellulitis is
an infection of the soft tissues of the orbit posterior to the orbital septum, differentiating it from preseptal cellulitis, which is an
infection of the soft tissue of the eyelids and periocular region anterior to the orbital septum. Patients with orbital cellulitis
frequently complain of fever, malaise, and a history of recent sinusitis or upper respiratory tract infection. Other signs include:
Conjunctival chemosis, Decreased vision, Elevated intraocular pressure, Pain on eye movement. The patient with orbital cellulitis
should be promptly hospitalized for treatment, with hospitalization continuing until the patient is afebrile and has clearly improved
clinically. Medical management is successful in many cases. Consider orbital surgery, with or without sinusotomy, in every case of
subperiosteal or intraorbital abscess formation.

136.Ring lesion in eye: (missing options)


A - Acanthamoeba infection
Answer: A | Reference: http://eyewiki.aao.org/Acanthamoeba_Keratitis#Signs
It could be Kayser-Fleischer Ring (brownish-green copper ring)

137.keratitis caused by parasites: (missing options)


Answer: Acanthamoeba | Reference:
http://www.cdc.gov/parasites/acanthamoeba/health_professionals/acanthamoeba_keratitis_hcp.html

Acanthamoeba keratitis is characterized by pain out of proportion to findings. In one study, 95% of patients complained of pain.
Patients may also complain of decreased vision, redness, foreign body sensation, photophobia, tearing, and discharge. Symptoms
may wax and wane; they may be quite severe at times.
Reference: http://eyewiki.aao.org/Acanthamoeba_Keratitis
- Skin manifestations: ulcers, nodules, or subcutaneous abscesses.
- CNS: meningeoencephalitis ;Mental status changes, Seizures, Hemiparesis, Fever, Headache, Meningismus, Visual disturbances,
Ataxia, Nausea and vomiting, Hallucinations, Personality change, Photophobia, Sleep disturbances.
Disseminated disease without CNS involvement may manifest as skin lesions, sinusitis, pneumonitis, or a combination.
Reference: Medscape

138.what test is used for distant vision: (missing options)


Answer: visual acuity

139.Red eye pic after pharyngitis and fever, what is your dx? (missing options)
Answer: adenovirus conjunctivitis
Evidence: Pharyngitis caused by adenovirus is common among young children and military recruits. Patients with pharyngitis present
with sore throat (more intense than that of a common cold), high fever, dysphagia, and red eyes
Reference: http://emedicine.medscape.com/article/225362-clinical
140.(pic) disc cup ask for diagnosis: (missing details of Q, missing options, answer not known)
Answer: ? (The normal cup-to-disc ratio is 0.3. A large cup-to-disc ratio may suggest glaucoma or other pathology)

141.(pic) of cornea after nail injury, diagnosis? (missing options)


Answer: corneal abrasion

142.Sudden eye swelling redness and pain, hazy cornea: (no stem of Q, missing options)
Answer: Acute angle closure glaucoma | Reference: introduction to clinical emergency medicine text book ; UpToDate

143.Case of chronic eye irritation and Watery secretion + eye Enotropia Enotropia: (no stem of Q, missing options)
Answer: conjunctivitis

152
144.diabetic female come to the Ophthalmologist with Rt eye pressure 44 mmHg, Lt eye pressure 22 mmHg , examination of
retina of Rt eye show disc to rim ratio and nasal deviation of retinal vessels .what is the most likely Dx? (missing options)
Answer: Glaucoma | Reference: Toronto

145.URTI + conjunctivitis with hyperemia + watery discharge what other ocular symptom you expect: (missing options)
A - epithelial nummular keratitis
Answer: A

146.Ectropion pic: (no stem of Q, missing options)


Answer: Ectropion is an abnormal eversion (outward turning) of the lid margin away from the globe.

147.In retinal detachment where the fluid collect?


A. Photoreceptors and retinal pigment epithelium
Answer: A
Cleavage in the plane between the neurosensory retina and the RPE.
•Three types
1. rhegmatogenous (most common) caused by a tear or hole in the neurosensory retina, allowing fluid from the vitreous to
pass into the subretinal space, tears may be caused by PVD, degenerative retinal changes, trauma, or iatrogenically,
incidence increases with advancing age, in high myopes, and after ocular surgery/trauma
2. Tractional Š caused by traction (due to vitreal, epiretinal, or subretinal membrane) pulling the neurosensory retina away
from the underlying RPE, found in conditions such as DR, CRVO, sickle cell disease, ROP , and ocular trauma
3. Exudative, caused by damage to the RPE resulting in fluid accumulation in the subretinal space, main causes are
intraocular tumors, posterior uveitis, central serous retinopathy
reference: Toronto notes OP 24

148.Scenario of a patient with sky blue sclera?


A. Osteogenesis imperfect
Answer: A.
Osteogenesis imperfecta (OI) is a genetic disorder characterized by bones that break easily, often from little or no apparent cause.
http://www.oif.org/site/PageServer?pagename=fastfacts

149.Watery discharge congested eye preaurcual lymph node enlarge diagnosis?


Answer: Viral conjunctivitis
Explanation: From the giving symptoms the cause most probably Viral conjunctivitis (adenovirus).
Reference: http://emedicine.medscape.com/article/1191370-workup
http://www.merckmanuals.com/professional/eye-disorders/conjunctival-and-scleral-disorders/viral-conjunctivitis
Reference: http://bestpractice.bmj.com/best-practice/monograph/500/diagnosis/tests.html

153
150.corneal ulcer diagnosed by?
Answer: Diagnosis is made by slit-lamp examination.
Reference: http://www.merckmanuals.com/professional/eye-disorders/corneal-disorders/corneal-ulcer

151.Young with sudden unilateral painless eye blindness what the DIAGNOSIS ?!
Answer: review the pic in old Q
Explanation: the most common reason for painless sudden visual loss is ischemia?
Reference: http://emedicine.medscape.com/article/1216594-overview

152.Neonate presented to the hospital with eye discharge ..the mother had a vaginal discharge before delivery ,,, what is the
organism
Answer : Gonorrhea
Explanation: there are many other differentials such as silver nitrate solution and chlamydial, staphylococcal, and herpetic
infections.
Reference: http://emedicine.medscape.com/article/1192190-overview#a5

153.Patient c/o follicular keratosis, what you will look to check? Eye
a) vit. A deficiency
Answer: A

154.patient sensitive to light what is your action:


A) wear sunglasses protect form sunlight
Answer: Incomplete

155.Scenario, flashes & floaters in the eye, how to treat?


Answer: posterior vitreous detachment
http://www.geteyesmart.org/eyesmart/diseases/floaters-flashes/causes.cfm

156.Injury of vagus nerve cause which of eye symptoms??


A) One of the choises was ptosis
Answer:??

157.Herpes keratitis of eye, scenario with picture, how to treat?


Answer:
The mainstay of therapy is antiviral treatment either in the form of topical therapy with trifluridine 1% eight to nine times a day or
oral administration of acyclovir or valacyclovir for 10 to 14 days. If trifluridine drops are used, care is to be taken to ensure antiviral
drops are discontinued within 10-14 days due to corneal toxicity. Epithelial debridement of the dendrites may also be utilized in
conjunction with antiviral therapy to help reduce viral load. Topical corticosteroids are contraindicated in the treatment of active
HSV epithelial keratitis.
http://eyewiki.aao.org/Herpes_Simplex_Virus_Keratitis#Management

158.A picture of glaucomatous cupping.

154
http://www.optic-disc.org/tutorials/glaucoma_evaluation_basics/page13.html
normally 0.2- 0.4 Physiological cupping =0.5, but could be pathological if there are abnormalities by the other investigation or there
is difference btw two eyes; Pathological > 0.5

159.Treatment of high myopia, name of complicated ophthalmological surgeries?


Answer: Treatment of high myopia: Eyeglasses or Contact Lenses or Refractive Surgery.

160.Which dye used to examine the cornea?


Answer: fluorescein dye
Reference: https://www.nlm.nih.gov/medlineplus/ency/article/001017.htm

161.Cover test +, what is the complication of the condition?


Answer: Amblyopia
the patient has Squint and the treatment is batching but if the treatment was ignored or delayed the patient will develop amblyopia.
162.Patient develop 2 cm dome shape mass in the dorsum of the hand , it's cover by keration, What’s the diagnosis?
A. Keratoacanthoma.
Answer: A
Medscape:
Lesions typically are solitary and begin as firm, roundish, skin-colored or reddish papules that rapidly progress to dome-
shaped nodules with a smooth shiny surface and a central crateriform ulceration or keratin plug that may project like a
horn.
Most keratoacanthomas occur on sun-exposed areas. The face, neck, and dorsum of the upper extremities are common
sites.

163.Cover test to right eye cause lateral movement in left eye?


A. strabismus.
Nystagmus

164.Glaucoma drugs side effect?


Answer:
http://www.glaucoma.org/treatment/glaucoma-medications-and-their-side-effects.php

165.Vesicles on the eye and forehead?


a. Herpes zoster ophthalmicus.
Answer: A
http://www.aafp.org/afp/2002/1101/p1723.html

155
166.baby with inward deviation of rt eye , most likely due to :
Answer: refraction error
Explanation: Most strabismus is caused by Refractive error, muscle imbalance
Reference: http://www.msdmanuals.com/professional/pediatrics/eye-defects-and-conditions-in-children/strabismus

167.a case about one patient who had a bruit over the eye!
Answer: cavernous sinus thrombosis

168.Young ,very rapid reduce vision:


a-Glaucoma
reference: http://www.kellogg.umich.edu/patientcare/conditions/glaucoma.html

169.retinal tear between which layers?


Neuroretina and its pigment epithelium (RPE)
Reference:Lecture Notes Ophthalmology

170.patient with unilateral bacterial conjunctivitis , what is the antibiotic ?


A.topical erythromycin
Answer:A
The condition is usually self-limiting, although a broad-spectrum antibiotic eye drop will hasten resolution.Abe that can be used:
Trimethoprim with polymyx B, Gentamicin, Tobramycin,Neomycin, Ciprofloxacin, Ofloxacin ,Gatifloxacin ,Erythromycin.
Fluoroquinolones should be reserved for more serious infections to minimize bacterial resistance.
Reference: http://emedicine.medscape.com/article/1191730-treatment

171.Patient having esoteopia, what part of brain responsible for that ?!


Strabismus: http://patient.info/doctor/strabismus-squint

The fibers of the oculomotor nerve arise from a nucleus in the midbrain, which lies in the gray substance of the floor of the cerebral
aqueduct and extends in front of the aqueduct for a short distance into the floor of the third ventricle
https://en.wikipedia.org/wiki/Oculomotor_nucleus

172.patient compain of ptosis how can. Repaire m ?

173.A patient is complaining of the severe eye pain and ciliary congestion post operatively. What is the most likely diagnosis?
Answer is: no choices
Possible answer: Endophthalmitis
- Endophthalmitis is a common post ophthalmic operations complication.
Source: Toronto notes “page OP23”
A good source to differentiate between ciliary & conjuctival congestion: https://pgblazer.com/difference-between-
conjunctival-congestion-and-circumcorneal-congestion/

174.Ectropion or entropion (I’m not sure) 25 degrees abnormality, treatment:


Answer: no choices
156
- Entropion: (inversion of an eyelid) is caused by age-related tissue relaxation, post-infectious or posttraumatic changes,
or blepharospasm. Eyelashes rub against the eyeball and may lead to corneal ulceration and scarring. Symptoms can
include foreign body sensation, tearing, and red eye.
- Ectropion: (eversion of the lower eyelid) is caused by age-related tissue relaxation, cranial nerve VII palsy, and
posttraumatic or postsurgical changes. Symptoms are tearing (due to poor drainage of tears through the nasolacrimal
system) and symptoms of dry eyes.
- Treatment of Ectropion and entropion: surgical repair.
Source: http://emedicine.medscape.com/article/1844045-overview#showall

175.What is the pathophysiology of retinal detachment?


o Retinal detachment occurs due to a break in the retina (called a retinal tear) that allows fluid to pass from the vitreous
space into the sub retinal space between the sensory retina and the retinal pigment epithelium.
http://emedicine.medscape.com/article/798501-overview
176.Girl with crossed eyes?
o Squint

177.baby born with white eye opacity can’t see what behind lens:
- congenital cataract
A cataract is an opacification of the lens. Congenital cataracts usually are diagnosed at birth. If a cataract goes undetected in an
infant, permanent visual loss may ensue.
Reference: http://emedicine.medscape.com/article/1210837-overview

178.patient with herpetic keratitis what associated with:


- cornea h....
Read more about Herpetic Keratitis: http://emedicine.medscape.com/article/1194268-clinical#b3

179.one has blow trauma to the eye present with sub-conjunctival hemorrhage and weak up ward gaze?
Orbital base (blowout) fracture
Reference: http://emedicine.medscape.com/article/1284026-overview#a10

180.Picture of entropion, what is dx


Entropion is a malposition resulting in inversion of the eyelid margin. The morbidity of the condition is a result of ocular surface
irritation and damage.

181.Best way to reduce gonococcal conjunctivitis.


The best way to avoid it is to treat the mother and if necessary to prevent a vaginal delivery in a gonorrhea infected mother.
Erythromycin and tetracyclin drops are effective as prophylactic agents.

182.A scenario of patient with Herpetic dendrites and a picture also.


157
They asked what is associated with this condition. [Exactly the same picture]

Answer: Typically, patients with HSV keratitis present with blurry vision, extreme photophobia, pain, redness, tearing, foreign body
sensation, and conjunctival hyperemia. Sometimes vesicular blepharitis (blisters on the eyelid) follows, symptoms worsen, vision
blurs, and blisters break down and ulcerate, then resolve without scarring in about a week. Toronto note 2015

183.Eye pain & itching after 4 month regimes of TB drug.... Which drug cause that?
Answer: Ethambutol

184.COPD developed DM and acute closure glaucoma>>> which treatment:


I think they want you to avoid anticholinergic.

185.Proliferative diabetic retinopathy.

186.Q about chlamydia conjunctivitis, about prevention: (all options about water sanitization and vectors) (not related to delivery
)
by water cholorization
st
Answer: I tried to search something not related to delivery but couldn’t find except for these below. The 1 is delivery related.

Educate parents or care providers to wash their hands frequently to prevent transmission of neonatal conjunctivitis. Educate
pregnant women on the importance of regular examinations to detect and treat sexually transmitted infections such as herpes
simplex, gonorrhea, and chlamydia in order to decrease the incidence of neonatal conjunctivitis.
158
Prophylaxis against conjunctivitis — In most of the United States, neonatal prophylaxis against gonococcal conjunctivitis is
routinely performed at birth, and required in the majority of states. However, neonatal ocular prophylaxis is not effective in
preventing neonatal chlamydial conjunctivitis [31,40-44]. The diagnosis and treatment of chlamydial infections in pregnant
women is the best method for preventing chlamydial disease

Eye care:
Ocular infections in newborns caused by Chlamydia trachomatis are common in the United States [5]. The agents also used for
gonococcal prophylaxis are not effective in preventing neonatal chlamydial conjunctivitis. Povidone-iodine appears to be
significantly more effective against C. trachomatis than silver nitrate or erythromycin.
Technique — After wiping each eyelid with sterile cotton gauze, the prophylactic agent is placed in each of the lower conjunctival
sacs [5]. The agent should be spread by gentle massage of the eyelids, and excess solution or ointment can be wiped away after one
minute. The eyes should not be irrigated after the application because doing so may reduce efficacy.
Reference: http://cursoenarm.net/UPTODATE/contents/mobipreview.htm?6/19/6449?source=see_link&anchor=H6#H6
http://cursoenarm.net/UPTODATE/contents/mobipreview.htm?7/35/7728

187.Pic of eye (there is proptosis and redness of upper eyelid on one eye) what is diagnosis;
A. Orbital cellulitis
B. Chalazion
Answer: A
In orbital cellutis, the patient would have from fever Conjunctival chemosis, Decreased vision, Elevated intraocular pressure and
Pain on eye movement. A chalazion is usually a painless swelling on the eyelid that has been present for weeks to months.

188.A case about pathological myopia.

159
Orthopaedics

160
1. Patient is referred to Orthopedics because of abnormally high bone density scan. She has no history of fractures. She takes
analgesics and is waiting for hip replacement surgery. Her dietary calcium and serum biochemistry are normal. What is the
cause of her high density result? (no numbers were provided).
A- Osteoarthritis.
B- Paget’s disease.
C- Osteoporosis.
D- Osteopenia.
Answer: B
Paget disease is a localized disorder of bone remodeling that typically begins with excessive bone
Resorption followed by an increase in bone formation. This osteoclastic overactivity followed by
Compensatory osteoblastic activity leads to a structurally disorganized mosaic of bone (woven bone),
Which is mechanically weaker, larger, less compact, more vascular, and more susceptible to fracture
Than normal adult lamellar bone.
Reference: http://emedicine.medscape.com/article/334607-overview

2. High density bone in DEXA with scheduled total hip replacement:


A - osteoporosis
B - osteomalacia
C - osteoarthritis
D - paget disease

Answer: C (osteoarthritis)
Evidence: Bone density is actually HIGHER rather than LOWER in osteoarthritis.
Low bone density is the telltale sign of osteoporosis, a skeletal disorder
characterized by weakened bones due to excessive loss of bone mass.
Osteoarthritis, on the other hand, is characterized by increased bone density
and bony growths (osteophytes) in conjunction with articular cartilage degeneration
Reference: http://www.orthop.washington.edu/?Q=patient-care/articles/arthritis/osteoarthritis.html

Symptoms of OA: joint pain or tenderness, Stiffness, loss of flexibility, Grating Sensation, & bone Spurs
Tests & Diagnosis: X-ray will show narrowing of the space between the bones in the joint,
Reference: http://www.mayoclinic.org/diseases-conditions/osteoarthritis/basics/tests-diagnosis/con-20014749
Http://emedicine.medscape.com/article/330487-differential
Artefactual causes of raised BMD—no true increase in bone mass:
OA
DISH
AS
Vertebral fractures
Vascular calcification
Thalassaemia major
Abdominal abscesses
Gallstones
Renal calculi
Gluteal silicone implants
Gaucher’s disease
Intestinal barium
Surgical metalwork
Laminectomy
Vertebroplasty and kyphoplasty
http://www.ncbi.nlm.nih.gov/pmc/articles/PMC3651616/

161
3. 37 years old presented with back pain. On examination: there was tenderness when palpating para-spinal muscles,
neurovascular exam was normal. What is the treatment? – (MRI showed mild spinal stenosis)
A. Physiotherapy
B. Bio feedback
C. Cortisone
D. Surgery
Answer: A

4. Patient with fracture humerus what the nerve injury?


A. Axillary
B. Median
C. Ulnar
D. Radial
Answer: A & D
Mid-shaft fractures may damage the radial nerve
Surgical neck fracture may damage axillary nerve

Evidence:
Important tips to remember: Fracture of humerus at:
- Surgical neck: axillary nerve and posterior humeral circumflex artery.
- Mid-shaft: radial nerve (loss of wrist extension) and profunda brachii artery.
- Supracondylar region: median Nerve and brachial artery.
- Medial epicondyle: ulnar nerve.

5. Humerus fracture, how will he present?


A. Waiter’s tip hand
B. Wrist drop
C. Claw hand
Answer:
Mid shaft humerus > radial nerve injury > wrist drop
Surgical neck > axillary nerve
Medial epicondyle > ulnar nerve > claw hand
Supra condylar > median n > ape hand

6. Injury to the surgical neck of humerus cause injury to:


A - Radial A
B - Median
C - Ulnar
D - Brachial plexus (axillary)

Answer: D
Evidence:

162
- This type is proximal humerus injury
- A, B and C go with distal and humeral shaft injury. Assessment: Perform a neurological examination, particularly examining the
axillary nerve Assess and brachial plexus injury through distal neurological examination. Reference: Toronto Notes 2015

The surgical neck of the humerus is a constriction below the tubercles of the greater tubercle and lesser tubercle. It is much more
frequently fractured than the anatomical neck of the humerus. A fracture in this area is most likely to cause damage to the axillary
nerve and posterior circumflex humeral artery. Reference: Wikipedia: http://www.orthobullets.com/trauma/1015/proximal-
humerus-fractures

7. Man was hit by a car, cleared by trauma team, vitally stable with 2 pictures: one showing open fracture of tibia and fibula and
in the other there is bleeding in the shin. How to manage?
A. External fixation
B. Internal fixation
C. Drainage with fixation (didn't mention the type of fixation in this choice)
D. IV antibiotic
Answer: B

8. Football player came to the doctor with pain in the knee after the other player hit him from the side of the knee, valgus test
is positive, what ligament is injured?
A. Fibular (lateral) collateral ligament
B. Tibial (medial) collateral ligament
C. ACL
D. PCL

Answer: B
Medscape: Evocative testing of collateral ligaments:
Valgus stress testing of the MCL: The patient is in the supine position with the knee flexed 25-30 º. The examiner places one hand on
the lateral knee and grasps the medial ankle with the other hand. Then the knee is abducted. Pain and excessive laxity indicate
stretching or tearing of the MCL.

- Mechanism of injury of lateral collateral ligament contact injury, such as a direct blow to the medial side of the knee, or a
noncontact injury, such as a hyperextension stress, may result in a varus force across the knee injuring the LCL
- LATERAL COLLATERAL LIGAMINT TESTED BY VARUS STRESS TEST
- MEDIAL COLLATERAL LIGAMENT TESTED BY VLULGAS STRESS TEST

9. Case of tibial collateral ligament injury: (no stem of Q, missing options, answer not known)
A - Valgus stress testing of the MCL
B - Varus stress testing of the LCL

Recommendations
Injury severity
Medial collateral ligament (MCL)* Lateral collateral ligament (LCL)**
Grade I [< 5 cm - Compression, elevation, & cryotherapy
Treatment similar to that for MCL injuries + hinged
laxity (partial - Short-term use of crutches with weight-bearing–as–
brace for 4-6 weeks
tear)] tolerated (WBAT) ambulation & early ambulation
- short-hinged brace blocking 20% of extension but allows
Grade II (5 - 10 full flexion Treatment similar to that for MCL injuries + hinged
cm laxity) - WBAT with closed-chain exercises to allow for brace for 4-6 weeks
strengthening of knee musculature without putting stress

163
on the ligaments
- Initially non–weight-bearing (NWB) on the affected
- Surgical treatment due to rotational instability
Grade III [> 10 cm lower extremity
(involvement of posterolateral corner of the knee)
laxity (complete - A hinged braced should be used, with gradual
- Bracing & physical therapy for ~ 3 months to
tear)] progression to full weight-bearing (FWB) over 4 weeks
prevent instability
(may require 8-12 weeks to heal)
Answer: ?
Evidence:
* All MCL injuries should be treated with early range of motion (ROM) and strengthening of musculature that stabilizes the knee
joint. Conservative measures usually are adequate, but, if the patient fails to progress with treatment, a meniscal or cruciate
ligament tear is suggested
** heal more slowly difference in collagen density
Reference: Medscape: http://emedicine.medscape.com/article/307959-treatment#d9

10. Pt with long scenario had trauma to knee. The knee displaced to anterior. (+ anterior drawer test) Which structure is injured?
A - anterior cruciate ligaments
B - posterior cruciate ligaments
C - medial meniscus
D - lateral meniscus

Answer: A
Evidence:
The pivot shift test is performed by extending an ACL-deficient knee, which results in a small amount of anterior translation of the
tibia in relation to the femur. During flexion, the translation reduces, resulting in the "shifting or pivoting" of the tibia into its proper
alignment on the femur. It is performed with the leg extended & the foot in internal rotation, & a valgus stress is applied to the tibia.
Reference: http://emedicine.medscape.com/article/89442-overview

11. Description of anterior cruciate ligament tests?

Answer:
The knee is flexed at 20–30 degrees with the patient supine. The examiner should place one hand behind the tibia
and the other grasping the patient's thigh. It is important that the examiner's thumb be on the tibial tuberosity. The
tibia is pulled forward to assess the amount of anterior motion of the tibia in comparison to the femur. An intact
ACL should prevent forward translational movement ("firm endpoint")
Primary function Resists anterolateral displacement of the tibia on the femur
Secondary function Resists varus displacement at 0 degrees of flexion
Reference: orthobullets

Anterior drawer test: The patient should be supine with the hips flexed to 45 degrees, the knees flexed to 90 degrees and the feet
flat on table. The examiner grasps the proximal lower leg, just below the tibiofemoral joint line, and attempts to translate the lower
leg anteriorly. The test is considered positive if there is a lack of end feel or excessive anterior translation relative to the
contralateral side.

Lachman test: with the knee flexed 20–30°, the tibia is displaced anteriorly relative to the femur; a soft endpoint or greater than 4
mm of displacement is positive (abnormal).

12. Lachman test, if positive it indicates what?


A. ACL
164
B. PCL
C. Medial meniscus
D. Lateral meniscus
Answer: A
The Lachman Test is a commonly used in orthopedic examinations to test for anterior cruciate ligament (ACL) integrity.
Http://physicaltherapyweb.com/lachman-test-orthopedic-examination-knee/

13. Young boy was playing sports with his friends, then presented to hospital with left shoulder pain, the arm is adduction with
internal rotation. This boy has dislocation in which area?
A. Inferior
B. Subglenoid Anterior
C. Subacromial Anterior
D. Supracondylar Posterior

Answer: D
The type of sholder dislocation to present with adduction and internal rotation is posterior, which is less common than anterior
dislocation.
Http://www.wheelessonline.com/ortho/posterior_shoulder_dislocation
14. Military soldier with flat foot. Which of the following will be sustained?
A - Flexor retinaculum
B - Extensor retinaculum
C - Spring ligament
D - Achilles tendon

Answer: C
Evidence:
Medial arch (navicular) injury: Injuries to the navicular bone and related structures jeopardize the structural integrity of the medial
foot arch and thus can lead to severe disability and chronic pain. Therefore, navicular injury is considered high risk, particularly for
runners. Such injuries include navicular stress fractures, tendinopathy of the posterior tibialis tendon insertion onto the navicular,
traumatic separation of an accessory navicular, and partial or complete tears of the attachment of the plantar calcaneonavicular
(spring) ligament.
Reference: http://www.uptodate.com/contents/evaluation-and-diagnosis-of-common-causes-of-foot-pain-in-adults#H13378296
Clinical conditions:
- posterior tibial tendon dysfunction: calcaneonavicular ligament is the most likely ligament to be attenuated in a patient with a
type II flatfoot deformity secondary to posterior tibial tendon dysfunction
- acute spring ligament tear:
= acute trauma can occur with forceful landing on flat foot
= the tibialis posterior tendon is usually normal
Reference: http://www.orthobullets.com/foot-and-ankle/7005/ankle-ligaments

15. Adult with mid-clavicular fracture. X-ray showed overlapped bone fragments. How will you treat?
A - internal fixation
B - external fixation
C - splint
D - figure 8

Answer: A
Evidence:
Nondisplaced midshaft clavicle fractures: arm sling or figure-of-eight dressing (arm sling is better)

165
Displaced mid-shaft fractures: Although non-operative treatment is a viable option to treat displaced mid-shaft fractures, operative
repair should be considered in patients with multiple risk factors for non-union, especially significant fracture displacement or
clavicle shortening. Reference: http://www.aafp.org/afp/2008/0101/p65.html

16. A patient was diagnosed with scoliosis, based on the cobb angle, when to do an orthopaedic referral?
A - > 5 degrees
B - > 10 degrees
C - > 15 degrees
D - > 20 degrees

Answer: B
References: Toronto notes. Http://www.medscape.com/viewarticle/449233_5
Evidence:
The term “Cobb Angle” is used worldwide to measure and quantify the magnitude of spinal
deformities, especially in the case of scoliosis. The Cobb angle measurement is the “gold standard” of
scoliosis evaluation endorsed by Scoliosis Research Society. It is used as the standard measurement to
quantify and track the progression of scoliosis.
Treatment:
• based on Cobb angle
Ƒ < 25°: observe for changes with serial radiographs
Ƒ > 25° or progressive: bracing (many types) that halt/slow curve progression but do NOT reverse deformity
Ƒ > 45°, cosmetically unacceptable or respiratory problems: surgical correction (spinal fusion)
Referral: Any child should be referred immediately when there is suspicion of scoliosis. Radiographs should be left to the specialist to
order. Plain anterior/posterior and lateral views are inadequate and may lead to a missed diagnosis of scoliosis

17. Patient with septic arthritis. The labs showing resistance to the antibiotic. What will you do? (answer not known)
A - stop antibiotic
B - start vancomycin
C - add vancomycin
D - gentamicin

Answer: ? (THIS QUESTION IS NOT COMPLETE, IF THERE IS SURGERY OPTION IT WILL BE THE ANSWER. IF NOT WE SHOULD KNOW
THE CULTURE AND THE RESULTS OF INVESTIGATION) | Reference: http://emedicine.medscape.com/article/236299-medication#2
18. What is the type of elbow joint? Or (Which of these joint is hinge joint?)
A - Ball and socket joint.
B - Saddle joint.
C - Pivot joint.
D - Hinge joint.

Answer: D (elbow is a hinge joint) | Reference: http://www.teachpe.com/anatomy/joints.php

19. What type of joint is the hip joint?


A - Ball and socket joint.
B - Saddle joint.
C - Pivot joint.
D - Hinge joint.

Answer: A
20. Type of intervertebral disc joint?
Answer: secondary cartilaginous, gliding joint.
References:
166
Https://en.wikipedia.org/wiki/Cartilaginous_joint
Http://www.innerbody.com/image_skel07/skel32.html#full-description

21. Osteoporosis, how to prevent spinal compression fx?


A - Daily vit D
B - Aerobic exercise
C - Weight bearing
D - Walking
Answer: A
Evidence: Vitamin D is used for prevention and treatment, and prevents total number of fractures
Reference: http://www.uptodate.com/contents/osteoporotic-thoracolumbar-vertebral-compression-fractures-clinical-
manifestations-and-
treatment?Source=search_result&search=osteoporotic+vertebral+compression+fracture&selectedtitle=1~150#H1565673

22. Occipital headache and neck pain can't do lateral flexion weakness of deltoid bilaterally on x-ray
A. osteophyte
B. neck strain
C. cervical spondylitis
D. brachial plexus neuropathy

23. Renal or small cell cancer stage III with bone pain what is the immediate action?
A. MRI only
B. Radiotherapy
C. IV steroid and MRI
D. No immediate action

Answer: A?
Not sure about the answer (whether immediate or not) but mostly it is

24. Man has a RTA. He was conscious, oriented, alert, had pain in all of his 4 extremities, what’s the type of shock?
A - irreversible
B - neurogenic
C - cardiogenic
D - haemorrhagic

Answer: B

25. Pt. Presented with deformity in the hand, xray showed fracture in the radius, they put picture of
xray and the hand. How will you fix it?
A - Close reduction ... Below elbow
B - Closed reduction. ... Above elbow
C - Open reduction ... Below elbow
D - Open reduction above elbow

Answer: A (Q not clear; it says fracture in the radius but didn’t specify radial head, shaft or distal)
Evidence:
For the distal radius fracture the treatment depend:
- if extra-articular and/or <5mm shortening we will do closed reduction and cast
- if its intra-articular we can do either fixation (ORIF) and cast
- Most distal radial fractures are casted with below elbow

167
References:
https://www2.aofoundation.org/wps/portal/surgery?Showpage=redfix&bone=Radius&segment=Distal&classification=23-
A1.1&treatment=&method=Cast&implantstype=&approach=&redfix_url=1285238880882&Language=en
- http://www.orthobullets.com/trauma/1027/distal-radius-fractures#4604
- http://www.orthobullets.com/trauma/1027/distal-radius-fractures

26. Woman complaining of left hand tingling mainly at thumb and index on exam there was mild atrophy of thenar muscle
Tinel’s test was positive which nerve may be affected?
A - Radial nerve.
B - Median nerve.
C - Musculocutaneous nerve.
D - Ulnar nerve.

Answer: B
27. What is the most accurate test for carpal tunnel syndrome:
A - Tinel
B - Compression test
C - Durkan's carpal test
D - Phallens test

Answer: B (Sensitivity 89%, specificity 96%)


References: uptodate & Medscape: http://www.uptodate.com/contents/carpal-tunnel-syndrome-clinical-manifestations-and-
diagnosis?Source=outline_link&view=text&anchor=H5#H5

Answer: B | Reference: http://www.medicinenet.com/script/main/mobileart.asp?Articlekey=16687

28. Patient with numbness of index finger when he uses scissors, what is the diagnosis:
A - Osteoarthritis.
B - Ducyptus.
C - Carpal Tunnel Syndrome.
D - Cubital Tunnel Syndrome.

Answer: C

29. Carpal tunnel syndrome. Pt can't work and write which muscles affected:
A. Thenar eminence
B. Interossei palmar muscle
C. Interossei dorsal muscle

Answer: A Compression of the median nerve as it runs deep to the transverse carpal ligament (TCL) causes atrophy of the thenar
eminence, weakness of the flexor pollicis brevis, opponens pollicis, abductor pollicis brevis

30. A patient is complaining of pain and numbness in his left thumb and index with loss of sensation over the dorsum 1/3 of left
hand. What will you do to confirm the diagnosis?
A. Nerve conduction studies
B. CT
C. MRI
Answer: A
The patient’s symptoms and signs match the distribution of median nerve, so carpal tunnel syndrome (CTS) is the main differential
diagnosis. Electromyography (EMG) and nerve conduction studies are the first line investigation in suspected CTS. An MRI is useful
preoperatively if there is any space occupying lesion. But before doing an MRI, ultrasound should be done.
168
Reference http://emedicine.medscape.com/article/327330-workup#c7

31. (Picture) 4 y/o boy with distal ulnar & redial bone fracture the skin was slightly injured by the bone what is the management?
A - closed reduction with cast below elbow
B - closed reduction with cast above the elbow
C - open reduction with plaid
D - Aggressive debridement and irrigation and Fracture stabilization

Answer: D| Reference: http://www.orthobullets.com/trauma/1004/open-fractures-management


32. Falls onto an outstretched hand leads to:
A - Clavicle fracture.
B - Colles fracture.
C - Shouder Dislocation.
D - Mith fracture.

Answer: B
Colle’s: Extra-articular transverse distal radius fracture (~2 cm proximal to the radiocarpal joint) with dorsal displacement ±
ulnar styloid fracture.
Smith’s: Volar displacement of the distal radius (i.e. Reverse Colle’s fracture)

33. A girl fell on outstretched hand. X-ray shows open fracture of radius and ulna. What is the treatment?
A. Internal fixation
B. External fixation
C. Drainage and fixation (didn’t mention type of fixation in this choice)
D. Antibiotic
Answer: A

34. Pt fall on outstretched hand, & he can't flex the distal part of his second finger? Which muscle is injured?
A - Flexor pollicis longus.
B - Flexor digitorum superficialis.
C - Flexor digitorum profundus.
D - Extensor carpi radialis longus.

Answer: C (flexor digitorum profundus for the proximal & distal joints, & flexor digitorum superficialis for the proximal joints)
Reference: Wikipedia

35. A 13 years old boy with right hip pain for 3 months, x-ray shows degenerative
collapse of femoral neck, WBC = 17,
ESR is normal, ALP is normal:
A. Perthe diseases
B. Hip arthritis
C. Tumor
D. Developmental Dysplasia of the HIP (DDH)

Answer: A (Also known as Legg–Calvé–Perthes disease (LCPD) or Coxa Plana and it is


a paediatric disease causing an idiopathic avascular necrosis of the femoral head.
Features: Antalgic gait ± pain, hip pain. Diagnostic features on X-Ray is a collapsed
femoral head) | Reference: Kaplan surgery

169
36. Pt after RTA, no abduction and lateral rotation of the arm. What is the origin of the affected nerve?
A - Medial plexus
B - Lateral plexus
C - Lower plexus
D - Root

Answer: D (Axillary nerve palsy, but the answers are not clear)
Evidence: Erb’s Palsy or Upper plexus or C5 root injury: Shoulder abduction, extension and external rotation are lost.
Reference: http://www.physio-pedia.com/Brachial_plexus_injury#Function

37. Old Patient with back pain. Spine DEXA results is provided below:
(Cervical = 0.05 ; Thoracic = < - 1.5 ; Lumbar = < - 2.8 ; Sacral = < - 1.3). What is the diagnosis?
A - Osteopenia
B - Severe osteopenia
C - Osteoporosis
D - Established osteoporosis

Answer: C (> - 1 is normal, - 1 to - 2.5 is osteopenia, < or equal to - 2.5 is osteoporosis)


Evidence: Bone mineral density T-score criteria for osteoporosis and low bone mass
Diagnosis T-score
Normal ≥-1.0
Low bone mass (osteopenia) Between -1.0 and -2.5
Osteoporosis ≤-2.5
Severe (established) osteoporosis ≤-2.5 and fragility fracture
Reference: uptodate:
http://www.uptodate.com/contents/image?Imagekey=ENDO%2F53999&topickey=ENDO%2F2056&source=see_link

38. Which of the following is most common non traumatic fracture in osteoporosis?
A - Vertebral fracture.
B - Statin induced myopathy in old lady.
C - Colles fracture.
D - Ulnar fracture.

Answer: A (Vertebral fracture is the most common clinical manifestation of osteoporosis) | Reference: uptodate

39. Old male not known to have any medical illness presented with right knee swelling, no fever or tenderness, what
investigation will you order?
A - Arthrocentesis.
B - Knee MRI.
C - X-Ray.
D - CT.

Answer: C | Reference: approach to non-traumatic knee effusion: http://www.aafp.org/afp/2000/0415/p2391.html

40. Which part of body bone is forming the ankle?


A - Calcanues.
B - Cuboid.
C - Talus.
D - Cuneiforms.

170
Answer: C (Ankle joint: the fibula, the tibia and the talus)

The true ankle joint is composed of three bones, anterior view: the tibia which forms the inside, or medial portion of the ankle; the
fibula which forms the lateral or outside portion of the ankle and the talus underneath.
The true ankle joint is responsible for the up-and-down motion of the foot.

41. Examiner asked a patient to stand on his toes, what is the nerve he examined?
A - Femoral nerve.
B - Deep peroneal nerve.
C - Tibial nerve.
D - Superficial peroneal nerve.

Answer: C (Toe Dorsifelxion by Deep peroneal nerve ; Toe Plantar flexion by Tibial nerve)
Reference: http://www.orthobullets.com/spine/2002/lower-extremity-spine-and-neuro-exam

42. A patient can't do dorsiflexion & eversion: (Foot drop) + loss of sensory of first web space between 1st and second toes
A - Femoral nerve.
B - Deep peroneal nerve.
C - Tibial nerve.
D - Superficial peroneal nerve.

Answer: B

Explanation: deep peroneal nerve is responsible for dorsiflexion of the foot and sensation of the first web space. Injury can cause
foot drop.
Reference: http://teachmeanatomy.info/lower-limb/nerves/deep-fibular/

- common peroneal causes foot drop and its course is close to the tibia plateau posteriorly at the lateral side (tibia plateau = head)
- if the question stated that there is numbness in the dorsum of the foot, then it is the deep peroneal

43. Patient with 1st metatarsal joint pain, red, tender, hot joint? What is the cause?
A - Staphylococcus aureus.
B - Monosodium urate crystal.
C - Ca pyrophosphate crystal.
D - Haemarthrosis.

Answer: B
- Gout is caused by monosodium urate monohydrate crystals.
- Pseudogout is caused by calcium pyrophosphate crystals and is more accurately termed calcium pyrophosphate disease.

44. Bilateral shoulder and hip stiffness and pain. What is the diagnosis:
A - Polymyalgia rheumatica.
B - OA.
C - Fibromyalgia.
D - Frozen shoulder.

Answer: A
45. Loss of adduction of fingers caused by injury to:
A - Ulnar nerve.
B - Median nerve.
C - Radial nerve.

171
D - Axillary nerve.

Answer: A

46. Patient with Osteoarthritis which type of exercise is the best:


A - High repetition and high resistance.
B - Low repetition and high resistance.
C - High repetition and low resistance.
D - Low repetition and low resistance.

Answer: D

47. Limping child, painful left hip (x-ray given)?


A - Slipped Capital Femoral Epiphysis.
B - Perthe's disease.
C - Developmental Dysplasia of the Hip (DDH).
D - Osgood-Schaltter Disease.

Answer: A
Evidence:
Klein's Line On AP view, line drawn along supero-lateral border of femoral neck
should cross at least a portion of the femoral epiphysis. If it does not, suspect SCFE

48. What is the artery supplying the gluteal region?


A - Internal pudendal artery.
B - Internal iliac artery.
C - External iliac artery.
D - Femoral artery.

Answer: B

49. An elderly patient came with bilateral decreased range of motion of knee joint. There was surrounding muscle wasting.
Examination shows crepitus. What is the Diagnosis:
A - Osteoarthritis
B - Rheumatoid arthritis.
C - Septic Arthritis.
D - Patella dislocation.

Answer: A
50. Mother and father were holding their son from his hand and they elevate them over the ground, then the child complain of
pain in his elbow he was flexing his arm with pronation he didn't let anybody touch it. What was the injured ligament?
A - Quadrant ligament.
B - Annular ligament.

172
C - Sheet between radius and ulnar bone.
D - Medial collateral ligament.

Answer: B | Reference: Pulled elbow: http://orthoinfo.aaos.org/topic.cfm?Topic=A00717

51. Patient complaining of hip pain after long periods of using the hip it keeps him awake at night and have prolonged hours of
stiffness in the morning:
A - Osteoporosis.
B - Osteoarthritis.
C - Hip fracture.
D - Muscle strain.
Answer: B

52. Knee aspiration in young boy showing labs result: WBC > 75, >25 neutrophils. Typical scenario. What is the treatment?
A - Oral antibiotic.
B - IV antibiotic
C - Surgical drainage with IV antibiotic.
D - Rest & pain killer if needed.

Answer: C
Evidence:
Medical management of infective arthritis focuses on adequate and timely drainage of the infected synovial fluid, administration of
appropriate antimicrobial therapy, and immobilization of the joint to control pain.

53. Shoulder dislocation, was put into place then loss of sensation over lateral arm occurred. What’s the nerve affected?
A - Axillary nerve.
B - Radial nerve.
C - Median nerve.
D - Ulnar nerve.

Answer: A

54. Pain in the snuff box would most likely to be due to fracture of (other version: What is snuff box fracture?)?
A - Scaphoid.
B - Head of hamate fracture.
C - Colle’s fracture.
D - 1st metacarpal fracture.
Answer: A

55. What is the most commonly injured Carpal bone? (missing options)
A - Scaphoid.

Answer: A
Evidence:
Scaphoid (palpated in anatomic snuff box) is the most commonly fractured carpal bone and is prone to avascular necrosis owing to
retrograde blood supply.
Dislocation of lunate may cause acute carpal tunnel syndrome. A fall on an outstretched hand that damages the hook of the hamate
can cause ulnar nerve injury.

173
Reference: FA USMLE step 1

56. Post-menopausal with fractures and osteoporosis, what is the best drug?
A - Calcitonin.
B - Calcitriol.
C - Alendronate.
D - Estrogen.

Answer: C (Alendronate is the first line for postmenopausal osteoporosis)

57. Old man on steroids and osteoporosis, what to give:


A - Calcitonin.
B - Calcitriol.
C - Alendronate.
D - Raloxifene.

Answer: C

58. Old lady postmenopausal with osteoarthritis and risk for osteoporosis, what will you do:
A - Calcium, TSH, dihydroxy vit D.
B - Bisohosphonate, vit D, calcium.
C - DEXA scan.
D - Estrogen level.

Answer: C
Thyroid-stimulating hormone level: Thyroid dysfunction has been associated with osteoporosis.
Http://emedicine.medscape.com/article/330598-overview

59. Child with radial and ulnar fracture and 1cm laceration in the volar aspect of the hand, what is most likely damaged nerve?
A - Radial nerve.
B - Median nerve.
C - Ulnar nerve.
D - Axillary nerve.

Answer: C

60. An elderly woman who has chronic back pain which increases while walking uphill and decreases while walking down hill,
what is the most likely diagnosis?
A - Claudication.
B - Lumbar Strain.
C - Lumbar Spine Stenosis.
D - Pott Disease.

Answer: B (lumbar spine stenosis)

174
Reference: https://www.laserspineinstitute.com/back_problems/neurogenic/symptoms/

61. In which condition bone age is greater than chronological age?


A - Constitutional growth delay.
B - Growth hormone deficiency.
C - Precocious puberty.
D - Hypothyroidism.

Answer: C
Evidence:
An advanced bone age is common when a child has had prolonged elevation of sex steroid levels, as in precocious puberty or
congenital adrenal hyperplasia

62. Female with leg pain when she walks 300 m. Relief by rest.
A - Claudication.
B - DVT.
C - Venous insufficiency.
D - Ligament rupture.

Answer: A
Read more about Vascular Claudication Versus Spinal Stenosis.

63. 10 years old child complaining of pain in tibial tubercle he has just had growth spurt. What is the cause of the pain:
A - Osgood schlatter disease.
B - Legg-perthes-canal disease.
C - Stress fracture.
D - Tibial splint.

Answer: A (partial avulsions or microavulsions of chondrofibro-osseous tibial tubercle)

64. Pt with tibiofibular fracture, plaster of paris applied, came back with extreme pain, what’s the sign to your dx:
(What is the first sign of compartment syndrome of the leg?)
A. Loss of pulse
B. Loss of sensation

175
C. Loss of movement
D. Pain out of proportion
Answer: D
Patients with compartment syndrome typically present with pain whose severity appears out of proportion to the injury. The pain is
often described as burning. The pain is also deep and aching in nature and is worsened by passive stretching of the involved muscles.
The patient may describe a tense feeling in the extremity.
(http://emedicine.medscape.com/article/307668-clinical)

CLINICAL FEATURES — The signs and symptoms of acute compartment syndrome (ACS) generally appear in a stepwise fashion,
although the timing of the appearance of specific findings varies. Important clues to the development of ACS include rapid
progression of symptoms and signs over a few hours and the presence of multiple findings consistent with the diagnosis in a patient
at risk. Therefore, serial evaluation is of great importance in patients at risk for ACS. However, the limitations of the physical
examination for identifying ACS must be emphasized; any tense painful muscle compartment represents a possible ACS.

Symptoms of ACS can include the following:

Pain out of proportion to apparent injury (early and common finding)


Persistent deep ache or burning pain
Paresthesias (onset within approximately 30 minutes to two hours of ACS; suggests ischemic nerve dysfunction)

Examination findings suggestive of ACS include the following:

Pain with passive stretch of muscles in the affected compartment (early finding)
Tense compartment with a firm "wood-like" feeling
Pallor from vascular insufficiency (uncommon)
Diminished sensation
Muscle weakness (onset within approximately two to four hours of ACS)
Paralysis (late finding)

65. Non athlete heard a click at the left leg calf when he Forcefully plantar flex his left foot while the leg is fully extended. On
examination, he can't plantar flex his foot against resistance & can't walk by toe-foot walking of the affected site. What is the
injured structure?

A. Calcaneal ligament
B. Quadriceps ligament
C. Plantar ligament
D. Femoris rectus
E. Achilles tendon rupture

Answer: E?
Http://www.orthobullets.com/foot-and-ankle/7021/achilles-tendon-rupture
66. Superficial palmar arch artery of:
A. Ulnar
B. Radial "deep arch"
C. Anterior
D. Posterior
Amswer: A
Https://en.wikipedia.org/wiki/Superficial_palmar_arch

176
67. Pt present with pain of inferior heal which getting worse at morning & improve at the end of the day.
A. Plantar fasciitis
B. Calcaneal spur
C. Calcaneal fracture
D. Achilles tendinitis

Answer: A
Plantar fasciitis is the most common cause of pain on the bottom of the heel.
Although many people with plantar fasciitis have heel spurs, spurs are not the cause of plantar fasciitis pain. One out of 10 people
has heel spurs, but only 1 out of 20 people (5%) with heel spurs has foot pain. Because the spur is not the cause of plantar
fasciitis, the pain can be treated without removing the spur
Http://orthoinfo.aaos.org/topic.cfm?Topic=a00149

68. Pt with open fracture in left arm. After open reduction & internal fixation pt suddenly cant extend his forearm, wrist or hand.
Which of the following nerve affected?
A. Radial nerve in spiral groove
B. Median nerve in cubital fossa
C. Ulnar nerve on medial epicondyle
D. Median nerve in lateral epicondyle
Answer: A
- Ulnar N passed post to medial epicondyle if injured >> claw hand and sensory lost over medial aspect of the hand.
- Radial N passed anteriorly to the lateral epicondyle within the cubital fossa of the forearm if injured >> depend on the site (
if in the anterior spiral groove >> wrist drop )
Http://www.orthobullets.com/pediatrics/4008/medial-epicondylar-fractures--pediatric

The radial nerve provides motor innervation to the extensors of the forearm, wrist and fingers.
Reference: http://emedicine.medscape.com/article/1244110-clinical
Kaplan USMLE step 1 – Anatomy

Radial nerve innervates all muscles of the posterior compartments of arm and forearm (extensors). Primary action: extend MP joints,
wrist and elbow.
Radial n. Injury: mid-shaft of humerus at radial groove or lateral elbow:
Loss of forearm extensors of the wrist and MP joints (wrist drop)
Weakened supination
Sensory loss on posterior forearm and dorsum of thumb

69. What is the first thing in management of depressed fracture?


A. Discharge the patient
B. Antibiotics
C. IV fluid
D. Lateral CT scan
Answer: A
Treatment of skull fractures is primarily conservative , consists of observation to rule out any ongoing complications such as CSF
leak, seizure, or infection. Medical interventions such as anticonvulsant and antibiotic prophylaxis are not routinely given to
patients with isolated skull fractures.
Http://bestpractice.bmj.com/best-practice/monograph/398/treatment/details.html

177
70. Patient is a golf player came medial epicondylitis what to do?
A. Refer to ortho.
B. Stop playing golf.
C. Recommend surgery for the patient
D. X-ray
Answer: B
Toronto:
Rest, ice, nsaids
Use brace/strap.
Physiotherapy, stretching, and strengthening.
Corticosteroid injection
Surgery: percutaneous or open release of common tendon from epicondyle (only after 6-12 mo of conservative therapy)

71. Patient sustain a trauma to the jugular foramen & hypoglossal canal, which function will be preserved ;

A. Raising the shoulders


B. Thoracic sensation
C. Vocal cords movement
D. Taste sensation of the tongue (i forget which part ; anterior or posterior )
Answer:
•Hypoglossal canal: Hypoglossal (CN XII) Inspect tongue for signs of lateral deviation, atrophy, fasciculations, asymmetry of
movement and strength
•Jugular foramen: IX(glossopharyngeal),X(vagus)(these two control autonomic nervous system+ vocal cords) ,XI(accessory)(control
some shoulder muscles).

72. Women wearing high-heels one of them was broken and her leg goes outward what is the ligament affected
A. Sprain
B. Cacinofibuler
C. Deltoid
D. Anterior Talofibular ligament
Answer: C
Medial ligament Complex (deltoid ligament) is associated with eversion injuries and the lateral ligament complex (Anterior
Talofibular, Calcaneo fibular, Posterior Talofibular) are associated with inversion injuries.

Reference: Toronto notes

[The medial, or deltoid, ligament is stretched in an Eversion (medial) ankle sprain.]


Evidence: "deltoid ligament injury occurs with pronation (eversion) trauma leading to forced external rotation & abduction of ankle"
Reference: http://www.orthobullets.com/foot-and-ankle/7005/ankle-ligaments

73. Girl with high heels, RT heel breaks with inversion of RT foot, what’s the tendon injured (other versions: A lady wearing high
heels fell & had ankle sprain, which tendon is affected? ; Twisted ankle what is the most common ligament?)? (missing
options)
A - calculotibialis
B - anterior tibialis

Answer: Anterior talofibular ligament is stretched in an Inversion (lateral) ankle sprain


Evidence:

178
- The most common injury is ankle sprain rather in supination most common or in pronation less common. There is no tendon
rupture or tear in this injury. There is ligament injury and are lateral ligaments complex...
ATFL = Anterior talofibular lig (most common), PTFL=posterior talofibular lig, CFL=calcanofibular lig
- The most common type of ankle sprain occurs when the foot is inverted too much, affecting the lateral side of the foot. When this
type of ankle sprain happens, the outer, or lateral, ligaments are stretched too much. The anterior talofibular ligament is one of the
most commonly involved ligaments in this type of sprain. Approximately 70-85% of ankle sprains are inversion injuries.
Reference: http://www.orthobullets.com/foot-and-ankle/7005/ankle-ligaments ;
http://www.iaaf.org/download/download?Filename=5b2291a7-258c-4635-bbc0-
8657eec73740.pdf&urlslug=Chapter%2010%3A%20Specific%20injuries%20by%20anatomic%20site

74. Flat shoulder 1/3 middle arm sensation lost:


A. Ape palsy
B. Complete claw
C. Partial claw
D. Wait until 12 w
Answer: most likely C
The question has missing details, but in general, cervical cerclage is done at 12 to 14 weeks for women with a history-based
diagnosis of cervical insufficiency.
Reference: http://www.uptodate.com/contents/cervical-insufficiency

75. If you take blood sample from thigh how to prevent sciatica nerve?
A. Upper medaila
B. Lower medial
C. Upper lateral
D. Lower lateral

Http://www.medscape.com/viewarticle/551320_3

76. Unilateal knee pain , swelling , middle age i think Joint apsirate
Wbc : nl Neut : 80% Did not mention crystals :
A. RA
B. Gout
C. Septic arthritis
D. Psudogout ??

77. Patient have parotitis, pain with eating that radiate to the ear, with nerve transmit pain with eating?
A. 8
B. 9
C. 10
179
D. 7

Answer: D, http://teachmeanatomy.info/head/organs/salivary-glands/parotid/

78. Lesion in forehead, what LN will be enlarged?


A. Mastoid
B. Auriculomandibular (should be pre-auricular or submandibular)
C. Superficial cervical
D. Parotid

Answer: should be submandibular


In general, the face, scalp, and ear –> drains into occipital, retroauricular, parotid, buccal, submandibular, submental, superficial
cervical l.n.
Http://images.slideplayer.com/16/5126607/slides/slide_11.jpg
http://images.slideplayer.com/12/3440402/slides/slide_9.jpg
http://teachmeanatomy.info/neck/vessels/lymphatics/

79. Patient come with back pain when awake from sleep he had stiffness for almost 30 minutes and it's resolved he take i think
paracetamol or other thing in examination there is a para spinal muscle stiffness When you did a spine CT you found mild
lumbar stenosis what your management:
A. epidural steroid injection
B. physiotherapy
C. ibuprofen
D. bed rest

Answer:

80. Hyperextension injury developed pain in distal phalanx, tender palms, cant flex distal phalanx:
A. Rupture superfascialis tendon
B. Rupture flexor digitorum profundus tendon
C. Extra articular fracture of distal phalanx
D. Intra articular fracture of middle phalanx
Answer: b. Rupture flexor digitorum profundus tendon. American Family Physician
Http://www.aafp.org/afp/2006/0301/p810.html & http://www.aafp.org/afp/2001/0515/p1961.html

81. A female patient injured her left index. Now she is unable to flex the distal part of the finger with an associated swelling.
What is the structure most likely injured?
A. Fracture to the distal phalanx bone
B. Fracture to the proximal phalanx bone
C. Torn flexor digitorum profundus (FDP)
D. Torn flexor digitorum superfisialis (FDS)
Answer: C
FDP is responsible for flexing the distal part of the finger while the FDS is responsible for flexing the fingers at the middle.
The Hand Examination and Diagnosis by American Society for Surgery of the Hand

82. Someone had a dislocation of mandible and the doctor tried to retract the mandible inferiorly. Which muscle retract the
muscle to its normal position
180
A. Medial ptyrgoid
B. Lateral ptyrgoid
C. Masters
D. Temporlais
Answer: D
Reference: http://teachmeanatomy.info/head/muscles/mastication/

83. Picture of tibial fracture proximal peripheral perfusion normal and normal sensation the best next to investigate?
A - Angiography
B - MRI
C - Doppler US
D - X-Ray

Answer: D
Evidence:
The standard protocol is to obtain anteroposterior and lateral radiographs of the injured leg. CT scans and MRI are reserved for
more complex injuries. MRI has particular value in higher tibial fractures that may extend into the knee joint or are suspected of
involving the tibial plateau. MRI also helps delineate associated meniscal or ligamentous injuries.

84. ‫ صورة ساق مكسوره من تحت الركبه النبض موجود‬duplex is .83


What to do?
A. Angiograph
B. Venography
C. Duplex US
D. MRI
Answer: A
Angiography remains the gold-standard investigation for the further investigation and delineation of vascular injury. In most
traumatic injury settings, angiography is best performed in the operating room, with the surgeon exposing the vessel proximal to the
injury for control and expediency. Transfer to the radiology suite should be restricted to haemodynamically stable patients with
proximal or torso injuries. Angiography may be used to treat certain selected injuries, and where expertise and technical facilities
are available. Proximal control may be possible with an angioplasty catheter prior to transfer to the operating room.

85. Pt with trauma, femoral/hip fracture. Lt leg short and laterally rotated. What muscle is responsible for lateral rotation:
A- gluteus maximus
B- rectus femorus
C- gracilus
D- adductor Magnus
Answer: A

86. Pt fall on his elbow what will you see in lateral x ray (no other info)
A) Radial line anterior to acetabulum (
B) Some line bisect with acetabulum
C) Anterior fat
D) Posterior fat
Answer: D
Supracondylar Fracture: extension type most common (95-98%), occur most commonly in children aged 5 to 7, mechanism of injury:
fall on outstretched hand, posterior fat pad sign:
Lucency along the posterior distal humerus and olecranon fossa is highly suggestive of occult fracture around the elbow
Http://www.orthobullets.com/pediatrics/4007/supracondylar-fracture--pediatric

181
87. An osteochondroma patients developed a mass (or pain) in his pelvis. Histology shows cartilaginous tumor…..what is the
most likely diagnosis?
A. Ewing’s sarcoma
B. Chondroblastoma disease
C. Chondrosarcoma
Answer: C
Explanation: osteochondroma (exostosis) is the most common benign tumor. It can –rarely- transform to chondrosarcoma which is a
malignant cartilaginous tumor that commonly occurs in the spine or pelvis. Ewing’s sarcoma histology show anaplastic blue cells.
Reference First Aid USMLE step 1

Chondrosarcoma is a cancer of cartilage cells. It is the second most common primary bone cancer. This cancer is rare in people
younger than 20. After age 20, the risk of getting a chondrosarcoma goes up until about age 75. Women get this cancer as often as
men. Chondrosarcomas can develop anywhere there is cartilage such as the pelvis, leg bone or arm bone. Occasionally,
chondrosarcoma will develop in the trachea, larynx, and chest wall. Other sites are the scapula (shoulder blade), ribs, or skull.
Osteosarcoma: is the most common primary bone cancer. This cancer starts in the bone cells. It most often occurs in young people
between the ages of 10 and 30, but about 10% of osteosarcoma cases develop in people in their 60s and 70s. It is rare in middle-
aged people, and is more common in males than females. These tumors develop most often in bones of the arms, legs, or pelvis.
Ewing tumor: is the third most common primary bone cancer, and the second most common in children, adolescents, and young
adults. The most common sites for this cancer are the pelvis, the chest wall (such as the ribs or shoulder blades), and the long bones
of the legs or arms.
Link: http://www.cancer.org/cancer/bonecancer/detailedguide/bone-cancer-what-is-bone-cancer

88. Patient with history of trauma to the shoulder. He is unable to raise his hand above his head. (There’s an accompanying MRI
picture of the shoulder with an arrow pointing at a small white spot within the joint on the top part)

A. Acromioclavicular junction
B. Shoulder dislocation
C. Rotator cuff muscle tear

Answer: C

Acute trauma to the shoulder suggests joint dislocation, fracture, or tendon tear.
Rotator cuff tendonitis and other inflammatory/degenerative joint disease are the results of chronic repetitive overhead
activity.
Conventional shoulder MRI is a T2-weighted image most of the time. Hyper-intensity on T2-weighted shoulder MRI can
mean: tumors, tendinitis, tendon tears, post-traumatic edema and inflammation.
The MRI image described in this question is probably a partial thickness rotator cuff tendon tear.
Source: http://emedicine.medscape.com/article/401714-overview

89. Muscle responsible of unlocking the knee?


A. Gastrocnemius
B. Fibularis
C. Popliteus muscle
Answer: C

Reference:
Https://books.google.com.sa/books?Id=0h7ftcb02euc&pg=PA412&lpg=PA412&dq=popliteus+muscle+unlock&source=bl&ots=m0sw
f6rnme&sig=-g9Jv-
182
cwfhslndy9vjmrc8micow&hl=ar&sa=X&ved=0ahukewj08_2Py8zJAhUD_hikhyq2dei4kbdoaqgcmae#v=onepage&q=popliteus%20mus
cle%20unlock&f=false

90. Muscle which make the knee flexion from upright position is?
Https://en.wikipedia.org/wiki/Knee

- The popliteus muscle at the back of the leg unlocks the knee by rotating the femur on the tibia, allowing flexion of the knee.
- The quadriceps femorus muscle group (rectus femoris, vastus lateralis, vastus medius, and vastus intermedius) crosses the knee via
the patella and acts to extend the leg.
- The hamstrings (semitendinosus, semimembranosus, and biceps femoris) flex the knee and extend the hip (except for the short
head of the biceps femoris).

Source: Boundless. “Muscles that Cause Movement at the Knee Joint.” Boundless Anatomy and Physiology. Boundless, 26 May.
2016. Retrieved 30 May. 2016 from https://www.boundless.com/physiology/textbooks/boundless-anatomy-and-physiology-
textbook/muscular-system-10/muscles-of-the-lower-limb-107/muscles-that-cause-movement-at-the-knee-joint-579-9335/

91. What is the type of fracture that has several segments in the diaphysis?
A - compound
B - impacted
C - comminuted

Answer: C | Reference: The pathophysiology of fractures Textbook + picture

92. A patient with Tibial fracture with abnormal ABI. What will you do next?
A - MRI
B - CT
C - Angiography

Answer: C
Evidence:
- Tibial fracture with ABI/PVR abnormal, Needs vascular team involvement and urgent vascular imaging (CT angiography)
Reference: Tibial Plateau Fractures – Initial Management Guidelines:
http://www.medicine.virginia.edu/clinical/departments/orthopaedics/orthopaedic-education/residency%20programs/treatment-
protocols/Tibial_Plateau_Fractures_2013_Initial_Management_Guidelines.pdf

93. Old lady complaining of right femur pain when she goes upstairs and downstairs. Examination of the right hip: flexion,
extension and rotation are normal but abduction caused pain. What is the best investigation to do?
A - Radiological assessment.
B - Blood testing.
C - Rheumatoid factor.

Answer: A
Evidence:
(Pain while abduction of Rt hip) Most probably to be posterior hip dislocation because all movements are intact (flexion internal
rotation...) Except abduction so we need radiological views to confirm the diagnosis
Reference: Netter’s concise orthopaedic anatomy

183
94. Football player he can't raise his big toe. What muscle affected:
A - Deep muscle under foot
B - Superficial muscle under foot
C - Tibia

Answer: A
Evidence:
(Pain while raising up the big toe) Deep muscles under foot responsible for dorsiflexion of big toe most probably the diagnosis will be
turf toe, which is common in football players (it sprains to the ligaments around the big toe)
Extensor hallucis longus muscle is responsible for Dorsiflexion of the great toe
Reference: Miller’s review of orthopaedic

95. Child with fracture of forearm (x-ray shows open fracture of radius & ulna), what is the treatment? (answer not known)
A - Closed reduction
B - Open reduction
C - Surgical intervention

Answer: A
O.R.I.F with plates and screws; closed reduction with immobilization usually yields poor results for displaced forearm fractures
(EXCEPT in children) | Reference: Toronto Notes 2015
open reduction internal fixation is treatment of choice in fracture radius and ulna together
References: Prof. Mohammed Jalaat Alfaisal & http://emedicine.medscape.com/article/1239187-treatment#d10

96. Fracture of distal ulna and radius with a pic what's the immediate management
A. Debridement, irrigation and fixation
B. Close reduction and cast above the elbow
C. Close reduction and cast below the elbow
Answer: B.
Close reduction and cast above the elbow. The majority of distal radial fractures is a below elbow cast. If the ulna is involved, a sugar
tong or an above elbow cast may be considered, in order to control forearm rotation. To limit forearm pronation and supination and
provide stabilization of the fracture and allow ossification. Http://www.aafp.org/afp/2009/0901/p491.html &aofoundation

97. Blood supply of post. Leg compartment:


A - tibial
B - Common fibula
C - Superfacial fibula

Answer: A (posterior tibial artery)


Evidence:
- The posterior tibial artery is the larger of the terminal branches of the popliteal artery. It descends deep to soleus, where it can be
exposed by splitting gastrocnemius and soleus in the midline, then becomes superficial in the lower third of the leg and passes
behind the medial malleolus between the tendons of flexor digitorum longus & flexor hallucis longus.
- posterior compartment of leg has two parts (superficial and deep):
= Superficial contains: Gastrocnemius, soleus, plantaris
= Deep contains: popliteus, flexor hallucis longus, flexor digitorum longus and tibialis posterior
Reference: Review of orthopedics by Miller + Gray's anatomy

98. What is the most specific sign for osteoporosis?


A - ↑ bone density

184
B - subchondral cyst
C - compression fracture

Answer: C (compression fracture)


Evidence:
Fractures are the most dangerous aspect of osteoporosis. Debilitating acute and chronic pain in the elderly is often attributed to
fractures from osteoporosis and can lead to further disability and early mortality. These fractures may also be asymptomatic. The
most common osteoporotic fractures are of the wrist, spine, shoulder and hip. The symptoms of a vertebral collapse ("compression
fracture") are sudden back pain. Reference: https://en.wikipedia.org/wiki/Osteoporosis

99. Fracture of the distal one-third of the humeral shaft commonly associated with which type of radial nerve injury?
A - neuropraxia
B - neurotemesis
C – axontemesis

Answer: A (Neuropraxia)
Evidence: Neuropraxia is the answer if closed fracture, while neurotemesis is the answer if open fracture + radial nerve palsy
Reference: http://www.orthobullets.com/trauma/1016/humeral-shaft-fractures

100.Patient complained of progressive wrist pain since 2 months and increased in the past 1 month, give history of caesarean
delivery. On exam there is numbness and normal range of motion. What is the Rx?
A - Thumb cast
B - Whole are cast
C - Surgical decompression

Answer: A (Thumb cast)


Evidence:
- looks like pregnancy induced carpal tunnel. If that’s the case, it should resolve after delivery. Since there is no muscle wasting and
weakness, it’s not that severe
- start with conservative management in carpal tunnel syndrome, no improvement/severe consider surgery

101.Which nerve is affected in case of fibular fracture?


A - common peroneal n
B - deep peroneal
C - superficial peroneal

Answer: common peroneal nerve


References:
- https://www.nlm.nih.gov/medlineplus/ency/article/000791.htm
- http://teachmeanatomy.info/lower-limb/nerves/common-fibular-nerve/

102.Trendelenburg gait features:


A - internal rotation with adduction
B - internal rotation with ab
C - external rotation with abduction
Answer: A
Evidence: Since the Trendelenburg gait develops as a result of weakness in Abductor muscle, the affected side tend to be in adduced
position with internal rotation, based on the pictures below:

185
103.Cancer patient c/o bone pain if you suspect mets what is the best radiological test?
A. Bone scan
B. MRI
C. Positron immersion tomography.
Answer: A. (Bone scan . step up to medicine)

104.Most common site of metastasis:


A. Bone
B. Kidney
C. Breast

Answer: A
The most common sites of cancer metastasis are, in alphabetical order, the bone, liver, and lung.
Reference: http://www.cancer.gov/about-cancer/what-is-cancer/metastatic-fact-sheet#q3

105.Young man close the door on his nail, the colour of which became blue under the nail. What will you do?
A - evacuate hematoma
B - remove nail
C - reassure it well resolved by itself

Answer: C (if painful, decompression is indicated)


References: http://www.webmd.com/skin-problems-and-treatments/bleeding-under-nail?Page=2 ;
Http://emedicine.medscape.com/article/82926-overview#a3

106.Young adult playing golf, came with Hx of atrophy of thenar muscles and tenderness at medial epicondyle, first evaluate:
A - X-ray
B - CT scan
C - reassure

Answer: A or C? (Golfer’s elbow; no need for imaging initially) | Reference: http://emedicine.medscape.com/article/97217-workup


Evidence: Plain x-rays are necessary to rule out a bony supracondylar process in the distal humerus or other post traumatic or
degenerative pathology at the elbow as the cause for compression.
Reference: http://www.orthoontheweb.com/nerve_compression.asp +

186
107.Physician in the clinic tells the child to bend forward and hang his both hands freely. This test is used in detection of?
A - Rectal Prolapse
B - Sexual Abuse
C - Scoliosis

Answer: C
Reference: http://www.physio-pedia.com/Adam's_forward_bend_test

It is called ADAM test which is a screening test for scoliosis.


Http://www.orthobullets.com/spine/2053/adolescent-idiopathic-scoliosis

108.Scoliosis what radiological method will you order:


A - Xray
B - CT scan
C - MRI

Answer: A
Evidence: PA spine radiographs — Radiographs are required to confirm the diagnosis of scoliosis, evaluate the etiology (congenital,
neuromuscular, idiopathic), determine the curve pattern and measure the magnitude (Cobb angle), and to evaluate skeletal maturity
(to determine the risk for progression).
Reference: uptodate: http://www.uptodate.com/contents/adolescent-idiopathic-scoliosis-clinical-features-evaluation-and-
diagnosis?Source=machinelearning&search=scoliosis&selectedtitle=1~150&sectionrank=2&anchor=H15#H15

109.Which part of bone is firstly affected in haematogenous osteomyelitis?


A - epiphysis.
B - metaphysis.
C - diaphysis.

Answer: B | References: http://patient.info/doctor/osteomyelitis-pro +


Http://www.uptodate.com/contents/overview-of-osteomyelitis-in-adults?Source=outline_link&view=text&anchor=H1#H1

110. Pt can't do abduction at hip joint, site of injury?


A - Gluteal maximus.
B - Inferior gluteal artery.
C - Head of femur.

Answer: A

111.Fracture of the tibia, displaced transverse, mx?


A - Close redaction.
B - Open with nail.
C - Antibiotic.

Answer: B
112.Elderly having knee pain for years and he is on NSAID when needed. Now he developed Left foot oedema. What is the best
investigation?
A - Echo
B - CBC
C - Doppler

Answer: A

187
113.Patient with active osteoarthritis with knee swelling on ex there is effusion but no pain not hot with crepetus during
movement. What is the best next? (answer not known)
A - Bilateral knee x Ray and ibuprofen
B - Aspirations of fluid
C - Bone…

Answer: ?

114.Child with FOOSH. He has pain on affected arm and shoulder. X-ray shows midclavicular fracture with segments overriding.
What's the mang:
A-sling
B-surgery
C-closed reduction
Answer B
Http://www.uptodate.com/contents/clavicle-
fractures?Source=see_link&sectionname=FRACTURES+OF+THE+MIDDLE+THIRD+%28MIDSHAFT%29+OF+THE+CLAVICLE&anchor=H6
#H6

115.A thyroid-disease patient developed pain while moving her shoulder and decreased range of motion. What is the diagnosis?
A. Impingement syndrome
B. Rotator cuff tear
C. Adhesive capsulitis
Answer : C

116.Best x ray to show fracture rib


A-AP
B-PA
C- Oblique
Answer: c
45° oblique view on expiration is recommended for radiographic imaging of patients with clinical signs of fracture, e.g. Evaluation of
lower rib fractures, while 45° oblique view during fast breathing is recommended for suspected upper rib fractures.
Http://www.ncbi.nlm.nih.gov/pmc/articles/PMC3529706/

117.Trauma to the knee ABI was low next step?


A. Angio Doppler
B. Ct
C. MRI
Answer: A
If ABI in knee injury less than 0.9 the patient should undergo arterial duplex ultrasound
http://www.orthobullets.com/trauma/1043/knee-dislocation

118.X ray Pic of fibula fracture which was multiple, + other pic for the leg of the pt
And gave hx and asked what u will do also,,
A. -Discharge home
B. -Debriment internal fixation
C. -External fixation

119.(photo & x-ray of open fracture). Pt had MVA and presented to ER with this legion. After stabilizing the pt what you will do? :
A. Give oral antibiotic & discharge

188
B. Discharge home no need for further management
C. Immediate surgery for debridement & ORIF

Answer: C
Http://orthoinfo.aaos.org/topic.cfm?Topic=A00582

120.Median nerve injury cause:


A. Ape hand
B. Clow hand
C. Waiter's tip deformity
Answer: A

121.Acute knee swelling with balloted patella, what to do next


A. ESR
B. CBC
C. Arthocentesis
Answer: missing data
Http://www.aafp.org/afp/2000/0415/p2391.html

122.What is the priority in pt management :


A. Airway
B. Breathing
C. Circulation
Answer: A

123.8 year-old male with left hip externally rotated with decrease in range of motion and decrease abduction of the hip. X-ray
identical to this one. What is your diagnosis :
A. Developmental dysplasia of the hip (DDH)
B. Legg–Calvé–Perthes disease
C. Slipped capital femoral epiphysis

Answer: A
Explanation:
DDH is initially a clinical diagnosis that’s suspected when there’s a limited abduction or asymmetry between gluteal
folds. Physical signs include Barlow and Ortolani’s test. In x-ray, patients show broken shenton’s line & femoral neck
above Hilgenreiner’s line.
Legg-Calvé-Perthes disease: idiopathic avascular necrosis of femoral head that is commonly found in boys 4-10 years
old. Initially patients present with painless limp.
Slipped capital femoral epiphysis: separation of proximal femoral epiphysis through growth plate that presents with
painful limp
Reference: First aid step 2CK
124.X-ray for displaced tibial fracture with another pic of skin sinus with discharge, received iv ab and tetanus toxiod , what is ur
next management :
A. discharge on oral antibiotic
B. external fixation
C. surgical debridement with internal nail fixation

Answer: C

189
Explanation: the picture is probably is of the open fracture and since it is described as sinus then size most likely <1 cm or size 1-10
cm, respectively type 1 or type 2 in the Gustilo classification. After primary debridement, Intramedullary nailing is the best option for
Gustilo-Anderson types I, II, and III fractures.
Reference: http://emedicine.medscape.com/article/1249761-treatment#d10

125.Patient presented with distal thigh pain erythema and warmth seems inflamed but the knee is completely normal there’s
limited range of motion with tenderness which of the following would be helpful in establishing the diagnosis:

A.C-reactive protein (or ESR)


B.X ray
C.Blood culture

Answer: b
Http://emedicine.medscape.com/article/336054-overview#a6

126.Epileptic patient had an episode of seizure presented to ER with arm in adduction and internal rotation what do you expect
him to have: Hx of epi
A. Anterior dislocation
B. Posterior dislocation
C. Inferior dislocation

Answer: B
Posterior shoulder dislocation usually associated with seizures.

Http://lifeinthefastlane.com/posterior-shoulder-dislocation/

Stress fracture – READ ABOUT


Https://www.merckmanuals.com/professional/injuries-poisoning/sports-injury/stress-fractures

1. Mechanism of injury – A blow to the anterior portion of the shoulder, axial loading of an adducted and internally rotated
arm, or violent muscle contractions following a seizure or electrocution represent the most common causes of posterior
shoulder dislocation [25-27].
2. Examination – Examination reveals prominence of the posterior shoulder with flattening anteriorly. The coracoid process
appears prominent. The patient holds the arm in adduction and internal rotation and is unable to externally rotate (picture
3 and movie 1) [28].
3. Imaging studies – Radiographic evidence of a posterior shoulder dislocation on a standard anteroposterior (AP) view is
subtle and may go undetected in up to 50 percent of cases [29]. Clues to the diagnosis include the "light bulb" sign, rim sign,
and trough line sign

127.16 years old boy known case of sickle cell anemia presented to with painful right hip pain for several weeks (this was the
scenario and it was for several weeks ). What is the most likely diagnosis:
A. Avascular necrosis
B. Stills disease
C. Tumor
** osteomyelitis was not in the choices

Answer: A
A condition caused by interruption of blood supply. Remember ASEPTIC mneumonic:
Alcohol, AIDS
190
Steroids (most common), Sickle, SLE
Erlenmeyer flask (Gaucher’s)
Pancreatitis
Trauma
Idiopathic/ Infection
Caisson’s (the bends)
Http://www.orthobullets.com/recon/5006/hip-osteonecrosis

Explanation: femoral head avascular necrosis pain most commonly localized to the groin area, but it may also manifest in the
ipsilateral buttock or greater trochanteric region. Approximately 30% of all patients with SCD have hip pathology by age 30 years.
The other choice, Adult Still's disease is a rare illness in which almost all patients have high fevers, rash, and joint pain. It may lead to
long-term (chronic) arthritis.
Reference: http://emedicine.medscape.com/article/205926-clinical
Https://www.nlm.nih.gov/medlineplus/ency/article/000450.htm

128.Pt with bilateral deltoid muscle weakness and other symptom?


A. Brachial plexus neuropathy
B. Neck spasm
C. Cervical spondylopthy
Answer: The main cause could be due to cervical radiculopathy or Brachial injury (Erb’s Palsy) depending on the case given.
However, Cervical spondylosis represents the natural degenerative process of the cervical motion segment (intervertebral disc and
facets) often leads to the clinical conditions of cervical radiculopathy, cervical myelopathy, disco genic neck pain.
Cervical Radiculopathy
A clinical symptom caused by nerve root compression in the cervical spine C5 radiculopathy
Deltoid and biceps weakness
Diminished biceps reflex
Or Part of a brachial plexsus injury
Http://www.orthobullets.com/spine/2030/cervica
l-radiculopathy
Http://www.orthobullets.com/trauma/1008/brac
hial-plexus-injuries
Http://www.orthobullets.com/spine/2029/cervica
l-spondylosis

129.Acute knee swelling with balloted patella,


what to do next (ortho)
A. ESR
B. CBC
C. Arthocentesis
Answer: According to aafp, the approach of any swollen knee algorithm by order is
1- Hx
2- PE
3- Radiograph
4- Arthocentesis

130.Girl with Right knee swelling Aspirations results show (?) Wbcs and what best management:

Answer: ? (missing details of Q, missing options, answer not known)

191
Evidence:
- if the WBC 15,000 to 200,000 the diagnosis is (Septic Arthritis)
- Empiric intravenous antibiotic treatment of septic arthritis should be based on the organism found in the Gram stain of the synovial
fluid, or on the suspicion of a pathogen from the patient's clinical presentation.

References:
- all about the knee: http://www.aafp.org/afp/2000/0415/p2391.html#abstract
- management of septic arthritis according the gram stain
- http://www.aafp.org/afp/2011/0915/p653.html#sec-3

131.Case with pic: old male complaining of cervical pain abnormal gait and his wife (‫الحظت انو االشياء بتطيح من يدو كتيير‬
A. Cervical stenosis ( my answer)
B. Spinal tumor
C. Stroke

132.Peroneal nerve is branch of ?


A. Femoral
B. Posterior tibial
C. Anterior tibial
Answer:?

The common peroneal nerve is a branch of the sciatic nerve. It has 2 branched the superficial and deep peroneal nerves.

Reference: Orthobullet

133.Child with history of comminuted clavicular # due to MVA, ttt :


192
A. Arm sling
B. Close reduction
C. ORIF
Answer: C
Reference: http://www.orthobullets.com/trauma/1011/clavicle-fractures
134.Stiffness with pain in shoulders and osteopenia:
A. bursitis
B. arthritis
C. adhesive capsulitis

Answer: question and choices incomplete, but the writer put adhesive capsulitis as an answer.
Explanation: The loss of passive range of motion (ROM) is a critical element in establishing the diagnosis of a true frozen shoulder
(adhesive capsulitis). Although conditions such as subacromial bursitis, calciifc tendinitis, and partial rotator cuff tears can be
associated with significant pain and loss of active ROM, passive ROM is preserved. Therefore, patients with those conditions should
not be classified as having a frozen shoulder. Osteopenia of the humeral head may be noted as a result of disuse.
Reference: http://emedicine.medscape.com/article/1261598-overview#a5
Http://www.aafp.org/afp/2000/0601/p3291.html

Explanation: Bone scans (Nuclear scintigraphy) are useful in a wide range of diseases. A common reason to obtain a bone scan is in
the evaluation of pain, in which a bone scan can help determine whether the source of the pain is from bone pathology or from the
soft tissues. For example, a long-distance runner may have foot pain due to a fracture or a sprain. The bone scan can help determine
if a bone injury or a tendon sprain is the cause of the pain. Bone scans can also be useful in the evaluation of systemic diseases such
as cancer or nonspecific widespread bone pain.
Reference: http://emedicine.medscape.com/article/2109077-overview#a1

135.Case scenario about child fall on his toys and twisted his ankle, they mention that he could not walk and want to be held by his
parents:
A. Soft tissue swelling of the ankle
B. Spiral fracture of tibia
C. Dislocation of fibula

ANSWER: A
- Soft-tissue injuries fall into two basic categories: acute injuries and overuse injuries.
1- Acute injuries are caused by a sudden trauma, such as a fall, twist, or blow to the body. Examples of an acute injury
include sprains, strains, and contusions.
2- Overuse injuries occur gradually over time, when an athletic or other activity is repeated so often, areas of the body do
not have enough time to heal between occurrences. Tendinitis and bursitis are common soft-tissue overuse injuries.
- Http://orthoinfo.aaos.org/topic.cfm?Topic=A00111

136.Case scenario about child falling on his toys and twisting his ankle, they mentioned that he couldn't walk and want to be held
by his parents:

A. Soft tissue swelling of the ankle


B. Spiral fracture of tibia
C. Dislocation of fibula

Answer is B
Fibular dislocation is pretty rare.
Ankle sprains & strains don’t cause inability to bare weight, only pain on walking
Source: http://www.orthobullets.com/trauma/1045/tibia-shaft-fractures
193
Http://www.orthobullets.com/foot-and-ankle/7028/low-ankle-sprain

137.Temporal arthritis pt "clear 194cenario" this pt is in higher risk of:


A. CAD
B. Blindness
C. Brain tumor
Answer: B
Around 50% of patients with GCA experience visual symptoms!
Http://emedicine.medscape.com/article/332483-clinical#showall
http://www.merckmanuals.com/professional/musculoskeletal-and-connective-tissue-disorders/vasculitis/giant-cell-arteritis

138.Non athletic patient after 15 of football playing he her a click sound when he do a plantar flexion while the knee extended
after that he can not do plantar flexion against resistance or stand on the tiptoe what is the tendon rupture :
A. olecranon
B. quadriceps
C. rectus femoris

139.Patient had injury to the soles of left foot what is the first structure u can see>>
A. Tibias posterior
B. Extensor halluces lounges tendon
C. Extensor digiturum longus
Answer: a. Tibias posterior.

140.Loss of sensation of the 1st lumbrical in the dorsal side of hand


A. Radial
B. Ulnar
C. Median.
Answer: c. Median. Medscape
Http://emedicine.medscape.com/article/1285060-overview
141.Injury in 1st layer of plantar foot, which is the structure may be affected?
A- Musculocutaneous nerve
B- Abductor hallucis
C- Flexor digitorum longus
Answer: B
First Plantar Layer: Most superficial of all the layers: Muscles
- Abductor hallucis
- Flexor digitorum brevis (FDB)
- Abductor digiti minimi

Reference: http://www.orthobullets.com/foot-and-ankle/7003/layers-of-the-plantar-foot

142.Carpenter 80 yrs. Can’t raise shoulder. X-ray showing osteopenia. Diagnosis


A. Glionhumoral arthritis
B. Acromial arthritis
C. Bursitis
194
Answer: C
The patient is carpenter means doing repetitive movements.
Reference: http://emedicine.medscape.com/article/2145588-clinical

143.Child was playing with his toy, as he was walking around he tripped and twist his leg, since then he does not want to walk
and want to be carried around. What is wrong with him?
A. Fracture of the tibia
B. Fracture of the femur
C. Soft tissue swelling of the ankle
Answer: Most likely C. Question is not clear but ankle strain is more likely in this case.

144.Pt with leg pain for a while. X-ray shows periosteal elevation and onion skin. Wts the mng?
A- IV antibiotic
B- Warm comp, rest and elevation
C- Steroids
Multilayered periosteal reaction, also known as a lamellated or onion skin periosteal reaction, demonstrates multiple concentric
parallel layers of new bone adjacent to the cortex, reminiscent of the layers on an onion. The layers are thought to be the result of
periods of variable growth.
It has been associated with:
osteosarcoma
acute osteomyelitis
Ewing sarcoma
Langerhans cell histiocytosis (LCH)

145.Hyperextension of DIP will cause?


2 choice Rupture tendon muscle,
A. Flexor digotorum profondus
B. Intra articular fracture of PIJ join
C. Extra articular fracture of DIJ

146. Fracture of head and neck of fibula which artery is affected?


A. Deep something
B. Superficial something

- Superior segment of Circumflex fibular artery which comes from posterior tibial A.
- Also anterior tibial A passes medial to fibula neck.
* If the question is asking about the nerve -_-, the nerve is common peroneal nerve.

147.A patient present with painful limping, fixation of femur at 30 degrees with limited internal rotation and maybe shortening
not sure. What is the most likely diagnosis?
A. Legg–Calvé–Perthes disease
B. Slipped capital femoral epiphysis
Answer: B
Explanation Both diseases have limited abduction and internal rotation, the difference is in the type of limp and age group
Legg-Calvé-Perthes disease: idiopathic avascular necrosis of femoral head that is commonly found in boys 4-10 years old.
Initially patients present with painless limp.
195
Slipped capital femoral epiphysis: separation of proximal femoral epiphysis through growth plate that presents with painful limp
Reference First aid step 2ck

Type I Salter-Harris epiphyseal injury at proximal hip most common adolescent hip disorder. The risk factors are: male, obese (#1
factor), hypothyroid (risk of bilateral involvement). The clinical features of acute slipped capital femoral epiphysis : sudden, severe
pain with limp. The clinical features of chronic slipped capital femoral epiphysis are : groin and anterior thigh pain ,positive
Trendelenburg sign on a affected side, tender over joint capsule and restricted internal rotation, abduction, flexion (Whitman’s sign:
obligatory external rotation during passive flexion of hip)

Reference: Toronto notes

148.Femoral tumors contain tumors and calcification


A. Osteosarcoma
B. Ewing’s

Answer: ?
Merck Manual
Osteosarcoma (osteogenic sarcoma): is the 2nd most common primary bone tumor and is highly malignant. It is most common
among people aged 10 to 25, although it can occur at any age. Osteosarcoma produces malignant osteoid (immature bone) from
tumor bone cells. Osteosarcoma usually develops around the knee (distal femur more often than proximal tibia) or in other long
bones, particularly the metaphyseal-diaphyseal area, and may metastasize, usually to lung or other bone. Pain and swelling are the
usual symptoms.
Findings on imaging studies vary and may include sclerotic or lytic features. Diagnosis requires biopsy. Patients need a chest x-ray
and CT to detect lung metastases and a bone scan to detect bone metastases
Ewing’s sarcoma: of bone is a round-cell bone tumor with a peak incidence between 10 yr and 25 yr. Most tumors develop in the
extremities, but any bone may be involved. Ewing’s sarcoma tends to be extensive, sometimes involving the entire bone shaft, most
often the diaphyseal region. About 15 to 20% occur around the metaphyseal region. Pain and swelling are the most common
symptoms.
Lytic destruction, particularly a permeative infiltrating pattern without clear borders, is the most common finding on imaging, but
multiple layers of subperiosteal reactive new bone formation may give an onion-skin appearance. X-rays do not usually reveal the
full extent of bone involvement, and a large soft-tissue mass usually surrounds the affected bone. MRI better defines disease
extent, which can help guide treatment. Many other benign and malignant tumors can appear very similarly, so diagnosis is made
by biopsy. At times this type of tumor may be confused with an infection. Accurate histologic diagnosis can be accomplished with
molecular markers, including evaluation for a typical clonal chromosomal abnormality
149.A young person with pain in forearm worsening day by day, xray shows "onion peel" best investigation?
A. MRI
B. CT
Answer: A
It's Ewing sarcoma; MRI for all bone tumor or US except Osteoid osteoma with CT

150.Child sitting always in w position what the change in bone: (missing options)
A - metatarsus adductus.
B - femoral anteversion (femoral torsion)

Answer: B [femoral anteversion (femoral torsion)]


Evidence: In Toeing: metatarsus Adducts, internal tibial torsion, femoral anteversion; Out
toeing: femoral retroversion. References: VMA course +
http://www.wheelessonline.com/ortho/femoral_anteversion +
http://www.childrenshospital.org/conditions-and-treatments/conditions/femoral-anteversion/symptoms-and-causes
196
151.Old age with R hip pain when palpate the joint with normal adduction flexion and abduction best next management is
A. NASID
B. Physiotherapy

152.A child with avascular necrosis of the head of femur. What should be done?
A - surgical correction
B - keep immobile for 6 weeks
C - keep externally rotated

Answer: ? (it depends on further details in the question)


Evidence:
AVN (Avascular Necrosis) or Osteonecrosis:
Commonest Presentation: Pain (inguinal area then radiated to the buttocks and thigh).
Management:
- Nonoperative management → essentially, with analgesics and pharmacological agents,
- Restricted patient weight bearing with the use of a cane or crutches has not been shown to affect the natural history of the disease
and is useful only in controlling symptoms, poor outcome (only 15% resolved with nonoperative intervention). Although nonsurgical
treatment options like medications or using crutches can relieve pain and slow the progression of the disease, the most successful
treatment options are surgical. .
- Joint-preserving procedures → In early stage 0 to II lesions, in young active patients, core decompression is the most conservative
surgical procedure that offers the best chance at preserving the femoral head.
- Joint replacement
Reference: http://orthoinfo.aaos.org/topic.cfm?Topic=a00216

Evidence:
Until recently, most children with Perthes’ disease were treated with a plaster cast or brace, or surgery. However, it is now known
that at least half of cases heal well without any treatment, particularly children aged five and under, and milder cases
The aim of treatment is to promote the healing process and to ensure that the femoral head remains well seated in the hip socket as
it heals and remodels. Treatments advised can depend on
- child's age. Younger children (age 6 and below) have a greater potential for developing new, healthy bone.
- degree of damage to the femoral head. If more than 50% of the femoral head has been affected by necrosis, the potential for
regrowth without deformity is lower.
- stage of disease at the time your child is diagnosed. How far along the child is in the disease process.
Treatments may include ‘observation’, bed rest and crutches, a plaster cast or special leg brace, or surgery
Reference: http://www.perthes.org.uk/what-is-perthes-disease/

Explanation: patient has Legg-Calvé-Perthes disease: avascular necrosis of the proximal femoral head. Medscape: Initial therapy
includes minimal weight bearing and protection of the joint. Best practice: in patients <5 years, mobilization with monitoring. In 5-7
years, with <50% necrosis, mobilization with monitoring; if necrosis >50%, surgery is the treatment.
References: http://emedicine.medscape.com/article/1248267-treatment#d10
Http://bestpractice.bmj.com/best-practice/monograph/751/treatment.html

References: http://www.ncbi.nlm.nih.gov/pubmed/3047257 &


http://emedicine.medscape.com/article/333364-treatment

197
153.Pt had posterior hip dislocation? (no stem of Q, missing options)
A - internal rotation adduction
B - external rotation abduction

Answer: A | Reference: http://slideplayer.com/slide/4311100/

154.Best exercise for osteoporosis? (missing options)


A - Low resistance exercise and conditioning
B - Low resistance and highly repetitive weight bearing

Answer: B
Evidence: Low-impact (resistance) and weight-bearing exercises is recommended e.g. Walking/jogging
Reference: Medscape: http://emedicine.medscape.com/article/330598-overview#showall

155.Boy was playing football with barefoot and was injured in his sole, and Dr found that it is only superficial plantar injury which
structure may be affected: (missing options)
A - posterior tibialis tendon
B - adductor hallucis longus

Answer: abductor hallucis longus, there is no adductor hallucis longus | Reference: consultant

156.To Prevent fracture caused by osteoporosis in old: (missing options)


B - Vit D
C - *other medication*

Answer: (Bisphosphonate is the first-line agent) | References: http://emedicine.medscape.com/article/330598-treatment#d13 &


http://www.uptodate.com/contents/prevention-of-
osteoporosis?Source=machinelearning&search=prevention+of+osteoporosis&selectedtitle=1~150&sectionrank=1&anchor=H28#H2
8
157.Old female with lytic bone lesion, high ALP, what’s the tx? (missing options)
A - Bisphosphonate
B - selective oestrogen

Answer: A | Reference: http://emedicine.medscape.com/article/1253331-treatment#d10


158.Elderly patient with lower back pain, what is the drug cause relaxation of muscle? (missing options)
A - Diazepam
B - Clorcarpine

Answer: A (diazepam) | Reference: http://www.webmd.com/back-pain/muscle-relaxants-for-low-back-pain

159.12 y/o Obese can’t bear weight with left hip external rotation, x-ray provided? (missing options)
A - Slipped capital femoral epiphysis
B - Fracture of femoral neck

Answer: A (12~ y/o, obese, limping & thigh is externally rotated slipped capital femoral epiphysis) | Reference: Kaplan surgery

160. Which one of these is a feature of osteoporosis? (missing options)


A - Decreased bone mass.

198
B - Decreased mineralization of bone.

Answer: A

161. Femoral neck # his leg was rotated laterally which muscle responsible: (missing options)
A - Rectus femoris.
B - Gluteas maximus.

Answer: Iliopsoas muscle | Reference: Rockwood and Green's Fractures in Adults page1570

162. Boy was playing football with barefoot and was injured in his sole, and Dr found that it is only superficial plantar injury
which structure may be affected: (missing options)
A - Posterior tibialis tendon.
B - Adductor halluces longus.

Answer: flexor digitorum longus

163.Pt c/o mild neck pain then sudden he c/o electrical pain on left arm and weakness and loss of tendon reflexes ? Diagnosis ?

A- cervical disc prolapse


B- polymyalgia rhumatica

Answer: A
Explanation: spontaneous onset neck pain is More likely to be associated with cervical spondylosis. Weakness and Decreased
reflexes are a sign of radiculopathy and cervical spondolysis. Polymyalgia rheumatica doesn't cause neurological symptoms.

Reference: http://bestpractice.bmj.com/best-practice/monograph/577/diagnosis/history-and-examination.html

164.Young male, complains of pain in elbow and shoulder since (1 w or 1 month , not sure ) he can't full extend his elbow , also
mention he squash player what is diagnosis :
A. Olecranon bursitis
B. Fracture
Squash is usually associated with tennis elbow, the question has missing details. If there's a direct trauma to the elbow it might be
olecranon bursitis.
165.Function of ACL
A. Medial rotation of tibia in relation to femur
B. Prevents anterior (forward) movement of the tibia off of the femur

ANSWER: B
- ACL attaches to anterior aspect of the tibia and courses superiorly, posteriorly, and laterally to attach to the lateral condyle
of the femur
- ACL prevents Anterior Displacement of the tibia under the femur
- Tension of the ACL is greater when the knee is extended and resists hyperextension
- It is weaker than the Posterior Cruciate Ligament
- KAPLAN USMLE ANATOMY

199
166.After fracture which one will be elevated?
A. CK
B. ALP
CK is found in the mitochondria and cytoplasm of skeletal muscle (predominantly), cardiac muscle, brain, and other visceral tissues.
The 2 subunits can form 3 isozymes: CK-MM, CK-MB, and CK-BB. Skeletal muscle, myocardium, and neuronal tissue are the main
sources of CK-MM, CK-MB, and CK-BB, respectively.
Increased CK is predominantly used to diagnose neuromuscular diseases and acute myocardial infarction. Neuromuscular disorders
include myopathies, muscular dystrophy, rhabdomyolysis, drug-induced myopathies, neuroleptic malignant syndrome, malignant
hyperthermia, and periodic paralyses.
Http://emedicine.medscape.com/article/2074023-overview#a2
Alkaline phosphatase is abnormal in Cholestasis, hepatocellular enzyme induction, canalicular injury, children during bone growth,
bone disease, pregnancy (placenta origin)
Http://www.medscape.com/viewarticle/710045_3

Explanation: Orthopedic trauma such as fractures and compartment syndrome can cause rhabdomyolysis by direct injury and
disruption of muscle tissue. This causes leakage of cellular contents such as CK and potassium.
Reference http://emedicine.medscape.com/article/1007814-overview#a1

167.After fracture which u must do for evaluate bone density?


A. X ray
B. Dexa
Http://www.uptodate.com/contents/bone-density-testing-beyond-the-basics

168.What the first management of risk factor?


A. Dexa
B. Biphosphate.
Bone mineral density (BMD) measurement is recommended in the following patients:
Women age 65 years and older and men age 70 years and older, regardless of clinical risk factors
Postmenopausal women and men above age 50–69, based on risk factor profile
Postmenopausal women and men age 50 and older who have had an adult-age fracture, to diagnose and determine the degree of
osteoporosis
The NOF recommends that pharmacologic therapy should be reserved for postmenopausal women and men aged 50 years or older
who present with the following:
A hip or vertebral fracture (vertebral fractures may be clinical or morphometric [eg, identified on a radiograph alone])

200
T-score of –2.5 or less at the femoral neck or spine after appropriate evaluation to exclude secondary causes
Low bone mass (T-score between –1.0 and –2.5 at the femoral neck or spine) and a 10-year probability of a hip fracture of 3% or
greater or a 10-year probability of a major osteoporosis-related fracture of 20% or greater, based on the US-adapted WHO
algorithm for calculating fracture risk ( FRAX)
Http://emedicine.medscape.com/article/330598-overview

169.Strongest ligament of hip prevent injury? (Prevent overextension of the hip)


A) Iliofemoral
B) Ischiofemoral
Answer: A
The iliofemoral ligament is not only stronger than the two other ligaments of the hip joint, the ischiofemoral and the pubofemoral,
but also the strongest ligament in the human body
Https://en.wikipedia.org/wiki/Iliofemoral_ligament

Evidence: Iliofemoral ligament: Located anteriorly. It originates from the ilium, immediately inferior to the anterior inferior iliac
spine. The ligament attaches to the intertrochanteric line in two places, giving the ligament a Y shaped appearance. It prevents
hyperextension of the hip joint during standing by screwing the femoral head into the acetabulum.
Reference: http://teachmeanatomy.info/lower-limb/joints/the-hip-joint/

170.Young male was stabbed in his back can't (abduct or adduct) his right thigh what is the affected muscle?
A. Magnus
B. Longus

Http://teachmeanatomy.info/lower-limb/muscles/thigh/medial-compartment/

171.Surgeon did suprarenal (procedure?), What he will hit anteriorly to it?


A. Kidney
B. Tail of pancreas

172.Muscle causes ankle dorsiflexion?


A) Tibialis anterior muscle
B) Gastrocnemius muscle
Answer: A
The tibialis anterior muscle is located alongside the lateral surface of the tibia. It is the strongest dorsiflexor of the foot. To test
the power of the tibialis anterior, the patient can be asked to stand on their heels (innervated by deep peroneal nerve).
Gastrocnemius muscle Actions: It plantar flexes at the ankle joint, and because it crosses the knee, it is a flexor there (Innervated by
Tibial nerve).
Http://teachmeanatomy.info/lower-limb/muscles/leg/anterior-compartment/

173.Patient complain of lower back pain after lifting heavy objects !


- The vast majority of people with low back pain improve within 4 to 6 weeks without treatment or with simple measures that
can be performed at home.
- Patients who are treated for acute back pain with bed rest have more pain and slower recovery than ambulatory patients.
-Therapy should focus on temporary symptomatic relief, to maximize patient comfort. We suggest a trial of short-term
treatment with either an NSAID or acetaminophen .

201
Reference: http://www.uptodate.com/contents/treatment-of-acute-low-back-
pain?Source=outline_link&view=text&anchor=H25#H25

174.Patient MVA and come with fracture of femur , tibia and fibula what is your action
A. Refer to orthopedic
Answer: supportive then refer
http://www.ncbi.nlm.nih.gov/pmc/articles/PMC3524792/

175.Patient has winging of the scapula. Which part of the trunk is affected?
Answer: long thoracic nerve
Explanation: it innervates serratus anterior which connects scapula to thoracic cage. Can be injured in mastectomy leading to
winging of scapula and lymphedema.
Reference first aid usmle step 1

176.Neck injury and cannot abduct arm > axillary


A. Shoulder dislocation
http://patient.info/doctor/shoulder-dislocation

177.A 3 year old girl with a typical history of elbow dislocation, which of the fallowing ligaments is affected?!
(No choices listed)

178.Child with painless limping. (missing options, answer not known)


A. Perthes disease

Answer: ?
Evidence:
Differential diagnosis of painless limping in children:
Developmental dysplasia of the hip
Neuromuscular disease (Cerebral palsy and muscular dystrophy)
Lower limb length discrepancy
Perthes disease: Typically present with painless limp, but may be associated with groin or anterior thigh pain (It become painful as it
progresses)
References: http://www.medscape.com/viewarticle/490135_4 ; http://www.aafp.org/afp/2000/0215/p1011.html

179.A male patient running 20 km complaining of upper leg pain. (missing options)
A - stress fracture

Answer: A | Reference: http://orthoinfo.aaos.org/topic.cfm?Topic=a00112

180.Athlete presented with severe painful plantar flexion which prevents him from raising the foot. Which ligament is affected?
A - Plantar fascia (Plantar fasciitis) (missing options)

Answer: A
Evidence:
The ligament affected depends on the mechanism of the injury:
Anterior Talofibular Ligament (ATFL) Primary restraint to inversion in plantar flexion

202
Calcaneal Fibular Ligament (CFL) Primary restrain to inversion in neutral or dorsiflexed position
Deltoid Ligament injury occurs with pronation (eversion) trauma leading to forced external rotation and abduction of ankle
Reference: http://www.orthobullets.com/foot-and-ankle/7005/ankle-ligaments

181.Septic arthritis organism? (no stem of Q, missing options)


A - Staph aureus

Answer: A
Evidence:
Staphylococcus aureus is the cause of the vast majority of cases of acute bacterial arthritis in adults and in children ≥ 2 years.
Neisseria gonorrhoeae is the most common pathogen (75% of cases) among younger sexually active individuals.
Reference: http://emedicine.medscape.com/article/236299-overview

182.Constrictor with repeated hand use: (missing options)


A - Lateral epicondylitis

Answer: A
Evidence:
Lateral epicondylitis (tennis elbow): Repetitive extension (backhand shots) or idiopathic- pain near lateral epicondyle.
Medial epicondylitis (golfer’s elbow): Repetitive flexion (forehand shots) or idiopathic - pain near medial epicondyle.
Reference: http://orthoinfo.aaos.org/topic.cfm?Topic=a00068

183.A man arrived to ER after MVA the neck of the femur was fractured. Which of the following may happen in sequence?

Answer: ? (missing options, answer not known)


Possible answer Injury of sciatic nerve; blood supply to femoral head—avascular necrosis (AVN)
Reference: Surgical recall 6th edition
184.Female patient with vertebral compression fracture. Which one of the following will prevent osteoporosis? (missing options)
A - Vit D supplement

Answer: A
Evidence:
Some of the most important treatments for preventing osteoporosis include diet, exercise, and not smoking, Calcium intake,
Vitamin D intake
Who needs treatment with a medication? — People with the highest risk of fracture are the ones most likely to benefit from drug
therapy. In the United States, the National Osteoporosis Foundation (NOF) recommends use of a medication to treat
postmenopausal women (and men ≥50 years) with a history of hip or vertebral fracture or with osteoporosis (T-score ≤-2.5).
Reference: http://www.uptodate.com/contents/osteoporosis-prevention-and-treatment-beyond-the-basics

185.A patient presented with Trendelenburg gait. Which muscle is responsible? (After MVA the patient walk toward the
unaffected side)
a. Gluteus medius (you need to know which site)

Explanation: A Trendelenburg gait, in which there is weakness of the hip abductors (gluteus medius muscle), is characterized by
trunk shift over the affected hip. Trendelenburg gait is caused by a neuromuscular weakness caused by a damage or a neuronal
injury of the superior gluteal nerve.
Reference: http://www.physio-pedia.com/Trendelenburg_Gait

203
186.Function of anterior forearm muscle: (missing options, answer not known)

Answer: ?
Evidence:
- Superficial group: 5 muscles: flexor carpiradialis, Flexor carpiulnaris, flexor digitorum superficialis, palmaris longus & pronator teres
- Deep group: 3 muscles; flexor digitorum profundus, flexor pollicis longus and pronator quadratus.
- All responsible for flexion and pronation
Reference: Zuckerman handbook of fractures

187.Type of bone hunger. (missing options, answer not known)

Answer: ? There are no types!


Evidence: Hungry bone refers to severe and prolonged hypocalcaemia which occurs after parathyroidectomy or less commonly
thyroidectomy despite normal or even elevated level of parathyroid hormone (PTH). The fall in serum calcium is primarily due to
functional or relative hypoparathyroidism leading to increase calcium influx of Ca+ into bone in patient without end-stage renal
disease Reference: uptodate and Toronto Notes

188.(Long scenario) Patient with low back pain, loss of sensation, weakness and loss of reflexes, what will do next?

Answer: ? (missing options, answer not known) Urgent MRI possible answer
Evidence:
The most common indication for the use of imaging procedures (MRI or CT) is the clinical setting of Low back pain complicated by
radiating pain (radiculopathy, sciatica), as well as in cauda equina syndrome (bilateral leg weakness, urinary retention, saddle
anesthesia), neurogenic claudication and/or spinal stenosis. MRI of the lumbar spine has become the initial imaging modality of
choice in complicated LBP, displacing myelography and CT in recent years.
Reference: http://www.guideline.gov/content.aspx?Id=35145
Evidence:
- first-line: NSAID, physical therapy, steroid injection
- persists for 3-6 mo or severe weakness/autonimc dysfunction surgery
Reference: Orthobullet: http://www.orthobullets.com/spine/2037/lumbar-spinal-stenosis

189.Boutonnière deformity (BD)? (no stem of Q, missing options, answer not known)
Answer: ?
Evidence: can manifest itself acutely after trauma, but most bds are found weeks
following the injury or as the result of progressive arthritis. The proximal interphalangeal
(PIP) joint of the finger is flexed, and the distal interphalangeal (DIP) joint is
hyperextended. Reference: Medscape

190.Osteoporosis t score: (missing options)


A - > - 2.5 osteoprosis

Answer: > - 2.5 | Reference: 3rd edition UQU last touch Q 61

191.Q about old female with recurrent fracture: (no stem of Q, missing options)
A. Oestrogen Deficiency

204
Answer: oestrogen improves calcium absorption and reduces the amount of calcium lost in urine, so deficiency of oestrogen will
lead to osteoporosis. An osteoporosis-related fracture is at high risk of recurrent fractures.

192.Tx of medial epicondylitis

Answer: conservative physiotherapy | Reference: miller

193.Pt doesn’t complain of anything & has sudden knee swelling. What’s the best
thing to do? (missing options)

Answer: X-ray
Evidence:
- acute knee swelling warrants x-ray
- if traumatic/sport injury Ottawa knee rule to decide if x-ray is needed
- Ottawa knee rule for trauma x-ray: > 55 yrs old, inability to flex 90°, inability to bear weight, tenderness
Reference: http://www.medscape.org/viewarticle/714758

194.Mechanism of Osgood Schlatter disease: (missing options)

Answer: osgood–schlatter disease or syndrome (tibial tubercle apophyseal traction injury and epiphysitis of the tibular tubercle) is
an irritation of the patellar ligament at the tibial tuberosity.
- It is characterized by painful lumps just below the knee and is most often seen in young adolescents.
- Risk factors include excess weight and overzealous conditioning (running and jumping).
- Diagnosis is made clinically
- Treatment is conservative with RICE (rest, ice, compression, and elevation) and, if required, acetaminophen.
Reference: http://www.orthobullets.com/sports/3029/osgood-schlatters-disease-tibial-tubercle-apophysitis

o Osgood-Schlatter disease:
o MOI: Overuse apophysitis of the tibial tubercle. Causes localized pain, especially with quadriceps contraction, in active
young boys.
o Treatment: Decrease activity for 2–3 months or until asymptomatic. A neoprene brace may provide symptomatic relief.

195.Muscle that extends the knee? (missing options)

Answer: The main muscle for extension is the quadriceps femoris, which is the most important muscle in stabilizing the knee joint.
The quadriceps is made up of the vastus medialis and lateralis, rectus femoris, and vastus intermedius.
Reference: http://emedicine.medscape.com/article/1898986-overview#a2

196.Chronic Gout? (no stem


of Q, missing options)

Answer: allopurinol
Evidence: picture
Reference: Toronto note

205
197.Patient presented with knee swelling and pain they did x ray and aspiration and found negative birefringence needle like.
What are you going to discharge the patient with? (missing options, answer not known)
A - Allopurinol

Answer: A? (NSAID/steroid is used for acute flares, allopurinol is not used in the acute setting)
Reference: picture

198.Pseudogout type of crystals? (no stem of


Q, missing options)

Answer: calcium pyrophysphate crystal


Evidence: picture
Reference: Toronto note

199.Signs of osteoporosis on X-ray? (missing


options)

Answer: joint space narrowing, osteophytes,


subchondral sclerosis, and subchondral cysts
Reference:
http://emedicine.medscape.com/article/330598-
workup#c9

200.Carpal tunnel syndrome vs. Thoracic outlet obstruction? (no stem of Q, missing options)

Answer: ?
Evidence:
o Carpal tunnel syndrome: Reference: Step 2 CK page 221
Entrapment of the median nerve at wrist caused by decrease space of the carpal tunnel leading to parasthesia, pain, paralysis.
Causes: overuse of wrist flexors, associated with DM, thyroid dysfunction, pregnant, middle aged women.

o Thoracic outlet syndromes (TOS):


a constellation of signs and symptoms arising from compression of the upper extremity neurovascular bundle by various
structures in the area just above the first rib and behind the clavicle, typically within the confined space of the thoracic outlet.
Causes: rib anomalies, muscular anomalies, or a result of injury. Cervical ribs predispose the patient to TOS after
hyperextension-flexion (whiplash) injury. The absence of a rib anomaly makes the diagnosis of arterial TOS less likely. Many
patients with nerve TOS have a prior history of neck trauma or repetitive occupational physical stress. Similarly, venous TOS is
highly associated with repetitive movements, particularly with repetitive overhead upper extremity movements.
Reference: uptodate: http://www.uptodate.com/contents/overview-of-thoracic-outlet-
syndromes?Source=search_result&search=thoracic+outlet+syndrome&selectedtitle=1%7E36

206
201.Retired farmer recently he develops pain in the left arm, which with time progressed until he can't sleep on his left side, by
examination found to have severe decrease in motion, what does he have? (missing options, answer not known)
A - osteoporosis of the bone
B - spondylitis (arthritis that affects the spine only)

Answer: ? (can be rotator cuff injury, bursitis, …)


202.Male pt complains of weakness in flextion of both Rt knee and Rt hip which muscle is affected? (missing options)
A - Sartorius

Answer: A | Reference: http://www.orthobullets.com/anatomy/10055/sartorius

203.Active osteoarthritis in knee pain best exercise: (missing options)


A - Quadriceps Ms strengthen.
Answer: A (Quadriceps and Hamstring Ms lengthening + strengthening)
References: http://emedicine.medscape.com/article/330487-treatment#d10 & http://www.medscape.com/viewarticle/510997_2

& http://emedicine.medscape.com/article/327330-clinical#b4

204.Tx of de Queverian syndrome? is a tenosynovitis of the sheath or tunnel that surrounds two tendons that control movement
of the thumb))
Answer:
Evidence:
Usually to start with conservative interventions, which include a forearm-based thumb spica splint with the interphalangeal joint
free along with short-term nonsteroidal anti-inflammatory drugs (nsaids). We suggest a local glucocorticoid injection for patients
whose symptoms have not resolved with conservative management. Most patients recover with this intervention.
Patients who present with severe symptoms may benefit from a glucocorticoid injection at the initial presentation. Surgical release is
generally reserved for patients who have not improved with conservative therapy and one or two glucocorticoid injections.
Reference: http://www.uptodate.com/contents/de-quervain-
tendinopathy?Source=outline_link&view=text&anchor=H660498#H660498

Forearm-based thumb spica splint with the interphalangeal joint free as well as a concurrent trial of nonsteroidal antiinflammatory
drugs (NSAIDS) for pain relief
If persistante local glucocorticoid injection
For patients with persistent symptoms despite splinting and one or two glucocorticoid injections, surgical therapy may help relieve
symptoms

205.The most common cause of olecranon bursitis is? (missing options)


A - Repeated injury

Answer: A (it is called student’s elbow; repeated injury like leaning on hard surfaces e.g. Tables)
Reference: http://www.nhs.uk/Conditions/Bursitis/Pages/Causes.aspx

206.Pt with +ve sign of Finkelstein test what is your management? (missing options, answer not known)

Answer: Most likely is thumb splint +NSAID (conservative tx) (+ve Finkelstein sign in case of De Quervain's tenosynovitis)
Reference: http://orthoinfo.aaos.org/topic.cfm?Topic=a00007

207.Patient with loss of shoulder passive and active movement: (no stem of Q, missing options)

207
Answer: adhesive capsulitis

208. Soldier walks 1000 miles developed pain on foot: (no stem of Q, missing options)

Answer: Spring ligament


209.Male with lumbar and femoral fractures. Osteoporosis on xray. Alkaline phosphates level abnormal but the rest is fine.
What’s the most likely cause? (missing options, answer not known)

Answer: ?

210.Pt complaining of lower back pain after lifting heavy objects


Answer:
Muscle strain

211.Knee reflex?
Answer: L 3-4 but mainly L4

212.Patient complaining of pain on distal palmar aspect of finger and ask about blood supply?
A. Deep and superficial Palmar arch

213.Shingles at umbilicus level what is the dermatome;


A. 10

214.Patient was complaining of leg pain after running and relax after stretch of the muscle, the problem in which muscle ?
A. Soleus
Answer: Gastrocnemius

208
215.Case of cervical disc present with numbness, shoulder pain and stiffness.........

216.Patellofemoral pain syndrome


Http://emedicine.medscape.com/article/308471-overview

217.Patient with neck pain and occipital headache, no history of trauma, there is a limitation in neck movement. On examination,
there is weakness in the upper shoulder, what is the diagnosis?
A. Cervical spondylosis

ANSWER: A
- Sign and Symptoms: Neck Pain is often accompanied by stiffness, with radiation into the shoulders or occiput "Headache"
which may be chronic or episodic
- In cervical spondylotic myelopathy, the most typical examination findings are suggestive of upper motor dysfunction,
including hyperactive deep tendon reflexes, ankle and/or patellar clonus, spasticity (especially of the lower extremities), the
Babinski sign, and the Hoffman sign.
- Http://emedicine.medscape.com/article/1144952-clinical#b4

218.Case scenario of young adult with avascular necrosis of the head of humerus, what's the best treatment?

Answer: best treatment option depends on radiograph staging.


Stage I & II: core decompression
Stage III, IV and V: hemiarthroplasty vs. Total shoulder replacement
Source: http://www.healio.com/orthopedics/curbside-consultation/%7b267d23fd-dc67-4212-a421-
6fd1a8c48b62%7D/what-are-the-indications

219.What’s the nerve responsible for elevation of arms?

Answer: axillary nerve


Axillary nerve supplies the deltoid muscle which is the main abductor of the arm (abducts arm beyond the initial 15° done
by supraspinatus muscle; supraspinatus is supplied by suprascapular nerve)

220.Obturator nerve injured which muscle affected?


A. Adductor muscle (longus Magnus brevis gracilis

The obturator nerve is responsible for the sensory innervation of the skin of the medial aspect of the thigh.

209
[2]
It is also responsible for the motor innervation of the adductor muscles of the lower extremity (external obturator. adductor
longus, adductor brevis, adductor magnus, gracilis) and the pectineus (inconstant). It is, notably, not responsible for the innervation
of the obturator internus, despite the similarity in name.
+
Http://teachmeanatomy.info/lower-limb/nerves/obturator-nerve/

221.Flat foot I think want the type of ligament??


A. Posterior Tibial Tendon
Acquired Flatfoot: There are numerous causes of acquired adult flatfoot, including fracture or dislocation, tendon laceration, tarsal
coalition, arthritis, neuroarthropathy, neurologic weakness, and iatrogenic causes. The most common cause of acquired adult
flatfoot is posterior tibial tendon dysfunction
Http://emedicine.medscape.com/article/1235600-overview#a8

222.Patient present with Hip and shoulder pain ESR high (polymyalgia rheumatic case ) in addition to symptoms what else can be
?? (proximal muscle tenseness)

Polymyalgia rheumatica (PMR) is a relatively common chronic inflammatory condition of unknown etiology that affects elderly
individuals.
Several diagnostic criteria for PMR exist. One set of diagnostic criteria is as follows:

Age of onset 50 years or older


Erythrocyte sedimentation rate ≥40 mm/h
Pain persisting for ≥1 month and involving 2 of the following areas: neck, shoulders, and pelvic girdle
Absence of other diseases capable of causing the same musculoskeletal symptoms
Morning stiffness lasting ≥1 hour
Rapid response to prednisone (≤20 mg)
Approximately 15% of patients with PMR develop giant cell arteritis (GCA), and 40-50% of patients with GCA have associated PMR.
Http://emedicine.medscape.com/article/330815-clinical

223.Osteoporotic Patient on medication that work by atp-analouge.

Answer: (Etidronate).
There are two classes of bisphosphonate: the N-containing and non-N-containing bisphosphonates. The two types of
bisphosphonates work differently in killing osteoclast cells.
Non-N-containing bisphosphonates: Etidronate, Clodronate, Tiludronate
The non-nitrogenous bisphosphonates (disphosphonates) are metabolised in the cell to compounds that replace the terminal
pyrophosphate moiety of ATP, forming a non-functional molecule that competes with adenosine triphosphate (ATP) in the cellular
energy metabolism.
Https://en.m.wikipedia.org/wiki/Bisphosphonate

224.2pic for hips limbs one painful another painless with progress painful (perth,hip dysplasia ,osteomyelitis) all same choice
A. Metaphyseal displaced.
B. Avascular necrosis of head of femur.
C. Fracture head of femur.

Answer: ?
- Legg-Calvé-Perthes disease (LCPD) is avascular necrosis of the proximal femoral head resulting from compromise of the tenuous
blood supply to this area. LCPD usually occurs in children aged 4-10 years. The disease has an insidious onset and may occur after an

210
injury to the hip. In the vast majority of instances, the disorder is unilateral. The earliest sign of LCPD is an intermittent limp
(abductor lurch), especially after exertion, with mild or intermittent pain in the anterior part of the thigh.
Http://emedicine.medscape.com/article/1248267-overview#a8

225.Internal iliac artery injury

Anatomy: The internal iliac artery supplies the walls and viscera of the pelvis, the buttock, the reproductive organs, and the medial
compartment of the thigh. The vesicular branches of the internal iliac arteries supply the bladder. It is a short, thick vessel, smaller
than the external iliac artery, and about 3 to 4 cm in length.
Https://en.m.wikipedia.org/wiki/Internal_iliac_artery

Most major iliac artery injuries are due to penetrating trauma. Intraoperatively, patients with iliac artery injuries present with active
pelvic bleeding or with a retro- peritoneal hematoma in the lateral aspect of the pelvis. Pelvic fractures associated with severe blunt
trauma may cause disruption of multiple branches of the internal iliac arteries and veins. This may result in fatal hemorrhage and
must always be kept in mind as a source of occult major bleeding.
Http://www.atcs.jp/pdf/2003_9_5/337.pdf

226.Lytic femur lesion & osteoporosis skull?


Answer: Paget disease
It is a chronic disorder of the adult skeleton in which bone turnover is accelerated in localized areas. Normal matrix is replaced with
softened and enlarged bone.
Sx: There are usually no symptoms for a prolonged period. If symptoms occur, they develop insidiously, with pain, stiffness, fatigue,
and bone deformity.
Signs: skull enlargement bitemporally and frontally (frontal bossing); dilated scalp veins; nerve deafness in one or both ears or
vertigo; headaches; angioid streaks in the fundus of the eye; a short kyphotic trunk with simian appearance; hobbling gait; and
anterolateral angulation (bowing) of the thigh, leg, or humerus, often with warmth and tenderness. Deformities may develop from
bowing of the long bones or osteoarthritis. Pathologic fractures may be the presenting manifestation.
Characteristic x-ray finding:
- Increased bone sclerosis
- Abnormal architecture with coarse cortical trabeculation or cortical thickening
- Bowing
- Bone enlargement

227.Lower back pain radiant to thighs what the investigation


In acute sciatica the diagnosis is based on history taking and physical examination and treatment is conservative (non-surgical).
Imaging may be indicated at this stage only if there are indications or “red flags” that the sciatica may be caused by underlying
disease (infections, malignancies) rather than disc herniation.
Although some authors favor magnetic resonance imaging above other imaging techniques because computed tomography has a
9 10
higher radiation dose or because soft tissues are better visualised, evidence shows that both are equally accurate at diagnosing
11
lumbar disc herniation. Radiography for the diagnosis of lumbar disc herniation is not recommended because discs cannot be
visualized by x rays.

228.Most significant risk factor for OP?


Strong Risk factors:
Prior fragility fracture

211
Female gender
White ancestry
Older age (>50 years for women and >65 years for men)
Low BMI
Fhx of maternal hip fracture
Loss of height
Postmenopause
Secondary amenorrhoea
Primary hypogonadism
Smoking
Excessive alcohol use
Prolonged immobilisation
Low calcium intake
Vitamin D deficiency
Glucocorticoid excess
Corticosteroid use
BMJ http://bestpractice.bmj.com/best-practice/monograph/85/diagnosis.html

229.Tx of osteoporosis
A. Calcium, bisphosphonates, vitamin D. Step up to medicine

230.Patient was involved in an Accident lost lateral rotation of lower limb Which muscle are affected?
A. Gluteus maximus, Gluteus medius (anterior fibres laterally rotate),Piriformis, Obturator externus, Obturator internus,
Gamellus superior, Gamellus inferior and Quadratus femoris. Anatomyzone
http://anatomyzone.com/3d_atlas/musculoskeletal/lower-limb/hip-lateral-rotators/

231.Patient (long scenario) has low back pain. Investigations revealed fracture. He also has high temperature and night sweats.
What is the most likely diagnosis?
Answer: TB
Reference:
Pott disease (spinal TB): the most common symptom is local pain. In a classic text, the description of the patient with spinal TB
reads: "The muscle spasm, which extends beyond the diseased area, sometimes produces the well-known erect posture and
'aldermanic' gait. Constitutional symptoms, fever, and weight loss are present in less than 40 percent of cases. The most important
complication of spinal tuberculosis is cord compression during the active phase of the infection resulting in paraplegia (Pott's
paraplegia)
Http://cursoenarm.net/UPTODATE/contents/mobipreview.htm?9/62/10208#H6

232.A patient complains of gradual neck pain and loss of side-to-side movement. Neck x-ray showed osteophytes and narrowed
joint space. What is the most likely diagnosis?
Answer: Cervical spondylosis
Http://radiopaedia.org/articles/cervical-degenerative-spondylosis-grading
Http://emedicine.medscape.com/article/306036-clinical

233.Patient complaining of elbow pain that is radiating down and tenderness with dorsiflexion of wrist. Diagnosis?
Answer: Lateral epicondylitis (Overuse injury)

212
234.Pregnant after delivery, with epidural anesthesia she started to complain of Loss of sensation at medial side of the thigh,,
which nerve affected?
A. Obturator Nerve

235.Young patient came with effusion, tenderness in the knee, febrile ,next step?
" there was 5 Qs similar to this q , hip effusion " septic arthritis"
DIAGNOSIS — The definitive diagnostic test is identification of bacteria in the synovial fluid. In the setting of suspected joint
infection, synovial fluid aspiration should be performed (prior to administration of antibiotics); fluid should be sent for Gram stain
and culture, leukocyte count with differential, and assessment for crystals
236.Pain in the midline of the plantar foot with walking?

Pain in the midfoot can occur along the medial arch. The navicular bone serves as a keystone for arch stability, and any pain directly
over the navicular raises the possibility of serious injury. Such injuries include navicular stress fractures, tendinopathy of the
posterior tibialis tendon insertion onto the navicular, traumatic separation of an accessory navicular, and partial or complete tears of
the attachment of the plantar calcaneonavicular (spring) ligament.

Pain over the navicular with an unusual bony prominence suggests a possible accessory navicular, present in greater than 20 percent
of patients. Pain over the plantar surface of the arch can arise from an acute strain or longitudinal arch collapse.

A more common location for midfoot pain is over the dorsum of the articulation of the first tarsometatarsal (Lisfranc) joint, and
sometimes such pain extends to the articulations of the second and third tarsometatarsal joints as well. Injuries to the
tarsometatarsal joints are often referred to as Lisfranc injuries. When pain occurs along the dorsum of the Lisfranc joint, particularly
following acute trauma, a serious injury such as a fracture or fracture dislocation may have occurred. Diagnosis is made by
radiograph. Ganglions can develop at the dorsal midfoot and cause more localized pain.

The lateral midfoot includes the cuboid and the base of the fifth MT. Pain at the base of the fifth metatarsal is common, but pain
over the cuboid less so. Pain in the lateral midfoot caused by instability of the ligaments that stabilize the cuboid may only be
elicited by dynamic assessment of the midfoot articulations. The examiner can grasp the base of the fifth MT with one hand and the
cuboid with the other and see if excessive motion occurs with vertical displacement of the fifth MT on the affected but not
unaffected side. Tenderness at the base of the fifth MT suggests injury and warrants imaging with plain radiographs, as these injuries
can be difficult to diagnose and manage.

237.Which tendon medial to Dorsalis pedis artery?


Answer: extensor hallucis longus tendon
The dorsalis pedis artery pulse can be palpated readily lateral to the extensor hallucis longus tendon (or medially to the extensor
digitorum longus tendon) on the dorsal surface of the foot, distal to the dorsal most prominence of the navicular bone which
serves as a reliable landmark for palpation

213
Incomplete Questions

1. Skull fracture? (missing details of Q, no stem of Q, missing options, answer not known)
- C-spine.

Answer: ?

2. 7 years old presented with back pain. Had a similar attach 1 year ago? What is the diagnosis? (missing options, no answer)
A. Ancklyosing spondulitis.

Answer: ?

3. Numbness of the lateral side of hands and fingers in computer programmer female that is confirmed by phalen test , in
which position would u splint the hand :
Dorsiflexion

Explanation: Wrist splints with the wrist joint in neutral or slight extension (to be worn at nighttime for a minimum of 3-4 wk) have
some evidence for efficacy. Since choices are incomplete. If neutral is one of the choices, then it is the most likely answer. If not,
then dorsiflexion.
Reference: http://emedicine.medscape.com/article/327330-treatment

4. Read about when to do bone sciantography : to determine wether the pain from the bone or from its surrounding tissue .

5. Bisphosphonate which drug make boneclast ??

6. Old women came for check up and said decrease calcium in food and consider she think has risk factor of osteoporosis

7. Five years limping with pain over five months no Hx of trauma or fever?

8. Many Q about bisphosphonate may be more than 4. Read about it spinal osteoporosis in x-ray show Erosion spine

9. Between symphysis pubis and inguinal. What artery inferior?

A. Inferior mesenteric?
Not sure

10. Nerve between heads of pronator teres muscle? Median nerve

214
Anesthesia

215
1. In epidural anesthesia the anesthesiologist hit just lateral to spinal processes which structure he will injure/hit first?
A. ligamentum flavum
B. Posterior longitudinal ligament
C. Anterior longitudinal ligament
D. interspinous ligament.

Answer: A
http://www.frca.co.uk/article.aspx?articleid=100361

2. What is the fasting duration for non-breast milk?


A. 8 hours
B. 6 hours
C. 4 hours
D. 2 hours

Answer: B
8 hours (solid)
6 hours (formula)
4 hours (breast milk)
2 hours (clear liquid)
Reference: Toronto notes.

3. What is the sign that indicate inadequate general anesthesia?


A. Dry skin.
B. Hypotension.
C. Bradycardia.
D. Dilated Pupils.

Answer: D
Reference: Toronto Notes

4. Pregnant on delivery she has hypotension and dyspnea. Which type of anesthesia will be given?
A. Pudendal n
B. Local cervical
C. General anesthesia
D. epidural ( Newly added )

Answer: General anesthesia


Reference: UpToDate

5. During labor, the anesthesiologist injects analgesic drug at L3-L4, after 2 days the mother still have pain in the site of
injection. Which ligament is affected?
A. Anterior longitudinal ligament.
B. Posterior longitudinal ligament.
C. Ligamentum flavum.
D. Interspinous ligament.

Answer: D
In this Q: we don`t know if the doctor used spinal or epidural but in general pain at site of injection = inflammation of supraspinous
lig or interspinous lig. (Answered by an anesthesia consultant at KFMC)
Note: The associations between back pain and epidural analgesia are unclear.
216
- Local tenderness at the site of epidural or spinal placement are relatively common & usually clears within several days to 3 weeks
and may be related to superficial irritation of the skin or periosteal irritation or damage.
- Although short-term back pain is common, it does not appear to be related to the use of regional analgesia. Similarly, no causal
relationship exists between the use of epidural analgesia and the development of long-term postpartum backache.

6. Station +1 80% effacement and 4 cm dilatation and Rupture of membrane with clear fluids, : what anesthesia to give?
A. VS
B. B-Pudendal,
C. Para cervical,
D. GA

Answer: D

7. to increase hyperbaric in intrathecal analgesia put patient in :


A. lateral
B. supine
C. trundlberg
D. antitrendulberg

Answer: C
Hyperbaric solutions goes with gravity
Hypobaric solutions goes against gravity
So to increase hyperbaric solution’s spread the patient is put in antitrendulenberg position

8. Which of these analgesia has x100 the strength of morphine?


A. Fentanyl
B. Tramadol

C. NSAID

Answer: A
Reference: Toronto Notes.
Extra notes:
Common Side Effects of Opioids: Nausea and vomiting, Constipation, Sedation, Pruritus, Abdominal pain, Urinary retention,
Respiratory depression..
When prescribing opioids, consider: Breakthrough dose, Anti-emetics, Laxative.
Fentanyl is a powerful synthetic opiate analgesic similar to but more potent than morphine. It is typically used to treat patients with
severe pain, or to manage pain after surgery.
Reference: https://www.drugabuse.gov/drugs-abuse/fentanyl

9. Mallampati class 3 what you will see?


A. Soft palate and uvula
B. Soft palate and base of uvula
C. Soft palate,uvula and tonsillar pillars

Answer: B

10. Women in labor (long scenario) with low Hgb & platelet >> what is the Type anesthesia?
A. GA
B. Para cervical
C. Pudendal block

217
11. pregnant lady with hypotension , what type of anesthesia you will given :
A. Pudendal
B. Epidural
C. General
Answer: ?

12. When you are going to intubate unconscious male but there was difficulty with ventilation with bag-mask, what to do:
A. Proceed to intubation
B. Apply cricoid pressure
C. Head tilt

Answer: C
- A Patient’s head should be extended to make the airway patent (check the positioning section in the first reference)
- Mask seal – Mask seal requires reasonably normal anatomy, absence of facial hair, lack of interfering substances, such as
excessive vomitus or bleeding, and the ability to apply pressure to the face with the mask.

Reference : http://emedicine.medscape.com/article/80184-overview & https://www.das.uk.com/guidelines/cvci.html

13. Patient came from RTA , in preparing of setting of intubation patient became desatting more and ambo bag could not
maintain saturation, which should be done ?
A. More jaw thrust
B. More head tilt
C. Precede immediate for intubation

Answer: A or B
According to up to date, if the patient's oxygenation cannot be maintained, immediate rescue by cricothyrotomy is necessary.
Reference :
http://www.uptodate.com/contents/the-failed-airway inadults?Source=outline_link&view=text&anchor=H351136572#H351136572

14. Best medication for chronic pain disease?


A. Ibuprofen
B. Acetaminophen
C. There is another choice started with ci >>>.
Answer: b
http://www.webmd.com/pain-management/tc/chronic-pain-medications

15. What’s usually giving with analgesics to reduce side effects?


A. Cimetidine
B. Metoclopramide

Answer: B
Explanation: the question probably means opioid analgesics; with which we usually give antiemetic like metoclopramide (dopamine
antagonist). Cimetidine is a histamine H2 antagonist for GERD and peptic ulcer, and is not an antiemetic therefore, not the answer.
References: http://www.anzca.edu.au/Documents/Acute-Pain-final-version- page 65

16. During delivery something happened C/S was required, what type of anesthesia?
A. Pudendal.
B. General.

Answer: epidural and spinal anesthesia, B/c in General anesthesia the drugs are given to the mother will affect the infant.
Reference: http://www.uptodate.com/contents/c-section-cesarean-delivery-beyond-the-basics

218
17. Best MS relaxant in sever back pain?
A. Diazepam
B. Metaloxone

Answer: B
The goal of muscle relaxants is to normalize muscle excitability, decrease pain, and improve the motor function. They exert their
pharmacologic effect centrally at the level of the spinal cord, the brainstem, or the cerebrum.
http://www.emedexpert.com/classes/skeletal-muscle-relaxers.shtml

18. Elderly, asthmatic, what is the best induction


A. Propofol
B. ketamine

Answer: A
Propofol is considered to be the agent of choice for induction of anesthesia in asthmatics.
http://www.ncbi.nlm.nih.gov/m/pubmed/11050961/

19. How to prevent a high level of anesthesia?


A. Anti-Trendelenburg
B. Trendelenburg

Answer: A?
If the level of spinal anesthesia is not fixed, the Trendelenburg position can alter the level of spinal anesthesia and cause a high
level of spinal anesthesia in patients receiving hyperbaric local anesthetic solutions. This can be minimized by raising the upper
part of the body with a pillow under the shoulders while keeping the lower part of the body elevated above heart level.
Reference: http://www.nysora.com/index.php?news=3424

20. Which anesthetic agent provides sedation with sub-anesthetic dose (in other version: an analgesic in mild doses)
A. Ketamine
B. Pentamin
C. Midzpam

Answer: A

21. Patient with Sever asthmatic attack have O2 and inhaled beta agonist what to give?
A. intubation
Answer: A

22. Direction of IM injection related to sciatic nerve? safest to use the upper outer quadrant.

23. Patient with dilated pupil tachycardia cause? Sympathomimetic

24. Which Ligament when you do epidural anesthesia

25. Earliest sign of local anesthetic toxicity:


A. Tongue and circumoral numbness


Answer: A
Reference: Toronoto Notes.

219
Reference: http://www.ncbi.nlm.nih.gov/pubmed/25197290

26. Same of common about breastfeeding what is indicative of poor anesthesia??

Answer:
"awareness with recall" (AWR) refers to both intraoperative consciousness and explicit recall of intraoperative events. Other terms,
such as "intraoperative awareness during general anesthesia," "anesthesia awareness," or simply "awareness," are used as
synonyms.

Anesthetic underdosing — The most important contributing factor for AWR is underdosing of anesthesia relative to a given patient's
specific requirements.
This can occur for the following reasons [21-23]:
●There is a mistake or failure in the delivery of anesthesia
●The anesthetic technique results in inadequate anesthesia
●It is judged unsafe to administer sufficient anesthesia
●A specific patient's needs are underappreciated

Ref: UpToDate

220
Extra information

221
222
223
224
Š_Œ
SMLE
KSAU-HS
Question Bank
1st Edition
Pediatrics

This is an accumulative effort from King Saud bin Abdulaziz University for Health Sciences (2016-17/
Batch 9) interns to organize and answer what have been collected previously from SMLE Q Bank
2015-16

We would like to acknowledge:


- King Saud bin Abdulaziz University for Health Sciences (2016-17/Batch 9) interns for their huge efforts in
accomplishing this project
- SMLE Q Bank Group
‫ مدونة طالب طب سعودي‬-

‫جهد بشري قابل للخطأ والصواب‬


For any comments, kindly contact us at
SMLE2016.17@gmail.com

Highlighted in yellow are queried questions


Highlighted in red are some repeated questions

2
Pediatrics

3
1. What is the injection that is routinely given to new-born to inhibit hemorrhage:
A. Vitamin K
B. Vitamin C
C. Vitamin D
D. Vitamin E
Answer: A

2. Cellulitis occurring about the face in young children (6-24 months) and associated with fever and purple skin
discoloration is MOST often caused by
A. Group A beta hemolytic streptococci
B. Haemophilus influenzae type B
C. Streptococcus pneumoniae
D. Staphylococcus aureus
E. Pseudomonas
Answer : A

3. Breastfeeding mother with HCV treated by interferon for more than one year, what is the risk of breastfeeding on the
infant?
A. Cracked Nipple
B. Mother with anemia
C. Infant complain of oral candidiasis
D. Not follow up of infant immunization
Answer: A
CDC – Hepatitis C:
HCV has not been shown to be transmitted through breast milk, although HCV-positive mothers should consider abstaining from
breastfeeding if their nipples are cracked or bleeding.
Uptodate: There is no evidence that breastfeeding is a risk for infection among infants born to HCV infected women
Antiviral treatment of pregnant women is not recommended. Ribavirin teratogenic in animal models. Interferon increase
spontaneous abortion in animal models.
Medscape: peginterferon alfa 2a: unknown if distributed in human breast milk. Large molecular weight

4. Milestone, baby pulls himself to stand, crawls without difficulty, which age is he?
A. 8 months
B. 10 months
C. 12 months
D. 14 months
Answer: A
Table: Kaplan Pediatrics Lecture Notes
Http://www.babycentre.co.uk/a6477/milestone-chart-seven-to-12-months

5. 2 years old presented with fever for one month with the pic, lab shows Pancytopenia, what is the cause?
A. Leishmania
B. Leukemia
C. Malaria
D. Brucellosis
Answer: B
Brucellosis, malaria and leishmanial also cause pancytopenia, but it seems the pic shows sign of leukemia.

6. A boy dreamed of a bad dream and he woke up crying and feared but he does not remember any of the dream which
stage of sleep he was in:
A. 1

4
B. 2
C. 3
D. 4
Answer: D
Night terrors happen during deep non-REM sleep. Unlike nightmares (which occur during REM sleep), a night terror is not
technically a dream, but more like a sudden reaction of fear that happens during the transitions from the deepest stage of non-
REM sleep to lighter REM sleep, a stage where dreams occur. Night terrors usually occur about 2 or 3 hours after a child falls
asleep. Unlike nightmares, which kids often remember, kids won't have any memory of a night terror.

7. Which of the following will you find in a croup patient?

A. Nasal flaring
B. Expiratory wheezing
C. Tachypnea
D. Lip cyanosed
Answer: all?? Answer may depend on cues in the question

8. What is the condition in which the baby will have bone age more than chronological age?

A. Hypothyroid
B. Chronic kidney disease
C. Congenital adrenal hyperplasia
D. Reckitt
Answer: C
Medscape: A bone-age study is useful in evaluating a child who develops precocious pubic hair, clitoromegaly, or accelerated
linear growth. Patients who have these symptoms because of adrenal hyperplasia have advanced skeletal maturation.
Http://emedicine.medscape.com/article/919218-workup#c5

9. Child with croup. What will you find in chest auscultation?


A. Wheezing
B. Crepitation
C. Gasp
D. Silent chest ( not sure about this option)

Answer: A
Http://www.merckmanuals.com/professional/pediatrics/respiratory-disorders-in-young-children/croup

10. Child presented with heaves on examination, Ecg shows RBBB


On ECHO: right ventricle motion abnormality and right ventricle hypertrophy
What is the most likely cause?
A. Mitral prolapse
B. ASD
C. VSD
D. Coarctation of aorta

Answer: D
Not enough clues but this looks like coarctation of the aorta because of the right ventricular hypertrophy and the fact that septal
defect was not seen on echo.
Http://emedicine.medscape.com/article/895502-workup#c4

5
11. One month boy came for vaccine. His older sister (6years old) had renal transplant and now is on immunosuppressive
medication. Which vaccine is contraindicated? (Doesnt say for the baby or for the girl)
A. MMR
B. Oral polio
C. Salk polio
D. Influenza
Answer: B. Due to the risk of spread OPV for immunocompromised girl.
OPV (is a live vaccine).

12. 10 days neonate present with lethargy , irritability , fever , signs of meningitis which organism is causative :
A. Listerea monocytogens
B. Streps pneumonia
C. Staph aureus
D. N-menningitidis

13. Bacterial meningitis in 14 month child I think? Gram positive cocci, what is the management?
A-amoxicillin
B-amoxicillin and gentamicin
C-ceftriaxone and vancomycin
D-vancomycin
Answer : C-ceftriaxone and vancomycin

Age or Predisposing Feature Antibiotics


Age 0-4 wk Ampicillin plus either cefotaxime or an aminoglycoside
Age 1 mo-50 y Vancomycin plus cefotaxime or ceftriaxone*
Age >50 y Vancomycin plus ampicillin plus ceftriaxone or cefotaxime plus vancomycin*
Impaired cellular immunity Vancomycin plus ampicillin plus either cefepime or meropenem
Recurrent meningitis Vancomycin plus cefotaxime or ceftriaxone
Basilar skull fracture Vancomycin plus cefotaxime or ceftriaxone
Head trauma, neurosurgery, or CSF shunt Vancomycin plus ceftazidime, cefepime, or meropenem
CSF = cerebrospinal fluid.

*Add ampicillin if Listeria monocytogenes is a suspected pathogen.


Reference : http://emedicine.medscape.com/article/232915-treatment#d8

14. Cystic fibrosis


A. Chromosome 7 long q arm
B. Chromosome 7 short p arm
C. Chromosome 7 short q arm
D. Chromosome 7 long p arm

Answer: a
Autosomal recessive, cftr gene found on the long (q) arm of chromosome 7 resulting in a dysfunctional chloride channel on the
apical membrane of cells. Reference: toronto notes

6
15. Mother brought her 2 years old child to the er with history of upper respiratory tract infection for the last 3 days with
mild respiratory distress. This evening the child started to have hard barking cough with respiratory distress. On
examination: rr 40/min, associated with nasal flaring, suprasternal & intercostal recessions. What is the most likely
diagnosis?
A. Viral pneumonia
B. Bacterial pneumonia
C. Bronchiolitis
D. Acute epiglottitis
E. Laryngotracheobronchitis (croup)

Answer: e
Reference: 3rd edition uqu > pediatrics > q 27

16. Child known case of dm 1, lost his consciousness at school. The last insulin injection is unknown.
A. Take him to the hospital
B. Iv ranger lactate
C. Im glucagon
D. Insulin

Answer: c
Because the patient could have hypoglycemic attack so give him shot of glucagon to increase his blood sugar.
Nb: hypoglycemia is more dangerous than hyperglycemia .. You may give insulin and kill the patient ..
Reference: http://www.healthofchildren.com/g-h/hypoglycemia.html

17. A child with the history of repeated infections, failure to thrive and anemia. His older brother also has same condition.
What is the most likely diagnosis?
A. Nutritional anemia
B. Leukemia
C. Lymphoma
D. Haemoglobinopathy

Answer: ?
Repeated infection, failure to thrive, anemia and family history of same condition make hemoglobinopathy the best answer.
Reference : http://www.uptodate.com/contents/overview-of-the-clinical-manifestations-of-sickle-cell-
disease?Source=machinelearning&search=haemoglobinopathy&selectedtitle=7%7e150&sectionrank=1&anchor=h13#h19

18. During delivery, when the doctor cut the umbilical cord, bleeding doesn’t stop. Which of the following factors is deficient
in this case?
A. Factor x
B. Factor xi
C. Factor xii
D. Factor xiii

Answer: d
The bleeding diathesis in inherited factor xiii (fxiii) deficiency is severe in most patients. Bleeding from the stump of the umbilical
cord within the first days to weeks of life is a characteristic sign.
Http://emedicine.medscape.com/article/960515-clinical

19. When does girls get puberty as compared to boys?


A. 1 to 2 yrs before
B. 2 to 3 yrs before
C. Same time when boys do
D. After boys
7
Answer: a
➢ Average age for girls: 10 and a half years old.
➢ Average age for boys: 11 and a half to 12 years old.

20. 6 year old child with fever, malaise, lymphadenopathy and hepatosplenomegaly. Shown is the cbc and bone marrow
aspiration slide. What is the most likely diagnosis?

Wbc 30,000 (4,500-11.000)

Hgb 9

Platelet 50,000 (150,000-400,000)

Peripheral blast cells 25%

A. Sickle sequestration.
B. Leukemia.
C. Malaria.
D. Leishmaniasis.

Answer: b
The diagnosis of acute lymphoblastic leukemia (all) is made when at least 30% lymphoblasts (french-american-british [fab]
classification) or 20% lymphoblasts (world health organization [who] classification) are present in the bone marrow and/or
peripheral blood.
Approximately 15% of patients with all have a t(9;22) translocation (i.e, philadelphia [ph] chromosome).
Reference: http://emedicine.medscape.com/article/207631-workup#c11

21. Child that throws a ball at you and draws a straight line and stacks “few” cubes on each other (they didn’t mention the
number of cubes). What is the age?
A. 12 months
B. 14 months
C. 18 months
D. 24 months

Answer: d
Check the table below reference.
th
Nb. Drawing straight lines begins at the age of 2 years. Reference: illustrated textbook and nelson 7 ed.

8
22. A child that can raise his head slightly when prone and smiles. He turns his head 180 degrees and has head lag when you
pull him to sit. How many old is he?
A. 4 weeks
B. 8 weeks
C. 12 weeks
D. 16 weeks

Answer: b
Check the table of q35. Same reference.

23. A child with rheumatic heart disease allergic to penicillin. What prophylaxis should be given before a procedure?
A. Iv amoxicillin
B. Iv vancomycin + iv gentamicin
C. Oral vancomycin + gentamicin
D. Oral amoxicillin

Answer: b (depends on the type of procedure and the ability to tolerate oral medications)
Most probable, the answer is b. Since amoxicillin is type of penicillin and gentamicin generally not given po.
Patients with rheumatic heart disease and valve damage require a single dose of antibiotics 1 hour before surgical and dental
procedures to help prevent bacterial endocarditis. Patients who had rheumatic fever without valve damage do not need
endocarditis prophylaxis. Do not use penicillin, ampicillin, or amoxicillin for endocarditis prophylaxis in patients already receiving

9
penicillin for secondary rheumatic fever prophylaxis (relative resistance of po streptococci to penicillin and aminopenicillins).
Alternate drugs recommended by the american heart association for these patients include po clindamycin (20 mg/kg in
children, 600 mg in adults) and po azithromycin or clarithromycin (15 mg/kg in children, 500 mg in adults).
Reference: color atlas of infective endocarditis p:81 and http://emedicine.medscape.com/article/891897-treatment
http://www.cps.ca/documents/position/infective-endorcarditis-guidelines

24. Dtap vaccine is against what?


A. Whooping cough, tetanus, diphtheria
B. Pertussis, tetanus, measles
C. Rubella, tetanus diphtheria
D. Whooping cough, rubella, diphtheria
Answer: a
Diphtheria, tetanus, and pertussis are serious diseases caused by bacteria. Diphtheria and pertussis are spread from person to
person. Tetanus enters the body through cuts or wounds. Diphtheria causes a thick covering in the back of the throat. It can lead
to breathing problems, paralysis, heart failure, and even death. Tetanus (lockjaw) causes painful tightening of the muscles,
usually all over the body. It can lead to "locking" of the jaw so the victim cannot open his mouth or swallow. Tetanus leads to
death in up to 2 out of 10 cases. Pertussis (whooping cough) causes coughing spells so bad that it is hard for infants to eat, drink,
or breathe. These spells can last for weeks. It can lead to pneumonia, seizures (jerking and staring spells), brain damage, and
death. Reference: http://www.cdc.gov/vaccines/hcp/vis/vis-statements/dtap.html

25. 11 years old patient with rheumatic fever and cardiac involvement. For how long he will require prophylaxis?
A. 5 years
B. 6 years
C. 10 years
D. 15 years
Answer: d or c
➢ Rheumatic fever with carditis and clinically significant residual heart disease requires antibiotic treatment for a
minimum of 10 years after the latest episode; prophylaxis is required until the patient is aged at least 40-45 years and is
often continued for life.
➢ Rheumatic fever with carditis and no residual heart disease aside from mild mitral regurgitation requires antibiotic
treatment for 10 years or until age 25 years (whichever is longer).
➢ Rheumatic fever without carditis requires antibiotic treatment for 5 years or until the patient is aged 18-21 years
(whichever is longer)
➢ Depends on uptodate the answer is c see the pic.

10
References:http://emedicine.medscape.com/article/236582-medication + http://emedicine.medscape.com/article/891897-
overview#showall + uptodate

26. Which of the following congenital heart disease is the least associated with infective endocarditis?
A. Asd
B. Vsd
C. Pda
D. Pulmonary stenosis

Answer: a
Similar question: http://gradestack.com/dr-bhatia-medical/infective-endocarditis-is/0-3042-3177-15646-sf
➢ Valvular aortic stenosis – 13.3 percent
➢ Coarctation of the aorta – 3.5 percent
➢ Primum atrial septal defect –2.8 percent
➢ Ventricular septal defect (vsd) –2.7 percent
➢ Tetralogy of fallot (tof) –1.7 percent
➢ No child with secundum atrial septal defect, patent ductus arteriosus (pda), or pulmonic stenosis had ie after surgery.

27. A boy came to your clinic with yellow discoloration of the eyes noticed 3 days back and hepatomegaly. His liver enzymes
are increased. What is the diagnosis?
A. Hepatitis a
B. Hepatitis b
C. Hepatitis c
D. Hepatitis d

Answer: a
Hepatitis a the only type of hepatitis that reveal tender hepatomegaly. Reference: step up to pediatrics p:88

28. What is the triple antitoxoid?


A. Tetanus, diphtheria, whooping cough

11
B. Tetanus, diphtheria, tb
C. Diphtheria, pertussis, colorectal ca
D. Diphtheria, tetanus, rabies.

Answer: a
Reference: http://www.who.int/vaccine_safety/initiative/tools/dtp_vaccine_rates_information_sheet.pdf

29. Which of the following is most typically seen in 4 years old baby?
A. Print name.
B. Stand on one foot briefly.
C. Copy triangle and square.
D. Toilet trained.

Answer: d
Check the table at the end of pediatrics section.

30. Child can set without support, cruises around furniture, uses chair to stand, say dada, crawl stairs. What is the age of
this child ?
A. 8 months.
B. 10 months.
C. 12 months.
D. 15 months.

Answer: b
Check the table at the end of pediatrics section.

31. (long scenario) child have 1mm defect in muscular atrial septum. What you will do?
A. Surgical repair.
B. Catheter repair.
C. Reduce after load.
D. Watchful waiting.
Answer: d
80-100% spontaneous closure rate if asd diameter <8 mm. Reference: toronto notes.

32. A baby 8 month old breastfeed for 6 month normally. He devolved vomiting and jaundice after fruit juice. What
component in the juice the baby is allergic to?
A. Glucose
B. Fructose
C. Sucrose
D. Galactose
E. Phenylalanine

Answer: b
Hereditary fructose intolerance due to deficiency of fructose-1,6-biphosphate aldolase b in the liver.
Reference: http://reference.medscape.com/article/944548-overview
12
33. What is the most common cause of acute bronchiolitis?
A. Respiratory syncytial virus (rsv)
B. Adenovirus
C. Parainfluenza
D. Mycoplasma pneumonia.
Answer: a
Reference: fa for the usmle step 2 ck and nelson 7 th p: 357

34. To which part of body it can go ? (continuation of the previous question )


A. Spleen.
B. Bladder.
C. Kidney.
D. Liver.
Answer: d
Rsv may be recovered from extrapulmonary tissues, such as liver, cerebrospinal fluid, or pericardial fluid
Reference: uptodate

35. Breastfeeding after delivery should start:


A. Immediately
B. 6 hr
C. 8hr
D. 24 hr

Answer: a
Reference:https://www.betterhealth.vic.gov.au/health/healthyliving/breastfeeding-when-to-start

36. 26 years old female g1p1 brought her two weeks old baby who cries a lot. He is on breastfeeding since birth, he stop
crying at night when she gave him formula milk. On examination the baby looks normal except for increase gurgle
sound. What is the most likely diagnosis?
A. Paralytic ileus.
B. Lactose malabsorption.
C. Increase bowel gases.
D. Breast milk jaundice.

Answer: or c?

37. 5 years old girl with uncomplicated cystitis. What is the management?
A. Oral amoxicillin
B. Iv cephalosporin
C. Im ceftriaxone
D. Sodium …

Answer: a
Empirical therapy should be initiated for symptomatic children and for all children with a urine culture confirming uti. For an
older child who does not appear ill but has a positive urine culture, oral antibiotic therapy should be initiated. For a child with
suspected uti who appears toxic, appears dehydrated, or is unable to retain oral fluids, initial antibiotic therapy should be
administered parenterally, and hospitalization should be considered. Neonates with uti are treated for 10 to 14 days with
parenteral antibiotics because of the higher rate of bacteremia. Older children with uti are treated for 7 to 14 days. Initial
treatment with parenteral antibiotics is determined by clinical status. Parenteral antibiotics should be continued until there is

13
clinical improvement (typically 24 to 48 hours). Commonly used parenteral antibiotics include ceftriaxone or gentamicin. Oral
regimens include a cephalosporin, amoxicillin plus clavulanic acid, or trimethoprim sulfamethoxazole.
th
Reference: nelson 7 p: 373-374

38. 7 years old girl was brought by her mother, she developed pubic hair and her height 70th percentile and weight 50th
percentile. On examination there are no signs of puberty except pubic hair. Abdominal, chest, cardiac and renal
examination were normal. What is the most likely diagnosis?
A. Congenital adrenal hyperplasia
B. 45x (turner syndrome).
C. Premature adrenarche.
D. Normal puberty.

Answer: c
Premature adrenarche is when these changes begin early, before age 8 for girls and age 9 for boys.
Reference:http://www.medscape.com/viewarticle/759350_3

39. A child with normochromic normocytic anemia + splenomegaly. Blood smear was attached showing clear spherocytosis.
Which of the following will be abnormal?
A. Plt
B. Reticulocytes
C. Wbc
D. Mcv

Answer: b
Reference: uptodate under the topic " hereditary spherocytosis: clinical features; diagnosis and treatment"

40. What is the most common intraabdominal tumor in children?


A. Hepatoma
B. Rhabdomyosarcoma
C. Ewing tumor
D. Wilms tumor

Answer: d
Reference: http://www.ncbi.nlm.nih.gov/pubmed/15321038

41. What is the single most important risk factor for cerebral palsy?
A. Prematurity
B. Birth weight less than 1.5 kg
C. Prenatal asphyxia
D. Genetic mutations.

Answer: a
“prematurity is the single most important risk factor for cerebral palsy”
Reference:https://books.google.com.sa/books?Id=bonllhartfac&pg=ra5-pt6753&lpg=ra5-pt6753&dq=#v=onepage&q&f=false

42. Baby with recurrent infection tb, aspergillosis all type of infection with history o brothers death at 3 year with same pr
give? Repeated
A. Influenza
B. Bcg
C. Varicella
D. Polio
14
Answer: im influenza

43. A child with hepatosplenomegaly, current infection. Brother died at 3 years with septic shock. How to give vaccination?
A. Give all.
B. Don’t give until 3 years.
C. Don’t give live vaccines.
D. Don’t give killed vaccines.
Answer: c

44. Trauma to the fibular head what is the nerve injured?


A. Common peroneal nerve
B. Presbycusis
C. Otosclerosis
D. Otitis media
Answer: a

45. Child can know color but with difficulty in making square
A. 2 years
B. 3 years
C. 4 years
D. 5 years
Answer: b
46. What is the age of child should be know few word ?
A. 6 month
B. 8 month
C. 12 month
D. 24 month
Answer: d

47. Bacterial meningitis in 14 month child , gram positive cocci, what is the management?
A. Amoxicillin
B. Amoxicillin and gentamicin
C. Ceftriaxone and vancomycin
D. Vancomycin
Answer: ceftrixone and vancomycine c
Http://www.fpnotebook.com/mobile/neuro/id/bctrlmngtsmngmnt.htm

48. Child with fever, general swelling and dark colored of urine which best evaluate for this pt:
A. Us
B. Rft
C. Urine culture
D. Urine specimen

Answer: d

Source: urinalysis and sediment examination are crucial in the evaluation of patients with acute nephritic syndrome. Look for
the following:

• Protein
• Blood
• Red blood cells (rbcs)
• White blood cells (wbcs)
• Dysmorphic rbcs
• Acanthocytes
15
• Cellular (ie, rbc, wbc) casts
• Granular casts
• Oval fat bodies
Http://emedicine.medscape.com/article/239278-workup

49. Mother and her child visited the pediatrician for 6 months vaccination, however the mother stated that her child was
hospitalized after receiving the 4 months vaccination he develop anaphylaxis , what is the right thing to do?
A. Test the child for which antigen is allergic from
B. Give him steroid/antihistamine post the vaccination
C. Vaccinate him and discharge home
D. Vaccinate him and hospitalize the child for 1 hour
Answer: a

50. What is the injection that is routinely given to new-born to inhibit hemorrhage?
A. Vitamin k
B. Vitamin c
C. Vitamin d
D. Vitamin e

Answer: a
(toronto note)

51. 10-cellulitis occurring about the face in young children (6-24 months) and associated with fever and purple skin
discoloration is most often caused by
A. Group a beta hemolytic streptococci
B. Haemophilus influenzae type b
C. Streptococcus pneumonie
D. Staphylococcus aureus
E. Pseudomonas

answer : c by pediatric doctor


(hib can lead to a violaceous or blue-purple color but it is not diagnostic).

Note: the most common organism can cause cellulitis at 6-24-month old is streptococcus

52. Attention deficit hyperactivity disorder ( they give me the symptom not the diagnosis ) child what is the manegment?
A. Ecitalpram
16
B. Atomoxetine
C. Olanzapine
D. Clonazepam
answer : b
Http://www.merckmanuals.com/professional/pediatrics/learning-and-developmental-disorders/attention-deficit-hyperactivity-
disorder-add,-adhd

53. Which syndrome is associated with coarctation of aorta:


A. Down
B. Patau
C. Edward’s
D. Turner
Answer: d
Http://www.merckmanuals.com/professional/pediatrics/chromosomal-anomalies/sex-chromosome-anomalies

54. Child with aspirin intake overdose … what kind of acid-base balance:
A. Metabolic alkalosis
B. Metabolic acidosis
C. Respiratory alkalosis
D. Respiratory acidosis

Answer : b
The net effect of these changes in most adults is respiratory alkalosis or a mixed respiratory alkalosis-metabolic acidosis. Pure
metabolic acidosis is unusual in adults, but is more common in children . Children with mild to moderate acute salicylate
poisoning lose the respiratory drive and are more likely to present with mixed metabolic and respiratory acidosis .
Respiratory alkalosis first in oncent at first then metabolic acidosis.

55. Child presented with croup presentation (barking cough … etc.), what is the management?
A. Inhaled steroid
B. Inhaled epinephrine and oral steroid
C. Oral steroid with antibiotics
D. Empirical antibiotics
Answer : b
usmle first aid step 2

56. During otoscopy examination of a child, pulling the ear at which direction is going to help to see tympanic membrane?
A. Anterior and inferior
B. Posterior and inferior
C. Anterior and superior
D. Posterior and superior
Answer: b

Straighten the patient’s ear canal by pulling the pinna up and back in children 3 years of age and older and down and back in
children younger than 3 years of age.
In general: in children, the auricle should be pulled downward and backward.
Http://www.atitesting.com/ati_next_gen/skillsmodules/content/physical-assessment-
child/equipment/ap_ear_nose_throat.html
Http://www.meddean.luc.edu/lumen/meded/medicine/pulmonar/pd/pstep18.htm

57. What is the daily fluid requirement for 15kg baby?


A. 1000
B. 1200

17
C. 1400
D. 1600

Answer: b
Explanation:
1. For the first 10 kg we give 100 ml per each kilo
2. For the second 10 kg (from 10 to 20 kg) we give 50 ml per each kilo
3. For every kilo after 20 kg we give 20 ml per each kilo
So this baby is 15 kg which will be (1000 ml from the first 10 kg) and (5 kg multiplied by 50 = 250 ml) so the baby’s daily
requirement will be 1250 ml per day.

58. What is the most common intra-abdominal tumor in children?


A. Hepatoma (scc)
B. Rhabdomyosarcoma
C. Ewing tumor
D. Wilms tumor

Answer: d
Explanation: although other tumors may occur at such age, wilm’s tumor of the kidney is the most common intra abdominal
malignancy of childhood.

59. What is the single most important risk factor for cerebral palsy?
A. Prematurity
B. Birth weight less than 1.5 kg
C. Prenatal asphyxia
D. Genetic mutations.

Answer: a
Ref : uptodate

60. 12 year boy came to the clinic complaining that he is short comparing to his colleagues, his height was 155 and his weight
is similar to 10 years boys.
His mother height is 145, father is 178, what is the expected height of the pt.
A. 140_155
B. 155_160
C. 160_175
D. 175_195
Answer : c - closest-
To calculate mid-parental height:
For boys: [paternal height + (maternal height + 5 inches or 13 centimeters)] / 2
178 + (145+13) /2 = 336/2=168 +- 8 = 160-176 cm (range)
For girls: [maternal height + (paternal height – 5 inches or 13 centimeters)] / 2

61. Baby full term flax..irritable..‫ كالسيميا هايبو عالمات عندو‬give


A. Ca
B. Mg
C. Ringer
D. Glucose
Answer : a
Http://emedicine.medscape.com/article/241893-treatment

62. 11-year-old (typical bacterial meningitis case) which ab× will be given:
A. Ceftriaxone and gentamycin
B. Ampicillin and gentamycin

18
C. Penicillin and gentamycin
D. Vancomycin

Answer:
3rd-generation cephalosporins (ceftriaxone or cefotaxime) for s. Pneumoniae and n. Meningitides and vancomycin for penicillin-
resistant strains of s. Pneumoniae and for s. Aureus

63. Infant with mother for routine checkup, when the mother put the baby he was laughing and when he saw the doctor he
tried to reach his mother what is most likely his age
A. 2 month
B. 4 month
C. 6 month
D. 8 month
Answer: c
Https://www.msdmanuals.com/professional/pediatrics/growth-and-development/childhood-development

64. Case of 12 years old boy on skate downstairs he felt and had perineum trauma, with bruises over the scrotum, perineum,
lower abdomen. Retrograde cystourethrogram show extravasation of the dye. Where is the injury?
A. Penile urethra
B. Urinary bladder
C. Prostatic urethra
D. Ureters.
Answer: a
Anterior urethral (composed of the penile and bulbar urethra) injuries. This type of injury is seen most commonly in blunt
trauma, but is not usually associated with pelvic fractures. It results from a strong blow to the perineum that causes the bulbar
urethra to be crushed against the inferior border of the pubic symphysis. This typically occurs in a fall astride, a straddle injury
from a vehicle accident, an assault, or from bicycle handlebars.

65. 2year old child got otitis media after urti. Treatment:
A. Observe.
B. High dose ibuprofen.
C. Amoxicillin 45 mg/kg/day for 5 days.
D. Amoxicillin 90 mg/ kg/ day for 10 days.✅
Answer: d

19
Http://www.uptodate.com.sci-hub.cc/contents/acute-otitis-media-in-children-
treatment?Source=search_result&search=otitis+media&selectedtitle=2~150

66. Absence of moro reflex in right side of infant is due to.


A. Intracranial hemorrhage.
B. Neonatal hypoglycemia.
C. Neonatal sepsis.
D. Erbs palsy.✅
Answer: d

Erbs palsy causes asymmetrical moro reflex. Intracranial hemorrhage causes poor moro reflex.
Https://www.msdmanuals.com/professional/pediatrics/perinatal-problems/birth-injuries

67. 13 years old with enteric fever. Allergic or resistant to chloramphenicol (i forgot). Treatment is:
A. Double chloramphenicol.
B. Add ciprofloxacin.
C. Ciprofloxacin alone (orally)
D. Im ceftriaxone
Answer: d
Antibiotic resistance is common and increasing, particularly in endemic areas, so susceptibility testing should guide drug
selection. In general, preferred antibiotics include ceftriaxone 1 g im or iv q 12 h (25 to 37.5 mg/kg in children) for 14 days
Https://www.msdmanuals.com/professional/infectious-diseases/gram-negative-bacilli/typhoid-fever#v11560044

68. Child woke up screaming, nightmare and crying parents asked the child he doesn’t remember anything what sleep stage?
A. 1
B. 2
C. 3
D. 4
Answer: c
In the current rules, nrem stage 3 and nrem stage 4 are combined as stage n3
Http://www.uptodate.com.sci-hub.cc/contents/stages-and-architecture-of-normal-
sleep?Source=search_result&search=sleep+stages&selectedtitle=1~113
Confusional arousals, sleep terrors, and sleepwalking are the most significant parasomnias associated with nrem sleep. They are
also termed disorders of partial arousal as they result from incomplete arousal from nrem sleep. Typically, they occur at the
transition from deep nrem (stage n3) sleep into the lighter stages of nrem sleep (n1 or n2) or into the awake state.
Http://www.uptodate.com.sci-hub.cc/contents/sleepwalking-and-other-parasomnias-in-
children?Source=search_result&search=sleepwalking+and+other+parasomnias+in+children&selectedtitle=1~150

69. Blood film for girls came abdominal pain cough splenomegaly dx;
A. P.malaria
B. P.falcifom
C. P. Oval
D. Mp. Something
Answer:
Depends on blood film

20
70. Newborn in endemic area of cretinism suspect to have cretinism what's your action:
A. Iodine supplements
B. Repeat thyroid fun after 1 or 2 month
C. Thyroxine✅
D. ?
Answer: c

Treatment should be initiated in an infant with a clearly positive screening test as soon as confirmatory blood samples have
been drawn, pending results. In cases in which screening tests are borderline, a treatment decision can be made after results of
the confirmatory tests return. Http://www.uptodate.com.sci-hub.cc/contents/treatment-and-prognosis-of-congenital-
hypothyroidism?Source=see_link

71. Neonate with physiological jaundice her parent brought the baby because the color changed from yellow to greenish
what you suspect disease: (i’m not sure about q)
A. Abo
B. G6pd
C. Due to oxidation of bilirubin
D. The treatment start to work
Answer,‫الموجود عن مختلفه االختبار في االجوبه‬
Http://www.medicalnewstoday.com/articles/165358.php

72. Treatment of EBV (in scenario there patient with tonsillar exudates, lymphadenopathy, splenomegaly)
A. Oral acyclovir
B. Oral antibiotic
C. Iv acyclovir
D. Supportive ttt
Answer: d

21
73. Most common cause of precocious puberty in girls
A. Idiopathic ✅
B. Adrenal tumor
C. Ovarian tumor
D. Functional ovarian cyst"
Answer: a
Idiopathic —central precocious puberty is idiopathic in 80 to 90 percent of cases of girls, but in only 25 to 60 percent of boys
(uptodate)

74. Child sit and support his head, laughing and cooing :
A. 4
B. 6 ✅
C. 8
D. 16
Answer: b
Baby can support head steadily at 4 months and start sitting with full support at 6 months. (milestones source: nilson)

75. Child his height and weight below normal besides growth hormone what you will order:
A. Somatomedin c
B. Aldosterone
C. Insulin
D. Testosterone
Answer: a
If case is due to growth hormone deficiency which more likely here so the answer is a and the the height should be fallen more
than 2.5 sd below mean for chronological age sex and ethnicity. (below 25% in growth chart)
Somatomedin c is another name to insulin like growth factor (igf-1)
(uptodate)

76. Thalassemia causing stillbirth, which type is it?


A. Two normal beta chains and 2 normal alpha chains
B. Two normal beta chains and 4 abnormal alpha chains ✅
C. 2 abnormal beta chains and 4 normal alpha chains
D. 4 abnormal beta chains and 2 normal alpha chains

Answer: b
This is alpha thalassemia major this will cause loss of all alpha chain and it will lead to hydrop fetalis and consequently cause
stillbirth. (uptodate/medscape)

77. About cyanotic heart disease.


A. Tof
B. Asd
C. Vsd
D. Pda
Answer: a

78. Baby sit briefly, crawl , move object from hand to hand , but can't do pincer grasp
A. 4 month
B. 6 month
C. 7 month
D. 9 month
Answer : c
At 6m starts to sit & transfer objects, crawl start at 7m,, immature pincer grasp at 9m (milestones: nilson)
79. Oropharyngeal maculopapular rash .. Also rash in palm and foot ..?
A. Cmv

22
B. Ebv
C. Coxsackievirus
D. Vaccina virus
Answer: if hfmd then answer is c

80. 7 years old girl was brought by her mother, she developed pubic hair and her height 70th percentile and weight 50th
percentile. On examination there are no signs of puberty except pubic hair. Abdominal, chest, cardiac and renal
examination were normal. What is the most likely diagnosis?
A. Congenital adrenal hyperplasia
B. 45x (turner syndrome).
C. Premature adrenarche.
D. Normal puberty.
Answer: c
Http://www.medscape.com/viewarticle/759350_3

81. Child with vomiting and diarrhea. On exam .. Sunken eyes.. Skin turgor.. Depressed fontanelle.. The degree of
dehydration is
A. 5%
B. 10%
C. 15%
D. 20%
Answer: b
Degree of dehydration for uptodate:
●mild dehydration (3 to 5 percent volume loss) – a history of fluid losses may be the sole finding, as clinical signs may be absent
or minimal.
●moderate dehydration (6 to 10 percent volume loss) – signs and symptoms are now apparent and can include the following:
tachycardia, orthostatic falls in blood pressure, decreased skin turgor, dry mucous membranes, irritability, decreased peripheral
perfusion with a delay in capillary refill between two and three seconds, and deep respirations with or without an increase in
respiratory rate. There may be a history of reduction in urine output and decreased tearing, and, in infants, an open fontanelle
will be sunken on physical examination.
●severe dehydration (>10 percent volume loss) – such children typically have a near-shock presentation as manifested by
hypotension, decreased peripheral perfusion with a capillary refill of greater than three seconds, cool and mottled extremities,
lethargy, and deep respirations with an increase in rate. Severe hypovolemia requires immediate aggressive isotonic fluid
resuscitation to restore the effective circulating volume (ecv) and prevent ischemic tissue injury.

82. Child presented with petechiae and his platelets is 15, otherwise healthy. What will you do for him?
A. Bone marrow aspiration
B. Splenectomy
C. Ivig
D. Steroid ✅
Answer: d

23
83. Child can run , say short stories , what is the age:
A. 2 years.
B. 3 years.
C. 4 years
D. 5 years.
Answer: d
A child starts to run at 18 months, and tell a story at 5 years.

84. Baby came to the emergency with abnormal movement and teeth delay.
Glucose is 5 (normal range from (5-10)
Calcium is low
Magnesium is low
Phosphate is high
What is the best management?
A. Calcium
B. Glucose
C. Magnesium
D. Ringer lactate
Answer: a , according to pediatric resident

85. Baby came with barking coughing ( croup case ) what you will hear on pulmonary auscultation?
A. Bronchial breathing
B. Decreased breath sounds
C. Prominent increase inspiratory sound
D. Increase exploratory wheezing
Answer: c , inspiratory stridor “uptodate”

86. Child with septic arthritis came to er with kness pain , swelling . Management:
A. Oral antibiotic for 14 days
B. Broad spectrum iv antibiotic
C. Surgical drainage and iv antibiotic
D. Antipyretic till the result of aspiration culture
Answer: c
Toronto note

24
According to illustrated textbook of pediatrics: a prolonged course of antibiotics is required, initially intravenously. Washing out
of the joint or surgical drainage may be required if resolution does not occur rapidly or if the joint is deep-seated, such as the
hip.
However joint aspiration is indicated prior to starting antibiotics and repeated until joint aspirate is clear.

87. Which syndrome is associated with coarctation of the aorta :


A. Down syndrome. (50% atrioventricular septal defect )
B. Patau syndrome. ( 80% with vsd , pda , asd )
C. Edward syndrome ( 60% with vsd , pda , asd )
D. Turner syndrome.
Answer: d
Turner syndrome is associated with the following :
*congenital heart defect particularly coarctation of aorta , bicuspid aortic valve
*hypothyroidism
*renal anomalies
Toronto note

88. Kwashiorkor:
A. Low protein and low carbohydrate. (marasmus)
B. High protein and low carbohydrate.
C. Low protein and high carbohydrate.
D. High protein and high carbohydrate.
Answer: c
Illustrated textbook

89. 7years girl presented with bilateral symmetrical lower limb weakness, emg showed ( i can't remember ) ,knee reflexes
showed absolute bilateral knee reflex absence , she had hx one week ago of gi symptoms , what is the best treatment ?
A. Plasmapheresis
B. Immunoglobulin
C. Steroids
D. Methotrexate
Answer:
I think this is a case of guillain - barre syndrome the treatment is iv immunoglobulin if the patient has significant weakness ,
plasmapheresis if progression continue both will hasten the recovery.

90. Bacterial meningitis in 14 month child gram positive cocci, what is the management?
A. Amoxicillin
B. Amoxicillin and gentamicin
C. Ceftriaxone and vancomycin
D. Vancomycin
Answer: c illustrated textbook of pediatrics and kaplan pediatrics

91. Which of the following is a cyanotic heart disease?


A. Asd
B. Vsd
C. Pda
D. Ductus arteriosus
Answer: d
It is the formation of the membrane between the aorta and pulmonary artery instead of spiral, it will grow straight so the artery
will leave from the lv while the aorta from rv which normally should be vice versa. The patient needs another defect to stay alive
such as pda, vsd or asd
Http://emedicine.medscape.com/article/891096-overview

92. Child sit and support his head , laughing and cooing :

25
A. 4
B. 6
C. 8
D. 16
Answer: b
(http://www.babycentre.co.uk/a6476/milestone-chart-one-to-six-months)

93. Child his height and weight below normal besides growth hormone what you will order:
A. Somatocin c
B. Aldosterone
C. Insulin
D. Testosterone
Answer: a

94. Child with epilepsy on anticonvulsant what you will change in his vaccines
A. Change opv to ipv
B. Dtp
C. Remove all vaccines
D. Remove all live vaccines
Answer: b
Source: http://www.vaccines.gov/basics/safety/should/

95. Baby said hi when he entered the clinic, imitates his mother, feeds his doll, refers to himself “me” and say “eye”:
A. 12 months
B. 15 months
C. 18 months
D. 24 months
Answer: d
Illustrated textbook

96. Baby thirsty with tachycardia, sunken eye. Volume loss


A. 1%
B. <3%
C. 5-9%
D. 9%
Answer: d
Symptom Degree of dehydration

Mild (< 3% body weight Moderate (3-9% body weight Severe (>9% body weight lost)
lost) lost)

Mental status Normal, alert Restless or fatigued, irritable Apathetic, lethargic,


unconscious

Heart rate Normal Normal to increased Tachycardia or bradycardia

Quality of pulse Normal Normal to decreased Weak, thready, impalpable

Breathing Normal Normal to increased Tachypnea and hyperpnea

Eyes Normal Slightly sunken Deeply sunken

26
Fontanelles Normal Slightly sunken Deeply sunken

Tears Normal Normal to decreased Absent

Mucous Moist Dry Parched


membranes

Skin turgor Instant recoil Recoil < 2 seconds Recoil >2 seconds

Capillary refill < 2 seconds Prolonged Minimal

Extremities Warm Cool Mottled, cyanotic

Http://emedicine.medscape.com/article/801012-clinical#showall

97. (case of intussusception) child came with colicky abdominal pain, vomiting, bloody stool. Us showed doughnut sign.
What is the most important step in management of this case ?
A. Urgent surgery referral
B. Ngt decompression
C. Iv fluid resuscitation
D. Barium enema

Answer: c
Iv fluid resuscitation, then decompression, afterward, air enema or barium if air in available (ref. Master the board udlme step 2
pediatric chapter)

98. Prim gravida, diet control gdm, on prolonged second stage of labor. She did full flexion of her hip. The head of the baby
descent during contraction and going up during relaxation, one nurse applied pressure on fundus, while another nurse
applied pressure on the supra pubic area. What is the cause of her delayed labor?
A. Full flexion of the hip.
B. Apply pressure on the fundus
C. Apply pressure on the suprapubic area
D. Something irrelevant
Answer: ???
Https://www.google.com.sa/url?Sa=t&rct=j&q=&esrc=s&source=web&cd=16&cad=rja&uact=8&ved=0ahukewigl_-
gwarnahwdtrqkhsttdrwqfghnma8&url=http%3a%2f%2fwww2.warwick.ac.uk%2ffac%2fmed%2fabout%2fglobal%2fetatmba%2ft
raining%2ftanzania%2fprolonged_labour.ppt&usg=afqjcnfbl0sbnm0iri8gvvqlfcnb7sgctq&sig2=edhoihkp_gyenfnbcpsspa
-
Http://emedicine.medscape.com/article/273053-overview

99. Which vaccine is contraindicated in hiv patient


A. Opv
B. Varicella
C. Mmr
D. Hbv
Answer: c or d
The current cdc guidelines are to give these to live vaccine because the they are less virulent than what is in the nature, the
paitents should have cdc count of more than 200. Http://hivinsite.ucsf.edu/insite?Page=kb-03-01-08#s4.7x

100.12 y.old girl htn:


A. 120/80
B. 40/99

27
C. Above 90 percentile
D. Above 95 percentile
Answer: D
Height needs to be determined to answer the question. Http://www.nhlbi.nih.gov/files/docs/guidelines/child_tbl.pdf

101.Young , vesilce ,pastule on back like a band :


A. Shingles
B. Chicken box
C. Herpes
D. Coxsackievirus
Answer:a
Http://emedicine.medscape.com/article/1132465-overview

102.Infant ( i think was 4 months old ) with hemangioma in upper eyelid of 1 eye ( left) covering his eye, when to do the
surgery ?
A. Immediately
B. 1 month
C. 3 month
D. 6 month

Answer: a
Toronto:

103.5 years old girl with uncomplicated cystitis, how to treat:


A. Oral ampicillin
B. Iv cephalosporin
C. Im ceftriaxone
D. Iv ...

Answer: c
Medscape: for parenteral therapy in a patient who is not allergic to cephalosporins, initial treatment may consist of a single
dose of ceftriaxone (75 mg/kg iv/im q12-24h). If the patient has cephalosporin allergy, initial treatment may be with gentamicin
(2.5 mg/kg iv/im as a single dose)
Toronto: oral cephalixen if outpatient, iv gentamycin or ampicillin if inpatient.
Uptodate: for e. Coli coverage, we suggest a second-generation (eg, cefuroxime, cefprozil) or third-generation cephalosporin
(eg, cefdinir, cefixime, cefpodoxime, ceftibuten) for empiric therapy because of increasing rates of e. Coli and other pediatric
uropathogen resistance to trimethoprim-sulfamethoxazole (tmp-smx), amoxicillin-clavulanate, and first-generation
cephalosporins]. However, depending upon local resistance rates, these agents may be acceptable alternatives to second- or
third-generation cephalosporins. Second- and third-generation cephalosporins have excellent activity against e. Coli and most
other gram-negative uropathogens. However, they are not effective in treating gram-positive uropathogens (eg, enterococcus)

104.Infant with maculopapular rash over his face with purple discoloration, what is the causative organism:
A. –gbs
B. Strept. Pneumonia
C. -staph.aureus

28
D. –rsv
Answer:

105.Child present to er with fever and sore throat for one week. Now he has paroxysms` cough and cyanosis at end of cough ?
A. Epiglotitis
B. Sinusitis
C. Croup
D. Bronchitis
Answer:c

106.What is the effect of polio (ipv& opv) on body?


A. All lead to the formation ag in the anterior horn
B. All lead to the formation of the ab in the serum which fight the virus
C. They all enter the intestinal mucosa where the entry of the virus is
D. They all lead to the formation of interferon gamma
Both b&c can be correct answer

107.16 months post-partum present with progressive loss of hearing in rt ear , and now in lt , conductive h , dehiscent
semilunar canal :
A. Glue ear,
B. Otosclerosis
C. Tympanosclerosis
D. Meniere disease
Answer: b

108.Baby hypotension sever vomiting and watery diarrhea what is the electrolyte abnormality
A. L na,
B. H na,
C. H k ,
D. Hyperglycemia
Answer: a

109.Baby that can support his head, laughing, cooping. How old is he!
A. 4 months
B. 8 months
C. 12 months
D. 16 months
Answer: a

110.A 6 weeks old baby pale, jaundice on examination there is palpable spleen 2 cm below the costal margin. Lab shows total
bilirubin =205 mg\dl, direct bilirubin = 60 mg/dl, positive direct & indirect combs test. Peripheral blood smear(attached
photo shows spherocytosis)
A. Spherocytosis i.
B. Gilbert disease j.
C. Abo incompatibility
D. Crigler najjar syndrome
Answer: should be autoimmune

111.Which of the following congenital heart disease is secondary to failure of spiral rotation of the heart septum?
A. Transposition of great artery
B. Asd
C. Vsd
D. Pda

29
Answer: a
Misalignment of the sepatient um can cause the congenital heart conditions tetralogy of fallot, persistent truncus arteriosus,
dextro-transposition of the great arteries, tricuspid atresia, and anomalous pulmonary venous connection

112.(long scenario for cerebral palsy). On examination there is crossing of lower limb when child suspend by the axilla. Which
type of cp does the patient have
A. Hemiplagia
B. Diplagia
C. Quadriplagia
D. Athetoid
Answer: c
Scissoring is seen in spastic cerebral palsy ( diaplegic & quadriplegic) (https://www.cerebralpalsy.org.au/what-is-cerebral-
palsy/types-of-cerebral-palsy/spastic-cerebral-palsy/)

113.Child admitted with sore throat and bilateral knee pain?


A. Juvenile rheumatoid arthritis
B. Rhumatoid arthritis
C. Other choices i did not remember it
D. Osgood shatler disease
E. Influenza

Answer: a
Http://www.webmd.com/rheumatoid-arthritis/understanding-juvenile-rheumatoid-arthritis-basics

114.Child eating a lot of milk but he does not eat meat, mcv hypochromic microcytic anemia , how will you manage this child?
A. Oral vitamins + iron
B. Trial of iron then then observe
C. Folic acid
D. Iron
Answer: a

115.Child complain of unilateral scrotal swelling , does not transilluminate , what is your plan ?
A. Discharge the patient
B. Give antibiotic
C. Do laparotomy
D. U/s and think about surgery

Answer: d
Http://www.merckmanuals.com/professional/genitourinary-disorders/symptoms-of-genitourinary-disorders/painless-scrotal-
mass

116.Baby sit briefly, crawl, move object from hand to hand, but can't do pincer grasp
A. 4 month
B. 6 month
C. 7 month
D. 9 month
Answer: b (toronto notes 2014 p5)

30
117.Newborn with congenital adrenal hyperplasia present with;
A. Hirsutism
B. Infantile acne
C. Abdominal striae
D. Dehydration
Answer: d
There are three main types and the presentation depends on the type nd patient gender.
Http://emedicine.medscape.com/article/919218-overview it is easier and more exam directed if you read from first aid.

118.About cyanotic heart disease.


A. Tof
B. Asd
C. Vsd
D. Pda
Answer: a
Source: master the boards: usmle step 2, second edition, pediatric page 412

119.Baby with 5 min after birth assessing ( hr 120 ,breath irregular and grasping , acrocynotic , cough and grimace , flexing all
limbs not moving ) apgar score :
A. 6
B. 7
C. 8
D. 9

Answer: c
Explanation: apgar score is assessed at 1 and 5 minutes and scored based on the table.

Reference: toronto notes 2015, page p66, pediatrics

120.8 years old girl, parent complains that she looks older than her classmates , wt and hight above the 95th percentile ,
otherwise normal :
A. Reevaluate after 12 months
B. Obesity medications
C. Life style modification
D. Surgical intervention .

Answer: c
Explanation: this is most likely due to obesity. Familial tall stature also known as constitutional tall stature is the most common
cause of tall stature. The second most common cause is nutritional. The height as well as the weight are at higher percentile.

31
Again the bone age is marginally to moderately advanced so final predicted height is not much. Nutritional tall stature is
managed by life-style changes and avoidance of bad dietary practices.
Reference: http://www.ncbi.nlm.nih.gov/pmc/articles/pmc3850425/

121.Failure of obliteration of pharyngeal arch 2,3,4 leads to?


A. Cervical cyst
B. Branchial fistula
C. Ectopic thymus
D. Parathyroid

Answer: b
Explanation: 2nd through 4th clefts form temporary cervical sinuses, which are obliterated by proliferation of 2nd arch
mesenchyme. Failure of obliteration leads to branchial cleft cyst within lateral neck.
Reference: first aid 2015, page 564

122.4 month child witch developmental milestone?


A. Try to crawling
B. Roll over from side to side
C. Sit without support
D. Complete fixation of the head
Answer: d
Explanation:

Reference kaplan step 2ck pediatrics 2014

123.Calculate glasgow coma scale: child crying and confused+ respond when calling his name+ withdraw to pain=>
A. 9
B. 10
C. 11
D. 12

Answer: c

32
124.Child football player on short acting β-agonist 5 time a week use + on zafirlukast
A. Long β agonist
B. Inhaled steroid
C. Theophylline
D. It’s okay no thing is needed

Answer: b
Explanation: the child has uncontrolled asthma for which we should add steroids to his management

125.Baby can say a few words or 3 words! I'm not sure


A. 1
B. 2
C. 3
D. 4

Answer: a
It should be 18 months
Http://www.cdc.gov/ncbddd/actearly/milestones/milestones-18mo.html

126.Child sit and support his head , laughing and cooing :


A. 4weeks
B. 6weeks
C. 8weeks
33
D. 16weeks

Answer: d
Http://www.thebump.com/a/baby-milestone-chart

127.Child his height and weight below normal besides growth hormone what you will order:
A. Somatomedin c
B. Aldosterone
C. Insulin
D. Testosterone

Answer: a
Somatomedin c, which is another name for insulin-like growth factor 1 (igf-1), while (human growth hormone) hgh levels vary
throughout the day depending on diet and activity levels, somatomedin c levels in the blood are more stable, making its
measurement a fairly reliable indicator of how much hgh the pituitary gland is producing overall.
Http://kidshealth.org/en/parents/somatomedin-test.html
Https://en.m.wikipedia.org/wiki/somatomedin

128.Rheumatic heart dx prophylactic for 10 yrs old boy with no cardic involvement?
A. 3m
B. 6m
C. 6 yrs
D. 10 yrs

Answer: d
Http://www.aafp.org/afp/2010/0201/p346.html

129.16 yr from aferica (ginia) with painless neck mass for 5 weeks developed cough , fever , urs:
A. Burkit lymphoma
B. Infectious mononucleosis !!!
C. Hugging lymphoma
D. Lym dx

Answer:a
Burkitt lymphoma, or small noncleaved cell lymphoma, is a highly aggressive b-cell non-hodgkin lymphoma characterized by the
translocation and deregulation of the c-myc gene on chromosome 8. Burkitt-like lymphoma (bll) is considered to be a
morphologic variant of burkitt lymphoma
Http://emedicine.medscape.com/article/1447602-overview
34
130.Neonate take immunoglobulin from his mother this is
A. Active artificial immunity
B. Passive artificial immunity
C. Active neutral immunity
D. Passive neutral immunity
Answer: D
Naturally acquired passive immunity can be provided during pregnancy, and through breast-feeding
Naturally acquired active immunity occurs when the person is exposed to a live pathogen, develops the disease, and becomes
immune as a result of the primary immune response. Artificially acquired active immunity can be induced by a vaccine, a
substance that contains the antigen.
Artificially acquired passive immunity is a short-term immunization by the injection of antibodies, such as gamma globulin, that
are not produced by the recipient's cells. Naturally acquired passive immunity occurs during pregnancy, in which certain
antibodies are passed from the maternal into the fetal bloodstream.
Http://www.infoplease.com/encyclopedia/science/immunity-active-passive-immunity.html
Https://en.wikipedia.org/wiki/passive_immunity

131.What measurement a you should take to relieve an infant abdominal colic?


A. Antispasmodic drugs
B. Increase bottle feeding
C. Warm baths
D. Prevent child abuse

Answer: C
Reassure the parents and drug treatment generally has no place in management of colic unless gerd appears likely; although the
anticholinergic agent dicyclomine hydrochloride is effective against colic, it has rare but serious adverse effects and cannot be
recommended. Http://emedicine.medscape.com/article/927760-overview

132. Child less than 50 percentile, polyuria, constipation, low na, k, cl, where is the primary defect?
A. Na, cl channel
B. K chanel
C. H+ reabsorption
D. H+ secretion
Answer: a

A case of barrter syndrome ; chloride is passively absorbed along most of the proximal tubule but is actively transported in the
talh and the distal convoluted tubule (dct). Failure to reabsorb chloride results in a failure to reabsorb sodium and leads to
excessive sodium and chloride (salt) delivery to the distal tubules, leading to excessive salt and water loss from the body.
Http://emedicine.medscape.com/article/238670-overview#a2

133.Child can now the color when you point at them, ride tricycle but cannot copy square what is his age ?
A. 2 years old
B. 3 years old
C. 4 years old
D. 5 years old

Answer: b or c
At the age of 3 the child can: ride a tricycle

134.Child was diagnosed with dm type 1 presented with frequent hypoglycemic attack at different time during the day, he
was diagnosed 6 month back, and he is compliant to his diet and treatment , the most likely cause of his symptoms is:
A. Brittle diabetes
B. Dawn phenomena
35
C. Smogyi phenomena
D. Honey moon period

Answer: a

Brittle diabetes is severe instability of blood glucose levels with frequent and unpredictable episodes of hypoglycemia and/or
ketoacidosis that disrupt quality of life.
Honey moon period is known to be glycemic control can be achieved with little or no insulin treatment as residual cells are still
able to produce insulin.
Dawn phenomenon and smogyi effect cause hyperglycemia.
Reference: http://www.uptodate.com/contents/the-adult-patient-with-brittle-diabetes-mellitus
+toronto notes +
Http://www.ncbi.nlm.nih.gov/pubmed/21717414

135.Which of the following is an early sign of puberty in boy


A. Pubic hair
B. Penis enlargement
C. Scrotal enlargement
D. Testicular enlargement

Answer: c or d
Many references mention that scrotal and testicular enlargement are the first sign of puberty in males.
Reference: http://www.merckmanuals.com/home/men-s-health-issues/biology-of-the-male-reproductive-system/puberty-in-
boys

136.Long scenario about child with dysmorphic feature developmental delay seizure disorder on anticonvulsant therapy
other details … (ped)
A. Ipv instead of opv
B. Deter dtp
C. Deter all live vaccine
D. Deter all vaccine
Answer:b
The contra indications of dtp and ipv are evolving unstable neurologic disease,hyporesponsive/hypotonic following previous
vaccine, anaphylactic reaction to neomycin or streptomycin.

137.Tonsilitis case:
A. Amoxicillin / clavilonic
B. Vancomycin
C. Ciprofloxacillin
D. Trimethoprime / sulpha
Answer: a
To treat tonsillitis penicillin v or amoxicillin or erythromycin (if penicillin allergy) x 10 d are used
Reference: toronto notes

138.What is the milestone of 4 year child ?


A. 40 - 60 word
B. Count to 4 and
C. Tell story
D. Say mama baba

36
Answer: c
Reference: http://www.cdc.gov/ncbddd/actearly/milestones/milestones-4yr.html

139.10 years old child with difficulty to eat for 2 years , multiple time aspiration in the past two weeks , on total parenteral
nutrition 1000 calorie and protein daily with lab result attached (low albumin and anaemia ) what is your action ?
A. Continue same tpn dose
B. Continue tpn with increase dose to 2000
C. Insert gastrostomy tube, same calorie
D. Insert gastrostomy tube decrease calorie to 80

Answer:b

Since the patient is complaining of aspiration; gastronomy can cause aspiration as a complication. Since the patient is anemic
and has hypoalbunimia continuing the same dose is wrong.
Reference: toronto notes

At the age of 4 the child can: name the colors and copy a square

References : kaplan usmle step 2 pediatric and http://www.cdc.gov/ncbddd/actearly/milestones/

140.Child with red urine and constipation. Urine analysis increase rbcs, wbcs, protein
A. Uti
B. Hsp
C. Hemolytic uremic
D. Post streptococcus glomurenephritis
Answer: d

37
In post streptococcus glomurenephritis urinalysis—rbcs, rbc casts, protein 1–2 +, polymorphonuclear cells.
References: toronto notes and kaplan usmle step 2 pediatric

141.12 year old girl htn:


A. 120/80
B. 40/99
C. Above 90 percentile
D. Above 95 percentile

Answer: d
Stage i hypertension is diagnosed if a child’s bp is greater than the 95th percentile but less than or equal to the 99th percentile
plus 5 mm hg. Stage ii hypertension is diagnosed if a child’s bp is greater than the 99th percentile plus 5 mm hg.
Reference: http://emedicine.medscape.com/article/889877-overview

142.Young , vesicle ,pustule on back like a band :


A. Shingles
B. Chicken box
C. Herpes
D. Coxsackievirus

Answer: a
Herpes zoster is primarily a disease of adults and typically begins with pain and paresthesia in a dermatomal or bandlike pattern
followed by grouped vesicles within the dermatome several days later
Reference: http://www.clevelandclinicmeded.com/medicalpubs/diseasemanagement/dermatology/common-skin-infections/

143.Newborn with congenital adrenal hyperplasia present with;


A. Hirsutism
B. Infantile acne
C. Abdominal striae
D. Dehydration
Answer: D

38
rd
Reference: step up to medicine 3 edition.

144.Baby with vomiting and diarrhea , looks ill , cries with tears , capillary refill 3 sec , vital signs were normal almost :
A. Moderate dehydration give something 10 ml infusion
B. Moderate dehydration give …
C. Sever dehydration give …
D. Sever dehydration give another something (couldn't remember the exact sentences but these were the choices )

Answer: b? (question and choices incomplete)


Explanation: patient most likely has moderate dehydration which is usually treated using oral rehydration solution. Mild
dehydration: minimal findings: may have slightly dry buccal mucous membranes, thirst, slightly decreased urine output.
Moderate: similar symptoms with increased severity and tachycardia, little or no urine output, lethargy, sunken eyes and
fontanelles, loss of skin turgor. Severe: more severe symptoms, rapid, thready pulse; no tears; cyanosis; rapid breathing; delayed
capillary refill; hypotension; mottled skin; coma. For mild/moderate: oral rehydration. For severe or failed oral: iv isotonic bolus
20 ml/kg.
Reference: http://www.merckmanuals.com/professional/pediatrics/dehydration-and-fluid-therapy-in-children/dehydration-in-
children

145.A child that can raise his head slightly when prone and smiles. He turns his
Head 180 degrees and has head lag when you pull him to sit. How many old is
He?
A. 4 weeks
B. 8 weeks
C. 12 weeks
D. 16 weeks
Answer : b

146.Case: mother come to doctor because worry about her child my be had dehydration what is most tool exam of child ?
A. History
B. Clinical
C. C.t
D. Ultrasound

147.5 years unwell with lower limb ecchymosis, positive cd10 (calla)?
A. Aml
B. All
C. Hodgkin
D. Ebv

39
Answer: b
cd10 also called common acute lymphoblastic leukemia antigen (calla), one of first markers to identify leukemic cells in children
(hence its name)
found on all cells which derive from pre-b lymphocytes
http://www.pathologyoutlines.com/topic/cdmarkerscd10.html
http://www.ncbi.nlm.nih.gov/pubmed/2957002

148.Child with stridor, slightly relieved by epinephrine what will you do next?
A. Steroids
B. Antihistamines
C. Antibiotics
D. Bronchodilators

Answer: a definitive treatment is treating the underlying disorder but emergency care involves: a mixture of helium and
o2 (heliox), nebulized epinephrine, and dexamethasone (10 mg iv, then 4 mg iv q 6 h) may be helpful in patients in whom airway
edema is the cause.
Http://www.merckmanuals.com/professional/pulmonary-disorders/symptoms-of-pulmonary-disorders/stridor

149.10 months, set without support, can't walk or crawl, say mom to hold him , grip pen what is the delay:
A. Socially
B. Fine motor
C. Gross motor
D. Speech
Answer: c
Http://image.slidesharecdn.com/doc1-141125004358-conversion-gate01/95/nelsons-developmental-milesotones-table-1-
638.jpg?Cb=1416876339

150.Child can hold his head when you pull him up, looks at his hands, laughs, his age in weeks
A. 4
B. 8
C. 12
D. 14
Answer: d (4 months)
Http://image.slidesharecdn.com/doc1-141125004358-conversion-gate01/95/nelsons-developmental-milesotones-table-1-
638.jpg?Cb=1416876339

151.Child who can understand few commands, points to what he needs, walks without support, crawl upstairs, his age in
months
A. 6
B. 9
C. 15
D. 24
Answer: d (24 months)
Http://image.slidesharecdn.com/doc1-141125004358-conversion-gate01/95/nelsons-developmental-milesotones-table-1-
638.jpg?Cb=1416876339

152.The fastest way to clean child stomach with iron overdose?


A. Gastric lavage
B. Ipecac syrup (not recommended by medscape)
C. Enema something ?
D. ?

40
Answer: decontamination trough ng lavage and whole bowel irrigation (with polyethylene glycol) then antidote (for severe
cases) by deferoxamine.
Http://www.rch.org.au/clinicalguide/guideline_index/iron_poisoning/
step up pediatrics + merck manual

153.Patient increase foot size 39 >> 41.5 and increase size of hand and joint which hormone?
A. Thyroxine
B. Prolactin
C. Acth
D. Somatotropin hormone “ known as growth hormone”
Answer: d
Explanation: all other answers are not applicable for above description.

154.Case of intussusception) child came with colicky abdominal pain, vomiting, and bloody stool. Us showed doughnut sign.
What is the most important step in management of this case?
A. Urgent surgery referral
B. Ngt decompression
C. Iv fluid resuscitation
D. Barium enema
Answer: c- iv fluid resuscitation (ref. Master the board)

155.A one month old child with total bilirubin of 200 and direct bilirubin of 80. What's the cause?
A. Gilbert syndrome
B. Crigler-najjar syndrome
C. Choledocal cyst
D. Abo incompatibility
Answer: c
All options are examples of unconjugated hyperbilirubinemia except for choledocal cyst. It presents as jaundice, acholic stools in
early infancy and palpable mass in right upper quadrant with hepatomegaly.
Http://emedicine.medscape.com/article/172099-overview

156.Child with eczema use topical steroid not affected what we add:
A. Oral antibiotic
B. Topical antibiotic
C. Sulfa something
D. Tacrolimus
Answer: d
Reference: http://emedicine.medscape.com/article/1049085-treatment

157.Minimal probability of a baby getting beta thalassemia from carrier parents? 0


A. 0%
B. 25%,
C. 50%,
D. 75%,
E. 100%
Answer: b

41
Carriers of thalassemia minor are usually clinically asymptomatic but sometimes have a mild anemia. When both parents are
carriers there is a 25% risk at each pregnancy of having children with homozygous thalassemia.
Http://www.ncbi.nlm.nih.gov/pmc/articles/pmc2893117/

158.A child complains of bilateral knee swelling, fever and pharyngitis, “all at the same time i think”. His labs show high esr
with no other abnormality. What is the most likely diagnosis?
A. Juvenile ra,
B. Septic arthritis,
C. Acute rheumatic fever,
D. Infectious mono
Answer: c
Acute rheumatic fever is diagnosed according to modified jones criteria: 2 major or 1 major 2 minor plus evidence of preceding
step infection (history of scarlet fever, group a strep pharyngitis culture, positive rapid antigen detection test, anti-streptolysin o
titers)
Major criteria Minor criteria
Sydenham chorea Prolonged pr interval
Transient migratory arthritis Elevated esr
Rheumatic subcutaneous nodules Arthralgia
Erythema marginatum Elvated crp
Panacrditis Fever
Toronto notes 2015

159.Child with barking cough and another sign indicate respiratory infection. What cause of this patient
A. No croups or laryngotracheitis in answers
B. Pertussis
C. Epiglottitis
D. ????
Answer: c
Barking cough is classic for croup which presents along with stridor. Pertussis presents with a whooping cough. In epiglottitis the
patient look toxic, in sniffing position and drooling. Barking cough differential diagnosis includes epiglottitis.
Toronto notes 2015
Http://emedicine.medscape.com/article/962972-differential
42
160.Regarding pals, which is true in pediatric resuscitation?
A. 30 compression 2 breath
B. 15 compression 1 breath
C. 10 compression 2 breath
D. 5 compression 1 breath
Answer: a
The ratio is 30:2 if single rescuer and 15:2 if two rescuers.

161.Four years old child c/o fever and maculopapular rash associated with auricular and occipital lymph node, he only
received his birth vaccines. Dx?
A. Mumps
B. Measles
C. Rubella
D. Chicken pox
Answer: c
Rubella clinical picture is asymptomatic infection, mild coryza, erythematous discrete skin rashes usually fade after 3 days,
lymphadenopathy (most common) sub-occipital, post-auricular, and cervical. +\- splenomegaly.
Reference: manual of clinical pediatrics.

162.15 years old male come for routine check up, labs hb: 10 mcv: 69 mch: 20 wbc: normal. Diagnosis?
A. Ida
B. Beta thalasemia trait
C. Anemia of chronic disease
D. Scd
Answer: b
The patient is having hypochromic microcytic anemia.

163.4-year-old child, language development


A- Can say 50-100 (i believe he is too old for that)
B- 2 phrase / words sentences (2 years old can do that)

43
C- Use past tense (why not) >> my answer i went with the rule (don’t underestimate a child power)
D- Tell a story
Answer: d
Link:
Http://www.asha.org/public/speech/development/45/

164.Neonate + prolonged bleeding after circumcision, aptt high, pt, bleeding time and platelet are normal, condition is most
likely due to deficiency in
A. V
B. Vii
C. Viii
D. X
Answer: c

165.Milestone, baby pull himself to stand crawl without difficulty, which age
A. 8 months
B. 10 months
C. 12 months
D. 14 months
Answer: b

166.Which one of these disease likely to exhibit cyanosis in later life?


A. 6 year with coartcation of aorta (lower limbs only)
B. Vsd (reversal of the shunt occurs early in childhood)
C. Trancus arteriosis (early cyanotic)
D. Asd (reversal of the shunt occurs late in adulthood)
Answer: d
Cyanosis in patients with asd is usually associated with either concomitant pulmonary valve stenosis resulting in elevated right
heart pressures, and thus right-to-left shunt, or eisenmenger syndrome. In addition, transient reversal of the atrial pressure
gradient and transient cyanosis can be induced by respiratory maneuvers such as valsalva and cough.

167.Clear case of dka in a child what's the initial management?


A. Electrolytes replacement
B. Fluid replacement
C. Bicharb
D. Insulin infusion
Answer: b

44
168.Child with symptoms of dka abg ph 7.24 pco2 lower than normal hco3 lower than normal. What is it?
A. Compensated metabolic acidosis
B. Compensated metabolic alkalosis
C. Uncompensated metabolic acidosis
D. Uncompensated metabolic alkalosis
Answer: a

169.15 month baby lp show gram- positive cocci in chain. What antibiotics?
A. Vancomycin alone
B. Ampicillin alone
C. Ceftriaxone and vancomycin
D. Ampicillin and gentamicin
Answer: c

170.Cow milk over breast milk more in what?


A. Fat.
B. Calorie.
C. Protein.
D. Iron
Answer: c
All mammalian species produce milk, but the composition of milk for each species varies widely and other kinds of milk are often
very different from human breast milk. As a rule, the milk of mammals that nurse frequently (including human babies) is less
rich, or more watery, than the milk of mammals whose young nurse less often. Human milk is noticeably thinner and sweeter
than cow's milk.
Whole cow's milk contains too little iron, retinol, vitamin e, vitamin c, vitamin d, unsaturated fats or essential fatty acids for
]
human babies. Whole cow's milk also contains too much protein, sodium, potassium, phosphorus and chloride which may put a
strain on an infant's immature kidneys. In addition, the proteins, fats and calcium in whole cow's milk are more difficult for an
[52][55][56]
infant to digest and absorb than the ones in breast milk. evaporated milk may be easier to digest due to the processing
of the protein but is still nutritionally inadequate. Some infants are allergic to cow's milk protein, this problem affecting infant
formulas derived from cow's milk.

171.Hypertension in child ?
A. More than 120/70
B. More than 140/90
C. More than 90th percentile.
th
D. More than 95 percentile.
Answer: d
●normal bp – both systolic and diastolic bp <90th percentile.
●prehypertension – systolic and/or diastolic bp ≥90th percentile but <95th percentile or if bp exceeds 120/80 mmhg (even if
<90th percentile for age, gender, and height).
●hypertension – hypertension (htn) is defined as either systolic and/or diastolic bp ≥95th percentile measured on three or more
separate occasions. The degree of htn is further delineated by the two following stages.
•stage 1 htn – systolic and/or diastolic bp between the 95th percentile and 5 mmhg above the 99th percentile.
•stage 2 htn – systolic and/or diastolic bp ≥99th percentile plus 5 mmhg.

172.4 weeks old infant, mother happy he never cries (i.e. He is lethargic). On examination: jaundice + umbilical hernia +
distended abdomen + coarse face features + bulging frontal fontanel. Diagnosis?
A. Congenetal hypthyroidism
B. Gilbert's syndrome
C. Cerebral palsy
D. Rickets

45
Answer: a

Congenital hypothyroidism:
The vast majority (more than 95 percent) of infants with congenital hypothyroidism have few if any clinical manifestations of
hypothyroidism at birth. This is because some maternal thyroxine (t4) crosses the placenta, so that even in infants who cannot
make any thyroid hormone, umbilical cord serum t4 concentrations are about 25 to 50 percent of those of normal infants. In
addition, many infants with congenital hypothyroidism have some, albeit inadequate, functioning thyroid tissue.

173.Q about cerebral palsy with typical feature patient had spastic paralysis of all limbs except upper limbs had less paralysis.
What type of cp the baby had:
A. Mixed
B. Diplegic
C. Hemiplegic
D. Quadriplegic
Answer: b
Http://www.michigancerebralpalsyattorneys.com/about-cerebral-palsy/

174.Baby diagnosed with cystic fibrosis. ..he has + sweat chloride test his brother is normal, to confirm diagnosis of cystic
fibrosis?
A. Ctfr gene in parent
B. Ctfr in sibling
C. Chloride test. .parent
D. Chloride test in sibling
Answer: a

175.Child with dehydration, depressed anterior fontanel, and decreased skin turgor. What is the percentage of dehydration?
A. 5
B. 10
C. 15
D. 20
Answer: c

Medscape

176.The most common drug used to treat juveniles rheumatoid arthritis is:
A. Paracetamol
B. Penicillamine
C. Systemic steroid
D. Aspirin
Answer:?
The answer should be any nsaid. Aspirin used to be the most common but aspirin is no longer the drug of first choice because of
the increased frequency of gastric toxicity and hepatotoxicity when compared to other nsaid medications.
Http://emedicine.medscape.com/article/1007276-medication - showall
46
177.Child with aspirin intake overdose ...what kind of acid base balance:
A. Metabolic alkalosis
B. Metabolic acidosis
C. Respiratory alkalosis
D. Respiratory acidosis
Answer: b
Acid-base disorders in salicylate toxicity adults: metabolic acidosis and respiratory alkalosis
children: metabolic acidosis
if fasting=>starvation ketosis may develop
salicylates directly stimulate the respiratory center to cause hyperventilation (respiratory alkalosis) which is dose-dependent.
This stimulation is much more pronounced in adults than in children.
Http://www.anaesthesiamcq.com/acidbasebook/ab8_6c.php
Initial respiratory alkalosis followed by metabolic acidosis
The most common abnormality, especially in adults, is a mixed acid-base disturbance (a primary respiratory alkalosis plus a
primary metabolic acidosis)

178.Kid dm type 1 brought to school clinic unconscious , last insulin dose unknown what to do?
A. Iv dextrose
B. Rl
C. Insulin
D. Urgent referral to hospital
In the school clinic i.v set could be available :-/ so i.v dextrose 25%

179.Pneumococcal 13 conj type


A. Toxoid
B. Conjogoid
C. Inactivated
D. Live
Answer: b ?
Reference: kaplan pediatrics
Explanation: it’s inactivated; polysaccharide based, and conjugated vaccine!!

180.4m old baby for vaccine she said at 2m he had fever and erythema and swelling
A. Give 4m same dose
B. Allergic test
C. Not give dtp
D. Antipyretic and antihistamine prophlactic
Answer: (give the vaccine of 4m as it is , after vaccine for local reaction give cool compressor with acetaminophen or nsaids ,
antipyretic if developed fever.)

181.Child who's lethargic and losing his concentration, Hgb is 10.5, what to give?
A. IM iron.
B. Oral ferrous sulphate.
C. Fortified cereal.
Answer: B
The patient is symptomatic, and the initial treatment for IDA is oral Ferrous Sulfate
Http://www.fpnotebook.com/hemeonc/peds/pdtrcanm.htm

182.8 y/o girl came with her father on wheel chair with pain in her knee. On examination there was limited flexion and
extension, they didn't mention any thing about knee effusion.
BP 105/58
HR: 101
47
Temp: 39.6
RR: 27
How to diagnose?
A. RF
B. ASO
C. Arthrocentesis
Answer: B
Anti-streptolysin O (ASO) titre and throat swab should be considered if rheumatic fever is possible (Jones criteria). Otherwise,
arthrocentesis should be considered if septic arthritis is possible.

183.A child can roll over, sit triploid, attempt to take object. Which month?
A. 6
B. 9
C. 2
Answer: A

Reference Kaplan pediatrics lecture notes

184.Angular chelosis is due to deficiency in what vitamin?


A. B12
B. B6
C. B2
Answer: C
Explanation: B2 deficiency manifests as angular chelosis, stomatitis and glossitis. B12 deficiency manifests as megaloblastic
anemia and peripheral neuropathy. B6 deficiency also manifests as peripheral neuropathy.
Reference Master the board

185.Treatment of post-streptococcal glomerulonephritis in children with edema and HTN?


A. High dose of antibiotic
B. Diuretic for edema
C. Diuretic for HTN
Answer: B&C
The mainstay of treatment of Poststreptococcal Glomerulonephritis is symptomatic treatment starting with salt and water
restriction then loop diuretics to control both the edema and the HTN.
Reference: http://emedicine.medscape.com/article/240337-treatment

186.Surgeon want to treat child diagnosed with PDA. During procedure, the surgeon is at risk to injure?
A. Left Phrenic Nerve
B. Vagus Nerve
C. Left Recurrent Laryngeal Nerve

Answer: C
Https://www.openanesthesia.org/pda_repair_recurrent_laryng_n_injury/

187.Fluid required to give a 10 Kg neonate, loss 5% of his body weight?


A. 1000
B. 1500
C. 2500
Answer: B
Deficit: 5% so (50 ml/kg) = 500 ml
Maintenance: (100ml / kg / 24h for the first 10 kg) = 1000 ml

48
Total: 1500 ml
Nelson Essentials page: 108:
The first step in caring for a dehydrated child is to assess the degree of dehydration, which dictates the urgency of the situation
AND the volume of fluid needed for rehydration.
Mild Moderate Severe
Infant 5% of body wt 10% 15%
Older child-adolescent 3% 6% 9%

188.6 years old with blood pressure above 95th percentile - +ve fhx of HTN - radial pulse intact femoral pulse is absent ( cause
)
A. Essential hypertension
B. Renal artery stenosis
C. Coarcotation of the aorta

Answer: C
Http://emedicine.medscape.com/article/895502-clinical#b4

189.A young boy presented with diarrhea sometimes bloody , Wight loss , arthritis , anemia the diagnosis is :
A. Crohn’s
B. UC
C. Celiac
Answer: B
((Step Up to Medicine, 4E, p164 / davidson's medicine ,20 E, P462))

190.1 month Baby come with abdominal distension and constipation since birth what would you do next :
A. X-ray
B. Biobsy
C. Rectal manometrey
Answer: I think the diagnosis is Hirschsprung's disease and barium enema is the imaging study of choice ((First Aid for the
USMLE Step 2 CK ,8Ed,p368 ))

191.Pediatric patient known case of Asthma since age 2 years. He is now asymptomatic with rare uses of albuterol inhalers,
he came for counseling, he had contact sport participation, what is the best question to ask to know his response:
A. “Are keeping up with your friends”
B. “How frequent do use inhaler”
C. “Presence night symptoms (cough)”
Answer: C

192.12-hour newborn developed jaundice. Which investigation will you order?


A. Hb electrophoresis
B. Osmotic fragility test
C. G6PD screening
Answer: C

193.1 month Baby come with abdominal distension and constipation since brith what you do next
A. Xray
B. Biobsy
C. Rectal manometrey
Plain x.ray is the most important initial test in investigating child with constipation

49
194.8 year child come with 30 BMI what can you do for him
A. Surgery for obesity
B. Give him drug for obesity
C. Ressure and Live style
Life style modification and multidisciplinary approach is most important step in treating obesity in pediatric population

195.Peadiatric case croup , from where take swap ?


A. Naophangeal swap
B. Nasal swap
C. Phangeal swap
Answer: A

196.Child in the hospital play and come to his parents say stories , draw head and hands and legs ‫الرسمه اللي فيها ارجل وايدي بشكل‬
‫ خطوط‬what is the age of this child:
A. 3yrs
B. 4yrs
C. 5yrs
Answer: C

197.Neonate with sign of sepsis which empirical antibiotic :


A. Ampicillin
B. Gentamicin
C. Cefotaxime
Answer: C?

198.Child in the school K/c of DM loss of copiousness , last insulin dose not known , what should you do:
A. Give IV Dextrose ‫في ارقام بس نسيتهم‬
B. SC insulin
C. Urgent transfer to hospital
Answer: A

199.Child is complaining of severe throbbing unilateral headache, aggravated by light. What is the most likely diagnosis:
A. Migraine
B. Cluster headache
C. Stress headache

Answer: A

200.An adolescent boy came to the clinic with unilateral gynecomastia. No other complaints. Everything was normal. How
will you manage?
A. Reassure
B. Give hormonal therapy
C. Breast us

Answer: a
Gynecomastia (transient development of breast tissue) is a common self-limited condition seen in 50% of male during puberty
(but any discharge from nipple or fixed mass should be investigated). Reference: toronto notes.

201.Months old infant his parents were not able to bring him for his 4 months vaccination. What will you do?
A. Arrange for 4 months vaccination
B. Give missed vaccination and next appointment
C. Give vaccine together during next appointment

Answer: ? Give missed vaccination (4 months) and arrange for the a next appointment (for the 6 months vaccination)
50
Children who have received some of their vaccines shots and then fallen behind schedule can catch up without having to start
over. Reference: http://www.medicinenet.com/childhood_vaccination_schedule/page3.htm

202.14 years old boy comes to your clinic with swollen lips. He has similar episodes since 3 or 5 years. Deficiency of which of
the following caused his presentation?
A. Factor d
B. Anaphylatoxin inhibitor
C. C1 esterase inhibitor

Answer: c
Hereditary angioedema is an autosomal dominant disease caused by low levels of the plasma proteins c1 inhibitor (c1-inh).
th
Reference: medscape. Nelson 7

203.2 weeks neonate passed unformed stool. What will you do?
A. Prescribe formula milk.
B. Give oral rehydration solution.
C. Prescribe lactose-free milk.

Answer: b
th
Reference: rome iii criteria for pediatric functional gastrointestinal syndrome, nelson 7 p:421

51
204.Pediatric patient with classical symptoms of diabetes + elevated blood glucose . What will you do next?
a. Urine dipstick
b. Genetic testing
c. Hba1c

Answer: a
Urine dipstick to pick up dka early and to prevent further complications.
Reference: http://guidelines.diabetes.ca/browse/chapter34

205.Baby with tachypnea, cough, hemoptysis and bilateral lung infiltrates. What is the treatment?
A. Steroid
B. Antibiotic
C. Surgery

Answer: b
N.b : the child may present with life-threatening hemorrhage (>8 ml/kg every 24 hours or 200 ml every 24 hrs). Massive
hemoptysis can quickly progress to acute respiratory distress in a child. These children require multiple procedures to stabilize
the airways and to control blood loss. Intravenous fluids and blood products are given to prevent cardiovascular collapse.
Reference: hemoptysis in children; see: http://medind.nic.in/ibv/t10/i3/ibvt10i3p245.pdf

206.Which of the following is a cyanotic heart disease in children?


A. Asd
B. Vsd
C. Tetralogy of fallot
Answer: c

52
th
Reference: nelson 7 p: 495

207.A 6 years child of positive HBV mother not taken any vaccine except BCG after delivery. What will you give him:
A. MMR,OPV,HBV,Varicella
B. HIB, MMR,OPV,HBV,PCV
C. HIB, MMR,OPV,HBV,….
Answer= All can be given.
Http://www.cdc.gov/vaccines/schedules/hcp/imz/catchup-shell.html

208.Image of cells for a 2 years old with pancytopenia and something else. What is the most likely diagnosis?
A. Malaria
B. Leishmaniasis
C. Leukemia

Answer: ? Depends on the image.


209.A 2 years old child can run and play ball. Socially active but he refused to share his toys with other children. What you will
say to his parents?
A. Delay motor
B. Social impairment
C. Well developed and normal response
53
Answer: c

210.2 month boy present with 2 cm of hemangioma in the back .. Wt is the ttt
A. Close f/u **
B. Excision
C. Beta blocker
]
Answer: the vast majority of infantile hemangiomas do not require any medical or surgical intervention. Treatment options for
clinically significant hemangiomas include the following:
• Laser surgery
• Surgical excision
• Medication
Http://emedicine.medscape.com/article/1083849-overview

211.20 days infant diagnosis as meningitis, his culture show gram negative bacilli. Which of following could be the organism?
A. Hemophiles influenza
B. E.coli
C. Neisseria meningitides
Answer: b
Note: group b streptococci (gbs) are the most commonly identified causes of bacterial meningitis, implicated in roughly 50% of
all cases. Escherichia coli accounts for another 20%. Thus, identification and treatment of maternal genitourinary infections is an
important prevention strategy.[5] listeria monocytogenes is the third most common pathogen, accounting for 5-10% of cases; it
is unique in that it exhibits transplacental transmission.[6]

- n . Meningitides it is gram negative diplococcus


- h . Influenza it is gram negative coccobacili
Ref : medscape

212.Patient with ostium secundum atrial septal defect. What you will see in his ecg?
A. Prolonged pr interval
B. Left axis deviation
C. Right axis deviation
Answer c
Http://emedicine.medscape.com/article/890991-workup#c6 an ecg demonstrates sinus rhythm, often with evidence of right
atrial enlargement manifested by tall, peaked p waves (usually best seen in leads ii and v2) and prolongation of the pr interval.
The qrs axis is slightly directed to the right (+100º), and the precordial leads reveal right ventricular enlargement of the so-called
volume overload type that is characterized by an rsr' pattern in leads v3 r and v1 with normal t waves.
The qrs duration may be mildly prolonged because of right ventricular dilation. This mimics the finding in right ventricular
conduction delay. A significant proportion (20-40%) of children with secundum atrial septal defect may not have abnormal ecg
[14]
findings. Uncommonly, a patient with a secundum atrial septal defect may demonstrate a superior qrs axis with right
ventricular enlargement, mimicking findings observed in the ecg of a patient with an ostium primum atrial septal defect

213.Best way to take urine sample:


A. Midstream
B. Foley catheter
C. Bag
Answer: midstream clean catch
Mid-stream urine sample means that you don’t collect the first or last part of urine that comes out. This reduces the risk of the
sample being contaminated with bacteria from:
54
• Your hands
• The skin around the urethra (tube that carries urine out of the body)

214.7month baby with you discover that the baby has vsd and asymptomatic otherwise healthy. What are you going to do?
A. Close observation
B. Surgery
C. Follow up after 6 months
Answer: follow up after 6 months
- no intervention is usually required for patients with small defects. These patients are typically asymptomatic and have a
reasonable expectation of spontaneous closure or decrease in the size of the defect over time.
-patients who continue to have a murmur, but are otherwise asymptomatic and growing well at the 8- to 10-week visit, are seen
again by the pediatric cardiologist at approximately 12 months of age.\
-if the murmur persists at the 12-month and the patient remains asymptomatic and clinically stable, no further intervention is
required. Echo follow-up is typically performed at three years of age for patients with membranous defects. In those with a
muscular defect, no echo is required if the patient remains asymptomatic.
-asymptomatic patients with residual small defects are usually followed every two to five years for overall assessment.
Http://www.uptodate.com/contents/management-of-isolated-ventricular-septal-defects-in-infants-and-children

215.Patient with bronchiolitis ..what is the organism ?


A. Adenovirus
B. Rsv
C. Parainfelunza
Answer: rsv
Bronchiolitis is typically caused by a virus. Respiratory syncytial virus (rsv) is the most common cause.

Http://www.uptodate.com/contents/bronchiolitis-and-rsv-in-infants-and-children-beyond-the-basics

216.8-month child breastfeed for 6 months normally, after he drank fruit juice he became sick and jaundice what should he
avoid?
A. Fructose
B. Galactose
C. Phenylalanine

Answer: a
Https://www.nlm.nih.gov/medlineplus/ency/article/000359.htm
Fructose: fruit sugar
Galactose: (composed of same elements as glucose): milk sugar

217.Infant paraumbilical hernia, what you will do?


A. Put plastic in mid abdomen
B. Reassurance
C. Do hernia repair before start school
Answer: c
Http://pedsurg.ucsf.edu/conditions--procedures/umblical-hernia.aspx
"conservative treatment is indicated under the age of two years when the hernia is symptomless. Parental reassurance is all that
is necessary. Ninety-five per cent of hernias will resolve spontaneously. If the hernia persists beyond the age of two years it is
unlikely to resolve and surgical repair is indicated." on the treatment of umbilical hernia in children (bailey & love's)

55
218.Neonate with deafness, rash .. What the cause ?
A. Rubella
B. Measles
C. Cmv

Answer: a
Ref: https://pedclerk.bsd.uchicago.edu/page/torch-infections
Rubella infection manifestation:
“blueberry muffin” rash due to extramedullary hematopoiesis
Cataracts
“salt and pepper” retinopathy
Radiolucent bone disease (long bones)
Ugr, glaucoma, hearing loss, pulmonic stenosis, patent ductus arteriosus, lymphadenopathy, jaundice,
hepatosplenomegaly, thrombocytopenia, interstitial pneumonitis, diabetes mellitus

219.What is characterized about x fragile syndrome?


A. Obesity
B. Macrogonadisim
C. Smal
Answer: fragile x syndrome (frax):
1 in 200 in frequency.
Most common cause inherited mental retardation.
Features:
Characteristic craniofacial finding (large head, prominent forehead, jaw, ear)
Characteristic neurobehavioral profile including (mental retardation, autism spectrum disorder, pervasive developmental
disorder)
Macro-orchidism
Mild connective tissue disorder including (joint laxity, patulous eustachian tubes, mitral valve prolapse)
reference : essential nelson of pediatrics

220.What come with turner syndrome?


A. Hypothyroid
B. Dm
C. Addison's
Answer: a
(reference: nelson essential of pediatric)

221.A baby 6 month show regurgitation after every meal he esophagus ph is low he is normally developing what is the rx?
A. Close follow up
B. Surgical fundal
C. Esophageal manometry
Answer:(no treatment is required may be a because i think this is a case of normal physiological gastroesophageal reflux which
happen in baby younger than 8months & presented with effortless regurgitation but otherwise the baby is normal
reference: nelson

222.What is the most common congenital abnormally cause infective endocarditis ?


A. Asd
B. Vsd
C. Tetralogy of fallout
56
Answer: (may be c because according to the references the cause of ie is unrepaired cyanotic congenital heart disease
referece: nelson essential + webs

223.Patient can't take bcg vaccine because he deficiency in


A. Il
B. Tnf gama
C. Inf gama
Answer: c

224.9-month old baby cannot sit by himself he is fisting his hand and crossing his leg most likely his presentation of?
A. Normal child
B. Cp
C. Down syndrome
Answer: b
Signs of hypertonia include keeping the hands fisted, keeping the legs extended and crossing the legs or ankles.
-down syndrome >> hypotonia

225.Child can support his head when sit and loving when stare to him or cooing
A. 4wk
B. 8wk
C. 12month
Answer: b

226.Developmental milestones q: 5 words, hop on one leg ?


A. 48 m
B. 36 m
C. 24 m
Answer: a

227.The most part in impaction of foreign body in the:


A. Left bronchus
B. Right bronchus
C. Bifurcation
Answer: b

228.Boy with glomerulonephritis after week he developed hemoptysis :


A. Heno choinlin purpra
B. Good pasture syndr.
C. Rapid deterotion
Answer: b

229.Child history of hip pain x-ray of hip shows effusion what is the next step of management?
A. Aspiration
B. Antibiotic
C. Us

Answer: a
Ultra-sound guided hip aspiration to rule out other causes, since transient synovitis is a diagnosis of exclusion.

57
Http://www.orthobullets.com/pediatrics/4030/transient-synovitis-of-hip

230.3 years old his parents has tb as a pediatrician you did ppd test after 72 hr you find a10mm enduration in the child this
suggest
A. Inconclusive result
B. Weak positive result
C. Strong positive result
Answer : c

Reference patient.info website

231.Child with high-grade fever for 5 days and sore throat ,on examination there was tonsillitis and white patches on the
gingiva. No ln enlargement, aso is negative. The most likely causative organism is:
A. Coxsackievirus.
B. Herpes simplex virus.
C. Ebv.
Answer : b
Acute herpetic gingivostomatitis
This is a manifestation of primary hsv-1 infection that occurs in children aged 6 months to 5 years. Adults may also develop
[5]
acute gingivostomatitis, but it is less severe and is associated more often with a posterior pharyngitis.
Infected saliva from an adult or another child is the mode of infection. The incubation period is 3-6 days.clinical features include
the following:
• Abrupt onset

58
• High temperature (102-104°f)
• Anorexia and listlessness
• Gingivitis (this is the most striking feature, with markedly swollen, erythematous, friable gums.)
• Vesicular lesions (these develop on the oral mucosa, tongue, and lips and later rupture and coalesce, leaving ulcerated
plaques.)
• Tender regional lymphadenopathy
• Perioral skin involvement due to contamination with infected saliva
Course: acute herpetic gingivostomatitis lasts 5-7 days, and the symptoms subside in 2 weeks. Viral shedding from the saliva
may continue for 3 weeks or more.
Medescape

232.A case of a child with delayed developmental milestones flat nasal bridge wide forehead short hand and feet what is the
cause
A. Genetic
B. Dietry insufeciency
C. Metabolism disorder
Answer : a
Genetic

Https://quizlet.com/17898748/paediatric-syndromes-flash-cards/

233.15 month old child brought by the parent with peripheral and central cyanosis, He was diagnosed with cardiac problem
but the parent doesn't know what exactly, most likely diagnosis?

A. Tetralogy of fallot
B. Pda
C. Coaratacation of the aorta

Answer: a
Explanation: this is a baby with a cyanotic heart disease which includes most commonly tetralogy of fallot, tricuspid atresia,
pulmonary atresia, and severe forms of ebstein's anomaly of the tricuspid valve. Acyanotic heart diseases include asd, vsd, and
pda.
Source: http://pediatricheartspecialists.com/articles/detail/cyanotic_heart_defects

234.Child abused sexually, in vaginal examination hymen rupture in which of the following indicate rape?
A. 6 o'clock
B. 8 o'clock
C. 3 o'clock
Answer: a
Source: forensic book.
Https://books.google.com.sa/books?Id=clemgip2794c&pg=pa210&lpg=pa210&dq=position+of+hymen+rupture&source=bl&ots
=gs4sigf2xj&sig=tmyzqialhewueohptxvma_badl0&hl=en&sa=x&ved=0ahukewi7t4f96tbjahwkcbokhwgwapcq6aeiodae#v=onepag
e&q&f=false

235.Medical director discovered cretinism in 90% of children in his village, when he analyzed the water he found that it is
deficient in iodine. The director wants to prevent and manage (the question was asking about how to treat them)
cretinism. What he is going to do initially?
A. Iodine supplementation
B. Thyroxin supplement (levothyroxine).
C. Tsh and t4 in 2 weeks.
Answer: b
Explanation: initial management should be supplementation of thyroxin, if the question was asking about a definitive solution
then we may choose to supplement water sources with iodine, however they are asking about initial management.

59
236.What is the most common cause of hearing loss in children?
A. Eustachian tube dysfunction
B. Prenatal maternal infection
C. Otitis media with secretion

Answer : most probably c


source: genetic defects are the most common cause of hearing impairment in newborns.
Ear infections, including secretory otitis media , and accumulation of earwax are the most common causes of hearing
impairment in infants and older children.
In older children, other causes include head injury, loud noise (including loud music), use of aminoglycoside antibiotics (such as
gentamicin) or thiazide diuretics, certain viral infections (such as mumps), tumors or injuries that damage the auditory nerve,
injury by pencils or other foreign objects that become stuck deep in the ear, and, rarely, autoimmune disorders.
Http://www.merckmanuals.com/home/children's-health-issues/ear,-nose,-and-throat-disorders-in-children/hearing-
impairment-in-children

237.Baby with rash in diaper area was going to different private hospitals which they gave him 3 different steroid with no
improvement, when you examine him you noticed that he has sattallite spots in the thigh and buttucks .what you will
give
A. Antifungal
B. Topical steroid
C. Topical antibiotics
Answer : a
Source: if candidiasis is suspected or proven by potassium hydroxide (koh) preparation or culture, an antifungal agent effective
against yeast is indicated. The author has good experience in using hydrocortisone cream (1%) twice daily and antifungal
(nystatin cream, powder, or ointment; clotrimazole 1% cream; econazole nitrate cream; miconazole 2% ointment; or
amphotericin cream or ointment) cream after every diaper change or at least 4 times per day.
Http://emedicine.medscape.com/article/911985-treatment

238.2 weeks infant came with history of sob , sweating in forehead, what cardiac anomaly he has?
A. Vsd
B. Asd or tof (one of them not sure) → ( asd, asymptomatic, tof : immediately after birth, severe cyanosis is seen in
patients with tetralogy of fallot and pulmonary atresia or severe pulmonary stenosis)
C. Trans-position of great vessels → usually born at term, with cyanosis apparent within hours of birth.
D. Pda → 3 week to 6-week-old infants can present with tachypnea, diaphoresis, inability or difficulty with feeding, and
weight loss or no weight gain.

Answer : qs need more details, most probably a


moderate vsds
Babies may have excessive sweating as a consequence of increased sympathetic tone. This sweating is especially notable during
feeds. An important symptom is fatigue with feeding. Because feeding results in a need for increased cardiac output, this activity
may unmask exercise intolerance in a baby. Rapid breathing (tachypnea) at rest or with feedings is usually present.

Http://emedicine.medscape.com/article/892980-clinical
All information from medscape website.

239.Boy with glumerilonephritis after week he develop hemoptysis :


A. Henoch-schonlein purpura
B. Goodpasture syndrome
C. Rapid deterioration
Answer : b
explanation: if not good-pasture syndrome it could be wegner’s granulomatosis vasculitis

240.Baby born full term flax-....enlarge labia the cause is :


60
A. Estrogen
B. Hcg
C. Progestrone
Answer: a
Newborn girls temporary changes in the vaginal area:
-the labia, may look puffy as a result of estrogen exposure in neonate.
-there may be a white fluid (discharge) from the vagina. This is called physiologic leukorrhea.
-there may also be a small amount of bleeding from the vagina.
-these changes are common and should slowly go away over the first 2 months of life.
Https://www.nlm.nih.gov/medlineplus/ency/article/001911.htm

241.Rx of neonate with apgar score 3


A. Chest expansion
B. Ventilation
C. Iv fluids

Answer: ventilation.
Explanation: resuscitation should be started with room air or a blend of o2 and air and titrated to achieve o2 saturations within
the target range, which increases over the first 10 min of life.

61
Source: http://www.msdmanuals.com/professional/pediatrics/perinatal-problems/neonatal-resuscitation

242.Child with eye itching for one month (no other symptoms in scenario), and have hx of asthma, what's the d?

A. Bacteria conjunctivitis
B. Viral conjunctivitis
C. Venral conjunctivitis

Answer: c
Explanation

62
243.Child had bee sting. No symptoms other than edema and erythema at the site of the bee sting. Management will be
A. Oral steroid
B. Antihistamine
C. Admission and observation
Answer: a
Explanation: a typical local reaction to a sting (1 to 5 cm) may be treated with cold compresses.
Large local reactions (5 to 10 cm) treatment is based upon symptoms:
Oral prednisone 40 to 60 mg to reduce significant swelling.
(nsaids) can reduce pain.
Pruritus can be treated with oral antihistamines and high potency topical corticosteroids until the itching subsides.

Source: http://www.uptodate.com/contents/bee-yellow-jacket-wasp-and-other-hymenoptera-stings-reaction-types-and-acute-
management?Source=machinelearning&search=hymenoptera&selectedtitle=1~91&sectionrank=2&anchor=h4#h4

244.Young girl came with s/s of anemia, there was splenomegaly, investigation: both direct and indirect coomb's test were
positive, most likely diagnosis?
A. Autoimmune hemolytic anemia.
B. Spherocytosis.
C. Thalassemia.
Answer: a

Autoimmune hemolytic anemia is diagnosed by detection of autoantibodies with the direct antiglobulin (direct coombs) test.

Https://www.msdmanuals.com/professional/hematology-and-oncology/anemias-caused-by-hemolysis/autoimmune-hemolytic-
anemia

245.Which of the following used in sca to increase production of hbf:


A. Folic acid
B. Hydroxycarbamide
C. Forget other choices

Answer: b
Explanation: hydroxyurea (hydroxycarbamide) is used to elevate fetal hemoglobin.
Folic acid replenishes the depleted folate stores necessary for erythropoiesis. Folic acid supplementation is well established in
the treatment of chronic hemolytic anemia. Although it is proposed that folate in anemia raises hemoglobin levels and helps
provide a healthy reticulocyte response, the use of folic acid in patients with scd is not well supported by the primary literature

Http://www.uptodate.com.sci-hub.cc/contents/hydroxyurea-and-other-disease-modifying-therapies-in-sickle-cell-
disease?Source=see_link&sectionname=use+of+hydroxyurea&anchor=h5#h5

63
246.Boy came for routine checkup, there was a murmur, echo done with only 2mm vsd what is your management??
A. Surgical repair
B. Watchful waiting
C. Medication can't remember the name
Answer: b
Explanation:
Small <4 mm
Moderate 4 to 6 mm
Large >6 mm

Small vsds, particularly muscular septal defects, often close spontaneously during the first few years of life. A small defect that
remains open does not require medical or surgical therapy. Asymptomatic patients with residual small defects are usually
followed every two to five years for overall assessment.
Http://www.uptodate.com.sci-hub.cc/contents/pathophysiology-and-clinical-features-of-isolated-ventricular-septal-defects-in-
infants-and-children?Source=see_link&sectionname=complications&anchor=h25#h25

247.Long history (case of meningitis) child receive iv penicillin g, lp showed gram –ve diplococcal, family concerned about his
young brother:

A. Admitted him to the hospital + observation


B. Give him iv penicillin
C. Oral rifampicin

Answer: c
Explanation:

Rifampin 600 mg (for children > 1 mo, 10 mg/kg; for children < 1 mo, 5 mg/kg) po q 12 h for 4 doses
Ceftriaxone 250 mg (for children < 15 yr, 125 mg) im for 1 dose
In adults, a fluoroquinolone (ciprofloxacin or levofloxacin 500 mg or ofloxacin 400 mg) po for 1 dose

Https://www.msdmanuals.com/professional/infectious-diseases/neisseriaceae/meningococcal-diseases

248.New born totally healthy with left thigh bruise all examination normal.
Prolonged pt, ptt
Your dx:
A. Hemophilia
B. Factor 10 deficiency
C. Idiopathic thrombocytopenic purpura
Answer: b

64
249.Child with dental caries, what u will advise him?
A. Oral antibiotic
B. Diet modification✅
C. Antiseptic mouthwash
Answer: b

Caries management with restorative therapy (eg, fillings) is the preferred therapeutic approach in many countries. However,
restorative therapy must be combined with preventive measures, since restorations have relatively short durability and new
caries may form at the margins of restorations if the causes of the disease persist.

The key for the prevention and control of dental caries and advanced periodontitis is the active promotion of oral hygiene. The
components of such a regimen include:

●regular brushing with a fluoridated toothpaste and dental flossing after each meal
●dietary counseling to reduce the ingestion of sugar-rich foods or beverages
●use of topical fluorides and oral antimicrobial rinses, such as chlorhexidine for high-risk patients
●modification of risk factors, such as smoking cessation
●overcoming the reluctance for regular visits to dental professionals

Http://www.uptodate.com.sci-hub.cc/contents/complications-diagnosis-and-treatment-of-odontogenic-
infections?Source=search_result&search=dental+caries&selectedtitle=1~91

250.Pediatric patient is always fatigued, he sleeps well at night with no orthopnea or pnd. What is deficient enzyme?
A. Pyruvate kinase
B. Pyruvate dehydrogenase complex
C. Pyruvate decarboxylase
Answer: a

251.Asthmatic patient on inhaled corticosteroid and prn short acting beta 2 agonist which he use 3 times daily. What is next
step in the management?

65
A. Long acting beta 2 agonist
B. Interleukin inhibitors
C. Mast cell stabilizer
Answer: a

252.Child had symptoms of mild infection (i thing urti) 4 days ago. And taking antibiotics. Now he’s ok. No fever good feeding
and active. Came for routine vaccination. What will you do?
A. Give him the vaccine
B. Wait until he finish the antibiotics course
C. Give the vaccine except …
Answer: give him the vaccine
Http://www.cdc.gov/vaccines/hcp/admin/contraindications-vacc.html

253.Sign of fetal distress?


A. Blood loss
B. Early deceleration
C. Late deceleration
Answer: c

A late deceleration is a reflex fetal response to transient hypoxemia during a uterine contraction
Http://www.uptodate.com.sci-hub.cc/contents/intrapartum-fetal-heart-rate-
assessment?Source=search_result&search=fetal+distress&selectedtitle=1~80#h27

254.45 days old child k/c of g6pd deficiency. Presented with jaundice and bilirubin=25 and his weight at 10th percentile. What
is the cause of jaundice?
A. Physiological
B. Hemolysis
C. Low birth weight (birth weight was 2.51 k)
Answer: hemolytic

Gucose-6-phosphate dehydrogenase (g6pd) deficiency is an x-linked enzymatic defect common in blacks that can result in
hemolysis after acute illnesses or intake of oxidant drugs (including salicylates and sulfonamides).

255.Child with vomiting + diarrhea after meal... Had similar attack one month ago after he came from camping with school.
What is the most important thing help you in diagnosis?
A. Pus in stool.
B. History of dairy products.
C. Lactose breath test.

Answer: c

66
256.Child 5 months, nurse said he has developmental delay in milestones, what would you do to reassure the mother?
A. Pincer grasp wave bye bye
B. Reach things✅
C. Sit without support
Answer: b
Https://www.msdmanuals.com/professional/pediatrics/growth-and-development/childhood-development

257. Thalidomide in pregnancy, what does it cause {ob/gyn?}


A. Tooth.
B. Spina bifida
C. No bone or ear or eye
Answer: c
Http://www.thalidomide.ca/recognition-of-thalidomide-defects/

258.2 week old baby with strong cough and 2 episodes of him losing consciousness, on exam there is intercostal retractions.
O2 sat was 90? What do you do?
A. Inhaled steroids
B. Ventilator support
C. Antibiotics
Answer

259. Gingival hypertrophy side effect


A. Carbamazepine
B. Phenytoin
C. Phenobarbital
Answer: b

67
260.Neonate 2 week old with umbilical hernia and jaundice, what will you do?
A. Check newborn metabolic screening results
B. Baby’s tsh and t4 levels
C. Mother’s tsh and t4 levels
Answer: b (congenital hypothyroidism)

Newborn screening detects the majority of infants with congenital primary hypothyroidism
infants with abnormal screening results are recalled for confirmatory serum tsh and free t4

Http://www.uptodate.com.sci-hub.cc/contents/clinical-features-and-detection-of-congenital-
hypothyroidism?Source=search_result&search=congenital+hypothyroidism&selectedtitle=1~86

261.Child presented with erythematous pharynx, with cervical lymph nodes and rapid strplysin test negative and low grade
fever with positive ebv. It next step
A. Give antibiotics and anti-pyretic
B. Give antipyretic and fluids ✅
C. Do culture and sensitivity
Answer: b
Rx:
Supportive: rest, hydration, saline gargle, analgesic for sore throat
2-avoid contact sports if splenic enlargement for 6-8 wks.
If there is airway obstruction (due to tonsillar enlargement): admit + steroid
68
Acyclovir doesn’t reduce duration of sx and nor result in earlier return to school/work

262.About an adolescent who missed or didn't receive vaccine? Can’t remember exact choices
A. 3 times 2 weeks apart
B. 3 times 4 weeks apart
C. 3 times 6 months apart
Answer: (it depends on the vaccine type and the dose interval between doses)
Example: between 1st and 2nd doses it’s often a minimum of 4 weeks apart except for hepatitis a (6 months apart) and
meningococcal (8 weeks apart) and varicella (3 months apart in younger than 13 years old)
Catch up immunization schedule according to vaccine link for:
http://www.cdc.gov/vaccines/schedules/downloads/child/catchup-schedule-pr.pdf

263.Commonest sexual anomaly in boys


A. Hypospadias
B. Micropenis
C. Cryptorchidism ✅
Answer: c
Cryptorchidism is the most common congenital abnormality of the genitourinary tract
Hypospadias affecting 1 in every 250 children, 1 in every 125 live birth
(uptodate)
Micropenis may be seen in baby with ambiguous genitalia but not as common as first two
Most appropriate answer here is cryptorchidism

264.A case of absence seizure. Epidural fentanyl causes


A. Demyelination.
B. Neurotoxic metabolite.
C. Something to gamma receptors
Answer: b?? Systemic opioid may cause neurotoxicity at higher doses
Another q asked before:
Clear case of absence seizure, what happens if we give fentanyl?
Answer: fentanyl-induced epileptiform activity on the electrocorticogram
Http://www.epilepsy.com/information/professionals/diagnosis-treatment/procedures-epilepsy-patients/general-anesthetics-4

265.A mother took her child to the well baby clinic, he was laughing with his mother. When the doctor came he cried. How
old is he?
A. 2 mo
B. 4 mo
C. 6 mo ✅
Answer c
Starting laughing loudly spontaneously to familiar voice at 4m but development of stranger anxiety at 6m (milestone: nilsson)

266.Side effect of dtp vaccine:


A. Fever
B. Generalised body pain
C. Erythema at injection site ✅
Answer: c if most common also initial finding.
Adverse reactions to diphtheria-tetanus-pertussis vaccines may include, (from most to least frequent) [1-3,6,60]:
●mild local and systemic reactions (e.g. Erythema at site of injection or local pain and skin irritation)
●entire limb swelling ●persistent, inconsolable crying (≥3 hours) ●hypotonic-hyporesponsive episode (collapse or shock-like
state) ●seizures ●fever ≥105ºf (40.5ºc) ●anaphylaxis (uptodate)

267.Child with generalized tonic clonic 5min, fever 38 , what is best mang?!
A. Control fever
69
B. Administer diazepam
C. Phenytoin
Answer: a
(simple febrile seizure less than 5 min treat symptomatically) if it recur or >5 min administer iv diazepam
A child's first seizure may be caused by an acute illness, such as a metabolic derangement or infectious disorder, and be
nonrecurrent, or may represent the beginning of epilepsy. A decision must be made about initiating chronic antiepileptic drug
(aed) treatment if a potentially reversible acute cause is not found during the evaluation.
Febrile seizures are the most common neurologic disorder of infants and young children, simple febrile seizures, defined as
generalized seizures lasting less than 15 minutes and not recurring during a 24-hour period, represent the majority of febrile
seizures. Simple febrile seizures are generalized, last less than 15 minutes, and do not recur in a 24-hour period. The most
common seizure type is generalized clonic, but atonic and tonic spells are also seen.
Rx : no need for rectal benzodiazepine as most child return to baseline less than 5 min if it was >5 min administer iv diazepam
but otherwise in less than 5 min treat symptomatically
(uptodate)
268.Central treatment of child with kawasaki is
A. Aspirin
B. Iv immunoglobulin
C. Nsaid
Answer: b
But better and more effective treatment with both iv immunoglobulin and aspirin
http://www.uptodate.com/contents/kawasaki-disease-initial-treatment-and-
prognosis?Source=search_result&search=kawasaki+disease+children+treatment&selectedtitle=1~150
(uptodate)

269.Child develop glove and stock numbness in foot and hand . Which is the best management:
A. Plasma exchange
B. Immunoglobulin
C. Steroid
Answer: b (if it is guillain-barré syndrome)
The main modalities of therapy for gbs are intravenous immune globulin (ivig) and plasma exchange (also called
plasmapheresis). Ivig is preferred to plasma exchange in children because of the relative safety and ease of administration,
although there are no reliable data suggesting that one or the other is superior. (uptodate)

270.Child presented with runny nose.. Sore throat. . Feel of fullness of ears with no fever.. On examination nose congested
and erythematous tonsils.. Diagnosis
A. Urti
B. Croup
C. Otitis media
Answer: a
-otitis media there is usually triad of otalgia, fever, and conductive hearing loss
- croup presented with characteristic barking cough and fever
- urti could present with different non specific signs and symptoms and according to etiology it could be viral with low grade
fever or no fever.

271.Preterm baby breast fed.. The mother is complaining of crying .. She only give him formula at night to calm him ..on exam
his abdomen is distended ..dx?
A. Increase abdominal gasses
B. Decrease motility
C. Incomplete nerve growth
Answer: a
Increase abdominal gases due to faulty feeding techniques or immaturity of gut (more likely in this baby)

70
Http://www.uptodate.com/contents/infantile-colic-clinical-features-and-
diagnosis?Source=machinelearning&search=infantile+colic+preterm&selectedtitle=1~150&sectionrank=1&anchor=h9932632#h
20171425

272.2 pelvic x-rays for children with similar choices


A. Metaphyseal displaced
B. Avascular necrosis of head of femur
C. Fracture head of femur
Answer: need more information

273.Child presenting with a picture of intussception, which of the following is the most appropriate step in management?
A. Barium enema,
B. Surgery,
C. Observation
Answer: a

Patients aged 5 months to 3 years who have intussusception rarely have a lead point (ie, idiopathic intussusception) and are
usually responsive to nonoperative reduction (which includes therapeutic enemas). Older children and adults more often have a
surgical lead point to the intussusception and require operative reduction (medscape
http://emedicine.medscape.com/article/930708-treatment#d1)

274.An 18 months old child with vsd of 2 mm, grade 2 pan systolic murmur, which the of the is the most appropriate next
step in management?
A. Surgery,
B. Observation
C. Catheterisation
Answer: b
Children with small ventricular septal defects (vsds) are asymptomatic and have an excellent long-term prognosis. Neither
medical therapy nor surgical therapy is indicated. Prophylactic antibiotic therapy against endocarditis is no longer indicated in
most cases. (medscape http://emedicine.medscape.com/article/892980-treatment)

275.A baby 6 month show regurgitation after every meal the esophagus ph is low he is normally
Developing what is the rx ?
A. Close follow up
B. Surgical fundal
C. Esophageal manometry

Answer: a

71
276.Neonate with deafness , rash .. What the cause ?
A. Rubella
B. Measles
C. Cmv
Answer: a
The classic triad presentation of congenital rubella syndrome consists of the following
Sensorineural hearing loss is the most common manifestation of congenital rubella syndrome. It occurs in approximately 58% of
patients. Hearing impairment may be bilateral or unilateral and may not be apparent until the second year of life.
Ocular abnormalities including cataract, infantile glaucoma, and pigmentary retinopathy occur in approximately 43% of children
with congenital rubella syndrome. Both eyes are affected in 80% of patients, and the most frequent findings are cataract and
rubella retinopathy. Rubella retinopathy consists of a salt-and-pepper pigmentary change or a mottled, blotchy, irregular
pigmentation,
Congenital heart disease including patent ductus arteriosus (pda) and pulmonary artery stenosis is present in 50% of infants
infected in the first 2 months' gestation.
Skin manifestations, including blueberry muffin spots that represent dermal erythropoiesis and dermatoglyphic abnormalities

277.5 years old girl with uncomplicated cystitis. What is the management :
A. Oral amoxicillin
B. Iv cephalosporin
C. Im ceftriaxone
D. sodium something
Answer: a
(patient >2y with uncomplicated cystitis , afebrile , no anomalies or underlying medical problem empirically can be given: orally
2nd generation or 3rd generation cephalo and if we suspect gram +ve enterococcus give amoxicillin ) uptodate
"treat lower-urinary tact infection (cystitis) with amoxicillin, trimethoprim-sulfamethoxazole, or nitrofurantoin (if no fever)"
(kaplan pediatrics)
278.Child with asd which finding on examination:
A. S3 with galob
B. Ejection systolic mur
C. Split s2
Answer: c, (fixed split s2 specific for asd and found almost in all patients with large left-to-right shunt)
Ejection systolic murmur at the left upper sternal border is correct also

72
279.Fever, abdominal pain, watery diarrhea, vomiting ,12 y child:
A. Rotavirus
B. Adenovirus
C. Norovirus
Answer: a is the most common, if there was urt symptoms go with b. If adult or the case is epidemic chose norovirus

280.Stiping up immedate after meal ,2 m old , all normal what will you do?
A. CT Abdomen
B. Assurance
C. PPI
Answer: b ( if the meaning of stipping is posseting it is considered normal. While if it indicates regurgitation gerd should be
considered and assurance mostly will do for the management unless conservative measures are mentioned in the choices such
as putting the child in mire upright position while feeding and increasing solid intake more than the fluid. If severe consider ppi)
more reading from illustrated textbook of pediatrics p219.

281.Long history (case of meningitis) child receive iv penicillin g, lp showed gram –ve diplococci, family concerned about his
young brother:
A. Admitted him to the hospital + observation
B. Give him iv penicillin
C. Oral rifampicin

Answer: c

Toronto note

282.New born totally healthy with left thigh bruise all examination normal. Prolonged pt , ptt your dx:
A. Hemophilia ( normal patient , high ptt)
B. Factor 10 deficiency
C. Idiopathic thrombocytopenic purpera (both pt , ptt normal )
Answer: b

73
283.3 and half years old uncircumcised boy , toilet trained , came with his parent with uti
What is the best method to obtain urine culture:
A. Catheter.
B. Clean cath
C. Suprapubic
Answer b: urine collection sample :
In toilet trained children >> midstream clean catch sample
In non toilet trained >> obtained via suprapubic tap or catheterization
First aid of the pedantics

284.Which heart disease is common in down syndrome:


A. Vsd.
B. Atrioventricular septal defect.
C. Coarctation of the aorta.
Answer: b
According to medscape the most common congenital cardiac defect is endocardial cushion defect (43%), which results in
atrioventricular septal defect (avsd)/av canal defect

285.12-old boy came to the clinic with his parents , they are complaining that he is obese and has strong appetite, all his
milestones were 2-3 months later than normal , pe / small hands and feed compared to rest of body , the cause of his
problem is :
A. Genetic
B. Metabolic
C. Natural
Answer: a
Prader-willi syndrome , toronto note

286.2 weeks baby had uti and sensitivity was +ve for amoxicillen, what investigation you should order before starting rx:
A. Renal us
B. Abdominal x ra
C. Urethral culture
Answer: a
All children under age of 5 and all male children should have a renal us to identify anatomical abnormalities including
hydronephrosis , dilatation of the distal ureter , or bladder hypertrophy and to rule out pyelonephritis .
74
Note : vcug indication >>
Female <5 y with uti
Female > 5y presenting with second uti
All males
Febrile uti
First aid of pediatrics

287.Child fall on his abdomen and felt a severe right abdominal pain and tenderness, what is the diagnosis:
A. Liver contusion
B. Acute cholecystitis
C. Acute pancreatitis
Answer: a
The mechanism indicates a liver injury which happens commonly after blunt injuries (spleen injury is more common with blunt
trauma than liver injury). And the severe pain might have resulted after bleeding into the peritoneal space causing peritonism.

288.Child had bee sting. No symptoms other than edema and erythema at the site of the bee sting. Management will be
A. Oral steroid
B. Antihistamine
C. Admitted and observation
Answer: b
Mercks manual

289.What is the treatment of juvenile idiopathic arthritis?


A. Paracetamol.
B. Aspirin.
C. Systemic steroid
Answer: nsaid is the first line,
Second line :methotrexate
Indications for corticosteroids: overwhelming inflammation, systemic illness, and bridging to methotrexate.
First aid 2ck

290.Child present with signs of meningitis, what complication will suspected if patient not treated?
A. Seizures
B. Somatization symptoms
C. At least 2 gi symptom
Answer: seizures .up to date

291.Possible causes of child with distress, in bronchoscope take biopsy


A. Cystic fibrosis
B. Deficiency of anti trypsin
C. Loss of ciliary kinesis
Answer : ??????

75
292.How to prevent infection for small baby during examination!
A. Wear gloves
B. Gown
C. Hand wash
Answer: c

293.A neonate has eye infection gm -ve diplococci ?


A. Iv ciprofloxacin
B. Im something
C. Local antibx

Answer: ceftriaxone 25–50 mg/kg iv/im, single dose (max 125 mg)
alt: cefotaxime, single dose

294.A few months old baby came to receive his vaccination. The baby was delivered at 34 weeks’ gestation. The baby’s
weight is low with a 90 g weight gain every month. Regarding vaccination of this case:
A. Delay 2 months from usual
B. As usual

C. Half dose

Answer: b
The data indicate that pis should follow the same vaccination schedule as that generally used for full-term infants, without
correcting for prematurity and regardless of birthweight.
Http://www.medscape.com/viewarticle/775544_1

295.Infant periumbilical hernia , what you will?


A. Put plastic in mid abdomen
B. Reassurance
76
C. Do hernia repair before start school

Answer: b
Umbilical hernias: most umbilical hernias do not cause any symptoms and do not require surgical repair until approximately age
5 years. For that reason, almost all umbilical hernias in young children and infants are managed by simple observation.
Http://emedicine.medscape.com/article/932680-treatment

296.Baby with cellulitis and purple rash:


A. Staph aureus
B. Strep pneumococcus
C. Group a beta hemolytic strep

Answer c
In individuals with normal host defenses, the most common causative organisms are group a streptococci (gas) and s aureus.
Group b streptococcus cellulitis occurs in infants younger than 6 months, because their immune responses are not fully
developed, and it may also be seen in adults with comorbidities such as diabetes or liver disease. For infantile cellulitis,
presentations may include sepsis.
Http://emedicine.medscape.com/article/214222-overview#a4

297.Patient with “pertussis” best swap


A. Nasal swab
B. Nasopharyngeal
C. Tracheal
Answer b
The results of blood culture are uniformly negative because b pertussis grows solely in the respiratory epithelium.
The culture specimen should be obtained by using deep nasopharyngeal aspiration or by holding a flexible swab (dacron or
calcium alginate) in the patient's posterior nasopharynx for 15-30 seconds or until a cough is produced.
Http://emedicine.medscape.com/article/967268-workup#c9
298.A mother took her child to the well baby clinic, he was laughing with his mother. When the doctor came he cried. How
old is he ?

A. 2 mo
B. 4 mo
C. 6 mo
Answer: c stranger anxiety
Source: toronto notes 2014 p5

299.Q about truncus arteriosus & bulbus cordis , cause ??!!


A. Vsd
B. Asd
C. Tetralogy of fallot
Answer: the note form the one who provided the question (if there is transposition of great vessels (tga) choose it, if not choose
tetralogy of fallot (tof))

300.Child with nephrotic syndrome on steroid for 3-6wks or 6 months has vaccine?
A. Give the vaccine
B. Stop the steroids
C. 3month and give appointment
Answer: a or c
Routine childhood vaccines with live virus strains are contraindicated during steroid therapy and for a minimum of 1 month
afterward. But other than that it is not contradicted. It is recommended to give in addition to the routine dead influenza yearly
and pneumococcus vaccine every 5 years. Http://emedicine.medscape.com/article/982920-treatment#d20

77
301.The most common complication of mump?
A. Hearing loss
B. Encephalitis
C. Sterility
Answer:b
The most common complication is orchitis. It is usually unilateral, and rarely will cause sterility even if bilateral. The second most
common is aseptic meningitis or encephalitis. Http://www.cdc.gov/mumps/hcp.html#complications and
https://www.mja.com.au/journal/2008/189/8/mumps-resurgent-disease-protean-manifestations

302.The most common organism in whooping cough


A. Bordetetella
B. Pertussis
C. H. Infeluanza
Answer: b
Whooping cough is characteristic of pertussis infection. Http://emedicine.medscape.com/article/967268-overview

303.Boy has a cat developed itching for a month with red eye and watery with discharge no lymphadenopathy and general
exam normal:
A. Cat scratch
B. Dermtitis allergic
C. Conjunctivitis
Answer:b
A is not related to the eye. C there is only two cases of conjuctvitis from cats to humans so the answer is b because it also the
symptoms of allergy. Http://icatcare.org/advice/cat-health/chlamydophila-felis-infection-feline-chlamydophilosis

304.Baby with vomiting and abdominal distension examination gush of stool history of decrease feeding and constipation,
history of barium enema what next?
A. Colectomy
B. Hydrostatic reduction
C. Leveling colostomy
Answer: ????

305.Asthmatic boy on muntelukast present to er, he has symptos everyday excacerbated by exercise what to give for
maintenance
A. Oral steroid daily with long acting when needed
B. Inhaled steroid twice with short acting when needed
C. Muntelukast with long acting
Answer: b

78
Courtesy https://www.guidelinesinpractice.co.uk/jan_15_holmes_asthma

306.Child, obese, had left hip pain and limbing with x-ray showing slept femoral hip, no hx of trauma, dx:
A. Slipped capital femoral head
B. Fracture
C. Osteomyelitis
Answer: a
Toronto:
Scfe: risk factors: male, obese

307.1-month baby come with abdominal distention and constipation since birth what you do next?
A. Xray
B. Biopsy
C. Rectal manometry
Answer: a
Biopsy: is diagnostic
Enema: if no signs of obstruction
Kub: if signs of obstruction (distention here)

308.8 year child come with 30 bmi what can you do for him
A. Surgery for obesity
B. Give him drug for obesity
C. Reassure and live style
Answer: c

79
309.Six day infant irritable poor feeding twitching of muscles bulging fontanile
A. Blood culture,
B. Lp
C. Iv antibiotic
Answer: b

310.Baby born on 35th week of gestation. His mother brings him for vaccination. Which of the following is true?!
A. Give the vaccine at time as schedule
B. Delay his vaccination 2 months
C. Give vaccine at time, but half the dose
Answer: a

311.High cretinism incidence in one area, low iodine in the supplied water, investigator want to interfere for prevention ,
what is best initial management to treat suspected child with cretinism ?

A. T4 & tsh after 2 weeks


B. Daily thyroxine
C. Iodine supplementation
Answer: c
One of the reasons of crenrtism is lack of iodine in the diet of the child.

312.5 year old girl came with uncomplicated cystitis


A. Give oral amoxicillin g.
B. Give iv ceftriaxone h.
C. Give iv ciprofloxacine
Answer: a (should be oral ceftriaxone toronto 15)

313.Child came complaining of pallor & fatigue. He has short stature & multiple cafe au lait spots over his skin. Lab shows
(wbc= normal, rbc= low, hb= low, retc= 9)
A. Iron deficiency anemia
B. Sca
C. Fanconi anemia
Answer: c
Fanconi anemia is aplastic anemia + congenital anomalies like short stature
Café au lait seen in fanconi anemia and neurofibromatosis ( http://emedicine.medscape.com/article/960401-overview)

314.Parent bring their child after mid night (3 a.m.) Child complain of barking cough, dyspnea, fever & inspiratory stridor. Sp
o2 =92% in room air. Which of the following symptoms are of concern?
A. Faring of ala nasi u.
B. Expiratory stridor v.
C. Blue color of lips
Answer: c
Cyanosis may suggest acute epiglottitis which is emergency condition (toronto 15 ot45)

315.A 18 month old child present to opd having diarrhea & flatulence. Looking pale, below 25th percentile for weight. Baby
was completely normal at age 12 month when he weaned from breast feeding.
A. Celiac disease
B. Cystic fibrosis.
C. Hirschsprung disease
Answer: a

316.Lactating mother with active hepatitis b.

80
A. Encourage breast feeding
B. Stop breast feeding
C. See level of hbs ag then decide

Answer: a
All women with hepatitis b are encouraged to breastfeed their babies since the benefits of breastfeeding outweigh the potential
risk of transmitting the virus through breast milk. In addition, since all newborns should receive the hepatitis b vaccine
at birth, the risk of transmission is reduced even further.
Https://www.cdc.gov/breastfeeding/disease/hepatitis.htm

317.Child can walk without support ,crawling ,build 3 cubes point to something he interested in , so what is the age of the
child ?
A. 9month
B. 15 month
C. 2 years
Answer: b

318.Parent brought their baby to your clinic , u noticed testicular asymmetry and was tender on palpation , the testes where
palpable in scrotum bilaterally , next step :
A. Transillumination
B. Surgery
C. Watch it while the baby is craying and coughing .

Answer: b
Explanation: in the pediatric population, there is a higher likelihood for testicular torsion if the testis is high riding compared
with the other side. While abnormal lie can help in diagnosis, fewer than 50% of cases demonstrate horizontal lie. In an older
patient, a physical examination may reveal a swollen, tender, high-riding testis with abnormal transverse lie and loss of the
cremasteric reflex. Surgical exploration is mandatory unless torsion can be excluded.
th
Reference: illustrated textbook of paediatrics 4 edition, page 350
Http://emedicine.medscape.com/article/2036003-overview

319.Baby on antibiotic developed watery diarrhea, what is most likely organism:


A. C. Perferingas
B. C. Difficle
C. Rota virus

Answer: b
Explanation: the diagnosis of c difficile colitis should be suspected in any patient with diarrhea who has received antibiotics
within the previous 3 months, has been recently hospitalized, and/or has an occurrence of diarrhea 48 hours or more after
hospitalization. Symptoms include: mild to moderate watery diarrhea that is rarely bloody, cramping abdominal pain, anorexia,
and malaise.
Reference: http://emedicine.medscape.com/article/186458-overview

320.18 months old girl delivered premature her wt was 2.6kg .she is healthy but last 2 days c/o irritability and fatigue .mother
shift feeding from breast feed to cow milk feeding at age of 9 month , she has hypochromic microcytic anemia, this
patient complaint due to ?
A. Premature
B. Cow milk feeding
C. Bone marrow defect

Answer: b
Explanation: cow milk fed infants suffer from iron deficiency anemia due to low iron content in cow's milk.

81
th
Reference: nelson essentials of pediatrics 7 edition, page 97, table 31-1

321.Neonate with gradual cyanosis, in the beginning there is parasternal murmur without cyanosis, there is right ventricular
hypertrophy , right axis deviation, on x-ray there is small heart with pulmonary vessel (something i didn’t recall) what is
the diagnosis ?
A. Transposition of great vessels
B. Tetralogy of fault
C. Other acyanotic heart disease

Answer: b
Transposition of great vessels Tetralogy of fallot
Most common cyanotic heart disease Most common cyanotic disease
presenting at birth
Single and loud s2 Paroxysmal hypercyanotic attack (tet spell)
Egg on string appearance in cxr Systolic thrill along left sternal border, harsh systolic murmur
along upper sternal border, boot shaped heart in cxr, right axis
deviation on ecg
Reference kaplan step 2ck pediatric 2014

322.Child with bronchiolitis what is the treatment?


A. Ribavirin
B. Acyclovir
C. Other antiviral
Answer: a
Explanation: the initial management of bronchiolitis is by means of supportive therapy. This is achieved by maintaining
oxygenation, hydration, and ventilation in case apnea developed. Pharmacologic therapy has limited role in management
however ribavirin aerosol is indicated in following conditions to minimize or prevent need of ventilation: congenital heart and
pulmonary disease, immunosuppressed patients & infants less than 6 weeks.
Reference http://emedicine.medscape.com/article/961963-treatment#d10

323.A mother with hepatitis c infant comes to your clinic asking about breastfeeding. How will you advise her?
A. Treat the baby then breastfeed.
B. Continue breast feeding.
C. Stop breastfeeding.

Answer: b
Reference: http://www.cdc.gov/breastfeeding/disease/hepatitis.htm

324.Boy 10 yrs bmi 30 , wt &ht above 95 percentile what is the best management?
A. Refer to surgery
B. Lifestyle modification
C. Re evaluate after 12 m

Answer: b
325.10 years old child with diarrhea. What is the correct about oral rehydration solution?
A. Start with 50 in first 4 hour then maintenance 100 per day
B. Start with 50 in first 4hours then maintenance 50 per day
C. Start with 100 in first 4 hours then maintenance 100 per day
Answer: a
Explanation:

82
Reference nelson’s essentials of pediatrics

326.Pediatric with holosystolic murmur in left 3rd intercostal space, with hyper dynamic heart and loud s2 what’s the dx:
A. Vsd
B. Asd
C. Pda

Answer: a

327.What are the most common cause of ftt :


A. Psychosocial
B. Cystic fibrosis
C. Asthma

Answer:a

Child (usually infant) not fed adequate calories.emotional or maternal deprivation concurrent with nutritional deprivation. Leads
to neglect of infant; psychosocial deprivation most common reason in all age groups.for diagnosis feed under supervision (may
need hospitalization) for 1 week the baby should gain >2 oz/24 hours over the week. Reference: kaplan usmle step 2 pediatric

328.4 y\o child developed resistance or not respond to chloramphenicol in salmonella infection you will
A. Repeated dose of chloramphenicol
B. Ciprofluxocin
C. Im ceftrixone

83
Answer is: c
Recommended regimens for the treatment of typhoid fever included ampicillin, trimethoprim-sulfamethoxazole, or
chloramphenicol in case of resistance quinolone, macrolide, and third-generation cephalosporin antibiotics are preferred for
empiric therapy pending sensitivities. Reference: http://emedicine.medscape.com/article/2281

329.Child with anemia a picture showed spherocytosis , what investigation you will do?(ped)
A. Electrophoresis
B. Osmotic fragility test
C. G6pd levels
Answer is: b
Spherocytes are present in :hereditary spherocytosis, immune hemolytic anemia, post-transfusion. This is a case of hereditary
spherocytosis and to confirm the diagnosis osmotic fragility test should be preformed. Reference: kaplan usmle step 2 pediatric
+toronto notes

330.Child with arthritis , fever, epistaxis, gingival bleeding, results put is low , hgb is low.. Appropriate investigation?

A. Bone marrow aspiration


B. Electrophoresis
C. Anti dsdna

Answer:a
The causes of pancytopenia are: aplastic anemia, mds , myelofibrosis ,leukemia,tb,amyloidosis, sarcoidosis or drugs (e.g.
Chemotherapy).

331.Child e elective oral surgery ,all lab within normal, on examination there is murmur 2/6 best hear in left sternum or (
suprasternal ) not sure , this murmur appear in sitting disappear in supine . Otherwise normal: what the appropriate
initial step:
A. Give antibiotics before surgery
B. Postpone surgery
C. Reassurance
D. Some thing
Answer: c
"changing the child’s position from supine to sitting, then to standing, and finally to squatting during the examination will
change the flow and is useful in helping to define innocent murmurs, treatment is to reassure"
Reference: http://circ.ahajournals.org/content/111/3/e20.full

332.Q about truncus arteriosus & bulbous cordis, cause?


A. Vsd
B. Asd
C. Tetralogy of fallot
Answer: if there is toga choose it, if not choose tof

333.Baby with vomitting and abdominal distension examination gush of stool history of decrease feeding and cosnstipation,
history of barium enema what next?
A. Colectomy
B. Hydrostatic reduction
C. Leveling colostomy

Answer: c (hirschsprung disease)

84
Reference: http://emedicine.medscape.com/article/929733-treatment#d7

334.Case senario the result was (high lactic acid ) : defecincy of :


A. Pyruvate kinase
B. Pyruvate dehydrogenase
C. Pyruvate carboxylase

Answer: c
Reference: https://ghr.nlm.nih.gov/condition/pyruvate-carboxylase-deficiency
Http://emedicine.medscape.com/article/125014-overview

335.Like mump senario :"can't remember "


A. Sajigran
B. Hypersensitvity vasculitis
C. Autoimmune....

Answer: A

336.Fever, malaise , maculopapular rash over the body & behind the ear; causative organism?
A. Rubella
B. Measles
C. Mumps

Answer: A,B, i think it can be rubella also

337.Yellowish discoloration and caries on occlusal surface of a child teeth, what will u do?
A. Oral antibiotics
B. Hexidine mouth wash
C. Improve diet
Answer: c

338.Mother changes her baby diaper many times a day. Labs all within the normal except (na) was low. What is diagnosis?
A. Acute proximal renal tubules acidosis
B. Acute distal renal tubules acidosis
C. Congenital chloride diarrhea

Answer: c
- Congenital chloride diarrhea: serum electrolyte levels may be within the reference range, especially in neonates and
treated patients. However, typical findings include low concentrations of serum chloride, sodium, and potassium.
- Http://emedicine.medscape.com/article/945263-workup
- Renal tubular acidosis: serum sodium expect to be within normal ranges
- Http://bestpractice.bmj.com/best-practice/monograph/239/diagnosis/tests.html

339.10 years old baby boy diagnosed with type 1 diabetes , presented to emergency department with 3 days history of not
feeling well and losing weight.
Investigations showed high blood sugar and ketone in the urine. What is best initial thing to do?
A. Electrolyte replacement
B. Fluid replacement
C. Insulin

Answer: b
85
- Step (1): always start with iv fluid then insulin

-
- Step (2): correct potassium and bicarbonate
- Step (3): monitoring therapy: should continue until resolution of dka. Criteria for resolution are:

Plasma glucose <11.1 mmol/l (<200 mg/dl) (at this point, insulin can be decreased by 50%)
Serum bicarbonate >18 mmol/l (>18 meq/l)
Venous ph >7.3
Anion gap <10.

- Http://bestpractice.bmj.com/best-practice/monograph/162/treatment/details.html

340.Girl after eating seafood developed rash with severe itching and diarrhea, the mechanism of reaction:
A. Complement
B. Cell mediated
C. Immediate

Answer: c
- Food-induced anaphylaxis: type i hypersensitivity reaction is mediated by food proteins binding to food-specific ige,
leading to mast-cell degranulation. Mast cells release mediators such as tryptase and histamine, which give rise to the
symptoms of anaphylaxis.
- Typical symptoms include urticaria, angio-oedema, throat tightness, hoarseness, pruritus, vomiting, nausea, diarrhoea,
wheezing, respiratory distress, and hypotension, usually within 2 hours of ingestion of the inciting food.
- The most common food allergens in adults are peanuts, tree nuts, shellfish, and fish. In children, milk, eggs, soya,
wheat, peanuts, and tree nuts
- Http://bestpractice.bmj.com/best-practice/monograph/841/overview/aetiology.html

341.Baby can walk when he held by one hand and good pincer grips but he cannot put things in the bottle. What is his age?
86
A. 9 m
B. 12 m
C. 15 m

Answer: a

342.1st q, child developed generalized edema with fever and dark urine, all labs normal except low calcium and low albumin,
what is dx?
A. Minimal change disease
B. Mesangial
C. Nephropathy
Answer: dark probably means hematuria. Glomerulonephritis?

343.Hcv infant, mother asking about breastfeeding?


A. Treat the baby then breastfeed.
B. Continue breast feeding.
C. Stop breastfeeding.
Answer : b
Having hcv-infection is not a contraindication to breastfeed. Hcv is transmitted by infected blood, not by human breast milk.

344.Rx of neonate with apgar score 3 what is the single most important step?
A. Chest expansion
B. Ventilation
C. Iv fluids
Answer:
87
If baby scores 3 or less, he may need immediate lifesaving measures
Indication of neonatal ventilation :
1. Apneic or gasping following initial steps and tactile stimulation
2. Hr<100/min in a spontaneously breathing baby
Ventilation (by bag and mask ventilation) of the lungs is the single most important and most effective step in cardiopulmonary
resuscitation of the compromised newborn.Chest compression: if hr is below 60/min after 30sec of bag & mask ventilation
345.Child with rta had head injury, admitted to icu. After 12h he give urine 100ml\h.
Lab show :
Sereum osmolarity : high
Serum na : high
Urine osmolarity : low
What is the cause of this condition :
A. Central diabetes inspidus
B. Renal diabetes inspidus
C. Fluid overload in the icu.
Answer: ??

346.Short stature child. Lab showing: low gh. What also must be assessed in this patient:
A. Prolactine
B. Somatomedine c
C. Glocuse
Answer: b
Insulin-like growth factor 1 (igf-1), formerly called somatomedin c. Igf-1 is used by physicians as a screening test for growth
hormone deficiency and excess in acromegaly and gigantism.

347.2 month old with ida what is the route of treatment?


A. Oral ferrous sulfate
B. Prenatal iron
C. Transfusion of packed erythrocyte (sure answer from canadian bank)
Answer: c

348.Case senario about child fall on his toys and twisted his ankle , they mention that he couldn't walk and want to be held
by his parents:
A. Soft tissue swelling of the ankle
B. Spiral fracture of tibia
C. Dislocation of fibula
I think is a?

349.Years old girl has type 1 dm , her wight 40 kg ( below 50th percentile ) and his hight 150 cm ( below 95th percentile ) , she
has no signs of secondary sexual characteristics of puberty, you want to perform annual sceening in clinic for:
A. Ophthalmology
B. Growth hormone!?
C. Ct to renal

350.10 years old baby boy diagnosed with type 1 diabetes , presented to emergency department with 3 days hx of not feeling
well and losing weight
Investigations showing high blood sugar and ketone in the urine
What is best initial thing to do:
A. Electrolyte replacement
B. Fluid replacement
C. Insulin

351.Young boy presented with diarrhea some time bloody, wight loos, arthritis, anemia the diagnosis is:
88
A. Crohn’s
B. Uc
C. Celic
In a large, retrospective review of ibd patients, joint complications were found in 16% and 33% of those with ulcerative colitis
and crohn's disease, respectively.
Http://www.medscape.com/viewarticle/550723_2

352.Month baby come with abdominal distension and constipation since birth what will you do next:
A. Xray (initial)
B. Biobsy (definitive)
C. Rectal manometrey

353.Child post-meal complains of nausea and abdominal distention, cramps, she oriented and cooperative, this symptom
after he came from school 1 month ago, vital signs at admission normal but after that bp decreased, normal temperature.
How to reach diagnosis?
A. History of food intolerance
B. Pus and stool examination
C. Stool culture

354.Boy fall on his mandibular, which muscle forceful close the jaw
A. Masster
B. Pterygoid
C. Temporalis

355.Child plays with his father’s watch , once father check it, the battery were missing his child looks normal but drooling ,
what is your action?
A. Endoscopy
B. Activated charcoal
C. X-ray chest ,abdomen

Answer: a
When a foreign body is strongly suspected on clinical grounds, visualization by endoscopy, which has the added advantage of
allowing removal of the object, may be the most efficient method of management.
" as a result, esophageal button batteries have emerged as the most critical indication for emergent endoscopy in children"
Medscape: http://emedicine.medscape.com/article/801821-workup#c5
Article:
http://www.naspghan.org/files/documents/pdfs/cme/jpgn/management_of_ingested_foreign_bodies_in_children_.28.pdf

356.Father complain of his child teeth there were brown caries , what you do?
A. Change food
B. Orla antibiotic cream
C. Systemic fluoride

Answer: a

357.8 months child crawl ,pincer grasp, wave bye ,say mama to carry him , brought by his mother from special clinic she said
that her child didn't pass the developmental exam ,what you will tell her?
A. Fine motor delay
B. Gross motor
C. Social
Answer: may be b

358.6 year old with duchene muscular dystrophy what to see?

89
A. Atrophied muscles of lower limbs
B. Fasiculation
C. Gower sign’climb on him self

Answer: c
The gower sign is a classic physical examination finding in md and results from weakness in the child's proximal hip muscles.
Although the gower sign is a classic physical examination finding in duchenne md, it is by no means pathognomonic; other types
of md and disorders with proximal weakness may also cause this sign.
Medscape: http://emedicine.medscape.com/article/1259041-clinical

359.4 yo girl came with her father for bilateral esotropia what is management ?
A. Observe
B. Glasses
C. Botulism …
Answer :b
Esotropia is a type of strabismus or eye misalignment. The term is derived from 2 greek words: ésò, meaning inward, and trépò,
meaning turn. In esotropia, the eyes are crossed; that is, while one eye looks straight ahead, the other eye is turned in toward
the nose. This inward deviation of the eyes can begin as early as infancy, later in childhood, or even into adulthood.
Medscape: http://emedicine.medscape.com/article/1198784-overview
Treatment modalities used to realign the eyes include spectacles (sometimes with prism or bifocal), strabismus surgery (eye
muscle surgery), and botulinum toxin (less frequently utilized).
Http://www.aapos.org/terms/conditions/48

360.Pediatric case of broncheactitis whats the most important ?


A. Physiotherapy
B. Home oxygen
C. Steroid

In addition to the treatment of an identified underlying disorder in patients with bronchiectasis, therapy is guided at reducing
the airway secretions and facilitating their removal through cough. Antibiotics can be used to prevent and treat recurrent
infections, usually based on the findings at bronchoalveolar lavage. Secretions can be mobilized with chest physiotherapy and
mucolytic agents. Inhaled corticosteroids may have a role in regulating the host response and halting inflammatory damage to
the lung.
Http://emedicine.medscape.com/article/1004692-treatment#d8

361.12 yo bilateral lower lung infiltration treatment (ttt)?


A. Ciprofloxacin
B. Azithromycin
C. Penicillin

Answer: b
nd rd
ttt of atypical pneumonia (bilateral infiltration). Typical causes lobar pneumonia and ttt is amoxicillin or 2 – 3 gene.
Cephalosporin. Step up pediatrics

362.14 year old girl patient ectric (jaundice) and sore throat, there is blood film shows (spherical shape rbc) what is your
diagnosis?
A. G6pd deficiency
B. Spherocytosis
C. Sickle cell anemia

Answer: b
http://www.aafp.org/afp/2004/0601/p2599.html

90
363.Child with paroxysmal cough, "whoop"
A. Pertussis
B. Epiglottis
C. Croup

Answer: a paroxysmal cough(whooping cough) is characteristic of pertussis.


Http://www.merckmanuals.com/professional/pediatrics/symptoms-in-infants-and-children/cough-in-children

364.Child hgb is low, rbc show spherocytes…..what type of anemia?

A. Microcytic hypochromic
B. Microcytic hyperchromic
C. Macrocytic anemia
Answer: non of above - normocytic normochromic
Explanation: rbc show spherocytes = hereditary spherocytosis (type of hemolytic anemia)

Link: http://kidshealth.org/en/parents/anemia.html#

365.Behcet's disease:
91
A. Vasculitis<<
B. Ulcer
C. ...test
Incomplete question

366.Radiosensitive tumor ?
A. Seminomas
B. Yolk sac
C. Embryonal carcinoma
Answer: a
Explanation: radiosensitive tumours are tumours which respond well to radiotherapy.
Radiotherapy may be effective alone, or may require the addition of cytotoxic chemotherapy as in the more advanced stages of
a wilms tumour and hodgkin's disease.
This group includes:
Malignant lymphomas
Seminomas
Medulloblastoma
Neuroblastoma
Wilm's tumour
Early cervical carcinoma
Vaginal carcinoma
Most head and neck tumours
Link: http://www.gpnotebook.co.uk/simplepage.cfm?Id=1993342989

367.Like mump senario :


A. Sajigran
B. Hypersensitvity vasculitus
C. Autoimmunce
Incomplete question

368.14 years old child known case of scd present to the er due to abdominal pain , pale he had a urti few days back , vital :
Hgb 34 (normal 110-120)
Wbcs : high
Fever
Reticulocyte =8
Splenomegaly
What your initial management:
A. Splenectomy
B. Packed rbcs transfusion
C. Antibiotic

369.A case if croup child is tachypneic , tachycardic , retrosternal retraction we started on racemic epinephrine inhaled it is
improving the symptoms initially what your next step:
A. Antibiotic
B. Steroid
C. Oxygen

370.What is the reliable method to diagnose all:


A. Lymph node involvement
B. Anemia and thrombocytopenia
C. Bone marrow blast cell
Answer: c, bone marrow aspiration and biopsy (definitive for confirming leukemia)
92
Http://emedicine.medscape.com/article/207631-overview

371.4 y/o boy thumb sucking


A. Observation
B. Palatal crib
C. Reward ++
Answer: c. Reward. Use positive reinforcement. Identify triggers. Offer gentle reminders. Mayoclinic
Http://www.mayoclinic.org/healthy-lifestyle/childrens-health/in-depth/thumb-sucking/art-20047038?Pg=2

372.Child can roll over, sit triploid, attempt to take object. Which month?
A. 6
B. 9
C. 2
Answer: a

Usmle step 2 pediatrics kaplan lecture notes

373.A child tells stories, runs, and plays father roleplay. How old is the child?
A. 2
B. 1
C. 4
D. 5
Answer: c

Usmle step 2 pediatrics kaplan lecture notes

374.A baby setting in his mother’s lap unsupported. When the doctor spoke, the baby turned around, laughed and babbled to
the doctor. How old is the baby?
A. 2 months
B. 4 months
C. 6 months
Answer: c

Usmle step 2 pediatrics kaplan lectures notes

375.Fetal demise in 30 week. What's the cause?


A. Abnormal 2b and normal 2a
B. Abnormal 4b and normal 4a
C. Normal 4b abnormal 4 a
Answer: c
93
The fetus has α-thalassemia with 4 gene deletions which leads to hydrops fetalis. Hydrops fetalis (fetal hydrops) is a serious fetal
condition defined as abnormal accumulation of fluid in 2 or more fetal compartments, including ascites, pleural effusion,
pericardial effusion, and skin edema. The condition is incompatible with life.
Http://emedicine.medscape.com/article/955496-overview#a3

376.18 month-old female child presents with anemia. What is the most likely diagnosis?
A. Homozygous β-thalassemia
B. Homozygous α-thalassemia
C. Carrier α-thalassemia
Answer: a
Β-thalassemia major manifests by age 1-2 years old with symptoms of severe anemia.
Merck manual

377.Patient 10 yrs with diarrhea. Bloody. Joint pain


A. Ulcerative colitis
B. Crohn’s
C. Celiac
Answer: a
Reference: http://emedicine.medscape.com/article/930146-overview

378.6 years old child, came for pre school checkup, on exam he looks normal except for grade iii heart murmur along the
sternal border “no specific location or timing of the murmur provided”, no thrill, the murmur accentuated with supine
position, dx?
A. Still’s murmur
B. Vsd
C. Asd
Answer: a
The most common innocent murmur is a still murmur, which is characteristically loudest at the lower left sternal border and has
a musical or vibratory quality that is thought to represent vibrations of the left outflow tract. Characteristics that are more likely
to be associated with an innocent murmur include a systolic (rather than diastolic) murmur; soft sound; short duration; musical
or low pitch; varying intensity with phases of respiration and posture (louder in supine position); and murmurs that become
louder with exercise, anxiety, or fear.
Reference: http://www.aafp.org/afp/2011/1001/p793.html

379.What type of insulin is used in dka?


A. Nph
B. Regular
C. Glarigine
Answer: b

380.A girl with type 1 dm diagnosed 3 years ago, she is 12 years did not show signs of puberty, what should she repeat
annually?
A. Growth hormone
B. Celiac testing
C. Ophthalmology exam
Answer: a
Not ophthalmologic examination because usually retinopathy occurs after puberty has started in children with type 1 dm.
Read more in ada guidelines: http://www.ndei.org/ada-diabetes-management-guidelines-children-adolescents-type-1-diabetes-
type-2-diabetes.aspx

94
381.11 years old child had severe diarrhea 3 weeks ago, now the child presented with bilateral lower limbs weakness and
numbness, dx?
A. Polio
B. Gbs
C. Muscular dystrophy
Answer: b
The typical patient with gbs presents 2-4 weeks following a relatively benign respiratory or gastrointestinal illness with
complaints of finger dysesthesias and proximal muscle weakness of the lower extremities.
Reference: http://emedicine.medscape.com/article/315632-overview

382.Child c/o fever, bloody stool, and tenesmus, abd exam showed abd distention,
Dx?
A. Ascaris
B. Amebiasis
C. Geirdiasis
Answer: b
The symptoms are often quite mild and can include loose feces (poop), stomach pain, and stomach cramping. Amebic dysentery
is a severe form of amebiasis associated with stomach pain, bloody stools (poop), and fever.
Reference: https://www.cdc.gov/parasites/amebiasis/

383.A child with gum bleeding, erythema papules in mouth. Swab showed multinucleated giant cell. Which organism
A. Coxsackie virus
B. Staph
C. Herpes
Answer: c
Reference: http://emedicine.medscape.com/article/218580-clinical#b1

384.5 years old girl presenting with hepatospleenomegally, pale, decreased level of consciousness, v\s indicating shock, labs:
hb: 5 platlets: 65, what's your important next investigation?
A. Abdomen us
B. Reticulocyte count
C. Bm biopsy
Answer: b
The patient has splenic sequestration. Splenic sequestration occurs with highest frequency during the first 5 years of life in
children with sickle cell anemia. It is a medical emergency. This complication is characterized by the onset of life-threatening
anemia with rapid enlargement of the spleen and high reticulocyte count.

385.Case of thalassemia >> splenomegaly + frontal protrusion


Labs: hb electrophoresis, hb a2, hb f high
A. B thalassemia major
B. B thalassemia minor
C. Alpha thalassemia
Answer: a
Reference: http://emedicine.medscape.com/article/206490-workup#c7

386.Neonate developed cyanosis (2nd or 3rd week after delivery) and there is finding on on auscultation i cannot remember
exactly but they didn’t mention about machinery murmur the qs about the management:
A. Nsaids
B. Steriod
C. Prostaglandin e1
95
Answer:
Prostaglandins are utilized to maintain the patency of the ductus arteriosus until surgical ligation is performed. When surgical
ligation is not indicated, prostaglandin inhibitors (eg, nonsteroid antiinflammatory drugs [nsaids]) are used to close the ductus
arteriosus.
Intravenous (iv) indomethacin or iv ibuprofen is used in the neonate and in premature infants.

387.Child etc ... Can not take the object in 2 fingers, how old is he?
A. 4 months
B. 6 months
C. 9 months
Answer: b
Link: http://connectability.ca/2011/03/21/practical-strategies-for-developing-fine-motor-skills/

388.Child complains of abdominal pain for 2 weeks with diarrhea that is occasionally bloody and malaise. What is the most
appropriate test for diagnosis?
A. Abdominal ct
B. Barium enema
C. Abdominal us
Answer: most likely a deepening on the options.

389.Pediatric patient k/c of asthma since age 2 years he is now asymptomatic with rare uses of albuterol inhalers, he came
for counseling, he had contact sport participation, what is best question to ask to know his response:
A. “are keeping up with your friends”
B. “how frequent do use inhaler”
C. “presence night symptoms (cough)”
Answer: b “not sure”

390.Cyanotic newborn with scaphoid abdomen. Heart sounds identified in the right side only. What's the most likely
diagnosis?
A. Dextrocardia with situs inversus
B. Transposition of great vessels
C. Diaphragmatic hernia
Answer: c
Infants frequently exhibit a scaphoid abdomen, barrel-shaped chest, and signs of respiratory distress (retractions, cyanosis,
grunting respirations). In left-sided posterolateral hernia, auscultation of the lungs reveals poor air entry on the left, with a shift
of cardiac sounds over the right chest. Http://emedicine.medscape.com/article/978118-overview

391.Most common intra-abdominal tumor in infants?


A. Wilm's tumor
B. Neuroblastoma
C. Lymphoma
Answer: b
Most common b then a
- neuroblastoma was the most common tumor constituting 46.7% of all cases, followed by wilms’ tumor (26.7%)
http://adc.bmj.com/content/97/suppl_2/a196.3
- neuroblastoma is the most common intra-abdominal malignancy of infancy, the most common cancer in infancy, and the most
common extracranial solid tumor of childhood. Neuroblastoma is the third most common malignancy in children up through 14
years of age, behind acute lymphocytic leukemia and cancers of the brain and central nervous system.
Http://emedicine.medscape.com/article/439263-overview#showall

392.Baby can sit roll from prone to supine and back play handle object but can't pick things b/ 2 fingers age
96
A. 4 months
B. 6 months
C. 8 months
Answer: b
Http://www.cdc.gov/ncbddd/actearly/milestones/milestones-6mo.html

393.Child with head trauma, admitted to icu and received fluid after hours she started to have high urine output 100ml/hr.
What would be the cause?
A. Nephrogenic di
B. Central di
C. Siadh
Answer: information not sufficient

394.Child with barking cough, what is best site to take swab?


A. Throat
B. Pharynx
C. Nasopharynx
Answer:

395.3 years old his parents has tb as a pediatrician you did ppd test after 72 hr you find a 10mm induration in the child this
suggest
A. Inconclusive result
B. Weak positive result
C. Strong positive result
Answer: c

Http://www.cdc.gov/tb/publications/factsheets/testing/skintesting.htm

396.Child with high-grade fever for 5 days and sore throat, on examination there was tonsillitis and white patches on the
gingiva. No ln enlargement, aso is negative. The most likely causative organism is:
A. Coxsackie virus.
B. Herpes simplex virus.
C. Ebv.

97
Answer : b
[4]
Clinical features include the following :

Abrupt onset of illness


Fever
Listlessness or irritability
Inability to eat and/or drink
Gingivitis (with markedly swollen, erythematous, and occasionally bleeding gums)
Increased drooling in infants due to pain on swallowing
Vesicular lesions on the tongue, buccal mucosa, and palate with extension, at times, to the lips and face (these may rupture
and coalesce to form large, ulcerated areas.)
Tender submandibular or cervical adenopathy
Http://emedicine.medscape.com/article/964866-clinical

397.Child with painful ear and runny nose and mild cough what will u give him?
A. Antibiotic
B. Antihistamine and decongestant
C. Paracetamol
Answer: a

Source: http://emedicine.medscape.com/article/994656-overview

398.Child with cola colored urine, which test should you perform first?
A. Urinary microscopy.
B. Renal function test.
C. Renal biopsy.
Answer: a- urinary microscopy.

399.Neonate was on breast feeding after 2 weeks has irritability and decrease feeding diagnosed with meningitis .. What is
the organism ?
A. Niesseria
B. Listeria
C. Strepococcal pneumonia ( i chose listeria )
Answer: b
Gel
G: gbs
E: e.coli
L: listeria
Http://emedicine.medscape.com/article/1176960-overview#a5

400.A woman has +ve hep b surface antigen delivered a baby in a hospital and received immunogloben and b vaccine within
the first 12 hour ,, what are the recommendation for breast feeding ??
A. No breast feeding
B. Breast feeding is okay
C. Breast feeding after 12 hours
Answer: c
Reference: kaplan pediatrics
Explanation: mothers with hbv infection are free to breast feed their
Infants after the neonate has received the appropriate recommended vaccination.

98
401.Child presented with something in the eye with lab result of incense of wbc what is the diagnosis
A. Leukemia
B. Neroblastoma
C. Not clear

402.Treatment of meningitis? Pediatrics


A. Ampicillin
B. Doxycycline

Answer: A
Reference: http://emedicine.medscape.com/article/232915-treatment#d8

403.Children diagnosed to be hypertensive :


A. BP Above 90th percentile
B. BP Above 95th percentile
Answer : A

404.Child with URTI is complaining of bleeding from nose, gum and bruising the treatment is:
A. Prednisolone
B. IVIG
Answer : B ( could be ITP)??

405.Most common cause of epistaxis in children ?


A. Self-induced trauma
B. Polyps
Answer: A.
Reference: Medscape

99
406.Late complication of meningitis in children?
A. Seizure
B. Cranial nerve palsy
Answer: A
Seizures are a common complication of bacterial meningitis, affecting almost one third of the patients. Persistent seizures,
seizures late in the course of disease, and focal seizures are more likely to be associated with neurologic sequelae.
Http://emedicine.medscape.com/article/961497-clinical#b3

407.Patient with lower limb weakness and sensation also angular stomatitis?
A. Vitamin B1 (thiamin)
B. Vitamin B3 (niacin)
Answer: Vitamin B12
Vitamin B12 deficiency can present with lower limb weakness and loss of sensation with angular stomatitis. Vitamin B2 and B3
can present with angular stomatitis but usually not with lower limb weakness. Vitamin B1 deficiency causes beriberi which
causes mainly lower limb weakness and paresthesia or cardiac diseases (wet beriberi).
Reference: http://emedicine.medscape.com/article/1152670-clinical

408.Child develops petechia in the legs, he has history of URTI. Labs showed: normal CBC expect platelet count was low.
What is the next step in management?
A. Platelet transfusion
B. Immunoglobulin
Answer: B
Http://emedicine.medscape.com/article/202158-overview

409.6 years old boy with fever, sore throat, developed pink maculopapular rash all over his body and pericarditis, what's the
Dx?

A. Still' disease (uptodate: Still’s disease” has become the eponymous term for systemic juvenile idiopathic
arthritis: characterized by daily quotidian fever, rash, and arthritis)
B. Kawasaki Disease
Answer: B
Medscape: Kawasaki disease produces prolonged fever (often abrupt in onset and preceded by several days of nonspecific
symptoms) along with a constellation of clinical features that includes the following: Nonexudative bilateral conjunctivitis (90%),
Anterior uveitis (70%), Erythema and edema on the hands and feet; the latter impedes ambulation, Strawberry tongue and lip
fissures, Myocarditis and pericarditis
Diagnostic criteria established by the American Heart Association (AHA) are fever lasting longer than 5 days and 4 of the 5
following main clinical features:
Changes in the peripheral extremities: Initial reddening or edema of the palms and soles, followed by membranous
desquamation of the finger and toe tips or transverse grooves across the fingernails and toenails (Beau lines)
Polymorphous rash (not vesicular): Usually generalized but may be limited to the groin or lower extremities
Oropharyngeal changes: Erythema, fissuring, and crusting of the lips; strawberry tongue; diffuse mucosal injection of the
oropharynx
Bilateral, nonexudative, painless bulbar conjunctival injection
Acute nonpurulent cervical lymphadenopathy with lymph node diameter greater than 1.5 cm, usually unilateral
Http://emedicine.medscape.com/article/965367-overview

410.2 Years child with neck mass, has has ptosis , miosis , anhidrosis and
Heterochromia. What is your diagnosis (case of Horner's syndrome):
A. Wilms
100
B. Neuroblastoma

Answer: B
Http://emedicine.medscape.com/article/988284-clinical

411.Child brought with diarrhea and abdominal pain. Distended abdomen lab otherwise normal. What is the diagnosis?
A. IBD
B. Carbohydrates intolerance

Answer: both may present with the symptoms listed in the question.

412.Milestone: A child says baba and walks holding furniture, and a lot of other features, how old is this child?
A. 12M
B. 10M
Answer: Saying mama/baba is at 9-11mo but it is not specific because he walks and throw objects at 12mo . ((First Aid for the
USMLE Step 2 CK, 8Ed ,p361 ))

413.Most common intra-abdominal tumor in infants?


A. Wilm's tumor
B. Neuroblastoma
Answer: B
Medscape: Neuroblastoma is the most common intra-abdominal malignancy of infancy, the most common cancer in infancy,
and the most common extracranial solid tumor of childhood
Http://emedicine.medscape.com/article/439263-overview

414.Child with pain when chewing and headache, he complains loss of weight and decrease appetite? (Tempromandibular
joint dysfunction)? What is the complaint?
A. Hearing loss
B. Facial palsy
Answer: Temporomandibular Joint Syndrome?
Medscape: http://emedicine.medscape.com/article/809598-clinical

415.Child come with case of immune diffcancy with presented with lump in the groin and lap was given what is the diagnosis
?? Theres CH50 in Q
A. Severe combined immune diffciancy
B. Chronic granulamtae disise
More explanation is needed but the presentation of lump make chronic granulamatouis disease more possible.

416.2 y child had mass in flank lead to displace the collection system Dx?
A. Wilms
B. Nuroblastoma
Answer: B

417.Diet causes Kawashioker?


A. Low protein
B. High Carbs
Answer: B

101
418.Peadi case barking cough after giving racemic Ebenverin what next step ?
A. Antibiotics
B. Steroid
Answer: B

419.Baby has sckle cell anemia and recive blood transfusion , what about vaccintion ?
A. Take vaccine ‫يعني ياخد الفاكسين عادي‬
B. Dont give vaccie

420.Child presented with petechiae and his platelets is 15, otherwise healthy. What will you do for him?
A. Splenectomy
B. Ivig

Answer: ? Observation
Observation vs. Pharmacologic intervention highly debated; spontaneous recovery in >70% of cases within 3 months.
Treatment with iv ig or prednisone if mucosal or internal bleeding, platelets <10, or at risk of significant bleeding.
Life-threatening bleed: additional platelet transfusion ± emergency splenectomy
Supportive: avoid contact sports and asa/nsaids. Reference: toronto notes.

421.4 weeks old boy with acute onset forceful non bilious vomiting after feeding. On abdominal examination: there is olive
mass at epigastric area. What is the 1st investigation should you do?
A. Ph monitoring
B. Abdominal us

Answer: b (the diagnosis of choice is us and the most accurate test is an upper gi series)
Hypertrophic pyloric stenosis: when the vomiting persists, other clinical and biochemical findings may occur such as
dehydration, hypochloremic alkalosis and unconjugated jaundice. Late clinical manifestations include weight loss and visible
gastric peristaltic activity with a palpable pyloric ‘olive’. Prompt us diagnosis is important as these late findings make the infants
suboptimal candidates for surgery. Reference: pubmed.

422.Child came with wheezing and cough, diagnosed to have asthma and his dr. Prescribed beclomethasone space inhaler or
nebulizer?? Twice daily. You will be worried about:
A. Growth retardation
B. Extraocular problem

Answer: a
Reference: 3rd edition uqu > pediatrics > q 58
Nb. Corticosteroids inhalers can lead to oral thrush (yeast infection of the mouth).

423.Child presented with asthma exacerbation. The patient did not respond to beta agonist. What is your next step?
A. Aminophylline
B. Systemic steroid

Answer: b
Reference: first aid usmle step 2ck

424.A child presented with fever and coryza, then watery diarrhea.
A. Adenovirus

102
B. Rotavirus
Answer: a
Adenovirus: http://emedicine.medscape.com/article/211738-clinical
Rotavirus: http://emedicine.medscape.com/article/803885-clinical

425.What’s the most common minimum side effect of dtp vaccine?


A. Low grade fever
B. Erythema over the injection site

Answer: b
Reference: http://www.cdc.gov/vaccines/vac-gen/side-effects.htm#dtap

426.8 years old boy with petechiae all over his body. Lab results: low platelets and high creatinine level. What is the
diagnosis?
A. Itp
B. Ttp

Answer: check below

103
th
Depends on nelson 7 the answer is probably hus. Hus if no fever or altered mental status. Hus = microangiopathic hemolytic
anemia + thrombocytopenia + renal failure. Ttp= hus + fever and/ or altered mental status.
th
Reference: nelson 7 p: 560

427.A child came to you with café au lait spots in face and neck. Which of the following features can strengthen your
diagnosis?
A. Port-wine stain.
B. Axillary freckling.

Answer: b
Clinical diagnosis requires the presence of at least 2 of 7 criteria to confirm the presence of nf1. The 7 clinical criteria used to
diagnose nf1 are as follows, in the absence of alternative diagnoses:
● Six or more café-au-lait spots or hyperpigmented macules =5 mm in diameter in prepubertal children and 15 mm
postpubertal
● Axillary or inguinal freckles (>2 freckles)
● Two or more typical neurofibromas or one plexiform neurofibroma
● Optic nerve glioma
● Two or more iris hamartomas (lisch nodules), often identified only through slit-lamp examination by an
ophthalmologist
● Sphenoid dysplasia or typical long-bone abnormalities such as pseudarthrosis
● First-degree relative (eg, mother, father, sister, brother) with nf1
104
Reference: http://emedicine.medscape.com/article/1177266-overview

Nf2: meningioma, schwannoma, glioma, neurofibroma, posterior subcapsular lenticular opacities accompanied by external
signs: hearing loss, ringing in the ears, and balance problems associated with vestibular nerve lesions, visual deficits and cranial
nerve palsies.
th
Reference: medscape and nelson 7 p: 645

428.(long scenario) child brought by his father in wheelchair complaining of knee swelling and history of falling on his knee.
What is the best investigation ?
A. Joint aspiration.
B. X-ray.

Answer: b
A plain radiograph of the affected joint should be performed to rule out fractures, periostitis, avascular necrosis, bone tumors,
and bone dysplasias.
Reference: uptodate

429.What is the best initial investigation in a child presented with croup ?


A. X-ray
B. Pharyngeal swab

Answer: a
Diagnosis of croup is clinical, based on the presence of a barking cough and stridor, and can be aided with a chest x-ray
showing a "steeple sign". Reference: uptodate + master the boards.

430.Malnourished baby with fair coiled hair and abdominal distension. What is the most likely diagnosis?
A. Kwashiorkor
B. Marasmus

Answer: a

431.Child with meningitis what to do immediately?


A. Antibiotics
B. Lumbar puncture
Answer: a

105
Reference: toronto notes.

432.A child with developmental delay and constipation. Labs: low na , low k and low ..?? What is the most likely diagnosis ?
A. Cah
B. Congenital chloride diarrhea
*other choices include syndromes
Answer:?
Bartter syndrome?
Some hypokalemic patients develop rhabdomyolysis, especially following exercise. Hypokalemia slows gastrointestinal motility;
potassium levels less than 2.5 meq/l may cause an ileus. Hypokalemia impairs bladder function, potentially leading to urinary
retention. Hypokalemia causes polyuria by producing secondary nephrogenic diabetes insipidus. Chronic hypokalemia may
cause kidney damage, including interstitial nephritis and renal cysts. In children, chronic hypokalemia, as in bartter syndrome,
leads to poor growth.
Reference: http://emedicine.medscape.com/article/238670-overview - showall
Http://emedicine.medscape.com/article/238670-overview#showall
And nelson 7th p: 116

433.A child with flat buttocks. What investigation will you do (celiac)?
A. Antibody…
B. Intestine biopsy

Answer: a
Antibody testing, especially immunoglobulin a anti-tissue transglutaminase antibody (iga ttg), is the best first test, although
biopsies are needed for confirmation; in children younger than 2 years, the iga ttg test should be combined with testing for igg-
deamidated gliadin peptides. Reference:http://emedicine.medscape.com/article/171805-workup

434.Child with chronic diarrhea , endoscopy showed sickle shaped parasite adherent to the bowl wall , what is it?
A. A:giardia
B. B:entemebea histolytica
Answer: a

106
Biopsy specimens from duodenum are often teeming with sickle-shaped giardiatrophozoites,which are tightly bound by the
concave attachment disc to the villus surface of the intestinal epithelial cells
Reference : medescape and nelson p:366

435.Breastfeeding mother known history of seizure with phenytoin ask about breast feed?
A. Reassuranse
B. Feeding after 8 hrs
Answer: reassuranse
Breast-feeding considerations

Phenytoin is excreted in breast milk; however, the amount to which the infant is exposed is considered small. The
manufacturers of phenytoin do not recommend breast-feeding during therapy. Phenytoin, carbamazepine and valproate are
probably safe

Reference: Http://www.uptodate.com/contents/phenytoin-drug-information?Source=see_link&utdpopup=true

436.Child with croup what is the best initial investigation to diagnosis?


A. X-ray
B. Pharyngeal swab
Answer: the diagnosis of croup is clinical, based upon the presence of a barking cough, hoarseness, and stridor. Neither
radiographs (if x-ray is performed, it will show steeple sign) nor laboratory tests are necessary to make the diagnosis. However,
radiographs may be helpful in excluding other causes if the diagnosis is in question.
Confirmation of etiologic diagnosis is not necessary for most children with croup, since croup is a self-limited illness that usually
requires only symptomatic therapy. When an etiologic diagnosis is necessary, viral culture and/or rapid diagnostic tests that
detect viral antigens are performed on secretions from the nasopharynx or throat.
Http://www.uptodate.com/contents/croup-clinical-features-evaluation-and-
diagnosis?Source=search_result&search=croup&selectedtitle=2%7e59#h23

437.Young patient compline of scrotal pain, examination is normal, us normal, urine analysis show pyuria . What is the next
step?
A. Refer to surgery
B. Give him azithromycine and cefxime
Answer: give him azithromycine and cefxime
107
This is a case of epididydimitis
This patient has epididymitis. In males 14-35 years of age, the most common causes are neisseria gonorrhoeae and chlamydia
trachomatis. The recommended treatment in this age group is ceftriaxone, 250 mg intramuscularly, and doxycycline, 100 mg
twice daily for 10 days. A single 1-g dose of azithromycin may be substituted for doxycycline. In those under age 14 or over age
35, the infection is usually caused by one of the common urinary tract pathogens, and levofloxacin, 500 mg once daily for 10
days, would be the appropriate treatment.
But testicular torsion cause severe pain without urinary symptoms diagnosis by color doppler ultrasonography will show a
normal-appearing testis with decreased blood flow. And it need ergent surgical intervention

438.Picture of growth chart all parameters was low what is the dx?
A. Genetic
B. Gh def
Answer: depends on the chart
• If the weight and hight proportionally small >> chromosomal “genetic”
• If the wight fall more that the hight “ftt” >> due to chronic illness , lack of intake
• If hight fall more than the wight “shrt stature” >> endocrine .
• Ref : uptodate

439.Patient with barky cough ......what is the organism?


A. Parainflunza
B. H.infelunza
Answer:
Croup (laryngotracheitis) characterized by inspiratory stridor, barking cough, and hoarseness. Is chiefly caused by parainfluenza
virus.
Http://www.uptodate.com/contents/croup-approach-to-management

440.Patient with respiratory sx..between attack ..he looks apathetic. .what u to investigate?

A. Sputum culture
B. Sweet chloride.

Answer:

441.7 y child and an u & l respiratory tract infection since birth with generalized joint pain aches his uncle and brother have
the same condition labs show high creatinine and bun and +ve urine protein
A. Idiopathic fibrosis
B. Autoimmune
Answer: ?

442.12 year-old mild jaundice, splenomegaly , echogenic shadow of gall bladder


A. Sca
B. Thalassemia
Answer: hereditary spherocytosis

443.Child ingested a caustic material he present to er crying drooling what to do 1st?


A. Maintain airway
B. Activated charcoal
Answer: (a - i think, because activated charcoal is contraindicated in causatic material ingestion
reference: http://emedicine.medscape.com/article/813772-treatment#d10

444.A child alert, anterior fontanelle depressed, how much dehydration:


108
A. 5 - 9
B. >9
Answer: a
Mild 5%: normal fontanelle
Moderate 6-10%: sunken slightly
Severe >10 %: sunken significantly
Http://www.utmb.edu/pedi_ed/core/fluids&electyrolytes/page_09.htm

445.Adolescent male with swollen parotid and salivary gland with dry eye and dry mouth, labs hla, ana and rf are positive
which of the following is appropriate treatment?
A. Physostigmine
B. Artificial eye and saliva drops
Answer: b this is case of sjogren’s syndrome
Http://www.uptodate.com/contents/treatment-of-dry-eye-in-sjogrens-
syndrome?Source=outline_link&view=text&anchor=h3377156#h3377156
Http://www.uptodate.com/contents/diagnosis-and-classification-of-sjogrens-syndrome
- sjögren’s syndrome (ss) is a chronic autoimmune inflammatory disorder characterized by diminished lacrimal and salivary gland
function. The diminished exocrine gland function leads to the “sicca complex,” a combination of dry eyes and dry mouth.
-ss is most common in women in their 50s and 60s but can affect adolescents and young adults, as well as men.
-prominent parotid and lacrimal glandular enlargement, may result from ss.
-anti-ro/ssa and/or anti-la/ssb antibodies, “rheumatoid factor and ana might be +ve but not might be -ve not specific”.
-treatment of dry mouth due to salivary gland hypofunction aims to alleviate symptoms and prevent complications with artificial
tears and saliva.

446.3 year old child with uti admitted what investigation to be done ?
A. Us
B. Cystoscope

Answer: us
Http://www.uptodate.com/contents/urinary-tract-infections-in-infants-older-than-one-month-and-young-children-acute-
management-imaging-and-prognosis

447.Child with croup what is the best initial investigation to diagnosis?


A. X-ray
B. Pharyngeal swab
Answer a- xray
Clinical diagnosis; if the diagnosis is unclear, patients should have ap and lateral x-rays of the neck and chest; subepiglottic
narrowing (steeple sign) seen on ap neck x-ray supports the diagnosis.
Http://www.merckmanuals.com/professional/pediatrics/respiratory-disorders-in-young-children/croup

448.Children diagnosed to be hypertensive :


A. Bp above 90th percentile
B. Bp above 95th percentile
Answer : b
sbp and/or dbp>= 95th percentile for sex , age , height on >= 3 occasions
(toronto note )

449.Child with urti is complaining of bleeding from nose, gum and bruising the treatment is:
A. Prednisolone
B. Ivig

Answer : a

109
explanation: itp (idiopathic thrombocytopenic purpura) can occur in anyone at almost any age, but these factors increase your
risk:
1. Women are about twice as likely to develop itp as men are.
2. Recent viral infection. Many children with itp develop the disorder after a viral illness, such as mumps, measles or a
respiratory infection.

- common medications used to treat idiopathic thrombocytopenic purpura include:


• Corticosteroids. The first line of therapy for itp is a corticosteroid, usually prednisone, which can help raise your
platelet count by decreasing the activity of your immune system. Once your platelet count is back to a safe level, you
can gradually discontinue taking the drug..
• Intravenous immune globulin (ivig). If you have critical bleeding or need to quickly increase your blood count before
surgery, you may receive medications, such as immune globulin, given intravenously.
• Thrombopoietin receptor agonists. The newest medications approved to treat itp are romiplostim (nplate) and
eltrombopag (promacta). These drugs help your bone marrow produce more platelets, which helps prevent bruising
and bleeding. Possible side effects include headache, joint or muscle pain, dizziness, nausea or vomiting, and an
increased risk of blood clots.
• Biologic therapy. Rituximab (rituxan) helps reduce the immune system response. It's generally used for people with
severe itp, and in those who corticosteroids don't help. Possible side effects include low blood pressure, fever, sore
throat and rash
Soure:http://www.mayoclinic.org/diseases-conditions/idiopathic-thrombocytopenic-purpura/basics/treatment/con-20034239

450.Child came with wheezing and cough and diagnosed with asthma and his dr. Prescribe to him beclomethasone nebulizer
what most worried side effect of using it:
A. Growth retardation
B. Extaoriculer problem
Answer : a- growth retardation
The most worrying potential systemic effects are adrenal insufficiency, growth suppression, glaucoma and osteoporosis
Reference
http://www.ncbi.nlm.nih.gov/pubmed/17602347

451.Child with septic arthritis how to manage


A. Drainage and abx
B. Broad spectrum iv abx
Answer: a

Medical management of infective arthritis focuses on adequate and timely drainage of the infected synovial fluid, administration
of appropriate antimicrobial therapy, and immobilization of the joint to control pain.

Http://emedicine.medscape.com/article/236299-treatment

452.Mumps in child what is the most common complication ?


A. Orchitis
B. Meningitis
Answer: b
The most common complication of mumps in children is meningitis, sometimes associated with encephalitis, and in young adults
orchitis. Http://www.who.int/biologicals/areas/vaccines/mmr/mumps/en/

453.Which of the is mostly associated sign with croup:


110
A. Dysphonia
B. Cyanosis
Answer: a hoarseness
Uptodate

454.Child ate a number of iron tablets presented with severe symptoms including constipation and bloody stool nausea and
vomiting and drowsiness how would you treat him:
A. Iv desferoxamine.
B. Dialysis
Answer: a
Explanation: answer may be desfroxamine which is an antidote to iron (chelating agent) used to bring down iron levels, it is also
used as treatment of hemochromatosis (disease of iron over-absorption in the gut), another example to chelating agents is
penicilliamine used for the copper overload in wilson’s disease.

455.2-month old has diarrhea and his mother is worried from dehydration what will you advice the mother?
A. Change milk
B. Oral rehydration solution

Answer: b
Read about kawasaki disease???

456.What is the commonest cause of htn in adolescent :


A. Idiopathic
B. Renal
Answer: a
Explanation: disease or renal artery stenosi (secondary to renal involvement) .. Most childhood hypertension, particularly in
preadolescents, is secondary to an underlying disorder (table 27). Renal parenchymal disease is the most common (60 to 70
percent) cause of hypertension.23
Adolescents usually have primary or essential hypertension, making up 85 to 95 percent of cases.23 table 323–25 shows causes
of childhood hypertension according to age.
Source: http://www.aafp.org/afp/2006/0501/p1558.html

457.Boy with asymptomatic hematuria the most important investigation in this case is :
A. Cbc
B. Urine analysis
Answer: b
Hematuria cause

111
458.Girl came with upper arm bone pain investigation was given with anemia and high bilirubin and high reticulocyte count
what most appropriate next step:
A. Hb electrophoresis
B. Arm x-ray

112
Answer: a
Explanation: we are suspecting sickle cell anemia in this patient due to anemia (from sickle shaped rbc rupture), high bilirubin
(from rbc hemolysis) and high reticulocytes (young rbc trying to compensate rbc loss), and most importantly bone pains that can
be explained by the occurrence of a vaso-occlusive crisis known to happen to sicklers.

459.An infant came with cyanosis during feeding and crying (there was many thing in scenario) what is the management:
A. Prostaglandin
B. Surgical repair
Answer: depend in the full history with which cyanotic congenital heart disease
Tetralogy of fallot (most likely the scenario about it and the treatment is surgical)
• 10% of all chd, most common cyanotic heart defect diagnosed beyond infancy
• embryologically, a single defect with hypoplasia of the conus causing:
vsd + right ventricle (rv) outflow tract obstruction (rvoto) (e.g. Pulmonary stenosis) + overriding aorta + rvh
• infants may initially have a l ~ r shunt and therefore are not cyanotic but the rvoto is progressive, resulting in increasing r ~ l
shunting with hypoxemia and cyanosis
• history: hypoxic "tet" spells
• primary pathophysiology is hypoxia, leading to increased pulmonary vascular resistance
(pvr) and decreased systemic resistance, occurring in exertional states (e.g. Crying, exercise)
• paroxysm of rapid and deep breathing, irritability and crying
• hyperpnea, increasing cyanosis often leading to deep sleep and decreased intensity of
murmur (decreased flow across rvoto)
• peak incidence at 2-4 months of age
• if severe may lead to seizures, loss of consciousness, death (rare)
• management: 0 2, knee-chest position, fluid bolus, morphine sulfate, propanolol
• physical exam: single loud s2 due to severe pulmonary stenosis (i.e. Rvoto)
• investigations
• ecg: rad, rvh
• cxr: boot shaped heart (small pa, rvh), decreased pulmonary vasculature, right aortic
arch (in 20%)
• treatment: surgical repair within first two years of life, or earlier if marked cyanosis, "tet" spells, or severe rv outflow tract
obstruction
ebstein's anomaly
• congenital defect of the tricuspid valve in which the septal and posterior leaflets are malformed and displaced into the rv
leading to variable degrees of rv dysfunction, ts, tr or functional pulmonary atresia ifrv unable to open pulmonic valves
• ra massively enlarged, interatrial communication and patent foramen ovale (pfo) often exists allowing r ~ l shunting
• tr and accessory conduction pathways (wpw) are often present - often associated with arrhythmia
• cause: unknown, associated with maternal lithium and benzodiazepine use in 1st trimester
• treatment: in newborns, consider closure of tricuspid valve + aortopulmonary shunt, or transplantation • in older children,
tricuspid valve repair or valve replacement + asd closure
transposition of the great arteries (tga)
• 3-5% of all congenital cardiac lesions, most common cyanotic chd in neonate
• parallel pulmonary and systemic circulations
• systemic: body ~ ra ~ rv ~ aorta ~ body
• pulmonary: lungs ~ la ~ lv ~ pulmonary artery ~ lungs • physical exam
• no murmur if no vsd
• newborn presents with progressive cyanosis unresponsive to oxygen therapy as the ductus
arteriosus closes and mixing between the two circulations diminishes; severe hypoxemia,
acidosis, and death can occur rapidly
• if vsd present, cyanosis is not prominent and infant presents with chf after a few weeks of
life
• investigations
• ecg: rad, rvh
• cxr: egg-shaped heart with narrow mediastinum ("egg on a string") • treatment

113
• prostaglandin e1 (prostin vr'") infusion to keep ductus open until septostomy or surgery (arterial switch procedure)
• infants without vsd must be repaired within 2 wks to avoid weak lv muscle
toronto notes: pediatrics p23

460.Which of the following is true regarding deutschen muscular dystrophy:


A. Positive gower’s sign
B. Muscle atrophy on biopsy
Answer: a

461.Baby was delivered 30 weeks and has respiratory symptoms what will be the main cause?
A. Asthma
B. Decreased pulmonary surfactant
Answer: b

In premature infant surfactant production is often inadequate to prevent alveolar collapse and atelectasis, which result in
respiratory distress syndrome

462.Baby 2 weeks old present with jaundice started 7 days ago... High indirect bilirubin 19.3... How to manage?
A. Phototherapy
B. Transfusion
Answer: a
Phototherapy

For neonates born at ≥ 35 wk gestation, phototherapy is an option when unconjugated bilirubin is >12 mg/dl (> 205.2 μmol/l)
and may be indicated when unconjugated bilirubin is > 15 mg/dl at 25 to 48 h, 18 mg/dl at 49 to 72 h, and 20 mg/dl at> 72 h.
Phototherapy is not indicated for conjugated hyperbilirubinemia.
For neonates born at < 35 wk gestation, threshold bilirubin levels for treatment are lower because premature infants are at a
greater risk of neurotoxicity. The more preterm the infant, the lower the threshold

Exchange transfusion
For term infants, specific indications are serum bilirubin ≥ 20 mg/dl at 24 to 48 h or ≥25 mg/dl at > 48 h and failure of
phototherapy to result in a 1- to 2-mg/dl (17- to 34-μmol/l) decrease within 4 to 6 h of initiation or at the first clinical signs of
kernicterus regardless of bilirubin levels. If the serum bilirubin level is > 25 mg/dl when the neonate is initially examined,
preparation for an exchange transfusion should be made in case intensive phototherapy fails to lower the bilirubin level.

Https://www.msdmanuals.com/professional/pediatrics/metabolic,-electrolyte,-and-toxic-disorders-in-neonates/neonatal-
hyperbilirubinemia
463.4 months on breastfeeding, came with 2 days hx of lethargy constipation, fever, response weak when light directed to his
eyes (long scenario), cause?
A. Hypothyroidism
B. Infantile botulism
Answer: b
Infants typically present with constipation and poor feeding. This presentation is followed by progressive hypotonia, and
weakness. Loss of deep tendon reflexes appears to occur more commonly in type b infection [37]. Cranial nerve dysfunction is
manifested by decreased gag and suck, diminished range of eye movement, pupillary paralysis, and ptosis. Autonomic signs
include decreased tearing and salivation, fluctuating heart rate and blood pressure, and flushed skin.
Http://www.uptodate.com.sci-hub.cc/contents/neuromuscular-junction-disorders-in-newborns-and-
infants?Source=see_link&sectionname=infant+botulism&anchor=h14#h14

464. Developmental milestone expected in a 3 year old?


A. Climbs stairs
B. Catches ball with foot
114
Answer: a
Https://www.msdmanuals.com/professional/pediatrics/growth-and-development/childhood-development

Child presents with hematuria, history of constipation 2 weeks ago and now she has incontinence. Urinalysis showed rbcs
and wbcs. What’s the most likely diagnosis?
A. Post-streptococcal gn
B. Hsp
Answer: uti

A history of new onset of incontinence, dysuria, frequency, or urgency suggests urinary tract infection (uti)
465.A case of pediatric nephrotic syndrome with edema, what will you give him?
A. Steroid trial
B. Diuretic
Answer: (a or b) we need more information. The best treatment here (steroid trial) and if they said initial treatment it should be
with diuretic to control edema. And symptomatic treatment is mainstay in children who fail to respond to steroid.
Rx:
Idiopathic ns is most common form of ns in children, according to response to steroid are classified as:
1- steroid sensitive: initially (no relapses) prednisone 60 mg/m2, if relapses or frequent the iskdc recommends a prednisone dose
of 60 mg/m2 per day (maximum of 60 mg/day) be initiated when a patient has relapsed and continued for three days
(according to uptodate) we administer daily prednisone, 40 to 60 mg/m2, until proteinuria has disappeared for four to five days,
followed by alternate-day therapy with tapering by 15 to 20 mg/m2 every other day to the patient's steroid threshold
2- steroid resistance: 1st kidney biopsy then genetic screening, in non-genetic causes we suggest combinations of cyclosporine
or tacrolimus and prednisone.
For symptomatic management of nephrotic syndrome in children
1- edema: salt and fluid restrictions, diuretics (options are: furosemide, thiazide and amiloride)
2- hypercoagulability: preventative measures include regular ambulation, avoidance of hemoconcentration resulting from
hypovolemia, avoidance of central venous catheter if possible. Treatment of venous thromboembolism — anticoagulation is
most often initiated with low molecular weight heparin.
3- infection: abx, most common organisms are strept. Pneumonia followed by e. Choli
4- hyperlipidemia: statins should be used with caution, and dietary modification
5- hypertension: acei
The majority of children with nephrotic syndrome (ns) will respond to steroid therapy. However, symptomatic management is
important in the early course of therapy, as response to steroid therapy may take several weeks, and it is the mainstay of
therapy in children who fail to respond to steroids. (uptodate)

466.Mother with gbs and had a baby who has irritability and agitation and fever. What will you do?
A. Give antibiotics
B. Do cultures ✅
Answer: b
Best next step do cultures (spinal and blood) then treat with iv abx (penicillin or ampicillin)
Http://www.cdc.gov/groupbstrep/about/symptoms-diagnosis-treatment.html

467.Baby 2 weeks old present with jaundice started 7 days ago .. High indirect bilirubin 19.3 .. How to manage
A. Phototherapy
B. Transfusion
Answer: a
But we should know more details about baby (term or preterm and tb level)
But for initial rx in general we go with phototherapy
Management of neonatal hyperbilirubinemia: overview
-phototherapy starting initially as prevention also to reduce the total bilirubin later, it should be started initially
-exchange transfusion should be preserved for severe cases with total bilirubin >25 mg/dl or with severe symptoms
For term infants without risk factors:

115
-phototherapy indications:
•24 hours of age: >12 mg/dl (205 micromol/l)
•48 hours of age: >15 mg/dl (257 micromol/l)
•72 hours of age: >18 mg/dl (308 micromol/l)
-exchange transfusion indications:
•24 hours of age: >19 mg/dl (325 micromol/l)
•48 hours of age: >22 mg/dl (376 micromol/l)
•72 hours of age: >24 mg/dl (410 micromol/l)
•any age greater than 72 hours : ≥25 mg/dl (428 micromol/l)
Term infants with risk factors or late preterm infants without risk factors
-phototherapy indications:
•24 hours of age: >10 mg/dl (171 micromol/l)
•48 hours of age: >13 mg/dl (222 micromol/l)
•72 hours of age: >15 mg/dl (257 micromol/l)
-exchange transfusion indications:
•24 hours of age: >16.5 mg/dl (282 micromol/l)
•48 hours of age: >19 mg/dl (325 micromol/l)
•≥72 hours of age: >21 mg/dl (359 micromol/l)

Late preterm infants with risk factors


-phototherapy indications
•24 hours of age: >8 mg/dl (137 micromol/l)
•48 hours of age: >11 mg/dl (188 micromol/l)
•72 hours of age: >13.5 mg/dl (231 micromol/l)
-exchange transfusion indications
•24 hours of age: >15 mg/dl (257 micromol/l)
•48 hours of age: >17 mg/dl (291 micromol/l)
•≥72 hours of age: >18.5 mg/dl (316 micromol/l)
(uptodate)

468.5 years old boy presented with sore throat, he was discharged home, culture was done showing group a meningococcus,
the physician called the family to inform them he finds that the child is asymptomatic, the best treatment is:
A. Penicillin ✅
B. Single dose ceftriaxone
Answer: a or b
"a range of antibiotics can treat the infection, including penicillin, ampicillin, chloramphenicol and ceftriaxone. Under epidemic
conditions in africa in areas with limited health infrastructure and resources, ceftriaxone is the drug of choice."
Http://www.who.int/mediacentre/factsheets/fs141/en/

469.When do gilbert disease patients have jaundice?


A. At birth
B. With concurrent disease ✅
C. While feeding
Answer: b
The most common inherited disorder of bilirubin glucuronidation is gilbert syndrome (also known as meulengracht disease).
Gilbert syndrome is a benign condition that has also been called "constitutional hepatic dysfunction" and "familial nonhemolytic
jaundice" . Although many patients present as isolated cases, the condition can also run in families. It is characterized by
recurrent episodes of jaundice and may be triggered by, among other things, dehydration, fasting, intercurrent disease,

116
menstruation, and overexertion . Other than jaundice, patients are typically asymptomatic. The hyperbilirubinemia in patients
with gilbert syndrome is unconjugated. (uptodate/toronto)

470.Baby, 3rd day after delivery got a purulent eye discharge what is the organism:
A. Chlamydia
B. Gonorrhea
Answer: b (ophthalmia neonatorum)
Chlamydia: the incubation period for c. Trachomatis conjunctivitis is 5 to 14 days after delivery. Presentation before five days is
unusual, discharge watery then mucopurulent
Gonorrhea: infection typically causes a purulent conjunctivitis, with profuse exudate and swelling of the eyelids. The infection
usually becomes manifest two to five days after birth
(uptodate)

471.7 year girl pupic hair growing, what is best investigation:


A. Fsh
B. Ct scan
Answer: b (best investigation for definitive imaging of adrenals if high suspicion of tumor.) (medscape)
Similar q with different answer:

472.Child ingested a caustic material he present to er crying drooling what to do 1st?


A. Maintain airway
B. Activated charcoal
Answer: a
Ref. E medicine
In the treatment area, patients suspected of ingesting a caustic substance should be triaged to a high priority for prompt
evaluation and treatment. This includes prompt evaluation of airway and vital signs as well as immediate cardiac monitoring and
intravenous access. Intravenous fluids and blood products may be required in the event of significant bleeding, vomiting, or
third spacing.

Airway control
because of the risk of rapidly developing airway edema, the patient’s airway and mental status should be immediately assessed
and continually monitored. Equipment for endotracheal intubation and cricothyrotomy should be readily available. Gentle
orotracheal intubation or fiberoptic-assisted intubation is preferred. Blind nasotracheal intubation should be avoided due to the
increased risk of soft-tissue perforation.
If possible, it is best to avoid inducing paralysis for intubation because of the risk of anatomical distortion from bleeding and
necrosis. If a difficult airway is anticipated, iv ketamine can be used to provide enough sedation to obtain a direct look at the
airway.
Cricothyrotomy or percutaneous needle cricothyrotomy may be necessary in the presence of extreme tissue friability or
significant edema.

Gastric emptying and decontamination


do not administer emetics because of risks of re-exposure of the vulnerable mucosa to the caustic agent. This may result in
further injury or perforation.
Gastric lavage by traditional methods using large-bore orogastric ewald tubes are contraindicated in both acidic and alkaline
ingestions because of risk of esophageal perforation and tracheal aspiration of stomach contents.
In large-volume liquid acid ingestions, nasogastric tube (ngt) suction may be beneficial if performed rapidly after ingestion.
Pyloric sphincter spasm may prolong contact time of the agent to the gastric mucosa for up to 90 minutes. Ngt suction may
prevent small intestine exposure. Esophageal perforation is rare. Ngt suction may be of particular value following ingestion of
zinc chloride, mercuric chloride, or hydrogen fluoride, unless signs of perforation are present. This should be done after
consulting with a regional poison control center.
Activated charcoal is relatively contraindicated in caustic ingestions because of poor adsorption and endoscopic interference.
Dilution

117
dilution may be beneficial for ingestion of solid or granular alkaline material if performed within 30 minutes after ingestion using
small volumes of water. Because of the risk of emesis, carefully consider the risks versus benefits of dilution.
Do not dilute acids with water; this would result in excessive heat production.

473.Physician in the clinic tell the child to bend forward and hang his both hands freely. This test is used in detection of ?
A. Rectal prolapse
B. Scoliosis
Answer : b
Adams forward bend test
Patient has to bend forward, starting at the waist until the back come in horizontal plane,with feet together , arms hanging and
the knee in extention, the palms are hold together , th examiner stand at the back of the patient and look along the horizental
plane of the supine.

474.Newborn came with cyanosis and tachypnea , there was decrease in breath sound in the left side and the heart sound
was louder in the right side.
What is the diagnosis?
A. Dexteocardia
B. Diaphragmatic hernia
Answer: b
475.Pediatric with murmur 2/6 disappear in spine position going for oral surgery?
A. Consult cardiology
B. Reassuring it's innocent
Answer: b (disappear with position)

476.27- 10 year old boy with chronic abd pain and bloody diarrhea , wt loss . Dx ?
A. Ulcerative colitis
B. Cronhs dz
Answer: a, uc more common in young age and more associated with bloody stool
Not enough info to distinguish between the two, since both can affect children and cause bloody diarrhea and weight loss.

477.3 year old child with uti admitted what investigation to be done ?
A. Us
B. Cystoscope
Answer: a
Question is not complete according to pediatric specialist but start with a non invasive test so a is more appropriate

118
478.Croup in details if recurrent after half hour of treatment what next mangement ?
What is the second line tx after epinephrine
A. Epinephrine can be repeated every 15-20min with monitoring the heart
B. O2 supplement and consider intubation if there is danger of airway compression
Answer: not clear (some references mention that epinephrine can be repeated every 2 hours however others prefer initiating
intubation once the child is considered not responsive to the treatment based on the child’s clinical presentation including paco2
and the severity of the illness in general)

479.Supra renal mass with mets to lung in child


A. Neuroblastoma
B. Wilms tumor
Answer: b
Neuroblastoma >> rarely metastasize to the lung

480.1700g baby at birth but now his weight is ok


A. Give half dose vaccine
B. Give full dose
Answer: b (give vaccine as scheduled)

481.Cr, k , bun > child 4 y


A. Daylysis
B. Ivf
Answer:
incomplete question

482.An infant came with cyanosis during feeding and crying (there was many thing in scenario ) what is the management:
A. Prostaglandin
B. Surgical repair
Answer: the scenario is missing a lot of details but i think they mean transposition of the great vessels .
The management will be different if it is with or without vsd:
1- if it was without vsd:
* the first thing to do is to give pge1 to keep the pda open
* early balloon septostomy
*arterial switch procedure is definitive
2-with vsd :
119
*pa band to control the increase pulmonary blood flow
*arterial switch with vsd closure procedure is definitive
First aid of the pediatrics
)all patients with transpostioning of great vessels require mixing of blood through maintaining the potency of ductus arteriosis
and in 20% of patients septostomy can be life saving. Arterial switch is mandatory in the first few days). Illustrated textbook of
pediatrics
If tetralogy of fallot surgery is done from 6 months of age if the infant is severely cyanosed a shunt might be used (modified
blalock–taussig shunt).

483.Boy medically free complaint of hematuria on urine analysis next step:


A. Repeat urine analysis.
B. Bladder scop
Answer: a
This is a case of microscopic hematuria which mean : > 3 rbcs on urinalysis of at least two separate sample
Limited to these two choices bladder scop is not indicated in the investigation for hematuria.
The most common cause of hematuria in children is uti
A source for the choice a could not be found

484.Patient with features of turner syndrome, they usually have:


A. Cardiovascular abnormality
B. High cholesterol
Answer: a

485.Newborn with red lump on left shoulder .. It is


A. Lipoma
B. Hemangioma
Answer: b
Https://en.wikipedia.org/wiki/hemangioma

486.Child ate a number of iron tablets presented with severe symptoms including constipation and bloody stool nausea and
vomiting and drowsiness how would you treat him
A. Iv deferoxamine.
B. Dialysis
Answer:a

Source: illustrated textbook of pediatrics.


Http://www.uptodate.com/contents/acute-iron-poisoning

487.14-month child come with murmur. O/e pt has vsd measure 4 ( or 2 ) mm , what will do?
A. Decrease after load
B. Reassure him
Answer :
Anatomic definition – however, one needs to consider the size of the patient especially in the neonate.
•small <4 m •moderate 4 to 6 mm •large >6 mm
Managment of small defect :

120
If the murmur persists at the 12-month cardiology visit and the patient remains asymptomatic and clinically stable, no further
intervention is required. Echocardiographic follow-up is typically performed at three years of age for patients with membranous
defects. In those with a muscular defect, no echocardiography is required if the patient remains asymptomatic
Up to date
According to medscape: in children with moderate or large vsds, medical therapy is indicated to manage symptomatic
congestive heart failure (chf) because some vsds may become smaller with time.
If the size is 4 = moderate and symptoms usually present with moderate and large so,,,

488.Child with positive hydrochloride test and some complication of disease, his brother is normal with no sign & symptoms.
A. U will do cftr for his parents
B. U will do chloride test
Answer: b
Http://www.uptodate.com/contents/image?Imagekey=peds/50886&source=graphics_search&rank=2~87&search=

489.40 - 60 word, count to 4 and


A. Tell story
B. Say mama baba
Answer: a

He should have already said (mama and baba) since he was 12 months old. So the closest answer is story telling which is in 5
years old. Kaplan pediatric development and toronto notes pediatrics.

490.Child complain of petechial rash all over the body . On examination there was a palpable spleen . There is a history of
urti, what’s the diagnosis?
A. Itp

121
B. Hsp

Answer: a

Toronto:
Hsp:
Vasculitis of small vessels • often have history of urti 1-3 wk before onset of symptoms. Clinical triad: 1) palpable purpura, 2)
abdominal pain, 3) arthritis
• skin: palpable, non-thrombocytopenic purpura in lower extremities and buttocks, edema, scrotal swelling
• joints: arthritis/arthralgia involving large joints associated with painful edema
• gi: abdominal pain, gi bleeding, intussusception
• renal: microscopic hematuria, iga nephropathy, proteinuria, htn, renal failure in <5%

Itp:
• 50% present 1-3 wk after viral illness (urti, chicken pox)
• sudden onset of petechiae, purpura, epistaxis in an otherwise well child suggestions
• clinically significant bleed in only 3% (severe bleed more likely with platelet count <10) with <0.5% risk of intracranial bleed •
no lymphadenopathy, no hepatosplenomegaly • labs: thrombocytopenia with normal rbc, wbc • bone marrow aspirate only if
atypical presentation (≥1 cell line abnormal, hepatosplenomegaly) • differential diagnosis: leukemia, drug-induced
thrombocytopenia, hiv, infection (viral), autoimmune (sle, alps)

Evb:
Classic triad: febrile, generalized non-tender lymphadenopathy, pharyngitis/tonsillitis (exudative) • ± hepatosplenomegaly • ±
periorbital edema, ± rash (urticarial, maculopapular, or petechial) – more common after inappropriate treatment with β-lactam
antibiotics • any “-itis” (including arthritis, hepatitis, nephritis, myocarditis, meningitis, encephalitis樀

491.Preterm baby complain of sob, x-ray showed gross ground appearance + air bronchogram, this is due to?
A. Pneumonia.
B. Low surfactant. (ards)

Answer: b

First aid:
• Ards is the most common cause of respiratory failure in preterm infants.
It’s caused by surfactant deficiency which leads to: poor lung compliance, alveolar collapse, atelectasis

492.Initial management of juvenile ra


A. Intra articular corticosteroid
B. Paracetamol
Answer: none
The intial therapy depends on the disease progression. However, usually the initial therapy is nsaid other than aspirin, then
dmards.
Http://www.uptodate.com/contents/systemic-juvenile-idiopathic-arthritis-treatment

493.Baby can sit without support, he can grasp, says mama, wave bye bye to the dr. Which developmental milestone he has
defect in?
A. Gross motor
B. Fine motor

Sit ~> gross motor


Grasp ~> fine motor
Say mama ~> speech and languag

494.Child with diarrhea, vomiting, abdominal pain, and bloating. What is the possible organism:
122
A. Entamoeba histolytica
B. Giardia lamblia

Answer: b
Toronto, Kaplan

495.Child with long history of enuresis, what is the most important investigation to be done:
A. Vcug
B. Urinalysis
Answer: b
Kaplan

496.Child come with case of immune deficiency with presented with lump in the groin and lap was given what is the diagnosis
?? There’s ch50 in q
A. Severe combined immune deficiency
B. Chronic granulomatous disease

Ch50 is a screening test for total complement activity. Levels of complement may be depressed in genetic deficiency, liver
disease, chronic glomerulonephritis, rheumatoid arthritis, hemolytic anemias, graft rejection, systemic lupus erythematosis,
acute glomerulonephritis, subacute bacterial endocarditis and cryoglobulinemia. Elevated complement may be found in acute
inflammatory conditions, leukemia, hodgkin's disease, sarcoma, and behcet's disease.

497.Child present with crying and lobulated breathing + retraction of intercostal muscle what to do
A. Prepare for intubation
B. Clam the baby down

498.Sign of duodenal atresia in x-ray


A. Bird peak appearance r.
B. Double babble appearance

Answer: b

499.Child came by his parents , they said that their son have recurrent lip swelling(angiooedema)so deficiency of which ?
A. Hereditary c1-inh deficiency
B. Deficiency anaphylactoid inhibitor

Answer: a
Http://emedicine.medscape.com/article/135604-overview#a2

500.Which of the is mostly associated sign with croup:


A. Dysphonia
B. Cyanosis
Answer: a

501.Child with hairless spot, mother noted she was pulling her hair when stressed, what to give her?
A. Lithium
B. Lorazepam
Other antiepileptic mentioned
Answer: ssri :citalopram, fluvoxamine, escitalopram, paroxetine, sertraline, fluoxetine
(uptodate) trichotillomania is an intriguing psychosomatic entity in which there is an irresistible desire to pull out the hair from
the scalp, eyelashes, eyebrows and other parts of the body. The process results in an instant release of tension, a sense of relief
and security. However, non-scaring alopecia is its clinical presentation. The development of trichobezoar following ingestion of
the pulled hair is its salient complication in a few cases. Subsequently, it may cause symptoms pertaining to the gastrointestinal
tract culminating in intestinal obstruction, perforation, pancreatitis and obstructive jaundice. The rapunzel syndrome

123
(trichobezoar) may occur when gastrointestinal obstruction is produced by a rare manifestation of a trichobezoar with a long tail
that extends to or beyond the ileocecal valve. In most cases in children, trichotillomania +/- trichobezoar is a habit disorder and
thus has a better prognosis. However, in adults the psychopathology is usually deeper and thus entails a poor prognosis. The
diagnosis is made after taking a thorough history, noting the clinical features and evaluating a hair-root examination, where
telogen hair is (almost) completely lacking, which distinguish trichotillomania from other hair disorders. Treatment modalities
vary in childhood and adult varieties. Apart from psychotherapy, the drug treatment involves several agents including selective
serotonin reuptake inhibitors (ssris) and domipramine. Trichobezoar/rapunzel syndrome requires surgical intervention.

502.Child with itchy scalp and scales, other classmates are affected, what is the most likely diagnosis?
A. Tinea capitis
B. Scabies
Answer: a
Tinea capitis is a highly contagious disease caused by superficial fungal infection of the skin of the scalp, eyebrows and
eyelashes. Pruritus usually is minimal but may be intense at times. Alopecia is common in infected areas.
Source: http://emedicine.medscape.com/article/1091351-clinical

503.Baby ride tricycle draw circle but can’t draw square:

A. 3 years
B. 4 years

Answer: a
Illustrated textbook

504.Child with mild jaundice ,splenomegaly and echogenicity in the gallbladder .what is the type of anemia?!
A. Sickle cell anemia
B. Thalassemia
Answer: a toronto notes

505.A case of pediatric nephrotic syndrome with edema, what will you give him ?
A. Steroid trial
B. Diuretic
Answer: a most common in children is minimal change dis treat it by steroids. First aid step2 ck

506.Baby was delivered 30 weeks and has resp symptoms what will be the main cause?
A. Asthma
B. Decreased pulmonary surfactant
Answer: b

507.Baby brought by his parents to er complaining of sob & drooling saliva ................... Diagnosis:
124
A. Croups
B. Epiglottitis
Answer: b.
The presentation of sob with drooling points towards epiglottis which is an infection of epiglottis in toddler and young school
age. However croup usually presents with stridor and barking cough in preschoolers and in winter usually. Kaplan pediatrics
(respiratory disease chapter)

508.2 year child didn't complete his vaccination ( 10 months is the last one ) present with fever bilateral swelling
pediauricular , unable to swallow , dysphasia , enlarged tonsils and spleen and lymph nodes...( no infectious
mononucliosis nor mump in the choices):
A. Diphtheria
B. Streptococcus pharyngitis
Answer: a
Diphtheria pharyngitis will cause all the prementioned signs and symptoms in addition to a pathognomic grayish membrane.
Http://emedicine.medscape.com/article/963334-clinical#b4

509.Y/o amnorrhea ,short stature ,htn,broad neck ,also parent short stature dx:
A. Familial
B. Turner
Answer: b
Https://ghr.nlm.nih.gov/condition/turner-syndrome

510.1 month old baby , has constipation science birth , best diagnostic investigation ?
A. Manometry
B. Full thickness intestinal biopsy .

Answer: b
Explanation: hirschsprung disease should be considered in any newborn with delayed passage of meconium or in any child with
a history of chronic constipation since birth. The definitive diagnosis of hirschsprung disease is confirmed by a full-thickness
rectal biopsy demonstrating absence of ganglion cells.
Reference: http://emedicine.medscape.com/article/178493-overview

511.Full term baby on breast milk and gaining weight , presented 10 days after birth with yellowish discoloration , indirect bili
19.5 , direct 2.7 ,what to do next :
A. Blood exchange
B. Phototherapy

Answer: b
Explanation: phototherapy is an effective and safe method for reducing indirect bilirubin levels. In term infants, phototherapy is
begun when indirect bilirubin levels are between 16 and 18 mg/dl. For blood exchange, a level of 20 mg/dl for indirect bilirubin
for hemolytic infantsw weighing more than 2000 g. Asymptomatic infants with physiologic or breast milk jaundice may not
require exchange transfusion, unless the indirect bilirubin level exceeds 25 mg/dl.
th
Reference: nelson essentials of pediatrics 7 edition, page 221

512.Baby with blood jelly stool ( was case of intussusption ) what will u do :
A. Us
B. Barium enema

Answer: b
Explanation: currant jelly stool is associated with intussusception. The traditional and most reliable way to make the diagnosis
of intussusception in children is to obtain a contrast enema (either barium or air). Contrast enema is quick and reliable and has
the potential to be therapeutic.
125
Reference: http://emedicine.medscape.com/article/930708-workup#c10

513.5 years old baby presented with sever lower limb pain , growth parameters under the 5th percentile , low hg :
A. Osteomyelitis
B. Vasocclusive crisis

Answer: b
Explanation: the lower limb pain (vaso-occlusive crisis), growth retardation, and low hemoglobin all are symptoms of sickle cell
disease. The most common clinical manifestation of scd is vaso-occlusive crisis. During childhood and adolescence, scd is
associated with growth retardation, delayed sexual maturation, and being underweight.
Reference: http://emedicine.medscape.com/article/205926-clinical

514.What is the features that seen in patient with congenital adrenal hyperplasia?
A. Hirsutism
B. Dehydration

Answer: b
Explanation: depends on which enzyme in cortisol synthesis pathway is defective. Presentation of 21-oh deficiency (most
common). Divided into classic deficiency with salt wasting and dehydration (50%): inadequate aldosterone resulting in failure to
thrive, hyperkalemia, hyponatremia, hypoglycemia, acidosis (most male presentations)
Classic deficiency without salt wasting: simple virilizing, female ambiguous genitalia
Non-classic: androgen excess (e.g. Amenorrhea, precocious puberty, hirsutism, etc.)
References: http://emedicine.medscape.com/article/919218-clinical
Toronto notes 2015, page p30 (pediatrics)

515.What u will see on physical examination of pt with croup ??


A. Presence of inspiratory sounds
B. Presence of expiratory wheez

Answer: a
Explanation: the characteristic croup signs include: hoarseness, barking cough, and inspiratory stridor develop, often suddenly,
along with a variable degree of respiratory distress.
Reference: http://emedicine.medscape.com/article/962972-clinical

516.Child 3 year old, what can he do?


A. Clime stair.
B. Trace triangular.

Answer: a
Explanation:

Reference kaplan step 2ck pediatrics 2014

517.Modified dukes criteria:


A. 1 major 2 minor
B. 1major 3 minor

126
Answer: b
Explanation:
2 major criteria and 0 minor criteria
1 major criteria and 3 minor criteria
0 major criteria and 5 minor criteria
Reference http://reference.medscape.com/calculator/endocarditis-diagnostic-criteria-duke

518.What is the type of murmur in atrial septal defect?


A. Fixed split in s2
B. Ejection systolic click

Answer: a.
Explanation:
Auscultation in septal defects
Atrial septal defect Fixed wide splitting of s2
Ventricular septal defect Harsh holosystolic murmur loudest at tricuspid area

Ejection systolic clicks are associated with congenital aortic or pulmonary stenosis.
Reference http://emedicine.medscape.com/article/892980-clinical#b2
Http://emedicine.medscape.com/article/162914-clinical#b2
Macleod’s clinical examination

519.Asystole first treatment in a child?


A. Epinephrine
B. Atropine.
Answer: a
Explanation: epinephrine remains the drug of choice for asystole in children. Atropine is not indicated.)
Reference: http://www.aafp.org/afp/1999/1015/p1743.html

520.A three weeks old boy presented with scrotal asymmetry. In examination both testicles were palpable in the scrotum.
(that’s it, with no more information). What are you going to do next in examination:
A. Trans-illumination test
B. Let the patient to cry/cough to see if the is any bulging

Answer: a
Http://www.mayoclinic.org/diseases-conditions/hydrocele/basics/tests-diagnosis/con-20024139

Child ate honey then he develop fluccid paralysis :


Answer: clostridium botulinum
Botulism is a broad term encompassing 3 clinical entities caused by botulinum toxin. Propagation of this toxin under different
circumstances can lead to food-borne, wound, or infant botulism. Infants with botulism are afebrile, suck poorly, and are
lethargic and listless. They often are described as being floppy.[9] constipation is almost always the first sign of infant botulism.
Http://emedicine.medscape.com/article/961833-clinical

521.Initial management of juvenile ra


A. Intra articular corticosteroid
B. Paracetamol

127
Answer: nsaids!
Most with pauciarticular disease respond to nonsteroidal antiinflammatory drugs (nsaids) alone (kaplan pediatrics)

522.2 month old boy with diarrhea and the mother afraid from dehydration? The best advice ?
A. Changing milk
B. Ors

Answer: b
Oral rehydration therapy (ort) is a type of fluid replacement used to prevent or treat dehydration especially that due to diarrhea.
It involves drinking water with modest amounts of sugar and salt added (an oral rehydration solution or ors) while continuing to
eat. Routine therapy also includes supplemental zinc.
Https://en.m.wikipedia.org/wiki/oral_rehydration_therapy

523.Say few words at the age of which ?


A. 24m
B. 12m
Answer: b
Https://www.healthychildren.org/english/ages-stages/baby/pages/developmental-milestones-12-months.aspx

524.Neonate with bilious vomiting, seed stool diarrhea he passed meconium in day 2 what is the cause?
A. Hirschsprung disease
B. Allergy to formula milk

Answer: b

525.Newborn with umbilical hernia. What should tell the mother ?


A. Reassure
B. Should treat it before school

Answer: a or b!
Explanation: most umbilical hernias do not cause any symptoms and do not require surgical repair until approximately age 5
years. For that reason, almost all umbilical hernias in young children and infants are managed by simple observation.
Reference: http://emedicine.medscape.com/article/932680-treatment

526.Newborn girl ( i did not remember the complain but there is enlargment of clitoris ( i think this is congenital adrenal
hyperplasia, cah ) ask about treatment ?
A. Fluid
B. Hydrocortisone

Answer: b (for treatment of cah, but if complaint is dehydration, answer is a)


Explanation: correct any abnormalities in fluids, electrolytes, or serum glucose. Provide glucocorticoids/mineralocorticoids as
necessary, extra glucocorticoids in times of stress. Psychosocial support.
Reference: toronto notes 2015, page p31, pediatrics

527.Long history of child with history of infectious mononucleosis( upper respiratory tract infection with
hepatosplenomegaly) what is your next step ?

A. Cbc
B. Monospot test
Answer: b

128
Heterophile test antibodies (e.g:monospot test)are sensitive and specific for ebv heterophile antibodies, they are present in
peak levels 2-6 weeks after primary ebv infection, and they may remain positive in low levels for up to a year.
Reference: http://emedicine.medscape.com/article/222040-overview

528.Child presented with convulsion after ge:


A. Salmonella
B. Shigella

Answer: b

Generalized seizures have been reported occasionally among young children with shigellosis, and usually resolve without
treatment. Children who experience seizures while infected with shigella typically have a high fever or abnormal blood
electrolytes (salts), but it is not well understood why the seizures occurs.

Http://www.cdc.gov/shigella/general-information.html#definitions-symptoms

529.Sabin (polio vaccine) iv & oral both do:


A. Increase intestinal iga secretion
B. Attach on anterior horn to prevent viral attachment
The choices are incomplete, but in general, opv increases iga secretion and shedding of the virus into the feces whereas ipv
induces "serum" antibodies.
Reference: http://www.ncbi.nlm.nih.gov/pubmed/6740072

530.A boy draws his brother .till a story. Play a role as father how old:
A. 4
B. 5
Answer: b
Reference: kaplan pediatrics, page 40 (playing pretend + tells a story is 5)

531.Baby brought by his parents to er complaining of sob & drooling saliva ................... Diagnosis:
A. Croups
B. Epiglottitis
Answer: b
The clinical triad of drooling, dysphagia, and distress is the classic presentation. Fever with associated respiratory distress or air
hunger occurs in most patients. Drooling occurs in up to 80% of cases.
Reference: http://emedicine.medscape.com/article/963773-clinical#showall

532.2 year child didn't complete his vaccination ( 10months is the last one ) present with fever bilateral swelling pediauricular
, unable to swallow , dysphasia , enlarged tonsils and spleen and lymph nodes...( no infectious mononucliosis nor mump
in the choices):
A. Diphtheria
B. Streptococcus pharyngitis

Mmr is a two-shot series of vaccines usually given during childhood. A child should receive the first shot when he is
between 12-15 months, and the second when he's between 4-6 years of age.
Reference : http://www.aafp.org/afp/2004/0315/p1465.html
Http://www.ncirs.edu.au/consumer-resources/mmr-decision-aid/symptoms-of-measles-mumps-and-rubella/

129
533.Mouth ulcer :
A. Hsv 1
B. Hsv2

Answer: a
** hsv type 1 is usually associated with orofacial disease, and hsv type 2 is usually associated with genital infection
Reference: http://www.clevelandclinicmeded.com/medicalpubs/diseasemanagement/dermatology/common-skin-infections/
534.Baby can sit w/o support , he can grasp says mama , wave bye bye to the dr. Which developmental millstone the baby
has defect in?! (peds)
A. Gross motor
B. Fine motor

Answer: by the scenario given the baby is probably 9mo .however the question has missing clues. What type of grasp? Does this
baby can only sit without support? Reference: nelson

535.# year child what is the normal development? (age??)


A. Say from 100- 150 words
B. Can tell a short story
Question is missing important part of the history!

536.7 days baby weight 3.02 was 3.5 mom is concern about her baby weight, although she feed him:
A. A-reassure the mom it's normal babies lose weight in the first couple of weeks
B. B-routine test and reassure
Answer: a

537.11 months old infant , dark foul smell stool mixed with blood and mucus , what will u do investigation :
A. Something like teqneium scan .
B. Ultrasound

Answer: (incomplete question and choices!)

130
Explanation: in infants with stool mixed with blood and mucus, main ddx include: bacterial diarrhea, presents with fever,
abdominal pain and bloody diarrhea, bacteria determined by stool culture. Intussusception, presents with paroxysmal
abdominal pain, vomiting, sausage shaped abdominal mass, and currant jelly stool, diagnosed by contrast enema. Meckel
diverticulum, mostly asymptomatic, but may present with bloody stool, intussusception, volvulus, or diverticulitis, diagnosed by
technetium scan.
th
Reference: illustrated textbook of paediatrics 4 edition, pages 225-227, 229
Http://emedicine.medscape.com/article/1955984-clinical

538.Most sexual differentiation in male:


A. Micropenis
B. Hypospadias

Answer: question and choices incomplete


Explanation: diseases of sexual differentiation (dsd) vary in frequency, depending on their etiology. In general, cah is the most
common cause of ambiguous genitalia in the newborn. Mixed gonadal dysgenesis (mgd) is the second most common cause of
dsds. In 46, xy babies there might be defect in testicular synthesis of androgens or androgen resistance in target tissues.
Hypospadias occurs at a rate of 1 case per 300 live male births; in fewer than 1% of patients, hypospadias occurs in combination
with undescended testes. Clinicians should suspect the possibility of a dsd in patients with both hypospadias and cryptorchidism.
References: http://emedicine.medscape.com/article/1015520-overview#a4
Toronto notes 2015, page u38

539.Mild stone said baba and walk holding furniture and alot of other features: (missing q)
A. 12m
B. 10m

540.Baby came to u with sore throat , u obtain culture and send the baby home , the culture grows mennigicoccus , u call the
parents and they say their baby is asymptomatic , what will u do :
A. 10 days oral ampicillin
B. One dose im ceftriaxone sorry i forgot the rest of choices

Answer: choices incomplete (answer most likely missing)


Explanation: chemoprophylaxis is not recommended for close contacts of patients with evidence of neisseria meningitidis only in
nonsterile sites (e.g., oropharyngeal). Furthermore, there is no indication to treat persons who are asymptomatic
nasopharyngeal carriers. However, if in the exam re-assurance is not among the choice, rifampin, ciprofloxacin, and ceftriaxone
90%-95% effective in reducing nasopharyngeal carriage of n. Meningitidis and are acceptable for chemoprophylaxis
Reference: http://www.cdc.gov/vaccines/pubs/pinkbook/mening.html

541.Milestone for baby who can said baba and walk holding furniture and many other features:
A. 12m
B. 10m

Answer: a
- See the above table for developmental milestones

542.Cyanotic heart disease?


A. Coarctation of the aorta
B. Truncus arteriosus

131
Answer: b
- Https://yhdp.vn/uptodate/contents/mobipreview.htm?32/44/33482

543.Infant + symptoms of meningitis,


What is the next?
A. Lp
B. Iv antibiotics
Answer: a?
Treatment with antibiotics is usually started immediately after the blood tests and lumbar puncture are performed.
- up-to-date

544.Milestone : child crawl, hand to hand but no pincer grip :


A. 7 m
B. 9 m
Answer : a

545.Child vaccine, his sister recently kidney transplant, which vaccine should he
Avoid ?
A. Opv
B. Measles
Answer : a
Administration of opv is associated with a low incidence of paralytic poliomyelitis in vaccines. Also, individuals in close contact
with recently inoculated vaccines may be at a small risk of developing paralytic poliomyelitis because poliovirus can be shed in
the feces (and possibly from the pharynx) for 6-8 weeks after opv administration.

546.Child with mild intermittent asthma, he is using short acting b agonist 1 time\ week. Nowadays, he is using it 4 times \
week without any benefit, what will you add :
A. Long acting b agonist
B. Short acting inhaled steroid
Answer: b
Asthma stepwise approach: https://goo.gl/ozwmhf

547.Neonate + white eye reflex. Posterior segment information not available ?


A. Congenital cataract
B. Congenital glucoma
Answer: a?
Congenital cataract: dim red reflex, leukocoria, decrease vision, nystagmus, squint, photohobia.
Congenital glaucoma: megalocornea, diffuse corneal edema, epiphora, high intracranial pressure.

132
548.Child with om take amoxicillin and follow up now with opd on examination there is fluid behind t.m?
A. Amoxicillin
B. Wait and watch
Forget other choices
Answer: on uoq similar q but for 30 yrs the answer is wait and watch

549.In cpr for infant you'll use:


A. Atropine
B. Epinephrine
Answer is: a or b (not sure)
Epinephrine is to be used for infants/children with bradycardia and poor perfusion that is unresponsive to ventilation
and oxygenation. While atropine is to be used in bradycardia caused by increased vagal tone or cholinergic drug
toxicity.
Depends on the cause, so either can be true.
Source: http://www.medscape.com/viewarticle/726741_3

550.Young child present with his family (a picture of baby, his age seems to be in months, his head is tilted to one side). Baby
was normal at birth with normal non-complicated delivery, when you try to stretch sternocleidomastoid the baby begins
crying. All reflexes and movements are normal. What is the diagnosis?

A. Cervical rib
B. Infant torticollis

Answer: b
Cervical rib is mostly asymptomatic in individuals who have it, and it is almost always an incidental finding. Symptoms
rise in case of trauma or injury leading to thoracic outlet syndrome.
The cause of congenital torticollis in infants is not known. Intrauterine and perinatal events that have been proposed to
contribute to its development include malposition of the head in utero and injury to the sternocleidomastoid muscle
(scm) that results in fibrosis.
Torticollis typically develops by two to four weeks of age, after which the child prefers to hold the head tilted to one
side.
Source: https://yhdp.vn/uptodate/contents/mobipreview.htm?42/49/43798

551.2 days old baby with urine smelling like burned sugar. What’s the diagnosis?
A. Maple syrup disease
B. Phenylketonuria
Answer: a
Maple syrup urine disease and phenylketonuria are both inborn errors in the metabolism of amino acids.
Phenylketonuria accumulation of phenylalanine in body fluids, leading to intellectual disability and “mousy odor”.
Maple syrup urine disease accumulation of leucine, isoleucine and valine and their corresponding keto-acids leading
to encephalopathy, progressive neurodegeneration, and “maple syrup or burned sugar smelling urine”.
Sources: http://emedicine.medscape.com/article/947781-overview
Http://health.mo.gov/living/families/genetics/newbornscreening/pdf/maplesyrupquickfacts.pdf

552.Child presented to er with hx of testicular pain, what is next step?


A. U/s
B. Surgery counseling

133
Testicular torsion is a clinical diagnosis. If the history and physical examination strongly suggest testicular torsion, the patient
should go directly to surgery without delaying to perform imaging studies.
When a low suspicion of testicular torsion exists, color doppler and power doppler ultrasonography can be used to demonstrate
arterial blood flow to the testicle while providing information about scrotal anatomy and other testicular disorders.
Http://emedicine.medscape.com/article/2036003-workup#c10

553.Baby 18 month old with delay speech other exam normal


A. Hearing
B. Development

554.Child presented with something in the eye with lab results of high wbc. What is the diagnosis?
A. Leukemia
B. Neuroblastoma

It could be leukemia with eye infiltration but the answer could be in the 2 missing choices.
In neuroblastoma, there could be anemia or thrombocytopenia but not leukocytosis.

555.Child brought to you with complain of yellwish occlusive area of the teeth what u will give :
A. Antiseptic mouth wash
B. Fluride supplement

556.Case of 2 months baby , his parent feeds him with bottle of milk while he sleeping , developed with plaque and
discoloration of his teeth:
A. Nursing bottle carries!?
B. Can't recall other answers

557.Baby was pale with petechial


A)chlormphenicil (aplastic anemia)
B) vancomycin
Answer: incomplete question

558.Child with earache and fever ?

A. Otitis media
B. Otitis externa
Answer: stem not very clear, if associated with fever then most likely aom , if severe pain with tragus palpation then most
probably oe - depends upon more details in
Link: http://emedicine.medscape.com/article/994656-overview
The key physical finding of oe is pain upon palpation of the tragus (anterior to ear canal) or application of traction to the pinna
(the hallmark of oe).

Link: http://emedicine.medscape.com/article/994550-overview

559.Child with arthritis, fever, epistaxis, gingival bleeding, results plt is low , hgb
Is low.. Appropriate investigation?
A. Bone marrow aspiration
B. Electrophoresis
C. Anti dsdna
Answer: most important is blood smear not mentioned in choices
Explnanation: http://bestpractice.bmj.com/best-practice/monograph/795/diagnosis/differential-diagnosis.html

134
Https://labtestsonline.org/understanding/conditions/dic/start/2

560.African teen with jaw mass histopathology showed starry sky appearance (case of burkitt's lymphoma) what's that
pathophysiology?
A. Overexpression of c-myc
B. Overexpression of n-myc
Answer: a (medscape)
All types of burkitt lymphoma are characterized by dysregulation of the c-myc gene by one of three chromosomal translocations
t(8:14), the most common. Or less commonly t(8:2) and t(8:22). Overproduction of the c-myc product may also change the
lymphocytes into cancer cells.
Link: http://emedicine.medscape.com/article/1447602-overview

561.(long scenario), child with pansystolic (holosystolic) murmur. What is the cause?
A. ASD
B. VSD
Answer: b (table 5 in link)
Link: http://www.aafp.org/afp/2011/1001/p793.html

562.Years old girl failed in math exam. Then she had palpitations, tachypnea and paresthesia. This is ? (age missing)
A. Hyperventilation syndrome
B. Conversion
Answer: a
Explanation: hyperventilation syndrome (hvs): is a condition in which minute ventilation exceeds metabolic demands, resulting
in hemodynamic and chemical changes that produce characteristic dysphoric symptoms.
Cause of hvs is unknown, but some persons who are affected appear to have an abnormal respiratory response to stress,
sodium, lactate, and other chemical and emotional triggers, which results in excess minute ventilation and hypocarbia.
Link: http://emedicine.medscape.com/article/807277-overview

563.Child come with watery diarrhea during the stool microscope you found a flagellated parasite. What is the mechanics of
the diarrhea:
A. A/ secretions a lot of fluid
B. B/ decrease the absorption of fluid
Answer: b
Explanation: marked or moderate partial villous atrophy in the duodenum and jejunum can be observed in histologic sections
from asymptomatic individuals who are infected. In addition to disrupting the mucosal epithelium, effects in the intestinal

135
[5, 19]
lumen may contribute to malabsorption and the production of diarrhea. nevertheless, diarrhea can occur in individuals in
the absence of obvious light microscopic changes in small intestinal structure.
Varying degrees of malabsorption of sugars (eg, xylose, disaccharides), fats, and fat-soluble vitamins (eg, vitamins a and e) may
contribute to substantial weight loss. The histopathologic response to giardiasis varies and imperfectly correlates with the
clinical symptoms

Link: http://emedicine.medscape.com/article/176718-overview#a3

564.Child present with abdominal mass and some urinary symptoms but i can not remember after ct the report say that there
is a large mass involved the renal collecting system what your next step :
A. 24 h collection of catecholamine
B. MRI

565.Safe vaccine you can give it to immunocompromised patient:


A. -BCG
B. -HBV
Answer: safe: pneumococcal, meningococcal, hib,
Contraindicated: all live vaccines. Oral polio, rota , mmr, smallpox , bcg , varicella , opv
Cdc: http://www.cdc.gov/vaccines/pubs/pinkbook/downloads/appendices/a/immuno-table.pdf

566.Child with vomiting presented dehydrated and a small mass was felt in the epigastric area, what is his acid-balance?
A. Hypochloremic metabolic alkalosis
B. Metabolic acidosis
Answer: a

The patient is having (pyloric stenosis), also known as infantile hypertrophic pyloric stenosis (ihps), is the most common cause of
intestinal obstruction in infancy causing functional gastric outlet obstruction. Hypochloremic hypokalemic metabolic alkalosis is
the classic electrolyte and acid-base imbalance of pyloric stenosis.
- Classically, the infant present with non-bilious vomiting or regurgitation, which may become projectile.
- The emesis may become brown or coffee color due to blood secondary to gastritis or a mallory-weiss tear at the gastro-
esophageal junction.
- Signs of dehydration and malnutrition might be seen, such as poor weight gain, weight loss, marasmus, decreased
urinary output, lethargy, and shock.
- May develop jaundice, which is corrected upon correction of the disease.

567.18 months’ baby says baba mama what you will do for him
A. Developmental assessment
B. Bone age
Answer: a
Baby at 18 months should say several single words. Saying mama baba should be at 12 months.
Reference: http://www.cdc.gov/ncbddd/actearly/milestones/index.html

568.Cretinism.
A. Iodine supplementations
B. Thyroxine
Answer: a
Congenital hypothyroidism (cretinism) is inadequate thyroid hormone production in newborn infants. It can occur because of an
anatomic defect in the gland, an inborn error of thyroid metabolism, or iodine deficiency. The mainstay in the treatment of
congenital hypothyroidism is early diagnosis and thyroid hormone replacement.
Reference: http://emedicine.medscape.com/article/919758-overview

136
569.Pediatric patient is complaining of red eye and fever, later he develops pink rash on the face which spreads to upper and
lower limbs, there is also white papule in the mouth, what is the diagnosis?
A. Rubella
B. Meningococcal rash
Answer: a
The typical course of rubella exanthem (rash) that starts initially on the face and neck and spreads centrifugally to the trunk and
extremities within 24 hours. It then begins to fade on the face on the second day and disappears throughout the body by the
end of the third day.
Reference: http://emedicine.medscape.com/article/968523-clinical#b5

570.Child presented to clinic, very tall and very thin “ not the exact wording of the question” .exam showed cardiac
abnormality and pectus excavatum, don’t remember other details. Dx?
A. Marfan
B. Charg syndrome
Answer: a

571.Child come with sever symptom of croup the doctor give epinephrine then relive but after time again symptom?
What you will do?
A. Repated epinephrine
B. Steroid
Answer: b
The benefits of corticosteroids and nebulized epinephrine for moderate to severe croup have been demonstrated in meta-
analysis and randomized controlled trials, respectively, specific pharmacologic intervention depends upon the severity of
symptoms:
o For children with moderate stridor at rest and moderate retractions or more severe symptoms, we recommend
administration of dexamethasone (0.6 mg/kg, maximum of 10 mg) by the least invasive route possible: oral if oral
intake is tolerated, intravenous if iv access has been established. A single dose of nebulized budesonide is another
option, particularly for children who are vomiting.
o For children with moderate stridor at rest and moderate retractions, or more severe symptoms, we recommend nebulized
epinephrine in addition to dexamethasone.

572.7-year girl developed pubic hair and axillary hair and complain from acne and breast develop not enlargement of clitoris
what dx?
A. Central cause of puberty (something same this)
B. Gonadotropin realizing tumor
Answer: no clear options!
Etiology of precocious puberty:
• usually idiopathic in females (90%), more suggestive of pathology in males (50%)
• central (gnrh dependent)
Hypergonadotropic hypergonadism; hormone levels as in normal puberty
Premature activation of the hpg axis
Differential diagnosis: idiopathic or constitutional (most common in females), cns
Disturbances (tumors, hamartomas, post-meningitis, increased icp, radiotherapy), nf,
Primary severe hypothyroidism
• peripheral (gnrh independent)
Hypogonadotropic hypergonadism
Differential diagnosis: adrenal disorders (cah, adrenal neoplasm), testicular/ovarian tumor, gonadotropin/hcg secreting
tumor (hepatoblastoma, intracranial teratoma, germinoma) exogenous steroid administration, mccune-albright
syndrome, aromatase excess syndrome, rarely hypothyroidism (van wyk-grumbach syndrome).

573.16 y/o female presented with history of irregular period, that came every 5 weeks then, 6 weeks?

137
A. Normal changes or physiological changes
B. Uterian fibroids
Answer: a
Long cycles most often occur in the first 3 years postmenarche, the overall trend is toward shorter and more regular cycles with
increasing age. By the third year after menarche, 60% to 80% of menstrual cycles are 21 to 34 days long, as is typical of adults.
An individual's normal cycle length is established around the sixth gynecologic year, at a chronologic age of approximately 19 or
20 years.
Http://pediatrics.aappublications.org/content/118/5/2245

574.Child say few words what age


A. 12m
B. 24m
Answer: b
18 months:
Says several single words.
2 years:
Says sentences with 2 to 4 words.
Link: http://www.cdc.gov/ncbddd/actearly/milestones/

575.Child with sever rheumatic fever &cardiac involvement what to give for short period?
A. Im penicillin monthly
B. Large does aspirin + ... Orally steroid
Answer: a
The primary goal of treating an arf attack is to eradicate streptococcal organisms and bacterial antigens from the pharyngeal
region. Penicillin is the drug of choice in persons who are not at risk of allergic reaction. A single parenteral injection of
benzathine benzylpenicillin can ensure compliance. Oral cephalosporins, rather than erythromycin, are recommended as an
alternative in patients who are allergic to penicillin.
Corticosteroids should be reserved for the treatment of severe carditis. After 2-3 weeks, the dosage may be tapered, reduced by
25% each week.
Link: http://emedicine.medscape.com/article/333103-treatment

576.2 months baby brought to hospital parents complain that the baby is spitting what you will do?
A. Reassurance
B. CT abdomen
Answer: a
Spitting up, sometimes called physiological or uncomplicated reflux, is common in babies and is usually (but not always) normal.
Babies often spit up when they get too much milk too fast. This may happen when baby feeds very quickly or aggressively, or
when mom’s breasts are overfull.
Link: http://kellymom.com/hot-topics/reflux/

577.3 years old came to the er with dry cough, he was crying and hoarseness of voice was present, what is your
management?
A. Oxygen mask
B. Nebulized cool mist
Answer: refer to croup management

578.Child presented to the emergency room with non productive cough and inspiratory stridor. What is the worst sign we
should worry about?
A. Bluish lip color
B. Expiratory stridor
Answer: a

138
579.4 days baby present with bilious vomiting, he had abdominal distention, poorly feed on examination abdominal
distention (no other findings in examination in the q)
A. Mid volvulus
B. Allergic to formula
Incomplete mcq

580.Child with recurrent URTI with pseudomonas, and atypical organism. Whats the cause:
A. Cystic fibrosis
B. Low CD4
Answer: a

Typical respiratory manifestations of cf include a persistent, productive cough, hyperinflation of the lung fields on chest
radiograph, and pulmonary function tests that are consistent with obstructive airway disease. The onset of clinical symptoms
varies widely, due to differences in cftr genotype and other individual factors, but pulmonary function abnormalities often are
detectable even in the absence of symptoms. As an example, in a cohort of infants largely identified by newborn screening, 35
percent had respiratory symptoms (cough,
Wheezing, or any breathing difficulty); mean pulmonary function scores were abnormal by six weeks of age and declined during
the subsequent two years.
Transient infection of the airway with pathogenic bacteria often occurs early in life. Eventually, over years and varying widely
among individuals, chronic airway infection with either staphylococcus aureus or gram negative bacteria is established, often
with radiographic evidence of bronchiectasis. S. Aureus and nontypeable haemophilus influenzae are common pathogens during
early childhood, but pseudomonas aeruginosa is ultimately isolated from the respiratory secretions of most patients.

581.Baby present with his family (pic of baby his age is months head tilted to one side) he was normal on birth with normal
not complicated delivery, when you try stretch sternocleidomastoid he was crying all reflexes and movement normal?
A. Cervical rib
B. Infant torticollis
Answer: b

Torticollis develops within the first few days or weeks of life may result from hematoma, fibrosis, and contracture of the
sternocleidomastoid (scm) muscle. A non-tender mass may be noted in the scm, usually in the midsegment. Torticollis is a
frequent cause of plagiocephaly (flattening of one side of the head) and asymmetric facies – merck manual
https://www.merckmanuals.com/professional/pediatrics/congenital-craniofacial-and-musculoskeletal-
abnormalities/congenital-neck-and-back-abnormalities

582.What you'll see on physical examination of patient with croup?


A. Presence of inspiratory sounds
B. Presence of expiratory wheeze
Answer: a
139
Merck manual - https://www.merckmanuals.com/professional/pediatrics/respiratory-disorders-in-young-children/croup

583.Infantile colic:
A. Decreased peristalsis
B. Increased gases
Answer: b
The cause of infantile colic remains unclear. Gastrointestinal disorders have been implicated in colic because of the infant’s leg
position and grimacing during a crying spell. Excessive crying or increased gas production from colon function can result in
intraluminal gas formation.
Http://www.aafp.org/afp/2004/0815/p735.html

584.Child with mild trauma develop hemarthrosis, in past history of similar episode dx?
A. Platelets dysfunction
B. Clotting factor deficiency
Answer : b hemophilia

585.CPR in child according to American heart association in presence of 02 rescuer:


A. 15 compression and 2 ventilation
B. 30 compression and 2 ventilation
Answer: a

Two breaths can be given after every 30 chest compressions. If someone else is helping you, you should give 15 compressions,
then 2 breaths.
Http://www.webmd.com/first-aid/cardiopulmonary-resuscitation-cpr-for-children?Page=2

586.Baby with decreased air in the lung, abdomen looks scaphoid, what is the diagnosis?
A. Diaphragmatic hernia.
B. Dextocardia.
Answer a

587.Patient with barking cough, red epiglottis is caused by:


A. Pertussis
B. Hib
Answer:
Barking cough and red epiglottis comes with croup and the most common cause is parainfluenza type 1,2. In epiglottitis it
appears as cherry-red swollen and the cough is whooping, the most common organism is hib

588.The main tx for Kawasaki :


A. IVIG
B. Aspirin
Answer: A
Cost-benefit analysis reveals that IVIG treatment of KD is one of the most cost-effective medical therapies available, leading to
tremendous short- and long-term savings
UPTODATE

589.A child who ate honey develops progressive paralysis symptoms?


A. C. Botulism
B. C. Perfringens
Answer: A

140
Honey can contain the bacteria that causes infant botulism so, children less than 12 months old should not be fed
honey. Honey is safe for persons 1 year of age and older.

Reference: http://www.cdc.gov/nczved/divisions/dfbmd/diseases/botulism/

590.Patient with barking cough and 38 C temp which of the following symptoms is associated with this disease?
A. Cyanosis
B. Wheezing

Answer: A
Clinical Features of Croup:
Age: 4 mo-5 yr

Preceded by URTI symptoms

Generally occurs at night
 with improving in the morning
Biphasic stridor and croupy cough (bark, spasmodic, and hoarseness)
Appear less toxic than epiglottitis

Supraglottic area normal

Rule out foreign body and subglottic stenosis

“Steeple-sign” on AP x-ray of neck

If recurrent croup, think subglottic stenosis
In severe cases, cyanosis.
Auscultation reveals prolonged inspiration and stridor. Rales also may be present, indicating lower airway involvement.
Breath sounds may be diminished with atelectasis.
Low grade Fever (38-39°C)
References: Toronto note, Medscape, Merck manual

591.Child take overdose of isoniazid and toxicity symptoms?


Answer: seizure include status epilepticus
Http://www.uptodate.com/contents/isoniazid-inh-poisoning?Source=outline_link&view=text&anchor=h25#h25

592.What is the 24h maintenance dose for a 25kg child?


Answer: 1.6 L
To Calculate the daily maintenance use the following formula:
(100 ml for each of the first 10kg) + ( 50ml for each kg 11-20) + (20 ml for each additional kg)
Reference: http://www.sickkids.ca/Nursing/Education-and-learning/Nursing-Student-Orientation/module-two-clinical-
care/paediatric-iv-therapy/Calculating-Maintenance-Fluid-Rates/index.html

593.The fluid deficit is the percentage of dehydration multiplied by the patient’s weight
Or 50 ml/kg, 100ml/kg, 150ml/kg

594.Maintenance: (100-50-25) ml/kg/24h rule

595.A child with sickle cell anemia and bilateral hip pain. What is the possible diagnosis?
A. Avascular necrosis of femoral head

Answer: A

596.Intranasal influenza vaccine ----> the name of the vaccine


141
Answer: Live attenuated influenza vaccine (LAIV)
Live, attenuated (temperature-sensitive mutant)vaccine → trade name =flue Mist

597.Patient with thumb sign on X-ray:


A. Ceftriaxone and clindamycin
Answer: A
Thumb sign on X-ray
The thumb sign is a manifestation of an oedematous and enlarged epiglottis which is seen on lateral soft-tissue radiograph of
the neck, and it suggests a diagnosis of acute infectious epiglottitis. This is the radiographic corollary of the omega sign.
Http://radiopaedia.org/articles/thumb-sign-of-epiglottitis

598.Mother brings her first & only infant was fatigued, not move or gaze from light direct on his eye what the Dx?
A. Infantile botulism

599.Baby milestones: can hold his head and when he looks at his flying hand he laughs and coos
-If he asked about the age it would be 2 months ((Toronto notes,2015,p5 \ first aid USMLE step2,ED8,p361……..))

Case of marasmus
Answer:

600.Child prested with cyanosis and murmur ( case of the transposition of greet vessel )

601.TOF present with


A. VSD
This not a clear Q but there are hints to answer that Q Initial abnormal finding is pulmonary stenosis murmur.
Cyanosis is the most common symptoms. Tet (hypoxic) spill is distinctive and important feature of TOF.

602.Child 9 y ..Rx of sever avascular necrosis?

603.Pedia case neonat has jundice admitted due to physiolgical jundice then disharge , still appear jundice what is case ?
A. Deudenal atresia
604.(Prolong physiological jaundice associated with hypothyroidsm and GIT obstruction )

605.The most common parotid tumor in pediatrics?


Answer: ?
Pleomorphic adenoma then hemangioma.
Reference: http://www.ncbi.nlm.nih.gov/pubmed/10680869
*mucoepidermoid if they asked about the most common malignant tumor.

606.Which of the following is the most common heart abnormality to get infective endocarditis?
a. Tetralogy of fallot

Answer: a
In children, cyanotic heart disease is still the most common cause of endocarditis, and the risk does not diminish after surgical
repair as prostheses carry their own risk. Reference: http://www.ncbi.nlm.nih.gov/books/nbk2208/

142
Bacterial endocarditis can occur with many heart defects but is most common in aortic valve lesions, a patent ductus arteriosus
(unrepaired), tetralogy of fallot, ventricular septal defects, coarctation of the aorta, and mitral valve prolapse with mitral
regurgitation. Reference: bacterial endocarditis by ernest g. Brookfield, m.d.

607.Baby with greasy looking rash on face.


a. Seborrheic dermatitis

Answer: a

608.What is the most common site for mump?


a. Parotid

Answer: a
Reference: http://reference.medscape.com/article/966678-clinical

609.Typical case of poststreptococcal glomerulonephritis (psgn). What will you do to confirm diagnosis?
Answer: ?
Occurs 1-3 wk following initial primary gas infection of pharynx or skin.
Diagnosis is confirmed with elevated serum antibody titers against streptococcal antigens (asot, anti-dnaseb), low serum
complement (c3) reference: toronto notes
If it glomerular involvement occurred in less than a week after urti and c3 is normal, then it’s due to iga nephropathy

610.The most common chromosomal abnormality in a new infant:


a. Down syndrome (trisomy 21)

Answer: a
Most common abnormality of autosomal chromosomes. Reference: toronto notes.

611.Child was having rhinorrhea and then developed episodes of cough followed by vomiting. Which of these vaccination
may prevent him from having this disease?
a. Dtap
Answer: a
612.(a case of pertussis) how to diagnose of pertussis?
A. Nasopharyngeal swab

Answer: a

613.Tetralogy of fallot findings:


a. Ventricular septal defect (vsd) + overriding of the aorta + pulmonary stenosis + right ventricular hypertrophy (rvh)

Answer: a
Mnemonic: prove. Ps, rvh, overriding of the aorta, vsd.

614.A case of tof. How does it appear on x-ray and echo?


Answer:?
➢ Ecg: right axis deviation, rvh
➢ Cxr: boot shaped heart, decreased pulmonary vasculature, right aortic arch (in 20%)
143
Reference: toronto notes

615.Turner features:
a. Thick skin neck

Answer: a
Short stature, low set mildly deformed ear, triangular face, flat nasal bridge, epicanthal fold, web neck with or without cystic
hygroma, shield like chest with wide internipple space, puffiness of hands & feet, internal malformation mainly coarctation of
aorta in heart, horseshoe kidney. Reference: nelson.

616.A child was on clindamycin developed abdominal pain and watery diarrhea.
a. Clostridium difficile

Answer: a
Pseudomembranous colitis

617.Child came with rt abdominal pain, jaundice, palpable tender liver, dx?
a. Hav
Answer: a

618.Neonate born in home by a midwife presented with umbilical bleeding. What does he need?
a. Vit k injection

Answer: a

619.Turner syndrome chromosomes?


a. 45x
Answer: a

620.Child with fever, malaise, lns enlargement & mouth ulcers. What is the diagnosis?
a. Herpes simplex virus infection type 1 (hsv 1)

Answer: a
Reference: toronto notes.

621.10 years old girl presented with fatigability, diarrhea and glossitis. What is the diagnosis?
a. Vitamin b12 deficiency

Answer: a
th
Reference: textbook of oral medicine. Nelson 7 p:512

144
622.6 years old boy presented with gingivitis, petechiae and rash. What is the diagnosis?
a. Vitamin c deficiency

Answer: a
th
Reference: nelson 7 page:95

145
623.15 years old has dm with dehydration?
a. Dka

Answer: a
th
Reference: nelson 7 page:574

624.A boy with rickets (picture was provided). What is the deficient vitamin?
a. Vit d

Answer: a

625.Which of the following diseases has mendelian mode of inheritance?


a. Alpha thalassemia

Answer: a
Autosomal recessive

146
626.Child presented with bronchiolitis. What is your management?
a. Give oxygen
Answer: a
Treatment of bronchiolitis is supportive (kaplan)

627.Mode of inheritance of neurofibromatosis.


Answer:?
Autosomal dominant.
th
Reference: nelson 7 edition ch186p:645

628.Infant suffer from groin rash that spare folds?


Answer:?
Irritant diaper dermatitis: shiny, red macules/patches, no flexural involvement.
Differential dx for groin rashes are: psoriasis, seborrheic dermatitis and langerhans cell histocytosis.
th
Reference: toronto notes. Nelson 7 p: 657

629.7 years old boy developed flu after receiving flu vaccine. His father asked you about the reason. How will you reply?
a. Live attenuated vaccine has small risk of infection

Answer: a
Reference: http://www.cdc.gov/vaccines/vac-gen/side-effects.htm#dtap

630.Boy presented with unilateral nasal obstruction and foul smelling. What is the diagnosis?
a. Foreign body

Answer: a, age is very important. Infant+ unilateral nasal obstruction + foul smelling = foreign body. Progressive nasal
obstruction concerning for growing mass. Anosomia + sinusitis= nasal polyps
Reference: step up pediatric p:525

631.What is the inheritance mode of fanconi anaemia?


Answer:
Autosomal recessive.
th
Reference: nelson 7 p: 517

632.Patient diagnosed with congenital adrenal hyperplasia. What is the next step?
Answer:
Iv hydrocortisone
Initial management is hydration , long term is oral steroids. Management in children:- 1- glucocorticoid is nenecessary in
children who have classic 21-hydroxylase deficiency and in symptomatic children with non-classic 21-hydroxylase deficiency. The
goal of the tx is to replace deficient steroids while minimizing adrenal sex hormone and iatrogenic glucocorticoid excess. 2-
mineralcorticoid and nacl: mineralocorticoid replacement is recommended in all pts who have the classic form of cah, whether
or not it is the salt-losing form. E.g fludrocortisone. 3- monitoring therapy: the response to the tx should be evaluated monthly
st
in the 1 3 months then every 3 months in infant and every 6 months in thereafter. Response to therapy is monitoring by
measuring serum 17-hydroxyprogesterone, androstenedione, plasma rennin activity and growth velocity.
Reference: uptodate under the topic : "treatment of classic cah due to 21-hydroxylase deficiency in infant and children"

147
633.In atrial septal defect (asd) there will be:
A. Fixed s2 split
Answer: a
The pathophysiology and amount of shunting depend on the size of the defect and the relative compliance of the both
ventricles. Even with large asds and significant shunts, infants and children are rarely symptomatic. A prominent right ventricular
impulse at the left lower sternal border (llsb) often can be palpated. A soft (grade i or ii) systolic ejection murmur in the region
of the right ventricular outflow tract and a fixed split s2 (due to overload of the right ventricle with prolonged ejection into the
pulmonary circuit) are often audible. A larger shunt may result in a mid-diastolic murmur at the llsb as a result of the increased
volume passing across the tricuspid valve.
th
Reference: master the boards and nelson 7 p:492

634.Child with thumb sign on lateral xray. What is the diagnosis?


a. Epiglottitis
Answer: a
Reference: step up pediatrics p: 512

635.Bilateral parotid swelling.


Answer: ?
Http://www.ncbi.nlm.nih.gov/pmc/articles/pmc3528147/

Local disease Systemic disease

148
● Mumps - more commonly children than ● Sarcoidosis
adults ● Tuberculosis
● Parotitis ● Alcoholism
● Uveoparotid fever ● Myxoedema
● Sialectasis - especially if related to eating ● Cushing's disease
● Sjogren's syndrome ● Diabetes/insulin resistance - about 25% of patients with
● Tumour infiltration overt or latent diabetes have bilateral asymptomatic
enlargement of the parotid glands
● Liver cirrhosis
● Gout
● Bulimia nervosa
● Hiv in children may cause bilateral parotid enlargement

636.Child presented with recurrent nasal congestion, rhinorrhea, sneezing, tearing eyes,..what is the diagnosis?
a. Allergic rhinitis

Answer: a the hallmarks of allergic rhinitis are clear: thin rhinorrhea; nasal congestion; paroxysms of sneezing; and
pruritus of the eyes, nose, ears, and palate. Postnasal drip may result in frequent attempts to clear the throat, nocturnal cough,
and hoarseness. It is important to correlate the onset, duration, and severity of symptoms with seasonal or perennial exposures,
changes in the home or school environment, and exposure to nonspecific irritants, such as tobacco smoke. Reference: nelson
th
7 p: 283

637.What is the causative organism of croup?


a. Parainfluenza
Answer: a

Reference: step up pediatrics p:512

638.Rheumatic fever prophylaxis?


a. Im penicillin monthly
Answer: a
An injection of 0.6-1.2 million units of benzathine penicillin g intramuscularly every 4 weeks is the recommended regimen for
secondary prophylaxis for most us patients. Administer the same dosage every 3 weeks in areas where rheumatic fever is
endemic, in patients with residual carditis, and in high-risk patients.
Management of acute rheumatic fever consists of benzathine penicillin to eradicate the beta-hemolytic streptococcus, anti-
inflammatory therapy with salicylates, and bed rest. Additional supportive therapy for heart failure or chorea may be necessary.
Long-term penicillin prophylaxis, preferably with intramuscular benzathine penicillin g, 1.2 million u every 28 days, is required.
Oral regimens for prophylaxis generally are not as effective. The prognosis of acute rheumatic fever depends on the degree of
permanent cardiac damage.
th
Reference:http://emedicine.medscape.com/article/891897-treatment#showall reference: nelson 7 p: 502

149
639.What can increase fetal hemoglobin in sickle cell anemia (no hydroxyurea in the options)
a. Folic acid

Answer: ?
● The hbf inducers: - can be grouped in several classes based on their chemical structures and mechanisms of action
including
Hypomethylating agents (eg; 5-azacytidine and decitibine)
Short chain fatty acids: histone deacetylase inhibitors (eg; sodium butyrate)
Chemotherapeutic agents (eg; hydroxyurea)
Stem cell factor and erythropoietin
Azacytidine, butyric acid, erythropoietin and hydroxyurea

Reference: advances in pediatric

640.10 years old child got strep throat infection followed by rheumatic fever. He was treated early without any
consequences. For how long he'll continue the rheumatic fever prophylaxis ?
a. 15 years

Answer: ?
8 - 11 years (requires antibiotic treatment for 5 years or until the patient is aged 18-21 years (whichever is longer))
Refer to q58

641.(long scenario) baby with cavernous hemangioma and have pleural effusion. (he have other findings). What you will find
in this baby?
A. Pulmonary hemangioma.

Answer: ?

642.A child is always using abnormal sitting habits (w-setting), what is the effect on the bones?
Answer: ?
Internal femoral torsion (femoral anteversion), it's the most common cause of intoeing > 2 years of age.
Management: observation, takes 1-3 years to resolve. Surgery only if significant at > 10 years of age.

150
th
Reference: pediatrics kaplan for step 2 and nelson 7 p: 676.

643.What is the gene responsible for neurofibromatosis?


Answer: ?
➢ The nf1 (peripheral neurofibromatosis or von recklinghausen disease) gene is cytogenetically located on the long (q)
arm of chromosome 17, band q11.2 (17q11.2). The nf1 gene encodes for a cytoplasmic protein called neurofibromin
1, which is a ras -gtpase–activating protein that suppresses tumor growth, primarily by inhibiting ras activity.
➢ The gene for nf2 (bilateral acoustic neurofibromatosis or central neurofibromatosis) is cytogenetically located on the
long (q) arm of chromosome 22, band q12.2 (22q12.2). The nf2 gene codes for the protein neurofibromin 2, also
called merlin or schwannomin. Reference: medscape.

644.A case of ambiguous genitalia. Which hormone you would like to check?
Answer:
17-hydroxyprogesterone (hormone) should be measured promptly in all infants with nonpalpable gonads presenting with
genital ambiguity to exclude congenital adrenal hyperplasia (cah) due to 21-hydroxylase deficiency. This is the most common
cause of genital ambiguity and can lead to life-threatening adrenal insufficiency within the first weeks of life. Reference:
uptodate

645.A child presented with croup. What to give in er?


a. Epinephrine

Answer: a
Refer to the table at the end of pediatrics section

646.Epiglottitis case.
a. Intubate

Answer: a
Refer to the table at the end of pediatrics section
➢ Comparison between croup, tracheitis and epiglottitis:

151
Reference: toronto notes

647.Roll from prone to supine and vise versa + crab with only two fingers. What is the milestone.
Answer:?
A. 4 - 6 months
Refer to the table at the end of pediatrics section

648.Pediatric blunt trauma with duodenal coiled spring sign. What will you do?
Answer:
Duodenal hematoma:
In the pediatric age group, duodenal injury from a blunt abdominal trauma resulting in an intramural hematoma is rare. This
case illustrates the characteristic delayed presentation of a duodenal hematoma following a blunt injury from a motor vehicle
accident and the associated sequelae of delayed pancreatitis. Total parenteral nutrition and gastric decompression provide an
effective conservative treatment of the gastric outlet obstruction associated with this injury. Surgical intervention is only
reserved for those patients who continue to show the clinical and radiological signs of complete high obstruction despite
conservative management for three weeks.
For more details, see: http://www.ncbi.nlm.nih.gov/pmc/articles/pmc3088392/

649.Child sucking his thumb what to do?


Answer:
Reward we he doesn't do it

See:http://www.ncbi.nlm.nih.gov/pubmed/2676311

152
650.A child presented with recurrent nasal congestion, rhinorrhea, sneezing and tearing eyes, what is the most likely
diagnosis?
a. Allergic rhinitis

Answer: a
The hallmarks of allergic rhinitis are clear: thin rhinorrhea; nasal congestion; paroxysms of sneezing; and pruritus of the eyes,
nose, ears, and palate. Postnasal drip may result in frequent attempts to clear the throat, nocturnal cough, and hoarseness. It is
important to correlate the onset, duration, and severity of symptoms with seasonal or perennial exposures, changes in the
home or school environment, and exposure to nonspecific irritants, such as tobacco smoke.
th
Reference: nelson 7 p: 283

651.Child with bad smell and tooth is good :::: tonsillitis with crept
Answer:
Tonsils —the tonsils may be involved in the pathogenesis of bad breath in a small percentage of cases (perhaps 3 percent) [10].
Tonsillectomy based solely upon a complaint of bad breath should be avoided [33].
Some patients complain of small stones on their tongue or tonsils when they cough that have a foul odor (and often lead
patients to assume that they must have terrible breath). These stones are "tonsilloliths" that form in crypts of the tonsils.
Ref : uptodate

652."2qs about developmental milestone" 2 times


Baby can smile at which age : 2monthe ,4 or 6
Answer:
Social smile →2 months

653.Child w fever, coryza, diarrhea:


Answer: adenovrius
Typical symptoms and signs among children with rotavirus infection include vomiting, nonbloody diarrhea, and fever .
Ref: uptodate
Fever, rhinorrhea, cough, and sore throat, usually lasting 3-5 days, are typical symptoms of adenoviral ard medecape

654.Scenario a child didn't take hbv vaccine what u will give him
Answer: if the vaccine is not given after birth the baby may start the course of hepatitis b vaccines (in combination with other
childhood vaccines) beginning at six weeks, then at four and six months of age.

655.2qs about coarctation of aorta (straight forward)


Coarctation of aorta: radio femoral delay
Baby with deference in the bp in upper and lower extremities
Answer:
A clinical diagnosis of coarctation of aorta is made if there is an absent or delayed femoral pulse (when compared with the
brachial pulse). If there’s a murmur may be associated with other cardiac defects, such as pda, aortic stenosis, or ventricular
septal defect (vsd).
Http://www.uptodate.com/contents/clinical-manifestations-and-diagnosis-of-coarctation-of-the-
aorta?Source=search_result&search=coarctation+of+aorta&selectedtitle=1%7e126

153
656.Formula milk comparing to breast milk contain more ... What
Answer:
It’s contain more protein 1.5-1.9 g , carbohydrate 7-8.6 g , sodium 0.65-1.1 mmol , calcium 0.88-2.1 mmol , phosphorus 0.9-1.8
mmol and iron 8-12.5 umol
Ref: illustrated textbook of paediaterics page 206 - table 12.2

657.Treatment of choice for kawasaki


Answer: guidelines by the american heart association (aha) and the american academy of pediatrics (aap) are available for the
treatment of patients who fulfill the diagnostic criteria for kawasaki disease (kd) and for those who do not
The recommended initial therapy includes intravenous immune globulin (ivig; 2 g/kg) administered as a single infusion over 8 to
12 hours and aspirin (initial dose of 30 to 50 mg/kg daily divided into four doses).
Ivig : to reduce risk of coronary artery aneurysm
Aspirin : reduce risk of thrombosis

At risk of coronary arteries aneurysm within the first week of illness in about one third of affected children
Http://www.uptodate.com/contents/kawasaki-disease-initial-treatment-and-
prognosis?Source=search_result&search=kawasaki&selectedtitle=2%7e150

658.Child with jaundice, elevated direct bilirubin


Answer:
1-torontto note p72 (check the picture below)

2- http://www.pathophys.org/neonatal-hyperbilirubinemia/

154
659.Case of gonorrhea eye infection in a newborn.. What to give
Answer: infants with gonococcal ophthalmic disease should be hospitalized and observed for response to therapy and for
disseminated disease. Presumptive treatment should be started after obtaining cultures in infants with organisms seen on gram
stain or in those with negative gram stain, but who are considered to be at high risk (eg, mother with no prenatal care, history of
stds, or substance abuse).

Treatment consists of a single dose of ceftriaxone (25 to 50 mg/kg, not to exceed 125 mg, intravenously or intramuscularly) . A
single dose of cefotaxime (100 mg/kg, intravenously or intramuscularly) is an alternative option and is preferred for neonates
with hyperbilirubinemia and those receiving calcium-containing intravenous (iv) solutions (eg, parenteral nutrition) .

Topical antibiotic therapy alone is inadequate and is not necessary when systemic treatment is provided. The eyes should be
irrigated frequently with saline until the discharge clears.

Asymptomatic infants of untreated mothers — asymptomatic infants whose mothers have untreated gonococcal infection are at
high risk for acquiring infection. These infants also should receive systemic treatment with a single dose of ceftriaxone (25 to 50
mg/kg, up to a total dose of 125 mg, administered intravenously or intramuscularly) or cefotaxime (100 mg/kg, administered
intravenously or intramuscularly) and should be evaluated for chlamydial infection .

Prevention — the most effective measure to prevent both gonococcal and chlamydial infections is to diagnosis and treat these
infections in pregnant women. In addition, prophylactic antibiotic eye therapy reduces the risk of gonococcal conjunctivitis;
however, it is not effective in preventing c. Trachomatis conjunctivitis.

Neonatal prophylaxis —. Prophylaxis can be administered up to one hour after birth to facilitate infant-family attachment. The
following are regimens recommended by the american academy of pediatrics.

●erythromycin (0.5 percent) ophthalmic ointment


●tetracycline (1 percent) ophthalmic ointment; tetracycline ophthalmic ointment is not available in the united states

Http://www.uptodate.com/contents/gonococcal-infection-in-the-newborn

660.A child who came for 6 mo vaccination appt, his family report he had an anaphylaxis shock at 4 months vaccination: what
vaccine to give and what not to give? Or do an allergy test first? Or reassure and give them all?
Allergy test
Answer: http://www.uptodate.com/contents/allergic-reactions-to-vaccines
Http://vaccine-safety-training.org/vaccine-reactions.html

661.Mother came to you that her child compline of spells for seconds
Answer: absence (petit mal): usually only seen in children, unresponsive for 5-10 s with arrest of activity, staring, blinking or eye-
rolling, no post-ictal confusion; 3 hz spike and slow wave activity on eeg

Seizure mimics
• benign paroxysmal vertigo
• breath holding
• hypoglycemia
• narcolepsy
• night terror
• pseudoseizure
• syncope
• tia
• tic
Reference: toronto note
155
662.Which of the following influenza vaccines is given intranasally
Answer: http://www.cdc.gov/flu/about/qa/nasalspray.htm

663.Croup (parainfluenza)
Answer: look for a child 1 to 2 years of age. Croup usually occurs in the fall or winter. Fifty percent to 75% of cases are caused by
infection with parainfluenza virus; the other common causative agent is influenza virus. The disease begins with symptoms of
viral upper respiratory infection (e.g.rhinorrhea, cough, fever). Roughly 1 to 2 days later patients develop a “barking” cough,
hoarseness, and inspiratory stridor. The “steeple sign” (describes subglottic narrowing of the trachea; is classic on a frontal
radiograph of the chest or neck.
Treatment:
N mild cases: outpatient management with cool mist therapy and fluids.
N moderate cases: may require supplemental o2, oral or im corticosteroids,
And nebulized racemic epinephrine.
N severe cases (eg, respiratory distress at rest, inspiratory stridor): hospitalize
And give nebulized racemic epinephrine
Reference: usmle secrete- first aid

664.Young patient swallowed pins, what are you going to do. ( serial xrays)
Answer: the peak incidence of foreign body ingestion is between the ages of 6 months and 3 years.1 2 most ingested foreign
bodies pass through the gastrointestinal tract without difficulty, especially once they have reached the stomach.
Children with foreign body ingestion typically do not require laboratory testing.
Laboratory studies may be indicated for workup of specific complications, such as potential infection
Chest/abdominal radiography
• Most foreign bodies ingested by children are radiopaque (in contrast to inhalation, in which most are radiolucent).
Http://adc.bmj.com/content/84/2/165.full
Http://emedicine.medscape.com/article/801821-workup

665.Apgar score:
Answer:
Rapid scoring system that helps evaluate the need for neonatal resuscitation.
Each of 5 parameters:appearance (blue/pale, pink trunk, all pink) ,pulse (0, < 100, > 100),grimace with stimulation (0,
grimace,grimace and cough), activity (limp, some, active), respiratory effort (0, irregular, regular) is assigned a score of 0–2
At 1 and 5 minutes after birth.
N scores of 8–10: typically reflect good cardiopulmonary adaptation.
N scores of 4–7: indicate the possible need for resuscitation. Infants should
Be observed, stimulated, and possibly given ventilatory support.
N scores of 0–3: indicate the need for immediate resuscitation.
Reference: first aid

666.Pneumococcal conjugate 13 what type of vaccination ??


Answer:

156
Pneumococcal conjugate vaccine (called pcv13) protects against 13 types of pneumococcal bacteria.
Pcv13 is routinely given to children at 2, 4, 6, and 12–15 months of age. It is also recommended for children and adults 2 to 64
years of age with certain health conditions, and for all adults 65 years of age and older. Your doctor can give you details.
Http://www.cdc.gov/vaccines/hcp/vis/vis-statements/pcv13.html

667.2-year-old complain of papule on the foot no itching pink pale not respond for antifungal?
a. Granuloma
Answer:
Not clear question

668.Down syndrome have bilateral ventricular enlargement


Answer:
Children with down syndrome are at a much higher risk for congenital heart disease. As a comparison: the incidence of
congenital heart disease in the general population is 0.8 percent. The incidence of congenital heart disease in children with
down syndrome is between 40-60 percent.
Some heart defects can be left alone with careful monitoring while others require surgery to correct the problem.
The following types of heart defects in children with down syndrome are discussed below.
• Atrioventricular septal defects (avsds) – these are the most common in children with down syndrome.
• Ventricular septal defects (vsds)
• Atrial septal defects
• Patent ductus arteriosus
• Tetralogy of fallot

• Http://downsyndrome.nacd.org/heart_disease.php

669.Q about cf chromosome


Answer: chromosome 7 called cftr **

670.Nine month child .. Take all vaccine regularly what vaccine should be given ?
Answer: 9 months’ vaccines: measles, meningococcal conjugate quadrivalent (mcv4)
Ref: according to saudi national vaccination schedule ->

671.6-month baby can't sit, hypotonia, crossed lower limb. Which vaccine should be modified?
a. Change opv vaccine to ipv
Answer

672.15 kg child .. What is the daily water requirement ?


Answer: 15kg = 1250 ml.
Source: http://faculty.olin.edu/~jcrisman/service/kwtwebnews/nutrition/fluid.htm
Holliday segar fluid requirement calculation:
1 to 10 kg: 100ml\kg
11 to 20 kg : 1000 ml + 50 ml\kg, for each kg over 10kg.

157
Over 20kg: 1500 ml + 20 ml\kg for each kg over 20kg.
• 1kg = 2.2lbs.

673.Typical case of turner, what another finding you find?


a. Cardiovascular abnormality.
Answer: a

Turner syndrome
features :
a)characteristic facial appearance (low set mildly malformed ears,triangular face,flattened nasal bridge epicanthal fold )
b)webbing of the neck with or without cystic hygroma
c)shield like chest with widened internipple distance
d)internal malformations may include congenital heart defect (coarctation of the aorta is most common anomaly
followed by bicuspid aortic valve
,post stenotic aortic dilation with aneurysm may develop )and renal anomaly (horseshoe kidney )
e)short stature is cardinal feature
f)hypothyroidism
reference :essential nelson of pediatrics

674.Nf1 gene responsible for ?


a. Neurofibromatosis
Answer: a
Nf1 gene located on chromosome 17q11.2
and coding for a tumor suppressor gene
responsible for neurofibromatosis type 1
reference : essential nelson of pediatrics

675.Autosomal recessive disease both parents are carrier and phenotype normal what the chance they have a kid with a
disease ?
a. 25
Answer: a
Ref: wikipedia

676.A baby with bilateral renal agenesis: (oligohydramnios)


Answer: with bilateral renal agenesis: (oligohydramnios which is a sign for the disease during prenatal diagnosis step
reference:: http://www.ncbi.nlm.nih.gov/m/pubmed/264077/

677.Child with high fever 2 weeks and abdominal distention and weight loss
a. Bm
Answer: ?

678.Child with typical symptoms of epiglottitis. (tripod position) diagnosis?

158
Answer:
Http://www.uptodate.com/contents/epiglottitis-supraglottitis-clinical-features-and-diagnosis#h28
679.Child typical symptoms of croup. ( laryngotracheobronchitis ) what is the organism?
a. Parainfluenza virus
Answer: a

680.Baby was playing with his father watch, suddenly his father looks the watch is not working, baby become agitated and
refuse food what you will do?
a. I chose upper gi endo
Answer:

681.Qs about osteosarcoma


Answer: http://www.uptodate.com/contents/osteosarcoma-epidemiology-pathogenesis-clinical-presentation-diagnosis-and-
histology?Source=search_result&search=osteosarcoma&selectedtitle=1~105

682.Qs about child development


Answer: http://www.uptodate.com/contents/developmental-behavioral-surveillance-and-screening-in-primary-
care?Source=search_result&search=child+development&selectedtitle=1~97

683.Baby wave his hand bye bye which developmental milestone reflect
Answer 9 months
Http://2.bp.blogspot.com/-qyxe36dat4o/ufvu_fhzn9i/aaaaaaaaabq/e46pvqmrzcy/s1600/developmental+milestones.jpg

684.3 months baby with history of bronchiolitis, what is the cause?


a. Rsv
Answer: a
Usmle first aid step 2

159
685.4 years old brought by his parents, height < 5th percentile, they ask if he will remain short. What you will do initially :
b. Parental height
answer : parental height

686.8 years old girl presented with fever, numerous bruises over the entire body and pain in both legs. Physical examination
reveals pallor and ecchymosis and petechiae on the face,trunk and extremities. Findings on complete blood count
includes a haemoglobin of 6.3 g/dl, white cell count of2800/mm3 and platelet count of 29,000/mm3. Which of the
following would be the most appropriate treatment?
Answer :dx>> acute lymphoblastic leukemia…. Treatment from torronto>>

160
161
687.What is the most common abdominal tumor in children?

Answer: wilms tumor (master the boards)

688.A child presented with diaper rash with sattalie lesion he was given local creams and steroid but didn’t work:
c. Local antifungal
Answer: a
An antifungal treatment may be prescribed if the child is diagnosed with a yeast infection. This type of treatment is not
recommended without consulting the child's provider first. Antifungal treatments are available as a cream, ointment, or powder.
The treatment is usually applied two or three times per day, beneath a skin ointment or paste, until the rash is gone.
Http://www.uptodate.com/contents/diaper-rash-in-infants-and-children-beyond-the-basics#h14

689.What is the most common cause of facial cellulitis in pediatrics?


Answer: group a streptococus beta hemolitic

690.Which of the following is a feature of tetralogy of fallot?


a. Pulmonary stenosis

Answer: a is correct.
Explanation: teratology of fallot consists of 4 pillers:
1. Pulmonary stenosis
2. Vsd
3. Over-riding of the aorta
4. Right ventricular hypertrophy

691.Child came to the clinic with his mother was having ball, the doctor asked him to throw the ball to him , he through it to
the doctor and he went away to take the ball back
what is the developmental milestone of this child ?

Answer : 2 years = 24 month


Movement/physical development
Stands on tiptoe
Kicks a ball
Begins to run
Climbs onto and down from furniture without help
Walks up and down stairs holding on
Throws ball overhand
Makes or copies straight lines and circles
Http://www.cdc.gov/ncbddd/actearly/milestones/milestones-2yr.html

692.Best diagnosis tool for thalassemia is .


d. Electrophoresis
Answer: a
Evaluation for hemolytic anemia if suspected
Peripheral smear
Electrophoresis
Dna testing (prenatal diagnosis)

162
Https://www.merckmanuals.com/professional/hematology-and-oncology/anemias-caused-by-hemolysis/thalassemias

693.Treatment of de queverian syndrome?


Answer: rest, warm soaks, and nsaids may help in very mild cases. Local corticosteroid injections and a thumb spica splint help
70 to 80% of cases. Surgical release of the first extensor compartment is very effective when conservative therapy fails.
Source: https://www.msdmanuals.com/professional/musculoskeletal-and-connective-tissue-disorders/hand-disorders/de-
quervain-syndrome

694.5 months child: milestone


Https://www.msdmanuals.com/professional/pediatrics/growth-and-development/childhood-development

695. Diagnosis of infective endocarditis?


Blood culture
Answer
The duke diagnostic criteria
A definitive clinical diagnosis can be made based on the following:

2 major criteria
1 major criterion and 3 minor criteria
5 minor criteria

Major blood culture criteria for ie include the following:

Two blood cultures positive for organisms typically found in patients with ie
Blood cultures persistently positive for one of these organisms, from cultures drawn more than 12 hours apart
Three or more separate blood cultures drawn at least 1 hour apart

Major echocardiographic criteria include the following:

Echocardiogram positive for ie, documented by an oscillating intracardiac mass on a valve or on supporting structures, in the
path of regurgitant jets, or on implanted material, in the absence of an alternative anatomic explanation
Myocardial abscess
Development of partial dehiscence of a prosthetic valve
New-onset valvular regurgitation

Minor criteria for ie include the following:

Predisposing heart condition or intravenous drug use


Fever of 38°c (100.4°f) or higher
Vascular phenomenon, including major arterial emboli, septic pulmonary infarcts, mycotic aneurysm, intracranial hemorrhage,
conjunctival hemorrhage, or janeway lesions
Immunologic phenomenon such as glomerulonephritis, osler nodes, roth spots, and rheumatoid factor
Positive blood culture results not meeting major criteria or serologic evidence of active infection with an organism consistent
with ie
Echocardiogram results consistent with ie but not meeting major echocardiographic criteria

Http://emedicine.medscape.com/article/216650-overview#a1

696.Initial treatment of juvenile rheumatoid arthritis is


163
a. Aspirin.
Answer:
Initial drug therapy for children with polyarticular jia (formerly called polyarticular onset juvenile rheumatoid arthritis [ra])
should be aggressive in order to control the inflammatory process and relieve symptoms as quickly as possible while
minimizing drug side effects.
Patients with all but very mild disease activity are usually treated with a disease-modifying antirheumatic drug (dmard),
initiated either at the time of diagnosis or after one to two months of nonsteroidal anti-inflammatory drug (nsaid) therapy due
to ongoing signs and symptoms of active disease.

High disease activity — a dmard is typically started without a trial of nsaids in patients with high disease activity because these
children almost uniformly require more therapy than an nsaid alone.

Http://www.uptodate.com.sci-hub.cc/contents/polyarticular-juvenile-idiopathic-arthritis-
treatment?Source=search_result&search=juvenile+rheumatoid+arthritis&selectedtitle=8~150

697.Child with egg allergy, what vaccine should avoid?


Answer: influenza
The majority of the intramuscular inactivated influenza vaccines (iivs) and the intranasal live-attenuated influenza vaccine
(laiv) are cultured on fluid from chicken embryos. As a result, there is a small amount of egg protein in these vaccines. Thus,
there is a theoretical risk of inducing an allergic reaction when administering the influenza vaccine to an individual with egg
allergy.

Http://www.uptodate.com.sci-hub.cc/contents/influenza-vaccination-in-individuals-with-egg-
allergy?Source=search_result&search=egg+vaccine&selectedtitle=1~150

698.Baby her mother give him honey, then he develop botulism


Answer:
Spores of clostridium botulinum bacteria, found in dirt and dust, can contaminate honey. Infants typically present with
constipation and poor feeding. This presentation is followed by progressive hypotonia, and weakness.

699.Baby with diarrhea and he should take vaccine, what u will do?
a. Delay vaccine
Answer:
In general can be safely administered to children with mild illness, with or without fever (eg, upper respiratory infection, otitis
media, and diarrhea)
For more information http://www.cdc.gov/vaccines/hcp/admin/contraindications-vacc.html

700.Which babies should not get rotavirus vaccine or should wait?


Answer:
Contraindications:
o Severe allergic reaction (e.g., anaphylaxis) after a previous dose or to a vaccine component.
o Severe combined immunodeficiency (scid)
o History of intussusception

701.Boy return from africa and got meningitis. Cause?


Answer: meningococcal meningitis

The largest burden of meningococcal disease occurs in an area of sub-saharan africa known as the meningitis belt, which
stretches from senegal in the west to ethiopia in the east.
Http://www.who.int/mediacentre/factsheets/fs141/en/

164
702.Hearing loss after urti?
Answer:

703.Child with sore throat, fever, palm and sole rash, what is the most likely dx?
Answer= hand, foot, and mouth syndrome (a coxsackie viruses)

704.Child walks freely, tells story, draw his brother as circles and lines, how old:
Answer? 3 years

Http://www.ldonline.org/article/6313?Theme=print

705.Most common cause of epistaxis in children?


Answer: a
a. Self inducing trauma.

The most common causes of epistaxis are


Local trauma (eg, nose blowing and picking)
Drying of the nasal mucosa

706.Pic of child of history support the diagnosis of henoch schonlein purpura. Treatment:
a. Supportive therapy.

Answer:

The vast majority of patients with henoch-schönlein purpura (hsp), also called immunoglobulin a vasculitis (igav), recover
spontaneously. Thus, care is primarily supportive and includes adequate hydration, rest, and symptomatic relief of pain.
Http://www.uptodate.com.sci-hub.cc/contents/henoch-schonlein-purpura-immunoglobulin-a-vasculitis-clinical-manifestations-
and-diagnosis?Source=see_link

707.6 month infant turns bluish when feeding. Auscultation shows holosystolic murmur with single s2. Diagnosis:
165
a. Tga.
Tga present with single s2 but with no murmur. (unless there’s vsd)
Answer: tof
Tof: most common cyanotic heart disease in children, cyanosis usually absent at birth but develop over first 2 year of life, infant
are often asymptomatic until 4-6 month
Auscultation: single s2 with systolic murmur on upper and left sternal border
Tga: most common cyanotic heart in infant cyanosis usually within few hours after birth.
Auscultation: loud single s2. They may not be a murmur if no svd present

708.Ebv pharyngitis what is treatment:


a. Iv acyclovir

Answer?

Primary ebv infections rarely require more than supportive therapy. Even in clinical situations where an antiviral or
immunomodulatory treatment would be desirable, it is not clear that ebv responds.

Http://www.uptodate.com.sci-hub.cc/contents/clinical-manifestations-and-treatment-of-epstein-barr-virus-
infection?Source=search_result&search=ebv+children&selectedtitle=1~150
Similar questions from uqu 4th edition:

709.Case of boy with greasy stool no matter what he eat; recurrent chest infection; sweat test +ve (not sure if the test was
mentioned in the case)
a. Cystic fibrosis ✅
Answer: a
Cf: clinical features in infants and children:
-meconium ileus (20%) - recurrent respiratory symptoms (45%) -failure to thrive (28%)
- others: sinus disease, pancreatic disease: steatorrhea (insufficiency), obstruction of ilium, rectal prolapse, infertility,
musculoskeletal, recurrent dvt, and nephrolithiasis
(uptodate)

166
Diagnostic criteria — both of the following criteria must be met to diagnose cystic fibrosis (cf)

●clinical symptoms consistent with cf in at least one organ system, and

●evidence of cystic fibrosis transmembrane conductance regulator (cftr) dysfunction (any of the following):
•elevated sweat chloride ≥60 mmol/l (on two occasions)
•presence of two disease-causing mutations in cftr, one from each parental allele
•abnormal nasal potential difference

(uptodate)

710.Child drink cow milk his hemoglobin low and mcv low which type of anemia he will has:
a. Iron deficiency anemia ✅
Answer: a
Iron deficiency anemia in infants and children common causes are related to nutrition: e.g:
-introduction of unmodified cow's milk (non-formula cow's milk) before 12 months of age
-occult blood loss secondary to cow's milk protein-induced colitis (source: uptodate)

711.Child has itching and all students in his class got the same infection:
a. Sarcoptes scabie ✅
Answer: a
Disease: scabies. Organism: mite sarcoptes scabiei
Highly contagious. The prominent clinical feature of scabies is itching, it is often severe and usually worse at night. The pruritus
is the result of a delayed type-iv hypersensitivity reaction to the mite, mite feces, and mite eggs.
To diagnose it you need:
-widespread itching that is worse at night
-a pruritic eruption with characteristic lesions and distribution
-other household/ school with similar symptoms (contact)
-diagnosis of scabies can be confirmed by microscopic visualization of the mite, eggs, larvae, or feces in scrapings of papules or
burrows examined under oil immersion.
Organism is mite called sarcoptes scabiei, transmitted by close person to person contact
(uptodate)

712.Child who has repeated infections in chronic granulomatous disease:


a. They are most likely caused by staph and strept ✅
answer: a?
In general, the organisms that infect patients with cgd are catalase producing. More with catalase positive.
(mnemonic: space: staphylococcus aureus, pseudomonas, aspergillus, candida and enterobacter)
(uptodate)

713.Pyloric stenosis:
a. Hypochloremic hypokalemic metabolic alkalosis ✅
answer: a
Most common electrolyte in infant with hypertrophic pyloric stenosis due prolonged symptoms are low chloride potassium and
elevated bicarbonate and ph>7.45
(uptodate and net.)

714.2 q about meningitis how to interpretation of types of meningitis and how to treat children with meningitis?
Answer: ?

167
For more information refer to uptodate:
http://www.uptodate.com/contents/search?Search=child+meningitis&sp=0&searchtype=plain_text&source=user_input&sea
rchcontrol=top_pulldown&searchoffset=&autocomplete=true

715.Scenario about down syndrome: single palmar crease...etc what's the diagnosis?
Answer : down syndrome
For more please refer to this link http://www.uptodate.com/contents/down-syndrome-clinical-features-and-
diagnosis?Source=search_result&search=down+syndrome&selectedtitle=1~150

716.Case about neonate algorithm for resuscitation very long case


Answer;

717.Neonatal apgar score:


Answer: ?

Uptodate aout apgar score http://www.uptodate.com/contents/neonatal-resuscitation-in-the-delivery-


room?Source=preview&language=en-us&anchor=h7&selectedtitle=3~92#h7

718.During school screening boy with asthma .. He is ok .. He use sprayer when need only.
Answer: incomplete?

719.13 y o with hypertension ?


a. Salt and thiazide
Answer: a
Adolescent and children htn from uptodate
Http://www.uptodate.com/contents/treatment-of-hypertension-in-children-and-
adolescents?Source=machinelearning&search=adolescent+hypertension&selectedtitle=2~150&sectionrank=2&anchor=h13#h13

720.Calculates the deficit for child case


Answer: incomplete q
First, assess degree of dehydration

168
Then, replace deficit

Source: toronto notes 2015


Dehydration http://www.uptodate.com/contents/clinical-assessment-and-diagnosis-of-hypovolemia-dehydration-in-
children?Source=search_result&search=dehydration+in+children&selectedtitle=1~150

721.A young patient with fever, sore throat, cervical lymphadenopahty, exudative pharynx, and maculopapular rash over
body?
A. Ebv/infectious mononucleosis
Answer:a

Early signs include fever, lymphadenopathy, pharyngitis, rash, and/or periorbital edema. Relative bradycardia has been
described in some patients with ebv mononucleosis, but it is not a constant finding. Later physical findings include
hepatomegaly, palatal petechiae, jaundice, uvular edema, splenomegaly, and, rarely (1-2%), findings associated with splenic
rupture. Cns findings associated with ebv mononucleosis are rare but usually occur later in the course of the illness. Splenic
tenderness may be present in patients with splenomegaly.pulmonary involvement is not a feature of ebv infectious
mononucleosis. The classic presentation of ebv infectious mononucleosis in children and young adults consists of the triad of
fever, pharyngitis, and lymphadenopathy.the pharyngitis due to ebv infectious mononucleosis may be exudative or
nonexudative.
169
722.Child presenting with vesicular lesions in hands, palms, and sole of feet, which of the following most likely cause?
Answer: coxackie virus
Hand-foot-and-mouth disease (hfmd) is an acute viral illness that presents as a vesicular eruption in the mouth (see the image
below), but it can also involve the hands, feet, buttocks, and/or genitalia. Coxsackievirus a type 16 (cva16) is the etiologic agent
involved in most cases of hfmd, but the illness is also associated with coxsackievirus a5, a7, a9,

723.Chronic granulmatous disease child what is the defect:


Answer: defective phagocyte napdh oxidase✅
Chronic granulomatous disease (cgd) is a rare (∼1:250,000 births) disease caused by mutations in any one of the five
components of the nicotinamide adenine dinucleotide phosphate (nadph) oxidase in phagocytes. This enzyme generates
superoxide and is essential for intracellular killing of pathogens by phagocytes.
Ref: emedicine.

724.Nine month child .. Take all vaccine regularly what vaccine should be given ?
Answer:
9 month vaccine : measles

725.Baby wave his hand bye bye which developmental milestone reflect ?
Answer : wave bye bye by 9-12m of age reflect social development

726.Most important cause of cerebral palsy:


a. Asphyxia
Answer:
Prematurity 78%
Iugr 34%
Intrauterine infection 28
Antepartum hg 27%
Placenta pathology 21%
Multiple pregnancy 20%
Uptodate
Another classification is:
80% prenaral mostly due to vascular occlusion, cortical migration disorders or structural mal-development of the brain during
gestation.
10% perinatal due to hypoxic-ischemic injuries during delivery.
10% postnatal due to meningitis for example
Illustrated textbook of pediatrics for more reading go to page 53.

727.11 years child previously normal, presented with cyanosis, echo showed ventricular hypertrophy i cant remember rt or
left, what is the diagnosis:
a. Vsd
Answer: it is important to know which ventricle coz the diagnosis will change
Rvh>> fallots tetralogy
Lvh>> tricuspid atresia , pulmonary atresia
According to the patient age it most likely fallots tetralogy ( it is the most common cyanotic congenital heart disease in children )
170
728.Parents had a child with cleft palate and they are asking about recurrence rate of cleft palate i their children
Answer: 4%
While if the q was about spina bifida the chance to have another baby is 1%
4% ( source: medscape) and 9% if two children were affected previously)
Vma lectures

729.About description of scarlet fever rash


Answer:
* 24-48 h after pharyngitis , rash begins in the groin ,axillae , neck ,antecubital fossa:pastias lines may be accentuated in flexural
area .
*within 24 h , sandpaper rash becomes generalized with perioral sparing , non-pruritic , non-painful,blanchable.
*rash fades after 3-4d may be followed by desquamation

730.Boy was playing football barefoot glass injured his feet what will be affected?
a. Tendons or nerves
Answer: ???

731.Baby with pic. Of rickets?


Answer: vit. D

171
Https://en.wikipedia.org/wiki/rickets

732.Infantile colic character:


Episodes of crying for more than three hours a day, for more than three days a week for a three-week duration in an otherwise
healthy child between the ages of two weeks and four months,happen in the evening and for no obvious reason , associated
[2]
symptoms may include legs pulled up to the stomach, a flushed face, clenched hands, and a wrinkled brow. the cry is often
high pitched (piercing)
Wikipedia
Colic is commonly described as a behavioral syndrome in neonates and infants that is characterized by excessive, paroxysmal
crying. Colic is most likely to occur in the evenings, and it occurs without any identifiable cause. Compared with regular crying,
colicky crying is more turbulent or dysphonic and has a higher pitch.
Medscape

733. Picture of 1 month baby with raised and scale what is the cause
a. Eczema
Answer:
It may be seborrheic dermatitis : its caused by pityrosporum ovale ,present as sever ,red dipper rash with yellow scale ,thick
crust “cradle cap” may be seen on the scalp .

734.Girl with rt knee swelling aspirations results show elevated wbcs and what best management:
Answer : the q is not complete
Causes of acute monoarthritis
Septic arthritis
Reactive arthritis including the initial presentation
Of acute rheumatic fever and post infectious
Arthritides
Hemarthrosis
Traumatic joint effusion
Bone tumors and acute leukemia
Juvenile arthritis (systemic onset or enthesitis
If juvenile idiopathic arthritis (pauciarticullar ) girl> boy ,ass with increase risk of iridocyclitis .it will present with fever,nodules
,erythematous rashes ,pericarditis ,fatigue
For the criteria of juvenile idiopathic arthritis read kaplan pediatrics lecture notes page 173

172
Lab : positive rf in 15% .ana may be positive , increase esr ,wbc ,platelet
Tx : nsaid is the first lines. Methotrexate is second line
Http://medind.nic.in/icb/t10/i9/icbt10i9p997.pdf
If the results of joint aspiration indicated septic arthritis treat with long course of iv antibiotics and joint aspiration and
sometimes surgical drainage if joint is deep like hip or the resolution is not rapid.

735.Treatment of bronchiolitis?
Answer: treatment is primarily supportive
Treatment according to kaplan pediatrics lecture notes: supportive care; hospitalize if respiratory distress; may give trial of beta-
2 agonist nebulization. And no steroids
Treatment according to first aid 2ck:
Mild: as out patient using fluid and nebulizers as needed ,
Severe: inpatient with contact isolation ,hydration ,o2

736.10 years old boy with hip pain?


Answer: approch to patient with hip pain
Http://www.uptodate.com/contents/image?Imagekey=peds%2f89604&topickey=peds%2f2856&source=see_link

737. Case of fanconi syndrome


Answer:

738.Child present with asd, wht u will hear ?


a. Splitting s2
Answer: wide and fixed ,split s2 and systolic ejaection murmur at the left upper sternal border
Reference: first aid ck2

173
739.Child with cesarian delivery present with signs of distress what is the cause
Chest x-ray show air bronchogram with infiltration
a. Pneumonia ?
Answer : respirtory distress syndrome cause: surfactant deficiency leads to poor lung compliance and alveolar collapse.
X-ray : ground-glass appearance and air bronchogram are characteristic findings in rds.

740.Children present with signs of intestinal obstruction


Answer : intestinal obstruction !!!
Not clear ?

741.Child her mother has hep. B surface antigene positive, after 3 months he becomes positive, he received bcg vaccine, what
can give him now?
Answer: ?????

742.Child has itching and all student in his class got the same infection: sarcoptes scabiei
Answer:
Scabies
Human scabies is an intensely pruritic skin infestation caused by the host-specific mite sarcoptes scabiei var hominis. A readily
treatable infestation, scabies remains common primarily because of diagnostic difficulty, inadequate treatment of patients and
their contacts, and improper environmental control measures.
(http://emedicine.medscape.com/article/1109204-overview#a3)

743.Milestone > baby healthy run to the doctor play a role model as his father can’t complete a sentence can’t eat with spoon

Answer: 18 months

744.Chid drink cow milk his hemoglobin low and mcv low which type of anemia he will has:

A. Iron deficiency anemia


Answer:
Consumption of cow's milk (cm) by infants and toddlers has adverse effects on their iron stores, a finding that has been well
documented in many localities.
(http://www.ncbi.nlm.nih.gov/pubmed/22043881

174
745.Preventing child from drinking before bed, encourage to go to toilet before bed, all these measures to help child with?
A. Enuresis

746.Child above 90th percentile in height, long scenario and cardiac abnormality with fatigue…etc ?
A. Marfan syndrome

747.Kid with renal failure has post-streptococcal glomerulonephritis with casts, you will find?

Answer:
Post-streptococcal glomerulonephritis (psgn) is usually diagnosed based upon:

1. The clinical findings of acute nephritis include hematuria with or without red blood cell casts (brown urine), variable
degrees of proteinuria, periorbital or generalized edema, and hypertension.
2. Recent post-streptococcal infection is most commonly demonstrated by serological markers for elevated antibodies to
extracellular streptococcal antigens. The streptozyme test, which measures 5 different streptococcal antibodies, is
positive in more than 95% of patients with apsgn due to pharyngitis.
Antistreptolysin (aso)
Antihyaluronidase (ahase)
Antistreptokinase (askase)
Antinicotinamide-adenine dinucleotidase (anti-nad)
Anti-dnase b antibodies
Please refer to the source for additional information.
Http://emedicine.medscape.com/article/980685-workup

748.Child is doing fine, his brother died while heading to work. What we should investigate for?
A. Hypertrophic cardiomyopathy


Answer:
Instantaneous or sudden death may result from dysfunction/abnormalities of the heart and its vessels, noncardiac vessels,
pulmonary system, and central nervous system (cns). Heart related causes include arrhythmias, cardiomyopathies,
inflammatory, valvular diseases & acute myocardial infarction.
Http://emedicine.medscape.com/article/1680282-overview#a5

749.A child with hydrocephalus progressively increasing in between the 3rd and 4th ventricle - which area is blocked?

Answer: one of the most common causes of hydrocephalus is "aqueductal stenosis." in this case, hydrocephalus results from a
narrowing of the aqueduct of sylvius, a small passage between the third and fourth ventricles in the middle of the brain.
Http://www.ninds.nih.gov/disorders/hydrocephalus/detail_hydrocephalus.htm

750.A scenario about short stature, webbed neck what's chromosome?

Answer: 45x (turner syndrome)


Http://emedicine.medscape.com/article/949681-overview

751.A scenario about down syndrome: single palmar crease...etc what's the diagnosis?
Answer: down syndrome

175
752.Calculates the deficit for child case?
Answer:

753.Treatment of juvenile rheumatoid arthritis?


Answer: pharmacologic management consisting of:
Nonsteroidal anti-inflammatory drugs (nsaids),
Disease-modifying antirheumatic drugs (dmards),
Biologic agents,
Intra-articular and oral steroids
Source: http://emedicine.medscape.com/article/1007276-treatment

754.Two q about meningitis, interpretation of types of meningitis and how to treat children with meningitis?
Source: http://emedicine.medscape.com/article/961497-treatment#d10

755.Cystic fibrosis mode of inheritance?


Answer: autosomal recessive

756.Vaccine of hep. A missed second dose what to do?


Answer: have it as soon as possible but you don't need to start with the first dose again
Source: family medicine consultant.

757.Normal 10 y/o child, his parents worry about height of child.............? What the most things to ask??
- can't remember choices
Details: constitutional delay of growth and adolescence (cdga) and familial short stature (fss), the two most common entities
associated with short stature, are characterized by deceleration of linear growth during the first 2 or 3 years of life.

758.What is the milestone of 4 year child?

759.Boy with hypopigmented lesion in back and extremity becomes lighter with sun exposure treatment topical steroid
antibiotic ...
:antifungal

176
A fungal infction called pityriasis versicolor. The organism is called malassezia furfur. A common, benign, superficial cutaneous
fungal infection usually characterized by hypopigmented or hyperpigmented macules and patches on the chest and the back.
And it usually recur. Treated with topical antifungal

760.Neonate came with decreased feeding & activity + fever. On examination, baby is hypotensive, what’s the diagnosis?
A. Septic shock.

Answer: a

761.Child with symptoms. On examination, there's strong pulse in the upper limbs and absence in lower limbs. What’s the
diagnosis?
A. Coarctation of aorta.

Answer: a
First aid:
• Asymptomatic htn (upper extremity htn).
• Classical physical examination finding is a systolic bp that is higher in the upper extremities. The difference in bp in
right and left can indicate the point of coarctation.

762.Daily fluid requirement for child 10 kg ..... 1000 ml /day

763.Child is given ffp heparin what's the dx?!


-dic e thrombus

• toronto:
•in hemorrhage: replacement of hemostatic elements with platelet transfusion, frozen plasma, cryoprecipitate ƒ maintain
platelets >50,000/mm3 and hemoglobin >80 g/l ƒ 4-5 units of ffp if inr >1.5 or aptt >38 ƒ 10 units of cryoprecipitate if fibrinogen
<100 mg/dl ƒ 1 adult dose of buffy-coat platelets if <10,000 (<20,000 if febrile, <50,000 before invasive procedure)
• in thrombotic phase: ufh or lmwh in critically ill, non-bleeding patients
•ffp: depletion of multiple coagulation factors (e.g. Sepsis, dic, dilution, ttp/hus, liver disease), emergency reversal of life-
threatening bleeding secondary to warfarin overdose

764.Dehydrated child 25 kg , maintenance is ....1500


Answer: maintenance rule.. You should know it by now
- 4,2,1
- 4 ml/kg/h for the first 10 kg.
- 2 ml/kg/h for the second 10 kg.
- 1 ml/kg/h after that.

765.Case scenario , they mentioned mother ht & father ht . And they asked about excepted ht of the child?!
(mother ht+ father ht ) / 2 + or - 5

766.In developing country to prevent dental caries add to water?


A. Floride
177
Answer: a

767.Baby with oral thrush and ask about diagnosis?


A. Oral candidiasis.

Answer: a, kaplan
768.Child with white reflex in both eyes:
A. Congenital cataract.

Answer: a

769.Child present with fever and sore throat what of the following suggest viral cause?
A. Rhinorrhea and mucus secretion

770.Calculate fluids replacement for child for 24 hr?


Answer depends on body weight
For 24 hours use the rule: 100:50:20
For hourly replacement use: 4:2:1

771.What is contraindication of nursing woman to lactate her child by breast when she has hepatitis c virus ?

Answer: crackled nipple


Not absolute contraindication
Https://www.cdc.gov/breastfeeding/disease/hepatitis.htm

772.2 month old has diarrhea and his mother is worried from dehydration what will you advice the mother:
Change milk

Answer: oral rehydration solution

773.A child presented with diaper rash with satellite lesion he was given local creams and steroid but didnt work:

Answer: local antifungal


(case of candida diaper rash)

774.What is the most common cause of facial cellulitis in pediatrics?


Answer;
Facial cellulitis in pediatric practice is now largely due to
(1) trauma/loss of skin integrity with secondary infection,
(2) dental problems,
(3) severe sinusitis.

775.To prevent hemorrhage in newborn we give at birth which vit :

Vit k
178
Explanation: im vitamin k is recommended for all neonates within 6 h of birth to reduce the incidence of intracranial
hemorrhage due to birth trauma and of classic hemorrhagic disease of the newborn
Reference: http://www.msdmanuals.com/professional/nutritional-disorders/vitamin-deficiency,-dependency,-and-
toxicity/vitamin-k

776.Newborn after 2weeks c/o bilateral conjunctivitis ,chest infiltration , lung crepitation, what is organism ??
A. Chlamydia

Explanation: chlamydia trachomatis is the most common cause of sexually transmitted genital infections in the united states.
Infants born vaginally to infected mothers with genital disease are at risk for acquiring c. Trachomatis, which usually presents as
conjunctivitis and/or pneumonia.
Reference: http://www.uptodate.com/contents/chlamydia-trachomatis-infections-in-the-newborn

777.Child drink cow milk his hemoglobin low and mcv low which type of anemia he will has:

Answer: iron deficiency anemia


Iron deficiency anemia develops when body stores of iron drop too low to support normal red blood cell (rbc) production.
Inadequate dietary iron, impaired iron absorption, bleeding, or loss of body iron in the urine may be the cause. Iron equilibrium
in the body normally is regulated carefully to ensure that sufficient iron is absorbed in order to compensate for body losses of
iron.
Http://emedicine.medscape.com/article/202333-overview

778.Diabetic or (gdm) pregnant in her 40 or 42 ga nvd on examination there was an absent moro reflex on the lt. Side of the
baby dx?
A. Erb's palsy

Answer: birth-related (obstetrical) brachial plexus injuries” the mother is diabetic so big baby is a risk factor”
Causes of absent infant moro reflex:
Cerebral palsy
Damage to spinal cord and brain
Clavicle fracture
Broken shoulder bone
Erb's palsy
th th
Erb-duchenne paralysis involves the 5 & 6 cervical nerves & is the most common & usually mildest injury. The infant cannot
abduct the arm at the shoulder, externally rotate the arm or supinate the forearm. The usual picture is one of painless
adduction, internal rotation of the arm, and pronation of the forearm. The moro reflex is absent on the involved side,& the
th
hand grasp is intact. Reference: table : step-up of pediatric page414 , nelson essential of pediatric 7 edition page 202

179
779.Preventable cause of gastroenteritis by vaccine?

A. Rota virus
Answer: a. Http://emedicine.medscape.com/article/964131-medication#1

780.Least cardiac anomaly associated with infective endocarditis?

A. Asd
Answer:
The most common chd group among children with ie was cyanotic chd lesions, present in 34% of cases. Although the next most
frequent defects among cases were atrial septal defect (16%) and ventricular septal defect (15%), their proportions were
reduced in comparison with controls (27% each) http://www.medscape.com/viewarticle/819866

781.Neonate came with decrease feeding & activity + fever , o/e baby is hypotensive dx?

Septic shock
Reference: http://emedicine.medscape.com/article/979128-overview

782.Preterm baby c/o sob x-ray showed gross ground appearance + air bronchogram, this is due to?!

Pneumonia
Low surfactant "
Answer: respiratory distress syndrome

180
th
Illustrated text book of pediatric 4 edition, pg 162.

783.Child with decrease uop, tea colored urine , generalized swelling next investigation :

This is a case of acute glomerulonephritis. The investigations include(in order): ua,cbc,bun,esr and complement . Streptozyme
testing may be useful. Imaging studies are helpful in some patients, for assessment of clinical signs suggesting extrarenal
involvement.

Reference:medscape http://emedicine.medscape.com/article/239278-workup

Reference: toronto notes

784.4 y o boy, developmentally normal but came with his mother with the complaint that he doesn't stop sucking his thumb .
What's the best way to prevent this habit ?
A. Reward not doing so *

785.Dehydrated child 25 kg, calculate the maintenance


A. 1500 (wrong)
Answer: 1600 ml/day. 10 x 100 = 1000
10 x 50 = 500
5 x 20= 100
So the maintenance is 1600 ml

Body weight Ml / kg / day Ml / kg / hr


First 10 kg 100 4
Second 10 kg 50 2
181
Each additional kg 20 1

Reference: http://reference.medscape.com/calculator/maintenance-fluid-calculation-child

786.Six year old girl with pan-systolic murmur

The causes of holosystolic murmur are: mitral regurgitation, tricuspid regurgitation and vsd.

Reference: kaplan usmle step 2 pediatric

787.A child can walks alone and build three cubes. How old is he?
a. 18 months
Answer: a

Reference: kaplan usmle step 2 pediatric

788.A 3 year old patient with uti is febrile what to do:


Answer: US
Patients with cystitis do not present with fever. This is a case of pyelonephritis and patients present with abdominal or flank
pain,fever,malaise,nausea,vomiting, and diarrhea. To diagnose • u/a, urine c&s
• cbc and differential
• imaging indicated if suspicious of complicated pyelonephritis or symptoms do not improve with 48-72 h of treatment:
abdominal/pelvic u/s or ct
reference: toronto notes+kaplan pediatric

789.7 days old baby presented with vomiting and fever ,culture shows catalase positive[negative] ,gram positive in chain
,beta hemolytic .mother had hx of infection before delivery .the tx is :
A.ampicillin.
Answer: a
This is a case of neonatal sepsis caused by group b streptococcus. The treatment if no evidence of meningitis: ampicillin and
aminoglycoside until 48–72-hour cultures are negative.
If meningitis or diagnosis is possible: ampicillin and third-generation cephalosporin (not ceftriaxone)
Reference:kaplan pediatric
Flow chart for gram positive and negative bacteria
http://www.courses.ahc.umn.edu/pharmacy/6124/handouts/gram_pos_neg.pdf

790.Which enzyme is deficient in phenylketonuria?


Phenylalanine hydroxylase (pah)
Deficiency of phenylalanine hydroxylase prevents conversion of phenylalanine to tyrosine leading to build up of toxic
metabolites . Reference: toronto notes

182
791.A young child presented with painful lesion in the back of her mouth and soft palate

A. Herpangina
Answer:a
An acute febrile illness associated with small vesicular or ulcerative lesions on the posterior oropharyngeal
structures.herpangina is one of many manifestations of enterovirus infection. Patients present with: fever(may be the first
apparent symptom), sore throat and pain upon swallowing , headache or backache.
Reference: http://emedicine.medscape.com/article/218502-clinical#b4

792.What is the most common pathogen in patient with chronic granulomatous disease?
A. Staphylococcus arues
Answer: a
Unusual infections with catalase-positive organisms:s. Aureus (most), nocardia sp. ,s.marcescens, b. Cepacia, aspergillus or
c.albicans.
Reference: kaplan pediatric

793.Child with fever. Rash, greyish macule in moth dx???

A) measles
"the manifestations of the 3-day prodromal period are cough, coryza, conjunctivitis, and the pathognomonic koplik spots (gray-
white, sand grain-sized dots on the buccal mucosa opposite the lower molars"
Reference: text above taken exactly from nelson text book of pediatrics, page 330

794.The most common complication of mump? Hearing loss encephalitis sterility

Answer: complications include:


Inflammation of the testicles (orchitis) in males who have reached puberty.
Inflammation of the brain (encephalitis)
Inflammation of the tissue covering the brain and spinal cord (meningitis)
Inflammation of the ovaries (oophoritis) and/or breasts (mastitis) in females who have reached puberty
Deafness
Reference: http://www.cdc.gov/mumps/about/complications.html

795.Rota virus how to confirm the diagnosis ?

A-stool antigen
Reference : https://www.cdc.gov/rotavirus/clinical.html

183
796.Child development under 50 percentile, and delay teeth growth, lab normal except for ca "low", management?
Look for the cause
A) calcium

797.Case (young or kid) bilateral knee pain then rash starts on legs thigs and buttocks = typical
Hsp

Answer: these have missing information and options however == henoch-schonlein purpura (hsp) is a disease involving
inflammation of small blood vessels. It most commonly occurs in children. The inflammation causes blood vessels in
the skin, intestines, kidneys, and joints to start leaking. The main symptom is a rash with numerous small bruises,
which have a raised appearance, over the legs or buttocks.
Subsequently, symptoms develop, of which the following are the most common:
Rash (95-100% of cases), especially involving the legs; this is the hallmark of the disease
Abdominal pain and vomiting (35-85%)
Joint pain (60-84%), especially involving the knees and ankles

Reference: http://emedicine.medscape.com/article/984105-overview
Http://www.webmd.com/skin-problems-and-treatments/henoch-schonlein-purpura-causes-symptoms-treatment

798.Baby with diarrhea , no vomiting , how to manage ?


Ors
Question and choices incomplete

Explanation: mild dehydration: minimal findings: may have slightly dry buccal mucous membranes, thirst, slightly decreased
urine output. Moderate: similar symptoms with increased severity and tachycardia, little or no urine output, lethargy, sunken
eyes and fontanelles, loss of skin turgor. Severe: more severe symptoms, rapid, thready pulse; no tears; cyanosis; rapid
breathing; delayed capillary refill; hypotension; mottled skin; coma. For mild/moderate: oral rehydration. For severe or failed
oral: iv isotonic bolus 20 ml/kg.
Reference: http://www.merckmanuals.com/professional/pediatrics/dehydration-and-fluid-therapy-in-children/dehydration-in-
children

799.Baby was jaundice then become greenish :


Oxidation of the bilirubin

Question and answers incomplete


Explanation: if oxidation is the answer then this is the best explanation i could find!: bilirubin is created by the action of
biliverdin reductase on biliverdin, a green bile pigment that is also a product of heme catabolism. And when bilirubin, is oxidized
it reverts to become biliverdin (green pigment) once again.
Reference: https://en.wikipedia.org/wiki/bilirubin

800.Baby presented with abdominal bloating and constipation , inv shows increase ca+ . I forgot the choices
No choices available

801.15 yo female , no period still , on examination slight breast buds with wide spaced areola , fine pubic hair on labia majora
, (not sure if they mention a normal growth parameters in the question and unfortunately i forgot the choices :/ ) .
184
Question and choices incomplete

802.Senario about renal tubular acidosis in child


Question incomplete and no choices

803.Milestone for baby can hold his head and when he looks at his flying hand, he laughs and coos
A. 4 months

* this is my answer because there were no choices for this question + see the above table for developmental milestones

804.Infant with gonococcal eye infection what's the treatment


A. Gonococcal ophthalmia neonatorum (neonatal conjunctivitis)

Answer: a
- Infants with gonococcal ophthalmic disease should be hospitalized and observed for response to therapy and for
disseminated disease.
- Treatment consists of a single dose of ceftriaxone (25 to 50 mg/kg, iv or im). A single dose of cefotaxime (100 mg/kg, iv
or im)
- Topical antibiotic therapy alone is inadequate and is not necessary when systemic treatment is provided
- The eyes should be irrigated frequently with saline until the discharge clears.
- Https://yhdp.vn/uptodate/contents/mobipreview.htm?38/34/39462#h9

805.Patient with a family history of multiple sudden cardiac death. This patient has marfan's syndrome features. What might
be the cause of death?
A. Ruptured aortic aneurysm

Answer: a
- Http://bestpractice.bmj.com/best-practice/monograph/514/follow-up/prognosis.html
- Http://bestpractice.bmj.com/best-practice/monograph/514/follow-up/complications.html

806.Intranasal influenza vaccine (the name of the vaccine)


A. Flumist quadrivalent (laiv4)

Answer: a
- Indicated for active immunization to prevent influenza a and b viruses in healthy children, adolescents, and adults aged
2 to 49 years.
- Se (children): runny nose/nasal congestion (46-50%), cough (36-40%), irritability (16-20%), headache (16-20%)
- Contraindications:
Hypersensitivity to eggs, sulfites
Iv/im administration
Children/adolescents receiving aspirin
Asthma, reactive airway disease
Diabetes, renal dysfunction
Hemoglobinopathies
Immunodeficiency diseases
185
Patients on immunosuppressants
Any chronic pulmonary/cardivascular disorder

- Http://emedicine.medscape.com/article/219557-medication#4
- Http://www.cdc.gov/flu/protect/vaccine/vaccines.htm

807.Newborn meningitis, most common organism.


Answer : group b strep is the most common followed by e. Coli followed by listeria.

808.15 month child with meningitis, treatment. (table)

809.Pincer grasp develops at 9 months

810.Purulent discharge from newborn's eye, what's the organism;


Gonorrhea
Gonococcal conjunctivitis tends to be more severe than other causes of ophthalmia neonatorum. The classic presentation is
severe bilateral purulent conjunctivitis.

811.Child eat many tablet of paracetamol tablet, which of the following increase risk of poisoning :
A- glutathione depletion
Answer : a

812.Child with vasoocclusive crises last week now came for hep a vaccine , what to do ?
Hepatitis a and hepatitis b vaccines are indicated for those who require repeat transfusions, such as sickle cell anemia
Http://www.phac-aspc.gc.ca/publicat/cig-gci/p03-chroni-eng.php

813.Child with air on bowel wall?


Necrotic entrocolitis

186
Necrotizing enterocolitis (nec) is a serious gastrointestinal disease of
Neonates. Nec is characterized by mucosal or transmucosal necrosis
Of part of the intestine.
Pneumatosis intestinalis—gas in the bowel wall that displays
A linear or bubbly pattern—is present in 50-75% of patients.)
Http://emedicine.medscape.com/article/411616-overview#a2

814.Child complaining of bilateral knee pain after that he developed purple rash over lower limb // dx ???
Henoch-schönlein purpura : the rash is most often seen on the legs, buttocks, elbows and around the waistline. It affects both
sides of the body. Joints, particularly the knees and ankles, can become swollen, tender, warm and painful. The inflammation
will gradually clear over time and there is not any lasting damage to the joints. The joint pains tend to come on after the rash
has appeared in most people.
Http://patient.info/health/henoch-schonlein-purpura-leaflet

815.Case of 2 month old baby, his parents feed him with bottle of milk while he is sleeping, baby developed plaque and
discoloration of his teeth, what’s the cause?

Answer: only one choice mentioned (nursing bottle carries)


Causes of teeth discoloration in infants: inadequate dental & gum hygiene, frequent consumption of sugary liquids
(milk, juice, formula), iron-fortified formula or iron supplements, genetically weak enamels, newborn jaundice, poor
feeding practices.
Early childhood dental carries (also known as baby bottle carries or nursing bottle carries) is an infectious disease
caused by the interaction of bacteria, mainly streptococcus mutans and sugary foods on tooth enamel.
It is one of the most common communicable diseases in childhood.
More on the topic: http://www.medscape.com/viewarticle/740461_2

816.Pediatric patient come with barking cough what is the best diagnosis ?
Answer: chest x ray ---> show steeple sign in croup patient
Croup is a common, primarily pediatric viral respiratory tract illness. As its alternative names, laryngotracheitis and
laryngotracheobronchitis, indicate, croup generally affects the larynx and trachea, although this illness may also extend to the
bronchi. It is the most common etiology for hoarseness, cough, and onset of acute stridor in febrile children.
Http://emedicine.medscape.com/article/962972-overview

817.Case of barking cough?

Answer: croup
Croup usually begins with nonspecific respiratory symptoms (ie, rhinorrhea, sore throat, cough). Fever is generally low grade
(38-39°c) but can exceed 40°c. Within 1-2 days, the characteristic signs of hoarseness, barking cough, and inspiratory stridor
develop, often suddenly, along with a variable degree of respiratory distress.
Http://emedicine.medscape.com/article/962972-clinical

818.Case of drooling of saliva?

Answer: epiglottis
Epiglottitis is an acute inflammation in the supraglottic region of the oropharynx with inflammation of the epiglottis, vallecula,
arytenoids, and aryepiglottic folds

187
Http://emedicine.medscape.com/article/763612-overview

819.3 y.o. Mile stone

820.Croup case what will you hear by pulmonary auscultation?


The obvious respiratory distress and harsh inspiratory stridor are the most dramatic physical findings. Auscultation reveals
prolonged inspiration and stridor. Rales also may be present, indicating lower airway involvement.

821.Child with diarrhea, no vomiting and his mother worried about dehydration what will you advice her
Answer: oral fluid therapy is effective, safe, convenient, and inexpensive compared with iv therapy. Oral fluid therapy is
recommended by the american academy of pediatrics and the who and should be used for children with mild to moderate
dehydration who are accepting fluids orally unless prohibited by copious vomiting or underlying disorders (eg, surgical
abdomen, intestinal obstruction).
Http://www.merckmanuals.com/professional/pediatrics/dehydration-and-fluid-therapy-in-children/introduction-to-
dehydration-and-fluid-therapy-in-children

822.Child or infant come with abdominal mass compressing collecting system, lung mets?
Answer: wilms tumor, most common pedia. Renal malignancy (2-3 y), painless abd. Mass is common presenting sign,
common metastasis site is lungs. Step up pediatrics

Incomplete questions:

823.10 year old with ass:


A. Vsd

824.Child spitting milk, growing fine..what is the dx what will u do next?

825.27days neonate ,, with pale stool , high conjugate ,, diagnosed with neonatal jaundice and treated with fluroscenc. >>
what is the cause :

188
826.Child 2-4 mths ? Vominting ? Wt height 50 th centile , wt to do? Reassure

827.2 year old baby only says mama and baba. His brother had history of speech delay until 3 years old. (not sure if question
is about etiology or diagnosis)

Answer: no choices
Speech and language delay may be due to: hearing loss, global developmental delay, anatomical deficit (e.g., cleft
palate), environmental deprivation, and familial.
A hearing test and assessment by a speech and language therapist are the initial step
Source: illustrated
More sources or speech delay:
Https://yhdp.vn/uptodate/contents/mobipreview.htm?25/46/26343
Http://www.ncbi.nlm.nih.gov/pmc/articles/pmc2491683/table/t1/

828.Child with mild persistent asthma. Visits the er once every month. He is on ventolin. What to add to his medication?
Answer: no choices
According to guidelines, mild persistent asthma is treated with low dose inhaled corticosteroids (ics).
Alternatives include a leukotriene receptor antagonist (ltra)
Type of medications depend on child’s age
Source: https://www.nhlbi.nih.gov/files/docs/guidelines/asthma_qrg.pdf

829.Child with swelling inside the leg then developed skin manifestations. What is the side effect of medication you well use?
Answer: not sure
Source on evaluation of inguinal swelling in children: https://yhdp.vn/uptodate/contents/mobipreview.htm?21/42/22185

830.Child there is widening of his leg


Most of answer contain rickets

831.Child with hirschsprung disease what to do?


Consult with pediatric surgeons and pediatric gastroenterologists. Genetic consultation may be indicated (if a heritable or
chromosomal anomaly is suspected).
Surgical management of hirschsprung disease begins with the initial diagnosis, which often requires a full-thickness rectal
biopsy. Traditionally, a diverting colostomy was created at the time of diagnosis, and definitive repair was delayed until the child
grew to a weight of 10 kilograms.
Http://emedicine.medscape.com/article/178493-treatment#showall

832.Child with labored breath what to do? (incomplete - no options)


Answer: difficult to answer without options but every approach starts with pat & abcde assessment.
Explanation:
The assessment of a child follows a very simple process, with the approach adjusted according to the age and developmental
characteristics of the child. The initial assessment, consisting of the internationally recognized pediatric assessment triangle
(pat) and the hands-on abcde (airway, breathing, circulation, disability, exposure) assessment,
Full approach found in link
Link: http://airway.jems.com/2011/10/evolving-assessment/

189
833.What is central treatment of kawasaki?
a. Ivig + high dose asprin. Bmj http://bestpractice.bmj.com/best-practice/monograph/236/treatment.html

834.Diet causes kwashiorkor?


a. Aetiology remains ill defined, but is not entirely because of a deficiency of protein in the diet, as commonly
suggested. It is typically associated with a corn-based diet, recent weaning, measles, or diarrhoeal illness. Bmj
http://bestpractice.bmj.com/best-practice/monograph/1022/basics/aetiology.html

835.Child walks freely , tells story , draw his brother as circles and lines , how old is he?
a. 4 years old. Cdc: https://www.cdc.gov/ncbddd/actearly/pdf/parents_pdfs/milestonemomentseng508.pdf

836.90% of children in a village have cretinism


a. Levothyroxine and iodine supplement. Medscape http://emedicine.medscape.com/article/919758-overview

837.Child with blue dot in testis and painful mass in inguinal area?
Answer: testicular appendage torsion

838.A child present with s&s of leukemia with calla +ve? Dx?
Answer:all

839.An infant has rsv infection. What is the treatment?


Answer: ribavirin
The initial management of bronchiolitis is by means of supportive therapy. This is achieved by maintaining oxygenation,
hydration, and ventilation in case apnea developed. Pharmacologic therapy has limited role in management however ribavirin
aerosol is indicated in following conditions to minimize or prevent need of ventilation: congenital heart and pulmonary disease,
immunosuppressed patients & infants less than 6 weeks.
Http://emedicine.medscape.com/article/961963-treatment#d10

840.Child with non bilious vomiting and olive mass, what you will use in diagnosis?
a. Sonography
The patient is having pyloric stenosis.
Reference: http://emedicine.medscape.com/article/929829-overview

841.Neonate continuously crying, ultrasound showed doughnut sign, how to manage?

Doughnut sign in us indicates intussusception. The management of intussusception is:


- For all children, start iv fluid resuscitation and nasogastric decompression asap.
- Then non-operative reduction: therapeutic enemas can be hydrostatic, with either barium or water-soluble contrast, or
pneumatic, with air insufflation.
- If non-operative reduction is unsuccessful or if obvious perforation is present, promptly refer the infant for surgical care
(surgical reduction).

190
**the presence of peritonitis and perforation revealed on plain radiographs are the only 2 absolute contraindications to non-
operative reduction.

842.5 years old, with recurrent infection, greasy diarrhea, slow growing, he's jaundiced with positive sweat chloride test?
A- Cystic fibrosis

Cystic fibrosis (cf) is a disease of exocrine gland function that involves multiple organ systems but chiefly results in chronic
respiratory infections, pancreatic enzyme insufficiency. Requirements for a cf diagnosis include either positive genetic testing or
positive sweat chloride test findings and 1 of the following: typical chronic obstructive pulmonary disease (copd), documented
exocrine pancreatic insufficiency & positive family history.

- Gi symptoms: meconium ileus, abdominal distention, intestinal obstruction, increased frequency of stools, failure to
thrive (despite adequate appetite), flatulence or foul-smelling flatus, steatorrhea (greasy stool), recurrent abdominal
pain, jaundice, gi bleeding.
- Respiratory symptoms: cough, recurrent wheezing, recurrent pneumonia, atypical asthma, dyspnea on exertion, chest
pain.
- Genitourinary symptoms: undescended testicles or hydrocele, delayed secondary sexual development, amenorrhea.

843.Child with renal disease, his family are afraid that he become like his father on dialysis. He is also known to have snhl,
what is the disease?
A- Alport syndrome

Alport syndrome encompasses a group of inherited, heterogeneous disorders involving the basement membranes of the kidney,
cochlea and the eye. They present with:
Renal manifestations: hematuria, proteinuria, hypertension.
Hearing impairment: sensorineural hearing loss
Ocular manifestations: anterior lenticonus, dot-and-fleck retinopathy, posterior polymorphous corneal dystrophy, temporal
macular thinning.

844.Child hematuria. Hearing loss


A. Alport

845.Child bilious vomiting and constipation since birth diagnosis? Rectum biopsy. The patient has hirschsprung disease
Reference: http://emedicine.medscape.com/article/178493-overview

846.Type of htn in adolescent? Essential hnt


Reference: http://www.aafp.org/afp/2006/0501/p1558.html

847.Child with thumb sign, dx? Epiglotitis


Reference: http://emedicine.medscape.com/article/763612-overview

848.Child with epiglotitis (swollen epiglottis) with its s&s, what is the treatment?
Vancomycin, ceftriaxon, others
Answer: combination therapy with a third-generation cephalosporin (eg, ceftriaxone or cefotaxime) and an antistaphylococcal
agent active against mrsa (eg, clindamycin, vancomycin)
Reference: http://www.cursoenarm.net/uptodate/contents/mobipreview.htm?29/20/30016

849.Patient with cough and vomiting after cough, fever for 3 weeks, dx?

191
Pertussis
Reference: http://emedicine.medscape.com/article/967268-overview

850.Scenario of child with guillain-barre syndrome had viral gastroenteritis 3 weeks ago, asked about prognosis?
Answer: residual weakness
Read more about prognosis: http://emedicine.medscape.com/article/315632-overview#a6

851.Opv and ipv common mechanism of action


- The inactivated polio and oral polio vaccine produces antibodies in the blood to all three types of poliovirus. In
the event of infection, these antibodies prevent the spread of the virus to the central nervous system and
protect against paralysis.

- Oral polio vaccine also produces a local, mucosal immune response in the mucous membrane of the intestines.
In the event of infection, these mucosal antibodies limit the replication of the wild poliovirus inside the
intestine.

852.Von will brand disease >>>> pathophysiology


Factor viii
The aptt is mildly prolonged in approximately 50% of patients with vwd. The prolongation is secondary to low levels of fviii
because one of the normal functions of vwf is to protect fviii from degradation.
Reference: http://emedicine.medscape.com/article/206996-overview

853.Bcl bcr gene associated with:


- Chronic myelogenous leukemia (cml)
The diagnosis of cml is based on the following: histopathologic findings in the peripheral blood and philadelphia (ph)
chromosome in bone marrow cells.
Reference: http://emedicine.medscape.com/article/199425-overview

854.12 years old with myopia, pectus exavatum, congenital heart disease, height > 90th percentile weight <50th percentle,
what's your diagnosis?
A- Marphan's syndrome
Answer: a

Marfan syndrome (mfs) is a spectrum of disorders caused by a heritable genetic defect of connective tissue that has an
autosomal dominant mode of transmission. The defect itself has been isolated to the fbn1 gene on chromosome 15, which
codes for the connective tissue protein fibrillin. Abnormalities in this protein cause clinical problems of the musculoskeletal,
cardiac, and ocular system. Skeletal deformities such as thoracolumbar scoliosis, thoracic lordosis, and pectus excavatum.
In the cardiovascular system, aortic dilatation, aortic regurgitation, and aneurysms. Ocular findings include myopia,
cataracts, retinal detachment, and superior dislocation of the lens.

855.Child with facial swelling hypoalbuminemia, high cholestrol?


Nephrotic syndrome.
Nephrotic syndrome is defined by the presence of nephrotic-range proteinuria, edema, hyperlipidemia, and
hypoalbuminemia.
Reference: http://emedicine.medscape.com/article/982920-overview

192
856.Premature presented with abdominal distention and air on x-ray ?
A- Necrotizing enterocolitis
Answer: a
Link http://radiopaedia.org/articles/necrotising-enterocolitis-1

857.5 year old child present with odor of adult and pubic hair what investigation will do ?
Answer:
• investigation: initial screening tests: bone age, serum hormone levels (estradiol, testosterone, lh, fsh,tsh, free t4, dhea-s, 17-
oh-progesterone)
Secondary tests: mri head, pelvic u/s, β-hcg, gnrh, and/or acth stimulation test

858.Newborn presented with vomiting and enlarged clitoris, lab was included showing hypokalemia and hypernatremia.
What is the most likely diagnosis?
A- Congenital adrenal hyperplasia
Answer: a if it is hyperkalemia not hypo
Http://emedicine.medscape.com/article/919218-clinical

859.Child complaining of bilateral knee pain after that he developed purple rash over lower limb. Diagnosis?
Answer: hsp
Http://emedicine.medscape.com/article/984105-clinical

860.Parent came for consuling. Most common mendilne inheritance?


Answer: thalassemia

The alpha and beta thalassaemias are the most common inherited single-gene disorders in the world with the highest
prevalence in areas where malaria was or still is endemic.

861.Adolescent (12-14) with band like headache. He is stressed in school?


Answer: tension headache

862.Child with white plaque?


Answer: oral thrush, candidase

Candidiasis — oropharyngeal candidiasis or thrush is a common local infection seen in young infants, older adults who wear
dentures, diabetics, patients treated with antibiotics, chemotherapy, or radiation therapy, and those with cellular immune
deficiency states, such as the acquired immunodeficiency syndrome (aids). Patients receiving inhaled glucocorticoids for asthma
or rhinitis are also subject to this complication.
Thrush can manifest in several ways. The pseudomembranous form is the most common and appears as white plaques on the
buccal mucosa, palate, tongue, or oropharynx. The atrophic form, which is often found under dentures, is characterized by
erythema without plaques. Candidiasis may also present with a beefy red tongue and associated soreness.

863.Multiple fractures in child?

193
Osteogenesis imperfecta, (oi) is a group of genetic disorders that mainly affect the bones. The term "osteogenesis imperfecta"
means imperfect bone formation. People with this condition have bones that break easily, often from mild trauma or with no
apparent cause. Multiple fractures are common, and in severe cases, can occur even before birth. Milder cases may involve only
a few fractures over a person's lifetime.

864.3 questions were about milestone


o 1st q the mentioned: tricycle (3 years see above)
o 2nd q: smile pull to sit (3 months – 4 months see above)
o 3rd q: language development.

865.Child mild persistent asthma visit er once every month he is on ventolin what to add to his medication?
Answer: low dose cs according to asthma ttt scale

866.Child with oral and tonsillar ulcers and vesical, fever. Dx:
Answer: herpangina

Herpangina is an acute febrile illness associated with small vesicular or ulcerative lesions on the posterior oropharyngeal
structures (enanthem). Herpangina typically occurs during the summer and usually develops in children, occasionally occurring
in newborns, adolescents, and young adults. Herpangina is one of many manifestations of enterovirus infection and can occur in
association with enteroviral exanthem, aseptic meningitis, encephalitis, acute flaccid paralysis, and other clinical syndromes.
Herpangina is a pharyngeal infection typically caused by various enteroviruses. In recent years, coxsackievirus a16, enterovirus
71, and coxsackievirus b have been implicated most often.

867.27 days neonate, with pale stool, high conjugate, diagnosed with neonatal jaundice and treated with fluorescence. What
is the cause?
Http://emedicine.medscape.com/article/974786-overview#a7

868.4 years old brought by his parents, height < 5th percentile, they ask if he will remain short. What you will do initially :
A) parental height
Answer : i think check somatmedin c

869.8 years old girl presented with fever, numerous bruises over the entire body and pain in both legs. Physical examination
reveals pallor and ecchymosis and petechiae on the face, trunk and extremities. Findings on complete blood count
includes a haemoglobin of 6.3 g/dl, white cell count of 2800/mm3 and platelet count of 29,000/mm3. Which of the
following would be the most appropriate treatment? Answer :

The hemophilias are a group of related bleeding disorders that most commonly are inherited. Inherited bleeding disorders
include abnormalities of coagulation factors and platelet function; the most common of these disorders is von willebrand
disease. However, when the term "hemophilia" is used, it most often refers to the following two disorders:
Factor viii deficiency (hemophilia a)
Factor ix deficiency (hemophilia b, also called christmas disease)
Https://yhdp.vn/uptodate/contents/mobipreview.htm?35/56/36746

870.Month old with wide anterior fontanel, large protruded tongue, diagnosis?
A. congenital adrenal hypothyroidism

194
Answer : congenital hypothyroidism (cretinism)

871.Child with oral and tonsillar ulcers and vesicles, fever. Dx:
A-herpangia
Answer : a

Though herpangina can be asymptomatic, symptoms usually associated are high fever and sore throat. A small number of
lesions (usually 2 - 6) form in the back area of the mouth, particularly the soft palate or tonsillar pillars. The lesions progress
initially from red macules to vesicles and lastly to ulcerations which can be 2 – 4 mm in size. The lesions heal in 7 – 10 days.

872.Child with delay in walking, on examination there is bowing in his legs, labs showing normal ca normal phosphor and
elevated alkaline phosphates, what is the diagnosis?
A- rickets.
Answer : a
Calcipenic rickets is often but not always associated with low serum calcium levels, while phosphopenic rickets is characterized
by low serum levels of phosphorus.
Early on in the course of rickets, the calcium (ionized fraction) is low. However, this level is often within the reference range at
the time of diagnosis,the phosphorus level is invariably low for age, unless recent partial treatment or recent exposure to
sunlight has occurred. Alkaline phosphatase levels are uniformly elevated.
Reference:
Http://www.uptodate.com/contents/overview-of-rickets-in-children?Source=outline_link&view=text&anchor=h1#h1
Http://emedicine.medscape.com/article/985510-workup#c8

873.Child with red bulging tympanic membrane, ear pushed down and forward, what is the dx?
Answer : mastoiditis - from lecture note

874.Child with generalized swelling, … long scenario of nephritic syndrome … you are suspecting minimal change
nephropathy, what you will find in the biopsy?
(options are long "2 lines long for each")
Answer: -ve em , focal fusion , loss of foot process

875.teenage girl presents with palpation, SOB, parasthesia and light headache, she failed in math test.
A. Hyperventilating syndrome.
Answer: A

876.Long case about child 2 months with respiratory symptoms his brother dies from pneumonia and her sister does not have
any medical hX. Investigations. Show T cell 0 low all IG Dx

877.Child with sickle cell, what is the lifelong treatment to prevent infections?
A- penicillin and immunization.

878.Child presented with cyanosis and murmur ( case of transition of great vessel)

879.Child with pain and swelling in his hands and foot (sickle cell disease),…. Forgot the question…..

880.Child with vesicles in his oral mucosa, what is the diagnosis?


A- herpes simplex type 1.

195
881.(3 scenarios about testicular torsion)

882.Child ride tricycle can't copy square what is the age


Answer: 3 years

Extra tables:

196
Reference: toronto notes

197
➢ Vaccinations

Reference: ministry of health

198
4/5
SMLE
KSAU-HS
Question Bank
1.10 Edition
OB-GYN Questions

This is an accumulative effort from King Saud bin Abdulaziz University for Health Sciences (2016-17/
Batch 9) interns to organize and answer what have been collected previously from SMLE Q Bank
2015-16

We would like to acknowledge:


- King Saud bin Abdulaziz University for Health Sciences (2016-17/Batch 9) interns for their huge efforts in
accomplishing this project
- SMLE Q Bank Group
‫ مدونة طالب طب سعودي‬-

‫جهد بشري قابل للخطأ والصواب‬


For any comments, kindly contact us at
SMLE2016.17@gmail.com

Highlighted in yellow are queried questions


Highlighted in red are some repeated questions

2
Obstetrics
&
Gynecology

3
1. Long scenario of pregnant in 1st trimester while chick up she had high blood pressure in next visit high blood pressure
but lower than the 1st visit, diagnosis?
A. Essential hypertension
B. Gestational hypertension
C. Chronic hypertension
D. Preeclampsia
E. Answer: C
Source: medscape http://emedicine.medscape.com/article/261435-overview#a7

2. A pregnant lady with pneumonia develops igg, what type of of immunity will the baby acquire?
A. Active natural
B. Active artificial
C. Passive natural
D. Passive artificial

Answer: C
Passive immunity is the transfer of active immunity, in the form of readymade antibodies, from one individual to another.
Passive immunity can occur naturally, when maternal antibodies are transferred to the fetus through the placenta, and can also
be induced artificially, when high levels of human (or horse) antibodies specific for a pathogen or toxin are transferred to non-
immune individuals.

3. Breastfeeding mother with HCV treated with interferon more than one year what the risk of breastfeeding on infant?
A. Nipple cracking
B. Mother with anemia
C. Infant complain of oral candidiasis
D. Not follow up of infant immunization
Answer: A
Hepatitis C is not transmitted through breast milk. However, the Centers for Disease Control recommends that mothers with
HCV infection should consider abstaining from breastfeeding if their nipples are cracked or bleeding
Reference: https://www.drugs.com/breastfeeding/interferon-alfa.html

4. A married lady presented with periumbilical abdominal pain, guarding, sever pain on rectal exam and low grade fever.
What is the most likely diagnosis?
A. Ovarian torsion
B. Ectopic pregnancy
C. Appendicitis
D. Cholecystitis
Answer: B
Explanation: In appendicitis there will be a clear discerption of shifting of pain mcburney’s point since all other symptoms are
similar?

5. Premature birth, the woman at risk for it if cervical effacement is?


A. 10mm
B. 20mm
C. 30mm
D. 40mm
Answer: < 25 mm. C?
USMLE step 2 CK (page 69): Risk factors of preterm labor: short trans-vaginal cervical length of < 25 mm
In OBGYN clinic: < 3 cm (30 mm)

4
6. Pap smear found epithelial cells
A. HIV
B. HPV
C. HSV1
D. HSV2

Answer: B
I think the question means dysplastic epithelial cells which are caused by HPV infection.

7. OCP effects on liver:


A. Hepatoma
B. Adenoma
C. HCC
D. Hepatic hyperplasia
Answer: A

8. Most common site of gonococcus in female:


A. Cervix
B. Urethra
C. Pharnix
D. Rectum
Answer: A

9. Pregnant lady with daily symptom of cough and wheeze and nocturnal once a week
She's on albuterol What's the management?
A. Short with inhaled steroid
B. Short with oral steroid
C. Long with inhaled steroid
D. Long with inhaled steroid again
Answer: A or B (Depends on stepwise management or severity)
Asthma
http://www.webmd.com/asthma/systemic-corticosteroids-for-quick-relief-during-asthma-attacks

10. Female after menarche at what age the bone will stop growing?
A. 6 months
B. 12 years
C. 24 years
D. 36 years
Answer: B
Girls will usually stop growing earlier than boys, around age 11 or 12.
Https://www.care.com/a/when-do-girls-stop-growing-20150731040419

11. Adenomyosis diagnosed by :


A. MRI
B. US
C. Endometrial specimen
D. Uterine biopsy by hysterectomy

5
12. Best antibiotic for breast feeding is?
A. Chloramphenicol
B. Azithromycin
C. Cimetidine
D. Ciprofloxacin

13. 48 Y Women with fibroid 5 or 6 cm asymptomatic:


A. Hysterectomy
B. Myomectomy
C. Regular follow up yearly
D. Follow up and CBC every 2 month

14. Multigravida in labor with 60% effacement and dilated cervix (5 cm). After 1 hour she still has 60% effacement but the
cervix dilates to 6 cm. What will you do for her?
A. Expectant management
B. Oxytocin
C. Cervix ripening
D. Artificial rupture of membranes
Answer: A Obs/gyne consultant.
Prolongation of active phase is diagnosed if cervical dilation is <1.2 cm/h in a primipara or <1.5 cm/h in a multipara. Arrest is
diagnosed if cervical dilation has not changed for >2 h. However, treatment is directed at assessment of uterine contraction
quality. If hypotonic give oxytocin.
Otherwise see the indications for induction of labor.

15. 42 years old female complaining of amenorrhea, night sweat and flushing for the last 6 months. What is the most likely
diagnosis?
A. Hypothyroid
B. Hyperprolactinemia
C. Congenital adrenal Hyperplasia
D. Pheochromocytoma
Answer: b

16. 21 years old Female with negative pap smear. You should advise her to repeat pap smear every:
A. 6 months
B. 12 months
C. 18 months
D. No repeat

Answer: 3 year
Cervical cancer screening guidelines:
Age <21: no screening REGARDLESS of sexual activity
Age 21: Start Pap test with cytology alone without HPV testing.

Frequency of screening:
Age 21-29: repeat Pap every 3 years (no HPV testing for this group)
Age 30-65: repeat Pap smear every 3 years OR repeat Pap every 5 years if both cytology and HPV testing (the latter is the
preferred method of screening in this age group)

Stop screening:
After age 65 if negative Pap smear for past 10 years AND no history of CIN 3 or more severe diagnosis.
Reference: American Cancer Society (ACS) guidelines, ACOG guidelines, also in Kaplan.
6
17. How ectopic pregnancy occurs at the cellular level?
A. Disappearance of zona pellucida.
B. Fertilization at ampulla tube.
C. Persistence of Zona pellucida.
D. Fast division of blastomere.
Answer: A
As cilia degenerate the amount of time it takes for the fertilized egg to reach the uterus will increase. The fertilized egg, if it
doesn't reach the uterus in time, will hatch from the non-adhesive zona pellucida and implant itself inside the fallopian tube,
thus causing the pregnancy.
Reference: Wikipedia and Clinical reproductive medicine & surgery book - textbook of clinical embryology
Alterations in molecular signaling between the oocyte and the implantation site is makes ectopic pregnancy more likely.
Reference: BMJ Best Practice
So probably A is the right answer

18. Nursing mom wants to conceive but not in the coming two years. What will you recommend for her?
A. Vaginal ring
B. Combined OCP
C. Progestin injection
D. Patch

Answer: C

19. A patient with ectopic pregnancy of 2.5*3.0 size. Hcg is 5000. The patient is stable. What will you do?
A. Wait and watch
B. Laparotomy
C. Laparoscopy
D. D & C

Answer: C
Reference: Toronto Notes OB24 + 25

20. A couple came to your clinic. They are trying to conceive for the last 3 months with no success. The girl had
appendectomy before marriage. She also has an aunt who is her uncle’s wife (not blood related) with down syndrome.
What should be done?
A. Try some more
B. Clomiphene
C. Laparoscopy
D. Semen analysis

Answer: A

21. A 50 years old lady came with signs and symptoms of menopause. What picture describes his report best?
A. Increased LH and FSH
B. Decreased FSH and LH
C. Increased FSH decrease LH
D. Increase LH decrease FSH

Answer: A

7
22. A long scenario of a lady with vaginal infection, has strawberry cervix. What is the organism?
A. Trachomatis
B. Bacterial vaginosis
C. Gonorrhea
D. Trichomonas vaginalis

Answer: D

23. What is the role of metformin in PCOS?


A. Decrease glucose level
B. Decrease insulin resistance
C. Anti-androgenic
D. Menstrual regulation
Answer: B
The hallmark mark of PCOS is insulin resistance.

24. Hormonal replacement therapy prevents which of the following?


A. Postmenopausal symptoms
B. Osteoporosis
C. Coronary artery disease
D. Stroke

Answer: A
Indications of HRT: primary indication is treatment of menopausal symptoms (short-term).
HRT is not used to treat osteoporosis, although if used they decrease risk of osteoporotic fractures
Reference: Kaplan

25. 46 years old woman comes with amenorrhea for 6 months and flushes at night that disturbs her sleep. What is the best
investigation to make your diagnosis?
A. LH
B. FSH
C. Estrogen
D. Progesterone
Answer: B

26. Premature menopause starts before...?


A. 30
B. 35
C. 40
D. 45
Answer: C

27. A pregnant lady with gestational diabetes. What medication will you prescribe for her?
A. Metformin
B. ...zide
C. ...zone
D. Insulin

Answer: D

8
28. A pregnant lady had a child with 3500 grams with the use of forceps, presented to you 20 days postpartum with whitish
vaginal discharge but with no itching or cervical tenderness. On examination cervix is pink. Microscopic examination
reveals epithelial cells with leukocytes. What would you do for your patient?
A. Dipstick urinalysis
B. Pelvic ultrasound
C. Reassure
D. Metronidazole
E. Culture discharge

Answer: C
Presence of epithelial cells and few numbers of wbcs is normal. Also, a normal discharge does not have an offensive odor and is
not associated with vaginal irritation, itching, or burning.
Reference: http://www.ncbi.nlm.nih.gov/books/NBK288/

29. A pregnant lady with a positive OGTT, what is your action?


A. Repeat the test
B. Check hga1c
C. Start insulin
D. Do a random blood glucose

Answer: C
One abnormal OGTT is enough to diagnose GDM and the treatment is diet and exercise. If diabetes is not controlled, insulin is
initiated.
Reference: BMJ Best Practice and Master the Boards

30. Pregnant G1P0 who has a history of travelling 1 year ago, came for check up. Result shows HIV +ve. What is the action in
this case ?
A. Acyclovir for the mother during 1 week.
B. (something) given to the baby after delivery.
C. (something) given to the mother and baby after delivery.
D. Acyclovir is contraindicated.

Answer: zidovudine for mother and baby


In an HIV-infected pregnant woman who has never been exposed to antiretroviral medication, HAART should be started as soon
as possible, including during the first trimester. Combination antiretroviral therapy should be offered in all cases. As zidovudine
(ZDV) is the only agent specifically shown to reduce perinatal transmission, it should be used whenever possible as part of the
highly active antiretroviral therapy (HAART) regimen.
All HIV-exposed infants should receive zidovudine.
Reference: http://emedicine.medscape.com/article/1385488-overview#showall

31. What is Adenomyosis?


A. Presence of endometrial tissue and gland in Uterine Ligament.
B. Presence of endometrial tissue and gland in Uterine Muscle.
C. Presence of endometrial tissue and gland in Cervix.
D. Presence of endometrial tissue and gland out Uterus.

Answer: B
It occurs when endometrial tissue, which normally lines the uterus, exists within and grows into the muscular wall of the uterus.

9
32. What is the most common sign and symptom in placental abruption?
A. Vaginal bleeding
B. Uterine tenderness
C. Uterine contractions
D. Fetal distress
Answer: A
Placental abruption is mainly a clinical diagnosis with all the above findings. The most common symptom is dark red vaginal
bleeding with pain during the third trimester of pregnancy (80%) and abdominal or uterine tenderness (70%).

Bleeding may occur at various times in pregnancy:


Bleeding in the first trimester of pregnancy is quite common and may be due to the following: miscarriage (pregnancy loss)
ectopic pregnancy (pregnancy in the fallopian tube)
Bleeding in late pregnancy (after about 20 weeks) may be due to the following: placenta previa or placental abruption.
Reference: alqassim Booklet. Q84, also BMJ Best Practice

33. (long scenario) female patient with bacterial vaginosis. What is the most appropriate treatment?
A. Ceftriaxone.
B. Clindamycin.
C. Ampicillin.
D. Fluconazole.

Answer: B
Refer to the table at the end of OB/GYN section

34. 34-year-old lady pregnant, complaining of amenorrhea, bleeding, and abdominal pain. B-hcg done showed levels of 1600,
she was given methotrexate. One week later she still has abdominal pain despite analgesia. B-hcg done showed 6000
units. What is the best management?
A. Continue methotrexate.
B. Exploratory laparoscopy.
C. Salpingectomy
D. Salpingostomy

Answer: B
35. A mother delivered her first baby with cleft lip and palate. What is the percentage of recurrence for her next pregnancy?
A. 1%
B. 4%
C. 15%
D. 20%
Answer: B
Reference: Handbook of Genetic Counseling/Cleft Lip and Palate

36. Pregnant female is HIV positive. What is the most likely mode of transmission to the baby?
A. Through the placental
B. Through the blood cord
C. By breast feeding
D. Through hand contamination of mother
Answer: C
Reference: http://www.prn.org/index.php/transmission/article/mother_to_child_hiv_transmission_296

01
37. What is the best place to take a cervical sample for Pap smear?
A. Endocervix Cancer,
B. Exocervix
C. Transformation
D. Vaginal vault
Answer: C
The transformation zone is the site of origin for most cervical neoplasia and should be the focus of cytology specimen collection.
References: http://www.cytopathologyear-old rg/specimen-collection-adequacy-requisition/
Https://books.google.com.sa/books?Id=0flwgd3ojlec&pg=PA11&lpg=PA11&dq=-#v=onepage&q&f=false

38. Pregnant lady, everything was normal except hemoglobin was low. What is the next step?
A. Iron
B. Nothing
C. Folate
D. B12

Answer: A
Iron supplementation is almost universally recommended during pregnancy to correct or prevent iron deficiency. Reference:
http://www.ncbi.nlm.nih.gov/pmc/articles/PMC1447059/

39. Cervical insufficiency: the canal is less than


A. 10 mm
B. 20 mm
C. C-30 mm
D. D- 40 mm
Answer: B
We make a diagnosis of cervical insufficiency in women with one or two prior second-trimester pregnancy losses or preterm
births and cervical length <25 mm on TVU examination or advanced cervical changes on physical examination before 24 weeks
of gestation. Risk factors for cervical insufficiency support the diagnosis.
Reference: http://www.uptodate.com/contents/cervical-
insufficiency?Source=search_result&search=cervical+insufficiency&selectedtitle=1%7E36

40. What is the best drug given to prevent postoperative thromboembolism?


A. LMW heparin
B. Uf heparin
C. Warfarin
D. Eno….
Answer: B
Unfractionated heparin (UFH) may be preferred if the patient is likely to have immediate surgery because of its shorter half-life
and reversibility with protamine compared with LMWH. (Medscape)

41. 30 years old female has 1 child ,Want to delay pregnancy 3 years later, she didn't want OCCP nor intravaginal Device.
Doctor advise her for transdermal patch, What is the best advice to tell the patient about the patch?
A. A- decrease compliance
B. B- increase blood clot
C. C- less effective than OCCP (same effect )
D. D- less skin complication (causes skin irritation )
Answer: B
Http://www.mayoclinic.org/tests-procedures/ortho-evra/basics/risks/prc-20013014

00
42. Best way to diagnose bacterial vaginosis:
A. Gram stain
B. Dark field microscopy
C. PCR
D. Culture

Answer: A
Gram's stain — Gram's stain of vaginal discharge is the gold standard for diagnosis of BV
BV can be diagnosed by the use of clinical criteria (i.e., Amsel’s Diagnostic Criteria)
Demonstration of clue cells on a saline smear is the most specific criterion for diagnosing BV.
Obtaining routine vaginal cultures in patients with BV has no utility, because this is a polymicrobial infection and some women
may have asymptomatic carriage of G vaginalis organisms.

Reference: http://emedicine.medscape.com/article/254342-workup http://www.cdc.gov/std/tg2015/bv.htm

43. Pregnant woman with significant edema in his hand and foot, Blood pressure 160/110, what will you do?

A. Give him diuretic


B. Low diet salt
C. Labetalol
D. Observation in hospital

Answer: C
If a pregnant woman's blood pressure is sustained greater than 160 mm Hg systolic and/or 110 mm Hg diastolic at any
time, lowering the blood pressure quickly with rapid-acting agents is indicated for maternal safety.
Anticonvulsant therapy may be undertaken in the setting of severe preeclampsia (primary prophylaxis) or in the setting
of eclamptic seizures (secondary prophylaxis). The most effective agent is IV magnesium sulfate; phenytoin is an
alternative, although less effective, therapy.
Labetalol has a more rapid onset of action, may be given orally or parenterally, and is generally preferred as a first-line
agent.
Reference: http://emedicine.medscape.com/article/261435-overview#a21

44. Pregnant lady everything normal except hemoglobin low, next step?
A. Iron
B. Nothing
C. Folate
D. B12
Answer: A
Iron deficiency anemia accounts for 75-95% of the cases of anemia in pregnant women. While folate deficiency is much less
common than iron deficiency. A woman who is pregnant often has insufficient iron stores to meet the demands of pregnancy.
Encourage pregnant women to supplement their diet with 60 mg of elemental iron daily. The clinical consequences of iron
deficiency anemia include preterm delivery, perinatal mortality, and postpartum depression. Fetal and neonatal consequences
include low birth weight and poor mental and psychomotor performance.
Reference: http://emedicine.medscape.com/article/261586-overview

45. Prolong labor. She might have post-partum hemorrhage, How to asses this patient?

A. Visual blood loss


B. Hematocrit count
C. Pulse
D. Hemoglobin level

02
Answer: Incomplete question (A or C)
Hemoglobin and hematocrit values remain unchanged from baseline immediately after acute blood loss.
PPH usually manifests with such rapidity that diagnostic procedures are almost entirely limited to a physical
examination of the involved structures.
Caregivers consistently underestimate visible blood loss by as much as 50%.
We make the diagnosis of PPH in postpartum women with bleeding that is greater than expected and causes symptoms
(eg, pallor, lightheadedness, weakness, palpitations, diaphoresis, restlessness, confusion, air hunger, syncope) and/or
results in signs of hypovolemia (eg, hypotension, tachycardia, oliguria, oxygen saturation <95 percent).
Diagnosis may be delayed in symptomatic women without heavy vaginal bleeding who are bleeding internally, such as
intra-abdominal bleeding related to a cesarean delivery or a broad ligament or vaginal hematoma due to a sulcus
laceration.

Reference: http://emedicine.medscape.com/article/275038-overview#a10
Http://emedicine.medscape.com/article/432650-workup

46. Pregnant woman in 3rd trimester have high blood glucose level despite close observation What is the suspect cause?

A. Neonate hyperglycemia
B. Neonate hypoglycemia
C. Mother hyperglycemia
D. Mother hypoglycemia

Answer: C
This woman complaining of gestational diabetes mellitus

Reference: http://www.medscape.com/viewarticle/710578

47. Pregnant on labor, on pelvic examination you fell the orbital margin and the nose, what is the presentation of this fetus?

A. Mento anterior
B. Mento posterior
C. Lateral mento-transvers
D. Medial mento-transvers

Answer: B
Face presentations are classified according to the position of the chin (mentum):
Left Mento-Anterior (LMA), Left Mento-Posterior (LMP), Left Mento-Transverse (LMT)
Mento anterior is the most common presentation.

48. Pregnant lady, had an outbreak asking for all vaccination can be given, what you will give?

A. Influenza
B. MMR
C. Rubella
D. Varicella

Answer: A
Two vaccines are routinely recommended during pregnancy:
o Flu (influenza) shot.
o Tetanus toxoid, reduced diphtheria toxoid and acellular pertussis (Tdap) vaccine
Certain vaccines should generally be avoided during pregnancy, including:
o Varicella (chickenpox)
03
o Human papillomavirus
o Measles, mumps and rubella
o Zoster
Reference: http://www.mayoclinic.org/healthy-lifestyle/pregnancy-week-by-week/expert-answers/vaccines-during-
pregnancy/faq-20057799

49. A G3P0 female with 3 recurrent abortions, on investigating the last abortion you found an aneuploidy 45X. What are the
chances of having this abnormality in her next pregnancies?
A. 30%
B. 40%
C. 50%
D. 60%

Answer: A
04
Turner syndrome (45X) is frequently observed and is the most common chromosomal abnormality observed in spontaneous
abortions. Turner syndrome accounts for 20-25% of cytogenetically abnormal abortuses
(http://emedicine.medscape.com/article/260495-overview#a5)

50. Pregnant Women with hypotension and low platelet what is type of anesthesia you prefer ?
A. Epidural
B. Pudendal block
C. Spinal
D. General
Answer: D
General anesthesia is indicated for maternal with medical condition like thrombocytopenia. While one of the complications of
epidural and spinal is hypotension. (Kaplan page 124)

51. MMR vaccine in breastfeeding : -


A. Harm to baby
B. Safe to baby
C. Delayed feeding 72 hours
D. Live attended vaccine
Answer A & D
According to Centers for Disease Control and Prevention, the MMR vaccine is live attenuated vaccine and also it is safe for baby.
(http://www.cdc.gov/vaccines/vpd-vac/measles/faqs-dis-vac-risks.htm)

52. Female pregnant with twins , in her 34 week ( she had some complication I forget it ) the doctor said that she needs
emergency CS cuc the presentation of twin A might result in fetal complications , what is the presentation of twin
A. Twin Cephalic- breech
B. Transverse-cephalic
C. Breech-cephalic
D. Cephalic-cephalic
Answer C
In general, if the first (presenting) twin is in the cephalic (vertex) presentation, labor is allowed to progress to vaginal delivery,
whereas if the presenting twin is in a position other than cephalic, cesarean delivery is often performed.
Twin gestations in which the first twin is in the breech presentation (20% of all twin deliveries) are most often delivered via
cesarean delivery (Obstetrics and Gynecology, sixth edition, Charles R. B. Beckmann, page 186-187)

53. Female pregnant in her 24 week , came to you for her first prenatal visit , lab are provided all were normal except that
the Hgb is slightly low ( I think it was 10) , what will you do ?
A. Nothing
B. Folate
C. Vit B
D. Iron
Answer D
Anemia in pregnancy is generally defined as an Hct less than 30% or a hemoglobin of less than 10 g/dl. (Obstetrics and
Gynecology, sixth edition, Charles R. B. Beckmann, page 151)
WHO recommendations:
Daily oral iron (30-60mg) and folic acid (0.4 mg) supplementation is recommended as part of the antenatal care to
reduce the risk of low birth weight, maternal anaemia and iron deficiency.
In settings where anemia in pregnant women is a severe public health problem (40% of higher), a daily dose of 60 mg of
elemental iron is preferred over a lower dose.

05
If a woman is diagnosed with anemia in a clinical setting, she should be treated with daily iron (120 mg of elemental
iron) and folic acid (400 μg or 0.4 mg) supplementation until her haemoglobin concentration rises to normal (1, 2). She
can then switch to the standard antenatal dose to prevent recurrence of anemia.
(http://www.who.int/elena/titles/guidance_summaries/daily_iron_pregnancy/en/)

54. Patient with Invasive Cervical Cancer, you want to stage her cancer, which of the following tests you should perform?
A. Proctoscopy, colonoscopy, hysteroscopy
B. Proctoscopy, cystoscopy, hysteroscopy
C. Proctoscopy, gastroscopy, hysteroscopy
D. Proctoscopy, laparoscopy, hysteroscopy
Answer: B. Proctoscopy, cystoscopy, hysteroscopy

Accordig to “FIGO” staging of cervical cancer:


Stage I is carcinoma strictly confined to the cervix; extension to the uterine corpus should be disregarded.
Stage IA: Invasive cancer identified only microscopically. All gross lesions even with superficial invasion are stage IB
cancers.
Stage IB: Clinical lesions confined to the cervix
(Obstetrics and Gynecology, sixth edition, Charles R. B. Beckmann, page 385)
Stage 2, 3, & 4 is extend not invasive.
Based on the above we need to be sure the invasive cervical cancer is confined to cervix by examine the close organ to it so, I
choose option “B”

55. Single Female came to your clinic one day after condom rupture during vaginal intercourse, she is worried about
becoming pregnant. What you will do?
A. A.Pregnancy test
B. B.Wait and arrange for appointment after one week
C. C.Give post-coital contraception
D. D.Give progesterone only contraception
Answer: C
EMERGENCY CONTRACEPTION
• hormonal EC (Yuzpe® or Plan B®, usually 2 doses taken 12 h apart) or post-coital IUD insertion
• hormonal EC is effective if taken within 72 h of unprotected intercourse (reduces chance
Of pregnancy by 75-85%), most effective if taken within 24 h, does not affect an established
Pregnancy
• post-coital iuds inserted within 5 d of unprotected intercourse are significantly more effective
Than hormonal EC (reduces chance of pregnancy by ~99%)
*Yuzpe® method = 98% (within 24 h), decreases by 30% at 72 h

*“Plan B” levonorgestrel only= 98% (within 24 h), decreases by 70% at 72 h

56. Postpartum patient with bilateral breast engorgement and tenderness, what is the management?

A. Hot compression and continue breastfeed


B. Cold compression and stop breastfeed
C. Oral Dicloxacillin and continue breastfeed
D. Oral Dicloxacillin and pump the milk into bag and discard it
Answer: A. Hot compression and continue breastfeed

If engorgement is making it hard to breast-feed, use the following steps. They can relieve your symptoms and keep your milk
flowing.

06
Soften your breasts before feedings. You can apply a warm compress for a couple of minutes before you breast-feed.
Or you can use your hands or use a pump to let out (express) a small amount of milk from both breasts.
Try to breast-feed more often. Pump your breasts if your baby won't breast-feed. Take care to empty your breasts each
time.
Take ibuprofen (such as Advil or Motrin) to reduce pain and swelling. Ibuprofen is safe for breast-feeding moms when
taken as directed. But it's a good idea to check with your doctor before you take any kind of medicine while breast-
feeding.
If your breasts still feel uncomfortable after nursing, try a cold compress to reduce swelling. You can use a frozen wet
towel, a cold pack, or a bag of frozen vegetables. Apply it to your breasts for 15 minutes at a time every hour as
needed. To prevent damage to your skin, place a thin cloth between your breast and the cold pack.
If you are not breast-feeding, use one or more of these steps to relieve discomfort:
Do not pump or remove a lot of milk from your breasts. If your breasts are very painful, it's okay to remove just a little
bit to make you more comfortable.
Apply a cold pack to your breasts for 15 minutes at a time every hour as needed. To prevent damage to your skin, place
a thin cloth between your breast and the cold pack.
Take ibuprofen (such as Advil or Motrin) in addition to using non-medicine treatments. Be safe with ibuprofen. Read
and follow all instructions on the label.
Wear a bra that fits well and provides good support.
(http://www.webmd.com/parenting/baby/tc/breast-engorgement-overview?Page=2)

57. Increase in frequency of muenst:


A. Metrorrhagia
B. Hypermenorrhea
C. Metrorrhagia
D. D)Polymenorrhea
Answer:D
Hypermenorrhea : abnormally heavy or prolonged menstruation; can be a symptom of uterine tumors and can lead to anemia if
prolonged
Metrorrhagia: is uterine bleeding at irregular intervals, particularly between the expected menstrual periods
Polymenorrhea is the medical term for cycles with intervals of 21 days or fewer.
Oligomenorrhea is the medical term for infrequent, often light menstrual periods (intervals exceeding 35 days).
Menometrorrhagia (meno = prolonged, metro = uterine, rrhagia = excessive flow/discharge) may be diagnosed. Causes may be
due to abnormal blood clotting, disruption of normal hormonal regulation of periods or disorders of the endometrial lining of
the uterus

58. Patient pelvic inflammatory d with salpingitis , On ceftriaxone and no improvement , What is the cause?!
A. N gonorrhea
B. Chlamydia
C. Adenovirus
D. Herps
Answer: B

The organisms most commonly isolated in cases of acute PID are N gonorrhoeae and C trachomatis. Ceftriaxone is used for
Neisseria Tx.
Reference : http://emedicine.medscape.com/article/256448-medication#3
59. Best time to check chorionicity and amnionicity of twins?
A. Early 2nd trimester*
B. Late 2nd trimester
C. Early 3rd trimester
D. Late 3rd trimester
Answer : A
It is easiest to determine chorionicity and amnionicity in the first trimester.
Reference ; http://radiopaedia.org/articles/twin-pregnancy-1
07
60. Seven weeks pregnant lady c/o vaginal bleeding with tissue. Her cervix was open and you can see some product
of conception. Her fundal height is equal to 7 to 8 weeks.
A. Threatened abortion
B. Incomplete abortion✅
C. Missed abortion
D. Molar pregnancy
Answer : B (Incomplete abortion is a pregnancy that is associated with vaginal bleeding, dilatation of the cervical canal, and
passage of products of conception. Usually, the cramps are intense, and the vaginal bleeding is heavy. Patients may describe
passage of tissue, or the examiner may observe evidence of tissue passage within the vagina. Ultrasound may show that some of
the products of conception are still present in the uterus.) Medscape.

61. When does a pregnant patient do GDM ?


A. 12 weeks
B. 16 weeks
C. 20 weeks
D. 28 weeks
Answer : D

24-28 weeks for GDM screening (Medscape)

62. Definition of postpartum hemorrhage:


A. More than 500 ml post SVD
B. Less than 500 ml post CS
C. More than 500 ml post CS
D. Less than 500 ml post SVD
Answer : A
Postpartum hemorrhage is bleeding of 500 ml post spontaneous vaginal delivery and 1000 ml post CS

63. Nullipara with diabetes gestational diet. Normal contraction. During labor she full extension and one
nurse push the baby from fundus and the other nurse push the above symphysis pubis with no labor
thin the doctor do episiotomy. What response for delayed labor
A. Full extension
B. Pushing the fundus
C. Pushing symphysis pubic
D. Episiotomy
Answer: B

64. Fibroid was found in a healthy asymptomatic 52 year old woman, it was 5x6cm, what will you do?
A. Follow up every two months with ultrasound and CBC
B. Follow up regularly
C. Immediate myomectomy
D. Immediate hysterectomy
Answer: B
Conservative treatment if: minimal or asymptomatic, <6-8 cm or stable in size, not submucosal, currently pregnant. Toronto
We perform annual pelvic exams and, in patients with anemia or menorrhagia, check a complete blood count. Uptodate

65. Mother after ROM came to ER the patient give history of herpes infection 2 years back OE doctor see inactive H.
Simplex what will do:
A. Cs
B. Instrumental delivery
C. Sterile speculum examination
D. Give acyclovir
Answer:

08
It has been recommended that a cesarean section should be performed if active lesions are present at the onset of
labour
Http://www.ncbi.nlm.nih.gov/pmc/articles/PMC2094935/
Http://www.hindawi.com/journals/criog/2015/198547/

66. How to assess progress in labour:


A. Severity of uterine contraction
B. Descending of presenting part
C. Molding of head
D. Duration of contraction :
Answer:b
Vaginal examinations have become a routine intervention in labour as a means of assessing labour progress( affecment ,
dilatation,desnding of baby)

67. 26 year old female with three months history of bilateral clear fluid coming out of her breasts, it was obvious on
physical examination. Normal menstrual cycle. What investigation you would like to do?
A. Neuroimaging.
B. Mammogram.
C. Prolactin assay.
D. ACTH.
Answer: c

68. The most accurate diagnostic investigation For ectopic pregnancy:


A. Culdocentesis
B. Pelvic U/S
C. Endometrial biopsy
D. Serial B-HCG
E. Laparoscopy
Answer:
Combined transvaginal ultrasonography and serial quantitative beta-hcg measurements are approximately 96 percent
sensitive and 97 percent specific for diagnosing ectopic pregnancy. Therefore, transvaginal ultrasonography followed by
quantitative beta-hcg testing is the optimal and most cost-effective strategy for diagnosing ectopic pregnancy.
Http://www.aafp.org/afp/2005/1101/p1707.html

69. 24 years old G1P0 , she has gestational diabetes which is controled by diet only, and no other medical problems.
She is in the 2nd stage of labor which last more than 2 hours, normal uterine contractions, baby's head comes
down with eatch contraction and go back when uterus is relaxed, the mother's hip is maximally flexed, one
nurse is apllying suprapubic pressure, other nurse applying fundus pressure, the doctor decided to d o
episiotomy and deliver the posterior shoulder.
Which of the following will cause delay in delivery?

A. Suprapubec pressure
B. B ) fundus pressure
C. Hip flexion
D. Delivery of posterior shoulder.
Answer:b
70. G8P7 in 36 weeks of gestation, with past medical history of postpartum hemorrhage in each previous delivery that
required blood transfusion.
What should be done regarding this delivery?
A. Deliver patient in OR
B. Perform CS at 38 weeks
C. Give patient IV fluid before delivery to compensate for any hemorrhage that may happen
D. Perform active management of 3rd stage golabor

09
Answer: d

Third stage of labor: from the delivery of fetus till the delivery of placenta
Active management of the third stage: (1) Uterotonic medication administered within one minute after delivery of baby after
ruling out presence of another fetus; (2) controlled umbilical cord traction and counter traction to support the uterus until
separation and delivery of the placenta; (3) uterine massage after delivery of the placenta

The best preventive strategy is active management of the third stage of labor
Refrences:
Http://www.aafp.org/afp/2007/0315/p875.html
Http://www.acog.org/-/media/Districts/District-VIII/activemanagementthirdstagelabor.pdf?Dmc=1

71. What is the most common cause for postpartum hemorrhage?


A. Uterine atony
B. Multiparity
C. Multiple gestation
D. Macrosomia
Answer:a
Reference : toronto notes

72. A girl with bilateral ovarian abscess with fever ?


A. Immediate laparotomy
B. Laparoscopic
C. Trans us drain
D. Antibiotic
Answer:
Hospitalization, IV antibiotics, if not improved after 48 hr:
First-line therapy typically uses a second generation cephalosporin with anti-anaerobic activity and it is still inconclusive whether
additional anti-anaerobic coverage is needed above and beyond the second-generation cephalosporing, cefoxitin or cefotetan,
plus doxycycline , proceed to trans US drain.
Uptodate

73. Self breast examination decrease breast cancer by years ?


A. 1 year
B. 2year
C. 3 year
D. 4year
Answer:
Recommend CBE be performed at least every 3 years starting between ages 20 and 39 and annually starting at age 40. The
ACOG,[14] ACR,[28]and AMA[29] recommend starting CBE at age 40 and annually thereafter.
Ref: Medscape

74. Sign of fetal distress?


A. Blood loss
B. Early decelerations
C. Late decelerations*
D. Various decelerations
Answer ; C

21
75. Mother G2 p 1 with Rh + and father - the last baby + and what is % the baby will have - :
A. 50%
B. 25%
C. 100%
D. 0%"
Answer: A , it is an AD

76. 34 yo Female after examination with Pap smear you found ASCUS what is your next step:
A. Cone biopsy
B. Colposcopy
C. Repeat later
D. Do HPV test
Answer: D/B
HPV DNA testing is indicated with an atypical Pap smear (ASCUS - atypical squamous cells of undetermined significance).
BMG best practice

77. Gynecologist see atypical invasive cell on colposcopy , Otherwise the patient is fine . What is the initial
management for her?
A. Clinical staging
B. Conization
C. And ?
D. Surgical
Answer: B ?

78. A patient with blood only noticed when she wipes with toilet paper, where is the bleeding ?
A. Urethra
B. Vulva
C. Uterine body
D. Uterine cervix
Answer : B?

20
79. Vulvar carcinoma will present as a unifocal, ulcerative and lesion in :
A. Labia Majora
B. Clitoris
C. Mons pubis
D. Perineum
Answer: A

80. Multipara pregnant. Medically free.. She is term.. Effacement 90% .. 4 cm.. Regular contractions.. Spontaneous
rupture of membrane.. Suddenly baby is stress from 140 to 80 beats.. Which type of anesthesia would you use:
A. General anesthesia
B. Narcotics
C. Pudendal n
D. Upper utrian?
Answer:

81. Girl with bilateral overian abscess with fever ?


A. Immediate laprotomy
B. Laproscopic
C. Trans us drain
D. Antibiotic
Answer: Q encommlene
Http://www.uptodate.com/contents/management-and-complications-of-tuboovarian-
abscess?Source=outline_link&view=text&anchor=H26297647#H26297647

82. 10 week pregnant with DM Nephropathy and HTN. BLOOD PRESSURE is high (162/141) and +3 protein in urine.
What to do?
A. ACEI
B. Bed rest
C. Termination of pregnancy
D. Observation
Answer ??:
ACE inhibitors have been designated by the FDA as category D drugs -- meaning that they carry known fetal risks
http://www.uptodate.com/contents/pregnancy-in-women-with-diabetic-kidney-
disease?Source=outline_link&view=text&anchor=H11#H11

83. Long case Pregnant lady -almost at labour I think-with breech presentation . Face flexed, the lichoer is fair the
baby found to be small and the pelvis of mother has ??? Somthing I forgot but sure they didn't mention the type
of pelvis
Which of the following will prevent you from trying Ecv ?
A. Small baby
B. Fair lichor
C. Pelvic
D. Flexed face of baby
Answer :
*Absolute contraindication to external cephalic version :
If cesarean delivery is indicated for reasons other than breech presentation
Placenta previa or abruptio placentae
Nonreassuring fetal status,
Intrauterine growth restriction in association with abnormal umbilical artery Doppler index
5-isoimmunization
6-severe preeclampsia
7-recent vaginal bleeding
8-significant fetal or uterine anomalies
22
9-ruptured membranes,
10- fetus with a hyperextended head,
11- multiple gestations
Relative contraindications:
Relative contraindications include maternal obesity, small for gestational age fetus (less than 10%), and
[24]
oligohydramnios because they make successful ECV less likely. Previous uterine scar from cesarean delivery or
myomectomy may also be a relative contraindication for ECV.

84. 14 ys girl menarche at age of 12 she suffering from sever dysmenorrhea with normal amount what is the appropriate
action :
A- NSAID
B- acetaminophen
C- OCP
D- progesterone
Answer:a
Most patients with primary dysmenorrhea show subjective improvement with NSAID treatment ( first choice).
Primary dysmenorrhea: Menstrual pain in absence of organic disease begins 6 mo-2 yr after menarche (once ovulatory
cycles established)
References:
Toronto notes

85. Pregnant female in her 34 week , cervix is affect 80% and 1 cm dilated , fetal position I is + 1 , what type of Anastasia will
give ?
A-Pedundale nerve block
B-GA
C-Narcotic
D-Epidural
Answer:c

86. 62 years old came with vaginal bleeding. What is the most common benign cause of bleeding in this age?
A. Cervical erosion
B. Cervical polyps
C. Atrophic vaginitis
D. Endometrial Hyperplasia
Answer: C
But you need rule out other serious causes. Such as uterine cancer
Http://www.uptodate.com/contents/postmenopausal-uterine-bleeding

87. Pregnant at 1st trimester, which is of the following medication is contraindication;


A. Erythromycin
B. Nitrofurantoin
C. Tetracycline
D. Gentamicin
Answer: C
It causes abnormal tooth discoloration off the baby.
Http://www.ncbi.nlm.nih.gov/pubmed/1124176
88. Women in phenobarbital what you will do while breastfeeding:
A. Stop drug
B. Continue drug
C. Ween child 3 week before starting breastfeeding.
D. Stop it one month before starting breastfeeding.
Answer: B

23
During breastfeeding the baby will, however, continue to be exposed to the AED in varying concentrations depending
on the prescribed AED. If mothers receiving ethosuximide, phenobarbital or primidone choose to breastfeed, they
should exercise caution and closely monitor the infant for sedation, lethargy and any significant clinical findings.
(http://www.medscape.com/viewarticle/530483_7)

89. 43 year old female with irregular menses 3 months back & 1-2 days spotting, what next to do next ?
a. FSH
b. LH
c. HCG
d. US

Answer: A

90. 45 yrs female came to the primary healthcare and found to have this result
Hga1c > 7.8, Random blood sugar ( elevated )
** labs indicating that she has diabetes ** what labs you will request:
A. LFT
B. Fasting blood sugar
C. Full blood count
D. Urinalysis
Answer: D

91. Post cautery now complain if post coital bleeding. What is the source of bleeding?
A. Vulva.
B. Vagina.
C. Uterine cervix.
D. Uterine body
Answer:C

92. Women deliver baby (down syndrome) and she want to know about future pregnancy?
A. Aminocenthesis in next pregnancy
B. Keryotype of infant
C. Keryotype infant and mother
D. U/s in next pregnancy
Answer: B
Recurrence rate depends on type of down syndrome which is determined by karyotyping

93. Lady on cervical exam you visualize mass 00*00 cm what to do?
A. Cone biopsy
B. Excision
C. Reassure
D. Take sample for histopathology (or something like this )
Answer: D

94. A 43 year-old female undergone for a mammogram, and the result was negative, when do you advise her to get tested
again:
A. After 1 year.?
B. After 3 year.
C. After 5 year.
D. Other choices I cannot remember.
Answer: A

24
95. DVT in a pregnant woman what to do ?
a. Duplex U/S +bed rest +LMWH(low molecular weight heparin)
b. Venography +bed rest +LMWH(low molecular weight heparin)
c. Plethysmography +bed rest +LMWH(low molecular weight heparin)
d. Plethysmography +bed rest +LMWH(low molecular weight heparin)+warfarin

Answer: A
Http://emedicine.medscape.com/article/2056380-workup

96. A lady delivered a macrosomic baby, what is the reliable method of diagnosing postpartum hemorrhage?
A. Visual assessment of blood loss
B. Maternal pulse

C. Hemoglobin

D. Creatinine
Answer: B
Important organ systems to assess include the cardiovascular (heart murmur, tachycardia, strength of peripheral
pulses) the pulmonary system (evidence of pulmonary edema), and neurological systems (mental status changes from
hypovolemia).
The hemoglobin and hematocrit are helpful in estimating blood losses. However, in a patient with acute hemorrhage,
several hours may pass before these levels change to reflect the blood loss and platelet count.
Http://emedicine.medscape.com/article/796785-workup
Http://emedicine.medscape.com/article/796785-clinical#b4

97. A 42-year-old female complaining of amenorrhea, night sweat and flushing for the last 6 months. What is the most likely
diagnosis?
A. Hypothyroid
B. Hypoprolactinemia >> new choices
C. Congenital adrenal Hyperplasia
D. Pheochromocytoma
Answer: A
Primary hypothyroidism will lead to decreased level of T3, T4 and increase level of TSH. TSH and FSH both of them have
the same alpha unit, so when level of TSH is increased it will go and attach to FSH receptor and will work like FSH, this
will lead to appear of menopause symptoms.

98. When does a pregnant patient do GDM ?


A. 12 weeks
B. 16 weeks
C. 20 weeks
D. 28 weeks
Answer: D (24 -28 weeks)
Source: toronto notes 2014 OB6

99. Definition of postpartum hemorrhage:


A. More than 500 ml post SVD
B. Less than 500 ml post CS
C. More than 500 ml post CS
D. Less than 500 ml post SVD
Answer: A
Postpartum hemorrhage: loss of >500 ml post SVD or >1000 ml post CS
Source: Toronto notes 2014 OB 48

25
100.G5P5 women , has chlymedia and herpes , on examination she has cervical dysplasia , what is the most likely cause :
A- HPV
B- herpes
C- chlymedia
D- multiparity

Answer: A
Explanation: Worldwide, the human papillomavirus (HPV) has been detected in more than 90% of cervical carcinomas
and in as many as 99.7% of cervical neoplasias. Almost all precancerous and cancer lesions are associated with long-
term, persistent HPV infection.
Reference: http://emedicine.medscape.com/article/1618870-overview#a2

101.Pregnant women her amniotic fluid >2000 , atresia on which part cause that :
A- renal
B- ureteric
C- tracheal
D- esophageal

Answer: D
Explanation: Fetal anomalies, including esophageal atresia (usually associated with a tracheoesophageal fistula),
tracheal agenesis, duodenal atresia, and other intestinal atresias. Renal agenesis is associated with oligohydromnios
Reference: http://reference.medscape.com/article/975821-clinical#b5

102.Which of these agents can aggrevate the primary dysmenorrhea?


A- copper iud
B- levo IUD
C- magnesum
D- CCB
Answer: A
Explanation: Copper intra-uterine devices may cause painful and heavy periods.
Reference: http://www.merckmanuals.com/professional/gynecology-and-obstetrics/menstrual-abnormalities/dysmenorrhea

103.Hydrops fetalis in thalasemia case:


a. Normal 2 beta abnormal 4 alpha
b. Abnormal 2 beta normal 4 alpha
c. Normal 4 beta abnormal 2 alpha
d. Abnormal 4 beta normal 2 alpha
Answer: a

Master the board.

26
104.A pregnant lady in labor. Multipara and gravida, after presentation to you she had spontaneous rapture of amniotic
membrane. In examination she is 5 cm dilated with 100% effacement of the cervix with station zero . After three hours
still the same and no change. What is your management:
A. Expectant management
B. Oxytocin
C. Prostaglandin E2(or I2 not sure)
D. CS

Answer:b
Http://www.druglib.com/druginfo/syntocinon/indications_dosage/

105.After vaginal delivery the patient is complaining of urine coming out of the vagina during the micturition what is your
diagnosis:
A. A.viscovaginal fistula
B. B.urterovaginal fistula
C. C.uretherovainal fistula
D. D.rectovaginal fistula

Answer: c
Continuous inconteinence = vesico or uretero
Just during micturition URTHERO
Http://www.urology-textbook.com/urethrovaginal-fistula.html

106.Women in phenobarbital what you will do while breastfeeding:


A. Stop drug
B. Continue drug
C. Ween child 3 week before starting breastfeeding.
D. Stop it one month before starting breastfeeding.

Answer: B
Breastfeeding is a viable option for women with epilepsy who are being treated with aeds. Caution and clinical
monitoring should be exercised if the mother is using phenobarbital, primidone, ethosuximide or lamotrigine. If
mothers receiving ethosuximide, phenobarbital or primidone choose to breastfeed, they should exercise caution and
closely monitor the infant for sedation, lethargy and any significant clinical findings.
Http://www.medscape.com/viewarticle/530483_7

107.Women after hystrectomy she will receive :


A. Continuos estrogen and progesterone
B. Cyclic estrogen and progestron
C. Levonorgestrel
D. Estrogen only

Answer : D
Treating menopausal symptoms with estrogen alone is known as estrogen therapy (ET). ET improves the symptoms of
menopause, but it increases the risk of cancer of the uterus (endometrial cancer). Because of this, ET is only safe for
women who don’n have a uterus (such as those who have had a hysterectomy).
Http://www.cancer.org/cancer/cancercauses/othercarcinogens/medicaltreatments/menopausal-hormone-replacement-
therapy-and-cancer-risk

27
108.Best way to diagnose bacterial vaginosis:
A. Gram stain
B. Dark field microscopy
C. PCR
D.culture
Answer: A
Gram's stain — Gram's stain of vaginal discharge is the gold standard for diagnosis of BV
Http://www.uptodate.com/contents/bacterial-vaginosis#H5

109.Women post intercourse bleeding:


A. Uterine cervix
B. Uterine body
C. Valve
D. Vagina

Answer:- In premenopausal women, the source of vaginal bleeding after sex is usually the cervix. In postmenopausal women,
vaginal bleeding after sex may arise from the opening of the bladder (urethral meatus), the outer opening of the
vagina (labia) or the uterus, as well as the cervix.
Refrence: mayoclinic
Http://www.mayoclinic.org/symptoms/bleeding-after-vaginal-sex/basics/causes/sym-20050716

110.A woman who had spontaneous rupture of membranes came to the hospital stating that the fluid that came out was
clear. O/E her temp. 38.4 c and there's Pain score was 8 out of 10. On palpation of uterus when not in contraction, there's
tenderness. How to manage?
a. Give antipyretic
b. Give antibiotics while in labor
c. Don't do anesthesia
d. Do immediate CS.

Answer:
Once the decision to manage a patient expectantly has been made, the institution of broad-spectrum antibiotics should
be considered.
However if there is no sign of infection the management of PROM depends on the gestational age (go back to Kaplan
notes)
Reference: http://emedicine.medscape.com/article/261137-overview#a7 and Kaplan

111.CML associated with which translocation :


A- t (14; 18)
B- t ( 11;14)
C- t (8; 14)
D- t (9; 22).
Answer : D
CML is almost invariably associated with an abnormal chromosome 22 known as the Philadelphia chromosome, often
abbreviated as Ph, Ph(1), or Ph1[1,2]. The Philadelphia chromosome t(9;22)(q34;q11) results in the formation of a
unique gene product (BCR-ABL1)
112.A pregnant women exposed to MMR and she is Not vaccinated against MMR before ...what you have to give her ?
a. MMR vaccine
b. Immunoglobulin
c. No treatment.
d. IV immunoglobulin

Answer: D
28
MMR vaccine should not be given to a pregnant lady.
Reference: http://www.cdc.gov/vaccines/pubs/preg-guide.html

113.A women came with vaginal discharge and strawberry cervix and blood spots and other details what is the organism ?
a. Chlamydia trachomatis
b. Gonorrhea infections
c. Trichomonas
d. Bacterial vaginosis
Answer C

Reference: Kaplan USMLE Step 2 OBGYN

114.Women came with pruritus and itching … no discharge ,culture is negative , what to do
A. Give empirical therapy AB
B. Reassurance and no follow up
C. Reevaluation when symptoms come with consideration other inflammatory process
D. Referral to STD clinic
Answer: C

Reference: http://www.merckmanuals.com/professional/gynecology-and-obstetrics/symptoms-of-gynecologic-
disorders/vaginal-itching-and-discharge

115.Vaginal discharge with fishy odor and other details what is the treatment ?
A. Fluconazole
B. Metronidazole
C. Ampicillin
D. Ceftriaxone
Answer: B

This is bacterial vaginosis ( Clue cells, fishy odor and PH>4.5). Metronidazole or clindamycin are used to treat BV.
Metronidazole is safe in pregnancy.
Reference: Kaplan USMLE Step 2 OBGYN

116.Pregnant came for evaluation, TSH was low what are you going to give her:
a. Methinazole
a. Ptu
b. Radio active iodine
c. Thyroidectomy

Answer: A or B (depending on the trimester)


29
Methinazole and ptu can be used in pregnancy but Methinazole should not be used in the first trimester. PTU has a risk
of developing liver failure (rare) thus is should be limited to the first trimester. Radio active iodine is contraindicated in
pregnancy. Thyroidectomy could be used after the failure of medical therapy.
Reference: Kaplan USMLE Step 2 OBGYN

117.A patient with breast mass the become large with menses came to you asking for diagnosis FNA shows yellowish
secretion and it disappear after it what is diagnosis :
a. Galactocele
b. Ductasia [Duct ectasia]
c. Normal variant
d. Anovulatory
Answer: ?

118.Pregnant , full term, fully dilated , station +2 ,cephalic presentation, well rotation of the head, fetal bradycardia:
A. Ventouse
B. CS
C. Forceps
D. Nothing

Answer:A and C
Both of them can be used
Reference: Kaplan USMLE Step 2 OBGYN

119.RLQ pain 9 out of 10 , mass felt tender on examination , US mass 6cm:


a. Ectopic pregnancy
b. Cystic rupture
c. Appendicitis
d. Cystic torsion
Answer: D

Sudden and severe lower abdominal pain associated with adnexal mass is presumptive evidence of ovarian torsion

Reference: Kaplan USMLE Step 2 OBGYN

120.Women in her 30s , multipara 40 week with breach presentation ECV done 2 weeks ago , amniotic fluid index 12, now the
baby is on lateral position , why it is contraindicated ECV ?
A. Age of the patient
B. Previous ECV
C. Her AFI
D. Position of the fetus
Answer:?
ECV: repositioning of fetus within uterus under U/S guidance. Contraindications: previous T3 bleed, prior classical C/S, previous
myomectomy, oligohydramnios, PROM, placenta previa, abnormal U/S, suspected IUGR, HTN, uteroplacental insufficiency,
nuchal cord.
In this case there is no contraindication to repeat the ECV
Reference:Medscape and Toronto notes

121.Which of these cardiac diseases female fit for pregnancy:


a. MS 1cm
b. Sever MR
c. Eisenmenger

31
d. 20% ejection fracture
Answer: A
The most common acquired lesion in pregnancy is rheumatic heart disease and the most common of which is MS
Reference: Kaplan USMLE Step 2 OBGYN

122.What is the best antibiotic for breast feeding is?


A. Chloramphenicol → enters breast milk; discontinue drug or do not nurse
B. Azithromycin →Unknown whether drug is excreted into breast milk; use with caution
C. Cimetidine → not antibiotic & does enter breast milk
D. Ciprofloxacin → drug enters breast milk; use not recommended (American Academy of Pediatrics Committee states
that drug is compatible with nursing

Answer: None of the choices is “SAFE”. There could have been a 4th choice instead of cimetidine
In general, according to Mayo clinic, the following are safe during pregnancy
Amoxicillin
Ampicillin( all beta lactames are safe according to Kaplan)
Clindamycin
Erythromycin
Penicillin
Nitrofurantoin

123.27 weeks pregnant lady with history of UTI treated with antibiotics on week 12. Now she has symptoms of UTI and stat
abx (?), what is the best management?
A. Referral to cystoscopy
B. Start antibiotics if patient asymptomatic
C. Continue antibiotics then do culture
D. Referral to surgery
Answer: C
I believe B is wrong because I think the question is meant to be “ef patient is symmnomanec” to make sense. If so, B
would be wrong because bacteriuria/UTI in pregnant patients should be treated promptly even if she is
“asymmnomanec” due to the increased risk of pyelonephritis and other complications.
C is right because pregnant patients with UTI treatment success depends on complete eradication of the bacteria.
Patients should be followed up with culture after antibiotics course to insure that.
Source: http://emedicine.medscape.com/article/452604-treatment
124.Patient with Resistant slapingitis what is the organism?
A. Gonerra
B. Chlamydia
C. Strepto
D. E.coli

125.20 year-old pregnant lady was exposed to rubella virus 3 days ago. She was never vaccinated against rubella mumps or
measles, what's the best thing to do?
A. Give IG
B. Vaccine
C. Do nothing
D. Terminate the pregnancy
Answer: C

30
126.Pregnant with history of DVT, how to manage?
A. Heparin
B. Enoxaparin
C. Warfarin
D. No anticoagulant
Answer: B
Subcutaneous low molecular weight heparin (LMWH) is the preferred treatment for most patients acute DVT. The most
commonly used LMWH is enoxaparin

127.Which is at risk in uterine artery ligation


A. Pudendal n
B. Ovarian a
C. Vagina
D. Ureter
Answer: D
Ureters pass under uterine artery and under ductus deferens (renromerenoneal).“Waner (ureters) under the bridge (uterine
artery, vas deferens).”Gynecologec procedures involving ligation of the uterine vessels may damage the ureter. (First Aid – Step 1)

128.24 year-old was married for 9 month with regular heavy menses and pain. On examination, there is a nodule in cervix
and tenderness. What is the cause?
A. Fibroid-
B. Endometriosis
C. Cervical cancer
D. Vaginal cancer

129.A lady complained of vuvula vesicle that tender , no vaginal discharge?


A. Chancer
B. Syphilis
C. Hsv
D. Postherptic leison
Answer: A?

32
130.80% effecment , 4 cm dilated cervix on IV oxytocin, she is stable on CTG showingg variable acceleration?
A. Stop oxytocin
B. Give terbutaline
C. Change mother position
D. Expectant delivery
Answer: D

131.Best to confirm menopause?


A. FSH
B. LSH (not sure if this choice is correct! I think it should be LH)
C. Estrogen
D. Progesterone
Answer: A
(FSH) levels are higher than (LH) levels, and both rise to even higher values than those seen in the surge during the menstrual
cycle. The FSH rise precedes the LH rise. FSH is the diagnostic marker for ovarian failure. LH is not necessary to make the
diagnosis. Http://emedicine.medscape.com/article/264088-overview#a9

132.27 Gestation pregnant with monoamniotic twin, one of them died?


A. Give steroid and deliver
B. Wait to 34 then deliver
C. Wait 37 then deliver
D. Wait until SVD
Answer: (a or b)? Once choosing a conservative management, one should be aware, however, to the natural history of
approximately 90 % deliveries within 3 weeks from the time of diagnosis. Preterm delivery is therefore common and steroid
prophylaxis for lung maturity enhancement should be given.
Http://www.degruyter.com/dg/viewarticle.fullcontentlink:pdfeventlink/$002fj$002fjpme.2013.41.issue-1$002fjpm-2012-
0019$002fjpm-2012-0019.xml?T:ac=j$002fjpme.2013.41.issue-1$002fjpm-2012-0019$002fjpm-2012-0019.xml

133.70 year old with yellow discharge, foul smell not itching not sexually active:
A. Atrophic vaginitis
B. Candida
C. Bacterial vaginosis
D. Trhomanis
Answer: A This is due to estrogen deficiency in postmenopausal women.
Http://emedicine.medscape.com/article/257141-overview#a4

134.Breastfeeding mother haven't received MMR (rubella vaccine) what are you going to tell her?
A. MMR well hurt the baby
B. MMR is live attenuated bacteria
C. MMR can be received while breastfeeding
D. Stop breastfeeding for 48-72
Answer: C MMR vaccine can be given to breast-feeding mothers without any risk to their baby. Very occasionally rubella vaccine
virus has been found in breast milk but this has not caused any symptoms in the baby.
Http://www.nhs.uk/conditions/mmr/documents/rubella%20-%20questions%20and%20answers.pdf

135.For pap smear?


A. Three specimen from endocervix
B. 2 specimen from two different areas
C. One specimen from endocervix
D. One specimen from cervical os
Answer: A or C couldn’n find a reference for the # of the specimens
Http://www.cytopathologyear-old rg/specimen-collection-adequacy-requisition/

33
136.80% effacement, 4 cm dilated cervix on IV oxytocin, she is stable, CTG variable acceleration deceleration?
A. Stop oxytocin
B. Give terbutaline
C. Change mother position
D. Expectant delivery
Answer: A
If CTG is non-reassuring start 1 or more conservative measures:
Encourage the woman to change position and avoid being supine, offer oral or intravenous fluids, reduce contraction frequency
by stopping oxytocin if being used and/or offering tocolysis.
Https://www.nice.org.uk/guidance/cg190/resources/interpretation-of-cardiotocograph-traces-table-248732173
Http://geekymedics.com/how-to-read-a-ctg/

137.What prevent fracture in post menopause


a. Daily vitamin D supplements
b. Weight baring exercise
c. Decrease obesity
d. Answer: b. Weight bearing exercise. Step up to medicine

138.Increase risk of dysmenorrhea?


a. Copper releasing hormone
b. Levonorgestrel releasing hormone
c. Magnesium
d. Nifedipine
Answer: A

139.Pregnant with DVT previously, what to do:


a. Aspirin
b. Enoxaparin
c. Heparin
d. No anticoagulant
Answer: B.
Http://www.nhs.uk/conditions/pregnancy-and-baby/pages/dvt-blood-clot-pregnant.aspx

140.What is the mean age of menopause in normal women?


a. 48.4
b. 51.4
c. 53.4
d. 55.4
Answer: B- 51.4 (E medicine health) www.emedicinehealth.com/menopause/article_em.htm

141.62 years old came with vaginal bleeding. What is the most common benign cause of bleeding in this age?
a. Cervical erosion
b. Cervical polyps
c. Atrophic vaginitis
d. Endometrial Hyperplasia
Answer: c. Atrophic vaginitis. Atrophy account for (59%) of all postmenopausal bleeding histopathology. Uptodate.

142.Pregnant woman worries from tetanus for her baby. How will you advise her regarding vaccine?
A. Give to mother and child after delivery
B. Give to baby after delivery
C. Give as early as possible or before pregnancy
D. Give to mother after delivery
Answer: C

34
Http://www.who.int/reproductivehealth/publications/maternal_perinatal_health/immunization_tetanus.pdf

143.Pregnant lady (7th week) presented with RLQ pain, febrile (38.5) with tachycardia and hypotention. Labs: normal CBC (no
leukocytosis) UA: Normal … Diagnosis?
A. Ruptured appendix
B. Ruptured Ectopic Pregnancy
C. Ruptured Ovarian Cyst
Answer: B

Clinical manifestations of ectopic pregnancy typically appear six to eight weeks after the last normal menstrual period. The
classic symptoms of ectopic pregnancy are: pelvic\lower Abdominal pain, Amenorrhea, Vaginal bleeding. These symptoms can
occur in both ruptured and un-ruptured cases. Rupture may be present as sudden, severe pain, followed by syncope or by
symptoms and signs of hemorrhagic shock or peritonitis.

144.2*3 cm ampulla ectopic pregnancy, patient is hemodynamically stable. Management?


A. Laproscopy
B. Medical
C. Laprotomy
D. Observe
Answer: B
The management of hemodynamically stable ectopic pregnant patient is medically by Methotrexate. The criteria for
methotrexate treatment:
1-The patient must be hemodynamically stable, with no signs or symptoms of active bleeding or hemoperitoneum.
2-The patient must be reliable, compliant, and able to return for follow-up care.
3-The size of the gestation should not exceed 4cm at its greatest dimension (or exceed 3.5 cm with cardiac activity) on
ultrasonographic measurement - Exceeding this size is a relative, but not absolute, contraindication to medical therapy
4-Absence of fetal cardiac activity on ultrasonographic findings - The presence of fetal cardiac activity is a relative
contraindication
5-No evidence of tubal rupture - Evidence of tubal rupture is an absolute contraindication
6-β-HCG level less than 5000 miu/ml - Higher levels are a relative contraindication.

145.What structure you will feel laterally in p.v?


A. Ovaries
B. Pernial body
C. Ureter
D. Rectum
Answer: A
35
The purpose of the bimanual examination is to determine the size and nature of the uterus and the presence or absence of
adnexal masses.
Link: http://emedicine.medscape.com/article/1947956-technique#c4
Http://www.osceskills.com/e-learning/subjects/bimanual-vaginal-examination/

146.Pregnant lady 24 weeks GA, thyroid function test as the following:


TBG High, TSH Normal, TOTAL T4 high and Free T4 low
A) Pregnancy
B) Oral contraceptives use
C) Compensated euthyroid
D) Hyperthyroidism
Answer: A
A major contribution to the increased TBG concentration during pregnancy is the reduced plasma clearance of the protein
caused by changes in TBG glycosylation induced by estrogen. Total thyroxine (TT4) and total triiodothyronine (TT3)
concentration increase in the setting of pregnancy-induced increases in serum TBG concentrations. Free T3 (FT3) and free T4
(FT4) levels are slightly lower in the second and third trimesters. TSH levels are low-normal in the first trimester, with
normalization by the second trimester.
Link: http://www.ncbi.nlm.nih.gov/pmc/articles/PMC3625634/

147.22 years old women regular menses has severe lower abdominal pain what is the diagnosis?
A. Acute appendicitis
B. Ovary something
C. Ovary something
D. Gastroenteritis
Answer: ?
LINK: Causes of Acute abdominal pain in a young woman:
Https://www.ranzcog.edu.au/editions/doc_view/292-24-acute-abdominal-pain-in-a-young-woman-gynaecology-or-general-
surgery.html
148.Vaginal discharge. Treatment?
A. Metronidazole cream 4times 7days
B. Metronidazole tablet
C. Clindamycin tablet
D. Clindamycin cream
Answer: B

The 5-nitroimidazole drugs (metronidazole or tinidazole) are the only class of drugs that provide curative therapy of
trichomoniasis. Patients should be instructed to avoid intercourse until they and their partners have completed treatment and
are asymptomatic, which generally takes about a week. After single dose therapy or treatment of asymptomatic patients, the
couple should abstain from intercourse until both partners have waited at least seven days since taking the last antibiotic dose.
There are no studies on how long trichomonads remain viable after treatment is initiated or completed.
Oral is preferred to vaginal therapy since systemic administration achieves higher drug levels and therapeutic drug levels in the
urethra and periurethral glands, which serve as endogenous reservoirs of organisms that can cause recurrence. Cure rates for
vaginal therapy with metronidazole gel are ≤51 percent, which is significantly lower than with oral therapy, and therefore
vaginal therapy with metronidazole is not recommended

149.High d-dimer.acute case. What would you give this pregnant woman?
A. Infractionated heparin and warfrin
B. LMWH
C. Warfarin
D. Aspirin
Answer: B

36
150.Dew drops on rose petals vaginal lesions, dx:
A- Herpes simplex
B- Syphilis
C_ chanchroid lesion
D- herpangia
Answer: A
Herpes simplex and varicella zoster both cause this type of rash.
The skin lesions are characteristic for this disease. There are maculopapules, vesicles and scabs in various stages of evolution.
This is shown in the figure below.
There is shifting from maculopapules to vesicles over hours to days. Usually the trunk and face is affected and this shifts to other
regions of the body. The base of these vesicles are erythematous and they appear in crops i.e. Some are still developing while
others are healing. The classical sequence is macules, papules, clear vesicles, pustules, central umbilication and eventually crust
formation. The classical description of the lesion is a 'dew drop on rose petal' appearance. The rose petal refers to the reddish
irregular papule and the clear vesicle on it is the dew drop.

151.When to swap for GBS in pregnant ladies:


A-25 wks
B-30 wks
C-35 wks
D-40 wks
Answer: C
The CDC recommends GBS rectovaginal screening cultures for all pregnant women at 35 to 37 weeks of gestation.

152.What it the Side effect of post-menopausal hormonal therapy


A. Breast cancer
B. Uterine cancer
C.DVT
D. STROKE
Answer: C

Hormone Therapy and Breast Cancer


Although studies have been inconsistent, an emerging consensus appears to suggest that HT may slightly increase the risk for
breast cancer. This risk is similar to that associated with natural late menopause, and it comes into effect after at least 5 years of
continuous HT.

Hormone Therapy and Endometrial Cancer


In 1975, Zeil and Finle proved the relationship between the administration of exogenous estrogens and an increased incidence
of endometrial cancer. More recently, evidence from randomized controlled studies showed a definite association between HT
and uterine hyperplasia and cancer. HT based on unopposed estrogen is associated with this observed risk, which is unlike the
increased risk of breast cancer linked with combined rather than unopposed HT.
Continuous combined regimens have not been associated with an increased risk. However, cyclical regimens—even ones
involving 10-14 days of progestogens per month—do increase the risk after 5 years of usage.

Hormone Therapy and Thromboembolism


Various studies have shown concordance in the observation of an increased risk of thromboembolism with HT. The WHI study
demonstrated combined HT increased the risk of venous thrombosis and pulmonary embolism in 15 per 10,000 women per
year. The risk of venous thrombosis increased for women given an estrogen-only regimen.

Thus I believe that the strongest evidence in the literature currently is for DVT

37
153.22 year old lady healthy present to checkup she only complain of breast tenderness 3 days before menses /she never
had sexual contact and wish to be pregnant in next 2 year what is the best thing to do for screening?
A. Breast US
B.HPV
C. Pap smear
D. Colposcopy
Answer: C
She is most likely having premenstrual syndrome. No need for further testing regarding her breast tenderness. Pap smear should
be done at this age even if she is still virgin because it can be transmitted without intercourse.

154.Contraindication for breastfeeding?


A- HCV
B- HIV
C- Herpes zoster
D- Wart
Answer: B
Contraindication of breast feeding: human immunodeficiency virus (HIV) , antiretroviral medications, active tuberculosis,
human T-cell lymphotropic virus type I or type II , cancer chemotherapy agents , radiation therapy

155.24 married for 9 months with regular heavy menses and pain, on examination there is a nodule in cervix and tenderness,
what is the cause?
A- Fibroid
B- Endometriosis
C- Cervical cancer
D- Vaginal cancer
Answer: A

Http://www.merckmanuals.com/professional/gynecology-and-obstetrics/uterine-fibroids/uterine-fibroids

156.Pregnant type DM1 class f w/ nephrotic complication and control HYPERTENSION what is likely complication?
A) Preeclampsia
B) Stillbirth
C) Shoulder dystocia
D) Large for GA
Answer: A

Pregestational diabetes mellitus, as seen with T1DM patients, is a well-known risk factor for preeclampsia. The risk of developing
preeclampsia in gravid T1DM patients is between 12% to 15%, compared with 5% to 7% in the general population. In patients
with preexisting nephropathy the risk rises to as much as 50%.
Choice C and D are more common in type 2 DM.
Http://www.ncbi.nlm.nih.gov/pmc/articles/PMC3046748/

157.Pregnant woman, if not allergic, by which antibiotics you treat UTI?


A. Ampicillin
B. Sulfametha
C. Nitrofurantoin
D. I think ciprofloxacin
Answer: C

38
158.G3P2+0. Her first visit was on the 20th week of gestation. She has history of two premature deliveries. Her cervical length
was 30 mm. What is your appropriate management?
A. Strict bed rest
B. Terminate her pregnancy
C. Immediate cerclage
D. Inject her with progesterone
Answer: D or C
Several studies have indicated that the likelihood of preterm delivery increases with decreasing cervical length. A cervical length
of 25–30 mm before 32 weeks gestation seems to increase the risk of preterm delivery. If examination and ultrasound show that
you have an abnormally short cervix, and you’re less than 24 weeks pregnant, your practitioner may recommend “cerclage”, a
procedure in which she stitches a band of strong thread around your cervix to reinforce it and help hold it closed. However,
nhere’s a lot of controversy about whether cerclage should be used in this situation."

159.Patient g2p1 with twins GA41 weeks , +ve GBS management ?


A. C/s with AROM
B. Acyclovir
C. Prepare forceps for delivery
D. Abx
Answer: D Abx
Explanation: Mother must be given Abx to minimize risk of transmission to baby
According to the CDC, if you have tested positive and are not high risk, your chances of delivering a baby with GBS are:
1 in 200 if antibiotics are not given
1 in 4000 if antibiotics are given
To help protect their babies from infection, pregnant women who test positive for group B strep bacteria in the current
pregnancy should receive antibiotics through the vein (IV) during labor. Antibiotics help to kill some of the group B strep bacteria
that are dangerous to the baby during birth. The antibiotics help during labor only — they cannot be taken before labor,
because the bacteria can grow back quickly. Penicillin is the most common antibiotic that is given, but women who are severely
allergic to penicillin can be given other antibiotics. Women should tell their doctor or nurse about any allergies during a checkup
and try to make a plan for delivery. When women get to the hospital, they should remind their doctor and any staff if they have
any allergies to medicines.
Penicillin is very safe and effective at preventing the spread of group B strep bacteria to newborns during birth. There can be
side effects from penicillin for the mother, including a mild reaction to penicillin (in about 1 out of every 10 women). There is a
rare chance (about 1 out of every 10,000 women) of the mother having a severe allergic reaction that requires emergency
treatment.
Link: http://americanpregnancyear-old rg/pregnancy-complications/group-b-strep-infection/
Http://www.cdc.gov/groupbstrep/about/prevention.html

160.OCP:
A. Decrease the risk of ovarian cancer
B. Increase the risk of breast cancer
C. Decrease endometrial cancer
D. Increase risk of ectopic pregnancy
Answer: A
Explanation: combined oral contraceptive (COC) decrease the risk of ovarian cancer, endometrial cancer and colorectal cancer
It also increase the risk of cancers of the breast, cervix and liver.
Link: http://www.cancer.org/cancer/news/features/birth-control-cancer-which-methods-raise-lower-risk

161.34 years old lady, in 27 weeks pregnant present with upper and lower extremity edema her Blood pressure was 150/90
admitted for further investigation
What your management:
A. Low salt diet
B. BB
C. Reassure
D. Continue investing

39
Answer: Options might be wrong or scenario messing more details
Explanation: This could be preeclampsia:
Mild preeclampsia is defined as the presence of hypertension (BLOOD PRESSURE ≥041/91 mm Hg) on 2 occasions, at least 6
hours apart, but without evidence of end-organ damage, in a woman who was normotensive before 20 weeks' gestation.
All women who present with new-onset hypertension should have the following tests:
O CBC
O ALT and AST levels
O Serum creatinine
O Uric acid
O 24-hour urine collection for protein and creatinine (criterion standard) or urine dipstick.
- Management:
O Delivery is the only cure for preeclampsia. Patients with mild preeclampsia are often induced after 37 weeks' gestation.
O Before this, the patient is usually hospitalized and monitored carefully for the development of worsening preeclampsia or
complications of preeclampsia, and the immature fetus is treated with expectant management with corticosteroids to
accelerate lung maturity in preparation for early delivery.
O In patients with severe preeclampsia, induction of delivery should be considered after 34 weeks' gestation. In these cases,
the severity of disease must be weighed against the risks of infant prematurity. In the emergency setting, control of BLOOD
PRESSURE (Hydralazine, Labetalol, Nifedipine, Sodium nitroprusside) and seizures (ABC, Magnesium sulfate is the first-line,
Lorazepam and phenytoin may be used as second-line).
Link: http://emedicine.medscape.com/article/1476919-overview

162.Approach to vulvar carcinoma is ??


A) Clinical then ask about HPV
B) Histopatholgically then biopsy
C) Radiologically
D) Hematologically then tumor marker
Answer: B

163.Pregnant frequent abortions hx of recurrent herpes but NOW No lesions in vulva ,


Membranes ruptured one hour ago Clear & thin Wt to do?
A. CS
B. Instrumental
C. Give acyclovir
D. Speculum
Answer: A

164.To perform instrumental delivery you should first exclude what?


A. Cephalopelvic disproportion
B. Placental rupture
C. Breech presentation
D. Uterine rupture
Answer: A
See link for list of contraindications of instrumental delivery
Source: http://emedicine.medscape.com/article/263603-overview#a12

165.Old lady did hysterectomy and bilateral oophorectomy histology showed Ovarian germ cell theca something What other
findings :
A. chronic salpingitis
B. endometrial hyperplasia
C. uterine navus
D. Cervical something
Answer: B
Ref: http://emedicine.medscape.com/article/254489-overview#a6

41
166.Case about infertility, what's the initial evaluation
A. Temperature chart
B. Semen analysis
C. Refer to reproductive clinic
Answer: B
It depends on the case at hand. But most commonly, after taking proper Hx & Px, the first lab evaluation should be semen
analysis.
http://www.aafp.org/afp/2015/0301/p308.html

167.Patient post hystrectomy and oophorectomy to be started on HRT how you gonna give?
A- Cyclic estrogen & progestrone
B- Continuous estrogen & progesterone
C- Estrogen alone

Answer: C,
Post-Hysterectomy pts have no uteri so no point in giving PROGESTERON to prevent endometrial hyperplasia. Uptodate.

168.Post coital bleeding + vaginal discharge (no special color or odor was mentioned)
A- C. Trachomatis
B- Vaginosis
C- Candida

Answer: A

Postcoital bleeding:
The most common etiology is chlamydial infection(cervicitis), 2nd most common cause is Bacterial vaginosis.

Vaginal discharge (ddx):


Normal Bacterial Vaginosis (BV) Candidiasis Trichomoniasis
Symptoms - None - Discharge - Itching - Frothy discharge
- Odor that gets worse - Burning - Bad odor
after intercourse; may be - Irritation - Dysuria
asymptomatic - Thick, white discharge - Dyspareunia
- Vulvar itching and
Burning

Reference: uptodate, First Aid for Obstetrics and Gynecology.

169.38 years old female had amenorrhea for two months after 1 year of irregularity of the menstrual cycle. She is a mother of
3 children, she has one history of dilatation and curettage after Cesarean section. She has thinning in the vaginal
secretion labs were done and shows normal finding except high FSH & LH, low estrogen radiology revealed normal uterus
and cervix what is the diagnosis?

A- Premature failure of the ovary


B- Asherman's syndrome
C- Androgenic cause

Answer: A
40
In Primary ovarian insufficiency, ovaries do not regularly release eggs and do not produce enough sex hormones despite
high levels of circulating gonadotropins (especially follicle-stimulating hormone [FSH]) in women < 40. Diagnosis is
confirmed by high FSH and low estradiol levels. Typically, treatment is with combined estrogen/progestogen therapy.
Reference: Merck Manual: http://www.merckmanuals.com/professional/gynecology-and-obstetrics/menstrual-
abnormalities/primary-ovarian-insufficiency

170.50 years old Menopausal woman, high risk of breast cancer, with osteoporosis, what is the treatment of osteoporosis
here?
A. Vit D supplementation
B. Estrogen
C. Biphosphonate
Answer: C (Merck manual/uptodate)
Estrogen is contraindicated. All patients should have (risk factor modification, diet including daily supplementation of elemental
Ca and Vit D, exercise). Pharmacologic treatment bisphosphonates are first line drugs.

171.34 weeks gestation fetus with decreased fetal movements. What will you do?
A. Non stress test
B. Biophysical profile
C. Stress test
Answer: A
The nonstress test (NST) provides immediate reassurance of fetal viability and well-being. We suggest the basic evaluation of
pregnancies complicated by decreased fetal movement include review of the prenatal record and a nonstress test. Even in the
setting of a reactive nonstress test, we suggest obtaining an ultrasound examination within 24 to 48 hours if not recently
performed. *Uptodate

172.Which OCP will cause hypertension?


A. Estradiol levonorgesterel
B. Estradiol progesterone
C. Estradiol depressing....
Answer: A

st nd
173.1 c/s & 2 forceps delivery. Now pregnant with 34 weeks gestation & 50% effaced cervix. How you will deliver her?
A. C-Section
B. Induction with protein
C. Induction with syntocinon
Answer: ?

174.A patient has a breast mass. It is not related to cyclic pain. On exam, it is mobile. What is the most likely diagnosis?
A. Fibro-adenoma
B. Fibrocystic change
C. Intraductal papilloma (IDC)
Answer: A
Explanation: Fibroadenoma description matches the scenario in the question. IDC presents as nipple discharge, and is the most
common cause of spontaneous unilateral bloody nipple discharge
Reference Toronto notes

175.A patient presented to the gynecology clinic with malodorous vaginal discharge. What is the most likely diagnosis?
A. Trichomonas vaginitis (TV)
B. Gonorrhea
C. Chlamydia
Answer: A

42
Explanation: TV is the most common cause of vaginal complains worldwide. It is mainly characterized by diffuse malodorous
yellow-green discharge with vulvar irritation and characteristic strawberry cervix. Gonorrhea and chlamydia are sexually
transmitted disease that has mucopurulent discharge
Reference Kaplan OB/GYNE usmle step2

176.Case of vaginal prolapse. How to examine?


A. Speculum
B. US
C. Upright position
Answer: A
Uptodate:
APPROACH TO THE EXAMINATION: Pelvic Organ Prolapse is diagnosed with a pelvic examination. A medical history is also
important to elicit prolapse-associated symptoms, since treatment is generally indicated only for symptomatic prolapse.
Examination components: Physical examination of women with POP includes the following components:
Visual inspection
Speculum examination
Bimanual pelvic examination
Rectovaginal examination
Neuromuscular examination
177.Women at 20 w gestation with 2 previous premature births, what to do to prevent recurrence for this fetus?
A. Cerclage at 20 wk
B. Aspirin
C. Strict bed rest until full term
Answer: A
USMLE step 2 CK (page 69):
Interventions to prevent preterm delivery:
Singleton pregnancy:
a. Weekly IM 17-hydroxyprogesterone caproate (17-OH-P) if cervical length >= 25 mm with prior spontaneous PTB
b. Weekly IM 17-OH-P plus cervical cerclage placement if cervical length < 25 mm before 24 weeks with prior PTB
c. Daily vaginal progesterone if cervical length < 20 mm before 24 weeks but No prior PTB
Twin pregnancy: no interventions shown to have benefit

178.Hysterectomy granulosa theca tumor , what else to find ?


A. Condylomata acuiminata
B. Mole in uterus
C. Endometrial hyperplasia

Answer: C
Granulosa theca tumors are associated with elevated estrogen levels produced by the tumor, which will lead to endometrial
hyperplasia.
Http://emedicine.medscape.com/article/254489-clinical

179.When is the appropriate time to give rhogam to an Rh “-“ pregnant lady?


A. Before pregnancy and as soon as she knows she's pregnant
B. 3 days within delivery
C. At delivery and 2 weeks after

Answer: B
Rhogam should be given on 28 weeks gestation and 3 days within delivery.
Http://bestpractice.bmj.com/best-practice/monograph/669/treatment/details.html

43
180.Palpation of posterior vaginal fornix. What will you feel laterally?
A. Perineal body
B. Ovaries
C. Rectum

Answer: B
Http://emedicine.medscape.com/article/1947956-technique#c4

181.13 yo girl with normal second sexual character with absent uterus and vagina what's the cause:
A. Muller agenesis
B. Gonadal agenesis
C. Turner syndrome
Answer: A

182.Mother infected with HCV positive asking you about breastfeeding?


A. Continue, it's save
B. Discontinue and stop it
C. Give him HCV vaccine
Answer: A
A mother with hepatitis C, it is acceptable for you to breastfeed your child, according to both the Centers for Disease Control
and Prevention (CDC) and the American Academy of Pediatrics (AAP). Researchers do not believe the virus can be transmitted
through breast milk. Some studies did not find higher rates of hepatitis C in breastfed infants than in formula-fed babies.
Http://www.healthline.com/health-slideshow/hepatitis-c-and-pregnancy#6

183.Can use for pregnant women "?


A. Paracetamol
B. Aspirin
C. Ibuprofen

184.Women with mastitis


A. Stop breast feeding
B. Clean nipple with alcohol
C. Surgical drainage
THE ANSWER MAY BE MISSED OPTION CONTINUE BREAST FEEDING

185.How can stimulate breast feeding secretion?


A. Breast feeding
B. Increase fluid intake
C. Increase caloric intake

186.Primigravida week 16. She is RH negative. What is your next step?


a. US
b. Anti-D Rh immunoglobulin
c. Rh antibody titer
Answer: C
Rh antibody titer during the initial prenatal visit if she’s RH -
Unsensitized patients do not yet have antibodies to Rh positive blood. The goal is to keep it that way:
● So any time that fetal blood cells may cross the placenta, anti-D Rh immunoglobulin (rhogam) are given.
● Prenatal antibody screening is done at 28 and 35 weeks. Patients who continue to be unsensitized at 28 weeks should
receive anti-D Rh immunoglobulin prophylaxis.
● At delivery, if the baby is Rh positive, the mother should be given anti-D Rh immunoglobulin again.
44
The patient is considered sensitized if she has a titer level more than 1:4.
● If the titer is less than 1:16, no further treatment is necessary.
● If it reaches 1:16 at any point during the pregnancy, serial amniocentesis should be done. Serial amniocentesis allows for
evaluation of the fetal bilirubin level. Reference: Master the board. 2nd edition P461

187.Multigravida 34 week, her baby is breech, what you will do for her?
a. Expectant delivery
b. CS
c. External Cephalic Version
Answer: A
Expectant delivery until 36 week.
You should not perform ECV before 36 weeks, because the baby can turn into cephalic spontaneously.
Reference: Master the board

188.Which of the following non hormonal supplements will decrease the hot flashes in postmenopausal women?
a. Black Cohosh
b. Paroxetine
c. Bromocriptine
Answer: B (SSRI in general)
Ssris, venlafaxine, gabapentin, propranolol, clonidine. Reference: Toronto Notes
189.Why postmenopausal women develop osteoporosis?
a. Decrease progestin
b. Increase FSH
c. Decrease Estrogen

Answer: C

190.Young female complaining of whitish grey vaginal discharge. KOH test and clue test were positive. What is the diagnosis?
A. Gonorrhea
B. Bacterial Vaginosis
C. Trichomonas Vaginalis
Answer: B
Amsel criteria 3 out of 4 is diagnostic
Ph >4.5
Positive clue cells
Discharge is thin, grey and homogenous
Whiff test positive (KOH mount)
Reference: uptodate

191.20 years old sedentary female complaining of amenorrhea for the last 6 months and her BMI is 20.
A. Prolactinoma
B. Anorexia
C. Depression

Answer: C More information is needed to answer the question

192.A girl who hit puberty few months back and complains of spotting in between her periods. What will you tell her?
a. She has PCOS
b. She needs to take ocps
c. If tests were normal en’s not a disease

Answer: C
45
193.A patient with cervical carcinoma. What viruses are thought to be major culprits?
a. HPV 43 and 44
b. HPV 16 and 18
c. HPV 6 and 11

Answer: B

194.A woman with vaginal infection that grows gram negative diplococci. What is the organism involved?
a. N.gonorrhoeae
b. HSV
c. Candida
Answer: A
Reference: Toronto Note

195.What is the best way to know the date of pregnancy?


a. LMP
b. Ultrasound
c. Fundal height
Answer: B
The 3 basic methods used to help estimate gestational age (GA) are menstrual history, clinical examination, and ultrasonography
Early (first trimester) ultrasound is the most accurate way to determine gestational age. The first 2 are subject to considerable
error and should only be used when ultrasonography facilities are not available. Reference: Medscape.

196.A pregnant woman who has a child with down syndrome. She’s concerned about having another child with down
syndrome. What is the best test to rule out down syndrome in the second trimester?
a. Amniotic fluid sample
b. Chorionic villous sampling
c. Triple investigation
Answer: A, obs/gyne consultant
Second trimester screening tests include:
● Triple investigation is done between 15- 20 weeks. Its sensitivity is about 65% for trisomy 21. Patients with positive
screen should be offered U/S or amniocentesis for confirmation.
● Quadruple screen can improve the detection rate for Down syndrome to about 81%.
Second trimester confirmatory test: Amniotic fluid sample.
*See the tables at the end of OB/GYN section.
References: Toronto Notes and American Family Physician Journals http://www.aafp.org/afp/2007/0901/p712.html

197.Which of the following viruses can cross the placenta?


a. Rubella
b. Mumps
c. HBV
Answer: A
Hepatitis B transplacental infection is rare.

46
Reference: Kaplan
I think Rubella is the answer, because it is mentioned in all references I read, while Hep B was mentioned in few. Also, in
Medscape, they said it is a hypothesis that Hip B can cross placenta.Diseases that can cross the placenta:
● All TORCH; Toxoplasmosis Others: e.g. Syphilis Rubella
, CMV
, HSV
● HIV, Chicken pox, CMV, Erythema Infectiosum (Fifth Disease), Hepatitis B. Reference: Toronto Notes

198.(long scenario) Pregnant lady in 1st trimester (12 weeks) on iron trial, complaining of fatigue and shortness of breath.
CBC show: Hb = low , MCV = 70 , hematocrit = normal , reticulocyte = 10% . What is the most likely diagnosis?
A. Physiological.
B. Iron deficiency.
C. Thalassemia.

Answer: B

199.28 weeks pregnant (nuli), presents with generalized fatigue, BLOOD PRESSURE 162/95, 3+ protein in urine. What is your
next step?
a. Mgso4
b. Labetalol
c. Methyldopa

Answer: A

We recommend administration of antenatal corticosteroids for all pregnant women at 23 to 34 weeks


Betamethasone is given if < 34 weeks who are at increased risk of preterm delivery within the next seven days.
Reference: Master the Board + http://www.uptodate.com/contents/antenatal-corticosteroid-therapy-for-reduction-of-
neonatal-morbidity-and-mortality-from-preterm-delivery

200.Which infection has high mortality rate in pregnant?


a. Toxoplasmosis
b. Syphilis
c. CMV

Answer: C, Answerd by OBS/GYN consultant.


Question did not specify fetal or maternal mortality. Untreated syphilis has high fetal mortality rate (40%)

47
51- What antibiotic is safe in pregnancy?
a. Ciprofloxacin
b. Amoxicillin
c. Chloramphenicol

Answer: B
Some of the antibiotics that may be prescribed safely during pregnancy include: Amoxicillin, Ampicillin, Clindamycin,
Erythromycin, Penicillin, Gentamicin, Ampicillin-Sulbactam, Cefoxitin, Cefotetan and Cefazolin

201.Pregnant women (30 Something but definitely less than 38) with BLOOD PRESSURE baseline 80 now present with 140 or
160 (Not sure), LL edema. +2 proteinuria, no change in LFT (not sure). Admitted to the hospital. What will you give her?
a. Betamethasone
b. Labetalol
c. Mgso4
Answer: ? He question is not clear. The bottom line is that if the patient has severe preeclampsia give her mgso4 and
antihypertensives and definitive treatment is delivery. If only mild: conservative management and delivery 36 weeks
Reference: Kaplan
Refer to Q46
202.What is the drug of choice for eclamptic seizure?
A. Phenytoin.
B. Diazepam.
C. Magnesium Sulfate.

Answer: C
Although the definitive treatment is delivery, and the seizure should be controlled with magnesium sulfate.
Reference: Toronto Notes.

203.A pregnant lady presented with flank pain. On examination there was tenderness. Labs showed leukocytosis and positive
nitrate, what is the management?
a. Admission to treat pyelonephritis
b. Drink plenty of fluids
c. Start antibiotics

Answer: A
Reference: Kaplan Obstetrics and Gynecology USMLE step 2.

204.Which of the following is a side effect of ocps?


a. Breast cancer
b. Cervical cancer
c. DVT

Answer: C
Reference: Master the Boards.

205.Postmenopausal women complaining of itchy vulva and erythema of the labia majora and sometimes bleed. On
examination there is a pea shaped mass. What is the diagnosis?
A. Bartholin gland cyst.
B. Bartholin gland carcinoma.

48
C. Bartholin abscess.

Answer: B
It is generally recommended that women over age 40 with a Bartholin cyst or abscess undergo drainage and biopsy of the gland
to exclude the possibility of an underlying carcinoma.
Https://quizlet.com/53890288/obgyn-neoplasia-flash-cards/
More details are needed to answer the question. Although, old age and bleeding goes more with malignancy
Reference: BMJ Best Practice

206.Woman pregnant 42 Gestation was given gel for induction of labor. Later she was found to have 4 cm dilated cervix and
70% effaced. However, her contractions lasted 2 minutes and fetal HR dropped from 140 to 80. What to do?
a. Give Oxygen
b. Immediate c-section
c. Give some medication
Answer: B
Reference: Toronto Notes. OB47

207.A patient did a PAP smear and the result showed high grade intraepithelial cells. What will you do next?
a. Cone biopsy
b. Total hysterectomy
c. Colposcopy
Answer: C
HSIL (high-grade squamous intraepithelial lesion) includes: moderate dysplasia, carcinoma in situ, and CIN II and III. Colposcopy
and biopsy is indicated.
Reference: BMJ Best Practice
Refer to the chart at the end of OB/GYN section

208.What is the best treatment for premenstrual dysmorphic syndrome?


a. OCP
b. Fluoxetine
c. Bromocriptine
Answer: B
Reference: Master the Boards, and BMJ Best Practice

209.Sexually abused child vaginally, the hymen tear will be in which position?
a. 2 o'clock
b. 4 o'clock
c. 6 o'clock

Answer: C
Blunt penetrating trauma to the vaginal orifice produces a characteristic pattern of injury; bruising, lacerations and/or
abrasions are typically seen between the 4 and 8 o’clock positions of the hymen.
Reference: WHO -Child sexual abuse

49
210.A 19 years old white female. Weight: 52 kg & Height: 145 cm with no history of twins in the family, got spontaneously
pregnant by twins. What is the risk factor in this case?
A. Age
B. Race
C. Weight

Answer: B (None of the answers is correct)


Dizygotic twins are the most common. Identifiable risk factors include IVF, newly discontinued OCP, race (e.g. Certain African
regions), increased maternal age, geography, family history, or ovulation induction.
References: Kaplan Lecture Note; Ob/Gyn and Toronto Notes

211.A pregnant lady 34-week gestation presents with headache, epigastric pain and blurred vision with a BLOOD PRESSURE of
163/89. What is the best course of management?
a. Stabilize the general condition, magnesium sulphate, ….
b. Deliver immediately
c. Give magnesium sulphate, stabilize the general condeneon….

Answer: A, obs/gyne consultant said (stabilize then deliver).


Treatment is: ABC > mgso4 + hydralazine + Delivery
Refer to Q46

212.What is the best investigation to establish ectopic pregnancy?


a. HCG
b. Laparoscopy
c. Pelvic U/S

Answer: B
Β-HCG: 85% of ectopic pregnancies demonstrate abnormal β-HCG doubling
U/S: is only definitive if fetal cardiac activity is detected in the tube or uterus
Laparoscopy: for definitive diagnosis but it is invasive
Reference: Toronto Notes + Master the boards

213.40 year-old woman G3P1, history of 2 months pregnancy. Upon examination her uterus is large for gestational age. HCG:
very high, U/S: no fetus or heart sounds. She was diagnosed to have cancer which is sensitive to chemotherapy and easily
treated. What does she have? (She had molar or ectopic pregnancy before)
a. Endometriosis
b. Gestational Trophoblastic Disease.
c. Ovarian CA

Answer: B
Two main risk factors increase the likelihood for the development of GTD: 1) The woman being under 20 years of age, or over 35
years of age, and 2) previous GTD. Suspect gestational trophoblastic disease if uterine size is much larger than expected for
dates, women have symptoms or signs of preeclampsia, or β-hcg levels are unexpectedly high during early pregnancy or if
ultrasonographic findings suggest it.
Https://www.merckmanuals.com/professional/gynecology-and-obstetrics/gynecologic-tumors/gestational-trophoblastic-
disease

51
214. 35 weeks of gestation, come mention she doesn’t feel the baby movement. Fetus is dead. She is hypoxic with decreased
DLCO (75%, Normal 80%) Blood test: Low Hct, prolonged PT and PTT. What does she have?
a. Amniotic embolism
b. DIC
c. ITP

Answer: A
Amniotic fluid embolism causes DIC and ARDS together.
Amniotic fluid embolism is a clinical syndrome of hypoxia, hypotension, and coagulopathy that results from entry of fetal
antigens into the maternal circulation.
Https://www.merckmanuals.com/professional/gynecology-and-obstetrics/abnormalities-and-complications-of-labor-and-
delivery/amniotic-fluid-embolism

215.Women 34 weeks pregnant, however fundal height is 28cm. What is the most likely cause of IUGR?
a. GDM
b. Oligohydramnios
c. Polyhydramnios
Answer: B
Screening for IUGR in the general population relies on symphysis–fundal height measurements.
Discrepancy of greater than 3 cm between observed and expected measurements may prompt a growth evaluation using
ultrasound.

216.Anovulatory female. What will you give to induce ovulation?


a. Clomiphene
b. Danzo
c. Pulsatile push of LH
Answer: A
The most widely prescribed drug for ovulation induction to reverse anovulation or oligoovulation.
Https://www.uptodate.com/contents/ovulation-induction-with-clomiphene-beyond-the-basics#H3
Medical induction of ovulation: clomiphene citrate, human menopausal gonadotropins
(HMG [Pergonal®]), LHRH, recombinant FSH, and metformin.
Metformin may be used alone or in conjunction with clomiphene citrate for ovulation induction

217.Female with clear presentation of UTI. History of URTI. Urine analysis showed nitrate : + Esterase : +, what is the
organism:

A. Klebsiella pneumoniae
B. E.coli
C. Pseudomonas

Answer: E. Coli
-nitrate test is commonly used in diagnosing urinary tract infections (UTI). A positive nitrite test indicates that the cause of the
UTI is a gram negative organism, most commonly Escherichia coli
-A leukocyte esterase test (LE test) is a urine test for the presence of white blood cells and other abnormalities associated with
infection.
White blood cells in the urine usually indicate a urinary tract infection.also used to screen for gonorrhea and for amniotic fluid
infections.
-The combination of the LE test with the urinary nitrite test provides an excellent screen for establishing the presence of a
urinary tract infection (UTI).

50
218.A 17 yrs old girl vaginal delivery at home with perineum tear what is the injured ?
A. Coccygeal
B. Pubococcygeal
C. Ischial...
Answer: pubococcygeus
The muscles of the anus (corrugator cutis ani, the internal anal sphincter and the external anal sphincter)
* The medial muscles of the urogenital region (the superficial transverse perineal muscle, the deep transverse perineal muscle
and bulbocavernosus)
* The medial levator ani muscles (puborectalis and pubococcygeus)
* The fascia of perineum, which covers these muscles
* The overlying skin and subcutaneous tissue.

219.What the treatment of eclamptic seizure?


A. Magnesium sulphate
B. Diazolam
C. Carbamazepine
Answer: mgso4
As soon as eclampsia or severe preeclampsia is diagnosed, Mg sulfate must be given to stop or prevent seizures and reduce
reflex reactivity
Https://www.merckmanuals.com/professional/gynecology-and-obstetrics/abnormalities-of-pregnancy/preeclampsia-and-
eclampsia

220.Diagnosis of trichomonas vaginosis?


A. Yellow discharge
B. White discharge
C. Clue cell
Answer: A
Yellow-green, and frothy in trichomonas
Clue cells will be in bacterial vaginosis not in trichomonas

221.Adenomyosis treatment?
A. Hysterectomy
B. Ocp
C. Gonadotropin analogue
Answer: A

Hysterectomy is the definitive surgical treatment


First IUD (coil) may be offered, since it has been found that a low level slow release of progesterone can ease the symptoms of
adenomyosis
Adenomyosis advice association..

222.Pregnant with flank pain and tenderness, +nitrate and leukocytosis what is the plan
A. Admission to treat pyelonephritis
B. Drink plenty of fluid
C. Start antibiotics
Answer: A
Acute pyelonephritis is characterized by fever, flank pain, and tenderness in addition to significant bacteriuria.
Positive results for nitrites, leukocyte esterase, wbcs, red blood cells (rbcs), and protein suggest Pyelonephritis.
Http://emedicine.medscape.com/article/452604-overview

223.Severe symptoms of preeclampsia


A. Abdominal pain
B. High urea

52
C. High blood pressure
Answer: abdominal pain due to hepatic sub capsular swelling from edema which may develop to hematoma or Hepatic
rupture even it’s rare but can occur .(resource Lippincott Obs&gyne recall) the other options can occur in mild preeclampsia
too.

Severe preeclampsia:

Blood pressure: 160 mm Hg or higher systolic or 110 mm Hg or higher diastolic on two occasions at least six
hours apart in a woman on bed rest

Proteinuria: 5 g or more of protein in a 24-hour urine collection or 3+ or greater on urine dipstick testing of
two random urine samples collected at least four hours apart

Other features: oliguria (less than 500 ml of urine in 24 hours), cerebral or visual disturbances, pulmonary
edema or cyanosis, epigastric or right upper quadrant pain, impaired liver function, thrombocytopenia,
intrauterine growth restriction

We generally perform a two-layer rather than a single-layer uterine closure so is not an issue … if she had only one caesarean
section, and there were complications during the procedure CS is indicated . If there were no complications, a vaginal birth
after CS is possible but if previous 2 CS the 3rd should be CS

224.40 year old lady early pregnant, what is useful for her:
A- urine dip steak.
B- blood group and Rh factor
C- ultrasonography
Answer: C
8-12 weeks GA> blood group and Rh
(Toronto notes)

225.What to do after a Pap smear show atypical changes?


A. Hysterectomy
B.Guided calposcopy biopsy
C.Exsional biopsy
Answer: B
Http://www.mayoclinic.org/tests-procedures/pap-smear/basics/results/prc-20013038

226.When do you do US for screening of the fetus?


A-early 2nd trimester
B- late 2nd
C-early 3rd and late 3rd
Answer: B
Please see attached table

53
227.Complicated labor switch to C-section, when to give antibiotics?
A. Before C-section
B. After
C. During

Answer: A
All women undergoing elective or emergency Caesarean section should receive antibiotic prophylaxis. (I-A)
The choice of antibiotic for Caesarean section should be a single dose of a first-generation cephalosporin. If the patient
has a penicillin allergy, clindamycin or erythromycin can be used. (I-A)
The timing of prophylactic antibiotics for Caesarean section should be 15 to 60 minutes prior to skin incision. No
additional doses are recommended. (I-A)
If an open abdominal procedure is lengthy (> 3 hours) or estimated blood loss is greater than 1500 ml, an additional
dose of the prophylactic antibiotic may be given 3 to 4 hours after the initial dose. (III-L)
Prophylactic antibiotics may be considered for the reduction of infectious morbidity associated with repair of third and
fourth degree perineal injury. (I-B)
Reference: http://sogc.org/wp-content/uploads/2013/01/gui247cpg1009e_000.pdf

228.A pregnant in 32 weeks of gestation, she is in true labor, what to do:


A. Call neonatologist, give corticosteroids, strict bed rest
B. Call neonatologist, give corticosteroids, give fluids
C. Call neonatologist, give antibiotics, bed rest

Answer: A or B

54
229.Pregnant women in 1st trimester never got chickenpox, her antibody titer is zero, what is the best management?
A. Avoid exposure
B. Acyclovir 3
C. Varicella vaccine

Answer: A
In’s contraindicated to take varicella vaccine during pregnancy if patient got contact there is varicella zoster immune globulin
(varizig) that can help to reduce the risk of becoming infected with chickenpox.
Non immune pregnant woman can take the vaccine one month prior to being pregnant or after delivery.
Reference: http://www.cdc.gov/vaccines/pubs/preg-guide.htm

230.Smoker pregnant women, what is the possible fetus complication?


A. Macrosomia
B. Low birth weight
C. Transient tachypnea
Answer: B
Smoking during pregnancy has been associated with a host of complications, including low birth weight, premature rupture of
the membranes, placenta previa, placental abruption, and preterm birth, preterm premature rupture of membranes (PPROM),
ectopic pregnancy.
Reference: http://www.medscape.com/viewarticle/717666

231.30 weeks’ pregnant woman, BP: 170/120, what is the proper prophylactic management in addition to blood pressure
control?

A. Mg sulphate and deliver


B. Mg sulphate and wait till 34w
C. Call anesthesia and deliver

Answer: B

55
If a pregnant woman's blood pressure is sustained greater than 160 mm Hg systolic and/or 110 mm Hg diastolic at any
time, lowering the blood pressure quickly with rapid-acting agents is indicated for maternal safety.
Anticonvulsant therapy may be undertaken in the setting of severe preeclampsia (primary prophylaxis) or in the
setting of eclamptic seizures (secondary prophylaxis). The most effective agent is IV magnesium sulfate; phenytoin is
an alternative, although less effective, therapy.
Labetalol has a more rapid onset of action, may be given orally or parenterally, and is generally preferred as a first-line
agent.
Women with suspected, mild, or diagnosed preeclampsia remote from term or labile blood pressures due to chronic
hypertension and/or gestational hypertension should be hospitalized for close observation, bed rest, and frequent
fetal monitoring.
When preeclampsia develops remote from term (ie, < 34-36 weeks' gestation), attempts are often made to prolong
the pregnancy to allow for further fetal growth and maturation.

Reference: http://emedicine.medscape.com/article/261435-overview#a21

232.Best way to avoid transmission of tetanus in pregnant lady to her baby?


A. Newborn tetanus toxoid
B. Neonate anti tetanus
C. Give the mother early tetanus toxoid
Answer: C
ACIP recommends that providers of prenatal care implement a Tdap immunization program for all pregnant women. Health-care
personnel should administer a dose of Tdap during each pregnancy, irrespective of the patient's prior history of receiving Tdap.
Reference: http://www.cdc.gov/mmwr/preview/mmwrhtml/mm6207a4.htm

233.Patient has obesity, hirsutism, HTN, insulin resistance What is the diagnosis?
A. Kallman syndrome
B. Kleinfilter syndrome
C. Stein leventhal syndrome

Answer: C
o Polycystic ovarian syndrome also called:
o Chronic ovarian androgenism
o Hyperandrogenic anovulation (HA)
o Stein–Leventhal syndrome

234.Lady with metromenorrhagia ( dysfunctional uterine bleeding) from 6 month ago and abdominal pain interfere with her
activity, what is the best drug?
A. Hysterectomy
B. Oral contraceptive
C. Estrogen analogous

Answer: B is the best answer


Reference: http://emedicine.medscape.com/article/795587-treatment#d9

235.Which of the following Oral contraceptive can cause hyperkalemia?


A. Levonogestrel
B. Norethindrone
C. Yasmin

Answer: C

56
Yasmin (ethinyl estradiol+drospirenone “mrogesnen”) and Yaz causes hyperkalemia (rare side effect, but contraindicated in renal
and adrenal insufficiency)
Reference : (Toronto notes)

236.Pregnant in labor cervical opening 6 cm, which stage this manifestation?


A. Stage 1
B. Stage 2
C. Stage 3

Answer: A
Labour stages:
First stage: The time of the onset of true labor until the cervix is completely dilated to 10 cm.
o Early Labor Phase –The time of the onset of labor until the cervix is dilated to 3 cm.
o Active Labor Phase – Continues from 3 cm. Until the cervix is dilated to 7 cm.
o Transition Phase – Continues from 7 cm. Until the cervix is fully dilated to 10 cm
Second stage: The period after the cervix is dilated to 10 cm until the baby is delivered
Third stage: Delivery of the placenta.
Reference : http://americanpregnancyear-old rg/labor-and-birth/first-stage-of-labor/

237.A 39 weeks pregnant female .. Came with BLOOD PRESSURE 160/95 ..no proteinuria.. The previous visits the BLOOD
PRESSURE was normal .. What is your diagnosis?
A. Gestational hypertension
B. Chronic HTN
C. Preeclampsia

Answer: A
GESTATIONAL HYPERTENSION is diagnosed with sustained elevation BLOOD PRESSURE ≥ 140/90 mmhg after 20 weeks of
pregnancy without proteinuria. (Kaplan)

238.Early pregnant , what is useful for her


A. Urine dip steak
B. Blood group and rh factor
C. Ultrasonography
Answer: C
First trimester ultrasonography is used to confirm the presence of an intrauterine pregnancy, estimate gestational age, diagnose
and evaluate multiple gestations, confirm cardiac activity, and evaluate pelvic masses or uterine abnormalities (as an adjunct to
chorionic villus sampling, embryo transfer, or localization and removal of intrauterine contraceptives). It is also useful for
evaluating vaginal bleeding, suspected ectopic pregnancy, and pelvic pain. (Obstetrics and Gynecology, sixth edition, Charles R.
B. Beckmann, page 63)

239.Pregnant woman in third trimester she is in airline about 18 hours and developed of sudden chest pain , with dyspnea,
what is the cause ?
A. Pulmonary hypertension
B. .pulmonary embolism
C. Myocardial ischemia
Answer: B
Sob and chest pain symptoms of PE
Clinical signs and symptoms of PE are nonspecific. The classic symptoms of PE are dyspnea (82%), abrupt onset chest pain (49%),
and cough (20%). (http://emedicine.medscape.com/article/2056380-clinical#b5)

57
240.Young patient with 4 weeks amenorrhea , presented with abdominal pain and severe vaginal bleeding , the bleeding
from ?
A. A-Ovary
B. B-Fallopian tube
C. C-Cervix
Answer:Fallopian tube(webmed)
The classic symptoms associated with ectopic pregnancy are amenorrhea followed by vaginal bleeding and abdominal pain on
the affected side (Obstetrics and Gynecology, sixth edition, Charles R. B. Beckmann, page 142)

241.Complicated labor switch to c-section when to give antibiotics ?


A. Before c-section
B. After
C. During
Answer: a
Single dose prophylactic antibiotic should be used (e.g. Cefazolin 1-2g) (Kaplan page 133)

242.Pregnant lady miss pregnant symptom since 1 week and started complain of spot bleeding the most valuable
investigation in this condition is :
A. Hcg
B. Alpha feto
C. Us
Answer: C
US to distinguish between types of abortion, to identify the different types of placenta previa or placenta abruption. (master the
board: USMLE Step 2 CK)

243.You performed a pudendal nerve block on a woman in labor, which of the following structures will be fully sensitive and
not blocked by the anesthesias?
A. Perineal body
B. Urogenital diaphragm
C. Rectum
Answer: C
The sensory and motor innervation of the perineum is derived from the pudendal nerve, which is composed of the anterior
primary divisions of the second, third, and fourth sacral nerves. The pudendal nerve’s 3 branches include the following:
1. Dorsal nerve of clitoris, which innervates the clitoris
2. Perineal branch, which innervates the muscles of the perineum, the skin of the labia majora and labia minora, and the
vestibule
3. Inferior hemorrhoidal nerve, which innervates the external anal sphincter and the perianal skin
Http://emedicine.medscape.com/article/83078-overview#a1
The anal canal also has differing nervous innervations above and below the line. Above the pectinate line, the nerve
supply is visceral, coming from the inferior hypogastric plexus. As is it visceral, this part of the anal canal is only sensitive to
stretch. Below the pectinate line, the nerve supply is somatic, receiving its supply from the inferior rectal nerves (branches of
the pudendal). As it is somatically innervated, it is sensitive to pain, temperature, and touch.
Http://fitsweb.uchc.edu/student/selectives/Luzietti/Painful_anus_anorectal_anatomy.htm

244.Pregnant in the third trimester with history of recurrent herpes simplex, she is in labour, during exam no lesions what to
do?
A. CS

58
B. IV acyclovir
C. Do specular exam before

Answer: B

• Although treatment during primary infection lessens morbidity, it does not eradicate latent virus, which can
subsequently reactivate. Clinical recurrences are common and can be treated episodically or prevented with continual
antiviral suppression.
• Recurrent genital infection refers to the occurrence of genital HSV infection in a patient seropositive for the HSV type
recovered from the lesion. The risk of neonatal transmission at delivery is much lower than in patients with primary or
first episode genital infection.
• Transmission of herpes simplex virus (HSV) to neonates usually occurs during labor and delivery as a result of direct
contact with virus shed from infected sites (vulva, vagina, cervix, perianal area). Importantly, viral shedding can occur
when maternal symptoms and lesions are absent
• Suppressive antiviral therapy is suggested at 36 weeks of gestation through delivery for women with a history of
recurrent genital herpes to reduce the risk of lesions at the time of delivery
• Suppressive therapy reduces the risk of clinical recurrence of HSV and asymptomatic viral shedding at delivery, and
thus the need for cesarean delivery. However, the clinical impact on neonatal HSV is unknown.
Http://www.uptodate.com/contents/genital-herpes-simplex-virus-infection-and-pregnancy?Source=see_link

245.Pregnant women has GGT diagnostic what is your action ?


A. Do hga1c
B. Start insulin
C. Do Random blood Glucose
Answer: start insulin
Definitive diagnosis is based on OGTT if it is positive start insulin. (Kaplan page 91)

246.About the side effect of OCP ?


A. Breast cancer
B. Cervical cancer
C. DVT
Answer: DVT
Serious complications (such as venous thrombosis, pulmonary embolism, cholestasis and gallbladder disease, stroke, and
myocardial infarction) are more likely for women using high-dose formulations. However, these complications also can occur
occasionally in patients taking low-dose formulations. Hepatic tumors have also been associated with the use of high-dose oral
contraceptives. (Obstetrics and Gynecology, sixth edition, Charles R. B. Beckmann, page 226)

247.Female with no sexual hx presented with amenorrhea and thyroid disease has been excluded what first investigation
will you do ?
A. Pregnancy test
B. TSH
C. Prolactin
Answer: prolactin
First rule out pregnancy (no sexual hx) & then TSH (thyroid disease excluded in question)
The definitive method to identify hypothalamic–pituitary dysfunction is to measure FSH, LH, and prolactin levels in the blood. In
these conditions, FSH and LH levels are in the low range. The prolactin level is normal in most conditions, but is elevated in
prolactin-secreting pituitary adenomas. (Obstetrics and Gynecology, sixth edition, Charles R. B. Beckmann, page 316)

59
248.Pregnant with HIV on medication, used to have 400 copies and now 200 copies on labor
A. A.Spontaneous Vaginal delivery
B. B.Forceps delivery
C. C.CS
Answer: A
Awareness of maternal HIV status can help guide management of labor and delivery to minimize risk of transmission to the
fetus. Use of episiotomy or vacuum extraction or forceps may potentially increase risk of transmission by increasing exposure
to maternal blood and genital secretions.
Cesarean delivery performed before the onset of labor and rupture of membranes significantly reduces the risk of perinatal HIV
transmission. Planned cesarean delivery at 38 weeks of gestation to prevent perinatal transmission of HIV is recommended for
women who have a viral load >1000 copies/ml. (Obstetrics and Gynecology, sixth edition, Charles R. B. Beckmann, page 170)

249.Women with negative pregnancy test have vaginal bleeding, Hgb 9 mangement?
A. Blood transfusion
B. Progesterone
C. Conjugated Estrogen
Answer: b
The primary goal of treatment of anovulatory uterine bleeding is to ensure regular shedding of the endometrium and
consequent regulation of uterine bleeding. If ovulation is achieved, conversion of the proliferative endometrium into secretory
endometrium will result in predictable uterine withdrawal bleeding.
A progestational agent may be administered for a minimum of 10 days. The most commonly used agent is medroxyprogesterone
acetate. When the progestational agent is discontinued, uterine withdrawal bleeding ensues, thereby mimicking physiologic
withdrawal of progesterone. (Obstetrics and Gynecology, sixth edition, Charles R. B. Beckmann, page 319)

250.Missed period 2 months , high ah BCG , examination show 16 weeks GA .. US show fetus small for data ? Dx:
A. Choriocarcinoma
B. Hydatidiform
C. Placenta in site trophoblastic tumor
Answer: C
The placenta is critical for nutrient regulation and transportation from mother to fetus. Abnormalities in placentation or
defective trophoblast invasion and remodeling may contribute to fetal growth restriction as well as other disorders of
pregnancy. In addition, uterine anomalies (uterine septum or fibroids) may limit placental implantation and development and,
consequently, nutrient transport, resulting in inadequate nutrition for the developing fetus. Finally, the genetic composition of
the placenta is important and abnormalities such as confined placental mosaicism are associated with growth delay. (Obstetrics
and Gynecology, sixth edition, Charles R. B. Beckmann, page 191)

251.Postmenopausal women has hot flushes what altered enzyme is the reason behind her symptoms ?
A. TSH
B. Estrogen
C. Progenstron
Answer: B
The hot flush is the most common symptom of decreased estrogen production and is considered one of the hallmark signs of
perimenopause. (Obstetrics and Gynecology, sixth edition, Charles R. B. Beckmann, page 330)

252.Female with ovarian mass, hysterectomy was done, specimen showed thecal cell tumor, what other things you would
find in the specimen?
A. Moles in the uterus
B. Endometrial hyperplasia
C. Others
Answer:B

61
The leading role in the pathogenesis of endometrial hyperplasia is given relative or absolute hyperestrogenic, the absence of
antiestrogenic effects of progesterone or insufficient effect.
Causes of hyperestrogenic: anovulation caused by the persistence or atresia of follicles, hyperplastic processes in the ovaries or
hormoneproducing tumor of ovary (stromal hyperplasia, tecomates, granulosa tumor, theca cell tumor, etc)

253.Pregnent lady miss pregnant symptom since 1 week and started complain of spot bleeding the most valuable
investigation in this condition is :
A)HCG
B)alpha feto
C)us .....

Answer:C

254.Post menopose women pallor vagina and week what is the Treatment:
A)estrogen
B)proges
C)diet
Answer: A
The therapeutic standard for moderate to severe vaginal atrophy is estrogen therapy, administered either vaginally at a low
dose or systemically. There has been a relative lack of randomized controlled trials performed to date, but they have shown that
low-dose, local vaginal estrogen delivery is effective and well tolerated for treating vaginal atrophy.
Http://www.medscape.org/viewarticle/556471

255.During pv exam you found cervical mass 10×12mm what you will do :
A) Reassure.
B) Biopsy.
C) Test for human papilloma virus

Answer:
Https://my.clevelandclinic.org/health/diseases_conditions/hic_Genital_Warts/hic_Understanding_HPV
256.Girl 15 never had menses, Examination breast bed ,fine hair >n After 1 year come e increase height >6cm And gain
kilograms:
A) Primary amenorrhea
B) amenorrhea Precocious puberty
C) Constitutional
Answer: A
Primary amenorrhea is the failure of menses to occur by age 16 years, in the presence of normal growth and secondary sexual
characteristics. If by age 13 menses has not occurred and the onset of puberty, such as breast development, is absent, a workup
for primary amenorrhea should start.
Http://emedicine.medscape.com/article/252928-overview

257.Mother breastfeeding, needs MMR vaccine


A -Give the vaccine
B - Give and stop breastfeeding for 72 hours
C -MMR is harmful to the baby
Answer : A

Breast feeding does not interfere with the response to MMR vaccine, and your baby will not be affected by the vaccine through
your breast milk.
Reference : http://www.cdc.gov/vaccines/vpd-vac/measles/faqs-dis-vac-risks.htm

258.Old lady postmenopausal with osteoarthritis and risk for osteoporosis , what you will do :

60
A. Calcium , TSH , dihydroxy vit D
B. Bisphosphonate , vit D , calcium
C .DEXA scan
Answer: B
Bisphosphonates are first-line pharmacological therapy for postmenopausal women and men.
In postmenopausal women ,oestrogenis considered only for those at high risk for whom non-oestrogen medicines are
inappropriate.
Ref. BMG Best practice

259.Pregnant female in 24 weeks gestation, known case of chronic DM type 2 and chronic HTN, fundal height is 25 cm, which
one of the following is a complication of her pregnancy?
A. Preeclampsia.
B. Shoulder dystocia.
C. Large infant for gestational age.
Answer: A
Hypertension/preeclampsia (especially if pre-existing nephropathy/ proteinuria): insulin resistance is implicated in etiology of
hypertension

260.Patient doesn’t want to get pregnant for years. What will you give her?
A. Estrogen
B. Androgen
C. Estrogen and androgen
Answer: ?
Methods of long acting reversible contraception:
Available LARC methods include iuds and the subdermal implant:
1- Hormonal intrauterine device (Mirena - also known as IUC or IUS)
2- Nonhormonal intrauterine device with copper (US -paragard)
3- Subdermal contraceptive implant (US -Nexplanon/Implanon/Implanon NXT; internationally -Norplant/Jadelle)
4- Some shorter-acting methods are sometimes considered LARC:
- Depot medroxyprogesterone acetate injection (DMPA; US - - Depo Provera shot)
- Combined injectable contraceptive

62
261.Patient with amenorrhea and discharge from her breast with high prolactin level what to do next:
A - check estrogen level
B - exclude pituitary lesion
C - TSH level
Answer: b
Patients with hyperprolactinaemia or those diagnosed with hypogonadotrophic hypogonadism and neurological symptoms
should undergo neuroimaging to rule out an intracranial neoplasm.
Serum prolactin: elevated levels of circulating prolactin (hyperprolactinaemia), whether idiopathic or due to a pituitary
adenoma, result in hypogonadotrophic hypogonadism. For persistently elevated levels, neuroimaging is indicated to rule out
intracranial neoplasm
BMG Best Practice

262.Pregnant lady presenting lower limb swelling, HYPERTENSION and mild proteinuria, what will you give her?
A-Methyldopa
B-ACEI
C-ARB
Answer: A ???
The answer depends of the BLOOD PRESSURE reading and other details to distinguish b.w mild and severe pre-eclampsia.
Anyhow we gonna discuss the treatment for both.
Mild preeclampsia:
• < 37 weeks, expectant Rx in the hospital with no need for antihypertensive or mgso4 + close monitoring for both fetus
and mother.
• > 37 weeks, prompt delivery is the choice
Severe preeclampsia:
• Antihypertensive; labetalol or hydralazine
• Mgso4
• Prompt delivery regardless the gestational age

263.Had history of HPV when she was young, you did Pap test and found nothing what to do now:
A - do nothing
B - repeat every 5 years
C - repeat annually
Answer: C
Women with a negative Pap smear and a positive HPV test should have both tests repeated at 12 months. If both tests are
negative at that time, they can be returned to routine screening. If the HPV test remains positive, women should be referred for
colposcopy. For women with an abnormal Pap smear, irrespective of HPV testing status, appropriate evaluation should be
undertaken. It is important to note that women should continue to have cytological screening even if they have been immunised
for HPV
Ref BMG Pest practice

264.Nulliparous Patient came to ER with heavy bleeding 18 month history of heavy bleeding and trying to conceive for 1 year
Vitals given
Hgb= 10
Pregnancy test -ve; what to give:
A. Blood transfusion
B. Progesterone
C. Estrogen
Answer: B
*The right answer is progesterone the medication called duphastone
Or because she wants to get pregnant

63
Clomiphene if there’s.

265.Pregnant lady 11 weeks GA , come to weird about infectious disease outbreak in the school of her child that may
affect her pregnancy . What is the appropriate vaccine at that time?
A-rubella
B-varicella
C-influenza
Answer: c
Varicella & rubella vaccines are contraindicated during pregnancy.
Http://www.cdc.gov/vaccines/pubs/preg-guide.htm

266.Women had yellow watery foul smelling vaginal discharge


A- bacterial vaginosis
B- trichominus vaginalis
C- atrophic vaginitis
Answer: trichominus vaginalis. See table in Q115 Gyne section

267.Patient in labor and has pre-eclampsia. Mgso4 and hydralazine were given. Respiratory rate was 12. What do
you give?
A- Narcan
B- Naloxone
C- Atropine
Answer:
She has magnesium sulphate toxicity > respiratory depression so Calcium gluconate
Is the antidote for magnesium sulphate

268.What changes will occur during pregnancy


A) Increase tidal volume
B) Increase total lung capacity
C) Increase residual capacity
Answer: a
Respiratory changes during pregnancy: No Change in VC and FEV1. Decrease TLC, FRC and RF
Reference: toronto.

269.Patient complaint progressive wrist pain since 2 months and increased in the passed 1month , give history of
cesarean delivery on exam therenumbness and normal range of motion
What is the Rx?
A. Thumb cast
B. Whole are cast
C. Surgical decompression
Answer:
Symptoms usually resolve over period of week after delivery
We recommend nocturnal wrist splint, surgical decompression rarely done

64
Http://www.uptodate.com/contents/carpal-tunnel-syndrome-treatment-and-
prognosis?Source=outline_link&view=text&anchor=H956780404#H956780404

270.Pregnant is in her 38 weeks gestation with a blood pressure of 140 over 90. No proteinuria and
completely asymptomatic what will you do?
A-Immediate delivery
b-Antihypertensives
c-Observation (frequently)
Answer:c
This case consider as gestational hypertention ( bp<150/90 and no proteinuria)
Ttt: close blood pressure monitoring

271.Female patient known to have Bicornuate uterus present in labor , give History of kicking in lower abdomen and
on Examination there is round object in fundus on auscultation the heart positive in the umbilicus of his mother ,
what is the most likely presentation ?
A-Face
b-Vertex
c-Breach
Answer: C
272.Pregnant with past history of depression on Paroxitine for long time. She is asking the physician if she can use
this medication or not while she is pregnant. What you have to tell her ?
A. It is not safe because the risk of cardiac congenital malformation * [NOT SURE 100%]
B. It is not safe ...
C. It is safe ...
Answer: A
Https://www.drugs.com/pregnancy/paroxetine.html

Http://www.uptodate.com/contents/risks-of-antidepressants-during-pregnancy-selective-serotonin-reuptake-
inhibitors-ssris?Source=outline_link&view=text&anchor=H530686176#H530686176

273.Lady with metromenorrhagia, from 6 month ago and abdominal pain interfere with her activity , what is the best drug ?
A- hysterectomy
B- OCP
C- estrogen analogous
Answer:
-NSAID used for relief of mild to moderate pain. Inhibits inflammatory reactions and pain by decreasing activity of
cyclooxygenase, which is responsible for prostaglandin synthesis.
Acute bleeding (stabilize and IV estrogen or d&c)
Chronic bleeding:
- (anatomical or organic problem>>IV estrogen or d&c if no response after 24 hrs.
-(ocps -long progestin -NSAID)
Http://emedicine.medscape.com/article/255540-medication#7

274.Pregnant in labour with 6 cm dilation and 1+ effacement and spontaneous rupture of membrane, which analgesic to give
A. Pudendal
B. Cervical
C. General
Answer: paracervical
Active phase (stage 1) begins with cervical dilation acceleration, usually by 6 cm of dilation, ending with complete cervical
dilation. (Kaplan page 116)
Paracervical block is administered in the “acneve mhase” of labor. (Kaplan page 123)

65
275.Q about Ocp side effects on the liver ?

A. Hepatocellular carcinoma
B. Sinusoidal diletation
C. Hepatic adenoma

Answer: All but i think more C


Hepatocellular adenomas occur mostly in women of childbearing age and are strongly associated with the use of oral
contraceptive pills (ocps) and other estrogens (
Http://emedicine.medscape.com/article/170205-overview)

276.About pregnant lady with twins on 24 weeks and you discover one fetal death what to do?
A. Delivery + dexamethasone
B. Wait until 34 weeks
C. Wait until 37 weeks
Answer : wait until 34 weeks

277.Pregnant lady come to you with splenomegaly and platelet = 50 and uterus on the level of symphysis pubic , what is
Diagnosis :
A- idiopathic thrombocytopenic purpura
B- gestational thrombocytopenia
C- thromboembolic disease
Answer:

278.Female last delivery with forceps what is complicate :


A- primary postpartum hemorrhage
B- secondary postpartum hemorrhage
C- uterine inversion
Answer:
A primary PPH which occurs within the first 24 PP

279.Old female with endometrial biopsy showing: high grade hyperplasia with atypia. What is the best management?
a) • Trial of OCP.
b) • Total abdominal hysterectomy.
c) • Cauterization.
Answer: B

280.On examination: her cervix is dilated by 3 cm and effaced by 70% and fetal presenting part at 0 station. After 6 hours or
so, her cervix is dilated by 7 cm and effaced by 80%, but she is complaining of tenderness when palpated the uterus, her
temperature is 38. What is the best management?
a) • Give intrapartum antibiotics.
b) • Emergency CS.
c) • Wait for spontaneous delivery.
Answer:A
Ref :uptodate

281.Pregnant in 3rd trimester with pain and bleeding, CTG showed late deceleration. Uterus is distended. What is the
diagnosis?
A. Placenta previa.

66
B. Vasa previa.
C. Abruptio placenta.
Answer:
Causes of late deceleration:
Fetal hypoxia and acidemia, maternal hypotension or uterine hypertonus (Toronto notes OB35)

282.Epithelial cell source in female sample:


A. Chlamydia urethritis
B. Vulva contamination
C. Renal stones
Answer: B

283.Pregnant woman with chronic HYPERTENSION and uncontrolled DM she is on week 10 of gestation what is the
best action
A- bed rest
B- ACE inhibitor
C- termination of pregnancy
Answer: bed rest?
ACEI in contra indicated during pregnancy
284.Pregnant with vag bleeding she delivered normal baby but she bleeding from nose , gum and IV line
A-factor v liden
B-DIC
C- protine s c def
Answer: B

285.Patient G1P0 27ws came sick with high blood pressure was admitted in ICU for observation Dr. Was prescribe
magnesium sulphate what is the indication of such drug
A- prevent the seizer
B- control of high BP
C- something for renal management
Answer:A
Risk of seizure is highest in the 1st 24 hours post-partum -continue mgso4 for 12-24 hours after delivery
References: Toronto notes

286.Patient in routine checkup during pregnancy discover high Blood pressure in 3 time 160/... You will start :
A- methyl dopa
B- atenolol
C- labetalol ( not sure if it was within choices)
Answer:A or C
Prefered antihypertensive agents during pregnancy :Labetalol , Nifedipine, a-methyldopa.
But for severe HYPERTENSION (BLOOD PRESSURE >160/110), give one of labetalol, nifedipine, or hydralazine.
ACEI , ARB, atenolol , prazosin are all contraindicated during pregnancy. Toronto notes

287.Pregnant with chest infection :


A. Trimethoprim/sulphamethaxazol
B. Augmentin
C. Ceftraioxon
Answer:

288.Patient has history of gonoreheai think came with complain of infertility hystroscopy done with result of normal uterus
but dye cant be seen freely from tubes ( tubal block) what is the best way for lady to conceive :
A. In vitro fertilization and embryo transplant
B.induction of ovulation

67
C. Clomophen
Answer:A

289.25 ys old female came to ER with sever Rt lower abdominal pain she has history of aminorrhea for 2 month what is the
diagnosis :
A. Rupture tubal pregnancy.
B. Acute appendicitis
C. Diverticulitis
Answer:a

290.Lady came with history of infertility. She has BMI of 30 ( and other features of PCO) which of the following will help her
to conceive :
A) wt reduction
B) in utero fertilization
C) Clomophine
Answer: C
Clomiphene citrate, human menopausal gonadotropins, LHRH, recombinant FSH, and metformin. Toronto notes
Induction
291.Pregnant woman diagnosed with gestational diabetes what is the treatment should be started
A) Insulin
B) Metformin
C) Sulphanylurea
Answer: A.
First line is management through diet modification and increased physical activity and initiate insulin therapy if glycemic targets
not achieved within 2 wk of lifestyle modification alone

292.Female pregnant lady with hyperthyroidism but not symptomatic what is the management? I DON"T KNOW
A) MMI theantithyroid
B) PTU the antithyroid
C) b blocker
Answer:
PTU recommended in 1st trimester, MMI during 2nd and 3rd trimester.
Symptomatic treatment with β-blockers.
Toronto notes

293.Female patient she’s 30 yers old she did pap smear yearly for 9 years ,, all normal ,, this time pap smear showed low
grade …. What is your management I DON”T KNOW
A) remove the lision by electrical something
B) repeat after 1 year
C) colposcopy
Answer: C

294.65 years old female patient at night she wake up and want to urinate but she urinate before arrive to path : #ob
A-urgency
B-overflow
C-or...or…
Answer: a
URGE INCONTINENCE: urine loss associated with an abrupt, sudden urge to void

295.18 year-old girl presented to the clinic complained of amenorrhea for almost 2 consecutive months. She denied the
pelvic examination. What is your next step ?

68
A. TRH, TSH, T4, T3
B. Brain MRI
C. B-Hcg Urine Test
Answer:c

296.Best way to diagnose Adenomyosis ?


A. Histology section of hysterectomy
B. Endometrial Biopsy
C. Pelvic MRI
Answer:A
The only definitive diagnosis is by histologic confirmation of the surgically excised tissue. Kaplan

297.Menopause lady came with vaginal spotting , on examination there was cystic nodule ir )cervix examination showed
tumor what to do?
A. -cone biopsy
B. -Directed biopsy
C. -Pap smear
Answer:
298.Q: pregnant lady with hypotension, what type of anesthesia you will give her:
A- pedundal.
B- epidural.
C- general.
Answer: C

299.Young lady came to clinic complained of not being pregnant for 2 years. She has dysmenorrhea. What is your diagnosis ?
A. Endometriosis *
B. Endometritis
C. Leiomyoma
Answer: a

300.Female abdominal pain examination tender nodular retroverted uterus what investigation?

A. Laparoscopy
B. Hysteroscopy
C. Hysterosalpengiogram
Answer: A

301.Multiparous with cervical dysplasia, has chlamydia and HSV 2, what is the cause of her dysplasia ?

A. Chlamydia
B. HSV
C. HPV

Answer: C
(Uptodate) Human papillomavirus (HPV) is the major etiologic agent of cervical precancer and cancer. The association between
HPV and cervical neoplasia is so strong that most other behavioral, sexual, and socioeconomic covariables have been found to
be dependent upon HPV infection and do not hold up as independent risk factors.
●HPV infection is necessary but not sufficient to develop cervical neoplasia. The two major factors associated with development
of high-grade CIN and cervical cancer are the subtype of HPV and persistent infection. Environmental factors (eg, cigarette
smoking) and immunologic influences also appear to play a role.

69
●Low-oncogenic-risk HPV subtypes, such as HPV 6 and 11, do not integrate into the host genome and only cause low-grade
lesions (eg, low-grade SIL and CIN 1) and benign genital warts
●Hegh-oncogenic-risk HPV subtypes, such as 16 and 18, are strongly associated with high-grade lesions, persistence, and
progression to invasive cancer, but also cause low-grade lesions.
●The primary approach to prevention of CIN and cervical cancer is HPV vaccination. Although HPV is a sexually transmitted
infection, condoms are only partially protective. For women with CIN, appropriate monitoring and treatment are used as
secondary prevention of cervical cancer.

302.Recurrent Bartholin gland cysts after incision & catheter placement, how to manage:

A. I&D
B. Incision & catheterization
C. Marsupialization

Answer: C
This procedure is reserved for recurrent abscesses. The acute abscess is drained prior to marsupialization. This procedure
consists of a wide incision of the mass followed by suturing the inner edge of the incision to external mucosa. This complicated
procedure is usually performed by a gynecologist or urologist in the OR.
Http://emedicine.medscape.com/article/1894499-overview
303.Pregnant lady at 34 weeks present with regular contractions, dilated cervix 3 cm, fetal station .., US reveals a back toward
the cervix in transverse lie with echo lucent area behind the placenta , what to do ?

A. Tocolytic
B. Induce labor
C. CS

Answer: we need to know if the membrane ruptured


Transvers lie:
• INTACT membrane:
-Before and early labor: ECV at 38-39 weeks follows by artificial rupture of membrane
If ECV unsuccessful ~> CS
- active labor: CS
• Ruptured membrane:
- >= 34 CS
- < 34 expectant management
• Second twin transverse: internal podalic version to breech then deliver while membrane is still intact
•Prenerm labor: regular contractions (2 in 10 min) • cervix >2 cm dilated, 80% effaced, or documented change in cervix
• Tocolytics: requirements (all must be satisfied) ƒ .preterm labor ƒ .live, .immature fetus, .intact membranes, .cervical dilatation
of <4 cm ƒ .absence of maternal or fetal contraindications. (Kaplan OBS and gyne)

304.Pregnant in labor, about 80% effacement, 4 cm dilation, +1 fetal station, rupture of membrane, (they give lab values
which was low Hb & low platelet) what type of anesthesia?
A. General anesthesia
B. Para cervical
C. Pudendal

Answer: C
Http://emedicine.medscape.com/article/83078-overview#a2

305.Epithelial cells in female urine sample;


A. Chlamydial urethritis
B. Outer vulva contamination

71
C. From the cervix
Answer:
Generally 15-20 squamous epithelial cells/hpf or more indicates that the urinary specimen is contaminated. Mostly none of the
choices above is true because such a finding usually found if the disease in the upper urologic system.
Http://emedicine.medscape.com/article/2074001-overview#a2

306.Pap shows ASCUS, estrogen trial for some duration, Pap again show ASCUS; what is next
A. Colposcopy
B. Hysterectomy
C. F/U in next year

Answer: A
If the patient had a pap smear with ASCUS then repeated after 4-6 months with the same result do colposcopy.
Kaplan obs and gyne diorders of the cervix and uterus

307.Scenario, about a female has bright red spots what's the source of this blood?!
A. Uterine
B. Cervix
C. Vulva
Answer: ???
• heavy bleeding: uterus
• staining, spotting, light bleeding: genital tract
• brown: uterus, cervix, upper vagina
• red: genital tract
• postcoital: cervical

308.Case of Premature Preterm Rupture of Membranes at 32 weeks what to do?


A. Sterile speculum exam
B. Vaginal exam
C. Chemical investigation of liquor

Answer:
Original Answer: C
If the presentation of this patient gush of fluid or leakage (American Associated Family Medicine)
NCBI Answer: A
Preterm PROM is largely a clinical diagnosis. It is typically suggested by a history of watery vaginal discharge and confirmed on
sterile speculum examination. The traditional minimally invasive gold standard for the diagnosis of ROM relies on clinician ability
to document 3 clinical signs on sterile speculum examination: (1) visual pooling of clear fluid in the posterior fornix of the vagina
or leakage of fluid from the cervical os; (2) an alkaline ph of the cervicovaginal discharge, which is typically demonstrated by
seeing whether the discharge turns yellow nitrazine paper to blue (nitrazine test); and/or (3) microscopic ferning of the
cervicovaginal discharge on drying.
Http://www.ncbi.nlm.nih.gov/pmc/articles/PMC2492588/

309.HELLP syndromes:

A. Hypertension,........, low enzyme


B. Hypertension, ,....., high enzyme

70
C. Hemolysis , Elevate liver enzyme, low platelet

Answer: C
HELLP syndrome, named for 3 features of the disease (hemolysis, elevated liver enzyme levels, and low platelet levels)
Http://emedicine.medscape.com/article/1394126-overview

310.Seven weeks pregnant lady c/o vaginal bleeding with tissue. Her cervix was open and you can see some product of
conception. Her fundal height is equal to 7 to 8 weeks.
A. Threatened abortion
B. Incomplete abortion
C. Missed abortion
Answer : B
Complete abortion: no product of conception found.
Incomplete abortion: some product of conception found
Inevitable abortion: product of conception intact, dilated cervix, vaginal bleeding.
Threatened abortion: product of conception intact, no cervix dilatation, intrauterine bleeding.
Missed abortion: death of fetus, but all products of conception present in uterus.
Septic abortion: infection of the uterus and surrounding area.
Source: Master the boards USMLE step 2 CK
311.Old lady postmenopausal with osteoarthritis and risk for osteoporosis, what you will do:
A. Calcium ,TSH ,dihydroxy vit D
B. Bisphosphonate, vit D, calcium
C. DEXA scan
Answer: C We recommend pharmacologic therapy for postmenopausal women with a history of fragility fracture or with
osteoporosis based upon bone mineral density (BMD) measurement (T-score ≤-2.5)
Uptodate

312.Patient with polyhydramnios what atresia?


A. Kidney
B. Esophagus
C. Duodenal
Answer: C

313.Lady atypical cervical cell, the doctor can't see cervix well in colposcopy, what the appropriate next:
A. Repeated pap smear
72
B. Repeated colposcopy
C. Cone biopsy
Answer: D or C

314.Female with DM well controlled, she wants to get pregnant, to avoid the complication, DM control should be…
A. Started before pregnancy
B. 1st trimester
C. 2nd trimester

Answer: A

315.Pregnant lady in the 8 weeks gestation came with history of bleeding for the last 12 hours + abdominal pain, she passed
tissue. On Examination, os is opened, uterus is 7-8 weeks in size, Diagnosis?
A. Incomplete abortion.
B. Threatened abortion.
C. Molar pregnancy.

Answer: A

Kaplan:

316.Pregnant women is Rh +ve and her baby is Rh -ve .what will happen to the mother?
A. No reaction.
B. Mild hemolysis.
C. Hydrops fetalis.

Answer: A

RESOURCE: PREVIOUSLY WRITTEN!


It's due to Rh- mother and Rh+ baby..
Rh disease is also called erythroblastosis fetalis during pregnancy. In the newborn, the resulting condition is called
hemolytic disease of the newborn (HDN).
Some of the more common complications of Rh disease for the fetus and newborn baby include the following:
o Anemia: (in some cases, the anemia is severe with enlargement of the liver and spleen)
o Jaundice: yellowing of the skin, eyes, and mucous membranes.
o Severe anemia with enlargement of the liver and spleen
o Hydrops fetalis: this occurs as the fetal organs are unable to handle the anemia. The heart begins to fail and
large amounts of fluid build up in the fetal tissues and organs. A fetus with hydrops fetalis is at great risk of
being stillborn.

73
317.Female after delivery started to develop pelvic pain, fever, & vaginal discharge -,
There's test mentioned in the Q . What’s the diagnosis?
A. PID
B. Bacterial vaginosis
C. Vaginal yeast

• >20% clue cells = squamous epithelial cells dottedwithcoccobacilli(Gardnerella) • Paucity of WBC • paucityoflactobacilli •
Positivewhifftest:fishyodorwithadditionof KOH to slide (due to formation of amines)

318.Pregnant lady in her 8 week of gestation came and complain that she loss pregnancy sensation & there is vaginal
spotting. What to do to establish the diagnosis
A. Mother serum AFP
B. Trans vaginal US
C. Serum b HCG
Answer: B

319.Which of the following is contraindicated for assistant delivery by forceps?


A. Breach presentation
B. Face presentation
C. Cephalopelvic disproportion
Answer: C

320.Pregnant lady in her 41 wk of gestation admitted for delivery induction. After oxytocin was given she start having
contraction and there is 4 cm dilation & 60 % effacement. After one hour there is 8 cm dilatation of the cervix & 80%
effacement. Baby pulse is 120-140, also there is acceleration & variability. What is the correct action to do
A. Expectant delivery
B. Stop oxytocin
C. Go immediately for CS
Answer: A

321.Which of the following drug safe during pregnancy


A. Erythromycin
B. Cephalosporin
C. Warfarin surgery
Answer: A

322.Patient presented with lower diffuse abdominal pain , the cervix was normal during P/E there was chandelier sign
positive (Cervical motion tenderness) , what is the most likely diagnosis ?
A. Pelvic inflammatory disease
B. Uterine abnormality
C. Ectopic pregnancy
Answer: A
Cervical motion tenderness is more common with PID
In cases of ectopic pregnancy Typically presents with RLQ pain. PID can exist concurrently with ectopic pregnancy.
Positive pregnancy test will guide search for ectopic pregnancy: hcg hormone level is high in serum and urine.Ultrasound reveals
an empty uterus and may show a mass in the fallopian tubes.

74
323.Lady presenting with lower abdominal pain when you did U/S you found tubuloovarian abscess , so what you will do ?
a. Emergent laparotomy
b. Aspiration of the abscess by laparascopy C. CT guided aspiration
c. IV antibiotic

Answer: C
Https://en.wikipedia.org/wiki/Tubo-ovarian_abscess

324.Pregnant in 2nd trimester hx of tiredness in first now she is ok all labs normal ex hemoglobin level 10 so management:
A. Iron
B. Folic acid
C. None
Answer: A

325.Young female has severe attack of headache anxiety and palpitation she also have lost weight and her skin looks (i forgot
the word) which test will order:
a. Brain MRI
b. Urine catecholamine
c. TSH

Answer: C

326.Pregnant is in her 38 weeks gestation with a blood pressure of 140 over 90. No proteinuria and completely asymptomatic
what will you do:
a. Immediate delivery
b. Antihypertensives
c. Observation (frequently)

Answer: A
Pre eclampsia case
At >36 weeks' gestation: delivery is the most sensible approach.
Http://bestpractice.bmj.com/best-practice/monograph/326/treatment/step-by-step.html

327.Lady with lower abdominal pain. Vaginal examination reveals suprapubic and fornices tenderness with purulent vaginal
discharge?
A. Acute cervitis
B. Acute salpingitis
C. Acute appendicitis

Answer: B
Explanation: PID is an upper genital tract infection most commonly caused by chlamydia and gonorrhea
The initial infection is acute cervicitis which has no symptoms. However, vaginal exam reveals signs most commonly as
mucopurulent cervical discharge or a friable cervix.
Acute salpingo-oophritis: patient complains of bilateral abdominal/pelvic pain often after menses. On examination: there is
mucopurulent discharge and cervical motion tenderness.
Reference: Kaplan OB/GYN step2 lecture notes

328.During delivery the cord is before the fetal head, management?

75
A. C/S.
B. Vacuum.
C. Forceps.

Answer: A
Explanation: The gold Standard obstetrical management of cord prolapse in the setting of a viable pregnancy typically involves
immediate delivery by the quickest and safest route possible. This usually requires cesarean section, especially if the woman is in
early labor to avoid fetal compromise or death from compression of the cord. However, vaginal delivery may be a reasonable
option in select cases when delivery is imminent.
Reference: http://cursoenarm.net/UPTODATE/contents/mobipreview.htm?38/0/38926?Source=see_link#H1942694

329.Small for gastitional date us show storm form appearance:


A- Complete hydatiform
B- Partial hydatiform
C- Somthing ca

Answer: A
Explanation: U/S findings in molar pregnancies: if complete: no fetus (classic “snow snorm” due to swelling of villi(. If partial:
molar degeneration of placenta ± fetal anomalies, multiple echogenic regions corresponding to hydropic villi, and focal
intrauterine hemorrhage
Reference: Toronto Notes 2015, Page GY45

330.Pregnant on 16 weeks ,Rh (-) what u will do ??


A- amniocentesis
B- give her rh antibody
C- coombs test

Answer: C
Explanation: for RH incompatibility, routine screening with indirect Coombs test at first visit for blood group, Rh status, and
antibodies.
Reference: Toronto Notes 2015, page OB8, Obstetrics

331.Women c/o vesicles of vulva and cervix ? What is organism ?


A- herpes simplex
B- gonorrhea
C- trichomonas

Answer: A
Explanation: Herpes simplex present with painful vesicular lesions on the vulva and cervix. Gonorrhea and trichomonas do not
cause ulcers.
Reference: Kaplan OB/GYN lecture notes 2014, page 57, and 203-204

332.Women in postpartum , she said that she complete her family and ask about contraception . She is exclusively breast
feeding her newborn .and she said that the menses in previous came at 10th month postpartum ?
A- give her OCP
B- progestrone injection
C- wait as previous until 10month

Answer: B
Explanation: for breast feeding women current evidence indicates that progestin-only contraceptives do not appear to have an
adverse effect on breast milk volume during the first 6 weeks postpartum and also have no adverse effect on infant growth and

76
development throughout the duration of their use. Generally recommended that breastfeeding mothers not use hormonal
methods that contain estrogen.
Reference: http://www.medscape.com/viewarticle/565623_5

333.Contraindications to do instrumental delivery


a. Placenta abruption!
b. Face presentation!
c. Breech presentation

Answer: b

334.Case of gonorrhea, what are you going to give his close contacts:
A. Rifampin chemoprophylaxis
B. Isolate all contacts for 4 weeks
C. Meningiococal vaccine

Answer:b
Http://www.cdc.gov/std/tg2015/gonorrhea.htm

335.Most common complication after hystrectomy:


a. Bleeding 1-3%
b. Bladder injury 2%
C. Ureter injury 1.6%

Answer: A
The most serious postoperative complication of hysterectomy is hemorrhage, which occurs in 1% to 3% of patients. Ureteral
injuries are common, the incidence is reported to be 0.5% for hysterectomy performed for benign disease and up to 1.6% for
laparoscopically-assisted hysterectomy. Bladder injuries occur in up to 2% of hysterectomy cases.
Http://www.medscape.com/viewarticle/582384_4

336.Pap shows ASCUS, estrogen trial for some duration , Pap again show ASCUS ; what is next
a. Colposcopy
b. Hysterectomy
c. F/U in next year
Answer:

77
USMLE Step2CK 8th edition pg 339

337.Pregnant lady at 34 weeks present with regular contractions , dilated cervix 3 cm, fetal station .., US reveals a back
toward the cervix in transverse lie with echo leucent area behind the placenta , what to do ?
A. Tocolytic
B. Induce labor
C. CS

338.46 year old woman G2P2 expressed that she want to get pregnant again, but she had amenorrhea since 7 months now.
What will consider before you can tell her wither she can or can't get pregnant?
a. LH and FSH Level
b. Estrogen level
c. Prolactin level

Answer: Maybe based on the age the patient can have peri -menopause but she’s not considered menopause until loss of
menses for 12 months?

Is it safe for a mother infected with hepatitis C virus (HCV) to breastfeed her infant?
Yes. There is no documented evidence that breastfeeding spreads HCV. Therefore, having HCV-infection is not a contraindication
to breastfeed. HCV is transmitted by infected blood, not by human breast milk. There are no current data to suggest that HCV is
transmitted by human breast milk. However, HCV-positive mother's nipples and/or surrounding areola are cracked and
bleeding, she should stop nursing temporarily.

339.Pregnant lady with nausea and vomiting and abdominal pain, what’s your first priority in management?
A- IV fluids
B- Pain management

78
C- IV Antibiotics
Answer : A

340.HCV infant, mother asking about breastfeeding?


A- Treat the baby then breastfeed.
B- Continue breast feeding.
C- Stop breastfeeding.
Answer: B
Ref.: http://www.cdc.gov/breastfeeding/disease/hepatitis.htm

341.Pregnant women in labor pain her abdomen distended to xyphoid process US show breach presentation ,intact
membrane , fully dilated and effaced , station 0 what is the best management of the case
a. Cesearion section
b. Amniotomy
c. Extension breach delivery
Answer is :?

Criteria for Vaginal Breech Delivery : Frank or complete breech, GA >36 wk • EFW 2,500-3,800 g based on clinical and U/S
assessment (5.5–8.5 lb) • Fetal head flexed • Continuous fetal monitoring • 2 experienced obstetricians, assistant, and
anesthetist present • Ability to perform emergency C/S within 30 min if required
C/S recommended if: the breech has not descended to the perineum in the second stage of labor a er 2 h, in the absence of
active pushing, or if vaginal delivery is not imminent a er 1 h of active pushing
Contraindications to vaginal breech delivery:cord presentation,clinically inadequate maternal pelvis, fetal factors incompatible
with vaginal delivery

342.Best modality in diagnosis DVT in pregnancy


a. D-dimer
b. Doppler US
c. Impedance Plethysmography

Answer:B or C

In pregnancy, compression US should be performed with the patient in the left lateral decubitus position and with Doppler
analysis of flow variation during respiration to maximize the studies ability to diagnose pelvic DVT. D-dimer increases
progressively throughout gestation,[38] adding to the difficulty in selecting an appropriate cut off value for reasonable
specificity in pregnancy. Impedance plethysmography is both sensitive and specific for the diagnosis of proximal deep vein
thrombosis in symptomatic patients when venography is the reference standard.In contrast, impedance plethysmography is not
sensitive for the detection of proximal deep vein thrombi in asymptomatic high-risk patients such as patients who have recently
undergone hip arthroplasty or stabilization of hip fracture.

Reference: http://www.medscape.com/viewarticle/410882_2
Http://emedicine.medscape.com/article/2056380-workup#showall
Reference: Toronto notes

343.Minimal test to check in preeclampsia :


A. Creatinine , liver enzymes , platelet
B. Creatinine, liver enzyme , Htc -platelet, uric acid , liver enzymes
C. Platelet , uric acid , creatinine

Answer: B

79
Elevation in hemoconcentration is shown by elevation of Hb, HCT, BUN, serum ceraitnine and serum uric acid.
Reference: Kaplan USMLE Step 2 OBGYN

344.7 weeks gastation woman c/o bleeding associated with tissue passage, cervix is open:
A) Inevitable abortion
B) Threatened abortion
C) Incomplete
Answer: C
The answer is incomplete abortion which involves bleeding, passage of some tissue and cervical dilation. * refer to the picture
below taken from first aid gynecology.

Monozygot twin presentation (twin A/twin B)will dengioras in :


A) Cephalic. Cephalic
B) ransver. Cephalic
C) Breach. Cephalic
Answer: A
According to Mudaliar and Menon's Clinical Obstetric book, 10th edition, page 184:
The most common presentation is Cephalic-Cephalic then Cephalic-Breech
Reference:
https://books.google.com.sa/books?Id=jvgworkjhhcc&pg=PA186&lpg=PA186&dq=identical+twins+breech+cephalic+most+com
mon&source=bl&ots=-56D-
lyqqc&sig=MC2qZQVy4Q6940lVTNw7cAxnBs0&hl=ar&sa=X&ved=0ahukewi3j_Xp5q_nahwcvbqkhshyan0q6aeictap#v=onepage&
q=identical%20twins%20breech%20cephalic%20most%20common&f=false

345.Pregnant women (38 w) her BLOOD PRESSURE is 140/90, no proteinuria ... What is the appropriate treatment?
A) Do CS
B) Observation
C) Some thing
Answer: B
"Therefore, pregnant patients should be started on antihypertensive therapy if the SBP is greater than 160 mm Hg or the DBP is
greater than 100-105 mmhg."
Reference: http://emedicine.medscape.com/article/261435-overview#a21

346.What’s true regarding cervical cancer?


81
A. Most of the lesion progress to malignancy
B. CIN requires months to progress to malignancy
C. Pap smear decrease its incidence (my answer)

347.A patient 39 week in labor. You ran a reactive cardiotocography (CTG), on examination you feel orbital margin nose and
chin. How will you manage her?
A. Delivered her in operating room
B. Emergency c/s
C. Oxytocin

Answer: C
- According to the question, it is a FACE Presentation.
- Continuous electronic fetal heart rate monitoring is considered mandatory because of the increased incidence
of abnormal fetal heart rate patterns and/or nonreassuring fetal heart rate patterns.
- Fetuses with face presentation can be delivered vaginally with overall success rates of 60-70%
- Cesarean delivery is performed only for the usual obstetrical indications.
- Oxytocin can be used to augment labor
- Forceps may be used if the mentum is anterior
- Http://emedicine.medscape.com/article/262341-overview#a4

348.Pregnant 8 week of gestation presented with severe abdominal pain followed by heavy bleeding. Examination revealed
tense abdomen. What is most likely diagnosis?
A. Threatened abortion
B. Ectopic pregnancy
C. Ovarian failure
Answer: B
- Ectopic pregnancy: A fertilized ovum implanting and maturing outside of the uterine endometrial cavity. The
most common site being the fallopian tube (Oviduct-Distal Ampulla)
- EP Triad: Secondary amenorrhea + Unilateral abdominal or pelvic pain + Vaginal bleeding
- Symptoms generally appear 6 to 8 weeks after the last normal menstrual period, but they can occur much
later in cases of non-tubal ectopic pregnancy
- To confirm the diagnosis: B-hcg titer > 1500 miu + No intrauterine pregnancy is seen with vaginal sonogram
- Http://bestpractice.bmj.com/best-practice/monograph/174/basics/definition.html

349.Patient with preterm symptoms. Cephalic presentation. CTG done showed contraction every 10 min vaginal exam
showed 1 cm -3 station managed with hydration and steroid.
What is the best next step to confirm the diagnosis?
A. Vaginal exam
B. CTG
C. Lung maturation
Answer: A (Sterile speculum examination)
Preterm labor triad (used to confirm the diagnosis): Pregnancy 20-36 weeks + 3 or more contractions in 30 min + cervical
dilation of 2 cm or more. Other common symptoms: abdominal pain\ low back pain\ vaginal bleeding
- KAPLAN Step 2: Obstetrics and Gynecology

350.25 year-old female was diagnosed with pelvic inflammatory disease 3 years ago which was completely resolved. She
presented with inability to conceive for 3 months of trying. Investigation were normal, and semen analysis was normal as
well

80
Her BMI is 35
LH and FSH were low
How will you manage her?
A. Induction ovulation and IVF
B. Induction ovulation and normal conceive
C. Advice here to reduce Her BMI to 23 and trying to get pregnant

Answer: C (NOT SURE)


Since this patient has high. Women with elevated baseline weight or body mass index (BMI) greater than 27 kg/m2 and
anovulatory infertility should be advised to lose weight
- Https://yhdp.vn/uptodate/contents/mobipreview.htm?13/10/13482
- Https://yhdp.vn/uptodate/contents/UTD.htm?14/5/14425?Source=see_link

351.A case of pregnant lady whose baby was breech and small in size. She came at 38 weeks of gestation for external cephalic
version. Upon US, bicornuate uterus was discovered, baby’s head is flexed. Doctor decided to do CS, why?
A. Bicornuate uterus
B. Baby’s head flexed
C. Size of baby
Answer: A
Obstetric outcomes in bicornuate uterus depends on the length of the muscular septum i.e. Whether the
bicornuate uterus is partial or complete. Outcomes were worse in complete bicornuate uterus.
Small baby size and flexed head are not indications for cesarean delivery so the only option is bicornuate uterus.
Source: http://emedicine.medscape.com/article/273534-overview

352.A case of pregnant lady who’s complaining of severe pain and bleeding. US was done and showed fibroid and viable
fetus, what to do?
A. Hysterectomy
B. Termination of pregnancy
C. Analgesia
Answer: C
Fibroid pain during pregnancy is usually managed conservatively by bed rest, hydration, and analgesics. Use nsaids with
caution. Rarely, severe pain may necessitate additional pain medication (narcotic analgesia), epidural analgesia, or
surgical management (myomectomy).
It is rare for fibroids to be treated surgically in the first half of pregnancy. If necessary, however, several studies have
reported that antepartum myomectomy can be safely performed in the first and second trimester of pregnancy.
Source: http://www.ncbi.nlm.nih.gov/pmc/articles/PMC2876319/

353.A female cannot get pregnant, and tried for 3 months and she is normal, regular menstrual cycle, and husband is normal
what to do?
A. Try more
B. Semen analysis
C. Genetic study
Answer: A
Infertility is defined as not being able to get pregnant (conceive) after one year of unprotected sex. Women who do not have
regular menstrual cycles, or are older than 35 years and have not conceived during a 6-month period of trying, should consider
making an appointment with a reproductive endocrinologist—an infertility specialist.
Http://www.cdc.gov/reproductivehealth/infertility/

rd
354.A pregnant lady in her 3 trimester is complaining of swelling in her lower limbs. What will you do?
82
A. Venogram <bed rest <heparin
B. Doppler, bed rest >heparin
C. Clinical, bed rest < warfarin
Answer: B
DVT risk factors is as follows: Age, immobilization longer than 3 days, pregnancy and the postpartum mereod…enc . The American
Academy of Family Physicians (AAFP)/American College of Physicians (ACP) recommendations for workup of patients with
probable DVT. In patients with intermediate to high pretest probability of lower-extremity DVT, ultrasonography is
recommended
Http://emedicine.medscape.com/article/1911303-treatment

355.Site of fibroid that cause abortion?


A. Submucosal
B. Intramural
C. Serosal

Answer: A
A recent systematic review found the spontaneous miscarriage rate to be higher in women with submucosal and intramural
fibroids (in descending order) undergoing IVF, compared with women with no fibroids; however, only 11 controlled studies were
analysed (Klatsky et al., 2008)."
Medscape: http://www.medscape.com/viewarticle/753718_4

356.3 months pregnant woman scenario of Bacterial vaginosis what to give?


A. Oral metronidazole
B. Ceftriaxon
C. Cream
Answer: oral metronidazole
Bacterial vaginosis is a clinical syndrome caused by excessive growth of bacteria that may normally be present in the vagina, The
etiology is polymicrobial in nature; with a ph of more than 4.5, Gardnerella vaginalis and anaerobes become the prominent
associated organisms. Treat BV occurring in pregnant women to reduce the risk of pregnancy-associated complications related
to infection (premature labor unresponsive to tocolytic therapy). Oral metronidazole or clindamycin is recommended;
clindamycin cream should be avoided during the second half of pregnancy
Medscape: http://emedicine.medscape.com/article/235054-overview

357.Pregnant with asthma scenario came with SOB, what to do?


A. Chest X-ray
B. Spirometry
C. CT
Answer: B
Lung function tests, such as spirometry, are useful for distinguishing the shortness of breath associated with a worsening of
asthma from the normal shortness of breath that many women experience during pregnancy. (uptodate)

358.Lady with lower abdominal pain. Vaginal examination: fornices tenderness, suprapubic tenderness and purulent vaginal
discharge. What is the diagnosis ?
A. Acute cervicitis
B. Acute salpingitis
C. Acute appendicitis
Answer: B
Salpingitis is an infection and inflammation in the fallopian tubes. It is often used synonymously with pelvic inflammatory
disease (PID).
Symptoms: (1) Lower abdominal pain is usually present. The pain is typically described as dull, aching or crampy, bilateral, and
constant; it begins a few days after the onset of the last menstrual period and tends to be accentuated by motion, exercise, or
coitus (2) Abnormal vaginal discharge is present in approximately 75% of cases. (3) Unanticipated vaginal bleeding.
Physical signs: (1) Cervical motion tenderness (2) Uterine tenderness (3) Adnexal tenderness

83
359.Assessment of delivery by?
A. Number of contractions
B. Force of contractions.
C. Fetal station.
Answer: C
Evaluation of status of labor, including a description of uterine activity, cervical dilation and effacement, and fetal station and
presentation, unless vaginal exam deferred; evaluation of fetal status, including interpretation of auscultation or electronic fetal
monitoring strips.

360.36 weeks gestational age experiencing uterine contraction every 3 minute, each contraction lasting 30 second,
hypertensive (BLOOD PRESSURE 160\ 100). What will you do:
A. CS
B. Induce labor
C. Tocolytic
Answer: A

361.Massive bleeding 2 hours after delivery. What is the management


A. Ringer lactate
B. Blood transfusion
C. Methergine
Answer: B

362.Pregnant women complaining of UTI at 12 weeks of gestation then treated // now complaining of dysuria , ... She take ()
medication for 4 days ,, what u will do :
A. Give her small dose Abx till delivery
B. Change drug
C. Treat even asymptomatic
Repeated without choices

363.Lactating women with mastitis?


A. Continue breastfeeding
B. Draining
C. Antibiotic
Answer: Supportive treatment includes the use of analgesics and warm compresses. Breasts should be frequently emptied of
milk through continued nursing or pumping (should be initiated with the uninfected breast). If symptoms are not improving
within 12-24 hours or if the woman is acutely ill, antibiotics should be started. If an abscess develops, consider irrigation and
debridement along with IV antibiotics.
Http://emedicine.medscape.com/article/2028354-overview

364.Female after birth was experiencing excessive crying for short period then resolved.
A. Maternal blues
B. Postnatal depression
C. Postpartum psychosis
Answer: most likely A
blues: Symptoms peak on the 4-5 day after delivery and last for several days, spontaneously remit within the first 2 postpartum
weeks
depression: > 2 wks, develops most frequently in the first 4 months following delivery but can occur anytime in the first year.
Http://reference.medscape.com/article/271662-overview#showall

84
365.Pregnant lady came to the clinic at 10 week with 2 hr bleeding, examination revealed close Os, the fundus is palpable ?
Cm above symphysis pubis. What is the cause?
A. Rupture cyst
B. Inferiorly located placenta
C. Ectopic pregnancy
Answer: probably C Sx of ectopic preg.: early pregnancy bleeding, pelvic pain, adnexal tenderness or mass, uterus may be
slightly enlarged (but less than anticipated based on date of LMP or # of weeks)
Could be abortion (threatened) because there is no cervical dilation. Merck Manual

366.G7P0 Gave birth to 4 Kg Baby, blood loss was 800 ml


A- primary hemorrhage
B- secondary hemorrhage
C- tertiary hemorrhage
Answer: A
Primary hemorrhage – bleeding that occurs within the intra-operative period.
Reference: http://teachmesurgery.com/post-operative/early/haemorrhage/

367.Pregnant lady has asthma, before pregnancy she used to have 3 attacks/ week using salbutamol, now she is 30 weeks
pregnant and her symptoms relieved spontaneously, why does her asthma improve with pregnancy?
A- Increase TV
B- Progesterone effect on smooth muscles
C- improve expiratory I dnk what
Answer: A
Reference: http://emedicine.medscape.com/article/796274-overview#a2

368.The doctor convert to Caesarean Section, what is the case to do CS?


A- Age of fetus
B- Presentation
C- Rupture of membrane
Answer: B
Indications for CS:
Possible indications include:
- Cephalopelvic disproportion
- Malpresentation - eg, breech, transverse lie.
- Multiple pregnancy.
- Severe hypertensive disease in pregnancy.
- Fetal conditions: distress, iso-immunisation, very low birth weight.
- Failed induction of labour.
- Repeat caesarean section.
- Pelvic cyst or fibroid.
- Maternal infection (eg, herpes, HIV)

85
369.Which of the following tests are minimum requirements for screening of pre-eclampsia:
A-Platelets count, creatinine level & liver enzymes
B- HCT, 24 urine protein & liver enzymes
C- HCT, creatinine level & liver enzymes
Answer: A
In a patient with new-onset hypertension without proteinuria, the new onset of any of the following is diagnostic of
preeclampsia:
- Platelet count below 100,001/μl
- Serum creatinine level above 1.1 mg/dl or doubling of serum creatinine in the absence of other renal disease
- Liver transaminase levels at least twice the normal concentrations
- Pulmonary edema
- Cerebral or visual symptoms
Please read more about the criteria for diagnosing pre-eclampsia: http://emedicine.medscape.com/article/1476919-overview

370.Pregnant g3p2 in labor, cervical dilatation 3cm, effacement 100% membrane rupture, after 3 hours still 3 cm,
A. C-section
B. Oxytocin
C. Waiting
Answer: She still in latent phase (less than 4 cm), so wait but should not exceed 14 hours the phase.

371.A Triad of ascites pleural effusion and ovarian mass, what is the most likely tumor?
A- Sex cord stromal tumors
B - Epithelial tumors
C- Germ cell tumors
Answer: A
Meigs syndrome is defined as the triad of benign ovarian tumor with ascites and pleural effusion that resolves after resection of
the tumor.

372.Child vaginal bleeding + breast +mass in pelvic?


A- Ovarian teratoma
B- Granulosa theca
C- Yolk sac tumor
Answer: B
Patients usually present with precocious pseudopuberty (70-80%) and have secondary sex characteristics at a very early age.
These may include increased linear growth, breast enlargement, clitoral enlargement, pubic hair development, increased vaginal
secretions, and vaginal bleeding.

373.Pregnant lady, asymptomatic, UA: 50,000 CFU bacteria WBC: 2 Diagnosis?


A- Cystitis
B- Pyelonephritis
C- Asymptomatic Bacteriuria
Answer: C

Asymptomatic bacteriuria is defined as isolation of a specified quantitative count of bacteria in an appropriately collected urine
specimen from an individual without symptoms or signs of urinary tract infection.

86
374.Pregnant women complaining of UTI at 12 week then treated, now complaining of dysuria, She take () medication for 4
days, what u will do:
A- Give her small dose Abx till delivery
B- Change drug
C- Treat even asymptomatic
Answer: C

375.How to obtain a pap smear:


A) 3 specimens from internal canal
B) Two specimens from different sites
C) One specimen from cervical os
Answer: B
Http://www.cytopathology.org/specimen-collection-adequacy-requisition/

376.Pregnant women with no prenatal history. Present with regular uterine contractions every 5 mins, cervical dilation 10
cm. On examination baby is breech and neck is extended. What to do?
A. CS
B. Vaginal delivery
C. Breech extraction
Answer: I thick B

377.A woman after peuperium developed fecal & urine incontinence Ddx
A) partial perineal injury
B) pp sepsis
C) fistula
Answer: A
Link: http://www.ncbi.nlm.nih.gov/pmc/articles/PMC3279110/

378.Pregnant lady in the week 41 of gestation, effacement is 50%, 2 cm dilated for the past 2 weeks, now the effacement is
60%, dilated 3 cm, the fetal condition is good based of CTG findings, what is your next step in her management?
A- give oxitocin and amniotomy.
B- give …. And amniotomy.
C- give epidural then CS.
Answer: A .from Toronto note Obstetrics , give oxitocin and amniotomy .conservative(not sure

379.Primary amenorrhea (Not sure what’s the question here)


A. Non-steroidal
B. Combined oral
C. Mini pill (progesterone only)
Answer is: NOT SURE
For primary amenorrhea, hormone therapy, consisting of an estrogen and a progestin, is recommended for women
with estrogen deficiency.
A typical regimen consists of an estrogen with a dosage equivalent to 25 mcg/day of transdermal estradiol given
unopposed (i.e., no progesterone) daily for 6 months with incremental dose increases at 6-month intervals until the
required maintenance dose is achieved.
Cyclic progesterone therapy, given 12-14 days per month, should be instituted once vaginal bleeding begins.
Source: http://emedicine.medscape.com/article/252928-medication#1

87
380.Which are long cell process
A- interphase
B- pro
C- pre

Answer: B, prophase
Chromosomes become visible, the nucleolus disappears, the mitotic spindle forms, and the nuclear envelope disappears.
Http://www.phschool.com/science/biology_place/biocoach/mitosisisg/prophase.html

381.Lady diagnosis with lichen sclerosis presented with a mass in her labia majora,
What is the type of cancer?
A. Adenosquamous carcinoma
B. Squamous carcinoma
C. Adenocarcinoma
Answer: B
Explanation:http://www.aafp.org/afp/2002/1001/p1269.html

382.Female patient (long scenario), deep dyspareunia?


A) PID
B) vaginitis
C) vulvitis.
Answer: A
(Not enough information to determine answer, patient age and symptoms were not mentioned)
Incomplete question

383.Best investigation of ovarian cancer:


A. U/S
B. CA125
Answer:

Uptodate : -
The only way to diagnose ovarian cancer is with exploratory operation.
-Intra operative biopsy is not usually done , biopsy may spread cancer cells.
-Ca125 is for monitoring ( pre and post operative ) - it is not diagnostic and not specific.

88
-Imaging like u/s is recommended initially , but can not be diagnostic .
References:
Http://www.uptodate.com/contents/ovarian-cancer-diagnosis-and-staging-beyond-the-
basics?Source=outline_link&view=text&anchor=H4#H4
For screening :Check the NICE guide lines
http://www.nice.org.uk/guidance/cg122/resources/ovarian-cancer-recognition-and-initial-management-35109446543557

384.8 weeks pregnant presented with vaginal bleeding and no pain. What will you do to approve your diagnosis?
A. Vaginal os
B. Β-HCG
Answer: A
Explanation: The patient is suspected to have abortion. For which, vaginal os examination and ultrasound are indicated for
confirmation.
Reference Kaplan OB/GYNE usmle step 2

385.Pregnant lady has flu symptoms. What will you give?


A. Oseltamivir
B. Zanamivir
Answer: A
Explanation: Oseltamivir is preferred for treatment of pregnant women. Pregnant women are recommended to receive the
same antiviral dosing as non-pregnant persons
Reference http://www.cdc.gov/flu/professionals/antivirals/antiviral-dosage.htm

386.A pregnant lady is taking iron. She came complaining of weakness and fatigue. Her labs show HB low and MCV low. What
is the most likely diagnosis?
A. Iron deficiency anemia
B. Hypothyroid
Answer: A

387.Woman gave birth and developed fever, abdominal pain and nausea. What is the diagnosis?
A. PID
B. Candida
((Robbins an Cotran pocket basis pathology,7E,p532))
Answer: Most common cause of postpartum fever is endometritis. Risk factors: ER CS after prolonged rupture of membranes
and prolonged labor. Findings: moderate to high fever, exquisite uterine tenderness. Managed with multiple agent IV
antibiotics.
Kaplan LN CK OBGYN 2013

388.46 year old patient, wants to get pregnant what will you check?
A. LH and FSH level
B. TSH

Answer: A

389.Patient with insomnia and other sx relieved on the first day of menses what is the tx?
A. SSRI
B. Oral medroxyprogesterone acetate

89
Answer: A
This seems like premenstrual syndrome. The first line treatment of which are ssris
Http://bestpractice.bmj.com/best-practice/monograph/419/treatment/details.html

390.30 year-old Diabetic Patient present with swelling of vulva and white discharge odorless
A. Trachominos vaginalis
B. Candida albicans

Answer: B
Http://emedicine.medscape.com/article/213853-clinical

391.Married female want not to be pregnant now; based on statistical studies what you will prescribe?
A. Progesterone only
B. Combined estrogen progesterone.
Answer: B
Progesterone maintains the lining of the uterus, which makes it possible for a fertilized egg to attach and survive. Makes cervical
mucous accessible by sperm. Allows the embryo to survive. Prevents immune rejection of the developing baby. Allows for full
development of the fetus through pregnancy. Helps the body use fat for energy during pregnancy.
Progesterone only used if women want to breastfeed her baby. If not so combined.

392.Post menopausal women + mass on US the patient taking vaginal estrogen what is the diagnosis?
A. Lieomyoma
B. B-lieomyosarcoma
Answer: both lieomyoma and leiomyosarcoma not common in post menopause and can differentiate btw sarcoma + oma by
present of
5-10 mitosis /10 high power field +atypia or more than 10 with or witout atypia ((Robbins an Cotran pocket basis
pathology,7E,p541))

393.Pregnant present with bloody discharge in 10 week and fundus hight is 16 what is cos
A. Ruptue of cyssti
B. Ectopic pregnancy
To approach pregnant with bleeding you have to decide is early pregnancy bleeding or late (antepartuem) bleed which judjed by
a cutoff of time 20 weeks in that case the bleeding is early and there is 2 of most common causes of early pregnancy bleeding
which is ectopic pregnancy and abortion in that threatened abortion is more convincing than other options

394.Elevated in menopause lady?


A. Progesterone
B. LH

395.Mass out of vagina with coughing and defecation?


A. Ureterovaginal prolapse or
B. Rectovaginal prolapse (rectocele )

396.Pregenat 32 Weeks did 2 ceaserian & 2 sponteious vaginealdelvery , now what mannent for delvery ?
A. Elective c/s at ..
B. Sponteous Vaginal delvery

91
397.Pregnant female with UTI which drug if safest during all trimesters:
A. Ampicillin
B. Nitrofurantoin Other choices contraindicated

398.Lactating mother complaining of breast tenderness, hotness and redness, diagnosed to have bacterial mastitis. What will
you recommend for her?
a. Continue breast feeding, hot compressor and antibiotic.
b. Discontinue breast feeding and give antibiotic to mother and baby.

Answer: A
Heat or ice packs, continued nursing/pumping, antibiotics (dicloxacillin/cephalexin) (erythromycin if pen-allergic).
Reference: Toronto notes and 3rd Edition UQU > Obstetrics and Gynecology > Q 385

399.A patient with premature rupture of membranes for more than 18 hours (long scenario with a lot of details). Which of
the following give the patient high risk for GBS infection?
a. Rupture of membranes for more than 18 hours.
b. Family hx of GBS infection.

Answer: A

400.Female came to infertility clinic because she cannot conceive. She has a history of three elective abortion and D&C in
the past. She refused to be examined. What is the most likely diagnosis?
a. Sheehan Syndrome
b. Asherman Syndrome
Answer: B

401.45 years old gravida 4 para 3, week 8 pregnant. Last pregnancy she had a down syndrome baby so she’s asking for
checkout regarding Down syndrome. What are the complications that you are going to tell her when you take consent?
a. Rupture of amniotic sac
b. Unintended miscarriage

Answer: B
Chorionic villus sampling (10 - 12 wk):1-2% risk of spontaneous abortion. Reference: Toronto Notes

402.Pregnant woman in 3rd trimester, with vaginal infection (discharge), after delivery the baby got eye infection
(conjunctivitis and discharge). What is the most likely cause?
a. Chlamydia
b. Gonorrhea

Answer: B
Gonococcal conjunctivitis tends to occur 2-7 days after birth but can present later.
Chlamydial conjunctivitis usually has a later onset than gonococcal conjunctivitis; the incubation period is 5-14 days.
Reference: http://emedicine.medscape.com/article/1192190-clinical

403.Pregnant at 5 weeks of gestation. Cervical incompetence was diagnosed. What will you do?
a. Cerclage now
90
b. Cerclage at 12-13 weeks

Answer: B
Usually at the end of the 1rst trimester and removed in the third trimester.
Emerging evidence indicates that progesterone suppositories are superior to cerclage in preventing preterm labour late in
pregnancy. Reference: Toronto Notes.

404.Female G2P2 complaining of irregular menstruation for 6 months, history reveal normal babies with normal deliveries
but she did D&C after the second delivery for retained part of placenta, investigations (I can’t remember), what is your
diagnosis ?
A. Asherman’s syndrome.
B. Polycystic ovary.

Answer: A
Intrauterine adhesion (or intrauterine synechiae) is a condition in which scar tissue develops within the uterine cavity.
Intrauterine adhesion accompanied by symptoms (eg, infertility, amenorrhea or hypomenorrhea) is also referred to as
Asherman syndrome. In’s primarily caused by curettage for pregnancy complications.
Reference: Uptodate

405.Pregnant lady with a history of 2 NSVD and 1 CS. How will you manage?
a. Natural vaginal delivery trial
b. Admit at 38 weeks for CS

Answer: A

406.40 weeks of gestation primigravida presents with hypoxia, drowsiness and agitation for 6 hours, LL edema (Long scenario
with blood tests). What does she have?
a. Amniotic Emboli
b. PE

Answer: B
During pregnancy, risk is increased because venous capacitance and venous pressure in the legs are increased, resulting in stasis,
and because pregnancy causes a degree of hypercoagulability.
Https://www.merckmanuals.com/professional/gynecology-and-obstetrics/pregnancy-complicated-by-disease/thromboembolic-
disorders-in-pregnancy
Http://emedicine.medscape.com/article/261226-overview#a2

407.While giving birth she lost sensation in the medial thigh: what nerve?
A- Pudendal,
B-obturator
Answer:B
-Lithotomy positioning during delivery or in gynecologic/urologic procedures also has been associated with compressive
femoral neuropathy.
The sensory branch of the femoral nerve, the saphenous nerve, innervates skin of the medial thigh and the anterior and medial
aspects of the calf.
-The cutaneous branch of the obturator nerve supplies the skin of the middle part of the medial thigh.
Ref : http://emedicine.medscape.com/article/1141793-overview#a7

408.G8P7 in operation room she tell you that she had after all previous pregnancies severe postpartum hemorrhage
what you will do you do to pt:
92
A- give her crystalloid I.V during C/S or labour.
B- active third stage.
Answer: B
Routine oxytocin administration in third stage of labor can reduce the risk of PPH by > 40%
(Toronto notes)

409.Positive culture of budding yeast in urine what is the management?


A. Flucanazole
B. Caspofungin

Answer: A
Increasing are the numbers of fungal UTI, particularly those caused by Candida spp, and, to a lesser extent, by
Aspergillus spp and Cryptococcus neoformans.
Candiduria is a condition most often found in elderly, hospitalized, or immunocompromised patients.
Candida albicans is the most common species isolated, accounting for more than half of all fungal infection cases.
The mainstay of antibiotic treatment for candiduria is the azolic compounds, mainly fluconazole 200 mg orally daily for
2 weeks
Reference: http://emedicine.medscape.com/article/213853-medication#2

410.Pregnant lady missed pregnant symptom since 1week and started compline of spot bleeding, the most valuable
investigation in this condition is:
A. Hcg-alpha
B. Feto-ultrasonography

Answer: B
Http://emedicine.medscape.com/article/404971-overview

411.Pregnant in 40 weeks gestational age, did not follow up ,, examination and ultrasound reflect breech presentation, in
progressive labor pain, cervical full dilation and full effacement , intact membrane, Engagement zero, what to do ?
A. Amniotomy
B. CS

Answer: B
Http://emedicine.medscape.com/article/262159-overview#a3

412.Female has 3 children, doesn't want to get pregnant anymore, has a history of endometriosis in ovary, She removed it,
now she has another one in the right ovary, how to manage this patient?
A. Hysterectomy bilateral salbingo oophrectomy ,
B. Aspiration of the mass ..?

Answer: A
Definitive: bilateral salpingo-oophorectomy +/- hysterectomy
Reference (Toronto notes GY14)

413. Pregnant woman in labour room , when she delivered her baby, sudden onset of bleeding from vagina , the baby is not
infected , after 2 hours mother onset bleeding from mouth and nose , what is the cause ? -
A. DIC
B. Deficiency in factor llx

93
Answer: A
Abruptio placentae, amniotic fluid embolism, sepsis, and severe preeclampsia are obstetric conditions associated with
disseminated intravascular coagulopathy (Obstetrics and Gynecology, sixth edition, Charles R. B. Beckmann, page 138)

414.Patient with Hx of lower transverse incision and double uterine fold suture and she is at 37 wk and doing fine
A. A-CS
B. B-SVD
Answer :B
Criteria for trial of labor include patient consent, nonrepetitive cesarean indication (e.g. Breech , plcanta previa), Previous low
segment transverse uterine incision, clinically adequate pelvis. (Kaplan page 135)
"trial of labor after cesarean" means that you plan to go into labor with the goal to deliver vaginally, but still may need to C-
section

415.Female was diagnosis with ovarian cancer , she haven't ever used OCP , what will you tell her daughters ?
A. OCP can protect you from ovarian cancer
B. Bilateral oophorectomy is recommended for you
Answer A
Oral contraceptives that prevent ovulation appear to provide significant protection against the occurrence of ovarian cancer.
(Obstetrics and Gynecology, sixth edition, Charles R. B. Beckmann, page 408)

416.Pregnant in 40week suddenly become drowsy seizure what is the cause :


A. PE
B. Amniotic Emboli.

Answer: B
A woman in the late stages of labor becomes acutely dyspneic with hypotension; she may experience seizures quickly followed
by cardiac arrest. Massive DIC-associated hemorrhage follows and then death. Most patients die within an hour of onset.
Currently no definitive diagnostic test exists. The United States and United Kingdom AFE registries recommend the following 4
criteria, all of which must be present to make the diagnosis of AFE:
Acute hypotension or cardiac arrest
Acute hypoxia
Coagulopathy or severe hemorrhage in the absence of other explanations
All of these occurring during labor, cesarean delivery, dilation and evacuation, or within 30 minutes postpartum with no
other explanation of findings
(http://emedicine.medscape.com/article/253068-overview#a5)

417.Tamoxifen for breast cancer has metrohagia, US showed thick endometrium what to do next ?
A. Endometrial biopsy
B. CA 125
Answer:
An endometrial biopsy is needed to confirm a diagnosis of endometrial cancer. A biopsy removes a small sample of the lining of
the uterus (endometrium) to be looked at under a microscope. (http://www.webmd.com/cancer/tc/endometrial-cancer-exams-
and-tests)

Tamoxifen is used to treat women with estrogen receptor-positive breast cancer. It can be used in conjunction with
chemotherapy. It is also given as a 5-year course of preventive treatment following surgery. (Obstetrics and Gynecology, sixth
edition, Charles R. B. Beckmann, page 293)

94
Tamoxifen is a selective estrogen receptor modulator and acts as an estrogen receptor antagonist in the breast and as an
agonist in the uterus and bone. Studies of tamoxifen therapy for 5 years have demonstrated a 50% reduction in local breast
recurrence, 30% decrease in mortality, and prevention of new breast cancer development. The risks of endometrial cancer and
thromboembolic events due to hypercoagulability are increased two to four times with tamoxifen but occur in fewer than 1% of
women taking the medication. (General Gynecology, The Requisites in Obstetrics and Gynecology, Sokol, 2007ed, page 514)

418.Pregnant lady with positive nitrite and leukocyte esterase and E.coli?
A. Penicillin !!!
B. Advise her to drink a lot of fluid.
Answer: A
Uncomplicated UTI
Ƒ first line: amoxicillin (250-500 mg PO q8h x 7 d)
Ƒ alternatives: nitrofurantoin (100 mg PO bid x 7 d)
Ref. Toronto notes page OB19
419.Pregnant 40 g.a did not follow up ,, examination and ultrasound reflect breech presentation ,,, in progressive labour
pain.. Cervical full dilation and full effacement , intact membrane .. Engagement zero ,, what to do ?
A. Amniotomy
B. CS
Answer: CS
In light of recent studies that further clarify the longterm risks of vaginal breech delivery, the decision regarding mode of
delivery should depend on the experience of the healthcare provider. Cesarean delivery will be the preferred mode for most
physicians because of the diminishing expertise in vaginal breech delivery. Planned vaginal delivery of a term singleton breech
fetus may be reasonable under hospital-specific protocol guidelines for both eligibility and labor management. The following
criteria have been suggested for vaginal breech delivery:
Normal labor curve
Gestational age greater than 37 weeks
Frank or complete breech presentation. Because of the risk of umbilical cord prolapse, vaginal delivery of a fetus in the
footling breech position is not recommended.
Absence of fetal anomalies on ultrasound examination
Adequate maternal pelvis
Estimated fetal weight between 2500 g and 4000 g
Documentation of fetal head flexion. Hyperextension of the fetal head occurs in about 5% of term breech fetuses,
requiring cesarean delivery to avoid head entrapment.
Adequate amniotic fluid volume (defined as a 3-cm vertical pocket)
Availability of anesthesia and neonatal support.

If a vaginal breech delivery is planned, the woman should be informed that the risk of perinatal or neonatal mortality or short-
term serious neonatal morbidity may be higher in it than in a cesarean delivery, and the maneenn’s informed consent should be
documented. (Obstetrics and Gynecology, sixth edition, Charles R. B. Beckmann, page 110-111)

420.Best prenatal screening in the first trimester for Down syndrome ?


A. Chorionic villous biopsy
B. Amniocentesis
Answer: A
Chorionic villus sampling (CVS) is a diagnostic procedure performed under US guidance. In this procedure we aspirate by
catheter from pregnant uterus between 10-12 weeks gestation. The tissue is sent to laboratory for karyotyping.
Amniocentesis is diagnostic procedure performed after 15 weeks (second trimester).
(Kaplan page 30)

421.Postmenopausal lady came with vaginal spotting , on examination there was cystic nodule in her labia majora , what is
the diagnosis ?
95
A. Bartholin cyst
B. Bartholin carcinoma
Answer: B
Bartholin carcinoma on average occurs in women over the age of 50; however, any new Bartholin mass in a woman over the age
of 40 should be excised. Treatment of diagnosed Bartholin cancers is radical vulvectomy and bilateral lymphadenectomy.
Recurrence is disappointingly common, and a 5-year overall survival rate of 65% is noted. (Obstetrics and Gynecology, sixth
edition, Charles R. B. Beckmann, page 373)

422.Lady with metromenorrhagia , from 6 month ago .. And abdominal pain interfere with her activity , what is the best
drug?
A. OCP
B. I think "" estrogen analogous
Answer: A
• medical treatment of endometriosis
NSAIDS (e.g. Naproxen sodium – Anaprox®)
Pseudopregnancy:
o Cyclic/continuous estrogen-progestin (OCP)
o Medroxyprogesterone (Depo-Provera®)
o Dienogest (Natazia®)
Pseudomenopause
o 2nd line: only short-term (<6 mo) due to osteoporotic potential with prolonged use,
Unless combined with add-back therapy (e.g. Estrogen/progesterone or SERM); if
Long-term use required, add-back estrogen+progesterone
o Danazol (Danocrine®): weak androgen
Side effects: weight gain, fluid retention, acne, hirsutism, voice change
o Leuprolide (Lupron®): gnrh agonist (suppresses pituitary)
Side effects: hot flashes, vaginal dryness, reduced libido
Can use ≥02 mo with add-back progestin or estrogen
Surgical
o Conservative laparoscopy using laser, electrocautery ± laparotomy
Ablation/resection of implants, lysis of adhesions, ovarian cystectomy of endometriomas
o Definitive: bilateral salpingo-oophorectomy ± hysterectomy
o ± follow-up with medical treatment for pain control not shown to impact on preservation of fertility
o Best time to become pregnant is immediately after conservative surgery
Ref:Toronto notes page GY14

423.A female had an IUD inserted 2 years ago, now she's complaining of lower abdominal pain and vaginal discharge which
was foul smelling few days ago. On examination you found a right 9-adnexal mass. A gram stain of the cervical discharge
showed a gram positive beading bacillus, what is the most likely causative organism?
A. Perfringens
B. Bacteroides fragilis

Answer: not enough answering options


But most likely it is Gram-positive branching filaments of Actinomyces species. (http://www.medscape.com/viewarticle/407784)
424.Female patient obese with regular menstrual cycle , on PE/ she had acne , other examination is normal , what
investigation will you order ?
A. TSH
B. ACTH
Answer:
Although this patient is having regular menstrual cycle, this is mostly a case of Polycystic ovarian syndrome, in PCOS:
Obesity (in 40% to 50% of patients)
Signs of virilization, including deepening voice and masculinization of body habitus
96
Cutaneous signs of androgenization, such as hirsutism, acne, oily skin, and male pattern baldness; acanthosis nigricans (gray-
brown, velvety discoloration of skin, usually at the neck, groin, and axillae)
(according to Clinical Key)
In PCOS Determining the LH/FSH ratio is useful because a ratio of 3:1 is virtually diagnostic of PCOS; however, a normal ratio
does not exclude the diagnosis, as LH levels fluctuate widely throughout the course of a day.
Also Thyroid-stimulating hormone (TSH) is measured to rule out hypothyroidism

425.Pregnant lady with vaginal discharge caused by n. Gonorrhea , which of the following is associated with this case :
A- chlamydia
B- HSV
Answer:A
Tests for chlamydial infection should be done in all patients. It is particularly important that these should be performed in
mothers with untreated gonorrhea, and mother and newborn infant should be tested
(clincalkey)

426.Which organism can cause vaginal infection:


A-HSV 1
B- HSV 2
Answer: b
HSV 2
There are two main subtypes of herpes simplex virus (HSV): HSV-1, which classically causes oral herpes, and HSV-2, which
classically causes genital herpes; however, both types can cause symptoms in almost any anatomic site.
(clincalkey)

427.42-year-old patient with PCOS, nulligravida, she never took any medication to regulate her period, endometrial biopsy
showed endometrial hyperplasia, what is the cause?
A. Old age.
B. Unopposed estrogen.
Ans: unopposed estrogen.
The leading role in the pathogenesis of endometrial hyperplasia is given relative or absolute hyperestrogenic, the absence of
antiestrogenic effects of progesterone or insufficient effect.

428.Patient with secondary dysmenorrhea + infertility?


A. Endometriosis.
B. Leiomyoma.
Answer: a

The most common cause of secondary dysmenorrhea is endometriosis. The main clinical features are dysmenorrhea, chronic
pelvic pain and infertility
Reference ; http://www.ncbi.nlm.nih.gov/pmc/articles/PMC3935015/

429.Patient with postpartum depression what is Tx:


A.triptaline
B.psychotherapy

Answer : b
Sertraline, paroxetine & nortriptyline are the safest & most effective in PPD. Psychological treatments for PPD are often the
treatment of choice for women, as they are effective for the treatment of depressive symptoms and do not involve the risks of
exposure to medications.
Reference : http://www.ncbi.nlm.nih.gov/pmc/articles/PMC3039003/

97
430.Female patient present with laughing and coughing passing out urine O/E there is bulging in labia majora ( stress
incontinence)
A. Cystocele
B. Urethrocele
Answer: A (sure 100%)

98
431.Pregnant lady week 12 discovered to have small fibroids, what should she expect?
A-Asymptomatic
B-Degenerates
Answer : A

Most fibroids are asymptomatic. However, severe localized abdominal pain can occur if a fibroid undergoes so-called “red
degeneraneon,” torsion (seen most commonly with a pedunculated subserosal fibroid). Pain is the most common complication of
fibroids in pregnancy, and is seen most often in women with large fibroids (> 5 cm) during the second and third trimesters of
pregnancy.

Some studies have suggested that small and large fibroids (≥ 6 cm) have different growth patterns in the second trimester (small
fibroids grow whereas large fibroids remain unchanged or decrease in size), but all decrease in size in the third trimester.
Fibroids that did increase in volume, the growth was limited almost exclusively to the first trimester, especially the first 10
weeks of gestation, with very little if any growth in the second and third trimesters.
Reference : http://www.ncbi.nlm.nih.gov/pmc/articles/PMC2876319/

432.Pregnant with placenta abruption what's suspected complication :


A-Fetal distress
B-Other "
Answer:
For the baby
Born at low birthweight
Preterm delivery
Asphaxia ‘ fetal distress ‘
Fetal death & stillbirth.

433.Patient with severe painful vesicles on genital area:


A- syphilis
B- HSV
Answer: B
Genital ulcers in HSV are multiple, painful ulcers start as vesicular lesions and progress to ulceration, then crusted lesions.

434.2 years female C/O atrophic vaginitis , weakness of pelvic floor muscle , urine incontinence What is the most
appropriate management for her?
A-Kegel exercise.
B-Surgical
Answer: Kegel Exercise

Reference : uptodate

435.Contraindication to IUD
A-Active PID
B-Coagulation abnormality
Answer: A
Reference: Toronto

436.1ry dysmenorrhea, what is the first line of management:


A- nsaids
B- Acetaminophen
Answer: A
Reference: uptodate

99
437.Women after CS have persistent hypotension , What is best management
A. Normal saline
B. IV dopamine
Answer:

438.Women develop gestational diabets and doctor give her insulin after delivery she is at risk to have :
A- Dm type 1
B -dm type 2
Answer: b

The mother is at risk of developing type 2 diabetes in the next 10 to 20 years


Reference : toronto notes

439.Which medication decrease effect of OCP?


A-anti epileptic
B-anticoagulant
Answer: a
Http://www.aafp.org/afp/2008/0901/p634.html50--treatment of cap in pregnancy
Http://www.ncbi.nlm.nih.gov/pubmed/11945109

440.16 year old female no menstrual cycle yet, all other features are present. What is the diagnosis?
A. Mullerian agenesis.
B. Ovarian agenesis
Answer :
Mullerian agenesis : to development of mullerian duct ( no vagina, uterus, cervix) but present with primary amenorrhea
and secondary sexual chch

441.17 year old deliver her baby in the home with help of her friend,, what type of perineal muscle tear ?
A. Pubococcygeus
B. Ischiocavernosus
Answer

442.Delivery , baby developed distress , what type of anesthesia given to mother :


A. General anesthesia ?
B. Narcotic analgesia ?
ANSWER : B

443.Prevent congental heart disease in preg >


A-rubilla vaccin✔‫الكيس ناسيه حامل ماكانت اذا‬
B- amniocen
Answer:
To decreade risk of congintal heart disease :
1-Viral Infections – Women who contract rubella (German Measles) during the first three months of pregnancy have an
increased risk of having a baby with a heart defect ,women should avoid becoming pregnant for one month after
receiving the MMR vaccine. If a woman is not vaccinated, she should talk to her health care provider about any possible
risks.
2-Take 400 micrograms of folic acid supplement a day during the first trimester (first 12 weeks) of your pregnancy
3-Avoid drinking alcohol or taking drugs.
4-If you have diabetes, make sure it's controlled.

011
444.Baby born full term flax-....enlarge labia the cause is :
A)estrogen
B)hcg .....
.proges..‫الباقي نسيت‬
Answer :

445.When amniotic fluied less than 400 it is :


A- oligohydro
B- poly

Answer:
At 12 weeks' gestation, the average volume is 60 ml.2 By 16 weeks, when genetic amniocentesis is often performed,
the mean volume is 175 ml.2, 15 From 20 weeks on, there is greater variance of amniotic fluid volume. Based on
numerous studies using dye or para-aminohippurate dilution, radioactive isotopes, and actual collection of amniotic
fluid at amniotomy, it has been determined that amniotic fluid volume increases steadily throughout pregnancy to a
maximum of 400–1200 ml at 34–38 weeks; however, wide variation does exist

446.The most effective way to prevent cardiac anomaly in pregnancy is ?


A- smoking cessation
B-genetic screen
Answer:A
Women who smoke during early pregnancy are more likely to have a child with congenital heart defects
(http://americanpregnancy.org/birth-defects/congenital-heart/

447.Lady in labour of breach presentation cervix fully dilated membrane i think rupture but no preceding in labor for i
think 2hs what will do
A- continuo with vaginal delivery
B- cs
Answer:B
C/S recommended if: the breech has not descended to the perineum in the second stage of labor after 2 h, in the absence of
active pushing, or if vaginal delivery is not imminent after 1 h of active pushing. Toronto notes.

448.Pregnant lady fall from stairs presented to ER with severe abdominal pain and back pain abdomen was tender distended
there was vaginal black like blood with fetal distress:
A- abruptio placente
B- uterine rupture
Answer: A
Sudden onset, constant, localized to lower back and uterus ± fetal distress. Toronto notes
449.Breastfeeding mother known history of seizure on phenytoin, Ask about breastfeeding?
A- Reassurance.
B- Feeding after 8 hours.
Answer: A
450.Patient with PCO (ocps?) Was on progesterone and now is off it, at risk of wt:
A. Endometrial ca
B. Cervical ca
Answer:
Women who use oral contraceptives have been shown to have a reduced risk of endometrial cancer. This protective effect
increases with the length of time oral contraceptives are used and continues for many years after a woman stops using oral
contraceptives.
Cervical cancer: Long-term use of oral contraceptives (5 or more years) is associated with an increased risk of cervical cancer
(http://www.cancer.gov/about-cancer/causes-prevention/risk/hormones/oral-contraceptives-fact-sheet#q5)

010
451.Which drug is safe for pregnant women:
A. Cimeitidine
B. Cefoxizime

Answer: A
Cimetidine has been assigned to pregnancy category B by the FDA.
(http://www.drugs.com/pregnancy/cimetidine.html)
Cefoxizime is also category B!!

012
452.Pregnant, developed edema from inguinal to ankle what to give her?
A. Heparin
B. Warfarin

Answer: A
(Uptodate)Pregnancy and the puerperium are well-established risk factors for deep vein thrombosis (DVT) and pulmonary
embolism (PE), which are collectively referred to as venous thromboembolic disease (VTE).
●Ineneal managemenn of susmecned VTE dureng mregnancy demends on nhe degree of clenecal susmeceon, whenher annecoagulaneon es
contraindicated, and whether PE, DVT, or both are suspected.
●For mregnann women, we recommend adjusned dose subcunaneous low molecular weight heparin (SC LMWH), rather than
adjusted dose intravenous unfractionated heparin (IV UFH) (Grade 1B) or vitamin K antagonists (Grade 1A). We recommend
against the use of oral direct thrombin inhibitors (eg, dabigatran) or anti-Xa inhibitors (eg, rivaroxaban, apixaban) in pregnant
women (Grade 1C).
●We suggesn nhan annecoagulann nheramy connenue at least six weeks postpartum (Grade 2C). We suggest a total duration of
anticoagulant therapy of at least three to six months for women whose only risk factors for VTE were transient (eg, pregnancy)
(Grade 2C). Patients with persistent risk factors for VTE may require longer therapy.
●Thrombolynec nheramy should be reserved for pregnant or postpartum patients with life-threatening acute PE (ie, persistent and
severe hypotension due to the PE))

453.How to diagnose vulvar cancer?


A. Biopsy of the lesion
B. Something about viral antigen detection
Answer: A
Http://emedicine.medscape.com/article/264898-overview

454.Pregnant patient with anemia, MCV high, what will you give her?
A. Iron
B. Folate

Answer: B
If these are the only available choices, this is the answer because folate deficiency causes macrocytic anemia.
Https://www.nlm.nih.gov/medlineplus/ency/article/000551.htm

455.Lactating women the doctor prescribed phenytoin for seizures regarding breast feeding she should
A. Stop breast feeding
B. Feed after 8 hours
Answer: A
Phenytoin may cause harm to an unborn baby, but having a seizure during pregnancy could harm both mother and baby.
Tell your doctor right away if you become pregnant while taking this medicine.
If you become pregnant while taking phenytoin, your name may be listed on a pregnancy registry. This is to track the
outcome of the pregnancy and to evaluate any effects of phenytoin on the baby.
Phenytoin can make birth control pills less effective. Ask your doctor about using non hormonal birth control (condom,
diaphragm with spermicide) to prevent pregnancy while taking this medicine.
Phenytoin can pass into breast milk and may harm a nursing baby. You should not breast-feed while you are using this
medicine.
Http://www.drugs.com/phenytoin.html

456.Which of the following is a risk of Staphylococcus Saprophyticus vaginal infection?


A. Septicides in condoms
B. Douching habits
Answer: A


013
There is a strong association between the use of condoms coated with nonoxynol 9 and the occurrence of UTI, which suggests
that vaginal spermicides interfere with the normal vaginal flora and promote colonization by S. Saprophyticus.
Douching habits is associated with stds (gonorrhea, chlamydia, syphilis, trichomoniasis, or herpes simplex virus-2).
Http://cid.oxfordjournals.org/content/40/6/896.full
Http://www.ncbi.nlm.nih.gov/pubmed/8571993

457.Female delivered her baby 4 months ago breastfeeding needs contraception and concerned about not having her period?
A. Reassure and counsel about contraception
B. Order prolactin level
Answer: B
Women who breastfeed have a delay in resumption of ovulation postpartum. This is believed to be due to prolactin-induced
inhibition of pulsatile gonadotropin-releasing hormone release from the hypothalamus.
Source: http://www.uptodate.com/contents/overview-of-postpartum-care

458.Mother with GBS and had a baby who has irritability and agitation and fever. What will you do?
A. Give antibiotics
B. Do cultures
Answer:B http://www.cdc.gov/groupbstrep/about/symptoms-diagnosis-treatment.html

459.You have a patient and you took her permission to examine her. What are you doing?
A. Taking informed consent
B. Being efficient in you job
Answer: The very act of a patient entering a doctor's chamber and expressing his problem is taken as an implied (or implicit)
consent for general physical examination and routine investigations. But, intimate examination, especially in a female, invasive
tests and risky procedures require specific expressed consent. Expressed (explicit) consent can be oral or written.

460.Case of female underwent vaginal hystrectomy what the artery most be effected :
A. Overian a
B. Femoral a
Answer: A
Anatomical position of these vessels. Also if uterine artery is mentioned as choice it is more common to be injured in such a
procedure.Femoral is in the thigh while ovarian is within the procedure site. (Kaplan anatomy pelvis chapter)

461.White vaginal discharge pseudohyphae what is the treatment


A. Metronidazole
B. Miconazole
Answer: B
It is a topical antifungal of -azole family. A is an antibacterial and certain parasites.

462.Pregnant present with bloody discharge in 10 week and fundus height is 16 what is cos
A. Ruptue of cyssti
B. Ectopic pregnancy

Answer: the scenario mentioned goes more with Molar Pregnancy.


Kaplan:
The most common symptom is bleeding prior to 16 weeks of gestation.
The most common sign is fundus larger than dates.

014
463.Primary amenorrhea normal breast spared axially and pubic hair:
A. Turner
B. Androgen insensitivity syndrome
Answer: B

464.Scenario about 1ry dysmenorrhea .what's the 1st linesigns and symptomsf ttt?!
A. Nsaids
B. Ocps

465.Most common cause of secondary amenorrhea with high LH and FSH :


A- pregnency
A- Menopause

466.Polycystic ovaries syndrome fertility what is the cause?


A- Endometrial.
B- Ovarian.
Answer; B

467.Pregnant on iron therapy has fatigue and SOB , Hb is low , MCV is low ,retics=10%, What is the diagnosis
A) IDA
B) Thalassemia
Answer: A, due to iron treatment, reactional elevation in retics count

468.40 year-old Patient was normal cycle, now heavy and bleeds intermittently, wt to do to dx:
A- pap
B- colposcopy
Answer : A

AUB investigations ; in addition to lab tests, you have to screen for cervical cancer,& possibility of cervicitis by gonorrhea or
chlamydia. For patient younger than 45 years old with unopposed prolonged estrogen exposure ( e.g. Obesity ) or with
persistent AUB despite medical management , u have to take an endometrial biopsy.
Frequent, heavy or prolonged AUB in women > 45 years necessitate endometrial biopsy.
Http://www.uptodate.com/contents/approach-to-abnormal-uterine-bleeding-in-nonpregnant-reproductive-age-
women?Source=outline_link&view=text&anchor=H16090927#H16090927

469.After CS, on the 5th day there was discharge from the wound. In examination abdominal structure can been seen
through the wound. What is the diagnosis
A.bowel fistula
B.wound dehiscence

Answer:b
Https://en.wikipedia.org/wiki/Wound_dehiscence

015
470.Scenario female c/o amenorrhea , with normal breast development & normal pubic hair . O/E no uterus & cervix. Dx?!
A. Mulleirn duct
B. Gonadal dysgenesis.

The question is incomplete but depending on the answers given it seems en’s primary ammonorhea.
Refrence: Kaplan notes OBGYN pg 233

471.Pregnant women 36 week with N&V and uterine contraction each 4-5 min lasting for 30-40 seconds, cervix dilated 6,7 or
4 ,effacement 70% what is the best managemnt ? BLOOD PRESSURE 170/
A. Urine dipstick
B. Tocolytic
There was no delivery or mgs4 And i think there was no anthypertension medication
"It should be mgslphate then delivery "

472.Young lady with abdominal pain, bloating, what treatment to give? Question missing information. Is it always or before
period or sudden
A- TCA
B- Antispasmodic
Answer: B

473.Asystole first treatment in a child?


A- Epinephrine and CPR
B- Atropine.
Answer: A ( Epinephrine remains the drug of choice for asystole in children. Atropine is not indicated.)
Ref.: http://www.aafp.org/afp/1999/1015/p1743.html

474.During delivery something happened C/S was required, what type of anesthesia?
A- Pudendal.
B- General . If not already on epidural
Answer: epidural and spinal anesthesia, B/c in General anesthesia the drugs are given to the mother will affect the infant.
Ref.: http://www.uptodate.com/contents/c-section-cesarean-delivery-beyond-the-basics

016
475.Women with metromenorrhagea otherwise normal what is the treatment ?
A. Mefenamic acid
B. NSAID
C. OCP
Answer:C

Treatment of DUB includes replacing the lacking hormone using OCP.If not successful nsaids can be used.
Reference: Kaplan USMLE Step 2 OBGYN

476.Pregnant lady e epilepsy what will caus difict in baby:


A) Seizure
B) Anticonvalsant

477.Asymptomatic trichomonas when will you treat:


A) Pregnancy
B) Start immediately
Answer: B
Treatment should be instituted immediately and, whenever possible, in conjunction with all sexual partners with metronidazole
(safe in pregnancy)
Reference: http://emedicine.medscape.com/article/230617-medication

478.Pregnant, not following in prenatal clinic , not aware if she had any disease , present the clinic with high blood pressure ,
what is most propable Diagnosis :
A- preeclampsia
B- chronic hypertension
Answer: question incomplete
Explanation: the answer is chronic hypertension if she presents before 20 weeks of pregnancy. The answer is gestational
hypertension if she presents after 20 weeks with systolic >140 or diastolic >90. The answer is pre-eclampsia if she presents after
20 weeks with hypertension and proteinuria or has adverse outcomes like headache, renal dysfunction, IUGR, or systolic >160,
or diastolic >100. The answer is eclampsia if she pre-eclampsia with seizures (not caused by other neurologic diseases).
Reference: Toronto Notes 2015, page OB17
Pap smear testing is taken from:
Answer: transitional zone
Https://en.m.wikipedia.org/wiki/Pap_test

479.Long scenario of pregnant G1P0 lady has SOB ,,,, allergic to pollen ,,,,, what will u do :
A- CXR
B- AB
Question and choices incomplete

480.What investigation will help reach diagnosis of polycystic ovary disease (PCO)?
A. FSH /LH
B. CT of Theca cells of ovary

Answer: A
The diagnosis is suspected in the presence of irregular menstrual bleeding, obesity, hirsutism and infertility. PCO is confirmed
with LH to FSH ratio, which is in the range of 3:1 The normal LH to FSH ration in ovulatory patient is 1.5: 1
KAPLAN Lecture NOTES

017
481.A post- menopausal women had a mass on ultrasound. The patient is taking vaginal estrogen. What is the diagnosis
A. Lieomyoma (uterine fibroids)
B. Lieomyosarcoma
Answer: A
Most common cause of post-menopausal bleeding is leiomyoma. First task is to exclude endometrial neoplasia. A biopsy finding
of benign endometrium or a vaginal ultrasonography finding endometrial thickness < 4 mm reliably rules out endometrial
hyperplasia or cancer.
Https://en.wikipedia.org/wiki/Uterine_fibroid

482.When to advice against breastfeeding?


A. HIV
B. Hepatitis C with cracked nipples
Answer: A
HIV is an absolute contraindication
There are no sufficient data to say whether Hepatitis C with cracked nipple is contraindicated but it is advised against.
Source: http://www.cdc.gov/breastfeeding/disease/
Http://www.cdc.gov/breastfeeding/disease/hepatitis.htm

483.27 year-old lady symptomatic (do not remember). US show bilateral ovarian cysts. What’s next in assessment?
A. CA125
B. Histopathology
Answer is: A
Any pelvic mass should be assumed to be a cancer until proven otherwise, particularly in a patient with a prior history
of breast cancer or a family history of breast/ovarian cancer.
Most ovarian cysts are discovered incidentally in pregnant ladies who come for their regular pregnancy US.
CA 125 is often recommended for postmenopausal women with an ovarian cyst.
CA 125 may be recommended for premenopausal women whose ovarian cyst appears very large or suspicious for
cancer on ultrasound.
CA 125 does not give definitive diagnosis of ovarian cancer but it is considered one of the earliest steps toward
diagnosis, as elevated levels of CA 125 necessitate further more definitive investigation modalities.
Histopathological analysis is used for definitive diagnosis.
Source: http://emedicine.medscape.com/article/255865-workup#showall

484.Pregnant relative contraindication of ( methyl…) post partum hemorrhage ?


A. DM
B. HTN

485.15 year-old girl came with her mother complaining that there did not menstruate yet. There is breast bud and pubic hair
(normal secondary sexual characteristics ) :
A. Primary amenorrhea
B. Secondary amenorrhea
Answer: A (my answer although it should be 16yo)
Primary amenorrhea is the failure of menses to occur by age 16 years, in the presence of normal growth and secondary sexual
characteristics. If by age 13 menses has not occurred and the onset of puberty, such as breast development, is absent, a workup
for primary amenorrhea should start. Secondary amenorrhea is defined as the cessation of menses sometime after menarche
has occurred. Oligomenorrhea is defined as menses occurring at intervals longer than 35 days apart.
Medscape: http://emedicine.medscape.com/article/252928-overview

018
486.Previously trying to get pregnant, now 4 weeks of amenorrhea, breast tenderness...etc. Scenario of pregnant came to u
what will u order to test?
A- Progesterone
A- TSH
Answer: Beta hcg
Quantitative hcg testing, often called beta hcg (β-hcg), measures the amount of hcg present in the blood. It is used to screen for
pregnancy.

487.Pregnant, early with closed os, no adnexal masses, came with mild spotting.. (scenario clearly denying ectopic)
A. Implantation bleeding
B. D & C
Answer: A
Small amount of spotting associated with the normal implantation of the embryo into the uterine wall, called implantation
bleeding. This is usually very minimal, but frequently occurs on or about the same day as your period was due.

488.Multiparous, 3 hours in labor. 0 station, child in vertex position, 5 cm dilated, 2 contractions in 10 min, monitor is
normal, what to do?
A. Observation
B. Oxytocin
Answer: A?
She’s in the first stage of labor (active), Management includes:
- Periodic assessment of the frequency and strength of uterine contractions and changes in cervix and in the fetus' station and
position.
- Monitoring the fetal heart rate at least every 15 minutes.

489.Post hysterectomy lady is doing fine. Her urine output is …., temperature …. , urinary catheter removed. What will make
the doctor not discharge this patient ?
A. Inadequate urine output
B. Fever
Answer: A (by urologist resident)

490.70 year-old with dysfunctional uterine bleeding. What is the treatment?


A. OCP
B. Hysterectomy *
Answer: B

491.Case of 40 wk gestation, 5cm cervical dilation, ruptured membrane, CTG contraction is good, 3 hours what is your action?
A. Prostaglandin E1(cervical cream)
B. Oxytocin
Answer: B The optimal management of poor labor progression in the active phase is to confirm that the patient is in the active
phase (cervix is at least 5 to 6 cm), administer oxytocin, and wait four hours.
Http://cursoenarm.net/UPTODATE/contents/mobipreview.htm?17/43/18097#H16

492.Female after giving birth she said that she was going crazy. She was checking her baby's bed 15 times looking for snakes
although she knows that there is no snake.
A. Psychosis
B. Obsession
Answer: most likely B not psychosis https://iocdf.org/expert-opinions/postpartum-ocd/

019
493.Obese female with uncontrolled DM, presented with menorrhagia how to investigate?
A. MRI
B. Endometrial biopsy

Answer: B This procedure is used in women who are at risk for endometrial carcinoma, polyps, or hyperplasia. Include those
with HTN, DM, chronic anovulation (eg, PCOS), obesity, atypical glandular cells (AGUS) on Pap smear, new-onset menorrhagia,
and those older than 70 years or any woman older than 35 years with new-onset irregular bleeding (especially if nulliparous).
Http://emedicine.medscape.com/article/255540-workup#c6

494.40 years old female came for pap smear, everything normal in imaging, history and examination what to tell her?
A. Pap smear is not indicated
B. Indicated annually
C. Every 5 years
D. 3 normal and then no indication
Answer: C
Http://emedicine.medscape.com/article/1947979-overview#a2

495.Lady complains of painful vulvar vesicles, no vaginal discharge?


A. Chancre Syphilis
B. HSV Post-herptic lesion

Answer: B because it's painful vesicles, chancres are painless ulcers


http://wps.prenhall.com/wps/media/objects/6356/6509531/tools_7e/table11-3_7e.pdf

496.Risk for neonate to have GBS?


A. Intrapartum fever 39
B. Rupture of membrane 18 h
Answer: both are correct but B might be a more common cause.
"more common in the setting of prematurity and prolonged rupture of the membranes"
http://emedicine.medscape.com/article/229091-overview#a4
http://www.cdc.gov/groupbstrep/about/transmission-risks.html
Relative contraindication of methergine? HYPERTENSION (it is used to manageof uterine atony, postpartum hemorrhage)

001
497.A postmenopausal woman has her last period 4 years ago. She is now complaining of recent intermittent red vaginal
bleeding. Vaginal examination is normal except for small amount of red blood on the cervical os. What is your next step?
A. Dilatation and curettage
B. Endometrial biopsy
Answer: B
An endometrial biopsy is considered the gold standard for evaluation of post-menopausal bleeding. Endometrial biopsy can be
obtained with an endometrial pipelle in the outpatient setting, or by hysteroscopy and curettage (with or without dilatation) in
either the outpatient or inpatient setting
Http://www.ranzcog.edu.au/doc/doc_view/2099-54-postmenopausal-bleeding.html

498.Long scenario, woman with grayish-white vaginal discharge with fishy smell, Dx?
A- Bacterial vaginosis
B- Candida albicans
Answer: A
Reference: http://familydoctor.org/familydoctor/en/diseases-conditions/vaginal-discharge/symptoms.html

499.Nulliparous postmenopausal women presented with spotting what u will do next:


A- us
B- uterine biopsy
Answer: B
Screen all women with postmenopausal vaginal bleeding (PMB) for endometrial cancer.
Reference: http://www.jfponline.com/specialty-focus/gynecologyobstetrics/article/postmenopausal-bleeding-first-steps-in-the-
workup/940b4bf896fa86b753fd5696c0ab7d82.html

500.One question about endometriosis She not want to conceive What is the definitive Treatment?
A) Total abdominal hysterectomy (TAH)
B) Ablation
Answer: A
Link: http://www.sciencedirect.com/science/article/pii/S1110569011001348

501.Which ligament prevent uterine prolapse?


A) Round ligament
B) Uterosacral ligament
Answer b
The uterosacral ligaments are by far the most important ligaments in preventing uterine prolapse.

502.Pregnant at 10 weeks GA, came ē RLQ pain (no other symptoms), vitals was normal except for tachycardia, CBC was
normal, what's the Dx:
A- Ruptured appendicitis
B- Ruptured tubal pregnancy
Unclear MCQ

503.Pregnant taking sulfunurea what the congenital defect:


A. Teeth
B. Renal

000
Sulfonylureas are antidiabetic drugs widely used in the management of diabetes mellitus type 2. They act by increasing insulin
release from the beta cells in the pancreas.
Effect on fetus:
The safety of sulfonylurea therapy in pregnancy is unestablished. Prolonged hypoglycemia (4 to 10 days) has been reported in
children born to mothers taking sulfonylureas at the time of delivery. Impairment of liver or kidney function increase the risk of
hypoglycemia, and are contraindications. As other anti-diabetic drugs cannot be used either under these circumstances, insulin
therapy is typically recommended during pregnancy and in hepatic and renal failure, although some of the newer agents offer
potentially better options.

504.Pathophysiology of PCOS:
A. Increased insensitivity of androgen
B. Increased androgen activity
Answer: B

Toronto note 2015

505.Q about deceleration in fetal assessment?


A- Good prognostic factor
B- Bad prognostic factor
Answer: b
Toronto note 2015

506.Complication of screening for Down syndrome?


A) Rupture of membrane
B) Abortion
Answer: B
Https://yhdp.vn/uptodate/contents/mobipreview.htm?27/32/28169#H12

507.Lady comes to you at 20 days postpartum complaining of yellowish odorless


Vaginal discharge and the cervix is pink to red color, her pregnancy was normal with no complication; what you will do?
A. Reassurance
B. Do culture
Answer: A

508.Pregnant lady at 32 weeks comes with regular uterine contraction, fetal head at -2 what is your action?
A. Bed rest
B. Give her steroid
Incomplete Question

509.Kallman syndrome how to dx


A. Gonadotropin
B. Urinary something
Answer: A

002
We can diagnosis KS by exclusion, physical Ex (delay puberty + insomnia), The main biochemical parameters in men are low
serum testosterone and low/normal levels of the gonadotropins LH and FSH. In women low serum estradiol and low/normal
levels of LH and FSH. Normal anterior pituitary function & Normal appearance of the hypothalamus and pituitary region on MRI.
Http://emedicine.medscape.com/article/122824-workup

510.What to see under microscope in bacterial vaginosis


A) Cell with multiple inclusion
B) Giant cell
Answer:
Demonstration of clue cells on a saline smear is the most specific criterion for diagnosing BV. Clue cells are vaginal epithelial cells
that have bacteria adherent to their surfaces. The edges of the squamous epithelial cells, which normally have a sharply defined
cell border, become studded with bacteria. The epithelial cells appear to be peppered with coccobacilli.
Http://emedicine.medscape.com/article/254342-workup
Https://yhdp.vn/uptodate/contents/mobipreview.htm?29/56/30602

511.40 year-old female, completed her family. She has endometrioma. Presented complaining of mild dysmenorrhea and
severe pain during intercourse. What is the most appropriate management?
A. Removal of the cyst and ablation of the endometriosis lesions
B. TAH + BSO
Answer : B Oophorectomy
Definitive surgery — Definitive surgery involves hysterectomy, with or without removal of the fallopian tubes and ovaries.
Definitive, rather than conservative, surgery for treatment of endometriosis should be considered when (1) incapacitating
symptoms persist following conservative surgery and medical therapy, (2) moderate to severe disease is present and future
pregnancy is not desired, or (3) hysterectomy is indicated for coexisting pelvic pathology [ 1 ]. The decision to perform a
definitive procedure is primarily dependent upon the patient's interest in maintaining child-bearing potential.
Https://yhdp.vn/uptodate/contents/mobipreview.htm?14/29/14810

512.Postmenopausal women with bone metastasis, BMD T score -3. Came with vertebral fractures. What is your appropriate
management?
A. Estrogen
B. Bisphosphonate
Answer: B ?
Bisphosphonates: This group of drugs works best in cases where metastasis is weakening the bone.
How it works. You receive bisphosphonates by IV infusion every 3 to 4 weeks. These drugs help with bone metastasis by:
Slowing bone damage and reducing the risk of bone fractures
Easing bone pain
Reducing high levels of calcium in the blood
Http://www.webmd.com/cancer/treating-bone-metastasis-breast-cancer

513.A 14 years female, with 6 month history of lower mid abdominal pain , the pain is colicky radiate to the back and upper
thigh, begin with onset of manse and last for 2-4 days, , physical examination of abdomen and pelvis normal, normal
secondary sex development, what is the most likely diagnosis?
A) Primary dysmenorrhea
B) Secondary dysmenorrhea.
Answer: A
Primary dysmenorrhea refers to the presence of recurrent, crampy, lower abdominal pain that occurs during menses in the
absence of demonstrable disease that could account for these symptoms.
Secondary dysmenorrhea has the same clinical features, but occurs in women with a disorder that could account for their
symptoms, such as endometriosis, adenomyosis, or uterine fibroids.
Http://emedicine.medscape.com/article/253812-clinical

003
514.Premenstrual syndrome :
A) More in the first half of menses
B) More in the 2nd half of menses
Answer: B

515.Patient with odorless vaginal disch, grey-white. Spores on wet mount.


A-Candida
B-Othet opts
Answer: A candida

Http://emedicine.medscape.com/article/2188931-overview

516.Pregnant woman..what type of fibroid will put her in high risk of


Miscarriage?
A. Submucosa
B. Intramural
Answer: Intramural
Explanation: Types of Fibroids:
● Intramural fibroids, the most common, grow in the wall of the uterus.
● Subserosal fibroids grow on the outside of the uterus. As they grow larger, they can cause pain due to their size or
pressure put on nearby organs.
● Submucosal fibroids grow just underneath the uterine lining and can crowd into the uterus cavity and lead to heavy
bleeding and other more serious complications.
● Pedunculated fibroids grow on small stalks inside or outside the uterus.
It's possible to have more than one type of fibroid.
Http://www.webmd.com/women/uterine-fibroids/ss/slideshow-fibroid-overview
It is known that the frequency of spontaneous miscarriage is increased in pregnant women who have fibroids. One study found
that the rate of miscarriage in women with fibroids was 14% compared with 7% in women without fibroids. Research has found
that the size of a fibroid does not affect the risk of miscarriage, but that the number of fibroids may do. Multiple fibroids
increase the risk of miscarriage (miscarriage rate found to be 23%) compared with a single fibroid (miscarriage rate was 8%). It is
also thought that the position of the fibroid in the uterus is important. Early spontaneous miscarriage happens more often when
the fibroids are in the main body of the uterus when compared with fibroids being in the lower segment of the uterus or those
fibroids which are intramural or submucosal. It is not known exactly how fibroids cause miscarriage. Possible mechanisms
include: increased irritability of the uterus, mechanical compression by the fibroid and/or damage to the blood supply to the
growing placenta or foetus.
Link: http://www.emedmd.com/content/fibroids-pregnancy

517.Nulli pregnant Decrease amount of urine, SOB?


A) Acute amniotic emboli-
B) acute glomerulonephritis
Answer: B
Explanation: Only reasonable answers from choices above - incomplete question

004
518.Regards cervical cancer Wt true ?
A)Pap smear decrease incidence dramatically
B)CIN start as low grade then high grade
Answer: BOTH!

519.How to differentiate a large ovarian cyst from ascites?


A. Dull anteriorly and resonant laterally
B. Resonant anteriorly and dull laterally

Answer:a (midabdominal dullness and lateral tympany)

Reference: https://books.google.com.sa/books?id=qw4iksfm8x8c&pg=pa472&lpg=pa472&dq#v=onepage&q&f=false

520.Nulli pregnant,decrease amount of urine, sob :


A. Acute amniotic emboli
B. Acute glumerularnephritis

521.Radio sensitive tumor:


A. seminomas << yolk sac embryonl carcinoma
Answer:a
Radiosensitive tumours are tumours which respond well to radiotherapy. Radiotherapy may be effective alone, or may require
the addition of cytotoxic chemotherapy as in the more advanced stages of a wilm's tumour and hodgkin's disease.
This group includes:
Malignant lymphomas
Seminomas
Medulloblastoma
Neuroblastoma
Wilm's tumour
Early cervical carcinoma
Vaginal carcinoma
Most head and neck tumours
Reference: http://www.gpnotebook.co.uk/simplepage.cfm?id=1993342989

005
522.woman with recurrent uti , why ?
A. because it cleans itself of the from anus to vulva
Answer: cervical cap usage
Recurrent urinary tract infection (uti) refers to ≥2 infections in six months or ≥3 infections in one year.
Risk factors for recurrent uti :

•nhe frequency of sexual intercourse ( strong rf )


•smermecede use ( strong rf )
•women with recurrent uti have been shown to have an increased susceptibility to vaginal colonization with
uropathogens compared with women without a history of recurrences.
•melvec anatomy may predispose to recurrent uti in some women, with a shorter distance from the urethra to the
anus being associated with increased risk.
•among postmenopausal women, mechanical, and/or physiological factors that affect bladder emptying are
associated with recurrent uti.
Reference: uptodate

523.What is papanicolaou smear? (choices: how many sample and how many area?)
Answer: https://www.nlm.nih.gov/medlineplus/ency/article/003911.htm

524.DVT in pregnant. What will you do?


A. Compression ultrasound

Answer: A
Reference: http://emedicine.medscape.com/article/2056380-workup#c10

525.Ascites Bilateral ovarian mass, what is the most likely tumor?

Answer:

Reference: uptodate

526.60 years old female with vague Sx


Labs : ALP high , DEXA scan showed mild osteoporosis , what to give ?
A. Bisphosphonate
Answer: A

006
527.Hirsutism obese female and irregular menses:
A. PCOS
Answer: A

528.Asymptomatic female has 7 cm cyst in ovary the period is normal


A. Follicular cyst
Answer: A
((Robbins an Cotran pocket basis pathology,7E,p541))

529.Gray Virginal discharge what can u see in microscpy ( there is no clue cell or hyphee in the answer )
Gray vaginal discharge is more indicative of bacterial vaginosis which characterized by clue cells in microscopy.
Clue cell is epithiail cell covered or surrounded by bacteria giving it that stippled appearance

530.Pregnant with n. N. Gonhorea Tt :


Ceftrixone +azthromycn or doxcyclin (due to co infection of chlamedia)

531.Tamoxifen SE or Complications?
A. Uterine Bleeding

532.Pregenat healthy come to prenatal follup every thing is normal Lab show : nitrat +ve , WBC increase , what is the dignosis
?
A. Asymptomaic bacturia

533.Pregenat , has +ve protein in urin and i think hypertention , she recevid mg sulphate , what is prevent ( or treat ) :
A. Sezure

534.Which of the following muscles will be affected in perineal tear during normal vaginal delivery?
Answer: ? (it depends on the degree of the perineal tear)
1st degree: vaginal mucosa affected only
2nd degree: involvement of perineal body muscles which includes:
● Bulbocavernous
● Superficial transverse perineal muscle
● Pubococcygeus muscles
3rd degree: involvement of the external anal sphincter and/ internal anal sphincter.
4th degree: extent through the anal mucosa.

007
Reference: uptodate

535.What is the most common cause of vaginal bleed?


Answer: anovulation
The most common specific causes in adult women who are not known to be pregnant are:
● Anovulatory (dysfunctional uterine) bleeding > most common
● Complications of an early, undiagnosed pregnancy
● Submucous myoma

536.In polycystic ovarian syndrome, which of the following will be found on blood test?
a. FSH:LH 1:3

Answer: A
Other findings (increased DHEA-S, androstenedione and free testosterone (most sensitive)).

008
537.Mom wants to know if her baby is having thalassemia or not. How you will investigate her antenatally?
Answer:
Screening tests for high risk population: CBC (MCV and MCH), Hb electrophoresis or high performance liquid chromatography
(HPLC)
Confirmatory tests: Chorionic villus sampling (CVS): between 10-12 weeks, Amniocentesis: between 15 weeks to term.
Reference: Toronto Notes

538.Woman with endometriosis. What is the best way to diagnose?


a. Laparoscopy
Answer: A

539.Pregnant with monochorionic twins in week 27. One of them died, what to do?
Answer: delivery
If fetal assessment after 26 weeks of gestation suggests impending death rather than demise of one twin of a monochorionic
pair, we suggest prompt delivery of both twins rather than expectant management given the high risk of neurologic impairment
in the surviving co-twin. Reference: http://www.uptodate.com/contents/twin-pregnancy-
prenatalissues?Source=search_result&search=monochorionic+twins+one+f+them+die&selectedtitle=1%7E150#H24

540.Patient known case of PCOS and wants to get pregnant.


a. Clomiphene citrate

Answer: A
Reference: Toronto Notes

541.Which of the following is equally effective to laparoscopy in a patient with unruptured small ectopic pregnancy?
a. Methotrexate

Answer: A
Reference: Toronto Notes

542.Megaloblastic anemia in pregnant.


a. Give folate

Answer: A
Reference: http://www.ncbi.nlm.nih.gov/pubmed/8612357

543.Female patient came with signs and symptoms of PCOS. Lab results: FSH= 1.5, LH= 10. What is the most likely
Diagnosis?
a. PCOS
Answer: A

544.A patient presented to the ER with severe RLQ pain and positive B-HCG. What is the Diagnosis?
a. Ruptured ectopic pregnancy.

Answer: A

009
545.63 years old female. Pap smear showed atypical squamous cells of undetermined significance (ASCUS). You gave her
local estrogen and after one week pap smear results still showing ASCUS. What will be your next step?
Answer: colposcopy + biopsy
Refer to the chart at the end of OB/GYN section.

546.What is the most common cause of secondary amenorrhea?


a. Pregnancy

Answer: A

547.Typical case of PCOS. Biopsy showed Endometrial hyperplasia. What is the cause?
a. Unopposed estrogen
Answer: A
Reference: Medscape

548.Multipara, 38 weeks pregnant. Cervical os is 7 cm with cord prolapse?


a. CS

Answer: A
549.Diabetic female complaining of itchy vaginal discharge?
a. Candidiasis

Answer: A
Refer to the table at the end of OB/GYN section.

550.A female patient presented with green vaginal discharge and pruritus?
a. Trichomonas

Answer: A
Refer to the table at the end of OB/GYN section.

551.A female patient can’t get pregnant for 3 years. Recently she developed breast milk. What is the most likely diagnosis?
a. Hyperprolactinemia.

Answer: A

552.30-year-old lady having whitish vaginal discharge, odorless and labial erythema. What is the most likely cause?
A. Candida infection.

Answer: A
Candida vulvovaginitis: Erythematous, excoriated vulva/vagina with thick white discharge without odor.

021
553.What is the best time to estimate the chorionicity of the twins on ultrasound?
Answer: 10-13 weeks.
Assessment of chorionicity: Ultrasonography is an effective prenatal tool for determining amnionicity and chorionicity. The
optimal time for performing the ultrasound examination is in the first trimester after 7 weeks (sensitivity ≥98 percent), with
lower but acceptable accuracy in the early second trimester.
Reference: http://www.uptodate.com/contents/twin-pregnancy-prenatal-issues

554.20 years old female, presented with amenorrhea, short stature and webbed neck. Which hormone will be affected?
a. Decreased estrogen

Answer: A
Reference: Uptodate.

555.A pregnant during labor. Her cervical opening is 6 cm. Which stage?
Answer: Active Phase

First Stage of Labor:


The first stage of labor is the longest and involves three phases:
Early Labor Phase –The time of the onset of labor until the cervix is dilated to 3 cm.
Active Labor Phase – Continues from 3 cm. Until the cervix is dilated to 7 cm.
Transition Phase – Continues from 7 cm. Until the cervix is fully dilated to 10 cm.
Americanpregnancy.org

556.Women with itchy, whitish vaginal discharge. KOH shows pseudohyphae. What is the treatment?
a. Ointment Miconazole

Answer: A
Candidal vaginitis: Vaginal vulvar pruritus, burning, or irritation (which may be worse during intercourse) and dyspareunia are
common signs, as is a thick, white, cottage cheese–like vaginal discharge that adheres to the vaginal walls.
Https://www.merckmanuals.com/professional/gynecology-and-obstetrics/vaginitis,-cervicitis,-and-pelvic-inflammatory-disease-
pid/candidal-vaginitis

557.Women developed itchy rash with whitish vaginal discharge after nitrofurantoin course for UTI. How will you treat this
patient?
a. Miconazole
Answer: A
020
558.Calculate day of delivery.
Answer:
EDC using Naegle’s Rule: ƒ 1st day of LMP + 7 d – 3 mo ƒ e.g. LMP = 1 Apr 2013, EDC = 8 Jan 2014 (modify if cycle >28 d by adding
number of d >28) Reference: Toronto Notes.

559.Woman with bilateral ovarian abscess, what is the diagnosis?


Answer:
Pelvic inflammatory disease
The major complication of pelvic inflammatory disease are tuboovarian abscess,chronic pelvic pain,infertility and ectopic
pregnancy

560.Trichomonas vaginosis read about treatment?


Answer: metronidazole
Refer to table below

561.Yellow secretion pv with pmn >10 ?


Answer: trichomonas
Symptoms range from none to copious, yellow-green, frothy vaginal discharge with soreness of the vulva and perineum,
dyspareunia, and dysuria.
If trichomoniasis is present in microscopic examination, numerous neutrophils are also present.
Http://www.merckmanuals.com/professional/infectious-diseases/sexually-transmitted-diseases-stds/trichomoniasis

562.DVT prophylaxis for pregnant woman?


Answer: heparin ..
By Gyne consultanat

563.Pregnant in 38th week .. By examination .. The fetus in breech position .. The cervix is closed. What is the next step?
Deliver her by c/s
Answer:

022
564.Risk factor of recurrent UTI in females
A- (whipping from back to front)
Answer:

565.Tubal ligation then c/o vginal spotting after 6 wk of amenorrhea


Answer:
Delayed complications of laparoscopic tubal ligation include the following:
• Failure
• Filshie clip complications
• Regret
• Ectopic pregnancy
• Menstrual changes
• Hysterectomy
• Sexual function
Ref: http://emedicine.medscape.com/article/1848429-overview#a6

566.Pregnant lady with lobar pneumonia what is the type of immune the baby will have?
Answer: there are no choices but the answer is “masseve emmuneny”

567.Typical case of PCOS what is skin manifestation associated with?


Answer:
Acanthosis nigricans
Http://www.ncbi.nlm.nih.gov/m/pubmed/17645376/

023
568.Female try to get pregnant for one year she is healthy and her husband is known to be healthy what to do 1st ?
Semen analysis
Answer :
Since the woman is healthy we should investigate man, most common factor for this is semen abnormalities, therefore, semen
analysis and culture.
Reference: http://www.cdc.gov/reproductivehealth/infertility/

569.A female postpartum with upper lateral quadrant mass, redness, tenderness,with +ve lymph nodes:
Answer: Mastitis
During lactation, enlarged intramammary and/or axillary lymph nodes may be seen. The hyperplastic nodes are felt to be related
to the bacterial seeding of the nipple by the infant during breastfeeding. These nodes are typically seen in the upper outer
quadrant of the breast and axilla.
Benign entities include galactocele, fibroadenoma, obstructed milk duct, mastitis with or without abscess, hyperplastic
intramammary and/or axillary lymph nodes, and granulomatous mastitis. Malignant diseases include pregnancy-associated
breast cancer and metastatic disease.
Reference: http://www.ncbi.nlm.nih.gov/pmc/articles/PMC3781252/

570.Patient present at 10 week with painless bleeding not part of conception, cervix closed, what is the diagnosis?
- Threatened abortion
Answer please see attached table

571.Patient with pelvic inflammatory disease, didn't respond to antibiotic after 3 days’ examination revealed fluctuating
mass, how to manage?
A. Laparoscopy

Answer: minimally invasive drainage or laparoscopy, depend on choices!


A case of tubo-ovarian abscess
Antibiotics are the mainstay of treatment for TOA. In some women, antimicrobial therapy must be combined with a
minimally invasive drainage procedure or surgical treatment

Reference: http://cursoenarm.net/UPTODATE/contents/mobipreview.htm?5/63/6128?Source=HISTORY

024
572.Hormonal changes at menopause in estrogen, LH, FSH.
Answer:
Follicle-stimulating hormone (FSH) levels are higher than luteinizing hormone (LH) levels, and both rise to even higher values
than those seen in the surge during the menstrual cycle. FSH is the diagnostic marker for ovarian failure. Estrogen levels begin
to fall.

573.Pregnant lady no symptoms UTI, no frequency no urgency no dysuria, Positive urine culture, Diagnosis?
A. Asymptomatic bacteriuria
Answer: A
Http://emedicine.medscape.com/article/2059290-overview

574.With heavy menses each 2 weeks, what to do?


A. Endometrial biopsy
Answer:Investigations:
CBC, serum ferritin
Β-hcg
TSH, free T4
Coagulation profile (especially in adolescents): rule out von Wellebrand’s disease
Prolactin if amenorrheic
FSH, LH
Serum androgens (especially free testosterone)
Day 21 (luteal phase) progesterone to confirm ovulation
Pap test
Pelvic U/S: detect polyps, fibroids; measure endometrial thickness (postmenopausal)
SHG: very sensitive for intrauterine pathology (polyps, submucous fibroids)
HSG
Endometrial biopsy: consider biopsy in women >40 yr
Must do endometrial biopsy in all women presenting with postmenopausal bleeding to exclude endometrial cancer
D&C: not for treatment; diagnosis only (usually with hysteroscopy)
Reference: (toronto notes)

575.Female in 12 weeks’ gestation age develop UTI treated, then at 27 weeks again had another UTI what you will do?

A. You will treat even if asymptomatic


Answer: A
Any asymptomatic UTI in pregnant women should be treated
Reference: http://emedicine.medscape.com/article/452604-treatment

576.Female in 12 wk gestation develop UTI treated then at 27 wk again had UTI what you will do ? E. Coli >100000 -you will
A. Treat if asymptomatic
Answer : treat even if asymptotic
Asymptomatic bacteriuria in pregnancy is more likely to lead to cystitis and pyelonephritis (Obstetrics and Gynecology, sixth
edition, Charles R. B. Beckmann, page 159)

577.What you should look for in case of gonorrhea?


A. Chlamydia trachomatis

Answer: A

025
Because chlamydia is also frequently found in conjunction with Neisseria gonorrhoeae infection, any patient with known or
suspected gonorrhea infection should also be evaluated for chlamydia. (Obstetrics and Gynecology, sixth edition, Charles R. B.
Beckmann, page 248)

578.About old female with recurrent fracture ..


A. Estrogen Def
Answer:
Estrogen affects the development of cortical and trabecular bone, although the effect on the latter is more pronounced. Bone
density diminishes at the rate of approximately 1% to 2% per year in postmenopausal women, compared with approximately
0.5% per year in perimenopausal women. (Obstetrics and Gynecology, sixth edition, Charles R. B. Beckmann, page 332)

579.63-Married female Patient with white vaginal discharge, odorless, Wet test was negative, KOH test showed
Psudohyphae, what is the diagnosis?
A. Vaginal candidiasis.

Anwer: Vaginal candidiasis


Vulvovaginal candidiasis is caused by ubiquitous airborne fungi. Approximately 90% of these infections are caused by Candida
albicans. The most common presenting complaint for women with candidiasis is itching, although up to 20% of women may be
asymptomatic. Burning, external dysuria, and dyspareunia are also common. The vulva and vaginal tissues are often bright red in
color, and excoriation is not uncommon in severe cases. A thick, adherent “connage cheese” discharge with a ph of 4 to 5 is
generally found. This discharge is odorless. Diagnosis requires either visualization of blastospores or pseudohyphae on saline, or
10% KOH microscopy, or a positive culture in a symptomatic woman. (Obstetrics and Gynecology, sixth edition, Charles R. B.
Beckmann, page 243)

580.Pregnant with repeated infection WBC 100000, treated and come again with same symptom what to do ? Treat her even
she is asymptomatic ?

Answer :treat even asymptomatic


Uptodate

581.42 with heavy menses each 2 wk what to do


Answer
CBC count, Iron studies, Coagulation factors, Human chorionic gonadotropin, Thyroid function tests and prolactin level, Liver
function and/or renal function tests, Hormone assays, pap smear, and cervical specimen.
(http://emedicine.medscape.com/article/255540-workup)

582.Pregnant women at 14 weeks with splenomegaly low platelet what is diagnosis ?


Answer:
3
*low platelet count (<150000/mm ) is most likely indicative to gestational thrombocytopenia (Kaplan page 39)
* Splenomegaly with pregnancy should be considered as a high risk pregnancy and it is to be dealt under the supervision of
senior obstetrician in a tertiary care centre because it can complicate the maternal and fetal outcome
(http://www.jemds.com/latest-articles.php?At_id=1540)

026
583.Follicular cell of ovary what it is orign embryologically???
Answer:
The outer ovarian cortex consists of follicles embedded in a connective tissue stroma. Embryologically, this stroma is the
medulla that originated as the gonadal ridge, while the cortex originated as coelomic epithelium. The medulla contains smooth
muscle fibers, blood vessels, nerves, and lymphatics. (Obstetrics and Gynecology, sixth edition, Charles R. B. Beckmann, page 40)

584.About Endometriosis best management? And treatment..


Answer:
Clinical manifestations of endometriosis fall into three general categories: pelvic pain, infertility, and pelvic mass. The goal of
therapy is to relieve these symptoms. There is no high quality evidence that one medical therapy is superior to another for
managing pelvic pain due to endometriosis, or that any type of medical treatment will affect future fertility. Therefore,
treatment decisions are individualized, taking into account the severity of symptoms, the extent and location of disease,
whether there is a desire for pregnancy, the age of the patient, medication side effects, surgical complication rates, and cost.
Treatment options include:
Expectant management
Analgesia
Hormonal medical therapy
o Estrogen-progestin oral contraceptives, cyclic or continuous
o Gonadotropin-releasing hormone (gnrh) agonists
o Progestins, given by an oral, parenteral, or intrauterine route
o Danazol
o Aromatase inhibitors
Surgical intervention, which may be conservative (retain uterus and ovarian tissue) or definitive (removal of the uterus
and possibly the ovaries)
Combination therapy in which medical therapy is given before and/or after surgery
(http://www.uptodate.com/contents/overview-of-the-treatment-of-
endometriosis?Source=search_result&search=Endometriosis&selectedtitle=1~150)

585.Questions about cervical cancer (staging, biopsy)


Answer:
Staging:
Stage I is carcinoma strictly confined to the cervix; extension to the uterine corpus should be disregarded.
o Stage IA: Invasive cancer identified only microscopically
o Stage IB: Clinical lesions confined to the cervix or preclinical lesions greater than stage IA
Stage II is carcinoma that extends beyond the cervix but has not extended onto the pelvic wall. The carcinoma involves
the vagina but not as far as the lower third section.
o Stage IIA: No obvious parametrial involvement. Involvement of as much as the upper two thirds of the vagina
o Stage IIB: Obvious parametrial involvement, but not onto the pelvic sidewall
Stage III is carcinoma that has extended onto the pelvic sidewall and/or involves the lower third of the vagina.
o Stage IIIA: No extension onto the pelvic sidewall, but involvement of the lower third of the vagina
o Stage IIIB: Extension onto the pelvic sidewall or hydronephrosis or nonfunctioning kidney
Stage IV is carcinoma that has extended beyond the true pelvis or has clinically involved the mucosa of the bladder
and/or rectum.
o Stage IVA: Spread of the tumor onto adjacent pelvic organs
o Stage IVB: Spread to distant organs
(Obstetrics and Gynecology, sixth edition, Charles R. B. Beckmann, page 385)

027
586.Group B strep positive pregnancy and she is in her 24 weeks , when to give prophylactic antibiotic ?
Answer: during labour
All women who are GBS positive by rectovaginal culture should receive antibiotic prophylaxis in labor or with rupture of
membranes. (Obstetrics and Gynecology, sixth edition, Charles R. B. Beckmann, page 165)

587.Pregnant lady no symptoms UTI, no frequency no urgency no dysuria Positive urine culture? Diagnosis?

Answer: Asymptomatic bacteriuria


A urine culture is obtained at the onset of prenatal care and patients with asymptomatic bacteriuria are treated with ampicillin,
cephalexin or nitrofurantoin. (Obstetrics and Gynecology, sixth edition, Charles R. B. Beckmann, page 159)

588.Multiple something? -bone marrow>>not sure 38)isotretinoin most feared complication -birth defect -( I don't know
about this question is it correct or not )
Answer:
Isotretinoin: This drug must not be used during pregnancy or by those who may become pregnant during treatment. Since this
drug can be absorbed through the skin and lungs and may harm an unborn baby, women who are pregnant or who may become
pregnant should not handle this medication or breathe the dust from the capsules.
You must have two negative pregnancy tests before starting this medication. You must have a monthly pregnancy test during
treatment with isotretinoin. If the test is positive, you must stop taking this medication and consult your doctor right away.
(http://www.webmd.com/drugs/2/drug-6662/isotretinoin-oral/details#precautions)

589.Pregnant in the third trimester with history of recurrent herpes simplex, she is in labour, during exam no lesions what to
do?
Answer : 1) CS IV acyclovir 2) Do specular exam before
In the absence of active lesions or prodromal symptoms, vaginal delivery should be allowed.
(http://emedicine.medscape.com/article/274874-overview#a1)

590.Primary amenorrhea , what is the next investigation ?


Answer:
Thyroid-stimulating hormone (TSH), b-HCG, prolactin, follicle-stimulating hormone (FSH), and luteinizing hormone (LH)
measurements are always the first line of testing. If hirsutism is predominant upon examination, include androgen testing:
measure testosterone, dehydroepiandrosterone sulfate (DHEAS), androstenedione, and 17-OH progesterone to determine the
organ of cause (eg, ovary vs adrenal gland).
If the history or physical findings suggest a chronic disease process, indicated tests may include of the erythrocyte sedimentation
rate (ESR), liver function tests, blood urea nitrogen (BUN) determination, creatinine determination, and urinalysis.
If the history and physical findings suggest a delay in puberty, assessing FSH and LH levels and determining bone age are
important in differentiating pubertal delays as a cause. (http://emedicine.medscape.com/article/252928-workup)

028
591.Pregnant lady with vaginal discharge and + KOH , what is Diagnosis :
A- bacterial vaginisis
Answer: A
Diagnosis is usually established by vaginal ph greater than 4.5, presence of clue cells (vaginal epithelial cells with borders that
are covered with small bacteria), examining the discharge microscopically; may show clue cells. A positive whiff test after adding
a drop of KOH to some of the discharge on a microscopic slide suggests bacterial vaginosis.
(clincalkey)

592.Type of fibroid cause abortion :


Answer: submucosal
Fibroids that bulge into the uterine cavity (submucous) or are within the cavity (intracavitary) may sometimes cause
miscarriages. The fertilized egg comes down the fallopian tube and takes hold in the lining of the uterus. If a submucosal fibroid
happens to be nearby, it can thin out the lining and decreases the blood supply to the developing embryo. The fibroid may also
cause some inflammation in the lining directly above it. The fetus cannot develop properly, and miscarriage may result.
Http://www.fibroidsecondopinion.com/fibroids-and-pregnancy/

593.Postmenopausal with osteoporosis and high Alkaline phosphatase ,What will you give :
A)
Answer:
Bisphosphonate
“DISCUSSION: Bisphosphonate treatment lowered ALP levels, and this decrease was strongly correlated with a decrease in BAP.
Among blood test data, the decrease in BAP had the strongest correlation with the ALP decrease.
CONCLUSION: For treatment of osteoporosis, ALP is an acceptable alternative to BAP. Elevated ALP in postmenopausal women is
mainly caused by high bone nurnover.”
Http://www.ncbi.nlm.nih.gov/m/pubmed/25534961/

594.Female with ductal carcinoma Doctor want treat her, what is the gene responsible for that cancer? (No BRCA1 in options)
Tp53
I don't remember the remaining
Answer:
029
Most breast cancers are associated with BRCA 1 & BRCA 2 mutations, but inherited changes in the TP53 gene greatly increase
the risk of developing breast cancer, as well as several other forms of cancer, as part of a rare cancer syndrome called Li-
Fraumeni syndrome (described below). These mutations are thought to account for only a small fraction of all breast cancer
cases. Noninherited (somatic) mutations in the TP53 gene are much more common than inherited mutations, occurring in 20 to
40 percent of all breast cancers
Https://ghr.nlm.nih.gov/gene/TP53#conditions

595.What is the antithyroid used in pregnancy?


Propylthiouracil
Answer:
PTU
Because of the risk of fetal abnormalities associated with methimazole, propylthiouracil may be the treatment of choice when
an antithyroid drug is indicated during or just prior to the first trimester of pregnancy.
(Merck manual)

596.Contraindication of external cephalic version?


Answer: Contraindications include multiple pregnancy, severe fetal anomaly, ruptured membranes, significant third-trimester
bleeding, and other indications for caesarean section (e.g., placenta praevia or uterine malformation). Ref. BMG Best Practice

597.Ovarian cancer tumor marker


Answer: CA125
CA125 levels >35 U/ml in post-menopausal women warrant concern for ovarian cancer. In the pre-menopausal patient this
serum marker yields little because elevated levels are associated with many benign conditions, such as uterine fibroids, PID,
endometriosis, adenomyosis, pregnancy, and menstruation. Levels >200 U/ml in pre-menopausal women warrant a referral to a
gynaecological oncologist for further evaluation.
Generally it is preferable to describe specific signs in lieu of declaring fetal distress that include:
Decreased movement felt by the mother
Meconium in the amniotic fluid ("meconium stained fluid")
Non-reassuring patterns seen on cardiotocography:
Increased or decreased fetal heart rate (tachycardia and bradycardia), especially during and after a contraction
Decreased variability in the fetal heart rate
Late decelerations
Biochemical signs, assessed by collecting a small sample of baby's blood from a scalp prick through the open cervix in labor
Fetal metabolic acidosis
Elevated fetal blood lactate levels (from fetal scalp blood testing) indicating the baby has a lactic acidosis
Some of these signs are more reliable predictors of fetal compromise than others. For example, cardiotocography can give high
false positive rates, even when interpreted by highly experienced medical personnel. Metabolic acidosis is a more reliable
predictor, but is not always available.
Ref. Wikipedia

598.Mother close to delivery developed respiratory symptoms + fetal distress :


A- Amniotic fluid embolism
ANSWER: A

599.Long scenario of patient with symptoms of dysfunctional uterine bleeding; you diagnosed her what medication you'll
prescribe:
- OCP
Answer: A

031
Progestogens are first-line treatment for DUB, particularly when associated with anovulation. It Can also be delivered through
progesterone-containing iuds and contraceptive implants.
BMG Best practice

600.Mother after delivery sees snakes crawling into her baby bed :
Postpartum psychosis
Answer: A

601.12 week pregnant w high blood pressure:


Answer: pre-existing hypertension
Women who had high blood pressure before pregnancy – or are diagnosed with it before 20 weeks – have chronic
hypertension.

602.Pregnant works under sun, developed patches over the forehead and cheeks :
A- cholasma
Answer:
Melasma also known as Chloasma faciei is thought to be the stimulation of melanocytes (cells in the epidermal layer of
skin that produce a pigment called melanin) by the female sex hormones estrogen and progesterone to produce more
melanin pigments when the skin is exposed to sun.

603. Causes of fetal distress:


Answer:
1- cord prolapse
2-placenta abruption
3-anemia
4-oligo/polyhydramnios
5-DM/PET
6-IUGR
7-post term

604.Endometriosis typical case presentation and asking about the diagnosis


Answer:
Gold standard is laparoscopy

605.30*A female presenting with hirsutism, acne, and irregular menstruation. What is the most probable finding?
High androgen
Answer:
PCO ( 1- high androgen , insulin resistance, acanthosis nigra)
To diagnosed PCO
1- US 2- lab ( high: testosterone , androgen, insulin
Low : progesterone , increase ratio LH/FSH > 2:1

606.A pregnant lady in her first trimester did not have any vaccination for rubella what to do?
Answer:?
MMR vaccine unsafe during pregnancy so if patient not immunized give MMR vaccine after delivary

030
607.Pregnant in labor cervical opening 6 cm, which stage?
Answer:
First Stage:
Latent>> 3-4cm dilation
Active>> from 4cm to 10cm

608.Definition of leiomyoma?
Answer :
Leiomyoma: A benign tumor of smooth muscle, the type of muscle that is found in the heart and uterus. A leiomyoma
of the uterus is commonly called a fibroid.

609.Treatment of community acquired pneumonia in pregnancy?


Answer: ?
For pregnant women:
➢ Community acquired pneumonia and no features of severe disease: antipneumococcal beta-lactam (ceftriaxone,
cefotaxime, ampicillin-sulbactam) plus azithromycin
➢ Allergic reactions to cephalosporins: clindamycin plus aztreonam, unless they have severe pneumonia.
➢ Severe pneumonia and past reactions to cephalosporins: vancomycin plus azithromycin plus aztreonam.
Reference:http://www.uptodate.com/contents/treatment-of-respiratory-infections-in-pregnant-women

610.Vaginal infection lead to neonatal conjunctivitis


Answer:
The most common bacteria that can cause serious eye damage are gonorrhea and chlamydia. These can be passed from
mother to child during birth.
Time of onset:
Neisseria gonorrhoeae: Delivery of the baby until 5 days post-birth (Early onset)
Chlamydia trachomatis: 5 days post-birth to 2 weeks (Late onset - C.trachomatis has longer incubation period)
Wikipedia, Medline pluse

611.Pregnant early, low appetite nausea fatigue. Blood: low Hb, high MCV, MCHC, high TIBC: whats the reason
Answer: macrocytic anemia( high MCV, High MCHC ) due to vitamin b12 or folate deficiency . While High TIBC is due to
pregnancy

612.Missed period 2 months , high bhcg , examination show 16 weeks GA ,U/S show fetus small for data ? Dx?
1- choriocarcinoma (raising or plateau HCG )
2- hydatidiform (large for date and BHCG>100,000)
3- placenta in site trophoblastic tumor(low BHCG)
Answer:

613.Woman with IUD and came with vaginal pain and discharge, (what organism)?
Answer:
Actinomyces infections in association with IUD use have been reported.
And PID ( Actinmuces ) is the most common infection associated with IUD ( resource : First aid Obs&Gyne )

032
614.What is the best treatment of endometriosis?
Answer:

Ref: Toronto notes

615.Patient complain of scanty pubic hair and primary amenorrhea & secondary sexual character & develop breast with
bilateral growing swelling what is the diagnosis ?
ANDROGEN INSUFFICIENCY ?
Answer: please see table

033
616.Endometriosis (Presentation, investigations, treatment)

Reference: http://emedicine.medscape.com/article/271899-overview

617.Patient with BPH+HYPERTENSION what TTT? (best management)

Answer: alpha blocker


Alpha-blockers are medicines that are mainly used to treat high blood pressure (hypertension) and problems with passing urine
in men who have enlargement of the prostate gland. (http://patient.info/health/alpha-blockers)

618.Female with 3 months meses loss what is best action :


A- reassurance
Answer: incomplete question

619.Pregnant lady with hypertension. You're mostly concerned about:


IUGR.**
Answer:

620.Q about the polycystic ovarian Investigation ?

621.Pap smear in old postmenopausal lady, showed ASCUS, was given estrogen cream, FU PAP showed ASCUS again. What’s
your next step?
A-Colposcopy
Answer : A
She had to go for CS , epidural anesthesia is not possible if the cervix is dilated more than 5 cm.
(confirmed by student get full mark in Oby/Gyne )

034
622.Same weight for 6 months , amenorrhea :
A-Eating disorder ?
Answer?

623.Which part of the female urethra is more susceptible to be damaged ( in an intervention I can't remember what)
Answer :??

624.A patient who’s 36 weeks pregnant started having contractions lasting 30 secs. CTG was done and was good.
What to do ?
• Give tocolytics
Answer :

625.If a Pap smear shows HPV, the cytology will show what:
Answer: see chart in OB/Gyne section

626.HYPERTENSION in third tri >> only (gast hypertension )✔


Answer:

627.Pregnant patient came with high blood pressure was given magnisum sulfate, which of the following is sign of
low maginsum in the body ? ( sizure wasn't included)
Answer :
Symptoms of magnesium deficiency include hyperexcitability, muscular symptoms (cramps, tremor, fasciculations,
spasms, tetany, weakness), fatigue, loss of appetite, apathy, confusion, insomnia, irritability, poor memory, and
reduced ability to learn. Moderate to severe magnesium deficiency can cause tingling or numbness, heart changes,
rapid heartbeat, continued muscle contractions, nausea, vomiting, personality changes, delirium, hallucinations, low
calcium levels, low serum potassium levels, retention of sodium, low circulating levels of parathyroid hormone (PTH),[4]
and potentially death from heart failure.[5] Magnesium plays an important role in carbohydrate metabolism and its
deficiency may worsen insulin resistance, a condition that often precedes diabetes, or may be a consequence of insulin
resistance.
Https://en.wikipedia.org/wiki/Magnesium_deficiency_%28medicine%29

UNREVISED QUESTIONS :

628.Pregnent lady in 3 trimester suddenly she developed LL swilling from hip to toes best investigation is :
Answer:
Dopplex , because it is above the knee

629.Which vaccination cant give to pregnant woman?


Answer: all live vaccines (measles, mumps, polio, rubella yellow fever, and varicella )

630.When a couple should seek help regarding infertility ?


Infertility: inability to conceive or carry to term a pregnancy after 1 yr of regular, unprotected intercourse. Toronto notes

631.Primary amenorrhea , what is the next investigation


Http://www.uptodate.com/contents/evaluation-and-management-of-primary-amenorrhea

035
632.Diabetic female c/o itchy vaginal discharge ?
Answer: Candidiasis
Candidiasis: Predisposing factors include:
• Immunosuppressed host (DM, AIDS, etc.)
• Recent antibiotic use
• Increased estrogen levels (e.g. Pregnancy, OCP)

633.Pregnant lady in labor she has preeclampsia ,she already given magnesium sulphate after that she had respiratory
distress with tachypnea(I think side effects of Mg sulphate) , what you are going to give ?
Answer: Ca gluconate (antidote)
Https://en.wikipedia.org/wiki/Calcium_gluconate

634.18 year-old married missed her period for two months, came with rt sided abd pain wts thedx:
A. Ruptured ectopic
Answer:

635.Similar question to previous Q but what's the test to order:


A. Urine hcg
Answer:

636.Vaginal bleeding week 10 fundus 15cm closed os what is the diagnosis


A. Threatened abortion.

036
637.Patient hirsutism, obese x-ray shows cystic ovary, she wants to conceive, best Treatment?!
A. Clomiphene citrate
It is most probably a PCOS case, so for them to pregnant if infertile they need clomiphene and metformin. (Master the boards
step2 CK OBS Gyen chapter)

638.Pregnant women with UTI what is the best drug:


A. Nitrofurantoin
Answer: Nitrofurantoin is an antibiotic that is used for treating urinary tract infections caused by several types of bacteria. It is
effective against E. Coli, Enterobacter cystitis, Enterococcus, Klebsiella, and Staphylococcus aureus.
Pregnancy category: B; contraindicated at term.
(http://reference.medscape.com/drug/macrobid-macrodantin-nitrofurantoin-342567#6)

639.Pregnant lady, fall from stairs, and started to have vaginal bleeding, Diagnosis?
A. Placenta abruption

Answer: is a complication of pregnancy, wherein the placental lining has separated from the uterus of the mother
Prior to delivery. It is the most common pathological cause of late pregnancy bleeding.

640.Female had vaginitis coming with fever & rash.


A. Toxic shock syndrome
Answer: Toxic shock should be considered in any individual who presents with sudden onset of fever, rash, hypotension, renal or
respiratory failure, and changes in mental status. STSS (Streptococcal Toxic Shock Syndrome) most commonly occurs in women,
usually those who are using tampons, TSS develops within 5 days after the onset of menstruation.
Http://emedicine.medscape.com/article/169177-clinical

641.Minimal investigation for Monitor pre-eclampsia.


Answer:
Hypertension plus one or more of the following after the 20th week of pregnancy:
Proteinuria
Thrombocytopenia
Impaired liver function
Signs of kidney trouble other than protein in the urine
Pulmonary edema
New-onset headaches
Visual disturbances
Http://www.mayoclinic.org/diseases-conditions/preeclampsia/basics/tests-diagnosis/con-20031644

642.Female with severe pain during her period and heavy bleeding on examination nodules in uterosacral ligament:
A. Endometriosis.
Answer: A
Signs and symptoms of Endometriosis: Cyclic pelvic pain, abnormal heavy bleeding and nodular uterus or adnexal masses.
Diagnosis: laparoscopy (dark brown clusners of leseons called Endomenreoma “Chocolane Cysn”)
Treanmenn: nsaeds, ocms, Danazol “androgen derevaneve”, leumrolede acenane “leumron” bonh are used no decrease FSH & LH.
Source: Master the boards: USMLE STEP 2 CK

037
643.25 years old female with 2 cm mass in upper right breast movable and firm –negative family history, for 2 months:
A. Fibroadenoma.
Answer: A
Source: Toronto Notes 2014

644.Child take overdose of isoniazid and toxicity symptoms?


Answer: seizure include status epilepticus
Http://www.uptodate.com/contents/isoniazid-inh-poisoning?Source=outline_link&view=text&anchor=H25#H25

645.25 years old lady presented with severe abdominal pain and regular menstruation, in US mass in the ovary with hair,
what is the next step?(dermoid cyst)
A. Laparoscopy
Answer: A

646.Breastfeeding mother on phenytoin ;


A. Continue breast feeding

Answer: A
Http://www.aafp.org/afp/2001/0701/p119.html

647.12-week pregnant w high blood pressure?


Answer:

Http://earlypregnancy.net/tag/disorders-with-pregnancy

038
648.25 year-old c/o lower abdomen. Cramp, 6 weeks from the last normal period. She has vaginal bleeding but no passage of
tissue. Diagnosis?
A. Ectopic pregnancy

Answer: ???
• commlene aborneon: bleedeng + commlene massage of nessue
• encommlene aborneon: exnremely heavy bleeding, cramps, passage of tissue noticed
• messed: no bleeding
• nhreanened: vagenal bleedeng +- cramps
• enevenable: Increaseng bleedeng and cramms ± rumnure of membranes
US must be done to decide the type.

(Master the boards step2 CK OBS Gyen chapter)

649.Treatment of dysmonrhea
It depends on the cause and type.

650.RH -ve mother and +ve baby what is the complecation at dlivery
Answer: Hemolytic disease of the newborn (HDN).

651.Pregnant woman with vaginal bleeding - you have to the types of abortion
Http://www.medbullets.com/step2-3-obstetrics/20373/spontaneous-abortion read it. It is an explanation of abortion.

039
652.2Qs stages of labor

653.Early Pregnat varicella vaccine:


Avoid exposure
Neither inactivated nor live-virus vaccines administered to a lactating woman affect the safety of breastfeeding for women or
their infants. Http://www.cdc.gov/vaccines/pubs/preg-guide.html#bfeeding

654.Clear Hx about OCD and ask about dx

Toronto:

655.Female present with oligomenorrhea. She had 3 periods in the last year" she had acne, hirsutism, Body wt 60 , pv
normal,dx?
A. PCOS.

Answer: A

First Aid:
Diagnosis of PCOS requires fulfillment of two of the following three (Rotterdam Criteria):
- Polycystic Ovaries (via US).
- Oligo- and\or anovulation.
- Clinical and\or biochemical evidence of hyperandrogenism.

041
656.Scenario about PCOS - they asked what invx u will screen for?
A. Glucose Tolerance & Lipid Profile.

Answer: A
First Aid:
Women with PCOS are at increased risk for the following:
DM2: Acanthosis nigricans.
Metabolic Syndrome: Insulin Resistance, atherogenic dyslipidemia, and HTN.

657.Patient with history of amenorrhea for 6 weeks presented with abdominal pain. On Examination, there's fluid in Douglas
pouch, what’s the Diagnosis?
A. Ectopic Pregnancy.

Answer: A
Kaplan:
The classic triad of unruptured ectopic pregnancy:
o Amenorrhea.
o Vaginal bleeding.
o Unilateral pelvic\abdominal pain.
Ruptured ectopic pregnancy is associated with intraperitoneal bleeding and irritation.

658.Breech presentation at 34 weeks,,?! ECV at 36 wks


First Aid:
• Endometrial biopsy: consider biopsy in women >40 yr.
• Must do endometrial biopsy in all women presenting with postmenopausal bleeding to exclude endometrial cancer.

659.Female present with defemenization "breast atrophy & deeping of voice" they found to have ovarian cancer, diagnosis?
- Sertoli Leydig Cell

Answer: A
Toronto:

660.Mother worry about radiation from microwave if exposed to her child, what to tell her?
A. Not all radiation is dangerous & microwave is one of them.
Answer: A

040
661.Young female with recurrent UTI what you should do to reduce the recurrence:
A. Educate patient not to wipe from back to front.

662.Grey Virginal discharge what can u see in microscpy ( there is no clue cell or hyphee in the answer
A. Intrepithelium
Note: Clue cells: vaginal squamous epithelial cells

663.Pregnant with N.gonorrhea what is the Rx:


Toronto:
Single dose of ceftriaxone 250 mg IM plus single dose of azithromycin 1g PO or doxycycline 100 mg PO bid for 7d
If pregnant: above regimen or alternate cephalosporin, or single dose of azithromycin 2 g PO

664.6 months post-partum irritable intolerance to heat :


A. Post-partum thyroiditis

665.36 yr old female use condom as contraceptive. She complains of nausea & amenorrhea. What is first investigation to do?
A. Beta HCG

666.If screening for breast cancer is limited only to postmenoupse women. Which young women carrying the following gene
will be missed from the screening:
Answer: BRCA 1

667.Female can't conceive, had symptoms and signs of PID , what to do :


Hysterosalpingogram

Explantion: Pelvic inflammatory disease affects the tube, and the best investigation for tubal factors is hysterosalpingogram (it
can be therapeutic)
Reference: Toronto Notes 2015, page GY21

668.A pregnant lady in her first trimester did not have any vaccination for rubella what to do?
A. Don’n give Rubella vaccine during pregnancy.

Answer: a
Vaccines Should Pregnant Women
Avoid (live attenuated vaccine) Vaccines Can Be Given During the Pregnancy

Hepatitis A Pneumococcal Hepatitis B

MMR OPV\IPV Influenza (Inactivated)

Varicella HPV Vaccine Tdap

Http://www.webmd.com/baby/pregnancy-is-it-safe-to-get-vaccinations

669.Which OCP cause Hyperkalemia?


042
Answer: Drospirenone
Ethinyl estradiol: Reduces LHRH release from hypothalamus, reduces gonadotropin release from pituitary; increases synthesis of
DNA, RNA, and various proteins in target tissues; other possible mechanisms include changes in cervical mucus that cause
inhibition of sperm penetration and endometrial changes that reduce likelihood of implantation
Http://reference.medscape.com/drug/yasmin-yaz-drospirenone-ethinyl-estradiol-342768#4

670.Female present e oligomeorrhea " she had 3 periods in the last year" she had acne $ histirusim Body wt 60 , pv
normal,dx?!
PCOS

Answer: The diagnostic criteria for PCOS should include two of the following three criteria: chronic anovulation,
hyperandrogenism (clinical/biologic), and polycystic ovaries
Refrence: http://emedicine.medscape.com/article/256806-overview

671.Scenario about PCOS they asked what investigation u will screen for?!
Glucose tolerance+ lipid profile
Answer: Women with PCOS are at increased risk for the following:
Type 2 DM
Inslulin resistance
Infertility
Metabolic syndrome- insulin resistance, obesity, atherogenic dyslipidemia, and HTN
Evaluate for metabolic abnormalities :
2hr glucose tolerance test
Fasting lipid and lipoprotein levels (total cholesterol, HDL, LDL,triglycerides)
th
Reference: USMLE Step2CK 8 edition pg330

672.In which situation, the hepatitis c positive mother should not breastfed her infant Lack of hepatitis c vaccine
Cracked nipples

Answer:
Data are insufficient to say yes or no. However, HCV is spread by infected blood. Therefore, if the HCV-positive mother's nipples
and/or surrounding areola are cracked and bleeding, she should stop nursing temporarily. Instead, she should consider
expressing and discarding her breast milk until her nipples are healed. Once her breasts are no longer cracked or bleeding, the
HCV-positive mother may fully resume breastfeeding. Https://www.cdc.gov/breastfeeding/disease/hepatitis.htm

673.40 years c/o heavy bleeding & intercyclic bleeding , not pregnant not using ocp. & not sexual active from a year, dx?!
a. Anovulatory cycle
[1]
Answer: Abnormal uterine bleeding (formerly, dysfunctional uterine bleeding [DUB] ) is irregular uterine bleeding that occurs
in the absence of recognizable pelvic pathology, general medical disease, or pregnancy. It reflects a disruption in the normal
cyclic pattern of ovulatory hormonal stimulation to the endometrial lining.
Reference: http://emedicine.medscape.com/article/257007-overview

Dysfinctional Uterine Bleeding: If the pregnancy is negative, there are no anatomic causes for bleeding and coagulopathy is ruled
out, then the diagnosis of hormonal imbalance should be considered. Mechanism: the most common cause of DUB is
043
anovulation. Anovulation results due to unopposed estrogen. With unopposed estrogen, there is continous stimulation of the
endometrium with no secretory phases.
Reference: Kaplan lecture notes, Gynecology pg231

674.Female present e defemenization " breast atrophy & deeping of voice" they found to have ovarian cancer , dx?!
Sertoli leyding cell

Refrence: https://www.nlm.nih.gov/medlineplus/ency/article/001172.htm

675.Increase amniotic fluid:


Duodenal atresia

676.Pregnant women with UTI what is the best drug:


Answer: nitrofurantoin

677.A patient who’s 36 weeks pregnant started having contractions lasting 30 secs. CTG was done and was good. What to do?
Give tocolytics
If the contractions are for labour then deliver because she is term

678.History of PID, presented with secondary infertility, what to do?


A- Colposcopy
Answer : Laparoscopy

679.You’re a gynecologist in clinic, a lady come to you with profuse vaginal discharge, diagnosis?
Ref: Master the Boards

680.Which of the following will increased the risk of GABH ?


High fever postpartum

Answer:?
681.What is the most common complication of HYPERTENSION in pregnancy?
A. IUGR
044
Answer:A
IUGR Macrosomia

Causes Maternal:(Asymmetric) DM
HTN Prolonged gestation
Small vessel disease (SLE) Male gender
Malnutrition Multiparty
Tabacco
Alcohol
Placental:(Asymmetric)
Infarction
Abruption
TTTS
Fetal:(symmetric)
Infection
Aneuploidy
Structural anomalies

Reference: Kaplan USMLE Step 2 OBGYN

682.During labor patient complain of severe pain in right thigh relief after labor what’s the nerve?
A. Cutaneous branch of femoral nerve

Answer:A
The lateral femoral cutaneous nerve exits the pelvis under the inguinal ligament and then passes medial and inferior to the
anterior superior iliac spine. It is a pure sensory nerve which supplies the anterolateral thigh. Injury to the lateral femoral
cutaneous nerve causes burning, pain, or numbness of the anterolateral thigh, known as meralgia paresthetica syndrome.4 The
lateral femoral cutaneous nerve is at risk of injury during prolonged pushing with hip flexion as the nerve is compressed under
the inguinal ligament.

Reference: http://www.mncyn.ca/wp-content/uploads/2013/08/volume31.pdf

683.Case of threaten abortion how will you manage?


A. Repeated US until viability of the fetus confirmed
Answer: A

684.2 scenario about abruptio placenta 2 scenario about early pregnancy bleeding ddx of early pregnancy bleeding:

• Physiologic bleeding: spotting, due to implantation of placenta – reassure and check serial -hcgs
• Abortion (threatened, inevitable, incomplete, complete)
• Abnormal pregnancy (ectopic, molar)

045
Trauma (post-coital or after pelvic exam)
• Genital lesion (e.g. Cervical polyp, neoplasms)
Reference: Toronto notes

046
685.Female with history of lichen sclerosis present with lesion what will you do?
A.Take biopsy
Answer: A

In non-neoplastic disorders of vulvar epithelium biopsy is necessary to make diagnosis and/or rule out malignancy. In lichen
sclerosis subepithelial fat becomes diminished; labia become thin, atrophic, with membrane-like epithelium and labial fusion
causing pruritus, dyspareunia, burning.
Reference: Toronto notes

686.Pregnant primi 34w didn't gain weight the doctor start to think of preeclampsia what finding support that:
A. Elevated BP
Answer:A

One of the risk factors of preeclampsia is nulliparity. The classic symptoms of pre-eclampsia include a frontal headache, visual
disturbance and epigastric pain. However, the majority of women with pre-eclampsia are asymptomatic. Hypertension is usually
the first sign.Rapidly progressive oedema of the face and hands may suggest pre-eclampsia.

687.Woman with abdominal pain they perform laparotomy then a dark blood when they open (I don’t remember if there’s
bilateral adnexal mass or not) What is the Dx:
Chocolate hemorrhagic cyst ???

688.Normal 10 year-old child, his parents worry about height of child ............. ? What the most thing to ask ?

689.Treatment of dysmenorrhea
Answer: Nonsteroidal anti-inflammatory drugs (nsaids)
Reference: http://emedicine.medscape.com/article/253812-medication

690.RH -ve mother and +ve baby what is the complication at delivery
Acute hemolysis
Reference: http://emedicine.medscape.com/article/797150-overview

047
691.Pregnant woman with vaginal bleeding - you have to know the types of abortion

692.Early Pregnant varicella vaccine:


A-Avoide exposure
Answer:a
Reference; http://www.cdc.gov/vaccines/pubs/preg-guide.html#varicella

693.Lady ,12 month trying to conceive, regular menstruation , her husband > normal semen analysis & temperature is normal
What is the cause?

694.Pregnant in last trimester or in labour couldn't remember anyhow she was presented with vaginal bleeding , baby was
delivered with no complication but mother developed bleeding from nose and mouth ?
DIC << I choose this
Incomplete question and choices
Explanation: DIC is the most likely answer, especially if the bleeding was in the third trimester and was due to abruptio placenta.
The most common cause of DIC in pregnancy is abruptio placenta. In DIC, typically there is a history of blood loss through
bleeding in areas such the gingivae and the gastrointestinal (GI) system
Reference: Toronto Notes 2015, page OB26
Http://emedicine.medscape.com/article/199627-clinical

695.40 years old women , her period regular , no intercourse for 1 year , well till 3 months when she had heavy period and
intermenstrual bleeding : Sorry I forgot the choices :/ but I think they were asking about what to do next

048
Answer: Choices not available
Explanation: under the topic of menorrhagia in Medscape, Exclusion of pregnancy (the most common cause of irregular
bleeding in women of reproductive age and the first diagnosis that should be excluded before further testing or drug therapy).
Reference: http://emedicine.medscape.com/article/255540-overview

696.Female on tamoxifene for her breast cancer , and progesterone for 5 years , she stop the progesterone , she was having
fibroid that is 2*3 befor 5 years and now become 5*6 with increased homogenous thickness of the endomertium Sorry
for the unorganized senior but it was like that and again I forgot the choices :/ but most probably they were asking about
the diagnosis .

Answer: Choices not Available


Explanation: Tamoxifen has pro-estrogenic effects on the endometrium and is associated with a number of pathologies. It is
associated with an increased prevalence of endometrial hyperplasia (1-20%), endometrial polyps (8-36%), endometrial
carcinoma, cystic endometrial atrophy
Reference: http://radiopaedia.org/articles/tamoxifen-associated-endometrial-changes

697.Fast division of blastomere.


Answer:
As cilia degenerate , the amount of time it takes for the fertilized egg to reach the uterus will increase. If the fertilized egg doesn
t reach the uterus in time, it will hatch from the non-adhesive zona pellucida and implant itself inside the fallopian tube, thus
causing the ectopic pregnancy.

698.How to confirm Down syndrome?


Answer: chorionic villous sampling
Chorionic villus sampling (CVS) is performed at 10-13 weeks’ gestation; earlier testing is thought to be associated with a 1 in
300-1000 risk of fetal transverse limb deficiency, a small risk of maternal cell contamination, and a 0.5-1% risk of a fetal loss
after the procedure. The accuracy of CVS (96-98%) is less than that of midtrimester amniocentesis, because of confined
placental mosaicism and maternal-cell contamination.
Http://emedicine.medscape.com/article/943216-workup#c9

699.Pregnant at 36 weeks gestational age presented with painless vaginal bleeding. Examination revealed closed cervical os
without contractions. What would you see on US?
A. Low lying placenta
Answer: (Case of Placenta Previa)
Placenta Previa (PP) is defined as the placenta overlying the cervical OS. PP triad:
1. Late trimester bleeding
2. Lower segment placental implantation (seen in US)
3. NO Pain

700.40 year-old with irregular menses for 3 months presented with bleeding between menses. What is the diagnosis?
Answer: Anovulatory bleeding (chronic)?!!
- Anovulatory Bleeding Triad:
1- Irregular, Unpredictable vaginal bleeding
2- 30s – early 40s -year-old woman
3- Obese, hypertension

049
701.Melanocyte stimulating hormone released from?
A. Intermediate lobe of pituitary
Answer: http://www.ncbi.nlm.nih.gov/pubmed/1717055

702.Asymptomatic female has 7 cm cyst in her ovary. Her period is normal


A. Follicular cyst

703.Lateral vaginal examination what you feel?


A. Ovary

704.24 year-old lady presented with bright red bleeding, breast tenderness, and bad mood. Β-HCG is negative. No
abnormality on examination. What to do?
Answer: no choices
The initial approach to evaluation of non-pregnant reproductive-age women with abnormal uterine bleeding (AUB)
is to confirm that the source of bleeding is the uterus, exclude pregnancy, and confirm that the patient is
premenopausal.
Most reproductive-age women with AUB should be evaluated initially with: human chorionic gonadotropin (hcg),
CBC, hemoglobin and/or hematocrit, HORMONAL TESTING (i.e., thyroid, prolactin. FSH/LH, estrogen), bleeding
disorders testing.
Pelvic imaging is useful if a structural lesion (e.g., endometrial polyps) is suspected; it is not required in every
woman with AUB. Pelvic ultrasound is the first-line study.
More details on the topic in the following link:
Http://www.uptodate.com/contents/approach-to-abnormal-uterine-bleeding-in-nonpregnant-reproductive-age-
women

705.Most common cause of post-partum hemorrhage?


Answer: Uterine atony
PPH has many potential causes, but the most common, by a wide margin, is uterine atony, ie, failure of the uterus to
contract and retract following delivery of the baby.
Medscape:
Http://emedicine.medscape.com/article/275038-overview#a7

706.Pregnant have pyelonephritis:


Answer: nitrofurantoin
Pyelonephritis is the most common urinary tract complication in pregnant women, occurring in approximately 2% of all
pregnancies. Acute pyelonephritis is characterized by fever, flank pain, and tenderness in addition to significant bacteriuria.
Http://emedicine.medscape.com/article/452604-overview

707.Vaginal discharge with falgellated cell on microscopy?


Answer: trichomonas vaginalis
Humans are the only known host of T vaginalis. Transmission occurs predominantly via sexual intercourse. The organism is most
commonly isolated from vaginal secretions in women and urethral secretions in men. Women with trichomoniasis may be
asymptomatic or may experience various symptoms, including a frothy yellow-green vaginal discharge and vulvar irritation.
Http://emedicine.medscape.com/article/230617-overview

051
708.Patient on paroxetine and is stable, she got pregnant what you will do ?
Paroxetine is a selective serotonin reuptake inhibitor (SSRI), used for the treatment of depression and anxiety disorders.
Exposures to paroxetine in early pregnancy, indicated an increased risk (also 2%) of cardiovascular defects of relatively mild
types after maternal use of paroxetine.
Http://www.acog.org/About-ACOG/News-Room/News-Releases/2009/Depression-During-Pregnancy

709.Lady with cyclic abdominal pain, heavy bleeding, not on contraceptive, trying to conceive, what is the investigation:
A. Laparoscopy

710.Women treated in the past for pelvic Inflammatory disease. Now her US showed bilateral ovarian cyst.. During surgery
dark blood come from ovaries.. Dx?
Answer: Chocolate cysts (Ovarian endometriosis)
Chocolate cysts are affecting women during their reproductive period and may cause chronic pelvic pain associated with
menstrual periods (menstrual cramps, endometriosis). The chocolate cyst is the cyst of the ovary with intracavitary hemorrhage
and formation of a hematoma containing old brown blood.
Http://lakecharlesobgyn.com/Complete/246-Endometriosis-Chocolate-Cysts.aspx

711.Ovarian cancer marker?


A. CA125
Gynecologic Tumor Markers Tumor Marker Overview http://emedicine.medscape.com/article/269839-overview

712.Pregnant lady had placenta priva to know type by>> Transabdominal US OR Transvaginal US or MRI. UPTODATE
https://yhdp.vn/uptodate/contents/mobipreview.htm?34/13/35032

713.Mass out of vagina with coughing and defecation?


A. Uterine prolapse BMJ http://bestpractice.bmj.com/best-practice/monograph/659/diagnosis.html
Due to increase in intra-abdominal pressure (chronic obstructive airway disease, straining, constipation, heavy lifting, and hard
physical activity) can lead to pelvic organ prolapse (POP).

714.Anti-diabetic drug taken by ladies with PCOS:


A. Metformin BMJ http://bestpractice.bmj.com/best-practice/monograph/141/treatment.html

715.Elevated in menopause lady?


BMJ http://bestpractice.bmj.com/best-practice/monograph/194/diagnosis/tests.html

050
716.Vulvar cancer cause and treatment?
Human papillomavirus (HPV) may be a cause of some vulvar malignancies. Medscape.
Http://emedicine.medscape.com/article/264898-overview
Treatment options for vulvar cancer depend on the type and cancer stage. Mayoclinic
Surgery :(excision) (partial vulvectomy)(radical vulvectomy). Extensive surgery for advanced cancer: called pelvic
exenteration.
Reconstructive surgery. Surgery to remove nearby lymph nodes
Radiation therapy
Chemotherapy
Http://www.mayoclinic.org/diseases-conditions/vulvar-cancer/basics/treatment/con-20043483
Http://www.mayoclinic.org/diseases-conditions/vulvar-cancer/basics/treatment/con-20043483

717.Urge incontinence how to diagnose?


Diagnosis of OAB is made primarily on the person's signs and symptoms and by ruling out other possible causes such as an
infection.
Bmj http://bestpractice.bmj.com/best-practice/monograph/169/diagnosis/history-and-examination.html

052
718.Urge incontinence Concept of treatment?

BMJ
http://bestpractice.bmj.com/best-practice/monograph/169/treatment/details.html
053
719.Female with Tubo-ovarian abscess what is the treatment?
Treatment typically involves drainage of the abscess. The antibiotic regimen should include broad coverage against gram-
negative rods, 
enterococci, and anaerobes(clindamycin or metronidazole should be used with doxycycline as this provides
better anaerobic coverage than doxycycline alone*BMJ*). 
 Step UP to medicine. Http://bestpractice.bmj.com/best-
practice/monograph/195/treatment/details.html#expsec-519813

720.Rupture of membrane during pregnancy when to give antibiotics?


The American College of Obstetricians and Gynecologists (ACOG) recommends a seven-day course of intravenous ampicillin and
[2]
erythromycin followed by oral amoxicillin and erythromycin if watchful waiting is attempted before 34 weeks. Amoxicillin-
clavulanic acid increases the risk of fetal bowel death (necrotizing enterocolitis) and should be avoided in pregnancy.

BMJ
Http://bestpractice.bmj.com/best-practice/monograph/1002.html

721.DM pregnant her oral glucose tolerance test came positive what to do next.
Diagnose with GDM: nternational Association of Diabetes and Pregnancy Study Groups (IADPSG) criteria (endorsed by the
American Diabetes Association [ADA]) recommended that all women not known to have diabetes should undergo a 75-gram
OGTT at 24 to 28 weeks of gestation, with diagnosis of GDM based upon the finding of 1 abnormality.
Management 1st : diet, exercise + glucose monitoring if uncontrolled with dietary therapy, or marked initial hyperglycaemia
plus insulin therapy, then at 32 to 34 weeks’ gestation start antepartum fetal monitoring. BMJ
http://bestpractice.bmj.com/best-practice/monograph/665/diagnosis/tests.html

054
722.Fibroid during pregnancy does it Degenerate or stays asymptomatic Or what ?
Degenerative changes : Usually presents during pregnancy with acute abdominal pain with significant local tenderness over the
site of the mass.
Most common type of degeneration during pregnancy is red type, believed to occur due to rapid fibroid cellular growth that
exceeds the blood supply.
Usually requires admission to the hospital for pain therapy, usually with non-steroidal anti-inflammatory drugs (nsaids).
However, these drugs should be used with caution to avoid fetal problems, such as premature closure of the ductus arteriosus.
OR pregnancy loss: There exist a number of hypotheses regarding the mechanism of action of spontaneous abortion in patient
with uterine fibroids including altered uterine contractility, altered uterine vasculature, and/or supporting extracellular matrix
.BMJ http://bestpractice.bmj.com/best-practice/monograph/567/follow-up/complications.html

723.Typical of polycystic ovarian syndrome diagnosis .


BMJ http://bestpractice.bmj.com/best-practice/monograph/141/treatment.html

724.Endometriosis .. How diagnosis ?


st
1 transvaginal ultrasound (TVUS) BMJ http://bestpractice.bmj.com/best-practice/monograph/355/diagnosis/tests.html

th
725.Pregnant lady in her 27 gestational week has mono chorionic twin. One of them died, what will be your next step?
Answer: expectant delivery (do nothing)
In all circumstances, the use of steroids and magnesium sulfate should be considered for fetal lung maturity and
neuroprotection respectively. If the live twin is leading, well grown and is in a cephalic presentation, then vaginal delivery may
be considered. If the live twin is malpresenting or is growth restricted, or if the dead twin is leading, caesarean section is
preferred
Https://www.ranzcog.edu.au/editions/doc_view/848-54-fetal-death-of-a-twin.html

726.Post-menopausal with dysuria, frequency and supra pubic tenderness?


Interstitial cystitis
Reference: http://emedicine.medscape.com/article/2055505-clinical

727.Female has itching, white cheesy discharge?


Candida
055
728.Which oral contraceptive causes hyperkalemia?
A) Estradiol Drospirenone (from google and was in the choices)
Answer:
• Yasmin®: 30 μg ethinyl estradiol + 3 mg drospirenone (a new progestin)
• Yaz®: 20 μg ethinyl estradiol+ 3 mg drospirenone – 24/4-d pill (4 d pill free interval)
• Drospirenone has antimineralocorticoid activity and antiandrogenic effects
Link: http://www.medscape.com/viewarticle/759450_2

729.Etiology of Postpartum Pyrexia:


B-5W
Breast: engorgement, mastitis
Wind: atelectasis, pneumonia
Water: UTI
Wound: episiotomy, C/S site infection
Walking: DVT, thrombophlebitis
Womb: endometritis
Pharyngitis, Gastroenteritis

730.Gravida 1 para 1, came with history of severe abdominal pain and vaginal bleeding with passing some fetal tissues, by
examination the cervix was open with some tissues protruding
What is the conventional management?
A) Dilatation and curettage
Answer: A
In missed, incomplete, or inevitable abortion present before 13 weeks' gestation, the standard therapy has been suction D&C.
Link: http://reference.medscape.com/article/266317-treatment

731.Girl with depression in first 2 days of menstruation what dx? Mood swings, depression?!
Answer: PMS

732.Patient has hx of PID came with adnexal mass what next step?
Answer: Pelvic ultrasound is the first line imaging study for the evaluation of an adnexal mass

733.Transmission of HIV : Through breast feeding

MODES OF TRANSMISSION DRIVING THE EPIDEMIC — The major modes of acquiring HIV infection are [:
Sexual transmission, including via heterosexual and homosexual contact
Parenteral transmission, predominantly among injection drug users (IDU)
Perinatal transmission
Mother-to-child transmission — With high levels of HIV infection among young women, the potential exists for large numbers
of infected children, since infants can become infected in utero, at birth, or during breastfeeding. Such mother-to-child
transmission accounts for 90 percent of infection among children worldwide. In the most affected countries in the world, such
as in sub-Saharan Africa, 20 to 40 percent of pregnant women are HIV-infected, and one-third of their babies become infected.
Although antiretroviral use during pregnancy, at the time of delivery, and during breastfeeding can largely prevent this, only a
minority (25 percent or less) of affected mothers are able to access such antiretroviral prophylaxis

056
734.Pathogenesis of jaundice in newborn From a mother has - blood group and the newborn has +o.
Individuals are classified as Rh negative or positive based upon the expression of the major D antigen on the erythrocyte. The
original description of HDFN was due to Rh(D) incompatibility, which is associated with the most severe form of the disease
(hydrops fetalis).
PATHOGENESIS AND CONSEQUENCES OF ALLOIMMUNIZATION — By 30 days of gestation, the Rh(D) antigen is expressed as
part of the red blood cell (RBC) membrane, and, in contrast to most other antigens (eg, A,B,M,N), Rh(D) is only present on rbcs.
Maternal Rh(D) alloimmunization develops as a result of maternal immune system exposure to Rh(D)-positive rbcs. Once anti-D
igg antibodies are present in the pregnant woman's circulation, they can cross the placenta and opsonize fetal rbcs, which are
then phagocytized by macrophages in the fetal spleen.
Events that can cause maternal alloimmunization include:
Transplacental fetomaternal hemorrhage during any pregnancy
Injection with needles contaminated by Rh(D)-positive blood
Inadvertent transfusion of Rh(D)-positive blood
D-mismatched allogeneic hematopoietic stem cell transplantation
Transplacental fetomaternal bleeding accounts for virtually all cases of maternal Rh(D) alloimmunization. Tiny (0.1 ml) quantities
of fetal rbcs gain access to the maternal circulation in nearly all pregnancies, as demonstrated by studies using flow cytometry.
The frequency and volume of spontaneous fetomaternal hemorrhage increase with advancing gestational age and are highest at
delivery. Fetomaternal hemorrhage can also be associated with miscarriage, pregnancy termination, ectopic pregnancy, invasive
in-utero procedures, fetal death, maternal abdominal trauma, antepartum maternal hemorrhage, and external cephalic version.

735.18 year-old married missed her period for two months, came with rt sided abd pain what is the dx:
Answer: ruptured ectopic

736.Similar question but what’s the test to order:


Answer: Urine hcg
The most common clinical presentation of ectopic pregnancy is first trimester vaginal bleeding and/or abdominal pain. Ectopic
pregnancy may also be asymptomatic.
Normal pregnancy discomforts (eg, breast tenderness, frequent urination, nausea) are sometimes present in addition to the
symptoms specifically associated with ectopic pregnancy. There may be a lower likelihood of early pregnancy symptoms,
because progesterone, estradiol, and human chorionic gonadotropin (hcg) may be lower in ectopic pregnancy than in normal
pregnancy.
He pain associated with ectopic pregnancy is usually located in the pelvic area. It may be diffuse or localized to one side. In cases
in which there is intraperitoneal blood that reaches the upper abdomen or in rare cases of abdominal pregnancy, the pain may
be in the middle or upper abdomen. If there is sufficient intraabdominal bleeding to reach the diaphragm, there may be referred
pain that is felt in the shoulder. Blood pooling in the posterior cul-de-sac (pouch of Douglas) may cause an urge to defecate.
The timing, character, and severity of abdominal pain vary, and there is no pain pattern that is pathognomonic for ectopic
pregnancy. The onset of the pain may be abrupt or slow, and the pain may be continuous or intermittent. The pain may be dull
or sharp; it is generally not crampy. The pain may be mild or severe. Tubal rupture may be associated with an abrupt onset of
severe pain, but rupture may also present with mild or intermittent pain.
DIAGNOSTIC EVALUATION
Overview — The main goals and steps of the evaluation of a woman with a suspected ectopic pregnancy are:
1. Confirm that the patient is pregnant
a. Measurement of hcg is performed initially to diagnose pregnancy and then followed to assess for ectopic
pregnancy. For follow-up, hcg is measured serially (every 48 to 72 hours) to determine whether the increase is
consistent with an abnormal pregnancy. A single hcg measurement alone cannot confirm the diagnosis of
ectopic or normal pregnancy.
b. The initial test to diagnose pregnancy may be either a urine or serum hcg. Once a pregnancy is confirmed, if
ectopic pregnancy is suspected, the serum hcg is then repeated serially (typically every two days) to assess
whether the increase in concentration is consistent with an abnormal pregnancy
2. Determine whether the pregnancy is intrauterine or ectopic. Determine the site of the ectopic pregnancy.

057
3. Determine whether the structure in which the pregnancy is implanted (most commonly, the fallopian tube) has
ruptured and whether the patient is hemodynamically stable. Failure to diagnose ectopic pregnancy before tubal
rupture limits the treatment options and increases maternal morbidity and mortality.
4. Perform additional testing to guide further management (eg, blood type and antibody screen, pre-treatment testing for
methotrexate therapy).

737.Pregnant lady 39 weeks presented with high blood pressure for the first time. No proteinurea or seizures, wts her dx:
th
Answer: Gestational hypertension if it was not stated that it was present before pregnancy (before the 20 week) and/or
persists after pregnancy for more than 12 weeks postpartum
INTRODUCTION — There are four major hypertensive disorders that occur in pregnant women:
Preeclampsia-eclampsia – Preeclampsia refers to the syndrome of new onset of hypertension and either proteinuria or end-
organ dysfunction most often after 20 weeks of gestation in a previously normotensive woman.. Eclampsia is diagnosed when
seizures have occurred.

Chronic (preexisting) hypertension – Chronic hypertension is defined as systolic pressure ≥041 mmhg and/or diastolic pressure
≥91 mmhg that antedates pregnancy, is present before the 20th week of pregnancy, or persists longer than 12 weeks
postpartum.

Preeclampsia-eclampsia superimposed upon chronic hypertension – Preeclampsia-eclampsia superimposed upon chronic


hypertension is diagnosed when a woman with chronic hypertension develops worsening hypertension with new onset
proteinuria or other features of preeclampsia (eg, elevated liver enzymes, low platelet count).

Gestational hypertension – Gestational hypertension refers to elevated blood pressure first detected after 20 weeks of
gestation in the absence of proteinuria or other diagnostic features of preeclampsia. Over time, some patients with gestational
hypertension will develop proteinuria or end-organ dysfunction characteristic of preeclampsia and be considered preeclamptic,
while others will be diagnosed with preexisting hypertension because of persistent blood pressure elevation postpartum.

738.Patient with odorless vaginal disch, grey-white. Spores on wet mount.


Answer:Candida

The diagnosis of vulvovaginal candidiasis is based on the presence of Candida on wet mount, Gram’s stain, or culture of vaginal
discharge in a woman with characteristic clinical findings (eg, vulvovaginal pruritus, burning, erythema, edema, and/or curd like
discharge attached to the vaginal sidewall) and no other pathogens to account for her symptoms.
The vaginal ph in women with Candida infection is typically normal (4 to 4.5), which distinguishes candidiasis from
trichomoniasis or bacterial vaginosis.
Candida species can be seen on a wet mount of the discharge; adding 10 percent potassium hydroxide destroys the
cellular elements and facilitates recognition of budding yeast, pseudohyphae, and hyphae

739.Patient with foul vaginal disch, greenish color. Microscopy flagellate organisms. Whan’s the treatment: (trach vaginitis)
Answer: Oral metronidazole

740.Patient with dyspareunia and sever dysmenorrhea on examination post fornix nodule. What is the management?

This appears to be a presentation of endometriosis specifically post-vaginal endometriosis.


Treatment:
Medical treatment options include nonsteroidal analgesics, hormonal contraceptives, gonadotropin-releasing hormone (gnrh)
agonists, and aromatase inhibitors. As there are no data supporting one treatment or treatment combination over another, the

058
treatment choice is based upon symptom severity, patient preferences, medication side effects, treatment efficacy,
contraceptive needs, costs, and availability. Medical interventions do not improve fertility, diminish endometriomas, or treat
complications of deep endometriosis such as ureteral obstruction.
1. Nsaids: nsaids are be considered the first-line treatment for pelvic pain, including endometriosis-related pain. However,
there are no high-quality data reporting NSAID efficacy in treating pain due to endometriosis, nor have nsaids been
shown to be superior to other agents or to placebo
2. Combined (estrogen and progestin) contraceptives are the first-line treatment for most women with endometriosis-
related pain because they can be used long-term, are well-tolerated, are relatively inexpensive and easy to use, and
provide contraception and additional benefits including decreasing the risk of ovarian and endometrial cancers
3. Progestin-only therapy is another treatment option. The progestins most commonly used for the treatment of
endometriosis-related pain include medroxyprogesterone acetate (MPA) or the 19-nortestosterone derivatives
norethindrone acetate and dienogest
4. Gonadotropin-releasing hormone (gnrh) agonists — gnrh agonists include nafarelin, leuprolide, buserelin, goserelin,
and triptorelin. Gnrh agonists bind to receptors in the pituitary gland. Because gnrh agonists have a longer half-life than
native gnrh, the pituitary-ovarian axis is down-regulated and hypoestrogenism results. Endometriosis-related pain is
likely treated by the induction of amenorrhea and progressive endometrial atrophy. The hypoestrogenic state is the
main source of adverse effects, including hot flushes, vaginal dryness, decreased libido, mood swings, headache, and
decreased bone density. Negative effects can be reduced by add-back therapy, typically with oral norethindrone
acetate or a combination of estrogen and progestin (ie, COC pill)
5. Danzol: Danazol is a derivative of 17 alpha-ethinyltestosterone. It primarily inhibits the luteinizing hormone surge and
steroidogenesis, and increases free testosterone levels. Its mechanisms of action include inhibition of pituitary
gonadotropin secretion, direct inhibition of ovarian enzymes responsible for estrogen production, and inhibition of
endometriotic implant growth. While danazol is effective at treating endometriosis-related pain, it is not commonly
used because of androgenic side effects.
6. Aromatase inhibitors — We reserve aromatase inhibitor (AI) treatment for women with severe, refractory
endometriosis-related pain [49,50]. We inform women that treatment of endometriosis is an off-label use of these
medications. Typical treatments include oral anastrozole 1 mg once daily or oral letrozole 2.5 mg once daily. These
agents appear to regulate local estrogen formation within the endometriotic lesions themselves, in addition to
inhibiting estrogen production in the ovary, brain, and periphery (eg, adipose tissue). Disadvantages of ais include bone
loss with prolonged use and ovarian follicular cyst development.

741.Best medication for gestational diabetes mellitus is?


Answer: Insulin
Http://emedicine.medscape.com/article/127547-overview

742.Pregnant female present with bleeding and abdominal contractions started at night has history of mild hypertension.
Diagnosis?
Answer: Placental abruption
http://emedicine.medscape.com/article/252810-overview

743.Placenta abruption:
A) vaginal bleeding
Answer: a?

744.Idiopathic anovulation, drug to enhance ovulation?


Answer:
Clomiphene is a weak estrogen-like hormone that acts on the hypothalamus, pituitary gland, and ovary to increase levels of
follicle-stimulating hormone (FSH) and luteinizing hormone (LH, which is also important in the process of ovulation).
Ref : uptodate

059
745.Patient with foul vaginal discharge, greenish color. Microscopy flagellate organisms. What's the treatment: (trach
vaginitis)
A-Oral metronidazole
Answer: A 5-nitroimidazole drugs — The 5-nitroimidazole drugs ( metronidazole or tinidazole ) are the only class of drugs that
provide curative therapy of trichomoniasis.
Https://yhdp.vn/uptodate/contents/mobipreview.htm?1/53/1881

746.Pregnant lady 39 weeks presented with high blood pressure for the first time. No proteinuria or seizures, wts her dx:
A- Gestational hypertension
Answer: A
Https://yhdp.vn/uptodate/contents/mobipreview.htm?28/44/29383

747.Snowstorm appearance in pregnant what's the Dx?


Answer: seen in complete hydatidiform mole

748.Postmenopausal lady taking tamoxofin, which of the following u will carefully assess?
A- vaginal bleeding
Answer: A
Because it may increase the risk of uterine malignancy
Tamoxifen may increase the risk of the following, particularly in women over age 50 years:
Cancer of the uterus (endometrial cancer and sarcoma).
Blood clots within deep veins (deep vein thrombosis), usually in the legs, which can travel to the lungs (pulmonary embolism).
Reference:
Http://www.uptodate.com/contents/tamoxifen-drug-information?Source=outline_link&view=text&anchor=F224618#F224618

749.Endometriosis definition ?
The presence of tissue that normally grows inside the uterus (womb) in an abnormal anatomical location. Endometriosis is very
common and may not produce symptoms, or it may lead to painful menstruation. It has also been associated with infertility.
Endometriosis occurs most commonly within the Fallopian tubes and on the outside of the tubes and ovaries, the outer surface
of the uterus and intestines, and anywhere on the surface of the pelvic cavity. It can also be found, less often, on the surface of
the liver, in old surgery scars or, very rarely, in the lung or brain.
Endometriosis occurs in the reproductive years. The average age at diagnosis is 25-30.Endometriosis may be suspected by
during a physical examination; it is confirmed by surgery, usually laparoscopy; available treatments include medication for pain,
hormone therapy, and surgery
Http://www.medicinenet.com/script/main/art.asp?Articlekey=3240

750.Pregnant 10wks has bleeding and fetus delivered , os is opened and still some remnants ? What to do ?
With missed, incomplete, or inevitable abortion present before 13 weeks' gestation, the standard therapy has been suction D&C
(medscape).
Women with an incomplete, inevitable, or missed abortion can be managed surgically, with medication, or expectantly. All three
management approaches are effective, but treatment is completed more quickly with surgical management and involves fewer
medical visits. The choice of method is typically based upon patient preference

INCOMPLETE QUESTIONS:

061
751.Question about Mayer-Rokitansky-Küster-Hauser syndrome

This condition causes the vagina and uterus to be underdeveloped or absent. Affected women usually do not have
menstrual periods due to the absent uterus. Often, the first noticeable sign of MRKH syndrome is that menstruation
does not begin by age 16 (primary amenorrhea). Women with MRKH syndrome have a female chromosome pattern
(46,XX) and normally functioning ovaries.
You can read more about MRKH here:
Https://ghr.nlm.nih.gov/condition/mayer-rokitansky-kuster-hauser-syndrome

752.What is the treatment of gonorrhea?


Answer:
- This disease is caused by Neisseria Gonorrhoeae, a Gram- negative diplococcus
- Symptoms: lower genital tract infection → vulvovaginal discharge and itching.
Upper genital tract infection → bilateral abdominal and pelvic pain
- Disseminated gonorrhea is characterized by: dermatitis, polyarthralgia and tenosynovitis
- Diagnosis: Nucleic acid amplification test (NAAT) of either cervical discharge or urine
- Treatment: single dose of IM ceftriaxone + single dose of Azithromycin. In case of Bartholin abscess, it needs to
undergo incision and drainage with Word catheter
- KAPLAN Lecture Notes

753.HPV infectious mono

754.A lady in labor with 6 cm dilatation was given epidural anesthesia. The pain came back, so they gave her
(venylphantanile?) Baby started to be in distress. She was given fluids without improvement. What to give her next?
Answer: no choices
Ephedrine?? Is it antidote?

755.Characteristic for premenstrual syndrome. Which phase or behavioral or symptomatic abnormality? (Not sure whan’s the
question here)
Answer: no choices
Premenstrual syndrome (PMS) is a recurrent luteal-phase condition characterized by physical, psychological, and
behavioral changes of sufficient severity to result in deterioration of interpersonal relationships and normal activity.
More on PMS: http://emedicine.medscape.com/article/953696-overview

756.Case of vaginal watery brown discharge.


Answer: no choices
There are many causes for brown vaginal discharge but differentials depend on accompanying symptoms and clinical
presentation.
More on brown vaginal discharge: http://www.newhealthadvisor.com/Light-Brown-Discharge.html

060
757.Typical case of PCO. Diagnosis ?
Stein–Leventhal syndrome(other name)
Stein and Leventhal were the first to recognize an association between the presence of polycystic ovaries and signs
of hirsutism and amenorrhea (eg, oligomenorrhea, obesity).
PCO diagnostic criteria:
At least 2 of the following 3 features are required for PCOS to be diagnosed:
Oligo-ovulation or anovulation manifested as oligomenorrhea or amenorrhea
Hyperandrogenism (clinical evidence of androgen excess) or hyperandrogenemia (biochemical evidence of
androgen excess)
Polycystic ovaries (as defined on ultrasonography): 12 or more follicles in at least 1 ovary—measuring 2-9 mm
3
in diameter—or a total ovarian volume greater than 10 cm .
Http://emedicine.medscape.com/article/256806-overview#showall

758.What is most stimulus factor for milk secretion?


The baby's suckling at the breast does two things - it brings milk out with its suction, and, it sends an important message to
the mother's pituitary gland, via the nipples' nerve endings, the thoracic nerves, and the hypothalamus to send more milk
out!
Http://www.healthcentral.com/encyclopedia/hc/breastfeeding-3168758/

759.A drug that interferes with OCP? Anti-epileptic? Not sure


Http://www.mckinley.illinois.edu/handouts/pill_interactions_drugs.html

760.Case of incomplete abortion


One of the common complications of pregnancy is spontaneous miscarriage, which occurs in an estimated 5-15% of
pregnancies. Spontaneous miscarriages are categorized as threatened, inevitable, incomplete, complete, or missed. Signs of
incomplete miscarriage include the following: The cervix may appear dilated and effaced, or it may be closed, Bimanual
examination may reveal an enlarged and soft uterus. On pelvic examination, products of conception may be partially present in
the uterus, may protrude from the external os, or may be present in the vagina. Bleeding and cramping usually persist.
Http://emedicine.medscape.com/article/795085-clinical#b4

062
761.Pregnant G3P2 in labor, cervical dilatation 3cm ,, effacement 100% membrane rupture , after 3 hr still 3 cm ,, /(( c/s,,
oxytocin , waiting ) .

762.Embryonal alveolar from what? Sacule i think

763.A female has dyspareunia, dysmenorrhea in examination we found a cyst in the posterior fornix what your management?
A)danazol
Answer: A - But scenario is messing more options to determine accurate answer
This is probably endometriosis.
Signs & symptoms of endometriosis typically reflect the area of involvement, they include: Dysmenorrhea, Heavy or irregular
bleeding, Pelvic pain, Lower abdominal or back pain, Dyspareunia, Dyschezia (pain on defecation) - Often with cycles of diarrhea
and constipation, Inguinal pain, …
Laparoscopy is considered the primary diagnostic modality for endometriosis.
The most common sites in descending order: Ovaries, Posterior cul-de-sac, Broad ligament.
Treatment:
Medical: Combination oral contraceptive pills (cocps), Danazol..etc
● Danazol: a derivative of the synthetic steroid ethisterone that suppresses the production of gonadotropins and has
some weak androgenic effects. Danazol exhibits hypoestrogenic, hyperandrogenic effects that cause atrophy of the
endometrium, which can alleviate the symptoms of endometriosis.
Surgical: classified into:
● Conservative surgery: Drainage and laparoscopic cystectomy, Ablation,
● Semi Conservative surgery: hysterectomy and cytoreduction of pelvic endometriosis.
● Radical surgery: TAH-BSO

764.CH16 where u find?(pcos,...)

765.Ovarian tumor post hysterectomy what you will find in biopsy?


A. endomaterial hyperplasia
Reference: http://www.cancer.org/cancer/endometrialcancer/detailedguide/endometrial-uterine-cancer-risk-factors

766.Most common cause of Leukorrhea?


Answer: Estrogen imbalance
Leukorrhea is a thick, whitish or yellowish vaginal discharge.There are many causes of leukorrhea, the usual one being
estrogen imbalance. The amount of discharge may increase due to vaginal infection or stds, and also it may disappear
and reappear from time to time, this discharge can keep occurring for years in which case it becomes more yellow and
foul-smelling; it is usually a non-pathological symptom secondary to inflammatory conditions of vagina or cervix.
Reference: Wikipedia: https://en.wikipedia.org/wiki/Leukorrhea

767.Ovarian follicular cell origenate from ?? No answers for this one.


Answer: OVARIAN Follicular cells (granulosa cells) develop from the secondary SEX CORD (which develops in gonadal ridge).
Reference: Embryology websites, Basic embryology

768.A female patient with ovarian cancer and high CA125. What is the type of cancer?
Answer: Germinoma?
Epithelial tumors represent the most common histology (90%) of ovarian tumors. Five main histologic subtypes: Serous
(from fallopian tube), endometrioid (endometrium), Mucinous (cervix), Clear cell (mesonephros), Brenner
http://emedicine.medscape.com/article/255771-workup#c9
063
769.2 weeks infant came for routine checkup the doctor exam the baby and he looks well , but when the doctor ask
the mother about her baby she told somthing else she said the baby is not well he is confused and he has evil
power or somthing like that
What does the mother have
A-Post-partum psychosis
Answer: a
Postpartum psychosis has a dramatic onset, emerging as early as the first 48-72 hours after delivery. In most women,
symptoms develop within the first 2 postpartum weeks
The mother may have delusional beliefs that relate to the infant (eg, the baby is defective or dying, the infant is Satan
or God), or she may have auditory hallucinations that instruct her to harm herself or her infant.

Postpartum depression develops most frequently in the first 4 months following delivery but can occur anytime in the
first year and it interferes with the mother's ability to care for herself or her child

Postpartum blues: Symptoms peak on the fourth or fifth day after delivery and last for several days, but they are
generally time-limited and spontaneously remit within the first 2 postpartum weeks

Refrence : http://reference.medscape.com/article/271662-overview#a6

064
Œ_ Œ
SMLE
KSAU-HS
Question Bank
1st Edition
FM, PSYCH,EM,DERMA,Basic

This is an accumulative effort from King Saud bin Abdulaziz University for Health Sciences (2016-17/
Batch 9) interns to organize and answer what have been collected previously from SMLE Q Bank
2015-16

We would like to acknowledge:


- King Saud bin Abdulaziz University for Health Sciences (2016-17/Batch 9) interns for their huge efforts in
accomplishing this project
- SMLE Q Bank Group
‫ مدونة طالب طب سعودي‬-

‫جهد بشري قابل للخطأ والصواب‬


For any comments, kindly contact us at
SMLE2016.17@gmail.com

Highlighted in yellow are queried questions


Highlighted in red are some repeated questions

2
Table of Contents

Family Medicine, Research and Ethics .......................................................................................................... 4


Family medicine .....................................................................................................................................................5
Research .............................................................................................................................................................. 52
Ethics................................................................................................................................................................... 72
Psychiatry.................................................................................................................................................. 79
Emergency medicine.................................................................................................................................128
Dermatology ............................................................................................................................................163
Basic Science ............................................................................................................................................205
Anatomy............................................................................................................................................................ 206
Embryology ....................................................................................................................................................... 259
Histology ........................................................................................................................................................... 264
Physiology ......................................................................................................................................................... 267
Biochemistry...................................................................................................................................................... 270
Genetic .............................................................................................................................................................. 282
Microbiology ..................................................................................................................................................... 293
Immunology ...................................................................................................................................................... 306
Pathology .......................................................................................................................................................... 314
Pharmacology .................................................................................................................................................... 318

3
Family Medicine,
Research and Ethics

4
Family medicine

5
1. What is most common serious chronic infection found in expatriates coming to Saudi Arabia (Most common infection that is
found in expatriates before they start working)?
A - Hepatitis A
B - Hepatitis B
C - Hepatitis C
D - HIV
Answer: B
Evidence:
In Saudi Arabia, Hepatitis B infection was the most common cause (57.5%), followed by non-communicable diseases (21.2%) and
hepatitis C infection (17.4%). References: https://www.ncbi.nlm.nih.gov/pubmed/24975313 +
http://applications.emro.who.int/emhj/v19/07/EMHJ_2013_19_7_664_670.pdf?ua=1

2. The most difficult method to prevent transmission:


A - Person to person
B - Vector
C - Droplet
D - Airborne
Answer: D | Reference: 3rd Edition UQU last touch (Family medicine and statistics - Q 3)

3. At a day-care centre 10 out of 50 had red eye in the first week, another 30 developed the same condition in the next 2 weeks.
What is The attack rate (cumulative incidence)?
A - 40%
B - 60%
C - 80%
D - 20%
Answer: C
Evidence:
Attack Rate = cases due to a specific cause in a short period of time divided by the population at risk, often associated with an
epidemic situation such as food borne disease (cumulative incidence)
Reference: 3rd Edition UQU (Family medicine and statistics - Q 15)

4. The chairman of the public health wants to reduce the incidence of stroke. He is reading the literature but he is confused.
What is the best intervention?
A - Cholesterol level test for all population
B - Anti-Smoking campaign
C - Hypertension booth in the malls
D - Obesity booth in the mall
Answer: C

6
5. An adult came to you for varicella vaccine. How will you give it to him?
A - one dose now and one after 2 weeks.
B - one dose now and one after 3 months
C - one dose now and one after 6 weeks
D - just give one dose now
nd
Answer: C (For adult; two doses of varicella vaccine at least 4 weeks apart (the 2 dose can be given after 4 weeks or more. For
children; two doses of varicella vaccine at least 3 months apart)

6. Least harmful vaccine in immunocompromised patient:


A - BCG
B - pneumococcal vaccine
C - measles
D - Mumps
Answer: B
Evidence:
o Pneumococcal vaccine (both conjugate and polysaccharide) classified as a subunit vaccine, which is like inactivated vaccine can
be given to immunocompromised patients.
o BCG, measles and mumps are a live attenuated vaccine which should not be given to immunocompromised patient.
o MMR (measles, mumps and rubella) vaccine is the only exception of live attenuated vaccine type can be administered to
asymptomatic HIV/AIDS patient (not all immunocompromised patients)

7. A man who wants to travel. What is your advice? (other version: best prophylaxis against traveller’s diarrhoea; What would
you advise someone who is travelling to (?) to do most importantly, as precaution from Traveller’s diarrhoea):
A - Eat fruits and vegetables. (other version: fresh fruit and vegetable; Eat washed fruits and vegetables)
B - Eat vegetables that can be peeled. (other version: peeled fruit; Eat fruits that you can peel)
C - Drink iced water. (other version: drinks with ice)
D - (other version: daily antibiotic; Prophylactic antibiotics)
Answer: B | Reference: Oxford handbook of General practice, 3rd edition, Prevention of travel-related illness (P174)
Evidence:
At this time, prophylactic antibiotics should not be recommended for most travellers. Prophylactic antibiotics afford no protection
against nonbacterial pathogens and can remove normally protective microflora from the bowel, which could make a traveller more
susceptible to infection with resistant bacterial pathogens. A traveller relying on prophylactic antibiotics will need to carry an
alternative antibiotic to use in case diarrhoea develops despite prophylaxis. Additionally, the use of antibiotics may be associated
with allergic or adverse reactions in a certain percentage of travellers and may potentially contribute to drug resistance. The use of
prophylactic antibiotics should be weighed against the result of using prompt, early self-treatment with antibiotics when TD occurs,
which can limit the duration of illness to 6–24 hours in most cases. Prophylactic antibiotics may be considered for short-term
travellers who are high-risk hosts (such as those who are immunosuppressed) or who are taking critical trips (such as engaging in a
sporting event) during which even a short bout of diarrhoea could affect the trip.
Reference: http://wwwnc.cdc.gov/travel/yellowbook/2016/the-pre-travel-consultation/travelers-diarrhea
Travellers should be advised to eat only food that is fully cooked and served hot and fruit that has been washed in clean water and
then peeled by the traveller. Raw fruits that are eaten unpeeled (such as strawberries) or cut should be avoided, and fruits that are
eaten peeled (such as bananas) should be peeled by the person who eats them.

7
Reference: http://wwwnc.cdc.gov/travel/yellowbook/2016/the-pre-travel-consultation/food-water-precautions
8. An example of secondary prevention is:
A - Detection of asymptomatic diabetic patient (other version: Screening for HTN)
B - Coronary bypass graft
C - Measles vaccination
D - Rubella vaccination
Answer: A
Evidence:
o Primordial prevention: consists of actions to minimize future hazards to health and hence inhibit the establishment factors
(environmental, economic, social, behavioural, cultural) known to increase the risk of disease, E.g. improving sanitation. It
addresses broad health determinants rather than preventing personal exposure to risk factors, which is the goal of primary
prevention.
o Primary prevention: Action to protect against disease as immunization and to promote health as healthy lifestyle.
o Secondary prevention (screening): Identifying & detecting a disease in the earliest stage before symptoms appears.
o Tertiary prevention: Improves the quality of life of people with various diseases by limiting the complications.
Reference: 3rd Edition UQU > Family medicine and statistics > Q 98.

9. Healthy young adult with high cholesterol level. When will you follow him up again for dyslipidaemia? (answer not known)
A - 6 months
B - 12 months
C - 24 months
D - 36 months
Answer: A?
Evidence:
- Fasting lipid levels should be obtained 6-8 weeks following the initiation or alteration of therapy. For patients at goal, lipid profiles should be
performed annually. Reference: https://ghcscw.com/SiteCollectionDocuments/Clinical_Practice_Guidelines/8_CPG_DyslipidemiA.pdf.
- Less than two Cardiac Risk Factors with elevated lipids: Repeat lipid panel in 3-6 months. Reference: http://www.fpnotebook.com/cv/Lipid/Hyprchlstrlm.htm

10. 32-year-old athlete man, who has a family history of HTN and DM, came for check-up. He has good musculature and doing
weight-lifting exercises. His blood glucose level within normal range also his lipid profile except for total serum cholesterol
210 mg/dl. What is your action?
A - Start statin.
B - Change diet.
C - Check up in next few months.
D - Reassure him.
Answer: B (Borderline high cholesterol treated with diet modification)

11. Young female with height of 167 cm and weight of 153 kg. In which class of BMI?
A - Overweight.
B - Obesity class I.
C - Obesity class II.
D - Obesity class III.
Answer: D (Easy way to calculate BMI: Wt. 153 kg, Ht. 167 cm → 1.67 meter.
8
Step 1: Wt./Ht. (in meters) = 153/1.67 = 91.6
Step 2: Result of step one divided again by Ht. (in meters) = 91.6/1.67 = 54.8
Answer is 54 Which is extreme obesity)

12. The targeted glycated haemoglobin in a patient with type 1 DM should be [other version: (long scenario) old man with DM
and HTN, came for routine check-up. (CBC, FBS, Lipid profile… etc.). What is the targeted glycosylated haemoglobin]:
A - 0.065
B - 0.08 (other version: 0.070)
C - 0.095
D - 0.11
Answer: A
Evidence:
o Therapy in most individuals with type 1 or type 2 diabetes should be targeted to achieve a HbA1c ≤7.0% in order to reduce the
risk of microvascular and if implemented early in the course of disease, macrovascular complications.
o More intensive glucose control, HbA1c <6.5%, may be targeted in patients with a shorter duration of diabetes with no evidence
of significant CVD and longer life expectancy, to further reduce risk of nephropathy and retinopathy, provided this does not
result in a significant increase in hypoglycaemia.
o A HbA1c target <8.5% may be more appropriate in type 1 and type 2 patients with limited life expectancy, higher level of
functional dependency, a history of recurrent severe hypoglycaemia, multiple co-morbidities, extensive CAD, and a failure to
attain established glucose targets despite treatment intensification. Reference: Toronto notes.

13. 28-year-old girl came for check-up. She asks about when to do breast self-examination?
A - Not advised.
B - Advise every 4 months.
C - Advise every 12 months.
D - Advise 3 years.
Answer: A | Reference: Oxford General practice, Edition 4 (P:686)

14. Saudi Arabia has a good screening for military job. What is the infection that would prevent a man from getting the job
(other version: Which of these infections leads the workforce to be permanently dismissed)?
A - Hepatitis A virus
B - Hepatitis B virus
C - Hepatitis C virus
D - HIV
Answer: D | Reference: http://www.beforejoiningthemilitary.com/military-medical-requirements-and-disqualifying-conditions/

15. female patient G2P0 had two abortions, came to you in clinic asking to give her any vaccine to help her to get a baby, HCG is
positive. Which vaccine will you give her?
A - Varicella
B - Rubella
C - Mumps
D - Influenza
Answer: D (Varicella, Rubella and Mumps vaccines are all contraindicated in pregnancy) | Reference: UpToDate
9
16. Pregnant woman came to the clinic asking about tetanus risks and prevention. There is a history of contact with patients who
have tetanus. On examination: Normal and the measurement of uterine level from symphysis is 12 cm. What is the action to
prevent baby from tetanus?
A - Introduce Acyclovir to the mother at 18 weeks.
B - Tetanus immunoglobulin and vaccine to the baby after delivery.
C - Give Acyclovir to mother and baby after delivery.
D - Give tetanus vaccine to the mother.
Answer: D | Reference: http://www.cdC.gov/vaccines/pubs/preg-guidE.htm#tdap

17. best way of prevention (other version: What is the best measure in health care):
A - Screening program
B - Genetic counselling
C - Increase individual health awareness (other versions: improve personal something; individual education; environment
modification) or behavioural modification of the person (other version: behavioural modification for the person) (newly added)
D - (other version: environmental modification)
Answer: C [increase individual health awareness (or behavioural modification of the person)]

18. The most REVERSIBLE cause of stroke is?


A - obesity
B - DM
C - HTN
D - hyperlipidaemia
Answer: C

19. The best lifestyle treatment for hypertensive patient:


A. NA restriction less than 6
B. Give to him K
C. Run every day 1.5 Km
D. Do ******? 4 times per week

Answer A
Sodium reduction (≤2.4 g/day)
DASH diet (fruit and vegetables, whole grains, low sodium, low-fat proteins);
Maintaining waist circumference of <102 cm for men and <88 cm for women and weight loss to a BMI of about 25 kg/m^2
Increased physical activity consisting of at least 30 minutes of moderate intensity,
Dynamic aerobic exercise (walking, jogging, cycling, or swimming) 5 days per week to total 150 minutes/week, as tolerated or
recommended by physician
Limited alcohol consumption
http://bestpractice.bmj.com/bestpractice/monograph/26/treatment/details.html

20. Obese man with low calorie diet and intensive exercise could not lose weight. He has DM & HTN. What is the best way to
lose weight?
A. Surgery
10
B. Medication
C. Decrease calories
D. Increase intensity of exercise

Answer: b metformin?

21. You advice patient (with DM?) for walking:


A. Walk 1.5 km 4 days/week
B. Do brisk walking 30 min 3 days/week
C. Reduce sodium intake
D. Give potassium

Answer is B
People with diabetes should accumulate a minimum of 150 minutes of moderate- to vigorous-intensity aerobic exercise each
week, spread over at least 3 days of the week, with no more than 2 consecutive days without exercise
Reference: http://guidelines.diabetes.ca/browse/Chapter10#bib9

22. Type of exercise is recommended for coronary artery disease.


A. isometric
B. isotonic
C. yoga
D. dynamic

Answer: B, isotonic
Reference: SLE made easy

23. 30 age with risk of CAD which exercise benefit for her?
A. Yoga
B. heavy lifting
C. bicycle
D. low resistance
Answer: C.
Large muscle group, continuous exercise, such as walking, jogging, bicycling, swimming, group aerobics, and rowing, is appropriate
for cardiovascular endurance conditioning.
Reference: http://www.medscape.com/viewarticle/716347_3

24. patient with normal glucose level and HBA1c, he is 42 y/o male who’s smoked for 20 years, when do you do these tests
again?
A - after 3 months
B - 6 months
C - 12 months
D - 36 months

11
Answer: D (36 months)

25. 72 old patient started to have gradual memory loss since 2 yrs back, but he is capable of doing his daily activities (e.g.
dressing himself) but lately he started to forget the burner on, and his personality changed from kind and caring father to
aggressive. And he is irritable. What will you do?
A - Do cost effective Tx
B - Refer to geriatric
C - TCA trial
D - Give him Risperidone (antipsychotic)
E - Arrange to transfer him to caring facility true for severe case
Answer: B | Reference: (UQU sle) q151 p447 4th edition

26. pt takes one dose of varicella vaccine, and after one year presents to your clinic. What to do next:
A - give double dose
nd
B - give 2 dose
C - start over
D - antibody test
nd
Answer: B (give 2 dose)
Evidence:
nd
The varicella vaccine in adult take 2 dose with 4-8 w in between. you must catch up vaccination with a 2 dose for all adolescents
nd
and adults who may have missed a 2 dose.
Reference: http://www.cdc.gov/vaccines/schedules/hcp/imz/adult.html

27. Virus that can turn into a new virus and cause a pandemic?
A - Influenza
B - Rhinovirus
C - Parainfluenza
D - RSV
Answer: A

28. If mammogram free/clear of lesions, when to repeat it:


A. 2 y
B. 3 y
C. 4 y
D. 5 y
Answer: A (Correct answer depends on the woman’s age)
Evidence:
ACA recent screening guidelines dictate the following for women at the average risk:
Women ages 40 to 44 should have the choice to start annual breast cancer screening with mammograms if they wish to do so. The
risks of screening as well as the potential benefits should be considered.
Women age 45 to 54 should get mammograms every year.
Women age 55 and older should switch to mammograms every 2 years, or have the choice to continue yearly screening.
Screening should continue as long as a woman is in good health and is expected to live 10 more years or longer.

12
Reference: http://www.cancer.org/cancer/breastcancer/moreinformation/breastcancerearlydetection/breast-
cancer-early-detection-acs-recs

29. 2 years old child came with Hx of falling down. On x-ray # in radius. What will do for this child:
A - referral to paediatrics
B - referral to ortho
C - admission
D - splint for hand
Answer: B
Evidence:
Insufficient information; depends on the fracture type and growth plate involvement.
Forearm fracture is evaluated according to age, deformity of the hand, stability of the fracture
The younger the child, the more likely to do a simple closed reduction and casting
So I think orthopaedics are the ones who can evaluate better
Reference: http://emedicine.medscape.com/article/824949-workup

30. Vaccination can prevent hepatitis?


A - HBV vaccine
B - HCV vaccine
C - HDV vaccine
D - HEV vaccine
Answer: A (only HBV & HAV vaccines are available)

31. Female pt did mammogram which was negative, the doctor told her that it will repeated every 2 years, she insists to do it
regularly, how many years the cancer develop before detection by mammogram: (answer not sure of)
A. 1
B. 2
C. 4
D. 7

Answer: B?

32. 22 y/o female healthy with regular cycle, never had sex. History of bilateral breast pain 3 days before her cycle, no family
history of cancers. She said she wants to get pregnant within 2 years! What will you screen her for: (answer not sure of)
A - US breast
B - PaP Smear
C - human papilloma virus
D - Gram stain for (streptococcus I think)
Answer: A? (This is mostly the description of fibrocystic breasts. First tests to order are ultra sound and mammography)

33. (repeated Q in Family Medicine and Statistics. Q50) WHAT IS THE MOST PREVENTIVE METHODS TO PREVENT HUMAN
DISEASES ?
A. SCREENING

13
B. GENTIC COUNSELING
C. ENVITROMENT MODIFICATION
D. behavioral modification
Answer: d?

34. Pt e high level of cholesterol , what to avoid?! (FM)


A. Organ meat
B. Avocado
C. Chicken
D. White egg

Answer: A
Reference: http://www.webmd.com/cholesterol-management/foods-to-avoid-for-high-cholesterol

35. theoretically which of the following cancer will prevented by vaccination?


A. ALL
B. CML
C. adult T cell leukemia
D. myocois something
Answer : C

36. 45 years old male, came for regular check up, apart from bronchial asthma, his bp is 125/80, ha1c is 5.9, when is the next
time he should check his blood glucose level?
1. 3 months
2. 6 months
3. 12 months
4. 36 months

Answer: blood glucose level daily home screening


HA1C every 3 months .

37. What's the most common chronic infection found in expertise in Saudi Arabia?
A. HAV
B. HBV
C. HCV
D. HIV

Answer: B

38. Best time to do self breast examination?


A. Daily
B. Weekly
C. Monthly
D. Annually

14
Answer: C
http://www.hopkinsmedicine.org/breast_center/treatments_services/breast_cancer_screening/breast_self_e
xam.html

39. Best NSAIDs causing LESS GI symptoms?


A. Indomethicin
B. Aspirin
C. Ibuprofen
D. Celecoxib

Answer: D
http://www.ncbi.nlm.nih.gov/books/NBK45590/

40. The best lifestyle Treatment for HTN patients:


A. NA restriction less than 6
B. Give to him K
C. Run every day 1.5 Km
D. Do some thing 4 time per week (?)
Answer:
In order of effect on BP reduction: weight loss → ↓ saturated fat →Exercise regularly→↓salt intake →↓ alcohol. ((Step-Up to
Medicine, 4E , p439 ))

41. What will tell you tell a young lady in regard to breast ca?
A. Self-exam is obsolete now
B. Self-exam as well as mammography are important
C. Only mammography
D. CT scan

Answer: A
Screening should start at 45 years old with annual mammograms, then switch to mammograms every two years after age 55.
Reference: http://jama.jamanetwork.com/article.aspx?articleid=2463262

42. Mammogram screening delays breast cancer than self-breast examination by?
A. 1 year
B. 2 years
C. 3 years
D. 4 years
Answer:
http://www.cancer.gov/types/breast/hp/breast-screening-pdq

43. Lipid profile almost normal, just cholesterol of 212 (NL < 200)
When will you repeat the lipid profile?
A. 6 month

15
B. 12 month
C. 24 month
D. I forgot but (> 24 ) month
Answer: B and C. Check screening guidelines for dyslipidemia
https://yhdp.vn/uptodate/contents/mobipreview.htm?32/29/33241

44. Patient taking Metformin for MI, what contraindicative to give?


A. Steroid
B. Narcotics
C. Nitrates
D. Antibiotic
Answer: A
Certain drugs tend to produce hyperglycemia and may lead to loss of glycemic control. These drugs include the thiazides and other
diuretics, corticosteroids, phenothiazines, thyroid products, estrogens, oral contraceptives, phenytoin, nicotinic acid,
sympathomimetics, calcium channel blocking drugs, and isoniazid. When such drugs are administered to a patient receiving
Metformin hydrochloride tablets, the patient should be closely observed for loss of blood glucose control. When such drugs are
withdrawn from a patient receiving Metformin hydrochloride tablets, the patient should be observed closely for hypoglycemia.
https://www.drugs.com/pro/metformin.html#i4i_contraindications_id_inv-4e5eed3d-771e-4dd4-8239-be36a2cb4357

45. Female in her 40s her previous mammogram was normal when should she repeat it
A. One year
B. Two years
C. Three years
D. Four years
Answer: a. one year. Annual mammogram beginning at age 40 years for females. 
step up to medicine.

46. a man wants to lose wt have hx of DM he is on regular exercise &low calori diet but his wt not decreasing what will you
advice him:
A. Intense exercise
B. Use medication to loss wt
C. Wt bearing exercise
D. Low calori
Answer: B

47. Moderately over wt women what will advise her:


A. Wt bearing exercise
B. Resistance exercise
C. Yoga
D. Something

48. Hypertensive women 160/90 what will advise her:


A. Reduce salt less than 6g
B. Walk 1.3km 4tims per week
C. Birsik walking 30min 3time per week
16
D. Somthing
Answer: A
Reduce sodium intake to no more than 100 mmol/day (2.4 g sodium or 6 g sodium chloride; range of approximate SBP reduction, 2-8
mm Hg)
Reference: http://emedicine.medscape.com/article/241381-overview

49. Which patient has the greatest risk for developing stroke?
A. 55 year-old, male, hypertensive and obese
B. 50 year-old male, diabetic, hypercholesterolemic
C. 22 year-old female smoker

Answer: A
Explanation: Hypertension, which promotes the formation of atherosclerotic lesions, is the single most important treatable risk
factor for stroke
Reference: Step up to medicine

50. Old lady postmenopausal with osteoarthritis and risk for osteoporosis, what you will do:
A. calcium ,TSH ,dihydroxy vit D
B. bisphosphonate, vit D, calcium
C. DEXA scan
Answer: C We recommend pharmacologic therapy for postmenopausal women with a history of fragility fracture or with
osteoporosis based upon bone mineral density (BMD) measurement (T-score ≤-2.5)
Uptodate

51. heavy smoker pt for > 20 yr. came with white plaque in his tongue with no other symptoms. what is the correct action
A. take biopsy
B. observation
C. start immediately with chemotherapy
Answer: A
http://bestpractice.bmj.com/best-practice/monograph/621/diagnosis/tests.html

52. Parent came to your clinc with their obese child with BMI 33 ,So they are afraid of having their child a disease , they wanted
you to do lipid profile , after taking history you decided to do a lipid profile but why ?
A. Because his parents wish or need this test to be done
B. Because the child eating French fries daily
C. Because there is early family history of cardiac disease
Answer: C

17
53. Exercise for cardiac patients:
A. Yoga
B. Isometric
C. Isotonic
Answer: C
Reference: http://www.healthcentral.com/heart-disease/c/77/23289/heart-heart/

54. Commonest cause of HTN in adolescent


A. Essential
B. renal
C. cardiac
Answer: A

55. A 50 years old female patient travelled for 12 hours, at the end of the flight she could not wear her shoes, what investigation
should you do?
A. Pelvic CT
B. Abdominal US
C. Compression US
Answer: C

56. 47 y/o female, her only child was born when she was 35 y, now she develop epithelial ovarian cancer, no BARCA1 mutation,
what will you tell her daughter about her risk to develop cancer?
A. Family history has no risk
B. OCP has protective effect
C. Barrier contraceptive has protective effect

Answer: B
Risk factors for ovarian cancer:
- Age , BRCA1, BRCA2, Lynche Syndrome, infertility, Endometriosis, Family health history of breast, ovarian, colorectal, or other
cancer.
Protective factors for ovarian cancer :
- Oral contraceptives, Tubal ligation, Breastfeeding, Risk-reducing salpingo-oophorectomy, multiparity

Reference : uptodate & www.cdc.gov/cancer/knowledge/provider-education/ovarian/risk-factors.htm

57. Elderly patient with osteoporosis, most appropriate advice to lower the risk of compression fracture is?
A. Avoid obesity
B. Aerobic exercise
C. Vitamin D
Answer: C

58. Elderly female smoker what provides the most risk for osteoporosis fracture?
18
A. age
B. smoking
C. vitamin d deficiency

Answer: A
http://www.shef.ac.uk/NOGG/NOGG_Pocket_Guide_for_Healthcare_Professionals.pdf

59. do self brest ex every ?


A. month -
B. 3 month
C. 6 month

60. There is a new outbreak of TB. What will you do?


A - Give Ethambutol chemoprophylaxis.
B - Give Rifampicin chemoprophylaxis.
C & D - Other 2 irrelevant choices.
Answer: B (rifampicin: Prophylaxis for those exposed to people with N. meningitidis or HiB meningitis) | Reference: Toronto Notes

61. What is the most common problem faced by community medicine:


A - HTN
B - Coryza (other version: rhinitis)
C - UTI
Answer: B | Reference: FM consultant; OM al Qura, family medicine

62. Best types of carbohydrate in DM (other version: diabetic patient what type of carbohydrates is recommended):
A - Monosaccharide
B - Disaccharide
C - Polysaccharide
Answer: C
Evidence:
Simple carbs have only one or two sugars, so they are digested quickly, making blood glucose rise rapidly to a high peak, which is
what diabetics need to avoid. Examples of simple carbs include the sugars found in fruits and milk, the added sugars in processed
foods, and table sugar.
Complex carbs contain three or more sugars, so these take longer to digest and thus they cause a less rapid rise in blood glucose and
a lower peak. Examples of complex carbs include the fibres in spinach, watercress, buckwheat, barley, wild or brown rice, beans, and
some fruits.

63. Children who are living in a poor country with poor hygiene will have a high risk to hepatitis:
A - HAV
B - HDV
C - HEV
Answer: A

19
64. A school did a screening test for their students and they found that there is a good number of obese students. The school
doctor wants to know more about these students before educating their parents. What should you provide him with (other
version: Collect information about BMI, what is the most imp additional information)? (answer not known)
A - HDL\LDL
B - Girth measurement
C - Dietary habits (other version: eating habits)
th
Answer: C? | Reference: Oxford General practice, 4 edition, obesity (P:174)

65. Medical director discovered cretinism in 90% of children in his village, when he analysed the water he found that it is
deficient in iodine. The director wants to prevent and manage cretinism. What he is going to do initially?
A - Iodine supplementation
B - Thyroxin supplement (Levothyroxine)
C - TSH and T4 in 2 weeks
Answer: B (the mainstay in the treatment of congenital hypothyroidism is early diagnosis and thyroid hormone replacement)
Reference: Medscape.

66. What is the mineral found in water prevents teeth cavities?


A - Folic acid
B - Iron
C - Fluoride
Answer: C

67. The greatest risk factor for stroke?


A - DM
B - HTN
C - smoking
Answer: B

68. Young obese want to lose weight, advice?


A. Yoga
B. Bicycle
C. Weight lifting

Answer: B
Physical activities recommended for obese patients:
Walking
Bicycling
Jogging
Swimming
Running
Reference: http://bestpractice.bmj.com/best-practice/monograph/211/treatment/details.html

20
69. 12 years old girl with type 1 DM, her weight is 40 kg (below 50th percentile) and her height is 150 cm (below 95th percentile),
she has no signs of secondary sexual characteristics of puberty, you want to perform annual screening in clinic for what?
A. Ophthalmology
B. Growth hormone
C. CT renal

Answer is A
- Children who are on appropriate doses of insulin and, as a result, have well-controlled diabetes typically have patterns of growth
and development that are identical to those of otherwise healthy children. Under-insulinization and poor diabetes control can
result in growth delay and growth attenuation. When severe, chronic insulin deficiency results in Mauriac syndrome, also known
as "diabetic dwarfism." Children with Mauriac syndrome have decreased growth velocity, short stature, and delayed puberty,
and hepatomegaly. Improved insulin delivery usually results in normalization of growth but can produce rapid deterioration of
retinopathy and nephropathy, if undertaken too aggressively.
- Growth hormones in these patients will be normal, so no need to screen.
- Screening should be directed toward retinopathy (annual ophthalmic evaluation starting at 10 years old) and nephropathy
(annual urine sample to measure albumin/Creatinine ratio starting at 10 years old).
Source: http://www.medscape.com/viewarticle/412399
https://yhdp.vn/uptodate/contents/mobipreview.htm?39/29/40410#H15

70. 46 y/o male with no risk factors for diabetes and with normal BP and normal fasting glucose and normal HgbA1c (I forget
what the level, but for sure it wasn't in the prediabetic or diabetic range). When will the next screen be?
A. In 6 months
B. In 12 months
C. In 36 months

Answer: C

21
http://image.slidesharecdn.com/bowenpredmcme-150409130005-conversion-gate01/95/bowen-predm-
cme4915-13-638.jpg?cb=1428602546

71. (long scenario) how does hyperglycemia cause infection to occur?


A. allow bacteria to grow.
B. decrease immune response.
C. impairs phagocytosis

Answer: Chttp://www.ncbi.nlm.nih.gov/pubmed/16006275
Explanation: Studies have demonstrated impairment of host defenses, including decreased polymorphonuclear leukocyte
mobilization, chemotaxis, and phagocytic activity related to hyperglycemia.
Link: http://www.ncbi.nlm.nih.gov/pubmed/16006275

72. pt use diuretics and he developed muscle weakness and diarrhea what is the cause?
A. hyper k
B. hypo k
C. hyper Na

Answer: A
Severe hyperkalemia may present as muscle weakness, fatigue and slow heart rate (bradycardia). It is important to monitor
potassium blood levels and to have anelectrocardiogram performed.

73. 9 years old lab, GH NORMAL, INSULIN LIKE GROWTH FACTOR LOW, BONE AGE 7 Y, WHAT TO GIVE
A. Growth hormone
B. Octeroide acetate
C. Diet

Answer: A,
Insulin-like growth factor 1 (IGF-1), also called somatomedin C, IGF-1 is a hormone similar in molecular structure to insulin. It
plays an important role in childhood growth and continues to have anabolic effects in adults. A synthetic analog of IGF-
1, mecasermin, is used for the treatment of growth failure.
https://en.wikipedia.org/wiki/Insulin-like_growth_factor_1

74. Man with abnormal teeth order and marginated white patch on his tongue with ulcer Dx?
A. Toxic ingestion
B. Excessive growth of tonge cells
C. NO Oral thrush , either Aphthous ulcer

Answer: B
Explanation: Acromegaly: Symptoms Changes in facial features: of forehead and jaw, widening the spaces between teeth and
enlarging tongue
Link: http://pituitary.ucla.edu/acromegaly-89

22
75. Doctor informed pt about his disease, prognosis, medications, & side effect, what do you call this form of talk?
A. Improve communication
B. Patient Doctor relationships
C. Educational care

Answer: C?

76. DM , osteoarthritis on NSAID came for regular check up .. Bp found to be 160/something


Invest:
Inc cr +_ urea
Inc bicarb
Dec k
nl na
What is the cause of HTN :
A. Essential
B. NSAID induced
C. Primary hyper aldosteronism

Answer: B

77. A mother called you saying that her diabetic child is unconscious and that she missed the last 2 insulin injections. What would
you advise her?
A. You will call the ambulance to bring them to the hospital.
B. You will ask them to come immediately to the ER
C. Ask the mother to give the child sugar
Answer: A. Diabetic ketoacidosis: When there is not enough insulin circulating, the body cannot use glucose for energy. Instead, fat
is broken down and then converted to ketones in the liver. The ketones can build up excessively when insulin levels remain too low.
Common causes of ketoacidosis include a missed dose of insulin or an acute infection in a person with type 1 diabetes. Ketoacidosis
may be the first sign that a person has developed type 1 diabetes.

78. vaccine he should not receive?


A - Hep B
B - Varicella
C - DTaP
Answer: B (assuming the CD count is < 200, varicella isn’t given, but the rest are; if CD ≥ 200, all are given)
Evidence:
if CD4 > 200, MMR and varicella are only live vaccine that should be given to HIV pts
Don't give OPV to HIV pts or their contacts
Immunization recommendation for HIV patients by the CDC are:
If you have HIV infection and your CD4 count is ≥ 200, talk with your doctor about:
Influenza vaccine each year to protect against seasonal flu
Tdap vaccine to protect against whooping cough and tetanus
Pneumococcal vaccine to protect against pneumonia and other pneumococcal diseases
23
Hepatitis B vaccine series to protect against hepatitis B
HPV vaccine series to protect against human papillomavirus if you are a man or woman up to age 26 years
MMR vaccine to protect against measles, mumps, and rubella if you were born in 1957 or after and have not gotten this
vaccine or have immunity to these diseases
Varicella vaccine to protect against chickenpox if you were born in 1980 or after and have not gotten two doses of this vaccine
or have immunity to this disease
If you have HIV infection and your CD4 count is less than 200, talk with your doctor about:
Influenza vaccine each year to protect against seasonal flu
Tdap vaccine to protect against whooping cough and tetanus
Pneumococcal vaccine to protect against pneumonia and other pneumococcal diseases
Hepatitis B vaccine series to protect against hepatitis B
HPV vaccine series to protect against human papillomavirus if you are a man or woman up to age 26 years
Reference: http://www.cdc.gov/vaccines/adults/rec-vac/health-conditions/hiv.html

79. the community department want to decrease the incidence of stroke what is the appropriate action?
A - education the population about the Hypertension in mall
B - check for hyperlipidaemia in mall
C - check for BMI for people in malls
Answer: A (educate the population about hypertension)

80. effective way for detecting severity of pain with people who can’t communicate:
A. face .‫الرسمات‬.
B. number
C. verbal

Answer: B (number

81. patient with low back pain and normal neurological examination, MRI showed mild spinal stenosis. What is the Tx :
A - surgery
B - physiotherapy
C - biofeedback
Answer: B
Evidence:
Non-surgical treatments – For mild to moderate pain, more conservative treatment methods can include medications (analgesia),
physical therapy, and steroid injections. Reference: http://www.back.com/back-pain/conditions/lumbar-spinal-stenosis/

82. newly diagnose DM type 2 pt. came for his appointment at 10 a.m. but his doctor had critical case some he came to OPD late
(at 11;20 a.m.). the pt get angry. what is the right thing to do:
A. try to understand why the pt is angry
B. try to explain to pt why you are late
C. listen to pt with empathy

24
Answer: B

83. pathological rib fracture , spine T score -2.6 :


A- osteomalasia
B- osteoprosis
C- established osteoporosis

Answer: C
Explanation: T-score of –1 to –2.5 SD indicates osteopenia. T-score of less than –2.5 SD indicates osteoporosis. T-score of less than –
2.5 SD with fragility fracture(s) indicates severe (established) osteoporosis
Reference: http://emedicine.medscape.com/article/330598-workup#c10
http://bestpractice.bmj.com/best-practice/monograph/85/diagnosis/tests.html

84. Epidemic investigation (study ) what is the first step ?


A. Identifying population at risk
B. Count the cases
C. Confirme diagnosis

Answer: C
Epidemiologic Steps of an Outbreak Investigation:
Reference: http://www.cdc.gov/ophss/csels/dsepd/ss1978/lesson6/section2.html

85. Doctor informed pt about his disease, prognosis, medications , & side effect ...?
A. Improve communication
B. Patient Doctor relationships
C. Davo.. something
Answer: A?
instructional communication?

86. Young obese want to lose weight, advice


A. Yoga
B. Bicycle
C. Weight lifting
Answer: B

87. most common cause of dka in adult?


A. Missing insulin
B. Dietary
C. Increase physical activity

Answer: ?

25
The most common precipitating factor is infection, followed by noncompliance with insulin therapy.
Reference: http://www.aafp.org/afp/2005/0501/p1705.html
The most common scenarios for diabetic ketoacidosis (dka) are underlying or concomitant infection (40%), missed or disrupted
insulin treatments (25%), and newly diagnosed, previously unknown diabetes (15%). Other associated causes make up roughly 20%
in the various scenarios.
Reference: http://emedicine.medscape.com/article/118361-overview#a5

88. patient can't take bcg vaccine because he deficiency in


A. Il
B. Tnf gama
C. Ifn
Answer:
Uptodate: “studies among infants demonstrate bcg-associated induction of cd4+ and cd8+ t cells, interferon (ifn)-gamma+,
interleukin (il)-2+, tumor necrosis factor (tnf)-alpha+, and polyfunctional cd4+ t cells”
“studies in adults indicate that bcg induces cd4+, ifn-gamma responses, and ifn-gamma and tnf-alpha secreting cd8+ cells with
cytotoxic activity; data on polyfunctional t cells has been conflicting”
In infants & adults, bcg works on: cd4 & cd8 t cells, ifn-gamma, il 2, tnf-alpha.

89. A patient is concerned about microwave radiation and its risk of cancer on their children. They are asking for your advice,
what will you tell them? (missing options)
A - Microwave cause cancer but not in children
B - Microwave don't cause cancer
Answer: B (If you use a microwave oven in the correct way, there is no known harmful effect on humans)
Reference: http://www.cancerresearchuk.org/about-cancer/cancers-in-general/cancer-questions/radiation-microwaves-and-cancer

90. why do we give influenza vaccine every year?


A. bacterial drift.
B. bacteria develop resistance

Answer: A
Genetic drift: Random changes in the gene frequencies of a population from generation to generation. This happens as a result of
sampling error.
New flu vaccines are released every year to keep up with rapidly adapting flu viruses. Because flu viruses evolve so quickly, last
year's vaccine may not protect you from this year's viruses. After vaccination, your immune system produces antibodies that will
protect you from the vaccine viruses. In general, though, antibody levels start to decline over time — another reason to get a flu
shot every year. Link: http://www.mayoclinic.org/diseases-conditions/flu/in-depth/flu-shots/art-20048000

91. The type of Hepatitis B vaccine is ;


A. Recombinant
B. Inactivated

Answer: Probably option was not mentioned


Explanation: Conjugate vaccines
26
If a bacterium possesses an outer coating of sugar molecules called polysaccharides, as many harmful bacteria do, researchers may
try making a conjugate vaccine for it. Polysaccharide coatings disguise a bacterium’s antigens so that the immature immune systems
of infants and younger children can’t recognize or respond to them. Conjugate vaccines, a special type of subunit vaccine, get
around this problem. When making a conjugate vaccine, scientists link antigens or toxoids from a microbe that an infant’s immune
system can recognize to the polysaccharides. The linkage helps the immature immune system react to polysaccharide coatings and
defend against the disease-causing bacterium. The vaccine that protects against Haemophilus influenzae type B (Hib) is a conjugate
vaccine.

Link: http://www.historyofvaccines.org/content/articles/different-types-vaccines
http://www.vaccines.gov/more_info/types/

92. 35 y/o pt diagnosed with Dm.. you advise him. To do aerobic exercise How much minutes per week,?
A-120
B-180

Aiming for 30 minutes of moderate-to-vigorous intensity aerobic exercise at least 5 days a week or a total of 150 minutes per week. -
See more at: http://www.diabetes.org/food-and-fitness/fitness/types-of-activity/what
werecommend.html?referrer=https://www.google.com.sa/

93. patient presents with bilateral exophalmus and eyelid injection and intermittent ptosis, auscultation shows thyroid bruit,
normal vital sign? (there is some important information missing)
A. retinal vein ..
B. hyperthyroidism

Answer: B
http://emedicine.medscape.com/article/121865-clinical#b3

94. Hypoglycemia is more with


A. Glyburide
B. acarbose
Answer: A

27
95. female patient treated for hyperlipidemia investigation shown increase in CPK, what is the cause?
A. Atrovstatin
B. niacin
Answer: B
http://www.rxfiles.ca/rxfiles/uploads/documents/Lipid-CK-monitor.pdf
https://www.guidelines.co.uk/wpg/statins

96. African with HT diagnosed now protein+2 Wt ttt


A. Diuretic
B. ACEI - Correct Answer

Answer: B
Explanation: Mayo clinic - ACEI are safe to be given to those with kidney diseases as it protects the kidneys from excreting large
amount of fluids.

97. Military soldier want to deceive his boss, got hypoglycemic symptoms, what did he use?
A. Insulin before exercise
B. Metformin before exercises

Answer : A

98. High myopia best treatment?


A. Fakic intraocular lens>> **
B. Photolaser kertoectmy

Answer: incomplete question

99. Which important more to do for elderly at first time I am not sure if there previous hx of hypertension in his father ??
A. LIPID PROFILE
B. ECG

Answer: incomplete question

28
100.increase of which of the following prevalence cause reactivation of TB in developed countries? (missing options)
A - DM
B - HIV
Answer: B (if in developed countries; if in developing countries Diabetes Mellitus is a likely option)
Evidence: Major risk factors for TB activation include:
- HIV infection, recent contact with an infectious patient, initiation of an anti-tumour necrosis factor (TNF) treatment, receiving
dialysis, receiving an organ or hematologic transplantation, silicosis, being in prison, being an immigrant from high TB burden
countries, being a homeless person, being an illicit drug user. Reference: WHO
- People with a weak immune system, as a result of chronic diseases such as diabetes, are at a higher risk of progressing from latent
to active TB. Reference: WHO
- Some certain conditions and other factors associated with progression from LTBI to active TB. These conditions as: HIV infection,
DM other. Reference: CDC

101.mumps in child. what is the most common complication? (missing options)


A - orchitis
B - meningitis
Answer: B commonest A is the correct answer
Evidence: Complications:
• common: aseptic meningitis, orchitis/oophoritis
• less common: encephalitis, pancreatitis, thyroiditis, myocarditis, arthritis, GN, ocular complications, hearing impairment
Reference: Toronto note in (infectious diseases)
Epididymo-orchitis is the most common symptomatic extra-salivary mumps complication in adult males and may develop in as many
as 38% of infected post-pubertal males. It is manifested as pain & swelling of one or both testicles. Where aseptic meningitis occurs
in 4% to 6%. Reference: BMJ Best Practice: http://bestpractice.bmj.com/best-
practice/monograph/1037/diagnosis.html

102.enteric fever is best diagnosed in the first week of presentation by? (missing options)
A - blood culture
B - stool culture
Answer: A
Evidence:
A blood culture during the first week of the fever can show S. typhi bacteria. Urine and stool in second week.
Reference: https://www.nlm.nih.gov/medlineplus/ency/article/001332.htm

103.woman came for check-up; her husband is +ve for HBsAg. All investigations (HBcAb, HBsAg) for woman are -ve. Which marker
should be detected if +ve or -ve in this woman: (missing options)
A - HB IgM
B - HBeAg
Answer: A (HB IgM will be elevated first)

104.the most effective nonpharmacological treatment for hypertension? (missing options)


A - Low sodium diet
B - Decrease Wight
29
Answer: A
Evidence:
non-pharmacological (lifestyle measures) as well as pharmacological means. Lifestyle changes should be the initial approach to
hypertension management and include dietary interventions (reducing salt, increasing potassium, alcohol avoidance, and
multifactorial diet control), weight reduction, tobacco cessation, physical exercise, and stress management.
Reference: http://www.ncbi.nlm.nih.gov/pmc/articles/PMC3028941/

105.Doctor is asking the patient to perform Physical examination? (missing options)


A - Getting informed consent.
B - Show respect to the patient
Answer: explain what you will do and take permission

106.patient can't take BCG vaccine Because he deficiency in: (answer not sure of)
A - IL
B - gama INF
Answer: B?

107.the most effective way to prevent cardiac anomaly in pregnancy is? (# family ?) (missing options)
A - smoking cessation
B - genetic screen
Answer: most important is diabetic control because it's associated with septal hypertrophy

108.Patient entered the clinic, then the physician took the history from him, after that the physician told the patient "Would you
please let me examine you". This sentence is under category of? (missing options)
A - Taking Informed Consent
B - Respect the patient
Answer: A

109.18 years old he did not receive varicella vaccine what you will do? (missing options)
A - give 1 and other after 6 week
B - give 1 and other after 6 month
Answer: A
Evidence:
All children age 13 years and older as well as adults without evidence of immunity should also have documentation of 2 doses of
varicella vaccine, separated by a minimum interval of 4 weeks.
Reference: http://www.immunize.org/askexperts/experts_var.asp

110.Smoker c/o whitish lesion on mouth , not removed by wash ,What is Your diagnosis?
A- thrush
B- leukoplakia

Answer: B

30
Explanation: Oral leukoplakia is a white patch or plaque that cannot be rubbed off, cannot be characterized clinically or histologically
as any other condition, and is not associated with any physical or chemical causative agent except tobacco. Therefore, a process of
exclusion establishes the diagnosis of the disease.
Reference: http://emedicine.medscape.com/article/853864-overview

111.Tell the pt information about his disease and side effect of medication is kind of:
A. establish rapport
B. information

Answer: B

112.patient come from Africa , I forgot the symptoms :/ , what to do :


A. blood culture
B. stool culture

Answer: ?

113.Patient missed second dose varicella vaccine what to do ?


A. measure antibodies
B. give second dose

Answer: B
http://www.immunize.org/askexperts/experts_var.asp

114.Best first investigation to apply in pandemic disease?


A. Establish the diagnosis
B. Know the population who are in risk

Answer: B
It is important to know who is at risk to try to minimize spread.
https://goo.gl/o1rBD9

115.An 80 years old person with symptoms of vitamin B12 deficiency. What to do first?
A. Vitamin b12
B. Reticulocyte count
Answer: A
Step up to medicine: (macrocytic anemia – Vit B12 deficiency – Diagnosis)
Peripheral blood smear: macrocytic RBCs and Hypersegmented neutrophils
Serum Vitamin B12 level: low <100 pg/ml
Serum methylmalonic acid and homocysteine levels
Ab against IF
Schilling test
31
116.They want to stop screening for breast cancer for women under 47 to decrease unnecessary anxiety to the public. You didn’t
agree because there is a gene that cause cancer in young women. What is the gene?
A. APC
B. BRCA2
Answer: B
BRCA1 or BRCA2 mutation.

117.case femal age" 48 i think" when she rebeat mamogram screaing ?


A. 2 year
B. 4 year

118.most reliable screening for prostate cancer is:


A. PR examination
Answer: A
The American Cancer Society (ACS) recommends that men have a chance to make an informed decision with their health care
provider about whether to be screened for prostate cancer. The decision should be made after getting information about the
uncertainties, risks, and potential benefits of prostate cancer screening. Men should not be screened unless they have received this
information. The discussion about screening should take place at:
Age 50 for men who are at average risk of prostate cancer and are expected to live at least 10 more years.
Age 45 for men at high risk of developing prostate cancer. This includes African Americans and men who have a first-degree relative
(father, brother, or son) diagnosed with prostate cancer at an early age (younger than age 65).
Age 40 for men at even higher risk (those with more than one first-degree relative who had prostate cancer at an early age).
After this discussion, those men who want to be screened should be tested with the prostate-specific antigen (PSA) blood test. The
digital rectal exam (DRE) may also be done as a part of screening.
If, after this discussion, a man is unable to decide if testing is right for him, the screening decision can be made by the health care
provider, who should take into account the patient’s general health preferences and values.
Assuming no prostate cancer is found as a result of screening, the time between future screenings depends on the results of the PSA
blood test:
Men who choose to be tested who have a PSA of less than 2.5 ng/mL may only need to be retested every 2 years.
Screening should be done yearly for men whose PSA level is 2.5 ng/mL or higher.

119.Causes of hypertension:
A. BMI > 30
B. High intake sodium
Answer: A
http://bestpractice.bmj.com/best-practice/monograph/26/diagnosis/history-and-examination.html

32
120.Patients on estrogen develops high triglyceride (isolated, other tests were normal). What drug to give?
A. Fenofibrate
B. statin
Answers: A
Explanation: Fenofibrate is a fibric acid derivative that lowers triglycerides while statin lower both triglycerides and LDL.
Reference USMLE Step 2 CK.

121.Patient with back pain that improves with walking, you find that it is muscle strain how will you treat him?
A. physiotherapy
B. surgery

Answer: A

122.50 years old woman with no issues except for hga1c 7.3, ldl and triglyceride are high. What is the next test you want to
order?
A. Tst
B. Lft

Answer: ?
In diabetic patients with clinical cvd or over age 40 years, statin therapy should be added to lifestyle intervention regardless of
baseline lipid levels.
Reference: uptodate
All patients started on statins should have their ast and alt tested as a matter of routine monitoring, even if no symptoms are
present.
Reference: master the board usmle step 2 ck

123.what is the difference between dm type 1 and 2?


A. Endogenous insulin secretion
B. Weight

Answer: a

33
124.Pt obese with dm wt treatment:
A. metformin (rt answer)
B. B-other ops
Answer: a

34
125.elderly female complaining of depression, bilateral shoulder and hip pain. Normal blood workup.
A. Polymyalgia rheumatica
B. Fibromyalgia

Answer: b, normal blood workup

126.Man wants to increase awareness about stroke prevention, what will he do?
A. HTN campaign in mall

Answer: A
Repeated, HTN is the most important factor for stroke prevention.

35
127. 29 yr old female, came for her annual check up , her father was dx with dyslipidemia one year ago and she is anxious about
she will have the same thing , her lab were all normal except for high triglycerides , what will you give her ?
Answer: Fibrates
Therapy for high TG levels:
1. First-line therapy is weight loss, aerobic exercise, glycemic control in diabetics, 
and low-fat diet. 

2. Medications include fibrates, nicotinic acid, and fish oil. 

3. Statins should be considered even in patients with high triglyceride levels 
because of their cardioprotective effects. 

Reference: Step-Up of Medicine.

128.BMI?
(No choices listed)
Answer: Uptodate/WHO: screening for overweight and obesity should include measurement of BMI, waist circumference and
evaluation of overall medical risk. BMI correlates with percentage of body fat and body fat mass. It is defined as a person’s weight in
kilograms divided by the square of the person’s height in metres (kg/m2).
BMI Nutritional status
Below 18.5 Underweight
18.5–24.9 Normal weight
25.0–29.9 Overweight
30.0–34.9 Obesity class I
35.0–39.9 Obesity class II
Above 40 Obesity class III

http://www.euro.who.int/en/health-topics/disease-prevention/nutrition/a-healthy-lifestyle/body-mass-index-
bmi

129.treatment of gastitional DM is ?
A. Insulin

Answer: A
Gestational diabetes mellitus (GDM) is defined as glucose intolerance of variable degree with onset or first recognition during
pregnancy. Infants of mothers with preexisting diabetes mellitus experience double the risk of serious injury at birth, triple the
likelihood of cesarean delivery, and quadruple the incidence of newborn intensive care unit (NICU) admission.
Reference: http://emedicine.medscape.com/article/127547-overview

130.Which antidiabetic medication cause weight gain?


A. Sulfonylurea

Answer: A
Sulfonylureas (SUs) are the oldest and most widely used medications for the treatment of T2DM. Although SU therapy effectively
lowers blood glucose concentrations (average decrease in FPG of 2–4 mmol/l, accompanied by a decrease in HbA1c of 1–2%) by
36
stimulating insulin secretion from β-cells, Hypoglycemia is the most common and most serious adverse event associated with SU
therapy.
Reference: http://www.medscape.com/viewarticle/722513_3

131.What questionnaire to ask about alcohol intake (other version: case about addict person and ask which of the following
question is include in criteria of CAGE questionnaire)?
Answer: CAGE questionnaire is indicated in alcoholism
1. Have you ever felt you needed to Cut down on your drinking?
2. Have people Annoyed you by criticizing your drinking?
3. Have you ever felt Guilty about drinking?
4. Have you ever felt you needed a drink first thing in the morning (Eye-opener) to steady your nerves or to get rid of a hangover?
Reference: http://www.niaaA.nih.gov/research/guidelines-and-resources/recommended-alcohol-questions + Crash Course + https://en.m.wikipedia.org/wiki/CAGE_questionnaire

132.What is the most common cause of hepatitis in KSA? (missing options)


HBV
Answer: B | Reference: http://www.alliedacademies.org/articles/profile-of-viral-hepatits-in-saudi-arabiA.pdf

133.Vegetarian patient having angular cheilitis: (missing options)


A - Vit B6 deficiency
Answer: A
Evidence:
- Causes of angular chelitis: Vit B2, Vit B5, Vit B12, Vit B3, Vit B6 deficiencies, zinc and iron deficiency, …
- Vitamins deficiency among vegetarians: Vit B12, D, B6, …
- B12 deficiency is a common problem among people who follow a vegan diet.
Reference: Webmed + Wikipedia + Medscape.

134.What is the organism found in undercooked meat? (missing options, answer not known)
A - Entamoeba histolytica
Answer: ?
Organisms that can be found in raw meat:
o Beef: E. coli O157:H7, Salmonella, Shigella, Staphylococcus aureus and Listeria monocytogenes
o Poultry: Salmonella and Campylobacter
o Shellfish: Vibrio gastroenteritis,
Salmonellas, Plesiomonas shigelloides,
Staphylococcus and Bacillus cereus
Reference: Toronto Notes

135.What is the vaccine type of Hepatitis B?


(missing options, answer not known)
Answer: ? Subunit | Reference: the table at the
end of FM section
37
136.What is the vaccine type of Pneumococcal vaccine? (missing options, answer not known)
Answer: ? [Both conjugate and polysaccharide are classified as a subunit vaccine (which is like an inactivated vaccine)]
Reference: the table at the end of FM section.

137.definition of epidemiology: (missing options)


Answer: the study of the distribution and determinants of health related events (including diseases and application of this study to
control of diseases and the others health problem) | Reference: Om al Qura, family medicine

138.What’s the best approach in hx? (missing options, answer not known)
A - Open ended Q
Answer: ?

139.Isosorbide dinitrate side effect?


A. Headache Medscape

Answer: A
Explanation:

140.Well controlled DM, BP 3 readings 138/82, what to do next?


A. Nothing

Answer; We have to add antihypertensive as the BP goal for diabetics is < 130/80. ACEI is appropriate.

141.Anti-obesity medication how its work?


⬇apt -Lipes ??

Current and potential anti-obesity drugs may operate through one or more of the following mechanisms:
Appetite suppression-Catecholamines and their derivatives (such as phentermine and other amphetamine-based drugs) are the
main tools used for this, although other classes of drugs such as anti-depressants and mood stabilizers have been anecdotally
used for appetite suppression (see: bupropion and topiramate).
Drugs blocking the cannabinoid receptors may be a future strategy for appetite suppression.
38
Increase of the body's metabolism.
Interference with the body's ability to absorb specific nutrients in food. For example, Orlistat (also known as Xenical and Alli)
blocks fat breakdown and thereby prevents fat absorption.
https://en.m.wikipedia.org/wiki/Anti-obesity_medication

142.Best sport for cardiovascular patient?


Answer: no choices
Patients with genetic CVD (e.g. hypertrophic cardiomyopathy, long-QT syndrome, Marfan syndrome) can safely participate
in most forms of non-competitive recreational sport activities with moderate or low intensity. Examples: informal jogging
without a training regimen, biking on level terrain, or lap swimming, in which energy expenditure is largely stable and
consistent.
o Source: http://circ.ahajournals.org/content/109/22/2807.full
Patients with ischemic heart disease should perform moderate-intensity aerobic physical activity of at least 30 min duration
on most days of the week.
o Source: http://www.medscape.com/viewarticle/776877_4

143.Best sport for old DM?


Answer: no choices
People with diabetes should accumulate a minimum of 150 minutes of moderate- to vigorous-intensity aerobic exercise each week,
spread over at least 3 days of the week, with no more than 2 consecutive days without exercise
Source: http://guidelines.diabetes.ca/browse/Chapter10#bib9

144.Food poisoning prophylactic ...


The four C’s of prevention: cleaning, cooking, chilling & cross contamination (avoiding it)
Reference: http://www.nhs.uk/Conditions/Food-poisoning/Pages/Prevention.aspx

145.Young female wt and height was given and was clearly obese
Investigations? high cholesterol +_ TG
What else u will order?
Answer: TSH

146.Patient with pain in the right heel, more when he wakes up, relieved by the day. Diagnosis?
Answer:
Plantar fasciitis? Heel pain worsens by bearing weight on the heel after long periods of rest. Individuals with plantar fasciitis often
report their symptoms are most intense during their first steps after getting out of bed or after prolonged periods of sitting.
Improvement of symptoms is usually seen with continued walking.

39
147.Tx of traveller's diarrhoea? (missing options)
Answer: ciprofloxacin

148.Pt came with HTN and get controlled by medication after 1 week she came with dry cough which medication is prescribed?
A - Perindopril (ACE) (missing options)
Answer: A

149.23 years old female concern about breast cancer , her mother and sister had breast cancer , what to do or what to look for
I'm not sure :/ ?
A. BRCA

Answer: A
Explanation: primary care providers should screen women who have family members with breast, ovarian, tubal, or peritoneal
cancer with screening tools designed to identify a family history that may be associated with an increased risk for mutations in
breast cancer susceptibility genes (BRCA1 or BRCA2). Women with positive screening results should receive genetic counseling and,
if indicated after counseling, BRCA testing (Grade B)
Reference: Toronto Notes 2015, page FM4

150.a complaint for breast cancer screening for women under 52 years and older to decrease unnecessary anxiety to the public.
You’ll include those under 52 if they have which gene mutation?
A. BRCA2
Answer :A

151.HIV patient scenario but no dx (no cd4 count), which (question 18 has been moved to “Incomplete Questions”)

152.risk for osteoporosis: (missing options)


Answer: Age
Evidence: Osteoporosis risk factors are: Advanced age (≥50 years), Female sex, White or Asian ethnicity, Genetic factors, such as a
family history of osteoporosis, Thin build or small stature (eg, body weight less than 127 lb), Amenorrhea, Late menarche, Early
menopause, Postmenopausal state, Physical inactivity or immobilization, Use of certain drugs (eg, anticonvulsants, systemic steroids,
thyroid supplements, heparin, chemotherapeutic agents, insulin), Alcohol and tobacco use, Androgen or estrogen deficiency,
Calcium deficiency, Dowager hump. Reference: http://emedicine.medscape.com/article/330598-overview#a4

153.also about risk for osteoporosis: (no stem of Q, missing options)


Answer: Aging process

154.viral gastroenteritis prevented by which vaccine: (missing options)


A - Rota vaccine
Answer: A (rota vaccine prevent GI and given at 2, 4 month)

155.vaccination (no stem of Q, missing options, answer not known)


A - Rubella vaccination
40
Answer: ?

156.example of secondary prevention


A. Detection of asymptomatic diabetic

Answer: A

157.29 yrs old female, came for her annual check-up, her father was dx with dyslipidaemia one year ago and she is anxious about
she will have the same thing, he lab were all normal except for high triglycerides, what will you give her? (missing options)
Answer: diet and exercise for 3 month
Evidence:
Nonpharmacologic management of hypertriglyceridemia is generally the initial treatment for patients with this condition. This
primarily involves lifestyle modifications such as diet, exercise, weight reduction, smoking cessation, and limiting alcohol intake.
Reference: http://emedicine.medscape.com/article/126568-overview

158.What is the best sentence you ask the pt to know about his satisfaction about the asthma? (missing options, no answer)
Answer: ?

159.Vaccine of hep.a missed second dose what to do? (missing options)


Answer: ?
Evidence:
The second dose should be administered as soon as possible. The first dose does not need to be re-administered.
Reference: http://www.cdc.gov/hepatitis/hav/havfaq.htm#B3

160.splenectomy. what vaccine should the pt take: (missing options)


Answer: PVC + meningococcal vaccine + Hib + annual influenza vaccine
Evidence:
Spleen is an organ that remove damaged RBCs and also it's also protect the body against bloodstream infection by removing the
bacteria from the blood. (is important to fight infection) So, splenectomy or a non-functional spleen can worsen sepsis
Most common organism which causes sepsis is streptococcus pneumoniae, less commonly Neisseria meningitidis or Hib
So, the pt would need:
A - pneumococcal vaccine is recommended and it has 2 types: 1- PPSV23 ; 2- PCV13
B - Hib vaccine ; C - meningococcal vaccine _ meningococcal conjugated vaccine ; And Influenza vaccine annually.
Reference: http://www.uptodate.com/contents/preventing-severe-infection-after-splenectomy-beyond-the-
basics?source=outline_link&view=text&anchor=H1#H1

161.Which of the following vaccination is appropriate for asplenic patients: (missing options)
A - Pneumococcal vaccine

41
Answer: Pneumococcal vaccine, HiB, Meningococcal those 3 + annual influenza vaccine
Evidence: If you do not have a spleen or your spleen does not work well, talk with your doctor about:
• Influenza vaccine: each year to protect against seasonal flu
• Tdap vaccine: to protect against whooping cough and tetanus
• Hib vaccine: to protect against Haemophilus influenzae type b (Hib) if you were not previously vaccinated with the vaccine
• Pneumococcal vaccines: (both types) to protect against pneumonia and other pneumococcal disease
• Meningococcal vaccines: (both types) to protect against meningitis and other meningococcal disease
• Zoster vaccine: to protect against shingles if you are 60 years and older
• HPV vaccine: series to protect against human papillomavirus if you are a man up to age 21 or woman up to age 26
• MMR vaccine: to protect against measles, mumps, and rubella if you were born in 1957 or after and have not gotten this vaccine
or have immunity to these diseases
• Varicella vaccine: to protect against chickenpox if you were born in 1980 or after and have not gotten two doses of this vaccine or
have immunity to this disease
Reference: http://www.cdc.gov/vaccines/adults/rec-vac/health-conditions/asplenia.html

162.what is the anti influenza medication that is given intranasally?


Answer: ?
Live attenuated influenza vaccine (nasal spray flu vaccine) provides protection against four flu viruses: an influenza a (h1n1) virus, an
influenza a (h3n2) virus and two influenza b viruses.
Reference: centers for disease control and prevention.
http://www.cdc.gov/vaccines/hcp/vis/vis-statements/flulive.html

163.diabetic patient on metformin and another drug but still blood sugar not controlled. He is allergic to sulfa, what to add?

Answer: ?
➢ Metformin should be used as initial therapy for type 2 diabetes unless contraindications exist. Second-line agents include
sulfonylureas, dpp-4 inhibitors, glp-1 receptor agonists, meglitinides, and α-glucosidase inhibitors. Tzds are no longer
recommended because of potential increases in cardiovascular risk and fracture risk.
➢ The incretin agents are a reasonable second choice if the main problem is ppg elevation and if hypoglycemia and weight gain are
concerns. If fpg and postprandial glucose are elevated and if cost is a concern, sulfonylureas are a reasonable second-line agent.
➢ If a patient has symptomatic hyperglycemia, poor control despite two to three oral agents, or an a1c > 8.5%, insulin should be
considered.
Reference: http://m.clinical.diabetesjournals.org/content/30/2/72.full

164.37 years old presented with back pain. On examination there was tenderness when palpating paraspinal muscles,
neurovascular exam was normal. What is the treatment?

42
A. Physiotherapy

answer: a

165.obese patient recently diagnosed to have dm ii. He is following a diabetic diet regimen and he exercises regularly. When he
came to you in the next visit... His blood sugar was high and he gained 5 kgs... He was also complaining of thirst and hunger,
what would you give him:?
a. Metformin

Answer: a

166.a patient known case of htn controlled with ccbs. Developed bph. How will you treat?
a. Prazosin
Answer: a

167.patient known case of diabetes type 2 suffer from recurrent hypoglycemia. Which drug is responsible?
A. Sulphonylureas

Answer: a
Patients on sulfonylureas and meglitinides have the highest incidence of hypoglycemia because of their pharmacological action of
increasing insulin secretion. Of the sulfonylureas, glyburide presents the highest risk of hypoglycemia. Combination therapies,
especially those regimens containing a sulfonylurea, increase the risk of hypoglycemia.
Reference: davidson's and http://www.ncbi.nlm.nih.gov/pubmed/19421967

168.which of the following is a primary cause of osteoporosis?


A. Age
Answer: a
Primary type 1: most common in post-menopausal women, due to decline in estrogen, worsens with age.
Primary type 2: occurs after age 75, seen in females and males at 2:1 ratio, possibly due to zinc and vitamin d deficiency.
Secondary type 3: drugs (corticosteroids and chemotherapy) or other diseases (cushing syndrome, multiple myeloma,
ibd, renal failure, or ra),
Reference: toronto notes and http://www.medscape.com/viewarticle/410461_3

169.which of the following is considered to be beneficial in hypertensive patients who are diabetic?
A. Ace inhibitors

43
Answer: a
Hypertensive patients with dm and tight bp control by use of acei or b-blocker reduced risk of diabetic complications and death
related to dm and reduced risk of end-organ damage. Bp goal in diabetics is <130/80.
Reference: toronto notes and kaplan

170.cause of secondary htn

Reference: american academy of family physicians

171.scenario about diabetic what is the tx?


Answer: metformin

172.which type of carbohydrates is better for diabetic patient?


Polysaccharides
answer: polysaccharides, a type of carbohydrate that includes starch and cellulose, may benefit people with diabetes because they
help retard absorption of glucose
Reference: http://www.sciencedaily.com/releases/2009/07/090728172604.htm

173.Low back pain in the morning that resolves in 30 mins two cases, what to do to the patient?
Answer : physiotherapy to strengthen muscles

44
174.dm type 1 which of these confirm dx?
A. Acetone

Answer: a
Ketones are natural by-products of fat breakdown. If you have diabetes and produce little or no insulin then ketones in your urine
indicate that fat, and not glucose, is being used as an energy source. Urine ketones are not reliable for diagnosing or monitoring
diabetic ketoacidosis (dka), although they may be useful in screening. The plasma acetone level—specifically, the beta-
hydroxybutyrate level—is a more reliable indicator of dka, along with measurement of plasma bicarbonate or arterial ph as
clinically required
http://www.nps.org.au/conditions/hormones-metabolism-and-nutritional-problems/diabetes-type-1/for-
individuals/diagnosis
http://emedicine.medscape.com/article/117739-workup

175.smoking increases risk of:


A. bladder cancer

Answer: a
Smoking is the most important risk factor for bladder cancer. Smokers are at least 3 times as likely to get bladder cancer as
nonsmokers. Smoking causes about half of the bladder cancers in both men and women.
Reference: http://www.cancer.org/cancer/bladdercancer/detailedguide/bladder-cancer-risk-factors

176.you give pt aci for treating hypertension, what you will add?
answer: step 1 antihypertensive treatment with an angiotensin-converting enzyme (ace) inhibitor or a low-cost angiotensin-ii
receptor blocker (arb). If diuretic treatment is to be initiated or changed, offer a thiazide-like diuretic, such as chlortalidone. A
combination of ace inhibitors and diuretics instead of ace inhibitors alone is recommended for preventing recurrence of stroke
based on findings of (progress).

http://www.cleveland
clinicmeded.com/medi
calpubs/diseasemanag
ement/nephrology/art
erial-hypertension/
https://www.nice.org.
uk/guidance/cg127/ch
apter/1-
guidance?unlid=85464
02482015836347
http://www.pharmacology2000.com/cardio/antihyper/antihype.htm

45
177.q about pathophysiology of dm 1 i don't remember the options
Answer:

46
178.lipid profile of a patient shows high level and patient is on simvastatin what to add?
Answer:
Uptodate: “thus, in patients who do not achieve a particular ldl-c goal on statin therapy alone, we suggest not adding a nonstatin
lipid-lowering medication for primary prevention. That is, the patient should be maintained on statin therapy as his/her only lipid-
lowering medication.”

179.pt came with cough and she take anti cholesterol medication i cant remember the name , she started it since 3 weeks , the dr
should worry about what ? " not sure maybe it's 2 different q "
Answer: statin = lft

180.differentiate between dm type 1 and 2


Answer: see picture

47
181.a question about anti–dyslipidemia drugs.

Answer: see picture

48
Reference: first aid usmle step 1

182.Sign of osteoporosis on x-ray


Answer: dexa scan ??
Osteopenia
radiographic features
decreased bone density can be appreciated by decreased cortical thickness, loss of bony trabecula in early stages in radiography.
Bones like vertebra, long bones (proximal femur), calcaneum and tubular bones are usually looked for evidence of osteoporosis.
Plain film
not a sensitive modality, as more than 30-50% bone loss is required to appreciate decreased bone density on radiograph
vertebral osteoporosis manifests as:
pencilling of vertebrae
loss of cortical bone (picture frame vertebra) and trabecular bone (ghost vertebra)
compression fractures and vertebra plana
loss of trabecula in proximal femur area which is explained by singh's index (which can also be seen in the calcaneum
in tubular bones (especially metacarpals), there will be thinning of cortex
cortical thickness <25% of whole thickness of metacarpal signifies osteoporosis (normally 25-33%)

49
183.Target ha1c in type 1 dm ?
Answer: < 6.5

184.Patient diabetic for years and was just dx as htn, what to give:
A. Acei- pril drugs
Answer : ace inhibitors — are first-line therapy in all patients who have hf or asymptomatic lv dysfunction, in all patients who have
had an st elevation mi, in patients with a non-st elevation mi who have had an anterior infarct, diabetes, or systolic dysfunction, and
in patients with proteinuric chronic kidney disease
-in diabetic patients, combination treatment is commonly needed to effectively lower bp, so the discussion of which medication to
be used first is not always important. However, a blocker of the renin–angiotensin system (ras) should almost invariably be included
because of the evidence of its superior protective effects. Especially, in proteinuric diabetic nephropathy, ras blockade is clearly
indicated. Large hypertension and heart failure trials have also reported an impact on diabetes development in favor of ras blockade
(figure 3). So for persons with cardiovascular or kidney disease, including microalbuminuria, or with cardiovascular risk factors in
addition to diabetes and hypertension, an acei or arb should be started.
(http://www.medscape.com/viewarticle/769327_3)

185.Pt with high cholesterol on treatment, has muscles aches wt was she given:
A. Statins

Answer:
Fibrate + statins = more risk
Statin alone also known to elevate liver enzymes
(master the board 3rd edition page 69)
Statins are extremely well tolerated by most patients but can produce a variety of muscle-related complaints in some individuals.
The most serious risk of these drugs is rhabdomyolysis with acute renal failure and even death.
(http://www.medscape.com/viewarticle/771737_2)
-the major concern when using a statin-fibrate combination is the potential increased risk for myopathy and rhabdomyolysis.
(http://www.medscape.org/viewarticle/563490)

186.Patient with muscle and joint pain he take anti lipid medication :
A. Statin not sure

187.Case dm + htn , wt is ttt ?


A. Pril ( acei )

188.What is the developmental Screening tool that relies on the parents’ information only? (missing options)
Answer: Milestone
50
121. Calculation of BMI and what degree pf obesity
●Underweight – BMI <18.5 kg/m2.
●Normal weight – BMI ≥18.5 to 24.9 kg/m2.
●Overweight – BMI ≥25.0 to 29.9 kg/m2.
●Obesity – BMI ≥30 kg/m2. THEN
●Obesity class I – BMI of 30.0 to 34.9 kg/m2.
●Obesity class II – BMI of 35.0 to 39.9 kg/m2. (from here and above morbid obesity)
●Obesity class III – BMI ≥40 kg/m2. This type of obesity is also referred to as severe, extreme, or massive obesity.

51
Research

52
1. Epidemiological study to see the effect of smoking in lung cancer: 90% of lung cancer patients are smokers while 30% of those
who don’t have the disease are smokers. The specificity of smoking as a risk factor is (other version: What is the incidence
and prevalence predicting the lung cancer in relation to smoking no answer to other version):
A - 10%
B - 40%
C - 30%
D - 70%
E - 90%
Answer: D
Evidence: Specificity = 70 / (30 + 70) = 70%
Lung Cancer No Lung Cancer

Smoker 90 (A) 30 (B)

Non smoker 10 (C) 70 (D)

Important equations:
Prevalence of Disease = People who have the disease / Total × 100
Sensitivity = A / (A + C) × 100
Specificity = D / (D + B) × 100
Positive Predictive Value = A / (A + B) × 100
Negative Predictive Value = D / (D + C) × 100
Reference: an intern who had the same question in the exam and got 100% in statistics section

2. were tested positive with your test. Also figures showing the number of true negatives with your test. What is the specificity
of your new test? (missing details of Q, answer not known)
A - 40%
B - 50%
C - 60%
D - 80%
Answer: ?

3. You conducted a study in which a group of epileptic patients using carbamazepine for 10 years. Now you compare them with
their age equivalent healthy individuals. What is this type of study called?
A - case control study
B - retrospective cohort study
C - cross sectional
D - prospective cohort study

53
Answer: D
Evidence:
o Cohort study: compares a group with exposure (carbamazepine in this Q) to a group without such exposure. Done in two ways;
either Prospective (you give the exposure and follow the subjects for a specific time) or retrospective (the exposure occurred
already in the past, you look back in the history).
o Case control study: compares a group with disease to a group without disease.

4. 80 persons found to have Typhoid Fever in a population of 20,000 during the last 4 years. If we measure it on a population
of 100,000, what will be the incidence in one year?
A - 20
B - 100
C - 150
D - 200
Answer: B (Incidence in a population of 20,000 for 1 year is 20/20,000. Therefore, the incidence in a population of 100,000 for 1 year is 100/100,000)

5. Doctor start research to comparing between people had risk factors to MERS-CoV and control group 100 had risk factor 88
developed MERS-CoV 500 control group developed 6 what the ratio
between risk group and control group (other versions: a table that
has 100 exposed and 80 affected & 500 not exposed and 6 affected.
What’s the ratio of exposed to non-exposed; History of study for 2
groups, one has exposure to MERS virus, the other has not been
exposed:
exposure to MESR Non exposure to MESR
80 6 HAVE DISEASE
20 494 NO
100 500 TOTAL

what is the risk of exposure to MERS-virus; Cohort study – 2 groups: one


of them exposed to patient with positive MERSA and the other group
are not exposed to these patients):
A. A - 1:33
B. B - 1:44
C. C - 1:55
D. D - 1:66
Answer: D (1:66: [(80 ÷ 100) × (500 ÷ 6)] = 66)

6. Cohort study; calculate relative risk?


A. 33:1
B. 44:1
C. 55:1
D. 66:1

Answer: D

54
𝑐𝑎𝑠𝑒𝑠 𝑒𝑥𝑝𝑜𝑠𝑒𝑠𝑒𝑑 𝐶𝑎𝑠𝑒𝑠 𝑛𝑜𝑡 𝑒𝑥𝑝𝑜𝑠𝑒𝑑 80 6
RR= [ ]/[ ]= [ ]/[ ] = 66.66
𝑇𝑜𝑡𝑎𝑙 𝑒𝑥𝑝𝑜𝑠𝑒𝑑 𝑇𝑜𝑡𝑎𝑙 𝑛𝑜𝑡 𝑒𝑥𝑝𝑜𝑠𝑒𝑑 80+20 500
https://practice.sph.umich.edu/micphp/epicentral/relative_risk.php

7. Conduct study for obesity cause prostate cancer. The total men with prostate cancer is 1000 and the total of men without
prostate cancer is 1000. The obese men with prostate cancer is 50 and the non-obese men with prostate cancer is 10. What
statement is right?
A. OR=.52
B. OR=5.2
C. RR=.52
D. RR=5.2
Answer: B
OR = A * D / B*C = 50*990 / 950*10 = 5.2
https://www.medcalc.org/calc/odds_ratio.php

8. Relative risk of exposed to mers:


A. 33:1
B. 44:1
C. 55:1 (the right answer) 100% sure because i got 100% in research.
D. 66:1

9. There is a study to the effect on intensive insulin regimen in the reduction of neuropathy in the patient, the results are as
following
Event in the regular insulin regimen:0.092
Event in the intensive insulin regimen:0.022
Which of the following is correct?
A. patient expected event rate:0.07
B. odd ratio is 4.2
C. number needed to harm:7
D. number needed to treat:12

Answer:

10. In a cohort study. To study the association of some kind of lubricant and skin cancer.
The unexposed risk: 2
The exposed risk: 3
What can you calculate? Giving what’s mentioned above
A. Number needed to treat = 20
B. Number needed to harm = 20
C. Odd ratio..
D. RR = 0.7

55
Answer: Relative risk
Relative risk or risk ratio (RR) is the ratio of the probability of an event occurring (for example, developing a disease, being
injured) in an exposed group to the probability of the event occurring in a comparison, non-exposed group. Relative risk includes
two important features: (i) a comparison of risk between two "exposures" puts risks in context, and (ii) "exposure" is ensured by
[1][2]
having proper denominators for each group representing the exposure
Consider an example where the probability of developing lung cancer among smokers was 20% and among non-smokers 1%.

Disease status
Risk
Present Absent
a b
Smoker
c
Non-smoker d

Here, a = 20, b = 80, c = 1, and d = 99. Then the relative risk of cancer associated with smoking would be
R R = a / ( a + b )/ c / ( c + d ) = RR= {20/100}{1/100}}=20.}
Smokers would be twenty times as likely as non-smokers to develop lung cancer.
Source: https://en.wikipedia.org/wiki/Relative_risk

11. In a cohort study on lubricant oil use and urinary bladder CA done over 20 years
10000 exposed - 10000 non exposed
750 exposed got CA - 150 non exposed got CA
then they asked about the incidence in 1000 in one year?
A. A - 2.25
B. B - 45
C. C - 0.45
D. D - 0.225
Answer: A [Wrong/missing question specifics. (exposed or non-exposed)]
Correct Answer is: D (0.225%)
If they wanted to calculate the incidence proportion or risk, then it would be 0.225% in one year.
Number of new cases of disease or injury during specified period/size of population at start of period
Risk= [(750 + 150) / 20000] * 100 = 4.5% in 20 years
4.5 / 20 years= 0.225% in one year

12. Incidence of diseae (I can recall that disease )117/1000 population per year … Mean time disability >>>> 2.5 days Days lost
>>>>1.6 day So what is the prevalence of the mean time disability ?
A. 48 or 46
B. 73.1
C. 180 to 188(I can not remember the exaxt number)
D. 292.5
Answer: B

56
13. persons found to have Typhoid Fever in a population of 20,000 during the last 4 years. If we measure it on a population of
100,000, what will be the incidence in one year ?
A. 20
B. 100
C. 150
D. 200

14. Study about GDM association with malformation. 2 groups. 5000 control group, 50 developed. 1000 diseased group 20
developed.(not sure about the numbers) The best for risk factor association? (and calculate)
A. Relative risk =2
B. odds ratio=2
C. RR= ?
D. OR =?

OR used for case control study


Answer; B (By community specialist)
OR = ( 20*4950 ) / ( 50*980 ) = 2

15. Case control study between lung cancer and smoking:


A. Non-smoker 1
B. Low smoker 1.3
C. Moderate 3
D. Heavy 5

16. DM chohort study during 3 years of 10.000 DM pts , number increase about 2000 pt at the beginning of the study and 1000 at
the end , how much percent increase ??
A. 10.2
B. 12.2
C. 20.2
D. 22.2
Answer: Unclear scenario

17. Study comparing 2 groups of pregnant ladies on a specific diet, outcome of pregnancy ? What type of study ?
A. cohort study
B. Cross sectional
C. Historical cohort

Answer: A
Cohort studies are observational studies in which the starting point is the selection of a study population, or cohort. They are
classified based on a certain “risk factor” and the outcome of that risk is monitored.
https://www.iarc.fr/en/publications/pdfs-online/epi/cancerepi/CancerEpi-5.pdf

57
18. “Non response bias”. What is this study?
A - Cross sectional.
B - Cohort.
C - Case-control
Answer: A [Non-response bias (example of selection bias) occurs with cross sectional study]
th
19. In a study they are selecting every 10 family in the city, what is the type of study
A - systematic study
B - stratified study
C - non-randomised study
Answer: A (It differs from the one written in 3rd Edition UQU Family medicine and statistics - Q 19)
Evidence:
- Random sampling: preferred way of sampling, it is often difficult to do. It requires that a complete list of every element in the
population be obtained. Computer generated lists are often used with random sampling.
- Systematic sampling: every kth element is taken. This is similar to lining everyone up and numbering off "1,2,3,4; 1,2,3,4; etc".
When done numbering, all people numbered 4 would be used.
- Convenience sampling: readily available data is used. That is, the first people the surveyor runs into.
- Cluster sampling: accomplished by dividing the population into groups called clusters -- usually geographically. The clusters are
randomly selected, and each element in the selected clusters are used.
- Stratified sampling: divides the population into groups called strata. For instance, the population might be separated into males
and females. A sample is taken from each of these strata using either random, systematic, or convenience sampling.

20. Study compared effect of medication on baby's whose mothers were/were not taking the medication while pregnant, and
they followed them till birth and after developing. What is the type of the study?
A. Cohort
B. Cross sectional
C. History cohort

Answer: A

http://www.ncbi.nlm.nih.gov/pmc/articles/PMC2998589/

58
21. It's found that patients on type A insulin have a higher incidence of (not known) compared to people taking type B insulin.
Type A insulin event rate is 0.092. Type B insulin event rate is 0.022. Which one of the following is correct: (missing options)
A - Peep is 0.07 Odd ratio
B - Something cases to harm is 8 (?) number needed to harm (NNH)
C - Something cases to.... Is 14
Answer: ? (options incomplete)
Evidence:
Control event rate (CER) = c / c + d
Experimental event rate (EER) = a / a + b
(a) Relative Risk (RR) = EER / CER= [a / (a + b)] / [c / (c + d)]
(b) Relative Risk Reduction (RRR) = (CER – EER) / CER (commonest reported measure of dichotomous treatment effect)
(c) Absolute Risk Reduction (ARR) = CER - EER
(d) Number Needed to Treat (NNT) = 1/ARR
A certain risk reduction may appear impressive but how many patients would you have to treat before seeing a benefit? This
concept is called "number need to treat" and is one of the most intuitive statistics for clinical practice.
For example if:
Yes No
Exposed 8 (a) 992 (b)
Not Exposed 10 (c) 990 (d)

The RR = [(8 / 1000) / (10 / 1000)] = 0.8 making the RRR = [(1 - 0.8) / 1] = 0.2 or 20%. Although this sounds impressive, the absolute
risk reduction is only 0.01 - 0.008 = 0.002 or 0.2%. Thus the NNT is 1 / 0.002 = 500 patients. It is obvious that on an individual patient
basis the pre-intervention risk or probability is a major determinant of the degree of possible post-intervention benefit, yield, or risk
reduction.

22. Risk of un-exposed population 2, risk of exposed population 3 , what is true ;


A. Number to be treated 20
B. Patient event risk 2
C. Number to be harm ..

Answer:

23. Smoker with lung cancer 80, total smokers 100, smoker without lung cancer 6 total nonsmokers 500. Relative risk reduction?
A. 33:1
B. 55:1
C. 66:1

Answer:

24. risk of un-exposed population 2, risk of exposed population 3 , what is true ; (RESEARCH)
A. Number to be treated 20
B. Patient event risk 2
C. Number to be harm ..

Answer:
59
25. smoker with lung cancer 80, total smokers 100 , smoker without lung cancer 6 total nonsmokers 500.. Relative risk reduction
? (Research)
A. 33:1
B. 55:1
C. 66:1

Answer:

26. case control study showing this information: Odd ratio .75, Control rate .05 What’s correct :
A. Number needed to tx is 69
B. Number needed to harm 69
C. Relative risk is .12

Answer: ?

27. Which is true about null hypothesis? (missing options)


A - There is significant difference between tested population
B - There is no significant different
Answer: B
Evidence:
the term "null hypothesis" usually refers to a general statement or default position that there is no relationship between two
measured phenomena, or no association among groups

28. Case control study: (missing details of Q, no stem of Q, missing options)


A - Odd 0.05
B - Event 0.75
Answer: OR = 5.2

29. Standard deviation is


A. Study of variant
B. Determine the range
Answer : A
SD
http://www.mathsisfun.com/data/standard-deviation.html

30. The number of URTI cases per 1000 population this year is 117 cases. The average number of disability from work is 2.5 days.
What is prevalence of disability per 1000?
A. 292.5

Answer: A

31. What is the definition of attributable risk?


60
A. difference in rate of a condition between an exposed population and an Unexposed population.

Answer: A
In epidemiology, attributable risk is the difference in rate of a condition between an exposed population and an unexposed
population.[1] Attributable risk is mostly calculated in cohort studies, where individuals are assembled on exposure status and
followed over a period of time.

32. There is Q about cumulative incidence “I’m not sure about the Q but I'm sure about the numbers" the new case if some
disease for 2012 is 200 and the already existed disease is 80 out of population 8000000" what is the CI for 100000:
A. 3.5

Answer: A
New case 200
Old case 80
Total 280
So,
(280/8000000)* 100000= 3.5

33. Definition of epidemic curve?


Answer: Eepidemic curve" shows the frequency of new cases over time based on the date of onset of disease. The shape of the
curve in relation to the incubation period for a particular disease can give clues about the source
Reference:
http://sphweb.bumc.bu.edu/otlt/MPHModules/EP/EP713_DescriptiveEpi/EP713_DescriptiveEpi3.html

34. How to calculate attributable risk?

https://quizlet.com/133762520/ch-13-attributable-risk-flash-cards/

35. Cohort study, calculate the Relative Risk: (missing options, answer not known)

61
Diseased Healthy

Exposed A B

Not exposed C D

Answer: ? Relative Risk = [A / (A + B)] / [C / (C + D)]

36. with table of sensitivity, specificity, positive and negative predictive


values: (missing options)
Answer:
- Sensitivity: the probability that a diseased pt will have a positive test result
- Specificity: the probability that a non-diseased pt will have a negative test
result

Reference: UQU + picture

37. Attributable risk meaning: (missing options)


Answer: AR = the amount of disease incidence that can be attributed to a specific exposure
Evidence:
- Difference in incidence of disease between exposed and non-exposed individuals
- Incidence in non-exposed = background risk
- Amount of risk that can be prevented
Reference: http://www.pitt.edu/~super7/30011-31001/30101-30111.ppt

38. calculate the annual prevalence: (missing details of Q, missing options)


Answer: please see table of statistics

39. research question asking about the relative risk for exposure and none exposure to something. The numbers are 80 out of
1000 in exposure. 6 out 5000 in non exposure.
A. 66:1

correct answer is A, I got 100% in research.

40. Standard deviation


A. Variability
Answer: A
The most widely used measure of dispersion of a frequency distribution. It is equal to the positive square root of the
variance. Whereas the mean tells where the group of values are centered, the standard deviation is a summary of how

62
widely dispersed the values are around the center.
Reference: Kaplan USMLE Step 2 Epidemiology.

41. Study of relation of the bladder cancer and lubricating oil use , they take 10,000 male who use the oil and 10,000 who don't
and follow them for 20 years , from the one who use the lubrication oil 750 developed cancer and 150 who don't use the oil
developed the cancer , calculate the incidence per 1000 per one year .

Answer: 20000 * 20 = 400000 person-years. 750+150 = 950 events (cancer).


Incidence rate = event/person-time = 950/400000 = 0.00237 event per person-year
= 2.357 per 1000 person/year
Explanation: Incidence is a term which is used to measure disease frequency which occurs in a population over a period of time.
Incidence rate = The number of events divided by the amount of person-time observed (E/NT). Person-time means the number of
persons multiplied by the time they were followed up; if you followed 10 persons for 100 years, then person-time = 10 * 1000 =
1000 person year. In the answer above, the in incidence rate is 0.00237 events per 1 person per year, so, to make it per 1000 per
year, multiply by 1000.
Reference: https://www.ctspedia.org/do/view/CTSpedia/StudyIncidencePersonTime
http://www.cdc.gov/ophss/csels/dsepd/ss1978/lesson3/section2.html

42. best sentence describes case-control study:


A study that compares patients who have a disease or outcome of interest (cases) with patients who do not have the disease or
outcome (controls), and looks back retrospectively to compare how frequently the exposure to a risk factor is present in each group
to determine the relationship between the risk factor and the disease.
https://himmelfarb.gwu.edu/tutorials/studydesign101/casecontrols.html

43. Cohort study advantages: (missing options)


Answer: please see table below
Reference: http://www.ncbi.nlm.nih.gov/pmc/articles/PMC2998589/#!po=20.9677

63
Extra information

44. M
easu
re
of

Variability:
Answer:
- range: the difference between highest and lowest score (for
small data)
- variance: the degree of spread within distribution (stable
measure)
- standard deviation: measure how the average score
deviated away from the mean (most stable measure)

45. What's meant by stander deviation:


Answer:
- measure that is used to quantify the amount of variation or dispersion of a set of data values. A standard deviation close to 0
indicates that the data points tend to be very close to the mean (also called the expected value) of the set, while a high standard
deviation indicates that the data points are spread out over a wider range of values.
- positive square root of variance

64
46. Attributive risk
Attributable risk is the difference in rate of a condition between an exposed population and an unexposed population. Attributable
risk is mostly calculated in cohort studies, where individuals are assembled on exposure status and followed over a period of time.
Source: https://en.wikipedia.org/wiki/Attributable_risk

47. study of 1000 with congenital heart ,20 have gestational diabetes and control group of 5000 no congenital, 80 have
gestational diabetes What is odd ratio and RR
Answer:
http://www.graphpad.com/guides/prism/6/statistics/index.htm?stat_interpreting_results_contingen.htm

48. Definition of evidence based medicine (biosocial …. )


Answer: practice according available scientific evidence
-is about trying to improve the quality of the information on which decisions are based. It helps practitioners to avoid ‘information
overload’ but, at the same time, to find and apply the most useful information.
-The practice of evidence based medicine means integrating individual clinical expertise with the best available external clinical
evidence from systematic research.
Link: http://med.fsu.edu/index.cfm?page=medicalinformatics.ebmTutorial

49. Cohort study MCQ


A study design where one or more samples (called cohorts) are followed prospectively and subsequent status evaluations with
respect to a disease or outcome are conducted to determine which initial participants exposure characteristics (risk factors) are
associated with it. As the study is conducted, outcome from participants in each cohort is measured and relationships with specific
characteristics determined

50. Calculation of the incidence

51. Odds ratio calculation


The OR evaluates whether the odds of a certain event or outcome is the same for two groups. Specifically, the OR measures the ratio
of the odds that an event or result will occur to the odds of the event not happening. Clinically, that often means that the researcher
measures the ratio of the odds of a disease occurring or a death from a specific injury or illness happening to the odds of the disease
or death not occurring.
The odds ratio is used when one of two possible events or outcomes are measured, and there is a supposed causative factor.
OR = (a/b)/(c/d)
or: OR = (a × d)/(b × c)

52. Number needed to treat calculation


The NNT offers a measurement of the impact of a medicine or therapy by estimating the number of patients that need to be treated
in order to have an impact on one person.

The NNT is the inverse of the absolute risk reduction – the difference between the proportion or rate of events in the active
treatment intervention group (Pa) and the proportion of events in the control group (Pc):
65
Number needed to treat =1/(Pa-Pc)

53. Definition of Positive predictive value?


Answer: ppv: the probability that a pt with +ve test result has the disease.
Positive predictive value is the probability that subjects with a positive screening test truly have the disease. Negative predictive
value is the probability that subjects with a negative screening test truly don't have the disease.

54. best sentence describe case-control study:


Answer:
A study that compares patients who have a disease or outcome of interest with patients who do not have the disease or outcome,
and looks back retrospectively to compare how frequently the exposure to a risk factor is present in each group to determine the
relationship between the risk factor and the disease
One of the most significant triumphs of the case-control study was the demonstration of the link between tobacco smoking and lung
cancer
CROSS sectional: observational study that assesses risk factor and outcome at snapshot in time also called prevalence study

55. a study with some people exposed to MRSA patient and a control group who weren’t, calculate the risk (or) the ratio:
(accuracy of stem of Q not known, missing options, answer not known)
MRSA negative MRSA positive

Yes Contact 20 80 100

No contact 494 6 500

514 86 600

Answer: relative risk [a / (a + b)] / [ c / (c + d)] = [20 / (20 + 80) / 494 / (494 + 6)] = 0.809

56. Risk of infection among a population in a country in 2012: (not sure of numbers in stem, missing options, answer not known)
new cases 200, leave the country I think 12, died I think 20, total population 80,000,000
what is the risk in 100,000:
Answer: ?

57. patients with a specific disease in a thousand. 2.5 days are disability and 1.5 absent workers calculate the incidence of
disability annually: (missing options, answer not known)
Answer: ? Standard New
Treatment Treatment
58. definition of specificity (understand the definition very well
and read the Q with focus)
Event Happens a b
Answer: the quality or condition of being specific also called
true negative. Specificity is Event does not c d the probability that an individual
without the disease will test happen negative. It is the number of patients
who have a negative test and do not have the disease (true negatives) divided by the number of patients who do not have the

66
disease. A test with high specificity will infrequently identify patients as having a disease when they do not (ie, low false positive
results).
Reference: UpToDate

59. Positive predictive value means?


Answer: is the probability that subjects with a positive screening test truly have the disease

60. Attributable risk definition


Community lecture:
The incidence of disease in the exposed popula/on whose disease can be attributed to the exposure. AR = risk in exposed – risk in
non-exposed
•difference in rate of a condition between an exposed

61. Epidemiology, what is the annual prevalence rate?


Kaplan:
Prevalence rate: is the proportion of individuals with existing disease at a point in time (point prevalence). It is the
proportion of individuals with existing disease during a period of time (period prevalence)

62. What is the following of these is true ,specificity ,sensitivity ,Positive predictive value and Negative predictive value ? 60 940
1000 The true answer specificity is 940/1000 ( =94%) The other choices are wrong answer equation of sensitivity ,PPV ,NPV .

63. A question about the formula of relative risk And another to calculate the relative risk

64. patients with a specific disease in a thousand. 2.5 days are disability and 1.5 absent workers calculate the incidence of
disability annually

65. A question about the definition of specificity (it was written in a very complex english so understand the definition very well
and read the Q with focus)

66. Patient with high Ca and low Iga Igm what is the diagnosis (this Q was already submitted by one of our colleagues but the
answer in the (gathered smle) had multiple myeloma and an explanation to a subtype of it. In the exam they offer you both
multiple myeloma and it's subtype as choices and you have to choose between the two

67. What is the first step in epidemiology?


http://www.cdc.gov/ophss/csels/dsepd/ss1978/lesson6/section2.html

68. The definition of endemic or epidemic disease?


https://www.vocabulary.com/articles/chooseyourwords/endemic-epidemic/

69. incidence rate ?


67
70. case control study :

71. OR ( 0.2 OR 2 ) RR (0.2 or 2 )

72. definition of positive probability test?

73. definition of case report research ?

74. What's the causation of the study?


One of the options was the strength of the association

75. 1000-20, 5000-6, OR? 2.0


76. Incidence 117 of total 1000 patients. Mean of ... =2.5 days.
(Not sure what’s the question here)
- Incidence is 11.7%
- What is the prevalence?
- Prevalence is: incidence x duration of the disease = 11.7 x 2.5 = 29.25%
- NOT SURE

77. 2 groups disease (I think it was prostatic cancer) and non, exposed to smoking and non. (MISSING
INFORMATION)
A. OR = 5
B. RR =5
C. OR= .5
D. RR = .5
Answer: NOT SURE

78. Case control definition.


A study that compares patients who have a disease or outcome of interest (cases) with patients who do
not have the disease or outcome (controls), and looks back retrospectively to compare how frequently
the exposure to a risk factor is present in each group to determine the relationship between the risk
factor and the disease.

79. Positive predictive value definition?


Positive predictive value is the probability that subjects with a positive screening test truly have the
disease.

68
80. A study of pregnant women and their diet, one with a diet full of ... And the other group with a diet
without... then followed them up for any congenital disease in fetuses, is the study:
A) Cohort
B) Case control
Answer: A
Compares a group with a given exposure or risk factor to a group without such exposure. Looks
to see if exposure the likelihood of disease. (First Aid – Step 1)

81. Study we have 1000 DM 20 of them have congenital abnormal babies and 5000 control of non DM have 50
congenital abnormal babies... What detmermine acurcy of the study?
A. OD 2.0
B. RR 2.0
C. OD 0.2
D. RR 0.2

82. What most determine accuracy of a study..


A. Specifity
B. Sensitivity
C. Positive predictive value
D. Negative predictive value

83. Why we use standard deviation?


Answer: Measurement
is a measure that is used to quantify the amount of variation or dispersion of a set of data values (umm alqura)
https://en.m.wikipedia.org/wiki/Standard_deviation

84. You are reading a population study that states that 90% of lung cancer patient are smokers while 30% of
lung cancer patient are non-smokers. What is the specificity of using smoking as a predictor of lung cancer?
A. 30
B. 70

Answer: B (umm alqura)


Fundamentals of biostatistics:
https://books.google.com.sa/books?id=Iag8AAAAQBAJ&pg=PA52&lpg=PA52&dq=specificity+of
+smoking+as+a+predictor+for+lung+cancer&source=bl&ots=aUYw1B2DOG&sig=zFUWGkpr3vV

69
zwFtYTdlppxwMmQQ&hl=en&sa=X&redir_esc=y#v=onepage&q=specificity%20of%20smoking%
20as%20a%20predictor%20for%20lung%20cancer&f=true

85. Mortality rate?


Mortality rate, or death rate, is a measure of the number of deaths (in general, or due to a specific cause) in a
particular population, scaled to the size of that population, per unit of time. Mortality rate is typically expressed
in units of deaths per 1,000 individuals per year;
https://en.m.wikipedia.org/wiki/Mortality_rate
86. Cumulative ?60.1

87. A research about a disease, on 10000 population, in the beginning there were 2000 patients but after 3 years
of the study there were another 1000 patients, what is the incidence in one year? Answer: 10%
incidence = number of new cases of disease or injury during specified period divided by size of population
at start of period (1000 ÷ 10000)

88. Non response bias with? Answer: cross sectional

70
89. Research about HTN patients and fat consuming in diet, they found in that HTN patient there were 30 was
fat consumers, while in non HTN patients there were 10 Fat consumers. What is the effect ? The answer
was same numbers but: is it OR , RR, RD?

71
Ethics

72
1. If you successfully treated your patient. Then the patient brought to you an expensive watch. What would be your response?
A. Accept the gift and say thank you
B. Refuse the gift politely
C. ask him for more gifts.
D. Shout at him and say "I do not accept gifts"
Answer: B

*2. (long scenario) lady complaining about work partner. She is agitated and mad. What is your action? (answer not known)
1. Instruction to deal with him.
2. Empathy during session.
3. Avoid him.
Answer: ?

73
Family Medicine & Statistics (Incomplete Questions)

1. Normalization of the data

2. Exposed odds ratio of 3 and non-exposed odds ratio 2. Which of the


following is true?
A - Number needed to treat is ...
B - Number needed to harm is ...
C - Relative risk is ...

3. Case-control study, calculate the odds ratio:


Answer: ? [Odds ratio: (A / B) / (C / D) or (A * D) / (B * C)] | Reference:

4. attrubt risk factor"

5. You have an old pt he can't understand has answer you will take him to your clinic ask him open end question

*Notes about Questions:


there was question in statistic about:
o Odds Ratio
o relative risk
o association between 2 factor like 6:2
o The question was required numbers in answer
Read about:
o definition of Different types of research sampling
o odd ratio
o Relative risk
o Calculations:
Attributable risk
Relative risk

6. There were also missing questions about the following:


o Vaccination types
o Different types of research sampling
7. 2 statistical questions: one about sensitivity and specificity, & another one about mortality rate

8. pt diagnosed DM and complained on medication but he complained of hypoglycemia more than 3 times ..... Cause ,?
Honeymoon ,,

74
9. partly Diabetic patient on metformin has sulphonylurea allergy need to be on another extra drug for control which one you
choose. repeated
http://emedicine.medscape.com/article/117853-treatment#d9

10. 14years old boy presented with his father to the clinic for routine investigation his height was 174 (>90 centile ) and weight
was 45 ( > 25 centile ) everything was normal except that you found irregular pulse , vital : HR = 80 other was normal , ECG pic
was Attached. What is your management?
A) normal development
B)need a pacemaker
C) disfunction cardiomyopathy
D)syncopal

11. You have devised a new test to diagnose a disease and you want to check it Total Disease No disease
against a gold standard test. There were figures showing the number of those who
actually have the disease according to the gold standard test and the number of 100 80 20 Exposure
those who ,
500 6 494 No exposure

12. 17 y/o increase 20kg in 3 months screen for anabolic steroids 600 86 514

13. Pt took her anti osteoporosis drug then had a sever retrosternal pain. Don’t remember exact drugs, but belongs to
bisphosphonate.

14. Lipid lowering agent can cause sever muscle weakness

15. Pt on hypoglycemic agent with poor control blood sugar have sulpha drug allergy <<< which medication you give

16. investigation for bone density? DXA scan.

17. HTN with BPH what's the drug of HTN

18. Best description of case-control study.

19. Relative risk of exposed to MERS:


A. 33:1
B. 44:1
C. 55:1 (the right answer) 100% sure because I got 100%
D. 66:1

20. Gene in DM 2 <<<<

21. a drug to treat DM named incritin what's the MOA of it

22. Glipizid MOA

75
23. Research questions very difficult relative risk, Prevalence

24. MERS COV > 66:1

Extra information for statistical questions:

76
77
Extra information for vaccination questions:

78
Psychiatry

79
1. Clozapine is used in which disease in children
A - Bipolar
B - Depression
C - Substance abuse
D - Psychosis (Or schizophrenia)

Answer: D
Evidence:
Despite a higher incidence of adverse effects in children, clozapine appears to be a uniquely beneficial second-line agent for treating
children with refractory schizophrenia.
Reference: http://www.ncbi.nlm.nih.gov/pubmed/18220495

2. A patient presented with behavioural changes. His father died recently (3 days). He is walking around naked and saying that
his father asked him to do that then returns to his normal state. What is the most likely diagnosis?
A - Brief psychotic disorder
B - Schizophrenia
C - Schizophreniform
D - Schizoaffective

Answer: A
Evidence:

Brief psychotic disorder consists of delusions, hallucinations, or other psychotic


symptoms for at least 1 day but < 1 mo, with eventual return to normal
Brief psychotic disorder
premorbid functioning. It is typically caused by severe stress in susceptible
people.

One or more episodes of symptoms must last ≥ 6 mo before the diagnosis is


Schizophrenia
made.

Characterized by symptoms identical to those of schizophrenia but that last ≥ 1


Schizophreniform
mo but < 6 mo.

Schizoaffective disorder is characterized by significant mood symptoms,


Schizoaffective psychosis, and other symptoms of schizophrenia. It is differentiated from
schizophrenia by occurrence of ≥ 1 episodes of depressive or manic symptoms.
Reference: Merck Manual

3. Which of the following is a criteria of Somatization disorder?


A - intentionally symptoms
B - at least 2 GI symptoms must be present
C - more than 40 years’ old

80
D - loss of occupational / life function

Answer: B
Evidence: ≥8 physical symptoms that have no organic pathology including each of:
- four pain symptoms related to at least four different sites or functions
- two gastrointestinal symptoms, not including pain
- one sexual symptom, not including pain
- one pseudo-neurological symptom, not including pain (e.g. Numbness, paraesthesia)
- onset before age 30; extends over a period of years.
Reference: Toronto Notes

4. Most common affect symptom in premenstrual dysphoric disorder:


A. Irritability
B. Mood swings
C. Depression
D. Anxiety
Answer: B
Explanation: The most common behavioral symptom of is labile mood, occurring in more than 80 percent. Other frequent
behavioral complaints include irritability, tension, depressed mood, increased appetite (70 percent), and forgetfulness and difficulty
concentrating (more than 50 percent).
Reference:http://cursoenarm.net/UPTODATE/contents/mobipreview.htm?31/5/31838#H2

5. About male patient he covers the TV because he thinks that he gets instruction from the TV and the government is
watching him:
A - Mania
B - Depression
C - schizophrenia

Answer: C
Evidence: schizophrenia is characterized by hallucinations, delusions, disordered thoughts, behavioural disturbance and disturbed
social function. Reference: First Aid Step 2

6. 20s Female patient c/o Multiple GI symptoms such as Nausea, vomiting, diarrhoea, headache, fatigue, joint pain, urine
retention. The all lab result is normal. These symptoms for 2 years unchanged:
A - Somatisation disorder
B - Conversion disorder
C - GAD
D - Dissociative disorder

Answer: A
Evidence: Somatisation disorder:
The essential feature of a somatization disorder is several symptoms that not lead to any medical sense, with a pattern of many
physical complaints in persons younger than 30 years that occurs over several years and results in unnecessary medical treatment
and/or causes significant impairment in functioning.

81
All the following historical criteria are required for a diagnosis:
- Four different pain sites (e.g., head, abdomen, back, joints, extremities, chest, rectum) or painful functions (e.g., menstruation,
sexual intercourse, urination)
- Two gastrointestinal symptoms other than pain (e.g., nausea, bloating, vomiting, or intolerance of several different foods)
- One sexual or reproductive symptom other than pain (e.g., erectile or ejaculatory dysfunction, irregular menses, excessive
menstrual bleeding)
- One pseudoneurological symptom (e.g., impaired balance, paralysis, aphonia, urinary retention)
Reference: Medscape: http://emedicine.medscape.com/article/918628-overview#a2

7. Which one is known to cause restlessness, insomnia and drowsiness?


A - SSRI
B - MAOI
C - Tetracycline antidepressants
D - Tricycle antidepressants

Answer: A
Evidence: please see picture
Reference: Toronto notes

8. 72 y/o male disoriented and hallucinating and disorganized thinking had aortopopletial graft and symptom fluctuates in the
2 days, what is the cause?
A - multi infraction dementia
B - mania
C - dementia
D - delirium

Answer: D
Evidence: When taking the history of a patient with cognitive impairment, always clarify the
rate of onset and the course of the symptoms. Delirium is acute in onset, and usually short
term, while dementia has a very gradual onset and is a chronic disorder. Lewy body dementia and multi-infarct dementia are the
only dementias that feature transient episodes of impaired consciousness as a typical feature. Reference: Crash Course Psychiatry

9. An elderly male told you that he's always occupied by the idea that his backyard is invaded by aliens, although he knows that
aliens don't exist and no one is invading his back yard. He's afraid that he's "going insane". He has:
A - Delusions
B - Obsessions
C - Compulsions
D - Hallucinations

Answer: B
Evidence:
False beliefs based on incorrect inference about external reality that persist despite the evidence to the
Delusions
contrary and these beliefs are not ordinarily accepted by other members of the person's culture or subculture
Obsessions Involuntary thoughts, images or impulses

82
Repetitive mental operations (counting, praying or repeating a mantra silently) or physical acts (checking,
Compulsions
seeking reassurance, handwashing, strict rituals)
Hallucinations Perceptions occurring in the absence of an external physical stimulus
Reference: http://emedicine.medscape.com/article/292991-overview ; crash course

10. Long scenario when you read it you think it about psychiatry but I think it about premenstrual syndrome. What is the
treatment? (missing details of Q, answer not known)
A - SSRI
B - other psych medication
C - combined oral contraceptive
D - psych medication

Answer: A or C?
Evidence: Pharmacological therapies of PMS include: hormonal contraception (the combined pill, implants, patches), SSRIs, danazol
(a testosterone analogue) and gonadotrophin-releasing hormone analogues. No single treatment is universally effective, and studies
with all therapies have not produced consistent results. Current recommendations in the literature regarding oral contraceptive pills
are conflicting. Reference: Crash course+ http://emedicine.medscape.com/article/953696-medication

11. Patient talks about false belief & against culture (other version: false feeling not according to person's intelligence or culture),
Diagnosis?
A - illusion
B - delusion
C - schizoid (other version: Somatization)
D - hallucinations

Answer: B
Evidence: Delusions are false beliefs based on incorrect inference about external reality that persist despite the evidence to the
contrary and these beliefs are not ordinarily accepted by other members of the person's culture or subculture.
Reference: http://emedicine.medscape.com/article/292991-overview

12. Poor prognostic factor for schizophrenia:


A - family history
B - acute onset
C - Adolescence
D - mood included in his history

Answer: C
Evidence: The factors associated with a good prognosis are:
• Female sex; • Married; • Older age of onset; • Abruption onset of illness (as opposed to insidious onset); • Onset precipitated by
life stress; • Short duration of illness prior to treatment; • Good response to medication; • Paranoid subtype, as opposed to
hebephrenic subtype; • Absence of negative symptoms; • Illness characterized by prominent mood symptoms or family history of
mood disorders; • Good premorbid functioning. Reference: Crash Course Psychiatry

13. Middle age male presented in the ER with irritability, suspiciousness, overactive, and poor hygiene what is the diagnosis:
A - Schizophrenia

83
B - Bipolar Disorder
C - Psychoses
D - Acute anxiety state

Answer: C (We cannot judge by these symptoms ONLY its Schizophrenia or Bipolar disorder)

14. Patient came to you for check-up and tells you that he diagnosed two years ago with pancreatic cancer and asking you for
the medication but he lost the prescriptions, when you have looked in the system there wasn't any data about him. When
you face him with what you find, he runs away. What is the diagnosis?
A - Malingering.
B - Somatization.
C - Schizophrenia
D - Drug addict.

Answer: D (By exclusion)


Evidence:
- Somatization. Recurring, multiple, clinically significant physical complaints which result in patient seeking treatment or having
impaired functioning.
- Malingering: intentional production of false or grossly exaggerated physical or psychological symptoms, motivated by external
reward.
Reference: Toronto notes

15. Which one of the antipsychotic drugs is most likely to cause weight gain?
A - Ziprasidone.
B - Olanzapine.
C - Quetiapine.
D - Aripiprazole.

Answer: B (B, C, D causes weight gain but more common in is B) | Reference: Toronto notes

16. A man is behaving in a strange way after the death of his son. Random people on the road and ask them irrelevant questions.
He won’t listen when asked not to do that. He had no such behaviour before in life. Which of the following will be excluded
from your differentials?
A - Brief psychotic disorder
B - Schizophrenia
C - Schizophreniform
D - Schizoaffective

Answer: A
Evidence:

Brief psychotic disorder consists of delusions, hallucinations, or other psychotic symptoms


Brief psychotic
disorder
for at least 1 day but < 1 mo, with eventual return to normal premorbid functioning. It is
typically caused by severe stress in susceptible people.
84
Schizophrenia One or more episodes of symptoms must last ≥ 6 mo before the diagnosis is made.

Characterized by symptoms identical to those of schizophrenia but that last ≥ 1 mo but < 6
Schizophreniform
mo.

Schizoaffective disorder is characterized by significant mood symptoms, psychosis, and other


Schizoaffective symptoms of schizophrenia. It is differentiated from schizophrenia by occurrence of ≥ 1
episodes of depressive or manic symptoms.
Reference: Merck Manual

17. Which of the following is considered good prognostic factor for schizophrenia?
A - No identified cause
B - Family history
C - Insidious onset
D - Affective symptoms

Answer: D
Evidence: Good Prognostic Factors:
Acute onset, later age at onset, shorter duration of prodrome, female gender, good cognitive functioning, good premorbid
functioning, no family history, presence of affective symptoms, absence of structural brain abnormalities, good response to drugs,
good support system. Reference: Toronto Notes

18. A guy who is showing an erratic behaviour lately. He is drinking a lot, spending too much money and having involved in
sexual activities a lot. What is your diagnosis?
A - drinking problem
B - mania
C - drug use
D - schizophrenia

Answer: B
Evidence: A manic episode is defined as ≥ 1 wk of a persistently elevated, expansive, or irritable mood and persistently increased
goal-directed activity or energy plus ≥ 3 additional symptoms:
Inflated self-esteem or grandiosity
Decreased need for sleep
Greater talkativeness than usual
Flight of ideas or racing of thoughts
Distractibility
Increased goal-directed activity
Excessive involvement in activities with high potential for painful consequences (e.g., buying sprees, foolish business
investments)
Manic patients may be inexhaustibly, excessively, and impulsively involved in various pleasurable, high-risk activities (e.g. , gambling,
dangerous sports, promiscuous sexual activity) without insight into possible harm.
Reference: http://www.merckmanuals.com/professional/psychiatric-disorders/mood-disorders/bipolar-disorders
85
19. An alcoholic comes to you with symptoms of alcohol withdrawal. Last drink he consumed was 2 days back. What drug will
you give?
A - Naloxone
B - Diazepam
C - Nicotine
D - Disulfiram

Answer: B
Evidence:
- diazepam 10-20 mg IV/PO or lorazepam 2-4 mg IV/PO q1hr until calm
- thiamine 100 mg IM/IV then 50-100 mg/d
- magnesium sulfate 4 g IV over 1-2 h (if hypomagnesemia)
- admit patients with delirium tremens (DT), or multiple seizures.
Reference: Toronto Notes.

20. A male patient presented with depression. What is the first drug to be given in depression?
A - TCA
B - lithium
C - CBT
D - SSRI

Answer: D
Evidence: Selective serotonin reuptake inhibitors, or SSRIs (e.g. Sertraline, paroxetine, citalopram, fluoxetine), are recommended by
NICE (2009) as first line antidepressants because they
Have fewest side-effects. Reference: Crash Course Psych

21. What is the best initial pharmacological agent for generalized anxiety disorder:
A -SNRI
B - oxicarbazpine,
C - lithium,
D - bupropion

Answer: A (SSRI first then SNRI) | Reference: http://www.adaa.org/finding-help/treatment/medication

22. Obsessive compulsive disorder drug mechanism of action:


A. Increase serotonin reabsorption
B. Decrease serotonin reabsorption
C. Increase activity of serotonin
D. Decrease activity of serotonin
Answer: B
Ssris are believed to increase the extracellular level of the neurotransmitter serotonin by limiting its reabsorption into
the presynaptic cell. Https://en.wikipedia.org/wiki/Selective_serotonin_reuptake_inhibitor

86
23. Which of the following is the least to cause tardive dyskinesia:
A. Clozapine,
B. Risperidone,
C. Haloperidol,
D. Chlorpromazine
Answer: A
Clozapine is used to treat sever tardive dyskinesia (Step 2 CK FA 2007 p395)

24. Secondary phenomena in panic attack not in presenting complain?


A. Phobia
B. Chest pain
C. Epigastric pain
D. Palpitation

Answer: A

25. Obsessive compulsive disorder drug mechanism of action:


A. Increase serotonin reabsorption
B. Decrease serotonin reabsorption
C. Increase activity of serotonin
D. Decrease activity of serotonin
Answer: b
Reference: http://emedicine.medscape.com/article/1934139-treatment#d7

26. Patient has diarrhea, abdominal pain, gait abnormality but the labs were normal. What is the most likely diagnosis?
A. Somatization disorder (Somatic Symptom Disorder)
B. Hypochondriasis
C. Conversion disorder
Answer: A
Deficient information to make a definite diagnosis.

27. Patient panics when he gives a presentation, but acts normally with his friends.
A. Agoraphobia
B. GAS
C. performance anxiety
Answer: C
This is a case of social anxiety in which the person fears being ridiculed by his peers.
Reference: http://www.merckmanuals.com/professional/pediatrics/mental-disorders-in-children-and-adolescents/social-anxiety-
disorder-in-children-and-adolescents

28. A case of depression for 4 month, loss of appetite, abdominal pain and diarrhea and constipation, lost 6 kg of his weight. He
claims it is duo to IBS. He underwent embolectomy before 6 month. What is the diagnosis?
A. Dysthymia
B. Low mood
87
C. Major depression
Answer: C?

29. pt come clinic the result of investigation is lung cancer the pt say with anxiety (it's a mistake) Which of the following explain
the reaction of pt:
A. denial
B. anger
C. barogainer

30. Girl with BMI = 16 says “I'm fat”: (no stem of Q)


A - Depression
B - Anorexia bulimia
C - Anorexia nervosa

Answer: C
Evidence: Anorexia nervosa is characterized by overvalued ideas concerning body shape and weight, preoccupation with being thin
and intrusive dread of fatness. Reference: Crash Course Psychiatry

31. Patient turns to be erratic for 4 months, he said that people on TV know what he’s thinking about and they are talking about
him. In the last 2 months he started to claim that he has special power that no one has. What is the most likely diagnosis?
A - Uni-polar...depression
B - Bipolar...Mania
C - Schizophrenia

Answer: Schizophreniform | Reference: 3rd Edition UQU > Psychiatry Q 30 + Toronto Note

32. Patient after death of his sibling, cries and can't sleep for two days, best short acting treatment;
A. Imitriptan
B. Fluxetine
C. Lorazepam
Answer: C
A and B takes 4 wks. To start their effect while C will have an intermediate effect, but it shouldn’t be used for

33. A depressed patient on medication, but he had weight gain and erectile dysfunction. What is the medication responsible for
these side effects?

88
A - Sertraline.
B - Venlafaxine
C - TCA.

Answer: A
Evidence: Sertraline is an antidepressant in a group of drugs called selective serotonin reuptake inhibitors (ssris) and most common
side effects is decreased sexual desire or ability, failure to discharge semen (in men) and weight gain.
Reference: https://www.drugs.com/sfx/sertraline-side-effects.html
34. What is an alternative medication for severe depression?
A - triptyline
B - psychotherapy
C - electroconvulsive therapy

Answer: A (TCA; if you find RCT in the answers then choose it) | Reference: Toronto Notes
35. Difference between delirium & dementia?
A - impaired memory
B - state of consciousness
C - confusion

Answer: B | Reference: Toronto Notes

89
36. A patient was brought by his family with 3 months’ history of delusions, hallucination, that are gone by themselves now. No
mood disorders. What is the diagnosis?
A - Brief Psychotic disorder
B - Schizophrenia
C - Schinziform disorder

Answer: C

Brief psychotic disorder consists of delusions, hallucinations, or other psychotic


Brief psychotic
disorder
symptoms for at least 1 day but < 1 mo, with eventual return to normal premorbid
functioning. It is typically caused by severe stress in susceptible people.

Schizophrenia One or more episodes of symptoms must last ≥ 6 mo before the diagnosis is made.

Characterized by symptoms identical to those of schizophrenia but that last ≥ 1 mo but


Schizophreniform
< 6 mo.

Schizoaffective disorder is characterized by significant mood symptoms, psychosis,


Schizoaffective and other symptoms of schizophrenia. It is differentiated from schizophrenia by
occurrence of ≥ 1 episodes of depressive or manic symptoms.

37. Cardiac patient with depression on regular medications, started to have convulsion then coma, He took overdose of some
pills which his relative does not know it. What is the cause of this seizure?
A - SSRI
B - Digoxin
C - quinine

Answer: C

38. Old patient after surgery he loss concentration and other symptoms and after 2 days’ symptoms start to resolve?
A - Multi infarct dementia
B - Hallucinations
C - Delirium

Answer: C
39. Mania? (missing details of Q, no stem of Q)
A - Bipolar
B - affective disorder
C - Schizophrenia

Answer: A [It depends on the symptoms. To be more specific it’s Bipolar type I which is include at least 1 episode of Manic episode
(DSM-5 Page 126). Affective disorder is including depression, bipolar disorder, and anxiety disorder]

90
40. Patient in clinic suddenly he said he looks to left and said he saw his mother and no one can saw her excepatient me his
mother died when he was child?
A - audible hallucination
B - visual hallucination
C- delirium

Answer: B {These are hallucinations of the visual modality. They occur most commonly in organic brain disturbances (delirium,
occipital lobe tumours, epilepsy, dementia) and in the context of psychoactive substance use [lysergic acid diethylamide (LSD),
mescaline, petrol/glue-sniffing, alcoholic hallucinosis]} | Reference: Crash Course

41. Treatment of dementia?


A - edrophonium
B - revistagmine
C - Neostigmine

Answer: B
Evidence:
- Neostigmine: is a cholinesterase inhibitor indicated in Symptomatic control of myasthenia gravis; antidote for nondepolarizing
neuromuscular blocking agents after surgery; prevention and treatment of postoperative distention and urinary retention
- Edrophonium: is indicated in myasthenia gravis and respiratory depression
- Rivastigmine: is a cholinesterase inhibitor. It works by increasing the amount of a certain substance (acetylcholine) in the brain,
which may help reduce symptoms of dementia in patients with Alzheimer disease.
Reference: http://www.rxlist.com/enlon-drug/indications-dosage.htm

42. A patient had weight gain, because she can’t taste the food or smell. Examination is normal, she was seen by neuro and
psychiatry with no diagnosis. What’s her diagnosis?
A - Meningioma
B - Aneurysm
C - Malingering

Answer: C
Evidence: Malingering as the intentional production of false or grossly exaggerated physical or psychological problems. Motivation
for malingering is usually external (e.g., avoiding military duty or work, obtaining financial compensation, evading criminal
prosecution, or obtaining drugs). Reference: http://emedicine.medscape.com/article/293206-overview

43. Patient with irritably, has delusion and auditory hallucination, now he has flight of ideas: (no stem of Q)
A - neurosis
B - psychosis
C - dissociative disorder

Answer: B
Evidence: Psychosis refers to a mental state in which reality is grossly distorted, resulting in symptoms such as delusions,
hallucinations and thought disorder. Reference: Crash course

91
44. Patient turns to be erratic for 4 months, he said that people on TV know what he's thinking about and they are talking about
him. In the last 2 months he started to claim that he has special power that no one has. What is the most likely diagnosis?
A - Uni-polar...depression
B - Bipolar...Mania
C - Schizophrenia

Answer: Schizophreniform disorder


Evidence: Schizophreniform disorder is characterized by the presence of the symptoms of schizophrenia, but it is distinguished from
that condition by its shorter duration, which is at least 1 month but less than 6 months.
Reference: http://emedicine.medscape.com/article/2008351-overview

45. Maintenance bipolar drug: (answer not sure of)


A - sodium valvex
B- lithium
C- olanzapine

Answer: B or C?
Evidence: If maintenance treatment is indicated, NICE (2006) recommends lithium, valproate or olanzapine. The choice depends on
sex, physical comorbidity and patient preference. Reference: Crash Course Psychiatry

46. Patient was constantly seeking physicians believing that he has oesophageal carcinoma, all of his workup were negative and
all doctors reassured him (other version: young patient came to you wanting full investigations to roll out cancer. He is very
scared of cancer that it can destroy his job and future. No family hx of cancer) what is his psychiatric condition?
A - Conversion disorder
B - somatization disorder
C - hypochondriasis

Answer: C
Evidence: Diagnostic criteria for hypochondriasis include the following (DSM-IV-TR):
The patient has a preoccupying fear of having a serious disease.
The preoccupation persists despite appropriate medical evaluation and reassurance.
The belief is not of delusional intensity (as in delusional disorder, somatic type) and is not restricted to a concern about
appearance (as in persons with BDD).
The preoccupation causes clinically significant distress or impairment.
The preoccupation lasts for at least 6 months.
The preoccupation is not explained better by another mood, anxiety, or somatoform disorder.

On the other hand, somatization is characterized by early onset (< 30 y) and recurrent, multiple, physical complaints that result in
medical attention or significant impairment. Somatization disorder is best thought of as preoccupation with numerous and/or
sequential physical symptoms, not a preoccupation with a particular medical illness.
Reference: Crash Course Psychiatry + http://emedicine.medscape.com/article/290955-differential

Extra information about hypochondriasis:


92
*80. Patient have been having depression for the last 4 months with decreased energy concentration and have lost a considerable
amount of Kg in the last few weeks. He was diagnosed recently with IBS (he also retired 2 months before his depression) he looks
tired and mildly jaundiced what is the most likely diagnosis: (answer not sure of)
A - Major depressive disorder
B - Dysthymia
C - Depression due to general medical illness

Answer: C?

47. A lady had progressive sadness over the past 2 years. She has hopelessness, insomnia, decreased appetite, low self-esteem
and suicidal ideation. Diagnosis?
A. Minor depression
B. Depressive disorder
C. Dysthymia
Answer : C

93
Http://www.webmd.com/depression/guide/chronic-depression-dysthymia

48. Patient came with 3 hours hx of anxiety , diaphoretic , tachypneic , what is the cause ?
A- Sympathomimetic
B- organophostate
C- anticholinergics
Answer A

49. Depression patient came to psychiatrist the doctor asked him about his mentality what did the doctor examined?
A. Insight
B. Compliance
C. Depression

Answer A
Insight can be defined not only in terms of people's understanding of their illness, but also in terms of understanding how the illness
affects individuals' interactions with the world.
(http://bjp.rcpsych.org/content/160/6/850)

50. Patient afraid of diseases and germs, what diagnosis?


A. Obsessive compulsive
B. Anxiety disorder
C. Specific phobia

Answer: C
Http://www.apa.org/monitor/julaug05/fears.aspx

51. Patient with history of 2 years’ depression, decreased appetite, low self- esteem
A. Major depression
B. Depression something
C. Dysthymia
Answer: C
Source: Toronto notes 2014 PS11

52. A patient had weight gain, because she can’t taste the food or smell. Examination is normal, she was seen by neuro and
psychiatry with no diagnosis. What’s her diagnosis?
A. Meningioma
B. Aneurysm
C. Malingering
Answer: C
The most common goals of people who malinger in the emergency department are obtaining drugs and shelter. In the clinic or
office, the most common goal is financial compensation. And choices A&B can be diagnosed easily.
Http://emedicine.medscape.com/article/293206-clinical

53. Teacher has fear before start the class. She said it's normally major teacher mistake. Is it:

94
A. Agoraphobia
B. Specific
C. Social...
Answer: missing data

54. Someone afraid of disease, germ. Is it:


A. OCD
B. Specific phobia
C. Agoraphobia
Answer: B
It is fear of objects or situations, thee other possible is hypochondriasis but it is the fear of having certain disease despite normal
diagnostic tests.

55. Scenario about patient after the death of his father went out and do things, then he returns normal but he is confused about
what happen to him in the previous days, diagnosis?
A. Schizophrenia
B. Schizoaffective
C. Brief Psychotic Disorder.

Answer: C
Kaplan:

56. Patient loss interest in life .... -.... & suicidal idea for duration of 2 months, diagnosis?
A. Dysthymia
B. Major depression
C. Minor depression

Answer: B

95
First Aid:
Diagnosis requires depressed mood or anhedonia, and 5 of more from the following:
- Sleep (hypersomnia or insomnia).
- Loss of interest
- Guilt.
- Decreased energy or fatigued.
- Decreased concentration
- Weight and appetite (either increase or decrease)
- Psychomotor agitation
- Suicidal ideation

57. Patient think that he has cancer and did test to confirm he has no cancer and patient said it is significantly affect his life:
A. Factitious disorder
B. OCD
C. Hypochondriasis
Answer: C

58. Postpartum lady experienced low mood and tearing for one week only then she returned to her normal self?
A. Postpartum blues
B. Postpartum depression
C. Postpartum psychosis

Answer: A
Postpartum blues: first weeks of delivery, mood swings & tearfulness, care for self & baby present
Postpartum depression: first month of delivery, despair & hopelessness, care for self & baby neglected
Postpartum psychosis: first weeks of delivery, hallucination & bizarre behavior

59. Young patient with eating disorder. How you will manage:
A. Cognitive psychotherapy
B. Antidepressant
C. Interpersonal ….

Answer: A
Cognitive behavioral therapy (CBT) is considered optimal primary treatment for bulimia but may not be available. Selective
serotonin-reuptake inhibitors (ssris) or serotonin-norepinephrine reuptake inhibitors (snris) may also be used adjunctively to
CBT, or as an alternative when CBT is not available. Interpersonal psychotherapy (IPT) and dialectical behavioral therapy (DBT)
can be used as primary therapy, although less evidence supports their effectiveness.
Http://bestpractice.bmj.com/best-practice/monograph/441/treatment/step-by-step.html

60. Patient have been having depression for the last 4 months with decreased energy concentration and have lost a considerable
amount of Kg in the last few weeks. He was diagnosed recently with IBS (he also retired 2 months before his depression) he
looks tired and mildly jaundiced what is the most likely diagnosis:

96
A. Major depression disorder
B. Dysthymia
C. Depression due to general medical illness

Answer: C

61. Parents admitted to your clinic with their adolescent son, they are seeking help of bad behaviors of their son, during
assessment of adolescent by (H.E.A.D.S.S) Home Environment E Education Employment Activities Drugs Sexuality
Suicide/Depression what is the best one of communication to the adolescent?
A. Group therapy
B. Individual therapy*
C. Communicate to the adolescent with presence of his parents
Answer:
Http://www.bcchildrens.ca/Youth-Health-Clinic-site/Documents/headss20assessment20guide1.pdf

62. Patient speaks rapidly, before he demonstrates his idea he goes to talk about other idea.
A. Loose of association
B. Preservation
C. Flight if idea

Answer: A
• Flight of idea: a rapid shifting of ideas with only superficial associative connections between them that is expressed as a
disconnected rambling from subject to subject and occurs especially in the manic phase of bipolar disorder

Flight of idea: a form of formal thought disorder marked by abruption leaps from one topic to another, albeit with discernable links
between successive ideas, perhaps governed by similarities between subjects or, in somewhat higher grades, by rhyming, puns, and
word plays (clang associations), or innocuous environmental stimuli – e.g., the sound of birds chirping.
•loss of association: Failure to follow a train of thought to a natural conclusion. E.g. "Why does my computer keep crashing?", "Well,
you live in a stucco house, so the pair of scissors needs to be in another drawer."
Http://www.merriam-webster.com/medical/flight%20of%20ideas

63. Patient with history of chronic schizophrenia, but he is not compliant. What will you give him?
A- IV Lorazepam + IV haloperidol
B- Injection (something) of haloperidol
C- Oral haloperidol
Answer: B
Explanation: Long-acting injectable antipsychotics may be particularly useful for patients with schizophrenia who experience
frequent relapses due to non-adherence to antipsychotic medications.
Reference http://cursoenarm.net/UPTODATE/contents/mobipreview.htm?4/9/4241#H16350541

64. Patient speaks rapidly , before he demonstrate his idea he goes to talk about other idea

97
A. Loose of association
B. Preservation
C. Flight of idea

Flight of idea: a form of formal thought disorder marked by abruption leaps from one topic to another, albeit with discernable links
between successive ideas, perhaps governed by similarities between subjects or, in somewhat higher grades, by rhyming, puns, and
word plays (clang associations), or innocuous environmental stimuli – e.g., the sound of birds chirping. Loss of association: Failure to
follow a train of thought to a natural conclusion. E.g. "Why does my computer keep crashing?", "Well, you live in a stucco house, so
the pair of scissors needs to be in another drawer."

65. Teacher has fear before start the class. She said it's normally major teacher mistake . Is it:
A. Agoraphobia
B. Specific
C. Social ..
Answer: C
Reference : http://www.helpguide.org/articles/anxiety/phobias-and-fears.htm
Http://socialphobia.org/social-anxiety-disorder-definition-symptoms-treatment-therapy-medications-insight-prognosis

66. Someone afraid of disease , germ . Is it :


A- Specific phobia
B- Argo ..
C- OCD
Answer: A
Specific phobia, formerly called a simple phobia, is a lasting and unreasonable fear caused by the presence or thought of a
specific object or situation that usually poses little or no actual danger.
Reference: http://www.webmd.com/anxiety-panic/specific-phobias

67. A man walk in the street and and asks people and repeat even when he was asked to stop
A. Preservation
B. Flight of idea
C. Poor insight
Answer: A

Preservation is known as a sentence or phrase in repeated several times after it is no longer relevant.
Flight of idea is known as Thoughts rapidly jump from one topic to another
Insight is known as is the degree to which a person recognizes that he or she is unwell
Reference: Kumar and Clark’s Clinical Medicine + http://dysphrenia.hpage.co.in/disorder_of_thought_71402323.html

68. Patient complain of the following symptoms: impotence, inability to sleep, tachycardia and can't remember but seemed
anxious. What is the most likely diagnosis?

98
A. Generalized Anxiety disorder (GAD)
B. Depression
Answer: A
Explanation: For a patient to be labeled as depression patient he must have anhedonia in his symptoms as it is a major criteria for
diagnosis. GAD patients experience excessive worry out of proportion to event with fatigue, inability to concentrate & sleep
problems
Reference: Master the board step 2 CK

69. Elderly with psychosis, best treatment is:


A. haloperidol
B. Olanzepine
Answer: B
Answer: http://www.medscape.com/viewarticle/564899_4

70. Patient came with his wife. she said that he walks around the house 8 times to make sure its locked and wash his hand
many times before eating. The patient said this not affecting his life. Past history: he likes to stay alone all the time. What
is the diagnosis?
A. OCD
B. Generalized anxiety disorder

Answer: A
This may be a case of obsessive compulsive personality disorder because the patients seems to not be concerned with his condition.
https://www.verywell.com/ocd-vs-obsessive-compulsive-personality-disorder-2510584

71. Why SSRI is the drug of choice for depression? (missing options)
A - More tolerable and less side effects
B - Cheap

Answer: A
Evidence: Selective serotonin reuptake inhibitors, or ssris (e.g. Sertraline, paroxetine, citalopram, fluoxetine), are recommended by
NICE (2009) as first line antidepressants because they have fewest side-effects. Reference: Crash Course Psych
72. What medication that cause hypertensive crisis with Tyramine
A. MAOI
B. TCA

Answer: A
73. 40-50 Female presented with depression and she was given amitriptyline 30 mg (TCA). Later she came complaining of
dizziness. (other version: A patient with depression on TCA came with dizziness.) What will you do? (missing options)
A - Decrease dose to 10 mg
B - Switch to SSRI

Answer: B

99
74. Old man, lives in a nursery home, missed his wife recently. Also he had heart failure and osteoarthritis recently which makes
him more depressed. He is sad and having low mood. He doesn’t eat that lead to loss of 4 kgs of his weight in the last month.
Sometimes he forgets. What is the most likely diagnosis? (missing options)
A - Alzheimer’s disease
B - Depression

Answer: B
Evidence: Among the criteria for a major depressive disorder, at least 5 of the following symptoms have to have been present during
the same 2-week period (and at least 1 of the symptoms must be diminished interest/pleasure or depressed mood):
● Depressed mood: For children and adolescents, this can also be an irritable mood
● Diminished interest or loss of pleasure in almost all activities (anhedonia)
● Significant weight change or appetite disturbance: For children, this can be failure to achieve expected weight gain
● Sleep disturbance (insomnia or hypersomnia)
● Psychomotor agitation or retardation
● Fatigue or loss of energy
● Feelings of worthlessness
● Diminished ability to think or concentrate; indecisiveness
● Recurrent thoughts of death, recurrent suicidal ideation without a specific plan, or a suicide attempatient or specific plan for
committing suicide
Reference: http://emedicine.medscape.com/article/286759-overview

75. A woman is afraid that there is a snake in her child room. Last night she checked his room and opened his door like 50 times
and she is not relieved by that? (missing options)
A - Psychosis
B - Obsession

Answer: B
Evidence:
- Obsessions are involuntary thoughts, images or impulses.
- Compulsions are repetitive mental operations (counting, praying or repeating a mantra silently) or physical acts (checking, seeking
reassurance, handwashing, strict rituals).
Reference: Crash Course

76. A guy who is heroin addict. You want to start rehabilitation. What drug will you prescribe?
A - Diazepam
B - Methadone

Answer: B (missing options)


Evidence:
- Withdrawal: long-acting oral opioids (methadone, buprenorphine), α-adrenergic agonists (clonidine)
- Treatment of Chronic Abuse:

100
= psychosocial treatment (e.g. Narcotics Anonymous) usually emphasize total abstinence
= naltrexone or naloxone (opioid antagonists) to extinguish drug-seeking behaviour
= long-term treatment may include withdrawal maintenance treatment with methadone or buprenorphine
Reference: Toronto Notes.

77. Old man is walking on the street and asking random people the same question. When asked to stop he doesn’t stop. What
does he have? (missing options)
A - Loose of association
B - Delusion of control

Answer: Preservation
Evidence:
- Preservation is known as a sentence or phrase in repeated several times after it is no longer relevant.
- Flight of idea is known as Thoughts rapidly jump from one topic to another.
- Insight is known as is the degree to which a person recognizes that he or she is unwell.
Reference: Kumar and Clark’s Clinical Medicine + http://dysphrenia.hpage.co.in/disorder_of_thought_71402323.html

78. The doctor asked the patient: "do you think you are mentally ill?" The question is evaluating: (missing options)
A - Insight
B - Judgment

Answer: A
Evidence:
- Judgement is an assessment of real life problem-solving skills
= Testing: What should you do if you find a stamped (not cancelled) and addressed envelope?
- Insight is an understanding of their condition (including abnormal thoughts)
= Testing: Tell me about your illness and if it is worse now? What medications are you taking?
Reference: http://www.fpnotebook.com/neuro/Exam/insghtandjdgmnt.htm

79. Patient has loss of interest, suicidal attempt? (no stem of Q, missing options, answer not known)
A - Major depression
B - Minor depression (dysthymia)

Answer: ? (Although dysthymia was traditionally considered less severe than major depression, the consequences of dysthymia are
increasingly recognized as grave; they include severe functional impairment, increased morbidity from physical disease, and
increased risk of suicide) | Reference: http://emedicine.medscape.com/article/290686-overview

80. Man feeling depressed for 3 months, he is fighting with 2 of employee in job, 4 months ago he became the manager in his
job:
A - Depression

101
B - adjustment

Answer: B (no stem of Q, missing options)


Evidence: Adjustment disorder is a stress-related, short-term, nonpsychotic disturbance. The discomfort, distress, turmoil, and
anguish to the patient are significant. Feeling unable to cope is common at times of psychosocial stresses to which we need to
adaptation or adjust. For this diagnosis to be made, the emotional and/or behavioural symptoms need to occur within 3 months’
symptoms usually fully resolve within 6 months of onset, and if this is not considering another diagnosis. Reference: Crash Course

81. A patient presented with history of panic attack of job meeting and she became anxious. What is the most likely diagnosis?
A - specific anxiety disorder
B - panic attack

Answer: ? (missing options, answer not known)


Evidence: panic attacks must be associated with longer than 1 month of subsequent persistent worry about:
(1) having another attack or consequences of the attack, or
(2) significant maladaptive behavioural changes related to the attack.
To make the diagnosis of panic disorder, panic attacks cannot directly or physiologically result from substance use (intoxication or
withdrawal), medical conditions, or another psychiatric disorder. Other symptoms or signs may include headache, cold hands,
diarrhoea, insomnia, fatigue, intrusive thoughts, and ruminations.
Reference: http://emedicine.medscape.com/article/287913-overview

82. Woman feels abandoned by everyone and been hospitalized for trying to commit suicide many times. What is the type of her
personality disorder?
A - Borderline
B - Obsessive

Answer: A Evidence: Borderline personality disorder (BPD) is characterized by marked instability in functioning, affect, mood,
interpersonal relationships. Approximately 70-75% of patients with BPD have a history of at least one deliberate act of self-harm,
and the mean estimated rate of completed suicides is 9%. Reference: http://emedicine.medscape.com/article/913575-overview

83. Patient with fear of automobiles?


A- specific phobia.
B- GAD.
Answer A- specific phobia.

84. Female who experienced palpitation, sweating while she is in a meeting, she became anxious and refuses to attend any
meeting after that, what is the diagnosis?
A- specific phobia.
B- GAD.
Answer : B- GAD.

85. Patient continues to do something, if he tries to stop he become irritable.

102
A. Obsession
B. Compulsion
Answer: C
•obsessions are defined by unwanted repetitive ideas or impulses frequently well up in the mind of the person with OCD
• Compulsions are defined by in response to their obsessions, most people with OCD resort to repetitive behaviors called
compulsions. Https://www.psychologytoday.com/conditions/obsessive-compulsive-disorder

86. Potential side effect of amitriptyline:


A. Dystonia
B. Weight gain
Answer: B
Dizzy, Drowsiness, Dry Mouth, Headache, Increased Hunger, Taste Problem, Weight Gain http://www.webmd.com/drugs/2/drug-
8611/amitriptyline-oral/details/list-sideeffects

87. A patient presented with history of panic attack of job meeting and she became anxious. What is the most likely diagnosis?
A. Specific anxiety disorder
B. Panic attack
Answer: A

88. Euphoria, tachycardia, hallucination:


A. Amphetamine toxicity
B. Cannabis toxicity
Answer: A
A causes all the prementioned symptoms, But B only causes euphoria out of the prementioned symptoms.
Https://www.google.com.sa/url?Sa=i&rct=j&q=&esrc=s&source=images&cd=&cad=rja&uact=8&ved=&url=https%3A%2F%2Fquizlet.
com%2F112423974%2Fpsychiatry-rcsi-flash-cards%2F&psig=afqjcngutnauofkm0bk-upl0vd61yctckw&ust=1465916477516777

89. A mother noticed her daughter pulling her hair while studying or with stress and anxiety ( I think the diagnosis is
trichotillomania), what is the Rx:
A. Lithium
B. I don't remember the rest

Medscape: In children, ssris (e.g., fluoxetine, sertraline, and fluvoxamine) may be more advantageous as a medication choice than
tricyclic antidepressants (tcas) because of their milder adverse effects.

90. Premature ejaculation 26 year married look thin and sad;


A. SSRI
B. Injection in penis
Answer: A

91. What is the best drug for GAD (generalized anxiety disorder)?

103
A. SSRI
b. Benzodiazepine

Answer: A
Http://bestpractice.bmj.com/best-practice/monograph/120/treatment/details.html

92. 25 y/o c/o bad hygiene and he thinks that TV talks with him, what is your diagnosis?
A- schizophrenia
B- mania attack of bipolar

Answer: A
Explanation: The hallmark of schizophrenia is psychosis, such as auditory hallucinations (voices) and delusions (fixed false beliefs).
DSM-5: The presence of 2 (or more) of the following, each present for a significant portion of time during a 1-month period (or less if
successfully treated), with at least 1 of them being (1), (2), or (3): (1) delusions, (2) hallucinations, (3) disorganized speech, (4) grossly
disorganized or catatonic behavior, and (5) negative symptoms
Reference: http://emedicine.medscape.com/article/288259-overview#a2

93. Which of these drugs cause weight gain?


A- risperidone
B- olanzapine

Answer: B
Explanation: olanzapine is associated with weight gain with increased risk of DM and hyperlipidemia. Another antipsychotic that is
associated with weight gain is Aripiprazole.
Reference: Toronto Notes 2015, Page PS37, psychiatry

94. Patient on antidepressants, how long it will take to exert its effect?
A- 3-4 weeks
B- 4-6weeks

Answer:A or B
Explanation: Therapeutic response with most classes of antidepressants usually occurs in about 2 to 3 wk (sometimes as early as 4
days or as late as 8 wk).
Reference: https://www.msdmanuals.com/professional/psychiatric-disorders/mood-disorders/depressive-disorders
Antidepressants relieve emotional/cognitive symptoms in 2-6 wk (Toronto Notes)

95. Patient continues to do something, if he try to stop he become irritable. (Psychiatry)


A. Obsession
B. Compulsion
Compulsions as defined by (1) and (2)
(1) repetitive behaviors or mental acts that the person feels driven to perform in response to an obsession, or according to rules that
must be applied rigidly

104
(2) the behaviours or mental acts are aimed at preventing or reducing distress or preventing some dreaded event or situation;
however, these behaviours or mental acts either are not connected in a realistic way with what they are designed to neutralize or
prevent or are clearly excessive. (Toronto notes)

96. Treatment of Somatization?


A. Antidepressants
B. Frequent phone calls
Answer: during regularly scheduled brief monthly visits. Individual psychotherapy is needed.
Reference: Kaplan Psychiatry, Epidemiology and Ethics. 2013

97. Euphoria, tachycardia, hallucination :


A-Amphetamine intoxicate
B-Cannabis toxicity << my choice –Schizophrenia
Answer: b
Reference: http://patient.info/doctor/cannabis-use-and-abuse

98. A female complains that she’s not happy about her weight and she want to lose weight. some time she has urge to eat so she
eat excessive amount then vomit In exam she’s fatigue what’s the diagnosis :
A. Bulimia nervosa
B. Anorexia nervosa

Answer: A
Bulimia nervosa characterized by frequent binge-eating and purging and a self-image that is unduly influenced by weight Anorexia
nervosa characterized by failure to maintain a normal body weight, fear and preoccupation with gaining weight, unrealistic self-
evaluation as overweight,
Reference: Kaplan USMLE step 2 Psychiatry.

99. Obese boy is teased by his friends tells you that he wants to take pills and never wake up. What's your next step?
A - Immediate psychiatry referral (missing options)

Answer: A

100.What is the treatment of generalized anxiety disorder? (missing options)


A - Escitalopram

Answer: A
- lifestyle: caffeine and etoh avoidance, sleep hygiene
- psychological: CBT including relaxation techniques, mindfulness
- biological:
= ssris and snris are 1st line (paroxetine, escitalopram, sertraline, venlfaxine XL)
= 2nd line: bupropion (caution due to stimulating effects), buspirone (tid dosing)
= add-on benzodiazepines (short term, low dose, regular schedule, long half-life, avoid prn)
Reference: Toronto Notes

105
101.Elderly patient his wife died recently. He’s been depressed and having difficulty sleeping since that time. Which of the
following is a short term medication that is suitable for him? (missing options, answer not known)
A - Diazepam

Answer: ?
Evidence: Long acting benzodiazepines, with active metabolites should usually be avoided in the elderly e.g. Diazepam,
chlordiazepoxide, flurazepam, nitrazepam. When benzodiazepines are prescribed in older people, short acting benzodiazepines with
few active metabolites are preferable e.g. Lorazepam 0.5mg bd/tid, temazepam 10-20mg nocte, alprazolam 0.25mg bd/tid.
Reference: http://www.svhf.ie/documents/benzogoodpracticeguide.pdf

102.A patient presented with history of washing his hand a lot. What is the most likely diagnosis? (missing options)
A - Obsessive compulsive disorder

Answer: A
Evidence: Obsessions are involuntary thoughts, images or impulses. Compulsions are repetitive mental operations (counting, praying
or repeating a mantra silently) or physical acts (checking, seeking reassurance, handwashing, strict rituals).
Reference: Crash Course Psychiatry

106
103.What is the best factor for smoking cessation? (missing options)
A - The patient desire.

Answer: A [A smoker's motivation (drive, intention, desire) to stop smoking is a critical factor in whether or not they’re likely to quit]
Reference: http://bmb.oxfordjournals.org/content/52/1/157.full.pdf

104.A patient moved to a new school. What is the most likely diagnosis? (missing details of Q, missing options)
A - Adjustment syndrome

Answer: A
Evidence: Adjustment disorder is feeling unable to cope is common at times of psychosocial stresses to which we need to adapatient
or adjust (such as moving house, changing job or becoming a parent). Reference: Crash Course

105.A female patient says her new-born is evil and won’t live long. What is your diagnosis? (missing options)
A - Postpartum psychosis

107
Answer: A
Evidence: Postpartum psychosis (or puerperal psychosis) It is a severe episode of mental illness which begins suddenly in the days or
weeks after having a baby. Symptoms vary and can change rapidly. They can include high mood (mania), depression, confusion,
hallucinations and delusions. Postpartum psychosis is a psychiatric emergency, the patient needs hospitalization.

106.Male patient concerned about his sexual performance. During intercourse he develops palpitations & diaphoresis. Physical
exam: normal. Lab: within normal range. What is the diagnosis? (missing options)

Answer: PANIC
Evidence: Criteria for Panic Disorder (≥4): STUDENTS FEAR + the 3 Cs:
Sweating, Trembling, Unsteadiness, Dizziness, Depersonalization, Derealisation, Excessive heart rate, palpitations, Nausea, Tingling,
Shortness of breath, Fear of dying, losing control, going crazy, 3 Cs: Chest pain, Chills, Choking. Reference: Toronto Notes

107.Old patient with Alzheimer's dementia, became agitated and have hallucinations and delusions. What is the appropriate
drug in his case? (missing options)
A - Haloperidol

Answer: A
Evidence: Haloperidol: is a typical antipsychotic medication. It is used in the treatment of schizophrenia, tics in Tourette syndrome,
nausea and vomiting, delirium, agitation, acute psychosis, and hallucinations. Reference: Toronto notes

108.Patient think neighbours want to kill him. He said he can listen to them talking about him despite they are not around?

Answer: Delusion (missing options)


Evidence: Delusions are false beliefs of incorrect inference about external reality that persist despite the evidence to the contrary
Reference: http://emedicine.medscape.com/article/292991-overview

109.Panic disorders treatment: (missing options)

Answer: First-line pharmacologic therapies for panic disorder include selective serotonin reuptake inhibitors (ssris), serotonin–
norepinephrine reuptake inhibitors (snris), and tricyclic antidepressants (tcas). These 3 classes of agents are preferred over
benzodiazepines | Reference: http://emedicine.medscape.com/article/287913-medication

110.Diazepam used to treat which psychotics disorder in children: (no stem of Q, missing options)
Answer: Diazepam is frequently used as an anti-seizure medication, and it has been approved for anxiety in children as young as 6
months. As a class, benzodiazepines are most commonly used to treat anxiety disorders in child psychiatry. They are sometimes
used for adjunct treatment of bipolar mania and to treat catatonia. Overall, however, benzodiazepines are not as commonly used in
children as adults due to concerns about potential dependence. Also, benzodiazepines may be more likely to cause paradoxical
disinhibition in children compared to adults, especially in children with autism spectrum disorders and developmental delay.
Reference: http://whatmeds.stanford.edu/medications/diazepam.html
108
111.Patient see in cars and people in his plate food. What is the diagnosis ? (missing options)

Answer: visual hallucinations [(if there is no food in his plate); If there's food then it's an illusion!] | Reference: Crash Course Psych

112.A 23 YO patient who sees her nose very big, but all doctors see it’s normal, already done two plastic surgeries before for lips
& eyelids. (other version: Patient says she had a protruded jaw & she wants a cosmetic surgery, when you examine her jaw is
normal with no protrusion. You say she doesn't need the surgery She went already to 2 doctors who also refused to do the
surgery. She is done blepharoplasty & other cosmetic surgery before.) What is this condition called in psychiatry? (missing
options)

Answer: body dysmorphic disorder


Evidence: Body dysmorphic disorder (dysmorphophobia) is a variant of hypochondriacal disorder, in which patients are
Preoccupied with an imagined or minor defect in their physical appearance. Reference: Crash Course Psych
113.A patient cannot sleep, goes to work exhausted because she checks gas, door and kid’s stuff for school several times. What is
this condition called in psychiatry? (missing options)

Answer: OCD [Obsessions are involuntary thoughts, images or impulses. Compulsions are repetitive mental operations (counting,
praying or repeating a mantra silently) or physical acts (checking, seeking reassurance, handwashing, strict rituals)]
Reference: Crash Course Psych

114.Case of poor hygiene and hallucinating, what medication to give? (missing options)

Answer: antipsychotic
Evidence: Psychosis refers to a mental state in which reality is grossly distorted, resulting in symptoms such as delusions,
Hallucinations and thought disorder. Antipsychotic medications are Antipsychotics are of benefit in reducing positive symptoms
(e.g. Delusions and hallucinations). However, they have little or no benefit on negative symptoms (e.g. Apathy and social
withdrawal). Clozapine, which is the most effective antipsychotic known, but is not used first line because of its side-effects.
Antidepressants and lithium are sometimes used to augment antipsychotics in treatment-resistant cases. Reference: Crash Course

115. Psychiatric patient said TV talks to me. (no stem of Q, missing options)

Answer: delusion of reference

116.Best antidepressants drug in adults: (missing options)

Answer: fluoxetine (SSRI); SSRI is the best initial Rx in adult


Reference: http://www.uptodate.com/contents/depression-treatment-options-for-adults-beyond-the-basics

117.About lady scares of some places like streets and malls and she thinks that those places are dangerous:

Answer: Agoraphobia (no stem of Q, missing options)


Evidence: Agoraphobia literally means ‘fear of the marketplace’, i.e. Fear of public places. In psychiatry today, it has a wider

109
Meaning that also includes a fear of entering crowded spaces (shops, trains, buses, elevators) where an immediate escape is difficult
or in which help might not be available in the event of having a panic attack. Reference: Crash course

118.About lady has neurological symptoms, such as numbness, blindness, paralysis: (no stem of Q, missing options)

Answer: Conversion disorder


Evidence: Conversion disorder involves symptoms or deficits affecting voluntary motor or sensory function
DSM-5 are as follows:
One or more symptoms of altered voluntary motor or sensory function.
Physical findings provide evidence of incompatibility between the symptom and recognized neurological or medical conditions.
The symptom or deficit is not better explained by another medical or mental disorder.
The symptom or deficit causes clinically significant distress or impairment in social, occupational, or other important areas of
functioning or warrants medical evaluation.
Reference: http://emedicine.medscape.com/article/287464-clinical
119.#y/o came with depression what is your assessment: (missing detail of Q, missing options)

Answer: suicide | Reference: http://www.nimh.nih.gov/health/topics/depression/index.shtml

120.Eroded enamel & enlarged glands in a girl. What’s your diagnosis : (missing options)

Answer: bulimia Nervous


Evidence: Associated Features
• fatigue and muscle weakness due to repetitive vomiting and fluid/electrolyte imbalance
• tooth decay
• swollen appearance around angle of jaw and puffiness of eye sockets due to fluid retention
• reddened knuckles, Russell’s sign (knuckle callus from self-induced vomiting)
• trouble concentrating
• weight fluctuation over time
Reference: Toronto notes

121.What’s the approach to a 40 y/o lady with anxiety in work place because of conflicting with colleagues? (missing options)

Answer: Management of anxiety: Treatment usually consists of a combination of pharmacotherapy and/or


psychotherapy. Antidepressant agents are the drugs of choice in the treatment of anxiety disorders. Behavioural therapy and CBT
have demonstrated efficacy through controlled studies | Reference: http://emedicine.medscape.com/article/286227-treatment
122.Guy treated with haloperidol comes with generalized rigidity with up rolling of eyes. (missing options)
A - Acute dystonia

Answer: A (Acute dystonia)


Evidence: Dystonic reactions are reversible extrapyramidal effects that can occur after administration of a neuroleptic drug.
Symptoms may begin immediately or can be delayed hours to days. Although a wide variety of medications can elicit symptoms, the
typical antipsychotics are most often responsible. They reportedly arise from a drug-induced alteration of dopaminergic-cholinergic
balance in the nigrostriatum (i.e., basal ganglia). Reference: http://emedicine.medscape.com/article/814632-overview#a4

110
123.17 year old healthy guy who thinks TV is talking about him and he has super powers (other version: Old guy who thinks TV is
talking to him by God through the light): (no stem of Q, missing options)

Answer: delusion of reference & delusion of grandiosity

124.Withdrawal symptoms of smoking cessation start after: (missing options)

Answer: 3-5 days | reference: https://quitsmokingcommunity.org/how-to-quit-smoking/nicotine-withdrawal-timeline-symptoms/

125.A patient presented with history of repetitive closing of his garage around 10 times and moved around his house a lot. What
is the most likely diagnosis: (missing options)?
A - obsessive compulsive disorder

Answer: A (obsessive compulsive disorder)


Evidence: Obsessions are involuntary thoughts, images or impulses. Compulsions are repetitive mental operations (counting, praying
or repeating a mantra silently) or physical acts (checking, seeking reassurance, handwashing, strict rituals). Reference: Crash Course

126.Delusional disorder, what is it and what is its treatment: (missing options)

Answer: Delusional disorder is an illness characterized by at least 1 month of delusions but no other psychotic symptoms.
Treatment aims to establish an effective physician-patient relationship and to manage complications. Substantial lack of insight is a
challenge to treatment. If patients are assessed to be dangerous, hospitalization may be required.
References: http://emedicine.medscape.com/article/292991-overview#a6 +
Https://www.merckmanuals.com/professional/psychiatric-disorders/schizophrenia-and-related-disorders/delusional-disorder

127.Delirium treatment: (missing options)

Answer: primary illness should be mainly focused; nurse in well-lit room; make patient comfortable; less number of attendants; less
changes in staff; lower doses of Risperidone 1-2 mg in two or three divided doses; some sedative like zolpidem 10 mg at bed time
References: http://emedicine.medscape.com/article/288890-treatment

128.Definition of insomnia?

Answer: Insomnia describes sleep of insufficient quantity or poor quality. Primary insomnia is diagnosed when present for at least a
month, and not attributable to medical or psychiatric illness, substance misuse, or other dyssomnia or parasomnia.
Reference: Crash Course

129.Stress cause increase in catecholamine. What types of stress? (missing details of Q, missing options, answer not known)
A - Psych neural stress

Answer: ?

130.ADHD case, what is the treatment? (missing options)

111
Answer:
- Pharmacological management is indicated as first-line treatment in school-age children with severe ADHD.
The central nervous system (CNS) stimulant methylphenidate (Ritalin®, Concerta®, Equasym®).
- Psychosocial interventions are recommended in all cases, and are used first-line in children with mild to
Moderate ADHD, and in all preschool children

ADHD Symptoms:
1. Impaired attention includes: difficulty sustaining attention in work or play tasks; not listening when being spoken to; being highly
distractible – moving from one activity to another; reluctance to engage in activities that require a sustained mental effort (e.g.
Schoolwork); and being forgetful or regularly losing things.
2. Hyperactivity includes: restlessness; incessant fidgeting; running and jumping around in inappropriate situations; excessive
talkativeness or noisiness; and difficulty engaging in quiet activities. Impulsivity includes: difficulty awaiting turns; interrupting
others’ conversations or games; and prematurely blurting out answers to questions.

131. Alzheimer brain feature: (missing options)


Answer: Pathology involves neurofibrillary tangles, neuritic plaques with amyloid deposition, amyloid angiopathy, and neuronal loss.
MRI or CT may show atrophy | Reference: USMLE step 2 CK
Gross changes:
The cortex shrivels up, damaging areas involved in thinking, planning and remembering.
Shrinkage is especially severe in the hippocampus, an area of the cortex that plays a key role in formation of new memories.
Ventricles (fluid-filled spaces within the brain) grow larger.
Under the microscope:
Alzheimer's tissue has many fewer nerve cells and synapses than a healthy brain.
Plaques, abnormal clusters of protein fragments, build up between nerve cells.
Dead and dying nerve cells contain tangles, which are made up of twisted strands of another protein.
Reference: https://www.alz.org/braintour/early_stage.asp

132.Teenage girl had fight with her friend and now she didn't want to see or meet her, and if the friend come place the girl get
out of it?! What is the diagnosis: (missing options)
A - avoidance

Answer: A | Reference: http://emedicine.medscape.com/article/294

133.Patient with fear of motor vehicle, increasing in intensity, what is the diagnosis?
A. Specific phobia

134.Patient saying that she had a protruded jaw and she wants a cosmetic surgery, when you examine her her jaw is normal with
no protrusion and you say that she doesn't need the surgery She went already to 2 doctors who also refused to do the
surgery. She is done blepharoplasty & other cosmetic surgery before. What does she have?
A. Body dysmorphic disorder
112
Answer: Body dysmorphic disorder is preoccupation with an imagined defect in appearance or disproportionate concern with a
slight physical anomaly

135.A patient with multiple complains, CNS, GIT, what is the diagnosis?
Answer: somatization

113
136.Definition of insomnia?
Answer: According to guidelines from a physician group, insomnia is difficulty falling asleep or staying asleep, even when a person has
the chance to do so. People with insomnia can feel dissatisfied with their sleep and usually experience one or more of the following:
fatigue, low energy, difficulty concentrating, mood disturbances, and decreased performance in work or at school

137.A patient presented with history of repetitive coles of his garage around 10 times and moved around his house a lot. What is
the most likely diagnosis?
A. Obsessive compulsive disorder

138.Somatization disorder 2 q Ocd Tricyclic side effect Weight gain


Http://www.medbullets.com/step2-3-psychiatry/20644/somatoform-disorders

139.Antipsychotic drug cause weight gain:


A. Risperidone
the ts ol weipht paie haggees ie atoost attAntipsychotic but the one that are worse than Respirdone are Olanzapine and Clozapine.
Http://www.the-rheumatologist.org/article/when-steroids-cause-psychosis/3/

140.Patient talking to dr & all time patient looks to the right side, dr asked why?! He said his mother is there, diagnosis?
A. Visual Hallucination.
Answer: A

141.Scenario about body dysmorphic disorder.

Kaplan:
Definition: a disorder characterized by the belief that some body parts is abnormal, defective, or misshapen.
Most common concerns involve facial flaws.
Constant mirror checking.

142.Female develop extreme fear from zoo, park, sporting event, the fear prevents her from going out?
A. Agoraphobia

Answer: A
Toronto:

114
143.Best treatment for depression in adolescents and children?
A. Fluoxetine

Answer: Fluoxetine is approved in youth eight years and older and escitalopram is approved in youth 12 years and older.
Http://www.uptodate.com/contents/depression-treatment-options-for-children-and-adolescents-beyond-the-basics

144.A case about a 23 y o girl with fluctuating mood and always complain of being abounded by her psychiatrist. Diagnosis?
A. Borderline personality disorder.
Answer: A

145. Hallucination case what to give:


A. Antipsychotic
Answer:A
Iindications to use antipsychotic: schizophrenia and other psychotic disorders, mood disorders with or without
psychosis, violent behavior, autism, Tourette’s, dementai, OCD
Reference: Toronto notes

146.Feeling Hopelessness might mean:


A. Suicidal attempt
Answer: A
Symptoms associated with suicide hopelessness, anhedonia, insomnia ,severe anxiety,impaired concentration, psychomotor
agitation or panic attacks Reference: Toronto notes

115
147.Palpitation and shortness of breath, patient avoid getting out because he afraid the get the same symptoms:
A. Panic attack

Question and choices incomplete


Explanation: symptoms of panic attack include fear of dying, Palpitations, Sweating Trembling or shaking, Sense of shortness of
breath, and others. Symptoms usually peak within 10 min and dissipate within minutes thereafter. Panic disorder is occurrence of
repeated panic attacks typically accompanied by fears about future attacks or changes in behavior to avoid situations that might
predispose to attacks.
Reference: http://www.msdmanuals.com/professional/psychiatric-disorders/anxiety-and-stressor-related-disorders/panic-attacks-
and-panic-disorder

148.2 cases of obsessive compulsive disorder:


one of them was excessive washing of hand around 40 time.
the other about wife complain that her husband is checking the door locked or not a lot of times.

149.Patient came to the ER ,hyperarousal , suspicious Hyperactivity what is the most likely Diagnosis ?
A. Psychosis Mania
2 others (there was no drug abuse )

Incomplete question and choices!


1. Patient e manifestation of anorexia nervosa what's the lab associate with it?!!
A. Increased liver enzyme ?

2. ‫ واحد مدمن كل شوية يجي الطوارئ بشكوة غير ويبالغ في االعراض وفي الفحص يكون سليم لما يبعدوعنو ويحسب انهم مو شايفينو يرجع طبيعي جدا‬-malingering
(my answer) -there is NO addiction on the choices

3. What is the side of effect of the tricyclic antidepressant Ametreclen (Amitriptyline?):


A. Dystonia.

116
B. Hyper salivation.
C. Hyperpigmentation.
Answer: A (NOT SURE)
- Tricyclic antidepressants block muscarinic M1, histamine H1, and alpha-adrenergic receptors, and commonly cause cardiac
effects (orthostatic hypotension, bradycardia, arrhythmia), anticholinergic effects (blurred vision, dry mouth, constipation),
antihistaminic effects (sedation, increased appetite, weight gain), decreased seizure threshold, sexual dysfunction, diaphoresis,
and tremor.
- Answer is most probably A because others are wrong.
Source: https://yhdp.vn/uptodate/contents/mobipreview.htm?35/20/36169#H9

4. Drug not cause tardive dyskinesia ?


A. Olanzapine
Atypical antipsychotics (clozapine, risperidone, olanzapine, sertindole, quetiapine, ziprasidone, paliperidone, aripiprazole. Their
frequency and severity of Extrapyramidal side effects are significantly lower than seen with traditional neuroleptics. (Step Up to step
2 CK)

5. Stress cause increase in catecholamine and something what types of stress?


A. Psychoneural stress
Answer: A
6. SE of bupropion.
Tremor, dry mouth, anxiety, headache, increased risk of seizure

7. SE of TCA.
Anti-muscarinic effects: dry mouth, dry nose, blurry vision, lowered gastrointestinal motility or constipation

8. A girl with anxiety. What will you do?


A. Relaxation therapy
B. Behavioral therapy
C. Low dose alprazolam
Answer:

9. Patient his wife die 2 weeks ago ,, she did not have any illness, His appetite normal, Low mood?
Normal bereavement typically begins after the death of a loved one and includes feelings of sadness, worrying about the deceased,
irritability, sleep difficulties, poor concentration, and tearfulness. It typically lasts less than 6 months, but can go on longer.
Treatment is generally limited to supportive psychotherapy. (Master the board Step 2)

10. How we can assess cognitive impairment in old age ? ( short memory assess- IQ-

11. 20 years female come with her Dady complaining of tachycardia, abdominal pain and peripheral tingling after math exam
failure ,, diagnosis?
A. Anxiety
B. Depression
C. Oppsional deifint disorder
Answer: hyperventilation syndrome ?

117
Patients with acute hyperventilation syndrome (HVS) may present with agitation and anxiety. Most commonly, the history is one of
sudden onset of dyspnea, chest pain, or neurologic symptoms (e.g., dizziness, weakness, paresthesia’s, or near-syncope) after a
stressful event.
Http://emedicine.medscape.com/article/807277-clinical

12. 21 years female feels dizzy embarrassed when talking with classmates?
A. Low dose alozepam
B. Behavioral therapy
C. Anxiety therapy

13. Anxiety definition and gave me some disorders , which one included in this definition
A. Obsessive compulsive*
B. Dissociative
C. Bipolar
Answer: A
14. Amitriptan " TCA" side effect
A. Weight gain
Answer: A
Tricyclic antidepressants (amitriptyline, nortriptyline, imipramine), could cause Hypotension, dry mouth, constipation, confusion,
arrhythmias, sexual side effects, weight gain, GI disturbances. (Master the boards)

15. Long case about OCD (lock his car multiple times).

16. 50 years acute onset if disorientation, fluctuating mental status, other symptoms. Finally, he sees monkey in bed room? What
is your diagnosis?
A. Delirium
B. Schizophrenia
C. Psychosis
Schizophrenia is a mental illness that causes psychosis, but schizophrenia also has other symptoms. And it isn't the only cause
of psychosis. In some cases, other mental illnesses cause psychosis, including depression, bipolar disorder, dementia and borderline
personality disorder.
Psychosis: The disorder causes at least one psychotic symptom:
Delusions
Hallucinations
Disorganized speech
Grossly disorganized or catatonic behavior
Http://www.merckmanuals.com/professional/psychiatric-disorders/schizophrenia-and-related-disorders/brief-psychotic-disorder

17. Patient feels worry when he stands on class (agoraphobia) TREATMENT ?


A. Behavior
Agoraphobia is fear of and anticipatory anxiety about being trapped in situations or places without a way to escape easily and
without help if intense anxiety develops.

118
Treatment

1. Cognitive-behavioral therapy
2. Sometimes an SSRI
If untreated, agoraphobia usually waxes and wanes in severity. Agoraphobia may disappear without formal treatment, possibly
because some affected people conduct their own form of exposure therapy. But if agoraphobia interferes with functioning,
treatment is needed. Http://www.msdmanuals.com/professional/psychiatric-disorders/anxiety-and-stressor-related-
disorders/agoraphobia

18. Old with Alzheimer + hallucination TREATMENT : ola , halo , thio,(which type from antipsychotic drug will give him(
Http://www.aafp.org/afp/2003/0601/p2335.html

19. Phenytoin SE: hirsutism antiepileptic


Http://reference.medscape.com/drug/dilantin-phenytek-phenytoin-343019#4

20. Girl with BMI 16 and fine hair all over body
Answer: Anorexia Nervosa
Anorexia nervosa (AN) is a potentially life-threatening eating disorder characterized by the inability to
maintain a minimally normal weight, a devastating fear of weight gain, relentless dietary habits that prevent
weight gain, and a disturbance in the way in which body weight and shape are perceived
Http://emedicine.medscape.com/article/912187-overview

21. Generalized anxiety disorder, doctor want to give medication that is very effective but has minimal
abuse ability:
Answer: azatilopram
Selective serotonin reuptake inhibitors (ssris) are generally used as first-line agents, followed remotely by
tricyclic antidepressants (tcas).
Http://emedicine.medscape.com/article/286227-treatment#d8

22. Patient complaing of headache, he lost his wife recently, crying a lot and socially isolated, sleep is not
affected, I can’t remember the duration ?

A. Major depressive episode


B. Conversion
C. Dysthymia

Among the criteria for a major depressive disorder, at least 5 of the following symptoms have to have been
present during the same 2-week period (and at least 1 of the symptoms must be diminished interest/pleasure
or depressed mood):
- Depressed mood: For children and adolescents, this can also be an irritable mood
- Diminished interest or loss of pleasure in almost all activities (anhedonia)
119
- Significant weight change or appetite disturbance: For children, this can be failure to achieve expected
weight gain
1. Sleep disturbance (insomnia or hypersomnia)
2. Psychomotor agitation or retardation
- Fatigue or loss of energy
- Feelings of worthlessness
- Diminished ability to think or concentrate; indecisiveness
3. Recurrent thoughts of death, recurrent suicidal ideation without a specific plan, or a suicide attempting or
specific plan for committing suicide
4. Http://emedicine.medscape.com/article/286759-overview

23. Case of autism


Autism spectrum disorder (ASD) manifests in early childhood and is characterized by qualitative abnormalities
in social interactions, markedly aberrant communication skills, and restricted repetitive behaviors, interests,
and activities (rrbs).
Http://emedicine.medscape.com/article/912781-overview

24. Case of attention deficit hyperactivity disorder ?

Answer: Atomoxetine
Atomoxetine (brand name: Strattera) is a monoamine reuptake inhibitor predominantly affecting
norepinephrine which is approved for the treatment of attention deficit hyperactivity disorder (ADHD)
Https://en.m.wikipedia.org/wiki/Atomoxetine

Regarding medication for ADHD, stimulants are the first-line therapy and probably the most effective
treatment.
Http://emedicine.medscape.com/article/289350-treatment
25. Young male presented with history of delusion, behavioral changes with no mood disturbance for 3 months. Relieved
spontaneously without treatment.
A. Brief psychotic
B. Schizophrenia
C. Schizoaffective disorder
D. Schizophreniform Disorder
Answer: D,
- Brief psychotic: consists of delusions, hallucinations, or other psychotic symptoms for at least 1 day but < 1 mo.
- Schizoaffective disorder: characterized by significant mood symptoms, psychosis, and other symptoms of schizophrenia. It is
differentiated from schizophrenia by occurrence of ≥ 1 episodes of depressive or manic symptoms.
- schizophreniform: characterized by symptoms identical to those of schizophrenia but that last ≥ 1 mo but < 6 mo. Merck Manual

120
26. Psychiatric patient complaining of being "abandoned " by her psychiatrist. She was also complaining of being abandoned by
everyone before. ‫المتواجدين بعد كانوا يشتكون انه تصرفاتها متناقضة‬. What is that type of personality?
A. Schizophrenic
B. Borderline
C. Obsessive
Answer: B, patients with borderline personality disorder have an intolerance of being alone; they make frantic efforts to avoid
abandonment and generate crises, such as making suicidal gestures in a way that invites rescue and caregiving by others. Merck
Manual

27. How to manage psychotic patient non-complaint to his medication


A. Diazepam
B. Haloperidol IV injection
Answer: B, switch to a long-acting injectable antipsychotic for patients with poor adherence.
Http://www.psychiatrictimes.com/schizophrenia/switching-antipsychotics-why-when-and-how/page/0/4
long-acting preparations are useful for eliminating drug nonadherence. They may also help patients who, because of disorganization,
indifference, or denial of illness, cannot reliably take daily oral drugs.
Http://www.merckmanuals.com/professional/psychiatric-disorders/schizophrenia-and-related-disorders/schizophrenia

28. She is angry at her friend, she was taking another path when walking, not answering her calls, don’t drink or eat in same
place with her, this psychological behavior called: avoidance
Http://psychologydictionary.org/avoidance/

29. 20 years female feels dizzy embarrassed when talking with classmates? Behavioral therapy
30. 14 years female come with her farther complaining of tachycardia, abdominal pain and peripheral tingling after math exam
failure, diagnosis? Anxiety

31. Patient with grandiose something, flight of ideas, diagnosis ?


A) psychosis.
Answer: choices are not enough - mania wasn’t mentioned.
A delusion of grandeur or Grandiose delusion is the fixed, false belief that one possesses superior qualities such as genius, fame,
omnipotence, or wealth. It is most often a symptom of schizophrenia, but can also be a symptom found in psychotic or bipolar
disorders (manic state).

32. Patient with generalized anxiety disorder the doctor want to give him drug that had a lower abuse what to choose:
A. Buspirone

33. What is treatment of bipolar?


A- Olanzapine
B- Lithium

34. Female can't smell or taste , otherwise normal :


A. Malingering.
DSM-5 subtypes of factitious disorder [1]
i. Factitious disorder imposed on self
121
ii. Falsification of physical or psychological signs or symptoms, or induction of injury or disease, associated with identified
deception.
iii. The individual presents himself or herself to others as ill, impaired, or injured.
iv. The deceptive behavior is evident even in the absence of obvious external rewards.
v. The behavior is not better explained by another mental disorder, such as delusional disorder or another psychotic
disorder
vi. Factitious disorder imposed on another (previously factitious disorder by proxy)
vii. Falsification of physical or psychological signs or symptoms, or induction of injury or disease, in another, associated with
identified deception.
viii. The individual presents another individual (victim) to others as ill, impaired, or injured.
ix. The deceptive behavior is evident even in the absence of obvious external rewards.
x. The behavior is not better explained by another mental disorder, such as delusional disorder or another psychotic
disorder. BMJ

35. Mother after delivery sees snakes crawling into her baby bed :
A. Postpartum psychosis (symptom of Postnatal depression).
Psychotic symptoms (uncommon) These include hallucinations, delusions, confused and disturbed thoughts, and a lack of insight
and self-awareness.
Psychotic symptoms occurred in 4% of episodes in one study. [59] They substantially increase risk of self-harm or harm to the baby,
particularly if the delusions or hallucinations relate to the baby. BMJ http://bestpractice.bmj.com/best-
practice/monograph/512/diagnosis/history-and-examination.html

36. Patient had hx of schizophrenia noncompliance with oral medication?


A. Give Depot haloperidol

37. A teacher is nervous each time before he enters his class. He says it is only a matter of time before he makes a mistake. He
also complains of tremor, fear, sweating and palpitation. What is the most likely diagnosis?
A. Social phobia
B. Specific phobia
C. Anxiety
Answer: A
Social phobia is defined as marked and persistent fear of social or performance situations in which one is exposed to unfamiliar or
possible scrutiny by others; fearing he or she will act in a way that may be humiliating or embarrassing.
Toronto notes 2015

38. A Man is complaining of episodes of numbness and nervousness every time he sees cars that are also associated with tremor
and sweating. What is the most likely diagnosis?
A. Panic attacks
B. Social phobia
C. Specific phobia
D. Anxiety
Answer: C

122
Specific phobia is defined as marked and persistent fear that is excessive or unreasonable, cued by presence or anticipation of a
specific object or situation.
Toronto notes

39. Low mood, activity, sleep, eating?


A- Manic episode
B- Depressive
C- Schizophrenia
D- OCD
Answer: B

According to DSM 5, Major depressive disorder is diagnosed as at least 5 of the following symptoms have to have been present
during the same 2-week period (and at least 1 of the symptoms must be diminished interest/pleasure or depressed mood):
- Depressed mood: For children and adolescents, this can also be an irritable mood
- Diminished interest or loss of pleasure in almost all activities (anhedonia)
- Significant weight change or appetite disturbance: For children, this can be failure to thrive
- Sleep disturbance (insomnia or hypersomnia)
- Psychomotor agitation or retardation
- Fatigue or loss of energy
- Feelings of worthlessness
- Diminished ability to think or concentrate; indecisiveness
- Recurrent thoughts of death, recurrent suicidal ideation without a specific plan.

40. Case about female counting everything (stairs, words, steps) and she can stop but feels agitated?
A- Obsession
B- Compulsion
C- Anxiety
Answer: B

According to DSM 5:
Obsessions are defined by (1) and (2) as follows:
- Recurrent and persistent thoughts, urges, or images that are experienced, at some time during the disturbance, as intrusive
and unwanted, and cause marked anxiety and distress
- The person attempts to suppress or ignore such thoughts, impulses, or images or to neutralize them with some other
thought or action
Compulsions are defined by (1) and (2) as follows:
- Repetitive behaviors (e.g., hand washing, ordering, checking) or mental acts (e.g., praying, counting, repeating words
silently) in response to an obsession or according to rules that must be applied rigidly.
- The behaviors or mental acts are aimed at preventing or reducing distress or preventing some dreaded event or situation;
however, these behaviors or mental acts either are not connected in a way that could realistically neutralize or prevent
whatever they are meant to address, or they are clearly excessive.

41. Elderly patient already diagnosed with Alzheimer's disease development psychotic symptoms what is drug of choice:
A- olanzepine
B- resperidone
123
C-Haloperidol
Answer: B
Risperidone reduces psychosis in patients with Alzheimer disease.
Reference: http://www.aafp.org/afp/2011/0615/p1403.html

42. Patient with severe depression


A- TCA
B- ECT electro convulsive therapy
Answer: A
Reference: http://www.medscape.com/viewarticle/467185_4

43. Which anti psychosis can lead to obesity?


Answer: Olanzepine
Reference: http://reference.medscape.com/drug/zyprexa-relprevv-olanzapine-342979#4

44. 3 months post-partum female fair from snake come around her baby?
Postpartum psychosis
Reference: http://www.rcpsych.ac.uk/healthadvice/problemsdisorders/postpartumpsychosis.aspx

45. Which medication used for heroin withdrawal?


Answer:
The U.S. Food and Drug administration approved buprenorphine (branded drugs include Suboxone and Subutex) for
the use of opioid withdrawal and as a maintenance therapy.
Link:
http://www.nationaldrugstrategy.gov.au/internet/drugstrategy/publishing.nsf/Content/9011C92D2F6E1FC5CA2575B4
001353B6/$File/bupren4.pdf

46. Female patient around 50, complaining of palpitations, weight loss and insomnia without any reason?
A. Generalized anxiety disorder
B. Panic disorder
Answer: B

47. Female young developed a panic attack during meeting ,, since that she avoid attend meeting
A - Specific phobia
B - Panic disorder
C - Generalized anxiety disorder
Answer: A

48. Antidepressants cause insomnia...

49. Case of Adjustment disorder. What is the best treatment?


Answer: Supportive psychotherapy

124
50. What is the most commonly used medication for pediatric psychiatric illnesses?

The best-known and most-treated childhood-onset mental disorder is attention deficit hyperactivity disorder (ADHD). Children with
ADHD exhibit symptoms such as short attention span, excessive motor activity, and impulsivity which interfere with their ability to
function especially at school. The medications most commonly prescribed for ADHD are called stimulants. These include
methylphenidate (Ritalin, Metadata, Concerta), amphetamine (Adderall), dextroamphetamine (Dexedrine, Dextrostat), and
pemoline (Cylert).

51. Patient elderly with symptoms pf infections and delirium what to give:
Answer: Haloperidol

Delirium that causes injury to the patient or others should be treated with medications. The most common medications used are
neuroleptics. Benzodiazepines often are used for withdrawal states. Even though case reports showed evidence that cholinesterase
inhibitors may play a role in the management of delirium, larger trials and systematic review did not support this use.
Neuroleptics
The medication of choice in the treatment of psychotic symptoms. Older neuroleptics such as haloperidol, a high-potency
antipsychotic, are useful but have many adverse neurological effects. Newer neuroleptics such as risperidone, olanzapine, and
quetiapine relieve symptoms while minimizing adverse effects. Initial doses may need to be higher than maintenance doses. Use
lower doses in patients who are elderly. Discontinue these medications as soon as possible. Attempatient a trial of tapering the
medication once symptoms are in control. Neuroleptics can be associated with adverse neurological effects such as extrapyramidal
symptoms, neuroleptic malignant syndrome, and tardive dyskinesia. Doses should be kepatient as low as possible to minimize
adverse effects. Paradoxical and hypersensitivity reactions may occur.

52. A girl feel irritable, palpitations, and nervous every time the ask her to speak in the class, what’s the management?
A. Small dose of lorazepam
B. Behavioral therapy
C. Relaxing techniques
Answer: B

53. Social anxiety disorder treatment includes:


Answer: Behavioral therapy (first line), Pharmacological therapy with SSRI
Toronto note 2015

54. Patient has insomnia, afraid of lose his job with hx of repeated checks about the door and his children, if he didn't do this
routine. He can't sleep
A) OCD
B) Generalized anxiety
Answer: A

55. Bipolar patient type 1, started on lithium 6 weeks ago, c/o excessive thirst and urination. Serum lithium within the optimal
therapeutic range. What's the underlying cause of his symptoms?
A) Psychogenic polyuria
B) Central DI
C) Nephrogenic DI

125
Answer: C
-A search of the World Health Organization's adverse effect database revealed 359 reports of drug-induced diabetes insipidus.
Lithium was the most common cause (159 reports). Http://www.ncbi.nlm.nih.gov/pubmed/10612269
- Renal toxicity is common with chronic lithium therapy, with nephrogenic diabetes insipidus being the most severe
manifestation. Http://emedicine.medscape.com/article/815523-clinical#showall

56. Patient who eats a lot what is appropriate approach:


A) Cognitive behavioral therapy
B) Interpersonal therapy
Answer: A

57. Case of Adjustment disorder and asking about the best treatment.
A. Supportive psychotherapy

Answer: A

58. Patient whose wife died 2 weeks ago, she did not have any illness, His appetite normal with low mood
Answer: Incomplete MCQ

59. Case of neurosis what is investigation perform ?


A- Urinalysis
B- Urine culture
C- CBC
D- Polygraph
Answer: A

60. Patient with depression on antidepressant present with toxicity non elevated liver enzymes and some GI symptoms what
drug
A- aspirin
B- acetaminophen
C- Antidepressant mentioned in the scenario
D- NSAIDs
Answer:

61. case of poor hygiene and hallucination. What medication to give?


A. lithium
Answer: ?
Antipsychotic medications used to treat hallucinations include haloperidol, olanzapine and risperidone.
http://www.news-medical.net/health/hallucination-treatments.aspx

62. A patient reported that he was diagnosed with pancreatic cancer and had received chemotherapy yet with no improvement
and wants to do invx , his medical report was free (he was lying) , and when he confronted by reports , he run ? Dx :
most
probably malingering.

126
63. Patient with manifestation of anorexia nervosa what's the lab associated with it?
A. Increased liver enzymes.
Answer: a
• Low: wbc, plt, na, k, phosphate, hypoglycemia, t3
• Normal: hgb, esr, rft
• High: bun, lft, cholesterol
• Hypokalemic hypochloremic metabolic alkalosis: observed with vomiting
• Acidosis: observed in cases of laxative abuse

64. Patient with erectile dysfunction, what's the medication to avoid?


A- Ssri.
Answer: a

65. Patient with bulimia what’s lab finding?


A. Hypokalemia, metabolic alkalosis, high amylase
Answer: a

Increased serum amylase(is found in up to 30% of persons with significant vomiting because of hypersecretion from the salivary
glands.), sialadenitis. With significant vomiting, hypokalemic metabolic alkalosis is possible. Among patients with significant
laxative abuse, normokalemic metabolic acidosis may occur.
Reference: toronto notes medscape: http://emedicine.medscape.com/article/286485-workup

66. Somatization disorder 2 q ocd tryciclic side effect weight gain

67. Antipsychotic drug cause wight gain :


Respirdone, couldn’t remember the other choices
Mood-stabilizing antipsychotics commonly prescribed for bipolar disorder include olanzapine (zyprexa), risperidone (risperdal),
quetiapine (seroquel), aripiprazole (abilify), ziprasidone (geodon), lurasidone (latuda) and asenapine (saphris).
Reference : http://www.mayoclinic.org/diseases-conditions/bipolar-disorder/expert-answers/bipolar-medications-and-weight-
gain/faq-20058043
http://psycheducation.org/treatment/weight-gain-and-bipolar-disorder-treatment/

127
Emergency
medicine
128
1. A patient is brought to the emergency department after a motor vehicle accident. On examination, he has a femur fracture
that is associate with high alp in his labs. How would you confirm that this elevation in alp is not hepatic in origin?
A. Glucose 6 phosphate
B. ….. Kinase
C. Alanine transaminase
D. Gamma-glutyml transpeptidase
Answer: d
Obtaining a serum gamma glutamyl transferase (ggt) is a helpful next step. Found primarily in the liver and kidney, ggt rise usually
parallels changes in alp in liver disease; elevations in both alp and ggt help confirm the likelihood of a hepatic disorder. If ggt is
normal when alp elevation is noted, evaluation for bone disease, included bony metastases, should be pursued.
http://www.medscape.com/viewarticle/413420

2. Long scenario .. Patient brought to ER with multiple injuries in head . Chest , abdomen , proximal upper limb is
amputated blood profuse , hypotensive , on o2 face mask 6L , what to do
A. Tourniquet ,
B. Clamp vessels ,
C. OR ,
D. Mantain airway
Answer : Maintain Airway. Three goals exist in the emergency department treatment of the patient with hypovolemic shock as
follows: (1) maximize oxygen delivery - completed by ensuring adequacy of ventilation, increasing oxygen saturation of the blood,
and restoring blood flow, (2) control further blood loss, and (3) fluid resuscitation. Reference: Medscape.

3. In emergency department patient come with close head truma and loss of consciousness what is the first thing to do:
A. Intubation and hyperventilation
B. Asses airway

129
C. Pupillary response
D. Glascow coma scale
Answer: Assess Airway. Establishing and maintaining airway patency takes precedence over all other treatment. 
Reference: First
Aid Step 2 page 466

4. An alcoholic comes to you with symptoms of alcohol withdrawal. Last drink he consumed was 2 days back. What drug will
you give?
A. Naloxone
B. Diazepam
C. Nicotine
D. Disulfiram
Answer:B
Diazepam. Basic protocol for alcohol withdrawal treatment is diazepam 20mg PO every 2hrs til regression of severe withdrawal
symptoms + Thiamine 100 mg IM then 100 mg PO OD for 3 days in addition to hydration. Reference: Toronto notes.

5. Patient presented to ER with history of drug overdose and coma for the last 8 hours on examination absent gag reflex best
management is :
A. A.iv naloxone
B. B.gastric lavage
C. C.immediate endotracheal intubation
D. Charcoal
Answer: c
Airway protection – Airway protection by endotracheal intubation should be performed early in the poisoned patient with
depressed mental status, unless the cause is easily reversible (eg, opioid intoxication or hypoglycemia), because of the high risk for
aspiration and its associated complications, particularly when gastric decontamination procedures need to be undertaken
Http://www.uptodate.com/contents/general-approach-to-drug-poisoning-in-adults

6. Patient involved in MVA the impact especially in forehead examination forehead laceration & fx ,discharge from nose clear
positive glucose test ( I think mean CSF leak ) Which cranial n affected :
A. Optic
B. Olfactory
C. Ophthalmic
D. Oculomotor
Answer: D. Oclumotor nerve. Oculomotor deficits from injuries to cranial nerves III, IV, or VI can occur, as may facial nerve palsies or
hearing loss from injury to cranial nerves VII and VIII, respectively. Reference: http://www.uptodate.com/contents/skull-fractures-
in-adults#H10

7. Which of the following substance ingestion is a contraindication to gastric lavage?


A. Aspirin.
B. Benzodiazepine.
C. Some kind of bleach or cleaning products.
D. Maybe hydrocarbons.

130
Answer: C. Caustics ingestion (drain cleaners) is a contraindication to Gastric lavage. Reference: Master the board page:
602
Treatment includes high volume water and endoscopy to visualize degree of damage.

8. Child on picnic with family, presented to the ER with high suspicion of foreign body inhalation. The most common location in
the chest will be in?
A. Right Main Bronchus *
B. Left Main Bronchus
C. Carina of Trachea
D. Inlet of Larynx
Answer: A
Right Main Bronchus *

9. Case about someone stabbed in the 4th right intercostal space, what the structure behind it?
A. Right horizontal fissure
B. Right apical lung
C. Left something
D. Left something
Answer :A
Right horizontal fissure

10. Man make RTA he was conscious , oriented , alert , but his extremity ‫دم محتقنه‬
Type of shock ?
A. Irreversable
B. Neuro
C. Cardio
D. Hemorrhagic
Answer: D
Hemorrhagic

11. A patient comes to the emergency with sudden dypsnea. X ray picture was attached showing dark area
Without vascular marking. What is the diagnosis?
A. Pneumonia
B. Pulmonary edema
C. Pneumothorax
D. Pulmonary embolism

Answer: C (radiology master class


12. A guy was standing at a bus stop then fell. No previous history and no cardiac anomalies. What is the diagnosis?
A. Cardiac syncope
B. Fainting??!
C. Seizure
D. Vasovagal
Answer: D
Vasovagal cause fainting/ syncope. I don’t think “Fainting” was included in the choices provided in the exam.

13. Semiconscious polytrauma patient, has difficulty breathing on bag ventilator (Ambu bag), prepared for intubation. What is
131
the next step?
A. Go directly and intubate
B. Cricoid pressure
C. O2 supplements
D. Jaw thrust

Answer: C
Preoxygenation/Denitrogenation with 100% O2 for 3 min or 4 vital capacity breaths. Toronto Notes

14. A patient ate a wild mushroom. Which of the following will be inhibited?
A. RNA polymerase I
B. RNA polymerase II
C. RNA polymerase III
D. DNA Gyrase

Answer: B

15. Post-traumatic amnesia, vitally stable but he’s complaining of pain in all of his 4 extremities. Which type of shock might be
developed?
A. Neuro
B. Cardiac
C. Hypovolemic
D. Reversible

Answer: A
Reference: Toronto Notes

16. Which of the following babies needs immediate action?


A. Cough and wheezing.
B. Shallow spinal skin.
C. Absent lower pulse.
D. Yellowish discoloration.
Answer: C

17. 36 year old male having retrosternal chest pain that radiates to jaw and left arm for 20 minutes (vitals included). What
is the first thing to give him?
A. Aspirin.
B. Morphine.
C. Oxygen.
D. IV nitroglycerin.
Answer: A

Reference: Master the boards: USMLE step 2 CK

18. You are working as an emergency doctor and you got a call that there was an accident at the nuclear plant and 2 men were

132
exposed to nuclear radiation. They don’t have any complaints right now. What will you advise them?
A. Cover them up with dry thick blankets
B. Give them aspirin and ask them to keep their legs moving
C. Isolate them and advise them to stay calm
D. Give them antibiotics
Answer: C or D?

• In hospital casualty will be finally decontaminated and kept in a clean special ward. The decontamination room must be at the
entry of the hospital and should be sealed off from other premises and should have a separate ventilation system.
References: http://www.apiindia.org/pdf/pg_med_2008/Chapter-17.pdf
• Prevention and treatment of infections. References: http://emergency.cdc.gov/radiation/arsphysicianfactsheet.asp

19. Unconscious patient after ingestion of overdose sleep pills, breathing was reflex response breathing , you give him 2
breathing by mask , then you check pulse it was rapid and weak , what is
Next step:
A. Wait code blue team
B. Put patient in recovery position
C. Intubat and ventilation
D. Do CPR 5 cycle 30:2
Answer: C

Airway protection – Airway protection by endotracheal intubation should be performed early in the poisoned patient with
depressed mental status, unless the cause is easily reversible (eg, opioid intoxication or hypoglycemia), because of the high risk for
aspiration and its associated complications, particularly when gastric decontamination procedures need to be undertaken [40].
Tracheal intubation with mechanical ventilation is also indicated in the presence of severe acid-base disturbances or acute
respiratory failure. Particularly when intubating a severely acidemic patient, it is important to prevent the development of a
respiratory acidosis through inadequate minute ventilation. Occasionally, the management of high-grade physiologic stimulation
may require sedation and/or paralysis with mechanical ventilation to limit the extent of complications such as hyperthermia,
acidosis, and rhabdomyolysis. One rare exception to this important principle of aggressive airway management is salicylate
poisoning, in which mechanical ventilation should be avoided unless absolutely necessary.
Ref: uptodate

20. Child girl obese try to suicide and eat alot of drug because of her friend and came to the ER?

A. Referral immediate to the psychatry


B. Treatmen for acute depression
C. Something
D. Another something
Answer: A

21. Patient brought to ER with head trauma and loss of consciousness, what should you do first?

A. Check for pupillary reflex

B. B ) intubate and hyperventilate

C. Glascow coma scale


133
D. Check the airway.

Answer : d

22. Patient came from RTA and the staff preparing him for intubation he devolved low ventilation on bivalve what to d
o ? Repeated
A. A.Proceed to intubation
B. Jaw thrust
C. More head tilt
D. Cricoid pressure
Answer:A

23. Patient conscious with multiple injuries. How do you maintain airway?
A. Mask
B. Oro pharyngeal airway
C. Nasopharyngeal
D. Endotracheal Intubation
Answer : A

24. Patient presented to ER with history of drug overdose and coma for the last 8 hours on examination absent gag reflex best
management is :
A. Iv naloxone
B. Gastric lavage
C. Immediate endotracheal intubation
D. Coracol
Answer: C

25. Unconscious patient after RTA .. Ventilation with bag mask was difficult .. What to do ?
A. Exaggerate jaw thrust
B. Increase head tilt
C. Coricoid pressure
D. Intubation
Answer : D

26. Elderly asthmatic patient comes to regular check up. His wife has osteoporosis. He's recurrent kidney stones.he's taking
calcium and vitamin supplements. Labs show high ca, normal phosphate, PTH high. Most diagnosis Is:
A. Sarcoidosis.
B. Paget's disease.
C. Vit D intoxication.
D. Hyperparathyroidism.
Answer: D

27. A patient presented to ER with ingestion of multiple iron tablets. What is your next step?
A. Induce emesis
B. Ipecac syrup
C. Gastric lavage

134
D. Wait and monitor
Answer:D if asymptomatic wait 6 h then discharge
Management decisions — The management of patients who have severe symptoms and/or have ingested toxic amounts of iron is
straightforward. (See 'GI decontamination' above and 'Deferoxamine' above.)
Mildly symptomatic patients — Patients who have ingested nontoxic doses of elemental iron frequently experience mild
gastrointestinal symptoms such as abdominal pain, vomiting, and diarrhea. Those with an unintentional overdose of less than
40 mg/kg elemental iron and a normal physical examination may be observed.
Https://yhdp.vn/uptodate/contents/mobipreview.htm?2/57/2970

28. Patient came to ER, airway patient gasping ,, u gave 2 breath by mask , pulse is rapid & weak. What to do?
A. Waiting code team.
B. Put patient in recovery position.
C. Do CPR.
D. Intubation.
Answer: D
NO CLEAR RESOURSE

29. Patient come to ER unconscious :


A. Check pulse
B. Examine pupils
C. IV fluid infusion
D. Clear airway

Answer: D
Explanation: The first step in evaluating an acutely injured patient assesses the need for resuscitation. As taught throughout basic
life support training, remember the abcs (ie, airway, breathing, circulation).
Reference: http://emedicine.medscape.com/article/869733-overview#a2

30. In emergency department patient come with close head trauma and loss of consciousness what is the first thing to do:
A. Intubation and hyperventalition
B. Asses airway
C. Pupalliry responce
D. Glascow coma scale

Answer: B
Http://www.bmj.com/content/300/6734/1265

31. A 6-year-old girl, brought by parents to ER with history of falling from height. Not talking but crying, withdrawal from pain,
open her eye only in response to Doctor talking. Calculate GCS
A. 9
B. 10
C. 11
D. 12

135
Answer: B Medscape

32. Patients brought to the emergency department after a motor vehicle accident with a depressed skull fracture. The patient is
unconscious and difficult to intubate. What will be your next step?
A. Increase head tilt
B. Increase chin left
C. Cricoid pressure
D. Proceed to intubation
Answer: ??

33. Case with shock, baroreceptor is activated and he is having tachycardia which of the following will be decreased?
A. Heart rate
B. Venous capita
C. Coronary blood flow
D. Something irrelevant
Answer: B

34. Unconscious patient after road traffic accident, ventilation with bag mask (Ambu bag) was difficult. What to do?
A. Exaggerated jaw thrust
B. Increase head tilt
C. Corticoid pressure
D. Intubation
Answer: D
Uptodate: Most emergency department (ED) endotracheal intubations are performed on an emergent basis (ie, intubation cannot
be delayed or avoided). The universal emergency airway management algorithm© provides the recommended approach to
emergency intubation. This approach is based on two key assessments of the patient prior to intubation.

136
The first assessment is to determine if the patient has a "crash" airway (ie, presenting in extremis with little or no cardiovascular or
respiratory activity, and unlikely to respond to insertion of a laryngoscope). If so, the crash airway algorithm© is used
If the patient is not a crash airway, the next step is to determine if the patient presents a difficult airway. This requires assessment of
specific patient attributes to predict the likelihood of difficulty in performing any of the major procedures in airway management:
direct laryngoscopy and intubation, bag-mask ventilation, surgical airway management, and ventilation using an extraglottic airway.
If the patient is felt to be neither a crash nor a difficult airway, then rapid sequence intubation is the recommended method for
managing the airway
If, at any time during the evaluation or management of the difficult airway, an SpO2 cannot be maintained at 90 percent, or at least
held stable in a viable range, the difficult airway becomes a failed airway, and the failed airway algorithm© is followed

35. 1st line of management in HyPerCalcemia


A. pratyhriodexromy
B. Hem dialysis
C. Fluid adjacent
D. Dexoferrin
Answer: C

36. Patient presented with sever vomiting on exam he looks ill with dry mucous membrane, capillary refill=3 seconds and he
cries with tears. What is the appropriate management?
A. Severe and treat with dextrose
B. Moderate and treat with ORS
C. Moderate and treat with pedialyte (I chose this on as he cries with tears so its not severe but im not sure
D. This is a 5% dehydration
Answer: B or C

37. Patient presented with drug overdose comatose and with loss gag reflex what is the next step in Mx:
137
A. IV nalaxone
B. Intubate
C. Some Other drug
D. Gastric lavage

Answer: B

38. Pt presented 45 mints after iron tab ingestion what is the best initial management?
A. Charcoal
B. Gastric lavage
C. Tetaric emetic Inducing vomiting
D. observe

Answer: ???
It depends on the patient presentation. It should be treated as allergic reaction.

39. Pt presented 45 mints after iron tab ingestion what is the best initial management?
A. Charcoal
B. Gastric lavage
C. Tetaric emetic inducing vomiting
D. Observe
Answer: D
Reference: http://emedicine.medscape.com/article/801821-treatment

40. Healthy woman travelled for 18 hours, examination were completely normal, investigation done and D dimer was 350, they
give the normal value less than 250. What is the appropriate management?
A. Heparin
B. LMWH
C. Unfractionated heparin
Answer: B

41. …Old pt, diabetic dehydrated , lab high Na low k , high bicarbonate , ketones in urine Dx ?
A. metabolic syndrome
B. diabetic ketoacidosis
C. lactic acidosis

answer: B
http://emedicine.medscape.com/article/118361-overview

138
42. Pt with acetaminophen overdose ,conscious and cooperative, the best initial management?
A. Oral Acetylcysteine
B. IV Acetylcysteine
C. Charcoal
Answer: with an hour presented C if more or symptomatic A.
Adults toxic dose 7.5-10 g. Oral activated charcoal (AC) if the patient presents within 1 hour after ingesting a potentially toxic dose,
and clinically stable. I suggest reading: http://emedicine.medscape.com/article/820200-treatment#d1

43. A case of heat stroke?


A. Warm fluid
B. Core cooling whole body
C. Electrolytes replacement

Answer: B

Heatstroke is a medical emergency and continues to be one of the leading causes of preventable death in sports. Rapid reduction of
the core body temperature is the cornerstone of treatment because the duration of hyperthermia is the primary determinant of
outcome. Once heatstroke is suspected, cooling must begin immediately and must be continued during the patient's resuscitation.

Reference: Medscape: http://emedicine.medscape.com/article/166320-treatment

44. Road Traffic Accident (RTA) pt came to ER after 30 mins o/e chest there's sound bilateral + distended neck. Dx?! (cardio)
A. Hemothorax
B. Pneumothorax
C. Cardiac tamponade

Answer is C. Beck’s triad for diagnosing cardiac temponade : JVD, Hypotension, Distant heart sounds (muffled sound). In
pneumothorax and hemothorax breath sounds are decreased on the affected side. If chest sounds are not affected with distended
neck so this can be cardiogenic shock.

45. What will you give in case of methotrexate toxicity?


A. Folic acid
B. Folnelic acid (something like that)
C. Steroid
Answer: A
There are three antidotes that have been used for MTX toxicity: leucovorin, thymidine and glucarpidase.

139
Leucovorin (folinic acid) is the reduced and active form of folic acid. It selectively “rescues” normal cells from the toxic effects
caused by MTX’s inhibition of the production of reduced folates. The recommended dosage is 100 mg/m2 IV every 3 to 6 hours
until the plasma MTX level is less than 0.01 mcmol/L or for 3 days or longer if levels are not available.
Thymidine Its use is investigational and is only given along with other therapies
Glucarpidase (carboxypeptidase) is an antidote that has been used recently for MTX toxicity in combination with leucovorin. It
converts MTX to an inactive form and rapidly lowers MTX blood levels. It is given as a single bolus of 50 units/kg IV over 5
minutes. Leucovorin should be continued for 48 hours after glucarpidase administration
Hemodialysis and hemoperfusion have been used to lower MTX levels. Intrathecal overdoses require special measures including
cerebrospinal fluid drainage and exchange, steroids, and antidotes.

46. Pt with acetaminophen overdose, conscious and cooperative, the best initial management?
A. Oral Acetylcysteine
B. IV Acetylcysteine
C. Charcoal
Answer: C
Reference: http://emedicine.medscape.com/article/820200-treatment#d10

47. Child with traumatic injury to elbow. X-ray will show:


A. Posterior fat pad.**
B. Anterior fat pad.
C. Others epicondylar injuries I forgot.

Answer: Posterior Fat pad. Supracondylar fractures account for more than 60% of all elbow fractures in children.
More than 95% of supracondylar fractures are hyperextension type due to a fall on the outstretched hand. . If there is only minimal
or no displacement these fractures can be occult on radiographs and the only sign will be a positive fat pad sign. A visible fat pad
sign without the demonstration of a fracture should be regarded as an occult fracture. These patients are treated as having a
nondisplaced fracture with 2 weeks splinting.
In trauma, displacement of the posterior fat pad is virtually pathognomonic of the presence of a fracture.
Displacement of the anterior fat pad alone however can occur due to minimal joint effusion and is less specific for fracture.

Reference: http://www.radiologyassistant.nl/en/p4214416a75d87/elbow-fractures-in-children.html

48. Man brought to ER with suicidal attempt after ingestion 10 g of paracetamol 30 mins ago , he is conscious , oriented , what to
do ?
A. Activated charcol
B. N-acitylcystine
C. Obsreve
Answer: Activated Charcoal.
Usually start with Activated charcoal if the paracetamol ingestion is within 2 hours as it reduce the gastrointestinal absorption for
paracetamol, then give the antidote for paracetamil (N-acitylcystine) if the plasma paracetamol level plotted on or above the line
drawn between 100 mg/L at 4 hours and 15 mg/L at 15 hours after ingestion
Reference: http://patient.info/doctor/paracetamol-poisoning

140
49. Asprin toxicity with ABG : show low CO2 and low HCO3 ?
With acidic PH ?
Cause ?
A. Resp. Alkalosis followed by metabolic acidosis
B. Metabolic acidosis followed by resp. Alkalosis
C. Metabolic acidosis
Answer: Respiratory alkalosis progressing to metabolic acidosis is the key for aspirin overdose diagnosis. Aspirin interferes with
oxidative phosphorylation increasing lactate levels. Reference: Master the board page: 604

50. Patient came to ER comatosed, red cherry skin


A. Bacteremia
B. Insulin overdose
C. Carbon monoxide poisoning
Answer: Carbon monoxide poisoning. Medscape: http://emedicine.medscape.com/article/819987-clinical#b4

51. Elderly patient had motor vehicle accident, there is problem with ventilation. What is next step:
A. Exaggerated jaw thrust
B. More head tilt
C. Intubate immediately
Answer : C. Failure of ventilation is an indication for endotracheal intubation. Reference: Medscape.

52. Dog bite infections?


A. Gram negative bacillus bacterua
B. Virals
C. Parasite

Answer : A. The most common pathogens in dog bites are Pasteurella spp. (both Pasteurella multocida and Pasteurella canis),
Staphylococcus and Streptococcus spp., and the fastidious Gram-negative rod Capnocytophaga canimorsus (previously known as the
CDC and Prevention Group Dysgonic Fermenter-2).

Http://www.medscape.com/viewarticle/739023_4

Treatment is amoxicillin/clavulanate plus tetanus vaccination booster if more than 5 years since last injection. Reference: Master the
board.

53. Case head trauma on parietal lobe subdural hematoma which artery is injured ?
A. Superficial temporal .
B. Mid cerebral
C. Rt.cerebral

Answer: B Middle cerebral artery supplies parietal lobe. However, subdural hematomas are most commonly caused by tearing
of the bridging veins that drain from the surface of the brain to the dural sinuses. Arterial rupture can also result in Subdural

141
Hematoma and most of them were caused by injuries to small cortical arteries.

Reference:
http://www.uptodate.com/contents/subdural-hematoma-in-adults-etiology-clinical-features-and-diagnosis#H2

54. Patient is eating at a restaurant with his friend; he choked with a piece of meat, his friend who was an emergency assistant
performed Heimlich maneuver but without benefit. He then decided to perform a tracheostomy, during tracheostomy which
of the following structures will be cut?
A. Cricoid cartilage
B. Thyroid cartilage
C. Cricothyroid membrane
Answer: C.
(no reference for the answer )

55. What is the treatment of Aspirin toxicity?


A. A.Alkalinization of urine.
B. B.Gastric lavage.
C. C.Activated charcoal.

Answer: A. Alkalinization of urine. Reference: First Aid step 2 page 482

56. Patient with shock & Cherry red skin ;


A. Bacteremia
B. Septicemia
C. Carbon monoxidemia
Answer : c

57. Old man came to ER with SOB cough, fever, what is the next step of management
A. A.Put him in negative pressure room
B. B.Xray
C. C-Antibiotics
Answer:
X- Ray
Cough and fever with respiratory infection order CXR to notes any interstitial changes .
58. Healthy baby was in picnic with his family .. Suddenly he get SOB ..come to ER the chest osculation :RT wheezing in rt
hemithorax
What you will find in the CXR (foreign body),?!
A. -RT lower consolidation
B. Hyperinflation with midline shift. *
C. -RT Atlectasis

ANSWER:
-RT lower consolidation

59. Case of RTA and they found difficulty in intubation what should they do:
A. Just proceed to intubation whatever
B. Increase chin tilt .
C. Press on cricoids
142
Answer: Cricothyrotomy or flexible bronchoscopy. Reference: http://www.uptodate.com/contents/the-failed-airway-in-
adults#H351136572

60. Young man close the door on his nail color become blue under nail what will u do ?
A. Evacuate hematoma
B. Remove nail
C. Reassure ......
Answer: its according to the nail edges status. Evacuation is indicated in the presence of a painful subungual hematoma with the nail
edges intact.
Http://emedicine.medscape.com/article/82926-overview#a2

61. Man have 41 c temp with muscle spasm mangment ?


A. 
core cooling
B. 
syprying warm
C. 
warm IV

Answer:
management of fever :
Increase fluid intake
General : sponge bath , light clothing .
Acetaminophen /ibuprofen
Treat underlying cause

62. Head trauma on parietal lobe subdural hematoma which artery is injured ?
A. Superficial temporal .
B. Mid cerebral
C. Rt.cerebral .......
Answer : Middle cerebral artery supplies parietal lobe. However, subdural hematomas are most commonly caused by tearing of the
bridging veins

63. Patient ingested 20 gram of acetaminophen 8 hours ago, what is your next step?
A. Charcoal
B. N acetylcysteine
C. Gastric lavage
Answer: B
o Gastric lavage/emesis (if <2 h after ingestion)
▪ Oral activated charcoal (if the patient has a stable mental and clinical status and presents to the emergency department within 1
hour of ingestion)
• N-acetylcysteine (NAC, Mucomyst®) can be given PO or IV (most effective within 8-10 h of ingestion, but should be given no
matter when time of ingestion) > promotes hepatic glutathione regeneration

Draw a 4-hour serum acetaminophen concentration to determine the risk for hepatotoxicity, plot this value using the Rumack-
Matthew nomogram. Patients with acetaminophen levels below the “possible” line for hepatotoxicity on the Rumack-Matthew
nomogram may be discharged home after they are medically cleared. If the ingestion occurred with intent to do self-harm, a
thorough psychosocial, psychological and/or psychiatric evaluation is indicated before the patient can be discharged safely from the
medical care facility.
Admit patients with acetaminophen plasma levels above the possible line on the Rumack-Matthew nomogram for treatment with

143
N -acetylcysteine (NAC). Treat patients with evidence of hepatic failure, metabolic acidosis, coagulopathy, and/or encephalopathy in
an intensive care unit (ICU). Transfer patients with evidence of clinically significant hepatotoxicity to a medical facility with intensive
care support and organ transplant services.

Reference:
• Toronto Notes
• Http://emedicine.medscape.com/article/820200-treatment

64. Patient came to the ER with status asthmaticus. What will you use to intubate him?
A. Propofol
B. Ketamine
C. Midazolam

Answer: B
Studies suggest that ketamine and propofol have bronchodilatory properties and thus are suitable induction agents for the severe
asthmatic. Ketamine is preferable because it aids bronchodilation through both direct and indirect mechanisms and helps to
maintain blood pressure. Barbiturates, such as thiopental, should be avoided because they can exacerbate bronchospasm through
histamine release. Reference: http://www.uptodate.com/contents/emergency-airway-management-in-acute-severe-asthm
Toronto notes

65. Male patient presented to the ER after MVA, he has tachycardia, tachypnea, hypotension and cardiac arrhythmia. What
is your most correct next step is ?
A. FAST to check abdominal hemorrhage
B. Needle decompression
C. Pericardiocentesis
Answer: A

66. Anterior abdominal stab wound omuntam come through the wound ?
A. Fast
B. CT
C. Exploratory laparotomy
Answer: C

67. Patient work outdoor in temp 42 c .. Tired and Complain of crampy abdominal
Pain .. Lower limb pain and fever .. Otherwise normal ,, what to do ?
A. Electrolytes and oral replacement fluid
B. Warming
C. Cooling
Answer: A and C

68. Patient is bleeding the baro receptor activated result in increased tachycardia and decrease of ?
A. A-HR
B. B-Ventricular rate
C. C-Coronary artery flow
Answer: C

144
69. Patient came to you with history of bee sting with light heeded and shortness of breath what is the Rx ?
A. Oral histamine reassurance
B. Fluids and elevate the leg
C. Sc epinephrine , IV histamine
Answer:c

70. Man got a bee sting then his wife trying look for the epinephrine what it gonna inhibit?
A. Leukotriene release from macrophages
B. Cross reactivity with the cardiac..
C. Inhibit immunocomplex formation
Answer: inhibits tryptase release

71. Case of RTA and they found difficulty in intubation what should they do :
A. Just proceed to intubation whatever.
B. Increase chit tilt .
C. Press on cricoid
Answer: A


72. Patient has acute MI. Which of the following enzymes will be elevated?
A. Creatine kinase.
B. Alanine aminotransferase.
C. Alkaline phosphatase.
Answer: A

73. Patient had bee sting on the hand yesterday, he is presenting to you with redness and itching on the bite site. What is the
treatment?
A. Oral steroid.
B. Antihistamine.
C. Oral Antibiotic.
Answer: B

74. Patient with ischemic stroke presented after 6 hours. What is the treatment?
A. Tissue plasminogen activator.
B. Aspirin.
C. Clopedogril.
Answer : B

75. Patient came from RTA and the staff preparing him for intubation he devolved low ventilation on bivalve what to do ?
A. Proceed to intubation

145
B. Jaw thrust More head tilt
C. Cricoid pressure
Answer : A

76. Young healthy guy comes to ER after light headedness nausea and heavy breathing 20 mins before admission. What most
appropriate thing to do?
A. Alcohol concentration.
B. CT brain.
C. Other choices unrelated.
Answer: CT Brain. Alcohol intoxication as a cause of altered mental status is a diagnosis of exclusion and should be considered only
after ruling out more serious conditions such as head trauma, hypoxia, hypoglycemia, hypothermia, hepatic encephalopathy, and
other metabolic and physiologic derangements. 
Uptodate: http://www.uptodate.com/contents/ethanol-intoxication-in-adults#H4

77. RTA with hypovolemic shock signs , Hg low , what to give initially ?
A. Ringer lactate
B. Packed RBC
C. Whole blood transfusion
Answer: A

78. Known case of DM1 , last insulin time not known , was playing football , loos his consciousness for minutes , what will u give
him :
A. D5 with …
B. Ringer lactate with ..
C. Subcutaneus insulin

Answer: A?
Explanation: management of diabetic hypoglycemia: IV access and rapid blood glucose measurement. - D50W 50 ml IV push,
glucose PO if mental status permits. - if IV access not possible, glucagon 1-2 mg IM, repeat x 1 in 10-20 min. - O2, cardiac, frequent
blood glucose monitoring. - thiamine 100 mg IM. - full meal as soon as mental status permits
Reference: Toronto Notes 2015, page ER36, emergency medicine

79. Patient came to ER , airway patient gasping ,, u gave 2 breath by mask , pulse is rapid & week . What to do?!
A. Waiting code team
B. Put patient in recovery position
C. Do CPR

80. Patient present with pleuritic chest pain , examination shows decrease air entry in the right lower lung, attached chest x-ray
showing radiolucent right costophrenic angle, what is next?
A. Ventilation perfusion scan
B. Needle decompression
C. Chest tube

146
Answer: C
This is a case of hemothorax. Plain radiography of the upright chest may be adequate to establish diagnosis by showing blunting at
the costophrenic angle or an air-fluid interface if a hemopneumothorax is present.Chest pain and dyspnea are common
symptoms. It should be drained by tube thoracostomy.
Reference: http://emedicine.medscape.com/article/2047916-treatment

81. Sign of pericardial tamponade:


A. Pericardial rub
B. HTN
C. Muffled heart sound
Answer: c
Beck’s Triad
• Hypotension
• Increased JVP
• Muffled heart sounds

Classic Quartet of Tamponade


• Hypotension
• Increased JVP
• Tachycardia
• Pulsus paradoxus
Toronto note 2015

82. Patient came to ER with jaw fracture, plan for reduction and fixation, which of the following muscle forcefully closes the
jaw?!
A. Lateral prerygoid:
B. Masseter:
C. Temporalis:
When the lateral pterygoid contracts it pulls the articular disc and head of mandible forward onto the articular tubercle and is
therefore the major protruder of the lower jaw. When the lateral and medial pterygoids contract on only one side, the chin moves to
the opposite side. When opposite movements at the two temporomandibular joints are coordinated, a chewing movement results.
The masseter muscle is a powerful muscle of mastication that elevates the mandible.
The temporalis is a powerful elevator of the mandible. Because this movement involves posterior translocation of the head of
mandible from the articular tubercle of the temporal bone and back into the mandibular fossa, the temporalis also retracts the
mandible or pulls it posteriorly. In addition, the temporalis participates in side-to-side movements of the mandible.
Answer: B

83. Road accident presented with paralysis of 4 limbs and extremity pink and hot what's the shock?
A. Irreversable
B. Nurogenic
C. Hypotensive
Answer: B
147
84. In case of epistaxis what's first thing to do:
a. Insert tampon
b. Let him lye on his side lateral.
c. Pinch the fleshy part provide pressure.
Answer: C

85. In case of thyroid crisis what's the initial step:


A. Radioactive thyroid
B. Iodine thyroid
C. Propanolol
Answer: C
If there's one of the supportive measure choose it
Supportive measures:
If needed, immediately provide supplemental oxygen, ventilator support, and intravenous fluids. Dextrose
solutions are the preferred intravenous fluids to cope with continuously high metabolic demand.
Correct electrolyte abnormalities.
Treat cardiac arrhythmia, if necessary.
Aggressively control hyperthermia by applying ice packs and cooling blankets and by administering
acetaminophen (15 mg/kg orally or rectally every 4 hours).
Reference: Medescape

86. Patient brought to emergency department after a motor vehicle accident. He has weak heart sounds, silent right side of the
chest, distended neck veins. What is your next step?
A. Needle decompression
B. Cardiac window
C. Pericardiocentesis
Answer: A?
Unilateral absence of breath sounds, hyper resonance, tracheal deviation and raised JVP are signs of tension pneumothorax for
which needle decompression is indicated. However, the scenario seems incomplete. If the patient showed the full beck triad of
pericarditis (Hyoptension, muffled heart sounds and raised JVP), then a pericardiocentensis is indicated.
Toronto notes 2015

87. Why CT is best in blunt trauma:


A. is best in unstable patient
B. Better to detect on retro-peritoneum bleeding
Answer: B

148
88. Child girl obese try to suicide and eat alot of drug because of her friend, came to ER:
A. Immediate referral to psychiatry.
B. Treatment of acute depression.
Answer: A

89. Child swallowed battery , now it is lodged in esophagus . Ttt ?


A. 
Endoscopic removal
B. 
Wait ..
Answer:
Endoscopic removal

90. After paracetamole overdose 45 min



A. Charcoal✔

B. N acetyl cystine 

Answer: Usually start with Activated charcoal if the paracetamol ingestion is within 2 hours as it reduce the gastrointestinal
absorption for paracetamol, then give the antidote for paracetamil (N-acitylcystine) if the plasma paracetamol level plotted on or
above the line drawn between 100 mg/L at 4 hours and 15 mg/L at 15 hours after ingestion
Reference: http://patient.info/doctor/paracetamol-poisoning

91. CPR in child according to American heart association in presence of 02 rescuer:



A. 15 compression and 2 ventilation
B. 30 compression and 2 ventilation

Answer :
30 compression and 2 ventilation

92. Young female stayed out in sun at 42 degree .. She came to ER later tired.. Muscle cramp.. Vitally stable except T: 38 how to
manage?

A. Normal sline
B. Cold pack
Answer: Heat cramps treatment involves rest in a cool envirmoment with oral fluids, normal saline IV is reserved in case of
orthostatic hypotension. Reference: Toronto notes.

93. Patient presented with mild burn, what to do?


A. Apply cold water (or ice).

B. Apply butter.

Answer :
first degree
Treatment aimed at comfort
Topical creams (pain control, keep skin moist) ± aloe
Oral nsaids (pain control)

94. Basal skull fracture cavernous sinus affected>>not sure, which muscle is intact?
A. Trapezius
B. Sternocliedmastoid
Answer :

149
95. Child ate a number of iron tablets presented with severe symptoms including constipation and bloody stool nausea and
vomiting and drowsiness how would you treat him:
A. IV deferoxamie.
B. Dialysis
Answer :
Http://www.uptodate.com/contents/chelation-therapy-for-thalassemia-and-other-iron-overload-states?Source=related_link

96. Patient with Digoxin toxicity, what will order ? Same Q188 page 523 7th update but not choises
A. Lidocaine
B. Fb immunoglbulin
Answer:
Http://emedicine.medscape.com/article/154336-treatment

97. Patient ate from a restaurant. 2 days after that started to complain of diarrhea, vomiting and urticaria.
A. Food poisoning
B. Food allergy
Answer: ?
Food poisoning: http://emedicine.medscape.com/article/175569-overview
Food Allergy: http://emedicine.medscape.com/article/135959-overview

98. A guy presents in emergency with decreased oxygen carrying capacity of blood cells. Suicidal attempt is suspected. What is
the culprit?
A. Carbon monoxide
B. Cyanide

Answer: A

99. Soldier tries to have an excuse from the military, presented to the ER with symptoms of tremors, hypoglycemia. What is he
using?
A. Factitious injection of insulin.
B. Metformin
Answer: A

100.Acute attack of cluster headache, what is the abortive treatment?


A. Oxygen 100%
B. Subcutaneous Sumatriptan

Answer: A
Both Oxygen 100% and Subcutaneous Sumatriptan are effective. Reference: uptodate and toronto
Abortive Therapy for Cluster Headache: ergotamine, triptans (sumaptriptan, eletriptan, almotriptan, zolmitriptan), 100% O2,
prednisone, lithium. Prophylaxis: Verapamil
Abortive Therapy for Migraine Headache: ergotamine, triptans (sumaptriptan, eletriptan, almotriptan, zolmitriptan). Prophylaxis
(Given when there is 3 or more episodes per month): Propranolol (Best), CCB, TCA, SSRI, Topiramate, Botulinum toxins injections.
Reference: Master the Boards

150
101.Person escaped from fire in his flat from 3rd floor and jumped through the window. What will be the most life saving in this
case?
A. Maintain Airway.
B. Call for help.
Answer: B

102.4 members in the family developed nausea and diarrhea 8 hours after eating at a restaurant. Then after 48 hours they
improved. Stool test showed oxidase positive gram +ve bacilli. What is organism?
A. Shigella
B. Salmonella

Answer:
Shigella (gram –ve)
Salmonella (gram –ve)
Bacillus cereus is a Gram-positive aerobic or facultatively anaerobic, motile, spore-forming, rod-shaped bacterium that is widely
distributed environmentally. B. Cereus is associated mainly with food poisoning. Reference:
http://www.ncbi.nlm.nih.gov/pmc/articles/PMC2863360/
Reference: Toronto Notes

151
103.Depressed patient with HTN Brought by family to ER for drug overdose palpitation diaphoresis and ECG shows arrhythmia.
What is the possible drug ?
A. SSRI
B. Digoxin
Answer: A

152
104.Old patient with high cholesterol level measured before 4 months and he is on a diet. Came to ER complaining of chest pain.
What will concern you?
A. Current symptoms
B. Cholesterol level before 4 m
Answer: A

105.Man got bee sting ,, his wife gave him epinephrine .. What is the Immunologic action of epinephrine ?
A. Iterlukine..... Ect
B. Tryptanase .....ect
Answer: inhibits tryptase release

106.Patient with ECG showing bradycardia, what should you give?


A. Atropine.
B. Cardioversion.
Check the ACLS Algorithm .

153
107.In emergency department patient come with closed head trauma and loss of consciousness what is the first thing to do:
A. Intubation and hyperventilation
B. Assess airway
Answer: B

108.Baby with unknown numbers of paracetamol pills ingestion presented after 10 or 4 hours what to do :
A. N.ascetylecystin
B. Drug level

Answer: B
Explanation: The serum acetaminophen (APAP) concentration is the basis for diagnosis and treatment. It is important to measure,
even in the absence of clinical symptoms, because of the delay in onset of clinical toxicity. After a single ingestion, N -acetylcysteine
(NAC) therapy is guided by the serum APAP concentration. APAP levels 4 hours post ingestion >150 mcg/ml (>993 µmol/L) reflects
possible toxicity.
Reference: http://emedicine.medscape.com/article/820200-workup

109.DKA pt if fluids corrected rapidly. What's the complication?


A. brain edema
B. central potine mylinolysis
Answer

110.Patient ingested multiple drugs what should you do? (did not give time frame)
A. Gastric lavage
B. Activated charcoal
Answer: B
Reference: http://cursoenarm.net/UPTODATE/contents/mobipreview.htm?9/4/9281#H11055967

111.Case of opioid toxicity which to do first?


A. Iv naloxone
B. intubation
it depends on scenario of the case if airway is at risk then you must intubate (http://emedicine.medscape.com/article/815784-
treatment)

112.case of acetaminophen overdose since 45 min, pt conscious, what the appropriate next step:
A. Give activated charcoal
B. N acetylcysteine
Answer: A
Administer activated charcoal (AC) if the patient has a stable mental and clinical status, patent airway, and presents to the
emergency department within 1 hour of ingestion.

154
Reference: http://emedicine.medscape.com/article/820200-treatment

113. Overdose of aspirin can cause:


A. Diarrhea
B. constipation
Answer: B
Patients with mild intoxication frequently have nausea and vomiting, abdominal pain, lethargy, ringing in the ears, and dizziness.
More significant signs and symptoms occur in more severe poisonings and include high body temperature, fast breathing rate,
respiratory alkalosis, metabolic acidosis, low blood potassium, low blood glucose, hallucinations, confusion, seizure, cerebral edema,
and coma. The most common cause of death following an aspirin overdose is cardiopulmonary arrest usually due to pulmonary
[
edema.
https://en.wikipedia.org/wiki/Aspirin_poisoning

114. Pt presents with black vomiting after ingesting large amount of medication (suicidal attempt). What is the most likely
medication?
Answer: aspirin overdose

Aspirin overdose can cause coffee ground vomitus.

Reference: http://www.healthline.com/health/coffee-ground-vomitus#Causes2

115. What is the antidote for paracetamol toxicity?


A. N-acetylcysteine

Answer: A
Reference: http://emedicine.medscape.com/article/820200-treatment

116.High K wide QRS, what to do?


Give Ca gluconate
((Toronto Notes 2015 , NP13 ))

117. Bee sting treatment?
For local reactions:


• Provide supplemental oxygen
• Diphenhydramine limits the size of the local reaction.
155
• Clean the wound and remove the stinger if present.
• Apply ice or cool packs.
• Elevate the extremity to limit edema.

if generalized reactions developed; Treatment should include an initial intravenous (IV) bolus of 10-20 ml/kg isotonic crystalloids in
addition to diphenhydramine and epinephrine. 
Reference: http://emedicine.medscape.com/article/768764-treatment#d9

118.Child ingested iron what is treatment:


Answer: Whole bowel irrigation (WBI) is an effective method of GI decontamination for serious iron overdose in addition IV
deferoxamine therapy. Uptodate. 
Gastric lavage is NOT recommended because iron tablets are relatively large and become sticky
in gastric fluid, making lavage unlikely to be of benefit.

119.Female came to ER .. Agettated.. Pupil is dilated.. She is toxic with what?



A. Organophosphate..


Answer: agitation and dilated pupils (mydriasis) are characteristic for anticholinergics toxicity e.g. Atropine, TCA, carbamazepine.
Reference: Toronto notes, Emergency medicine.

120.At ER patient came with toxicity of acetaminophen for 3 hours .. What is the management?
Answer: Usually we start with activated charcoal to reduce the acetaminophen absorption if within 2 hours but since this patient
came after 3 hours we give directly the antidote N-acetylcysteine (NAC) to protect the liver. Reference: Medscape

121.Patient had trauma, presented with tachycardia , bradypnea and hypotension , what is the first thing you will do ? Needle
thoracotomy
Answer: Tension pneumothorax is suspected. So Immediate attention to the abcs (airway, breathing, circulation) while assessing
vital signs and oxygen saturation is paramount, particularly in patients with thoracic trauma. Definitive Management includes
Needle thoracostomy at 2nd intercostal space at Mid clavicular line followed by chest tube at 5th intercostal space at anterior
axillary line. Reference: Toronto notes.

122.Patient MVA and come with fracture of femur , tibia and fibula what is your action :
A- refer to orthopedic
Answer:
Parenteral analgesia should be administered when appropriate.pen fractures must be diagnosed and treated appropriately (by
orthopedician) Tetanus vaccination should be updated, and appropriate antibiotics should be given in a timely manner.
Http://emedicine.medscape.com/article/826304-treatment#d10

123.What medication you can give to prevent haluin withdrawal symptom ?


• Methadone
156
Answer:
Http://emedicine.medscape.com/article/287790-treatment

124.Patient came with Oragnophosphare intoxication, how to Treat?


Answer: give Atropine
Http://emedicine.medscape.com/article/167726-treatment

125.Common Antidotes :

126.Glasgow coma scale

127.Patient dose not complain of anything ,, has sudden knee swelling ? What is the best thing to do ?
Answer:
If knee swelling associated with pain , stiffness this indicates inflammatory cause , should role out life threatening condition as septic
joint .

128.Patient after road traffic accident developed chest pain. On examination: no lung sound and hyperresonance on one side.
What is the 1st thing to do?
A. Needle decompression

Answer: A
Tension pneumothorax

129.Treatment of bee sting.


Answer:
ABC management, epinephrine 0.1 mg IV over 5 min if shock, antihistamines, cimetidine 300 mg IV/IM/PO, steroids,
Β-agonists for SOB/wheezing 3 mg in 5 ml NS via nebulizer, local site management (ice packs). Reference: Toronto Notes and
Medscape.

130.Aspirin toxicity treatment


• Alkalinization of urine

Answer: A
Alkalinize the urine, use charcoal and (dialysis in severe cases). Master the Boards

131. What is the antidote for organophosphate poisoning?


• Atropine
Answer: A and pralidoxime
Reference: Master the Boards.
Organophosphate poisoning signs and symptoms in the mnemonic DUMBBELS: Diarrhea, Urination, Miosis, Bradycardia,
Bronchorrhea, Emesis (NV), Lacrimation, Salivation/Sweating.

132.Trauma due to MVA. The patient has spinal injury with hypotension and bradycardia. What is the type of shock?
A. Neurogenic

157
Answer: A

133.Contraindication for gastric lavage?


Answer:
• Initial resuscitation incomplete
• Risk assessment indicates good outcome with supportive care and antidote therapy alone
• Unprotected airway where there is a decreased level of consciousness or risk assessment indicates potential for these
complications during the procedure
• Small children
• Corrosive ingestion
• Hydrocarbon ingestion
Reference: http://lifeinthefastlane.com/ccc/gastric-lavage/+
Https://www.clintox.org/documents/positionpapers/gastriclavage.pdf

134.Aspirin toxicity resulting in which of the following?


• Respiratory alkalosis followed by metabolic acidosis
Answer: A
Also, renal insufficiency, elevated prothrompin time, confusion, seizure, coma and fever. Master the Boards

135.Young patient come to ER with palpitation, euphoria, visual hallucinations,


A. Amphetamin poisining

Answer: A
CLINICAL FEATURES — Clinicians should consider the diagnosis of methamphetamine intoxication in any diaphoretic patient with
hypertension, tachycardia, severe agitation, and psychosis. Acutely intoxicated patients may become extremely agitated and pose a
danger to themselves, other patients, and medical staff
Ref :uptodate

136.After intubation still low in spo2 wt u will do


Answer:
Causes of hypoxia occurring soon after intubation: Think DOPES:

Displacement of the endotracheal tube (ETT)
Obstruction of the
ETT
Patient — especially pneumothorax; also: pulmonary embolism, pulmonary edema, collapse, bronchospasm
Equipment —
ventilator problems
‘Stacked breaths’ — a reminder about bronchospasm and ventilator settings.
First step in managing an intubated and ventilated patient who is hypoxic: Disconnect the ventilator and administer high-flow 100%
oxygen using a bag-valve-mask.

137.Atropine side effect ? Dry mouth


Answer: Dry mouth, blurred vision, photophobia, glaucoma, constipation, urinary retention, and drowsiness.

138.Which type of insulin used in DKA ?


Regular ( short acting )
Answer: short acting
Initial bolus of 5-10 U short-acting/regular insulin (or 0.2 U/kg) IV in adults (controversial – may just start with infusion)
Followed by continuous infusion at 5-10 U (or 0.1 U/kg) per h. Toronto notes

158
139.Patient in ER MVA. First thing you do?
ABC
Answer: ABC

140.Gunshot with wound bowel perforation, What antibiotics you should give
Http://emedicine.medscape.com/article/195537-treatment#d9

141.Patient had bee sting and presented to you immediately with shortness of breath and lightheadedness. What is the
treatment?
• SC epinephrine.
Answer: im epinephrine. Master the boards

142.What is the degree of shock in this patient?

143.Patient with multiple trauma post MVA, vitally unstable. First step:
A-IV fluid

144.Patient presented to the ER, unable to talk, his face is blue, what is the next step in the management?
A- open mouth check for any foreign body.
Answer: A

145.Case of burn, how much fluids you will give in the 1st 8 hours? Answer:

159
146.Multiple sclerosis Present to ER what to give?
Answer : IV methylprednisolone

147.Case of acetaminophen overdose since 45 min, pt. Conscious, what the appropriate next step:
Give charcoal N acetylcysetine
Www.uptodate.com/contents/acetaminophen-paracetamol-poisoning-in-adults-treatment

148.Female c/o headache , she described it as the worst headache in her life ?
Ruptured berry aneurysm

Explanation: The central feature of classic Subarachnoid hemorrhage is sudden onset of severe headache (thunderclap headache),
often described as the "worst headache of my life." Intracranial saccular aneurysms (“berry aneurysms”) represent the most
common etiology of nontraumatic SAH; about 80% of cases of SAH result from ruptured aneurysms.
Reference: http://emedicine.medscape.com/article/1164341-overview#a3

149.4 years old boy was found on the floor of the kitchen , cyanosed , has peanut bean on his hand :
Aspiration

Explanation: Children are at risk for putting small toys, candies, or nuts into their mouths. Children aged 1-3 years chew incompletely
with incisors before their molars erupt, and objects or fragments may be propelled posteriorly, triggering a reflex inhalation.
Patients may be asymptomatic or may be undergoing testing for other diagnoses. If present, physical findings may include stridor,
fixed wheeze, localized wheeze, or diminished breath sounds. If obstruction is severe, cyanosis may occur.
Reference: http://emedicine.medscape.com/article/298940-clinical#b4

150.Road traffic accident came to ER not feeling any of his four limbs and limbs are pink and warm
Vital signs: Low pulse and BP, SpO2 96%
What type of shock?
Answer:

160
May be (neurogenic shock) resulting from spinal cord injury after the road traffic accident. Mont Reid: mechanism: loss of vasomotor
control, expansion of venous capacitance bed with peripheral pooling of blood
Manifestations: warm well perfused skin, low BP, bradycardia may be present.

151.An elderly patient presented to ER due to decreased level of consciousness, lethargy


PCO2: 20 mmhg
K: 2
pH: 7.2 (can't remember the rest of the labs and choices)
What does she have?
A. High anion gap metabolic acidosis with primary respiratory alkalosis

Answer: A. We can't be sure about the anion gap without the rest of the lab result otherwise the answer is correct

152.Vertical and horizontal nystagmus associated with?


A. Phencyclidine toxicity
Answer: A
Phencyclidine toxicity: Nystagmus (horizontal, vertical, or rotary) - Rotary often considered a hallmark of PCP intoxication (57-89%)
Http://emedicine.medscape.com/article/816348-clinical#showall

153.Child ingests unknown numbers of paracetamol, present to the er after 4 hours stable. What to give;
A. N acetylcysteine
http://emedicine.medscape.com/article/820200-overview

154.Digeorge syndrome ?
A. Thymic (some thing)
Reference: http://emedicine.medscape.com/article/886526-overview

155.head trauma with low urine osmolality?


Answer: central di can be an acute or chronic complication of head injury or subarachnoid hemorrhage.
Reference: medscape

161
162
Dermatology

163
1. A beach guard who stays most of the time under the sun, presented to you complaining of new papules over his nose and
cheeks. What is the most likely diagnosis?
A - Melasma.
B - Hamartoma.
C - Freckling.
D - Actinic keratosis.

Answer: D
Evidence: Premalignant lesions caused by sun exposure. Small, rough, erythematous or brownish papules or plaques. Risk of
squamous cell carcinoma is proportional to degree of epithelial dysplasia.
Reference: FA USMLE 1

2. A patient with a typical history of herpes zoster infection with dermatomal involvement (other version: Rash in the
dermatomal distribution of the trunk). (no stem of Q)
A - HSV.
B - Herpes Zoster.
C - Bacterial vaginosis.
D - HPV.

Answer: B

3. Patient with psoriasis involving 15% of his body with nail involvement. What is the best treatment for him?
A - Laser.
B - Topical steroid.
C - Topical vitamin D analogs.
D - Methotrexate.

Answer: D [Methotrexate can be used in severe psoriasis (involving more than 10% of body surface)]

4. Picture of a woman's face with comedones and papules on the cheek. Which of the following drugs should be avoided?
A - Retinoic acid.
B - Erythromycin.
C - Tetracycline.
D - Steroids.

Answer: D

5. Description of a man with red nodules and papules on the face involving mainly the cheeks and nose. Few telangiectasia’s are
also present. What treatment will you suggest?
A - Cold compresses
B - Oral doxycycline
C - Topical retinoic acid
D - Topical steroids

Answer: B (It’s a case of rosacea.)

164
6. 18 years old girl with vitiligo on the face and arm that is symmetrical started on medical treatment 3 years ago. She wants to
get married soon and wants the lesions go away. What will you do for her?
A - Split thickness graft
B - Continue medical treatment
C - Melanocyte transfer
D - Stop med and observe

Answer: B

7. A long scenario about a rash over the elbows, knees, and cheeks that is itchy, weeps and crusts which got better with
steroids. What is the diagnosis?
A - HSV
B - Staph
C - Fungal
D - Eczema

Answer: D
8. A patient complaining of itchiness that increases at night, superficial linear burrows around 4th & 5th digits, inflammatory
papules and nodules.
A - Atopic dermatitis
B - Allergic dermatitis
C - Scabies
D - Tinea

Answer: C

9. Which of the following drugs is used for acne rosacea: (answer not known)
A - Erythromycin
B - Clindamycin
C - Cephalexin

Answer: ?
Evidence:
- For mild rosacea: We can use topical metronidazole or azelaic acid creams and gels.
- For severe rosacea: doxycycline (first line therapy), tetracycline and erythromycin.
Reference: http://www.nhs.uk/Conditions/Rosacea/Pages/Treatment.aspx

10. Treatment of pyoderma gangrenosum?


A - Methotrexate.
B - Antibiotics.
C - Systemic Steroids.
D - Surgery
Answer: C | Reference: http://emedicine.medscape.com/article/1123821-treatment

165
11. Condyloma lata is caused by?
A - HSV.
B - HPV.
C - Spirochete Treponema.
D - Gonorrhoea.

Answer: C
(A Smooth, moist, painless, wart-like white lesions on genitals caused by spirochete Treponema pallidum in secondary syphilis)

12. Condyloma acuminata is caused by?


A - HSV.
B - HPV.
C - Spirochete Treponema.
D - Gonorrhoea.

Answer: B (Commonly HPV 6 and 11)

13. 30 years-old man came with erythema of the nose and pain in the right eye, there is erythema and nodules in left periorbital
area and forehead. What is the most likely diagnosis?
A - Systemic lupus Erythematosus.
B - Herpes zoster.
C - Rosacea.
D - Measles.

Answer: B

14. Schoolboy brought by his mother, he has 2x2 hair loss in the temporal area, hair around this area-clubbed hair. What is the
most likely diagnosis?
A - Trichotillomania.
B - Alopecia areata.
C - Tinea infection.
D - Telogen effluvium

Answer: D
15. (long scenario) 4 years old child presented with area of 3x3 hair loss, on examination of the area there is multiple pustule.
What is the most likely diagnosis?
A - Aplasia cutis congenita.
B - Staphylococcal infection.
C - Trichotillomania.
D - Tinea infection.

Answer: B
Evidence: (staph infection alone won't cause hair loss, its only cause pustules so in this case it's look like infection on top of an
original Diagnosis which had caused the alopecia which is missing in this scenario, if the child born with areas of alopecia the

166
diagnosis would be Aplasia cutis congenita which later could be complicated by secondary infection). Reference:
http://dermnetnz.org/lesions/aplasia-cutis.html

16. Patient with acne came with comedones and presence of pustules, what type of acne is it?
A. Obstructive
B. Inflammatory
C. Infectious
D. Obtrusive
Answer: B
Comedonal acne is obstructive and non-inflammatory. Papulopustular acne is inflammatory acne.

17. Best treatment of Pyoderma Gangrenosum?


A. Plasmapheresis
B. Steroid
C. Oral Antibiotics
D. Methotrexate
Answer: B
http://emedicine.medscape.com/article/1123821-treatment

18. Patient presented to hospital with 5 days of generalized skin eruption including soles & palms, what is the cause?
A. Drug Induced Reaction
B. Pityriasia
C. Erythema Nodosum
D. Erythema Marginatum
E. Erythema Multiform

Answer: E
http://bestpractice.bmj.com/best-practice/monograph/367/diagnosis/history-and-examination.html

19. Pyoderma gangrenosun treatment?


A. Steroid
B. D- pencillamin
C. Aspirin
D. Paracetamol
Answer: A
Therapy for pyoderma gangrenosum involves the use of anti-inflammatory agents, including antibiotics, corticosteroids,
immunosuppressive agents, and biologic agents. The prognosis is generally good; however, the disease can recur and residual
scarring is common.
http://emedicine.medscape.com/article/1123821-overview
http://www.dermnetnz.org/reactions/pyoderma-gangrenosum.html

20. Smoker came with whitish patches. What to do?


I think: Stomatitis nicotina
A. Excision

167
B. Antibiotic
C. Nothing to do
D. Close follow up

Answer: C or mostly D (both could fit because if the patient doesn't stop smoking leukoplakia will not disappear! Biopsy and follow
up needed if the patient don't stop smoking)

The only definitive treatment for nicotinic stomatitis is smoking cessation.

Reference: Stomatitis nicotina: Medscape. Wikipedia.

21. First line of defense in the skin is (other version: “First immunologic defense of skin is”):
A - mucous membranes
B - collagenous cell (other version: Blast cells)
C - keratinocytes.
D - areolar connective tissue (other version: melanocytes)

Answer: C

22. Patienthave nodulo-cystic acne, how to manage?


A - Clindamycin
B - Oral abx
C - Erythromycin
D - Isotrention

Answer: D
Evidence:
- Type I: comedonal, sparse, no scarring
- Type II: comedonal, papular, moderate to little scarring
- Type III: comedonal, papular, and pustular, with scarring
- Type IV: nodulocystic acne, risk of severe scarring (severe)

23. Oropharyngeal maculopapular rash. Also rash in palm and foot. Diagnosis?
A - CMV
B - EBV
C - coxsackie virus
D - Vaccina virus

Answer: C [typical description of hand, foot and mouth disease (coxsackie virus A16)]
168
Reference: http://www.uptodate.com/contents/hand-foot-and-mouth-disease-and-herpangina-an-overview

24. Child with maculopapular rash in face and nose, whites spot inside cheeks: (missing details of Q, answer not sure of)
A - mumps
B - measles
C - hsv1
D - vercialla zoster virus

Answer: B? (depends on vesicles: if present VZV; if absent measles + white spot inside mouth could be (koplike spot)
measles)

25. Case of Contact dermatitis. What is the Tx:


A - Topical steroid
B - Systemic steroid
C - systemic antibiotic
D - fluconazole

Answer: A

26. 21 y.o came with rash in all the body excepatientface, rash including palm and soles, had unprotected sex 8 weeks ago:
A - chancroid
B - chlamydia (other version: herpes)
C - syphilis
D - (other version: Leishmaniasis)

Answer: C (Common manifestation is rough reddish brown dots either to one part of the body or all specially bottom of feet & palm)
Evidence: List of rashes involved Palm & soles: Meningococcemia, Keratoderma blenorrhagica (Reiter’s syndrome), Typhus, Acral
lentiginous melanoma, Hand, Foot, and Mouth Disease, Mercury poisoning in children, Bacterial endocarditis, Tylosis, Rocky
Mountain Spotted Fever, Graft Versus Host Disease rash, Kawasaki, Measles, Toxic Shock Syndrome, Steven Johnson syndrome,
Secondary and Congenital Syphilis with its characteristic Copper colored rash.

27. Patientwith psoriasis, took a medication then developed generalized psoriasis covering all his body surface, what is the
percentage of the involved body surface?
A- 30%.
B-50%
C- 70%
D- 90%
Answer:D
Pustular psoriasis
Reference: http://www.kevinmd.com/blog/2014/05/mksap-64yearold-man-rapidly-spreading-rash.html

169
28. Hemangioma in the lift eye in an infant and it needs to be resected so it doesn't affect the vision, when will u do that? I
don’t know ! I didn’t answer anything
A. Immediately
B. 2 weeks
C. 3 months
D. 6 months

29. Patient with macula hypopigmentation and no history of chronic disease ?


A. Vitiligo
B. Albinism
C. Psoriasis
D. Melanoma
Ocular albinism 1: OA1 (X-linked recessive OA/Nettleshop-Falls type) involves the eyes only. Patients with OA 1 have normal
skin; however, it may be paler than first-degree relatives. Ocular findings in OA 1 are similar to those of OCA, with decreased visual
acuity, refractive error, fundus hypopigmentation, absent foveal reflex, strabismus, iris translucency, and posterior embryotoxon in
30% of patients (implying anterior segment dysgenesis).

Http://emedicine.medscape.com/article/1200472-clinical

30. Patientwith genitalia wart treated then she complains from pain with intercourse & bleeding the wart recurrent >> what the
source of bleeding??
A. Vulva
B. Vagina
C. Body uterus
D. Cervix

31. Baby with diaper rash, no satellite lesion ?


A. Barrier cream
B. Chang diaper frequntly
C. Steroid cream
D. Answer: not sure

While there are a several broad categories of causes of diaper rash, contact irritation is the most common
culprit. Effective treatments include frequent diaper changes, application of topical barriers (for example,
petroleum jelly), and rarely topical antibiotic/antifungal ointments, or low-potency hydrocortisone cream.
High-potency steroid creams, powders, and concentrated baking-soda/boric-acid baths and neomycin-
containing ointments are to be avoided.
Http://www.medicinenet.com/diaper_rash/article.htm
170
32. Itching only & CASE OF LADY WITH COMPLAINT OF pursuits .. Physical examination not towards specific infection What to do
?
A. Referral to sexual transmitted disease
B. Re evaluate if symptomatic with inflammatory manifestation
C. She is okay not need to come
D. Give treatment for all possible infections
Answer:

33. Contact with moulds in a new apartment her son develops a Rash in his hand and resolve completely, after a while he
develop multiple rash?
A-Maculo papular
B-wheal (urticaria)
C-Plaque
D-Cup
Answer: B

34. Child who has to have itchy papules, started as 1 papule then spread to the whole body, what is the treatment?
A. Steroid
B. Acyclovir
C. Antibiotics
D. Antiseptic
Answer: A
Pityriasis rosea (also known as pityriasis rosea Gibert ) is a skin rash. It is benign but may inflict substantial discomfort in certain
cases. Classically, it begins with a single "herald patch" lesion, followed in 1 or 2 weeks by a generalized body rash lasting up to 12
weeks. Treatment topical steroids.
Pityriasis rosea (PR) is a self-limited disease; treatment is supportive. The rash usually disappears in a few weeks, with no
sequelae. Pruritus is commonly associated with PR and often responds to topical zinc oxide and calamine lotion, oral antihistamines.
If the rash is severe, topical steroids can be applied. Systemic steroids are not recommended because they may exacerbate the
disease. Acyclovir may hasten resolution, especially if given within 1 week of rash, but the data are not conclusive.
Http://emedicine.medscape.com/article/1107532-overview

35. Diabetic woman with hypothyroidism developed oral white plaque (thrush) for long time with recurrent rash Ppd test
positive with old exposure What's the diagnosis :
A-Chronic mucocutanious candida infection
B-hypothyroid
C-Igm deficiency Digeorge disease
D-Chronic granulomatous ....
Answer: a
Reference: http://emedicine.medscape.com/article/1091928-clinical#showall

36. Hiv pt come with diffuse pasutle in skin an mouth tt by :


A. Topical strerid
171
B. Oral ab
C. Topical ab
D. Chemo & radiontherapy
Kaposi sarcoma is common in suppressed immunity patients like in hiv.

37. Treatment of Pyoderma Gangrenosum?


A. Corticosteroid
B. Methotrexate
C. Antibiotic
Answer: A
Uptodate: most patients with PG cannot be managed with local therapy alone. Systemic treatment is used as a first line intervention
in PG that is not limited to a few superficial ulcers. The systemic therapies utilized in PG are immunosuppressive and
immunomodulatory agents that calm the inflammatory process to allow for wound healing. Systemic glucocorticoids often induce a
rapid response and are typically used as initial therapy.

38. Women has lichen sclerosis which cancer she will get:
A. squamous cell carcinoma
B. Adenocarcenioma carcinoma
C. Adeno-squamous carcinoma
Answer: C
Squamous cell carcinoma.
Lichen sclerosus (LS) is a chronic inflammatory dermatosis that results in white plaques with epidermal atrophy and scarring. Lichen
sclerosus has both genital and extragenital presentations and also goes by the names lichen sclerosus et atrophicus (dermatological
literature), balanitis xerotica obliterans (glans penis presentation), and kraurosis vulvae (older description of vulvar presentation). An
increased risk of squamous cell carcinoma may exist in genital disease.
(http://emedicine.medscape.com/article/1123316-overview)

39. pic / pink patch under collar lumb:


A. Erythrasma
B. Fungal infection
C. Vetiligo
Answer:
The patches of erythrasma are initially pink, but progress quickly to become brown and scaly (as skin starts to shed), which are
classically sharply demarcated. Erythrasmic patches are typically found in intertriginous areas (skin fold areas - e.g. armpit, groin,
under breast).

40. Treatment of pyoderma gangrenosum


A. Methotrexate.
B. Antibiotics.
C. Systemic Steroids.
Answer: C

172
No specific therapy is uniformly effective for patients with pyoderma gangrenosum. In patients with an associated, underlying
disease, effective therapy for the associated condition may be linked to a control of the cutaneous process as well.
Topical therapies include gentle local wound care and dressings, superpotent topical corticosteroids, cromolyn sodium 2% solution,
nitrogen mustard, and 5-aminosalicylic acid.
Systemic therapies include corticosteroids, cyclosporine, mycophenolate mofetil, azathioprine, dapsone
Reference: http://emedicine.medscape.com/article/1123821-treatment

41. A case of a patient diagnosed to have cutaneous leishmania or Baghdad boil type which type of leishmania?
A. Kalazar
B. Donovani
C. Tropica

Answer : C
Reference: http://emedicine.medscape.com/article/220298-overview#a4

42. What is the main treatment for non-inflammatory acne (other version: First line treatment for non-inflammatory acne)?
A - Azelaic acid
B - Isotretinoin
C - Differin cream

Answer: B or C? (answer not sure of) (the best is isotretinoin but we should start with adapalene “diffrin cream” first)
References: European Handbook of Dermatological Treatments
According to SLE made easy isotretinoin is the main treatment for non-inflammatory acne

43. Old lady with migraine on medications. She’s been stable for 3 years till she went to ophthalmology to treat blepharitis with
eye drops and developed rash, big nose and big mouth.
A. Seborrheic dermatitis
B. Contact dermatitis
C. Rosacea
Answer: B

44. Commonest site for Lichen planus is:


A. Mouth
B. Abdomen
C. Inner wrist
Answer: C

45. Vesicular maculopapular rash on the trunk and also hands and feet?
A. HSV
B. CMV
C. RUBELLA
Answer: A ?? VZV: Chickenpox: Initially, papules, which evolve into vesicles (“dewdrops on a rose petal”) and eventually into
pustules and crusts; rash beginning on face and spreading inferiorly to trunk and extremities

173
46. Black spot at the sole of foot that pared with scalpel, what is the diagnosis?
A. Verruca
B. Heloma
C. Tyloma

Answer: A
http://www.foothealthfacts.org/footankleinfo/Plantar_Wart.htm

47. HIV patient came with diffuse pustules in skin and mouth, treatment is by:
A. Topical steroid
B. Oral AB
C. Topical AB
Answer: B

48. Patient on long term use of topical steroids. What is the most likely side effect? (missing options)
A - Scaling
B - Atrophy

Answer: B

49. What is the treatment for moderate to severe acne vulgaris?


A - Isotretinoin
B - Tetracycline
C - Clindamycin

Answer: A (Refer to the table at the end of Dermatology Section)

50. Patient with Positive HPV. What is associated with it?


A - Hyperkeratosis
B - Parakeratosis
C - Apoptosis

Answer: A

51. Patient with single hypopigmented lesion on the forearm, with Ulnar nerve thickness, what is the diagnosis:
A - Vitiligo
B - Amyloidosis
C - TB

Answer: I think it's associated more with leprosy because leprosy can present with Hypopigmented or reddish skin lesion with
Involvement of the peripheral nerves: Demonstrated by definite thickening of the nerve with/ without loss of sensation and/or
weakness of the muscles of the hands, feet or eyes supplied. While vitiligo rarely to be associated with nerve thickness
Reference: http://nlep.nic.in/pdf/manual3.pdf

174
52. Patient with a typical history of headache, fever and then rash. Which type of herpes is this:
A - HSV type 1
B - HSV type 2
C – Varicella

Answer: C (chickenpox)

53. A patient presented with macular papular rash and fever no vaccination before, Diagnosis? (answer not known)
A - Measles
B - Mumps
C - Rubella

Answer: A or C?
Evidence:
“Although the distribution of the rubella rash is similar to that of rubeola “measles”, the spread is much more rapid, and the rash
does not darken or coalesce”
Rubella: low grade fever, pruritic, lymphadenopathy, forchhiemer spots,
Measles: high grade fever, non pruritic, 3 Cs (cough, coryza, conectivitis), koplik spots
Mumps: low-grade fever, malaise, headache, myalgias, and anorexia. These symptoms are generally followed within 48 hours by the
development of parotitis
Reference: uptodate

54. How to decrease the keratosis by?


A - Wound trauma
B - Use abx
C - Avoid sun exposure

Answer: C (answer not sure of)

55. A patient diagnosed with cutaneous leishmania or bghlabar/bhagdad type, which type of leishmania? Or what’s the
organism?
A - Tropica
B - donovani
C - Post kala-azar dermal leishmaniasis

Answer: A
Evidence:
- Cutaneous leishmania (tropica, major, aesrhiopica, mexicana)
- Mucucutaneous leishmania (brazilienisis)
- Visceral leishmania (donovani, infantum ,chagasi)
References:
Http://www.uptodate.com/contents/image?Imagekey=ID%2F86988&topickey=ID%2F5689&rank=1%7E93&source=see_link&search
=leishmania&utdpopup=true

175
- http://www.uptodate.com/contents/clinical-manifestations-and-diagnosis-of-cutaneous-
leishmaniasis?Source=search_result&search=leishmania&selectedtitle=1~93

56. Patient present to the ER with erythema and peeling of the skin with fever: (missing detail in Q, answer not sure of)
A - Toxic epidermal syndrome
B - Nisseria medgitidis
C - septicemia

Answer: A?
- Stevens-Johnson Syndrome: Cutaneous blistering with mucous membrane involvement (especially lips); “Atypical lesions”: red
circular patch with dark purple center (targetoid), high fever, Sheet-like epidermal detachment in <10% of BSA (Nikolsky sign)
- Toxic Epidermal Necrolysis: Mucous membrane involvement, and severe blistering “Atypical lesions”: 50% have no target lesions
Diffuse erythema then necrosis and sheet-like epidermal detachment in >30% of BSA, high grade fever
Reference: Toronto notes

57. Symmetrical hypo pigmentation with no scaling for 1 year and it's progressing with no previous lesion:
A - Post inflammatory
B - Vitiligo
C - Leprosy

Answer: B

176
58. DM patientwith lesion papule on dorsum of right hand: (answer not sure of)
A - granuloma annulare
B - lichen plants
C - fungal infection

Answer: A?

59. What most commonly causes itching?


A - bile salt retention
B - eczema
C - pregnancy

Answer: B | Reference: http://www.uptodate.com/contents/pruritus-etiology-and-patient-


evaluation?Source=search_result&search=itching&selectedtitle=2~150#H690781

60. (pic) pink patch under wood’s lamp it fluorescent a coral red color Diagnosis:
A - erythrasma
B - fungal infection
C - vitiligo

Answer: A (Caused by G+ve bacteria Corynebacterium minutissimum)

61. Best treatment of moderate to severe acne vulgaris is: (answer not sure of)
A - Clindamycin
B - Tetracyclin
C - Erthromycin

Answer: B? (check table at the end)


Evidence: In general: acne are 3 types:
Comodones ‘ blackheads and whiteheads ‘ : topical retinoic acid.
Acne valgaris Papules, pustules Antibiotictopical for mild and oral as doxycycline & erythromycin for severe
Cystic acne with scars: isotretinoin

62. Ring lesion on the skin which of the following stain you are going to use ?
A. Indian ink stain
B. Gram stain
C. KOH stain

Answer: C
Suspicion of Tinea corporis
177
Http://www.consultantlive.com/skin-diseases/differentiating-common-annular-lesions-tinea-corporis-vs-granuloma-annulare

63. Hypopigmentation on trunk:


A. Antibiotics
B. Steroid
C. Selenium
Answer: C

64. Women has lichen sclerosis which cancer she will get:
A. Squamous cell carcinoma
B. Adenocarcenioma carcinoma
C. Adeno-squamous carcinoma

Answer: A
Lichen sclerosus is common chronic skin disorder that most often affects genital and perianal areas. People with lichen sclerosus
often have a personal or family history of other autoimmune conditions such as thyroid disease (about 20% of patients), pernicious
anaemia, or alopecia areata. Presents as white crinkled or thickened patches of skin that have a tendency to scar.
Http://www.dermnetnz.org/immune/lichen-sclerosus.html

65. Pic / pink patch under colar lumb:


A. Erythrasma
B. Fungal infection
C. Vetiligo

Answer: A
Erythrasma is a chronic superficial infection of the intertriginous areas of the skin. The incriminated organism is Corynebacterium
minutissimum, which usually is present as a normal human skin inhabitant. The typical appearance of erythrasma is well-
demarcated, brown-red macular patches. The skin has a wrinkled appearance with fine scales
Http://emedicine.medscape.com/article/1052532-clinical#b4

66. Rash on hear line and nasal fold non itchy , no past Hx , healthy one? (DERMA)
A. SLE
B. Seborrheic dermatitis
C. Acne valgaris

Answer: The mildest and most common form of scalp seborrheic dermatitis is dandruff, also known as pityriasis sicca, in which the
scalp shows fine, white, diffuse scaliness without underlying erythema (picture 2). Dandruff may be asymptomatic or accompanied
by mild pruritus. More severe forms of scalp seborrheic dermatitis present with visible inflammation, consisting of patchy, orange to
salmon-colored plaques covered with yellowish, greasy scales (pityriasis steatoides), mostly over the temporoparietal areas (picture
1E). Lesions may extend to the post-auricular areas where they often develop fissures, oozing, and crusting, and to the outer canal
and concha of the ear, sometimes with marked pruritus and superinfection (otitis externa).
178
67. Child with blister in trunk what diagnosis?
A. Herpes simplex
B. Varicella
C. Impetigo
In children, chickenpox usually begins as an itchy rash of red papules (small bumps) progressing to vesicles (blisters) on the stomach,
back and face, and then spreading to other parts of the body. Blisters can also arise inside the mouth
Http://www.dermnetnz.org/viral/varicella.html

68. Wheal with erythematous base, itching, lymph node enlargement, periorbital swelling, hepatosplenomegaly?
A. Rheumatic arthritis
B. Angioedema
C. Cholinergic urticarial

Answer: probably C
http://www.merckmanuals.com/professional/dermatologic-disorders/approach-to-the-dermatologic-patient/urticaria

69. What is the single most important thing for the prognosis of SLE :
A. Renal involvement
B. Arthritis
C. Wbcs
Answer: A
The disease course is milder and survival rate higher in persons with isolated skin and musculoskeletal involvement than in those
[50] [51]
with renal disease and CNS disease.
Http://emedicine.medscape.com/article/332244-overview#a6

70. Patient with penile lesion that is caused by using sulfa drugs describe the lesion?
A. Erythema
B. Ulcer
C. Fixed drug eruptions of the skin blistering.
Answer: c.
Fixed drug eruptions of the skin blistering. American Family Physician
Http://www.aafp.org/afp/2006/0101/p133.html

179
71. Patient with skin slightly elevated and mild itching. What is the most likely diagnosis?
A. Lichen planus
B. Band of neutrophil under skin
C. Erythema multiforme
Answer: A
Lichen planus: 6 Ps planar, purple, polygonal, pruritic, papules & plaques
Erythema multiform: targetoid lesion (erythematous macule that become centrally clear)
First Aid for the USMLE Step 2 CK

72. 17 years' boy soccer player' have hypopigmentation on the trunck and over the arm and the the pigmentation get lighter
with sun what you will give him:
A. Oral antibiotic
B. Topical antibiotic
C. Topical steroid
Answer: C

73. Which consider as atypical moles (dysplastic nevi)?


A) Irregular border
B) Smaller than 6 mm
C) Uniformity of color
Answer: A
Atypical moles often have a characteristic appearance, although individual lesions may not show all the findings. Typically, they are
large pigmented lesions and frequently measure 5-15 mm in diameter. Atypical moles are usually larger than common moles.

180
Borders are usually irregular, notched, and ill defined. Macular and papular areas may be present within a single lesion (also
described as a "fried egg" appearance). Color is highly variable and ranges from tan to dark brown to pink.
Http://emedicine.medscape.com/article/1056283-clinical - b1

74. Patientwith Hx of unprotected sex 8 weeks ago, came rash all over his body excepatientthe face, what is the organism:
A- Syphilis
B- Chlamydia
C- chancroid
Answer: A

75. A Picture of 1 month old baby with rash and scale what is the most likely cause? (missing options, answer not known)
A - Eczema
B - Seborrheic dermatitis

Answer: ?

76. Patient on long uses of topical steroids. What will happen?


A - scaling
B - atrophy

Answer: B
Evidence: long use of steroid Super potent and potent topical corticosteroids may induce atrophy, telangiectasia, easy bruising,
hypertrichosis and striae (local side effects). Reference: uptodate

77. A baby with multiple plaques on his face, abdomen and feet. What is the Diagnosis? (missing options, answer not known)
A - Basal cell carcinoma
B - squamous cell carcinoma

Answer: could be eczema or seborrheic dermatitis but the presence of the plaques on the feet (especially on soles) suggest
psoriasis. Other common DD: Tinea corporis and pityriasis rosea

78. Patient with a localized patch of hair loss: (missing options, answer not known)
A - male pattern
B - female pattern

Answer: ?
Evidence:
- male pattern hair loss: receding hairline and/or hair loss on the top and front of the head.
- Female pattern hair loss: diffuse thinning of hair on the scalp
- causes of localized patch of hair loss:
= Tinea capitis “the most common cause of hair loss in children. Patchy hair loss with some broken-off hairs visible just
above the surface of the scalp”
= Alopecia areata “appear literally overnight, or sometimes over a few days”

181
= Trauma including trichotillomania
= Telogen effluvium
= traction alopecia “thinning from tight braids or ponytails”

79. Patient came with lobulated nose, erythema over cheeks & (other features indicating rosacea type 4), what is the treatment?
A - Doxycycline.
B - Acyclovir.

Answer: A (Doxy. Other medications: Topical metronidazole, azelaic acid)


Reference: http://emedicine.medscape.com/article/1071429-treatment

80. Pregnant +ve hx of herpetic outbreak annually on examination everything is normal. What is the management?
A - Reassurance
B - Acyclovir

Answer: ? (missing options, answer not known)


Evidence: Acyclovir for symptomatic women, suppressive therapy at 36 week controversial C/S if active genital lesions, even if
remote from vulva ; Reference: Toronto notes

81. Young female, loss of hair half of her head and the skin is normal? (missing options, answer not known)
A - Alopecia
B – Trichomania

Answer: B? (not Enough details) | Reference: http://emedicine.medscape.com/article/1071854-overview#a6

82. Bee sting, when you give him epinephrine what you will inhibit? (missing options, options not complete, answer not known)
A - Interleukin…
B - Systemic inflammatory…

Answer: B?
Evidence: Epinephrine works on:
Alpha-1 causing generalized vasoconstriction
Beta-1 causing increased heart rate and cardiac contractility
Beta-2 causing increased bronchodilation and decreased release of histamine, tryptase, and other mediators of inflammation from
mast cells and basophils
Reference: http://www.medscape.com/viewarticle/726456_2

83. Patientdiagnosis with shingles around thoracic dermatome, what will you give him? (missing options)
A - Topical acyclovir and topical steroid

182
B - Oral steroid

Answer: Oral acyclovir


Evidence:
- compress with normal saline, Burow’s, or betadine solution
- analgesics (nsaids, amitriptyline)
- famciclovir or valacyclovir or acyclovir for 7 d; must initiate within 72 h to be of benefit; IV
- acyclovir for ophthalmic or disseminated Involvement
- gabapentin 300-600 mg PO tid for post-herpetic neuralgia
Reference: Toronto notes

84. Female recently used eye cream developed inflammation in face with redness: (missing options)
A - Contact dermatitis
B - Seborrheic dermatitis

Answer: A
Evidence:
- Seborrheic: condition affecting infant and cause yellow crusty greasy scaling.
- Contact: due to contact to material of clothing soap lotion chemical detergent and medication.

85. "infant patient have greasy inflamed areas in the scalp with yellowish excoriations. What is the diagnosis: (missing options)
A - seborrheic dermatitis
B - erythema toxicum"

Answer: A

86. Itching in lower limbs, otherwise normal: (missing details of Q, missing options, answer not known)
A - Tinea
B - Scabies

Answer: ? (maybe a sign of a serious internal disease not dermatological disease OR stress)

87. Vesicles highly suspected roundworms: (missing options, missing options, answer not known)
A - Ascaris
B - Tenea saginata

88. Patient with hyperpigmented non pruritic papules in the dorsum of the hands not resolved with antifungal:
A - tinea corpourus
B - lichen planus

183
Answer: ? (missing options, answer not known) (lichen planus: 6 Ps: pruritic, purple, polygonal, papules and plaques)

89. Patient is brought by his parent because of loss of hair. On examination: he had localized patch of
hair loss at temporal area, the end of hair looked broken and tapered. What is the diagnosis?
A - Alopecia Areata.
B - Trichotillomania.

Answer: B (trichotillomania)

90. Adult male on multiple drugs came with violaceous maculopapular lesion in the trunk, Diagnosis?
A - Erythema multiforme
B - Toxic epidermal necrolysis

Answer: Exanthematous (maculopapular) drug eruption (missing options)


Reference: http://www.aafp.org/afp/2010/0315/p726.html

91. Loose cornified fragments of the skin what is it?


A. Scales
B. Crustation
Answer: A

92. Bluish discolored cyst in lower lip?


A. Ranula
B. Mucocele
Answer: B
- mucocele is a collection of saliva in the oral mucosa. They are soft elevations whose color ranges from that of normal mucosa to
light blue or even white. Patients with mucoceles regularly state that the lesion “gets larger, then smaller, then larger again.” This
has become an important diagnostic sign. The mucosa of the lower lip and buccal mucosa are the most common sites, but any area
that contains intraoral salivary glands is a potential site.
- Ranulas are mucoceles that occur in the floor of the mouth and usually involve the major salivary glands. Specifically, the ranula
originates in the body of the sublingual gland, in the ducts of Rivini of the sublingual gland, and, infrequently from the minor salivary
glands at this location.
http://emedicine.medscape.com/article/1076717-clinical
https://dentistry.umkc.edu/Practicing_Communities/asset/OralLesions.pdf

93. Farmer have cyst on his dorsal of his right hand after 4 month his Dome-like cyst have keratin growth?
A. BCC
B. Keratocarcimoa
Answer:
184
keratoacanthoma ~> rapid evolution and spontaneous involution (4-6 months), has characteristic dome shape with a central plug of
keratin. Sun exposure, and exposure to chemical carcinogens such as tar,64 are thought to be etiologic factors.
- Squamous cell carcinoma is a malignant tumor arising from epidermal or appendageal keratinocytes or from the squamous
mucosal epithelium. There is often a history of damage by exogenous agents acting as carcinogens, such as sunlight, ionizing
radiation, local irritants, or arsenic ingestion. The tumor cells have a tendency toward keratin formation.
- The basal cell carcinoma is characteristically slow growing over months to years. The early lesions are round-to-oval papules or
nodules, often with an umbilicated center which may be ulcerated. The color is pink to red and often has a translucent or pearly
quality.
http://www.ncbi.nlm.nih.gov/books/NBK13437/
94. Patient is brought by his parent because of loss of hair. On examination: he had localized patch of hair loss at temporal area,
the end of hair looked broken and tapered. What is the diagnosis?
A. Alopecia Areata.
B. Trichotillomania.

Answer: B
It is an obsessive compulsive disorder characterized by the compulsive urge to pull out one's hair, leading to hair loss
Individuals with trichotillomania exhibit hair of differing lengths; some are broken hairs with blunt ends, some new growth with
tapered ends
Reference: http://emedicine.medscape.com/article/1071854-overview

95. Female patientobese with regular menstrual cycle, on PE/ she had acne, other exam is normal, what investigation will you
order?
A - TSH
B - ACTH

Answer: B (missing options) (examining for Cushing syndrome)

96. White patches after sun exposure treatment? (missing options)


A - Antibiotic
B - antifungal (selenium sulfide)

Answer: B [It's pityriasis versicolor (tinea flava)]

97. Female with pustules on her face, which type of acne is this?
A- inflammatory.
B- infectious.
Answer from toronto note derma: inflammatory type 3 .
Type 1: comedonal, sparse, no scarring .
Type 2: comedonal, papular, moderate -+ little scarring.
Type 3: comedonal,papular, and pustular with scarring.
Type 4: nodulocystic acne, risk of severe scarring.

185
98. Adult male on multiple drug came with violaceous maculopapular lesion in the trunk, Diagnosis?
A- erythema multiforme
B-toxic epidermal necrolysis
Answer : it could be toxic epidermal necrolysis .

99. Patient with hyperpigmented non pruritic papules in the dorsum of the hands not resolved with antifungal
A. Tinea corporis
B. Lichen planus
Answer: B?
Lichen planus is hyperpigmented lesions thats doesn’t resolved by antifungals but it cause pruritis!!
Treatment: Topical corticosteroids with occlusion or intradermal steroid injections.
Source: Toronto notes 2014 D17

100.Patientpresent with grey scale on erythematous base which mainly locate at hair line & behind the ear. There is no pain or
itching. What is the proper medication for this pt?
A. Ketoconazole ….mg twice a day
B. Other option are antibiotic, not anti-fungal
Answer:
Might be Seborrhoeic dermatitis
Http://bestpractice.bmj.com/best-practice/monograph/89/diagnosis/history-and-examination.html

101.Side effect of topical retinoid, a derivative of vitamin A:


A. Photosensitivity
B. Moist skin
Answer is A
- One of isotretinoin side effects is dryness of skin & mucous membranes.

186
Source: http://reference.medscape.com/drug/amnesteem-claravis-isotretinoin-343544#4

102.Patient with ulcer on the nose. Diagnosis?


A. Squamous carcinoma.
B. Basal cell carcinoma.
Answer: A?

103.Clear scenario of female patient with eczema


A. Psoriasis
B. Allergy

104.Scraping used in diagnosis which organism:


A. Scabies
B. Toxo... (not toxoplasmosis)
Answer: A

105.Single Red lesion on shoulder that keeps growing?


A. Strawberry Nevus
B. Pyoderma gangeronosum
Answer: A
Strawberry nevus (superficial hemangioma): characterized by phases of proliferation and involution as defined by a
rapid proliferation of blood vessels in the first year of life, followed by gradual regression of the vascular
component with replacement by fibrofatty tissue.
Pyoderma gangrenosum usually: a bite reaction, with a small, red papule or pustule changing into a larger,
ulcerative lesion. Often, patients give a history of a brown recluse or other spider bite, but they have no evidence
that a spider actually caused the initial event.
Reference: http://emedicine.medscape.com/article/1123821-clinical

106.Patient got skin incision by a knife, later he develops progressive pain, redness and swelling, what is the diagnosis:
A. Cellulitis
B. Necrotizing fasciitis
187
Answer: A
Non-purulent cellulitis is associated with the 4 cardinal signs of infection, as follows: Erythema, Pain, Swelling, Warmth.
Reference: http://emedicine.medscape.com/article/214222-overview

107.Pts playing tennis, something bites him (birds I think) after few minutes he has respiratory distress and he was collapsed
what is the immediate treatment for this patient?
A. Antihistamine
B. Epinephrine
Answer: B
Epinephrine maintains blood pressure, antagonizes the effects of the released mediators, and inhibits further release of mediators.
It is the drug of choice and it is usually well tolerated and potentially lifesaving.
Link: http://emedicine.medscape.com/article/135065-treatment#d9

108.Patient was started on Carbamazepine. Presented with large area of skin peeling with blisters. He looks toxic. There is
eosinophilia. What is the problem here?
A. Steven Johnson syndrome.
B. Toxic epidermal necrolysis.
Answer: b
Carbamazepine is known etiology of Toxic epidermal necrolysis.
Http://emedicine.medscape.com/article/229698-overview#a5

109.Old man smoker and alcohol drinker developed oral ulcer that bleeds when touching
A. Squamous cell carcinoma
B. Aphthous ulcer
Answer: A
Http://www.cancer.gov/types/head-and-neck/patient/lip-mouth-treatment-pdq
The most common cancer of the oral cavity is called squamous cell carcinoma and arises from the lining of the oral cavity (Over
95%). Http://www.ahns.info/resources/education/patient_education/oralcavity/
Aphthous ulcers more commonly affect young adults, and a familial tendency may exist. Smoking offers a somewhat protective
effect against recurrent aphthae. Other etiologic factors such as stress, physical or chemical trauma, food sensitivity and infection
have been proposed.
Http://www.aafp.org/afp/2000/0701/p149.html

110.Patient fever, mouth ulcer, and other symptoms, what's the Diagnosis:
A- HSV 1
B- HSV 2
Answer: A

111. Penile painless rash after use of sulfa drug description of the rash ?!
A. Papules
188
B. Vesicle

Answer: Bulla
This is a case of Stevens-Johnson syndrome. It is blisters or hive like rash with mucous membrane involvement. Frequently drug-
related (nsaids, anticonvulsants, sulfonamides, penicillin’s) occurs up to 1-3 week after drug exposure

Reference: Toronto note.

112. Which of the following is a skin manifestation associated with cystic fibrosis:
A. Sebohrric dermatitis
B. Dermatitis herpitiform
Answer: ?
113.Patient has skin eruption that affected all of his body including palms and soles?
A. Scleroderma
B. Something medication
Not complete

114.Black to brown lesion on sole of the foot: (missing options, answer not known)
A - helmata

Answer: differential diagnosis includes benign nevus, pigmented seborrheic wart, squamous cell papilloma, carcinoma or malignant
melanoma, capillary cavernous hemangioma and Kaposi's sarcoma. The differential is so broad & we need more details

115.Diaper rash that is resistant to topical steroids and has satellite lesions on thighs (well demarcated and red):
A. topical antifungal
Answer: Antifungal agents such as nystatin, clotrimazole, miconazole, ketoconazole, and sertaconazole are effective topical
therapies for diaper rash
Reference: uptodate

116.Scenario for patientwith scabies what is the Treatmentof choice? (missing options)

Answer:
1 - treated overnight with 1-2 applications of 5% permethrin from neck down + their contacts should be treated
2 - oral ivermectin
3 - symptomatic treatment for pruritus
Reference: FIRST AID USMLE STEP2 CK

189
117.The best test for hypersensitivity type 1: (missing options)
A - subdermal skin

Answer: A
Subdermal skin test (skin prick or skin scratch tests), also allergen-specific ige blood test or total ige tests, couldn’t find any article to
decide which is best however, skin testing is much more common
Reference: https://labtestsonline.org/understanding/conditions/allergies/start/2

118.2 year-old complain of papule on the foot no itching pink pale not respond for antifungal? (missing options)
A - Granuloma

Answer: A
Evidence: Granuloma annulare is a common skin condition with raised, flesh-colored bumps that appear in a ring. It may occur on
any part of the body (though most commonly on the sides or backs of the hands or feet). The bumps may be red at the beginning,
but this disappears as the ring forms. There is no itching or scaling.
References: http://www.uptodate.com/contents/granuloma-annulare & http://www.drgreene.com/qa-articles/ringworm-
lookalikes/

119.Isotretinoin most feared complication:


A - birth defect

Answer: A
Isotretinion (Accutane/roaccutane) is teratogenic, causing embryopathy in 20 - 30 % & spontaneous abortion is approximately 20 %
Reference: http://www.uptodate.com/contents/oral-isotretinoin-therapy-for-acne-vulgaris

120.Male with pustules and papules and telangiectasia, what is the diagnosis? (missing options)
A - Rosacea

Answer: A (Typical description)

121. Description of scarlet fever rash? (missing options)

Answer: The rash of scarlet fever is a diffuse erythema that blanches with pressure, with numerous small (1 to 2 mm) papular
elevations, giving a "sandpaper" quality to the skin. It usually starts in the groin and armpits and is accompanied by circumoral pallor
and a strawberry tongue. Subsequently, the rash expands rapidly to cover the trunk, followed by the extremities, and, ultimately,
desquamates; the palms and soles are usually spared. The rash is most marked in the skin folds of the inguinal, axillary, antecubital,
and abdominal areas and about pressure points. It often exhibits a linear petechial character in the antecubital fossae and axillary
folds, known as Pastia's lines.

190
122. Scabies, what’s the organism? (missing options)

Answer: Sarcoptes scabiei | Reference:


http://www.uptodate.com/contents/scabies?Source=search_result&search=scabies&selectedtitle=1~75

123. Purple, papule, polygonal rash on the flexors?


A - lichen planus.

Answer: A {Ps of Lichen Planus [Purple, Pruritic, Polygonal, Peripheral, Papules, Penis (i.e. Mucosa)]}

124.Old pt. With diffuse thinning of hair and without eyelash, diagnosis? (missing options)

Answer: alopecia areata | Reference: http://www.uptodate.com/contents/clinical-manifestations-and-diagnosis-of-alopecia-


areata?Source=see_link

Answer: ?
125. Alopecia in a boy who performs poorly in school: (missing options)
A - Trichotillomania

Answer: A (a compulsive disorder resulting in Alopecia from repetitive hair manipulation by the pt’s own hand)

126. Treatment of folliculitis: (missing options, answer not known)

Answer: ?
Evidence: scalp folliculitis:
= Topical antibiotics eg fusidic acid gel, clindamycin solution, erythromycin solution
= Mild topical steroid lotions or creams
= Oral antihistamines
= Oral antibiotics, particularly long-term tetracycline
= Oral isotretinoin – long-term low dose treatment.

127.Trichotillomania treatment: (missing options)

Answer: best answer & first line treatment is behavioral therapy and intervention; drugs: SSRI’s are used but aren’t beneficial

128. Treatment of acne rosacea? (missing options)

Answer:

191
- oral Antibiotic(Doxycycline) and topical Antibioticlike metronidazole for mild to moderate.
- trigger avoidance is key to long-term management
- avoid topical corticosteroids
- telangiectasia: treated by physical ablation; electrical hyfrecators, vascular lasers, and intense pulsed light therapies
- phymas: treated by physical ablation or removal; paring, electrosurgery, cryotherapy, laser therapy (CO2, argon, Nd:YAG)

129. Scrape of skin can be done in which of the following: (missing options)
A - scabies

Answer: A (Most effective drug for Rx is permethrin cream)

130.Multiple myeloma in the spine: (no stem of Q, missing options, answer not known)
- Histopath report

Answer: ?

131.Patient c/o rash over elbows, knees, and check?


Answer Eczema? If there's in cheek
Or it will be psoriasis

132.Male with pustules and papules and telangiectasia, what is the diagnosis?
A - Rosacea

Answer: A (Rosacea)

133.Well circumscribed lesion on erythematous base, arthritis?


This is a presentation of a rheumatological disease.
Subacute cutaneous lupus erythematosus can present in an annular form on sun-exposed areas or in a papulosquamous form

192
.

Skin lesions in Subacute cutaneous lupus erythematosus appear as Annular erythematous plaques with central clearing, often
mimicking annular psoriasis when associated with scales.
Reference: http://www.aafp.org/afp/2001/0715/p289.html

134.Patient has family history of allergy has scaling skin & itching in the face & anticubital fossa, diagnosis?
A. Atopic Eczema.
Answer: A
First Aid:
Look for a strong family history, usually the atopic triad:
- Asthma.
- Allergic Rhinitis.
- Eczema.
It can manifest as:
- In children: dry, scaly, pruritic, excoriated papules, plaques in the flexural areas and neck.
- In adults: lichenification and dry, fissured skin in flexural distribution. Often, there is a hand or eyelid involvement.

135.Ulcerative colitis skin lesions :


Most frequent skin lesions associated with IBD inculde erythema nodosum and pyoderma gangrenosum
Source: uptodate

136.An infant developed a rash that spares the folds. What is the possible diagnosis?
A. diaper dermatitis
Answer: A
The eruption may be patchy or confluent, affecting the abdomen from the umbilicus down to the thighs and encompassing the
genitalia, perineum, and buttocks. Genitocrural folds are spared in irritant dermatitis, but often involved in primary candidal
dermatitis.
Reference: http://emedicine.medscape.com/article/911985-clinical#showall

193
137.Retinoid (for acne) side effect?
Answer: avoid sun. From Medscape
Skin irritation with peeling and redness may be associated with the early use of topical retinoids and typically resolves within the
first few weeks of use. Topical retinoids thin the stratum corneum, and they have been associated with sun sensitivity. Instruct
patients about sun protection
http://emedicine.medscape.com/article/1069804-treatment

138.What is the difference between Scale and Crust?


Scale is an accumulation of loose cornified fragments of the Stratum corneum.
Crust is the presence of a dried exudate (serum, blood, pus) on the skin surface. ((First Aid for the USMLE Step 1 CK 2014,p430 ))

139.Rash started in the face and then spread:


A. Rubella
Answer: A
rubella ~> A deep, red, flat rash that starts on the face and spreads down to the trunk, arms, and legs. The rash starts as small
distinct lesions, which then combines to form one big rash. After 3 to 4 days, the rash will begin to clear, leaving a brownish
discoloration and skin peeling.
http://childrensnational.org/choose-childrens/conditions-and-treatments/skin-disorders/viral-exanthems-rashes

140.HIV patient had violacous skin lesion, biopsy showed spindle formation? Dx ?
A. kaposi sarcoma

Answer: ?

141.Patient came with skin lesion, wood lamp pink . Dx ?


A. fungal infection

Answer: ?

142.Redness and itching between the toes, what is the Dx ?


A. Scabies
Answer: A

194
143. bacterial disease with oral ulcer used steroid didn't improve what is next Tx
Reference: http://www.medscape.com/viewarticle/406932_11

144. Pyoderma with which disease? (missing options)

Answer: immunodeficiency pt.


Evidence: Commonly associated diseases include inflammatory bowel disease, eitherulcerative colitis or regional enteritis/Crohn
disease, and a polyarthritis that is usually symmetrical and may be either seronegative or seropositive. Hematologic
diseases/disorders are other commonly associated conditions; these include leukemia or preleukemic states, predominantly
myelocytic in nature or monoclonal gammopathies (primarily immunoglobulin A [iga])
Reference: http://emedicine.medscape.com/article/1123821-clinical

145. Keratitis caused by parasites: (missing options)

Answer: Acanthamoeba keratitis

195
Evidence: Acanthamoeba keratitis, or AK, is a rare but serious infection of the eye that can cause permanent vision loss or blindness.
This infection is caused by a tiny ameba (single-celled living organism) called Acanthamoeba. Acanthamoeba causes Acanthamoeba
keratitis when it infects the cornea, the clear dome that covers the colored part of the eye.

146. Infant suffer from groin rash, spare fold? (missing options)

Answer: diaper rash dermatitis, other names: Napkin dermatitis.

147.Lady with spot of hair loss over the scalp with normal underlying skin, what's the Diagnosis?
A- alopecia areata
Answer: A
Alopecia areata is a recurrent nonscarring type of hair loss that can affect any hair-bearing area and can manifest in many different
patterns.
The presence of smooth, slightly erythematous (peach color) or normal-colored alopecic patches is characteristic.
Referance:
Http://emedicine.medscape.com/article/1069931-clinical#b4

148.Case of lichen planus..


6 p’s: Purple, Pruritic, Polygonal, Planar, Papules, Plaques.
• Common sites: wrists, ankles, mucous membranes in 60% (mouth, vulva, glans), nails, scalp, genitals.
• Wickham’s striae: reticulate white-gray lines over surface; pathognomonic but may not be present.
• Increased risk of SCC in erosions and ulcers.
• It is associated with hepatitis
• Koebner phenomenonarin

149.Pic of patient lower limbs has rash on medial thigh , written that it was pinkish in color and itchy , and the patient is diabetic :
A. tenia cruris

Explanation: Tinea cruris, a pruritic superficial fungal infection of the groin and adjacent skin, is the second most common clinical
presentation for dermatophytosis. Presents as Scaly patch/plaque with a well-defined, curved border and central clearing. Pruritic,
erythematous, dry/macerated. Site: medial thigh.
Reference: Toronto Notes 2015, page D26, dermatology

150.Boy have lesion on his forarm erethmatous ,silvary scaling?


196
A. Psoriasis

Explanation: most common skin manifestations are scaling erythematous lesions. Well-demarcated, noncoherent, silvery plaques
overlying a glossy homogeneous erythema. Chronic stationary psoriasis (psoriasis vulgaris) is the most common type. This involves
the scalp, extensor surfaces, genitals, umbilicus, and lumbosacral and retroauricular regions.
Reference: http://emedicine.medscape.com/article/1943419-clinical#b2

151.Maculapapular rash , plt low , blood smear showed fragmented RBC , sciocyte The antibodies was targeting?
Answer: B-G glycoproteins Annex

152.Child has itching and all student in his class got the same infection:
Answer: sarcoptes scabiei
Human scabies is an intensely pruritic skin infestation caused by the host-specific mite Sarcoptes scabiei hominis. Burrows are a
pathognomonic sign and represent the intraepidermal tunnel created by the moving female mite. They appear as serpiginous,
grayish, threadlike elevations in the superficial epidermis.
Http://emedicine.medscape.com/article/1109204-overview

153.Case with hypopigmentation on the arm ( one area only ) + with symptoms on it (Can't remember them well its like
parathesia or something like this but im sure it not vitiligo ) DERMA
Vitiligo
A. Leprosy

154.Patientdevelop 2 cm dome shape mass in the dorsum of the hand , it's cover by keration Diagnosis?!
Kerato acanthoma

Answer
KERA TOACANTHOMA
Clinical Presentation :
Rapidly growing, firm, dome-shaped, erythematous or skin-coloured nodule with central keratin-filled crater, resembling an erupting
volcano
• often spontaneously regresses within a year, leaving a scar
• sites: sun -exposed skin
Reference: Toronto notes

155.A man who lives in the desert presented with skin lesion on the forearm. Microscopic evaluation of lesion shows Donovani.
What is treatment?
Answer: pentavalent antimonial drug (sodium stibogluconate OR meglumine antimonate) is the treatment of choice.
This is a case of localized cutaneous leishmaniasis usually transmitted by sandfly. Treatment with antimonial drugs will heal
lesions faster and prevent relapse, local dissemination, mucosal disease (usually), and transmission. Not all lesions require

197
treatment. Old World disease tends to be self-healing, and systemic treatment seldom is used. New World lesions more often
require systemic treatment.

156.A man came from india with diarrhea and then developed rash:
Answer: dermal leshmaniasis?

157.Derma description of a lesion that is loose and easy to fall defining of ( scales, other skin lesions) ?
Coxsackie virus in pediatric (Hand foot disease)?
Answer: question is missing a lot of information.

158. Erythematous papule with silver scaly when alcohol swab used pinpoint, bleeding. Diagnosis :
Answer: Psoriasis.
Psoriasis is a chronic inflammatory skin condition characterised by clearly defined, red and scaly plaques
(thickened skin). It is classified into several subtypes.
Http://www.dermnetnz.org/scaly/psoriasis.html

159. What is the treatment of folliculitis


Answer: oral Antibiotics
Folliculitis is the name given to a group of skin conditions in which there are inflamed hair follicles. The result
is a tender red spot, often with a surface pustule.
Http://www.dermnetnz.org/acne/folliculitis.html

160. Scaly erythematous rash on nasal folds and hair line ?

Answer: Sebrohic dermatitis


Seborrhoeic dermatitis (American spelling is ‘seborrheic’) is a common, chronic or relapsing form of
eczema/dermatitis that mainly affects the scalp and face. There are infantile and adult forms of seborrhoeic
dermatitis. It is sometimes associated with psoriasis (sebopsoriasis). Seborrhoeic dermatitis is also known as
seborrhoeic eczema.
Http://www.dermnetnz.org/dermatitis/seborrhoeic-dermatitis.html

161. Mid facial pain with derma manifestation ... Cant remember ?

Answer: Herpes zoster not sure


Reactivation of varicella-zoster virus (VZV) that has remained dormant within dorsal root ganglia, often for
decades after the patient’s initial exposure to the virus in the form of varicella (chickenpox), results in herpes
zoster (shingles).
198
Http://emedicine.medscape.com/article/1132465-overview

162.Old man have pladiasing blue cells on dermis grossly ulcer on top of nodules top of nose Diagnosis: Melanoma (nodular)

Http://www.merckmanuals.com/professional/dermatologic-disorders/cancers-of-the-skin/melanoma

163.Female recently used eye cream developed inflammation in face with redness: Contact dermatitis , Seborrhic dermatitis
Answer: Not Complete

164.Skin lesion "ringworm" which stain should be used?


A- Potassium chloride
Answer: potassium hydroxide
Tinea corporis often begins as a pruritic, circular or oval, erythematous, scaling patch or plaque that spreads centrifugally. Central
clearing follows, while an active, advancing, raised border remains. The result is an annular (ring-shaped) plaque from which the
disease derives its common name (ringworm)
Link: http://www.uptodate.com/contents/dermatophyte-tinea-infections

165.Boy have lesion on his forearm erythematous, silvery scaling?


Answer: Psoriasis
Http://emedicine.medscape.com/article/1943419-overview

166.What the cause of itching (just that):


A- Eczema

167.Patientwith hereditary angioedema what’s the defect:

PATHOGENESIS — In hereditary angioedema (HAE), angioedema results from excessive production of bradykinin, a potent
vasodilatory mediator. During episodes of angioedema in patients with HAE, plasma bradykinin levels have been shown to be
sevenfold higher than normal. Histamine and other mast cell mediators are not directly involved, which explains the lack of
response to antihistamines and distinguishes this form of angioedema from that associated with urticaria.

168.Patientscratching with linear marks and two blue dots at each end what does he have:
A. Scabies

169.Pityriasis Alba
199
Pityriasis alba is a low-grade type of eczema/dermatitis that primarily affects children.
The cause of pityriasis alba is unknown.
1. It often coexists with dry skin and atopic dermatitis.
It often presents following sun exposure, perhaps because tanning of surrounding skin makes affected areas more prominent.
No treatment is necessary for asymptomatic pityriasis alba.
A- A moisturising cream may improve the dry appearance
B- A mild topical steroid (hydrocortisone) cream may reduce redness and itch
C- Calcineurin inhibitors, pimecrolimus cream and tacrolimus ointment, may be as effective as hydrocortisone and have been
reported to speed recovery of skin colour.

170.Man live in desert present with skin lesion on forearm, microscope show donovani. What is treatment?
Answer: Leishmaniasis: many treatment options, see the link below
Http://emedicine.medscape.com/article/220298-treatment#showall

171.Premalignant lesion of SCC?


A) Actinic keratosis
Answer: A
Actinic keratosis (AK) is a UV light–induced lesion of the skin that may progress to invasive squamous cell carcinoma. It is by far the
most common lesion with malignant potential to arise on the skin. , http://emedicine.medscape.com/article/1099775-overview

172.Pediatric patient yellowish greasy area on the scalp, what is the Diagnosis:
Answer- Seborrheic dermatitis

173.Rash band like distribution, what is the Diagnosis:


Answer- Shingles

174.Child with hair loss circumcised shape. On microscope examination


Pickled... Diagnosis?
Answer: Maybe tinea Capitis

175.Using Steroid on the face. Will lead to?


Answer: Atrophy
Side effects — Topical corticosteroids are safer than systemic glucocorticoids. Nevertheless, cutaneous and systemic side effects can
occur, particularly with super potent and potent drugs, or extensive use of lower potency agents with or without occlusion. (See
"Major side effects of systemic glucocorticoids".)
Cutaneous — A number of cutaneous effects may occur with topical corticosteroid use:
Super potent and potent topical corticosteroids may induce atrophy, telangiectasia, and striae as early as two to three weeks
following daily application [25]. Intertriginous and thin-skinned highly penetrable areas are particularly susceptible to atrophy, which
usually recovers within weeks to months if therapy is discontinued as soon as atrophic change occurs.

200
176.a patient presented with macular papular rash and fever. (case of rubella)
Answer: ?
Clinical features: rash (pink, maculopapular rash 1-5 d after start of symptoms. The rash starts on face and spreads to neck and
trunk), prodrome of low grade fever and generalized, tender lymphadenaopathy especially occipital/retroauricular nodes. Diagnosis
is clinical. Star complex (sore throat, arthritis, rash) and positive serology for rubella igm. Most important complication is congenital
rubella syndrome.
Managament:
● For infected patients: symptomatic rx.
● For prevention: mmr vaccine.
● For rubella-exposed pregnant women: serologic testing.
Reference: toronto notes and medscape

177.what most commonly cause itching?


Answer:
Pruritus is a defining feature of atopic eczema. Daily pruritus is described in 87% to 91% of patients with atopic eczema.
Psoriasis patients, pruritus was the second most frequently reported symptom and was reported by 79%.
http://www.ncbi.nlm.nih.gov/books/nbk200924/
http://medicaljournals.se/acta/content/?doi=10.2340/00015555-0662&html=1

178.Case of thickened skin of forearm ask about test to do


A. Scl70
Answer: a to r/o scleroderma

179.Example for non-keratinized squamous epithelium?


Non-keratinized: non-keratinized surfaces must be kept moist by bodily secretions to prevent them from drying out. Examples of
non-keratinized stratified squamous epithelium include cornea, lining mucosa of oral cavity, esophagus, anal canal, foreskin, vagina,
and the internal portion of the lips.
https://en.wikipedia.org/wiki/stratified_squamous_epithelium

180.Xeroderma pigmentosime defect in?


Dna break repair gene

201
Incomplete & Missing Questions
1. Q: Angioedema
Answer: ? (Angioedema is form of urticaria in which there is deeper swelling in the skin which may take > 24 Hrs to clear)
2. Q: Case of scabies very clear
Answer: ? Maybe as Diagnosis there's night itchiness!
Tretment: permethrin 5% lotion. Alternative drug therapy includes precipitated sulfur 6% in petrolatum, lindane, benzyl
benzoate, crotamiton, and ivermectin; a possible new option is albendazole. Repeat application in 7 days.
Reference: http://emedicine.medscape.com/article/1109204-medication
3. Hidradenitis supportive of the buttocks: http://www.nhs.uk/conditions/hidradenitis-suppurativa/Pages/Introduction.aspx
4. Lichen planus (5 p's)
5. Management of Acne
6. Scabies
7. Hypo pigmented macule shin on sun how will Tx:
8. Boy with hypopigmented lesion in back and extremity
becomes lighter with sun exposure Treatment Topical
steroid Antibiotic ...
A-antifungal
Answer:a

Treatment of Tinea Versicolor


Effective topical agents include selenium sulfide, sodium sulfacetamide, ciclopiroxolamine, as well as azole and allylamine
antifungals.
9. Reference:
http://emedicine.medscape.com/article/1091575-
medication#2

10. Patient with groin abscess, after aspiration there was multiple cells, Cause?
A- Immunodeficiency - C5
Unclear Mcq

11. Dark brown shape in the foot? Diagnosis

12. Non pruritic pink eruption of the right foot no scales no history of infection.
Answer: Not complete

202
Extra information for treatment of acne:

203
204
Basic Science

205
Anatomy

206
1. patient with shoulder pain and pleurisy. Which part of the pleura causes radiation of the pain to shoulder?
A. Visceral
B. Mediastinal
C. Costal
D. Anterior

Answer: b
➢ Visceral pleura: insensitive to pain due to autonomic innervation.
➢ Parietal pleura:
● Costal and peripheral parts of diaphragmatic pleura are referred along intercostal nerves to thoracic and abdominal wall.
● Mediastinal and central diaphragmatic pleural pain referred to root of neck and over shoulder (dermatomes c3-c5).
Reference: lippincott's concise illustrated anatomy: thorax, abdomen & pelvis.

2. “right coronary artery dominance" is explained as right coronary artery giving branch to?
A. Circumflex
B. Anterior descending
C. Posterior descending
D. Marginal

Answer: c
Reference: http://www.cardiologysite.com/ppchtml/rca_dom.html

3. Patient developed dysphagia. On examination there was deviation of the uvula to the left side. Which nerve is affected?
A. Lt. Vagus
B. Rt. Vagus
C. Hypoglossal
D. Glossopharyngeal

Answer: b
In glossopharyngeal nerve (sensory) involvement, there will be no response when touching the affected side. With vagal nerve
damage, the soft palate will elevate and pull toward the intact side regardless of the side of the pharynx that is touched. If both cn ix
and x are damaged on one side (not uncommon), stimulation of the normal side elicits only a unilateral response, with deviation of
the soft palate to that side; no consensual response is seen. Touching the damaged side produces no response at all.
Reference: disorders of the nervous system-dartmouth.

4. What is the first structure you will hit after lumbar puncture?
A. Interspinous
B. Ligamintum flavum
C. Anterior spinal ligament

207
D. Posterior spinal ligament

Answer: a
skin
fascia and sc fat
supraspinous ligament
interspinous ligament
ligamentum flavum
epidural space and fat (epidural anesthesia needle stops here)
dura
Reference: usmle step 1 + wikipedia.
Note: best level for an lp is l3/l4 or l4/l5

5. Pain or numbness of 4 first fingers of hand. Which nerve affected?


A) ulnar
B) median
C) radial
D) brachial plexus
Answer: b -median.
Median nerve ( c6 – t1) innervates the flexor muscles in the anterior compartment of the forearm(excepation the flexor carpi ulnaris
and paright of the flexor digitorum profundus, innervated by the ulnar nerve). Also supplies innervation to the thenar muscles and
lateral two lumbricals in the hand.
Reference : http://teachmeanatomy.info/upper-limb/nerves/the-median-nerve/

6. Patellar knee jerk reflex maintained by:


A) l1-l2
B) l2-l3
C) l3-l4
D) l5-s1
Answer: c
Reference: usmle step 1

7. A patient with a stabbed wound to the gluteus. Examination: the patient tilt to the unaffected side while walking. Which
nerve is affected?
A. Femoral n
B. Obturator n
C. Superior gluteal n
D. Inferior gluteal n
E. Peroneal n
Answer : c

208
The trendelenburg gait pattern (or gluteus medius lurch) is an abnormal gait (as with walking) caused by weakness of the
abductor muscles of the lower limb, gluteus medius and gluteus minimus. People with a lesion of superior gluteal nerve have
weakness of abducting the thigh at the hip. The gluteus medius muscle is an interesting muscle as supplied by superior gluteal
Nerve, it works by lifting up the pelvis on the opposite side to prevent the hip on the swing leg from drooping down. With an absent
gluteus medius muscle contraction, the patient has to lean to this weak side to use the torso to lever the pelvis to rise on the
opposite side. This is called a trendelenberg gait. Reference : https://neckandback.com/conditions/walking-disorders-
how-nerve-and-joint-injuries-change-gait/ https://en.wikipedia.org/wiki/trendelenburg_gait
Https://en.wikipedia.org/wiki/superior_gluteal_nerve

8. Nerve to gluteus:
A) femoral
B) popliteal
C) ……obturator
D) …..obturator
Answer: ( insufficient information )

Reference : snell clinical anatomy by regions, p447

9. Patientwith trauma, femoral/hip fracture. Left leg short and laterally rotated. What muscle is responsible for lateral rotation:
A-gluteus maximus
B-rectus femorus
C-gracilus
D-adductor magnus
Answer: a
The lateral rotators are: the superior gemellus, inferior gemellus, obturator externus, obturator internus, quadratus femoris, gluteus
maximus and the piriformis.
Http://radiopaedia.org/articles/gluteal-muscles

209
10. A patient had a dislocation in his jaw, the doctor wanted to retract the mandible to his position which of the following
retracts the mandible in his normal position?
A) temporalis
B) lateral pterygoid,
C) medial ptergypoid,
D) masseter
Answer: a?
The temporalis is the most powerful muscle of the temporomandibular joint. Functionally, the muscle can be divided into two parts:
the anterior part runs almost vertically and moves the mandible forward (protrusion). The posterior part course almost horizontally
and pull the mandible backwards (retrusion). Spasm of the masseter, temporalis, and internal pterygoid muscles results in trismus,
preventing return of the condyle to the temporal fossa.
Reference: http://emedicine.medscape.com/article/149318-overview

11. Which part of the spermatic cord originate from internal oblique abdominal muscle ?
A. Internal spermatic sheath.
B. External spermatic sheath
C. Tunica vaginalis.
D. Cremaster muscle.
Answer: d
Aponeurosis of external oblique >>> external spermatic fascia
Internal oblique and its fascia >>> cremasteric muscle
Transversalis fascia >>> internal spermatic fascia

12. What is the position of the femoral vein to the artery?


A. Medial to anterior
B. Lateral
C. Posterior
D. Anterior
Answer: a

210
13. Which of the following muscles initiate unlocking of the knee during walking?
A. Plantaris
B. Tibialis anterior
C. Sartorius
D. Popliteus

Answer: d
The popliteus muscle in the leg is used for unlocking the knees during walking/standing by laterally rotating the femur on the tibia
during a closed chain movement. Reference: clinical kinesiology and anatomy.

14. Where does the lymph drain from the testicles?


A. Superficial inguinal
B. Deep inguinal
C. Para-aortic
D. Thoracic duct
Answer: c
Explanation: testicular lymph node drains into para-aortic lymph nodes. If question asked about scrotum then it is drained into
superficial lymph nodes. (i am not sure is it testicular or scrotum)

15. Patient with medial loss of sensation of one and half finger (palmar and dorsal surfaces). Which nerve is affected?
A. Median n
B. Ulnar n
C. Axillary n
D. Radial n
Answer: b

211
16. Which ligament pass inside inguinal canal
A. broad ligament
B. round ligament
C. uterosacral ligament
D. transverse cervical ligament
Answer: b: round ligament
The inguinal canal is a tubular structure that runs inferomedially and contains the spermatic cord in males and the round ligament in
females
Referance: medscape

17. Surgery in posterior triangle then develop loss of sensation in lower mandible ipsilateral which nerve is affected
A- lesser occipital nerve
B- grater occipital never
C- great auricle nerve
rd
D- 3 occipital
Answer: c

18. Knee dermatome?


A. L1:l2
B. L3:l4
C. L4:l5
D. L5:s1
Answer: l3:l4
Reference: medscape

212
19. Accessory nerve present in which triangle?
A. Posterior
B. Mental
C. Mandibular
D. Muscular

Answer: posterior cervical triangle

20. Patientafter rta , no abduction and lateral rotation of the arm.. . What is the origin of the affected nerve ?
A. Medial plexus
B. Lateral plexus
C. Lower plexus
D. Root
Answer: negative apley’s scratch test - suprascapular nerve (innervate supraspinatus and infraspinatus) and it is a branch of the
upper trunk.
Referance: snell’s clinical anatomy

21. What is the level of cardiac notch?


rd
A. 3
B. 4th
C. 5th,
D. 6th rip

Answer: b. 4th rib.


Reference: handbook of cardiac anatomy, physiology, and devices, page 58

22. Fecal incontinence which level is affected


A-above c2
B- below c2
C-above t12
D-below t12

213
Answer: d. Below t12

23. Inferior mesenteric artery supply?


A. Splenic flexure
B. Cecum
C. Sigmoid colon
D. Transverse colon
Answer c

24. Relationship of femoral vein to femoral artery?


A. Lateral
B. medial
C. anterior
D. posterior

answer: b
They are( van) from medial to lateral (vein then artery then nerve)
Reference : snell clinical anatomy by regions

25. Loss of sense of smell which lobe affected ?


A. Frontal
B. Occiptal
C. Parital
D. Temporal
Answer : frontal
Reference: uqu , ent chapter

26. Indirect hernia sac relation to cord?


A. Anterior medial
B. Anterior lateral
C. Posterior medial
214
D. Posterior lateral
answer: anteriomedial
recall

27. Child on picnic with family, presented to the er with high suspection of foreign body inhalation. The most common location in
the chest will be in?
A) right main bronchus
B) left main bronchus
C) carina of trachea
D) inlet of larynx
Answer: a
Same q in p.215 just add the 4th option

28. Patient lost sensation at the level of umbilicus after mva. The lesion will be at the level of
A) t4
B) t10
C) t11
D) t 12
Answer: b

29. Patientafter rta , no abduction and lateral rotation of the arm.. . What is the origin of the affected nerve ?
A. Medial plexus
B. Lateral plexus
C. Lower plexus
D. Root
Answer : c??
Deltoid m ..action : abducts arm ( 18 - 90 degree) teres minor m .. Action :- lateral rotate of arm. Both supply by axillary nerve from
posterior it is a part of the brachial plexus . It consists of contributions from all of the roots of the brachial plexus ( trunks )
Also, the upper trunk give suprascapular n which supply supraspinatus and infraspinatus
Supraspinatus action :- abducts of arm
Infraspinatus action :- lateral rotation of the arm

30. Superior rictus muscle >> eye movement


A. In down

215
B. Out down
C. Up in
D. Up out
Answer:c
Action of sr at the eye ball >> up + in
Movement of the eye when testing the muscle >> up + out
Ref: greys’ anatomy

31. Patient with pain in hands and fingers during typing, decrease of blood flow to which of the following will cause this pain?
A. Radial
B. Ulnar
C. Posterior interosseous
D. Anterior interosseous
Answer: b. Ulnar (superficial palmar arch), because q says ( fingers ) not specific one

32. If there was inferior mesenteric artery thrombosis. Which artery will not be affected! –
A. Descending colon -
B.sigmoid -
C.splenic -
D.cecum
Answer: c or d
D is the correct answer
Http://www.wesnorman.com/sup&infmesentericart.htm

33. Loss of sensation of anterior 2/3 of tongue which nerve affected?


A-facial
B-trigeminal
C-glossopharyngeal
D- hypoglossal
Answer: b
Sensory innervation of anterior 2/3 by trigeminal , taste intervention by facial
Sensory & taste innervation of posterior 1/3 by glossopharyngeal

34. Patient presents with otalgia, fever and sore throat? What is the nerve that refers ear pain?
A. Glossopharyngeal
B. Vagus
C. Sphenopalatine
D. Nasopalatine

Answer: a
Explanation: neuroanatomically, the sensation of otalgia is thought to center in the spinal tract nucleus of cn v. However, sensory
innervation of the ear is served by the auriculotemporal branch of trigeminal nerve, the first and second cervical nerves, the
jacobson branch of the glossopharyngeal nerve, the arnold branch of the vagus nerve, and the ramsey hunt branch of the facial
nerve.

216
Reference: http://emedicine.medscape.com/article/845173-overview#a6

35. Which of these not supplied by inferior mesentric artrey?


A- sigmoid ,
B- splenic flexure
C- cecum,
D- descending colon

Answer: c
Explanation: the inferior mesenteric artery (ima) is a branch of the abdominal aorta. It supplies the organs of the hindgut – the distal
1/3 of the transverse colon, splenic flexure, descending colon, sigmoid colon and rectum. The cecum is supplied by the superior
mesenteric artery.
References: http://teachmeanatomy.info/abdomen/vasculature/arteries/inferior-mesenteric/
Http://teachmeanatomy.info/abdomen/vasculature/arteries/superior-mesenteric/

36. Disease of spine got anterior chest abscess which nerve carry the infection
A- lateral cutaneus
B- anterior cutaneus
C- posterior cutaneus
D- diaphragmatic
Answer : b

37. Which layer in the scalp the nerves are found in :


A. Epicranial aponeurosis
B. Connective tissues
C. Skin
D. Pericranium

Answer: b
The superficial fascia is a fibrofatty layer that connects skin to the underlying aponeurosis of the occipitofrontalis muscle and
provides a passageway for nerves and blood vessels.
Http://emedicine.medscape.com/article/834808-overview

38. Non athlete heard a click at the left leg calf when he forcefully planterflex his left foot while the leg is fully extended.. On
examination, he can't planterflex his foot against resistance & can't walk by toe-foot walking of the affected site.. What is
the injured structure ? (anatomy)
A. Calcenear ligament
B. Quadriceps ligament
C. Planter ligament
D. Femoris rectus
Answer: C

39. Superficial palmar arch artery of :


A-ulnar

217
B-radial "deep arch"
C-anterior
D-posterior
Answer: a
Reference: http://www.healthline.com/human-body-maps/superficial-palmar-arch

40. Mid inguinal point ...artery posterior to it ?


A. External iliac
B. Femoral
C. Superior epigastric
D. Femoris

41. The nerve responsible for abduction of the eye?


A. 2
B. 3
C. 4
D. 6

Answer: d
Vi abducens: motor function: innervates the lateral rectus, which abducts the eye.
Https://quizlet.com/21466215/the-12-cranial-nerves-flash-cards/

42. Structure in front of left suprarenal gland?


A. Duodenum
B. Ivc
C. Pancreas
D. Kidney

Answer: c

43. A woman types on computers for living for long time, she developed pain and numbness over the hand and decrease of the
blood over palmar arch. What the artery is occluded?
A. Anterior interossios
B. Posterior interosseous
C. Ulnar
D. Radial

Answer: d
44. Case of carpal tunnel and difficulty in movement of fingers, what muscle is affected?
A. Medial lumbricle (should be lateral)
B. Dorsal interossius (ulnar n)
C. Palmar interossius (ulnar n)

218
D. Thenar (mixed median n and ulnar. Ulnar innervates the deep head of flexor policis brevis)

Answer: a ( if lateral) or d?
Clinically oriented anatomy / usmle step 1 – anatomy
Carpal tunnel median n in the hand (innervates: lumbricals for digits 2 and 3, the lateral lumbricals,

45. Location of saphenous vein:


A. Anterior to medial malleolus.
B. Posterior to medial malleolus.
C. Posterior to lateral malleolus.
D. Anterior to lateral malleolus.
Answer a
Http://emedicine.medscape.com/article/80393-overview

46. Which is Hinge Joint? (Anatomy-Ortho)


A. Shoulder
B. Elbow
C. Ankle
D. Knee
Answer: B
An another hinge joint example is the interphalangeal joints. Snell Anatomy

47. Frontal bone mass which lymph node going to be enlarged?


A- Mastoid
B- Sub mental
C- Submandibular
D- Parotid

Answer: C

48. Women complain of buttock pain In angiogram which artery most likely occluded?!
A. Internal ilic
B. External ilic
C. Femoral
D. Pudendal
Answer: A

49. Basal skull fracture with loss of sensation under the eye. Which of the following nerves is affected?
A. Frontal
B. Trochlear
C. Infraorbital
D. Supraorbital

Answer: C
219
After the maxillary nerve enters the infraorbital canal, the nerve is frequently called the infraorbital
nerve. This nerve innervates (sensory) the lower eyelid, upper lip, and part of the nasal vestibule.

50. Pectoralis major muscle was removed during mastectomy. Which action will be lost?
A. Abduction
B. Adduction
C. Extension
D. Internal rotation or flexion

Answer: B
Adduction and medial rotation http://www.orthobullets.com/anatomy/10008/pectoralis-major

51. patient lost sensation at the level of umbilicus after MVA . The lesion will be at the level of

A) T4
B) T10
C) T11
D) T 12
Answer : B

220
52. If there was inferior mesenteric artery thrombosis. Which artery will not be affected!
A. Descending colon *IMA
B. Sigmoid *IMA
C. Splenic *IMA
D. Cecum *SMA
Answer: D
Referenec: http://radiopaedia.org/articles/inferior-mesenteric-artery

53. 60 y.o smoker with lung cancer. When the tumor compresses the sympathetic innervation, it will lead to ?
A. Anhydrosis
B. Ptosis
C. Hydrosis
D. Myadrasis
221
Answer : B
- Pancoast tumors leads to compression of sympathetic ganglion causing Horner's syndrome.
- Horner's syndrome presents with : miosis(constriction of the pupils), anhidrosis (lack of sweating), ptosis (drooping of the eyelid)
and enophthalmos (sunken eyeball).

54. read about lower limb dermatom20-knee dermatome?


A. L1-L2
B. L3-L4
C. L4-L5
D. L5-S1
Answer : B

55. Patient developed dysphagia. On examination, there was deviation of the uvula to the left side. Which nerve is affected?
A. Right vagus
B. Left vagus
C. Right hypoglossal
D. Left glossopharyngeal
Answer: A
Lesions of the vagus nerve result in uvular deviation AWAY from the lesion. Lesions of the hypoglossal nerve result in the tongue
deviating TOWARDS the side of the lesion. The gag reflex involves 2 limbs: an afferent limb and an efferent limb. The
glossopharyngeal nerve (CN IX) carries afferent or sensory information, and the vagus nerve (CN X) carries efferent or motor
information.
http://bestpractice.bmj.com/best-practice/monograph/153/diagnosis/step-by-step.html

56. surgeon do operation (supra renal ) he injury anterior st ?


A. pancreas
B. duodnem
C. IVC
D. Kidney
222
Answer: C

57. glouteal Nerve or Artery come form ?


A. ext. ilic
B. nternal ilic
C. femoral
D. Bodenal
Answer: B

58. pt with thyroid goiter ,,,compress on external laryngeal nerve ,, what is the action affected:
A. tension of vocal cords
B. abuduction of vocal cords
C. adduction of vocal cords
D. Loss of sensation superior to vocal cords
Answer: A

59. Patient with drop foot, loss of sensation in 1st & 2nd metatarsal joints, what's the damaged nerve?
A. Common peroneal
B. Deep Peroneal
C. Tibial n.
D. Femoral
Answer: A

60. Liver biopsy :


A. Mid-axillary at 6th intercostal space
B. Mid-axillary at 7th intercostal space<<
th
C. Mid-axillary at 10
Answer: B **To choose the site, start with percussion over the right upper quadrant. The biopsy site is usually located in the
seventh or eighth intercostal space in the midaxillary line.
Reference: http://emedicine.medscape.com/article/149684-technique

61. Big and 2ed toe cannot dorsiflexion


A) deep pernoal n
B) post tibial n
C) sural n

Answer: a

223
Reference : snell clinical anatomy by regions

62. Muscle responsible for plantar flexion?


A) rectus femoris,
B) quads,
C) plantaris
Answer: c
Muscles used in planter flexion : gastocnemius ,soleus, plantaris, flexor hallucis longus, flexor digitorum longus ,tibialis posterior
Reference : brs gross anatomy

63. Patient complains of severe chronic constipation. Which muscle is important to be relaxed?
A) puborectalis
B) pubococcygeus
C) iliococcygeus
Answer: a
The puborectalis muscle is a muscular sling that wraps around the lower rectum as it passes through the pelvic floor. It serves an
important role in helping to maintain fecal continence and also has an important function during the act of having a bowel
movement. Paradoxical puborectalis syndrome occurs when the muscle does not relax when one bears down to pass stool. Resulting
in difficulty emptying the rectum, and sever constipation.
Reference : https://www.fascrs.org/patients/disease-condition/pelvic-floor-dysfunction-expanded-version

64. Patient was running and then felt pain in his left leg. The pain gets better with stretching the leg what is the muscle affected:
A) posterior tibial
B) gastrocnemius
C) soleus
Answer: b
The gastrocnemius is primarily involved in running, jumping and other "fast" movements of leg, and to a lesser degree in walking
and standing.
Reference : clinically oriented anatomy. P598

65. The ankle joint consists of what bones?


A) distal tibia, fabula, and talaus.
B) latral malloli, medial malloli, and talaus.
C) calclunus, tibia, and fibila.
224
Answer: a
Reference: brs gross anatomy

66. patient have blunt in right 4 intercostal space affected:


A) upper loop of lung
B) lower loop of lung
C) horizontal
Answer: c
In the anterior view, the horizontal fissure on the right side follows the contour of rib 4 and its costal cartilage and the oblique
fissures on both sides follow the contour of rib 6 and its costal cartilage.
Reference : https://web.duke.edu/anatomy/lab04/lab5_prelab.html

67. Waiter at a restaurant tripped on his knee and fractured his neck of the fibula while serving customers, sometime after he
noted dropping of his big toe while walking, which nerve got affected?
A) tibial n.
B) deep peroneal n.
C) common peroneal n
Answer: b or c?
The nerve begins at the apex of the popliteal fossa, then follows the medial border of the biceps femoris, running in a lateral and
inferior direction, over the lateral head of the gastrocnemius. To enter the lateral compartment of the leg, the nerve wraps around
the neck of the fibula, passing between the attachments of the fibularis longus muscle. Here, the common fibular nerve terminates
by dividing into the superficial fibular and deep fibular nerves. Common peroneal nerve gives 2 branches (superficial and deep) if the
superficial gets injured the clinical significance is that you cannot evert the foot and you will have sensation loss over some parts,
while if the deep peroneal is injured you will have a foot drop (similar to wrist drop in radial injury), you can claim it’s a
Common peroneal injury if all these symptoms present in the same case
Reference : http://teachmeanatomy.info/lower-limb/nerves/common-fibular-nerve/

68. Patient with lesion above the left eye brows , first lymph node to be examined is :
A) parotid
B) mental
C) submandibular
Answer: a
The lateral part of the face, including the lateral parts of the eyelids, is drained by lymph vessels that end in the parotid lymph
nodes. The central part of the lower lip and the skin of the chin are drained into the submental lymph nodes.
Reference : snell clinical anatomy by regions

69. What is the type of joint between the bodies of the vertebra?
A. Synovial
B. Cartilaginous
C. Suture

225
Answer: b

70. The radial pulse can be palpated lateral to which tendon:


A. Flexor carpi ulnaris
B. Flexor carpi radialis
C. Flexor digitorum profundus
Answer: b
It lies superficially between the tendons of brachioradialis and flexor carpi radialis. Proximally, the radial pulse is best appreciated
within the inverted v formed by the biceps tendon laterally and the bicipital aponeurosis medially. This inverted v also defines the
site where the radial recurrent artery (rra) branches off from the ra. Distally, the ra can be palpated between the radial styloid
laterally and the tendon of the flexor carpi radialis medially. Reference: annals of cardiothoracic surgery.
Source: wikipedia

71. Posterior vagal trunk supplies:


A. Esophagus
B. Jejunum
C. Descending colon
Answer: a
The posterior vagal trunk is a branch of the vagus nerve which contributes to the esophageal plexus. It consists primarily of fibers
from the right vagus, but also contains a few fibers from the left vagus. Reference: wikipedia.

72. What is the blood supply to posterior compartment of leg?


A. Tibial
B. Common fibular
C. Superficial fibular

Answer: a

73. Gluteal muscle what the nerve intervention :

226
Answer:
A. Gluteus maximus >> inferior gluteal nerve .
B. Gluteus medius >> superior gluteal nerve.
C. Gluteus monimus >> superior gluteal nerve.
Referance:http://teachmeanatomy.info/lower-limb/muscles/gluteal-region/

74. Muscle responsible of internal rotation of left lower leg:


a- maximums
b- gluteus medius or minimus
c- rectus femurs
answer: b

75. Loss of sensory sensation in the foot (big toe and 2nd toe). Which nerve is affected?
A. saphenous nerve
b. deep peroneal nerve
c. pudendal nerve
answer: b
snellen

76. What is the name of triangle between hyoid bone and two anterior belly of diagastric?
A. Submental triangle
B. Sub mandibular
C. Carotid
Answer: a. Submental triangle
- submental triangle: between the anterior belly of the digastric, superior to the hyoid bone, and the midline of the neck.
- submandibular (digastric) triangle: between the posterior and anterior bellies of the digastric muscle and inferior border of the
mandible. Its floor is formed by the mylohyoid, hyoglossus and middle constrictor muscles.
- muscular triangle: between the superior belly of the omohyoid, lower anterior margin of the sternocleidomastoid and the median
line of the neck.
- carotid triangle: between the posterior belly of the digastric, superior belly of the omohyoid and deep to the sternocleidomastoid
muscle.

227
Http://cosmos.phy.tufts.edu/~rwillson/dentgross/headneck/triangles/anterior%20triangle%20of%20the%20neck.htm

77. 44 years woman with hyperlidemia c/o of gluteal muscle atrophy, what is the most likely artery that affected by arthromotus
plaque?
A. Internal iliac artery

228
B. Internal pudendal artery
C. External iliac artery
Answer: a

78. Mountain climber who has hypoxia, which of the following liver zones is most affected by hypoxia?
A. Central of acini zone ii
B. Preiphral of acini zone ii
C. Sinusoidal
Answer: zone 3, the centrilobular zone iii has the poorest oxygenation, and will be most affected during a time of ischemia.

79. For LP you will insert the needle in:


A- l1- l2
B- l3 - l4
C- l5- s1
Answer: b
“l3/4 or l4/5 interspace” uptodate

80. Patient was running and then felt pain in his left leg. The pain gets better with stretching the leg what is the muscle affected:
A) posterior tibial
B) gastrocnemius
C) soleus
Answer: b
* calf strains are most commonly found in the medial head of the gastrocnemius.
Http://www.ncbi.nlm.nih.gov/pmc/articles/pmc2697334/

81. Absent gag reflex, loss of sensation posterior of the tounge:


A- glossopharyngeal nerve
B- vagus nerve
C- facial nerve
Answer: a

229
230
82. I think truma >> clear nasal discharge which affect
A) optic n
B) olfactory n
C) ophtham- ... )
Answer: b
Explanation: clear nasal discharge is a hint of csf due to fracture of the cribriform plate bone that contains nerve fibers of the
olfactory nerve.

83. Case head trauma on parietal lobe subdural hematoma which artery is injured?
A) superficial temporal .
B) mid cerebral
C) rt.cerebral .......
Answer : b

84. A woman complaining of left hand tingling mainly at thumb and index on exam there was mild atrophy of thinner muscle
tenil's test was positve which nerve may be affected?
A) radial nerve
B) median nerve
C) musculocutaneous nerve
Answer: b

85. Patientwith forehead multiple fractures which nerve is involved?


A- optic
B-olfactory,
C-ophthalmic

Answer c

86. Inferior horn of lateral ventricle which affect


A) hiccup
B) patman
C) cudate nucl✔.
Answer: c ?
Reference: textbook of human neuroanatomy
Explanation: roof of inferior horn of lateral ventricles is formed by tail of cudate nucli while the floor is formed by hippocampus.

87. Patientwith breast cancer and axillary ln involvement ..what muscle might be involved ?
A-pectoralis major ,
B-pecoralis minor ,
C-latismus dorsi
Answer b?
231
88. External laryngeal nerve action :
A. abduct cord
B. adduct cord
C. upper cord area sensation
Answer: b
-the cricothyroid muscle produces tension and elongation of the vocal folds by drawing up the arch of the cricoid cartilage and tilting
back the upper border of the cricoid cartilage lamina; the distance between the vocal processes and the angle of the thyroid is thus
increased, and the folds are consequently elongated, resulting in higher pitch phonation ( vocal folds adducted & stridulating as air is
forced between them)
- this muscle is the only laryngeal muscle supplied by the branch of the vagus nerve known as the external branch of the superior
laryngeal nerve

89. What type of radial nerve injury ?


A-nuropraxia
B-nurotemesis
C-axontemesis
Answer: a
Http://emedicine.medscape.com/article/1244110-overview
pathophysiology part

232
233
234
90. Which nerve will be affecting in a patent ductus arteriosus surgery?
A. Left vagus nerve
B. Phrenic nerve
C. Superficial nerves of the he

Answer: a
Explanation:
The left vagus nerve gives rise to the left laryngeal nerve in the superior mediastinum which courses under the aortic arch
Phrenic nerve arise from c3-4-5 and pass through the middle mediastinum laterally to give motor supply to the diaphragm
Reference usmle step1 anatomy lecture notes

91. Basal skull fracture , injury including the foramen ovale , which muscle will not be affected :
A- sternocloidomastoid
B- styloglossal
C- trapezius

Answer: b? (choices incomplete)


Explanation: in basilar skull fracture cranial nerves can be affected, and since foramen magnum is injured other foramen in the
temporal bone might be injured. A and c are supplied by the accessory nerve (cn xi) which exits through jugular foramen which is
bordered in part by the temporal bone. Styloglossal is supplied by the hypoglossal (cn xii) which exit through the hypoglossal canal in
the occipital bone. So, if the missing choice is a muscle not supplied by a cranial nerve, then it is the answer. If it is supplied by
cranial nerves, styloglossal is the likely answer.
References: http://emedicine.medscape.com/article/248108-overview#a10
Https://en.wikipedia.org/wiki/list_of_foramina_of_the_human_body

92. Thoracocentesis in midaxillary line :


A. 4th intercostal space
B. 5th intercostal space
C. Between 7 & 8 intercostal space

Answer: b??
Http://emedicine.medscape.com/article/80640-overview#a3
The optimal puncture site may be determined by searching for the largest pocket of fluid superficial to the lung and by identifying
the respiratory path of the diaphragm. Traditionally, this is between the 7th and 9th rib spaces and between the posterior axillary
line and the midline.

93. Cardiac notch in :


A. 4 th
B. 5 th
C. 2 nd
235
94. A question about superior laryngeal nerve if got injured what will be affected?
A) sensation above vocal cord
B) tension of vocal –abduction
C) adduction
Answer: a
The internal branch of superior laryngeal nerve (ibsln) provides general sensation, including pain, touch, and temperature for the
tissue superior to the vocal folds.
Link: http://emedicine.medscape.com/article/1923100-overview#a3

95. Compression of the external laryngeal nerve lead to loss of


A. Vocal cord abduction
B. Vocal cord adduction
C. Loss of sensation above vocal cord
Answer: a
Most of the muscles of the larynx receive their innervation via the recurrent laryngeal branch of the vagus nerve. Exception: the
cricothyroid muscle, which receives its innervation via the external laryngeal nerve. The posterior cricoarytenoid is the only abductor
of the vocal cords.

96. Patient with history of fall on his right leg, in severe pain, the pain decreases with passive stretch of his leg, which of the
following muscles is most likely affected??
A. Gastrocnemius
B. Popliteus
C. Soleus

Answer: a
Pubmed: the origin of the gastrocnemius and soleus are anatomically distinct arising from above and below the knee respectively.
This allows the examiner to isolate the activation of the muscles by varying the degree of knee flexion. With the knee in maximal
flexion the soleus becomes the primary generator of force in plantar flexion. Conversely with the knee in full extension the
gastrocnemius provides the greater contribution. This relationship allows for more accurate strength testing of the individual calf
muscles and enables the clinician to better delineate which muscle has been injured.
A similar approach is used to test pain and flexibility with passive ankle movements and stretching. In this case, the knee is again
placed in maximal extension and then subsequently in flexion while the ankle is passively dorsiflexed to cause relative isolated
stretch of the gastrocnemius and soleus respectively. Use of this technique for clinical isolation of the gastrocnemius and soleus is
key to determining the site of injury and guiding rehabilitate stretching and strengthening exercises as described below
Http://www.ncbi.nlm.nih.gov/pmc/articles/pmc2697334/

97. Cranial nerve supply skin of maxilla and mandible


A. 3
B. 4
C. 5

Answer: c

236
98. Gluteus muscle supplied by:
A- Internal iliac
B- External iliac
C- Femoral
Answer: A
They are supplied by superior gluteal arteries (all of the three muscles) and inferior gluteal arteries (only gluteus
maximus) which are branches of the internal iliac arteries

Reference: http://radiopaedia.org/articles/gluteal-muscles

99. Patient has fracture shoulder with winged scapula what is the nerve affected ?
A. Anterior
B. Posterior
C. Roots.
Answer: A. Anterior Rami of 5,6,7

100.Cancer of the lower lip. What are the lymph nodes that you will examine first?
A. Submental, submandibular.
B. Submental, Buccal.
C. Buccal, parapharyngeal.

Answer: A
The lymphatic drainage of both the upper and lower lips is primarily to the submandibular group of lymph nodes. To a lesser extent,
drainage may go to submental intraparotid, or internal jugular lymph nodes.
Reference: http://www.ncbi.nlm.nih.gov/pubmed/8460042

101.Case scenario, which nerve supply the pain when you chew ?
A. facial
B. vagus
C. auricotemportal and massitar branch of mandibular nerve
Answer: C

102.area of nail fold which nerve supply it :


A. meadin
B. ulnar
C. radius
Answer: A

103.pt with internal iliac artery injury ,, which part of body affected :
A. abdominal wall muscle
B. ront of thigh muscle
C. bladder

237
Answer: C

104.Patient who cannot flex his knee, absent ankle reflex, where is the level of injury?
A) l4-5,
B) l5-s1
Answer: b

Knee flexion Hamstrings (semitendinosus, semimembranosus, biceps femoris) Sciatic nerve L5, s1, s2

Reference: brs gross anatomy

105.What is the type of the pelvic bone:


A) pivot
B) ball and socket
Answer: b
The hip joint is a synovial joint formed by the articulation of the rounded head of the femur and the cup-like acetabulum of the
pelvis ( ball and socket joint).
Reference : thieme atlas of anatomy (2006), p 365

106.Which ligament prevent uterine prolapse?


A) round ligament
B) broad ligament
Answer:
Prolapse happens when the ligaments supporting the uterus become so weak that the uterus cannot stay in place and slips down
from its normal position. These ligaments are the round ligament, uterosacral ligaments, broad ligament and the ovarian ligament.
The uterosacral ligaments are by far the most important ligaments in preventing uterine prolapse.
Reference : kaplan obstetrics & gynecology ck2

107.In examination doctor touch posterior pharynx he want to test which nerve :
A) vagus
B) hypoglossal
Answer: a
Testing the gag reflex, afferent part by the glossopharyngeal cranial nerve nine (sensory part) and efferent part by the vagus cranial
nerve ten (motor part)
Reference : snell clinical anatomy by regions

108.Patient in dental clinic received local anesthesia and give numbness below eye , maxilla and part of the nose most likely
nerve blocked is :
A) sphenopalatine
B) infraorbital
Answer: b
Infraorbital beneath the orbital rim, and supraorbital for sensation above the orbital rim.
Reference : snell clinical anatomy by regions

238
109.Muscle of knee extension?
A) quadriceps
B) biceps femoris
Answer: a
Reference: rbs gross anatomy

110.Femoral neck # his leg was rotated laterally which muscle responsible
A) rectus femoris
B) gluteas maximus
Answer: b
The strong muscles of the thigh including the rectus femoris, the adductor muscles, and the hamstring muscles, pull the distal
fragment upward, so that the leg is shortened. The gluteus maximus, the piriformis, the obturator internus, the gemelli, and the
quadratus femoris rotate the distal fragment laterally, as seen by the toes pointing laterally.??
Reference : snell clinical anatomy by regions

111.Peroneal artery is a branch of which artery?


A. Femoral
B. Popliteal
Answer: b
Popliteal artery > posterior tibial artery > peroneal artery.

112.What is the narrowest part of the urethra?


A. Prostatic urethra
B. Membranous urethra

Answer: b

113.Knee trauma then can not fully extend the knee so which muscle is affected
A-quadriceps femoris
B-biceps femoris
Others i don't remember
Answer: extensors: quadriceps femoris
Ref: greys anatomy

114.Brain tumor compressed third portion of maxillary artery. Which of following artery will affect it:
A. Temporal artery
B. Sphenopalatine artery
Answer:sphenopalatine artery
Reference: greys anatomy

115.Fraction of head of tibia with drop feet which nerve?


A. Deep peroneal
B. Common peroneal
Answer: deep peroneal

239
116.Injury to temporal area and superficial temporal artery bleed which layer of the skull where vessels are ?
A. Peri-cranial
B. Epi-cranial aponeurosis
Answer: subgaleal hemorrhage? Galeal aponeurosis (epicarnial aponeurosis)

117.Upper outer mass in breast skin look like bakkering what is the cause ?
A. Pectoralis major
B. Crourp ligament

Answer: cooperligment

118.Patientwith right arm numbness and tingling in thumb and index fingers, symptoms increased with hands raised up. The
cause:
A. Thoracic inlet.
B. Thrombus
Answer: a

119.After surgery, patientloss of sensation of medial thigh, which nerve is affected?


A. femoral
B. obturator
Answer: obturator (l2 - l4)

120.What arises from the caudal part of the foregut and cranial part of midgut? (embryology)
A. Esophagus
B. Duodenum
answer b
midgut, forgut 》》duodenum /gastric /secum /
all Snellen
240
121.27 yo runner sudden painful left leg, pain relieved by stretching what is the muscle?
A. gastocnemus
B. soleus
Answer: b

122.Injury in knee, anterior displacement of leg bone which ligament affected?!


A- ant cruciate ligament
B- post cruciate
Answer: a

123.Patient present with signs of increased intracranial pressure: we will do ct scan to confirm it. What nerve examination can
help in diagnosis?
A- optic nerve
B- facial nerve
Answer:
↑ icp leads to cn iii and cn vi deficits (cn 3 & cn 6)
*fa step 2 ck- 8th ed.

124.Loss of sensation around the mouth and mandible, which nerve affected:
A- trigeminal nerve
B- facial nerve
Answer : a

125.Patientwhen walking and lifting his rt leg, his left hip drops, the doctor told the or that he has trendelenburg sign, which
muscle is affected?
A) left gluteus medius
B) right gluteus medius
Answer: b
Reference: grey’s anatomy for students.
Explanation: trendelenburg sign is occurs in people with weak or paralyzed gluteus medius or minimus muscle. The sign is
demonstrated by asking the patientto stand on one limb. When the patientstands on the affected limb, the pelvis severely drops
over the swing limb.

126.Which artery supplies av node & sa node?


A. Right coronary artery
B. Left coronary artery

Answer: a
Explanation:
Right coronary artery branches: sinoatrial and atrioventricular nodal arteries, posterior interventricular artery
Left coronary artery branches: anterior interventricular artery (left anterior descending) & circumflex artery
Usmle step1 anatomy lecture notes
Sinus node & atrioventricular (av node) nodal arteries: originates mainly from the right coronary artery

241
Http://www.pharmacology2000.com/cardio/cardio_risk/adult_cardiac_procedures/anatomy4.htm

127.Pharyngitis and tonsillitis pain from ?


A. Sphenoplatine nerve
B. Glossophrngyal nerve

Answer is b

128.Orbital mass with loss of sensation over lower eyelid , mandible , maxilla or zygomatic areas which nerve ?
A. Infra orbital
B. Maxillary
Answer: a
Https://en.m.wikipedia.org/wiki/infraorbital_nerve

129.Pain at right hand palmar arch test showing insufficient blood flow which could be injured
A. Radial
B. Ulnar
Answer: a
The deep palmar arch (deep volar arch) is an arterial network found in the palm. It is usually formed mainly from the terminal part of
the radial artery, with the ulnar artery contributing via its deep palmar branch, by an anastomosis. This is in contrast to the
superficial palmar arch, which is formed predominantly by the ulnar artery.
Https://en.m.wikipedia.org/wiki/deep_palmar_arch

130.Lymph nodes of breast walk along?


A. pectoralis major
B. pectoralis minor
Answer: a
Http://www.breastcancer.org/pictures/breast_anatomy/axillary_lymph_nodes

131.Question about cranial nerves and their location according to neck triangles?
One of them was accessory and another i can’t recall?
A. Anterior
B. Posterior
Answer. B
Cranial nerve in anterior triangle: facial [vii], glossopharyngeal [ix], vagus [x],accessory [xi], and hypoglossal [xii] nerves
Cranial nerves in posterior triangle: accessory nerve [cn xi]
Http://teachmeanatomy.info/neck/areas/posterior-triangle/
Http://teachmeanatomy.info/neck/areas/anterior-triangle/

132.Boy fight with 2 boys what system activated?


A. Sympathetic
242
B. Parasympathetic
Answer: a
Http://www.health.harvard.edu/staying-healthy/understanding-the-stress-response

133.Injury to internal iliac in MVA what is affected structure.


A) bladder
B) ovaries
Answer: a
Link: http://radiopaedia.org/articles/branches-of-internal-iliac-artery-mnemonic

134.Upper eyelid cancer which ln mets


A) parotid
B) deep cervical
Answer: ?
Source: http://www.eyeplastics.com/eyelid-anatomy-eyelid-blepharoplasty-or-eyelift-surgery.html
The lymphatic drainage of the eyelid is rather extensive. The majority of the upper eyelid and the outer half of the lower eyelid drain
into the pre-auricular lymph nodes, while a small part of the middle of the upper eyelid and the inner half of the lower eyelid drains
into the submandibular lymph nodes.

135.Hemorrhoids are:
A) veins
B) arteries
Answer: a
Link: http://www.health.harvard.edu/diseases-and-conditions/hemorrhoids_and_what_to_do_about_them

136.Man fall down from stairs on his face with many fracture on his jaw. Which muscle help in mastication?
A. Masseter
B. Temporalis
Answer: a

137.Function of acl
A. Medial rotation of tibia in relation to femur
B. Prevents anterior (forward) movement of the tibia off of the femur

Answer: b.

138.Muscle stretch thigh and leg:


A- Sarratus
B- Bicepsis femuris

139.Frontal bone fracture with concomitant infection. Which lymph node group should be examined?
A. Submental
B. Submandibular
243
Answer: B

140.Obturator nerve injury. Which muscle will get full paralysis?


A. Adductor Magnus
B. Adductor longus

Answer: B
NB. Adductor magnus has nerve supply from 2 nerves.

141.a patient has a trauma to his neck and he can’t abduct his shoulder, what is the nerve that was injured?
A. Axillary
B. long thoracic

Answer: B ??
Abduction is carried out by the deltoid(Axillary) and the supraspinatus in the first 90 degrees. From 90-180 degrees it is the trapezius
and the serratus anterior(long thoracic).
Reference: https://en.wikipedia.org/wiki/Shoulder_joint

142.Injury to jugular foramen which structure will not be affected all of option are muscle :
A. Strenocleidomastoid
B. Sphenioplatine and

Answer: sternocleidomastoid is least muscle affected during jugular foramen injury ,


-Jugular foramen muscle passing:
Sternocleidomastoid
Sphenopalatine
Digastric muscle
-If one of these choose SCM , if non of these choose it
Reference: by Neurosurgery consultant

143.Injury to temporal area and superficial temporal artery bleed which layer of the skull where vessels are ?
A. Peri cranial
B. Epi cranial aponeurosis
Answer: B

Reference:http://emedicine.medscape.com/article/881374-overview#a9

144.bronchial cancer mets to sympathetic plexus what the sign?


A. Ptosis
B. Dilated pupil
Answer : A

244
The answer is horners syndrome (ptosis ,anhydrosis,miosis)
-it results from an interruption of the sympathetic nerve supply to the eye and is characterized by the classic triad of miosis
(ie, constricted pupil), partial ptosis, and loss of hemifacial sweating (ie, anhidrosis).
-Horner syndrome can be congenital, acquired, or purely hereditary (autosomal dominant). The interruption of the
sympathetic fibers may occur centrally (ie, between the hypothalamus and the fibers’ point of exit from the spinal cord [C8
to T2]) or peripherally (ie, in cervical sympathetic chain, at the superior cervical ganglion, or along the carotid artery).
Reference: http://emedicine.medscape.com/article/1220091-overview#a3

145.Pt with melanoma above eyebrow which lymph node will examine ?
A. Cervical ..
B. Parotid lymph nodes
Answer:

146.also gluteal pain due obstrction where ?


A. ext ilic
B. intrnal ilic
Answer: B

147.Male patientcomplain of weakness in flexion of both rt knee and rt hip which muscle affected?
a) Sartorius
Answer: a
The sartorius is the longest muscle in the body. It is long and thin, running across the thigh in a inferomedial direction. At the hip
joint, it is a flexor, abductor and lateral rotator. At the knee joint, it is also a flexor.
Reference: http://teachmeanatomy.info/lower-limb/muscles/thigh/anterior-compartment/

148.Stab wound lateral to sternocleidomastoid, patient unable to do adduction of the shoulder and elevation over his head.
What is the nerve injured?
A) long thoracic
Answer: a
The serratus anterior is found more laterally in the chest and, forms the medial wall of the axilla. Function: the main action of
the serratus anterior is to rotatethe scapula, allowing the arm to be raised over 90 degrees. It also holds the scapula against the
ribcage – this is particularly useful when upper limb reaches anteriorly (e.g punching).
Innervation: it is innervated by the long thoracic nerve
Reference : http://teachmeanatomy.info/upper-limb/muscles/pectoral-region/

149.What is the name of the muscle that pass below lesser sciatic nerve canal?
A) pectenious m.
Answer: tendon of the obturator internus muscle
The following pass through the foramen: the tendon of the obturator internus ,internal pudendal vessels, pudendal nerve, nerve to
the obturator internus.
Reference : snell clinical anatomy by regions

245
150.Main action of anterior muscles of forearm?
Answer: flexion and pronation
In general, muscles in the anterior compartment of the forearm perform flexion at the wrist and fingers, and pronation. Reference :
http://teachmeanatomy.info/upper-limb/muscles/anterior-forearm/

151.Injury to deep peroneal nerve??


Answer: result in foot drop
Innervates the muscles in the anterior compartment of the leg, as well as some of the intrinsic muscles of the foot. Responsible
for dorsiflexion of the foot at the ankle joint. During the gait cycle for walking
Reference: brs gross anatomy

152.Dorsalis pedis pulse location?


A. Beside extensor hallucis longus.
Answer: a
Lateral to the extensor halluces longus and medial to the extensor digitorum longus tendon

153.Internal carotid artery branches.


Answer: ?
Http://www.bartleby.com/107/146.html

154.Which of the following nerves is responsible for adduction of fingers?


Answer:
Ulnar nerve. Reference: textbook of anatomy - upper limb and thorax

155.Which nerve supplies the frontal belly of the occipitofrontalis muscle?


A. Temporal branch of the facial nerve
Answer: a
Reference: endoscopic plastic surgery.

156.Which nerve is responsible for the gag reflex?


A. Glossopharyngeal
Answer: a
Afferent: glossopharyngeal ix.
Efferent: vagus x

157.Which nerve is responsible for tongue movement?


A. Hypoglossal
Answer: a

158.Muscle passing through the lesser sciatic foramen:


Answer?
246
Structures passing through the lesser sciatic foramen:
-piriformis
-inferior gluteal artery
-inferior gluteal nerve
-obturator internus
-pudendal nerve
-internal pudendal artery
-internal pudendal nerve
Reference: https://quizlet.com/63570717/structures-passing-through-greater-and-lesser-sciatic-foramen-
flash-cards/

159.In which scalp layer does vessels run?


A. Connective tissue (superficial fascia)

160.Origin of gluteal artery (scenario: ischemia in gluteaus area then ask for origin of gluteal artery)
A. Internal iliac artery
Answer: internal iliac artery
Reference: greys anatomy

161.Inferior alveolar n from facial nerve injury, during dental procedure? What's the manifestation?
Read about its function

Answer:
The main symptoms for the inferior alveolar nerve injury are: sensory paralysis of the lower lip on the affected side, the mental
region and the gingivae; stiffness, persistent pain; neuropathic pain such as allodynia; and pain and discomfort with occlusion.
Referance:http://www.aqb.jp/english/file/clinicalpracticepart3-6.pdf

162.Question about the thyroid gland and fascia related..


A. Deep cervical fascia
Answer: a

163.Patient has trouble dorsi flexing his ankle joint which structure is affected (1 question nerve and 1 muscle)
Answer: muscles: 1. Ant tibialis. 2. Extensor digitorum longus. 3. Extensor hallucis longus 4. Peronus tertius
Nerve affected: deep peroneal n.

164.Which of the following is not supplied by the inf. Mesenteric artery:


(splenic flexure, descending colon, transverse, cecum)
Answer: cecum

247
Reference: greys anatomy book

165.Artery in prepare inguinal hernia


Answer: testicular, cremasteric, and deferential a

166.3rd layer of scalp


Answer: epicranial aponeurosis
The scalp consists of five layers (seen in the image below):
1- the skin,
2- connective tissue,
3- epicranial aponeurosis,
4- loose areolar tissue
5-pericranium.
Referance: medscape

167.Tibia move for femur which ligament


Answer: acl?

168.Medial 1/3 loss of sensation in hand, where is the deformity


Answer: ulnar n

169.Unstable gate which artery response


Answer: if the gait he is talking about is waddling → gluteal medius and minimus affected → superior gluteal a.

170.Which ligament will be tearing during lp?


Answer: interspinous ligament
1- skin
2- facia and sc fat
3- surpaspinous ligament
4- interspinous ligament
5- ligamentum flavum
6- epidural space and fat (epidural anesthesia needle stops here)
7- dura

171.Phranic nerve paralysis:


A. Increased heart rate

Answer:
Clinical manifestations — patients with unilateral diaphragmatic paralysis are usually asymptomatic at rest, but may have
exertional dyspnea and decreased exercise performance [9,14]. However, patients with underlying or intercurrent lung disease
may experience dyspnea at rest. Orthopnea can also occur, but is not as intense as with bilateral diaphragmatic paralysis [7,15].
Unilateral diaphragm paralysis may also be associated with sleep-disordered breathing during rapid eye movement sleep
Referance: upodate

248
172.During laparoscopic surgery of inguinal hernia you find artery superficial going upward ?
A. Inferior epigastric artery.
Answer: a

173.Popliteal artery branch of?


A. Peroneal
Answer: femoral → external illiac → common iliac → abdominal aorta

174.Where do feel the pulse of factual artery?


Answer: located on the mandible (lower jawbone) on a line with the corners of the
Mouth (wiki)

175.Action of anterior compartment of the forearm muscle:


Answer: a a flex rest and flex finger

176.Trauma to the fibular head what is the nerve injured?


A. Common peroneal nerve
Answer: common peroneal n

177.From where peroneal artery come

Answer: post. Tibial a. > popliteal a > femoral a > external iliac

178.Which ligament comes before you reach epidural space?


A. Ligamintum flavum
Answer a

179.Indirect inguinal hernia weakness occur in which muscle


A. Transverse facialis

180.Stab wound lateral to sternocleidomastoid. Patient unable to do adduction of the shoulder and elevation over his head.
What is the narve injured
A- long thoracic
Answer: long thoracic?? (not sure, i don’t think its correct, long thoracic will affect the scapula & wing it, but not that..)

181.A 24 years old male was jogging and suddenly he had a muscle strain what is the most commonly affected muscle?
Answer: calf muscle

182.Gluteal artery from


A. Internal iliac artery
All snellen

249
183.Pressure on lesser omentum, what artery is compressed?
Answer: hepatic artery

184.Gluteus a. Supply?
A- internal ilia artery (is a branch from internal iliac)
superior gluteus artery supplies the iliacus, piriformis, and obturator internus muscles
inferior gluteus artery supplies the piriformis muscle, obturator internus, gluteus maximus and the superior hamstrings.
Answer: a

185.Long thoracic nerve is damaged, winged scapula is the result. Where does the long thoracic nerve originate from: a. Long
thoracic nerve is derived from ventral rami of c5, c6, c7 roots of brachial plexus
answer: a

186.Anatomy of maxillary artery


Answer: it branches from the external carotid artery
Http://study.com/academy/lesson/maxillary-artery-anatomy-branches.html

187.Patientc/o pain in second digit, which tendon affected?


Answer: ?

188.Qs about
* the anatomy of the arteries in the brain.
* big artery and branch what supply
* nerve and dermatome of upper and lower limbs

189.Blood supply in the hand, what make anterior arch?


Answer:
*superficial arch: predominant supply is ulnar artery. Minor supply from superficial branch of radial artery.
* deep arch: predominant supply is the deep branch of the radial artery. Minor supply from the deep branch of the ulnar artery.
Http://www.orthobullets.com/hand/6007/blood-supply-to-hand

190.What is posterior to the mid part of inguinal ligament or canal not sure
Answer: femoral a
Reference: teachmeanatomy
Explanation: the mid-inguinal point is halfway between the pubic symphysis and the anterior superior iliac spine. The femoral artery
crosses into the lower limb at this anatomical landmark.

191.Origin of gluteal artery ( scenario: ischemia in gluteus area then ask for origin of gluteal artery)
A) internal iliac artery
Answer: a

250
192.Occlusion to internal iliac artery, what area could be affected?
Answer: bladder

193.What is the artery supply posterior inferior nasal septum? (ent)


Answer: external carotid → internal maxillary → sphenopalatine a. → nasopalatine

194.Loss of sensation over the maxilla and mandible:


A) trigeminal
Answer:a

89- accessory n pass through which triangle


A- posterior triangle
Answer : a
Reference ( snell )

195.Stand on toes ..which nerve ?


A-tibial n .
Answer: a. Ref: grey’s anatomy

196.about kidney vein that drain to ivc (part of kidney)


Answer:
Both right and left renal vains drain into ivc
Right supra renal vain drains into ivc
Left supra renal vain dains into left renal vain
Ref: grey’s anatomy

197.Several dysfunctions can produce a positive trendelenburg test, including [6]


Superior gluteal nerve palsy
Lumbar disk herniation
Weakness of gluteus medius
Advanced degeneration of the hip
Legg-calvé-perthes disease
Ref :http://www.physio-pedia.com/trendelenburg_test

198.Innervation of stapedius?
Answer: facial
Explanation: the mastoid segment of facial nerve has s3 branches one of which is for stapedius.
Reference: http://emedicine.medscape.com/article/835286-overview#showall

199.Tumor or something compress on the 3rd part of the maxillary artery , which artery would has decreased flow :

251
A. Sphenopalatine

Explanation: maxillary artery has 3 parts. Third (pterygopalatine) part: anterior to lateral pterygoid muscle (six branches including
terminal branch). Branches: 1- posterior superior alveolar artery, 2- infraorbital artery, 3- artery of the pterygoid canal, 4- pharyngeal
artery, 5- greater (descending) palatine artery, 6- sphenopalatine artery - terminal branch Reference:
http://radiopaedia.org/articles/maxillary-artery

200.Loss of sensation of the posterior 2/3 if the tongue and no gag reflex
A. Ix

201.Artery supplying gluteal region :


o Superior gluteal artery which is the largest branch of the internal iliac artery

202.Which muscle passes through the sciatic foramen?


Answer: obturator internous*
Http://radiopaedia.org/articles/lesser-sciatic-foramen

203.Boundaries of sub mental tringle of the neck


Answer:
o Inferiorly – hyoid bone.
o Medially – imaginary sagittal midline of the neck.
Laterally – anterior belly of the digastric.
Link: http://teachmeanatomy.info/neck/areas/anterior-triangle/#submandibular_triangle

204.Scrotum layer anatomy?


(no choices listed)
nd
Answer: greys anatomy 2 ed
Layer anatomy from most superficial to deep:
External spermatic fascia
Cremasteric fascia
Internal spermatic fascia

Then: parietal then visceral layer of testis

205.Triangle of the neck?


Quadrangular area

252
A quadrangular area can be delineated on the side of the neck. This quadrangular area is subdivided by an obliquely prominent
[1]
sternocleidomastoid muscle into an anterior cervical triangle and a posterior cervical triangle.
Anterior cervical triangle
The anterior cervical triangle is bounded by the midline anteriorly, mandible superiorly, and sternocleidomastoid muscle
inferolaterally. This triangle is subdivided into 4 smaller triangles by the 2 bellies of the digastric muscle superiorly and the superior
belly of the omohyoid muscle inferiorly.
Submandibular triangle
The submandibular triangle is bounded by the mandible and 2 bellies of the digastric muscle. It contains the submandibular salivary
gland, hypoglossal nerve, mylohyoid muscle, and facial artery.
Carotid triangle
The carotid triangle is bounded by the sternocleidomastoid muscle, posterior belly of the digastric muscle, and superior belly of the
omohyoid muscle. It contains the carotid arteries and branches, internal jugular vein, and vagus nerve.
Muscular or omotracheal triangle
The muscular or omotracheal triangle is bounded by the midline, hyoid bone, superior belly of the omohyoid muscle, and
sternocleidomastoid muscle. It includes the infrahyoid musculature and thyroid glands with the parathyroid glands.
Submental triangle
The submental triangle is located beneath the chin, bounded by the mandible, hyoid, and anterior belly of the digastric muscle.
Posterior cervical triangle
The posterior cervical triangle is bounded by the clavicle inferiorly, sternocleidomastoid muscle anterosuperiorly, and trapezius
muscle posteriorly. The inferior belly of the omohyoid divides this triangle into an upper occipital triangle and a lower subclavian
triangle.
Occipital triangle
The occipital triangle is bounded anteriorly by the sternocleidomastoid muscle, posteriorly by the trapezius, and inferiorly by the
omohyoid muscle. The contents include the accessory nerve, supraclavicular nerves, and upper brachial plexus.
Subclavian triangle
The subclavian triangle is smaller than the occipital triangle and is bounded superiorly by the inferior belly of the omohyoid muscle,
inferiorly by the clavicle, and anteriorly by the sternocleidomastoid muscle. The contents include the supraclavicular nerves,
subclavian vessels, brachial plexus, suprascapular vessels, transverse cervical vessels, external jugular vein, and nerve to the
subclavius muscle.

206.Nerve supply of parotid gland?


The parotid gland receives both sensory and autonomic innervation. Sensory innervation is supplied by the auriculotemporal nerve,
a branch of the mandibular nerve. The autonomic innervation controls the rate of saliva production and is supplied by the
glossopharyngeal nerve.[4] postganglionic sympathetic fibers from superior cervical sympathetic ganglion reach the gland as
periarterial nerve plexuses around the external carotid artery, and their function is mainly vasoconstriction. The cell bodies of the
preganglionic sympathetics usually lie in the lateral horns of upper thoracic spinal segments. Preganglionic parasympathetic fibers
leave the brain stem from inferior salivatory nucleus in the glossopharyngeal nerve and then through its tympanic and then the
lesser petrosal branch pass into the otic ganglion. There, they synapse with postganglionic fibers which reach the gland by hitch-
hiking via the auriculotemporal nerve, a branch of the mandibular nerve.

207.Anasthesia subeodural betweel3 -l4 which ligament will be perforated?


I suspect that this is about the technique of epidural anesthesia because there is no such thing as subdural anesthesia.

253
Slow fahad see it`s lumber epidural dissection.......
1- skin
2- facia and sc fat
3- surpaspinous ligament
3- interspinous ligament
5- ligamentum flavum
6- epidural space and fat (epidural anesthesia needle stops here)
7- dura

208.Renal segment of inferior vena cava drain from the right side by which branch?
I have no idea what the “renal segment of the ivc” is.
Renal venous drainage is as follows:
The renal veins are veins that drain the kidney. They connect the kidney to the inferior vena cava. They carry the blood filtered
by the kidney.
There is one vein per kidney, that divides into 4 divisions upon entering the kidney:
o The anterior branch which receives blood from the anterior portion of the kidney and,
o The posterior branch which receives blood from the posterior portion.
Because the inferior vena cava is on the right half of the body, the left renal vein is generally the longer of the two.
[1]
Because the inferior vena cava is not laterally symmetrical, the left renal vein often receives the following veins:
▪ Left inferior phrenic vein
▪ Left suprarenal vein
▪ Left gonadal vein (left testicular vein in males, left ovarian vein in females)
▪ Left 2nd lumbar vein
This is in contrast to the right side of the body, where these veins drain directly into the ivc.
Often, each renal vein will have a branch that receives blood from the ureter.
Source: https://en.wikipedia.org/wiki/renal_vein

209.Gluteal muscle insufecincy ? Infer& superior glteal artery >> internal & external iliac artery .
Answer: ? Deficient information.

210.Ingunal lemphedenopathy in the medial part of the transverse group ? Anal area below pectinate line.

211.loss of sensation in medial part of thigh nerve affected.


Answer:?
Obturator nerve: Medial thigh
Anterior cutaneous branches of the femoral nerve: Anteromedial thigh

254
212.A patient presented with ischemia of the gluteus area. What’s the origin of gluteal artery?
A. Internal iliac artery

Answer: A

213.What area could be affected if there was occlusion of the internal iliac artery?
Answer: ?

214.Patient presents with blunt trauma that affect the inferior mesenteric artery, Which one of the following branches NOT
affected?
Answer:?
Inferior mesenteric artery has 3 branches:
- Left colic artery
- Sigmoidal arteries
- Superior rectal artery

215.Pt underwent endoscopic surgery for sinuses 2 days, pt developed numbness in infraorbital area which nerve is affected!?
(No choices listed)

255
216.Patient lost foot dorsiflexion, which muscle is affected ?
( can't remember the choices)

217.3rd layer of scalp:


Answer: aponuerosis

218.Patient head trauma, came superficial temporal artery laceration, what's the location of this artery in term of scalp layer?
Answer: aponuerosis

219.Blood supply of the appendix: the appendicular artery (appendiceal artery) is a terminal branch of the ileocolic artery.

220.3 years old child came to er with pronated arm flexed wrist which ligament affected?
Annular

221.Loss of sensation in the anterior aspect of leg and weak planter flexion what is the nerve root:
L5-s1 rechecked my answer

222.Parotid duct obstruction and pain during eating in ear what nerve responsible for this pain
A- facial nerve
Answer: a

223.What is the 3rd branch of maxillary artery?

256
224.The foramen than present between the 3rd and 4th ventricle is?
Answer: aqueduct of sylvius. The third ventricle also communicates with the fourth ventricle through the narrow cerebral (sylvian or
mesencephalic) aqueduct.

Http://www.cerebromente.org.br/n02/fundamentos/ventriiii_i.htm

225.Read about how to differentiate bw carpal tunnel syndrome and thoracic outlet obstruction
Answer:
Http://www.uptodate.com/contents/carpal-tunnel-syndrome-clinical-manifestations-and-
diagnosis?Source=search_result&search=carpal+tunnel+syndrome&selectedtitle=2~130
Http://www.uptodate.com/contents/overview-of-thoracic-outlet-
syndromes?Source=search_result&search=thoracic+outlet+obstruction&selectedtitle=1~150

226.Picture of sole hand shows


Answer: (insufficient information )

227.Nerve dermatomes of lower and upper limbs + cranial nerves


(not specific question)

228.Major arteries and their branches


257
(not specific question)

229.Flexion and pronationlling worse at morning. Dx?

230.Q's about the anatomy of the arteries in the brain.

258
Embryology

259
1. Which structure is made out of the Foregut and Midgut?
A. Duodenum
B. Esophagus
C. Stomach
D. Ilium

Answer: A
Explanation: Gut tube derivatives:
st nd
Foregut: Esophagus, stomach, 1 & 2 part of the duodenum, liver, pancreas, biliary apparatus & gallbladder
nd rd th
Midgut: 2 ,3 & 4 part of duodenum, jejunum, ileum, cecum, appendix, ascending colon & proximal 2/3 of transverse
colon
Hindgut: distal 1/3 of transverse colon, descending colon, sigmoid colon, rectum & anal canal above pectinate line
Reference: USMLE step1 anatomy lecture notes

2. Embrology question about inferior vena cava renal segment


A. Posterior cardinal
B. Supracardinal
C. Saccrocardinal
D. Subcardinal

Answer: D
Source: radiopedia website
Normal IVC has a complex embryological development with many embryological veins contributing to different parts:
Right vitelline vein: forms suprahepatic and hepatic segments of IVC
Right subcardinal vein: forms suprarenal segment
Right subsupracardinal anastomosis: forms renal segment
Right supracardinal vein: forms infrarenal segment
Right posterior cardinal vein: forms distal most IVC and its bifurcation into common iliac veins

3. Non keratinized stratified squamous epithelium:


A. Small bowel
B. Large bowel
C. Esophagus

Answer: C

4. Origin of right atrium embryo?


A. Cordis in heart
B. Change of transposition of GA
C. Ectopic cordis

Answer:
None of the above the origin of the right atrium is Sinus venosus
260
Primitive heart tube -has 5 dilatations.
1.truncus arteriosus-gives aorta and pulmonary trunk
2.bulbus cordis-smooth parts of rt & left ventricles
3.primitive ventricle-trabeculated parts of rt &left ventrcles.
4.primitive atrium- gives trbeculated parts of rt & left atria.
5.sinus venosus- gives the smooth part of rt atrium, coronary sinus, oblique vein of left atrium

5. Lung Embryology (about alveolar?)


Which phase the alvulo endo..epithilial of the lung become mature ?
A) Alvular
B),calvcalus,
C) Terminal sac

Answer: ?
Development of the lung can be divided into two phases, lung growth (structural development) and lung maturation (functional
development). Lung growth can be influenced by a host of physical factors. Lung maturation and the achievement of functionality is
primarily a biochemical process and is under the control of a number of different hormones. Lung growth proceeds through
gestation. There is progressive branching of the airways and finally development of alveolar spaces capable of gas exchange in the
last trimester. The surfactant system, composed of phospholipids that decrease surface tension within the alveoli and prevent
alveolar collapse during exhalation, develops in the last trimester, and reaches maturity by approximately 36 weeks. Lung growth
continues after birth as alveolar number continues to increase. The end result of the development of the lung is an organ with a
2
tremendously large surface area that is approximately 50-100 m , capable of exchanging oxygen and carbon dioxide across a very
thin membrane.
Reference: http://www.columbia.edu/itc/hs/medical/humandev/2004/Chpt12-lungdev.pdf

261
6. What is the origin of smooth part of right atrium?
A. Right sinus venosus
B. Left sinus venosus

Answer: A
Sinus venosus (R horn): smooth part of right atrium (sinus venarum) and the "valve" of the superior vena cava. The sino-atrial node.
Sinus venosus (L horn): coronary sinus, valve of coronary sinus.
Reference: http://www.nervenet.org/embryo/hderiv.html

7. Which hepatic cell produce extra matrix component ?


A. Stellate cell
B. Hepatocytes

Answer: a ( Stellate cell = Ito cell )

8. The embryogenic origin of tongue muscle?


A. Lingual
B. Occipital

Answer: B , Originating in occipital somites .


Ref : Langman’s Medical Embryology https://embryology.med.unsw.edu.au/embryology/index.php/Tongue_Development
http://www.embryology.ch/anglais/sdigestive/gesicht04.html

9. What arises from the caudal part of the foregut and cranial part of midgut? (Embryology)

C. Esophagus
D. Duodenum

Answer B
Midgut, forgut 》》duodenum /gastric /secum /
All snellen

10. Organ that originate from midgut and hindgut? (Embryology)


A. Duodenum
B. Transverse colon

Answer: transverse colon

262
11. Follicular cell of ovary what it is orign embryologically ???
Answer:
Cortical Cords , which split into isolated cell clusters ,with each surrounding one or more primitive germ cells . Germ cells
subsequently develop into oogonia , and the surrounding epithelial cells , descenants of the surface epithelium , form Folicular
cells .
Ref: Langman’s Medical Embryology .

12. Failure of 5th branchial arch development with 1st ,2nd and 4th something produces?

13. Failure of joining renal collecting duct with distal tubules.


Answer: ?
Multicystic Renal Dysplasia?
Reference: http://emedicine.medscape.com/article/982560-overview#showall

263
Histology

264
1. What is the strongest part of the vein?
A. Tunica intima
B. Tunica media
C. Tunica adventitia
D. Serosa (Not sure if it was there)

Answer: C
Source: flashcards of Quizlet

2. In which phase the endothelial lining of alveoli came in contact with blood vessels?
Alveolar phase.
A. Canalicular phase.
B. Pseudoglandular phase.
C. Saccular phase.

Answer: B
Source: http://www.embryology.ch/anglais/rrespiratory/phasen04.html

3. W
h
a
t

s

t
h
e

o
r
i
g
i
n

o
f juxtaglomerular apparatus?

265
A. Macula densa cell
B. Bowman capsule

Answer: A
The juxtaglomerular apparatus consists of three cells:
The macula densa, a part of the distal convoluted tubule of the same nephron.
Juxtaglomerular cells, which secrete renin.
Extraglomerular mesangial cells.

4. Which cell play as stem cell in tracheal epithelium?


A- Basal
B- Ciliated

Answer:

5. Calcitonin secreted from?


A. Parafollicular

Answer: parafollicular cell (C cell)

6. Melanine pigment from what cell :


A. Somatotrope

Answer:
Melanocyte, there are other cells able to produce melanin but of different embryonic origin (pigmented epithelium of retina, some
neurons, adipocytes)

7. What is the glomerular basement membrane barrier:

The kidney’s glomerular filtration barrier consists of two cells—podocytes and endothelial cells—and the glomerular basement
membrane (GBM)

Reference: http://www.ncbi.nlm.nih.gov/pmc/articles/PMC3334451/

8. Residual ribosomal RNA in which cell:


o RBC Reticulocyte. Ross Histology Text and Atlas 6th edition.

9. Melanin pigment from what cell :


o Melanocytes produce and distribute melanin into keratinocytes. Ross Histology Text and Atlas 6th edition.

266
10. Deepest layer of vein:
o Tunica intima.

Physiology

267
1. Organ responsible for multiorgan failure?
A. Heart
B. Lung
C. Kidney
D. Liver
Answer: a or D

2. Which system or organ will work in stress ?


A. Respiratory
B. Renal
C. Sympathetic
D. Parasympathetic

Answer: C

3. What the cell in the stomach responsible for production of vitamin B12?
A. Parietal cells
B. Chief
C. Global

Answer: A
Parietal cells secrete intrinsic factor which is responsible for absorbtion of vitamin B12.
Reference: http://www.merckmanuals.com/professional/nutritional-disorders/vitamin-deficiency,-
dependency,-and-toxicity/vitamin-b-12
Parietal cells do NOT produce vitamin B12. Parietal cells produce Intrinsic Factor which binds Vitamin B12 to be absorbed.
Davidson’s Medicine Page: 1020:
In the stomach, gastric enzymes release Vitamin B12 from food and at gastric ph it binds to a carrier protein termed R protein. The
gastric parietal cells produce intrinsic factor, a vitamin B12 binding protein, which optimally binds vitamin B12 at ph 8. As gastric
emptying occurs, pancreatic secretion raises the ph and vitamin B12 released from the diet switches from the R protein to intrinsic
factor. The vitamin B12-intrinsic factor complex binds to specific receptors in the terminal ileum.

4. With orthostatic hypotension what happen?


A. Decreas extracullar volum
B. Decrease interacullar volum

Answer: A
- ECF Volume Disturbance B Deficit (Dehydration) signs and symptoms:
Acute weight loss
268
Decreased skin turgor, Dry mucous membranes, Rough, dry tongue (longitudinal furrows in tongue)
Changes in behavior B agitation, restlessness, weakness
Flat neck veins in supine position
Weak thready pulse
Orthostatic hypotension
Slow-filling peripheral veins
http://www.austincc.edu/adnlev2/rnsg1443online/fluid_electrolytes_acid_base/summary_tables.htm
- Non-pharmacological measures are regarded as a cornerstone in the treatment of orthostatic hypotension. These measures consist
of chronic expansion of the extracellular volume or reducing the vascular volume in which pooling occurs.
Http://www.ncbi.nlm.nih.gov/pmc/articles/PMC2269496/

5. The corrected sodium of a patient is 138, what is the anion gap? (no Additional information)
Answer: ?

6. ABG question (compensated vs non compensated, respiratory acidosis vs metabolic acidosis)


Answer:

7. History of metabolic acidosis and lab showing decrease of ph and Hco3 how to compensate ?
A. CO2 wash out (hyperventilation)
Answer: A

8. Normal anion gap ?


o The normal value for the serum anion gap is 8-16 meq/L. Medscape
9. Melanocyte stimulating hormone released from?

10. Intermediate lobe of pituitary (Pars intermedia)

269
Biochemistry

270
1. Man eating rice only, he has gingival and tongue lesions. Which of the following deficiency you will find?
A. Vitamin a.
B. Vitamin c.
C. Thiamin(b1).
D. Niacin (b3).

Answer: b
In asia when white rice became more popular than brown rice, people started to develop a nerve damage disease called beriberi. It
can cause hypersensitivity of oral mucosa, burning sensation of the tongue and loss of taste.
References: http://www.dietobio.com/vegetarisme/en/vit_b1.html+
https://books.google.com.sa/books?id=ebqyie7bgsyc&pg=pa12&lpg=pa12&dq=#v=onepage&q&f=false

2. Patient irrigated, and weakness in lower and upper extremities, what is the cause?
A. Deficiency in vit d
B. Deficiency in vit a
C. Deficiency in b1
D. Deficiency in b3

Answer: c
Thiamine b1 deficiency (beriberi) cause poor memory, irritability, sleep disturbance, wernicke encephalopathy, korsakoff syndrome,
bilateral, symmetrical lower extremities paresthesias, decreased vibratory position sensation, absent knee and ankle jerk, muscle
atrophy.
http://emedicine.medscape.com/article/116930-clinical

3. Which of the following enhance non-heme iron absorption?


A. Coffee
B. Tea
C. Milk
D. Vitamin C
Answer: D

4. Which of the following is prescribed with iron supplement for better absorption?
A. Vitamin D
B. Vitamin E
C. Vitamin C
D. Calcium
Answer: C

271
5. Protein enter proteosome for degradation in form of ?
A. Folded N-terminal
B. Folded C-terminal
C. Unfolded N-terminal
D. Unfolded C-terminal
Answer: C
Https://www.ebi.ac.uk/interpro/potm/2006_8/Page2.htm

6. What is the source of energy to transfer molecules in and out of the nucleus?
A. ATP cytoplasmic
B. ATP nuclear
C. GTP cytoplasmic
D. GTP nuclear
Answer : a
Molecule movement requires energy in order to move solute against its electrochemical gradient. There are two ways in which a
flow of energy can be coupled to transporters: Primary active transport requires energy is provided by cytoplasmic atpase and
Sodium, potassium—atpase (Na, K—atpase) is present in plasma membranes.
Reference : http://www.biology-online.org/9/3_movement_molecules.htm

7. Which one has powerful molecular energy in the cell


A- ATP hydrolysis move component from cytoplasmic to nucleus
b- GTP hydrolysis move component from cytoplasmic to nucleus
C- ATP hydrolysis move component from nucleus to cytoplasmic
D- GTP hydrolysis move component from nucleus to cytoplasmic
Answer : B or C ??
Cellular GTP concentration dramatically changes in response to an organism’s cellular status, Biochemical analyses show that
PI5P4Kβ preferentially utilizes GTP, rather than ATP, for PI(5)P phosphorylation, and its activity reflects changes in direct proportion
to the physiological GTP concentration.
Reference : http://www.cell.com/molecular-cell/pdf/S1097-2765(15)00945-4.pdf

8. Which one of these is considered as sugar pentose?


A. Mannose
B. Arabinose
C. Erythrose
D. Glucose

Answer: B
Https://en.wikipedia.org/wiki/Pentose

9. Resting to vigorously exercise:


A. Pyrvate to lactate
B. Pyrvate to co2+o2

272
C. Decrease AMP
D. Decrease ndh/nadh
E. Aerobic Metabolism
Answer: B
Pyruvate is the starting molecule for oxidative phosphorylation via the Krebb's or citric acid cycle. In this process, all of the C-C and
C-H bonds of the pyruvate will be transferred to oxygen.

10. Problem in the phagocytosis in a child with normal immunoglobulin:


Which protein defect give problem in oxygenation:
A. Cytochrome 460
B. Lysosmal
C. Something with nitric
D. Decrease in NADH
Answer: D
Chronic granulomatous disease (CGD) is a rare (1:250,000 births) disease caused by mutations in any one of the five components of
the nicotinamide adenine dinucleotide phosphate (NADPH) oxidase in phagocytes. This enzyme generates superoxide and is
essential for intracellular killing of pathogens by phagocytes.

11. How does the protein enter the peroxisomes?


A. Folded with the help of C-terminal
B. Unfolded with the help of T-terminal
C. Folded with the help of T-terminal
D. Unfolded with the help of C-terminal

Answer: A
Peroxisomal proteins are fully folded in the cytoplasm and enter the organelle in folded form. The most common signal sequence
which directs proteins to peroxisomes is a C-terminal Ser-Lys-Leu tripeptide. Reference: Biochemistry and Molecular Biology
Textbook.
Http://global.oup.com/uk/orc/biosciences/molbiol/snape_biochemistry5e/student/mcqs/ch27/ Q5

12. What is the normal anion gap in mmol/l?


A. 13
B. 26
C. 95
D. 123
Answer : A
Normal anion gap 8-16 mmol/l or 3–11 meq/L

13. What is move the molecules in cell in and out


A. ATP cytoplasmic (ans by DR)
B. ATP neucliar
C. GTP cytoplasmic
D. GTP necular

14. Which one has powerful molecular energy in the cell?

273
A. ATP hydrolysis move component from cytoplasmic to nucleus
B. GTP hydrolysis move component from cytoplasmic to nucleus (ans by Dr)
C. ATP hydrolysis move component from nucleus to cytoplasmic
D. GTP hydrolysis move component from nucleus to cytoplasmic

15. About energy of the cell, which is correct?


A. ATP hydrolysis inside cytoplasm
B. ATP hydrolysis inside nucleus
C. GTP hydrolysis inside cytoplasm
D. GTP hydrolysis inside nucleus

Answer: C

16. How does the nucleus get energy?


A. Folded ATP c- terminal
B. Folded GTP c- terminal
C. Unfolded ATP N- terminal
D. Unfolded GTP N- terminal

17. A man who eat rice only, which vitamin will be deficient?
A) B1
B) B2
C) B6
D) B12
Answer A

Historically, beriberi has been common in regions where what is variously referred to as polished or white rice forms a major part of
the diet, which has its husk removed to extend its shelf life and palatability but has the side effect of removing the primary source of
thiamine (B1).

18. Molecule made in neuclolus ?


A. DNA,
B. Mrna,
C. Rrna
D. RNA
Answer: C

Three major components of the nucleolus are recognized: the fibrillar center (FC), the dense fibrillar component (DFC), and the
[4]
granular component (GC). The DFC consists of newly transcribed rrna bound to ribosomal proteins, while the GC contains RNA
bound to ribosomal proteins that are being assembled into immature ribosomes.

274
19. Mushroom inhibits which enzyme;
A. RNA Polymerase I
B. RNA Polymerase II
C. RNA Polymerase III
D. DNA ...
Answer: B
First aid biochemistry chapter in molecular biochemistry

20. The source of energy in nucleus :


A. Atp in cytoplasm
B. Atp from nucleus
C. Gtp from cytoplasm ?
D. Gtp from nucleus
The driving force is gtp in cytosol

21. Patient irriagate , and weakness in lower and upper extremities , what is the cause ?
A) Deficiency in vit D
B) Deficiency in vit A
C) Deficiency in B1
D) Deficiency in B3
Answer: C
Explanation: symptoms of Vit. B1; (beriberi) include weight loss, emotional disturbances, impaired sensory perception, weakness
and pain in the limbs, and periods of irregular heart rate. Edema (swelling of bodily tissues) is common.

22. Which of the following made in nucleolus:


A. DNA
B. Mrna
C. Rrna
Answer: mrna?

23. Organic phosphate, found in or effect of :


A. Liver
B. Kidney
C. Lung
Answer: C
Mnemonic devices used to remember the muscarinic effects of organophosphates are SLUDGE (salivation, lacrimation, urination,
diarrhea, GI upset, emesis) and DUMBELS (diaphoresis and diarrhea; urination; miosis; bradycardia, bronchospasm, bronchorrhea;
emesis; excess lacrimation; and salivation). Muscarinic effects by organ system include the following:
Cardiovascular - Bradycardia, hypotension
Respiratory - Rhinorrhea, bronchorrhea, bronchospasm, cough, severe respiratory distress
Gastrointestinal - Hypersalivation, nausea and vomiting, abdominal pain, diarrhea, fecal incontinence
Genitourinary - Incontinence

275
Ocular - Blurred vision, miosis
Glands - Increased lacrimation, diaphoresis
Ref : Medscape

24. Bond between DNA and trna:


A. Covalent
B. Ionic
C. Hydrophiclic
Answer: A?
There is no bond between DNA and TRNA

25. Synthesized inside the nucleus :


A- DNA
B- M RNA
C- T RNA Other types of RNA.
Answer: mrna. DNA is not in the nucleus in the metaphase.

26. What's the bound between T-RNA and m-RNA


A. Hydrophobic
B. Hydrogen
C. Convalent
Answer : B
Http://academic.brooklyn.cuny.edu/biology/bio4fv/page/antico.htm
Https://en.wikipedia.org/wiki/Transfer_RNA#Anticodon

27. After endoplasmic reticulum protein moves to:


A. Golgi apparatus
B. Mitochondria
C. Neocleas

Answer: A
Reference: USMLE step 1

28. Atlatic come for check up all thing normal except Xanthelasma on achllis tendon and cholesterol?
A - Ldl resptor
B - Apo ll
C - Apo c
Answer: A
LDL receptor genetic defects

29. What’s the longest part of the cell cycle?


A-Anaphase
B-Prophase
C-Cytokenesis
The longest phase is INTERPHASE
276
30. IN Exaggerated muscle contraction.
A. Pyruvate to lactic acid
B. NAD
C. NADPH

31. During transformation process of the protein. If translation begin & there is no end traffic code. What is the end destination
of this protein?
A. Lysosome
B. ER membrane
C. Cell membrane
Answer: A

32. Case senario the result was (High Lactic acid ) : defecincy of :
A. Pyruvate kinase
B. Pyruvate dehydrogenase
C. Pyruvate carboxylase
Answer: C
It is an enzyme that transform pyrovae to phosphoenopyruvate. Is deficiency causes lactic
acidosis.https://ghr.nlm.nih.gov/condition/pyruvate-carboxylase-deficiency

33. How does trna bind to codons in the mrna?


A. Covalent bond
B. Ionic bond
C. Hydrogen bond
Answer: C
Reference: http://depts.washington.edu/hhmibio/translationstudyguide.pdf

34. What converts pyruvate to lactate in glycolysis :


A. Increase NPH
B. Decrease AMP
Answer: not known
Explanation: Lactic Acid Fermentation: Reaction occurs in cells without mitochondria (RBC) or in cells when O2 is limited (muscle
cells during exercise). The entire purpose of this reaction is to convert the NADH back to NAD+ so that glycolysis can continue. This is
simply a redox reaction in which pyruvate is reduced to lactate by the enzyme lactate dehydrogenase. Lactate can then enter the
bloodstream and travel to the liver where it is converted back to glucose through the gluconeogenesis pathway.
Reference www.science.marshall.edu/frost/chapter14.pdf

35. Patient with lower limb weakness, sensation, and angular stomatitis?
A. Vitamin B1 Thiamin
B. Vitamin B3 Niacin

Answer: B12
Symptoms indicate Vitamin B12 deficiency
277
36. Dioxynucletides used in which investigation?
A. Western blot
B. DNA suquence
Answer: B

37. How does the body get rid of Lactic acidosis?


A. Excretion by kidney
B. Liver metabolism

Answer: B
In basic terms, lactic acid is the normal endpoint of the anaerobic breakdown of glucose in the tissues. The lactate exits the cells
and is transported to the liver, where it is oxidized back to glucose.
Reference: http://emedicine.medscape.com/article/167027-overview

38. Residual ribosomal RNA in which cell :


A. RBC
B. Reticulocyte
ANSWER: reticulocyte
Reference: https://en.wikipedia.org/wiki/Reticulocyte

39. Protein after transfer sequence what is last destination


A. Endoplasmic retuculum **
B. Cytosol
Answer: A

40. Type of protein form in neocleas:


A. Mrna
B. Trna
Answer: similar to question 4 page 286

41. What hormone increases body cells sensitivity and response to insulin?
A. Leptin
B. Lipase

Answer: A

278
Reference: http://www.ncbi.nlm.nih.gov/pmc/articles/PMC3602983/

42. Which of the following is considered in estimating the response of nucleoside reverse transcriptase inhibitors
A.HIV RNA level in the serum
B.genotype of HIV
Answer : A?

43. Mechanism of action Pyruvate kinase Electron movement:


Phosphenopyrovate to pyrovate and release ATP .

Answer:
The pyruvate kinase is a special case. Its intermediate product, which occurs immediately after transfer of the phosphate group from
phosphoenolpyruvate to ADP, is enolpyruvate. Removal of the phosphate group allows the enol group to rearrange itself into a keto
group. This second step of the reaction is sufficiently exergonic to offset the energetic cost of converting the phosphoester to the
anhydride, and it thus pushes the overall equilibrium of the reaction towards ATP formation.
Reference: http://watcut.uwaterloo.ca/webnotes/Metabolism/glycolysiscatalyticmechanisms.html

44. Rbcs deficient in lactic dehydrogenase. What is the enzyme responsible Pyruvate kinase ?
Reference: http://cmgm.stanford.edu/biochem200/regulation/
Http://emedicine.medscape.com/article/167027-overview

45. From rest muscle status to sudden contractions. What change in cellular level.
Answer:

279
46. ALP elevation, which enzyme confirms that elevation is from liver?
Answer: gamma-glutamyl transpeptidase (GGT) (more commonly) or 5'-nucleotidase (5'-NT),
Ref: http://www.liver.ca/liver-disease/diagnosing-liver-disease/liver-function-tests.aspx

47. Which of the following enzymes is in not functional?


A. HMG Coa..
ANSWER: ?
Plasma non functional enzymes:
Hepatic enzyme · ALT · AST · ALP · y-GT 2.
Cardiac enzymes: · Creatine kinase · CK-MB · LDH · AST 3.

280
Muscle enzyme: CK, LDH, AST, aldolase 4.
Pancreatic enzyme: amylase, lipase 5.
Bone Enzyme: ALP, ACP
Reference: http://nazimrmc.blogspot.com/2012/09/plasma-non-functional-enzyme.html

48. Questions about acid base balance


Low ph, low bicarbonate, high pco2: metabolic and respiratory acidosis✅
Low ph, low bicarbonate, others normal Metabolic acidosis ✅

49. Which is acidophilic:


A. Lace…
Answer:

50. Mechanism of action Pyruvate kinase Electron movement:


Phosphenolpyrovate to pyrovate and releases ATP .
Kaplan biochemistry

51. Rbcs deficient in lactic acid what is the enzyme responsible?


A. Pyruvate kinase
This enzyme deficiency will cause depletion of the end product ATP and lactate. So the ells membrane will be damaged and rupture.
Http://emedicine.medscape.com/article/2196589-overview#a3

52. Paracetamol overdose after 24 hrs what happened to livers>> depletes the liver's natural antioxidant glutathione
http://bestpractice.bmj.com/best-practice/monograph/337/treatment/step-by-step.html

53. Calcium metabolism.

281
Genetic

282
1. What is true about Thalassemia?
A. 4 abnormal beta chain genes, normal 2 alpha chain genes .
B. 4 normal beta chain genes, abnormal 2 alpha chain genes
C. 2 abnormal beta chain genes, normal 4 alpha chain genes
D. 2 normal beta chain genes, abnormal 4 alpha chain genes
Answer: Missing info

2. What is the longest cell cycle?


A-Prophase.
B-Cytokinesis.
C-Interphase
D-Metaphase
Answer: C
Interphase is usually longest, followed by prophase and telophase; metaphase/anaphase is usually shortest.
Reference : http://biolo1100.nicerweb.com/Locked/media/lab/mitosis/key1100.html

3. what is genetic beta thalassemia?


A. Inseration
B. Mutation
C. Depleting
D. Fharm shaft
Answer: B
Thalassemia refers to a spectrum of diseases characterized by reduced or absent production of one or more globin chains. Beta
thalassemia is due to impaired production of beta globin chains, which leads to a relative excess of alpha globin chains.
Reference : up to date

4. Progressive familial intrahepatic cholestasis, mode of inheritance?


A. Autosomal recessive
B. X-linked
C. Mitochondrial
D. Multifactorial
Answer: A
Progressive familial intrahepatic cholestasis (PFIC) is a class of chronic cholestasis disorders that begin in infancy and usually progress
to cirrhosis within the first decade of life. The condition was inherited in an autosomal recessive manner and was characterized by
hepatocellular cholestasis.
Reference : http://emedicine.medscape.com/article/932794-overview
Http://www.uptodate.com/contents/inherited-disorders-associated-with-conjugated-
hyperbilirubinemia?Source=outline_link&view=text&anchor=H20#H20

283
5. which gene is affected in IBD
( chromosomal )
A.13
B.14
C.15
D.16
Answer: D
Intracellular innate immune pathways recognizing microbial products in the cytoplasm – The IBD1 gene on chromosome 16
encodes the protein NOD2 (CARD15)
Reference : https://yhdp.vn/uptodate/contents/mobipreview.htm?43/26/44455

6. What is genetic beta thalassmia?


A - Inseration
B - Mutation
C - Depleting
D - Fharm shaft
Answer: B
The deficiencies in hemoglobin biosynthesis that characterize the various forms of thalassemia arise from mutations in or near the
two globin gene clusters.
Uptodate

7. What gene induce the apoptosis of the cells.


A. P53
B. CRP
C. Nitric oxide
D. TNF
Answer: A

8. Mutation gene in Sick cell disease is?


A. Single gene
B. Frame shift
C. Insertion
D. Deletion
Answer: A
These disorders involve mutations in the DNA sequences of single genes. As a result, the protein the gene codes for is either altered
or missing. Patients with sickle cell disease have a mutation in a gene on chromosome 11 that codes for the beta subunit of the
hemoglobin protein.
http://learn.genetics.utah.edu/content/disorders/singlegene/sicklecell/

284
9. Old patient agitated, urine incontinence, confused, impaired short memory, intact long memory, CT temporal and
hippocampal atrophy, which gene is affected
A. 13
B. 21
C. 18
D. X

Answer: B
Early-onset autosomal dominant AD:
The amyloid precursor protein ( APP) gene on chromosome 21
The presenilin-1 ( PS1) gene on chromosome 14
The presenilin-2 ( PS2) gene on chromosome 1
Late-onset: apolipoprotein E epsilon 4 (APOE E4)
Reference: http://emedicine.medscape.com/article/1134817-overview

10. Patient with dementia - most probably Alzheimer- which chromosome is defected?
A. 13
B. 15
C. 21
D. X
Answer: C
Explanation: The most common form of the disease –which strikes after age of 65- is linked to the apolipoprotein E (apoE) gene on
chromosome 19. However, mutations in genes found on chromosomes 1, 14, and 21 are also linked to rarer forms of the disease,
which strike earlier in life.

11. Which gene is affected in a old patient agitated, urine incontinence, confused, impaired short memory, intact long
memory, CT temporal and hippocampal atrophy?
A. 13
B. 21
C. 18
D. X

Answer: B
This case is Alzheimer’s disease. This disease has genetic factors 3 major genes for autosomal dominant AD have been
identified: amyloid precursor protein (chromosome 21), presenilin 1 (chromosome 14), presenilin 2 (chromosome 1).

Reference: Toronto notes

12. what is the gene for ductal carcinoma of breast?


A. P53
B. Bcl2
C. C-myth
Answer: B

285
Reciprocal expression of Bcl-2 and p53 in breast ductal carcinoma, The bcl-2+/p53-expression pattern was prevalent in histological
grade I and II tumors (77.4% and 59.3% respectively) and rarely present in histological grade III (6.3%).
Reference : http://www.ncbi.nlm.nih.gov/pubmed/8917421
Changes in other genes (OTHER THAN BRACA GENES):
TP53: The TP53 gene makes a protein called p53 that helps stop the growth of abnormal cells. Inherited mutations of
this gene cause Li-Fraumeni syndrome. People with this syndrome have an increased risk of breast cancer, as well
as other cancers such as leukemia, brain tumors, and sarcomas (cancers of bones or connective tissue). This is a
rare cause of breast cancer.
CHEK2: The Li-Fraumeni syndrome can also be caused by inherited mutations in the CHEK2 gene. Even when it doesn’t
cause this syndrome, it can increase breast cancer risk when it’s mutated.
PTEN: The PTEN gene normally helps regulate cell growth. Inherited mutations in this gene cause Cowden syndrome, a
rare disorder in which people are at increased risk for both benign and malignant breast tumors, as well as
growths in the digestive tract, thyroid, uterus, and ovaries. Defects in this gene can also cause a different
syndrome called Bannayan-Riley-Ruvalcaba syndrome that’s not thought to be linked to breast cancer risk. The
syndromes caused by mutations in PTEN can be grouped together as PTEN Tumor Hamartoma Syndrome.
CDH1: Inherited mutations in this gene cause hereditary diffuse gastric cancer, a syndrome in which people develop a
rare type of stomach cancer at an early age. Women with mutations in this gene also have an increased risk of
invasive lobular breast cancer.
STK11: Defects in this gene can lead to Peutz-Jeghers syndrome. People affected with this disorder develop pigmented
spots on their lips and in their mouths, polyps in the urinary and gastrointestinal tracts, and have an increased risk
of many types of cancer, including breast cancer.
PALB2: The PALB2 gene makes a protein that interacts with the protein made by the BRCA2 gene. Defects in this gene
can lead to an increased risk of breast cancer. It isn’t yet clear if PALB2 gene mutations also increase the risk for
ovarian cancer and male breast cancer.
Ref:http://www.cancer.org/cancer/breastcancer/moreinformation/breastcancerearlydetection/breast-cancer-early-
detection-risk-factors-you-cannot-change

13. Gene of (copper transport atpase) on which chromosome


A. 13
B. 9
C. 14
Answer: A
- The ATP7B gene is located on the long (q) arm of chromosome 13 at position 14.3.
- Researchers have identified more than 250 ATP7B gene mutations that cause Wilson disease. About half the mutations change one
of the protein building blocks (amino acids) used to make copper-transporting atpase 2.
Https://ghr.nlm.nih.gov/gene/ATP7B#location

14. Ionised radiation what will happen to DNA ?


A- deamination

286
B- de purantion
C- DNA break strain
Answer: C
Two things would happen to DNA due to radiation.First break down of it. The second formation of O 2 radicals which is more
destructive to DNA than the first.
Https://explorecuriocity.org/Explore/articleid/2212/radiation-effects-on-cells-dna-2212.aspx

15. What come with Turner syndrome ?


A. Hypothyroid
B. DM
C. Addison's
Answer : A
Https://en.wikipedia.org/wiki/Turner_syndrome

16. Mode of inheritance of marfan syndrome?


A. X-linked
B. Autosomal dominant
C. Autosomal recessive
Answer: B
Reference: http://emedicine.medscape.com/article/1258926-overview

Reference: http://learn.genetics.utah.edu/content/disorders/multifactorial/alzheimers/

17. Neurofibromatosia gene 1 located in :


A. Chromosome 17 q 22:11
B. Chromosome 17 p 22:11

Answer: A
Cytogenetic Location: 17q11.2, which is the long (q) arm of chromosome 17 at position 11.2
https://ghr.nlm.nih.gov/gene/NF1#location

Neurofibromatosis type 1 (NF1) is a multisystem genetic disorder that is characterized by cutaneous findings, most notably café-au-
lait spots and axillary freckling, by skeletal dysplasias, and by the growth of both benign and malignant nervous system tumors, most
notably benign neurofibromas.
http://emedicine.medscape.com/article/1177266-overview

18. which one enhance apoptosis?


A. P53
B. PCL-2
Answer: A
Reference: http://www.nature.com/onc/journal/v22/n56/full/1207116a.html

287
19. Gentics of hereditory hypercholestriema)
A. Apo c 11
B. Apo b100
Answer: ???

20. Which gene can induce apoptosis:


A. P53
B. BCL2
Answer: A

21. How does radiation damage DNA?


A. Deamination
B. Double strand DNA breaks*"
Answer: B
Radiation interaction may result in the cells failing to reproduce or cause ‘direct’ interference with a critical cellular system. In the
case of DNA, direct action occurs when alpha particles, beta particles or x-rays create ions that can: Chemically alter bases; Break
sugar phosphate backbones; or Break the hydrogen bonding connecting base pairs
Reference : http://explorecuriocity.org/Explore/articleid/2212/radiation-effects-on-cells-dna-2212.aspx

22. In SCA what is the genetic basic?


A. Point mutation
B. Deletion
Answer: A

23. Pt. with hypopigmented areas on trunk, the same lesion in his uncle, his wife pregnant, what's the possible mode of
transmission to his expected baby:
a. Autosomal trait.
b. X-linked trait.
Answer: A (no dominant trait )
Genetic conditions with an X-linked recessive inheritance pattern usually affect only the boys in a family. This is because they are
caused by altered or missing genes found on the X-chromosome. Girls are born with two X-chromosomes, so the effects of a working
gene on one can mask the effects of an altered or missing gene on the other. They will normally be unaffected carriers of the
condition. Boys have only one X-chromosome, so if they inherit one with an altered or missing gene, they will be affected.
- If one of two parents is affected by a genetic condition with a dominant inheritance pattern, every child has a one-in-two chance of
being affected. So on average half their children will be affected and half their children will not be affected and so will not pass on
the condition. However, as chance determines inheritance, it is also possible that all or none of their children will be affected.
Examples of genetic conditions that show a dominant pattern of inheritance are Huntington's disease, achondroplasia and
neurofibromatosis.
- If two parents are both carriers of a genetic condition with a recessive inheritance pattern, there is a one-in-four chance that each
child will be affected. So on average, one-quarter of their children will be affected. There is also a one-in-two chance that each child
will be an unaffected carrier, like the parents. Examples of genetic conditions that show a recessive pattern of inheritance are cystic
fibrosis, sickle-cell disease, Tay-Sachs disease and phenylketoneuria.
http://www.sciencemuseum.org.uk/whoami/findoutmore/yourgenes/whatcausesgeneticconditions

288
24. Case of neurofibromatosis, has axillary freckles (no dx), mode of inheritance.
A. Autosomal dominant
Answer : a
Neurofibromatosis, or NF, is the term given to two distinct neurocutaneous genetic conditions. Neurofibromatosis type 1, also
known as von Recklinghausen's NF or NF1 is the most common type. NF1 is an autosomal dominant genetic disorder.
Reference : https://yhdp.vn/uptodate/contents/mobipreview.htm?35/50/36650

25. Mode of transmission of polycystic kidney in adults:


Answer: autosomal dominant
ADPKD is the most common hereditary kidney disease, occurring in approximately 1 in every 400 to 1000 people. It often presents
later in life and is sometimes not diagnosed at all.
Reference : up to date

26. comparison between NF type I and type II ?

27. NF1 gene respons for ?


A. Nureofibromatosis type 1 (NF1)
NF1 is due to mutations in the NF1 gene, located at chromosome 17
Ref: uptodate
Reference : https://www.pinterest.com/goharkazanchan/pediatrics/

28. Autosomal recessive disease. Both parents are carrier and phenotype normal what the chance they have a kid with a
disease?
A. 25%
Answer:A

289
29. Adolescent with recurrent swelling of lips gene affected?
A. SERPING1 gene
Answer: A
Mutations in the SERPING1 gene cause hereditary angioedema type I and type II. The SERPING1gene provides instructions for
making the C1 inhibitor protein, which is important for controlling inflammation. C1 inhibitor blocks the activity of certain proteins
that promote inflammation.
Link: http://ghr.nlm.nih.gov/condition/hereditary-angioedema

30. 1st degree cousins came for pre-marriage counseling, they are worried about hereditary disease so they must be screened
for:
Answer: a-thalassemia

31. Long scenario.. What is the gene mutation for it? (Gilbert syndrome)
Answer: UGT1A1 on Chromosome 2q37.

32. Mode of inheritance for Wilson's disease The answer :-


A-autosomal recessive
Answer A
Https://en.wikipedia.org/wiki/Wilson%27s_disease#Genetics

33. couples will get marry, they were relatives, which screening test should be done before marriage?
Answer usually choose the answer which contains a test for an autosomal recessive disease.

290
34. Mendelian inheritance example?
A. Thalassemia
Http://cs.mcgill.ca/~rwest/wikispeedia/wpcd/wp/t/Thalassemia.html

35. Mendle’s Laws:


Answer: MENDEL'S LAWS OF INHERITANCE — Mendel noted that many physical characteristics of seeds and flowers were
transmitted from parental strains to offspring in a predictable and reproducible manner. From these observations, Mendel proposed
that traits manifest through the joint effects of two paired elements (or genes), with each parent contributing one element (allele)
to each offspring at conception

Law of segregation — The law of segregation states that paired parental copies of genes are separated from each other during
gamete formation, with each copy (ie, allele) segregating into separate gametes. This process of gamete formation is now known as
meiosis. (See "Principles of molecular genetics", section on 'Meiosis and sustained genetic diversity'.)
Law of independent assortment — The law of independent assortment states that genes segregate into gametes independent of
other genes such that the allelic status at one locus does not determine segregation of alleles at other loci. This is only true for genes
that are not linked to each other.
Law of dominance — The law of dominance distinguishes dominant, recessive, and co-dominant traits.
●A trait is considered dominant when it is manifest in heterozygous carriers (termed “heterozygotes”).
●A trait is considered recessive when it is observed only among those homozygous for the disease allele or with two different
mutations in the same disease-causing gene (compound heterozygotes). Heterozygotes for a recessive trait (carriers) are typically
phenotypically indistinguishable from non-carrier homozygotes. In X-linked disorders, however, carrier females can sometimes be
affected.
●A co-dominant or semi-dominant trait is one where both alleles contribute equally to phenotypic expression. Heterozygotes
manifest an intermediate phenotype between those of the two homozygous classes.
Ref : uptodate

36. Read about: down syndrome

37. Question about Marfan syndrome clinical features and diagnosis:


You can read about Marfan syndrome here: https://yhdp.vn/uptodate/contents/mobipreview.htm?20/3/20538

38. Chromosome in dm type 2 ?


A. Chromosome 20
Answer:
http://www.medscape.com/viewarticle/406038

39. Order to know what therapy you are going to give? Ki67 or her2
Ki67 = anastrozole
Her2 = trastuzumab or tamoxifen

291
40. Scenario ,,, they gave lab value of lactic acid which was high,,, they asked what's the enzyme defect in the patient
Notes: lactate dehydrogenase,
-congenital lactic acidosis is secondary to inborn errors of metabolism, such as defects in gluconeogenesis, pyruvate dehydrogenase,
the tricarboxylic acid (tca) cycle, or the respiratory chain.

41. Clear case of dessler syndrome ask about ecg change?


St segment elevation in all lead

292
Microbiology

293
1. Man came to the hospital after a wild cat bit him. What is the most likely organism?
A. Pasteurella caballi.
B. Pasteurella multocida.
C. Pasteurella canis.
D. Pasteurella avium.

Answer: b
Reference:
http://www.ncbi.nlm.nih.gov/pmc/articles/pmc3122494/
http://emedicine.medscape.com/article/224920-treatment

2. Patient with ventilator associated pneumonia. Culture showed lactose non-fermenting, gram negative motile… not the same
Q but they asked about an organism!!!
Bacilli producing greenish colony + Oxidase positive. What is the organism?
A. Haemophilus Influenzae
B. Streptococcus pneumoniae
C. Klebsiella or other gram negative bacteria
D. Pseudomonas aeruginosa

Answer: D
And read it!
There are many organisom causing VAP pseudomonas is the most common.
Reference: there is a well-written table in Infectious Disease Chapter, Toronto Notes. Go and read it!

3. Food poising, investigation shows Gram positive cocci?


A. Staph aurus
B. Shigella
C. Salmonella
D. Entero..

Answer: A
Merck Manual: Staphylococcal food poisoning is caused by ingestion of preformed heat-stable staphylococcal enterotoxins.
Staphylococci are gram + aerobic organisms.

4. Male was in swimming pool . 2 days later. Was vomiting, headache, fever, no neck stiffness. What is the possible bacterial?
A. S.Pneumoniea
294
B. H.Influenzea
C. N.Meningitides
D. Naegleria

Answer: D
Http://www.cdc.gov/parasites/naegleria/

5. Which lymphocytes increased in viral infection?


A. Basophils
B. Monocytes
C. Eosinophils
D. Lymphocytes

Answer: D
- basophils ~> An alteration in bone marrow function such as leukemia or Hodgkin's disease may cause an increase in basophils.
Corticosteroid drugs, allergic reactions, and acute infections may cause the body's small basophil numbers to decrease.
- monocytes ~> Diseases that cause a monocytosis include tuberculosis, malaria, Rocky Mountain spotted fever, monocytic
leukemia, chronic ulcerative colitis and regional enteritis.
- Eosinophils are associated with antigen-antibody reactions. The most common reasons for an increase in the eosinophil count are
allergic reactions such as hay fever, asthma, or drug hypersensitivity. Decreases in the eosinophil count may be seen when a patient
is receiving corticosteroid drugs.
- lymphocyte~> increase in many viral infections and with tuberculosis. A common reason for significant lymphocytosis is
lymphocytic leukemia. The majority of both acute and chronic forms of leukemia affect lymphocytes.

Http://www.rnceus.com/cbc/cbcdiff.html

6. Parasite in soil contamination:


A. Tenia saginatm
B. Ascaris-bancrofti
C. Schistosoma.

Answer: B
The main species that infect people are the roundworm (Ascaris lumbricoides), the whipworm (Trichuris trichiura) and hookworms
(Necator americanus and Ancylostoma duodenale). Source: http://www.who.int/mediacentre/factsheets/fs366/en/
And the most common is ascariasis: Source: CDC

7. A child with chronic diarrhea, endoscopy with biopsy showed Sickled shaped parasite. What’s the dx?
A. Entamyba histolytica
B. Giardia
C. Ancylostoma ·

295
Answer: C
Ancylostoma is called hookworms plus none of the other parasites looks sickle shaped
8. Food poisoning with abdominal cramps, nausea, vomiting is most likely caused by:
A. Salmonella
B. Staph
C. Enterococcus faecalis

Answer: B
Both salmonella and staph aureus cause the same symptoms but salmonella usually comes with fever.

9. What is the minimum dtap side effect ??


A) Redness at the injection site
B) generalize muscle pain
C) fever 38.3

Answer: A or C?? P.S. No low grade fever


Explanation: Common Side Effects of dtap Vaccines, Soreness or swelling where the shot was given, Fever, Irritability, Feeling tired,
Loss of appetite, Vomiting
Reference: CDC. Based on WHO half will get fever and half will get injection site reaction

10. HIV patient, presented with SOB and productive cough bronchoalviolar lavage showing ((something)) . Similar Q but no
Pnemocystis in answers !!
A) Pneumocystis jiroveci
B) Aspergillusfumigatus
C) Cryptococcus neoformans

Answer: C
The is bubble soape appearance so the answer is c

11. A patient came with eosinophilia and generalized body aches. What is the causative organism?
A. Entamoeba
B. Giardia
C. Oxyuris

Answer: helminth

12. Dog bite infection caused by?


a. Viral
b. Bacterial
c. Polymicrobial

296
Answer: c
Most common wound infection associated with dog bite is bacterial in origin ( Pasteurella multocida)
Http://www.medscape.com/viewarticle/739023_4

13. Dental caries caused by which organism?


A. Candida
B. HSV
C. Streptococcus mutans

Answer: C
The group of "mutans streptococci" was described as the most important bacteria related to the formation of dental caries.
Reference: Pubmed.

14. Long Scenario about old male bedridden on folly’s catheter he develop Gram -ve bacteria, what is the organism?

A. E.coli
B. Pseudomonas aergonsa
C. Strep. Puomonia

Answer : A ( step up to medicine UTI )

15. Which of the following vaccines is the least likely to harm an immunocompromised patient?
A. Measles vaccine
B. Mumps vaccine
C. Pneumococcal vaccine

Answer: C. Pneumococcal vaccine


Https://primaryimmune.org/wp-content/uploads/2011/04/Immunization-Of-The-Immunocompromised-Host.pdf

16. Patient after 24 h frome eating from resturant and found gram positive bacilli in food ,what is the organism ?
A- salmonella
B- shigella
C- B.cerus

Answer: C
Explanation: Bacillus cereus is a gram positive bacilli. Salmonella and shigella are gram negative.
Reference: First Aid 2015, page 119

17. Symptoms of UTI for 3 days + Foul smell urine for 2 days , what is the organism ?
A- E.coli
B- Kllebsiella pneumonia
297
C- Proteus

Answer: C?

Explanation: could not find the information in Medscape, merck, best practice, Toronto, and step up. However, proteus will turn
urine to alkaline ph, which is due to its ability to split urea into NH3 (ammonia) and CO2. And ammonia is known for its foul odor.

18. Case of giardiasis Dx:


A. 3 consecutive stool
B. -3 separated stool
C. Immunoassay

Answer:
Because Giardia cysts can be excreted intermittently, multiple stool collections (i.e., three stool specimens collected on separate
days) increase test sensitivity 1. The use of concentration methods and trichrome staining might not be sufficient to identify Giardia
because variability in the concentration of organisms in the stool can make this infection difficult to diagnose. For this reason, fecal
immunoassays that are more sensitive and specific should be used 2.

Refrence: http://www.cdc.gov/parasites/giardia/diagnosis.html

19. Gram positive cocci in clusters :


A-Indole
B-urea
C-coagulase

Answer:c
Reference; http://old.infectionnet.org/therapy-recommendations/vascular/empiric-therapy-of-bacteremia/gram-positive-cocci-in-
clusters/

20. Patient having ear discharge bacteria is non fermentar, oxidase + gram negative bacilli
A. Pseudomonas
B. Moraxella

Answer: A

298
21. Patient who is sexually active came with non specific urethritis, what is the most likely cause?
A. Chlamydia
B. Treponema pallidum

Answer: A
Chlamydia is the most common cause of non specific urethritis.

22. How to differentiate between gram positive Cocci ?


A. Catalase
B. Coagulase

Answer: A
To differentiate between gram + cocci we use catalase test. To differentiate catalase + cocci (staph) we use coagulase test

23. UTI patient, results showed, lactose + non fermenter gram - bacilli:
A. Klebsiella
B. Proteus

Answer: A
The Q is wrong it can’t be lactose+ non fermenter is should be fermenter. If that is the case, answer A is correct

24. Patient diagnosed with cutaneous leishmanial what is the organism?


A) Leishmania aka aza
B): Leishmania tropica

Answer: B

299
Reference: CDC and Wiki
Explanation: Leishmaniasis is caused by infection with Leishmania parasites. There are several different forms of leishmaniasis in
people. The most common forms are cutaneous leishmaniasis.

25. Child diagnose to have Giardiasis (Giardia Lamblia). What is the best investigation that confirms the diagnosis?
A. 3 stool analysis in consecutive days
B. 3 stool analysis in separated days
Answer :A

26. Organism undercooked beef ?


A.Entamoeba
B.Tinea

Answer : b

27. Organism undercooked beef?


a. Entamoeba
b. Tinea

Answer: B (Teina saginara - beef Tenia solium- pork)


Taeniasis in humans is a parasitic infection caused by the tapeworm species Taenia saginata (beef tapeworm), Taenia solium (pork
tapeworm), and Taenia asiatica (Asian tapeworm). Taenia solium tapeworm infections can lead to cysticercosis, which is a disease
that can cause seizures.
Http://www.cdc.gov/parasites/taeniasis/

28. Patient with meningitis and facial nerve palsy, what is the organism?
A. Haemophilus influenzae
B. Streptococcus pneumoniae

Answer: Borrelia burgdorferi


Explanation:
Microbiology of meningitis
Isolation suggests presence of some underlying disorders like
H. Influenzae paranasal sinusitis, otitis media, alcoholism, functional or
anatomical asplenia, or hypogammglobulinemia
S. Pneumonae Most common causative organism.
Agent for lyme disease, which is most common vector borne
disease in USA
B. Burgdorferi Triad of lyme disease: Meningitis, cranial neuritis,
radiculoneuritis.
Typical association with bilateral facial palsy

300
Reference http://emedicine.medscape.com/article/232915-overview#a4

29. Visceral leshmenia is caused by:


A- L.donovani
B- L.tropica

Answer: A
Explanation: Visceral leishmaniasis (kala-azar, Dumdum fever) is typically caused by L donovani in East Africa and the Indian sub-
continent, or by L infantum in Latin America, Europe, North Africa, and parts of Asia. Causing spiking fevers, hepatosplenomegaly,
and pancytopenia
Reference: http://bestpractice.bmj.com/best-practice/monograph/527/basics/aetiology.html
First Aid 2015, page 152

30. Girl prick her nail when she cut thr rose .the lesion become ulcerated then transmitted lesion on same lymphatic drainage ,
what the organism?
A- aspargillus
B- sporotrichosis

Answer: B
Explanation: Sporotrichosis is a subacute or chronic infection caused by the saprophytic fungus Sporothrix schenckii. The
characteristic infection involves suppurating subcutaneous nodules that progress proximally along lymphatic channels. Thorns of
plants usually provide the penetrating trauma that introduces the infection into the human host.
Reference: http://emedicine.medscape.com/article/228723-overview

31. Long case +ve giemsa stain TTT:


A. Minocycline
B. Benznidazole ( not sure about this drug)

Answer: Giemsa stain, is used in cytogenetics and for the histopathological diagnosis of malaria and other parasites. Depending on
this we need to know the anti-parasitic agents. The answer would be ALBENDAZOLE. However this is a missing question it
could be malaria!

Refrence: http://cursoenarm.net/UPTODATE/contents/mobipreview.htm?9/26/9632

32. Tuberculosis, that is the best culture media?


A. Blood agar
B. L.wenstein-Jensen medium
Answer: B
The Löwenstein–Jensen medium, more commonly known as LJ medium, is a growth medium for TB.
Reference: Wikipedia

301
33. After removal of phlegmon thy found Enteroccucus fecalis ? What antibiotic ?
A- ceftriaxone
B- ciprofloxacin

Answer: question and choices incomplete!


Explanation: phlegmon: spreading diffuse inflammatory process with formation of suppurative/purulent exudate or pus (Wikipedia).
Enterococci are gram-positive cocci, treatment of choice is ampicillin. Vancomycin can also be used in combination with gentamycin.
If choices do not include ampicillin or vancomycin, then ceftriaxone might be possible since it covers gram negative organism found
in the intestines.
Reference: http://emedicine.medscape.com/article/216993-treatment

34. Gram –ve bacilli, lactose fermenting & non lactose fermenting in urine culture?
Answer E. Coli?

35. UTI; lactose fermenter:


A. E. Coli
Answer: A

36. UTI urease +ve:


A. Proteus
Answer: A

37. Indole positive?


A. E coli

Answer: A
Indole test is test done to bacteria to determine the ability of the organism to convert tryptophan into the indole.

302
38. Treponema pallidum microscopy, to confirm:
A. Do dark field microscopy

Answer: A

39. TB culture media?


ANSWER:
Löwenstein–Jensen medium, more commonly known as LJ medium, is a growth medium specially used for culture of
Mycobacterium, notably Mycobacterium tuberculosis.

40. Isolated microorganism from urine, diplococci, kidney shape


A. Nisserrhea Gonnorrhea

Answer : A
Http://microbeonline.com/characteristics-shape-of-pathogenic-bacteria/

41. RNA hepatitis?


A. Hep C"
Answer: ?
All hepatitis viruses are RNA except HBV so I am not sure if the Q is correct

42. Clostridium perfringens?


A- crepitus under skin
Answer: A
Explanation: crepitus is formed because of gas escaping the necrotic tissue.

43. Infection that is more severe in immunocompromised rather than immunocompetent: (Micro)
A. Crypto.... Can't recall the rest
Answer :
It is now well known that people who are immunosuppressed secondary to HIV infection are at higher risk for Cryptosporidium
infection and that carriage of the parasite is associated with diarrheal disease in most cases. Furthermore, in those with diarrhea,
the disease is much more severe and prolonged than in otherwise healthy individuals
Ref: http://www.ncbi.nlm.nih.gov/pmc/articles/PMC118064/

44. Organism from animal bit?? No rabies in answers


A) Streptococcus mutalis<< not sure
Answer:

303
45. Boy swimming in river , after one day , onset of fever and weakness , what the most common organism cause infection ?
Answer: (Katayama's fever) Acute schistosomiasis
[Most people have no symptoms when they are first infected. However, within days after becoming infected, they may develop a
rash or itchy skin. Within 1-2 months of infection, symptoms may develop including fever, chills, cough, and muscle aches.]
Http://www.cdc.gov/parasites/schistosomiasis/disease.html

46. Monospot test +ve what is the diagnosis


Answer : EBV
Mononucleosis Spot Test
Https://labtestsonline.org/understanding/analytes/mono

47. Organisms that can be found in raw meat:


➢ Beaf: E. Coli O157:H7, Salmonella, Shigella, Staphylococcus aureus and Listeria monocytogenes
➢ Poultry: Salmonella and Campylobacter
Shellfish: Vibrio gastroenteritis, Salmonellas, Plesiomonas shigelloides, Staphylococcus and Bacillus cereus
Https://en.wikipedia.org/wiki/Taenia_saginata
Https://en.wikipedia.org/wiki/Raw_meat

48. Bacteria sexual like behavior ?


A-Conjunction
Answer : A
Ref: http://www.ncbi.nlm.nih.gov/books/NBK21942/

49. Female patient diagnosed with IBD on ceftriaxone with no benefit wt is the organism
Answer: Chlamydia

50. Typical presentation of coxsackievirus B virus?

Answer: More than 90% of coxsackieviruses infections are asymptomatic or cause nonspecific febrile illnesses. In neonates, they are
the most common cause of febrile illnesses during the summer and fall months. As a specific febrile illness coxsakievirus B is more
likely to cause asepctic meningitis than coxakievirus A. Some syndromes are almost caused exclusively by group A viruses
(herpangina, hand-foot-mouth disease), some others by group B (epidemic pleurodynia, myocarditis of the newborn).
Reference: http://virology-online.com/viruses/Enteroviruses5.htm
Http://emedicine.medscape.com/article/215241-clinical

304
51. Patient get beta lactemase organism , on amoxicillin , what to add :
Answer: clavulanic acid

Explanation: beta lactamase inhibitors Include Clavulanic Acid, Sulbactam, Tazobactam. Often added to penicillin antibiotics to
protect the antibiotic from destruction by β-lactamase (penicillinase)
Reference: First Aid 2015, page 181

52. Farmer had sand fly related infection , how to treat ?


Answer: myofestin

Explanation: Leishmaniasis is caused by an intracellular protozoa transmitted by the bite of a female sandfly. The treatment
mainstays of leishmaniasis are sodium stibogluconate and meglumine antimonite. Amphotericin B is also effective against resistant
organisms. Miltefosine is the sole oral agent that has been shown to be effective against leishmaniasis.
Reference: http://emedicine.medscape.com/article/220298-treatment

53. Type of HBV


A.DNA
Answer:A
The hepatitis B virus, a hepadnavirus, is a double stranded DNA virus, composed of a core (hbcag), surrounded by an outer
lipoprotein coat (also called envelope) containing the surface antigen (hbsag).
Reference: http://www.who.int/csr/disease/hepatitis/whocdscsrlyo20022/en/index2.html

54. Long case: ringworm, what's the test:


Answer: KOH
Answer: (Merck Manual) tinea is diagnosed by clinical appearance and by KOH wet mount of plucked hair and scale obtained by
scraping

55. bacterial sexual-like behavior?


A. Conjunction

Answer: a
Types of bacterial reproduction:
o Bacterial transformation: the dna molecule of the donor cell, when liberated by its disintegration, is taken up by
another recipient cell and its offspring inherit some characters of the donor cell.
o Bacterial transduction: the dna molecule that carries the hereditary characters of the donor bacterium is
transferred to the recipient cell through a group of phage particle.

305
o Bacterial conjugation: in which two bacteria lie side by side for as much as half an hour, during this period of time
a portion of genetic material is slowly passed from one bacterium, which is designated as a male to a recipient
designated as a female.
Reference: http://www.yourarticlelibrary.com/bacteria/3-types-of-sexual-reproduction-that-occurs-in-bacteria-
1869-words/6817/

56. Gram +ve cocci catalase +ve coagulase +ve grape like cluster
Answer: Staph aureus

57. Gram +ve Bacilli non aerobic:


Answer: Clostridium perfringens (spore forming)

58. Read about Epstein Barr virus??

59. Read about mononucleosis ??

Immunology
306
1. A patient with signs and symptoms of atopy. Which cells produce these mediators?
A. Mast cells
B. Nk cells
C. Macrophage
D. B cells

Answer: a

2. Patient with sx of persistent nasal obstruction and discharge since he transferred to this new house where the molds are on
the wall , in the hospital skin test was done and it was positive in less than 30 min whats this type of allergy:
C. Immediate
D. Delayed
E. Serum sickness reaction
F. Anaphylaxis

Answer: A
Type I reactions (ie, immediate hypersensitivity reactions) involve immunoglobulin E (ige)–mediated release of histamine and other
[2]
mediators from mast cells and basophils. Examples include anaphylaxis and allergic rhinoconjunctivitis.(Fast)

307
Reference: http://emedicine.medscape.com/article/136217-workup#c7

3. Cold agglutination test use which antibodies?


A-IgA
B-Igm
C-IgG
D-IgE
Answer: B
Reference: Master The Board.

4. You perform a skin hypersensitivity test for molds which came strongly positive within 30 minutes, what is the type of his
hypersensitivity reaction?
A. Immediate hypersensitivity reaction
B. Delayed hypersensitivity reaction
C. Contact dermatitis
D. Immune-complex reactions (serum sickness)
Answer: A. Immediate
Http://missinglink.ucsf.edu/lm/immunology_module/prologue/objectives/obj10.html

5. Viral knockout which of the following


A. IL2
B. IL3
C. IL4
D. IL5
Answer : C

6. What blood cell is responsible for blockage?


A. Basophil
B. Neutrophil
C. Eosinophil
D. Macrophage
Answer: ?

7. Which of the following vaccines is less harmful to immunocompromised patient?


A- Hepatitis recombinant B vaccine
B- bcg
C- salk polio
D- sabin polio
Answer: A

308
8. Man done vasectomy, change his mind want to reproduce again, they found antisperm antibodies what is the cause?
A. Antigen release or something like that?
B. Cross reactivity with viral infection
C. Inappropriate response of MHC II to antigen presenting
Answer: A
A man can make sperm antibodies when his sperm come into contact with his immune system. This can happen when the testicles
are injured or after surgeries (such as a biopsy or vasectomy). The testicles normally keep the sperm away from the rest of the body
and the immune system. Development of anti-sperm antibodies after vasectomy is thought to be related to breakdown of the
blood–testis barrier and leakage of sperm antigens from the epididymis. Sperm antigens have been found in the serum of men as
early as 2 weeks after vasectomy. Infertility after anti-sperm antibody binding can be caused by autoagglutination, sperm
cytotoxicity, blockage of sperm-ovum interaction, and inadequate motility. Each presents itself depending on the binding site of ASA.
Reference: http://www.glowm.com/section_view/heading/Long-Term%20Risks%20of%20Vasectomy/item/408#35035

9. Girl after eating seafood developed rash with severe itching and diarrhea, the mechanism of reaction
A. Complement
B. Cell mediated
C. Immediate

Answer: C

10. At which temperature does the blood bank preserve the blood?
A-37 C

309
B-4 C
C-22 C

Answer: B Reference: Medscape

11. What most commonly cause itching? (Medicine)


A-bile salt retention
B-eczema
C-pregnancy
Answer :B

12. Long case of eye trauma then repair then developed most likely endophthalmitis done enucleation, found in the uvea
lymphocyte and CD4 i think, what's is the cause?
A-cross reactivity
B-activation of lymphocytes
C-antigen release or something?
Answer :C?

13. HIV patient test to confirm?


A- PCR
B- western blot
C- ELISA
answer: B
14. A woman has +ve Hep B surface antigen delivered a baby in a hospital and received immunogloben and B vaccine within the
first 12 hour ,, what are the recommendation for breast feeding ?? (Pediatric)
A) no breast feeding
B) breast feeding is okay
C) breast feeding after 12 hours
Answer: c
Reference: Kaplan Pediatrics
Explanation: mothers with HBV infection are free to breast feed their
Infants AFTER the neonate has received the appropriate recommended vaccination.

15. Girl after eating sea food developed rash with sever itching and diarrhea , the mechanism of reaction:
A. Complement
B. Cell mediated
C. Antibody mediated
Most likely its Missed >> type I ige mediated
Answer: TYPES OF REACTIONS — Food allergy reactions may be ige-mediated, non ige-mediated, or both.
Reference: http://cursoenarm.net/UPTODATE/contents/mobipreview.htm?14/55/15217#H3

16. (long scenario) man traveled for business, went to his flat to relax, second day he noticed mold on his flat, he developed rash
all over his body. We took sample of mold and injected him, inflammation occurs after 30 minutes. What does it indicate?
A. Immediate immunity.

310
B. Late immunity.

Answer: A

17. Someone with cancer, took cells from him inject with cancer, then re inject to patient so they can be antigen for cancer. What
type of immunity?
A. Passive
B. Active immunotherapy
Answer: B
Active immunotherapy attempts to stimulate the immune system by presenting antigens in a way that triggers an immune response.
Reference: http://www.asbestos.com/treatment/immunotherapy/

18. Allergic reaction (Serum sickness) to animal protein derivative injection what will be found in blood of patients?
A. Ige
B. Igg
Answer: B
Serum sickness is one of type 3 hypersensitivity with igg predominance, although some igm may be found.

19. The best test for hypersensitivity type 1?


A. Subdermal skin injection ( my answer)
Answer: ?
Reference: Medscape
Explanation: skin prick test, intradermal test, or ige radioallergosorbent (RAST) tests can be used to identify Type 1 hypersensitivity.

20. Test for Sickle cell anemia?

311
A- Hb electrophoresis
B- bone marrow aspiration
Answer:A

21. Female patient ate from seafood restaurant , in the second day she developed diarrhea and nausea and urticaria , what does
she have ?
A. Food poisoning
B. Food allergy
Answer: B? ‫المحلولة بالمذكرة مكرر‬
Http://www.allergy.org.au/patients/food-allergy/allergic-and-toxic-reactions-to-seafood

22. Male just moved to new apartment in humidified area and there was dust , he was injected with mites and 30 mints later , he
developed allergy , what type of hypersensitive he has ?
A. Immediate
B. Delayed
Answer: ‫مكرر‬

23. (Long scenario) man traveled for business, went to his flat to relax, second day he noticed mold on his flat, he developed rash
all over his body. We took sample of mold and injected him, inflammation occurs after 30 minutes. What does it indicate?
A. Immediate hypersensitivity.
B. Delayed hypersensitivity
Answer: A
Type I reactions (immediate hypersensitivity) are ige-mediated. Type I reactions underlie atopic disorders (eg, allergic asthma,
rhinitis, conjunctivitis), anaphylaxis, some cases of angioedema, urticaria, and latex and some food allergies. Type I reactions develop
< 1 h after exposure to antigen.

24. Defective regulation of immunosuppression


A. Activated CD+4 cells activate other inflammatory cells like macrophages & B-cells or recruit more inflammatory cells by
stimulation of homing receptor on leucocytes & vascular epithelium.
Answer:?

25. What is the Immunological reaction to peanut allergy?


o Hypersensitivity reactions to foods are usually due to immunoglobulin (Ig) E-mediated reactions to food and/or
additives. Stepuptpmedicine

26. Rejection of transplanting liver mechanism


Answer:
There are three types of graft rejection that may occur. They include hyperacute rejection, acute rejection and chronic rejection.
Hyperacute rejection is caused by preformed anti-donor antibodies. It is characterized by the binding of these antibodies to antigens

312
on vascular endothelial cells. Complement activation is involved and the effect is usually profound. Hyperacute rejection happens
within minutes to hours after the transplant procedure. Unlike hyperacute rejection, which is B cell mediated, acute rejection is
mediated by T cells. It involves direct cytotoxicity and cytokine mediated pathways. Acute rejection is the most common and the
primary target of immunosuppressive agents. Acute rejection is usually seen within days or weeks of the transplant. Chronic
rejection is the presence of any sign and symptom of rejection after 1 year. The cause of chronic rejection is still unknown but an
acute rejection is a strong predictor of chronic rejections.
Reference: https://en.wikipedia.org/wiki/Liver_transplantation#Graft_rejection

27. BCG is a contraindication in patient with deficiency of which enzyme?

BCG Vaccine for prevention of tuberculosis should not be given to persons (a) whose immunologic responses are impaired because
of HIV infections, congenital immunodeficiency such as chronic granulomatous disease or interferon gamma receptor deficiency,
leukemia, lymphoma, or generalized malignancy or (b) whose immunologic responses have been suppressed by steroids, alkylating
agents, antimetabolites, or radiation.3 BCG Vaccine should not be administered to HIV-infected or immunocompromised infants,
children, or adults.
Prior to administration, the possibility of allergic reactions should be assessed. Allergy to any component of BCG Vaccine or an
anaphylactic or allergic reaction to a previous dose of BCG Vaccine are contraindications for vaccination.
BCG Vaccine is not a vaccine for the treatment of active tuberculosis.
BCG Vaccine should not be used in infants, children, or adults with severe immune deficiency syndromes. Children with a family
history of immune deficiency disease should not be vaccinated; if they are, an infectious disease specialist should be consulted and
anti-tuberculous therapy administered if clinically indicated.18

Chronic granulomatous disease (CGD) is a genetically heterogeneous immunodeficiency disorder resulting from the inability of
phagocytes to kill microbes they have ingested. This impairment in killing is caused by any of several defects in the nicotinamide
adenine dinucleotide phosphate (NADPH) oxidase enzyme complex, which generates the microbicidal respiratory burst. In CGD,
phagocytes ingest bacteria normally, but they cannot kill them

313
Pathology

1. Patient had HAV (its type A) after 6 months he came to check up what histopathology will see?
A. Peripheral necrosis
B. Central necrosis
C. Normal
D. Interseptal necrosis

314
Answer: C ?
Uptodate: Injury to the liver is secondary to the host’s immune response. Hepatocellular damage and destruction of infected
hepatocytes is mediated by HLA restricted, HAV specific CD8+ T-lymphocytes and NK cells.
Http://jcp.bmj.com/content/36/6/650.full.pdf

2. Most parts of liver become hypoxic?


A. Central venules
B. Peripheral
C. Sinusoids
D. Zone ??
Answer: A or D if it was zone 3
Explanation: Zone III is pericentral vein (centrilobular), and it is most susceptible to hypoxia and ischemia
Reference First aid USMLE step 1

3. A patient is coughing bloody frothy sputum. He has pulmonary edema, + hepatojugular reflux and lower limb edema.
Capillary pressure is 3 times more than oncotic pressure. What is the type of edema?
A. Venous
B. Arterial
C. Interstitial
D. Capillary

Answer: c
Pulmonary edema in heart failure patient caused by increase of pulmonary venous pressure lead to pulmonary venous distention
and transudation of fluid. Also, lead to pulmonary capillaries rupture.
Extra information:

Reference: rapid review pathology, 4th edition

4. How does minimal change glomerulonephritis look under microscope?


Answer: LM: normal glomeruli

5. Fragile x syndrome associated with


Answer: Large protruding ear, long face, high arched palate, hyperextensible finger joint, double joint thumbs, flat feet, soft skin,
post pubescent macroorchidism , hypotonia, language deficit, autism, ADHA, strabismus, seizures, fertility.

315
6. What part of the liver is affected during ( I think hypoxia) Zone 2 and 3 Central part of portal something
Answer: zone 3
Reference: USMLE step 1

7. Patient had hx of heart failure (orthopnea ,pnd and ankle swelling) what is the pathophysiology ? With tables!!!

8. Orthostatic hypotension pathophysiology :


A. Decrease extracellular fluid
B. Decrease intracellular fluid

9. What is the effect of high blood pressure on the kidney?

Hypertensive nephrosclerosis is a disorder that is usually associated with chronic hypertension. In addition to the level of blood
pressure, it is clear that individual factors are involved. As an example, black patients have an approximate eight-fold elevation in the
risk of hypertension-induced end-stage renal disease (ESRD) [1]; this increase in risk may persist even with "adequate" blood
pressure control. Although low birth weight and bias in diagnosis based upon the patient's race may be involved, the recent
recognition of an association between two independent sequence variants in the APOL1 gene on chromosome 22 and renal disease
in African Americans, including focal segmental glomerular sclerosis and hypertension-related ESRD, provides a much more likely
pathophysiologic mechanism [2] and suggests that hypertensive nephrosclerosis in blacks and whites may be distinct diseases. In
addition, the histologic features of hypertensive nephrosclerosis may be observed in patients with normal blood pressure.

Hypertensive nephrosclerosis is characterized histologically by vascular, glomerular, and tubulointerstitial involvement

Vascular disease — The vascular disease consists of intimal thickening and luminal narrowing of the large and small renal arteries
and the glomerular arterioles. Two different processes appear to contribute to the development of the vascular lesions:
●A hypertrophic response to chronic hypertension that is manifested by medial hypertrophy and fibroblastic intimal thickening,
leading to narrowing of the vascular lumen [4,5]. This response is initially adaptive by minimizing the degree to which the rise in
systemic pressure is transmitted to the arterioles and capillaries [5].

●The deposition of hyaline-like material (plasma protein constituents, such as inactive c3b, part of the third component of
complement) into the damaged, more permeable arteriolar wall [5].

Glomerulosclerosis — The glomeruli may show both focal global (involving the entire glomerulus) and focal segmental sclerosis:
●Global sclerosis is thought to reflect ischemic injury, leading to nephron loss. This can be further categorized histologically as either
solidified (in which the entire tuft is involved) or obsolescent (in which the tuft is retracted and Bowman's space is filled with
collagenous-type material). The solidified form is more commonly associated with African Americans than with Caucasians, and
might contribute to the increased prevalence of nephrosclerosis in African Americans [6].

●Focal segmental sclerosis is typically associated with glomerular enlargement, which can be a compensatory response to nephron
loss [7] but may also precede that loss [8]. However, the combination of hypertrophy and a rise in intracapillary pressure in these
glomeruli may gradually lead to hemodynamically mediated segmental sclerosis

316
317
Pharmacology

318
1. A culture was taken from the patient, it was shown to be resistant to beta lactam and sensitive to fluoroquinolones,
aminoglycosides and chloramphenicol. Which of the following antibiotic is contraindicated?
A. Levofloxacin
B. Chloramphenicol
C. Gentamycin
D. Azithromycin
Answer: A or D?
Fluoroquinolones have varying specificity for cytochrome P450, and so may have interactions with drugs cleared by those enzymes;
the order from most P450-inhibitory to least, is enoxacin > ciprofloxacin > norfloxacin > ofloxacin, levofloxacin, trovafloxacin,
gatifloxacin, moxifloxacin
Reference : http://www.medscape.com/viewarticle/418295_4

2. Ciprofloxacin MOA?
A-Inhibits RNA
B-Inhibits DNA
C-Inhibits cell wall synthesis
D-Other
Answer: B
The bactericidal action of ciprofloxacin results from inhibition of the enzymes topoisomerase II (DNA gyrase) and
topoisomerase IV (both Type II topoisomerases), which are required for bacterial DNA replication, transcription,
repair, and recombination.
Reference: http://www.drugs.com/pro/cipro-iv.html#ID_41750258-7c8d-4908-8782-cdac2405c884

3. Which of the following well break acute coronary thrombi, causing MI?
A) ASA
b) Heparin
c) Alteplase
d) Enoxaparin (LMWH)
Answer: C
Fibrinolytic (streptokinase, urokinase, tissue plasminogen activator “altplase “)
Initial therapy of ACS focuses on the following: Stabilizing the patient’s condition, Relieving ischemic pain (Nitrates & Beta blockers),
Providing antithrombotic therapy( Aspirin, Clopidogrel, Prasugrel, Ticagrelor, Glycoprotein iib/iiia receptor antagonists.
Cytokines, such as interleukin-1, tumor necrosis factor-alpha, interferon gamma, platelet-derived growth factors, and matrix
metalloproteinases are released by activated macrophages; they cause the intimal smooth muscle cells overlying the plaque to
become senescent and collagen cross-struts within the plaque to degrade.
319
Reference: Davidson's Principles and Practice of Medicine, 22nd ed ,P580

Reference : http://emedicine.medscape.com/article/1910735-overview

4. Bacterial meningitis in 14 month child I think? Gram positive cocci, what is the management?
A-amoxicillin
B-amoxicillin and gentamicin
C-ceftriaxone and vancomycin
D-vancomycin
Answer: C-ceftriaxone and vancomycin

Age or Predisposing Feature Antibiotics


Age 0-4 weeks Ampicillin plus either cefotaxime or an aminoglycoside
Age 1 mo-50 y Vancomycin plus cefotaxime or ceftriaxone*
Age >50 y Vancomycin plus ampicillin plus ceftriaxone or cefotaxime plus vancomycin*
Impaired cellular immunity Vancomycin plus ampicillin plus either cefepime or meropenem
Recurrent meningitis Vancomycin plus cefotaxime or ceftriaxone
Basilar skull fracture Vancomycin plus cefotaxime or ceftriaxone
Head trauma, neurosurgery, or CSF shunt Vancomycin plus ceftazidime, cefepime, or meropenem
CSF = cerebrospinal fluid.

320
*Add ampicillin if Listeria monocytogenes is a suspected pathogen.

Reference : http://emedicine.medscape.com/article/232915-treatment#d8

5. Ribavirin side effect ?


A... Lactate..
B- liver damage
C- renal damage
D- anemia
Answer: D
Commonly reported side effects of ribavirin include: hemolytic anemia, decreased hemoglobin, insomnia, dyspnea, lack of
concentration, irritability, and emotional lability.
Reference : http://www.drugs.com/sfx/ribavirin-side-effects.html

6. Drug for bladder or prostate oxybutynin


A-block muscarinic cholinergic
B- block b 2
C- activate a adrenergic
D- activat …
Answer: A
Oxybutynin chloride exerts a direct antispasmodic effect on smooth muscle and inhibits the muscarinic action of acetylcholine on
smooth muscle. Reference : http://www.drugs.com/pro/oxybutynin.html

7. What is the best drug given to prevent postoperative thromboembolism?


A) LW heparin
B) Uf heparin
C) Warfarin
D) Enoxaparin
Answer: A
Early prophylaxis in surgical patients with low-molecular-weight-heparin (LMWH) has been associated with significant reductions in
postoperative venous thrombosis
Reference : http://emedicine.medscape.com/article/1268573-overview

8. Which of this drug cause seizure?


A- isoniazid
B- ethambutol
C- ripaficin
D- Pyrazinamide
Answer : A

321
Common side effect of isoniazid nausea, vomiting, abdominal pain; seizures ,yellow skin or eyes; dark urine;
numbness or tingling in your hands or feet; blurred vision; or confusion or abnormal behavior. Allergic reaction
unusual weakness or fatigue
Reference: http://www.rxlist.com/isoniazid-side-effects-drug-center.htm

9. Female patient came with excessive facial and abdominal hair growth.. Which antiepileptic can cause this side effect?
A. Phenytoin
B. Carbamazepine
C. Phenobarbital
D. Valproic acid
E. barbiturates
Answer: A. Phenytoin
Not that significant https://www.drugs.com/sfx/phenytoin-side-effects.html

10. Which of the following is side effect of atropine :


A. vasoconstriction
B. decrease IOP
C. decrease urine output
D. dry of mouth
Answer:D

11. Treatment of chronic pain ;


A-Naloxone
B-Salicylate
C-Ibuprofen
D-Acetaminophen
Answer :D or B (Insufficient information)
Reference : uptodate , American Chronic Pine Association

12. A patient with heartburn taking antacids. She had rheumatic fever 1 week ago and was started on aspirin. What side effect
she can develop?
A-Constipation
B-Diarrhea
C-Dry mouth
D-Galactorrhea
Answer :B. (Source: Goodman Gilman)
Chronic administration of antacids may reduce serum salicylate concentrations in patients receiving large doses of aspirin or other
salicylates. The mechanism involves reduction in salicylate renal tubular reabsorption due to urinary alkalinization by antacids,
resulting in increased renal salicylate clearance
Reference : http://www.drugs.com/drug-interactions/acid-gone-antacid-with-aspirin-121-8977-243-
0.html?Professional=1

13. What is the mechanism of action of Orlistat?


A. Decrease cholesterol
322
B. Increase satiety
C. Decrease pancreatic enzymes
D. Increase BMR

Answer: C
It inhibits pancreatic and gastric lipases

14. Type 2 diabetic patient on glibiclizide and another oral hypoglycemic agent - i forgot what is was- heared about incretins in
the news and he wants to know more about it, what is its mechanism of action
A. Increases sensitivity of the receptors to insulin
B. Delay gastric emptying
C. Decrease gluconeogenesis "
D. Increase insulin secretion
Answer: D

15. Which of the following is side effect of morphine:


A- dry cough
B- nausea, vomiting
C- tachypnea
D- anxiety
Answer: b
The most common adverse events (seen in greater than 10%) reported by patients treated with morphine during therapy were
constipation, nausea, somnolence, vomiting, and headache.
[http://www.rxlist.com/avinza-drug/side-effects-interactions.htm]

16. Drug that reduced heart rate and peripheral resistance in HTN
A. Carvedilol
B. Hydralazine
C. Amlodipine
D. Thiazide
Answer: A

17. Side effect of anticholinergic drugs?


A. Diarrhea
B. Excess Salivation
C. Urine Incontinence
D. Blurred vision
Answer: D
Side effects:
Dryness of the mouth ---> Tachycardia ---> Blurring of vision ---> Difficulty speaking and swallowing ---> Dry hot skin ---> Difficulty in
micturition & Constipation ---> Ataxia, restlessness, hallucination and delirium, Dry mouth, urinary retention, constipation, blurred
vision, toxic-confusional states and many more.
Reference : uptodate Toronto Notes.

323
18. Which of the following medications is safe during breastfeeding?
A- chlormphenicol,
B- cimitidine,
C- erythromycin,
D- tetracycline
Answer: C

19. What drug is safe during pregnancy:


A-arythromycin
B-ciprofluxacine
C-cemetidine
D-One more opt
Answer: A
Ampicillin,Clindamycin,Erythromycin,Penicillin,Gentamicin,Ampicillin-Sulbactam
,Cefoxitin,Cefotetan,Cefazolin
Http://www.drugs.com/pregnancy/erythromycin.html

20. Pregnant with past history of depression on Paroxitine for long time. She is asking the physician if she can use this medication
or not while she is pregnant. What you have to tell her ?
A- It is not safe because the risk of cardiac congenital malformation *
B- It is not safe...
C- It is safe...
D- It is safe...
Answer: A

21. What is the rule of metformin in PCOS?


A) Decrease glucose level
b) Decrease insulin resistance
c) Anti-androgenic
d) menstrual regulation
Answer: b

22. Patient presented to ER with history of drug over dose and coma for the last 8 hours on examination absent gag reflex best
manegment is :
A-iv naloxone
B-gastric lavage
C-immediate endotracheal intubation
D-coracol
Answer : C
23. Attention Deficit Hyperactivity Disorder ( they give me the symptom not the diagnosis ) child what is the management?
A. Ecitalpram
B. Atomoxetine
C. Olanzapine
D. Clonazepam

324
Answer: B
First time: methylphenidate, dextroamphetamine
Atomoxetine (Strattera) has become a second-line

24. Which of the following is side effect of morphine :


A. dry cough
B. nausea , vomiting
C. tachypnea
D. anxiety
Answer: B. Sure (from some on got 100 in anesthesia)
Nausea, vomiting, constipation, lightheadedness, dizziness, drowsiness, increased sweating, or dry mouth may occur. Pain,
redness, or swelling at the injection site may occur if this medication is given into a muscle or under the skin. Reference :
http://www.webmd.com/drugs/2/drug-18436/morphine-intravenous/details#side-
effectshttps://www.drugs.com/sfx/morphine-side-effects.html

25. What is the best drug given to prevent postoperative thromboembolism?


A- LMW heparin
B- Uf heparin
C- Warfarin
D-eno….
Answer: B
Unfractionated heparin (UFH) may be preferred if the patient is likely to have immediate surgery because of its shorter half-life and
reversibility with protamine compared with LMWH. (Medscape

26. Case male with convulsion then loss his consciousness he is on antidepressants drug and drug for congestive heart failure O/E
all normal ECG shows dysarrythmia Which drug can cause this:
A. Quinine
B. Digitalis
C. SSRI
D. TCA
Answer: D

27. Affective half life of fluextine ?

A. 18 hr
B. 2 day
C. 4 day
D. 9 day
Answer: C
Time to clear out of the system

325
The body eliminates fluoxetine very slowly. The half-life of fluoxetine after a single dose is 2 days and after multiple dosing 4 days.
The liver then metabolizes fluoxetine into norfluoxetine, a desmethyl metabolite, which is also a serotonin reuptake inhibitor.
Norfluoxetine has an even longer half-life, i.e. 8.6 and 9.3 days for single and repeated dosage respectively.

Because of the long half-lives of fluoxetine and norfluoxetine, it may take up to 1 to 2 months for the active substance to disappear
from the body. There are no effective methods known to enhance the elimination of fluoxetine

Refrence:- http://www.emedexpert.com/facts/fluoxetine-facts.shtml

Other refrences:- Range and Dale’s Pharmacology

28. ECG with AF .. A patient present with irregular cardiac palpitation some duration in the past for which he was taking a drug,
lab values given demonstrating normal upper limit of aptt, normal PT, normal INR, normal platelet, low hemoglobin, what is
the mechanism of action of the drug ?
A. Anti thrombin III
B. Decrease factor VIII
C. Inhibit platelets aggregation
D. Decrease vit K dependent factors

29. Diabetic patient developed fever, productive cough and SOB. Labs show high WBC. CXR (picture was given which showed
lower lobe infiltrates + air-fluid level).The drug that will be given to the patient acts on which of the following?
A. DNA gyrase
B. 30 S ribosome
C. 50 S ribosome
D. Transpeptidase
Answer: C
Http://emedicine.medscape.com/article/428135-overview#showall
Http://www.uptodate.com/contents/lung-abscess?Source=outline_link&view=

326
Reference: Toronto Notes

30. Action of Neostigmine on muscarinic receptor:


A. Tachycardia
B. Bronchodilator
C. Pupil dilatation
D. Urinary incontinence
Answer: D
Http://missinglink.ucsf.edu/lm/drug_module/cholinergic_agonists/neostigmine.htm

31. Morphine side effect?


A. Tachypnea
B. Dry mouth
C. Nausea &vomiting
D. Anxiety
Answer: c. Nausea &vomiting.
Https://www.drugs.com/sfx/morphine-side-effects.html

32. Smoking withdrawal symptoms last for:


A- 1
B- 1-2
C- 2-4
D- 5-7 Days

33. Patient on diuretic develop palpation, which electrolyte lead to this:


A. Sodium
B. Potassium

327
C. Cl
D. Calcium
Answer; B

34. First symptoms of hypomagnesaemia is


A. Muscle paralysis
B. Hypotension
C. Loss of deep muscle reflex
D. Respiratory depression
Answer: C
35. Case is allergic to sulfa, shell fish, penicillin what you will give him?
A. Nitrofrounation
B. Tm/smz
C. Pencillin
D. Amoxicillin
Answer: A

36. Hypertensive patient, on antihypertensive medications, was given phytosterol. What is the mode of action of phytosterol?
A. Decrease plasma triglycerides
B. Decrease plasma cholesterol
C. Inhibit fatty acid synthesis
D. Decrease de novo synthesis of cholesterol

Answer: b
Phytosterols are plant sterols structurally similar to cholesterol that act in the intestine to lower cholesterol absorption.
Reference: pubmed.

37. Diabetic patient has history of weakness and dizziness. What anti-diabetes can cause that?
A. Sulfonylurea (glipizide)
B. Metformin
C. Thiazolidinediones (rosiglitazone)
D. Insulin

Answer: d
(symptoms are consistent with hypoglycemia)
➢ Sulfonylurea side effects: • hypoglycemia • weight gain.
➢ Metformin can cause unusual tiredness or weakness. Glucagon-like peptide (glp)-1 analogue: exenatide or liraglutide causes
muscle weakness.
● Pioglitazone, rosiglitazone and troglitazone can cause myalgia and muscle weakness.
References: toronto notes + mayoclinic + pubmed

38. A patient who was treated from tb, came to you complaining of eye pain. What is the cause?

328
A. Isoniazid
B. Rifampicin
C. Ethambutol
D. Pyrazinamide

Answer: c
Reference: http://www.drugs.com/sfx/ethambutol-side-effects.html

39. Which of the following drugs cause seizure?


A. Isoniazide
B. Ethambutaol
C. Ripaficin
D. Pyrazinamide

Answer: a
http://reference.medscape.com/drug/isoniazid-342564#4

40. What is the treatment of shigella?


A-Cefuxime.
B-Ampicillin.
C-Metronidazole
Answer: B
The following antibiotics are used to treat Shigella dysentery:Beta-lactams (Ampicillin, amoxicillin, third-generation cephalosporins),
Quinolones Macrolides( Azithromycin), Others: sulfonamides, tetracycline. Reference :
http://emedicine.medscape.com/article/968773-medication. All these antibiotics can be used: ampicillin, ciproflocxacin
,ceftriaxon, TMP-SMZ , azithromycin.

41. Patient diagnosed with MI and aspirin was giving what is mode of action
A. Anti platelet aggregation
B. Antithrombin III
C. Forget other choices
Answer: A
In the absence of an absolute contraindication, antiplatelet therapy with aspirin and platelet P2Y12 receptor blocker is indicated
in all patients with a non-ST elevation ACS
Reference : uptodate

42. DM pt. With necrotizing fasciitis .. They asked about antibiotic combination
A. penicillin/gentamycin
B. amoxil/erythromycin..??
C. piperacillin/tazobactam
Answer: A
Possible regimens for necrotizing fasciitis include a combination of penicillin G and an aminoglycoside (if renal function permits), as
well as clindamycin (to cover streptococci, staphylococci, gram-negative bacilli, and anaerobes).

329
Reference : http://emedicine.medscape.com/article/2051157-treatment

43. Side effect of nitroglycerin ..


A- bradycardia
B- headache
C- sexual dysfunction
Answer: B
The most common general side effect is headache.
Reference : http://www.drugs.com/sfx/nitroglycerin-side-effects.html

44. You need to know All The Antidepressant side effects


A) SSRI
B) MOI
C) TCA
Answer:
MOI: Dizziness, headache, drowsiness, sleep disturbances, fatigue, weakness, tremors, twitching, myoclonic movements,
Constipation, dry mouth, gastrointestinal disturbances, Weight gain, Postural hypotension, edema, Sexual disturbances
SSRI : Sexual dysfunction , Drowsiness , Weight gain ,Insomnia ,Anxiety, Dizziness ,Headache ,Dry mouth ,Blurred vision
,Nausea ,Rash or itching Tremor ,Constipation ,Stomach upset
1. TCA: arrhythmias; hypotension; tachycardia; seizures; ataxia; tremors; peripheral neuropathy; dyskinesia; disturbed
concentration; anxiety; insomnia; urinary retention, constipation; blurred vision; dry mouth, Skin rash; leukopenia,
thrombocytopenia, nausea; diarrhea; impotence, Alopecia; edema;
Reference : up-to-date , http://www.drugs.com/pro/amitriptyline.html

45. Which one of these drugs will cause hyperpigmentation?


A- ccb
B- amiodarone
C- digoxin
Answer: B
Pigmentation may be induced by a wide variety of drugs; nsaids, phenytoin, antimalarials, amiodarone, antipsychotic drugs,
dexamethasone.
Reference : http://www.dermnetnz.org/reactions/drug-pigmentation.html
(the answers were either anti-HTN or anticonvulsant, can't remember)

46. Angioedema is a side effect of which drug?


A. BB
B. CCB
C. ACEI
Answer: C
330
Http://www.medicinenet.com/ace_inhibitors/page2.htm

47. What’s given with NSAIDs to decrease its side effects?


A-Cimetidine
B-Pseudoephedrine
C-Don’t remember the rest but they ended in –ine
Answer:
The side effect (I believe) that we are concerned about here is Peptic Ulcer Disease. If ppis or Misopristol is the answer then choose
either (ppis are generally safer but no consensus). Don’t choose H2s as such an option is not supported by medical evidence. Bear in
mind that you might have to avoid ppis, for example when the patient is on clopidogrel (Coronary Artery patients..etc), You’d also
avoid Misopristol in patients who are pregnant or expected to be pregnant as it is an abortifacient medication.
Reference : uptodate

48. Which of the following medications used in treating osteoporosis and can cause epigastric & heartburn sensation?
A. Denosumab.
B. Risedronate.
C. Raloxifene.
Answer: B
Denosumab is a (Monoclonal antibody), Risedronate is a (Bisphosphonate), Raloxifene is a (Selective estrogen receptor modulator)
Reference: Kaplan Pharmacology for Step 1.

49. What is the mechanism of action of inhaled antiviral?


A. Inhibit DNA
B. Inhibit polysaccharide capsule.
C. Inhibit replication

Answer: C
Inhaled antiviral (Zanamivir; A neuraminidase inhibitors) is used for prevention and treatment of influenza A and B. It inhibits
influenza neuraminidase lead to decrease the release of progeny virus and inhibit virus replication.

50. Aspirin side effect


A. Dry mouth
B. Constipation
C. Diarrhea
Answer C

51. What is the contraindication for the use of prostaglandin F2 alpha, which is used in induction of labour:
A. Asthma
B. Hypertension
C. Diabetes mellitus"
Answer: A
Reference: http://www.ncbi.nlm.nih.gov/m/pubmed/6961113/

52. Which atypical antipsychotic that causes weight gain?


A. Quetiapine

331
B. Risperidone
C. Aripiprazole
Answer: B
Both Quetiapine and risperidone causes weight gain but is more common in risperidone.
Reference: http://www.ncbi.nlm.nih.gov/pmc/articles/PMC3998960/#!Po=17.6802

53. Which one of the following can be given to a patient with beta-lactam-resistant organisms?
A. Azithromycin
B. Vancomycin
C. Gentamicin

Answer: B
The Q is missing important information but with the given info the most propable answer is B
Reference: Dr. Yousef Alawlah

54. Long scenario of patients labs show low hg low platelet with normal reticulocyte which antibiotic cause this ?
A. Tetracycline
B. Cloroampincoal
C. Cefepime

Answer: A&C
Both tetracycline and cefepime can rarly cause hemolytic anemia and low platelet count. Cloroampincol is a wrong answer
because it causes aplastic anemia, which will show low reticulocyte

55. What anticoagulant can distract the clot?


A. Enaxbaren
B. Heparin
C. Aspirin
Answer: fibrinolytic ( streptokinase, urokinase, tissue plasminogen activator “ altplase “ )

56. Man got a bee sting then his wife gave him epinephrine what is it going to inhibit?
A. Luektrine release from macrophages
B. Cross reactivity with the cardiac…
C. Inhibit immunocomplex formation
Answer: MOA of epinephrine is vasoconstriction, increase cardiac output, increase respiratory rate, decrease mucosal edema,
bronchodialation, and decrease the release of mediators such as histamine and tryptase from mast cells and basophils, which
reduces urticaria.

57. Patient is taking Sublingual Isosorbide Dinitrate for myocardial infarction. What will be the side effects of the drug?
A- Hypotension
B- Hypokalemia
C- Heart Block
Answer: A. It is a nitrate, similar Q (headache was the right choice)

332
58. Nitate sublingual side effect ??
A- headache✔
B-hypokalemia
C- bronchospasm
Answer: If no hypotension in the choices >> then choose Headache .

59. Patient with asthma exacerbation, Which drug will decrease the mucous secretion more the broncho dilation?
A-oral steroids
B-ipratropuime
B-luekot
Answer: Omalizumab on Asthma, ipratropium in COPD

60. Case of Absent seizure, what’s the effect of injecting Phenytoin intrathecal:
A- inhibit secretion of substance P
B- seizure secondary of toxic metabolite
C- increase in seizure frequency
Answer: C

61. Treat of hypertg only ?


A- NA
B- fibrate
C- pyt
Answer: B http://www.medscape.org/viewarticle/563490

62. Long scenario of patients labs show low hg low platelet with normal reticylcyte which Antibiotic cause this ?
A. Tetracycline
B. cloroampincoal
C. cefepime
Answer : ‫ ا‬B?

63. Patient presented with vertigo what is the cause:


A.ethambutol
B.streptomycin
C.isoniazid
Answer: b
Http://www.drugs.com/sfx/streptomycin-side-effects.html

64. Most undesired side effect of anticholinergic agents?


A. Diarrhea
B. Excessive salivation
C. Blurred vision
Answer: C

65. Someone receive halpredol next day develop dyskinesia + up rolling eye
A. Acute dystonia ( within hrs )
333
B. Neuroleptic malignant syndrome
C. Tradtive dyskinesia (after months )

Answer: A , acute dystonia (oculogyric crisis). Treat with benzotropine or diphenhydramine


st th
1 Aid USMLE STEP2 CK 8 edition, pg506

66. MTREATMENT toxicity what to give?


A. Folic acid
B. Folinic acid
C. Steroid

Answer is B. Leucovorin (folinic acid).


Http://www.mdpoison.com/media/SOP/mdpoisoncom/toxtidbits/2009/January%202009%20toxtidbits.pdf

67. Patient on metformin , lab values given with low PH, normal urine osmolarity, no ketones ;(pharma)
A. DKA
B. Hyperosmolar hyperglycemia
C. Lactic acidosis

Answer: C
Lactic acidosis is the most common cause of metabolic acidosis in hospitalized patients. It is recognized as a potential complication
of metformin use, particularly in patients with risk factors such as renal dysfunction, liver disease, and heavy alcohol ingestion.

Refrence: http://www.medscape.com/viewarticle/827622
Http://www.medicinenet.com/metformin-oral/article.htm
68. Antibiotic that inhibit calcium and aluminium salt ?
G. Tetracyclin
H. Clhloroamphencol
I. Something pencillin
Answer:A
Absorption of tetracycline may be impaired by ingestion of cations like Ca2+, Mg 2+, Al3+, Fe2+/3+, and Zn2+ by chelating
tetracycline and form poorly soluble complexes and by alkaline ph

Reference:http://www.antibiotics-info.org/tetracycline.html

69. Patient with fever, abdominal pain, constipation then diarrhea, blood culture showed Gram negative lactose non fermenting,
oxidase negative, what AB group will you treat with?
A. DNA gyrase
B. 50 S ribosme
C. Transpeptidase

Answer: A
Uptodate: Fluoroquinolones are the only class of antimicrobial agents in clinical use that are direct inhibitors of bacterial DNA
synthesis. Fluoroquinolones inhibit two bacterial enzymes, DNA gyrase and topoisomerase IV, which have essential and distinct roles

334
in DNA replication. Ultimately, this action results in damage to bacterial DNA and bacterial cell death. Thus, fluoroquinolones are
bactericidal agents.
Https://yhdp.vn/uptodate/contents/mobipreview.htm?27/60/28618
Http://www.asp.mednet.ucla.edu/files/view/guidebook/microbiologyoverview.pdf

70. Which of the following should be avoided during pregnancy?


A. Amoxicillin
B. Cephalosporins
C. Fluoroquinolones
Answer: C
Http://www.merckmanuals.com/professional/infectious-diseases/bacteria-and-antibacterial-
drugs/fluoroquinolones

71. Which one of the following Alzheimer’s drugs causes hepatotoxicity?


A. Tacrine
B. Donepezil
C. Rivastigmine
Answer: A
All of these are acetylcholinesterase inhibitors all of which can cause GI dysfunction and muscle cramps. Tacrine is associated with
liver toxicity.
BRS pharmacology

72. A patient is on diuretics and digoxin. He is feeling palpitation and muscular weakness. What is the most likely electrolyte
disturbance associated with these symptoms?
A. K
B. Ca
C. Na
Answer: A
These symptoms are consistent with hypokalemia that is a side of both diuretics and chronic use of digoxin.
Toronto notes 2015

73. An elderly patients is currently on intranasal antiviral treatment for the influenza. What is the mechanism of action of this
antiviral?
A. Inhibit viral neuroaminidase
B. DNA gyrase
C. Polymerase
Answer A
The patient is most likely taking zanamivir that is an anti-influenza agent available intranasally. BRS pharmacology

74. Phenobarbital in lactating women :


A)Stop immediately
B) Continues
C) Fed after 8 hr

335
Answer: B?
Inter- and intrapatient variability in excretion of phenobarbital into breastmilk is extensive. Phenobarbital in breastmilk apparently
can decrease withdrawal symptoms in infants who were exposed in utero, but it can also cause drowsiness in some infants,
especially when used with other sedating drugs. Monitor the infant for drowsiness, adequate weight gain, and developmental
milestones, especially in younger, exclusively breastfed infants and when using combinations of psychotropic drugs. Sometimes
breastfeeding might have to be limited or discontinued because of excessive drowsiness and poor weight gain. If there is concern,
measurement of the infant's serum phenobarbital concentration might help rule out toxicity. Ref:
http://www.drugs.com/breastfeeding/phenobarbital.html

75. Metformin action at the cellular level:


A) inhibit tyrosine kinase
B)activate adenylate kinase
C) decrease ATP
Answer: It inhibits purified AMP deaminase . Furthermore , a known inhibitor of AMPD stimulated Glucose uptake and fatty acid
oxidation .
Http://www.ncbi.nlm.nih.gov/pmc/articles/PMC3398862/

76. Which one has inhibitory action?


A. Phosphtase A2
B. Cycloexagonase
C. Not sure in understand this question

77. Which anti-hypertensive drug can cause severe hypertension when


stopped abruptly?
A. Clonidine
B. Thiazide
C. Beta blocker
Answer: A
http://www.doctorshangout.com/profiles/blogs/clonidine-adverse-effects-and-rebound-hypertension

78. Diabetic on oral hypoglycemic with hypoglycemia :


A. Rosiglitazone
B. Metformin
C. Glyburide is sulfonylurea
Answer: C

79. Diabetic with poor control on glyburide 10 ml :


A. Increase the dose to 20
B. Add metformin
C. Switch to metformin
Answer: B (no insulin )

80. MTX toxicity what to give?

336
A. Folic acid
B. Folnelic acid
C. Steroid
Answer: ?
If one of the choices is Leucovorine it would be the answer
((according to uptodate for ministry of health in KSA + first aid USMLE ,25th edition,P409 ))

81. pt has duodenal ulcer and take antacid then developed constipation ,, which of these drugs cause constipation :
A. calcium carbonate
B. aluminum hydroxide
C. magnesium hydroxide

82. Antidote of paracetamol :


Acetylcysteine . Medscape
https://www.google.com/url?Sa=t&rct=j&q=&esrc=s&source=web&cd=4&cad=rja&uact=8&ved=0ahukewje_j
2g1sznahxl8rqkha4dazcqfggrmam&url=http%3A%2F%2Femedicine.medscape.com%2Farticle%2F820200-
treatment&usg=afqjcnhpcbnc2ojjdisiplv4ovzo_jznbq&sig2=7nrohvwidsidqjgsunoe9a

83. Pt with MI and take morphine (not sure about route of administration ) What is the side effect of morphine
A. Dry cough
B. Tachypnea
C. Anxiety

Answer: C
( side effects of morphin: http://reference.medscape.com/drug/ms-contin-astramorph-morphine-343319#4)
84. What two drugs are contraindicated together?
A. Digoxin and Levodopa
B. Sulpha
C. Tetracycline and aluminum
Answer: C
Reference:
Peer review
Drug.com: Absorption of Tetracyclines is impaired by antacids containing aluminum, calcium or magnesium and
preparations containing iron, zinc, or sodium bicarbonate.Concurrent use of Tetracycline Hydrochloride Capsules, USP may
render oral contraceptives less effective.
Medsacpe: aluminum hydroxide has serious interaction with tetracycline.

85. What is incretin drug mechanism of action?


A. Insulin resistant
B. Hepatic gluconeogenesis
C. Insulin release
Answer: C
337
These drugs work by mimicking the incretin hormones that the body usually produces naturally to stimulate the release of insulin in
response to a meal. They are used along with diet and exercise to lower blood sugar in adults with type 2 diabetes.
http://www.fda.gov/Drugs/DrugSafety/InformationbyDrugClass/ucm343516.htm

86. What is the mechanism of action of metformin on the cellular level?


A. decrease Muscle uptake of glucose
B. increase Muscle gluconeogenesis
C. Enhance muscle use of fatty acid oxidation

Answer: C
Decreases hepatic glucose production, decreasing intestinal absorption of glucose and improves insulin sensitivity (increases
peripheral glucose uptake and utilization)
Reference: UpToDate

87. Why we are concerned about anti hypertensive agents in elderly patients?
A. Hypotension
B. Hypokalemia
C. Cns side effect

Answer: a
Reference: http://www.ncbi.nlm.nih.gov/pubmed/11574742

88. a patient on digoxin and lisinopril or ccb??? What is the most probable electrolyte imbalance that will affect his ecg?
A. K
B. Na
C. Mg

Answer: a
Digoxin side effect.

89. A patient with an infection that is resistant to beta-lactam antibiotics, what antibiotic should be given?
A. Azithromycin
B. Vancomycin
C. Gentamicin

Answer: b
Reference: master the boards.

90. Which of the following drugs cause insomnia, irritability and restless?

338
A. Tca
B. Tetracyclin antidepressant
C. Ssri

Answer: c
Ssri side effects are cns- restlessness, tremor, insomnia, headache and drowsiness, abdominal- diarrhea, cramps and weight loss +
sexual dysfunction and impotence. Tca cause noradrenergic effect like tremor, tachycardia, sweating, insomnia and antihistamine
effects.
Reference: toronto notes psychiatry page 40 table 18

91. Man got a bee sting then his wife trying look for the epinephrine what it gonna inhibit?
A. Luektrine release from macrophages
B. Cross reactivity with the cardiac.
C. Inhibit immunocomplex formation

Answer:
Epinephrine is the drug of choice for anaphylaxis. It stimulates both the beta-and alpha-adrenergic receptors and inhibits further
mediator release from mast cells and basophils. It inhibits histamine, the proteases tryptase and chymase.
https://quizlet.com/8362963/immuno-block-4-practice-exam-flash-cards/
http://www.worldallergy.org/professional/allergic_diseases_center/anaphylaxis/anaphylaxissynopsis.php
http://www.ncbi.nlm.nih.gov/pmc/articles/pmc3343118/

92. Breast cancer patient who receives many medications (cyclophosphamide, fluorouracil, ..etc). In order to avoid the adverse
effect of having hemorrhagic cystitis which of the following will be given to this patient?
A. Aldesleukin
B. Mesna

Answer: b
Mesna: protects the bladder from damage that may be caused by some cancer medicines.
Reference: http://chemocare.com/chemotherapy/drug-info/mesna.aspx

93. Incretin role in dm:


A. Incease insulin sensitivity.
B. Stiumlate insulin secretion.
Answer: B
(Be careful it increase the secretion they play with phrasing you get confuse)

94. Which of the following toxins has many uses?


A. Botulinum
B. Tetanus

339
Answer: A
Http://www.ncbi.nlm.nih.gov/pmc/articles/PMC1128745/

95. Haloperidol toxicity, unrolling of eyes and dyskinesia, what is this?


A. Tardive dyskinesia
B. Neuroleptic malignant syndrome
Answer: B
(Emedicine) Administration of neuroleptic medications can result in any of the consequences listed in Physical; however, certain
combinations of medications (eg, lithium + haloperidol, anticholinergics + haloperidol), depot preparations (eg, fluphenazine and
haloperidol), and stronger neuroleptics (eg, haloperidol) are more likely to produce adverse effects, including neuroleptic malignant
syndrome (NMS)

96. Patient taking sympathomimetic what's the symptoms:


A- Hypotension.
B- Increased Heart Rate.

Answer: B

97. Side effects of atropine ?


A. Tachycardia
B. Bronchoconstriction

98. Absolute OCP contraindication?!


A- undiagnosed breast mass.
B- previous hx of DVT
Answer : B

Reference : Kaplan –Obstetrics & gynecology Step 2CK

340
99. Which of the following medication causes vertigo?
A) Ethambutol
B) Streptomycin
Answer: B
The following reactions are common: vestibular ototoxicity (nausea, vomiting, and vertigo); paresthesia of face; rash; fever; urticaria;
angioneurotic edema; and eosinophilia.
Reference : http://www.drugs.com/pro/streptomycin.html

100.What medication that cause hypertensive crisis with Tyramine


A) MAOI
B) TCA
Answer: MOAI
The blockade of maoia in the gastrointestinal tract is responsible for the "cheese reaction" associated with maois. This refers to a
severe hypertensive crisis that can occur after patients on maois ingest foods containing the sympathomimetic tyramine.
Reference : up to date

101.Aspirin toxicity resulting in


A. Respiratory alkalosis followed metabolic acidosis
B. Metabolic acidosis followed respiratory alkalosis
Answer : A
Salicylate toxicity can be grouped into phases: Phase 1 of the toxicity is characterized by hyperventilation leading to respiratory
alkalosis and compensatory alkaluria. Last as long as 12 hours. Phase 2, paradoxic aciduria in the presence of respiratory
alkalosis begin within hours and may last 12-24 hours. Phase 3 includes dehydration, hypokalemia, and progressive metabolic
acidosis. This phase begin 4-6 hours after ingestion in a young infant or 24 hours or more after ingestion in an adolescent or adult.
Reference : http://emedicine.medscape.com/article/1009987-overview

102.Most common side effect of Atropine? (No Dry Mouth in options?)


A- diaphragm paralysis
B-stimulate sweat gland
Answer: None
It has anti-cholinergic effect! Most of the side effects of atropine are directly related to its antimuscarinic action.
Dryness of the mouth, blurred vision, photophobia and tachycardia commonly occur with chronic administration of therapeutic
doses. Constipation and difficulty in micturition may occur in elderly patients.
Reference : http://www.rxlist.com/atropine-drug/side-effects-interactions.htm,The Goodman and Gilman Manual of
Pharmacology and Therapeutics (table)

341
103.Medication for acidity and heartburn that causes constipation.
A- Sodium chloride
B- Calcium…
Answer: aluminum hydroxylase
Aluminum salts (Amphogel, Alternagel). The most common side effect of antacids containing aluminum salts is constipation.
Reference : (medline) http://www.nytimes.com/health/guides/symptoms/heartburn/medications.html

104.Patient on lovastatin, his LDL and Cholesterol level in lower abnormal levels . What to add ?
A) Omega 3
B) Cholestyramine
Answer: ?
Patient who have low LDL require early treatment with very high doses of vitamin E Management in adults includes treatment of the
complications of the disorders.
Reference : http://emedicine.medscape.com/article/121975-treatment

105.Mode of action of aspirin:


A. Inhibit platelet aggregation
B. Acts on antithrombin
Answer: A
Aspirin inhibits platelet aggregation by irreversibly inactivating cyclooxygenase, a key enzyme in platelet prostaglandin metabolism.
Reference: Pubmed.

106.What is the mechanism of action of denosumab anti-resorptive properties:


A. Cases osteoclastic apoptosis
B. Inhibit RANKL

Answer: B
Activation of RANK by RANKL promotes the maturation of pre-osteoclasts into osteoclasts. Denosumab inhibits this maturation of
osteoclasts by binding to and inhibiting RANKL.

342
107.Regarding mechanism of action of Aspirin on enzymes?
A. Stimulate cyclooxygenase.
B. Irreversible inactivation of the cyclooxygenase (COX) enzyme

Answer: B
Irreversibly inhibits cyclooxygenase (both COX-1 and COX-2) enzyme.

108.Which medication decreases the effect of ocps?


A. Antiepileptic
B. Anticoagulant
Answer: A
Antiepileptics (barbiturates, phenytoin, and carbamazepine) are general inducers, they increase the metabolism of ocps thereby
decreasing their effect. Reference: kaplan Pharmacology for Step 1.

109.Elderly patient with low back pain, what is the drug that causes relaxation of muscle?
A) Diazepam
B) Clorcarpine
Answer: A

110.Patient diagnosed with duodenal ulcer he was prescribed medication 1 month ago now he have gynecomastia which
medication:
A. Cimetidine
B. Lansoprazole
Answer: A
Clinically, cimetidine can cause galactorrhea in women and gynecomastia, reduced sperm count, and impotence in men (Source:
goodman Gilman)

111.Patient with acute MI typical scenario, the analgesia he was given will work on: (most likely asking about morphine)
A. Inhibition of COX
B. Work on P-peptide or receptor can't remember exactly.

Answer: ?
Treatment of acute MI includes Aspirin, Nitrate, and morphine all of which play a role in analgesia. So, I am not sure which drug
MOA the Q wants.
- Aspirin inhibit cox
- Nitrate act as a vasodilator which will reduce ischemic pain
- Morphine acts on CNS by blocking mu receptors. Analgesia for MI Intravenous opiates (initially, morphine sulphate 5–10 mg
or diamorphine 2.5–5 mg) and antiemetics (initially, metoclopramide 10 mg) should be administered. Morphine sulfate,
an opioid agonist, is relatively selective for the mu receptor, although it can interact with other opioid receptors at higher
doses.
- Reference : http://www.rxlist.com/avinza-drug/clinical-pharmacology.htm

343
112.Symptoms of URTI in patient, keeping in mind current flu season you prescribed antiviral which works on:
A. Protease
B. DNA polymerase
Answer: The antiviral is likely to be Oseltamivir, which is a neuraminidase inhibitor (Flu virus surface protien). This option is not
available.

113.Positive culture of budding yeast in urine what is the management ?


A-Flucanazole
B-caspofungin
Answer: A

114.Which of the following will cause sedation in low dose ;


A) midazolam
B) lorazipam
Answer: A

115.Treatment for fever, cough, bilateral infiltration ?


A- Cipro
B- azithro
Answer: B..IF they meant legionella in the question.

116.Old patient with recent memory loss and poor self-care and social withdrawal, what to give him?
A) Neostigmen
B) Rivastigmen
Answer:B

117.Heparin mechanism of action:


A. Antithrombin 3
B. Inactive xa

Answer: A
Binds reversibly to antithrombin III (ATIII) and greatly accelerates the rate at which ATIII inactivates coagulation enzymes thrombin
(factor iia) and factor Xa
Http://www.drugbank.ca/drugs/DB01109

118.Comatose patient with respiratory depression and pinpointed pupil, drug abuse suspected what is the cause?
A. Cocaine
B. Opiates

Answer: B
Explanation:

344
Opioids Cocaine
Euphoria at first leading to apathy Psychomotor agitation, euphoria, impaired
CNS depression judgment
Symptoms of Constipation Tachycardia, fever, diaphoresis, HTN
overdose Pupillary constriction Pupillary dilations
Respiratory depression (could be Paranoia & hallucination (cocaine
life threatening) bugs/feeling of bug under skin)
Naloxone Haloperidol for agitation
Management
Symptom specific control
Reference First Aid USMLE step 2CK

119.Diabetic asking about the mechanism of action of incretin ?


A- increase insulin secretion
B- increase insulin sensevitiy

Answer: A
Explanation: glucagon-like peptide 1, GLP-1 (e.g. Exenatide and liraglutide). It regulates the rate through which nutrients transit the
gastrointestinal tract. And more importantly, enhances pancreatic insulin secretion in a glucose-dependent manner, ie, only when
glucose is high and not when glucose is normal or low.
Reference: http://www.medscape.org/viewarticle/541424_3

120.Clear case of absence seizure then asked what is going to happen to this patient if given fentanyl :
A. Glutamate receptors activation
B. Seizure activity due to toxic neurotransmitters release

Answer: B
Http://austinpublishinggroup.com/anesthesia-analgesia/fulltext/ajaa-v2-id1026.php
Http://www.epilepsy.com/information/professionals/diagnosis-treatment/procedures-epilepsy-patients/general-anesthetics-4

121.History of 4 time in 2 month of URTI with ant thyroid medication ?


A. Methimazole
B. Propronolol

Answer: A. Agranulocytosis – Agranulocytosis is a term used to describe a severe decrease in the production of white blood cells.
This condition is extremely serious, but affects only one out of every 200 to 500 people who take an antithyroid drug. (methimazole,
propylthiouracil) http://www.uptodate.com/contents/antithyroid-drugs-beyond-the-basics

122.Patient taking sympathomimetics what's the Sx?!


A. Increase HR
B. Hypotension
Answer: A

345
Reference: http://www.pharmacology2000.com/Autonomics/Adrenergics/sympclin1.htm

123.Which anti-lipid medications decrease LDL and Triglycerides, and increase HDL?
A. Statin
B. Fibrate

Answer: A
Statins block the production of cholesterol in the liver itself. They lower LDL and triglycerides, and have a mild effect in raising HDL.

124.Patient came to ER after eating a lot of drugs, came with eye dilatation What does he takes:
A) TCA
B) Organophosrous
Answer: A
Link: http://bestpractice.bmj.com/best-practice/monograph/342/diagnosis/step-by-step.html

125.Ant obesity medication "I don’t remember the name " MOA:
A. Inhibit the appetite center
B. Inhibit the lipase from pancreas
Answer: B if Orlistat
Orlistat inhibits pancreatic lipases, thereby reducing fat absorption from the gut by ∼30%.
Table from: http://www.cell.com/trends/neurosciences/pdf/S0166-2236(12)00206-8.pdf

126.Child with typical absence seizure, what would happen if he were given fentanyl?
A. Demyelination
B. Increase stimulation of excitatory GABA,
Answer: ??

346
127.Acetaminophen toxicity, what will happen or what we afraid of
A- Depletion of glutathione in liver
B- NADPH
Answer: A
Http://emedicine.medscape.com/article/820200-overview#a3

128.Which drug of anti-tb cause vertigo?


A) isoniazaid
B) ethamputal
Answer: I think streptomycin
Streoptomycin
Streptomycin injections are painful. Rash, induration, or sterile abscesses can form at injection sites. Numbness and tingling around
the mouth occur immediately after injection. Cutaneous hypersensitivity reactions can occur. Impairment of vestibular function is
uncommon with currently recommended doses. Hearing loss is less common than vertigo. Manifestations of damage to the 8th
cranial (auditory) nerve include ringing in the ears, ataxia, vertigo and deafness; damage usually occurs in the first 2 months of
treatment and is reversible if the dosage is reduced or the drug is stopped (2). Streptomycin is less nephrotoxic than other
aminoglycoside antibiotics. If urinary output falls, albuminuria occurs or tubular casts are detected in the urine, streptomycin should
be stopped and renal function should be evaluated. Haemolytic anaemia, aplastic anaemia, agranulocytosis, thrombocytopenia and
lupoid reactions are rare adverse effects.

129.Pt with high aptt what mechanism of action of that drug?


A. Unfractinated heparin
B. Antithrombin

Answer: heparin act indirectly by binding to antithrombin rather than binding directly to coagulation factors.
Reference: http://www.uptodate.com/contents/therapeutic-use-of-unfractionated-heparin-and-low-molecular-weight-
heparin?Source=outline_link&view=text&anchor=H3#H3

130.How to treat or avoid GIT complications of ASA (Aspirin) high dose?


A. Cimetidine
B. Misoprostol
Answer: B

131.Side effect of atropine:


A. Dry mouth.
Answer: A
Most of the side effects of atropine are directly related to its antimuscarinic action. Dryness of the mouth, blurred vision,
photophobia and tachycardia commonly occur with chronic administration of therapeutic doses. Anhidrosis also may occur and
produce heat intolerance or impair temperature regulation in persons living in a hot environment. Constipation and difficulty in
micturition may occur in elderly patients. Occasional hypersensitivity reactions have been observed, especially skin rashes which in
some instances progressed to exfoliation.
http://www.rxlist.com/atropine-side-effects-drug-center.htm

132.Antidot of digoxin over dose ? ( all answer one word name of medication )

347
A. Digibind

133.Enterococcus faecalis antibiotic and the patient is allergic to ampicillin , what to give -
A-Erythromycin
Answer: vancomycin
Vancomycin Used to treat enterococcal infections when ampicillin is contraindicated due to significant penicillin allergy and when
strains are resistant to ampicillin but susceptible to vancomycin. Target levels of 30-50 mcg/ml (peak) and 10-15 mcg/ml (trough) for
endocarditis and other serious infections.
Reference : http://emedicine.medscape.com/article/216993-medication#2

134.Female patient on anti-epileptic drugs presented with gum hypertrophy and excessive hair growth on face, chest back?
A:phenytoin
Answer : A
The major systemic side effects of phenytoin are gingival hypertrophy, body hair increase, rash, folic acid depletion, and decreased
bone density.
Reference : Uptodate, http://www.drugs.com/sfx/phenytoin-side-effects.html

135.TCA most associated with weight gain?


Answer : Amitriptyline is associated with a great tendency to gain weight. Doxepin, and Nortriptyline all have intermediate
tendency.
Reference: Sadock's clinical psychiatry and Kaplan psychiatry

136.Anti-depressant that causes constipation?


Answer : Amitriptyline.
Urinary retention, blurred vision, dry mouth and constipation are common anticholinergic side effects associated with tcas.
Amitriptyline Is the most anticholinergic.
Reference: Sadock's clinical psychiatry and Kaplan psychiatry

137.A female using OCP developed black discoloration on her face, what's your diagnosis?!
Answer : Melasma

Melasma is a chronic skin disorder that results in symmetrical, blotchy, brownish facial pigmentation. Hormone treatments—oral
contraceptive pills containing oestrogen and/or progesterone, hormone replacement, intrauterine devices and implants are a factor
in about a quarter of affected women. Reference : http://www.dermnetnz.org/colour/melasma.html

138.Patient eat a lot of aspirin tablets .. How will manage ?


Answer: nahco3
Decontamination — Activated charcoal (AC) effectively absorbs aspirin, and at least one initial dose (1 g/kg up to 50 g PO) should be
given to all alert and cooperative patients and all intubated patients via orogastric tube who present within two hours of ingestion,

348
Alkalinization of serum and urine — Alkalinization with sodium bicarbonate is an essential component of management of the aspirin-
poisoned patient
Reference : uptodate

139.Mechanism of aspirin in old man with pains.


Answer:
Intermediate doses (650 mg to 4 g/day) inhibit COX-1 and COX-2, blocking prostaglandin (PG) production, and have analgesic and
antipyretic effects.
Reference : uptodate

140.Why Aspirin cause hypothermia?


Answer: Intermediate doses (650 mg to 4 g/day) inhibit COX-1 and COX-2, blocking prostaglandin (PG) production, and have
analgesic and antipyretic effects.
Reference : uptodate

141.Action of heparin??
Answer:
Heparin acts at multiple sites in the normal coagulation system. Small amounts of Heparin in combination with antithrombin III
(Heparin cofactor) can inhibit thrombosis by inactivating activated Factor X and inhibiting the conversion of prothrombin to
thrombin. Once active thrombosis has developed, larger amounts of Heparin can inhibit further coagulation by inactivating thrombin
and preventing the conversion of fibrinogen to fibrin. Heparin also prevents the formation of a stable fibrin clot by inhibiting the
activation of the fibrin stabilizing factor.
Reference : http://www.drugs.com/pro/heparin.html

142.Paracetamol antidote?
Answer : N-acetylcysteine
The management of the acetaminophen-poisoned patient may include stabilization, decontamination, and administration of N-
acetylcysteine, a specific antidote. The duration of N-acetylcysteine treatment is determined by the type of ingestion and the
presence or absence of elevated serum alanine aminotransferase (ALT) concentrations.
Reference : http://www.uptodate.com/contents/acetaminophen-paracetamol-poisoning-in-adults-treatment

143.A patient presented to the ER with respiratory depression and pinpointed


the drug is cocaine I think
Answer : Opioid
Opioid use has increased significantly over the past ten years and so has the incidence of reportable adverse events, such as
respiratory depression and/or arrest.
Reference : http://www.medscape.com/viewarticle/813875_4

144.A patient presented to the ER with respiratory depression and pinpoint pupil
the drug is?
Answer: morphine

145.What is the mechanism of warfarin?


349
Answer : inhibiting synthesis of vitamin K
Warfarin acts by inhibiting the synthesis of vitamin K-dependent clotting factors, which include Factors II, VII, IX, and X, and
the anticoagulant proteins C and S. It blocks function of vit K epoxide reductase in the liver.
Reference : http://www.rxlist.com/coumadin-drug/clinical-pharmacology.htm

146.Patient HTN he complain of cough he want to change the drug to another one drug with same effect but no cough ?
Answer: he is using ACEI .. We give ARB instead
ACEI Side-effects include first-dose hypotension, cough, rash, angioedema, hyperkalaemia and renal dysfunction.
Angiotensin receptor blockers: block the angiotensin II type I receptor and have similar effects to ACE inhibitors; however, they do
not cause cough and are better tolerated.
Reference : Davidson's Principles and Practice of Medicine, 22nd ed, P611

147.Statin induced myopathy in old lady


Answer: (inadequate information)
Myopathy symptoms induced by statin therapy resolve relatively quickly; and can be reversed when the medication is discontinued.
It may present as rhabdomyolysis or severe muscle damage. Patients who present with clinically significant rhabdomyolysis require
hospitalization and IV hydration to prevent renal damage.
Reference : http://www.medscape.com/viewarticle/759844_8

148.Which of the following antibiotics is DNA gyrase and it works on what organism ?
Answer: Fluoroquinolones
Fluoroquinolones are the only class of antimicrobial agents in clinical use that are direct inhibitors of bacterial DNA synthesis. This
class have been developed with a broader spectrum of activity including better coverage of gram-positive organisms and even
anaerobes. It is available for systemic use include ciprofloxacin, gatifloxacin, gemifloxacin, levofloxacin, moxifloxacin, and ofloxacin.
Tthat antibiotic, which works in DNA gyrase, is Quinolone and one member of the family called Ciprofloxacin can cover
pseudomonas.
Reference : up to date

149.Epinephrine work on which cytokines


Answer :
Epinephrine is a direct-acting sympathomimetic drug that acts as an agonist at alpha and beta-adrenergic receptors. It produces
vasoconstriction to counteract the vasodilation and resulting hypotension associated with anaphylaxis. Epinephrine down-regulate
the release of histamine, tryptase
Reference : http://www.medscape.com/viewarticle/578750_2

150.Side effect of silver sulfadiazine ?


Answer: Leukopenia, pain, skin necrosis, erythema multiforme, skin discoloration, burning sensation, rashes, and interstitial
nephritis.

350
Reference : http://www.rxlist.com/silvadene-drug/side-effects-interactions.htm

151.Case of UTI methicillen sensitive ?


A-Cloxacillin

Answer : A
The term " methicillin – sensitive " or " methicillin- related Staphylococcus aurous" (MRSA)is somewate of a misnomer because we
don't acwally use methicillin. When this term is used think of the drugs oxacillin, cloxacillin, dicloxacillin, and nafcillin.
Reference: Kaplan -internal medicine Step 2CK

152.Mechanism of aspirin in pregnant lady with history of abortions


Answer:
Low doses (typically 75 to 81 mg/day) are sufficient to irreversibly acetylate serine 530 of cyclooxygenase (COX)-1. This effect
inhibits platelet generation of thromboxane A2, resulting in an antithrombotic effect.
Reference : uptodate

153.Glaucoma drugs side effect:


Answer:
Insufficient info, refer to the picture

351
154.A scenario telling that a patient has viral infection and you will use an inhaled antiviral. What anti-viral are you going to use?
A- Zanamivir
Answer: A
Http://emedicine.medscape.com/article/1966844-overview#a1

155.What is the action of Atropine?


A. Increase heart rate.
Answer: A

156.Aspirin inhibit which product formation?


A. Thromboxane A2
Answer: A
Aspirin irreversibly inhibits platelet cyclooxygenase 1 preventing the formation of prostaglandin H2, and therefore thromboxane A2

157.What is the mechanism of action Montelukast?


A. Inhibit lukotrine

Answer: A
Montelukast is a selective leukotriene receptor antagonist that inhibits the cysteinyl leukotriene cyslt1 receptor. This activity
produces inhibition of the effects of this leukotriene on bronchial smooth muscle resulting in the attenuation of bronchoconstriction
and decreased vascular permeability, mucosal edema, and mucus production.
Reference : uptodate

158.What antihypertensive medication increase uric acid and glucose:


A. Thiazide

Answer: A

159.Which of the following drugs causes weigh gain?


A. Risperidone
Reference: https://www.drugs.com/risperidone.html

160.Aspirin toxicity management:


A. Alkalization of urine
Answer: A
Alkalization of blood and urine keeps salicylates away from brain tissue and in the blood, in addition to enhancing urinary excretion.
Reference: http://emedicine.medscape.com/article/1009987-treatment#showall

352
161.Stress induced cortisol release, and decrease the immunity, what type of therapy?
Answer: Psychommunology?!

162.Atypical antipsychotic with least EPS risk


A. Clozapine
Answer: A
EPS is more with risperidone > olanzapine > quetiapine > clozapine & aripiprazole
Reference: http://www.medscape.com/viewarticle/756308_3

163.Hypertensive with BPH what to give?


A. Prazosin
Answer: A (alpha blocker)

164.Long scenario about peptic ulcer and ask about galactorrhea complication of which drug cause it?
A. Cimitedine

Answer: A

165.Clonidine decreases the effect of which of the following drugs?


Answer: ?
Clonidine may decrease the effectiveness of levodopa to control the symptoms of Parkinson disease.

166.What is the side effect of thalidomide in pregnancy?


Answer: ?
The use of thalidomide in humans has produced a high rate of severe and life-threatening birth defects such as phocomelia, amelia,
hypoplasticity of the bones, absence of bones, external ear abnormalities (including anotia, micropinna, small or absent external
auditory canals), facial palsy, eye abnormalities (anophthalmos, microphthalmos), and congenital heart defects.

167.What is the mechanism of polio vaccine (Both types; oral and IV)?
Answer: ?
Infection with poliovirus causes igm and igg responses in the blood, but mucosal iga is vital for blocking infection. This antibody
can neutralize poliovirus in the intestine, the site of primary infection. The live attenuated Sabin poliovirus vaccine (oral) is effective
because it elicits a strong mucosal iga response and provides intestinal immunity. On the other hand IPV is less effective in eliciting
intestinal immunity. Reference: Pubmed.

168.What are the antibiotics can be given safely during breast feeding?
Answer: ?

353
169.Which antidepressant can cause constipation?
Answer: ?
Tcas, SSRI, SNRI all cause constipation but TCA causes constipation more common than the others.

170.Aspirin can be used as an antipyretic but why does it cause hyperthermia if given in high doses?
Answer:?
Salicylates toxicity: Salicylates also uncouple cellular oxidative phosphorylation, resulting in increased oxygen consumption,
increased heat production, and fever with simultaneous decreased ATP production. Salicylates stimulate respiration directly and
indirectly. Uncoupling of oxidative phosphorylation leads to increased peripheral CO2 production and a compensatory increase in
minute ventilation, usually with no overall change in PCO2. Uncoupling of oxidative phosphorylation also leads to excessive heat
production, and salicylate toxicity is associated with hyperthermia. While it lowers Tempt. By preventing production of prostaglandin
E2
Reference: Critical Care Medicine - Perioperative Management, Reference : The Goodman and Gilman Manual of Pharmacology and
Therapeutics

171.What treatment of positive oxidase bacteria?

Answer:
*Antipseudomonal penicillins include ticarcillin and piperacillin
-*Cephalosporins with antipseudomonal activity include: Ceftazidime Cefoperazone Cefepime
Monobactam: aztreonam
Fluoroquinolones: Ciprofloxacin
Reference : uptodate

172.Mechanism of action of combined oral contraception & progesterone?


354
Answer: Progestogen negative feedback decreases the pulse frequency of
Gnrh release, which decreases the levels of FSH and LH and inhibits follicular development, preventing an increase in estradiol
levels. Progestogen negative feedback and the lack of estrogen positive feedback on LH secretion prevent a mid-cycle LH surge.
Inhibition of follicular development and the absence of a LH surge prevent ovulation.

173.Drug overdose pinpoint pupils. What to give


A. Naloxone
Answer: naloxone is the antidote for opioids overdose which is implied in the Q

174.Long scenario patient on metformin and sulfonylurea and not controlled.


A. Acarbose
Answer:

175.Which of ssris drugs suitable for young age and children?


A- fluoxetine.
Answer is A
Http://www.webmd.com/depression/selective-serotonin-reuptake-inhibitors-ssris-for-childhood-and-adolescent-depression

176.Patient with gram-negative diplococcic infection, which abx should be given?


Gram-negative diplococci are Neisseria / antibiotic third-generation cephalosporin (eg, ceftriaxone, cefotaxime) or pencilin .

177.Patient diagnosed as HTN and started meds came back with high glucose. Wt was he given:
- Thiazides
Uptodate:
Thiazide and Beta blockers are also associated with impaired glucose tolerance and an increased risk of new onset diabetes, with the
exception of vasodilating beta blockers such as carvedilol and nebivolol( perhaps thiazide is more important here)

178.Prophylactic of rheumatic disease Normal pt


A- I think the answer penicillin ( not sure)

Answer : A

179.Penicillin the first choice for treating group A streptococcus pharyngitis

Http://apps.who.int/medicinedocs/en/d/Js2252e/3.2.html

180.What is the antithyroid used in pregnancy?

A- propylthiouracil

Answer : A

355
Http://www.ncbi.nlm.nih.gov/pubmed/11005705

181.On patient with gout you will not use:


A. Spironolactone can't recall the choices

Answer : Thiazied
Http://www.medscape.com/viewarticle/489521_7

182.Dexamethasone toxicity is reverse by :

((digoxin-Fab)) http://www.ncbi.nlm.nih.gov/pubmed/25089630

183.Case of digoxin toxicity


Cardiovascular: Accelerated junctional rhythm, asystole, atrial tachycardia with or without block, AV dissociation, first-, second-
(Wenckebach), or third-degree heart block, facial edema, PR prolongation, pvcs (especially bigeminy or trigeminy), ST segment
depression, ventricular tachycardia or ventricular fibrillation
Central nervous system: Dizziness (6%), mental disturbances (5%), headache (4%), apathy, anxiety, confusion, delirium, depression,
fever, hallucinations
Dermatologic: Rash (erythematous, maculopapular [most common], papular, scarlatiniform, vesicular or bullous), pruritus, urticaria,
angioneurotic edema
Gastrointestinal: Nausea (4%), vomiting (2%), diarrhea (4%), abdominal pain, anorexia
Neuromuscular & skeletal: Weakness
Ocular: Visual disturbances (blurred or yellow vision)
Respiratory: Laryngeal edema
Symptoms
Confusion.
Irregular pulse.
Loss of appetite.
Nausea, vomiting, diarrhea.
Palpitations.
Vision changes (unusual), including blind spots, blurred vision, changes in how colors look, or seeing spots)
Reference :https://www.nlm.nih.gov/medlineplus/ency/article/000165.htm

184.Digitalis side effects


More common: Dizziness, fainting , irregular heartbeat or pulse, slow heartbeat
Rare: Black tarry stools, bleeding gums, pinpoint red spots on the skin, severe stomach pain
Reference : http://www.drugs.com/sfx/digoxin-side-effects.html

185.Ca channel blockers side effects

356
Common side effects of calcium channel blockers include: headache, constipation, rash, nausea, flushing, edema , drowsiness, low
blood pressure, dizziness, Sexual dysfunction, , and liver dysfunction .
Reference : http://www.rxlist.com/calcium_channel_blockers_ccbs-page3/drugs-condition.htm

186.Which antiparkinson cause hepatotoxicity?


Answer:
Tolcapone (Tasmar), used for Parkinson disease;
( medscape ) http://livertox.nih.gov/Tolcapone.htm

187.Which of those medication cause anxiety


A. TCA
Answer:
SSRI ( USMLE step 2 ck )

188.Which of bisphosphonate causes inhibition of osteoclast activity?


Answer:
Alendronate inhibits osteoclasts -mediated bone-resorption. Like all bisphosphonates, it is chemically related to inorganic
pyrophosphate, the endogenous regulator of bone turnover. But while pyrophosphate inhibits both osteoclastic bone
resorption and the mineralization of the bone newly formed by osteoblasts, alendronate specifically inhibits bone resorption
without any effect on mineralization at pharmacologically achievable doses. ( ref. Wiki )
Https://en.wikipedia.org/wiki/Alendronic_acid

189.Mechanism of action of propylthiouracil (PTU) (ttt of hyperthyroidism):


Answer: PTU inhibits the enzyme thyroperoxidase which normally acts in thyroid hormone synthesis by oxidizing the anion
− 0
iodide (I ) to iodine (I ), inhibit the conversion of T4 to the active form T3.
Reference: https://en.wikipedia.org/wiki/Propylthiouracil#Mechanism_of_action

190.Side effect of Nitroglycerin (ttt of stable angina ) :


Headache
Reference: http://www.webmd.com/drugs/2/drug-18030/nitroglycerin-oral/details#side-effects

191.Which of the following drug from biphosphonate work as ATP analog to suppers osteoclasts ? All the medication given end
with dronate ...........
(Answer: Clodronate + Etidronate+ Tiludronate )

There are two groups of bisphophonates: simple and nitrogen-containing. Simple bisphosphonates (e.g. Clodronate and
etidronate) induce osteoclast apoptosis. Clodronate creates a toxic analog of adenosine triphosphate, which targets the
mitochondria. For nitrogen-containing bisphosphonates, farnesyl diphosphate synthase (important in the cholesterol
biosynthetic pathway) is the direct intracellular target. Its inhibition suppresses protein geranylgeranylation, an essential
step in the cellular processes required for osteoclastic bone resorption.
Reference: http://www.orthobullets.com/basic-science/9058/bisphosphonates

357
192.DM patient his blood sugar not controlled by metformain and acarbose and he is allergic to sulpha what to add ?
Rivaglitazone

Choices incomplete!
Explanation: choices incomplete and the answer is likely missing. According to best practice, Meglitinides (e.g., repaglinide,
nateglinide) are an alternative to sulfonylureas, and can also be used as a first-choice secretagogue in people with known allergy to
sulphur-containing drugs. They increase the release of insulin.
Reference: http://bestpractice.bmj.com/best-practice/monograph/24/treatment.html

193.Most common anti diabetic medication causing hypoglycemia?


Glyburide carries the highest risk

194.DM patient his blood sugar not controlled by metformin and gli... What to add ??
Acarbose

Incomplete question and choices!


Explanation: the medication starting with gli is likely a new generation sulfonylurea (e.g. Glyburide or glipizide). In patients not
controlled on two drugs, switching to insulin is less expensive and less side effects than adding a third oral agent. If the question is
what to add, then we can add Thiazolidinediones (e.g. Rosiglitazone or pioglitazone) or Alpha-glucosidase inhibitors (acarbose). No
reference recommended one of these over the other!
Reference: http://www.uptodate.com/contents/management-of-persistent-hyperglycemia-in-type-2-diabetes-
mellitus
Http://www.aafp.org/afp/2001/0501/p1747.html

195.Side effects of bupropion?


Answer: Most common side effects in decreasing order include: headache, dry mouth, nausea, weight loss, insomnia, agitation,
dizziness, pharyngitis.
Reference: http://reference.medscape.com/drug/wellbutrin-zyban-bupropion-342954#4

196.Side effects of tricyclic antidepressants?


Answer: Dry mouth, blurred vision, constipation, urinary retention, drowsiness, increased appetite leading to weight gain, drop in
blood pressure when moving from sitting to standing, which can cause lightheadedness, increased sweating & may prolong QT
interval in ECG
Reference: http://www.mayoclinic.org/diseases-conditions/depression/in-depth/antidepressants/art-20046983

197.Abortive migraine medication?


Answer: Triptans
Explanation:

358
198.Treatment of herpes zoster?
Answer: Antiviral (acyclovir). Analgesia with nsaids or acetaminophen. If not enough narcotics can be added such as oxycodone. If
severe pain persists despite using narcotics steroids might be added.
Reference: http://cursoenarm.net/UPTODATE/contents/mobipreview.htm?1/5/1104

199.Diabetic patient come with fatigue and drowsiness, which drug group responsible for that:
Salphonylurea

Question and choices incomplete, answered by the one who wrote the question
Explanation: sulphonylureas adverse events include hypoglycemia and weight gain (none of the references mentioned fatigue and
drowsiness, unless symptoms were of hypoglycemia). Glucagon-Like Peptide Analogues (e.g. Exenatide and Liraglutide) cause
dizziness and muscle weakness. Thiazolidinedione (e.g. Rosiglitazone) may cause anemia which can present as fatigue.
Reference: Toronto Notes 2015, page E52

200.Gram neg: Aerobic: Non lactose-fermenting (Lactose negative): Oxidase (-):


Acinetobacter spp, Burkholderia spp, E. Coli, Proteus spp, Salmonella spp, Shigella spp, Serratia spp*, Stenotrophomonas maltophilia

201.Antacid MOA:

359
202.Patient take anti-lipid medication that cause hyperkalemia what is the medication:
A) Fibric Acid Derivatives clofibrate،fenofibrate cause arrhythmia
Answer: Statins very rarely causes hyperkalemia.
Http://m-learning.zju.edu.cn/G2S/ewebeditor/uploadfile/20120529102107199.pdf

203.Patient who has sulfa allergy. What is the best antidiabetic?


A) Meglitinides (repaglinide and nateglinide,)
Answer: A
Http://www.uptodate.com/contents/sulfonylureas-and-meglitinides-in-the-treatment-of-diabetes-
mellitus#H7
- The mechanism of action of the meglitinides closely resembles that of the sulfonylureas.
- Sulfonylurea Examples: (Glyburide, glipizide, glimepiride, tolazamide, tolbutamide) Contraindications: Sulfa allergy, type
1 diabetes, diabetic ketoacidosis http://emedicine.medscape.com/article/2172160-overview

360
204.Patient with angina took a pill under the tongue. What is its mechanism?
Answer: The patient took nitroglycerine sublingually. It causes vasodilation. It is achieved by activating guanylate cyclase and
increasing guanine nucleotide. This in turn activates cgmp-dependent kinase which dephosphorylates myosin light chain leading to
smooth muscle relaxation. Venodilation ↓ preload & ↓ cardiac work. Arterial dilatation ↓ after load. Both of which decrease O 2
requirements. It also decreases infarct size and post MI mortality.
USMLE Step1 Pharmacology Kaplan Lecture Notes

205.X-ray of the lung showed opacification with air fluid level? What is the MOA of Abx?
Standard treatment for an anaerobic lung infection is clindamycin (600 mg intravenously [IV] every eight hours, followed by 150 to
300 mg orally four times daily) based upon two published trials that demonstrated superiority of this agent compared with
parenteral penicillin
Mechanism of Action Reversibly binds to 50S ribosomal subunits preventing peptide bond formation thus inhibiting bacterial
protein synthesis; bacteriostatic or bactericidal depending on drug concentration, infection site, and organism

206.-phenytoin side effect Stevens–Johnson syndrome ?


Read about its clinical presentation:Typical prodromal symptoms of Stevens-Johnson syndrome are as follows:
Cough productive of a thick, purulent sputum
Headache
Malaise
Arthralgia

Patients may complain of a burning rash that begins symmetrically on the face and the upper part of the torso. The cutaneous
lesions are characterized as follows:

The rash can begin as macules that develop into papules, vesicles, bullae, urticarial plaques, or confluent erythema
The typical lesion has the appearance of a target; this is considered pathognomonic
In contrast to the typical lesions of erythema multiforme, these lesions have only 2 zones of color
The lesion’s core may be vesicular, purpuric, or necrotic; that zone is surrounded by macular erythema
Lesions may become bullous and later rupture, leaving denuded skin; the skin becomes susceptible to secondary infection
Urticarial lesions typically are not pruritic
Infection may be responsible for the scarring associated with morbidity
Although lesions may occur anywhere, the palms, soles, dorsum of the hands, and extensor surfaces are most commonly
affected
The rash may be confined to any one area of the body, most often the trunk

Signs of mucosal involvement can include the following:


Erythema
Edema
Sloughing
Blistering
Ulceration

361
Necrosis

The following ocular signs may be noted on slit-lamp examination:


Eyelids: Trichiasis, distichiasis, meibomian gland dysfunction, blepharitis
Conjunctiva: Papillae, follicles, keratinization, subepithelial fibrosis, conjunctival shrinkage, foreshortening of fornices,
symblepharon, ankyloblepharon
Cornea: Superficial punctate keratitis, epithelial defect, stromal ulcer, neovascularization, keratinization, limbitis,
conjunctivalization, stromal opacity, perforation
Ref: Medscape http://emedicine.medscape.com/article/1197450-overview

207.Post MI developed palpitation ECG pic (VT) Ttt:


A) Metoprolol Adenosine
Answer: A ( bb )
Metoprolol is a selective beta1-adrenergic receptor blocker that decreases the automaticity of contractions. During IV
administration, carefully monitor the patient's blood pressure, heart rate, and electrocardiograms (ecgs). Long-term use of
metoprolol has been shown to reduce
Http://emedicine.medscape.com/article/159075-medication

208.What antihypertensive decreases preload as well as causing vasodilatation:


Answer: ACEI , CARVIDILOL

209.Muscle relaxant
Answer:

210.Antidote for Opiate?


Answer: Naloxone

211.What is the mechanism of action of aspirin?


Aspirin induces a permanent functional defect in platelets, which can be detected clinically as a prolonged bleeding time. This
appears to be primarily, if not exclusively, due to irreversible inactivation of a key enzyme in platelet arachidonate metabolism
through acetylation of a critical serine residue near its catalytic site. This enzyme, cyclooxygenase (COX)-1, is responsible for the
formation of prostaglandin (PG)H2, the precursor of thromboxane (TREATMENT )A2. The non-linear relationship between
inactivation of platelet COX-1 and inhibition of TREATMENT A2-dependent platelet function by low-dose aspirin has important
implications: (i) a substantial reduction in platelet inhibition is associated with less than maximal inactivation of COX-1; (ii) recovery
of platelet function is disproportionately rapid, occurring within 3–4 days upon drug withdrawal, (iii) the requirement for virtually
complete and persistent inhibition of platelet COX-1 cannot be met by most traditional non-steroidal anti-inflammatory drugs
[7]
(nsaids), allowing their COX-2-dependent cardiotoxicity to be unmasked. Moreover, inhibition of TREATMENT A2-dependent
platelet function by aspirin leaves other platelet pathways [adenosine diphosphate (ADP)-P2Y12, thrombin-protease-activated
receptor (PAR)-1] largely unaffected, thus providing a rationale for dual or triple antiplatelet therapy in high-risk settings.

212.Best antibiotic for gram( - )bacilli?


Answer: Aminoglycosides

362
Http://www.uptodate.com/contents/gram-negative-bacillary-bacteremia-in-adults

213.Case which is group positive cocci in cluster methicillin sensitive what antibiotic
Answer: no options but General rule from master the board:
Any methicillin sensitive or resistant is Nafoxillin sensitive, resistant

214.Which of the emergent contraceptive drugs causes hypokalemia?

215.Which of the emergent contraceptive drug causes hypokalemia or hyperkelmia?

216.Medication prescribed to patient and he was asked to come after 2 weeks to check his INR , how does this medication act (
on which enzyme ) ?

217.What is the muscarinic effect after administration of anticholinergic drug?


Like sympathetic (atropine)

218.Aspirin in high dose cause hyperthermia through which mechanism: (read about it)
Salicylates are neurotoxic,
Cns toxicity is related to the amount of drug bound to cns tissue. It is more common with chronic than acute toxicity.
Acidosis worsens cns toxicity by increasing the amount of salicylate that crosses the blood-brain barrier and increases cns tissue
levels. there is an association between taking aspirin for viral illnesses and the development of reye syndrome.in child

363

Potrebbero piacerti anche